ATAR Notes: Forum

VCE Stuff => VCE English Studies => VCE Subjects + Help => VCE English & EAL => Topic started by: literally lauren on January 11, 2015, 10:35:55 am

Title: VCE English Question Thread
Post by: literally lauren on January 11, 2015, 10:35:55 am
ENGLISH Q&A AND PAST QUESTIONS DATABASE

To go straight to posts from 2020, click here.

What is this thread for?
If you have general questions about the VCE English (or EAL) course about what the tasks entail or how to improve in certain areas, this is the place to ask!

However, if you're looking for text-specific advice (eg. 'How could I talk about character X from text Y' or 'Is my interpretation of Z correct?') then please make a new thread on the English page. This section is designed to be an information and advice resource, so keep your questions fairly general, if possible.

If you're looking for essay marking and feedback, go to the English Work Submission and Marking


Who can/ will answer questions?
Everyone is welcome to contribute; even if you're unsure of yourself, providing different perspectives is incredibly valuable in English, - since we all know there's rarely one "right" answer ;)

Please don't be dissuaded by the fact that you haven't finished Year 12, or didn't score as highly as others, or you're advice contradicts something else you've seen on this thread, or w/e; none of this disqualifies you from helping others. And if you're worried you do have some sort of misconception, put it out there and someone else can clarify and modify your understanding! 

I will endeavour to help wherever I can, but there'll be a whole bunch of other high-scoring students with their own wealths of wisdom to share with you. So you may even get multiple answers from different people offering their insights.


Which questions get included in the database?
The questions in the tabs below are ones deemed helpful to multiple people (ie. dealing with common pitfalls and struggles.) That is not to say other questions are less valid, but these are just the most common areas that students need help with, so please have a read through some before asking your own questions. If there are points that haven't been covered yet, or you'd like a more detailed explanation for an existing question, post here and we can update the database for everyone.

There's also a list of useful Language Analysis vocab attached to this post :)


TEXT RESPONSE
Preparation
What to discuss in a T.R. essay + picking out evidence? (helpful for early in the year)
The 'goal' of T.R. essays (scroll down to second response)
Using background information
Understanding the text

Planning
Planning T.R. essays (scroll down to second response)

Essay Structure
Teachers "recommending" structures

Introductions
What should be included in the intro? 
Structuring introductions
How to make introductions more sophisticated 
What makes an introductions stand out
Using quotes in the introduction (2)

Body Paragraphs
Wording of topic sentences
Choosing evidence from the text
Finding paragraph weaknesses? (scroll down to third response)
Appropriate explanation of evidence in essays (2)

Ideas
Coming up with original ideas
The difference between 'surface level' themes and 'in-depth' themes
Making simple ideas sophisticated
Memorising ideas
Exploring the impact of major/minor characters

Conclusions
Structuring conclusions (2)

Contentions
Strengthening contentions

Areas of Study
The most important criterion
Layers of Text Response texts
Learning about Views and Values with Harry Potter
Addressing views and values (ft. Paddington Bear)
Branching out from the text and explaining the marking process

Prompts
What are 'implications?'
How to discuss implications (2)
Dealing with difficult prompts
Why 'How...' questions aren't as scary as you think
Addressing 'Do you agree?' prompts

Quotes
Embedding quotes
Shortening and integrating quotes
Memorising and categorising quotes
Memorising quotes for the exam

Exam Preparation
Choosing your exam text before the exam (2)
Making the most of rereading texts
Planning in the exam + choosing exam text

CONTEXT
Criteria
A translation of VCAA's criteria?
Why memorisation is a massive pitfall
How to ensure relevance
Explaining the target audience
Originality in context pieces
How to approach Context studying

Intoductions
Why you shouldn't mention Hitler ;) and the importance of originality in context + my sample intro
What should be in an introduction?

Body Paragraphs
Structuring body paragraphs (2) (3)

General Stylistic Advice
Forms and styles: playing it safe
A basic overview of the three styles
Equal opportunity writing styles? + waxing poetic about The American Dream
Choosing a style

Expository Writing
Balancing examples with discussion + responding to prompts + expository structure?
Abstract discussion in expository essays (with pretty colours because Lauren learnt how to format)
Tips for expository writing
What to include in expository pieces - ft. cake

Imaginative/Creative Writing
Benefits of the Imaginative style? (scroll down to second response)
An explanation of the 'show, don't tell' rule for creative writing
Made-up stories

Persuasive Writing
Structuring a persuasive piece

Ideas
Generating ideas (2)
Managing different ideas at once
Going beyond the text: The Very Hungry Caterpillar and Year 12 English
Ratio of examples to discussion + questioning the prompts?
Using close analysis (ie. symbols, motifs) in a Context piece?

Set Texts
Extent of set text usage (2) (3) (4)
Quoting the set text
'Jumping around' within the text, and why chronological exposition isn't worth much (scroll up a bit for an example involving a headless chicken)
Using texts from previous years

External Examples
Finding external examples
Tying in examples, and zooming in/out?
Trying to find examples (esp. for Encountering Conflict)
Categorising examples
Linking between examples
Sopistication of examples (scroll to third response) (2) (3)
Using risky examples (2)

Prompts
Questioning the prompts - with example (scroll to second response)
Dissecting prompts
Simplifying prompts - using the 2013 conflict prompt as an example
Unpacking prompts (2)
The importance of prompts


MISCELLANEOUS
Studying for English
Advice for Year 12s (2)
Improving in English
'Thinking' in English, with a chicken-flavoured analogy
Fascilitating self-improvement
Going beyond practice essays
Using high-scoring responses
Frequency of study
Typing or handwriting essays? (scroll down to second response)
Is a tutor necessary? (Spoilers! The answer is no.)
SAC Preparation (2)

Using Study Guides
Using study guides (2) (3)

Reading
Wider Reading
Reading academic journals?

Vocabulary
Acquiring new vocabulary (2)
Improving vocabulary (2) (3) (4) (5)
Circumlocating verbosity (2)
Vocab: evoke/provoke/invoke
Self-editing for expression
Improving clunky sentences (scroll down to second response)

General Writing Tips
Overcoming writer's block (2) (3) (4) (4) (5) (6)
Quality vs. Quantity: recommended word count?
Writing with clarity

Time Constraints
Dealing with time esp. in the exam (2) (3) (4)
Planning under timed conditions?
Timing of SACs
Cost/benefit analysis of planning under test conditions
How to prepare for time constraints
Concerns about timing + checklist to go through before a SAC (scroll down to second response)

Advice for non-Year 12s and other English subjects
Advice for Year 9/10 English?
General advice for year 11
English/EAL differences
English 3/4 without 1/2 (with Lit 1/2)


Exam Preparation
When to start studying for the exam
Exam Details (scroll down to second response)
Exam writing booklets
What to bring to the exam (2)
Assorted pre-exam questions 2015 (2)
How exam marking works
Day before the exam?

In the Exam
Order of essays (scroll down to second response) (2)
Timing in the exam
Title: Re: VCE English Question Thread
Post by: Leezy on January 11, 2015, 09:36:41 pm
Hey Lauren,
Would you recommend starting a text response essay with a quote as the first sentence in the introduction? My tutor says that examiners apparently love students starting with a quote, but my teacher tells me to avoid it.
Title: Re: VCE English Question Thread
Post by: cosine on January 11, 2015, 11:19:43 pm
I have started all my subjects besides english. Should this be worrisome?

Also, I think the reason why I haven't started is because I don't know where to begin... Someone who got 40 told me to read my books, but I cant even look at it lol :/
Title: Re: VCE English Question Thread
Post by: Paulrus on January 12, 2015, 12:33:29 am
Hey Lauren,
Would you recommend starting a text response essay with a quote as the first sentence in the introduction? My tutor says that examiners apparently love students starting with a quote, but my teacher tells me to avoid it.
I don't think there's a definitive answer to this, because each examiner is going to have their own preferences. That said, I'm more inclined to agree with your teacher. If the quote isn't highly relevant to the prompt, it'll make your essay look a bit pre-prepared - and if it is relevant, it'd be better used in a body paragraph, where you can actually get credit for using it to substantiate your ideas. So personally, I'd avoid it :)

Instead, if you're not keen on writing a completely new intro from scratch each time, you can try opening with a contextualising statement related to your text. I generally started with something like "Informed primarily by his own experiences as a soldier, Wilfred Owen's anthology 'The War Poems' elucidates/reflects on/illustrates _____", and that bit there would be altered depending on the prompt. That way it doesn't really feel generic, but you still have a solid and malleable base to work from for each essay. Another example is "Silhouetted against the backdrop of ____", but so many people start with that line that you'd be better off avoiding it lol.

I have started all my subjects besides english. Should this be worrisome?

Also, I think the reason why I haven't started is because I don't know where to begin... Someone who got 40 told me to read my books, but I cant even look at it lol :/
Ideally you should have started by now, but there's still plenty of time. You should definitely begin by reading your texts as soon as you can. I don't think you necessarily have to read all of them these holidays, but if you're feeling short on time you should familiarise yourself with your unit 3 texts at the very least. You'll find that the year goes extremely fast and you won't have much time to read them once it's begun.

Unfortunately, there's no easy way around it (except for just reading Sparknotes, but you know... don't do that), you're going to have to tackle those books. Take them a bit at a time if you need to - once you actually start them, they'll feel a lot less intimidating and you might find that you actually enjoy them :)
Title: Re: VCE English Question Thread
Post by: literally lauren on January 12, 2015, 01:02:14 am
Hey Lauren,
Would you recommend starting a text response essay with a quote as the first sentence in the introduction? My tutor says that examiners apparently love students starting with a quote, but my teacher tells me to avoid it.

I wouldn't say the examiners love it, but starting with a quote can be effective if done well. Note the difference here:
- "No longer would I be trampled, thrown about, or foiled." This determined shift marks a crucial turning point in the develpment of character in author's text.
- "No longer would I be trampled, thrown about, or foiled." In author's text, character undergoes many important changes in attitude.

The first would be safer, because it's explicitly stating the connection to the assessor. Put yourself in the shoes of a marker; you could read that second sentence and know what the student meant, but you're not allowed to give marks for the inferences you make.

If you're teacher isn't a fan of this style, then steer away from it for SACs, but if you want to keep it as an option for the exam, then by all means do so. Personally, I'd say unpacking the prompt is more important though. (Paulrus' example works really well for basically any prompt you come across, you'd just have to find something that works equally well with your specific texts.)

I have started all my subjects besides english. Should this be worrisome?

Also, I think the reason why I haven't started is because I don't know where to begin... Someone who got 40 told me to read my books, but I cant even look at it lol :/
It's never too late to start English ;D
At the very least, pleeeeeease read your texts. I promise it will help immensely to be able to think a step ahead. Even if you're reading through it in class, the things your teacher hints at and discusses will be way clearer if you already know the basic plot and character trajectories.

Places to start: (aside from reading your texts, which is very much step 1 in the whole process)
- Read over your Year 11 essays, marks, and feedback. See if there are common areas of weakness or if you don't understand where/ why you were wrong.
- Read some study guides (could be text-specific, or just general 'how to English' style things.) This can help clarify any conceptual errors you find in the reading of your own work.
- Read other people's essays. Depending which texts you're studying, there should be a few pieces floating around AN or the interwebs somewhere. Otherwise, you could also look at Language Analyses and try to familiarise yourself with the different ways of structuring essays.
- Sort out your oral. For most schools, this is the first SAC you'll do, but even if that's not the case, getting your speech or resources ready will save you a lot of time later. It's okay to wait until your teacher is explaining Text Response or Context concepts in class before you start analysing the texts or writing practice pieces, but the oral is pretty much self-directed, so get that out of the way, and you'll be able to dedicate more time to other subjects too.

Title: Re: VCE English Question Thread
Post by: JackSonSmith on January 15, 2015, 02:03:14 pm
Could someone please explain what this means and perhaps provide an example where this is evident. (regarding a 9 to 10 out of 10 for text response essay)

‘demonstrates an understanding of the implications of the topic, using an appropriate strategy
for dealing with it, and exploring its complexity from the basis of the text.’ 
Title: Re: VCE English Question Thread
Post by: ras on January 15, 2015, 02:50:51 pm
Could someone please explain what this means and perhaps provide an example where this is evident. (regarding a 9 to 10 out of 10 for text response essay)

‘demonstrates an understanding of the implications of the topic, using an appropriate strategy
for dealing with it, and exploring its complexity from the basis of the text.’

(anyone else please build on this answer I'm only going to answer it partially)

One way I always looked at this excelling at this criterion was ensuring that I addressed the nuances in a topic. For example, I studied Brooklyn and on a SAC got the topic 'Toibin's Brooklyn is an ultimately uplifting portrayal of love.' We were told that the high scorers on this SAC did not solely focus on romantic love in their essays, but also on other sorts of love, such as familial love, love for one's culture, love for one's country etc etc.

So essentially, I would see this criterion as looking at a topic, highlighting its key words, and brainstorming the implications of each key word, ensuring that you are exploring the nuances of the connection between these words and the text.

'An understanding of the implications of the topic' may also be referring to identifying how certain aspects of the text may contradict what is being stated in a topic. This may be addressed in the typical 'yes, but also no' approach to structuring a text response essay. It is extremely rare for an essay topic to be accurate for every section of a text, and examiners are often looking for you to recognise this and highlight the possibly problematic aspects of a topic. I always found this difficult to do well, as I would end up contradicting what I may have stated in earlier paragraphs.

In terms of 'appropriate strategies' for dealing with the complexities of a topic, I would usually aim to define key words in my intro. So if the word 'self-discovery' was in a topic I'd highlight in my intro that in my essay 'self-discovery' refers to a character gaining confidence, assertiveness, a clear idea of who they are, they stop being defined by others.

Hope some of this helps :D
Title: Re: VCE English Question Thread
Post by: literally lauren on January 15, 2015, 03:03:52 pm
Could someone please explain what this means and perhaps provide an example where this is evident. (regarding a 9 to 10 out of 10 for text response essay)

‘demonstrates an understanding of the implications of the topic, using an appropriate strategy
for dealing with it, and exploring its complexity from the basis of the text.’

I love this word!

Simply put, dealing with the 'implications' is the difference between writing about the words in the prompt, and writing about the ideas in the prompt.

Random thing from a 2012 practice exam to illustrate:
             'Which is more important to Antonio: to hunt and kill the jaguar, or to make amends for the past?'
This is from a film called The Old Man Who Read Love Stories. Don't worry if you haven't seen it; that's actually better for the purposes of this explanation.

So the simple approach typical of middle-band responses is to do this:
Which is more important to Antonio: to hunt and kill the jaguar, or to make amends for the past?
So there'll probably be one paragraph on the importance of the hunt for the jaguar and why that's important to Antonio, then one on how/why he makes amends for the past. The third might deal with these two in conjunction, or introduce a challenge by discussing other things that are important to the character. Whilst this is a safe approach, it's also very limiting.

In ras' method above, this 'key terms' approach works well because the definition of love is central to the way you would approach the question. Provided you were able to add sophistication in other ways, this is perfectly acceptable for certain prompt types, but if you need to look at overall implications/ 'between the lines' stuff and not just definition nuances, here's how:

Ask yourself: if what the prompt says is true, then what?
- If Antonio must prioritise either the hunt, or making amends, then the prompt suggests these two endeavours are mutually exclusive. Both cannot be of equal importance.
- If making amends cannot occur in the form of killing the jaguar, then that means the act of killing represents a refusal to atone for the past.
- Whichever option Antonio chooses, it is inferred that he does not care as much for the other.
- These external responses are indicative of what Antonio values/ finds more important. He cannot act in a state of cognitive dissonance; what he does reflects who he is.
- Antonio acts on what he finds important; his behaviour is a result of his beliefs
With each of these implications we can then ask ourselves 'is this the case in the text?' And filling in those blanks will give you a contention.

Up until you ask yourself this question, you don't need your textual knowledge. Don't bring up examples to demonstrate your points (yet!) Just examine the prompt using logic, and decypher what it is implying as well as denoting/ literally saying.

So a simple contention would be along the lines of 'Both the hunt and the idea of atonement are important to Antonio, but ultimately the hunt is more important.'
A better, more insightful contention would be: 'Antonio's prioritising of the hunt for the jaguar, despite it being an ultimately self-destructive pursuit, suggests his character does not intend to atone for his past mistakes.'
(Note, I'm not familiar with the text so these contentions might both be wrong, but we're looking for the structural differences in approach here, so it doesn't matter.)

As for the rest of that criterion, it's worth thinking about the "appropriate way to deal with the prompt" section. Basically this means that you can't just walk into any assessment writing a paragraph on pre-selected ideas, or in a rigidly pre-prepared manner. eg. The alternate prompt in the aforementioned prac exam was 'To what extent is belonging important in the film.' There aren't a lot of questions to ask here, so you'd have to add complexity in a different way, like defining belonging and examining all the different ways it is present or absent. You should also have a couple of structural methods at your disposal (eg. group paragraphs by thematic concern; start by talking about big ideas then 'zoom in' to the text, etc.) and then chose the most applicable one.
Title: Re: VCE English Question Thread
Post by: JackSonSmith on January 15, 2015, 10:12:24 pm
Thank-you both for taking the time to help me - and I'm sure, many others. I'll give more thought to the nuances that are contained within prompts and the implications of the ideas suggested. I guess having a dictionary and reading time has been accounted for by the VCAA.
Title: Re: VCE English Question Thread
Post by: cosine on January 15, 2015, 10:13:26 pm
Okay, so I miraculously read my book twice... now what? lol what should I do now?
Title: Re: VCE English Question Thread
Post by: literally lauren on January 15, 2015, 10:33:35 pm
Okay, so I miraculously read my book twice... now what? lol what should I do now?
Still more reading to go :)

Now that you have an understanding of the basic plot, it's time to start working through external resources to compliment your interpretation. Depending on which text you're studying, you should just be able to type '>title< VCE English resources' into google and get a fair few good sites. If you really wanted to, you could invest in some study guides too, though there should be enough free stuff available.

Using these, you can develop your own resources. Start with the basics (character maps, chapter summaries, quote banks, etc.) as these will give you a good basis for later analysis.

It kind of depends what text you're studying though. Like, if it's a novel, you'll want to spend a bit more time on summary exercises to make sure you've covered everything. Contrarily, if it's a play, then you'll have to think more about performativity, staging, and the ambiguities of performance.

The priority at the moment is ensuring you go into Year 12 with a solid understanding of the book you're studying, and the requirements of a T.R. essay. So long as you're comfortable with that, you should be fine :)
Title: Re: VCE English Question Thread
Post by: Auralee on January 16, 2015, 05:30:10 pm
Hi Lauren! I love writing and reading, however I have never been able to bring myself to read a novel twice. I am wondering how you persevere and tackle even the most sullen of VCE texts.

Also, I am really struggling with Language Analysis. I am a fairly proficient writer, however have always found difficulty in this part of English. I have read your sample essay, which by the way is out-of-this-world awesome, but I do not know how I can work towards that. I have also read through your guide but still have trouble in actually composing sentences and paragraphs that flow nicely and are cohesive. How would you suggest I proceed?

Title: Re: VCE English Question Thread
Post by: literally lauren on January 16, 2015, 08:32:12 pm
Hi Lauren! I love writing and reading, however I have never been able to bring myself to read a novel twice. I am wondering how you persevere and tackle even the most sullen of VCE texts.

Also, I am really struggling with Language Analysis. I am a fairly proficient writer, however have always found difficulty in this part of English. I have read your sample essay, which by the way is out-of-this-world awesome, but I do not know how I can work towards that. I have also read through your guide but still have trouble in actually composing sentences and paragraphs that flow nicely and are cohesive. How would you suggest I proceed?
VCE texts are not to be read for fun. Except Shakespeare, who is God.

I treated the second/third readings more like dissection exercises. Rather than sitting down to read them cover-to-cover, I'd constantly be stopping to google things, compile annotations, add quotes or other notes to my collections, or just making general scribbles.

Unfortunately some texts are just... well, sullen. Try and make them interesting, maybe by researching the history of the author or the zeitgeist. Otherwise, you can always make your analysis interesting by considering different points of view, eg. the audience of the time, and contemporary audiences (if applicable.)

With the exception of maybe three or four texts, I'd argue that every one on the list has some kind of interesting component to discuss.

Re: Language Analysis, it's great that you've already identified that you need to improve flow, now work out why your essays/ paragraphs/ sentences aren't flowing. eg. are you
a) running points of random analysis together with no clear thread, just because you want to cover as much of the article as possible?
b) jumping from the analysis of language to the contention without connecting threads?
c) cutting the analysis too short so that it feels stilted?
d) letting the analysis stretch out too long, or taking several sentences to say what could have been said in two?
e) not using the right linking words; so your ideas flow, but not the actual sentence structure?
f) not able to connect one discussion/ point of analysis to another?
g) not able to connect the stages of analysis (what-how-why: see first post for links to explanations)
h) any combination of the above, or other...?

Being a good writer helps immensely, and once you're aware of the specific problem, fixing it won't take long at all. Just narrow it down as much as possible. Biggest and most helpful question to constantly ask is 'do I not know what to write, or do I not know how to write it?' At the very least, this should help you to work out whether you need to develop your conceptual understanding of the task/ criteria, or if it's an issue with approach/ language/ vocab/ sentence structure.

Title: Re: VCE English Question Thread
Post by: JackSonSmith on January 17, 2015, 04:57:56 pm
I had a go at thinking, and have come up with a revised contention for my practice essay on "This Boy's Life". Am I on the right track?

Prompt: Discuss the conflict between Jack's desire for freedom and his desire to belong.

Original contention: Jack has an internal conflict between wanting to be free and wanting to belong.

Contention after thinking: Jack's desire to belong, despite it requiring him to betray his true self, suggests that he does not truly wish to be free – but to be free of the person that he was. (modeled after Lauren's example)
Title: Re: VCE English Question Thread
Post by: Auralee on January 17, 2015, 06:07:22 pm
Hi Lauren (again!) Thank you for that advice. I think that I will try and instead of 'reading the whole book again', maybe read the important bits or find other ways to enhance my understanding of the novel (e.g. research, reading other interpretations etc).

As for language analysis, I believe I know 'what' to write - I'm fairly good at identifying the techniques used by the writer - however am constantly struggling to find a way of fabricating a piece of writing that isn't just:

technique --> effect on audience

technique --> effect on audience

etc, etc etc,

Literally my essays just look like dotpoints right now, which obviously doesn't earn you the high scores.

Any suggestions in how I can generate more fluidity in my writing?

Much thanks,
Auralee
Title: Re: VCE English Question Thread
Post by: literally lauren on January 17, 2015, 08:31:09 pm
I had a go at thinking, and have come up with a revised contention for my practice essay on "This Boy's Life". Am I on the right track?

Prompt: Discuss the conflict between Jack's desire for freedom and his desire to belong.

Original contention: Jack has an internal conflict between wanting to be free and wanting to belong.

Contention after thinking: Jack's desire to belong, despite it requiring him to betray his true self, suggests that he does not truly wish to be free – but to be free of the person that he was. (modeled after Lauren's example)
Way better contention, man! Well done!
Notice how the second one is much stronger, not just because it's longer, but because it's got more content contained within it. You're not simply saying 'Jack has a conflict,' you're saying 'Jack wanting to belong required a betrayal of his identity, meaning he wanted to be free of his former self.' You've incorporated much more of the prompt, and you've opened up several more angles of inquiry.
With that said, don't be limited by this contention either. There are still some concepts you could examine (eg. what kinds of internal conflict does Jack experience, and how do they manifest themselves? Does one of these desires (freedom or belonging) win out in the end? Why do they come into conflict in the first place? etc.)
None of these are required, but don't feel you're stuck dealing with what you can sum up in a sentence. Cover as much as you can with the contention, but expand it later if you need to. :)

As for language analysis, I believe I know 'what' to write - I'm fairly good at identifying the techniques used by the writer - however am constantly struggling to find a way of fabricating a piece of writing that isn't just:

technique --> effect on audience

technique --> effect on audience

etc, etc etc,

Literally my essays just look like dotpoints right now, which obviously doesn't earn you the high scores.

Any suggestions in how I can generate more fluidity in my writing?
Alrighty: your current issue is that you're only using what and how statements. So you clearly need to involve the why component in some capacity, which I'll explain below, but just to clarify:
The what section is more than just the technique, and how is more than just the effect. You don't need to list a precise technique every time, you could simply quote, refer to tone, or comment on language generally. You definitely don't need to explain or define the technique; it's more about the language of the article than anything else. For the how section, think about integrating the context/ background information as well (where relevant.) You could also 'split the audience,' ie. comment on how language might affect different people in different ways. Don't delve too far into this, and don't make judgements without evidence, but if you're given the information, then you're allowed to use it for analysis.
For instance, let's say you were analysing a leaflet about the dangers of fast food, and there was a paragraph talking about how it damages your health. If you've been told this leaflet was distributed amongst a vegan yoga club, then the effect would be one of self-congratulations and pride. Whereas, if they were handed out at every Maccas and KFC in town, then clearly the author is angling to elicit more shame, regret, and humiliation.

Now let's unpack that troublesome 'why.'
The purpose of this component is to link your discussion back to the author's contention, thereby making it clear to the assessor that you understand how the language is persuasive and you're not simply pointing it out and leaving it at that. If the author is using a specific device in an attempt to elicit a certain response, why would he want audience members to feel this way, or believe a certain idea?
On the surface it's a fairly obvious question to answer, but it can require a lot of practice before you are able to tread the line between giving too much detail, and not enough.
Simply put, your analysis at the moment is as follows:
Author does this --> audience feels this    x repeat
What you need to do is round it off at the end like such:
Author does this --> audience feels this. Author does this for these reasons.

eg.
'The use of the childish epithet "liar liar, pants on fire" is designed to engender readers' scorn for the subject of the author's ridicule. This sense of scorn and disapproval for dishonesty is directed at the man inferred to be lying, therefore the author's unequivocal branding of him as a "liar" with firey pants forms part of a scathing attack upon the man's character.'

I think there are some other examples in previous posts. Let me know if this still isn't making sense and I might be able to explain this further.
Title: Re: VCE English Question Thread
Post by: knightrider on January 18, 2015, 12:57:19 pm
Hi Lauren in regards to the attachment below.

Does it mean that students doing units 3 and 4 English in 2017 will have a new study design.

Whereas students doing English  3/4 in 2016 will have the same study design  as before.
Title: Re: VCE English Question Thread
Post by: literally lauren on January 18, 2015, 04:08:03 pm
Thank you for bringing this up! I'd given up refreshing the VCAA page every week or so since I figured they wouldn't release any more details until later this year.
Okay, so I just read through all these new documents and my soul wants to die.
Yes, apparently this year is NOT the last year under the current system because VCE English teachers can't use numbers.

So the class of 2017 will be the first to experience the new system, meaning that next year during the transitional phase, Year 11s and Year 12s will be studying something different.
you'd think if that was the plan all along they'd say 'the accreditation period will end in 2017' but who needs logic when you're a curriculum authority -.-

Cheers to another year of Context! *facedesk*


On the plus side, the Literature course makes slightly more sense than the current one.
I'm still mad though  >:(
Title: Re: VCE English Question Thread
Post by: cosine on January 18, 2015, 05:08:13 pm
Okay, so I think i have a good understanding of the text i am reading now, well its a play...

So what now? I have read some online summaries, plots, character profiles and stuff... what do i do now?

Thank youuuu (ps, i dont know why, but im starting to like english, wtf....?)
Title: Re: VCE English Question Thread
Post by: RazzMeTazz on January 18, 2015, 05:31:50 pm
I was wondering if anyone could please distinguish between

1.) Campaign meeting
2.) Conference
3.) Convention
4.) Forum
5.) Summit

I am trying to choose the setting for my oral presentation, to see which will be best suited to my persona (Aboriginal welfare expert) but I'm unsure of which one would be most appropriate?

I have googled these, but I was still a little confused, as some seem a bit similar?

Thanks, any help would be appreciated!
Title: Re: VCE English Question Thread
Post by: JackSonSmith on January 18, 2015, 09:12:30 pm
I was wondering if anyone could please distinguish between

1.) Campaign meeting
2.) Conference
3.) Convention
4.) Forum
5.) Summit

I am trying to choose the setting for my oral presentation, to see which will be best suited to my persona (Aboriginal welfare expert) but I'm unsure of which one would be most appropriate?

I have googled these, but I was still a little confused, as some seem a bit similar?

Thanks, any help would be appreciated!

I would think that either a conference or forum would be best suited to your presentation. They, to me - carry the connotation of an open discussion about an issue. Summits sound a bit more global ie. "economics"/"health issues". Of course these are just my own interpretations of these words.
Title: Re: VCE English Question Thread
Post by: RazzMeTazz on January 18, 2015, 09:29:37 pm
I would think that either a conference or forum would be best suited to your presentation. They, to me - carry the connotation of an open discussion about an issue. Summits sound a bit more global ie. "economics"/"health issues". Of course these are just my own interpretations of these words.

Thanks! Yeah I was thinking conference as well, however since this is a persuasive speech, I wasn't sure if such speeches are given at conferences?

Title: Re: VCE English Question Thread
Post by: knightrider on January 19, 2015, 12:16:43 am
Thank you for bringing this up! I'd given up refreshing the VCAA page every week or so since I figured they wouldn't release any more details until later this year.
Okay, so I just read through all these new documents and my soul wants to die.
Yes, apparently this year is NOT the last year under the current system because VCE English teachers can't use numbers.

So the class of 2017 will be the first to experience the new system, meaning that next year during the transitional phase, Year 11s and Year 12s will be studying something different.
you'd think if that was the plan all along they'd say 'the accreditation period will end in 2017' but who needs logic when you're a curriculum authority -.-

Cheers to another year of Context! *facedesk*


On the plus side, the Literature course makes slightly more sense than the current one.
I'm still mad though  >:(

Thanks Lauren for clarification. :)
i was just wondering what did you mean by  this part."because VCE English teachers can't use numbers."
Title: Re: VCE English Question Thread
Post by: Auralee on January 19, 2015, 12:22:23 pm
Hi Lauren! Thanks for your advice... I'm still a little bit stuck on this but hopefully I'll get better with time.
Title: Re: VCE English Question Thread
Post by: StupidProdigy on January 19, 2015, 05:18:17 pm
Hey Lauren!
I'm kinda confused with my why I'm meant to read the white tiger at my school this year because I can't find it or a prompt for it on the 2014 exam? Just searching for some clarification. Thankyouuuu! :)
Title: Re: VCE English Question Thread
Post by: IndefatigableLover on January 19, 2015, 05:21:03 pm
Hey Lauren!
I'm kinda confused with my why I'm meant to read the white tiger at my school this year because I can't find it or a prompt for it on the 2014 exam? Just searching for some clarification. Thankyouuuu! :)
2015 is the first year that White Tiger is implemented for the English Exam :)
http://www.vcaa.vic.edu.au/documents/vce/english/vce_engeal_text_list.pdf

A small summary within the above document by VCAA:

Quote
Adiga, Aravind, The White Tiger, Atlantic Books, 2008 (1)
Set in modern-day India, The White Tiger follows Balram Halwai from his early life of rural poverty to his eventual
success as an entrepreneur and wealthy urbanite. Narrated as a series of letters to Chinese Premier Wen Jiabao, the
novel charts Balram’s journey out of the slums populated by the poor and lower castes, and celebrates his eventual
triumph as he breaks free from a life of servitude and obeisance. The novel explores the divisions between the rich
and the poor, and considers how social structures operate to reinforce class hierarchy. Adiga’s darkly comic novel
also raises questions about the reliability and integrity of the narrator, and asks whether success is ever possible
without moral compromise.
Title: Re: VCE English Question Thread
Post by: StupidProdigy on January 19, 2015, 05:25:09 pm
2015 is the first year that White Tiger is implemented for the English Exam :)
http://www.vcaa.vic.edu.au/documents/vce/english/vce_engeal_text_list.pdf

A small summary within the above document by VCAA:
Ohhh! Thankyou so much! I'm excited for that then, I loved the book 👍
Thanks again
Title: Re: VCE English Question Thread
Post by: literally lauren on January 19, 2015, 07:57:50 pm
Okay, so I think i have a good understanding of the text i am reading now, well its a play...

So what now? I have read some online summaries, plots, character profiles and stuff... what do i do now?
If you feel confident enough in your understanding of the text, you might look into reading some analyses (depending on which play you're studying, there might be proper academic analyses out there, or you may just have to use VCE level essays; both have their advantages.)
More importantly though, ask yourself 'is there anything I find difficult about the text?' Because now is a great time to clear those issues up. Or, more broadly than T.R. ask yourself what you find concerning about English. eg. does the idea of timed responses freak you out/ can you not fathom how to write an oral presentation/ do you not understand what Context even is? and who could blame you
Even the Study Design won't contain a comprehensive list of everything you need to know, so familiarise yourself with the criteria, and then fine-tune any problems or misconceptions afterwards. Ask yourself the question 'where should I go from here?' because that'll be way more beneficial than me simply suggesting a possible direction :)
(ps, i dont know why, but im starting to like english, wtf....?)
(http://1.bp.blogspot.com/_1XAzIkSK23s/TSIeffF2ZhI/AAAAAAAAABY/sGq0a4hwYdE/s320/a-cunning-plan.jpg)

I was wondering if anyone could please distinguish between

1.) Campaign meeting
2.) Conference
3.) Convention
4.) Forum
5.) Summit

I am trying to choose the setting for my oral presentation, to see which will be best suited to my persona (Aboriginal welfare expert) but I'm unsure of which one would be most appropriate?

I have googled these, but I was still a little confused, as some seem a bit similar?
1.) Campaign meeting = ever seen a news interview with a politician who's just won an election and is standing in a room full of screaming groupies who'd applaud anything he/she said? That's the kind of audience that would be at a campaign meeting; you're essentially 'persuading' people who already believe you (or else, why would they be a part of the campaign?) :P
2.) Conference = more of an open, moderate discussion with a balanced audience. This is a pretty broad term though, so it could apply to a variety of different scenarios.
3.) Convention = pretty much interchangeable with a conference; though conventions are more likely to have 'themes' or 'agendas,' perhaps attracting an audience with certain dispositions or proclivities.
4.) Forum = very generic term, could be anything.
5.) Summit = etymologically this is meant to be a meeting between heads of government, or international representatives, though the meaning has widened in modern usage. This is probably too global for your issue though.

Most of the (minor) distinctions are within the kind of audience you're speaking to, so I'd say a Conference or Forum would be the most ideal. Though I highly doubt any assessor would penalise you for saying you're speaking at a conference when it sounded more like something at a convention :P It's good that you're thinking about the context, but I wouldn't get too wrapped up in semantics :)

Thanks Lauren for clarification. :)
i was just wondering what did you mean by  this part."because VCE English teachers can't use numbers."

Just me being petty. Apparently the claim that the study design ends in 2014 actually means the Class of 2017 will be the first ones to go through the new system is something that makes perfect sense to VCAA, since they've offered no clarification as to the constant extensions.
Nvm... if I start ranting I won't stop. Just focus on which ever Study Design will apply to you and don't worry about whatever VCAA logic is :)

Hey Lauren!
I'm kinda confused with my why I'm meant to read the white tiger at my school this year because I can't find it or a prompt for it on the 2014 exam? Just searching for some clarification. Thankyouuuu! :)
Yeah, like IL said, it's a totally new text so there are very few resources available at the moment. At some stage throughout the year I'll update this thread once some samples start cropping up. There'll also be some original ones in around about August/September, or possibly over the June break depending on how busy I get :)
Title: Re: VCE English Question Thread
Post by: g1mp1e on January 22, 2015, 08:18:04 pm
Hey Lauren,

With regards to your lecture today, particularly with language analysis, I've previously been told to structure my ideas into different effects on stakeholders, and the different tonal shifts/language/etc. used for each stakeholder and how they vary according to whom the writer is "talking to". (e.g. in Martin Luther King's speech, he addresses the American people, the American government, and his fellow black Americans - and so you would structure your paragraphs around these three groups of targeted audiences). However, as you said today, we should perhaps aim to split our paragraphs into ideas and concepts raised. My question is, could you explain your way of splitting it into ideas and concepts a bit further, and would you recommend using one or the other, or can you use a mix of the two? If so, how would you go about doing this?

Awesome lecture today by the way!

Much thanks! :)
Title: Re: VCE English Question Thread
Post by: M_BONG on January 23, 2015, 12:13:32 am
Hey Lauren,

With regards to your lecture today, particularly with language analysis, I've previously been told to structure my ideas into different effects on stakeholders, and the different tonal shifts/language/etc. used for each stakeholder and how they vary according to whom the writer is "talking to". (e.g. in Martin Luther King's speech, he addresses the American people, the American government, and his fellow black Americans - and so you would structure your paragraphs around these three groups of targeted audiences). However, as you said today, we should perhaps aim to split our paragraphs into ideas and concepts raised. My question is, could you explain your way of splitting it into ideas and concepts a bit further, and would you recommend using one or the other, or can you use a mix of the two? If so, how would you go about doing this?

Awesome lecture today by the way!

Much thanks! :)
Both are commonly used. The stakeholders approach was the one I used because ANY article will have numerous stakeholders. Stakeholders don't have to be a person - it's just something the author has an opinion on. Eg. racism could a stakeholder in Luther King's speech. It's really easy to identify and it's effective because there is clear separation in what you are analysing.

For example, this is how you would use the stakeholder approach would work.
Body paragraph one: American society (analyse everything said about this)
Body paragraph two: Racism; repeat above
Body paragraph three: Racist people; repeat above.

I think Lauren's idea of splitting things into ideas and concepts (the 'argument' approach) is also quite good - although it might be tricky to sort things into arguments if it's not immediately clear what the author is arguing about. Also, it is a bit arbitrary because you run the risk of analysing arguments not techniques, if you don't master it well.

Title: Re: VCE English Question Thread
Post by: cosine on January 23, 2015, 07:31:16 am
Would reading books improve my english skills? And is it a good idea to read other books in year 12 besides our school ones? Thanks
Title: Re: VCE English Question Thread
Post by: brenden on January 23, 2015, 09:43:47 am
Would reading books improve my english skills? And is it a good idea to read other books in year 12 besides our school ones? Thanks
Yes and yes :).
Title: Re: VCE English Question Thread
Post by: cosine on January 23, 2015, 10:08:23 am
Yes and yes :).


Hahaha thank you :P
Title: Re: VCE English Question Thread
Post by: literally lauren on January 23, 2015, 11:06:57 am
Hey Lauren,

With regards to your lecture today, particularly with language analysis, I've previously been told to structure my ideas into different effects on stakeholders, and the different tonal shifts/language/etc. used for each stakeholder and how they vary according to whom the writer is "talking to". (e.g. in Martin Luther King's speech, he addresses the American people, the American government, and his fellow black Americans - and so you would structure your paragraphs around these three groups of targeted audiences). However, as you said today, we should perhaps aim to split our paragraphs into ideas and concepts raised. My question is, could you explain your way of splitting it into ideas and concepts a bit further, and would you recommend using one or the other, or can you use a mix of the two? If so, how would you go about doing this?

Awesome lecture today by the way!

Much thanks! :)

Now that the lecture is over, I'm in the process of compiling a full guide to the whole 'key players' thing, since I felt like a raced through it and I know some people will be totally unfamiliar with those concepts. It'll be posted here when ready :)

Stakeholders don't have to be a person - it's just something the author has an opinion on.
I think the issue many people have with this kind of terminology is that every teacher/ school has their own definitions. So what Zezima describes as stakeholders, I would call non-abstracted key players. Whereas the 'stakeholders' I learned about were the people with vested interests in the issue; <-- an essentially useless label because there's not always multiple persons or groups involved.
For example, this is how you would use the stakeholder approach would work.
Body paragraph one: American society (analyse everything said about this)
Body paragraph two: Racism; repeat above
Body paragraph three: Racist people; repeat above.
This would be a variation on the example I gave in the lecture, except I tended to expand the players to incorporate part of the contention (kind of like what you were doing with tone)
Eg.
BP1: The way American society should strive to be more inclusive
BP2: The damaging effects of racism
BP3: How racist people are extremely misguided
Or some such variation of the above. The exact focus would be up to you; the assessor's don't have a set list of right/wrong breakdowns, it's just about what suits your writing style, and what helps you give a full sense of the piece(s).

I guess it comes down to the semantics of what an 'argument' is; <-- a very interesting discussion that would be best to ignore for the sake of not over-complicating VCE English :P

So like I said yesterday, if you have a system that works for you, stick with it!
The reason I recommend having longer, more expansive ideas is because if you're a student who's been structuring by techniques, or barely considering structure at all, jumping straight into dividing arguments and ideas can be daunting, so that contention provides more focus. I've also found it way simpler for the more difficult articles where there's either only one real 'stakeholder' (a la 2011 exam) or when there are so many stakeholders that grouping them becomes unrealistic.

But I made a deliberate effort to explore other structures as well, since provided you're aware of the potential drawbacks in structuring by tone and/or argument, you'll be thinking on your toes and they shouldn't be too much of a concern.

Would reading books improve my english skills? And is it a good idea to read other books in year 12 besides our school ones? Thanks
Without a doubt, and good god yes.
Title: Re: VCE English Question Thread
Post by: cosine on January 23, 2015, 11:08:38 am
Hey lauren,

I am currently reading my book again and composing quotes and important things to note, is this a good idea?

Title: Re: VCE English Question Thread
Post by: literally lauren on January 23, 2015, 11:15:17 am
Hey lauren,

I am currently reading my book again and composing quotes and important things to note, is this a good idea?
Definitely, but you'll be adding to your quote bank throughout the year, so don't feel like you have to pick up on everything on the first/ second read-through. In fact, you kind of need to experience running into a prompt you can't handle in order to discover new themes/ Views&Values messages, so that you can then return to the text and look for how these themes or V&Vs would be present.
Start on the major stuff now, but keep it open as a work in progres (at least until you choose which text you'll write on for the exam.)
Title: Re: VCE English Question Thread
Post by: cosine on January 23, 2015, 11:23:01 am
Definitely, but you'll be adding to your quote bank throughout the year, so don't feel like you have to pick up on everything on the first/ second read-through. In fact, you kind of need to experience running into a prompt you can't handle in order to discover new themes/ Views&Values messages, so that you can then return to the text and look for how these themes or V&Vs would be present.
Start on the major stuff now, but keep it open as a work in progres (at least until you choose which text you'll write on for the exam.)

Alright thank you!!
Title: Re: VCE English Question Thread
Post by: g1mp1e on January 23, 2015, 02:00:49 pm
Now that the lecture is over, I'm in the process of compiling a full guide to the whole 'key players' thing, since I felt like a raced through it and I know some people will be totally unfamiliar with those concepts. It'll be posted here when ready :)
I think the issue many people have with this kind of terminology is that every teacher/ school has their own definitions. So what Zezima describes as stakeholders, I would call non-abstracted key players. Whereas the 'stakeholders' I learned about were the people with vested interests in the issue; <-- an essentially useless label because there's not always multiple persons or groups involved. This would be a variation on the example I gave in the lecture, except I tended to expand the players to incorporate part of the contention (kind of like what you were doing with tone)
Eg.
BP1: The way American society should strive to be more inclusive
BP2: The damaging effects of racism
BP3: How racist people are extremely misguided
Or some such variation of the above. The exact focus would be up to you; the assessor's don't have a set list of right/wrong breakdowns, it's just about what suits your writing style, and what helps you give a full sense of the piece(s).

I guess it comes down to the semantics of what an 'argument' is; <-- a very interesting discussion that would be best to ignore for the sake of not over-complicating VCE English :P

So like I said yesterday, if you have a system that works for you, stick with it!
The reason I recommend having longer, more expansive ideas is because if you're a student who's been structuring by techniques, or barely considering structure at all, jumping straight into dividing arguments and ideas can be daunting, so that contention provides more focus. I've also found it way simpler for the more difficult articles where there's either only one real 'stakeholder' (a la 2011 exam) or when there are so many stakeholders that grouping them becomes unrealistic.

But I made a deliberate effort to explore other structures as well, since provided you're aware of the potential drawbacks in structuring by tone and/or argument, you'll be thinking on your toes and they shouldn't be too much of a concern.
Without a doubt, and good god yes.

Alright cool, I'll be sure to try to use both then :P
Thanks Zezima and Lauren for the help!
Just another question regarding the year 12 oral, I am just looking for some ways to start off an oral? Lauren, you said in the lecture yesterday that we perhaps could use an anecdote, but are there any other interesting ways?
I quite like the idea of cutting straight to a news story/real life scenario, but looking for others.
E.g. I'm doing the issue of capital punishment.

Thanks again!
Title: Re: VCE English Question Thread
Post by: brenden on January 25, 2015, 03:01:09 pm
Alright cool, I'll be sure to try to use both then :P
Thanks Zezima and Lauren for the help!
Just another question regarding the year 12 oral, I am just looking for some ways to start off an oral? Lauren, you said in the lecture yesterday that we perhaps could use an anecdote, but are there any other interesting ways?
I quite like the idea of cutting straight to a news story/real life scenario, but looking for others.
E.g. I'm doing the issue of capital punishment.

Thanks again!
Recite what would be said to the prisoner pre-execution. "eg... "Applying sponge. Attaching electrodes. Do you have any last words?" (obviously not the words lol... not even sure if there's a procedure for it), and then be like "YO FOOLS WOULD YOU WANT THIS SAID TO YOU I DON'T THINK SO".
Title: Re: VCE English Question Thread
Post by: M_BONG on January 25, 2015, 03:54:40 pm
Recite what would be said to the prisoner pre-execution. "eg... "Applying sponge. Attaching electrodes. Do you have any last words?" (obviously not the words lol... not even sure if there's a procedure for it), and then be like "YO FOOLS WOULD YOU WANT THIS SAID TO YOU I DON'T THINK SO".
Bahahahha I like how you automatically assume that s/he would be against capital punishment ;)
Title: Re: VCE English Question Thread
Post by: Splash-Tackle-Flail on January 25, 2015, 04:08:29 pm
Hello everyone,

For English, my school is doing a new text for Unit 3: "I for Isobel". I have read it-  and imo it was painful to read :( but I did it in the end. However now I've read it once I really don't know what to do- and while there are some study guides on the internet, most of them are paid and/or aren't catered to the VCE course. What do you think I should do for this text? Especially since I'm quite set on doing war poems for the exam, but I still have to do a sac on I for isobel -.- .

Edit: Just read OP, and realised this probably relates to text specific advice (but could be extended to all new texts?) Sorry! Would it be possible to move this post to a new thread somehow?
Title: Re: VCE English Question Thread
Post by: literally lauren on January 26, 2015, 10:03:08 pm
Just another question regarding the year 12 oral, I am just looking for some ways to start off an oral? Lauren, you said in the lecture yesterday that we perhaps could use an anecdote, but are there any other interesting ways?
I quite like the idea of cutting straight to a news story/real life scenario, but looking for others.
E.g. I'm doing the issue of capital punishment.
Depends what you want the audience to feel. First, try to set up your issue in a more complex way; 'doing the issue of capital punishment' doesn't give you much direction. See if you can construct a contention that's more sophisticated than just 'CP = good/bad.' After that you can start to consider how you want to persuade their audience, eg. will your primary focus be on injustice, thereby appealing to the audience's trust (or lack thereof) in the legal system, or would you focus on the emotional/moral side? etc.
Anecdotes don't always have to be in the form of third person story telling either. You could use either the first or second person to make the issue more confronting, using real or hypothetical scenarios.
Another thing to consider is what you want the audience's response to be. This is slightly different from how you want them to feel, as the latter has more to do with attitudes and beliefs, whereas the response should be in the form of a behaviour or action. You don't have to go into a lot of depth telling people to write letters to the government or anything, but you couldn't have a persuasive speech that just rants about how good or bad capital punishment is. You have to do something with that momentum
eg.
1. Capital punishment is awful. Around the world, many people are killed for crimes they didn't commit, and this is unfair.
2. Capital punishment is awful and reflects poorly on society. Australia should not even be considering the prospect of bringing it back due to all the pain and injustice associated with it.
^Reductive arguments, but see how the second is more effective given its direction? This kind of bridges back around to having a strong contention, not just an interesting issue.
The main thing is that your intro is engaging, and relates to your arguments in a way that persuades the audience. Other than that, you have pretty much free creative license.

Hello everyone,

For English, my school is doing a new text for Unit 3: "I for Isobel". I have read it-  and imo it was painful to read :( but I did it in the end. However now I've read it once I really don't know what to do- and while there are some study guides on the internet, most of them are paid and/or aren't catered to the VCE course. What do you think I should do for this text? Especially since I'm quite set on doing war poems for the exam, but I still have to do a sac on I for isobel -.- .

Edit: Just read OP, and realised this probably relates to text specific advice (but could be extended to all new texts?) Sorry! Would it be possible to move this post to a new thread somehow?
IFI was on the English list ages ago (~2002 I think) and since the format for Text Response back in the day was ostensibly the same, you might be able to find some study guides or online materials around.
You don't have to rely on these though (see previous recommendations in response to cosine's questions.) Developing your own resources will probably be more helpful in the long run anyway.
If you're already fairly set on your Semester 2 text instead, then use your first text as an experimentation of note-taking and study techniques. Obviously the differences between a novel and a poetry collection (or a play, film, or other text for that matter) will necessitate a slightly different approach, but you can still use Semester 1 as an experience in fine-tuning the way you deal with the text, from the initial readings to the last-minute SAC preparation.
Work out whether you prefer spending several weeks analysing and dissecting the text before attempting full essays, or whether you like writing pieces alongside your study as you learn more and more. And you'll also be able to fine any general weak spots when it comes to Text Response :)
Title: Re: VCE English Question Thread
Post by: twinkling star on January 27, 2015, 12:02:11 am
I just wished to check this- is there a difference between how you write a text response for a Unit 3 SAC compared to a Unit 4 SAC?

'Cos the performance descriptor for the Unit 3 text response says 'Analyse, either orally or in writing, how a selected text constructs meaning, conveys ideas and values, and is open to a range of interpretations'

...whereas the performance descriptor for the Unit 4 text response says 'Develop and justify a detailed interpretation of a selected text'

And if there is a difference between the responses required for each unit, would you write in the Unit 3 or the Unit 4 style for the exam's section 1?

Thanks!  :)
Title: Re: VCE English Question Thread
Post by: knightrider on January 27, 2015, 12:18:24 am
When mentioned in criteria or questions what  "social, cultural and historical values are evident".

What does this actually mean.

what do they want us to state or expect us to include in our writing.
Title: Re: VCE English Question Thread
Post by: literally lauren on January 27, 2015, 10:04:53 am
I just wished to check this- is there a difference between how you write a text response for a Unit 3 SAC compared to a Unit 4 SAC?

'Cos the performance descriptor for the Unit 3 text response says 'Analyse, either orally or in writing, how a selected text constructs meaning, conveys ideas and values, and is open to a range of interpretations'

...whereas the performance descriptor for the Unit 4 text response says 'Develop and justify a detailed interpretation of a selected text'

And if there is a difference between the responses required for each unit, would you write in the Unit 3 or the Unit 4 style for the exam's section 1?

Thanks!  :)
Short answer: No, the Semester 1 and 2 Text Responses are the same, just on different texts.
Long answer: No, because the Study Design is stupidly written and offers schools a bunch of choices none of them take. Technically speaking, any one of the tasks you do in Semester 1 can be in oral form, as the Design requires 1/4 (that is, a L.A. T.R. Context piece, and persuasive/POV) except every single school I know choses to make the persuasive piece the oral task.
You're also technically allowed to bring in more resources for the Unit 3 text, though most teachers discourage this because they're trying to make the SACs throughout the year as similar as possible to the exam task unlike some other subjects *coughLiteraturecough*

This is kind of bridging into knightrider's question:
When mentioned in criteria or questions what  "social, cultural and historical values are evident".

What does this actually mean.

what do they want us to state or expect us to include in our writing.
Every T.R. text is chosen for, amongst other things, its ability to relate to a social or historical context. This might be in the time the text is set (eg. This Boy's Life, Stasiland) or the time the author is writing (eg. Cloudstreet, Brooklyn) or both (eg. Henry IV, White Tiger.) Most texts still support both discussions, so what the assessors are looking for is an awareness of how certain values can be seen or presented in the text. You can look at this from either the author or audience's perspective. Not every argument lends itself to this kind of discussion, and you shouldn't base your entire contention on a wider message, but the socio-historical stuff makes for good 'zoomed out' discussion. It's usually pretty hard to form a proper interpretation of the text without an understanding of what the author is trying to say overall; just don't rely too heavily on it in your essays.

Pro tip: If you're integrating some background information, weave it into a sentence with textual discussion
Rather than: 'The idea of body image is something that has plagued young people throughout modern age. The Very Hungry Caterpillar critiques the idea that self-worth should be derived solely from one's external appearance.'
Transition more fluidly: 'The Very Hungry Caterpillar critiques the idea that self-worth should be derived solely from one's external appearance, thereby forming part of Carle's commentary on the wealth of body dysmorphic problems in the modern age.'
In that first example, you risk not getting credit for the first sentence because it doesn't relate to the text; you're just commenting on the context. In the second, however; you can almost trick the assessor into seeing the connection just by combining the sentences.

So in terms of the actual range of interpretations part:
The words whilst, although, despite, and whereas are your greatest allies. You're not expected to spend a great deal of time on alternate interpretations, but an easy way of doing it is to say something like:
'Whilst the caterpillar's journey could be viewed as one of blissful ignorance, there is an underlying sense of self-realisation and enlightenment in the text.'
I've just drawn a distinction between my contention (self-realisation is important to the caterpillar's journey) and an alternate interpretation (the caterpillar is blissfully ignorant.)
Words like 'although,' 'whilst' etc. force you to add that extra clause as a way of challenging interpretations, so include them every so often and you should be fine for this category.

Of course, there's every possibility that your teacher will have a totally different 'interpretation' of the criteria ;) so check with them in case there are any bizarre requirements they want you to fulfill.
Title: Re: VCE English Question Thread
Post by: knightrider on January 27, 2015, 12:42:36 pm


This is kind of bridging into knightrider's question:Every T.R. text is chosen for, amongst other things, its ability to relate to a social or historical context. This might be in the time the text is set (eg. This Boy's Life, Stasiland) or the time the author is writing (eg. Cloudstreet, Brooklyn) or both (eg. Henry IV, White Tiger.) Most texts still support both discussions, so what the assessors are looking for is an awareness of how certain values can be seen or presented in the text. You can look at this from either the author or audience's perspective. Not every argument lends itself to this kind of discussion, and you shouldn't base your entire contention on a wider message, but the socio-historical stuff makes for good 'zoomed out' discussion. It's usually pretty hard to form a proper interpretation of the text without an understanding of what the author is trying to say overall; just don't rely too heavily on it in your essays.

Pro tip: If you're integrating some background information, weave it into a sentence with textual discussion
Rather than: 'The idea of body image is something that has plagued young people throughout modern age. The Very Hungry Caterpillar critiques the idea that self-worth should be derived solely from one's external appearance.'
Transition more fluidly: 'The Very Hungry Caterpillar critiques the idea that self-worth should be derived solely from one's external appearance, thereby forming part of Carle's commentary on the wealth of body dysmorphic problems in the modern age.'
In that first example, you risk not getting credit for the first sentence because it doesn't relate to the text; you're just commenting on the context. In the second, however; you can almost trick the assessor into seeing the connection just by combining the sentences.

So in terms of the actual range of interpretations part:
The words whilst, although, despite, and whereas are your greatest allies. You're not expected to spend a great deal of time on alternate interpretations, but an easy way of doing it is to say something like:
'Whilst the caterpillar's journey could be viewed as one of blissful ignorance, there is an underlying sense of self-realisation and enlightenment in the text.'
I've just drawn a distinction between my contention (self-realisation is important to the caterpillar's journey) and an alternate interpretation (the caterpillar is blissfully ignorant.)
Words like 'although,' 'whilst' etc. force you to add that extra clause as a way of challenging interpretations, so include them every so often and you should be fine for this category.

Of course, there's every possibility that your teacher will have a totally different 'interpretation' of the criteria ;) so check with them in case there are any bizarre requirements they want you to fulfill.

Thank you so much Lauren  :)
i was just wondering in terms of the response what criteria do the VCAA examiners usually stick to as they would have to be consistent with there marking.
Title: Re: VCE English Question Thread
Post by: literally lauren on January 27, 2015, 12:57:07 pm
Thank you so much Lauren  :)
i was just wondering in terms of the response what criteria do the VCAA examiners usually stick to as they would have to be consistent with there marking.
The most important be-all-and-end-all ultimately majorly critical 'do this or the English Gods will slap you' criteria is relevance. So if the socio-historical context is relevant to your discussion, then you can use it. However, if you're tying it in where it doesn't belong, especially if you're only doing it to ratchet up points for sounding sophisticated, the assessors are very likely to not only notice, but potentially penalise you.
Though you won't often lose marks directly for writing something of tangential relevance, you are indirectly missing an opportunity to gain marks in other areas. It also makes a bad impression on assessors, whose biggest pet peeve seems to be rote-learning formulaic responses. (No joke, every Assessor's Report since 2001 has bemoaned this approach!)

Only once relevance is assured can you get credit for the quality of ideas and writing. I'm generalising here, since I know there are assessors who find it acceptable to give credit for exploration and expression, even if what you're exploring and expressing isn't on the right track, but the most helpful way to approach the task is by prioritising relevance, and letting your discussion reflect that priority. :)
Title: Re: VCE English Question Thread
Post by: knightrider on January 27, 2015, 01:20:09 pm
The most important be-all-and-end-all ultimately majorly critical 'do this or the English Gods will slap you' criteria is relevance. So if the socio-historical context is relevant to your discussion, then you can use it. However, if you're tying it in where it doesn't belong, especially if you're only doing it to ratchet up points for sounding sophisticated, the assessors are very likely to not only notice, but potentially penalise you.
Though you won't often lose marks directly for writing something of tangential relevance, you are indirectly missing an opportunity to gain marks in other areas. It also makes a bad impression on assessors, whose biggest pet peeve seems to be rote-learning formulaic responses. (No joke, every Assessor's Report since 2001 has bemoaned this approach!)

Only once relevance is assured can you get credit for the quality of ideas and writing. I'm generalising here, since I know there are assessors who find it acceptable to give credit for exploration and expression, even if what you're exploring and expressing isn't on the right track, but the most helpful way to approach the task is by prioritising relevance, and letting your discussion reflect that priority. :)

Thank you so much Lauren  :)
really appreciate your time and effort into each response keep up your awesome work!! :) :D
Title: Re: VCE English Question Thread
Post by: Cogglesnatch Cuttlefish on January 28, 2015, 12:53:04 pm
How's religion in context essays? For example, the prompt is "conflict is inevitable" and I adopt the view that God creates conflict and some of the assessors may not necessarily believe in God. Are my chances of getting a high mark smitten as a result?
Title: Re: VCE English Question Thread
Post by: literally lauren on January 28, 2015, 12:59:54 pm
How's religion in context essays? For example, the prompt is "conflict is inevitable" and I adopt the view that God creates conflict and some of the assessors may not necessarily believe in God. Are my chances of getting a high mark smitten as a result?

If you're using that premise as the basis of your contention then you might have an issue, but if you're using the concept of religious faith as an example to demonstrate a point, then that should be okay.

(Assuming you're writing an expository/ expos-hybrid piece, you should be using other evidence anyway?)

I actually think it could be really interesting to look at the way different religions resolve the whole 'God is good, but bad stuff happens' dilemma, especially for the prompt you're writing on. I can't speak for your classroom teacher, but I know the end of year assessors are told not to let personal biases when it comes to political/ social/ religious issues skew their marking. Provided your writing is logical and interesting, I can't image them having a problem with it.
Title: Re: VCE English Question Thread
Post by: Cogglesnatch Cuttlefish on January 28, 2015, 01:15:53 pm
If you're using that premise as the basis of your contention then you might have an issue, but if you're using the concept of religious faith as an example to demonstrate a point, then that should be okay.

(Assuming you're writing an expository/ expos-hybrid piece, you should be using other evidence anyway?)

I actually think it could be really interesting to look at the way different religions resolve the whole 'God is good, but bad stuff happens' dilemma, especially for the prompt you're writing on. I can't speak for your classroom teacher, but I know the end of year assessors are told not to let personal biases when it comes to political/ social/ religious issues skew their marking. Provided your writing is logical and interesting, I can't image them having a problem with it.
Exactly what I was looking for. Thanks Lauren :D
Title: Re: VCE English Question Thread
Post by: shivaji on January 29, 2015, 03:07:11 pm
Hey Lauren,

Just wondering when analysing an editorial, how would you reference the author of it? I constantly use "the writer ...", are there any other better phrases which can be utilised?

Thanks! :D
Title: Re: VCE English Question Thread
Post by: maddihanna on February 01, 2015, 10:19:20 am
Hi all :)

Just a quick question about a prompt/structuring my response.
The prompt I have been given for my text response essay is one that I quite heavily disagree with and there are many pieces of evidence to prove why I do. I know I shouldn't completely favour my essay to one side, so I was just wondering if its enough to mention the 'affirmative' side only in my introduction?

The prompt is a "to what extent do you agree?" type.

Thanks in advance :)
Title: Re: VCE English Question Thread
Post by: paper-back on February 01, 2015, 10:36:05 am
During your second reading how do you what quotes you should highlight i.e. quotes of significance?
Title: Re: VCE English Question Thread
Post by: literally lauren on February 01, 2015, 11:32:49 am
Hey Lauren,

Just wondering when analysing an editorial, how would you reference the author of it? I constantly use "the writer ...", are there any other better phrases which can be utilised?

Thanks! :D
You can refer to him/her as 'the editor' as well. Technically you can even use the newspaper's title, eg. 'The Herald Sun contends that...' or just 'The article/editorial...'
Otherwise, just try to vary your sentence structure so that you're not repeating words too often. Something like 'the writer' isn't a big deal though, since you're expected to tie all your points back to how they're using language to persuade anyway :)

Hi all :)

Just a quick question about a prompt/structuring my response.
The prompt I have been given for my text response essay is one that I quite heavily disagree with and there are many pieces of evidence to prove why I do. I know I shouldn't completely favour my essay to one side, so I was just wondering if its enough to mention the 'affirmative' side only in my introduction?

The prompt is a "to what extent do you agree?" type.

Thanks in advance :)
It's perfectly okay to disagree, but your contention can't simply be 'yes' or 'no.' Ask yourself why you disagree, and try to come up with a more complex argument so that your essay doesn't sound one-sided. Not every Text Response has to deal with both sides (and you don't want to sound non-committal) so perhaps address the alternate interpretations but round things back to your overall contention.
eg. 'Though one could argue character X's transformation is the result of purely selfish intent, there is an undeniable compassion in his behaviour that contradicts this interpretation...'

Also, there's no real difference between a 'to what extent' or 'discuss' or 'do you agree' question. VCAA just vary the sentence structures sometimes :)

During your second reading how do you what quotes you should highlight i.e. quotes of significance?

It's impossible to get all the useful quotes in one go, so start with the simpler themes and major characters. As you study the text in class, you'll uncover more and more facets of the text. You might even find a prompt that deals with an idea or theme you haven't considered before; that's when you go back to the text and  try to find evidence (quotes) to support an interpretation.
Quote banks should always be a work in progress, so trust your instincts at the start and then see what you need to add or change later down the track.
Title: Re: VCE English Question Thread
Post by: RazzMeTazz on February 01, 2015, 09:21:36 pm
For the English Persuasive Oral SAC is it necessary to include a rebuttal paragraph?

Also what hand gestures would be recommended during the speech? I usually feel really awkward and I feel like my hands don't have anything to do during a speech (Especially a memorised one, because there are no cue cards to hold!)

Thanks in advance for any of the replies :)
Title: Re: VCE English Question Thread
Post by: M_BONG on February 01, 2015, 11:43:47 pm
For the English Persuasive Oral SAC is it necessary to include a rebuttal paragraph?

Also what hand gestures would be recommended during the speech? I usually feel really awkward and I feel like my hands don't have anything to do during a speech (Especially a memorised one, because there are no cue cards to hold!)

Thanks in advance for any of the replies :)
Nope, the beauty of persuasive speeches is that you don't need to be balanced. You can be as biased as you want. But, depending on what you are arguing and your target audience(s), you may want to address some of the points that the "other side" or your opponents are espousing to sound moderate (again, optional).

In terms of hand gestures, I always have a rule to do whatever makes you comfortable. For example, I *don't* force myself to have eye contact, hand gestures or colourful pitch - I try to do it naturally (within your comfort zone) and that is the way to make your delivery smooth and not robotic. Of course, if you are normally doing awkward things like shuffling around or bending your knees etc, you should try to correct it.

... Which leads on really well to my golden oral tip: TREAT IT LIKE A CONVERSATION - speak like you would speak normally.The BEST way to argue something is not to sound robotic, overly flamboyant or aggressive. Most people don't respond well to these two. The best orators are ones who try to bring themselves to your own level and speak to you as a fellow human being - not a punching back or a sponge absorbing random information.



Title: Re: VCE English Question Thread
Post by: Cogglesnatch Cuttlefish on February 02, 2015, 08:30:58 pm
Is it right to be commending people/institutes referred to in context pieces(expository mainly)? E.g "As stated by the renowned Abraham Maslow..." or "... draws parallels to the arduous energy expended by the late Martin Luther King Jr."
Title: Re: VCE English Question Thread
Post by: literally lauren on February 03, 2015, 12:07:31 am
Is it right to be commending people/institutes referred to in context pieces(expository mainly)? E.g "As stated by the renowned Abraham Maslow..." or "... draws parallels to the arduous energy expended by the late Martin Luther King Jr."
For context you can commend, critique or censure as much as you like. Just don't do anything like it in a Text Response essay, as it can sound sycophantic and isn't worth anything.
Topic sentences like '>Author's< novel '___' is a hauntingly beautiful evocation of the human condition...' just make assessors' eyes roll :)
Title: Re: VCE English Question Thread
Post by: brenden on February 03, 2015, 12:14:22 am
For context you can commend, critique or censure as much as you like. Just don't do anything like it in a Text Response essay, as it can sound sycophantic and isn't worth anything.
Topic sentences like '>Author's< novel '___' is a hauntingly beautiful evocation of the human condition...' just make assessors' eyes roll :)
B-b-but... I thought mentioning the human condition was an instant guarantee for full marks..?!
Title: Re: VCE English Question Thread
Post by: literally lauren on February 03, 2015, 12:19:56 am
B-b-but... I thought mentioning the human condition was an instant guarantee for full marks..?!

Ssssshhhhhh!!!!! We can't give away the secret!!


Nekminnit in this year's assessor's report: "An alarming number of students simply wrote the words 'human condition,' and nothing else on every page in their exam booklet, thereby royally screwing over our bell curves. VCAA is investigating the matter."
Title: Re: VCE English Question Thread
Post by: Callum@1373 on February 04, 2015, 03:04:29 pm
Hi Lauren  :D

I have a english context speech on whose reality and the prompt that we are doing is 'Our reality is influenced by place and time'. The book we are studying is The Shark Net. I don't know how to go about it, because of the word 'our'.

What I mean is i can think of numerous examples where Robert's reality is influenced by place and time, but what am i meant to do seeing as it is not ' A reality can be influenced' instead 'OUR reality...'. Does that mean that I have to talk about everyone, 100 people to show its 'our', 3 people to show its 'our' or can I just simply refer to a couple examples with different people.

I can see what most of my year is going to do - something along the lines of 'Robert lived in 'orderly' melbourne and then when he got to perth he thought it was isolated because his reality of what a place to live in was influenced by his place in melbourne.' I want to do something different and explore it a bit deeper  ;D 8)

Thanks for any help you can give  :) :) :)
Title: Re: VCE English Question Thread
Post by: cosine on February 05, 2015, 07:17:35 am
Hey

I got my sac date for my first english essay. Its on the 3rd of march, when would be suitable for me to actually start writing practice essays, better more what else should i do to REALLY prepare for this date? The text im reading is a play called 'Medea'. Thank you
Title: Re: VCE English Question Thread
Post by: StressedAlready on February 05, 2015, 08:14:04 pm
Okay so I am freaking out. Can you totally 100% confirm that 3&4 English in 2016 will still follow the old study design?! I seriously had a panic attack over this because I graduate next year and I'm just a little - okay, a lot - stressed about this change.
Title: Re: VCE English Question Thread
Post by: literally lauren on February 05, 2015, 09:46:30 pm
Hi Lauren  :D

I have a english context speech on whose reality and the prompt that we are doing is 'Our reality is influenced by place and time'. The book we are studying is The Shark Net. I don't know how to go about it, because of the word 'our'.

What I mean is i can think of numerous examples where Robert's reality is influenced by place and time, but what am i meant to do seeing as it is not ' A reality can be influenced' instead 'OUR reality...'. Does that mean that I have to talk about everyone, 100 people to show its 'our', 3 people to show its 'our' or can I just simply refer to a couple examples with different people.

I can see what most of my year is going to do - something along the lines of 'Robert lived in 'orderly' melbourne and then when he got to perth he thought it was isolated because his reality of what a place to live in was influenced by his place in melbourne.' I want to do something different and explore it a bit deeper  ;D 8)

Thanks for any help you can give  :) :) :)
You're absolutely right to want to challenge the prompt, and no, the evidence doesn't have to be the sole factor in your arguments. Obviously you need to do a substantial amount of exploration, but there's no requirement to 'prove' in English; you just need to suggest.
Some earlier posts contain information about questioning prompts (check the Context links on the first post in this thread) but for the example you've given, you can definitely consider the difference between our reality (ie. the objective one we all exist in) and our own personal realities.

Hey

I got my sac date for my first english essay. Its on the 3rd of march, when would be suitable for me to actually start writing practice essays, better more what else should i do to REALLY prepare for this date? The text im reading is a play called 'Medea'. Thank you
18th of Feb. 2:33 pm. Beginning practice essays at this exact time will guarantee full marks.
..I'm kidding of course. It totally depends on whether you prefer to read and research and think heaps before putting pen to paper, or whether you're the kind of learner who needs to practice articulating knowledge as soon as you acquire it. Remember, you can always write practice paragraphs in the meantime in order to consolidate your understanding of certain characters and themes. Writing full essays isn't the only way to learn, and you should be looking to improve your knowledge of the text as well as how you approach the Text Response criteria.

Okay so I am freaking out. Can you totally 100% confirm that 3&4 English in 2016 will still follow the old study design?! I seriously had a panic attack over this because I graduate next year and I'm just a little - okay, a lot - stressed about this change.
I can 100% confirm that the VCAA website states, in annoying equivocal jargon, that Year 12s in 2016 will be doing the current study design. There's really no point stressing over something like this anyway; 2/3 of the essay types are remaining the same, and it's looking like the third will only undergo minor changes. The marking scheme and a few odd SACs are the most significant alterations, unless you're doing EAL in which case there'll be a listening component like other SL subjects.
Tbh my sneaking suspicion is that this is a step towards a nationalised curriculum, and VCAA are just making minor changes here and there in preparation for a revamp in ~2021 or whenever.
Worst case scenario and VCAA actually implement the new design next year, everyone will be in the same boat, and the schools will guide you into things, so there's no sense worrying :)
Title: Re: VCE English Question Thread
Post by: Callum@1373 on February 06, 2015, 06:42:53 am
Thanks Lauren, I'll use that in my research  ;D
Title: Re: VCE English Question Thread
Post by: StressedAlready on February 06, 2015, 06:08:08 pm
Okay. I feel slightly better now. Panic attack over.

Because apparently some kids at my school were being told that they were being implemented next year by their teachers and I'd been told that they weren't and I was freaking out and gahhhhh. I feel slightly better now knowing everyone shall be in the same boat come 2016 English exam.

Worst case scenario, the whole state does horribly in English next year because they're ill prepared and the assessors feel bad and give us all a 50 SS because they screwed up the implementation.

Okay, so it's a little bit of wishful thinking on my part...
Title: Re: VCE English Question Thread
Post by: Talia2144 on February 06, 2015, 06:44:09 pm
Hi, can anyone please tell me if know any English tutorial class for year 11 student or where I can find one. Thank you
Title: Re: VCE English Question Thread
Post by: 2jzgte on February 07, 2015, 04:22:22 am
Hey

I got my sac date for my first english essay. Its on the 3rd of march, when would be suitable for me to actually start writing practice essays, better more what else should i do to REALLY prepare for this date? The text im reading is a play called 'Medea'. Thank you
Got my first essay on the same day, thinking of filling out character profiles to have a really in depth understanding of them and put together a few practice paragraphs. Thats all I've got in mind other than full practice essays based on prompts. Does anyone have any idea what aspects of the play they would want us to explore through the essay?
Title: Re: VCE English Question Thread
Post by: literally lauren on February 07, 2015, 03:08:09 pm
Okay. I feel slightly better now. Panic attack over.

Because apparently some kids at my school were being told that they were being implemented next year by their teachers and I'd been told that they weren't and I was freaking out and gahhhhh. I feel slightly better now knowing everyone shall be in the same boat come 2016 English exam.

Worst case scenario, the whole state does horribly in English next year because they're ill prepared and the assessors feel bad and give us all a 50 SS because they screwed up the implementation.

Okay, so it's a little bit of wishful thinking on my part...
Technically speaking, the new study design begins in 2016 because that's when the Year 11s (ie. class of 2017, currently year 10s) will start studying it. But if you're in year 11 now you have nothing to worry about.

Hi, can anyone please tell me if know any English tutorial class for year 11 student or where I can find one. Thank you
You can search the tutoring section of Atar Notes if you're looking for a one-on-one tutor, but I'm afraid I can't vouch for any of the particular groups (ie. commercially organised programs.)  You might be able to find an answer in some of the tutoring businesses thread.

Got my first essay on the same day, thinking of filling out character profiles to have a really in depth understanding of them and put together a few practice paragraphs. Thats all I've got in mind other than full practice essays based on prompts. Does anyone have any idea what aspects of the play they would want us to explore through the essay?
Sounds like a good plan for such a character-driven play. Just make sure you don't neglect the themes and values in the text, as going from character profiles to full essays can be a bit of a jump.
And unfortunately, there's no telling what your SAC might be on because they're all controlled internally, meaning every school (and occasionally every teacher) gets to choose their own focus. However, since it's in every teacher's best interests for their students to do well, they'll most likely choose/write a prompt relatively close to the areas you've been studying. So if you notice your teacher is bringing a lot of the discussion back to the idea of family an betrayal, for instance; don't be suprised if the prompt you get is 'Discuss the relationship between family and betrayal' :P
Title: Re: VCE English Question Thread
Post by: cosine on February 07, 2015, 05:56:46 pm
Would it be of much use if i posted my paragraph/short answer here for someone to edit for me and lend me some helpful advice, or not because the play i am doing (Medea by Euripides) you all may not have read? Thanks
Title: Re: VCE English Question Thread
Post by: literally lauren on February 07, 2015, 05:59:00 pm
Would it be of much use if i posted my paragraph/short answer here for someone to edit for me and lend me some helpful advice, or not because the play i am doing (Medea by Euripides) you all may not have read? Thanks

The English Work Submission and Marking board is the place to post your work if you want feedback :)
Title: Re: VCE English Question Thread
Post by: knightrider on February 08, 2015, 09:04:30 am
Hey Lauren.

How did you memorise your quotes for your sacs?

Also how did you group your quotes like how did you split them up?

What categories do you think are good ideas to be splitting your quotes into.Like themes etc.

Thanks  :)
Title: Re: VCE English Question Thread
Post by: literally lauren on February 08, 2015, 11:00:58 am
Hey Lauren.

How did you memorise your quotes for your sacs?

Also how did you group your quotes like how did you split them up?

What categories do you think are good ideas to be splitting your quotes into.Like themes etc.

Thanks  :)
Definitely split the quotes into categories. It'll depend on which text you're studying, as you could either go by characters (quotes said by, about, or to them) or by themes/ ideas.

My recommendations:
All About Eve - by characters
Burial Rights - by themes
Brooklyn - by themes
Cat's Eye - either (character maps would be helpful early on, but themes are better in the long run)
Cloudstreet - by themes
Henry IV Part 1 - either (I found themes worked better, even though there's an easy split between the four main characters)
I for Isobel - by themes
In the Country of Men - by themes
Mabo - by themes
Medea - by characters
No Sugar - by characters
Selected Harwood Poems - by theme (by poem is a good start, but breaking things into themes will definitely help for essays)
Stasiland - either
The Complete Maus - by themes
The Thing Around Your Neck - by themes
The War Poems - by theme (ditto with Harwood^, sort by poems at first, but aim for themes in the long run)
The White Tiger - by characters
This Boy's Life - by characters
Will You Please Be Quiet Please - by themes
Wuthering Heights - either, characters is probably easiest, but not conclusive

Regardless of what you're studying though, your first step should just be to go through the text chronologically and find as much as you can. Then, start grouping them into categories, but be flexible. Some will fit in multiple areas, so it's up to you whether you duplicate, or maybe just annotate the quotes so you remember.
A good quote bank will be a work in progress though, so keep in mind you may discover other important components of the text that you can't sufficiently cover with textual evidence. This is when you'll go back to the text and look for examples of a certain character or idea.

Basic memorisation techniques like reading, writing, and speaking the quote aloud will gradually cement them in you're mind, but the more you use them in essays, the easier it is to actually analyse and discuss the quotes, not just churn them out.

Don't panic about doing it all in one go though, especially if you're studying a novel or other dense text :)
Title: Re: VCE English Question Thread
Post by: cosine on February 08, 2015, 11:11:57 am
How can I practice for essays? People say do paragraphs, do this and that... But what do I do them on? Where am i meant to get the prompts and questions from?
Title: Re: VCE English Question Thread
Post by: literally lauren on February 08, 2015, 11:19:27 am
How can I practice for essays? People say do paragraphs, do this and that... But what do I do them on? Where am i meant to get the prompts and questions from?

HERE! Apologies if your text doesn't have a wealth of resources at the moment; I'll have to wait a few months before good resources start cropping up, or people post their own. Your school should also provide you with some sample prompts when the time is right.
But at this early stage, you could quite easily just write on characters or themes to start your thinking processes. There's no reason to worry about entire essays at this stage, and so proper VCAA-style prompts might not be the way to go, as these generally need fleshing out over multiple paragraphs. Most schools will assign basic stuff like 'Discuss the role of character X' or 'Discuss the importance of theme X' for starters, and then you can gradually build up to constructing arguments later :)
Title: Re: VCE English Question Thread
Post by: Maths Forever on February 08, 2015, 11:23:34 am
How can I practice for essays? People say do paragraphs, do this and that... But what do I do them on? Where am i meant to get the prompts and questions from?

Is this text new to the 2015 booklist for English?

If not, past 'Engage Education Foundation' (type in Google), 'VATE', 'Insight' or 'Neap' exams are great sources for topics.

Alternatively, try typing the name of your text into Google along with the word 'topics'.

Hope this helps!
Title: Re: VCE English Question Thread
Post by: pi on February 08, 2015, 11:44:32 am
FYI a guy from VCAA is talking about VCE English and is there to answer questions soon (starting within the next 20mins or so?) on ABC 774, they said a podcast will be uploaded after the session but yeah just to let y'all know :)

edit: starting now!

edt2: turned out to be a fairly general discussion with some anticlimactic question, probably not worth listening to later unfortunately

edit3: the type of music that played afterwards really makes me question why I listen to this station so much lmao
Title: Re: VCE English Question Thread
Post by: Callum@1373 on February 08, 2015, 08:46:13 pm
Hey, I notice a common theme in text response essays that are really good is that there are a lot of words to describe characters, their actions, themes etc. which I have never heard of. How do you learn to be able to find such good sophisticated words that are specific to the part of the text that you are discussing? ( I know that there is a lot more to a good TR essay apart from metalanguage, but still..)

For example, in a paragraph from someone on here i found: proclivity, amiable, pertinently, hedonistic, repartee, jovial (haha i think you get the point) :P
Title: Re: VCE English Question Thread
Post by: literally lauren on February 08, 2015, 11:05:56 pm
Hey, I notice a common theme in text response essays that are really good is that there are a lot of words to describe characters, their actions, themes etc. which I have never heard of. How do you learn to be able to find such good sophisticated words that are specific to the part of the text that you are discussing? ( I know that there is a lot more to a good TR essay apart from metalanguage, but still..)

For example, in a paragraph from someone on here i found: proclivity, amiable, pertinently, hedonistic, repartee, jovial (haha i think you get the point) :P

You could do what a lot of students do and just download & memorise a list of "sophisticated" words, but what I'd recommend is going through your own work (whether it's stuff from previous years, or pieces you write throughout this year) and replacing words you find yourself repeating, or believe to be too simple.
thesaurus.com is a wonderfully quick way of finding alternate words, and lets you browse their definitions.

Fair warning, you'll probably make a fair few mistakes as you adjust to words you may never have heard before, but the only way to improve vocab sophistication is to make these mistakes and learn from them. Some teachers can get frustrated when students deliberately use words they don't fully understand, but I'd argue it's the only way to improve.

After a point (esp. in actual SACs) you only want to be using language you're comfortable with, but challenge yourself early on, and you'll reap the rewards later :)
Title: Re: VCE English Question Thread
Post by: Callum@1373 on February 09, 2015, 06:37:06 am
You could do what a lot of students do and just download & memorise a list of "sophisticated" words, but what I'd recommend is going through your own work (whether it's stuff from previous years, or pieces you write throughout this year) and replacing words you find yourself repeating, or believe to be too simple.
thesaurus.com is a wonderfully quick way of finding alternate words, and lets you browse their definitions.

Fair warning, you'll probably make a fair few mistakes as you adjust to words you may never have heard before, but the only way to improve vocab sophistication is to make these mistakes and learn from them. Some teachers can get frustrated when students deliberately use words they don't fully understand, but I'd argue it's the only way to improve.

After a point (esp. in actual SACs) you only want to be using language you're comfortable with, but challenge yourself early on, and you'll reap the rewards later :)
Where were you when I chucked out my yr 10 english exams??? >:(
Title: Re: VCE English Question Thread
Post by: SE_JM on February 09, 2015, 01:40:52 pm
Hello Lauren,
I was jut viewing your 'power of ink' essay. It's really good! Really, really fantastic.
I was just wondering where is the original article?
I want to give it a go before referring to your example, but i can't find the article.
Any help?
Title: Re: VCE English Question Thread
Post by: literally lauren on February 09, 2015, 03:18:41 pm
Where were you when I chucked out my yr 10 english exams??? >:(
Haha, don't worry about it - you'll still have this entire year to reread and fine-tune your work. And you can do the 'alternate words' exercise with any piece of writing, be they VCE English essays, or professionally written compositions. It's just a matter of finding a starting point so your exploration isn't too aimless.
eg. 'starting point' --> 'genesis' --> 'nexus' --> 'nadir' --> 'zenith'
almost like word association, but you're challenging yourself to keep the connections going with synonyms, antonyms, or just associated words.

Hello Lauren,
I was jut viewing your 'power of ink' essay. It's really good! Really, really fantastic.
I was just wondering where is the original article?
I want to give it a go before referring to your example, but i can't find the article.
Any help?
VCAA took down the 2011 exam due to an incredibly amusing copyright violation, so there aren't any copies online like there are with previous years. I do have a scanned version from my Year 12 teacher (that has my annotations and scribbles, but better than nothing) so feel free to PM me your email address and I can link you a copy.
This offer is open to anyone, by the way... even though the 2011 exam is an awful practice exercise, it's good to know just how bad the exam could get :)
Title: Re: VCE English Question Thread
Post by: Eiffel on February 12, 2015, 03:57:59 pm
Im speaking on behalf of a friend who is doing English.

"With the oral, does doing badly/amazing affect our study score AT ALL. I mean sure it does have an affect but honestly does it mean anything? i got a 19/20 and many people in our year level got 18s-19s and couple of 20s and then those who got 15s and under, just in  terms of ranking and all that, lots of people did well including the non- academic ones who dont care (e.g. 15-16)
Title: Re: VCE English Question Thread
Post by: Maths Forever on February 12, 2015, 05:17:01 pm
Im speaking on behalf of a friend who is doing English.

"With the oral, does doing badly/amazing affect our study score AT ALL. I mean sure it does have an affect but honestly does it mean anything? i got a 19/20 and many people in our year level got 18s-19s and couple of 20s and then those who got 15s and under, just in  terms of ranking and all that, lots of people did well including the non- academic ones who dont care (e.g. 15-16)

In my school the persuasive oral presentation was worth 20 percent of the unit 3 SAC score. So 5 percent overall for the English study score. But usually the exam will determine how SACs are scaled, based on performance by the school and the individual student.
Title: Re: VCE English Question Thread
Post by: literally lauren on February 13, 2015, 09:33:11 am
Hi,
I'm currently studying stasiland, but I still don't really understand the text, even though I have read the book. I have a sac on it in 3 weeks and we've barely done anything in class (answering questions). And I have no idea how I'm able to write a good essay on it, if I don't understand the themes, and there's too many characters to remember. What can I do to gain a good understanding of the text?

Main thing to worry about is what you're not understanding, (which I realise is a hard question to answer, but necessary nonetheless.) If it's just a matter of grasping the plot and development of the many characters in a fairly long text, then you'll need to do some summative exercises, ie. chapter summaries, character maps/profiles (<-- defs do this for Stasiland. Insight have a good starter one here that you can add details to.)

Once that's under wraps you start to synergise it all, in order to understand the commonalities between the stories and begin to weave in the views and values of the text. If you're having difficulty here, start with study guides. There's plenty of info freely available on Stasiland at this point, so just googling 'Stasiland vce English resources' should be sufficient.

Start by just charting the major themes in the text, and connect each one to textual information (eg. the idea of sacrifice is linked to Herr Bohnsack, etc.) This will help your brain see the thematic concerns within the text, rather than as a separate list outside everything. When you feel confident enough in this step, you can move on to asking yourself the question 'so what?' Why has the author done this? What is she trying to say?

Piece together the views and values, and the whole text falls into place. From there, tackling essay prompts becomes a lot easier, because you'll be working from your contention down to the textual evidence that supports it, rather than listing every relevant piece of information that comes to mind, and then attempting to make sense of it in the conclusion.

To start with though, you have to go from the inside, out. Ask yourself as many questions as possible about what happens in the text and why, then move out to dealing with the significance of these events and what they imply, and finally out to what the text and the author are saying about a certain theme or character or message.

Start small - lest you be daunted by the enormity of a whole text and all its historical weight - that's my best advice :)
Title: Re: VCE English Question Thread
Post by: hwilome on February 15, 2015, 01:06:01 pm
Hi, Im a student who's doing EAL.

I have a few questions according to EAL.


1. I am finding it difficult to get the Main contention of an article. (e.g like this one http://m.heraldsun.com.au/news/opinion/linda-dessau-ticks-all-the-boxes-for-governor-role/story-fni0ffsx-1227216525121 ) It is unlikely for me to find the main points if I dont know what the article is REALLY about( Main Contention ). So is there a qay to practise finding the main contention of an article? I can ask my teacher to help but that's just too slow and she probably wouldnt be available all the time.


2. The 'Key Player' Method is particularly effective to make the entire L.A piece coherent. However it seems time-consuming, I'm an EAL student and we have to write a Note-Form-Summary, I dont think we have as much time outlining the Key Players as those in Mainstream do. What if I structure my L.A bodyparagraphs by the Main Points I used from my Note-Form-Summary? Will this be coherent as Key Player Method?


3. I have some confusions regarding to N.F.S, like how much details(dot points) should we add to each Main Point? What kind of N.F.S structure works for which kind of article/opinion piece? How to write a N.F.S for multiple pieces? How to decrease the time wasting on "Converting sentences to Note-Form"?


Many thanks, all replies are appreciated!
(≧▽≦)
Title: Re: VCE English Question Thread
Post by: M_BONG on February 15, 2015, 01:37:45 pm
Hi, Im a student who's doing EAL.

I have a few questions according to EAL.


1. I am finding it difficult to get the Main contention of an article. It is unlikely for me to find the main points if I dont know what the article is REALLY about( Main Contention ). So is there a qay to practise finding the main contention of an article? I can ask my teacher to help but that's just too slow and she probably wouldnt be available all the time.


2. The 'Key Player' Method is particularly effective to make the entire L.A piece coherent. However it seems time-consuming, I'm an EAL student and we have to write a Note-Form-Summary, I dont think we have as much time outlining the Key Players as those in Mainstream do. What if I structure my L.A bodyparagraphs by the Main Points I used from my Note-Form-Summary? Will this be coherent as Key Player Method?


3. I have some confusions regarding to N.F.S, like how much details(dot points) should we add to each Main Point? What kind of N.F.S structure works for which kind of article/opinion piece? How to write a N.F.S for multiple pieces? How to decrease the time wasting on "Converting sentences to Note-Form"?


Many thanks, all replies are appreciated!
(≧▽≦)
Hey!

1. There are no hard and fast rules to determine the contention of an article. However, a good article should make it bleedingly obvious what it is arguing. And that is what the contention is: what is the author sending a message/making a point on?

Practice will likely make it easier for you, but the way I always check the contention is look at the key places - the headline, the by-line and the last sentence. The last sentence is always a give away because the author will include something so that the reader will remember the article, and this is most likely related to their contention.


2. I actually find the key player method very time saving. I am not sure the exact requirements of EAL, but since you are required to write a piece of prose, this method can save a lot of time - you just find three groups that the author has an opinion on and sort all your analysis under this umbrella and I find this to be the most sophisticated approach.

However, the best thing to do is follow what you are good at. I obviously won't advocate for your "argument approach" (ie. sorting things by main points) but what I find with this is that it becomes really clunky and you have three separate, distinct paragraphs which actually don't link together, as a piece of prose.

Nevertheless, always do what you are comfortable with; but since it's early days, you can afford to explore around.


3. Not sure, sorry - probably need an EAL student to clarify.
Title: Re: VCE English Question Thread
Post by: literally lauren on February 15, 2015, 01:49:35 pm
Hi, Im a student who's doing EAL.

I have a few questions according to EAL.


1. I am finding it difficult to get the Main contention of an article. (e.g like this one http://m.heraldsun.com.au/news/opinion/linda-dessau-ticks-all-the-boxes-for-governor-role/story-fni0ffsx-1227216525121 ) It is unlikely for me to find the main points if I dont know what the article is REALLY about( Main Contention ). So is there a qay to practise finding the main contention of an article? I can ask my teacher to help but that's just too slow and she probably wouldnt be available all the time.
I'll go through this article later as an example if you need, but two questions you should ask yourself:
- If I were the author, and I was writing all this, what would I be trying to do? How do I want people to react to certain ideas? What's my best/worst case scenario?
and
- If I were a reader with no opinions or beliefs, and I just read this article and believed everything it said, what would my overall impression be? What values have I been given? What has been cast in a positive or negative light?

Quote
2. The 'Key Player' Method is particularly effective to make the entire L.A piece coherent. However it seems time-consuming, I'm an EAL student and we have to write a Note-Form-Summary, I dont think we have as much time outlining the Key Players as those in Mainstream do. What if I structure my L.A bodyparagraphs by the Main Points I used from my Note-Form-Summary? Will this be coherent as Key Player Method?
I'm in the process of writing up some EAL-geared resources cause I know it's a sparse area on AN and elsewhere.
With regardss to Key players, yes it's time consuming, but you're consuming time with a purpose. Even if it takes you five minutes of writing time just to tease out what some of the key players are and what the author says about them, that's five minutes well spent, because you've come to a conclusion that half the state won't even try to make. Identifying key players isn't just something that you can do, if you want; it's something that all good analyses definitely do, albeit in different ways. What's optional is the structure, so if you feel more confident going through things chronologically, then by all means do so. Just be aware of the risks, and practice with different forms (especially comparative ones) so you know you can handle whatever the exam throws at you.


Quote
3. I have some confusions regarding to N.F.S, like how much details(dot points) should we add to each Main Point? What kind of N.F.S structure works for which kind of article/opinion piece? How to write a N.F.S for multiple pieces? How to decrease the time wasting on "Converting sentences to Note-Form"?
Use the samples in the Assessor's Reports for EAL on the VCAA website, as these are the only good indicators I've found. Having not done EAL myself I can't vouch for any particular method, but something tells me most teachers would advocate a bunch of different approaches, meaning VCAA have to be flexible with what they accept.

I'll add your other quesitons to the list of stuff I'm trying to cover in the EAL post since it probably warrants more explanation than I have time for right now. Should be done in a couple of days :)

Do check with your teacher in the meantime, even if it takes ages and their advice is dubious. I have limited experience with EAL and most of my knowledge has been gleaned through other students and their teachers, so use your own as a resource too :)
Title: Re: VCE English Question Thread
Post by: hwilome on February 15, 2015, 03:19:33 pm
Thank you very very much Zezima and Lauren !!! :)
Title: Re: VCE English Question Thread
Post by: cosine on February 16, 2015, 07:55:27 pm
Anyone here a tutor for english 3/4? If so please PM me or if you know of someone good PM me their contact, really desperate for a tutor for english!
Title: Re: VCE English Question Thread
Post by: Apink! on February 19, 2015, 07:46:43 pm
Hello,
I often lose marks because I do not directly answer the prompt :P But if I do, I cannot come up with original ideas that will make my essay stand out
I'm currently studying "Shark Net" and "whose reality?"

How do you make sure that you DO answer the prompt? Many assessor's reports and my teachers always emphasize on making sure that we answer the prompt but I tend to wander off...And how do you come up with original ideas while answering the prompt? To what extent can you explore a prompt without seeming as you are not answering it?

Also, I am meant to do an expository style essay on a prompt (that is unseen). In the previous years when we did context, I would always do a creative piece or persuasive so i'm not used to the idea of expository... How would you structure your essay? What is the purpose of expository? I heard that you explore the issue at hand, so do you always need to consider both sides of the prompt?

Also, since the prompt is unseen, how can I prepare myself?

Gahh, I am so worried. Although I do love English, I am far from good ::)

Please help me :)
Title: Re: VCE English Question Thread
Post by: cosine on February 19, 2015, 07:51:54 pm
Same with me, I always fall under the impression that im not fully answering the prompt in an essay, and consequently i lose motivation throughout the essay and sort of give up at the end. What can I do to ensure I can analyse the prompt well, and be able to come up with good arguments/ideas? Thanks
Title: Re: VCE English Question Thread
Post by: literally lauren on February 20, 2015, 02:10:28 pm
Hello,
I often lose marks because I do not directly answer the prompt :P But if I do, I cannot come up with original ideas that will make my essay stand out
I'm currently studying "Shark Net" and "whose reality?"

How do you make sure that you DO answer the prompt? Many assessor's reports and my teachers always emphasize on making sure that we answer the prompt but I tend to wander off...And how do you come up with original ideas while answering the prompt? To what extent can you explore a prompt without seeming as you are not answering it?

Also, I am meant to do an expository style essay on a prompt (that is unseen). In the previous years when we did context, I would always do a creative piece or persuasive so i'm not used to the idea of expository... How would you structure your essay? What is the purpose of expository? I heard that you explore the issue at hand, so do you always need to consider both sides of the prompt?

Also, since the prompt is unseen, how can I prepare myself?

Gahh, I am so worried. Although I do love English, I am far from good ::)

Please help me :)
Same with me, I always fall under the impression that im not fully answering the prompt in an essay, and consequently i lose motivation throughout the essay and sort of give up at the end. What can I do to ensure I can analyse the prompt well, and be able to come up with good arguments/ideas? Thanks

Rather than thinking of it as "answering" the prompt, try to see your essays as taking the prompt a step further.
Let's say you're doing Conflict and the prompt is It is not what we learn, but how we learn from conflict that is important.

The simplistic, middle-band pieces would have contentions like 'yes' or 'no.' You don't want to fall into that chasm of simply repeating a bunch of evidence that demonstrates the same point. Your piece must have a contention, and it should be at least a sentence long (preferably more.) For instance, I might want to argue that 'how we learn' tells us more about who we are and what we value, whereas the lessons learned (ie. 'what we learn') have to be filtered through our understanding of the world. 'How we learn' is a process, 'what we learn' is simply the result; it's almost like 'what we learn' is a subset of the 'how.' Therefore the former is more important because it's an all-encompassing journey, not an end-goal.

Note how I haven't gone straight for the text, or any of my examples. I'm simply rationalising what the prompt is suggesting on an idea-level. Ultimately, you could boil down my contention to 'yes,' but the important thing is that I've got my reasoning here. I've done the development, and so the outcome is a way more powerful contention than it would have been if I'd just considered the prompt for 30 seconds and gone 'yeah, I guess that sounds about right.'

So that's the most beneficial thing you can do: develop your thinking so that you're not stifling yourself into repeatedly saying 'yes,' or just talking about the key words in the prompt.

But I know this seems risky when you don't know the right questions to ask, or if your discussion is actually relevant.

The question now becomes 'how do I maintain relevance while still conducting sufficiently broad and deep exploration?' To which I say the answer is weaving!
Usually I explain this with hand-drawn diagrams, so I'll see if I can upload something later when I have access to a scanner. Also, this is geared mostly towards expository pieces; different rules can apply for the other forms.

What you want to do is weave your discussion through to different levels. For anyone who knows anything about sewing, you'll know that the needle and thread have to be woven from one side of the material to the other, and back again. But if you prick the needle too close to the hole you just made, you can end up ripping the material and creating a hole that's too big, and can't be woven through. At this bottom level that the thread is trying to get to: you have the very close examples, often from the set text, though not always. This can have seemingly little to do with the Context itself, so the way you make it relevant is by slowly weaving it through the upper levels. However, using too much evidence to illustrate the same point is the equivalent of sewing too close together: you can't do any more sewing in that area, and you can't get to the next stage.

In the middle, you have 'theorisation,' which is where you're drawing conclusions from the examples, but not necessarily going all the way out to the Context yet. You might be discussing the meaning of an event, or comparing it to other occurrences for the sake of drawing parallels or contrasts. This is where you're actually passing through the material and connecting one level with another.

Where you chose to do this theorisation is incredibly important. I'll explain that more in the section on essay structure below.

Then towards the top you have the Context and the prompt, a.k.a. the umbrella that everything else must be under. This is where you start to tie everything together with a 'Therefore...' statement, and hammer home the relevance to the assessor. It's not enough to just conduct an exploration and assume your reader can piece together its relevance - YOU HAVE TO TELL THEM EXPLICITLY!

One of the worst things you can do in an expository piece is allow your marker to get to the end of a paragraph and ask 'so what?' You've brought up the example of post-war America and how people tried to learn from their mistakes... so what? You've reflected upon a past experience when you felt you learnt an important lesson... so what? You've tied together several historical and psychological examples about the way people learn in the midst of conflict... SO WHAT? Why are you saying this? What has this discussion taught us? What contention are you trying to reinforce here??

Answer these questions in your writing, and don't give your assessor the chance :)


This relates quite neatly to your question about essay structure as well. I usually recommend the following format to people as a starting point, as it'll make apparent the amount of discussion you have to do at different levels, as well as help you find areas of weakness in a very obvious way. Just a note for anyone else: even if you're dead set on writing an imaginative piece or w/e, you should write at least one expository essay anyway. It spells things out in the clearest way possible, which, when it comes to something as messy as Context, is pretty useful.

Overall (and this is a massive generalisation, not a hard-and-fast rule) you should aim for an even ratio of general abstract theorisation about the Context, to close evidence-based discussion.
Think of it like this:
(http://sewingschool.org/wp-content/uploads/2011/03/tension.jpg)
In the second image, there's not enough on the upper (ie. general Context stuff) level, meaning the final product is going to have a whole lot literal and metaphorical loose threads hanging at the bottom (evidence) level. The reverse could also be true: if you've gone a heap of talking about the Context but don't have anything to back it up, you won't be getting credit for all the loose loops up the top.

If you want your essay to be tight  ;) you have to not only balance the amount of discussion you do on either side, but you also have to know when and how to transition. Let's say you had a metre long piece of material to sew and you only made five stitches - one every 20 cm. It'd hardly look like an appropriate sewing job, and it wouldn't hold things together. Similarly, make too many stitches and you end up wasting thread (ie. wasting your time.) You also want to avoid making inconsistent transitions. Take a look at any piece of clothing you own; good stitching is evenly spaced out. Miss out on one of those spaces, and things fall apart.

To move away from my slightly tortured sewing metaphor now... you'd want your essay to look something like this:

   ---------------
-----------------    <-- intro exploring the prompt, all general
-----------------   <-- maybe an example at the end to lead into the B.P.

   ---------------       <-- body paragraph, T.S. starts off general
-----------------    <-- start integrating evidence
-----------------     <-- draw out some conclusions so you can link your discussion
-----------------     <-- more evidence, adding to, not repeating your point
-----------------     <-- lastly moving back out to the prompt and reinforcing your contention

   ---------------
-----------------    <-- conclusion tying things up, all general, though
-----------------          perhaps calling back to something mentioned in the intro (=bookending)

Here's an example I wrote in response to a similar question last year:

Quote
For example, the prompt: 'Our identities are always changing'
My paragraph might begin by looking at the idea of change, rather than just assuming the reader knows what I'm talking about. Obviously you don't have to give definitions, but it can be helpful to clarify
eg. 'Change is an inevitable part of our lives, but that is not to say all things are changing all the time. Often there are parts of our personas that remain stagnant until external events prompt us to reconsider ourselves, or to react in different ways.' {I haven't used any examples yet, I'm just breaking down some theory. Then I'd work on linking it to a specific idea or piece of evidence}: 'Nowhere is this change more obvious than in adolescence; a time of transition when we are forced to consciously reevaluate our selves in relation to society. In Bruce Beresford's Paradise Road, the majority of the cast are adult actors dealing with adult concepts, and it is easy to forget the children and teenagers that were likewise subject to the harsh conditions of POW camps in WWII. Following one of the choir's productions, we see boys as young as 15 and 16 being sent away to the men's camp as their mothers cry in the background 'no, please, he's just a child!' {I'm paraphrasing here, it's been ages since I saw the movie :P} One can only imagine the irreparable psychological damage this caused the boys. The separation of a child from their mother is incredibly traumatic, and a further indictment of how the brutality of war pervades all aspects of life. Who we are is, of course, a fluid concept, but our identity as a whole isn't entirely self-determined - we cannot ignore the role of external factors.'

Orange: abstract discussion
Purple: specific examples, from the text or otherwise
In hindsight, the conclusion I've drawn at the end of that paragraph is a bit dodgy, but that's mainly because I've only done two sentences of evidence-unpacking: your essays will obviously go into more detail, and hopefully draw from multiple sources, not just one scene in one text.


Hopefully that addresses the bigger questions, so to clarify the little ones in case you're still unsure:
- The purpose of the expository style is to expose facets of the prompt. It's giving you a literal, usually fairly straightforward assertion, and you have to take that and explore the implications.
- You don't need to consider 'both sides' because there are more than two sides. You need to consider many, but have your considerations fall under the umbrella of your contention. They're giving you an opportunity for limitless exploration, but you only have to explore what's most relevant to you.
- Rather than preparing for specific content like you would in other subjects (ie. 'this maths SAC is going to test these areas, in this format, probably with questions similar to last year' etc.) English is more about preparing a skillset that can handle anything. Collect examples, make a conclusive list of major prompts, write practice paragraphs and essays - do whatever you think will help you solidify your understanding.

Apologies for the verbosity, but 'how to answer a Context prompt' is a huge area, so I figured I'd tackle it from the ground up. If you have further questions, please let me know, as this definitely hasn't covered everything :)
Title: Re: VCE English Question Thread
Post by: M_BONG on February 20, 2015, 02:43:34 pm
Holy F, Lauren. You deserve a Noble Prize in Literature for writing all that! Also, a whole lot of awesome  and well-illustrated advice as well!
Title: Re: VCE English Question Thread
Post by: brenden on February 20, 2015, 05:02:22 pm
Rather than thinking of it as "answering" the prompt, try to see your essays as taking the prompt a step further.
Let's say you're doing Conflict and the prompt is It is not what we learn, but how we learn from conflict that is important.

The simplistic, middle-band pieces would have contentions like 'yes' or 'no.' You don't want to fall into that chasm of simply repeating a bunch of evidence that demonstrates the same point. Your piece must have a contention, and it should be at least a sentence long (preferably more.) For instance, I might want to argue that 'how we learn' tells us more about who we are and what we value, whereas the lessons learned (ie. 'what we learn') have to be filtered through our understanding of the world. 'How we learn' is a process, 'what we learn' is simply the result; it's almost like 'what we learn' is a subset of the 'how.' Therefore the former is more important because it's an all-encompassing journey, not an end-goal.

Note how I haven't gone straight for the text, or any of my examples. I'm simply rationalising what the prompt is suggesting on an idea-level. Ultimately, you could boil down my contention to 'yes,' but the important thing is that I've got my reasoning here. I've done the development, and so the outcome is a way more powerful contention than it would have been if I'd just considered the prompt for 30 seconds and gone 'yeah, I guess that sounds about right.'

So that's the most beneficial thing you can do: develop your thinking so that you're not stifling yourself into repeatedly saying 'yes,' or just talking about the key words in the prompt.

But I know this seems risky when you don't know the right questions to ask, or if your discussion is actually relevant.

The question now becomes 'how do I maintain relevance while still conducting sufficiently broad and deep exploration?' To which I say the answer is weaving!
Usually I explain this with hand-drawn diagrams, so I'll see if I can upload something later when I have access to a scanner. Also, this is geared mostly towards expository pieces; different rules can apply for the other forms.

What you want to do is weave your discussion through to different levels. For anyone who knows anything about sewing, you'll know that the needle and thread have to be woven from one side of the material to the other, and back again. But if you prick the needle too close to the hole you just made, you can end up ripping the material and creating a hole that's too big, and can't be woven through. At this bottom level that the thread is trying to get to: you have the very close examples, often from the set text, though not always. This can have seemingly little to do with the Context itself, so the way you make it relevant is by slowly weaving it through the upper levels. However, using too much evidence to illustrate the same point is the equivalent of sewing too close together: you can't do any more sewing in that area, and you can't get to the next stage.

In the middle, you have 'theorisation,' which is where you're drawing conclusions from the examples, but not necessarily going all the way out to the Context yet. You might be discussing the meaning of an event, or comparing it to other occurrences for the sake of drawing parallels or contrasts. This is where you're actually passing through the material and connecting one level with another.

Where you chose to do this theorisation is incredibly important. I'll explain that more in the section on essay structure below.

Then towards the top you have the Context and the prompt, a.k.a. the umbrella that everything else must be under. This is where you start to tie everything together with a 'Therefore...' statement, and hammer home the relevance to the assessor. It's not enough to just conduct an exploration and assume your reader can piece together its relevance - YOU HAVE TO TELL THEM EXPLICITLY!

One of the worst things you can do in an expository piece is allow your marker to get to the end of a paragraph and ask 'so what?' You've brought up the example of post-war America and how people tried to learn from their mistakes... so what? You've reflected upon a past experience when you felt you learnt an important lesson... so what? You've tied together several historical and psychological examples about the way people learn in the midst of conflict... SO WHAT? Why are you saying this? What has this discussion taught us? What contention are you trying to reinforce here??

Answer these questions in your writing, and don't give your assessor the chance :)


This relates quite neatly to your question about essay structure as well. I usually recommend the following format to people as a starting point, as it'll make apparent the amount of discussion you have to do at different levels, as well as help you find areas of weakness in a very obvious way. Just a note for anyone else: even if you're dead set on writing an imaginative piece or w/e, you should write at least one expository essay anyway. It spells things out in the clearest way possible, which, when it comes to something as messy as Context, is pretty useful.

Overall (and this is a massive generalisation, not a hard-and-fast rule) you should aim for an even ratio of general abstract theorisation about the Context, to close evidence-based discussion.
Think of it like this:
(http://sewingschool.org/wp-content/uploads/2011/03/tension.jpg)
In the second image, there's not enough on the upper (ie. general Context stuff) level, meaning the final product is going to have a whole lot literal and metaphorical loose threads hanging at the bottom (evidence) level. The reverse could also be true: if you've gone a heap of talking about the Context but don't have anything to back it up, you won't be getting credit for all the loose loops up the top.

If you want your essay to be tight  ;) you have to not only balance the amount of discussion you do on either side, but you also have to know when and how to transition. Let's say you had a metre long piece of material to sew and you only made five stitches - one every 20 cm. It'd hardly look like an appropriate sewing job, and it wouldn't hold things together. Similarly, make too many stitches and you end up wasting thread (ie. wasting your time.) You also want to avoid making inconsistent transitions. Take a look at any piece of clothing you own; good stitching is evenly spaced out. Miss out on one of those spaces, and things fall apart.

To move away from my slightly tortured sewing metaphor now... you'd want your essay to look something like this:

   ---------------
-----------------    <-- intro exploring the prompt, all general
-----------------   <-- maybe an example at the end to lead into the B.P.

   ---------------       <-- body paragraph, T.S. starts off general
-----------------    <-- start integrating evidence
-----------------     <-- draw out some conclusions so you can link your discussion
-----------------     <-- more evidence, adding to, not repeating your point
-----------------     <-- lastly moving back out to the prompt and reinforcing your contention

   ---------------
-----------------    <-- conclusion tying things up, all general, though
-----------------          perhaps calling back to something mentioned in the intro (=bookending)

Here's an example I wrote in response to a similar question last year:
In hindsight, the conclusion I've drawn at the end of that paragraph is a bit dodgy, but that's mainly because I've only done two sentences of evidence-unpacking: your essays will obviously go into more detail, and hopefully draw from multiple sources, not just one scene in one text.


Hopefully that addresses the bigger questions, so to clarify the little ones in case you're still unsure:
- The purpose of the expository style is to expose facets of the prompt. It's giving you a literal, usually fairly straightforward assertion, and you have to take that and explore the implications.
- You don't need to consider 'both sides' because there are more than two sides. You need to consider many, but have your considerations fall under the umbrella of your contention. They're giving you an opportunity for limitless exploration, but you only have to explore what's most relevant to you.
- Rather than preparing for specific content like you would in other subjects (ie. 'this maths SAC is going to test these areas, in this format, probably with questions similar to last year' etc.) English is more about preparing a skillset that can handle anything. Collect examples, make a conclusive list of major prompts, write practice paragraphs and essays - do whatever you think will help you solidify your understanding.

Apologies for the verbosity, but 'how to answer a Context prompt' is a huge area, so I figured I'd tackle it from the ground up. If you have further questions, please let me know, as this definitely hasn't covered everything :)
Let the upvotes rain.
Title: Re: VCE English Question Thread
Post by: Alter on February 20, 2015, 09:30:50 pm
Hey there guys.

Just to preface this, I should clarify that last year I did 1/2 Literature as opposed to mainstream English. As a result, I'm a bit behind in terms of having practised the different types of responses necessary for the subject, but I don't find them overly difficult to adapt to.

My question relates to doing a context piece. I should clarify that I am particularly interested in writing a creative piece for my SACs/exam as this is the text type I am most comfortable with, it being something I greatly enjoy and have done a lot in the past outside of school. That being said, I am not entirely sure how one is supposed to link into studied texts they've read while still maintaining cohesion and originality in a response. Are there any examples of how I can incorporate textual reference in a creative piece that I should read? To clarify, I'm doing encountering conflict with the texts The Lieutenant and Every Man In This Village Is A Liar.

On top of this, what are the best pieces of advice you can provide for someone who is keen on doing a creative piece for context, but has not had much experience with regular English before? I've read some threads on here for context which were quite helpful, but there don't seem to be a great deal of resources for people in my specific position. Thanks in advance.
Title: Re: VCE English Question Thread
Post by: literally lauren on February 20, 2015, 09:46:02 pm
Hey there guys.

Just to preface this, I should clarify that last year I did 1/2 Literature as opposed to mainstream English. As a result, I'm a bit behind in terms of having practised the different types of responses necessary for the subject, but I don't find them overly difficult to adapt to.
Dw man, in some ways Lit is better preparation for English because it helps you analyse and discuss things more closely. So long as you're across the basic 3 essay types in English, you're not really behind.

Quote
My question relates to doing a context piece. I should clarify that I am particularly interested in writing a creative piece for my SACs/exam as this is the text type I am most comfortable with, it being something I greatly enjoy and have done a lot in the past outside of school. That being said, I am not entirely sure how one is supposed to link into studied texts they've read while still maintaining cohesion and originality in a response. Are there any examples of how I can incorporate textual reference in a creative piece that I should read? To clarify, I'm doing encountering conflict with the texts The Lieutenant and Every Man In This Village Is A Liar.
Just so I can answer this properly, what sort of creative piece are you doing? Short story, letter, reflective piece, hybrid? There are slightly different rules for each format, so maybe give us a general overview of what one of your pieces might be dealing with. (You don't have to post a full essay or anything, just a brief outline)

Quote
On top of this, what are the best pieces of advice you can provide for someone who is keen on doing a creative piece for context, but has not had much experience with regular English before? I've read some threads on here for context which were quite helpful, but there don't seem to be a great deal of resources for people in my specific position. Thanks in advance.
Like I said above, write an expository piece anyway. You can definitely stick with creatives for your SACs and the exam, but expository pieces are the easiest way to grasp the requirements of the task, and the balance of your discussion. Not having done English 1&2 really isn't a disadvantage, and a lost of people I know (myself included) weren't exactly top-of-the-range in Year 11, but compensated by working hard in Year 12 when it mattered. Other than that, I'll wait till I know your preferred writing style before I go into more depth in terms of text integration :)
Title: Re: VCE English Question Thread
Post by: Alter on February 20, 2015, 10:20:53 pm
Thanks for the quick response, Lauren!

I'm glad that not doing 1/2 English won't be detrimental for my score. At my school, mainstream English seems to have a pretty poor reputation/stigma for being a subject done by people that aren't intelligent enough to do English Language or Literature do. As a result, the cohort seems to be typically weaker. That being said, there was a girl from my school last year who pretty impressively got a 50 in English despite having transferred from Literature in the same situation as me.

I see now that I was a bit vague with just 'creative writing'. I mean a short story, or possibly a hybrid (although I have very limited experience with the latter...). To be honest, anything that falls under the banner of a typical "narrative" / "short story" will work for me, although perhaps I'm being a bit naive in how distinct different types of creative pieces are.
Title: Re: VCE English Question Thread
Post by: Apink! on February 21, 2015, 07:53:53 am
Rather than thinking of it as "answering" the prompt, try to see your essays as taking the prompt a step further.
Let's say you're doing Conflict and the prompt is It is not what we learn, but how we learn from conflict that is important.

The simplistic, middle-band pieces would have contentions like 'yes' or 'no.' You don't want to fall into that chasm of simply repeating a bunch of evidence that demonstrates the same point. Your piece must have a contention, and it should be at least a sentence long (preferably more.) For instance, I might want to argue that 'how we learn' tells us more about who we are and what we value, whereas the lessons learned (ie. 'what we learn') have to be filtered through our understanding of the world. 'How we learn' is a process, 'what we learn' is simply the result; it's almost like 'what we learn' is a subset of the 'how.' Therefore the former is more important because it's an all-encompassing journey, not an end-goal.

Note how I haven't gone straight for the text, or any of my examples. I'm simply rationalising what the prompt is suggesting on an idea-level. Ultimately, you could boil down my contention to 'yes,' but the important thing is that I've got my reasoning here. I've done the development, and so the outcome is a way more powerful contention than it would have been if I'd just considered the prompt for 30 seconds and gone 'yeah, I guess that sounds about right.'

So that's the most beneficial thing you can do: develop your thinking so that you're not stifling yourself into repeatedly saying 'yes,' or just talking about the key words in the prompt.

But I know this seems risky when you don't know the right questions to ask, or if your discussion is actually relevant.

The question now becomes 'how do I maintain relevance while still conducting sufficiently broad and deep exploration?' To which I say the answer is weaving!
Usually I explain this with hand-drawn diagrams, so I'll see if I can upload something later when I have access to a scanner. Also, this is geared mostly towards expository pieces; different rules can apply for the other forms.

What you want to do is weave your discussion through to different levels. For anyone who knows anything about sewing, you'll know that the needle and thread have to be woven from one side of the material to the other, and back again. But if you prick the needle too close to the hole you just made, you can end up ripping the material and creating a hole that's too big, and can't be woven through. At this bottom level that the thread is trying to get to: you have the very close examples, often from the set text, though not always. This can have seemingly little to do with the Context itself, so the way you make it relevant is by slowly weaving it through the upper levels. However, using too much evidence to illustrate the same point is the equivalent of sewing too close together: you can't do any more sewing in that area, and you can't get to the next stage.

In the middle, you have 'theorisation,' which is where you're drawing conclusions from the examples, but not necessarily going all the way out to the Context yet. You might be discussing the meaning of an event, or comparing it to other occurrences for the sake of drawing parallels or contrasts. This is where you're actually passing through the material and connecting one level with another.

Where you chose to do this theorisation is incredibly important. I'll explain that more in the section on essay structure below.

Then towards the top you have the Context and the prompt, a.k.a. the umbrella that everything else must be under. This is where you start to tie everything together with a 'Therefore...' statement, and hammer home the relevance to the assessor. It's not enough to just conduct an exploration and assume your reader can piece together its relevance - YOU HAVE TO TELL THEM EXPLICITLY!

One of the worst things you can do in an expository piece is allow your marker to get to the end of a paragraph and ask 'so what?' You've brought up the example of post-war America and how people tried to learn from their mistakes... so what? You've reflected upon a past experience when you felt you learnt an important lesson... so what? You've tied together several historical and psychological examples about the way people learn in the midst of conflict... SO WHAT? Why are you saying this? What has this discussion taught us? What contention are you trying to reinforce here??

Answer these questions in your writing, and don't give your assessor the chance :)


This relates quite neatly to your question about essay structure as well. I usually recommend the following format to people as a starting point, as it'll make apparent the amount of discussion you have to do at different levels, as well as help you find areas of weakness in a very obvious way. Just a note for anyone else: even if you're dead set on writing an imaginative piece or w/e, you should write at least one expository essay anyway. It spells things out in the clearest way possible, which, when it comes to something as messy as Context, is pretty useful.

Overall (and this is a massive generalisation, not a hard-and-fast rule) you should aim for an even ratio of general abstract theorisation about the Context, to close evidence-based discussion.
Think of it like this:
(http://sewingschool.org/wp-content/uploads/2011/03/tension.jpg)
In the second image, there's not enough on the upper (ie. general Context stuff) level, meaning the final product is going to have a whole lot literal and metaphorical loose threads hanging at the bottom (evidence) level. The reverse could also be true: if you've gone a heap of talking about the Context but don't have anything to back it up, you won't be getting credit for all the loose loops up the top.

If you want your essay to be tight  ;) you have to not only balance the amount of discussion you do on either side, but you also have to know when and how to transition. Let's say you had a metre long piece of material to sew and you only made five stitches - one every 20 cm. It'd hardly look like an appropriate sewing job, and it wouldn't hold things together. Similarly, make too many stitches and you end up wasting thread (ie. wasting your time.) You also want to avoid making inconsistent transitions. Take a look at any piece of clothing you own; good stitching is evenly spaced out. Miss out on one of those spaces, and things fall apart.

To move away from my slightly tortured sewing metaphor now... you'd want your essay to look something like this:

   ---------------
-----------------    <-- intro exploring the prompt, all general
-----------------   <-- maybe an example at the end to lead into the B.P.

   ---------------       <-- body paragraph, T.S. starts off general
-----------------    <-- start integrating evidence
-----------------     <-- draw out some conclusions so you can link your discussion
-----------------     <-- more evidence, adding to, not repeating your point
-----------------     <-- lastly moving back out to the prompt and reinforcing your contention

   ---------------
-----------------    <-- conclusion tying things up, all general, though
-----------------          perhaps calling back to something mentioned in the intro (=bookending)

Here's an example I wrote in response to a similar question last year:
In hindsight, the conclusion I've drawn at the end of that paragraph is a bit dodgy, but that's mainly because I've only done two sentences of evidence-unpacking: your essays will obviously go into more detail, and hopefully draw from multiple sources, not just one scene in one text.


Hopefully that addresses the bigger questions, so to clarify the little ones in case you're still unsure:
- The purpose of the expository style is to expose facets of the prompt. It's giving you a literal, usually fairly straightforward assertion, and you have to take that and explore the implications.
- You don't need to consider 'both sides' because there are more than two sides. You need to consider many, but have your considerations fall under the umbrella of your contention. They're giving you an opportunity for limitless exploration, but you only have to explore what's most relevant to you.
- Rather than preparing for specific content like you would in other subjects (ie. 'this maths SAC is going to test these areas, in this format, probably with questions similar to last year' etc.) English is more about preparing a skillset that can handle anything. Collect examples, make a conclusive list of major prompts, write practice paragraphs and essays - do whatever you think will help you solidify your understanding.

Apologies for the verbosity, but 'how to answer a Context prompt' is a huge area, so I figured I'd tackle it from the ground up. If you have further questions, please let me know, as this definitely hasn't covered everything :)


Thank you so much! That really helps me clarify it a whole lot better.
If you could, could you please upload a good expository essay? I learn things by observing what great writer's had done and analyse it and then apply it to my own writing.
There were some on the essay thread, but not a lot of them are expository :P

Thank you Lauren, you explain it so much better :)

EDIT: Also my teacher is very keen on the idea of personal anecdotes in expository essay. She kinda implies that by putting a personal anecdote of some sort in the beginning of the essay, we will score better than generalising in the first paragraph. What do you think?

I personally do not like personal anecdotes. They are interesting, but often they don't seem like a  'must' think to put in a context expository essay. Should I do what my teacher says to get a better mark or should I just do my own thang?

Also, if you advise me on doing a personal anecdote/ creative bit in the first paragraph, how can I make sure my opening stands out with 35 or so of personal anecdotes/ creative stories? I want to be original :)
Title: Re: VCE English Question Thread
Post by: paper-back on February 21, 2015, 11:25:37 am
How can I improve on my expressiveness and fluency? Sometimes when others read my paragraphs, they don't seem to know what I'm trying address and it's becoming a real worry

Any tips would be much appreciated
Thanks!

Title: Re: VCE English Question Thread
Post by: Floatzel98 on February 21, 2015, 11:40:38 am
When we are writing context pieces, what do we need to include at the end when writing the summary of intent, or the explanation of the piece. How long does it need to be and does it affect the marking?
Title: Re: VCE English Question Thread
Post by: JackSonSmith on February 21, 2015, 02:04:35 pm
How can I improve on my expressiveness and fluency? Sometimes when others read my paragraphs, they don't seem to know what I'm trying address and it's becoming a real worry

Any tips would be much appreciated
Thanks!

Personally, I find that using short sentences helps. They ensure ideas are easy to follow.
Title: Re: VCE English Question Thread
Post by: literally lauren on February 21, 2015, 03:08:07 pm
Apink!: I'll try and upload something later tonight. I have a few good-ish expository essays, but I think it'd be more helpful if I wrote a piece from scratch.
With regards to personal anecdotes, no they're not compulsory, and the assessors don't really favour "petty personal stories" (not that all personal stories are petty, but it can be hard to go from 'My sister and I used to pull each others hair, but now we don't. Sometimes people learn lessons from conflict' :P) But you should absolutely so anything and everything your teacher suggests for the SAC. The exam approach might be a little different though.



Alter:

First of all, you need to be aware of the balance between a good Context piece, and a good short story. Here's what happens if you do too much of the former:
Jocelyn stood on the shore, a cool zephyr blowing through her hair. As she gazed at the setting sun, she wondered whether fear or prejudice is really at the heart of conflict. :/ It stops sounding like a believable story at the end there. That's because your piece has to have verisimilitude which is a lovely word that here boils down to 'tell a good story without being bogged down by nonsense.'

The easiest rule of thumb here is not to use the key words of the prompt, or the name of the context itself (eg. prompt is 'Our identity depends on our upbringing, not our values' --> the words 'identity,' 'upbringing' and 'values' are off-limits) as these are things you're meant to show in the story itself. (more on that later)

On the flip side, you still want to be ensuring marks for relevance, and you could write the most amazing short story in the world that totally blows the assessors out of their chairs - but if it has no relation to the prompt - 4/10 for effort, maybe :P
Regardless of your writing style, the prompt dominates everything. It's where your contention, ideas, and examples should all stem from.

The text is a subset of this, but how you integrate it is really up to you. If you find explicit connections easier, then you could write a POV from one of the characters, or rewrite another ending/ 'lost scene' from the text. But the better links will be idea-based ones; if the speaker in your story is going through a certain struggle, see if you can find parallels with someone from the set text.

Using the language of the author can also be very effective. For instance, Every Man has that famous quote about how "you can survive and not survive both at the same time," so if the idea of survival is relevant to your piece, consider integrating or adapting those words and including them in your work.

eg, let's say you were writing about spousal troubles and abuse: Lucy sat at the dinner table, petrified that she would sweat through her concealer and reveal the bruises. She must have looked too nervous, as Micheal took a firm grasp of her knee under the tablecloth. She felt his grubby fingers dig under the kneecap as though he was willing her bones to separate, but she knew she'd pay for her mistakes more severely once the guests had left. She glanced at the other couples and their affectionate tactility, their total comfort with one another. Micheal gripped her knee tighter. At least she was alive, she thought, but this was far from living.

For your SACs, you will have the opportunity to explain yourself fully in a Statement of Intention/ Written Explanation - a paragraph at the end that mentions your choice of form, style, language, audience, contention, and textual connection. So for the above, part of my SOI might read: 'I have used Megan Stack's questioning of what survival really means, and recontextualised it from a global, to a very personal setting.'

In the exam, you don't get this chance though. There are no SOIs, it's just the piece you write, and you have to rely on the assessor being smart enough to grasp the connections. I won't like - this is a massive challenge, and it's the reason I chickened out and went with a nice safe expository piece in the exam. I know people who can make it work, but it takes a lot of practice.

Story-time: Why Lauren abandoned imaginative pieces:
I wrote a piece around mid-year that was sent off to an external marker, and I thought the connections were pretty obvious. One of the films that used to be on the list (Paradise Road) had a line like 'The will to survive is strong, stronger than anything,' and one of the lines in my intro was 'Although the will to survive is strong, the pervasive effects of conflict... something something' and other such similar connections (I think I did this four or five times with the most major quotes that came to mind.) And I figured, that should be clear enough. Nuh uh. I got a 6/10, purely because the text "wasn't there" which scared me straight. It's definitely possible to circumvent these issues, but you need to be aware of the risks when making implicit links.
Moral of the story: assume everyone's an idiot and spell things out as much as the piece allows (without getting too clunky.) Don't assume your marker will fill in the blanks or connect the dots for you, because they won't :/
Since the priority of any context piece is the ideas it deals with, find a few points of interest in the text that you can generalise, and practice applying them. Treat the SOI as an opportunity to reiterate the connection though, not to do all the work for you :)



paper-back:

What kind of fluency issues are you having? If it's something like run-on sentences, then shortening them is definitely an option. But is your writing confusing because of the words you use/ how you express your ideas, or because of what you're actually trying to say. It may just be an issue with grammar and syntax, or there might be underlying issues with your ideas and/or contention depending on the piece. Maybe ask the people (students? teachers?) who said your piece was confusing and ask them how & why they're confused.

Whatever their response is, a good place to start is just to de-clutter your intentions. What are you trying to say, in the simplest, most basic terms? eg. 'okay, I wanna say how we know the character is evil because of how the director casts her in dark lighting and accentuates her sharp features.' --> The director's use of dark lighting to accentuate character X's sharp facial features contributes to her mysterious and sinister persona.

Start simple, and build it up, rather than just beginning a sentence without knowing where it's going to end. This should ensure that at the very least, you're getting your ideas across. After that, you can gradually work up to more complex sentence structures and better vocab.



Floatzel98:

Tbh a lot of schools make up their own rules for this, so I'll tell you what I've found to be most often the case, but it's worth clarifying with your teacher as well.
The standard acronym for what you should include is FLAP-C : Form (essay, speech, diary entry, etc.) Language (ie. what is appropriate for an address to US Congress vs. a high school essay competition,) Audience (same as above, consider the situational context of your Context piece,) Purpose (explained below) and Contention (also below.)
Your contention is what you, as a Year 12 English student, are saying about the prompt. But the purpose should be more imaginative.
Take this example of a creative editorial in a WA newspaper. Let's say the Context was 'Life and Experiences' or something like that; this author's contention would be that taking a life is inexcusable, no matter the circumstances. But his purpose is to address the inhumanity of execution and the hypocrisy of trying to attribute blame in this particular case. He doesn't end every paragraph by saying 'Therefore a human life is very valuable, and we should not deprive anyone of their right to live.' Instead, he communicates it through language and ideas. THAT'S what you need to do in your piece.

Re: marks, that's totally up to the school. Some places assign 5 or 10 marks to the SOI alone, other teachers just say 'do it, I'll tick it, and we'll move on with our lives.' It won't have a drastic effect on your marks, but it can help clarify your intentions. I think I remember hearing that it won't cost you any marks, but it might earn you some, if your teacher is generous :)
Title: Re: VCE English Question Thread
Post by: KingDrogba on February 21, 2015, 03:29:49 pm
Currently doing identity and belonging for context, have a strong list of ideas which i can bring into an expository essay but was wondering if anyone would share some other issues/movies/books/news articles which i could write about? Specifically to do with crisis and how that affects identity and belonging

Thanks for the help!
Title: Re: VCE English Question Thread
Post by: literally lauren on February 21, 2015, 03:32:51 pm
This might help :) I haven't updated it in a while, but these ideas might help spring you into other territory.
Title: Re: VCE English Question Thread
Post by: KingDrogba on February 21, 2015, 03:42:18 pm
You're too good to me
Title: Re: VCE English Question Thread
Post by: scarletmoon on February 21, 2015, 10:32:12 pm
For context, do examiners prefer that the outside examples you use are Australian?
Title: Re: VCE English Question Thread
Post by: literally lauren on February 22, 2015, 09:01:18 am
For context, do examiners prefer that the outside examples you use are Australian?

The examiners have many weird preferences, but I don't believe this is one of them. Looking back, I didn't bring up a single Australian example in my exam since I had better stuff at my disposal. If you have Australian evidence that works really well then by all means go for it, but pick it because it's interesting, not because it's Australian :)

If your teacher is pushing you down this direction for the SACs though, you should definitely be researching his/her "recommendations," and then maybe do a bit of your own research on the side for the exam, if you think it's necessary.

Having said that, keeping it country-specific would only be a weakness for a pure expository essay. If you were going for a slightly creative/hybrid twist, like a news article, or a piece for an essay competition, then a thematic focus like 'what are the true Australian values in times of crisis?' or 'is the idea of an 'Australian identity' dangerously exclusive?' could be quite useful. Even then, though, you could zoom out and compare our nation with others where applicable - it's really up to you :)
Title: Re: VCE English Question Thread
Post by: scarletmoon on February 22, 2015, 03:19:07 pm
The examiners have many weird preferences, but I don't believe this is one of them. Looking back, I didn't bring up a single Australian example in my exam since I had better stuff at my disposal. If you have Australian evidence that works really well then by all means go for it, but pick it because it's interesting, not because it's Australian :)

If your teacher is pushing you down this direction for the SACs though, you should definitely be researching his/her "recommendations," and then maybe do a bit of your own research on the side for the exam, if you think it's necessary.

Having said that, keeping it country-specific would only be a weakness for a pure expository essay. If you were going for a slightly creative/hybrid twist, like a news article, or a piece for an essay competition, then a thematic focus like 'what are the true Australian values in times of crisis?' or 'is the idea of an 'Australian identity' dangerously exclusive?' could be quite useful. Even then, though, you could zoom out and compare our nation with others where applicable - it's really up to you :)

Ok thankyou :)
Title: Re: VCE English Question Thread
Post by: appleandbee on February 22, 2015, 04:37:24 pm
Hey there!

I have a couple of questions:

1. For the oral, my school does things differently-we do the oral in groups (individual marks) and that the entire cohort does the same topic (Freedom of Speech is regards to section 18C and D in the Racial Discrimination Act). Would my approach be any different as a result? Is there anything to look out for/be aware of?

2. For the context (Whose Reality-Death of a Salesman), my teacher prefers current examples (apparently it shows that we're well read...), but the problem is that I feel that because the context is quite abstract, it's difficult to incorporate current examples.

3. For the prompt 'It is sometimes easier to live in a world of illusion, than it is to face reality', how should I address the point  that ' It is easier to face reality, than to live in illusion'? I personally wouldn't say facing reality is easier, but is necessary in order to live a meaningful life.
Title: Re: VCE English Question Thread
Post by: Eiffel on February 22, 2015, 07:19:03 pm
hey. Im usually a very competent student (talking about Eng Lang for this question - essentially same as Eng), and have been getting A+s during my VCE. However we had a sac for eng lang, and it had an essay component which i totally stuffed up. I havent received my mark yet but my teacher is a realyl harsh marker which makes it worse and am expecting 50-60% (havent got this low since year 7 lol). I know how ranking and all that works and if she marks harshly for everyone, but say the average of the class is sub 70-80s, if i do well in all my other assessments is 45+ feasible?

I did a 3/4 last year and didnt stuff any SACs up, but messed the exam quite a bit and ended up with 46, so i think SACs and Rank 1 helped a little? not too sure...
Title: Re: VCE English Question Thread
Post by: literally lauren on February 22, 2015, 08:13:55 pm
1. For the oral, my school does things differently-we do the oral in groups (individual marks) and that the entire cohort does the same topic (Freedom of Speech is regards to section 18C and D in the Racial Discrimination Act). Would my approach be any different as a result? Is there anything to look out for/be aware of?
I'm not entirely sure what you're saying here... are you all conveying the same contention as a group? Like, does one person speak per minute or something? Apologies, but I've never head of this format before so I can't really help; could you give me more information?

Quote
2. For the context (Whose Reality-Death of a Salesman), my teacher prefers current examples (apparently it shows that we're well read...), but the problem is that I feel that because the context is quite abstract, it's difficult to incorporate current examples.
Do you have trouble incorporating examples because they're current, or because you don't know how to incorporate examples? :) You'd use current news events or situations the same way you would any other story, so I doubt that's it. If you're having issues with general integration, it might be because you're going from abstract discussion (ie. 'Reality is a manifold concept...') to close details (eg. Tony Abbot's comments about how Indonesia owe us for giving them foreign aid were very misguided) without actually connecting the dots. The best way to do this is via theorisation. If you're using something political, look at some political theory or philosophical ideas, as this can make the transition much smoother.

eg. Reality is a manifold concept often riddled with truth, lies, and everything in between. And although we may regard our world as one with a roughly even balance when it comes to these concepts, often there are parts of society where mistrust and deception run rampant. Nowhere is this more evident than in the shady world of politics, where contests of popularity outweigh moral decisions and overrule conscientious objections. This was seen quite recently when many Australians expressed utter indignation over their Prime Minister's assertion that Indonesia should be compelled to release two Australian citizens on death row because "his country" had offered Indonesia aid almost a decade earlier. By implying that our aid was conditional, Tony Abbot essentially isolated himself from many voters who see disaster aid as an ethical obligation, not a political power move that deserves indemnity or recompense. And yet these people are not politicians, for the most part. Their reality is one not plagued by the same knife's-edge posturing and broadcastable rhetoric that people in power experience. So even if we could objectively judge Abbot's views to be ill-expressed or distasteful, we cannot hope to cast judgement upon an individual without some comprehension of the context they are in.  Hence, although our realities may overlap with one another's, an awareness of why untruths or embellishments might be necessary can assist us in reconciling misunderstandings, and closing the gap between different realities.
General context-y stuff
Theorisation and transitioning
Example/evidence-based discussion
Not every essay/paragraph has to weave in and out in this order, but practice like this first so you know how to conduct a flowing discussion.
{Also that paragraph came out weirdly pro-Abbot, this is just because of the prompt and areas I chose to explore. The views in this colourful explanation do not necessary represent those of the creator :)}
Quote
3. For the prompt 'It is sometimes easier to live in a world of illusion, than it is to face reality', how should I address the point  that ' It is easier to face reality, than to live in illusion'? I personally wouldn't say facing reality is easier, but is necessary in order to live a meaningful life.
What do you mean 'how do I address the point?' You've got a contention that's totally based on the prompt. You're challenging the prompt, and going against what it is suggesting, but this is totally fine; in fact, I'd probably recommend this over just churning out a long list of evidence to back up the prompt without any development beyond 'yup, I agree.'
Even if you don't necessarily agree with what you're writing, it can be useful to argue against yourself and build up the strength of your piece in that manner. You're more likely to find the flaws or logic-gaps in your writing if you're in this hyper-critical mode anyway.



hey. Im usually a very competent student (talking about Eng Lang for this question - essentially same as Eng), and have been getting A+s during my VCE. However we had a sac for eng lang, and it had an essay component which i totally stuffed up. I havent received my mark yet but my teacher is a realyl harsh marker which makes it worse and am expecting 50-60% (havent got this low since year 7 lol). I know how ranking and all that works and if she marks harshly for everyone, but say the average of the class is sub 70-80s, if i do well in all my other assessments is 45+ feasible?
If I said no, would you give up? Would you totally stop trying for English? Would you quit VCE and life and say goodbye to your family and run away to Mexico?
I understand that basically everyone on this website has numerical goals; perhaps even numerical prerequisites, but using them for anything other that study motivation is a dangerous pit to fall in.
Is a 44 worthless in your eyes? What's the difference between a 45 and a 46? If you knew you could put in all the effort in the world and still only get a 44, would you just not bother?

Truth be told, it's February. You've been at school for, what, three or four weeks? Nothing you could have done by this point will totally negate the possibility of you scoring highly.

I'm sure other people could jump in and tell you exactly how the system works and why a few percentages on each SAC are ultimately evened out, or even nullified, but I'm not going to do that because I barely understand the system because understanding the system isn't necessary knowledge.

It's comforting knowledge. It can console you when things don't always go to plan, but if you buckle down and focus on qualitative improvement, not quantitative, I guarantee you'll be pleasantly surprised with the results.

Don't let the numbers get you down, man :)
Title: Re: VCE English Question Thread
Post by: scarletmoon on February 22, 2015, 10:11:19 pm
What are some events that have happened in Australia that are applicable to "encountering conflict". Other than Bali nine or asylum seekers, I'm looking for something that's a little rare idk if that made sense...
Title: Re: VCE English Question Thread
Post by: literally lauren on February 22, 2015, 10:34:06 pm
Conflict is everything.

For something to happen, there must be conflict. Hence, everything is conflict, QED.

And even if this wasn't true, you can say just as much about the significance of the absence of conflict as its presence.

I think I've said this before in some other format, but basically: things happen, which must involve some form of conflict (even if it's only an early or late stage, like causes or resolutions.) This causes a change in who we are (identity) which in turn affects how we see ourselves, others, and the world around us (belonging // imaginative landscape.) And all of this happens in our subjective realities (whose reality.)

So all of the context areas of study can relate to pretty much anything. Some of the connections will be more strained than others (if you look at the context examples in the Resources thread - link is in one of the above comments as well - each example has between one and four of the context listed beside it. This doesn't mean an example would be invalid for the contexts not mentioned, just that you'd have to do a lot more work to get these to fit, and it might only be applicable for a few select prompts.)

If you're just looking to browse and don't want to read entire newspapers, the Google News app/site/thingo is a good way of targetted browsing. You can limit by publications (eg, no Herald Sun pieces) or by country (eg, only Australian publications) or by recency/ relevance (eg. only stuff from 2013 onward, and only pertaining to Australian politics, not international politics) and of course by keyword (eg. you're keen on science and want to write about genetic experiments, type 'genetics' and it'll spit back the most relevant stuff.)

Once the Examples Guide is updated, there'll be a list of similar resources up the top which should help with the research process. If nothing on that list at the moment takes your fancy, maybe start with your set text, extrapolate some ideas, and then think about how they might be relevant for our society.

You can almost turn it into an occasionally fun and challenging game of relating everything back to your context.
eg. My boyfriend is drinking a cup of tea, but he has no idea where those tea leaves came from, or how many hundreds of miles they've traveled only to be diluted in his dorky mug. Perhaps those leaves are the result of slave labour. He could be enabling conflict in the form of immense suffering without even realising it --> Sometimes it's not the deliberate acts of cruelty, but the unintentional actions and mistakes that have a profound impact in times of conflict  ;)
Title: Re: VCE English Question Thread
Post by: appleandbee on February 23, 2015, 12:32:30 am
I'm not entirely sure what you're saying here... are you all conveying the same contention as a group? Like, does one person speak per minute or something? Apologies, but I've never head of this format before so I can't really help; could you give me more information?
 

We've never done individual orals in our school, even for year 11. Everyone speaks for 3-4 minutes on a different aspect of the issue, discussing the different viewpoints as a group. As everyone’s doing the same issue, it will probably be difficult to separate people arguments-wise. It’s the first part of a two-part SAC, the second part being a language analysis on the same issue.


 Do you have trouble incorporating examples because they're current, or because you don't know how to incorporate examples? :) You'd use current news events or situations the same way you would any other story, so I doubt that's it. If you're having issues with general integration, it might be because you're going from abstract discussion (ie. 'Reality is a manifold concept...') to close details (eg. Tony Abbot's comments about how Indonesia owe us for giving them foreign aid were very misguided) without actually connecting the dots. The best way to do this is via theorisation. If you're using something political, look at some political theory or philosophical ideas, as this can make the transition much smoother.

eg. Reality is a manifold concept often riddled with truth, lies, and everything in between. And although we may regard our world as one with a roughly even balance when it comes to these concepts, often there are parts of society where mistrust and deception run rampant. Nowhere is this more evident than in the shady world of politics, where contests of popularity outweigh moral decisions and overrule conscientious objections. This was seen quite recently when many Australians expressed utter indignation over their Prime Minister's assertion that Indonesia should be compelled to release two Australian citizens on death row because "his country" had offered Indonesia aid almost a decade earlier. By implying that our aid was conditional, Tony Abbot essentially isolated himself from many voters who see disaster aid as an ethical obligation, not a political power move that deserves indemnity or recompense. And yet these people are not politicians, for the most part. Their reality is one not plagued by the same knife's-edge posturing and broadcastable rhetoric that people in power experience. So even if we could objectively judge Abbot's views to be ill-expressed or distasteful, we cannot hope to cast judgement upon an individual without some comprehension of the context they are in.  Hence, although our realities may overlap with one another's, an awareness of why untruths or embellishments might be necessary can assist us in reconciling misunderstandings, and closing the gap between different realities.
General context-y stuff
Theorisation and transitioning
Example/evidence-based discussion
Not every essay/paragraph has to weave in and out in this order, but practice like this first so you know how to conduct a flowing discussion.
{Also that paragraph came out weirdly pro-Abbot, this is just because of the prompt and areas I chose to explore. The views in this colourful explanation do not necessary represent those of the creator :)}

I find current political issues difficult to write about because they usually don’t provide me with much depth for discussion compared to philosophical theories and interesting stories. I was also unsure how to tie in current political issues with abstract discussion about reality (but you explained it pretty much).

Also how can I avoid my essay from becoming too philosophically dense? I plan of writing hybrid-feature article style. Most of my examples are philosophy based-Descartes, Schopenhauer/Buddhism, Iris Murdoch, Plato, surreal art-Salvador Dali, North Korea etc. Should I contextualise the philosophical theories with real world examples?

Another note about examples, should they strictly complement the book (Death of a Salesman), because I’m wary that my examples may be introduced in an awkward way and not blend in?

What do you mean 'how do I address the point?' You've got a contention that's totally based on the prompt. You're challenging the prompt, and going against what it is suggesting, but this is totally fine; in fact, I'd probably recommend this over just churning out a long list of evidence to back up the prompt without any development beyond 'yup, I agree.'
Even if you don't necessarily agree with what you're writing, it can be useful to argue against yourself and build up the strength of your piece in that manner. You're more likely to find the flaws or logic-gaps in your writing if you're in this hyper-critical mode anyway.


I wasn't challenging the prompt. It’s just that the word ‘sometimes’ suggests that ‘living in an illusion is easier than reality’ may not always be the case. I’m unsure of scenarios where facing reality is easier than living in an illusion because it may be better, but not necessarily easier.

Sorry for bombarding you with questions :P

Thanks so much! :D
Title: Re: VCE English Question Thread
Post by: cosine on February 23, 2015, 04:13:01 pm
Hey everyone

I have my first english sac on a text response on a play we read in class. What should i do right now to prepare please i feel like im lost :(
Title: Re: VCE English Question Thread
Post by: Apink! on February 23, 2015, 04:43:33 pm
Hi,
for an expository essay, and if the prompt was : the powerful changes reality,
Would you structure your essay like this:

1st paragraph: generalisation

Body paragraph1 - contention: powerful does change reality + evidence
Body paragraph 2- contention: powerful can only change exterior reality of someone but cannot change one's personal, interior reality +evidence
Body paragraph 3- contention: Sometimes it is not even clear who is more powerful +evidence

Long story short, would you have a contention you are trying to make for each paragraph and put evidence in for each?
Or would you have 1 main contention that you are trying to make and use the paragraphs to put in your evidence?

E.g
Main contention: powerful can only change exterior reality of someone but cannot change one's personal, interior reality

1st body paragraph: George Orwell's 1984 (example only, no separate contention)
2nd paragraph: Shark Net
3rd paragraph: something else

Also how would you do a conclusion for expository?
Title: Re: VCE English Question Thread
Post by: literally lauren on February 23, 2015, 08:09:52 pm
We've never done individual orals in our school, even for year 11. Everyone speaks for 3-4 minutes on a different aspect of the issue, discussing the different viewpoints as a group. As everyone’s doing the same issue, it will probably be difficult to separate people arguments-wise. It’s the first part of a two-part SAC, the second part being a language analysis on the same issue.
In that case, your approach won't be too different from normal English orals, but you will have to work closely with your group to ensure your arguments work well together as well as individually. It's worth chatting to your teacher as well, since I'm not sure whether they'd expect it to all flow like one 15 minute long speech from different people, or you hand over to one another (eg. 'And now Jacob will talk to you about the wider ramifications of this issue on a global scale...') or whether everything should be self-contained. But it sounds like you'll be able to do your own thing within a designated area (eg. just looking at the financial side of things, or the ethical/moral components of the issue) and so long as your contention is in harmony with the rest of the group, you should be fine.

Quote
I find current political issues difficult to write about because they usually don’t provide me with much depth for discussion compared to philosophical theories and interesting stories. I was also unsure how to tie in current political issues with abstract discussion about reality (but you explained it pretty much).

Also how can I avoid my essay from becoming too philosophically dense? I plan of writing hybrid-feature article style. Most of my examples are philosophy based-Descartes, Schopenhauer/Buddhism, Iris Murdoch, Plato, surreal art-Salvador Dali, North Korea etc. Should I contextualise the philosophical theories with real world examples?
Yes, exactly. You can only talk about theories for so long without it becoming dense and detached. Almost all good philosophical concepts will have links to real world examples, even if they're only hypothetical, but what you've listed here should be fine. The trick is to 'abstractify' things, so that you're not just talking about Salvador Dali's life, you're talking about how escapism can be more productive than mere avoidance, for instance. Not everything has to have a wealth of context and evidence, but it all has to be connected in a satisfying way. If you're doing a whole lot of theorisation with not a lot to back it up, that might be problematic.

Quote
Another note about examples, should they strictly complement the book (Death of a Salesman), because I’m wary that my examples may be introduced in an awkward way and not blend in?
If you're clever enough with your transitions then not all of them have to be. Remember, the priority is always the ideas here, so you need to find some common thread between DoaS and whatever else you want to discuss. It doesn't have to be very detail-oriented (eg. this character in the text was born in Ireland - Iris Murdoch was also born in Ireland!) Relate stuff back to the context and just weave your way around.
Having said that, it might pay to have some very closely related examples as your starting point so the transition isn't too severe. Most texts will either have real world comparisons to draw from, or an interesting backstory behind the author, or both, so maybe start there.

I wasn't challenging the prompt. It’s just that the word ‘sometimes’ suggests that ‘living in an illusion is easier than reality’ may not always be the case. [/quote] What you've described there is a challenge; you don't have to outright disagree with the entire prompt in order to challenge it. Challenging is more like complicating the discussion and making it more sophisticated, rather than denying some truth in the text or anything.
Quote
I’m unsure of scenarios where facing reality is easier than living in an illusion because it may be better, but not necessarily easier.
Sorry, this sentence has lost me... are you arguing that facing reality is easier, or isn't??



Hey everyone

I have my first english sac on a text response on a play we read in class. What should i do right now to prepare please i feel like im lost :(
Unfortunately there is no 'do the activity an instantly get full marks' secret for English. I know it's frustrating when you don't have a textbook or a list of questions to work off of sometimes, but that's because you need to direct your study!
Let's take this SAC for example; what's your absolute worst case scenario? What's the worst thing that could possibly come up on the SAC that would just make you weep and wail?
Maybe your answer is 'that I won't know enough quotes.' --> Go learn quotes then.
Maybe it's 'I can't say anything about this major character' --> Go read and write about that character.
Maybe it's 'I'm scared I won't know where to start' --> Practice some essay plans and introductions for unseen material.
Maybe it's all of those things, maybe you'll have a list of 29 different areas that you feel really weak and unconfident in, but a list of 29 definitive things to work on is better than just sitting there asking 'what do I do?' :)



Hi,
for an expository essay, and if the prompt was : the powerful changes reality,
Would you structure your essay like this:

1st paragraph: generalisation

Body paragraph1 - contention: powerful does change reality + evidence
Body paragraph 2- contention: powerful can only change exterior reality of someone but cannot change one's personal, interior reality +evidence
Body paragraph 3- contention: Sometimes it is not even clear who is more powerful +evidence

Long story short, would you have a contention you are trying to make for each paragraph and put evidence in for each?
Or would you have 1 main contention that you are trying to make and use the paragraphs to put in your evidence?
The second and third B.P.s are looking good, but the first one seems a bit weak. These paragraphs will be a few hundred words long each, and do you really want to devote a third of your discussion to a point as simple as 'power changes reality?' Your other points have more depth , but this one is essentially just saying 'yes' to the prompt, so you want it to be a bit more impactful.
In my experience, the first paragraph is an excellent place to clarify the definitions in the words/phrases you're using, especially for context where words like 'conflict' and 'reality' and 'power' get tossed around in essays so much that no one knows what they mean any more. Waaaaay too many essays at the end of the year will simply take a key word and run with it, never explaining what it means.
I'm NOT saying you should have an atrocious sentence like 'Webster's dictionary defines power as...' because that would be awful. But clarifying yourself can be done really effectively with synonyms. For example: 'The way we exert power and influence over other people can have a tremendous influence...' By adding that extra word 'influence,' I've clarified that I'm talking about social power and control over other people, and doing this once a paragraph or so will make things a lot clearer than branding everything as 'power' with no distinctions or complexity.

Quote
E.g
Main contention: powerful can only change exterior reality of someone but cannot change one's personal, interior reality

1st body paragraph: George Orwell's 1984 (example only, no separate contention)
2nd paragraph: Shark Net
3rd paragraph: something else
Now this is a slightly different problem. I'd say your first plan looks stronger because it's driven by ideas. This outline (and I know it's only a brief skeleton) is too limited by its evidence. From a Context perspective, it's much easier to go into a paragraph knowing what you're trying to say, than knowing what examples you'll draw from. Think about how much direction 'the powerful can only change exterior reality of someone but cannot change one's personal, interior reality' gives you as opposed to 'I'm going to write about Shark Net.'
Also, and this is a matter of personal preference, but I'd argue the stronger expository pieces will draw from more than one example per paragraph. It's not compulsory, and there are definitely examples that deserve their own independent paragraph of exploration, but your ability to compare, contrast, and draw parallels between different kinds of conflict gives you a lot more opportunities for an overall sophisticated contention.
Quote
Also how would you do a conclusion for expository?
To not lose credit: summarise your contention and main arguments, and end on a satisfying note.
To gain credit, try and make a bigger point. The main question a conclusion has to answer is 'so what?' What is the significance of the fact that power is important for changing reality? What does this mean about the nature of reality? It can take a lot of practice, but essentially you want to be zooming out as much as possible and say something impactful about your Context. It doesn't have to massively alter the way the reader thinks about reality, but it should be a concise summation of what you've explored, and the overall significance of the implications of your argument.

If that sounds vague, that's because it is. You can do anything you want in a conclusion, because a Context piece doesn't necessarily have to be an essay. It's convention to summarise at the end, but you could do this in many ways: with a creative example, a full-on imaginative POV from a character, narrative reflection, book-ending with a cool metaphor, whatever works! Maybe gauge what your teacher is a fan of and then experiment a little bit :)
Title: Re: VCE English Question Thread
Post by: appleandbee on February 23, 2015, 09:53:51 pm
Thanks so much for the detailed and straight to the point responses.  :D :D :D

I'm probably going to focus my hybrid piece (feature article in New York Times or something like that) on the American Dream (the ideas behind it), so hopefully my other examples can somehow flow from there.
Title: Re: VCE English Question Thread
Post by: Apink! on February 24, 2015, 09:46:30 am
Hi Lauren,
I wrote an expository essay on the prompt: "powerful control reality". I'm pretty sure it's horrendous at the moment (1st time writing one! :))
Could you have a look at it and give me honest criticism?  :) I'm sure I have a lot to improve on the linking with the context, fluency and a lot of things. Feel free to edit it and you can insert sentences/ phrases that would improve it.

Could you also do a sample conclusion for me that relates to the bigger picture?

 "first paragraph is an excellent place to clarify the definitions in the words/phrases you're using, especially for context where words like 'conflict' and 'reality' and 'power' get tossed around in essays so much that no one knows what they mean any more. Waaaaay too many essays at the end of the year will simply take a key word and run with it, never explaining what it means." Could you give me an example of this? :)

Please be really super harsh. Also, the text I am studying is Shark Net at the moment, so have I mentioned it enough in the expository? Should I be aiming for 5o: 5o ratio?

Powerful control reality:

Once it was within my sight, I could not resist her longing gaze, her transparency. Her sleek body gleamed under the dingy light, illuminating two creamy, butterscotch biscuits bound eternally by the sticky, fresh raspberry jam that made any child sigh dreamily. It was two in the morning, and my body was stiff and my lips were dry from cleaning the orphanage from five in the morning. I dressed the younger kids in starched, colourless clothes, swept the floors, polished the vase and replaced the flowers. My hands drew to the jar that sat coyly on the Master William’s mahogany desk like a magnet.  Grabbing one I hastily stuff it in my mouth and innocently resume to work. As I was munching quickly, my eyes find Madame Dowell and I freeze in shock. “No, Carolina.  Stealing is a sin, worse than adultery”. I was put outside to sleep that night. Next morning, I caught her eating from a jar. “Ahh, nothing more like a sweet treat to liven up the morning”. I gaze at her with faint distaste. “Go along, Carolina” Somehow, stealing from the jar was alright this time.

“powerful” figures around the world alter and manipulate their accounts of events to justify their actions or to prevent people from seeing the harsh reality.  Kim Jong Eun restricts every part of civilian life in order to keep them oblivious to the freedom they deserve, while the US falsified their progress information in the Vietnam War to the US civilians, to prevent them from hearing about embarrassing defeats.  However, on a smaller scale  parents are the most domineering figures in a child's life. Under the assumption that their child may take dramatic turns in life in exposure to confronting and promiscuous events, many parents restrict a child's reality of the world. This brings up Robert Drewe in the Shark Net. His father's ritual burning of the 'Mirror', as well as his revulsion towards  public display of anything that would be sexually suggestive that often surfaced outside through his occasional 'coughing fits', Drewe's curiosity and eagerness of this unentered territory grows feverishly in adolescence. This may have been a contributing factor to his early-received paternal role to a son because of his distorted world-view stemming from his parents' reality on intimate relationships.

However, it is not to say that dominant figures “control” reality because there are multitudes of realities to be considered. While the influenced individuals could uphold an outer veneer of a certain reality required from them from a higher power, essentially their personal, interior reality of the society and the world is often stagnant during lifetime. For example, in George Orwell’s 1984, a dystopian novel depicting a society under an utmost power of the Big Brother, the protagonist of the novel, Winston Smith looks no different from any other brain-washed citizens of the society. He works under the government institution responsible for erasing and editing past history for the benefit of the Party and uphold the outer appearance required to prevent being punished. While the “powerful” have encouraged him to adopt a persona and accept this suffocating reality of the society, it essentially does not “control” his reality as he believes, personally and privately that there is something that must be corrected in this society; his illegal diary keeping and search for the Brotherhood  attests this. Similarly in Shark Net, Eric Cooke is bound by the necessity to abide by the social rules as an individual working and living in a human society. As a result he is forced to fabricate his reality of the world by smiling and being friendly to his boss’s son, Robert. However, this social obligation does not “control” his own reality of the world, which was a severe resentment against society. 

While the more controlling individual may impact on another’s reality, sometimes it is not even clear who is more “powerful” one. In Enduring Love by Ian McEwan, Jed Parry upon encountering Joe Rose on a fatal balloon accident believes that the world is revolving around his imaginary love affair with Joe. Parry is suffering from De Clerambault’s syndrome which makes him to have intense delusions that Joe is in love with him   However, Joe, in response to this stranger’s obsessive demonstration of his undying love with letters, stalking and calling, cannot help but act detached, indifferent and quite cold. Jed accuses him for “playing” with him and lying for his amusement and cries “You are very cruel… But you’ve got all the power” In Jed’s mind, suffering from this condition Joe has the ability to transform him from his content self to an aggressive, messy fit. However, the continuous harassing from Jed eventually changes Joe as well- his marriage with Clarissa breaks down and he buys guns to protect himself from Jed’s capricious behaviour that often turns into life-threatening violence. Jed’s behaviour ultimately changes Joe’s reality on morality as well as his once happy marriage. They both change each other, but in this case the powerful figure is not so apparent.


[conclusion]

I know I haven't used a lot of external sources, but what else could I have used?
Also, if the prompt was: A child's world is shaped by their parent's reality
What kind of "big" ideas could I discuss, I don't think I would be able to produce a nice piece on this prompt (if I was given it on the day of the SAC) without repeating I said over and over again or just stating a few examples to support the same contention in the whole essay :-[

Thank you!!! ;D (so many questions :P)
You are a lifesaver

Title: Re: VCE English Question Thread
Post by: Callum@1373 on February 24, 2015, 10:05:30 am
Hey lauren  ;D I'm unsure on what makes up the paragraphs for an expository essay.

Like for T.R, it's pretty much 1 paragraph = 1 point that justifies contention.

Say you had a prompt for an expository piece that was: 'There are such things as colours'

What would be a better approach:

BP1: There are red colours
BP2: There are green colours
BP3: There are blue colours

I hope that weird analogy makes sense  :-\

------------------------------

or

BP1: Explaining how t.v was black and white in the olden days
BP2: Then talk about the history of scientists researching the electromagnetic spectrum and finding the wavelengths of visible light
BP3: Summarize what colour is
Title: Re: VCE English Question Thread
Post by: cosine on February 24, 2015, 03:46:06 pm
Where can I post my practice essay for someone kind to read? Thanks
Title: Re: VCE English Question Thread
Post by: heids on February 24, 2015, 04:11:56 pm
Where can I post my practice essay for someone kind to read? Thanks

English Work Submission and Marking
(and details on the way to post the essays)
Title: Re: VCE English Question Thread
Post by: cosine on February 24, 2015, 04:31:54 pm
Thanks bangla_lok

What elements do we need to include in the introduction of a text response?

I know we must include the author, title of the book and etc.. but how exactly do we embed these? Thanks
Title: Re: VCE English Question Thread
Post by: paper-back on February 24, 2015, 05:26:26 pm
Is it bad to memorise a good introduction and keep using that throughout all your text responses whilst just changing the arguments?
Title: Re: VCE English Question Thread
Post by: Paulrus on February 24, 2015, 05:59:19 pm
Hey lauren  ;D I'm unsure on what makes up the paragraphs for an expository essay.

Like for T.R, it's pretty much 1 paragraph = 1 point that justifies contention.

Say you had a prompt for an expository piece that was: 'There are such things as colours'

What would be a better approach:

BP1: There are red colours
BP2: There are green colours
BP3: There are blue colours

I hope that weird analogy makes sense  :-\

------------------------------

or

BP1: Explaining how t.v was black and white in the olden days
BP2: Then talk about the history of scientists researching the electromagnetic spectrum and finding the wavelengths of visible light
BP3: Summarize what colour is
I'd avoid using the first approach you outlined, because that seems similar to a lot of low-range text responses where people just say 'yes' to the prompt, and then use a different character for different paragraphs that all say the same thing. The way you'd choose your paragraph topics is honestly pretty similar to a text response. You're going to have a pervading contention, and then explore different facets of that within your paragraphs.
This is one I did towards the start-middle of last year if it helps:

'Some conflicts have a history that make them impossible to resolve'.
- Certain ideologies can become engrained in society’s collective psyche to the extent that it becomes impossible to challenge them. When people refuse to re-examine traditional perspectives simply because they are long-established, society itself begins to stagnate.
-Striking comparisons can be drawn between certain conflicts from completely different eras and cultures - the unfortunate implication is that any attempt at resolving conflict is ultimately fruitless, as it is continually destined to repeat itself.
- However, we are often able to learn valuable lessons through our history, allowing us to learn from past mistakes in conflict.

You get me? So you've got an overall contention (that tradition and history can be dangerous when they lead to complacency), and you're trying to examine different facets of the prompt. That way you're able to make pretentious sweeping statements about the human condition and score hella gud points with your assessors  ;)

Thanks bangla_lok

What elements do we need to include in the introduction of a text response?

I know we must include the author, title of the book and etc.. but how exactly do we embed these? Thanks
A very easy way to do it is to just start with a contextualising sentence that sums all that stuff up. In my exam I used "Informed by his own experiences as a soldier, Wilfred Owen's anthology 'The War Poems' elucidates/illustrates/demonstrates/whatever ______"
Your intro is the first impression you leave on the examiner, so you'll need to make it impressive, but the whole author and text name thing is just a structural requirement - don't worry too much about it, it's just kind of a box you have to tick. As long as you don't start your essay with "In [text] by [author], ____", you'll be fine :P

Is it bad to memorise a good introduction and keep using that throughout all your text responses whilst just changing the arguments?

Technically you could do this, but you'd need to make sure that you can do it in a way that sounds natural, because VCAA will crucify you if they think your writing is formulaic. In a standard intro though, you'd probably be better off just having a solid, malleable opening line that you can reuse and then continue from there. You don't really need to memorise a whole intro, but if you have a couple of phrases that you like using in every intro, then that's perfectly fine.
Title: Re: VCE English Question Thread
Post by: Callum@1373 on February 24, 2015, 06:52:04 pm
'Some conflicts have a history that make them impossible to resolve'.
- Certain ideologies can become engrained in society’s collective psyche to the extent that it becomes impossible to challenge them. When people refuse to re-examine traditional perspectives simply because they are long-established, society itself begins to stagnate.
-Striking comparisons can be drawn between certain conflicts from completely different eras and cultures - the unfortunate implication is that any attempt at resolving conflict is ultimately fruitless, as it is continually destined to repeat itself.
- However, we are often able to learn valuable lessons through our history, allowing us to learn from past mistakes in conflict.

You get me? So you've got an overall contention (that tradition and history can be dangerous when they lead to complacency), and you're trying to examine different facets of the prompt. That way you're able to make pretentious sweeping statements about the human condition and score hella gud points with your assessors  ;)
OK YES!!!!!! I have to ask you about the parts that I highlighted in red  ;D

So the prompt is saying pretty much that there are conflicts that because of past circumstances they cannot be resolved.
Then in your contention you give the implications of the prompt, that because some conflicts in history are complacent, it means unresolved conflict which is dangerous.

However, that third point is somewhat related but unrelated to the contention. What I mean is that nowhere in the prompt does it talk about being able to learn from past mistakes in conflict. But, learning from past mistakes from conflict is an implication, or facet of the prompt. It's an idea that is generated from the prompt, but isn't in the words of the prompts itself. That's why i made the words 'different facets' in red as well.

Because that last point is a facet of the prompt, like the prompt is a clue to that third point, but it is not explicitly stated in the prompt.

What i'm trying to say is, and i'm sorry but it's really hard to explain, is that what you have are three ideas that are somewhat suggested by the prompt, not written in big bold letters in the prompt. And that's what makes an essay good, it's what I need to know more about.

So say the 'other' way of tackling the prompt will call method #1
The other advanced way you have shown can be called method #2

For a prompt, say 'Humans are inventors of new technology'

#1 would be

#2 could be something more like

Ahh i really hope you can see what I mean. Is this the better way to do it? As in #2 is just a lot more exploration, and it is not just a bunch of points that re-word the prompt. But is this way of writing the essay too broad? Should it simply be three points that justify the prompt? Because with the example you gave, it seemed as if that third point was just an implication of the prompt, and not clearly written in the prompt at all. And saying that we can learn 'valuable lessons' doesn't seem to be told by the prompt at all, but is a well-written idea that you have thought of  :)

Sorry to drag on, but my teachers at school are going to spend 1 LESSON on how to write an expository piece, which is utterly stupid to me  ::) ::) ::)
Title: Re: VCE English Question Thread
Post by: Paulrus on February 24, 2015, 07:30:15 pm
I'm on my phone atm so sorry if this is kind of disjointed haha. If anything needs clarifying let me know!
OK YES!!!!!! I have to ask you about the parts that I highlighted in red  ;D

So the prompt is saying pretty much that there are conflicts that because of past circumstances they cannot be resolved.
Then in your contention you give the implications of the prompt, that because some conflicts in history are complacent, it means unresolved conflict which is dangerous.
Well it's not so much the conflicts themselves that are complacent, it's more the fact that an over-reliance on tradition can make us complacent in our ideologies. Like, if we just accept things as being right because they're long established, that can be dangerous cos we're not reexamining those beliefs - therefore that history can make it harder to resolve conflicts. If that makes sense :P



However, that third point is somewhat related but unrelated to the contention. What I mean is that nowhere in the prompt does it talk about being able to learn from past mistakes in conflict. But, learning from past mistakes from conflict is an implication, or facet of the prompt. It's an idea that is generated from the prompt, but isn't in the words of the prompts itself. That's why i made the words 'different facets' in red as well.

Because that last point is a facet of the prompt, like the prompt is a clue to that third point, but it is not explicitly stated in the prompt.

What i'm trying to say is, and i'm sorry but it's really hard to explain, is that what you have are three ideas that are somewhat suggested by the prompt, not written in big bold letters in the prompt. And that's what makes an essay good, it's what I need to know more about.
Haha sorry, I didn't really make the link explicitly clear in the topic sentence. It would have been more obvious in the essay but I don't think I have that any more.
Basically the idea is that learning from the past mistakes teaches us important lessons that actually assist in conflict resolution in the future - i.e. I'm basically saying "HOWEVER" and kind of challenging the prompt head on here by saying that history can actually make conflicts easier to resolve.
Try to pick apart the prompt and I think it makes it a lot easier to recognise the implications. For example, with this prompt you could ask yourself first off about the different kinds of conflict and how they might be affected by history (e.g lingering racial tensions from one event fuelling another, or the fact that the belief that the sun revolved around the earth had existed for so long that people refused to accept the heliocentric model, resulting in conflict of ideology). You could maybe think about the word resolve - how is conflict usually resolved, and how can this history hinder that? Are there instances where history can actually facilitate resolution?
Basically, the more questions you ask yourself about the prompt, the more implications you'll be able to draw out of it. It might take a while to get used to but it could help you come up with some sophisticated ideas
So say the 'other' way of tackling the prompt will call method #1
The other advanced way you have shown can be called method #2

For a prompt, say 'Humans are inventors of new technology'

#1 would be
  • We invented a telephone
  • We invented a lightbulb
  • We invented a calculator

#2 could be something more like
  • Humans through science over time have learnt how to take old technology and make it better
  • Technology takes much time and effort to make it better
  • Inventing technology, like fashion, is a non-stop process

Ahh i really hope you can see what I mean. Is this the better way to do it? As in #2 is just a lot more exploration, and it is not just a bunch of points that re-word the prompt. But is this way of writing the essay too broad? Should it simply be three points that justify the prompt? Because with the example you gave, it seemed as if that third point was just an implication of the prompt, and not clearly written in the prompt at all. And saying that we can learn 'valuable lessons' doesn't seem to be told by the prompt at all, but is a well-written idea that you have thought of  :)

Sorry to drag on, but my teachers at school are going to spend 1 LESSON on how to write an expository piece, which is utterly stupid to me  ::) ::) ::)
Haha I think approach 1 there is too narrow, but approach 2 is a bit too broad. I feel like the prompt itself is incredibly narrow though, and there's not really much room for exploration. Don't worry, you won't ever get anything that bad in VCE haha
The best way to approach that prompt would probably be to look at the way those concepts are interrelated. Like, the prompt is telling you that humans invent new technologies. I think the second paragraph might be a bit too tangential but the others work well - you could talk about how the invention of new technology is reflective of social progression and the betterment of humanity, etc. With that prompt you'd have to extrapolate heaps though because the prompt isn't really giving you much to work with at all. But yeah, just remember that you'll never get anything that bad in vce :P
Hopefully that helps a bit! Again I'm on my phone so this is a bit rushed, sorry. If Lauren or someone else has anything to add please do!
Title: Re: VCE English Question Thread
Post by: Callum@1373 on February 24, 2015, 08:34:51 pm
Ahh don't worry. I was just using that technology crap and the conflict as an example to show how I don't understand what you are meant to do with a context prompt. I can't figure out whether you answer a prompt as in yes i agree/disagree or if your meant to use it as a 'springboard' for ideas, but 'springboard' doesn't make sense to me. I don't get whether we are meant to make assumptions and explore aspects of it, as in draw inferences from it or if it is like asking does 1 + 1 =2, obviously yes.....
Title: Re: VCE English Question Thread
Post by: literally lauren on February 25, 2015, 10:46:38 am
Callum@1373:
I know Paulrus has addressed your question really well, but just with regards to your 'methods'
For a prompt, say 'Humans are inventors of new technology'

#1 would be
  • We invented a telephone
  • We invented a lightbulb
  • We invented a calculator

#2 could be something more like
  • Humans through science over time have learnt how to take old technology and make it better
  • Technology takes much time and effort to make it better
  • Inventing technology, like fashion, is a non-stop process
The problem with method 1 is that it's evidence based. You haven't made a point on an idea-level, you've just listed a bunch of cases that conform to the prompt's inference. Although you're allowed to agree with the prompt if you want, doing so blindly isn't a good way of tackling the task.
The issue with this example is that it's something that could be definitively proven, whereas a context prompt would be more like:
'Technology comes with downfalls as well as benefits'
This is more open for debate. So you can't just list evidence of technology having downfalls; you'd need to start with ideas and move outwards:
eg.
1: Technology amplifies human connection, for better and for worse (eg. it enables us to help one another much more efficiently, but it also allows us to harass, bully, and oppress people in a way that wouldn't have been possible 40 years ago)
2: Certain facets of technology exacerbates existing problems or disadvantages (eg. the 'great divide' gets bigger around the world; certain internet sites or phone apps seem to be inherently more dangerous and harmful than others.)
3: Circumventing the problems surrounding technology is getting harder because it is so omnipresent in our lives (eg. even news bulletins are plastered with 'tweet us @...' or 'like our fb page')

You shouldn't be able to boil down your essay to a dot-point list of evidence or examples. The focus in Context is always  your ideas; the evidence is there to support you, not do the work in place of your exploration.

Unlike what you've done in method 2 though, you don't just have to take your exploration in three totally different directions. Try to have each paragraph build on previous explorations. One way of doing this is ensuring the first paragraph is very open, preferably clarifying some of the key words in the prompt. From there, you can start constructing and challenging your arguments.



paper-back:
A 'good introduction' will be one that deals explicitly with the prompt as soon as possible, not one that sounds nice because it's got some impressive words or sentence structures. So you could definitely have a few rough sentence structures or formulae up your sleeve, but don't make it too generic. Original attempts at engagement with the prompt will trump an impressive but irrelevant formula every time :)
Title: Re: VCE English Question Thread
Post by: scarletmoon on February 25, 2015, 04:43:46 pm
So I have this prompt " we need some degree of conflict or tension to experience to richness of being fully alive" and I can't think of any good ideas for this prompt pls help  :'(
Title: Re: VCE English Question Thread
Post by: literally lauren on February 25, 2015, 05:10:00 pm
So I have this prompt " we need some degree of conflict or tension to experience to richness of being fully alive" and I can't think of any good ideas for this prompt pls help  :'(

First step: do you not know what to write, or do you not know how to write it. Since you've said you don't have any ideas, I'll assume it's the former and answer that question, but I might come back to the latter at the end.

As mentioned in posts on the previous page, you only need ideas to get you started, not examples. So it's a matter of exploring the prompt to get a contention, and some arguments out.

Assuming you understand the key words in the prompt, you should begin by going with your gut feeling: yes or no. Do you think conflict is necessary in life, or not? We'll extrapolate more later, but in order to give your planning enough direction, pick a rough contention to work with for now.

Next step is to ask why you think this. If conflict is a necessity, how do you know? And if it isn't, why not? Don't think in terms of examples (eg. it is a necessity because, like, people who don't bring up concerns in relationships often end up unhappy.) You want to be focusing on the general ideas for your initial framework.

This is where your questioning of the prompt and its implications will come into play (instructions on the first page of this thread if you need.)

Pick apart as much as you can, from the simple definitions (eg. 'What does it mean for something to be 'needed' to experience being alive?') to the more complex inferences (eg. 'What degree of conflict/tension is needed for us to experience the richness of life, and what degree renders us unable to experience this?')

You might only end up using four of these questions in your essay, but the more you ask, the better your thinking becomes.
Most people start moving into arguments at that stage, or even divide up paragraphs if you're feeling confident enough.

Then finally you start consulting your repository of examples to work out what evidence best demonstrates your points. The text will have to comprise at least one of these, so start there, but be prepared to draw from external examples as well. If you haven't begun investigating these avenues of evidence, that might be what's tripping you up at the moment. It can be hard to brainstorm ideas without an understanding of their practical component.

Just because I'm compiling examples for this thread at the moment, and I'm feeling generous, consider looking into
- The aftermath of wars - is this usually an 'absence' of conflict?
- 'The Hedonic Treadmill' theory - what is 'necessary' in life, and do we appreciate the 'richness' of it?
- Apocalypse predictions (Y2K, 2012, Harold Camping) - to what extent is this 'encountering/experiencing' conflict?
- PTSD, anxiety disorders - when tension doesn't help us experience life, but rather detracts from our ability to do so

Hope that helps :)
Title: Re: VCE English Question Thread
Post by: cosine on February 25, 2015, 05:17:00 pm
Lauren, i need your help urgently.

I know the book, i know the prompt, i know the arguments, i know the quotes. However, i dont know how to put them into words. I sit here for hours, literally, whilst the ideas pop up in my head, but I CANT get them to paper, i just cannot find the appropiate words to sentence these ideas of mine :(
Title: Re: VCE English Question Thread
Post by: heids on February 25, 2015, 05:24:13 pm
Lauren, i need your help urgently.

I know the book, i know the prompt, i know the arguments, i know the quotes. However, i dont know how to put them into words. I sit here for hours, literally, whilst the ideas pop up in my head, but I CANT get them to paper, i just cannot find the appropiate words to sentence these ideas of mine :(

I'm not Lauren :) but experienced exactly that all last year (except maybe I didn't have the ideas either :P ); I've stared through tears at a blank page for hours on many occasions!

One thing I found really helped is dot-pointing essays.  Start writing a dot-pointed plan; then make it more and more detailed (in dot-points, doesn't have to be beautiful language - just dump quotes, appropriate vocab, ideas etc. in dot points).  Finally you should get to the point where you have 1 dot point per real sentence you would write in an essay.  Then, turn each dot-point into a flowing sentence. Going a sentence at a time, it's not so hard anymore.

It takes ages, but it's early in the year; this helps you get confidence and you can gradually wean yourself from it.  Next time, transform dot-points to sentences earlier (i.e. 1 dot point covering 2 sentences).

Don't expect yourself to be perfect yet.  You have many months!
Title: Re: VCE English Question Thread
Post by: cosine on February 25, 2015, 05:33:52 pm
Bangla_lok

Thank you heaps. I know i have time to perfect myself, but my sac is next week and thinking that a week before the sac, that i cannot even write a single sentence about the book is destroying me. You see, why can't i write? Oh... :(
Title: Re: VCE English Question Thread
Post by: heids on February 25, 2015, 05:49:01 pm
Thank you heaps. I know i have time to perfect myself, but my sac is next week and thinking that a week before the sac, that i cannot even write a single sentence about the book is destroying me. You see, why can't i write? Oh... :(

I just want to promise that I had EXACTLY the same issues (embarrassing admission - I even broke down and cried in my first essay SAC....).  When trying to write essays, I would get to the middle of say my first body para, and then just get stuck.  I would alternately cajole, yell, cry, tell myself it didn't matter, and bite my pen (or my hand :( ) really really hard.  It didn't work.  I just couldn't write another word.  And yet every other subject I found so easy!

Take comfort; two weeks before my last SAC (50% of unit 4) I couldn't write an essay.  Had I tried the SAC that week, I would have got a 0, I am absolutely certain; I would have just broken down and not written a sentence (I had done that in my final year 11 exam :'( ).  I finally said to Mum - 'If this goes on, I will fail English.  Can you MAKE me sit there until I have written an essay?'  So, she did.  I took 4 hours on that essay - crying and pleading her so hard to let me off - but it was finally finished.  We repeated it every day; in my one-hour unseen SAC, I got 80%. 

In fact despite all my SAC average was 85%, 2nd in weak cohort.  For me, it worked out (somehow) though mainly because I got lovely examiners :P

All the best!  I wish I could convey how absolutely terrified and stuck I was with English - but words are simply inadequate.  Confess it openly to your teacher/parents/friends/anyone who can help, and just write.  Something, anything, no matter how rubbishy it is, no matter HOW long it takes.  Don't stress at the start if you keep repeating the same vocab and your sentences sound clunky. And dot-points really really do help get up a bit of confidence.  You will master yourself!

P.S. Such melodrama! :P
Title: Re: VCE English Question Thread
Post by: literally lauren on February 25, 2015, 05:51:37 pm
Lauren, i need your help urgently.

I know the book, i know the prompt, i know the arguments, i know the quotes. However, i dont know how to put them into words. I sit here for hours, literally, whilst the ideas pop up in my head, but I CANT get them to paper, i just cannot find the appropiate words to sentence these ideas of mine :(

Alrighty, that was good timing :P So unlike the above, this is a 'I know what to say but not how to say it' case.

For starters, are you able to communicate things on a very basic level? ie. using very simplistic language and basic sentence structure? I'm assuming since you have some idea of the arguments that this is possible, so next: could you communicate this verbally? Some people find it easier to convey concepts by speaking than they would writing it all out, and it means you don't have to worry about formal essay structures or anything. If you find yourself stumbling here, then it probably is an issue with your understanding of the content. Revisiting some study guides or sample analyses/essays will probably help you sort your approach out.

Alternatively, it might be an issue with the vocab you have at your disposal. If you find yourself struggling to think of synonyms to reword your thoughts, or otherwise unable to articulate your interpretations and ideas, you'll need to build up the language to do so. Start with the most basic form of expression (eg. character X does very bad things, which makes the audience feel like they don't like the character) and then, use every English student's best friend: thesaurus.com to explore synonyms of 'bad.' Don't treat this as a test, or a very quick, once-off exercise. When the thesaurus spits back a nice word that you think fits, look it up in dictionary.com; explore synonyms of that word; think about hoe using another word would change the meaning of the sentence (eg. character X does malicious things vs. character X does repulsive things - these don't mean the same thing.)

Do this more and more, and you'll find the same words keep cropping up over and over again. These are the kinds of words you want to be keeping track of in a workbook or notepad, and as you accumulate more synonyms, start interchanging basic words for sophisticated ones. But the only way to improve your confidence with these words is to find, document, remember, and practice implementing them. It can be a long process, but it's worth it in the long run, and the sooner you get started, the better.

Finally, if you're having issues with the actual essay writing part, then an extensive plan is probably your best bet. The process bangla_lok has outlined is excellent; start small and work up to big arguments.

Don't panic about not being able to just churn out 1000 words of magnificent prose just yet; you're only at the start of your studies, and you'll have to do a whole lot of fine-tuning and exploration throughout the year. The students who are able to write an essay with no hesitation or qualms are probably doing something wrong; you''re meant to have these troubles nice and early so you learn how to combat them before it's too late!

...that was a bit melodramatic... I meant before you get to a SAC or exam and don't have the resources to ask questions, or the time to contemplate your approach :) It's never really too late until ~30th of November or whenever the exam is this year.


edit: if it's of any comfort, you might not be the type of learner who benefits from churning out essays as a means of practicing. I would rarely even start writing pieces until a week before my SACs at the earliest, because I simply needed more time for my conceptual understanding to ferment and develop. In the meantime, I'd read heaps of analyses and other people's essays until I was completely familiar with the task and what I needed to do to get it right. But if I were to try writing an essay only a few weeks into studying the text, I'd either hit a total mental roadblock like you, or I'd just descend into mega-simplistic evaluations or summaries that wouldn't showcase my abilities, or help me learn.
By contrast, I had friends who would do nothing but write essays for English, no matter how mediocre they turned out, because for the sake of their marks, they found it more helpful to just get their brains in a writing mood, and worry about the content later.

It's best to work out where you fit on this spectrum and tackle your study from there :)
Title: Re: VCE English Question Thread
Post by: Apink! on February 25, 2015, 06:03:32 pm
Lauren, i need your help urgently.

I know the book, i know the prompt, i know the arguments, i know the quotes. However, i dont know how to put them into words. I sit here for hours, literally, whilst the ideas pop up in my head, but I CANT get them to paper, i just cannot find the appropiate words to sentence these ideas of mine :(
I am having a very similar problem like you. I started writing an essay at 3 and now it's 6 and I still haven't finished my essay (expository). I know it's my first time and all but I have a SAC next week and I am kinda blank on how I am going to write a full essay in like an hour. What do I do?
Oh my god
Title: Re: VCE English Question Thread
Post by: literally lauren on February 25, 2015, 06:11:10 pm
I am having a very similar problem like you. I started writing an essay at 3 and now it's 6 and I still haven't finished my essay (expository). I know it's my first time and all but I have a SAC next week and I am kinda blank on how I am going to write a full essay in like an hour. What do I do?
Oh my god

You have eight months to cut down on timing, don't worry about it now!

It's way easier to go from writing a good, sophisticated essay in three hours to a good essay in one hour, than it is to go from writing a mediocre essay in one hour to a good essay in one hour. Does that make sense?

Timing is easier to cut down on than sophistication is to build up. Develop your understanding and approach first, and not only will the timing just naturally get closer and closer to the exam constraints, but you'll find that if you're doing the task properly, you'll end up taking roughly the right amount of time anyway.

Also, it's not necessarily one essay per hour. If you know you can write one of the essays quicker (usually Language Analysis for most people, but can be Context as well, if that's your strength,) it'll give you more time to work on other areas. In my case, I knew I was find with L.A. and could write that in under 50 minutes, and I also knew there was no way I was ever going to write a Context piece in an hour, because I simply needed more time to think. So I got L.A. out of the way nice and quick, giving me over 10 minutes to plan and execute a good Context piece.

Exam approach is something I'll tackle in more detail later in the year, but at this point, please don't stress yourself out by thinking you should be at an end of year standard already. You wouldn't expect yourself to be able handle any other exam by the end of February; English is no different. You still have content to learn, strategies to develop, and ages to work all this stuff out :)
Title: Re: VCE English Question Thread
Post by: heids on February 25, 2015, 06:18:02 pm
I started writing an essay at 3 and now it's 6 and I still haven't finished my essay (expository). I know it's my first time and all but I have a SAC next week and I am kinda blank on how I am going to write a full essay in like an hour. What do I do?

As I said just before, you're not alone :) I and many others like me have struggled exactly like that.

Try identifying what it is that makes you slower; do you:
 - just sit there staring at the page and hating yourself?
 - try a million ways to write a sentence because the vocab/expression isn't good enough?
 - have no clue where your essay is heading?
 - struggle to come up with any evidence/quotes?
 - have no ideas of what to say?

Then ask yourself/others how best to eradicate that issue(s).

Always write a plan first; this way you don't just land up stuck with no clue. 

Remember, though it's really really hard, it's often better to just write.  Writing nothing gets 0; writing something that sounds stupid may get 4-6/10.  Don't try to come up with better words/expression; just write what you've got!

But also essays don't have to be finished (unless you want 10/10) - I think I finished in one SAC only, and only completed one of my pieces in the exam ::)  Look, if you write 3 good paragraphs, the teachers tend to expect you could have continued well :P .

If you get stuck in a paragraph, start the next paragraph on a new page (always remembering to number pages so the teacher doesn't get lost), you can come back later.  A technique I often used when I had NO CLUE what I was going to discuss - write an impressive-sounding sentence in the intro/topic sentence, but then don't write any of the paragraph, since you have no idea.  You can leave multiple paragraphs unfinished, but always ensure you complete one just to show you know how :)

This is of course only if you really struggle with time/getting stuck/ideas... and only for the SAC, not for when you're practising.
Title: Re: VCE English Question Thread
Post by: literally lauren on February 25, 2015, 06:26:40 pm
If you get stuck in a paragraph, start the next paragraph on a new page (always remembering to number pages so the teacher doesn't get lost), you can come back later.  A technique I often used when I had NO CLUE what I was going to discuss - write an impressive-sounding sentence in the intro/topic sentence, but then don't write any of the paragraph, since you have no idea.  You can leave multiple paragraphs unfinished, but always ensure you complete one just to show you know how :)

On a smaller scale, this can also work well if you've having serious expression issues.

In one of my SACs (Text Response, I think...) I was about halfway through paragraph two when I stumbled on a word; one of those tip-of-the-tongue situations, and I ended up wasting close to ten minutes just trying to think of this word.

What I should have done, and what I recommend others do, is to write a simple word in its place. Let's say I was trying to think of the word 'magnanimous' and it just wasn't coming to me: I'd write 'generous' or 'nice' or 'kind-hearted' in its place, and then put an asterisk on the side of the page next to that line. Then continue writing as normal, as it's more important to get the content out. At the end, if you have a few minutes to spare, you can scan through the margins of your piece and look for those asterisked areas. Then you could either look something up in the dictionary in the hopes that there's a synonym listed that will trigger your memory, or just stare into space until an epiphany hits you in the face.

The same should be done if you're a bad speller; rather than crossing out a word half a dozen times or pondering the spelling of a difficult word, get something out, even if it's a synonym or an obviously wrong version of the word you're going for, asterisk the side, and come back to it at the end.

A single misspelled word, or some slightly repetitive vocab isn't going to detract from your mark as much as an entirely missing paragraph could, so focus on content first, and refine little things like spelling and expression later :)

Bit off-topic, but little strategies like this may be of some use to those struggling with timing etc.
Title: Re: VCE English Question Thread
Post by: cosine on February 25, 2015, 08:17:16 pm
Thank you so much banga_lok and literary lauren, although im still not confident :(

Can you have a quick read of my introduction, from the play Medea, its a text response:

Euripides’ play, ‘Medea’, explicitly explores the instant repercussions that betrayal and guilt may lead to. The author’s depiction of the justice system is made evident, as the protagonist of the play, Medea, gets away from her barbaric actions unharmed, and guarded by the gods. Throughout the course of the play, both Jason and Medea demonstrate a degree of social injustice, as they both contribute to the corruption that Euripides condemns. However, after the concluding events, Medea’s guilt exceeds that of Jason’s, as she commits the most ‘unholiest of all deeds.’ Despite Jason instigating the conflict between them, Medea took it one-step further and surpassed Jason’s guilt, as her egocentric nature would not tolerate humiliation.
Title: Re: VCE English Question Thread
Post by: literally lauren on February 25, 2015, 09:40:06 pm
Thank you so much banga_lok and literary lauren, although im still not confident :(
That's alright, but turn that lack of confidence into something constructive, rather than allowing it to work to your detriment. In my book, not feeling confident is a better motivation for work that feeling confident; just give yourself some tangible targets to reach and those feelings will become more manageable.

If you're posting essays, please use the Submissions sub-forum instead, but I might start making an exception for practice paragraphs...
Euripides’ play, ‘Medea’, explicitly explores the instant this isn't really true of the text; the repercussions of both Jason and Medea's actions take awhile to come to fruition repercussions that betrayal and guilt may lead to. The author’s depiction of the justice system one of the noteworthy features of Medea is that there is no institutionalised justice system, and that Medea must seek her own (vigilante) justice is made evident, as the protagonist of the play, Medea, gets away from expression is a little weak (but at the very least, I know what you're trying to get across, so don't worry too much.) Something like 'escape the consequences of' might work better here her barbaric actions unharmed, and guarded by the gods. Throughout the course of the play, both Jason and Medea demonstrate a degree of social injustice, as they both contribute to the corruption that Euripides condemns. However, after the concluding events, Medea’s guilt exceeds that of Jason’s, as she commits the most ‘unholiest of all deeds.’ Despite Jason instigating the conflict between them, Medea took it one-step further and surpassed Jason’s guilt, as her egocentric nature would not tolerate humiliation. Everything else up until this sentence is okay, but you want to end your intro on a strong/ contention-driven note, rather than a specific piece of evidence. Since I don't know what prompt you're writing on, I can't really give you an example, but try and round this off so you don't end up repeating arguments (ie. 'Medea exceeds/surpasses Jason's guilt')

I think understanding the Text Response criteria might help put your mind at ease.

The number one thing teachers are looking for is your relevance to the prompt. If you're continually answering and exploring the question, you're on the right track, and you've pretty much already secured yourself a 6 or so. Secondly, they're looking for the quality of your ideas. If your only argument is that Medea did a bad thing by killing her children, therefore her character is worse - you're unlikely to score well in this area. Comparing and contrasting characters is usually a good starting point for adding sophistication (eg. yes, Medea commits multiple murders, but why is Jason's murdering of Medea's hopes less significant? What is Euripides saying about the two characters' actions? Do they cancel out?)
Lastly, you're assessed on your use of language and expression. This won't sway the mark as much as you think; it can definitely be an inhibiting factor for breaking into the A+ range, but if you're communicating solid ideas sufficiently, this criterion isn't really a big deal. Your language has to be fit for purpose; it doesn't have to comprise of stellar "ten-dollar" words that the assessor's never heard of.

Just based on this introduction, it seems like vocab is an issue for you, since you'll either lean on paraphrasing yourself, or using slightly colloquial language to get your point across. BUT YOU ARE GETTING YOUR POINT ACROSS! I can read that intro and have a good sense of your contention, and your main arguments, which is more than I can say for a lot of "high-scoring" responses in the Assessor's Reports that score 8s and 9s. I'm not saying that's how highly you'd score - especially since this is only an intro, and a minor part of the marking scheme, but I'm saying good clarity and sophisticated ideas are more important than sounding sophisticated. Don't beat yourself up about your writing not sounding like it's a profound explication of a classic text. It's a high school essay, and it's barely the start of the year.

Chances are, your teacher will be marking with leniency at this point since you're not expected to have the entirety of Text Response under wraps and ready for the exam right now. And in the even s/he's a strict marker, everyone at your school will be in the same boat, and the marks will be moderated in scaling.

Keep at it, and don't let a lack of confidence stop you from becoming more confident :)
Title: Re: VCE English Question Thread
Post by: vella97 on February 25, 2015, 10:06:16 pm
Hi Lauren,

I am a little bit stuck on how exactly to generate my ideas and get them down onto paper. Also with the styles, I am really confused as to which direction to take for my context piece (my SAC is in two weeks :l).
How did you generate discussion when you began to write? Would you recommend doing a full expository length piece or should I mix it up with a personal anecdote or example?

I am so stuck and don't really know how to prepare for my SAC!
Any tips?
(My context supporting text is Death of a Salesman for Whose Reality? by the way.)

Thank you so much :)
Title: Re: VCE English Question Thread
Post by: cosine on February 25, 2015, 10:07:58 pm
That's alright, but turn that lack of confidence into something constructive, rather than allowing it to work to your detriment. In my book, not feeling confident is a better motivation for work that feeling confident; just give yourself some tangible targets to reach and those feelings will become more manageable.

If you're posting essays, please use the Submissions sub-forum instead, but I might start making an exception for practice paragraphs...
I think understanding the Text Response criteria might help put your mind at ease.

The number one thing teachers are looking for is your relevance to the prompt. If you're continually answering and exploring the question, you're on the right track, and you've pretty much already secured yourself a 6 or so. Secondly, they're looking for the quality of your ideas. If your only argument is that Medea did a bad thing by killing her children, therefore her character is worse - you're unlikely to score well in this area. Comparing and contrasting characters is usually a good starting point for adding sophistication (eg. yes, Medea commits multiple murders, but why is Jason's murdering of Medea's hopes less significant? What is Euripides saying about the two characters' actions? Do they cancel out?)
Lastly, you're assessed on your use of language and expression. This won't sway the mark as much as you think; it can definitely be an inhibiting factor for breaking into the A+ range, but if you're communicating solid ideas sufficiently, this criterion isn't really a big deal. Your language has to be fit for purpose; it doesn't have to comprise of stellar "ten-dollar" words that the assessor's never heard of.

Just based on this introduction, it seems like vocab is an issue for you, since you'll either lean on paraphrasing yourself, or using slightly colloquial language to get your point across. BUT YOU ARE GETTING YOUR POINT ACROSS! I can read that intro and have a good sense of your contention, and your main arguments, which is more than I can say for a lot of "high-scoring" responses in the Assessor's Reports that score 8s and 9s. I'm not saying that's how highly you'd score - especially since this is only an intro, and a minor part of the marking scheme, but I'm saying good clarity and sophisticated ideas are more important than sounding sophisticated. Don't beat yourself up about your writing not sounding like it's a profound explication of a classic text. It's a high school essay, and it's barely the start of the year.

Chances are, your teacher will be marking with leniency at this point since you're not expected to have the entirety of Text Response under wraps and ready for the exam right now. And in the even s/he's a strict marker, everyone at your school will be in the same boat, and the marks will be moderated in scaling.

Keep at it, and don't let a lack of confidence stop you from becoming more confident :)

Thank you so much for that Lauren!

So do you think my language/vocab is a little weak? If so, how can I improve on this?

Thanks once again, i really appreciate the effort you put into this!
Title: Re: VCE English Question Thread
Post by: literally lauren on February 25, 2015, 11:32:47 pm
Hi Lauren,

I am a little bit stuck on how exactly to generate my ideas and get them down onto paper. Also with the styles, I am really confused as to which direction to take for my context piece (my SAC is in two weeks :l).
How did you generate discussion when you began to write? Would you recommend doing a full expository length piece or should I mix it up with a personal anecdote or example?
There's a fair bit of Context-related stuff on the previous page of this thread, and the very first page has a collection of FAQs for each area, so check those out first and see if your questions have already been answered.

How to generate ideas will depend on the style you're writing, and your chosen style will be dependent on your strengths, and your teacher's preferences (though not necessarily in that order :P) There are some explanations in earlier posts, but ultimately, since SACs are internally marked, you want to be catering to your teacher's whims.

I would definitely recommend writing an fully expository piece for practice, but if you want to vary it up with some imaginative paragraphs or creative anecdotal intros, go for it. There are no strict rules for Context aside from what your teacher assigns.

'Preparing' for the SAC is also a matter of preference. Much like the previous few questions, not everyone will find churning out essays to be the most helpful exercise. Reading, discussing, and even simply thinking can be much more useful exercises, and then you can start writing full pieces once you're comfortable.

My usual first response is to get someone to identify their worst-case scenario. What kind of prompts would screw you over completely? What sort of areas would leave you totally lost? Are there any key words in particular that you think you're totally ill-equipped to deal with? (eg. for WR: maybe you could handle anything to do with subjectivity and perception, but anything about truth or clashes of realities would be too hard to deal with.)

After that, you know where to start angling your study so you're not just aimlessly writing on prompts with no real direction.

So do you think my language/vocab is a little weak? If so, how can I improve on this?
Like I said above, start with the words and phrases you know you need to improve upon. For a text like Medea, you know you'll probably be dealing with concepts like vengeance and justice in almost every essay, so it's worth finding some synonyms for these. Brainstorm these (with the help of a thesaurus) and then consciously implement these words in your writing. You'll have to make this a deliberate process at first, but it'll become automatic before too long.  Going through some general 'good word' lists for Year 12 English can't hurt either.
Title: Re: VCE English Question Thread
Post by: appleandbee on February 26, 2015, 12:38:15 pm
Should I categorize examples for my context, eg. subjectivity, memories, illusion/perception and conflicting realities? I find that some of my examples suit illusion-related prompts and not memory-related prompts. I've seen some students manage to mould their examples to suit a range of prompts, but I'm personally more comfortable with examples for specific prompts.
Title: Re: VCE English Question Thread
Post by: literally lauren on February 26, 2015, 02:46:18 pm
Should I categorize examples for my context, eg. subjectivity, memories, illusion/perception and conflicting realities? I find that some of my examples suit illusion-related prompts and not memory-related prompts. I've seen some students manage to mould their examples to suit a range of prompts, but I'm personally more comfortable with examples for specific prompts.

Yes, but be flexible.

Most examples should link to three or four key areas (try and minimise the amount of highly specific ones that only relate to one minor area.) You should also have a few 'generalisable' ones up your sleeve that can be adapted to several vastly different prompts.

It depends what kind of writer you are; you could memorise 20 and then pick five that work on the day, or memorise seven broad ones and tailor the four or five most relevant ones to the prompt.

In fact, the process of grouping them can be a very helpful exercise in and of itself. Rather than just writing
eg. leadership spill --> deception; power; subjectivity
give yourself an opportunity to explore the potential of your examples. It won't just be about linking them in different ways - your essay should be examining the evidence from different perspectives depending on the point you want to make. And whilst there may be a couple of sentences here and there that can be the same, a fair chunk of your discussion will have to be altered each time to suit the prompt.
Whether you want to do this in the form of dot-point/sentences for each example, or a mindmap of connecting arrows linking the ideas together is up to you.

But yes, sorting through examples is definitely worthwhile, and it'll help you work out where the gaps are in your exploration. Don't be too strict with the categories though, and be prepared to add more throughout the year as you uncover more prompts and examples :)
Title: Re: VCE English Question Thread
Post by: kimmytaaa on February 26, 2015, 03:06:45 pm
can someone help me?
Right now I am doing mainstream English, but I want to do Lit next year. Does it matter if I didn't take unit 1/2 of Lit ? And whats the difference between English and lit?
Title: Re: VCE English Question Thread
Post by: literally lauren on February 26, 2015, 04:29:47 pm
Lit 1&2 is of some benefit, but you don't need it to do 3&4.

Simply put, English consists of 3 essay types (and an oral) and each of these three essays will be tested in the exam. It's 3 essays in 3 hours, but there's a bigger, more evenly spread cohort, so it's technically not as hard to score well.

Literature is 5 different essay types, only one of which you'll have to do in the exam. The exam is 2 essays in 2 hours, but a higher standard is expected, and the cohort is more competitive.

Essentially if you're English-inclined, you'll enjoy Lit more, but English objectively easier to score well in, especially if you consider yourself more maths/science-y. Lit is a little trickier to get a handle on unless you enjoy the idea of analysing texts and language.

In terms of assessment, I'm assuming you're at least a little familiar with mainstream English (exam = 1 Text Response, 1 Context piece, 1 Language Analysis.) Literature is a whole other kettle of fish: you have to do what's called a 'Passage Analysis' where you're given three extracts from the text you studied (you can see the VCAA Past Exams website for examples) and your task is to discuss how the language of the text is used to create meaning. So it's essentially the opposite of a Text Response essay:
TR: you're given a prompt/focal point and told to find evidence to back yourself up
Lit: you're given the evidence, and have to impose your own focus, analysing as you go.

There's a breakdown of Lit assessment with some comparison between how you'd approach English as well here. I know this is a pretty broad overview, let me know if you need any more clarification :)
 
Title: Re: VCE English Question Thread
Post by: Callum@1373 on February 26, 2015, 05:50:18 pm
Can someone guide me on how I write expository pieces? I'm spending hours staring at a prompt trying to make 'ideas' but they are always so weak. I have the sac on monday, my teacher can't help me the slightest. I have a very high level of understanding of my set text, I have external references up my sleeve - plato's cave, anecdotes, stanford prison experiment, etc and etc. I understand the context whose reality but I can't get anyone to tell me what the hell you have to do with the prompt. I've been told to agree/disagree, generate ideas (although i have no clue what it means to generate ideas from a prompt), and when i do come up with something and try to write on it nothing comes to mind. Like i study for methods and I got 100%, studied for chem and got 99% but i'm spending freaking ages on this silly context expository practice and am getting literally nowhere  ::)

EDIT: Looking at my english book now i have 8 prompts to which I have tried to explore and create a piece on but all I have to show for it is a bunch of shitty comments i have made about the prompt
Title: Re: VCE English Question Thread
Post by: Cogglesnatch Cuttlefish on February 26, 2015, 09:12:42 pm
Regarding prompts, what's the difference between a "do you agree?" and "discuss"?
Title: Re: VCE English Question Thread
Post by: literally lauren on February 26, 2015, 09:38:19 pm
Can someone guide me on how I write expository pieces? I'm spending hours staring at a prompt trying to make 'ideas' but they are always so weak. I have the sac on monday, my teacher can't help me the slightest. I have a very high level of understanding of my set text, I have external references up my sleeve - plato's cave, anecdotes, stanford prison experiment, etc and etc. I understand the context whose reality but I can't get anyone to tell me what the hell you have to do with the prompt. I've been told to agree/disagree, generate ideas (although i have no clue what it means to generate ideas from a prompt), and when i do come up with something and try to write on it nothing comes to mind. Like i study for methods and I got 100%, studied for chem and got 99% but i'm spending freaking ages on this silly context expository practice and am getting literally nowhere  ::)

EDIT: Looking at my english book now i have 8 prompts to which I have tried to explore and create a piece on but all I have to show for it is a bunch of shitty comments i have made about the prompt

In half-joking defense of English: yes you can get 100% on the methods test you study for, but that doesn't mean you'd necessarily get 100% every time. Nor does it mean you're going to get 100% on every other test.

You have to think of English essays (especially Context) as an entire area of study; you'll be gradually developing your abilities in a variety of ways; writing essays is only one of them.

In my experience, as much as Methods practice exams are an excellent way to assess your knowledge, 'spamming' them wouldn't actually help you learn new concepts; you'f have to make the effort to use sample answers or text book explanations etc. in order to refine your approach. Similarly, even if you understand the content, taking the test and actually demonstrating your knowledge is an entirely different skillset.

So in English terms, you can memorise the formulae, but you need to know how they work. And you can break this question this up between 'what is the purpose of a Context piece' and 'what does a good essay look like?'

For the first question, your aim is to conduct an interesting discussion using the prompt as your foundation. Be interesting, get marks.
There are a whole lot of other finicky criteria that teachers recommend, but what it comes down to is your ability to demonstrate an understanding of the implications of the prompt, drawing upon a variety of evidence and ideas to do so.

What might help at the moment (if you're not doing it already) is to focus on getting a contention together. Your piece must have an argument; it doesn't have to be overly persuasive, but it has to be concise and complex. With regards to agreeing/disagreeing, don't do either. No good context piece will 100% agree or disagree, and you're also not meant to fence-sit, (ie. two paragraphs = agree, two paragraphs = disagree) since that makes your discussion seem really weak. Ideally you'll be ~75% one way or the other (ie. either 'Yes, I mostly agree with the prompt, but I think there are important exceptions' or 'No, not necessarily, I think >this< is the case instead.') Only then can you hope to tie together your understanding of the text, your examples, and your sub-arguments.

Regarding what your essay should actually look like, consult some of the sample high-scoring responses in the Resource thread, or the colourful paragraphs in the previous page or two. But there are a hundred ways to do this right, so don't be disheartened if it takes some experimentation for you to find an approach that works for you; it's worth it in the end :)


Regarding prompts, what's the difference between a "do you agree?" and "discuss"?
There is no difference :) In every essay you will be discussing whether or not you agree, so it's just a way of VCAA to change up the questions slightly.
You could argue 'Do you agree'-type prompts invite contrary opinions more often (eg. things like 'This text is from the '70s and has no relevance to today's society' - you'd kinda be expected to say no) but there are massive exceptions, so focus on the ideas in the prompt rather than the framing.
Title: Re: VCE English Question Thread
Post by: Apink! on February 27, 2015, 07:43:23 am
Hello!
If anyone has the time, could you please have a look at my context essay I uploaded yesterday on the marking thread?
I need to improve, but am not sure which areas I need to.

Thank you! :) :)
Title: Re: VCE English Question Thread
Post by: kimmytaaa on February 27, 2015, 10:41:25 am
Lit 1&2 is of some benefit, but you don't need it to do 3&4.

Simply put, English consists of 3 essay types (and an oral) and each of these three essays will be tested in the exam. It's 3 essays in 3 hours, but there's a bigger, more evenly spread cohort, so it's technically not as hard to score well.

Literature is 5 different essay types, only one of which you'll have to do in the exam. The exam is 2 essays in 2 hours, but a higher standard is expected, and the cohort is more competitive.

Essentially if you're English-inclined, you'll enjoy Lit more, but English objectively easier to score well in, especially if you consider yourself more maths/science-y. Lit is a little trickier to get a handle on unless you enjoy the idea of analysing texts and language.

In terms of assessment, I'm assuming you're at least a little familiar with mainstream English (exam = 1 Text Response, 1 Context piece, 1 Language Analysis.) Literature is a whole other kettle of fish: you have to do what's called a 'Passage Analysis' where you're given three extracts from the text you studied (you can see the VCAA Past Exams website for examples) and your task is to discuss how the language of the text is used to create meaning. So it's essentially the opposite of a Text Response essay:
TR: you're given a prompt/focal point and told to find evidence to back yourself up
Lit: you're given the evidence, and have to impose your own focus, analysing as you go.

There's a breakdown of Lit assessment with some comparison between how you'd approach English as well here. I know this is a pretty broad overview, let me know if you need any more clarification :)
 


Thanks Lauren
Title: Re: VCE English Question Thread
Post by: scarletmoon on February 27, 2015, 10:15:26 pm
What are some websites that are similar to "The Conversation" and "The Age" ?
Title: Re: VCE English Question Thread
Post by: Apink! on February 28, 2015, 08:34:37 am
Hello!
I have a sac on Lit next week and I have no idea how to approach it. I know the structure in general terms: make language your central focus. I learn through examples and I tried reading them but I can't understand it (yes, that's how bad I am)
Please help me. The lit thread doesn't have the Q & A thread like here so I'm posting it here. Hopefully someone can help me? By the way, the text we are studying is Kinglake 350

Thank you!
Title: Re: VCE English Question Thread
Post by: AllG_ on February 28, 2015, 09:46:59 pm
Hi guys,
In terms of study guides, which companies produce the best ones? I'm stuck on getting either Insight or VATE. Also, are study guides for context theme useful? I'm doing Whose Reality if that helps.
Thanks!
Title: Re: VCE English Question Thread
Post by: literally lauren on March 01, 2015, 12:21:06 pm
What are some websites that are similar to "The Conversation" and "The Age" ?
What do you mean by 'similar to?' If you're just looking for general news sites, the Google News browser feature is a good way of comparing sources. Or if you're looking for similar opinions/ biaises, perhaps googling similar issues covered by these papers would be useful. Sorry, not entirely sure what you're looking for, but Google is probably better than me here :)

Hello!
I have a sac on Lit next week and I have no idea how to approach it. I know the structure in general terms: make language your central focus. I learn through examples and I tried reading them but I can't understand it (yes, that's how bad I am)
Please help me. The lit thread doesn't have the Q & A thread like here so I'm posting it here. Hopefully someone can help me? By the way, the text we are studying is Kinglake 350
You can make a new thread on the Lit boards if you need to ask something specific. I'm guessing your first SAC is the Adaptations and Transformations one? Make sure you're clear what the task is asking you to do first, and then it'll be easier to determine what you need to work on.

Hi guys,
In terms of study guides, which companies produce the best ones? I'm stuck on getting either Insight or VATE. Also, are study guides for context theme useful? I'm doing Whose Reality if that helps.
Thanks!
Definitely VATE, 100%. Insight is marginally helpful with the really basic textual stuff (plot, character maps/development, sometimes themes) but VATE is basically all analysis. Even their summaries are filled with good analysis, so unless you're struggling with the comprehension of your text, you''ll get much more use out of a VATE guide than Insight or NEAP or TSSM if they're still running.
(^That said, some of these companies produce the odd good guide every so often, it just depends on the text. Maybe suss out a sample first, or get a bunch of friends from school to pitch in $2 and you can share it around like clever cheapskates ;) )

Tbh my first resort would just be to google 'text name vce english resources/analysis/prompts' and you should get a fair few bits and pieces to use. If you're doing some of the more popular texts, then sites like English Works should have some good discussion. Otherwise you might have to rely on all those wonderful schools who upload resources and don't require you to have a school login to access them.
<3 Mentone Grammar and Brunswick Secondary ;D

In my experience, the Context guides are less helpful just because of the sheer enormity involved in Context studies. The published materials are either too rigidly diagnostic (if you're not doing a straight expository essay with the exact textual/external examples they're using, you're on your own,) or they try to cover everything and end up being too broad (eg. a single A5 page devoted to each different form and style, meaning you'll have about 30 pages that are fairly useless to you, and only ~400 words that you'll use form that section.)
Context is very much about originality, and although observing and adapting other people's approaches is important, you're better off working out where you sit first, and then using resources to build up your weaknesses, rather than starting someone else's method from scratch. More than any other essay in English, Context is really a matter of individual strengths, and you could attempt the exact same approach as someone else, but fall flat because it simply doesn't suit you.
If you've been dependent on study guides in previous years, and money is no issue, then these might help give you some groundwork in the early months. But if you're using them properly, guides will overstay their welcome before long, and you'll need to go beyond and develop your own interpretation and approach.

And in my incredibly un-biased opinion, you can get way better stuff from ATAR Notes anyway :)
Title: Re: VCE English Question Thread
Post by: Callum@1373 on March 01, 2015, 12:37:20 pm
Hey Lauren, I don't want to come across rude but could you take a look at my expository piece I posted in the work submission and marking board? I'm saying this because I've tried to use your advice to get my head around context and prompts, and I think i'm getting the grasp of it...

BTW, the teaching of context so far at my school was:

2 lessons on prep for english speech
1 lesson on whose reality
1 lesson on whose reality in relation to our text
1 lesson on expository writing
1 lesson on collecting examples

and wait for it.....

3 bloody useless lessons listening to damn speeches...........  ::)
Title: Re: VCE English Question Thread
Post by: Callum@1373 on March 01, 2015, 04:22:57 pm
Does anyone know an example in real life where because someone had been lured into believing a false memory, that it led to that person getting a false version of reality?
Title: Re: VCE English Question Thread
Post by: Joseph41 on March 01, 2015, 07:56:53 pm
Does anyone know an example in real life where because someone had been lured into believing a false memory, that it led to that person getting a false version of reality?

Hi Callum,

Have you come across Loftus and Palmer's studies on reconstructed memory? I didn't do English, so I'm not sure if it will be relevant, but it seems to fit the bill.

P.S. Grouse school!
Title: Re: VCE English Question Thread
Post by: Callum@1373 on March 01, 2015, 08:19:04 pm
Hi Callum,

Have you come across Loftus and Palmer's studies on reconstructed memory? I didn't do English, so I'm not sure if it will be relevant, but it seems to fit the bill.

P.S. Grouse school!
Oh wow, that's exactly the sort of thing I was looking for! Thank you very much for finding it!  ;D
Title: Re: VCE English Question Thread
Post by: literally lauren on March 01, 2015, 10:04:02 pm
Does anyone know an example in real life where because someone had been lured into believing a false memory, that it led to that person getting a false version of reality?

See also:
- Confirmation bias though this can occur in many forms and isn't necessarily 'luring' someone into falsehoods. Perhaps this could lead into talking about journalism ethics and the idea of 'leading questions' (the most extreme versions being things like 'Have you stopped beating your wife?')
- Drinking the Kool-Aid - best to just read up on this whole saga, but it makes for some interesting studies in group delusion and psychological manipulation.
- For a less sinister discussion, perhaps consider how we ourselves can alter memories, intentionally (suppression) or unconsciously (repression) which in turn affects how we view the past, present, and future.
Title: Re: VCE English Question Thread
Post by: Callum@1373 on March 02, 2015, 07:15:51 am
Yeah - those examples are good as they are based off confabulation.

ahahahahha - hey lauren have you stopped beating children?  ;D x1000
Title: Re: VCE English Question Thread
Post by: Apink! on March 02, 2015, 08:03:17 am
Hi Lauren!
Could you please have a look at my context essay as well on the thread?
I also tried to incorporate your advice into mine but I am not sure whether I did a good job of it or not.
It's been on there for a couple of days but no one answered :'(

Thank you! :)
Title: Re: VCE English Question Thread
Post by: JackSonSmith on March 02, 2015, 05:16:11 pm
How important is the inclusion of film techniques when writing an essay on Mabo? I was told that I wouldn't get over 14/20 if I did not include a plethora of detailed film technique analysis in my essays.
Title: Re: VCE English Question Thread
Post by: cosine on March 02, 2015, 06:00:52 pm
Instead of repeating myself and say 'In Euripides' play, 'Medea',..'
Is there any other good ways to start an introduction? And, after saying the above, what should I continue with?

Thank you
Title: Re: VCE English Question Thread
Post by: literally lauren on March 02, 2015, 06:58:33 pm
Hi Lauren!
Could you please have a look at my context essay as well on the thread?
I also tried to incorporate your advice into mine but I am not sure whether I did a good job of it or not.
It's been on there for a couple of days but no one answered :'(

Thank you! :)
Will do. It seems everyone has a big round of SACs at the moment so I haven't had much time, but I'll get there :)

How important is the inclusion of film techniques when writing an essay on Mabo? I was told that I wouldn't get over 14/20 if I did not include a plethora of detailed film technique analysis in my essays.
Fairly important, but you don't need to go overboard. Most teachers like to overemphasise film techniques or metalanguage because it's better to overdo it than underdo it. Just stick to whatever your teacher is suggesting for your SAC; if they say you must include seven film techniques in every paragraph, then that's exactly what you should do. Or maybe include eight if you're looking to challenge yourself :P After awhile, you'll probably get a sense for how necessary it is to delve into close, technique-based detail. If it feels like it's stunting your analysis, then maybe cut down, but if you feel like you're being to general, fit more in. It really depends on the essay, and the prompt.

Also, point of interest, the word 'plethora' is meant to mean 'too much of something' in a bad way (ie. you can't have a plethora of chocolate, but you can have a plethora of health problems as a result of said chocolate,) though nowadays it's used interchangeably with 'abundance,' unfortunately. :(

Instead of repeating myself and say 'In Euripides' play, 'Medea',..'
Is there any other good ways to start an introduction? And, after saying the above, what should I continue with?

Thank you
Just varying the sentence structure in your intro should be enough,
- 'Throughout Medea, Euripides...'
- 'Euripides' tragic play Medea...'
- 'In the tragic play Medea, Euripides...'
You'll only be doing this once per essay, so it's not like you'd lose marks for repetition. Alternatively, you could open with some historical/contextual information, and then segue into the prompt.

After that, your priority is to convey your contention, and perhaps sub-arguments. So long as your assessor can get to the end of your intro and know what you're arguing, you're fine. Some people like the obvious sign-posting (ie. '>first argument.< Furthermore, >second argument.< Moreover, >third argument.< However, >challenge.<') I tend to advocate for something smoother if you can manage it, but this method gets the job done. Try not to do the really obvious rewording-of-each-topic-sentence-in-the-intro since that can look lazy or repetitive.

Clarity is key, just get your ideas across, and open up the discussion for your body paragraphs.
Title: Re: VCE English Question Thread
Post by: vanessa14 on March 02, 2015, 07:01:02 pm
Hello!

I was just wondering, if you could give me some tips on how to make my analysis more critical? And I was wondering how you structured your essay paragraphs and maybe if you have an example to show me? Im always repeating myself and not doing much analysis...

Tips would be most helpful! Thank you.
Title: Re: VCE English Question Thread
Post by: literally lauren on March 02, 2015, 07:08:15 pm
Hello!

I was just wondering, if you could give me some tips on how to make my analysis more critical? And I was wondering how you structured your essay paragraphs and maybe if you have an example to show me? Im always repeating myself and not doing much analysis...

Tips would be most helpful! Thank you.

What do you mean 'more critical?' I'm guessing this is something your teacher has told you that you need to work on, in which case it's probably worth sitting down with them and getting a clearer picture of where you're going wrong.
Like, are your essays at the moment just summarising the text? Or are you taking too much knowledge for grated and not going into detail? Or are you just agreeing with the prompt and don't develop a complex contention? All of these things could be classed as 'not critical,' but you need to be specific if you want to know how to improve.

Essay structure is a matter of personal preference. I usually recommend opening with a general idea or concern in your Topic Sentence; interweaving discussion, analysis, and evidence throughout, and then ending by linking back to your contention. But there's a whole lot of room for originality in there, so it's just a matter of practicing to find your strengths and weaknesses.

With regards to repeating yourself, are you saying the same ideas over and over again, or are you just using the same words? The first is a problem with your understanding of the content, meaning you have to go back to the book and expand your knowledge. The second is a problem with expression, meaning you should consult some resources and start building up your vocabulary so you can tackle more analysis.
Title: Re: VCE English Question Thread
Post by: vanessa14 on March 02, 2015, 07:15:40 pm
What do you mean 'more critical?' I'm guessing this is something your teacher has told you that you need to work on, in which case it's probably worth sitting down with them and getting a clearer picture of where you're going wrong.
Like, are your essays at the moment just summarising the text? Or are you taking too much knowledge for grated and not going into detail? Or are you just agreeing with the prompt and don't develop a complex contention? All of these things could be classed as 'not critical,' but you need to be specific if you want to know how to improve.

Essay structure is a matter of personal preference. I usually recommend opening with a general idea or concern in your Topic Sentence; interweaving discussion, analysis, and evidence throughout, and then ending by linking back to your contention. But there's a whole lot of room for originality in there, so it's just a matter of practicing to find your strengths and weaknesses.

With regards to repeating yourself, are you saying the same ideas over and over again, or are you just using the same words? The first is a problem with your understanding of the content, meaning you have to go back to the book and expand your knowledge. The second is a problem with expression, meaning you should consult some resources and start building up your vocabulary so you can tackle more analysis.

I have a good understanding of the text but when It comes to analysing quotes, I seem to just be rewording it somehow and not doing critical analysis. I've tried sitting with my teacher but my essay marks are always the same, and no matter how I change my style of writing, it doesn't seem to get into that A band. I know my expression needs working on- so any advice for that?  ::) And yes I guess I don't go into as much detail as I'd like too while explaining quotes. And when I said repeating myself, I'd provide a quote, then explain it and then I write another sentence underneath all that, rewording what I wrote the first time, and it just goes downhill   :'(

Im just a little nervous for my SAC on wednesday!


Oh and one more thing  ::) Im not quite sure how to explain the metalanguage the author uses while i'm explaining the quotes I've chosen... any ideas?
Title: Re: VCE English Question Thread
Post by: literally lauren on March 02, 2015, 09:54:20 pm
Alrighty, let's break this up:
I have a good understanding of the text but when It comes to analysing quotes, I seem to just be rewording it somehow and not doing critical analysis.
A little trick you might find helpful is nominalisation, which basically means noun-ifying things.
Rather than saying: 'The character suggests she "does not love Romeo."'
Try instead: 'The character's suggestion that she "does not love Romeo"...'
See how that second sentence makes you keep going? Using the verb form for words like said, suggests, implies, confides, reveals, etc. can lead to lapsing into summary. Whereas, if you stop and force yourself to discuss the suggestion/ implication/ confidence/ revealing, etc. you're also forcing yourself into analysis.
It's only one minor change, and of course not every sentence should look like this, but if you get into the habit of analysing on a small level, it'll be easier to adapt to other kinds of helpful sentence structures.
I've tried sitting with my teacher but my essay marks are always the same, and no matter how I change my style of writing, it doesn't seem to get into that A band.
Assuming you're in Year 12, tough it out. Even if you think your teacher's advice is unhelpful, or wrong, they're still the ones marking your SACs! Most students in this position will be learning two courses; one is the methods for essay writing that their teacher mandates, and the other is what is objectively safe to write in the end of year exam. If you're lucky, there will be a lot of overlap, but for now, you have to focus on catering to your teacher.
If you want to improve, be as specific as possible!
There's a reason you can't just move into the A band by wanting to; numerical goals don't help you study. They might motivate you, but they don't give you anything tangible to work towards. Put yourself in a teacher's shoes and imagine a student came to you and said they wanted to go from a 6 to a 10. What would you say?
You might be able to point them in some general directions, but to actually help, you need to know what the problem is. Likewise if you're going to a teacher for help, make sure you have a clear purpose in mind, like 'I need to know how to structure my Topic Sentences, and whether I should make them really broad, or really specific.' Not 'I need to make this 7/10 essay a 10/10.'
I know my expression needs working on- so any advice for that?  ::)
What about your expression needs work? Are your sentences too long? Is your syntax all over the place? Is your vocab really weak? If you ask a specific question, you'll be able to get a more specific answer,
eg. 'My expression needs work.'
--> 'My ideas flow, but not my writing.'
--> 'I can't write long sentences.'
--> 'I'm not using enough conjunctive words (eg. 'and,' but,' 'therefore,' 'contrarily,' etc.)'
--> 'I should try using those words to combine similar ideas.'
And yes I guess I don't go into as much detail as I'd like too while explaining quotes. And when I said repeating myself, I'd provide a quote, then explain it and then I write another sentence underneath all that, rewording what I wrote the first time, and it just goes downhill   :'(
If you're aware of the problem, then just stop yourself from doing this :) Consciously remind yourself to discuss the quote, not just paraphrase it. Assume your marker has read the text - you don't need to contextualise quotes and go into heaps of detail before or after - just give them sufficient information so that they know what you're talking about, and move straight into analysing the significance of the quote on a broader level.

Oh and one more thing  ::) Im not quite sure how to explain the metalanguage the author uses while i'm explaining the quotes I've chosen... any ideas?
Not quite sure what you're asking here, but if it's just a general vocab issue, going through some sample word lists is probably your best bet. As a last minute thing, go through the essays you're writing/ have written and find instances where you haven't been able to explain a word properly. Look these words up in a thesaurus, and take note of any viable synonyms that you could use. Although you can acquire words passively, going out of your way to expand your vocab is often more productive. You don't want to be spending several minutes in a SAC just trying to word your response properly; sophisticated language is efficient language, so build it up early so you have a wealth to draw from if you need it :)
Title: Re: VCE English Question Thread
Post by: wmstudent on March 02, 2015, 10:03:31 pm
Just a quick question, can we use 'readers' in text response essays? Or is it not recommend if so why not? :)
Title: Re: VCE English Question Thread
Post by: literally lauren on March 02, 2015, 10:11:44 pm
Just a quick question, can we use 'readers' in text response essays? Or is it not recommend if so why not? :)
It depends on the format of the text you're studying.
If it's a novel, or a collection of short stories, then you can say 'readers' and 'author.'
If it's a play, then it's 'audience' and 'playwright.'
For films, 'audience' and 'director.'
For poetry, 'audience' and 'poet.'
Don't know if there are any other forms currently on the lists, but it's just a conventions thing. You won't be instantly marked down, but it's the sort of teensy-weensy thing than annoys English pedants, so you want to get it right just to avoid their wrath :)
Title: Re: VCE English Question Thread
Post by: vanessa14 on March 02, 2015, 10:45:43 pm
If you ask a specific question, you'll be able to get a more specific answer,
eg. 'My expression needs work.'
--> 'My ideas flow, but not my writing.'
--> 'I can't write long sentences.'
--> 'I'm not using enough conjunctive words (eg. 'and,' but,' 'therefore,' 'contrarily,' etc.)'
--> 'I should try using those words to combine similar ideas.'If you're aware of the problem, then just stop yourself from doing this :) Consciously remind yourself to discuss the quote, not just paraphrase it. Assume your marker has read the text - you don't need to contextualise quotes and go into heaps of detail before or after - just give them sufficient information so that they know what you're talking about, and move straight into analysing the significance of the quote on a broader level.

Okay so consciously reminding myself not to paraphrase it might just work! :) thanks for that suggestion. And yes I am in year 12 (: My expression needs some working on because (for example) I was referring a character doing something beneficial and I wrote, 'its gives out a positive image' Saying that out loud to myself does make me realise that the expression is funny, but how do I avoid doing this during my SAC? Sometimes I read over my essays but under exam conditions and with 2 minutes left, things like this slip my mind.  :(
I also tend to write things that are too informal... Are these minor errors? If so and if not, how do I avoid such things?

ps: thank you so much for the help you've provided me with already (: you're awesome!  ;D
Title: Re: VCE English Question Thread
Post by: literally lauren on March 02, 2015, 11:06:56 pm
Okay so consciously reminding myself not to paraphrase it might just work! :) thanks for that suggestion. And yes I am in year 12 (: My expression needs some working on because (for example) I was referring a character doing something beneficial and I wrote, 'its gives out a positive image' Saying that out loud to myself does make me realise that the expression is funny, but how do I avoid doing this during my SAC? Sometimes I read over my essays but under exam conditions and with 2 minutes left, things like this slip my mind.  :(
I also tend to write things that are too informal... Are these minor errors? If so and if not, how do I avoid such things?

ps: thank you so much for the help you've provided me with already (: you're awesome!  ;D
Two options:

1) Reread your work constantly. This is probably better in the long run since it teaches you to pick up on mistakes as you're making them, or just after. Perhaps make yourself reread every three or four lines depending on how bad the problem is, and then you can edit as you go.
I'd also recommend writing on every second line, since it'll let you make the necessary adjustments in a much neater way.

2) Allow yourself editing time after you've finished. This is what people usually do in timed constraints, though it has some drawbacks. For one, you might not get time to edit - then it becomes a question of 'is it better to have 3/4 of a really good essay, or a finished piece that's rushed and kind of average?' To which I would answer, the first one.
But if you find it easier, allocate maybe 10 minutes before your SAC finishes to go through your sentences with a fine toothed comb, even if you need to silently mouth the words or read really slowly; you'll get faster, and it'll get easier.

The fact that you know your grammar is iffy is actually a good sign. If you didn't know what the problem was, then you'd have issues with your understanding. But if, when you read your work, you know a sentence sounds clunky or has some letters/ words it shouldn't have, you'll probably be able to correct yourself. This may take some getting used to, and you may even want to go back to your old essays and correct those for practice, but the more you do it, the more likely you are to pick up on these sorts of errors as you're writing --> thereby cutting down on your editing time --> thereby giving you more writing time --> thereby letting you write more, and better :)

For formality, that's probably just a vocab concern. Usually the reason people use informal language is because they don't have a formal alternative up their sleeve; or they do, and they just don't spend time thinking about it. Go through your practice pieces and find instances of this informal language, and see if you can find a better word or phrase to communicate your point. If not, thesaurus.com :) Wherever possible, brainstorm some other possible words that could convey the same, or a slightly different point. Even if you don't use these in the very next essay you write, you have no way of knowing whether they'll come up in the next essay, or later in Context studies, or even in the exam.
Title: Re: VCE English Question Thread
Post by: vanessa14 on March 02, 2015, 11:55:05 pm
Two options:

1) Reread your work constantly. This is probably better in the long run since it teaches you to pick up on mistakes as you're making them, or just after. Perhaps make yourself reread every three or four lines depending on how bad the problem is, and then you can edit as you go.
I'd also recommend writing on every second line, since it'll let you make the necessary adjustments in a much neater way.

2) Allow yourself editing time after you've finished. This is what people usually do in timed constraints, though it has some drawbacks. For one, you might not get time to edit - then it becomes a question of 'is it better to have 3/4 of a really good essay, or a finished piece that's rushed and kind of average?' To which I would answer, the first one.
But if you find it easier, allocate maybe 10 minutes before your SAC finishes to go through your sentences with a fine toothed comb, even if you need to silently mouth the words or read really slowly; you'll get faster, and it'll get easier.

The fact that you know your grammar is iffy is actually a good sign. If you didn't know what the problem was, then you'd have issues with your understanding. But if, when you read your work, you know a sentence sounds clunky or has some letters/ words it shouldn't have, you'll probably be able to correct yourself. This may take some getting used to, and you may even want to go back to your old essays and correct those for practice, but the more you do it, the more likely you are to pick up on these sorts of errors as you're writing --> thereby cutting down on your editing time --> thereby giving you more writing time --> thereby letting you write more, and better :)

For formality, that's probably just a vocab concern. Usually the reason people use informal language is because they don't have a formal alternative up their sleeve; or they do, and they just don't spend time thinking about it. Go through your practice pieces and find instances of this informal language, and see if you can find a better word or phrase to communicate your point. If not, thesaurus.com :) Wherever possible, brainstorm some other possible words that could convey the same, or a slightly different point. Even if you don't use these in the very next essay you write, you have no way of knowing whether they'll come up in the next essay, or later in Context studies, or even in the exam.

Thank you so much! I'll definitely take these suggestions on board and try give myself 10 minutes to read through my SAC!  ::)
Title: Re: VCE English Question Thread
Post by: appleandbee on March 03, 2015, 12:38:32 am
Hello

How text- centric does my context piece have to be? Is one paragraph enough,  I'm just wondering because I've seen some pretty text dense ones out there.

Also can someone help me dissect the prompt 'It is difficult to look objectively at our lives because we are always looking from inside out'. I'm not too sure about whether I'm interpreting the prompt correctly.

Thanks!
Title: Re: VCE English Question Thread
Post by: literally lauren on March 03, 2015, 10:01:53 am
Hello

How text- centric does my context piece have to be? Is one paragraph enough,  I'm just wondering because I've seen some pretty text dense ones out there.

Also can someone help me dissect the prompt 'It is difficult to look objectively at our lives because we are always looking from inside out'. I'm not too sure about whether I'm interpreting the prompt correctly.

Thanks!
The text-centricity of your SAC pieces is at your teacher's discretion. They want you to use the whole text and nothing but the text, then stick it in every paragraph. Or if they're happy for you to mention it once or just reference the ideas, then that's all you need.
The danger is when you write an exam piece that's only catering to your teacher; for Context in particular you kind of have to learn two ways of writing throughout the year. SAC writing is simple because you have your marker there in front of you, and you can ask him/her whether you should do more or less of something and why.
In the exam, however; you're aiming to write something that could appeal to the majority of assessors, so that's where your understanding of the task requirements is critical.

Broadly speaking, one paragraph should be enough. That's all I did in my exam, and that's usually what I recommend to students. That doesn't mean the text-heavy ones are wrong by any means, just that they're maybe not as efficient as they could be.


For that Context prompt, what do you think you're misinterpreting? Are there words you don't understand, or is it the overall concepts or ideas that you find strange?
See if you can simplify the prompt's point and develop a contention around that :)
Title: Re: VCE English Question Thread
Post by: cosine on March 03, 2015, 10:27:33 am
Can someone read my hopeless intro please :(

Prompt: ‘Medea is ultimately careless of family ties, while the men – Creon, Jason and Aegeus – are obsessively concerned with their family interests’

In the tragic play, ‘Medea’, the perception of family differs accordingly to the characters. Euripides represents the social standards of 400BC, that women are self-centered, as the female protagonist Medea, the ‘loathsome creature’, delivers unholy deeds in the name of ‘moderation’. Although Medea demonstrates some love and compassion towards her family, there is a greater cause that is controlling her thoughts. The social views and values that she follows overthrows her love for her two children to the extent that she ‘sees no joy in seeing them.’ Whilst Medea is alienated in her own desires, Euripides depicts the men as the only civilized people in the play as they demonstrate affection towards their families. Aegeus and Creon display their utmost love for their family; however, Jason’s position is questionable, as he was willing to endure Medea’s exile.
Title: Re: VCE English Question Thread
Post by: appleandbee on March 03, 2015, 10:47:25 am
The text-centricity of your SAC pieces is at your teacher's discretion. They want you to use the whole text and nothing but the text, then stick it in every paragraph. Or if they're happy for you to mention it once or just reference the ideas, then that's all you need.
The danger is when you write an exam piece that's only catering to your teacher; for Context in particular you kind of have to learn two ways of writing throughout the year. SAC writing is simple because you have your marker there in front of you, and you can ask him/her whether you should do more or less of something and why.
In the exam, however; you're aiming to write something that could appeal to the majority of assessors, so that's where your understanding of the task requirements is critical.

Broadly speaking, one paragraph should be enough. That's all I did in my exam, and that's usually what I recommend to students. That doesn't mean the text-heavy ones are wrong by any means, just that they're maybe not as efficient as they could be.


For that Context prompt, what do you think you're misinterpreting? Are there words you don't understand, or is it the overall concepts or ideas that you find strange?
See if you can simplify the prompt's point and develop a contention around that :)

My teacher prefers more external examples, but the examiners reports contains text dense context pieces. She doesn't mind if our first context essay doesn't contain much external references because we didn't have that much time but I'm pretty well-versed with the external examples so one paragraph of the text works well for me.

Speaking of teachers, what is your opinion of using books outside the set text. My teacher doesn't like us using books outside of the set text as external examples because 'students too much story-telling as a result'. Obviously I would stick to the set text in my SACS but should I consider using them later on?

As for the prompt, I think I'm misinterpreting the main idea of the prompt. This is my interpretation 'We are looking at reality/the way things are from our own narrow perspective and as a result unable to view things objectively/big picture'
Title: Re: VCE English Question Thread
Post by: literally lauren on March 03, 2015, 12:54:43 pm
Can someone read my hopeless intro please :(

Prompt: ‘Medea is ultimately careless of family ties, while the men – Creon, Jason and Aegeus – are obsessively concerned with their family interests’

In the tragic play, ‘Medea’, the perception of family differs accordingly to the characters. Euripides represents the social standards of 400BC, that women are self-centered, as the female protagonist Medea, the ‘loathsome creature’, delivers unholy deeds in the name of ‘moderation’. Although Medea demonstrates some love and compassion towards her family, there is a greater cause that is controlling her thoughts. The social views and values that she follows overthrows her love for her two children to the extent that she ‘sees no joy in seeing them.’ Whilst Medea is alienated in her own desires, Euripides depicts the men as the only civilized people in the play as they demonstrate affection towards their families. Aegeus and Creon display their utmost love for their family; however, Jason’s position is questionable, as he was willing to endure Medea’s exile.
There's nothing hopeless about it. I would perhaps take issue with your interpretation at the end there, I don't think you could feasibly argue that Euripides is totally pro-men and rejects Medea's potential for caring. After all, this 'greater force' that you mention was born from her love for Jason, was it not? (<-- not rhetorical, this is a genuine debate you could have.)
Just try not to be too definitive with your readings. You don't want to fence-sit either, but you're expected to strike a balance between overly restrictive interpretations and the wishy-washy middle-ground.

My teacher prefers more external examples, but the examiners reports contains text dense context pieces. She doesn't mind if our first context essay doesn't contain much external references because we didn't have that much time but I'm pretty well-versed with the external examples so one paragraph of the text works well for me.

Speaking of teachers, what is your opinion of using books outside the set text. My teacher doesn't like us using books outside of the set text as external examples because 'students too much story-telling as a result'. Obviously I would stick to the set text in my SACS but should I consider using them later on?

As for the prompt, I think I'm misinterpreting the main idea of the prompt. This is my interpretation 'We are looking at reality/the way things are from our own narrow perspective and as a result unable to view things objectively/big picture'

The Assessor's Report does not contain the most effective, 'best' pieces. Quite often those essays will only score an 8 or 9. They're there to prove a point, which is why they're accompanied by some comments (usually) about what they've done right or wrong. Many people around the state are unable to integrate the text properly, so in an attempt to combat this, VCAA publish samples that, if anything, over-use the text.
If you look back through previous years, you'll see them emphasise different things with almost every essay they publish. Learn what you can from them, but definitely don't use them as a be-all-and-end-all formula.

Definitely steer clear of whatever your teacher dislikes, but in the exam, any external reference is fair game. I was notorious for using literary examples, and most of my expository pieces just looked like a massive, conflated series of book reports that vaguely pertained to Conflict. In fairness, I was warned that drawing exclusively from any one discipline or area is problematic, because you open yourself up to obvious loopholes,
eg. 'here's a bunch of fictional examples of my contention!' ...yeah, but they're all fictional
'here's a bunch of historical examples from across the globe!' ... yeah, but they're not current
'here's a bunch of personal anecdotes about my experiences' ... yeah, but you're just one person.
Context is meant to be a broad area of study; you're meant to unearth some universal truths, and you can't do that properly if you restrict yourself too much. Naturally, you don't want to over-correct and cram your essay with all the varied information you can think of, but try and find some solid links that let you explore depth and breadth sufficiently.

That interpretation of the prompt seems fine to me. You should be fine to start exploring sub-arguments and see how you go; if you hit a mental roadblock then perhaps revisit your contention and modify it slightly, but you're definitely addressing the main idea. 'We can't judge ourselves, because we are ourselves' is what it boils down to. 'Are we able to objectively assess our identities/realities, or are we too wrapped up in our own heads?' 'Can we judge our own realities with the same objectivity as we judge other peoples'?'
It's quite a good prompt, actually :)
Title: Re: VCE English Question Thread
Post by: Adequace on March 03, 2015, 04:35:02 pm
Hey Lauren,

I've been trying to read books to expand my vocabulary, but I can't seem to find time to read much of my book unless it is in the holidays. However, I still want to continue expanding my vocab.

Should I just go through lists of words and memorise them and their meanings? Should it be VCE-specific or just any sort of word?

I was planning on learning the words on an image you posted in an EL post. It was a circle with synonyms for particular words, good idea?
Title: Re: VCE English Question Thread
Post by: IndefatigableLover on March 03, 2015, 06:03:57 pm
Hi Lauren :)

So I have an oral SAC which needs to be completed for my AOS 1 text (a graphic novel [The Complete MAUS]) and I've chosen a topic which will be "exploring minor characters" within the graphic novel.

What should I be looking for when I'm determining the characters which I will be analysing as well as the type of analysis which I could be doing? Also is a supporting character the same as a minor character (because mainly I've been picking examples where the character at hand is made obvious but only features for 1-2 pages at most-ish compared to a character which occurs throughout the graphic novel as such).

Thanks!
Title: Re: VCE English Question Thread
Post by: literally lauren on March 03, 2015, 06:25:55 pm
Hey Lauren,

I've been trying to read books to expand my vocabulary, but I can't seem to find time to read much of my book unless it is in the holidays. However, I still want to continue expanding my vocab.

Should I just go through lists of words and memorise them and their meanings? Should it be VCE-specific or just any sort of word?

I was planning on learning the words on an image you posted in an EL post. It was a circle with synonyms for particular words, good idea?
Maybe it's just me, but I reckon reading would be more efficient than just sitting down and memorising words. There is an element of rote-learning, granted; but you have to acquire the words first.
I guess just be aware that vocab expansion doesn't have to be a 'sit-down-for-an-hour-and-learn-20-new-things' exercise; in fact it's most effective when you integrate the language in your essays or notes. I'd also say synonyms for words are more helpful that actual definitions. If you teach yourself to think in categories, you end up acquiring a bunch of other words along the way, and you'll eventually be able to distinguish between minute differences in connotations or associations.

eg. let's start with the word 'polite.' Instead of memorising the definition 'showing good manners or behaviour towards others,' you'd instead group it with a list like:
- affable
- cordial
- conciliatory
- genteel
- obsequious
- well-mannered
Some of these words are completely synonymous (eg. 'polite,' 'cordial,' and 'well-mannered' express exactly the same sentiment,) but the others have some interesting variations. 'Affable' is like cheerfulness when socialising; being 'conciliatory' means you're trying to reconcile or compromise with someone; 'genteel' is associated with upper-class respectability and refinement; and 'obsequious' is like 'sucking up' to someone, so it's more extreme than just being polite.

Not only does having more words help you write with more precision and efficiency, but this kind of attention to the associations of words is an immense help in Language Analysis.


If you're pressed for time, then stick to the VCE-oriented vocabulary (eg. tone words for L.A. (like that worksheet) or text-specific words for your T.R. texts.) But explore language as much as possible - it's a gradual process, but a worthwhile one.

Hi Lauren :)

So I have an oral SAC which needs to be completed for my AOS 1 text (a graphic novel [The Complete MAUS]) and I've chosen a topic which will be "exploring minor characters" within the graphic novel.

What should I be looking for when I'm determining the characters which I will be analysing as well as the type of analysis which I could be doing? Also is a supporting character the same as a minor character (because mainly I've been picking examples where the character at hand is made obvious but only features for 1-2 pages at most-ish compared to a character which occurs throughout the graphic novel as such).

Thanks!
I'm not sure if you've been given a specific prompt or thematic area to explore, but I'd guess that's what your teacher is expecting you to cover? So angling your exploration around an idea like 'The minor characters in Maus have a significant effect on our understanding of the text, or of theme X.' I guess Francoise
or Vlad and Anja's fathers might be interesting to look at; it depends on your interpretation of 'minor.' (Soz, it's been ages since I read Maus.) Since there are so many characters in the text, you pretty much have your choice of discussion topic. Just ensure you're going into enough depth with a few characters, and giving a sense of the full text as well :)

Title: Re: VCE English Question Thread
Post by: IndefatigableLover on March 03, 2015, 06:48:13 pm
I'm not sure if you've been given a specific prompt or thematic area to explore, but I'd guess that's what your teacher is expecting you to cover? So angling your exploration around an idea like 'The minor characters in Maus have a significant effect on our understanding of the text, or of theme X.' I guess Francoise
or Vlad and Anja's fathers might be interesting to look at; it depends on your interpretation of 'minor.' (Soz, it's been ages since I read Maus.) Since there are so many characters in the text, you pretty much have your choice of discussion topic. Just ensure you're going into enough depth with a few characters, and giving a sense of the full text as well :)
Thanks Lauren! Great to see you still remember things about Maus LOL xD

The prompt I've chosen is: "It is though the minor characters that the detail of the story is communicated." And yeah that's what I was wondering whether there was a particular definition of "minor" character since there are some which I would definitely like to talk about (provided I can find evidence to back up my points that is)!

Last question (so far that I can think of), but in the prompt, they've stated "that the detail of the story is communicated", what exactly do they mean by 'detail' and 'communicated' because I feel as though 'communicated' is the use of graphic novel techniques (except I'm not too sure if that's right) whilst 'detail' I don't really know what to think of..
Title: Re: VCE English Question Thread
Post by: literally lauren on March 03, 2015, 07:28:25 pm
Thanks Lauren! Great to see you still remember things about Maus LOL xD

The prompt I've chosen is: "It is though the minor characters that the detail of the story is communicated." And yeah that's what I was wondering whether there was a particular definition of "minor" character since there are some which I would definitely like to talk about (provided I can find evidence to back up my points that is)!

Last question (so far that I can think of), but in the prompt, they've stated "that the detail of the story is communicated", what exactly do they mean by 'detail' and 'communicated' because I feel as though 'communicated' is the use of graphic novel techniques (except I'm not too sure if that's right) whilst 'detail' I don't really know what to think of..
I think, if you feel up to it, you could actually conduct an interesting examination of what it means for a character to be 'minor.' You have the obvious surface level meaning which is 'a character that isn't in the text much,' but it's usually more complex than that. Infrequent characters can have a profound effect on the story, or even on other characters who in turn influence the story.
This could even bridge into an even bigger discussion about how characters have importance - is this assigned to them by the author, or just interpreted by the audience, or both? Usually the protagonist/antagonist/love interest etc. are pretty indisputable, but can we debate over the greater significance of a character?

AND you could even go so far as to question what makes a character 'a character.' The Greeks had a nice way of dividing up this concept:
είδος = literal character, as in, the person or thing itself, eg. Harry Potter, or Rodger Rabbit
χαρακτήρας = the spirit of the character, loosely translated today as the ethos of a person, eg. an immoral character, or a character of influence

You don't have to get too far down the rabbit hole of literary theory, but there's always more that can be found in a prompt :)

/nerdiness

For Maus, all the practice companies and VCAA themselves put a huge amount of focus on the fact that it's a non-conventional text form, so you'll see a lot of prompts that ask you to discuss whether the message is enhanced or impeded by the structure, or how the author uses certain features to communicate certain ideas.

The word 'communicated' here is fairly standard, but the word 'detail' has some potential. What is the 'detail' of the text? Is the prompt saying the minor characters just add background/flavour to the story, or that they contribute to the primary plot and concerns in a big way?

The wording is quite ambiguous, and I'd hazard the person writing it may not have considered all these eventualities, but feel free to take this in whichever direction you please. It might be worth asking your teacher whether you're getting too far away from the prompt just to get their feedback, but so long as your discussion always comes back to your contention, you should be golden :)
Title: Re: VCE English Question Thread
Post by: 99.90 pls on March 03, 2015, 08:11:16 pm
Hi!

A bit of an unorthodox question, but I need help understanding this article about Julian Assange. I'm currently trying to write a practice piece on this article, but I'm not a very cultured person, so the second half of the article beginning with "Arab governments are as concerned as any other..." doesn't make much sense to me. The first half of the article is fine though.

http://www.smh.com.au/federal-politics/political-opinion/julian-assange-is-the-ned-kelly-of-the-digital-age-20101207-18ob0.html

Thanks :)
Title: Re: VCE English Question Thread
Post by: 2jzgte on March 03, 2015, 09:27:11 pm
Can someone please help me analyse this essay prompt:
No amount of past hurt can condone Medea’s acts of cruelty. Rather than being a feminist hero, she is a vengeful witch. Discuss.
How would I address this prompt?
Title: Re: VCE English Question Thread
Post by: literally lauren on March 03, 2015, 09:34:03 pm
Ah yes, I remember this article from last year's round of MHS questions  ::)

Okay so the tricky thing about this article is it's subversion. It spends almost half the piece espousing something that's not the contention, namely, that Assange is an evil criminal.

It's in this second half of the article that you get a clearer picture of what the author is contending.
"So Arab governments are concerned..." <-- this section lists some of the 'revelations' to come from Wikileaks, but mocks how obvious and non-revelatory they are.
The author seems to be suggesting Assange is like Kelly in terms of their role in transporting the spoils of crime, but Assange isn't quite the mega-corporation-toppling-big-bad-outlaw that some might believe him to be.

It can be tricky going back to previous years' SACs since they rely on a lot of relevant-at-the-time understanding that most people, if they did know, have forgotten by now :P You'd probably be better off picking your own opinion pieces from the past few weeks and working with those.
This'll also help you catch up with the most relevant stories at the moment, so if you get something about counter-terrorism or foreign policy, you won't be totally stumped when the author brings up '18C' or 'Tony Abbot's shirt-fronting.'

Can someone please help me analyse this essay prompt:
No amount of past hurt can condone Medea’s acts of cruelty. Rather than being a feminist hero, she is a vengeful witch. Discuss.
How would I address this prompt?
The first half of this is badly worded, but essentially it boils down to 'We can't justify Medea's actions, regardless of her past. She's not a hero of women, she is a vengeful witch.'
First step: do you agree, or disagree with that sentiment?
Second step: why? What evidence in the text could you use to support your interpretation?

That should get you started; refer to the links on the first page of this thread for general advice on exploring and unpacking prompts :)
Title: Re: VCE English Question Thread
Post by: 2jzgte on March 03, 2015, 11:10:10 pm
Could someone please critique this introduction I just wrote.
No amount of past hurt can condone Medea’s acts of cruelty. Rather than being a feminist hero, she is a vengeful witch. Discuss.
Through the tragic play “Medea’, Euripides elicits the worst of Medea. Euripides was known to challenge the norms and bend the rules; this was clear cut in the composition of Medea’s fabricated personality which does not come together as a coherent and believable character. There is no doubt that revenge would have been a suitable path for Medea to take, but the way it was exacted and the ferociousness of Medea in doing so transcends all human limits. Considering the role of women in ancient Greece, Medea was not one to conform to society’s expectations; this however does not make her a feminist hero in any way. The injustices that Medea commits in the process of avenging herself are injudicious in comparison to the “sting of injustice” she has endured at the hands of the “arch-criminal” Jason.  The act of shedding kindred blood is by no means justifiable, her “lust for blood” is driven by her fury, “wrong a woman in love and nothing has a heart more murderous”. Her concern for the inequality of genders was merely a ruse which she uses to bind the Corinthian women in an oath of silence. Through manipulation and shear brutality Medea challenges the boundaries of monstrosity within human actions, driven by a clear ulterior motive, revenge.
Thanks for the reply earlier Lauren :)
Title: Re: VCE English Question Thread
Post by: Escobar on March 04, 2015, 01:33:15 pm
In text response, when a quote is given in the question, how should it be used in your essay?
Title: Re: VCE English Question Thread
Post by: literally lauren on March 04, 2015, 02:53:28 pm
In text response, when a quote is given in the prompt, how should it be used in your essay?

No strict rules, but I'd advice at least mentioning it in your first b.p.
There's a tendency for people to forget about the quote and just answer the question/statement for ~600 words, then you see them go 'oh shit, there was a quote' around about the third body paragraph where they try to cram it in. It looks clunky, and it stands out, so just blend it in whenever seems natural.

Dealing with the context of the quote is also a good idea; don't just briefly use it for evidence, actually delve into who said it, what precipitated it, what consequences came about as a result, that sort of thing.
Title: Re: VCE English Question Thread
Post by: Escobar on March 04, 2015, 04:16:02 pm
thanks  :)
Title: Re: VCE English Question Thread
Post by: vanessa14 on March 04, 2015, 07:28:07 pm
sometimes i have trouble coming up with topic sentences  :-X  I dont know where to start and what words to use  :'( any help?
Title: Re: VCE English Question Thread
Post by: literally lauren on March 04, 2015, 07:58:03 pm
sometimes i have trouble coming up with topic sentences  :-X  I dont know where to start and what words to use  :'( any help?

Do you not know what to write, or do you not know how to write it?

^That should always be the first question you ask yourself for problems like this.
For starters, don't think about the words. Don't worry about making it sound pretty and essay-ish, just get the ideas out as plainly as possible. It can be as clunky as 'This character does a lot of bad stuff in the book, but we still like him.' You'd obviously never want to write something that bad in an essay, but at the very least, you know what you want to express. Focus on the ideas you want to communicate, and then build it up piece by piece rather than all at once.

eg.
'The character does' --> are we talking about acts he perpetrates, or dark thoughts he has, or evil desires he harbors...? What does he 'do,' exactly?
'lots of bad stuff' --> 'Bad' how? Physically repulsive? Morally dubious? Psychologically confronting? Are these things bad for himself, or others? What kind of 'bad' are we talking about? (Looking up words like 'bad' in a thesaurus gives you an idea of just how broad the definition can be)
'but in the end we still like him' --> Do we admire him? Do we sympathise with him? Do we pity him? Do we only like him compared to other, worse characters? Do we like him in spite of his flaws, or because of them?

So after thinking about each of these weak areas ('does,' 'bad stuff' and 'we like him,') we can turn an embarrassingly simple sentence into a sophisticated one:
Character X commits many morally dubious acts throughout the text, however audiences are still inclined to sympathise with him because of his self-awareness and constant feelings of regret.

On the surface, it's as simple as that, but admittedly it does take practice and exposure before these words start coming to you naturally. I'm yet to meet a good writer who isn't also a good reader, so try and read widely so things like sentence structures and vocab can be naturally acquired, rather than painstakingly laboured over.
Title: Re: VCE English Question Thread
Post by: paper-back on March 04, 2015, 08:08:32 pm
Does anyone have any non-copyright "The White Tiger" prompts?
Title: Re: VCE English Question Thread
Post by: literally lauren on March 04, 2015, 08:19:34 pm
Some original ones in the Prompts thread.
Otherwise your best bet is to pester your teacher, since I haven't found very many resources for most of the new texts.
You could always write your own, too. It's quite a helpful exercise :)
Title: Re: VCE English Question Thread
Post by: vanessa14 on March 04, 2015, 10:47:30 pm
On the surface, it's as simple as that, but admittedly it does take practice and exposure before these words start coming to you naturally. I'm yet to meet a good writer who isn't also a good reader, so try and read widely so things like sentence structures and vocab can be naturally acquired, rather than painstakingly laboured over.

once again thank you so much  :) :) this is really helpful.
Title: Re: VCE English Question Thread
Post by: Adequace on March 04, 2015, 10:56:05 pm
Hey Lauren,

Do you mind giving me feedback on my oral? If you have time of course. University Deregulation Oral Feedback
Title: Re: VCE English Question Thread
Post by: theBRENDAN97 on March 05, 2015, 11:51:52 am
Hi Lauren, :)
This Friday at school we are watching a performance for the play No Sugar, just wondering what i should look out for in terms of construction, like what about stage direction, etc.
Thanks.
Title: Re: VCE English Question Thread
Post by: cosine on March 05, 2015, 05:26:31 pm
Jason's actions are supported by careful judgement; Medea's actions are supported by selfishness and passion

Can someone help me break down this prompt or look at my interpretation of it, thanks:

Paragraph 1: Initially as the play commences, Jason is depicted as the villain as we see his actions are lead by his desires
Paragraph 2: However, as the play proceeds we see him mature and come to realisation of the importance of family and decides to take care of his children
Paragraph 3: Medea thoroughly displays egocentric actions as they are driven by her fearsome passion

These are my paragraph arguments, any opinions? Thankyou
Title: Re: VCE English Question Thread
Post by: thaaanyan on March 05, 2015, 08:51:18 pm
hi, i was just wondering if anyone could help me with understanding how to weave in authorial/directorial b.ground in an intro?? like in what prompts do you think it's be acceptable and how would you do it?? thank you. i've been raised with stock standard TR formula (which i can do fine) but now im finding variation a bit difficult. i appreciate any help!! :)
Title: Re: VCE English Question Thread
Post by: JackSonSmith on March 05, 2015, 09:24:34 pm
hi, i was just wondering if anyone could help me with understanding how to weave in authorial/directorial b.ground in an intro?? like in what prompts do you think it's be acceptable and how would you do it?? thank you. i've been raised with stock standard TR formula (which i can do fine) but now im finding variation a bit difficult. i appreciate any help!! :)

It's my belief that things that are not specifically referenced in the text should not be in your essay. For example, in Mabo the director's own father was an activist so she would be able to sympathise with Eddie Mabo's family. However, when writing an essay, I would simply state that ''the director showcases the hardships faced by the Mabo family through...''.

Knowledge of the author's/ director's background can be useful when studying a text, but you should only discuss elements of a text that are at least implicitly implied and certainly not things that never see any mention.

Of course those are just my own interpretations/beliefs, sorry if I am wrong.
Title: Re: VCE English Question Thread
Post by: Cogglesnatch Cuttlefish on March 05, 2015, 09:42:16 pm
Can somebody help me unpack the following prompt:
"Contrary to expectations, it is the women of Cloudstreet that drive the narrative". Do you agree?
Im especially struggling with what the word "drive" is meant to mean in this context, and how "contrary to expectations" would be considered when formulating a contention/ideas and the like.
Title: Re: VCE English Question Thread
Post by: JackSonSmith on March 05, 2015, 09:54:23 pm
Can somebody help me unpack the following prompt:
"Contrary to expectations, it is the women of Cloudstreet that drive the narrative". Do you agree?
Im especially struggling with what the word "drive" is meant to mean in this context, and how "contrary to expectations" would be considered when formulating a contention/ideas and the like.

I'd look at:
Which women drive the narrative?
Is it their story? Their actions? Their words?
What may other expectations be?
Title: Re: VCE English Question Thread
Post by: kimmytaaa on March 06, 2015, 12:10:02 pm
Hi
I just want to know your opinions, so I have an English sac on the 16th or 17th but it has been week 6 without any practice essays to do and apparently all we do is summary of chapters of each chapter. On Tuesday next week, we are wasting our time watching the movie instead of doing anything. But the teacher said we are going to to the practise sac on Friday its a bit too late in my opinion, what do you guys think? On Friday next week, she is expecting us to do our practise essays in class and hand it up by the end of the lesson.
Title: Re: VCE English Question Thread
Post by: kimmytaaa on March 06, 2015, 12:43:47 pm
You could always do your own practice essays and submit them to your teacher. They're more than likely to correct them and you can always find topics if you ask your teacher, or look around online.
true, but my teachers don't even bother to look tho it so I use my tutors since they are more helpful than school teachers
Title: Re: VCE English Question Thread
Post by: pinklemonade on March 07, 2015, 10:35:04 am
Hey! I don't know if this question has been answered or not but I was wondering how would you write the conclusion for a comparative language analysis?
Would you talk about the first article, second, or both?

Thank you
Title: Re: VCE English Question Thread
Post by: scarletmoon on March 07, 2015, 11:14:46 am
I need help finding some external examples for the promp conflict can discriminate. I already have some examples for racial discrimination just wanting to find examples of other types of discrimination caused by conflict
Title: Re: VCE English Question Thread
Post by: literally lauren on March 07, 2015, 06:32:22 pm
Hey! I don't know if this question has been answered or not but I was wondering how would you write the conclusion for a comparative language analysis?
Would you talk about the first article, second, or both?

Thank you
You could get away with doing either, but it depends on the spread of the material. For something like the 2014 exam where the first piece was definitely the 'core,' you could just have dealt with that one section in your conclusion. However, if you had two articles of around equal length, it would be more conventional to draw on both of them when you wrap things up.
I'd recommend trying to use both anyway like you would in an intro, eg. 'Ultimately author X's appeals to a sense of justice and decency are designed to promote discord amongst people, whereas author Y's aggressive tone and pejorative language is indicative of his attempts to divide the local community...'

I need help finding some external examples for the promp conflict can discriminate. I already have some examples for racial discrimination just wanting to find examples of other types of discrimination caused by conflict
You could go for the typical forms of discrimination, like sexism, ageism, ableism, classism, heteronormativity, etc.
But it might be more interesting to unpack the idea of 'discrimination.' What is it, really? What is it based on? Why does it occur? Do our small judgments contribute to overarching ones (eg. my neighbour Joe is mean, therefore all people called Joe are mean) or do pre-existing stereotypes feed into our personal judgments? (eg. everyone knows people called Joe are mean, therefore my neightbour Joe must be mean.) Or both? Do we need a label to judge people?
And more importantly, when we look at the whole prompt 'Conflict can discriminate' some more important questions arise: how does conflict itself discriminate? Is this a good thing or a bad thing? Are we even aware of this? Is it a process we can fight or reject, or are we powerless to stop the discrimination of conflict.

Perhaps it's just my reading, but I'd see this prompt hinting towards the consequences of conflict more than the specific forms of discrimination. You don't want you piece to turn into a list of examples that illustrate the same point.

If conflict discriminates, that means conflict separates people and assigns them different fates. Some people are strengthened by conflict, some are destroyed by it. This prompt is suggesting that process to be innate; is that something you agree with?
Title: Re: VCE English Question Thread
Post by: stockstamp on March 07, 2015, 06:33:48 pm
Hoping someone can clarify this for me:

On the examiners report for English, provided are student examples from the exam that are labelled as "high scoring responses".
What exactly does this mean? Are the examples always a 10/10 - or are they anything from about an 8/10 to a 10/10.

I ask this because example 10/10 pieces seem to be a fairly useful way of identifying the standard of work I should work towards (I understand that english isn't a science and that I'm not following a formula for a perfect essay, but it would still be nice to have some examples of what the examiners like).

As an extension to this, are the example responses on ATAR Notes roughly a 10/10?

Thanks!
Title: Re: VCE English Question Thread
Post by: Burt Macklin on March 07, 2015, 08:22:45 pm
Hi Lauren,

After doing a practice Context SAC for Encountering Conflict, I've found that I've having difficulty finishing a piece under the time limits because I'm struggling to think of relevant external examples. Do you think it's just a matter of practicing to improve on this?

Also, I also find that my paragraphs are basically being written as arguments for my contention. (My intro is basically like "This is my contention" "Because, X, Y and Z".) Is this the way to go or should I be doing more exploration of the prompt?

Thanks!
Title: Re: VCE English Question Thread
Post by: JackSonSmith on March 07, 2015, 09:54:08 pm
Hi Lauren,

After doing a practice Context SAC for Encountering Conflict, I've found that I've having difficulty finishing a piece under the time limits because I'm struggling to think of relevant external examples. Do you think it's just a matter of practicing to improve on this?

Also, I also find that my paragraphs are basically being written as arguments for my contention. (My intro is basically like "This is my contention" "Because, X, Y and Z".) Is this the way to go or should I be doing more exploration of the prompt?

Thanks!

Hello, I'm not Lauren but maybe I can help.
Your ability to write quickly would have a correlation with how well you know your examples. Knowing your external examples well before an essay should increase your confidence  and in turn your speed when writing.

As for paragraph structure, I think the quality of your ideas would be the most important part. There's no set way in which you should structure your piece. At the end of the day, your arguments should be an extension / support of your main contention, so there's nothing wrong with a your approach. 
Title: Re: VCE English Question Thread
Post by: pinklemonade on March 07, 2015, 10:07:42 pm
Hey Lauren,
Just another quick question about comparative LA - how would you structure one with 2 articles of the same length?
I just read one that someone else posted and they had their intro, 6 body paras and conclusion..
This seems like a bit too much, is there another way of doing this?
Title: Re: VCE English Question Thread
Post by: 99.90 pls on March 07, 2015, 10:12:07 pm
Hey Lauren,
Just another quick question about comparative LA - how would you structure one with 2 articles of the same length?
I just read one that someone else posted and they had their intro, 6 body paras and conclusion..
This seems like a bit too much, is there another way of doing this?

If you're referring to this essay, I had to analyse a third cartoon as well. But I prefer to keep my ideas separate as opposed to combining them into one big mush. The number of paragraphs doesn't really matter, as long as each paragraph explores an idea thoroughly from start to finish, I suppose

It doesn't seem that bad when you look at it on paper.

(http://i.imgur.com/EZGEnC2.jpg)
Title: Re: VCE English Question Thread
Post by: pinklemonade on March 07, 2015, 10:38:32 pm
If you're referring to this essay, I had to analyse a third cartoon as well. But I prefer to keep my ideas separate as opposed to combining them into one big mush. The number of paragraphs doesn't really matter, as long as each paragraph explores an idea thoroughly from start to finish, I suppose

It doesn't seem that bad when you look at it on paper.

(http://i.imgur.com/EZGEnC2.jpg)

Ohh nah I was referring to this essay, but thank you! Just wasn't sure if it was possible to write that much in the restricted amount of time we're given
Title: Re: VCE English Question Thread
Post by: literally lauren on March 08, 2015, 10:41:01 am
Hoping someone can clarify this for me:

On the examiners report for English, provided are student examples from the exam that are labelled as "high scoring responses".
What exactly does this mean? Are the examples always a 10/10 - or are they anything from about an 8/10 to a 10/10.

I ask this because example 10/10 pieces seem to be a fairly useful way of identifying the standard of work I should work towards (I understand that english isn't a science and that I'm not following a formula for a perfect essay, but it would still be nice to have some examples of what the examiners like).

As an extension to this, are the example responses on ATAR Notes roughly a 10/10?
The 'upper-range'/'high band' essays at the end of the year can be anything from an 8 upwards. For the record, 'upper-mid range' usually means between 6-8, though sometimes the exact numbers are debatable. You can occasionally get 'borderline' responses that could be either a 7 or an 8, for instance. In the exam this just evens out to 15/20.

The important thing when working with examples is to work with multiple. There's an obvious danger in only using one or two essays as your point of reference; there are 101 ways of scoring 10/10, so by only studying a few of them, you're severely limiting your abilities.

The essays on ATAR Notes (the ones in the Resource thread at least) usually have their marks listed. If it's a fairly recent piece with a still-active user, you might be able to PM them if you really wanted to find out, but speaking from experience, you can learn just as much from an 8/10 piece than you could from a 10/10. So long as you have a well-developed sense of what separates a god essay from a bad or mediocre one, you'll be able to take what you need from other people's work and incorporate it into your own :)

Hi Lauren,

After doing a practice Context SAC for Encountering Conflict, I've found that I've having difficulty finishing a piece under the time limits because I'm struggling to think of relevant external examples. Do you think it's just a matter of practicing to improve on this?

Also, I also find that my paragraphs are basically being written as arguments for my contention. (My intro is basically like "This is my contention" "Because, X, Y and Z".) Is this the way to go or should I be doing more exploration of the prompt?

Thanks!
Like JackSon said, knowing your stuff will help cut down on time significantly. Are you practicing to an essay-per-hour time limit? Because unless your SAC is under those conditions, there's no reason why you should aim that high at this stage of the year. I know it's easy to get caught in the 'how the hell am I going to do this in an hour, this one essay took me three hours over a week' mentality, but trust me, it's a necessary process.
Look at it this way: in other subjects, you're not expected to complete an exam-level task in February/March. The SACs are modified to suit what you've learned thusfar. English isn't really like that. The prompts you get for a T.R./Context SAC could feasibly be on this year's exam, but that doesn't mean you'll write the same way. Everything leading up to the exam is a formative process, and you should be more concerned with developing your skills than conforming to strict limits.

Yes it's a matter or practice, but you won't be practicing 'cutting down on time.' You'll be practicing how to write well and hit the criteria efficiently; the timing is incidental for now.

The other thing I'd recommend is to work out why you take so much time (idk if you're 10 minutes or an hour over, but this applies either way.) Do you take ages to get started, or spend 15+ minutes on a plan? Do you get stuck after the first few paragraphs and run out of ideas/examples to use? Do you spend ages just putting your thoughts into words, and have to reread every sentence to ensure it makes sense?
Answering 'yes' to any one of these questions gives you a totally different path of study (ie. efficient brainstorming and preparation; idea development and example collecting; and vocab/expression respectively.)

For your second question, it depends what sort of piece you're writing. Your ''Because X Y Z" arguments could work well provided they're fleshed out enough. But you don't want to have three paragraphs that are all arguing the same thing.

eg. Prompt: 'Conflict changes how we view the world.'
--> B.P.1: It changes how we view other people.
--> B.P.2: It changes how we view our environment.
--> B.P.3: It changes how we view ourselves.
^A tad oversimplified, but you'd be surprised how many essays could be boiled down to something that plain. The trick is to have a complex contention that is based on, but not limited to the prompt.

Hey Lauren,
Just another quick question about comparative LA - how would you structure one with 2 articles of the same length?
I just read one that someone else posted and they had their intro, 6 body paras and conclusion..
This seems like a bit too much, is there another way of doing this?
Structure for L.A. SACs should be geared to whatever your teacher wants. Some love many paragraphs that cover as much of the article as possible, others prefer a standard essay format with 3-4 body paragraphs and clearly outlined focus points. I tend to advocate for the bigger paragraphs since the assessors seem more concerned with depth than breadth recently, but both are still important, so go for whatever you feel would give you the best opportunity to demonstrate your skills.
Roughly 800 words is the guideline. How you break this up is totally up to you (...and your teacher) :)

Title: Re: VCE English Question Thread
Post by: cosine on March 08, 2015, 10:43:47 am
Hey lauren

What scores am i looking for a 35 minimum?

I know it depends on my cohort, exams etc.. But surely there's some sort of sac average that would lead to a 35. Thankyou, just worried :3
Title: Re: VCE English Question Thread
Post by: literally lauren on March 08, 2015, 11:00:12 am
Hey lauren

What scores am i looking for a 35 minimum?

I know it depends on my cohort, exams etc.. But surely there's some sort of sac average that would lead to a 35. Thankyou, just worried :3

uuuurrrrgggghhhh numbers -.-

This is the English Board, friend.

Quality > quantity. And that goes for feedback too.

I could tell you that 8/10s usually amounts in a ~40 SS, but that information is useless because averaging 8/10 throughout the year means nothing unless you're aware of what you're doing right and wrong. A kid at my school who was getting solid 10s for everything ended up with a 45 because he got unlucky in the exam. By contrast, one of my friends was hovering around 7s and 8s all year, but got her shit together and pulled of a 48. Numbers are malleable, and somewhat unpredictable. Your understanding isn't. If you go into the exam confident in the knowledge that you know how to handle prompts and best showcase your abilities, then you'll be in a much better boat than someone who strolls in thinking 'I'll be fine, I've been getting 9s since mid-year, I'm almost guaranteed a 40+'

AAAAND... a lot of teachers mark easier early in the year. Getting an 8 right now might only be worth a 6 in the exam. Not all of them do this (I'm certainly not a fan of it,) but it's quite common.

This, all coupled with the fact that you could be getting 8/10 for different reasons every time you write means... well, let's say you had two essays worth 8/10. The first had excellent ideas, but poor expression and quote usage. The second had mediocre ideas, but excellent structure and evidence integration. You figure you can just pick the most relevant parts from each of these essays and apply them to the prompt in the exam. But chance, you pick the ideas from essay 2, and the structure from essay 1, which would bump your mark down to maybe the 5-6 region.
However, if you manage to combine the best parts of both pieces, you might even score higher than an 8.

This is why knowing why you're getting a certain score is infinitesimally more valuable than the score itself.
Title: Re: VCE English Question Thread
Post by: Splash-Tackle-Flail on March 08, 2015, 12:16:24 pm
With Text response, it seems as though when they give two prompts for a text, that one prompt is harder than the other- would the prompt difficulty have any influence on their marking scheme? Also, for analysis prompts (going by these categories http://www.vcestudyguides.com/types-of-essay-topics) I've been told that my essays for these types of prompts are almost too analytical and explicit in my analysis that it sounds more like a Language Analysis (e.g. "Through the author's use of the metaphor, (insert metaphor here), he/she exemplifies to readers the importance of ___). Do you have any tips on how to more subtly talk about what the author does to convey a certain meaning? Apparently I needed to focus more on the readers concerns and values rather than the language used to show what she is concerned about.
Title: Re: VCE English Question Thread
Post by: KingDrogba on March 08, 2015, 04:32:34 pm
With a comparative language analysis piece, what structure would you suggest to write with? Some teachers say analyse them as completely separate pieces (Eg: Analyse the first article on its own, then analyse the second comparing/contrasting to the first) whilst another teacher told me to compare/contrast in the same paragraph
What should i do?
Title: Re: VCE English Question Thread
Post by: scarletmoon on March 08, 2015, 04:56:37 pm
When starting essays with a quote, what is the maximum length of the quote? For example is this quote " conflict is the gadfly of thought, it stirs us to observation and memory. It shocks us out of sheeplike passivity and sets us at noting and contriving" too long to put at the start of an essay?
Title: Re: VCE English Question Thread
Post by: izzywantsa97 on March 08, 2015, 06:51:39 pm
With Text response, it seems as though when they give two prompts for a text, that one prompt is harder than the other- would the prompt difficulty have any influence on their marking scheme? Also, for analysis prompts (going by these categories http://www.vcestudyguides.com/types-of-essay-topics) I've been told that my essays for these types of prompts are almost too analytical and explicit in my analysis that it sounds more like a Language Analysis (e.g. "Through the author's use of the metaphor, (insert metaphor here), he/she exemplifies to readers the importance of ___). Do you have any tips on how to more subtly talk about what the author does to convey a certain meaning? Apparently I needed to focus more on the readers concerns and values rather than the language used to show what she is concerned about.

My teacher is a VCE marker and she says that they do take into consideration sometimes that some essay topics are harder! And for using metalanguage, generally start with the idea and then how the writer demonstrates it, and end with why the writer wants to send this message to the reader and the effect :)
Title: Re: VCE English Question Thread
Post by: appleandbee on March 08, 2015, 07:01:42 pm
If I'm doing a feature article for context, for the audience section is it enough to say 'readers of The Age' etc.? On that note, what kind of newspaper/magazine would it be appropriate to have a moderately sophisticated feature article (but with a couple of current/societal examples) focusing on 'how illusions/dreams/ feeling/internal self affect the way one perceives reality'?
Title: Re: VCE English Question Thread
Post by: thaaanyan on March 08, 2015, 09:13:10 pm
If I'm doing a feature article for context, for the audience section is it enough to say 'readers of The Age' etc.? On that note, what kind of newspaper/magazine would it be appropriate to have a moderately sophisticated feature article (but with a couple of current/societal examples) focusing on 'how illusions/dreams/ feeling/internal self affect the way one perceives reality'?

is this for your statement of intention????
often i'd suggest adding more depth than "readers of the age," you could still say that but then maybe follow up with something like "who have an interest in understanding the working nature of the realities and the illusions which constitute our identity and also professors of philosophy who wish to enhance their understanding regarding reality as a blah blah" (or something like that) the way i always figure is that if you add in the secondary specification you can also talk about how the language you use impacts your exact specific audience like "complex and jargonistic language in order to appeal to the academic nature of the reader's....."
if you need a magazine i'd make up one; eg. "The Science of Reality" a magazine which looks at the fabric of the universe around us through a rational lens and aims to both morally and scientifically quantify the essence/meaning of reality. totally fine to do that in context.

if i have totally misinterpreted what you're saying please ignore me haha. but yeah hope i helped! goodluck :)
Title: Re: VCE English Question Thread
Post by: Burt Macklin on March 09, 2015, 09:42:44 pm
Hi Lauren - again,

So I've figured out why I'm taking so much time (mostly because of running out of ideas/examples and inefficient brainstorming). I'm in the process of collecting more examples but am not too sure on what exactly I should do.

I've been providing some general overviews on examples, but I seem to have trouble relating them to ideas. I dunno, I feel like they aren't complex enough??  :-X Like I'm not going deep enough with the ideas?

(e.g. for Malala Youfazai - I've related her and the events associated with her to: culture and religion causing conflict, individuals showing courage and bravery amidst/after conflict, "positive" consequences)

Can you give an example of how to extrapolate ideas out of examples well?

Title: Re: VCE English Question Thread
Post by: literally lauren on March 09, 2015, 11:49:19 pm
With Text response, it seems as though when they give two prompts for a text, that one prompt is harder than the other- would the prompt difficulty have any influence on their marking scheme? Also, for analysis prompts (going by these categories http://www.vcestudyguides.com/types-of-essay-topics) I've been told that my essays for these types of prompts are almost too analytical and explicit in my analysis that it sounds more like a Language Analysis (e.g. "Through the author's use of the metaphor, (insert metaphor here), he/she exemplifies to readers the importance of ___). Do you have any tips on how to more subtly talk about what the author does to convey a certain meaning? Apparently I needed to focus more on the readers concerns and values rather than the language used to show what she is concerned about.
The difficulty of prompts is a matter of preference. Although some are objectively simple or difficult, VCAA are quite good at disguising things. Often a short, to-the-point statement can have a whole lot of hidden implications, while a complex looking structural prompt with an embedded quote or some difficult vocab can actually be boiled down to a very straightforward question. For 'analysis' type questions, you'll pretty much just have to transition between close textual evidence (structural features, language, meta-devices, that sort of thing) and the wider text's messages.
I suppose the best way to force yourself into this is to use the format of the question. Structural ones will usually begin with 'How...' as in 'How does the author's use of X create a sense of Y?' Occasionally some part of the equation will be missing and you'll have to fill in the blanks, eg. 'Discuss the author's use of X' or 'How does the author create Y?'
It all comes down to two questions though: if you've made a statement about the text, how do you know, and why is this important?

eg. Starting point: The character of Marlin in 'Finding Nemo' is initially unsympathetic.
How do I know: He's portrayed as an overbearing, didactic parent who does not allow his son to explore and grow, as is exemplified through the juxtaposition of Nemo with his more liberated peers. (--> seahorse, that other fish... a pink octopus I think?)
Why is this significant: This early representation serves as a contrast to the Marlin we see at the end of the film, thereby highlighting the importance of trusting one's child in order to be a cautious parent, but not overly so.


With a comparative language analysis piece, what structure would you suggest to write with? Some teachers say analyse them as completely separate pieces (Eg: Analyse the first article on its own, then analyse the second comparing/contrasting to the first) whilst another teacher told me to compare/contrast in the same paragraph
What should i do?
For your SAC, do whatever your teacher wants :)
For the exam, (and if your teacher is open to whatever method you prefer) I'd advocate the key player approach (explanation in the first page in this thread if you need.) Simply put, each paragraph focuses on a key idea or argument that the author uses to manipulate the audience. This allows you to transition between each article with ease, provided that's something your teacher is looking for.

So long as you're analysing language, you're in the clear. So comparing the articles isn't technically a part of the criteria; you could simply write a paragraph on each and be fine. It all depends on how fussy your teacher is.

When starting essays with a quote, what is the maximum length of the quote? For example is this quote " conflict is the gadfly of thought, it stirs us to observation and memory. It shocks us out of sheeplike passivity and sets us at noting and contriving" too long to put at the start of an essay?

There are no strict rules for Context essays, so the quote can be as long as you need. That one seems perfectly fine; it'd only be a problem if you were quoting more than three or four sentences. Even then, it's a writing task, not an 'essay' per se. If you want to intersperse your essay with multiple quotes, you can do that too. Just don't make the quotes do the work for you - your own exploration is what the assessors want to see :)

If I'm doing a feature article for context, for the audience section is it enough to say 'readers of The Age' etc.? On that note, what kind of newspaper/magazine would it be appropriate to have a moderately sophisticated feature article (but with a couple of current/societal examples) focusing on 'how illusions/dreams/ feeling/internal self affect the way one perceives reality'?

Assuming this is for your Statement of Intention (?) I guess that should suffice. Mind you, my teacher was pretty chill with the Statement thing, we were just told to write something valid so he could tick it off. However, I know some schools give more credence to the SOI, so it's probably worth checking with your teacher.

If you're looking for a precise publication that you could use, The Age or The Australian would be fine. You could always browse a few major online/print companies until you find one that writes the kind of pieces you're emulating. That would probably help you refine your writing style as well as give you some idea of the format and structural specifics :)

Hi Lauren - again,

So I've figured out why I'm taking so much time (mostly because of running out of ideas/examples and inefficient brainstorming). I'm in the process of collecting more examples but am not too sure on what exactly I should do.

I've been providing some general overviews on examples, but I seem to have trouble relating them to ideas. I dunno, I feel like they aren't complex enough??  :-X Like I'm not going deep enough with the ideas?

(e.g. for Malala Youfazai - I've related her and the events associated with her to: culture and religion causing conflict, individuals showing courage and bravery amidst/after conflict, "positive" consequences)

Can you give an example of how to extrapolate ideas out of examples well?
If that's the case, then make your brainstorming/notes better before attempting essays. It's perfectly fine, especially at this stage of the year, to need a whole heap of exploration for your evidence. In the exam, it'll probably be enough for you to scribble 'Malala,' 'Aztecs' and 'Spiderman' just to jog your memory.

For now though, spend some time unpacking things.
So that you know what you should be relating these ideas to, it's also probably worth going through a list of prompts and finding key thematic areas (eg. causes of conflict, how people's responses to conflict differ, why conflict can unite or divide people, etc.) That way, when you're thinking about your evidence, you'll know what parts of it are most significant.

Most of the research you do for your examples will just be a means of familiarising yourself until you're confident discussing it. In your essay, you'll just be giving your reader whatever is relevant to your discussion, so you should always know a little bit more than what you're writing.

There's also a chance that you're not choosing the right examples. Some are inherently less complex and relatable, which is not to say that you can't use them, but rather that it's better you leave these ones for later in the year after you're comfortable with how evidence integration and Context pieces work.

Try and broaden your discussion as much as possible, and then hopefully things will start to make more sense as you go :)
Title: Re: VCE English Question Thread
Post by: chansena on March 10, 2015, 05:41:04 pm
Hi ALL!

For context I am looking at writing a Feature Article, and think through a feature article it will allow me to explore context more. But i do not have an example piece. Could someone please post/ PM an example piece of a feature article preferably identity and belonging (but i don't mind if its anything else too ) I just need an example at the moment so i can use it as a guide. :)

Thanks


 
Title: Re: VCE English Question Thread
Post by: Eiffel on March 10, 2015, 05:47:17 pm
honestly speaking, if you get 50% on a sac are your chances of 50 gone? lets assume the class overall did well (say 80% ave)
Title: Re: VCE English Question Thread
Post by: Splash-Tackle-Flail on March 10, 2015, 07:20:38 pm
The difficulty of prompts is a matter of preference. Although some are objectively simple or difficult, VCAA are quite good at disguising things. Often a short, to-the-point statement can have a whole lot of hidden implications, while a complex looking structural prompt with an embedded quote or some difficult vocab can actually be boiled down to a very straightforward question. For 'analysis' type questions, you'll pretty much just have to transition between close textual evidence (structural features, language, meta-devices, that sort of thing) and the wider text's messages.
I suppose the best way to force yourself into this is to use the format of the question. Structural ones will usually begin with 'How...' as in 'How does the author's use of X create a sense of Y?' Occasionally some part of the equation will be missing and you'll have to fill in the blanks, eg. 'Discuss the author's use of X' or 'How does the author create Y?'
It all comes down to two questions though: if you've made a statement about the text, how do you know, and why is this important?

eg. Starting point: The character of Marlin in 'Finding Nemo' is initially unsympathetic.
How do I know: He's portrayed as an overbearing, didactic parent who does not allow his son to explore and grow, as is exemplified through the juxtaposition of Nemo with his more liberated peers. (--> seahorse, that other fish... a pink octopus I think?)
Why is this significant: This early representation serves as a contrast to the Marlin we see at the end of the film, thereby highlighting the importance of trusting one's child in order to be a cautious parent, but not overly so.


Thank you so much! loved the example hah!
Title: Re: VCE English Question Thread
Post by: literally lauren on March 10, 2015, 09:53:51 pm
honestly speaking, if you get 50% on a sac are your chances of 50 gone? lets assume the class overall did well (say 80% ave)
Honestly speaking, I have no earthly clue what anyone's chances of a 50 are. 'Assuming the class did well' is pretty vague; you could 100% everything from now on which would bump you up the rankings, but realistically, going from 50% to 100% doesn't happen overnight. Plus, there's so much score-changing behind the scenes, so I can't possibly say definitely yes or definitely no. I wouldn't even be confident guessing even after all the SACs are over, let alone after the very first one :P

So I'll resort to my default response to questions like this: does it matter? If you knew you could work like crazy and the very best you could end up with was a 49, would you stop trying? I get that 50 is the aim, and I genuinely applaud you for aiming so high, but constantly obsessing over whether a 50 is possible throughout the year is not the best way of going about things. By all means be aware of your capabilities, but putting the numbers out of your head and just getting work done is far more efficient, and you'll probably get a pleasant surprise come results day.
Title: Re: VCE English Question Thread
Post by: Eiffel on March 10, 2015, 10:25:10 pm
Very true haha

I haven't got 50 yet (%) but I just feel bad about my sac haha....
Just gotta put the hard work in , and I guess doing awesome on the exam will help significantly.

Do you think teachers will ever accept bribes? Honest question
Title: Re: VCE English Question Thread
Post by: cosine on March 10, 2015, 10:33:51 pm
This is how I feel right now for english. There's no hope for me. It seems as much as I try I just see no improvement haha
Title: Re: VCE English Question Thread
Post by: heids on March 10, 2015, 11:19:30 pm
This is how I feel right now for english. There's no hope for me. It seems as much as I try I just see no improvement haha

Well, Magic worked for me 8) so keep your hopes up
Title: Re: VCE English Question Thread
Post by: izzywantsa97 on March 11, 2015, 04:47:18 pm
We have our first essay sac tomorrow and basically I need help incorporating metalanguage/author's craft in. How are you supposed to do it?
Title: Re: VCE English Question Thread
Post by: chansena on March 12, 2015, 05:03:25 pm
We have our first essay sac tomorrow and basically I need help incorporating metalanguage/author's craft in. How are you supposed to do it?

What i find useful is always answering,  how does the author do this? I use this when making a new point.  This allows one to explore metalanguage and give an insightful response

Hope this helps :)
Title: Re: VCE English Question Thread
Post by: Eiffel on March 12, 2015, 11:46:23 pm
how often are you guys doing practice essays etc?
Title: Re: VCE English Question Thread
Post by: scarletmoon on March 13, 2015, 03:15:27 pm
how often are you guys doing practice essays etc?

I'm going to aim to do at least 1 essay a week or a fortnight
Title: Re: VCE English Question Thread
Post by: JackSonSmith on March 13, 2015, 09:39:41 pm
Here is a link to my personal favorite essay of all time.  (The first essay in the report)

http://www.vcaa.vic.edu.au/Documents/exams/english/english_assessrep_11.pdf


What would the weaknesses of this essay be? What are your opinions of this essay?

Opinions are welcome from everyone.
Title: Re: VCE English Question Thread
Post by: literally lauren on March 14, 2015, 12:50:19 am
I haven't got 50 yet (%) but I just feel bad about my sac haha....
Just gotta put the hard work in , and I guess doing awesome on the exam will help significantly.

Do you think teachers will ever accept bribes? Honest question
This is how I feel right now for english. There's no hope for me. It seems as much as I try I just see no improvement haha
Okay, I know the posts above are at least partly in jest, but I'm going to answer these seriously because I think there's an underlying problem here that's unique to English subjects.

Most subjects consist of a fairly straightforward series of tasks: sit down, do exercises 1-5, Qs. a-g; memorise vocab list from chapter 4; do a practice exam, that sort of thing. Many people would argue you can treat English the same way, and that it's just a matter of finding the right activity to work on.

This approach isn't invalid. Plenty of people have scored very highly by simply understanding the system, writing heaps (/"spamming" essays as I'm told da kidz are calling it) and memorising chunks of whatever works.
I feel like this is perpetuated by a lot of schools, tutors, and even professional companies because it's a comfort. Being told you can write a certain number of essays and be certain of a reasonably good mark is nice to hear. And whilst it's not totally untrue, I do think it's contributing to some serious misconceptions regarding the subject.

The best kept secret, I've found, is that although you may score well if you know what to write, you will definitely score well if you know how to think.

This is an unpopular view for good reasons; it's not like a teacher with a class of 30 can feasibly teach students how to think within 8 months, ~4 hours a week, whilst also conforming to a syllabus and the pressure of exams. I'll admit I've caved to this pressure as well with a few of my students, and end up just focusing on criteria and assessment because of time restrictions or other issues.

But English is a subject that rewards thinking.

You can rote-learn. You can know your texts inside-out and study high-scoring responses and annoy your classroom teacher until they give you so much feedback you can predict what they're going to say. What's more, you're going to feel good about this, because you're physically doing tangible work that your brain associates with progress. Even if you're hitting some mental blockades with the content, it'll still feel like improvement because you're working at it.

For some people this is the only way they learn. Quite a few people in my year level would be writing essays from day one. They were atrocious essays, and were in no way indicative of their abilities, but for them, it was a way of consolidating their knowledge.

But the only reason why this works is because of the (often unintentional) tangential benefits.

Doing the 'class-assigned' kinds of activities is a slow-but-effective way to better your thinking. So why do this when you could go for a fast-but-effective method?

Well, partly because it's going to feel slower. You'll be doing unfamiliar things, and for a long time, you'll probably be doing them badly. However, as someone who's seen the system from both sides now, I've concluded this is a much better way of tackling the course.

For starters, let's clarify what's meant by 'thought.' We all know what it means, but strangely it's not a word that gets tossed around in English classes quite so much as 'juxtaposition' or 'inclusive language' might. This can be attributed to it's abstract-ness: your teacher can't see you thinking, save for seeing the result of the process in essay-form.

So when you're getting essay feedback, you're receiving criticisms for the product of your thinking, right? (-Excluding handwriting issues or minor structural things that you maybe weren't aware of.) Here's where problems start to arise...

For anyone who's worked in retail/ hospitality, you'll probably be familiar with copping blame for things totally out of your control. I used to work in a chicken and chip shop, and I'd have customers who would come in and complain about everything under the sun; chips aren't cooked right, the salt is to salty, the chickens are too small, why does this salad have lettuce in it? etc. etc.
I was but a mere server-girl, and so, realistically, if these bitterly displeased customers actually wanted their problems solved, they would have addressed the root of the problem and not hurled abuse and/or utensils at me.

Your English teachers, in this somewhat tortured metaphor, are akin to my old chicken shop patrons. They're not trained to consider the source of their discomfort, be that a misunderstanding of the text or the fact that our shop was drastically understaffed most nights; instead, their natural inclination is to blame what is made apparent:  the wrong words in a body paragraph, or a slightly dirty fork.

When you get comments on your essay like 'needs development' or 'I'm not sure about this,' what your teachers are actually-sort-of-kinda-but-not-really-trying-to-say is change your thinking! But that sounds weird because it's easier for them to focus on your essay itself, and it's less strenuous for students to obsess over numerical scores or criteria than it is to consider the possibility of mind-altering-drugs-study instead.

Now prepare yourselves for
Lauren's foolproof guide for How to Think Good
To demonstrate this we're going to look at a textual excerpt. You don't have to know anything about it, in fact it's better if you don't. I'm adapting this from an Andrew Bovell play called 'Speaking in Tongues' if anyone's interested.
         VALERIE: [answering machine] John, it's me... Valerie. I wish you'd let me do the message.
                       You sound so... I don't know... distant.

To learn how to think properly for English (/Lit, which is what I usually use this example for, but tomaito tomahto) all you have to do is answer this question: What do you know about Valerie and John?
That's all. But fair warning, my answer to this question is over 2000 words long, and that's all without reference to the play this came from.

Most people will fumble for a starting point at first, like 'well, we know she's talking to John on an answering machine, and that she wants to do the message instead of him.' Later, once you get past the basic, denotative stuff, you'll end up in 'assumptions' territory, eg. maybe they're not getting along, and that's why he sounds distant and she's not allowed to do the message. Keep building on this, and eventually you'll reach full blown implications: John is trying to maintain some semblance of power in their marriage by exerting control over petty things like which of them record an answering machine message. Meanwhile, Valerie is able to undermine his authority through criticism; she is still able to voice her objections, meaning he does not have complete command over her.
That's not to say there is a 'right' answer. You could go in a completely different direction, eg. The fact that the two are communicating via an answering machine - an innately indirect form of conversation - suggests they are not able to engage with one another on any level. Both John and Valerie are "distant," and without artificial conduits like answering machines between them, their relationship has very little holding it together.

Evidently what I'm talking about here is more like overthinking than just thinking, but perhaps that's appropriate.
Let me be clear: this will not directly help you. You should not spend 200 words in an English body paragraph analysing two sentences from the text/article. This is not about a subject-specific skillset, this is about rewiring your brain to look at things differently.

In the above exercise, I extrapolated from two lines of dialogue and concluded that the couple had a serious communication barrier between them, and were likely in the midst of some confusion regarding the power balance between them. I could be wrong, but that's not the point. The point is that I can justify my thinking.

I had a teacher who conducted a similar exercise in class and ended it with 'of course you couldn't say something ridiculous like 'this excerpt suggests John wants to grow a beard' or anything.' But I disagree. 
John's lack of control over Valerie signifies his emasculation ,which is exacerbated by Valerie's implied criticism. She is able to express her wants in no uncertain terms, and her power is marked by a stereotypically feminine "I wish you'd let me" brand of passive aggression. Thus, it seems logical then for John to gravitate towards physicality as a means of reasserting himself with something equally gender codified; perhaps a handlebar mustache, or even a proper, fully-fledged man-beard - an ideogram of his patient but firm dominance.
Note: I would never seriously write that in an essay. This was an exercise in thinking, and taking my analysis further than the surface level. Do this often enough, and you begin to get a feel for what actually belongs in an essay, and what's just conjecture.

I know this is quite text-heavy, but a willingness to read is just important as a willingness to write for English :)

Yes, it's frustrating when you're not making obvious numerical improvements, and yes, working out what constitutes as "useful" study is a lot tough in English than it is in other subjects, but it ultimately boils down to your willingness to engage with the material.

Think about stuff, and learn how to demonstrate this thinking in the best way possible. No magic required :)

Title: Re: VCE English Question Thread
Post by: literally lauren on March 14, 2015, 01:35:56 am
Here is a link to my personal favorite essay of all time.  (The first essay in the report)

http://www.vcaa.vic.edu.au/Documents/exams/english/english_assessrep_11.pdf


What would the weaknesses of this essay be? What are your opinions of this essay?

Opinions are welcome from everyone.
I'm assuming you mean the Bypass Text Response piece?
so glad that text isn't on the list anymore. The Chief Assessor's buddy (also an English teacher) wrote it. Freaking nepotism man, the system's corrupt -.-

Firstly, this would be a pretty solid 10. I mean, I'd give it a 9-ish (~9.3 to be exact,) but it's very well constructed, well expressed, I just think it has some significant faults.

Let's look at the introduction:
Quote
In his meandering tale Bypass: the story of a road, Michael McGirr leads his readers on a journey down 'Australia's main street,' there's no need to quote this; it accomplishes nothing the Hume Highway ensuring that the stretch of bitumen is seen in a unique and refreshing way. This is a filler sentence. I know a lot of people use these just to kick off their intros, and I know some teachers even advocate for them, but personally I find them kinda irritating. General summations of the text and what it accomplishes don't tell me anything new or interesting, and engaging with the prompt from the outset would have been a much better starting point. From his bicycle saddle, McGirr is able to make use of his keen eye for detail as he this could easily be summed up with a couple of words in a much less clunky manner, eg. 'conveys' or 'hones in on.' It's also teetering on that dangerous brink of fawning! >:( >:( >:( Any sentence that's saying something like 'The author's expert elucidation of the human psyche is skillfully crafted to make readers weep with appreciation and joy' is so pointless it makes me weep with boredom. Admittedly, since this text was actually being marked by the author of the poxy text I guess this might have been a strategic thing, but VCAA have since raised their standards, so I'd advice everyone else to steer clear observes, ponders, and enlightens on w.c. You can't 'enlighten on' something the intricacies of human behaviour. Not only does his unique and honest narrative structure bleh, get to the point detail his journey from Sydney to Melbourne, it also offers an insight into the personal and spiritual journey that McGirr has embarked on. FINALLY we get some facet of the prompt being addressed! This is a good starting point, it's just a shame it's the third sentence instead of the first  ::) His physical journey is accompanied by anecdotes from the past, historical insights and aspects of his immense knowledge of literature, as well as the constant embellishment of self-deprecating humour. It's a bit 'list-y' and I'm squinting to see the relevance, but this is getting better. From all Big no-no. Never use absolutes like this, no matter how confident you are aspects of the journey that McGirr includes in his memoir minor quibble, but I can see it a lot in this piece: it's very inefficient. Here the student has written "the journey that McGirr includes in his memoir" instead of just "McGirr's journey" or even just "the text." There's a lot of redundancy and over-clarification - which admittedly is better than under-clarification, but is still a slight flaw his readers are able to learn more of the man himself. In this way, Bypass: the story of a road proves not only to be a detailed account of a bike ride *sigh* have I mentioned how glad I am that this text is gone? but a collection of opportunities to lean about McGirr's own character.
Based on this paragraph alone: meh/10.
Several redundancy issues, not a whole lot of development of the contention (if you can even call "yes, McGirr's character is revealed in his book" a contention,) and it's really not doing anything promising.

The strength is definitely in the body paragraphs, but even then, not entirely perfect. I think having 4 fairly short paras works to the student's disadvantage here because there's very little opportunity for deeper exploration. The majority of points, especially in the second, third, and fourth paras hinge on one or two examples, which is a shame because some of the exploration is well-handled. But when you're trying to draw big conclusions about the author as a character, just giving a really rough overview of key points of evidence is a bit problematic.

There are some missed opportunities for linking sentences at the ends of body paragraphs, and it kind of leaves too much up to the reader to project the analysis onto the essay. The last line is a rather neat summation of the piece (even though it breaks the stringent rule about never EVER using the word 'prove' in English - nothing is ever 'provable,' just 'inferable.')

Whilst I would definitely recommend reading high-calibre essays often, be careful not to glorify certain pieces. The idea should be to read heaps and take little bits and pieces here and there (like, ooh, that kid writes a mean topic sentence, and wow, this essay has some nice quote integration - *STEAL*)
But you don't want to be replicating any approach or format just because someone else scored well with it, especially because you'll be writing on a different prompt for a different (and invariably in this case, better) text.
Title: Re: VCE English Question Thread
Post by: KingDrogba on March 14, 2015, 11:11:58 am
What scores do you generally need to achieve a 40 in English?
Both Sacs and exams.

Cheers
Title: Re: VCE English Question Thread
Post by: literally lauren on March 14, 2015, 01:00:48 pm
What scores do you generally need to achieve a 40 in English?
Both Sacs and exams.

Cheers

Generally, 8/10s.
HOWEVER...
- SAC marks will change depending on cohort & exam performance
- getting 8/10 is much easier with some teachers than others
- some teachers mark harshly to scare you into working, and others mark too easy to boost confidence
- getting two or three 8/10s in a row doesn't guarantee you'll get the same score next time
- most people's performance is dependent on prompts (a.k.a. luck on exam day)
- thinking 'oh sweet, I'm getting 8s/9s, imma end up with a 45' is erroneous, and probably the best way to lull yourself into a false sense of security
- likewise, thinking 'Gosh dangit, I'm only getting 7s; oh well, no hope for me then' is also untrue and unhelpful

I think I've made my point :) If you obsess over scores, it will only work to your disadvantage. Focus on the work, and be pleasantly surprised by the end result.
Title: Re: VCE English Question Thread
Post by: xleannenguyen on March 15, 2015, 08:02:07 pm
Hi Lauren,
I have trouble 'analysing' or going more in-depth when it comes to Text Response.
I also have difficulties trying to write with clarity and tightening my expression
Is there some kind of structure/formula when writing with complexity?

Thanks :))
Title: Re: VCE English Question Thread
Post by: chansena on March 17, 2015, 06:43:15 pm
Hi,

I am trying to write sentences with a didactic tone, but i am struggling.

I have this thus far.

Children have become immune to their phones, back in the day we would play outside. Is this an example of a didactic tone ?

Could i also have some more examples please . 

Thanks!!


Title: Re: VCE English Question Thread
Post by: StupidProdigy on March 18, 2015, 05:55:02 pm
Hi Lauren,
I'm not sure if this sentence makes sense? It's just a topic sentence for a paragraph I'm trying to write but I'm not confident I even understand fully what I have said (mainly the extols the virtue of family bit)
'Adiga extols the virtue that family is the most critical feature to an individual and essentially connotes that a “man” must place family at the foremost of his life.'
Thankyou!
Title: Re: VCE English Question Thread
Post by: S33667 on March 19, 2015, 09:44:11 am
Hi Lauren,

I just want to thank you for the recommendation to watch Utopia as part of Whose Reality.   After season 1 I was hooked but couldn't quite work out the relevance to my area of study. 

I've just finished Season 2 - All I can say is WOW, talk about messing with my head and making me see reality from different perspectives.   A brilliant TV series and really thought provoking stuff  :)

I'm devastated there's no Season 3  :-X
Title: Re: VCE English Question Thread
Post by: literally lauren on March 19, 2015, 11:56:02 am
Hi Lauren,
I have trouble 'analysing' or going more in-depth when it comes to Text Response.
I also have difficulties trying to write with clarity and tightening my expression
Is there some kind of structure/formula when writing with complexity?

Thanks :))

Are these two issues related? As in, do you have issues with clarity because you're not going into enough depth?
'Clarity' is a fairly ambiguous terms, so let's try and break that down. Firstly, can you understand your own writing? Is it clear to you and not to your teacher, or can you notice the faults and confusion as you read? If it's the former, then you'll need your teacher to explain why specifically your writing isn't clear, and what you can do to fix it. But if you can tell there's something wrong, great! This means you have the right grammar rules in your head; you just get lost because you're rushing or forget to self-edit when you're following the flow of your ideas. If you know you're doing something wrong, you'll probably also have the same intuition when it comes to fixing your mistakes.

With regards to depth - do you not know what to write, or do you not know how to write it? Again, if it's the former, then perhaps sit down with your teacher and try to discuss the text, or read some online resources incl. other people's essays if they're available to you. Having trouble with what to write is an issue with content and understanding, so go back to the text and even comprehension resources if you have to (eg. depending on your text, you might find it helpful to do chapter summaries or annotations that highlight these points where going in-depth is necessary.)

However, if you've got the ideas sorted and just can't put them into words, then chances are you need to work on your expression and/or vocab. Try and be as specific as possible with where you're having trouble. One question many people find helpful is to ask 'when exactly do I hesitate/stop?' If you find yourself pausing at the ends of sentences with no idea where to go, or you finish a paragraph and feel like you've got nothing else to say next, then that's a problem that could be solved with adequate planning and thinking ahead. Whereas, if you stop mid-sentence and feel you don't have the right words at your disposal, then of course your vocab probably needs work.

Now that you've isolated an area that you need to work on, see if you can hone in on the specifics as to where and why you're going wrong :)

Hi,

I am trying to write sentences with a didactic tone, but i am struggling.

I have this thus far.

Children have become immune to their phones, back in the day we would play outside. Is this an example of a didactic tone ?

Could i also have some more examples please .
Not sure why a didactic tone is necessary, but to break it down: didactic means 'designed to teach,' so the tone you're going for would be a sort of aggressive school teacher barking commandments at you. You want to go for very definitive and informative sentences; I'd imagine most newspaper opinion pieces would be good examples of this :)

Hi Lauren,
I'm not sure if this sentence makes sense? It's just a topic sentence for a paragraph I'm trying to write but I'm not confident I even understand fully what I have said (mainly the extols the virtue of family bit)
'Adiga extols the virtue that family is the most critical feature to an individual and essentially connotes that a “man” must place family at the foremost of his life.'
Thankyou!
Mostly okay, but there are a few little issues. 'extols the virtue' is a little clunky, but understandable. 'Connotes' means 'hints at,' but it's usually meant to be used when referring to single words or phrases (eg. the word 'leader' connotes strength and influence, or the phrase 'deaf as a doorknob' connotes ignorance or being unwilling to consider new ideas.) In this case it might be better to use a word like 'suggests' or 'shows.'

Hi Lauren,

I just want to thank you for the recommendation to watch Utopia as part of Whose Reality.   After season 1 I was hooked but couldn't quite work out the relevance to my area of study. 

I've just finished Season 2 - All I can say is WOW, talk about messing with my head and making me see reality from different perspectives.   A brilliant TV series and really thought provoking stuff  :)

I'm devastated there's no Season 3  :-X
You're very welcome! Hope it didn't scar you too much :s In terms of linking it to WR, I probably wouldn't use it as an expository-type example, but funnily enough it relates really well to the idea of utopia and ideal realities. Without spoiling too much for those who haven't seen it, suffice it to say that the two conflicting groups in the series both believe they're doing what's right, and the show plays with that idea in an interesting way.

You could link it to some of the real world ideologies that take a similar view to The Network, and perhaps even link this into a discussion of terrorism and whether these realities can be effective, or the extent to which their coming into conflict is a beneficial. Mainly just a source of inspiration for you to get started :)
Title: Re: VCE English Question Thread
Post by: S33667 on March 19, 2015, 12:37:07 pm

You could link it to some of the real world ideologies that take a similar view to The Network, and perhaps even link this into a discussion of terrorism and whether these realities can be effective, or the extent to which their coming into conflict is a beneficial. Mainly just a source of inspiration for you to get started :)

This is exactly how it's helped me !   I've been trying to explore ideas around Daesh and foreign fighters.   I've been really focused on propaganda and was a bit lost how people could buy into it all.   But Utopia has really made me think deeper about the impact of an ideologies on your reality.   (and how easy it is to be swayed to a different belief system ... no spoilers, but there was a point where I was wondering if Leporidae was right and should be successful)
Title: Re: VCE English Question Thread
Post by: literally lauren on March 19, 2015, 01:24:54 pm
This is exactly how it's helped me !   I've been trying to explore ideas around Daesh and foreign fighters.   I've been really focused on propaganda and was a bit lost how people could buy into it all.   But Utopia has really made me think deeper about the impact of an ideologies on your reality.   (and how easy it is to be swayed to a different belief system ... no spoilers, but there was a point where I was wondering if Leporidae was right and should be successful)

I think that's the mastery of the series - to take such an unequivocally immoral concept and frame it in such a way that you end up almost aligning yourself with the 'evildoers.' It might also be worth juxtaposing this with some psychological theories about people's beliefs and how rigid realities can be - Justification theory, Fallibilism, and Confirmation bias come to mind as some starting points. In fact, even if you're planning on writing an imaginative piece, this kind of research can really help inform your writing. Wikipedia links above should provide you with a rough outline, but you could always do further reading if something piques your interest; let me know if anything doesn't make sense :)
Title: Re: VCE English Question Thread
Post by: Splash-Tackle-Flail on March 26, 2015, 05:42:59 pm
In text response, when writing a prompt, do we have to cover all major events in the novel? Also, would evidence not covered in class (aka minor evidence i guess) create a better impression of your essay to examiners, rather than just using the events discussed by the teacher? How relevant does our response have to be to a prompt, as far as trying to discuss ideas that may go off on a tangent?

Thanks!
Title: Re: VCE English Question Thread
Post by: heids on March 26, 2015, 05:55:18 pm
In text response, when writing a prompt, do we have to cover all major events in the novel? No.  Only evidence relevant to your discussion.
Also, would evidence not covered in class (aka minor evidence i guess) create a better impression of your essay to examiners, rather than just using the events discussed by the teacher? The examiner won't know what the teacher taught you.  Still, using minor events shows deeper, wider knowledge of the text rather than shallow surface level, while turning points/major events show you understand the 'big moments', the crux of what's going on.  So I'd use a mixture.  Definitely analyse in depth at home by yourself, do your own investigation rather than just relying on the teacher.  It'll add complexity and insight to your ideas.
How relevant does our response have to be to a prompt, as far as trying to discuss ideas that may go off on a tangent? Relevance is vital!  You can try to weave in stuff you know well, but still have to make it as relevant as possible.  Every single exam report has stressed 'don't be formulaic', i.e. don't rely on pre-prepared somewhat irrelevant stuff.  Everything you say should be trying to discuss the implications of the prompt, not what you would have liked the prompt to be.

(sorry I'm not Lauren :P)
Title: Re: VCE English Question Thread
Post by: Splash-Tackle-Flail on March 26, 2015, 06:56:49 pm
(sorry I'm not Lauren :P)

Still really helpful haha! Thank you so much!
Title: Re: VCE English Question Thread
Post by: JackSonSmith on March 26, 2015, 07:31:35 pm
I just wanted to clarify something.

So long as you reference ideas in your selected text and respond to the prompt, you can write anything in context?
Title: Re: VCE English Question Thread
Post by: literally lauren on March 26, 2015, 08:00:30 pm
I just wanted to clarify something.

So long as you reference ideas in your selected text and respond to the prompt, you can write anything in context?

*suspicious glance* Define "anything."
Title: Re: VCE English Question Thread
Post by: scarletmoon on March 26, 2015, 08:58:13 pm
How do you convert 28/30 into a score out of 10?
Title: Re: VCE English Question Thread
Post by: JackSonSmith on March 26, 2015, 08:58:19 pm
*suspicious glance* Define "anything."

Some ideas I had: expository/persuasive essay, opinion piece, speech, collection of short stories, brochure?, cartoons?
Title: Re: VCE English Question Thread
Post by: JackSonSmith on March 26, 2015, 08:58:50 pm
How do you convert 28/30 into a score out of 10?

Probably 9/10, as 28/30 = 0.933.
Title: Re: VCE English Question Thread
Post by: literally lauren on March 26, 2015, 10:33:06 pm
Some ideas I had:
expository/persuasive essay ✓ this would be absolutely fine, and is the most popular form

opinion piece/ speech ✓ ditto, very common, good opportunity if you're a naturally persuasive writer

collection of short stories ✓ valid, but keep in mind this is a ~1 hour writing task, so don't over-reach

brochure? - not sure how you'd do this without folding up your exam papers (and there's probably some ridiculous rule about that) or needing to format weirdly. You can definitely create an informative/expository style piece that would contain the same information as a brochure, but the idea is to have a cohesive piece of writing rather than little tidbits.
Having said that, I did have a few students last year who wrote newspaper/ opinion pieces and actually wrote in columns down the page (ie. they would have taken up maybe five pages of exam paper, but each one would be vertically divided down the middle with a little gap, so the formatting is permissible provided the content is valid.)

cartoons? - I doubt you'd get away with this. I'd love for it to be allowed, but to be honest VCAA have only recently even acknowledged cartoons and multimodal texts as a form of literature; I don't think they'd accept it for a "writing task."
If the concepts and creative progression in graphic novels inspires you, by all means implement that in your writing, but don't stray too far from the idea of an essay if you want to remain in safe territory.

So I guess I should change my Context motto to: 'Anything* goes!*subject to terms and conditions
Title: Re: VCE English Question Thread
Post by: scarletmoon on March 27, 2015, 04:22:15 pm
Is it bad to start a conclusion with "In essence..."? I like it but my teacher told me to avoid conclusions that start like this and I'm not sure why.
Title: Re: VCE English Question Thread
Post by: Chang Feng on March 28, 2015, 09:39:00 pm
Just wondering, how would you go about structuring your context notes? like i know for text response, you should do character profiles, and thematic analysis. so for context would you do the same things or, what should you do???
THANKS
if that makes sense
Title: Re: VCE English Question Thread
Post by: JackSonSmith on March 28, 2015, 10:54:23 pm
Just wondering, how would you go about structuring your context notes? like i know for text response, you should do character profiles, and thematic analysis. so for context would you do the same things or, what should you do???
THANKS
if that makes sense

So far, I've been making my context notes and filling them under events. eg. Vietnam war: Conflicts that are in parallel with events in Every Man in This Village is a liar.

Essentially I'm just putting my ideas down and then I plan to group them and organise them into categories at a later date, once I have more confidence with my knowledge.
Title: Re: VCE English Question Thread
Post by: Escobar on March 29, 2015, 12:11:54 pm
this may seem like a weird question but...
this is from Edrolo:
(http://i.imgur.com/E04yipp.png)
as you can see, the answer is A) and C)
how is the answer A) when it is asking for money, not talking about saving money?
Title: Re: VCE English Question Thread
Post by: JackSonSmith on March 29, 2015, 12:54:11 pm
this may seem like a weird question but...
this is from Edrolo:
(http://i.imgur.com/E04yipp.png)
as you can see, the answer is A) and C)
how is the answer A) when it is asking for money, not talking about saving money?

Perhaps they are appealing to the 'hip-pocket' of the team. eg. the team needs donations to stay afloat.
Title: Re: VCE English Question Thread
Post by: literally lauren on March 29, 2015, 09:19:59 pm
Is it bad to start a conclusion with "In essence..."? I like it but my teacher told me to avoid conclusions that start like this and I'm not sure why.
It's not inherrently bad, but if your teacher doesn't like it, then avoid at all costs.
You might be using it in a strange way, ie. you use a linking phrase but the ideas themselves don't link; you can't just say 'In essence,' at the start unless you're actually talking about the essence of the author's point or your contention. Otherwise it's just a regular conclusion with an odd starting phrase.
In the exam, no teacher would dock marks just for using a word or phrase like this, you just have to be careful that your usage is appropriate.

Just wondering, how would you go about structuring your context notes? like i know for text response, you should do character profiles, and thematic analysis. so for context would you do the same things or, what should you do???
THANKS
if that makes sense
First, go through a bunch of prompts for your Context (there's a big collection in the Resources thread on the main English page if you need) and pick out the major thematic areas you find

eg. Encountering Conflict
- causes of conflict
- the role of power and its effects
- whether conflict is fair/ justifiable
- the lessons we learn from conflict
        ect.

Under each of these umbrella sections, start fleshing out your examples and research. There'll be some overlap, so it's up to you whether you double up and explain the same examples in different ways multiple times, or whether you just type a big version out once and then put (refer to...) in other sections where relevant.
This'll also be an excellent way of finding out where the gaps in your knowledge/ research are. If you find that you have absolutely no examples for, let's say, 'the victims of conflict,' then if you got a prompt pertaining to this area you'd be in trouble. Not only does this help you collate all your examples, but if can help you brainstorm what else you should be reading and exploring for the sake of covering as much of the context as possible.

this may seem like a weird question but...
this is from Edrolo:
as you can see, the answer is A) and C)
how is the answer A) when it is asking for money, not talking about saving money?
I've never heard of Edrolo before so I have no idea what their quality is like, but this seems like the sort of pedestrian analysis the assessors have been criticising lately. Just looking at a dollar sign and calling it an 'appeal to hip-pocket nerve' is like calling anything with a question mark in it a 'rhetorical question' - even if it's true, it's not analysis!

I guess if you're abstractifying the idea of an appeal to financial concerns to mean 'any technique pertaining to the mere mention of money' then this might be accurate, but I'd say you're more likely to elicit an eye roll from an exam marker if you used the 'hip-pocket nerve' as part of your analysis here.

I'm getting strong vibes from Assessor's Reports and the teacher grapevine that the exam will be gearing well away from this kind of technique-labelling anyway. You'd be much better off practicing commentary on kinds of language (eg. imperative, superlative) or understanding the context of persuasion. Techniques are still a good starting point, but they're just a starting point.

edit: JackSon's suggestion would make sense too, but Answer C seems much more sensible. Idk why they've said that  ::)
Title: Re: VCE English Question Thread
Post by: Escobar on March 29, 2015, 09:40:14 pm
Thanks for your reply :)
our school paid for access to Edrolo, which is a website with lectures
the person in the video is supposed to be an assessor and have a lot of qualifications so seeing that answer was really strange
Title: Re: VCE English Question Thread
Post by: literally lauren on March 29, 2015, 10:11:28 pm
Thanks for your reply :)
our school paid for access to Edrolo, which is a website with lectures
the person in the video is supposed to be an assessor and have a lot of qualifications so seeing that answer was really strange
Since the question was out of context, maybe their explanation made more sense, but objectively speaking it seems like a fairly weak (if not totally wrong) analysis.
Doesn't mean the rest of the content isn't valuable though - almost all teachers have their own strange preferences and recommendations that don't really gel with an exam-perspective, but they're still capable of giving good advice in other areas.
Still, never hurts to double check here or with your teacher if you're ever unsure about something :)
Title: Re: VCE English Question Thread
Post by: Chang Feng on April 01, 2015, 10:47:56 am
THanks.
and also for context, are we required to have quotes from the set text. Or do we need to only take evidence from set text and then analyse how this is relevant to the broader context ie encountering conflict.
Title: Re: VCE English Question Thread
Post by: Callum@1373 on April 02, 2015, 01:12:33 pm
How should I learn the play Macbeth? We are starting it for T.R week 2 of next term, and I'd like to know the play before then. My problem is with Shakespeare plays, just reading the Shakespearean English confuses me, and if I just read it alongside the modern translation I find it incredibly boring and don't get the 'big picture' of what is happening.

Thanks  :D
Title: Re: VCE English Question Thread
Post by: scarletmoon on April 02, 2015, 09:57:31 pm
I'm not sure what this prompt means "Brooklyn is in essence an exploration of place"
Title: Re: VCE English Question Thread
Post by: JackSonSmith on April 02, 2015, 10:38:49 pm
I'm not sure what this prompt means "Brooklyn is in essence an exploration of place"

Perhaps it's about one's place in society. The importance of belonging to a place. Identifying with a place. Where we place our values. etc.

Just my own interpretations.
Title: Re: VCE English Question Thread
Post by: StupidProdigy on April 02, 2015, 11:54:28 pm
I'm not sure what this prompt means "Brooklyn is in essence an exploration of place"
Make it what you want it to mean. As long as it is structured well and obviously relates to the text still. There is often questions out there like this which allow you to show your own interpretation of the prompt and show the examiner (or whoever) an alternate discussion . Just make sure to define the key terms the way you see them and are going to explore them :)
Title: Re: VCE English Question Thread
Post by: Chang Feng on April 04, 2015, 01:03:20 pm
For language analysis.
When they say to analysis the writer's different approach. what does it mean by approach??.
Thanks you.
if you understand what i mean
Title: Re: VCE English Question Thread
Post by: Chang Feng on April 04, 2015, 01:06:52 pm
For a text response essay.
for every evidence you provide, such as a specific scene, how much explanation do you need to give it. cause teacher said i have tendency to not explain things in enough detail before i move on to next evidence.
Also for the end of year exam are we required to have multiple interpretation for text response and if so how would you implement it in.Also do we need social, historical context for end of year text response essay to.
thanks
Title: Re: VCE English Question Thread
Post by: JackSonSmith on April 04, 2015, 03:17:42 pm
For a text response essay.
for every evidence you provide, such as a specific scene, how much explanation do you need to give it. cause teacher said i have tendency to not explain things in enough detail before i move on to next evidence.
Also for the end of year exam are we required to have multiple interpretation for text response and if so how would you implement it in.Also do we need social, historical context for end of year text response essay to.
thanks

Generally speaking, it's recommended that you explain why/how a piece of evidence fits into/supports your contention.
When it comes to interpretations, it's always going to be best if you use your own unique interpretation of a text. What do you think/believe, based on the evidence from the text.
It is often beneficial to show an awareness of the time period and setting of a text and how they influence key events and characters. I'd certainly consider attitudes and beliefs at the time.
Title: Re: VCE English Question Thread
Post by: literally lauren on April 04, 2015, 09:32:31 pm
THanks.
and also for context, are we required to have quotes from the set text. Or do we need to only take evidence from set text and then analyse how this is relevant to the broader context ie encountering conflict.
Quotes are not essential (unless your teacher says they are.)
In the exam, ideas are what counts, so it's enough to just take a key element or occurrence (or possibly multiple if you prefer the 'breadth over depth' approach) in the text and discuss the relevance of that.
In the SAC, whatever your teacher wants is the priority, so if s/he is saying 'put 7 quotes in your introduction and then 5 per paragraph' then that's exactly what you should do :) If they're flexible, then do whatever you like. Some people find using the language of the text to be a very effective way of branching into discussion, but it depends what you're studying and what kind of writer you are. Suffice it to say: no, it's not a Text Response, so quotes aren't necessarily a necessity :)

How should I learn the play Macbeth? We are starting it for T.R week 2 of next term, and I'd like to know the play before then. My problem is with Shakespeare plays, just reading the Shakespearean English confuses me, and if I just read it alongside the modern translation I find it incredibly boring and don't get the 'big picture' of what is happening.
No Fear Shakespeare do the side-by-side translations, so if you're not a fan of the Bard then I'd either get a copy of their version or just read it online. But unless you're a die-hard-English-fanatic, that whole 'you have to read the text first before you read summaries/resources' is nonsense. Read a summary, watch the sparknotes video, watch a film adaptation if you find an accurate one. Watch this.
Read this.
After a battle three witches tell Macbeth (the 'Thane of Glamis' = kind of like an Earl or Nobleman) that he will become 'Thane of Cawdor' (a higher title) and eventually King; and that his friend Banquo will never be king, but his son will. Later the king declares Macbeth 'Thane of Cawdor,' so Macbeth starts to think the prophecy is true. He talks to his wife (Lady Macbeth) who is more power-mad than he is and ever so slightly a hardcore-sociopath, so she encourages him to kill the current king; he does, then feels immensely guilty because kings are 'chosen by God' so he has committed what he believes to be an 'unnatural' act. Macbeth becomes king and plans to have Banquo and his son killed by murderers. The get Banquo, but his son (Fleance, pronounced 'Flee-anse') escapes. Later at a feast, Banquo's ghost rocks up to scare Macbeth, which is largely interpreted to be a manifestation of his guilt, and foreshadows his eventual madness. The witches come back and tell Macbeth he can't be killed by any man who's born from a woman's lady parts, and that he'll be king until trees can walk - he takes this as assurance that he'll be alive and king forever, but madness and guilt have set in and begin to take their toll. Meanwhile his enemy Macduff is ammasing an army, so Macbeth has his family killed just because. Lady M, also wracked with guilt, sleepwalks and says some wonderfully poetic stuff about having bloodstains on her hands, then dies (possibly... this is debatable - she disappears, let's put it that way.) Macduff and his soldiers sneak up on the castle using tree branches to disguise their numbers (ie. trees are walking, hint hint!) Then:: PLOT TWIST: Macduff was a caesarean and not "of woman born" technically speaking, so he kills Macbeth.
The biggest question you're going to have to answer is 'Who is responsible for the tragedy?' to which my response is usually the metaphor of a weed growing; the witches sowed the seed of jealousy and greed, Macbeth's character was the perfect environment for it to grow, and for his ambition to turn ugly, then Lady Macbeth tendered the plant until it became an all-consuming brambles that destroyed pretty much everyone.
Don't worry if you don't instantly grasp the entire texts; most schools do a read through in class so people aren't lost, and there's no shame in using study guides for texts like these. Or any texts really. But let me know if there are any lines in particular that don't make sense... I've studied this text three times now and my copy is so full of annotations it's coming apart at the seams and I think there's more written on post-it-notes than there is in the actual play  ::) #Englishlyf

I'm not sure what this prompt means "Brooklyn is in essence an exploration of place"
Ergh, this is one of those 'The text is about _____' kinds of prompts that VCAA are really fond of  >:( They're kinda boring and hard to write on, but it can be boiled down to 'Discuss the exploration of place in Brooklyn' if that makes it easier.
BUT this is one of those instances where italicisation is critical!
IMPORTANT: For anyone studying: Brooklyn, Cloudstreet, Henry IV, Mabo, Medea, Stasiland, or Wuthering Heights: There is a world of difference between the title of your text, and what the title refers to!!!
VCAA have tried to trick people in the past with this.
In the example given here, think about how different your contention would have to be if you were arguing
- the place Brooklyn is an exploration of place, meaning that the characters there are able to explore the physical place and come to some sort of conclusion about their identities
vs.
- the novel Brooklyn has 'exploring place' as one of its key themes (which, for those who've read it means you'd be discussing Enniscorthy as well as Brooklyn, the place.)
So the difference between: 'Brooklyn' is an exploration of place and Brooklyn is an exploration of place is potentially huge.

I'm assuming it's the former case here, so you should structure your discussion accordingly :)

Make it what you want it to mean. As long as it is structured well and obviously relates to the text still. There is often questions out there like this which allow you to show your own interpretation of the prompt and show the examiner (or whoever) an alternate discussion . Just make sure to define the key terms the way you see them and are going to explore them :)
I mostly agree, but be careful not to 'topic dodge' by going off on too distant a tangent.Provided you're working within the confines of the prompt, you're fine, and totally free to define words however you like.

For language analysis.
When they say to analysis the writer's different approach. what does it mean by approach??.
Thanks you.
if you understand what i mean
Do you mean when you're given multiple articles and have to contrast their approaches? Because this just means discussing their contentions and the means by which they express them; ie. 'Where Author A contends that ___, Author B, by contrast, argues that ____' and 'Author A uses inclusive language in order to... However, Author B uses...' etc.

For a text response essay.
for every evidence you provide, such as a specific scene, how much explanation do you need to give it. cause teacher said i have tendency to not explain things in enough detail before i move on to next evidence.
Also for the end of year exam are we required to have multiple interpretation for text response and if so how would you implement it in.Also do we need social, historical context for end of year text response essay to.
thanks
a) you need to give as much explanation as is necessary. You can assume your assessor has read the text, but if you jump around too much then it can get confusing. Best to defer to your teacher here because they'll be more familiar with your writing than I am.
b) you should try to acknowledge different interpretations where you can, but this isn't big part of the criteria. It mainly just means not being too definitive with your writing (ie. don't say 'This scene proves that the character is jealous' - there is no 'proof' in English.)
c) no T.R. essay has to have socio-historical stuff, but it can help in certain contexts. It depends on the prompt, mostly. If you're asked to discuss a character's relationship or motivations, then spending ages talking about 1950's America or Jacobean England isn't wholly useful; whereas if you're told to discuss the author's views and values, or look at what the text says about society, then showing some awareness of the life and times of the author might be a good idea.
Just don't get too far away from the text and the prompt :)
Title: Re: VCE English Question Thread
Post by: stockstamp on April 05, 2015, 09:56:48 pm
Should it matter if you branch out from an essay topic a bit?

I'll give some background to this question - we recently completed our first English SAC at school, and I received a lower mark than I was hoping for, and to be honest, lower than I expected (before you write me off as arrogant, please listen read!).

Below is a fairly generic representation of the topic we were given.

"Event A is driven solely by cause B. To what extent do you agree?
         
             To which I responded with something like this...
  1. Cause B is influential - here's why it's influential - here's how it came about
  2. Though Cause B was influential, Cause B itself was actually the product of something much broader, which due to it's impact on Event A, is of a much higher significance than Cause B
  3. Both of my previous paragraphs are relevant, but here's what I think actually drove Event A

As you can see, the first two paragraphs stuck fairly closely to the topic, and made specific mention of what the topic included, whereas the 3rd paragraph was entirely original - I didn't completely agree with the topic ("To what extent do you agree? ..."), therefore I wrote about what I actually thought, even though it was different to where the topic would lead you.

(If this is a bit ambiguous, I can provide the actual essay topic, and what I actually wrote about if necessary.)

Fundamentally, my question is this: Could that third paragraph be a problem?

The  feedback we got for the SAC was pathetic - all we received were numbers.
The Criteria: (each out of 10)
  1. Close analysis and understanding of the chosen text             9
  2. Ability to interpret text in response to the task                       9
  3. Control of the conventions of the English language                10

Yes, this doesn't look like a bad mark on the surface, but a lot of students got 26+/30 who don't write anything original, and don't usually show much depth of thought. Point and click. (This is not an assumption - I have proof read their work in the past) Typically, Criteria 1 and 2 are my strengths! (based on feedback from past few years) I'm better at thinking about, and exploring a topic than using long and fancy vocab - yet I lost marks for the first 2, and actually got full marks for criteria 3. Both my second and third (especially third) paragraphs were original - something the examiners supposedly look for - no one else would have had the 'same' ideas.

English is meant to reward people who think, and my frustration is that I believe thinking cost me marks here - my argument was original, it was relevant and showed logic and depth of thought. But the marks I got lead me to believe that I was penalised because I branched out from the topic. According to my markers, I didn't 'interpret' the prompt properly.

Again, if specifics are required - either because you don't understand what I'm asking, or because you're sceptical (I'm not concerned if you are) - I will provide.


What is your opinion?
Title: Re: VCE English Question Thread
Post by: JackSonSmith on April 06, 2015, 10:52:04 am
Should it matter if you branch out from an essay topic a bit?

I'll give some background to this question - we recently completed our first English SAC at school, and I received a lower mark than I was hoping for, and to be honest, lower than I expected (before you write me off as arrogant, please listen read!).

Below is a fairly generic representation of the topic we were given.

"Event A is driven solely by cause B. To what extent do you agree?
         
             To which I responded with something like this...
  1. Cause B is influential - here's why it's influential - here's how it came about
  2. Though Cause B was influential, Cause B itself was actually the product of something much broader, which due to it's impact on Event A, is of a much higher significance than Cause B
  3. Both of my previous paragraphs are relevant, but here's what I think actually drove Event A

As you can see, the first two paragraphs stuck fairly closely to the topic, and made specific mention of what the topic included, whereas the 3rd paragraph was entirely original - I didn't completely agree with the topic ("To what extent do you agree? ..."), therefore I wrote about what I actually thought, even though it was different to where the topic would lead you.

(If this is a bit ambiguous, I can provide the actual essay topic, and what I actually wrote about if necessary.)

Fundamentally, my question is this: Could that third paragraph be a problem?

The  feedback we got for the SAC was pathetic - all we received were numbers.
The Criteria: (each out of 10)
  1. Close analysis and understanding of the chosen text             9
  2. Ability to interpret text in response to the task                       9
  3. Control of the conventions of the English language                10

Yes, this doesn't look like a bad mark on the surface, but a lot of students got 26+/30 who don't write anything original, and don't usually show much depth of thought. Point and click. (This is not an assumption - I have proof read their work in the past) Typically, Criteria 1 and 2 are my strengths! (based on feedback from past few years) I'm better at thinking about, and exploring a topic than using long and fancy vocab - yet I lost marks for the first 2, and actually got full marks for criteria 3. Both my second and third (especially third) paragraphs were original - something the examiners supposedly look for - no one else would have had the 'same' ideas.

English is meant to reward people who think, and my frustration is that I believe thinking cost me marks here - my argument was original, it was relevant and showed logic and depth of thought. But the marks I got lead me to believe that I was penalised because I branched out from the topic. According to my markers, I didn't 'interpret' the prompt properly.

Again, if specifics are required - either because you don't understand what I'm asking, or because you're sceptical (I'm not concerned if you are) - I will provide.


What is your opinion?

I would be very interested in reading your essay if you have it.
Title: Re: VCE English Question Thread
Post by: literally lauren on April 06, 2015, 11:05:59 am
Should it matter if you branch out from an essay topic a bit?
Theoretically: no; so long as you're careful.

I'll give some background to this question - we recently completed our first English SAC at school, and I received a lower mark than I was hoping for, and to be honest, lower than I expected (before you write me off as arrogant, please listen read!).

Below is a fairly generic representation of the topic we were given.

"Event A is driven solely by cause B. To what extent do you agree?
For a prompt like this, I'd say you're meant to question absolutes like "solely." An essay that just takes this statement as fact and provides evidence to support it would be quite weak, regardless of how well the discussion was conducted - it'd still be reductive. Having said that, it may depend on the circumstances, so could you let me know which text & prompt you were dealing with.
(I know I tell people to minimise the text-specific nature of their inquiries, but this is one instance that is highly dependent on each individual case :P)

To which I responded with something like this...
  1. Cause B is influential - here's why it's influential - here's how it came about
  2. Though Cause B was influential, Cause B itself was actually the product of something much broader, which due to it's impact on Event A, is of a much higher significance than Cause B
  3. Both of my previous paragraphs are relevant, but here's what I think actually drove Event A
This seems safe, but it could have come across as contradictory if you didn't have a clear, overarching contention. The trouble with 'challenge' paragraphs is that they can subvert your discussion if you don't do them properly; they're meant to give the illusion of challenging your points whilst in reality strengthening your contention.
eg.
Prompt: The tragic downfall of Charlie the Octopus was solely the result of his pride.
Para 1: Charlie's pride is dangerous - here's how it affects him and other characters.
Para 2: His pride is very complex - here's how and why it came about.
Para 3: The tragedy is actually due to lots of different things, not just his pride.
Contention: ?

A structure like this doesn't allow for much complexity because you're limiting yourself to three separate discussions without giving yourself much chance to tie them all together.

As you can see, the first two paragraphs stuck fairly closely to the topic, and made specific mention of what the topic included, whereas the 3rd paragraph was entirely original - I didn't completely agree with the topic ("To what extent do you agree? ..."), therefore I wrote about what I actually thought, even though it was different to where the topic would lead you.
If this was the case, you probably should have been challenging the prompt from the start rather than going along with it for half an essay and then introducing a contrary point. You are allowed to disagree with the prompt*in the exam

Fundamentally, my question is this: Could that third paragraph be a problem?
Yes, but it's more likely that your contention was problematic rather than one specific paragraph letting you down. (This is just conjecture for the moment, so just based on the plan you've outlined) I think you may have lost sight of the implications of the prompt and just been focusing on individual arguments.

The  feedback we got for the SAC was pathetic - all we received were numbers.
The Criteria: (each out of 10)
  1. Close analysis and understanding of the chosen text             9
  2. Ability to interpret text in response to the task                       9
  3. Control of the conventions of the English language                10
Urgh, I hate it when teachers do this; I feel your pain, man. If possible, maybe sit down with your teacher and ask him/her whether there are any specific areas where you could improve? Most will be open to this, and it's a good way to show them you're committed to improving.
If not, it'll be up to you (/ATAR Notes :) ) to find these areas of weakness.
   1. If you've lost a mark here, it's probably the result of some minor interpretational errors throughout your piece. 'Errors in interpretation' is a kind of strange phrase because you'd assume it means 'you thought this character died and they didn't' or 'you thought these characters were in love when they weren't' but more often it's about precision of wording. If there are two characters having a minor disagreement and I call it a 'fight' or 'clash,' this can fall under the umbrella of being an 'interpretational error' even though what's actually wrong is my word choice. Do this a couple of times in a row, and it all adds up to one mark lost (so you may have to pinpoint several 'mistakes' rather than there just being one easily identifiable paragraph/sentence where the mark was compromised.)
   2. *facepalm* I hate vague criteria so much... This is essentially the same as the first, though I'd say this has more to do with how you conduct your discussion in relation to the prompt. So if, in your teacher's opinion, you've gone off-topic or you lacked a cohesive focus, then that could've put you on the 9/10 side rather than the 10/10.

Yes, this doesn't look like a bad mark on the surface, but a lot of students got 26+/30 who don't write anything original, and don't usually show much depth of thought. Point and click. (This is not an assumption - I have proof read their work in the past)
That sounds frustrating and unjust, but short of some Freaky-Friday-teacher-student-brain-swap, there's nothing you can do to change other people's marks. Most of the time, students like this get the marks they deserve at the end of the year when exam criteria comes into play, so I wouldn't worry about it for now. The only thing that's in your control is your own score, so just do what you can to beat your own record rather than other peoples'.

Typically, Criteria 1 and 2 are my strengths! (based on feedback from past few years) I'm better at thinking about, and exploring a topic than using long and fancy vocab - yet I lost marks for the first 2, and actually got full marks for criteria 3. Both my second and third (especially third) paragraphs were original - something the examiners supposedly look for - no one else would have had the 'same' ideas.
Firstly, your strengths will likely fluctuate, and whilst it's good you're aware of what you're confident in, you won't always be losing marks in the same area every time. Secondly, yes the examiners look for originality (and yes, your method is definitely preferable to the passive 'Yes, because A B and C' approach a lot of other students take) but that's more of a secondary component of the criteria. No. 1 is RELEVANCE! Id what you're writing isn't relevant, you could be doing absolutely everything else perfectly but that wouldn't matter.

My English teacher said he once marked an exam piece on Shakespeare's Richard III, the first line of which was 'Richard III is a tragic play written by William Shakespeare, just like Macbeth' and from there on, it was a Macbeth essay. He said it was the best Macbeth essay he'd ever read, but it was still on the wrong text. From memory he ended up taking it to the Chief Assessor like 'wtf do I do with this?' See- the technical criteria VCAA operate on contains things like 'ability to use language appropriate to the task' and objectively speaking, the student did that perfectly. But relevance rules everything, and that kid got a 1/10 cause at least he got the author right.

Thirdly, you have no idea whether or not other people will have the 'same' ideas; don't rely on this to boost you up. Maybe your class is comprised of sheep who never think independently, but that won't mean the rest of the state will be. Maybe they wrote totally different ideas, but they did it better. Even though you're being compared to the state's standards at the end of the year, for now and for the sake of self-improvement, best to put other people's capabilities out of your mind and concentrate on your own work in a vacuum.

English is meant to reward people who think
English is meant to do a lot of things. Unfortunately English is run by English teachers who are flawed at best and downright petty at worst. I'm not saying this to make you cynical or pessimistic, but you need to be aware of this subject's... subjectivity. For the most part, if you're hitting the criteria then you're safe, but you'll still be relying on the assessors to recognise what you're doing is right.

Given we're now in the 7th year of this study design, VCAA have essentially weeded out any ridiculously biased exam markers and the standards are pretty clear now, but if you really want to do well: you have to think like an assessor, not like whoever wrote the criteria.

and my frustration is that I believe thinking cost me marks here - my argument was original, it was relevant and showed logic and depth of thought. But the marks I got lead me to believe that I was penalised because I branched out from the topic. According to my markers, I didn't 'interpret' the prompt properly. Again, if specifics are required - either because you don't understand what I'm asking, or because you're sceptical (I'm not concerned if you are) - I will provide.
If you want a second/exam-based opinion I'm happy to offer one, but the most helpful thing to do at this juncture would be to talk to your teacher/markers whenever possible. Even if it turns out what you're doing is completely acceptable from an exam point of view, you'll still have three or four more in-house SACs to complete where you're writing for your teachers, not the examiners. It appears you're in the unfortunate situation of needing to learn two ways of writing; one that hits your teacher's preferences, and an objectively 'safe' way to write for the end of the year. Luckily the two shouldn't be too radically different (your teacher hasn't given you fours when you deserved tens) but you'll still need to split your mindset between these two modes of writing.

And even if your teacher is an exam marker - no teacher marks SACs the same way they mark exams. For SACs they're trying to mould how you write, and are allowed to let their inherent biases come to the surface. In the exam, they can't afford to do this.

Let's say you had one of those weirdly restrictive teachers who said 'unless you use 7 quotes per paragraph I'll never give you full marks.' In the SACs they could enforce this, but when they're marking exam papers, there are different standards. Every essay is marked at least twice by two different people, so if the marks are too far apart, it goes to a third assessor and they take the closest of the scores.
eg.
Suppose I wrote an essay that one teacher (A) gave a 5/10 because it pissed her off for some reason, while another teacher (B) gave it a 10/10 because I'm wonderful. My essay would then get sent to a third assessor (C,) who agrees with B that I'm wonderful and gives me a 9/10. My score would end up being 19/20; assessor A's name goes on a 'watchlist' of sorts, and if they're continually assigning suspicious or questionable marks, they're kindly asked to step down from their role as an exam marker. Like I said, most of these wacky markers have been gotten rid of over the years, so nowadays you won't see much disparity, but it still pays to be aware of the process.

You certainly seem like a competent writer, and I'm sure the rankings/SAC scores will even themselves out in the end, so I'd say don't panic too much. Just be clever about how you approach your SACs and be aware of your audience. Some subtle conversations with your teacher about what they like to see wouldn't go astray either :)

But thank you so much for being specific with your question! This is way easier to respond to than 'I only got 28/30! Whyyyy?'

If you want me to be more specific, let me know what that prompt was and I can deconstruct it with you.
Title: Re: VCE English Question Thread
Post by: izzywantsa97 on April 06, 2015, 02:03:52 pm
When language analysis articles contain quotes, are we supposed to analyse the persuasive devices within that quote?
Title: Re: VCE English Question Thread
Post by: stockstamp on April 06, 2015, 02:34:38 pm
This is outstanding feedback!
After reading it I'm beginning to think that perhaps my biggest problem (among others - including irrelevance) was not having a clearly defined contention. That's essentially because I didn't really have a one-sided view. Considering your feedback, I would say that my SAC did not deserve full marks - but when considering the reasonably high marks that some other students got, I honestly believe I was marked much more stringently.

Usually when I write, I land somewhere in the middle of the given essay topic. Which is sort of what happened in the SAC. But would this be a problem generally - do you need to ultimately agree/disagree with a prompt?

In response to your suggestion about going through it with my teacher - I will do that, but it will be useless. All my teacher will do is point out some areas that she thinks don't have good enough expression. To which I will respond by reminding her that I got full marks for expression....
I actually asked her if I could take the essay to another teacher (who I've had in previous years) - she didn't allow it....

I think at this stage it's probably best to give a specific example; the SAC we completed was done under test conditions in class (though we were given the topic early), so all I have is a rough draft; Although the ideas were essentially the same, the writing changed a lot in test conditions, and the draft itself is very rough.

However, I have an essay characteristically very similar that was done as coursework, but not a SAC. The feedback I got for that suggests a similar issue - perhaps there's a slight irrelevance in my writing, but certainly there is no clear overall contention. I landed on both sides of the argument.
But once again, I would say the strength of this essay is that it's original, and explores the topic with a bit of depth.

This lost marks for the same criteria, and I'm not certainly not saying it deserved full marks, but it suffered similar scrutiny. I'll include it below.

P.S. I understand that you've already vested a lot of time into answering my question so if you don't want to respond then by all means don't. The feedback you already gave was immensely useful.
But - I once heard the almighty Christopher Hitchens say "We should pursue this, we've barely got our trousers off". So if you're feeling a bit that way, my essay is below :)


Medea Text Response
Medea suggests that the world is depressing and chaotic, where evil triumphs and innocence suffers.
To what extent do you agree?


The world created in Euripides’ Medea is one where there is an irrefutable presence of evil and suffering; darkness pervades the play at many points, but whether or not it is the dominant characteristic of the play is less certain. Although Medea is ultimately triumphant in her quest for vengeance on Jason, the idea that ‘evil’ itself is triumphant in the overall sense of the play at its conclusion is much less true. The evidence for this is that despite the turmoil of the play, there is still an obvious existence of love, compassion and grief; emotions which are polarised against evil. Furthermore, while Medea herself is the primary instigator of the most tragic event in the play, she could also be described as an innocent victim, for it was Jason’s betrayal of her that allowed for the ensuing evil in the play. Essentially, this could be attributed to that fact that Euripides appears to have intentionally constructed a somewhat depressing and unjust world within Medea – an idea which can be supported in multiple forms.

Medea is perhaps the only character within Euripides’ tragedy who could be described as triumphant. While all of the characters (including Medea) experience suffering, directly or indirectly, Medea is the only character who actually achieved something. In her sycophantic interactions with Aegeus, Medea secured a safe haven she could flee to after committing her crimes – something of a minor triumph – and her primary triumph; seeking revenge on Jason, the need for which drove her actions throughout the entire play. Jason’s exclamation that “you [Medea] have destroyed me, lady!” [1310] is the moment that ultimately confirms Medea’s triumph. To suggest, however, that because Medea is triumphant, therefore evil is triumphant, implies that Medea is wholly evil, which may not necessarily be true. It was revealed very early in the play that pertaining to Medea was the capacity for acts of an evil nature. She successfully “persuaded Pelias’ daughters to kill their father” [11], she betrayed her own family prior to the beginning of the play and the nurse’s repeated reference to “that savage temperament of hers” [105] are all reasons that the audience may judge her as innately evil. The strongest evidence within the play that seems to suggest Euripides intended for Medea to be perceived as evil is when the chorus become polarised against her (something rather unusual to Greek tragedy) and “beg [her] not to murder [her] children” – a statement that likely reflects Euripides’ view. Yet in spite of this, it is also reasonable for one to regard Medea as not evil, but rather a victim of unforgiving circumstances paired with an overly passionate personality. In the moments leading up to the tragic climax, Medea appears to have a moment of clarity where she exclaims “Ah, stop, my heart, do not do this deed!” [1057] and describes her plan as a ‘terrible’ crime. She continues in this episode of compassion to even comment on how she “loved to hug them!” [1057]. The revelation in this scene is that Medea may not in fact be inherently evil; rather she was traumatised to the extent that she was driven solely by passion. In her own words, “passion is master of my reason” [1079]. So although whether or not Medea should be described as intrinsically evil is decided by the audience, it is clear that the suggestion that she is triumphant is not paired with the statement that ‘evil’ is triumphant.

While most of the characters of Medea are not portrayed as particularly honourable, it is the vehement nature of Medea that allows her to potentially be labelled as evil. And though Medea is the character who experiences triumph, the force of ‘evil’ itself is not triumphant; the contrary is in fact much more probable. Evil is present in a multitude of scenes throughout the play, but presence alone is not enough to evil to be the leading thematic characteristic in the play. The presence of love is far more prominent. Medea’s methods of exacting vengeance are entirely dependent on the reality that love exists between Jason and his children, Jason and Medea and Creon and his daughter; if it did not exist Medea would not be able to cause any emotional suffering. At the moment where Medea confronts Jason, the justification he provides for his actions is the claim that he “wanted to raise my sons in a manner worthy of my house” [562]. Many audiences would doubt the sincerity of this claim, but the sheer anguish and grief found in his exclaims of “O children, my dear, dear children!” [1396] after their death might in fact be sufficient enough evidence to validate his apparent wishes of acting for the benefit of his children, regardless of whether or not selfishness was involved. Whichever perspective one takes however, the conclusion of the play alone makes it undeniable that Jason loved his children. This sense of fraternal love, although it lapses for the majority of the play, is also shown to exist in Medea, despite the horrific nature of her actions. The driving feature of Medea was her passion, which is why this love appeared to be so repressed, but in the moments where Medea’s passion receded to a less influential feature, Euripides showed it still existed. “O how I love to hug them” [1075] she cries in this moment. Love is also to be found in Medea through Creon, and the love he bears for his daughter, which is so strong that upon seeing her death Creon is so devastated that he wishes he could “share your [Glauce’s] death” [1210]; it pained him to the extent that he could not even bear to live. The final obvious reference to love in the play is the love Medea supposedly bore for Jason, but realistically, any feelings she may have once had should be dismissed as a mere extension of her passion, far removed from the concept of deep and genuine love. Yet even with that truth in mind, the sincere despair found within the statements of Creon and Jason display an inherent sense of strong morality and the capability for love and compassion. Without these fundamental values the play could scarcely be labelled a tragedy; they are essential to evoking an emotional response from the audience and their overall relevance effectively disproves the idea that ‘evil’ is triumphant. As a force, the evil in Medea is countermanded by the overwhelming evidence for love in combination with the sense of rancour the audience would regard Medea (undisputably the most ‘evil’ character) with.

The primary feature of Medea that leaves absolutely no ambiguity is that of suffering, and specifically, the unjust suffering of the innocent. The suffering of innocents takes place in the form of Medea’s children, where she murders them out of her own selfish desire for revenge. This was the incident that ultimately polarised both the audience and the chorus against her; the children were innocent in all sense and their murder was a crime that Medea herself described as ‘terrible’. Yet the sense of injustice aligned with these actions permeates the entire play to the extent that these murders may in fact simply have been the materialisation of the generally chaotic and depressing world created by Euripides. The children’s death were but one of many examples of innocence suffering; some would perceive Glauce and Creon as innocent, but furthermore and most significantly, Medea herself may be a victim of this dark world. At the opening of the play, the Nurse explained that “Jason has betrayed my lady [Medea] and his own children for a princess’ bed” [18], causing Medea to feel “the sting of injustice” [110]. In Medea’s acrimonious feministic speech she highlighted the suffering of women as a result of the underlying sexism Greece. The ancient Greek societal paradigm of the ideal woman was one where they were forced to take “a master to play the tyrant with [their] bodies” []; a concept that modern audiences would recognise as appalling. Aside from these specific examples of where innocents suffer, Euripides provided several comments that essentially prove that he intended for the world of Medea to be somewhat depressing. The seemingly pessimistic statement given by the messenger (who existed purely to provide commentary on the play) – “When fortune’s tide flows towards him, one man may surpass another in prosperity, but you should not call him happy” [1231] - suggests that Euripides may have been deliberately attempting to remove happiness from the play; suffering of innocents and the depressing characteristics were an inevitability, designed by Euripides. This may have been done simply to further dramatize the play for the benefit of the audience, or possibly it was a reflection of Euripides’ personal views. Regardless of the reason for the depressing nature of the play, the innocent do suffer in many forms.

Ultimately, it is clear that the world described by Euripides in Medea is one where suffering, evil and darkness have a significant presence. The suggestion that evil is triumphant, however, essentially depends on the interpretation of the context of evil. If ‘evil’ refers specifically to Medea, then it is triumphant, as Medea is the only character who experiences anything that could be described as a triumph. If ‘evil’ is a more generic reference to the concept of ‘evil’ as a force though, it is far from triumphant. Though the play is characteristically dark and depressing, the power of love still seeps through much of the play.

Title: Re: VCE English Question Thread
Post by: literally lauren on April 06, 2015, 10:57:07 pm
When language analysis articles contain quotes, are we supposed to analyse the persuasive devices within that quote?
You can discuss the fact that the author chooses to include a certain quote, but in terms of conducting an analysis on what the author of the quote was intending, then no.

eg. Article by Author A: In fact, many experts would agree with this assertion, such as Person B who was quoted in The Financial Review as saying "this is a great idea; only morons disagree!"
Your analysis: Author A invokes the opinion of Person B who A describes as an "expert" in order to accentuate the inferred accuracy of the claim that...
Wrong analysis: Author A includes a quote from person B who uses certain rhetorical devices in order to get his point across, like attacking dissenters as "morons" which compels readers to...

-

...considering the reasonably high marks that some other students got, I honestly believe I was marked much more stringently.
This might have been intentional. Again, I'm not your teacher so I don't know what standards they're using, but I do know that not every teacher applies the same objective end-of-year standards to all students. Some people benefit from ridiculously high (even unattainable?) standards as this encourages them to work harder; others prefer to be given credit for every minor improvement they make in order to show them what they need to do to change the mark. (Basically one group of people will prefer to get 6/10 all year from an exam perspective and then gradually work up to 10/10, whereas others like getting 10/10 from a March/April perspective, and then having the criteria and standards change as they do.) To give your teacher the benefit of the doubt, perhaps s/he ismarking you by different standards, but is doing this deliberately for the sake of your improvement  ???
 
Usually when I write, I land somewhere in the middle of the given essay topic. Which is sort of what happened in the SAC. But would this be a problem generally - do you need to ultimately agree/disagree with a prompt?
Think of it like a spectrum:
AGREE l--------*--------l--------*--------l DISAGREE
You don't want to be at either end of the scale, because that makes your essay too reductive (there are never objectively true or false statements when it comes to interpretation: you couldn't say 'Medea's fate is completely and utterly attributable to her inability to forgive; nothing more, nothing less' as though that was a blanket truth about the text with no exceptions.)
But you also don't want to fence-sit! ie. have an interpretation that's right in the middle of the spectrum, because it comes across as noncommittal (eg. 'Medea's fate is slightly sometimes because of this but also lots of other factors but she deserves it, but she doesn't necessarily, but Jason's to blame, but maybe he isn't...' etc. etc.) <-- that's an extremely oversimplified example obviously, but that's how a contention can seem.
Ideally you want to be at the (*) locations where you've got a clearly definied focus but you're not reduced to being blind to alternate points of view.

In response to your suggestion about going through it with my teacher - I will do that, but it will be useless.
Do it anyway.
a) because even if it's a superficial gesture that you don't learn anything from, at least it makes it seem like you're dogged enough to take time out of your life to speak to her about your approach and
b) if you ask the right questions in the right way, it will be at least somewhat useful.

All my teacher will do is point out some areas that she thinks don't have good enough expression. To which I will respond by reminding her that I got full marks for expression.... I actually asked her if I could take the essay to another teacher (who I've had in previous years) - she didn't allow it....
Trust me, I totes know where you're coming from on the 'unhelpful teacher' front; sometimes you just have to swallow your pride and be a tad manipulative. Without sounding too judgmental, is there a chance your teacher took your request to get a second opinion as an insult about her own impartiality or competence? There is a world of difference between approaching a teacher and saying 'Hey Miss, I know I've done fairly well on this SAC but I was wondering if you could clarify a few points for me... you said I've used poor expression in these places: do you have any strategies or examples so I can do this better?' and 'Hey Miss, I don't think this SAC mark is what I deserve; mind if I go next door and consult with this other teacher who I like better?' I'm sure you weren't that obvious, but still, think about the questions you're asking. (Make them as specific as the ones you're asking here - because these are spot on!) Your first priority is remaining on your teacher's good side; your second priority is getting her to answer your questions and help you... but you can't get the second without the first.

I won't correct your whole essay for the sake of time, but just some general notes:
Medea suggests that the world is depressing and chaotic, where evil triumphs and innocence suffers.
To what extent do you agree?
Hmm... this is clunkily worded...
Okay, the way I see it - it comes down to clauses. You could either view this as:
1) Medea suggests the world is depressing and chaotic whereby evil triumphs and innocence suffers
or
2) Medea suggests the world is depressing and chaotic because evil triumphs and innocence suffers.
Frustratingly, both would be feasible from a grammatical point of view, but I think the first meaning is semantically more sound. So where you've interpreted this as conveying a causal link between A and B, the prompt is in fact just one big statement about the world of Medea, ie instead of [world=depressing/chaotic] because of [the triumph of evil and the suffering of innocence]; it's [world=depressing/chaotic/evil-triumph/innocent-suffering]
Does that make sense?

You also seem to be able to use definitive language (eg. 'The evidence for this is that...') without having a very definitive contention which is why your teacher may keep harping on about expression even though you're getting full marks in that criteria. And I know you said this was a rough draft, but I can understand an even-handed assessor docking a few points for expression here and there.

The criteria are not independent! Sometimes screwing up one can have a carry-over effect, and sometimes doing one thing wrong can hide a whole lot of other problems under the surface. This is most often the case for issues of expression; they're the easiest thing in the world for teachers to notice because they don't have to pay attention to what you're conveying - just the grammar, spelling, and syntax you've used to convey it. So you could get a whole page full of corrections regarding expression and think 'oh cool, I'm doing everything else right, I've just gotta fix my shitty writing.' Wrong. That's just the first 'level' you have to address before you start dealing with all the other issues that crop up.

This is what I meant when I said 'you won't always be losing marks in the same area every time.' Not only will your approach vary for every prompt you write on, but you'll also be demonstrating different skills or imperfections each time, so don't be too perturbed if you're suddenly losing marks in an area when you weren't before.

Incidentally: your school might enforce different rules for now, but there's no need to reference page or line numbers in your exam :)

The third paragraph here seems a little out-of-focus, almost as though it's just a discussion of innocence in the text generally and not about how the suffering of innocents pertains to the depressing or chaotic nature of the world of the text (hence why you should always be rounding your points back to the WHOLE prompt rather than just a single aspect of it.)

But once again, I would say the strength of this essay is that it's original, and explores the topic with a bit of depth.
Relevance > Originality every time. You don't want to focus too much on originality because after a point it just gets risky. There's no requirement to blow the assessor's mind with a never-before-seen interpretation; it's enough to talk about the prompt with sophistication and in a way that flows logically. Based on the few Medea essays I've read so far, I'd say your interpretation is pretty standard (which is not in any way a bad thing; it just means that you won't necessarily stand out on that level alone.) And you shouldn't want to. The markers can't give you full credit for all the little things you're doing right if you've totally shell-shocked them with a world view they've never considered before, so rather than reaching for the stars, think of your English essays like neat little criteria-fulfilling devices that are being programmed to do everything the assessor wants. Originality is a very small part of that very big picture.
Title: Re: VCE English Question Thread
Post by: heids on April 11, 2015, 10:23:31 am
^ Please post essays requiring feedback in the English Work Submission and Marking board so the questions thread doesn't get cluttered; I'll try to get to it there, if someone else more qualified doesn't mark it first :)

EDIT: Eek, sorry, just realised you did put it there ::)...
Title: Re: VCE English Question Thread
Post by: JackSonSmith on April 13, 2015, 05:40:31 pm
Ways to "spice" up context essays

My English teacher has told me that assessors do not like "straight" essays and that they are pushing more more creative/imaginative pieces.

I was wondering, are there any good ways to integrate some "salt and pepper" into a plain essay, as these are what I am comfortable writing?
Title: Re: VCE English Question Thread
Post by: literally lauren on April 13, 2015, 07:32:26 pm
Ways to "spice" up context essays

My English teacher has told me that assessors do not like "straight" essays and that they are pushing more more creative/imaginative pieces.

I was wondering, are there any good ways to integrate some "salt and pepper" into a plain essay, as these are what I am comfortable writing?

I believe the assessors are pushing creativity, but this doesn't necessarily mean they want people to write in a completely imaginative style.

It depends how radically you want to change what you're doing at the moment; if your expository skills are bang on, then it would just be 'salt and pepper' seasoning, as you put it, that could help you stand out. But if your pieces are lacking development or complexity, then these creative elements would require more careful thought and might be trickier to implement straight away.

Some options: (mainly focusing on what to do structurally; let me know if you need clarification with content as well)

If any of these pique your interest and you want more info or some examples of how they work, let me know and I'll try and hunt down some of my old pieces or other links :)
Title: Re: VCE English Question Thread
Post by: JackSonSmith on April 13, 2015, 09:17:22 pm
I believe the assessors are pushing creativity, but this doesn't necessarily mean they want people to write in a completely imaginative style.

It depends how radically you want to change what you're doing at the moment; if your expository skills are bang on, then it would just be 'salt and pepper' seasoning, as you put it, that could help you stand out. But if your pieces are lacking development or complexity, then these creative elements would require more careful thought and might be trickier to implement straight away.

Some options: (mainly focusing on what to do structurally; let me know if you need clarification with content as well)
  • bookend
    For those who aren't familiar with what a 'bookend' is:
    (http://img.designswan.com/2010/01/bookEnd/2.jpg)
    They're basically devices for keeping a stack of books together on a shelf, with or without stylised designs like above. In essay terms, this translates to doing something at the start and at the end of your piece that 'holds it all together,' so to speak. It could be just a brief quote or idea, or it could be an entire paragraph, but the aim is to have something that frames your piece by being on either end of your essay.
  • 'thread'/weaving
    This is similar to bookending, only it will occur constantly. In essence you have a 'thread' that is continually woven back and forth throughout your piece (which also serves to 'hold things together' and provide cohesion.) For instance, you might maintain a focus on a certain culture, eg. Malaysia, so that all of your examples revolve around this idea.
     --> P1: Dutch colonisation and how their culture changed; P2: military culture (compulsory army service at age ~17 I think); P3: globalisation and its impact on more remote lifestyles ... obvs haven't lined to the context here, and you could use supplementary points where needed, eg. juxtapose their colonisation with more/less brutal ones in other nations.
  • reincorporation
    Kind of the halfway point between a bookend and a thread; this is where you bring up a similar idea/quote/point/metaphor/thing a couple of times, but give it a different meaning each time. It shouldn't be as in-depth as a properly woven 'thread,' but it also doesn't have to occur just at the start and the end like a bookend does.
  • interpolation
    Also known as 'interruption' or 'criss-crossing' where you are essentially writing two pieces (eg. a normal expository essay + a creative POV short story) but you alternate between paragraphs. If done properly, this should result in a dual piece that explores similar concepts in two different ways, ie. the expository sections takes care of the necessary theorisation and links to context, and the creative parts show these ideas in practice.
  • blend
    This one's really open ended, but it essentially just means turning a purely expository piece into more of a hybrid (which is gradable; there are different levels of 'hybridisation' so you don't have to go all in.) Often this is where news feature articles or creative background contexts (as in, pretending you're a war correspondent or a major world leader) come into play.

If any of these pique your interest and you want more info or some examples of how they work, let me know and I'll try and hunt down some of my old pieces or other links :)

Thank-you for your very helpful reply.

Are there any examples of interpolation and blend? They sound interesting but seem somewhat difficult to implement.
Title: Re: VCE English Question Thread
Post by: lisax3 on April 15, 2015, 04:42:40 pm
Is it possible to over analyse in language analysis? Such as over analysing what the effect of a particular technique has on the audience?
Title: Re: VCE English Question Thread
Post by: HighTide on April 15, 2015, 06:45:31 pm
Is it possible to over analyse in language analysis? Such as over analysing what the effect of a particular technique has on the audience?
I was also wondering the same. My language analysis doesn't really have a grouped structure but instead from top to bottom  to build up the intended effect which positions the readers to do something overall... My teacher said that it depends on the significance of the effect and suggested I pick out the predominant persuasive techniques.
Title: Re: VCE English Question Thread
Post by: literally lauren on April 15, 2015, 09:59:26 pm
Is it possible to over analyse in language analysis? Such as over analysing what the effect of a particular technique has on the audience?
Yes, definitely. Though there's a distinction to be drawn between over analysis and wrong analysis.

If you start delving into the audience's psyche and talking about complex mind manipulation techniques - that's over analysis.

But more likely this problem will arise when you're trying to assert something that can't be justified, eg. the author's use of the word 'sad' is likely to make the audience spiral into a state of eternal melancholy from which they will never recover - which would be wrong.

It really depends on the context, and I don't want to say 'steer clear entirely' because getting into an overly analytical mindset is actually really helpful for English as a whole. The trick is to know when to reign it in.
If you post an example of what you mean (or of what your teacher pointed out in your work, as I'm assuming is the case?) I might be able to clarify further.

I was also wondering the same. My language analysis doesn't really have a grouped structure but instead from top to bottom  to build up the intended effect which positions the readers to do something overall... My teacher said that it depends on the significance of the effect and suggested I pick out the predominant persuasive techniques.
If you're looking for a means of structuring things, I can recommend the key player method (check the first post in this thread for links to explanations - then you can get back to me if anything doesn't make sense.)

Structuring by techniques isn't the worst way of doing things, but it sounds like your current approach would be more conducive to building up an overall analytical piece rather than writing something that reads like it's a bullet point list of separate analyses all crammed together in an arbitrary paragraph.

But as usual, listen to your teacher for SACs. Be the system, beat the system  ;D
Title: Re: VCE English Question Thread
Post by: thaaanyan on April 16, 2015, 11:24:37 am
Hey Lauren,
I really like the idea of interpolation and blend, but i'm not sure how to go about analysing a prompt with the format. like with interpolation, does the story have to be connected to the expository? or it completely a different piece? are they any examples? i've been hunting through the resource thread and i can't seem to find any!
thank you :)
Title: Re: VCE English Question Thread
Post by: literally lauren on April 16, 2015, 12:25:41 pm
Okay turns out the examples I had from Year 12 aren't as great as I remembered :p looking through them now there's so much stuff that looks clunky and out of place, so I might just write up a new one over the weekend so I can demonstrate the different ways of doing things. I'll post it in the Resource & samples thread when I'm done :)
Title: Re: VCE English Question Thread
Post by: chansena on April 16, 2015, 06:35:00 pm
Okay turns out the examples I had from Year 12 aren't as great as I remembered :p looking through them now there's so much stuff that looks clunky and out of place, so I might just write up a new one over the weekend so I can demonstrate the different ways of doing things. I'll post it in the Resource & samples thread when I'm done :)

Hi,

Did you by any chance have a hybrid essay or a feature article? I'm looking at doing one for context 

If you could upload one or PM me one that would be great :)

Title: Re: VCE English Question Thread
Post by: HighTide on April 16, 2015, 08:20:31 pm
Okay turns out the examples I had from Year 12 aren't as great as I remembered :p looking through them now there's so much stuff that looks clunky and out of place, so I might just write up a new one over the weekend so I can demonstrate the different ways of doing things. I'll post it in the Resource & samples thread when I'm done :)
Hello, so I take it from "Be the system to beat the system" that you adhered to the structure (assumption) set by your school or was not far off? If so, was your ability to do a comparative analysis much simpler when you followed the normal structure?
The thing is, with my structure of language analysis, there's no comparison of the techniques or key arguments but more a comparison of the approach, intended effect and positioning... Would I be better off trying the standard structure?
Thanks.
Title: Re: VCE English Question Thread
Post by: literally lauren on April 16, 2015, 08:37:59 pm
Hi,

Did you by any chance have a hybrid essay or a feature article? I'm looking at doing one for context 

If you could upload one or PM me one that would be great :)
Will upload on Saturday with the other bits and pieces :)
#still writing year 12 essays to procrastinate from writing uni essays.....

Hello, so I take it from "Be the system to beat the system" that you adhered to the structure (assumption) set by your school or was not far off? If so, was your ability to do a comparative analysis much simpler when you followed the normal structure?
The thing is, with my structure of language analysis, there's no comparison of the techniques or key arguments but more a comparison of the approach, intended effect and positioning... Would I be better off trying the standard structure?
Thanks.
I'm not quite sure what you mean by 'normal structure,' but yes, I caved to the recommendations my school provided (though admittedly I got lucky with my teacher and he was pretty chill about our individual approaches) and the way I wrote in the exam was slightly different, but not by much.

In terms of comparison, it really isn't a big deal. There aren't any marks assigned to it, and you should never compare at the expense of analysis. In fact many of the exams in previous years (5/7 in the current study design, actually) haven't involved any comparison between different articles at all - it's just been one core text and then one or two visuals. The only exceptions were 2011 *spits* and 2014 which is probably a better one to practice :)

When you do need to compare articles, you can either do it by transitioning between them by commenting on their overall contention (ie. Author A argues ____, whereas Author B disagrees and instead suggests ____) or you can find a connection on the language/technique level and say 'Author A uses _____ in order to... However, Author B uses a different technique to the same effect//or// uses the same technique to a different effect.'
There's no inherent advantage in doing it one way or the other, and so long as your piece shows an awareness of both levels (ie 1. the broad, big picture contentions and sub-arguments and 2. the close features and language employed by the author(s)) then you should be fine.

Hope that answers your question, though I don't know what 'standard structure' you're referring to. If it's the key player method I mentioned, that's not technically 'standard;' it's just what I wholeheartedly recommend as being the most efficient way to order your essay, whether you need to do comparative analysis or not :)
Title: Re: VCE English Question Thread
Post by: HighTide on April 16, 2015, 09:15:11 pm
Will upload on Saturday with the other bits and pieces :)
#still writing year 12 essays to procrastinate from writing uni essays.....
I'm not quite sure what you mean by 'normal structure,' but yes, I caved to the recommendations my school provided (though admittedly I got lucky with my teacher and he was pretty chill about our individual approaches) and the way I wrote in the exam was slightly different, but not by much.

In terms of comparison, it really isn't a big deal. There aren't any marks assigned to it, and you should never compare at the expense of analysis. In fact many of the exams in previous years (5/7 in the current study design, actually) haven't involved any comparison between different articles at all - it's just been one core text and then one or two visuals. The only exceptions were 2011 *spits* and 2014 which is probably a better one to practice :)

When you do need to compare articles, you can either do it by transitioning between them by commenting on their overall contention (ie. Author A argues ____, whereas Author B disagrees and instead suggests ____) or you can find a connection on the language/technique level and say 'Author A uses _____ in order to... However, Author B uses a different technique to the same effect//or// uses the same technique to a different effect.'
There's no inherent advantage in doing it one way or the other, and so long as your piece shows an awareness of both levels (ie 1. the broad, big picture contentions and sub-arguments and 2. the close features and language employed by the author(s)) then you should be fine.

Hope that answers your question, though I don't know what 'standard structure' you're referring to. If it's the key player method I mentioned, that's not technically 'standard;' it's just what I wholeheartedly recommend as being the most efficient way to order your essay, whether you need to do comparative analysis or not :)

Sorry, my bad. "Standard" structure was referring to the key argument-based analysis' or technique-based analysis'. I've never heard of one based on key players... Would you be mixing up the arguments and the techniques to show their intended effect on the audience? So like a build up?
Title: Re: VCE English Question Thread
Post by: RazzMeTazz on April 16, 2015, 10:57:23 pm
How important is a sophisticated vocabulary, in order to do well on the English exam? :)

If you had a normal vocabulary but well-written pieces with good ideas etc, would the fact that you aren't the most sophisticated of writers, be a major issue?
Because often teachers tell me I am too verbose, and that I would be better off and clearer if I used simpler words, but  when I read  high-scoring sample responses, or even examples from teachers - the wording does tend to be quite sophisticated?

Thanks!

Title: Re: VCE English Question Thread
Post by: DJA on April 17, 2015, 01:02:06 am
How important is a sophisticated vocabulary, in order to do well on the English exam? :)

If you had a normal vocabulary but well-written pieces with good ideas etc, would the fact that you aren't the most sophisticated of writers, be a major issue?
Because often teachers tell me I am too verbose, and that I would be better off and clearer if I used simpler words, but  when I read  high-scoring sample responses, or even examples from teachers - the wording does tend to be quite sophisticated?

Thanks!



My take on this - from my guide to expression - yes shameless self-promotion ;)
Vocabulary is definitely important to good expression. If you’re using words such as ‘like’, ‘good’, ‘bad’ and ‘shows’ all the time it’s not going to make a good impression on an examiner. You should definitely have synonyms in your mind. Again reading helps a ton as when you read, you unconsciously expand your vocabulary. However, what also helps is actually writing down useful vocabulary. For English, I compiled interesting vocabulary at the back of my writing/note book whenever I read study guides, model essays or critical readings. If I didn’t know what a word meant, I would google it. Then when I wrote essays, I would open to my vocab and challenge myself to use some of the words I had learnt in my writing. Give it a shot!

Remember however that there IS a difference between sophisticated language and excessive and pretentious language. Use words that are appropriate, that you could even use in conversation but that aren’t so hard to understand that you sound like a wanker.
Everyday language: Good
Sophisticated: Magnificent
Pretentious: Splendiferous

Basically a sophisticated vocabulary is good to have but not the be it end all to score well on the English Exam. Honestly if you're using less complex words but convey your ideas and argument with clarity, you're going to do better than that guy or gal who throws around huge words and ties their ideas into knots or worse loops (don't you hate those essays which go round and round and don't actually make a point?)

So yes if you had a normal vocabulary but well-written pieces with good ideas etc - you're going to do well. That being said, it's good to have a few words that are appropriate to the text, the author or the time of the text you are studying that are more 'sophisticated' and that you can use to 'show off' a bit in exams. Lauren has said it already in the past and it's so true - Examiners are ONLY HUMAN - they can be unfortunately swayed by writing that 'sounds' sophisticated.

And yes the highest scoring responses will often have quite sophisticated vocabulary - BUT note that when you read these top scoring pieces, you will see (hopefully) how clear the writing is, how good the ideas are etc.

tl;dr: Know appropriate sophisticated words to the text you are studying and use them in your writing but do not throw in big words for the sake of it - prioritise structure and clarity of ideas and argument over trying to sound sophisticated.

ALSO it's 1 am so forgive me if this is a ramble because it is. DJA out
Title: Re: VCE English Question Thread
Post by: sjayne on April 17, 2015, 02:08:52 pm
How important is a sophisticated vocabulary, in order to do well on the English exam? :)

If you had a normal vocabulary but well-written pieces with good ideas etc, would the fact that you aren't the most sophisticated of writers, be a major issue?
Because often teachers tell me I am too verbose, and that I would be better off and clearer if I used simpler words, but  when I read  high-scoring sample responses, or even examples from teachers - the wording does tend to be quite sophisticated?

Thanks!

The closer you get to the 40+ scores the more important your vocab is.  I personally don't have a thesaurus in my head but that didn't necessarily affect my marks.

You want your work to sound good but you also want to write well so lots of vocab and poor discussion won't get you a high score. Similarly, good writing and  the vocab of a year 7 student isn't going to do anyone any favours. Use your words but use them well.
Title: Re: VCE English Question Thread
Post by: JackSonSmith on April 17, 2015, 08:29:51 pm
Open question to everybody.

What are your views/ recommendations about the use of comparisons and analogies in context pieces.

For example, studying encountering conflict, would I be able to successfully compare people to trees. ie. if someone is rooted in the place that they grew up in with their beliefs passed to them by their fore-bearers, they are unlikely to change their views, which in turn leads to long lasting conflict

etc.
Title: Re: VCE English Question Thread
Post by: RazzMeTazz on April 18, 2015, 12:57:44 pm
Thanks for the wonderful advice DJA and sjayne!!

:) It really helped me out a lot!
Title: Re: VCE English Question Thread
Post by: chansena on April 19, 2015, 01:07:05 pm
Will upload on Saturday with the other bits and pieces :)
#still writing year 12 essays to procrastinate from writing uni essays.....

Hi

I am not sure where to look for the files you have uploaded in particular the hybrid essay / feature article. Could you please direct me to where they are :)
 
 
Title: Re: VCE English Question Thread
Post by: scarletmoon on April 19, 2015, 06:46:12 pm
I need to write a persuasive essay on the prompt, "In conflict, anything is justifiable" I need some help on external resources I can use to support this prompt
Title: Re: VCE English Question Thread
Post by: chansena on April 19, 2015, 07:23:25 pm
I need to write a persuasive essay on the prompt, "In conflict, anything is justifiable" I need some help on external resources I can use to support this prompt

-For encountering Conflict I always liked to use a soldier / child soldier concept as you could always mold the concept to any encountering conflict prompt. (Boko Haram)

-You could also use government leader possibly

- A mother and father figure protecting their child therefore they justify their actions / behavior

Hope this helped  :)





Title: Re: VCE English Question Thread
Post by: vanessa14 on April 21, 2015, 09:08:47 pm
Hey everyone,

I was just wondering, for language analysis, if I get 4 or 5 texts (short), 2 with the same contention and 2 with the opposing contention, and one that is an image on its own, can I say the image supports both contentions or do I have to choose one?

Just a bit confused on what to do! Im writing a practise essay and I'm not sure if I can use the image for both contentions...

thanks to anyone who chooses to help, it will be greatly appreciated.
Title: Re: VCE English Question Thread
Post by: heids on April 22, 2015, 08:53:36 am
I was just wondering, for language analysis, if I get 4 or 5 texts (short), 2 with the same contention and 2 with the opposing contention, and one that is an image on its own, can I say the image supports both contentions or do I have to choose one?

Just a bit confused on what to do! Im writing a practise essay and I'm not sure if I can use the image for both contentions...

With the image, you always have to figure out its own contention (or what you imagine its contention could be).  Spend a while looking at it and thinking what is could be trying to make you feel or think.  It won't be trying to persuade the audience in two totally opposite directions at once!  If you're comparing the image with other articles, you're trying to find how it 'argues' similarly or differently to the other articles. So, you can CONTRAST it with two of the articles (how it argues differently/in opposition to them) and COMPARE it with the other two articles (how it backs up their contention; try to find a quote from these articles that show how the image tries to argue the same thing).

Alternatively, it may be arguing a different thing altogether (or at least have a slightly different way of arguing); if so, you can contrast it with all the articles ;D

Remember, the image doesn't necessarily totally agree with the accompanying article(s) - if you can point out slight differences, your analysis is deeper and better.  Remember to make your links up if you can't figure out what the image is actually showing ;)
Title: Re: VCE English Question Thread
Post by: literally lauren on April 22, 2015, 06:06:07 pm
Sorry, my bad. "Standard" structure was referring to the key argument-based analysis' or technique-based analysis'. I've never heard of one based on key players... Would you be mixing up the arguments and the techniques to show their intended effect on the audience? So like a build up?
Yes, it's essentially a 'build up' of analysis so it gives your piece a bit of direction - meaning you don't have to just float between random points of analysis and you can comment on overall persuasion by the end. It's not necessarily like 'mixing up the arguments' though; if anything you're structuring by argument (eg. one para on how the author manipulates the idea of censorship, one paragraph on how he wants us to feel about governmental responsibility, and one on the idea of freedom and security.)
Might be best to check some of the earlier explanations (pg 1 of this thread) and let me know if you're still confused :)

Open question to everybody.

What are your views/ recommendations about the use of comparisons and analogies in context pieces.

For example, studying encountering conflict, would I be able to successfully compare people to trees. ie. if someone is rooted in the place that they grew up in with their beliefs passed to them by their fore-bearers, they are unlikely to change their views, which in turn leads to long lasting conflict

etc.
DO IT!!! I'm a sucker for these kinds of metaphors/puns/all-round-bad-humour, so that sort of stuff can work wonders for making your writing more engaging.
It all depends what kind of piece you're writing; if it's a mega-conventional & formal essay, then this might seem a little out of place, but if it's more hybrid-y then you can get away with a lot more.

And the usual do-as-your-teacher-says-for-SACs-disclaimer-here.

Having said that, making the metaphors too cliched or forced can be problematic. It also depends how much you're utilising the comparison, ie. whether you're just making an offhand reference, or constantly revisiting the metaphor to strengthen the link.

Like most things in Context I'm afraid it just comes down to 'it depends.'  ::) It could complement your style and give the assessors a glimmer of hope as their coffee stained eyes drift to the piles of essays they still have to get through, or it could just make them grumble with irritation.

Ironically, it's best to contextualise these sorts of options because it all comes down to how you write, so I guess see how your teacher reacts when marking your essays? :)

I am not sure where to look for the files you have uploaded in particular the hybrid essay / feature article. Could you please direct me to where they are :)
Yeah, so... my precious little laptop gave itself an aneurysm and so the drafts I had are now on a hard drive I'm desperately trying to recover. Or, more accurately, a hard drive that my tech-inclined mate is desperately trying to recover while I prod him with sticks. I'm on a phone at the moment so writing whole essays isn't really an ideal option.
Suffice it to say that for feature articles, it'd be better to look at actual feature articles (ie. non-Context-based, like in The Australian or an online magazine) to get a sense for how the genre operates. It's a really 'open' form of writing, so having a go yourself and then adapting your method later is probably better than attempting to appropriate someone else's.
As for the hybrid, that is something I want to post an example of just because I know it's not often explained or demonstrated, so hopefully by this weekend I'll have either recovered the one's I was working on before, or will have written a new one :)

It'll be posted here when I'm done: English Resources and Sample High Scoring Responses
Title: Re: VCE English Question Thread
Post by: cosine on April 22, 2015, 06:19:23 pm
What exactly goes into an expository essay, and what's the structure like?
Title: Re: VCE English Question Thread
Post by: literally lauren on April 22, 2015, 09:03:19 pm
What exactly goes into an expository essay, and what's the structure like?
That's a little like asking 'what exactly goes into a cake'?

Are you baking a sponge cake, or a bedazzled chocolate cake complete with complex layers and fancy swirled icing flowers, or a cake made out of fish skeletons and earwax?

Sponge recipe:
Plain and simple; everyone can make a vanilla sponge essay. It's all about getting the right measurements (ie. a balance between abstract, context-based discussion and close examination of evidence and examples.) Generally speaking it's best to start broad and work your way in, so following a standard introduction that unpacks the prompt, you'll commence with the first of 3 or 4 body paragraphs that begins by outlining a key idea, then gradually moves into a discussion of the examples (textual and otherwise) before finally rounding it all back off to the overall point that is your contention. There are some kind of formulaic ways of doing this, but a sponge cake made properly can be perfectly satisfying, so if you'd rather stick to this recipe then all you'll really have to do is fine-tune your essay writing abilities and collect as many varied examples as you need.

Chocolate cake complete with complex layers and fancy swirled icing flowers recipe:
This is slightly more ambitious for obvious reasons, and usually involves a lot more creative freedom in terms of how you construct things. There are still 'rules' in the sense of not adding 27 bags of chocolate melts (ie. cramming in heaps of external examples and giving your assessor type two diabetes) or forgetting the eggs (ie. not having any links to the prompt and turning your piece into a dry, inedible mess) but on the whole you can do what you like. There are many different ways to make a chocolate cake delicious, and there are many different ways to make the structure of an expository essay interesting.

Cake made out of fish skeletons and earwax recipe: (for disaster)
But it is of course possible to follow every structural 'rule' and still screw things up. I mean, technically if you mix a bunch of stuff in a bowl, pour it into a cake tin and chuck it in the oven for about half an hour, you've followed the instructions... but in reality you need a certain degree of common sense when it comes to what ingredients to include. For example, shredded coconut is fine in theory, but if your guests are allergic to coconut (or there are 7 years worth of Assessor's Reports telling you to stop using coconut unless you can do it properly) then you may have to rethink your approach.
This is why, as much as I hate doing so, I have to answer 'it depends' to so many questions about what Context should be. I could tell you 'don't use quotes; they're clunky, rarely well integrated, and often feel forced' and that'd be true, but there are heaps of examples of students using quotes well and in a way that contributes to the strength of their piece as a whole. Not every rule has exceptions, but there are so few rules anyway that it's tough to know what's appropriate and what crosses VCAA's arbitrary lines.


The best thing you can do at this point is to just start writing! No matter how directionless or 'wrong' it feels, just get some stuff down on paper, give it to your teacher/AN and let someone suggest a direction for you. In my experiences it's easier to make Context suit you - that is, playing to the strengths of your own writing style and manipulating your approach so that you're hitting the criteria in your own way. There are safer ways of doing this and there are risker ways, but overall Context pieces are like this whole spectrum of possible options... and not even a linear spectrum! It's like a 3D graph of different variations and combinations that work in some conditions but not others. So start with what you know and just see where that takes you.
Title: Re: VCE English Question Thread
Post by: cosine on April 24, 2015, 08:06:00 pm
Thanks so much lauren!!

My sac is next week and I have not yet written any practice essays, nor any form of practice. Where can I start? I mean my teacher hasn't given us any prompts so thats a disadvantage.
Title: Re: VCE English Question Thread
Post by: chansena on April 24, 2015, 08:28:09 pm
My sac is next week and I have not yet written any practice essays, nor any form of practice. Where can I start? I mean my teacher hasn't given us any prompts so thats a disadvantage.


You want to start by doing some research into some external examples which you can use in your context. This could include famous people, quotes, poems, songs anything which you can tie into Identity and Belonging. You probably want to know about 6-8 different examples and have a few spares up your sleeve.

From there you need to decide which style of writing you are going to do
Creative, expository or persuasive.


The research stage is really crucial as it forms the backbone I suppose to your context piece.

And then it comes down to doing some practice pieces.

But I also like to practice breaking down prompts as I am quite slow to write context pieces. I practice by doing a plan for several topics.

Also the English Resources Page also has several ways of approaching context  English Resources and Sample High Scoring Responses

Hope this helped :)


Title: Re: VCE English Question Thread
Post by: heids on April 25, 2015, 08:17:55 am
My sac is next week and I have not yet written any practice essays, nor any form of practice. Where can I start? I mean my teacher hasn't given us any prompts so thats a disadvantage.

I totally understand because (especially for my evil enemy Context >:() I always felt like this soon before English SACs.

Chansena's advice is great.  Firstly you should have some background ideas and examples - from the text and external - about your context, a step that always overwhelmed me so I got into context SACs clueless.  This close to the SAC, I wouldn't stress so much about this, as I think you need to practise planning and writing an essay.  See Lauren's list of examples to help you.  When the SAC is over, please start researching for the future.

For prompts, see Re: Prompts and Sample Language Analysis Articles.  With a really long list of prompts, I never knew which to start with.  So I'd put them in a numbered list, and then use a random integer generator to randomly select one.  I suggest you decide to do say 10 plans + 2 full essays/pieces before the SAC (that was arbitrary, do however many you want/can), and randomly select prompts for these.  (Obviously cross off a prompt once you've done it).

Plans are great.  If you're doing expository, brainstorm the prompt, trying to think of it from all angles, etc. etc., and trying to throw in examples from your text and external examples.  Then gather your notes into a proper plan: write your contention, your 3-4 main ideas, and then dot-point how each paragraph will run - the examples you'll use, and what you'll draw out of them etc.  The more detailed you do it, the more valuable it is.  But it's not as scary or time-consuming as actually writing an essay.  It's crucial to write lots of plans and/or essays, since you'll be exposed to a random prompt in the SAC and have to actually write it.  Doing lots of random samples beforehand really helps boost your confidence and skills :D.

Finally, don't think it's too late to do anything and panic.  Write a list of what you want to achieve before the SAC (i.e. specific things, like 'write 5 detailed plans on randomly selected prompts') and tick them all off as you go.  Remember, anything you do - even if you just write one essay or come up with one external example the night before - is still going to be beneficial.  Try not to feel so overwhelmed that you don't even start.  I totally understand how tempting it is to do this, but breaking it down into small specific steps can help :).  Come up with something to do EVERY DAY between now and then.

EDIT: inserted a couple of smileys to make you feel better :D
Title: Re: VCE English Question Thread
Post by: cosine on April 25, 2015, 01:01:59 pm
I totally understand because (especially for my evil enemy Context >:() I always felt like this soon before English SACs.

Chansena's advice is great.  Firstly you should have some background ideas and examples - from the text and external - about your context, a step that always overwhelmed me so I got into context SACs clueless.  This close to the SAC, I wouldn't stress so much about this, as I think you need to practise planning and writing an essay.  See Lauren's list of examples to help you.  When the SAC is over, please start researching for the future.

For prompts, see Re: Prompts and Sample Language Analysis Articles.  With a really long list of prompts, I never knew which to start with.  So I'd put them in a numbered list, and then use a random integer generator to randomly select one.  I suggest you decide to do say 10 plans + 2 full essays/pieces before the SAC (that was arbitrary, do however many you want/can), and randomly select prompts for these.  (Obviously cross off a prompt once you've done it).

Plans are great.  If you're doing expository, brainstorm the prompt, trying to think of it from all angles, etc. etc., and trying to throw in examples from your text and external examples.  Then gather your notes into a proper plan: write your contention, your 3-4 main ideas, and then dot-point how each paragraph will run - the examples you'll use, and what you'll draw out of them etc.  The more detailed you do it, the more valuable it is.  But it's not as scary or time-consuming as actually writing an essay.  It's crucial to write lots of plans and/or essays, since you'll be exposed to a random prompt in the SAC and have to actually write it.  Doing lots of random samples beforehand really helps boost your confidence and skills.

Finally, don't think it's too late to do anything and panic.  Write a list of what you want to achieve before the SAC (i.e. specific things, like 'write 5 detailed plans on randomly selected prompts') and tick them all off as you go.  Remember, anything you do - even if you just write one essay or come up with one external example the night before - is still going to be beneficial.  Try not to feel so overwhelmed that you don't even start.  I totally understand how tempting it is to do this, but breaking it down into small specific steps can help :).  Come up with something to do EVERY DAY between now and then.

EDIT: inserted a couple of smileys to make you feel better :D

Thank you heaps, and many thanks for the smileys too haha :)
Title: Re: VCE English Question Thread
Post by: tiff_tiff on April 27, 2015, 01:11:15 pm
Does this make sense:
We cannot see ultraviolet radiation, nor x rays, for our cones are not sensitive to thee

don't think I used thee in the right context, but it sounds right :D
Title: Re: VCE English Question Thread
Post by: kimmytaaa on April 27, 2015, 02:42:22 pm
Hi Lauren
I have been having an English tutor that helps me, for my last sac he gave some examples and I did some practice essays for him to correct. When he was correcting it, he added a few notes and give some advice so that I can improve or work on next time, but when it came to the actual sac I did really poorly.But for that sac, we were given the prompt before hand so we got to do our planning at home before the sac date. My school teachers asked everyone to do a practice essay for her to correct but it wasn't compulsory and when I handed it up to her she said it was good but when it came to the actual sac, my teacher said is expected more from me. My tutor said if he didn't correct the practice essay, it won't be that good. I have a really bad habit of writing and writing so I do sometimes repeat things a lot but I don't see that mistake on my own. My tutor does points out those problems but I can't seem to improve on it. Is there a way to improve on this issue?
Title: Re: VCE English Question Thread
Post by: Callum@1373 on April 30, 2015, 08:44:44 pm
Received a B+ on my Language Analysis  :-[ Absolutely devastated.

Can anyone please for the love of god tell me that the comments I received suggest I should have got a much higher grade?

Quote
An intelligent and well constructed Language Analysis that demonstrates a very high understanding of persuasive techniques and how these devices are used to position readers.

Quote
The introduction contains all of the necessary information required in this kind of essay. The contention and tone are clearly and accurately identified.

Quote
The body paragraphs offer a thoughtful analysis of specific words and phrases and the manner in which they are used in an attempt to persuade

Quote
Excellent analysis of the visual material

Quote
The essay does not contain a concluding paragraph. This has affected the overall grade.
This is strange. I have been told before that All marks in language analysis come from the analysis. I did conclude my essay my summarizing the last techniques the author used and how readers were made to feel after they read the opinion piece.
Quote
The written expression is sophisticated and highly fluent

I feel like my teacher has legitimately slapped me in the face...after all that hard work to receive a bloody B+ in a damn year 11 essay...
Title: Re: VCE English Question Thread
Post by: heids on April 30, 2015, 10:01:22 pm
Does this make sense:
We cannot see ultraviolet radiation, nor x rays, for our cones are not sensitive to thee

don't think I used thee in the right context, but it sounds right :D

No, 'thee' means 'you' as an object (e.g. 'I love thee').  'Them' would fit in this sentence, but were you trying to rhyme or something ??? ?

Is there a way to improve on this issue?
I'm literally not Lauren, so wait for her to answer properly :-\  All I can say is to write some essays, and then afterwards imagine you're the tutor and go through and tear it apart.  Don't stress about finding ways to FIX the issues, just try to FIND the issues.  Then go to the tutor, get them to go over your issues, check if you've got them right, and ask for their help in fixing them.  Repeat this again and again - always try to find your own issues before you ask the tutor to point them out.  Before long, you'll be more aware of them as you write.

Received a B+ on my Language Analysis  :-[ Absolutely devastated.

Can anyone please for the love of god tell me that the comments I received suggest I should have got a much higher grade?  I feel like my teacher has legitimately slapped me in the face...after all that hard work to receive a bloody B+ in a damn year 11 essay...

Firstly: year 11, don't be devastated.  Even if your hard work hasn't paid off on this one SAC, it will pay off in year 12.

Secondly, just ask your teacher.  I agree, everything sounds really positive and deserving of something better, but obviously your teacher will have some reason which only he/she can explain, so just discuss it with him/her.  Unless she's one of those ones that has an innate aversion to handing out As and A+es.  In which case appeal to the school for a different teacher next year.  (I had a similar experience in year 12 lang analysis with 15/20 and not a single criticism written, discussed it with them and ended up with 17/20 and rank 1).

Thirdly... I like never, ever finished essays... always ran out of time (or more accurately, ideas, so I pretended I could have finished but ran out of time ;) )  I think the only SAC I finished in year 12 was the lang analysis one, then I didn't finish any of the other SACs or the exam.  Yet my marks ended up A+... I wouldn't have thought not finishing was too bad.
Title: Re: VCE English Question Thread
Post by: scandin9 on May 01, 2015, 09:05:34 pm
Hi,

My teacher says that we must mention persuasive techniques in the introduction; how can this be accomplished without listing?
Title: Re: VCE English Question Thread
Post by: heids on May 04, 2015, 09:25:56 am
My teacher says that we must mention persuasive techniques in the introduction; how can this be accomplished without listing?

I tended to put in the really major things, the major approaches the author used, the major ways the author tried to persuade throughout the article rather than just in one instance (e.g. the author mocks the opposition, or heavily relies on personal anecdote, or constantly uses experts/statistics etc. to add weight to his argument, or appeals strongly to nostalgia and tradition).  Remember, 'techniques' doesn't necessarily mean those horrible textbook 'named' techniques like 'rhetorical questions', 'appeal to hip-pocket nerve' etc.  I put in up to about 3 major approaches.  Often TONE is a really big factor to look at (you're analysing the LANGUAGE, not just argument techniques).

For instance, normally inclusive language is really minor (and sounds like basic technique identification) so you would end up listing if you put something like that in the intro; so normally you avoid it.  However, the occasional piece might be really really focused on inclusive language, appealing to group solidarity etc. as a major way of trying to persuade the audience, so you might include this in the intro.

So just stick to really major things that shape a lot of the article, and briefly show what their aim is and how they're trying to persuade the audience - you're showing that you understand the big-picture, overall dynamics of the piece.
Title: Re: VCE English Question Thread
Post by: StupidProdigy on May 04, 2015, 04:32:24 pm
For context is it bad to have the first sentence being a quote (expository essay)? It's a general broad quote not about my text, but the context and the prompt more generally
Title: Re: VCE English Question Thread
Post by: RazzMeTazz on May 05, 2015, 08:18:26 pm
Could someone please explain what is meant by the persuasive technique of 'rationalisation' in language analysis?

:) Thanks
Title: Re: VCE English Question Thread
Post by: literally lauren on May 05, 2015, 09:07:53 pm
For IDB I'm trying to think of an example for yes ones identity and belong is shaped constantly. 

and an example for yes ones identity does change but for the worst

I cant think of an external / real world example, event / quotes .


Any ideas ?
For the first instance, think about whether you'd consider yourself the same person now compared to who you were three years ago. Now would you say there was a dramatic shift in your identity at some point, or did you just gradually change in little increments every time? And for the second, you could consider any 'good person gone bad' stories or examples whereby someone who was once very moral, considerate, and level-headed then becomes callous, foolish, or naive.
Before looking for very specific examples, try to narrow down what you're talking about exactly so that you end up guiding yourself to an answer. So, instead of saying, 'what examples are there of continuous change,' ask yourself how do I know? Start with your premise or contention (in this case, one's identity is shaped constantly,) and then ask 'how do I know?' You may need to ask this question several times to get to the core of your point, but this'll help open up the discussion for you so that you can begin building upon what you know.

For context is it bad to have the first sentence being a quote (expository essay)? It's a general broad quote not about my text, but the context and the prompt more generally
That would be fine so long as you used it well. Most essays that open with quotes tend to just stick something profound that they googled before the SAC right at the start, and then write a piece that doesn't deal with the kinds of ideas that the quote raised.
Different teachers will have different preferences though; I know some who say 'no, don't, it's clunky and awful' and other's who've said 'it's a wonderful, engaging way to begin and everyone should do it,' so consult your education professional to see if opening-quotes are right for you :)

Could someone please explain what is meant by the persuasive technique of 'rationalisation' in language analysis?

:) Thanks
It depends on the context. The word itself means 'to make rational' (obviously from 'rational' + '-ise' + 'ation') so on the surface it would be used to refer to when the author makes something seem rational. This could be in the form of justification (eg. 'The author rationalises the concept of fearing change and explains that it is a natural human instinct.') Alternatively, the author might be reasoning or showing the logic behind something (eg. the equivalent of 'showing your workings' in maths: 'The government isn't allowing these people access to basic needs, and if those basic needs aren't met, they will die. So how can we support a government that knowingly allows people to die?') This'll usually be present in the form of 'leading logic' where the author creates this chain of A --> B --> C --> D etc. and implies that the final result is a natural, unavoidable consequence of A (eg. if I don't buy a 24 pack of pens for my exam, I might run out of pens, then I won't be able to finish, then I'll fail Year 12, then my parents will disown me, then I'll be forced to live on the streets, then I'll have to eat pigeons to survive. So if I don't buy these pens, I'll have to eat a pigeon. QED.) It's obviously a fallacy, but it's very common in Language Analysis pieces :)

Otherwise, if your teacher was giving you a specific example or told you to use this technique for a certain case, let me know and I might be able to explain it in more detail. Analysis rarely just stops at the technique level, so what you say after you identify the device is much more important.
Title: Re: VCE English Question Thread
Post by: scandin9 on May 05, 2015, 11:29:25 pm
Hey Lauren,

Do you have any resources on Year Of Wonders?
Title: Re: VCE English Question Thread
Post by: literally lauren on May 05, 2015, 11:49:22 pm
I do!
That was one of my Year 12 texts, though I didn't do it in the exam. I haven't posted anything for it because no Year 12s are studying it and very few schools select it for younger year levels, but I'll have a look through what I've got and see if there's anything worth scanning. I'll certainly have a practice essay or two, and maybe a few articles/ discussion question-answer type things :)
Title: Re: VCE English Question Thread
Post by: Callum@1373 on May 06, 2015, 05:17:46 pm
Hey Lauren,

I've finished reading Macbeth for text-response - I used a combination of watching the movie, reading the modern translation while reading the Shakespeare at once and I have a solid understanding of the plot.

What should my next step be? Thanks   :)
Title: Re: VCE English Question Thread
Post by: scandin9 on May 07, 2015, 08:15:33 pm
Hey Lauren :),

The resources for Year of wonders would be extremely helpful as I can't seem to find many academic articles on it.

Thanks in advance!
Title: Re: VCE English Question Thread
Post by: TheAspiringDoc on May 08, 2015, 06:35:55 pm
Hi all :)
So I really like English (i.e. reading, writing, debating and communicating etc.) and was wondering if you guys could give me some tips on how I could further my knowledge in the area. I read texts such as The Da Vinci Code, The Lost Symbol, The Kite Runner, The Hunger Games, Wild Cat Falling, Autobiographies, non-fiction, the newspaper, and AN essays people have posted up. The biggest issue is that while in english class at school, everyone mucks around and I end up being quite unproductive.. I'm also worried that students at selective schools are getting an unfairly large advantage over me and my low-grade private school..
Any tips/ ideas/ advice?
Thanks!! :)
Title: Re: VCE English Question Thread
Post by: literally lauren on May 08, 2015, 07:17:26 pm
Hey Lauren,

I've finished reading Macbeth for text-response - I used a combination of watching the movie, reading the modern translation while reading the Shakespeare at once and I have a solid understanding of the plot.

What should my next step be? Thanks   :)
For Macbeth, as well as many Shakespeare tragedies, there's only one real question: who's to blame?
You may think you know the answer now, but academics have bickered over this for centuries, so I guarantee there's more to say. See if you can answer that question, and let the discussion take you in different directions rather than concentrating on 'finding' the 'right' answer. For instance, if you think Lady M plays a significant part, then is it because she exerts control over Macbeth's character, or is she powerful in her own right? Does her power enhance or belittle Macbeth's? Does her degree of influence change at all? If so, what does this say about the blame we attribute? ~etc.

Also, there's no shortage of resources out there for Macbeth, so read heaps in order to expand your analysis and interpretation of the text :) Let me know if anything doesn't make sense... I think I've done Macbeth four times now over the course of my English-life  ::)

Hey Lauren :),

The resources for Year of wonders would be extremely helpful as I can't seem to find many academic articles on it.

Thanks in advance!
Sooooo, atarnotes won't let me upload the scans since they're too big  >:(  If you want, I could email them to you, otherwise I'll try and upload them in the actual Notes section (though I have no idea how soon they'll be published/ when you'll be able to access them though) - sorry!

Hi all :)
So I really like English (i.e. reading, writing, debating and communicating etc.) and was wondering if you guys could give me some tips on how I could further my knowledge in the area. I read texts such as The Da Vinci Code, The Lost Symbol, The Kite Runner, The Hunger Games, Wild Cat Falling, Autobiographies, non-fiction, the newspaper, and AN essays people have posted up. The biggest issue is that while in english class at school, everyone mucks around and I end up being quite unproductive.. I'm also worried that students at selective schools are getting an unfairly large advantage over me and my low-grade private school..
Any tips/ ideas/ advice?
Thanks!! :)
Read.
Read read read read read read read.

There's not much need to read VCE-level essays really, though I suppose you could peruse some Language Analysis or Text Response essays if you really wanted to. Those tend to be rather dry reading though (unless you're an oddball like me.)

What'll really help is reading. Novels of any genre, non-fiction texts on any subject matter - just go for whatever you're interested in. I owe at least half of my study score to the contents of my bookshelves.

The best thing you could do for yourself at this stage is to set yourself up with a solid intuition of grammar and sentence composition. Unfortunately, if you try and learn this deliberately, you'll likely just end up confused. Actually "learning" grammar is kind of counter-intuitive; most people are better off picking up on it subconsciously when they take in new information. So as great as it would be if you found a few authors or genres you really enjoy, the more you expose yourself to, the more chances you'll uncover something that will (implicitly) help you later down the line.

If you're really desperate, perhaps suggest a few areas of interest and I might be able to recommend something you like?
Title: Re: VCE English Question Thread
Post by: TheAspiringDoc on May 08, 2015, 07:42:45 pm



Read.
Read read read read read read read.

There's not much need to read VCE-level essays really, though I suppose you could peruse some Language Analysis or Text Response essays if you really wanted to. Those tend to be rather dry reading though (unless you're an oddball like me.)

What'll really help is reading. Novels of any genre, non-fiction texts on any subject matter - just go for whatever you're interested in. I owe at least half of my study score to the contents of my bookshelves.

The best thing you could do for yourself at this stage is to set yourself up with a solid intuition of grammar and sentence composition. Unfortunately, if you try and learn this deliberately, you'll likely just end up confused. Actually "learning" grammar is kind of counter-intuitive; most people are better off picking up on it subconsciously when they take in new information. So as great as it would be if you found a few authors or genres you really enjoy, the more you expose yourself to, the more chances you'll uncover something that will (implicitly) help you later down the line.

If you're really desperate, perhaps suggest a few areas of interest and I might be able to recommend something you like?
Anything with an element of uncertainty/mystery or any kind of book that forces you to think, whether it be about the plot or looking at real life society as a whole. I like things related to science but I'm guessing like most people there is a limited amount of pleasure I can draw from pages of chemical formulas and drawn out equations. I autobiographies of doctors in particular and I find adventure fiction pretty neat.
Sorry, that's probably quite a list to take in  ::)
Thanks!!  :D
Spoiler
PS #1 fan
Title: Re: VCE English Question Thread
Post by: literally lauren on May 08, 2015, 08:01:10 pm
 • The Adventures of Sherlock Holmes by Sir Arthur Conan Doyle
 • The Queen of Spades by Alexander Pushkin
 • The Curious Incident of the Dog in the Nighttime by Mark Haddon
 • The Doors of Perception by Aldous Huxley
 • The Strange Case of Dr Jekyll and Mr Hyde by Robert Louis Stevenson
 • Dangerous Book of Heroes (edited by the Iggulton brothers I think... basically a collection of really cool stories from different 'heroes' throughout history)
 • Awakenings by Oliver Sacks
 • The Spirit Catches You and You Fall Down by Anne Fadiman
 • The House of God by Samuel Shem
 • The Immortal Life of Henrietta Lacks by Rebecca Skloot
 • Complications: A Surgeon's Notes on an Imperfect Science by Atul Gawande
 • How Doctors Think by Dr. Jerome Groopman
 • Arrowsmith by Sinclair Lewis
 •The Good Nurse: A True Story of Medicine Madness, and Murder by Charles Graeber
 • Five Days at Memorial: Life and Death in a Storm-Ravaged Hospital by Sherri Fink
 • Medical Apartheid: The Dark History of Medical Experimentation on Black Americans by Harriet A. Washington
 • And The Band Played On by Randy Shilts


These are just my suggestions from the couple of shelves/piles I have in my eyeline :p Honestly, just wondering around some bookstores when you have the chance should net you a couple of good options. Perhaps google some of the above and see what takes your fancy (fishpond or TheBookDepository are good sites that do free shipping if you want to order some of them.) Otherwise your local library should have some interesting stuff. There's also a really good place at Fed Square every Saturday at about 10/11:00 onwards where vendors come and sell their wares. There's usually a lot to chose from, and I've never walked away empty handed.. though admittedly I do have a slight impulse-control problem when it comes to buying books... Anyway, it's held in 'The Atrium' in Fed Square, near the indoor entrance to the Art Gallery.Alternatively, if you want East from the station along Flinders St. then you should find it :)
Title: Re: VCE English Question Thread
Post by: Chang Feng on May 09, 2015, 06:29:33 pm
what form for context (ie feature article, etc) would be most similar to an essay style expository piece (basically the sample ones on the AN guide or Connect Education guide if seen those)? cause right now all i can write really is a generic expository essay style piece, but this would obviously have no purpose, so i was thinking which would be most similar and easiest to adapt from this essay form.
thanks.
Title: Re: VCE English Question Thread
Post by: cosine on May 09, 2015, 06:43:55 pm
WIth language analysis, is it okay if we refer to certain devices with more specificity?

For example:

The writer's use of metaphorical imagery causes..

Is it okay to refer to certain devices with verbs?
Title: Re: VCE English Question Thread
Post by: literally lauren on May 10, 2015, 10:49:12 am
what form for context (ie feature article, etc) would be most similar to an essay style expository piece (basically the sample ones on the AN guide or Connect Education guide if seen those)? cause right now all i can write really is a generic expository essay style piece, but this would obviously have no purpose, so i was thinking which would be most similar and easiest to adapt from this essay form.
thanks.

Probably a feature article, though you could also write an essay with a purpose (eg. an essay competition where the topic is 'how to prevent the conflict of climate change' or 'what do our actions towards asylum seekers say about our Australian identity?'/ whatever your Context may be.)

Purpose is more important for the SAC than the exam though, so it might be worth experimenting with some different forms and seeing what your teacher thinks suits you.

WIth language analysis, is it okay if we refer to certain devices with more specificity?

For example:

The writer's use of metaphorical imagery causes..

Is it okay to refer to certain devices with verbs?
Actually, I'd be even more specific than that! What kind of 'metaphorical imagery' is the writer using? What metaphor is being called upon? Just labeling something 'metaphorical imagery' doesn't tell us much.
And you can absolutely comment on verbs or other speech forms (eg. 'the author's repeated use of verbs like 'planning' 'thinking' and 'believing' further strengthen his call to action...') Specificity is key, so you should definitely try to zoom into the actual language behind the technique as much as possible (within reason) :)
Title: Re: VCE English Question Thread
Post by: scarletmoon on May 10, 2015, 11:31:44 am
HELP
I need external ideas for the prompt "Anything is justifiable in conflict"
I need to do an oral presentation on this soon
Title: Re: VCE English Question Thread
Post by: cosine on May 10, 2015, 11:53:54 am
Thanks Lauren!

Also what if I can pick up a technique, and I think, for example, it's sarcasm being used to draw attention on how ridiculous something is, what if the examiner/marker does not agree with me or does not think that's the reason of the use of sarcasm in that context?
Title: Re: VCE English Question Thread
Post by: literally lauren on May 10, 2015, 12:51:43 pm
HELP
I need external ideas for the prompt "Anything is justifiable in conflict"
I need to do an oral presentation on this soon

Your external ideas can come from anywhere, so there's not much point in me listing random things. Perhaps suggest some areas of interest/recommendations from your teacher to get yourself started.

Or, if you'd rather develop this yourself, first focus on your contention. Are you suggesting that 'yes, for the most part, anything is justifiable in conflict' or 'no, even if conflict is present, we can't say anything is justified'? Once you've got a general outline, ask yourself 'how do I know?' Why do I think this?
When you have these values in your head, it makes sense that they've come from somewhere, right? So when you argue something like 'conflict can have negative effects on people,' you're inclined to draw from what you know, and start sorting through your brain for examples of people suffering negative consequences. These may not be the only or exact examples you end up using, but at least it gives you a starting point.

Some questions to ask yourself that might help formulate ideas:
 - Do people have different ideas or what is and isn't justifiable?
 - Who does the 'justifying?'
 - Does conflict make things justifiable, or do we grant people more leniency in times of conflict?
 - Is it possible for certain things to be morally/ethically justifiable, but not legally justifiable (or vice versa?)
 - Does the fact that 'anytihng is justifiable' mean we are entitled to do anything?
 - Are things dependent on their context, or can we say, as a general rule: this is justifiable but this is not - regardless of the circumstances?

Thanks Lauren!

Also what if I can pick up a technique, and I think, for example, it's sarcasm being used to draw attention on how ridiculous something is, what if the examiner/marker does not agree with me or does not think that's the reason of the use of sarcasm in that context?
If you've argued your point effectively, then it's rare for the assessors to take marks off for disagreeing. Of course, this understanding will be strengthened throughout the year, and it'll get to a stage where you know saying something like 'therefore the author attacks other people to make them more sympathetic' is probably unlikely.

Unlike the other essays, L.A. doesn't leave much room for error. You'd have to be saying something pretty out-there for an assessor to disagree with your analysis. Worst case scenario, if you're not entirely sure about the effect of something, you could use a word like 'may' or 'perhaps' (eg. 'here, the author makes use of sarcasm; perhaps in an attempt to draw the audience's attention to the ridiculous nature of the situation') but this should be done sparingly, as it can make you seem uncertain about your interpretation.

I guess my recommendation would be to practice writing on different articles until you're no longer at risk of making comprehension errors of misconceptions :)
Title: Re: VCE English Question Thread
Post by: cosine on May 10, 2015, 12:58:46 pm
Thanks Lauren, again :P

Where can I get good/interesting (non-pointless) articles where I can read them and annotate so that I can be quick at it!? :)
Title: Re: VCE English Question Thread
Post by: literally lauren on May 10, 2015, 01:14:27 pm
Thanks Lauren, again :P

Where can I get good/interesting (non-pointless) articles where I can read them and annotate so that I can be quick at it!? :)

It depends what you're wanting to practice.

If you need to get better at comprehending contentions and just understanding the author's arguments, then going for shorter texts will be more efficient. The comments section in The Australian (underneath the cartoon, next to the editorials; not the really short/twitter comments on the far right, but the centre of the page that contains Letters to the Editor and other bits and pieces) is quite good for this.

On the other hand, if you need to practice picking out techniques, then you could go for the longer editorials, or perhaps even online magazine articles. Generally though, for the SACs you should seek materials from your teachers (especially SACs from past year levels) and beyond that, for exam practice, you should just use official practice exams.

There's no shortage of company papers (CSE, Insight, Neap, VATE <-- esp. this one) that are really good for practice purposes, or you could even go through the past VCAA exams and see how you fare.

With regards to annotations, it's not all about being quick, though. You kind of need to develop a plan of attack for how you'll approach the task under timed conditions. Are you going to go through the whole article and highlight every single persuasive device you find? How are you going to structure your essay? Will you write whole words/sentences in your annotations so you can refer back to them, or will  you just write really brief reminders to yourself so as to devote as much time to the essay as possible? Often the people who do extensive annotations aren't actually being as efficient as they could. As a practice exercise, doing heaps of annotations can help you get used to recognising what you need to analyse, but be smart about the process. Don't just annotate for the sake of doing so - work out what's most helpful to you and take things from there :)
Title: Re: VCE English Question Thread
Post by: cosine on May 10, 2015, 01:23:15 pm
It depends what you're wanting to practice.

If you need to get better at comprehending contentions and just understanding the author's arguments, then going for shorter texts will be more efficient. The comments section in The Australian (underneath the cartoon, next to the editorials; not the really short/twitter comments on the far right, but the centre of the page that contains Letters to the Editor and other bits and pieces) is quite good for this.

On the other hand, if you need to practice picking out techniques, then you could go for the longer editorials, or perhaps even online magazine articles. Generally though, for the SACs you should seek materials from your teachers (especially SACs from past year levels) and beyond that, for exam practice, you should just use official practice exams.

There's no shortage of company papers (CSE, Insight, Neap, VATE <-- esp. this one) that are really good for practice purposes, or you could even go through the past VCAA exams and see how you fare.

With regards to annotations, it's not all about being quick, though. You kind of need to develop a plan of attack for how you'll approach the task under timed conditions. Are you going to go through the whole article and highlight every single persuasive device you find? How are you going to structure your essay? Will you write whole words/sentences in your annotations so you can refer back to them, or will  you just write really brief reminders to yourself so as to devote as much time to the essay as possible? Often the people who do extensive annotations aren't actually being as efficient as they could. As a practice exercise, doing heaps of annotations can help you get used to recognising what you need to analyse, but be smart about the process. Don't just annotate for the sake of doing so - work out what's most helpful to you and take things from there :)

Thanks Lauren, again!

What would you recommend for a beginner? Short or long ones? :)
Title: Re: VCE English Question Thread
Post by: literally lauren on May 10, 2015, 01:42:04 pm
Thanks Lauren, again!

What would you recommend for a beginner? Short or long ones? :)

Probably best to start with short ones and work your way up.

Alternatively, the 2008 VCAA exam is an excellent starting point, and it'd be considered fairly short by current exam standards, so maybe check that one out :)
Title: Re: VCE English Question Thread
Post by: tiff_tiff on May 11, 2015, 05:20:12 pm
hey lauren,

I'm reading this article:
and i'm not quite sure what the author is trying to say about the 'artificial haemoglobin' in comparison to the new stem cel ideology.
I get that the article is trying to lead readers more into the stem cell version, because they say its the 'new idea', but they didn't really say what's wrong with the 'artificial haemoglobin' version. is it just presented there for the author to seem like she knows her science?

and also, they didn't mention anything about Douay's earlier process?

[i bolded the relevant part]

Could Stem Cells Breathe New Life into the Field of Blood Substitution?
Immature cells' regenerative prowess injects new excitement into the field March 17, 2014 |By Dina Fine Maron

More than a century after scientists embarked on the quest to find an alternative to the blood coursing through our veins, the dream still will not die. Not after a major study dealt a seemingly fatal blow to the field—determining that the top synthetic blood candidates at the time were all more likely to kill you than to save your life. Not after billions of dollars in public and private investments dried up. And not after multiple companies ran aground.
Starting in 2011, however, the moribund field received yet another revival, this time from a group of French researchers with a new approach to boosting blood supplies. Their principal insight: don’t try to re-create millions of years of evolution. Instead, they proposed to piggyback off of what nature already made by coaxing stem cells into taking on the job.
The appeal of creating blood alternatives is obvious. Certainly after a battlefield trauma or a car accident a ready transfusion of artificial blood that could theoretically work with any blood type and not require refrigeration would be a welcome medical tool. A synthetic product outlasting the typical 42-day shelf life of red blood cells and sidestepping even the miniscule risk of transmitting a blood-borne disease would also be high on the medical wish list. But such a product has not yet been created and proved safe in humans.
It’s not for lack of trying. Although blood cells serve multiple roles in the body and have complex interactions with other cellular materials, most synthetic blood products have aimed to just stick to the bare basics—shuttling oxygen from the lungs to different vital organs and then bringing carbon dioxide back to the lungs to be exhaled. When the red cell count gets low, bodily organs may not get the oxygen they need, making a person weak and eventually resulting in serious health problems. The most popular approach taken to replicate that function has been to create artificial hemoglobin- based oxygen carriers, tapping proteins in red blood cells called hemoglobin that act as oxygen’s transport service, and chemically modifying them to increase oxygen-carrying capacity.
But the new idea is to get the body to grow its own substitute—a product that would not be the same as whole blood but could fit the bill in a pinch.

A Paris-based research group, headed up by Luc Douay, professor of hematology at University Pierre and Marie Curie Faculty of Medicine, has already had some success. They culled stemlike cells from blood circulating through a patient’s body and manipulated them into becoming red blood cells nearly identical to those that normally transport oxygen in the body. The team injected two milliliters of the stem-cell derived blood cells back into the patient—an amount far smaller than would be needed in a typical transfusion. The creations had stored well at refrigerated temperatures and circulated in the body with survival time on par with that of original red cells.
It’s an encouraging step forward for a field littered with odd and sometimes cringe-worthy efforts to get at the lifesaving power of blood. Animal to human blood transfusions received a short-lived audition in 1667. But the first human-to- human blood transfusion was not performed until 1818—before we learned about blood types and how and when the body rejects certain transfusions. Blood-product research also included attempts in the late 1800s to hook up ailing patients to infusions of fresh cow’s milk. Milk, like blood, had fats that emulsify in fluid, the reasoning went. Plus, milk would be safer than blood because it would not clot. When patients died, physicians figured it was due to other complications. Needless to say, milk injections, like those from animal blood, never really took off.
Although most people only get transfusions once or twice in their lives (if at all), individuals with conditions like sickle- cell anemia require consistent blood transfusions of red cells. But with each infusion there’s a small risk that the body could develop an infection, reject the foreign blood or form antibodies that will lead to the body rejecting and destroying certain bloods in the future. A key threat, however, is that each transfusion contributes to the risk of iron overload in the body. All red blood cells contain iron, but after the body takes what it needs it has no easy way to dispose of the excess. It gets stored, instead, in organs including the heart, liver and pancreas. That buildup of increased iron with each transfusion can damage the organs and eventually prove fatal.
10004 Article 4
The French researchers hope that using freshly created blood cells made from stem cells could help alleviate those iron buildup concerns. “We think it could be transfused at least three to five times less each year because of the efficiency of the transfusion,” Douay says.
The secret lies in the age of the red blood cells derived from stem cells. Although red cells from donors have a typical shelf life of 42 days, they are a mix of older and newer cells, which means a number of them may not last long in the body. With stem cell–derived options all of the blood product would be new, which could theoretically give patients more bang for each infusion. The only thing that would appear different to a patient receiving the transfusions, ideally, is that he would be receiving them less often. “If you have brand-new cells, you should be able to increase the intervals between transfusions so you can make it longer, says David Anstee, director of the International Blood Group Reference Laboratory in England. “You might be able to improve the quality of life in those situations.” It’s not a perfect fix because it would likely add months, not years, between transfusions, but it could be a start.
Moreover, the idea of using Douay’s earlier process, which involved growing the cells in culture, at a larger scale would be “delusional,” he says. To make just one unit of blood—roughly a pint—it would require growing cells in about 400 flasks that were about 30 centimeters by 20 centimeters, he says. But even with endless space for those flasks it would still be impossible because the constant pH and temperature controls that would be needed would be impossible to maintain. What would be needed, he says, is an automated, stirred large-scale bioreactor (something his team hopes to one day produce themselves). “Even something as seemingly simple as red blood cells that don’t have a nucleus evolved a structure and a function that is much more complicated than we can perceive by looking under the microscope,” says Jason Acker, associate director of development for Canadian Blood Services..
Title: Re: VCE English Question Thread
Post by: KingDrogba on May 11, 2015, 09:23:39 pm
How should you break down a text response prompt and then answer it?
What structure should i be looking as a guideline to my writing?
Title: Re: VCE English Question Thread
Post by: InNeedForHelp on May 12, 2015, 10:37:01 pm
Hi Lauren,

Regarding shortening quotes, my teacher says to shorten them to 5 or 6 words but I'm finding it hard to shorten quotes from "Maus". I find that if I try to shorten them, I lose the a lot of the significant of the quote so it doesn't prove the point I'm trying to make.

Also, for remembering quotes, should I write the quotes under categories of themes or categories of what the character says?

Thanks
Title: Re: VCE English Question Thread
Post by: YellowTongue on May 12, 2015, 11:00:41 pm
In general, how do you go about memorizing quotes? Do you just sit down a stare at them for ages, or is there a technique you use? Also, how many quotes should I try and memorise for each text that I'm studying?

Title: Re: VCE English Question Thread
Post by: literally lauren on May 13, 2015, 06:12:46 pm
hey lauren,

I'm reading this article:
and i'm not quite sure what the author is trying to say about the 'artificial haemoglobin' in comparison to the new stem cel ideology.
I get that the article is trying to lead readers more into the stem cell version, because they say its the 'new idea', but they didn't really say what's wrong with the 'artificial haemoglobin' version. is it just presented there for the author to seem like she knows her science?
That section you've bolded would probably be one of the bits you'd leave out in analysis. That's not to say there isn't some worthwhile language to discuss, but it just doesn't seem very central to the argument. Most of it is just an outline about the history of the process that serves as a backdrop for the 'new idea.' Overall, this doesn't seem like a very persuasive piece though. It's more like the sort of thing you'd see in a scientific journal than a VCAA exam :)

How should you break down a text response prompt and then answer it?
What structure should i be looking as a guideline to my writing?
There is no 'one' breakdown and there is no 'one' structure. In fact, it's better if you build off what you know and try to shape that into something suitable than adopt someone else's methods completely. There are some recommendations on the first page of this thread, and you could always read practice essays to get a sense for what works well and what doesn't, but ultimately there are so many things you could do right and wrong, sometimes it's best to just sump in and work on little things gradually rather than trying to formulate a fool-proof format before you know your own strengths and weaknesses.
Breaking down the prompt should help you open up the prompt and tease out some interesting ideas, and your structure should enable you to explore these ideas. Beyond that, it's pretty much up to you :)

Hi Lauren,

Regarding shortening quotes, my teacher says to shorten them to 5 or 6 words but I'm finding it hard to shorten quotes from "Maus". I find that if I try to shorten them, I lose the a lot of the significant of the quote so it doesn't prove the point I'm trying to make.

Also, for remembering quotes, should I write the quotes under categories of themes or categories of what the character says?

Thanks
In general, how do you go about memorizing quotes? Do you just sit down a stare at them for ages, or is there a technique you use? Also, how many quotes should I try and memorise for each text that I'm studying?


I'll deal with these together since it seems like there's some overlap.

With very few exceptions, no one really gets much of a learning benefit from just staring at words on a page. Association is a much stronger and more efficient way of remembering materials. For starters, grouping quotes is really valuable: try and do so either by themes or characters depending on your text, and then you'll quickly start to link certain ideas together. Plus, when you start writing essays on these ideas, the links get even stronger.
That said, there's no need to be testing yourself too early. No one expects you to simply read a text once, write some notes, and then be able to roll out quotes with complete confidence. The first few practice paragraphs or essays you write should definitely be making use of your resources: have the text open in front of you, refer to your notes as you go, and perhaps even use class discussion or teacher feedback (whatever is more helpful) to compliment your own exploration.

For the more hardcore amongst you, you might even try some more elaborate memory hacks. Like I said, association is the most reliable means of forging mental connections, but it's usually easier to associate unfamiliar material with familiar things. So rather than just listing a whole bunch of quotes and 'associating' it with a character's name or a theme isn't likely to help.

There's a memory contest every year where geniuses the world over try to beat the world record for the amount of shuffled card decks memorised; (I believe it stands at about 60 or so) and when these people are interviewed, they often talk about how they make connections in order to aid their memories. For instance, they'll assign each card a certain person place or thing, so if they have to memorise a sequence like: Jack of Clubs, Two of Diamonds, Nine of Hearts - then they'll tell a little story with the assigned meanings. Let's say the Jack of Clubs = Jack Dawson from Titanic; the Two of Diamonds = my mothers' diamond earings, and the Nine of Hearts = my dog because she's nine years old. In order to remember the sequence, I'd simply picture Jack Dawson wearing my mother's earings, patting my dog.
^That's a way more memorable, vivid image than an objective sequence.

So how would this help you in VCE?
You obviously wouldn't need to go to such an extreme, but the same principles can be applied on a small scale. For instance, one of the books I studied in Year 12 dealt with the theme of identity and self-perception/deception, so I wrote up a list of quotes that related to these concepts and pinned them up on my bathroom mirror (because the mirror is where you see yourself, and reminds me of things like vanity and self-awareness.) Not only did that mean I was seeing the quotes a couple of times a day, but I was also able to connect them to one another under that umbrella of 'identity and perception.'

You could do the same with some of your other themes: perhaps print off a list of quotes about the importance of place and setting and blue-tack them to your front door or bedroom window. Or grab a list of ones about relationships between characters and put them next to your photo frames of your own friends and family. You can get as creative as you want - the important thing is that the immersion works as both a deliberate and incidental form of learning; when you need to memorise more, you can consciously tell yourself to go read the lists and recite them, and when you're just lying in bed exhausted after a long day, your eyes might glance over the list on your bedside table. Gradual absorption is hard to notice, but it does happen, and it does help.

And of course, using the quotes in the context of your analysis (ie. practice essays) is also very beneficial.

In terms of shortening quotes, it does depend on the text. Maus does have a lot more dense sentences, but there should still be a 'core' for most of the quotes that'll be containing most of the critical information you need. There are exceptions, and sometimes you just need to quote about ten words or so, but for the most part, the unimportant stuff can be paraphrased. Also, don't discount the role of quote modification. Altering the grammar of the quote can be an easy way of cutting down irrelevant information. Just use [square brackets] to add new information or replace words/letters that don't fit, and use ... ellipses ... to omit details.

So if I wanted to alter the sentence: "Jimmy, why don't you just go away and abandon your foolish dream"
I could say: Suzie is very critical of her brother's attitudes, and questions why he doesn't "just... abandon [his] foolish dream" to fly an aeroplane.
I'm hoping this is familar to you; if not, let me know and I can explain this in more detail. Otherwise, perhaps give me an example or two of the kinds of quotes you're finding hard to shorten and I can walk you through it :)
Title: Re: VCE English Question Thread
Post by: InNeedForHelp on May 13, 2015, 06:57:09 pm
Thanks Lauren for the thorough reply,

A couple of the quotes are:

”Friends? Your friends...? If you lock them together in a room with no food for a week...then you could see what it is friends."

"It sounds like you're feeling remorse - maybe you believe you exposed your father to ridicule... And now that you're becoming successful, you feel bad about proving your father wrong."

Also, this might sound silly but do I have to include the page number in the quote and should I remember the entire quoted or a shortened version of the quote?

Thanks again!
Title: Re: VCE English Question Thread
Post by: heids on May 13, 2015, 07:13:02 pm
^Not answering, just linking you to another of Lauren's posts for more advice on quotes :): Re: English Q&A

But far more important (:P) is my own question: I want to start reading the news to get some vague understanding of what's going on in the world.  I'm the most ignorant current-events person you've ever met.  But I don't know where to start - I don't know what sites to look at, or how to cope with the fact that I start reading an article and don't have the slightest clue of what's going on because I have no background.  Can you point me to any good (+ free) site or newsfeed or give me any newspaper-reading advice Lauren?

Thought I'd throw the question here because reading the news is great for language analysis and external examples for context (not that I ever did it).  Thankye kindly, your help is always much appreciated!
Title: Re: VCE English Question Thread
Post by: literally lauren on May 13, 2015, 07:59:03 pm
Thanks Lauren for the thorough reply,

A couple of the quotes are:

”Friends? Your friends...? If you lock them together in a room with no food for a week...then you could see what it is friends."

"It sounds like you're feeling remorse - maybe you believe you exposed your father to ridicule... And now that you're becoming successful, you feel bad about proving your father wrong."

Also, this might sound silly but do I have to include the page number in the quote and should I remember the entire quoted or a shortened version of the quote?

Thanks again!
So for that first quote, you'd probably want to concentrate on the last part that sounds a bit like a threat: "then you could see what it is friends" and perhaps the bit before that as context. But this is a great idea of where paraphrasing can come in handy, eg. The character declares that "if you lock them together in a room', only then would their true loyalties be revealed. That way, you're capturing the essence of the quote without using the whole thing.

Same goes for the second:
eg. The character implies the other character is plagued by his relationship with his father, and questions whether he "exposed [his] father to ridicule" and as such, "feel[ s] bad about proving him wrong."
Even that simple modification can help minimise what you're quoting.

Ideally you'd want to be familiar enough with the original/whole quote to be able to use it in different contexts and modifying it differently each time, so I'd say memorise the whole thing and just be prepared to chop and change on the go.

No, you don't need page numbers, but I've seen teachers get snarky about not including them in SACs, so check with yours to make sure. But in the exam, no, it's a waste of time and it's not required.

^Not answering, just linking you to another of Lauren's posts for more advice on quotes :): Re: English Q&A

But far more important (:P) is my own question: I want to start reading the news to get some vague understanding of what's going on in the world.  I'm the most ignorant current-events person you've ever met.  But I don't know where to start - I don't know what sites to look at, or how to cope with the fact that I start reading an article and don't have the slightest clue of what's going on because I have no background.  Can you point me to any good (+ free) site or newsfeed or give me any newspaper-reading advice Lauren?

Thought I'd throw the question here because reading the news is great for language analysis and external examples for context (not that I ever did it).  Thankye kindly, your help is always much appreciated!
For a very brief overview, watching the commercial news (probably Channel 9, or The 7pm Project or w/e it's called nowadays) will give you a quick and often slightly skewed run-down of current stories. However, on busy news days, this often isn't sufficient to get a full sense of the interesting stories, and on slow news days, you have to watch a lot of dull filler like 'studies have shown brunettes are more likely to be good at tennis' or 'this family bought a puppy today' shout-out to the first 30 seconds of this

ABC is your best bet for more in-depth coverage; the 7:30 Report have pretty good exposés, and whilst they get criticised for their left-wing bias, I honestly don't notice it much, if at all... or maybe that's just my own leftist perspective talking :p

SBS is definitely the forerunner for international news, and they take a much more journalistic approach to major stories than, say, Channel 7. It's a tad drier than most other sources, but it's probably the most reliable.

When it comes to written material, it might be worth reading around and finding a few journalists whose writing style you like, or whose views you consider engaging.
I'm very partial to satire myself, so now that Shaun Micallef's Mad as Hell is over for this season, I tend to read a lot of SBS' Comedy paper 'The Backburner'
some really amusing examples here :)

It's also a matter of personal interest levels: you can't really go wrong if you're just reading heaps of different materials (Andrew Bolt notwithstanding) but if you're not interested in the subject matter, it's likely to lose you quite early on. Personally, I've never been a fan of commerce or business-based news, so financial discussions usually bore me unless they're linked to an intriguing political or legal situation. By contrast, I'm quite fascinated with diplomatic relations, especially when it comes to cultural differences or linguistic barriers, so whilst the Bali executions were circling around in the media, I  was reading heaps about Indonesia's attitudes to prisons and drugs, and why the Australian government's approach put them between a rock and a hard place. Even this latest Budget discussion; there've been some interesting critiques of the language Abbot and Hockey have employed - most notably the x100 repetitions of 'a fair go' in the past 24 hours - which is rather amusing.

I definitely wouldn't call myself well-informed when it comes to current events; I kind of just read enough to get by for the sake of VCE English which is kind of tragic  ::) but I consider the ability to make connections just as important as what you're discussing. So I may not have an intricate understanding of the machinations of the Labour party, but I've written quite a few paragraphs comparing the Gillard-Rudd-Gillard metaphorical backstabbing with the literal backstabbing of Caesar by Brutus.
Basically, a passable grasp on Australian politics and famous Roman betrayals is more preferable, to me, than a really in-depth grasp on just Australian politics.

Also, if you're reading for the purposes of getting a quick overview of an issue or event: read the title, the first paragraph, and then the first and possibly last lines of every other paragraph. This often works for books too, provided it's not anything like Pynchon or Wallace where 'paragraphs' can go on for entire chapters :p

I guess it's best to find something that makes 'news' as well as individual news stories interesting on a personal level. Failing that, just befriend someone who's news-minded and make conversation with them so you can leech of their knowledge  :D
Title: Re: VCE English Question Thread
Post by: KingDrogba on May 16, 2015, 05:52:32 pm
What's been the usual A or A+ cut off for unit 3 and 4? Just wondering and cant seem to find it anywhere
Title: Re: VCE English Question Thread
Post by: heids on May 16, 2015, 05:59:04 pm
What's been the usual A or A+ cut off for unit 3 and 4? Just wondering and cant seem to find it anywhere

Search for 'vcaa grade distributions [year/subject]'.  Here are 2013 and 2014 distributions.  A+ seems about 84-85, A more like 74-75%.

P.S. Thanks Lauren for your help above!  Wery much obliged to ye.
Title: Re: VCE English Question Thread
Post by: Chang Feng on May 18, 2015, 09:06:57 pm
what form best suits the below purpose of writing (for context):
so i'm writing an expository piece in which i am explaining the reasons for different ideas of the topic (ie conflict occurs between the powerful and powerless). -so some of my ideas are that since the powerful wants to maintain power they will dismiss any ideas that contradict them, and these ideas of contradiction usually come from the powerless individuals of society. so i would go on and explain why this happens, since like it is human nature to have a greed for power.... and then use set text to provide evidence of this. and then provide an evaluation of the consequences of such conflict. Then provide a solution/ action of what we individuals should do in the future/ why thus conflict cannot be erased and will keep lingering on in society and thus the consequences we individuals have to face.
(thanks for the advice).
Title: Re: VCE English Question Thread
Post by: Chang Feng on May 18, 2015, 09:17:37 pm
and is it acceptable to write an expository essay for context?? Since the audience, form, language choice and purpose must relate.
Title: Re: VCE English Question Thread
Post by: HighTide on May 19, 2015, 08:18:56 am
what form best suits the below purpose of writing (for context):
so i'm writing an expository piece in which i am explaining the reasons for different ideas of the topic (ie conflict occurs between the powerful and powerless). -so some of my ideas are that since the powerful wants to maintain power they will dismiss any ideas that contradict them, and these ideas of contradiction usually come from the powerless individuals of society. so i would go on and explain why this happens, since like it is human nature to have a greed for power.... and then use set text to provide evidence of this. and then provide an evaluation of the consequences of such conflict. Then provide a solution/ action of what we individuals should do in the future/ why thus conflict cannot be erased and will keep lingering on in society and thus the consequences we individuals have to face.
(thanks for the advice).
It does depend on which type you are better at doing. It is likely that most people will do expository essays in the exam or for SACs. So less individuals will do imaginative and persuasive styled essays. I have heard from our school that an ordinary expository will get less marks than an ordinary creative or persuasive. However, saying that the other two forms are also tough. And also, remember that a good essay is a good essay regardless of the style that you use. If you write a good expository essay, you will get a good mark. Simple as that   :)
If you are in your final year, it's probably recommended that by this time of year, you choose the style that you will do for the exam and improve at it.
and is it acceptable to write an expository essay for context?? Since the audience, form, language choice and purpose must relate.
You certainly may write an expository. Just ensure that you state your audience, form, language choice and purpose clearly in the statement of intention, and also refer to your text and key ideas. As long as you follow your SOI, your essay should be good given that it does explore the key ideas in the text.
Title: Re: VCE English Question Thread
Post by: biy on May 19, 2015, 08:53:18 pm
Can anyone give me some oral topics please? My oral presentation is in the first week of term 3 and I have not yet started :(
Title: Re: VCE English Question Thread
Post by: scarletmoon on May 19, 2015, 08:54:39 pm
Can anyone give me some oral topics please? My oral presentation is in the first week of term 3 and I have not yet started :(


http://www.vcestudyguides.com/oral-presentation-topics-for-2015
Title: Re: VCE English Question Thread
Post by: Apink! on May 20, 2015, 12:43:12 pm
How can I improve in English if I have no one to mark my essays?
I could post them on AN but I've noticed that it is not very active (the marking board) and I can't really ask my school teacher all the time.

If I don't know where I need to improve on and don't know what I have done well, is simply just writing an essay a day will help me?

I would appreciate your advice. Is there a way for my essays to be marked without annoying anybody?

p.s I can't get a tutor because I can't afford it!
p.p.s any advice will be appreciated


Title: Re: VCE English Question Thread
Post by: heids on May 20, 2015, 01:57:51 pm
^ I decided a couple of days ago to start marking stuff (that is, from now onwards, not catching up on past ones); I've always thought I wasn't good enough, but while my marking won't be high calibre, hopefully it'll be still helpful in pointing out major issues.  So go for it occasionally, I'll give it a go if I have time.  Can't hurt if you post and don't get a response, anyway.  How about you try giving bits of feedback to others?  You'd learn a lot out of that yourself, and give the board a boost! :D

Anyway, PLEASE READ ALL THIS ADVICE as it is the most important thing I could ever say to a year 12.  You get way more out of the year if you work actively for yourself, rather than relying on being spoonfed or giving up because you're 'disadvantaged' by your SES/school/teachers/parents/situation.  I followed this advice for my 3 best subjects (which was why they became my top subjects), and felt totally on top of them; but I didn't for English ==> was completely unsure, confused and terrified; I kept harping on the fact that "I didn't know" and "I didn't have enough resources" (I hadn't really looked at AN, unfortunately).

OK, so.  I understand it's frustrating having limited resources (I couldn't afford to pay anything other than school fees for English), but the most valuable skill you can learn is being able to teach yourself with the resources you've got (actually, you have access to more than you think, it just depends on how you use it).  Because if you can learn what is required of a good essay, you'll then be able to see your own strengths and weaknesses by yourself.


Essentially what I'm saying is, actively use your resources.  If you really try, you can teach yourself how to write a good essay, and what things to avoid.  You can teach yourself how to mark your own work and find your own issues.  (Never just write an essay without critically self-marking).  If you find a problem you can't solve, ask about it in this thread.  And use that occasional 'true' feedback from other experts to help you become even more of an expert!

Finally, try essay-swapping.  Give your friend an essay to mark, and you mark one of theirs.  Sure, it won't be the best feedback, but you'll be able to help each other; and actually, marking other people's work really helps you discover faults and try to come up with ways to fix them.

tl;dr (though I hope you did read >:( :P): RELY ON YOURSELF, and actively use the resources you already have to teach yourself: AN, high-scoring essays, feedback on essays in the WM&S board, Lauren's Q&A threads, etc.

P.S. Some day I will learn the skill of writing a short post.  Some day.
Title: Re: VCE English Question Thread
Post by: Apink! on May 20, 2015, 02:54:51 pm
Hi
I read everything!
You made me realise how much resources are actually floating on AN. Yes, I realise now that not being able to afford resources is just an excuse.
I will go on the English question thread now and take notes!

Thank you so much for an insightful advice. I couldn't have asked anything more
p.s you're awesome! :P
Title: Re: VCE English Question Thread
Post by: Adequace on May 20, 2015, 05:13:24 pm
^ I decided a couple of days ago to start marking stuff (that is, from now onwards, not catching up on past ones); I've always thought I wasn't good enough, but while my marking won't be high calibre, hopefully it'll be still helpful in pointing out major issues.  So go for it occasionally, I'll give it a go if I have time.  Can't hurt if you post and don't get a response, anyway.  How about you try giving bits of feedback to others?  You'd learn a lot out of that yourself, and give the board a boost! :D

Anyway, PLEASE READ ALL THIS ADVICE as it is the most important thing I could ever say to a year 12.  You get way more out of the year if you work actively for yourself, rather than relying on being spoonfed or giving up because you're 'disadvantaged' by your SES/school/teachers/parents/situation.  I followed this advice for my 3 best subjects (which was why they became my top subjects), and felt totally on top of them; but I didn't for English ==> was completely unsure, confused and terrified; I kept harping on the fact that "I didn't know" and "I didn't have enough resources" (I hadn't really looked at AN, unfortunately).

OK, so.  I understand it's frustrating having limited resources (I couldn't afford to pay anything other than school fees for English), but the most valuable skill you can learn is being able to teach yourself with the resources you've got (actually, you have access to more than you think, it just depends on how you use it).  Because if you can learn what is required of a good essay, you'll then be able to see your own strengths and weaknesses by yourself.

  • Read VCAA exam reports and know the criteria inside out
  • Read through advice threads in the Eng resources thread, and at VCE study guides etc. (there's a ridiculous amount of free stuff if you just search for it!), and watch vTextbook; don't just read/watch, actively take notes
  • Critically go through high scoring responses; break down their arguments into detailed dot-points; watch how they do topic sentences; steal phrases or ways of expressing themselves really clearly; go through how they're different to yours; and so on.  Pull 'em apart.
  • Go through feedback given to other essays (i.e. read through the Compilation of Text Response/Context/Lang Analysis Feedback stickies in the Work Submission board).  In all probability, you will have fallen into the same pitfalls or have the same high points as some of the markers point out to other people.  You can learn SOOO much to apply to yourself!!! TAKE NOTES.  It's almost as good as direct feedback on your own, since Ned Nerb and Darvell gave great feedback.
  • Read through Lauren's 50 in English, available for queries :) and this year's Q&A; sure, skip irrelevant stuff, but I tell you there is sooo much gold that you don't want to miss like I did last year.  And again, taken notes.

Essentially what I'm saying is, actively use your resources.  If you really try, you can teach yourself how to write a good essay, and what things to avoid.  You can teach yourself how to mark your own work and find your own issues.  (Never just write an essay without critically self-marking).  If you find a problem you can't solve, ask about it in this thread.  And use that occasional 'true' feedback from other experts to help you become even more of an expert!

Finally, try essay-swapping.  Give your friend an essay to mark, and you mark one of theirs.  Sure, it won't be the best feedback, but you'll be able to help each other; and actually, marking other people's work really helps you discover faults and try to come up with ways to fix them.

tl;dr (though I hope you did read >:( :P): RELY ON YOURSELF, and actively use the resources you already have to teach yourself: AN, high-scoring essays, feedback on essays in the WM&S board, Lauren's Q&A threads, etc.

P.S. Some day I will learn the skill of writing a short post.  Some day.
Well said!

Is this applicable to a Year 10 student? I've been trying to do what you've stated above and tried using their language used within their essays but I find that I'm trying to drastically write an essay that I'm capable right now. I also find that I can't fully understand interpret VCE study guides and then applying the knowledge becomes essentially impossible.

Additionally, I have an english exam in a week and it's a text response. I've decided to prepare earlier than my class since I don't think I'll be prepared if I go at my teachers pace. I've found a T.R structure Essay Formula- Text Response and will probably just follow this until the end of VCE. Good idea?

I've also lost my trust in my teacher and now find myself disregarding her information about essay structure as I think she's just trying to teach us how to write like a normal Year 10 student. Although, this could obviously work against me, should I just listen to her?
Title: Re: VCE English Question Thread
Post by: heids on May 20, 2015, 05:56:17 pm
Well said!

Is this applicable to a Year 10 student? I've been trying to do what you've stated above and tried using their language used within their essays but I find that I'm trying to drastically write an essay that I'm capable right now. I also find that I can't fully understand interpret VCE study guides and then applying the knowledge becomes essentially impossible.

Additionally, I have an english exam in a week and it's a text response. I've decided to prepare earlier than my class since I don't think I'll be prepared if I go at my teachers pace. I've found a T.R structure Essay Formula- Text Response and will probably just follow this until the end of VCE. Good idea?

I've also lost my trust in my teacher and now find myself disregarding her information about essay structure as I think she's just trying to teach us how to write like a normal Year 10 student. Although, this could obviously work against me, should I just listen to her?

This was intended for year 12 not year 10, but it's always an important thing, to take control of your own learning :)  However, you don't have to be at year 12 level until guess when... year 12.  Until then, you should be building up a solid foundation of smaller skills, rather than trying to jump beyond yourself immediately.  Don't try and be impressive before you've learnt the basics, this is what I did because I couldn't bear to turn out something boring/shallow/substandard.  It doesn't work.

These could include:
- Read more.  In general, i.e. I'm not telling you to read a million year 12 English essays.
- Vocab, expression, grammar etc.  Build up a vocab you're totally familiar with - i.e. you know exactly where it should be used and the shades of meaning.  Get teachers to criticise clunky or unclear expression.
- Writing with conciseness.  Learn to express yourself clearly and neatly; an exercise I found fun was writing a normal essay, recording the wordcount, and then editing it, trying to get the word count down as low as humanly possible while still retaining the same ideas.  You get to this mindset of 'come on, just onnneee word more...' and you learn lots of tricks for expressing things more concisely.
- Learn to embed quotes nicely. (1-6 words only).
- Learn effective methods of learning quotes, annotating a text, and researching and generating ideas.
- Analysis.  Make your stuff less descriptive and more meaty/insightful.  Develop skills with analysing quotes, characters, themes, literary techniques, and authorial values.
- Develop good time management.

I'm sure that formula is fine, but there's no set-in-concrete way.  Be willing to be flexible.  In the future, you'll learn a lot from reading other essays with different structures.  You can do whatever is best for you as long as it covers all aspects of the topic and involves deep analysis.  But for now, I'd stick to whatever structure your teacher gives.  Instead, make your essay stand out through deeper analysis (e.g. analyse literary techniques/structures, the author's values and message, and make more insightful discussion round themes and characters).

So, 'when in Year 10, do as the Year 10s do.'  There's no specific 'Year 12 Structure', and you're probably just learning a 'yes-yes-no' formula with TEEL in each paragraph.  That's fine.  It's not like that's a "wrong" structure and you'll need to do a sudden snap change when you hit year 12; you'll probably gradually develop your own more complex style of dealing with topics over time :).

Hope I kinda answered your questions, I don't think this really did everything you asked ::)
... anyway, if I'd had your perspective back in year 9-10, my year 12 life might have been a bit different.
Title: Re: VCE English Question Thread
Post by: Apink! on May 21, 2015, 08:49:00 am
Hi everyone!

I’m in some serious, deep black hole right now that it sucking my life out. I am completely pathetically lost. I can’t develop paragraphs that will lead to the main contention I am arguing on. I don’t even realise what is wrong. I would really appreciate  it if you could give me a hand. I really wanted to succeed in English but it’s not as easy as I thought… btw, I’m trying to write an expository essay for Context.

So, this is a prompt I was working on which made me have a mental breakdown
Shared experience does not mean that people see things the same way.{VCAA 2011}

My contention: “Experience[ing]” and “[see]ing” things are two different ideas. Experiencing implies a passive flow of events that have come across a person whereas “see[ing]” is an active process in which we apply our filtered ideas and values to view our experiences.  Although people may across same situations in life and experience “shared” memories, it is inevitable that we as human beings cannot remember the same events in the identical manner because of this.


First paragraph:
T.S: ?
Second paragraph
T.S: ???
Third paragraph:
T.S:  *insert confused look here*

What can include in my first, second and third paragraphs so that my essay “builds up” to my contention and not just have each paragraph assigned for one evidence each to reinforce my contention? (I’m really lost on how I can do this, and an example might really clear this up)

Also what could I talk about with the contention I have?

I used to be those students who would designate main paragraph 1,2 and 3 just finding difference evidence all leading to the same idea which led to my contention. Now I find myself unconsciously doing this (even though I don’t  want to) and it’s really difficult to get it out of my system.  Even though I know what is required of me, I don’t know the how part and I would really appreciate your help!

Also I realised that one of my most problematic area in context is not having a contention or not really basing my essay on a contention. I do “explore” but without a contention which will probs guarantee a pretty bad mark. For example if I had this prompt:  “powerful control reality”

Paragraph 1: even though a powerful figure may “influence” reality, they cannot “control” it because there are realities in which others cannot change

Paragraph 2: Sometimes it is not even clear who is more “powerful”. Often, people control each other’s realities and it’s not necessarily influenced only by those who the society views as “powerful”

Paragraph 3:  Powerful people change people’s realities all the time! Hey just look at the government and stuff…

So I have ideas and I “explore” them. But as you can see it doesn’t lead to a CONTENTION ):(or at least I don’t think so) unless… unless… these three points all somehow magically combine together leading to a jaw-dropping contention which I didn’t know about because I’m just subconsciously an English genius (no).  So what my question is what am I doing wrong and how can I fix it?

I’m sorry for asking you this question, because I should know but I don’t… ): ):

p.s I finally understood weaving today! So thanks for that (:
p.p.s You probably realised I liked using p.s. btw I am sincerely sorry for asking so many questions ):
Title: Re: VCE English Question Thread
Post by: Apink! on May 21, 2015, 07:06:44 pm
Bump! :P
Title: Re: VCE English Question Thread
Post by: vanessa14 on May 23, 2015, 09:35:43 pm
Does anyone have a structure for persuasive writing for context? I have no idea how to write a persuasive essay, and it seems like I've forgotten everything to do with context because I don't know where to start.

And for an imaginative/creative piece, how do we relate it to the main text we discussed in class? Does it have to relate to it, and be of a characters point of view, or can we just write a creative story relating to the prompt alone?
Title: Re: VCE English Question Thread
Post by: Coffee on May 23, 2015, 11:01:38 pm
Does anyone have a structure for persuasive writing for context? I have no idea how to write a persuasive essay, and it seems like I've forgotten everything to do with context because I don't know where to start.
The structure of a persuasive piece will depend largely on what form you choose to write in (e.g. speech, opinion piece, etc.) However, you will be required to create and build upon an argument so it should contain the following:

Introduction:

Body Paragraphs:

Rebuttal:

Conclusion:

Remember to also use your persuasive language techniques!

And for an imaginative/creative piece, how do we relate it to the main text we discussed in class? Does it have to relate to it, and be of a characters point of view, or can we just write a creative story relating to the prompt alone?
For a creative piece, such as a short story, you should be drawing on your selected text in some capacity. For example, if you were studying Identity & Belonging and your selected text was 'Skin' you might choose to draw on this through the use of symbolism (e.g. doll, white/black, segregation, etc.) However, you should never explicitly state your text (e.g ...like in Skin where...) but rather represent the 'big ideas' overtly - make it obvious that your story relates to the context and the ideas studied in your selected text.

In relation to what you can write about, it's pretty open. You might choose to write an additional scene or rewrite a scene or write an alternative ending. You might choose to write through the eyes of another character or object. (Apparently a girl wrote a creative piece on 'Skin' through the eyes of Sandra's doll - it was very well received too!) Alternatively, you may choose to write on your own concept or idea while exploring the themes represented in your context (eg. For 'Skin' you might write a narrative piece about two chess pieces, perhaps a king and queen, who want to be together but are on opposite sides) - It's quite clear how this relates to 'Skin'!
Title: Re: VCE English Question Thread
Post by: vanessa14 on May 24, 2015, 03:15:18 pm
Thank you so much! That was really helpful!

Remember to also use your persuasive language techniques!

Is there a specific way to include my techniques? I've only used a rhetorical question so far :'( ... Can you give me a quick example of any other techniques I could include and how to do it?
Title: Re: VCE English Question Thread
Post by: TheAspiringDoc on May 24, 2015, 03:21:53 pm
Thank you so much! That was really helpful!

Is there a specific way to include my techniques? I've only used a rhetorical question so far :'( ... Can you give me a quick example of any other techniques I could include and how to do it?
emotive language, repitition, facts, opinions, quotes, triple construction, metaphor, real life examples and placing people in the situation (sort of like retorical questions?)
I dunno though, there's probably more..
Title: Re: VCE English Question Thread
Post by: Coffee on May 24, 2015, 04:12:32 pm
Is there a specific way to include my techniques? I've only used a rhetorical question so far :'( ... Can you give me a quick example of any other techniques I could include and how to do it?

Here's a list of persuasive language techniques with examples: http://www.vcestudyguides.com/wp-content/uploads/downloads/2010/09/Persuasive-Techniques-Insight-Outcomes1.pdf

In terms of how to incorporate these PLTs into your piece, think back to the skills you have learnt in language analysis and your oral. If you're writing an opinion piece, editorial or speech; these skills are easily translatable. The only difference is rather than analysing the piece, you're writing it. In order to help generate ideas ask yourself:


Remember the purpose of a persuasive piece is to convince the reader that your point of view is correct.
Title: Re: VCE English Question Thread
Post by: biy on May 24, 2015, 09:17:48 pm
Hey everyone

Where/when/how do i start with my oral presentation? I have a topic chosen but have no idea where to start, and how.. :(
Title: Re: VCE English Question Thread
Post by: HighTide on May 24, 2015, 09:29:21 pm
Hey everyone

Where/when/how do i start with my oral presentation? I have a topic chosen but have no idea where to start, and how.. :(
I'm not sure if you meant how to begin the presentation or begin preparation. Anyway...
I take it you have also adopted a persona. So, begin by establishing your persona, the setting, the audience, the issue and your contention.
This was mine this year:

Good afternoon ladies and gentlemen. As the economic analyst for the Federal Department of Infrastructure and Regional Development, I thank you, Premier Daniel Andrews, for allowing me this opportunity to address the Victorian Cabinet regarding the East-West Link contract.
I hope to convince you that the East-West Link is an essential piece of infrastructure which must go ahead. I will start by reminding Premier Andrews that despite choosing to present in this way, he was not elected with a clear mandate to abandon the East-West Link but instead, was voted in, due in a considerable part, to preferential voting.

If you meant, where to begin... then try researching your topic. Get more arguments than you need and choose the strongest in a way that it still flows and covers the issue. Also ensure it meets the rubric (if your school gave you one).


Title: Re: VCE English Question Thread
Post by: Adequace on May 25, 2015, 12:47:11 pm
Hey,

I have a text reponse exam next week and I'm confused since my teacher says I have to address this hidden question no matter what. "How does the author create meaning", do I address this in all of my paragraphs including the introduction or do I only have to mention it once?

Should each of my body paragraphs be saying how the author uses characters and their actions to imply a message or can I talk about a character or theme specifically?

My teacher said to sign post my arguments in my introduction, but obviously at VCE level I shouldn't, or do it so obviously? Do I have to mention what I'll be saying in each paragraph in my intro, or can I just start off with a broad idea in my intros and then focus on specific events in each of my body paragraphs?
Title: Re: VCE English Question Thread
Post by: heids on May 25, 2015, 04:49:18 pm
@Apink!, I didn't answer because it looks kinda silly to state, 'I don't know' :P

I have a text reponse exam next week and I'm confused since my teacher says I have to address this hidden question no matter what. "How does the author create meaning", do I address this in all of my paragraphs including the introduction or do I only have to mention it once?

Should each of my body paragraphs be saying how the author uses characters and their actions to imply a message or can I talk about a character or theme specifically?
That's a weird way to put it... essentially, it just means that you've got to keep referring to the author and how they are intentionally doing things.  They intentionally structure the book the way they do, intentionally choose language, intentionally set up characters in certain ways (e.g. they might make a poor person very likeable, heroic and strong while a rich person is harsh and horrible, to challenge the upper classes' focus on money) etc etc.  They also probably have messages to that society or humankind in general.  Basically you've just got to be always showing awareness that the author is the driving force behind everything, and they're carefully creating their text to convey their views/values to the audience as powerfully as possible.  Though I bet the authors didn't mean 1/100 of the things that students attribute to them :P. Read this post.

So it's not a requirement of 'PUT THIS ONCE IN THE INTRO AND ONCE IN EACH PARAGRAPH', it's more changing your overall way of writing to constantly acknowledge authorial intent and reference the messages the author is trying to convey.

Quote
My teacher said to sign post my arguments in my introduction, but obviously at VCE level I shouldn't, or do it so obviously? Do I have to mention what I'll be saying in each paragraph in my intro, or can I just start off with a broad idea in my intros and then focus on specific events in each of my body paragraphs?
Huh!?  I know I wasn't great at English, but I signposted paragraphs' arguments in the intro...  Obviously that doesn't mean writing bland formulaic 'firstly, secondly, finally' statements in the intro and then rehashing them at the start and end of your body paras and in the conclusion; you should mix it up a bit more... but I don't think there's anything terribly wrong with it, I think you'll find yourself getting more and more subtle as you go so don't stress yet.

I feel like Lauren's mentioned that she wrote pretty vague intros without too much signposting because the ideas just flowed from her mind as she went, but that's just coz she's Lauren ::) :P.  Check here for her intro hints.

And note that paragraphs shouldn't be centred so much round EVENTS as IDEAS, read this post for a couple of hints. The events support the ideas.

EDIT: hoping I haven't said anything too badly wrong here :P
Title: Re: VCE English Question Thread
Post by: Adequace on May 25, 2015, 05:07:35 pm
@Apink!, I didn't answer because it looks kinda silly to state, 'I don't know' :P
That's a weird way to put it... essentially, it just means that you've got to keep referring to the author and how they are intentionally doing things.  They intentionally structure the book the way they do, intentionally choose language, intentionally set up characters in certain ways (e.g. they might make a poor person very likeable, heroic and strong while a rich person is harsh and horrible, to challenge the upper classes' focus on money) etc etc.  They also probably have messages to that society or humankind in general.  Basically you've just got to be always showing awareness that the author is the driving force behind everything, and they're carefully creating their text to convey their views/values to the audience as powerfully as possible.  Though I bet the authors didn't mean 1/100 of the things that students attribute to them :P. Read this post.

So it's not a requirement of 'PUT THIS ONCE IN THE INTRO AND ONCE IN EACH PARAGRAPH', it's more changing your overall way of writing to constantly acknowledge authorial intent and reference the messages the author is trying to convey.
Huh!?  I know I wasn't great at English, but I signposted paragraphs' arguments in the intro...  Obviously that doesn't mean writing bland formulaic 'firstly, secondly, finally' statements in the intro and then rehashing them at the start and end of your body paras and in the conclusion; you should mix it up a bit more... but I don't think there's anything terribly wrong with it, I think you'll find yourself getting more and more subtle as you go so don't stress yet.

I feel like Lauren's mentioned that she wrote pretty vague intros without too much signposting because the ideas just flowed from her mind as she went, but that's just coz she's Lauren ::) :P.  Check here for her intro hints.

And note that paragraphs shouldn't be centred so much round EVENTS as IDEAS, read this post for a couple of hints. The events support the ideas.
Thanks for clearing up my misconceptions.
Title: Re: VCE English Question Thread
Post by: biy on May 25, 2015, 05:58:55 pm
Hi guys

Im having troubles with how to write up language analysis paragraphs :( I know what to write, but I dont know how to write it
Title: Re: VCE English Question Thread
Post by: Coffee on May 25, 2015, 06:07:13 pm
Hi guys

Im having troubles with how to write up language analysis paragraphs :(

What are you caught up on? Can you be more specific?
Title: Re: VCE English Question Thread
Post by: biy on May 25, 2015, 07:06:21 pm
What are you caught up on? Can you be more specific?

Sorry about that :3

Well I am in the process of writing one up now. We have to compare multiple articles with each other. I have never done this before so I do not know how to structure my paragraphs, and what do I say in the introduction and conclusion?

Thank you coffee (nice name haha :P)
Title: Re: VCE English Question Thread
Post by: literally lauren on May 25, 2015, 09:54:50 pm
Hey,

I have a text reponse exam next week and I'm confused since my teacher says I have to address this hidden question no matter what. "How does the author create meaning", do I address this in all of my paragraphs including the introduction or do I only have to mention it once?

Should each of my body paragraphs be saying how the author uses characters and their actions to imply a message or can I talk about a character or theme specifically?

My teacher said to sign post my arguments in my introduction, but obviously at VCE level I shouldn't, or do it so obviously? Do I have to mention what I'll be saying in each paragraph in my intro, or can I just start off with a broad idea in my intros and then focus on specific events in each of my body paragraphs?
From the sounds of things, your teacher seems to be pushing the 'text as a construct' angle that's explained in the post bangali linked above
(Incidentally I was just about to write an explanation of that until I clicked and realised I already had :p ty!)
Essentially it just means the focus is on the text and not what happens in the text if that makes sense? It's not really a matter of consciously telling yourself to stop and write a painful sentence like 'Therefore this contributes to the author's overall message that >blah<' every paragraph; it's something that should be woven throughout your discussion.
Broadly speaking, the starts of your paragraphs can involve a 'views and values statement' about the author's intent; the bulk of your paragraphs should avoid it; and the last few sentences in your paragraph should definitely include some. The good message-analyses tend to need a bit of discussion building up to the overall point, ie. you can't just go into a sentence, guns blazing, and think 'right! I'm going to say something about the author's deepest darkest values.' Imagine if your body paragraph was that jarring:

In 'Goings on at Number Thirty-Two,' Paddington Bear awakes from his nightmare about being unable to open a jar of marmelade which can be seen as the author's attempt to distill a sense of simultaneous relief at the dream being over, and fear for what it may portend.

Not only is the first part of that section just pure summary, but the conclusion it draws is a little shaky. I haven't said anything about how the author creates a sense of relief/fear, or what it is about the chapter that contributes to this impression. That's something I've just left to the reader to infer - and examiners tend to take a 'snooze ya'lose' approach to this sort of thing. Students who are content to just mash together a point from the text with a point of analysis/V&V without making the reason for that link clear... well, they won't lose marks exactly, but they certainly won't score as highly as someone who justifies their thinking.

Well I am in the process of writing one up now. We have to compare multiple articles with each other. I have never done this before so I do not know how to structure my paragraphs, and what do I say in the introduction and conclusion?

Thank you coffee (nice name haha :P)

Essay structure-wise:

Intro:
- mention the titles, authors, and contentions of each piece (pretty much compulsory)
- dates/publication details are optional, but some teachers prefer them, and they're easy to include
- a brief statement about the issue or 'spark' is usually good to start with
- audience or tone can be mentioned, but it's more important that you bring them up in your body paragraphs for analysis
- don't mention specific techniques
- don't quote anything

Body paragraphs:
- you don't have to include every text in every paragraph, but every paragraph should include more than one text (my rule; not an official one, but highly recommended)
   eg. P1: Article 1 + Article 2
         P2: Article 2 + Article 3 + visual
         P3: Article 1 + Article 3 + visual
- language is always the focus, but think about how you might contrast the arguments in the different pieces. Where do they differ? Remember, differences are better to comment on than similarities. --> because if things are different, you get to discuss both of them (eg. Where Author A suggests that the decision was misguided and foolish, Author B instead contends that it was the best possible option...) whereas saying 'these two contentions are the same' doesn't give you much else to say. Also, schools tend to pick significantly different articles to give you more to discuss anyway :)
- Never compare at the expense of analysing. You don't have to be constantly flipping back and forth between articles. Discuss one for a little while (~100 words or so) then transition when you feel you've said what you need to.

Concl.
- revisit contentions if needed, and try to say something about how language has been used overall
- don't analyse anything new
- don't rank the articles (ie. 'this article is more persuasive because of its use of statistical evidence') That's worth nada.
- ending by saying something about the audience is a popular option, though other things can work too.

And as always, because this is marked internally, your teacher will likely have their own expectations or at least recommendations, so it's worth having a chat with them if you're still unsure :)
Title: Re: VCE English Question Thread
Post by: Chang Feng on May 25, 2015, 10:15:17 pm
How do you make an expository essay less text response like. Like how is an expository essay different from a text response. Also how would you go about making an expository less dry, and more interesting to the reader so that they are captivated by it. (cause apparently my expository essays are to text response like, but not sure how to change it and also a bit dry.
THanks for the advice. 
Title: Re: VCE English Question Thread
Post by: Callum@1373 on May 27, 2015, 06:15:33 pm
For the play macbeth, do I refer to the audience as 'viewers', 'playgoers', 'audience' or something else?
Title: Re: VCE English Question Thread
Post by: vanessa14 on May 27, 2015, 08:45:32 pm
Just a quick question  ::) For Context, how do I explain a quote I've chosen that relates to my prompt as an example?... I know I have to mention the author's name but is there a specific amount about what I have to say about the author or do I just mention his/her name and write the quote and then explain how it relates to the prompt?

And is a single quote enough for one paragraph? (I know that examples arent meant to take up the whole paragraph)

Any ideas?

Oh and if it helps, im writing a persuasive speech!  :)
Title: Re: VCE English Question Thread
Post by: Alter on May 27, 2015, 10:12:24 pm
Also a very straightforward question (though quite urgent)...

I have my English Oral SAC tomorrow. I feel very prepared as I've been refining and practising my speech since the beginning of the term. As a result, I can recite my entire piece without fault (i.e. I've memorised it quite well) to the extent that I can perform it without any cue cards/notes at all. With this being said, I would like to know which would be preferable of two possibilities: should I perform my speech without any aids at all, or should I perform it with cue cards in my hand without necessarily referring to them? I understand this might seem like a ridiculously stupid question, but could the skill of being able to interact/articulate your ideas without any aid make it seem to the assessor as if you're of a higher calibre, or does it make your presentation seem arrogant/disorganised?

The reason I ask it is because, while I would've gone with cards in hand to the actual speech to begin with, I was present at a VCAA Plain English speaking competition today. The winner was commended on his prepared speech above all, particularly for the fact that he did so without any notes. Is this a subjective assessment that the assessor will make, it is objectively better to be able to perform in this manner if you have very good interaction abilities when speaking? Thanks in advance and sorry for the unusually niche nature of this question.
Title: Re: VCE English Question Thread
Post by: literally lauren on May 27, 2015, 10:47:15 pm
How do you make an expository essay less text response like. Like how is an expository essay different from a text response. Also how would you go about making an expository less dry, and more interesting to the reader so that they are captivated by it. (cause apparently my expository essays are to text response like, but not sure how to change it and also a bit dry.
THanks for the advice.

A Context piece uses a text for evidence in order to say something about the Context itself. In Text Response, you're given a prompt that relates only to your chosen text, and told to discuss the ideas within that text.
So Context has no limits, because you're talking about the entirety of the Imaginative Landscape/ Whose Reality/ Conflict/ Identity and Belonging. But Text Response is just about the text itself, and you can't just go off on tangents about other themes or ideas.

For instance, the sentence: 'The desire for revenge is a fundamental part of human nature, even though we don't often like to admit it' would be perfectly fine in a Context essay, because it's dealing with a general concept. However, you couldn't use this in T.R. because it's not telling us anything about the book/film/play etc.

By contrast, something like: 'This character's quest for revenge can be seen as emblematic of his flawed, but relatable nature' would be fine in both Context and T.R. The difference here is that, after this sentence, a T.R. essay would stick with the character and link the discussion to the overall message of the book - while a Context piece would start to zoom out from the text and try to say something about IL/WR/C/Id&b.

In terms of 'spicing up' essays and making them more interesting, there's some general stuff here and here that might help get you started, but ultimately it just depends on your writing style and how 'creative' you want to get.

For the play macbeth, do I refer to the audience as 'viewers', 'playgoers', 'audience' or something else?
'Audience' is the conventionally accepted term, with 'playwright' (preferred) or 'author' being used for Shakespeare. You could use 'playgoers'/'theatre-goers' if you wanted, but only in moderation. This'd more likely be the case if you were saying something about Jacobean society at the time, and how those specific theatre-goers in Shakespeare's day would've responded to something; in contrast to how 'audience' members (read: anyone who is viewing the play) could judge a certain character or infer a certain idea.

Just a quick question  ::) For Context, how do I explain a quote I've chosen that relates to my prompt as an example?... I know I have to mention the author's name but is there a specific amount about what I have to say about the author or do I just mention his/her name and write the quote and then explain how it relates to the prompt?

And is a single quote enough for one paragraph? (I know that examples arent meant to take up the whole paragraph)

Any ideas?

Oh and if it helps, im writing a persuasive speech!  :)
It depends how well known the person is. If you're quoting, say, John Lennon, then you probably don't have to clarify which John Lennon you're talking about, so specifications like 'Renowned British pop singer, and counter-culture activist John Lennon who was born on...' is going to sound superfluous. However, a brief adjective or two can be a good way to sum up a certain person; eg.
In her monumental address to the United Nations summit, then political prisoner Aung San Suu Kyi said "quote"
The same can be done for lesser known figures, or even completely random people you found by searching 'quotes about conflict/reality/identity.' We've all done it :P
So long as it doesn't detract from the flow of your speech, you should be fine.

As to the ratio of quotes to examples/explanation, I'm afraid your teacher will have to answer that because it's very subjective. Technically you don't need any quotes at all in Context - you just need 'evidence' of some kind. The most important thing is that your essay is able to find a balance between Context-based discussion, and example-based discussion - how exactly you do this is totally up to you.

I've heard some teachers say things like 'you have to have three or more quotes in every paragraph' while others say 'never use quotes; they're clunky and awful' so definitely check with your teacher to see if they have a hard-and-fast-rule either way. If you're lucky, they'll be cool with whatever you choose, and so it'll be up to you to determine whether your speech feels balanced or not.
ie. does it seem like
a) there are two many quotes and you can't explore them in enough depth, meaning the rest of the speech is compromised because you don't have enough time to talk about the bigger ideas?
b) there aren't enough quotes, so you end up stretching out the ones you do have to the point where you're reading too much into them, or they become sort of over-worked - OR - there aren't enough quotes and the whole thing feels like a stack of huge, lofty ideas with nothing supporting it underneath
or c) none of the above, so pretty much fine :)

Also a very straightforward question...

I have my English Oral SAC tomorrow. I feel very prepared as I've been refining and practising my speech since the beginning of the term. As a result, I can recite my entire piece without fault (i.e. I've memorised it quite well) to the extent that I can perform it without any cue cards/notes at all. With this being said, I would like to know which would be preferable of two possibilities: should I perform my speech without any aids at all, or should I perform it with cue cards in my hand without necessarily referring to them? I understand this might seem like a ridiculously stupid question, but could the skill of being able to interact/articulate your ideas without any aid make it seem to the assessor as if you're of a higher calibre, or does it make your presentation seem arrogant/disorganised?

The reason I ask it is because, while I would've gone with cards in hand to the actual speech to begin with, I was present at a VCAA Plain English speaking competition today. The winner was commended on his prepared speech above all, particularly for the fact that he did so without any notes. Is this a subjective assessment that the assessor will make, it is objectively better to be able to perform in this manner if you have very good interaction abilities when speaking? Thanks in advance and sorry for the unusually niche nature of this question.
I'm of the opinion that having cue cards in front of you is fairly sensible, even if you feel like you've memorised everything entirely... it's a good security blanket in case your brain just flies out the window when you need it most. Also, for someone who never knew what to do with her hands whilst giving a speech unless I had something to hold, it can kind of detract from the awkwardness in a weird psychological-boundary-between-you-and-the-audience sort of way.
But, assuming public speaking isn't too terrifying a prospect for you, then you could go either way. You definitely won't be penalised for having cue cards (unless your teachers are especially cruel, but I've never heard of this happening) and whilst you would get some credit for being able to speak confidently without them, ultimately the marks come down to how good your content is and how well you present it. Things like body language and tone of voice are way mroe influencial factors in the marking scheme than whether you have the speech in front of you.

Ultimately it comes down to what kind of public speaker you are. If your speech is suited to a more personal, perhaps even borderline colloquial delivery where you talk directly to the audience (eg. 'Imagine you were in this situation...' or 'How can we abide this?') then a lack of cards might work in your favour. Whereas, if it's a very formal speech and you're suited to a very formal delivery style, then having your speech there is probably best.

Incidentally, it's kind of like how newsreaders today still have sheets of paper on their desks and occasionally shuffle them around a bit, even though they're almost completely reliant on teleprompters and ear-feeds. It's partly in case everything goes wrong and they need to rely on the hardcopy, but it's mostly about the way they deliver and transition between points.

If you're still undecided, wait and see what others in your class do tomorrow. If it seems like everyone's using cue cards, then maybe just stick with the system, but if there are a few solid speeches delivered without them, then by all means test that memorisation-prowess!

Good luck!
Title: Re: VCE English Question Thread
Post by: vanessa14 on May 27, 2015, 11:11:29 pm

It depends how well known the person is. If you're quoting, say, John Lennon, then you probably don't have to clarify which John Lennon you're talking about, so specifications like 'Renowned British pop singer, and counter-culture activist John Lennon who was born on...' is going to sound superfluous. However, a brief adjective or two can be a good way to sum up a certain person; eg.
In her monumental address to the United Nations summit, then political prisoner Aung San Suu Kyi said "quote"
The same can be done for lesser known figures, or even completely random people you found by searching 'quotes about conflict/reality/identity.' We've all done it :P
So long as it doesn't detract from the flow of your speech, you should be fine.

As to the ratio of quotes to examples/explanation, I'm afraid your teacher will have to answer that because it's very subjective. Technically you don't need any quotes at all in Context - you just need 'evidence' of some kind. The most important thing is that your essay is able to find a balance between Context-based discussion, and example-based discussion - how exactly you do this is totally up to you.

I've heard some teachers say things like 'you have to have three or more quotes in every paragraph' while others say 'never use quotes; they're clunky and awful' so definitely check with your teacher to see if they have a hard-and-fast-rule either way. If you're lucky, they'll be cool with whatever you choose, and so it'll be up to you to determine whether your speech feels balanced or not.
ie. does it seem like
a) there are two many quotes and you can't explore them in enough depth, meaning the rest of the speech is compromised because you don't have enough time to talk about the bigger ideas?
b) there aren't enough quotes, so you end up stretching out the ones you do have to the point where you're reading too much into them, or they become sort of over-worked - OR - there aren't enough quotes and the whole thing feels like a stack of huge, lofty ideas with nothing supporting it underneath
or c) none of the above, so pretty much fine :)

thank you so much! you're awesome honestly! :)
Title: Re: VCE English Question Thread
Post by: Alter on May 28, 2015, 07:53:32 pm
Whew, my SAC went relatively well. I ended up deciding to do my speech without any cue cards/notes and I didn't go blank/make any crucial mistakes, so I'm glad I made the decision in retrospect. I didn't have a chance to see if anyone else would present it like this before I did it (I was the first up of my group of ~8), but of the rest I've seen today (20ish) nobody else did this. Hopefully it can help separate it me from others who relied a ton on prepared material. Thanks a bunch :)
Title: Re: VCE English Question Thread
Post by: InNeedForHelp on May 28, 2015, 08:57:32 pm
I'm wondering if I should remember ideas of our studied text before heading in to an exam or is there something that just sparks your memory in the exam? It seems that I don't fully retain what I read on a less so obvious important part of the novel which may be an issue since I want to write an original essay.

Also, should I be learning the context of the quotes I'll be using in my essay? I found some good quotes on the Internet but have a vague understanding of the context but I do think I can link the quote up to a major theme.
Title: Re: VCE English Question Thread
Post by: Coffee on May 29, 2015, 07:14:45 pm
I'm wondering if I should remember ideas of our studied text before heading in to an exam or is there something that just sparks your memory in the exam? It seems that I don't fully retain what I read on a less so obvious important part of the novel which may be an issue since I want to write an original essay.

Also, should I be learning the context of the quotes I'll be using in my essay? I found some good quotes on the Internet but have a vague understanding of the context but I do think I can link the quote up to a major theme.

You should definitely be prepared when going into an exam so you should have a thorough understanding of the ideas and themes presented in the text. If you're struggling to retain information it's important you're employing good study habits. What works will obviously vary from person to person but you should be revising regularly. Making summary sheets based on characters, plot, themes, etc can be very helpful as are completing practise questions/essays. I personally find mind maps to be very helpful in displaying connections between characters/themes as I can visualise it in the exam. But experiment and see what works for you! Also, it is important you understand the context of the quotations so you're using them appropriately.
Title: Re: VCE English Question Thread
Post by: cosine on May 30, 2015, 06:45:02 pm
Guys, stressing over my oral :(

Alcohol advertisements should be banned from being publically viewed. But i have absolutely no ideas/arguments :( Someone pls help me, my oral is in 3 days!
Title: Re: VCE English Question Thread
Post by: HighTide on May 30, 2015, 07:32:20 pm
Guys, stressing over my oral :(

Alcohol advertisements should be banned from being publically viewed. But i have absolutely no ideas/arguments :( Someone pls help me, my oral is in 3 days!
Hi cosine,
Currently, the use of alcohol advertisements entices younger generations. As such you can look preventing the broadcasting of a negative message by the use of alcohol advertisements. This will probably be the key message but it can have a flow-on effect.
Example, the government and TAC are strongly condemning drink driving. Currently, alcohol advertising in public would hinder their attempts to discourage such behavior.
You can speak of building a better, alcohol-free life for the younger generations. To speak of this you can refer to role models in sport etc. For example, Would you show advertise alcohol publicly at a sports event? I.e. People disapproved of Shane Warne's actions following the 2015 World Cup where he asked many questions relating to alcohol...

Overall, the main idea is building a better future, and to do that you can paint a positive picture of the benefits of reducing alcohol consumption in youth. In other words, CONSIDER THE RIPPLE EFFECT.

Hope it helps.  :)
Title: Re: VCE English Question Thread
Post by: Floatzel98 on May 31, 2015, 08:22:32 pm
I have a couple of articles that i need to compare which are mainly comprised of just an anecdote the whole way through, and i don't really know how this is persuasive and/or effects the reader.  I know this effect is something i should have learned in year 10/11, but i just don't really know what to write about these articles.

Thanks guys :)
Title: Re: VCE English Question Thread
Post by: Coffee on May 31, 2015, 08:58:14 pm
I have a couple of articles that i need to compare which are mainly comprised of just an anecdote the whole way through, and i don't really know how this is persuasive and/or effects the reader.  I know this effect is something i should have learned in year 10/11, but i just don't really know what to write about these articles.

Thanks guys :)

Can you identify any PLTs within the anecdote? Has the author used emotive language or hyperbole in his/her retelling of a story? What about sensationalism or irony? Evidence? Bias? Has the author praised or attacked a specific group? Take a look at this PDF http://www.vcestudyguides.com/wp-content/uploads/downloads/2010/09/Persuasive-Techniques-Insight-Outcomes1.pdf that explains PLTs and their intended effect with examples.
Title: Re: VCE English Question Thread
Post by: Floatzel98 on May 31, 2015, 09:14:06 pm
Can you identify any PLTs within the anecdote? Has the author used emotive language or hyperbole in his/her retelling of a story? What about sensationalism or irony? Evidence? Bias? Has the author praised or attacked a specific group? Take a look at this PDF http://www.vcestudyguides.com/wp-content/uploads/downloads/2010/09/Persuasive-Techniques-Insight-Outcomes1.pdf that explains PLTs and their intended effect with examples.
Okay thanks :) For one of them, i can't really identify any link to his contention in his anecdote. It does still relate to the issue though. Can i still talk about how he is propositioning  the readers to feel about the issue for when he does reveal his contention? It might not make sense without the article here, but thanks for the help anyway. I had that PDF last year and i used it all the time, so thanks for showing me that again :)
Title: Re: VCE English Question Thread
Post by: Coffee on May 31, 2015, 09:59:46 pm
Okay thanks :) For one of them, i can't really identify any link to his contention in his anecdote. It does still relate to the issue though. Can i still talk about how he is propositioning  the readers to feel about the issue for when he does reveal his contention? It might not make sense without the article here, but thanks for the help anyway. I had that PDF last year and i used it all the time, so thanks for showing me that again :)
You can discuss the use of anecdotal evidence as long as it is relevant to the author's attempt to persuade. So if the author uses anecdotal evidence but it doesn't quite relate to the overall contention, but it's essentially setting the audience up to agree; then it is an attempt to persuade the reader to agree with his/her contention. Does that kind of make sense?

It's a bit hard to know what you mean without having read the article and I feel as though I've misunderstood your question so apologies if I've gone on a complete tangent.
Title: Re: VCE English Question Thread
Post by: knightrider on June 01, 2015, 01:37:17 am
Hey literally Lauren.

Just wondering what will you be covering in the unit 4 headstart lectures for english.
Title: Re: VCE English Question Thread
Post by: literally lauren on June 01, 2015, 05:42:33 pm
Okay thanks :) For one of them, i can't really identify any link to his contention in his anecdote. It does still relate to the issue though. Can i still talk about how he is propositioning  the readers to feel about the issue for when he does reveal his contention? It might not make sense without the article here, but thanks for the help anyway. I had that PDF last year and i used it all the time, so thanks for showing me that again :)
Coffee is right, so long as you can link the technique to the contention, you're fine. Language Analysis isn't just about pointing out the devices that the author is using; in fact the examiners have recently been *subtly* reminding students that they have to do more than just state 'the author uses a rhetorical question as seen in this quote: "_____".' It's way more important that you make an attempt to talk about how this language is used to persuade. On their own, techniques aren't persuasive. So, an 'analysis' that does nothing but point out a list of techniques with no concept of the overall arguments is going to be literally pointless :)

Let's take a sample bit of mediocre analysis as an example:

In order to further his contention that the government's proposal is a bad idea, the author uses inclusive language like "we don't want this" to make readers agree with his argument and reject the proposal.

The reason this wouldn't score well is because it's just too broad. For starters:
In order to further his contention that the government's proposal is a bad 'bad' in what way? What part of the contention are we talking about here? Is the proposal inefficient? Costly? Misguided? Exploitative? Useless? You should never just use generic words like 'bad,' because no technique can support an entire contention - it might help you argue a single point, but you can't say 'the author's contention can be seen in this technique'  idea, the author uses inclusive language like "we don't want this" this might be similar to what you're talking about; yes, it's inclusive language, but there's more to say about this language (much like how there's more to say about anecdotes than just 'this is an anecdote.') For instance, what group is the author including the audience in? Why might this be important? If you know something about the audience (eg. local community, group of schoolkids, parents, etc.) then how might they, in particular, be affected by this? to make readers agree with his argument ALWAYS avoid this sort of thing. If you're ever writing something that's so plain, it could fit into any language analysis essay; YOU NEED TO BE MORE SPECIFIC! Expressions like 'thus making the readers agree with him' or 'thereby strengthening his contention and making the audience see his point of view' are worth nothing! and reject the proposal. Why??? What has this language done to bring about this result? I've stated the author includes the reader... but I haven't done anything with that fact.

Instead, something like:

The author's use of inclusive language, as seen in the phrase "we don't want this," seeks to encompass readers in his own disapproval of the proposal. Furthermore, the sharp, laconic sentence acts as a matter-of-fact assertion, intended to compel readers to agree with the idea of the proposal being unwanted and harmful to the community.

is way more likely to be looked upon favourably because it takes you through a thought process, step by step.

Don't assume your marker will do the thinking and fill in the gaps for you, because they won't. :)

Hey literally Lauren.

Just wondering what will you be covering in the unit 4 headstart lectures for english.
We're still working out the time-breakdown so I don't know how long I'll be spending on different areas (most likely 3x45 minute sessions, but idk) so at the moment it's looking like:
- One session dealing with 'sophisticated' interpretations of Section A texts and how to better your analysis (in both Sections A and C) in order to increase your understanding beyond the surface-level stuff.
- One session about what makes a good contention, incl. how to write proper topic sentences and get a reliable essay structure for all three Sections (though less of Section B since not everyone will be writing a conventional essay with topic sentences; I'll touch on it briefly since I recommend writing a few normal essays even if you do plan to write creatively/persuasively/hybrid-ly, but the focus will again be on A&C primarily)
- On session on improving expression and essay flow; making your writing 'better' regardless of what level you're at and how confident you feel. This'll be geared more so towards Sec. B since 'quality of writing' is a fairly major part of the marking scheme, but it'll have trickle-down effects for the other sections too.

I've been trying to incorporate all of the suggestions people have been sending me, and I'd welcome any more recommendations you have! There'll be time for questions and queries on the day, of course, but judging by the February lectures, I probably won't get a chance to address everyone's concerns - so the more I know beforehand, the more I can cover.

And please don't hesitate to shoot me a message if you think 'oh, it's only a problem for me' or 'she'll probably think this is a stupid question.'
Because a) if you're having trouble with something, chances are, someone else is too. And they're going to be so grateful to know that someone else brought it up and that they're not alone :p Also, you may be aware of a problem that others are experiencing without realising it - so I don't consider it 'time wasted' even if we're dealing with a seemingly small concern.
And b) I spent the first half of Year 12 English in a state of 'meh, this is fine, I'll get by' before caving and asking my teacher 'how the hell do I do any of this??' Then I just spent the next few months asking as many questions, stupid and otherwise, until I reached a point where I felt like I understood the task and how I could best meet the requirements. Self-study can get you far, but unless you know what to do and how to do it, you'll end up kicking yourself at the end of the year - trust me.
Title: Re: VCE English Question Thread
Post by: Apink! on June 02, 2015, 02:49:11 pm
If I had a prompt:
"one has little control over one's reality"
What would I discuss in each paragraph, and what would be a good contention?
I can't think of a sophisticated contention, and how to develop this contention coherently in each paragraph.
It's a context expository essay

Thank you in advance! (:
Title: Re: VCE English Question Thread
Post by: knightrider on June 02, 2015, 06:18:11 pm

We're still working out the time-breakdown so I don't know how long I'll be spending on different areas (most likely 3x45 minute sessions, but idk) so at the moment it's looking like:
- One session dealing with 'sophisticated' interpretations of Section A texts and how to better your analysis (in both Sections A and C) in order to increase your understanding beyond the surface-level stuff.
- One session about what makes a good contention, incl. how to write proper topic sentences and get a reliable essay structure for all three Sections (though less of Section B since not everyone will be writing a conventional essay with topic sentences; I'll touch on it briefly since I recommend writing a few normal essays even if you do plan to write creatively/persuasively/hybrid-ly, but the focus will again be on A&C primarily)
- On session on improving expression and essay flow; making your writing 'better' regardless of what level you're at and how confident you feel. This'll be geared more so towards Sec. B since 'quality of writing' is a fairly major part of the marking scheme, but it'll have trickle-down effects for the other sections too.

I've been trying to incorporate all of the suggestions people have been sending me, and I'd welcome any more recommendations you have! There'll be time for questions and queries on the day, of course, but judging by the February lectures, I probably won't get a chance to address everyone's concerns - so the more I know beforehand, the more I can cover.

And please don't hesitate to shoot me a message if you think 'oh, it's only a problem for me' or 'she'll probably think this is a stupid question.'
Because a) if you're having trouble with something, chances are, someone else is too. And they're going to be so grateful to know that someone else brought it up and that they're not alone :p Also, you may be aware of a problem that others are experiencing without realising it - so I don't consider it 'time wasted' even if we're dealing with a seemingly small concern.
And b) I spent the first half of Year 12 English in a state of 'meh, this is fine, I'll get by' before caving and asking my teacher 'how the hell do I do any of this??' Then I just spent the next few months asking as many questions, stupid and otherwise, until I reached a point where I felt like I understood the task and how I could best meet the requirements. Self-study can get you far, but unless you know what to do and how to do it, you'll end up kicking yourself at the end of the year - trust me.

Some wisdoms:
(http://www.dailythoughts.in/wp-content/uploads/2014/08/confucius-quotes-2.jpg)

Thanks very much Lauren  :)
Title: Re: VCE English Question Thread
Post by: Callum@1373 on June 02, 2015, 08:05:44 pm
If I had a prompt:
"one has little control over one's reality"
What would I discuss in each paragraph, and what would be a good contention?
I can't think of a sophisticated contention, and how to develop this contention coherently in each paragraph.
It's a context expository essay

Thank you in advance! (:
I guess the way you tackle the prompt is no defined way.

I think a good way to develop your contention is to start with a really basic idea say:

 - Everyone has a different reality and we jumble up our own with others.

Then advance it a bit more by saying something like:

 - Sometimes our reality however is not manipulated by others, but our own mind tends to change it

Then finish off with a stronger idea:

 - The implications of the prompt, e.g Losing touch with an objective reality can be dangerous

Hope I helped!  :)
Title: Re: VCE English Question Thread
Post by: KingDrogba on June 02, 2015, 08:54:28 pm
I am genuinely struggling to find examples of Identity and Belonging within the play 'The Summer of the Seventeenth Doll' i've read the play twice, and i need examples for an expository piece, does anyone have an ideas they are worth sharing?

I have an example to do with Roo, but struggle to discuss it.

Thank you!
Title: Re: VCE English Question Thread
Post by: Callum@1373 on June 03, 2015, 05:09:22 pm
Hey Lauren,

In regards to exploring the possibilities of a prompt, where can we go when we have that 'type' of prompt that has about 3 elements in it, e.g 'Reality is influenced by place, time and people', rather than those ones which are more of a central focus like 'Reality is an ever changing concept'

While I have put a lot of effort this year on focusing of my ability to explore the prompt and not just say I agree because a,b and c, I feel those one's with numerous elements are just forcing me to write:

 - Reality is influenced by place
 - Reality is influenced by time
 - Reality is influenced by people

How can I avoid doing this? Can I just be like stuff you teachers I'm gonna talk about place and then lead that onto another facet of the prompt, but not time or people?

Thanks  :D
Title: Re: VCE English Question Thread
Post by: thaaanyan on June 04, 2015, 09:08:44 pm
Hey Lauren,

In regards to exploring the possibilities of a prompt, where can we go when we have that 'type' of prompt that has about 3 elements in it, e.g 'Reality is influenced by place, time and people', rather than those ones which are more of a central focus like 'Reality is an ever changing concept'

While I have put a lot of effort this year on focusing of my ability to explore the prompt and not just say I agree because a,b and c, I feel those one's with numerous elements are just forcing me to write:

 - Reality is influenced by place
 - Reality is influenced by time
 - Reality is influenced by people

How can I avoid doing this? Can I just be like stuff you teachers I'm gonna talk about place and then lead that onto another facet of the prompt, but not time or people?

Thanks  :D

Hey! How's it going :)
Not Lauren but I hope I can help!
The prompt does generally provide for the easy three paragraph split which you've outlined above, and while simplistic if well executed it can still provide a powerful and captivating essay. You must address all aspects of the prompt though, no matter how you do it, so I wouldn't advise going down that hypothetical route.

Ultimately though your prompt is a springboard, it provides the basis for your discussion rather than dictating the confines within which your ideas must operate.
For example, you could contend that while, reality is a byproduct of facets of  individual's external environment (time, place, people) it can also be further influenced by our own introspection and the unique ability individuals have to develop/grow through challenging their own ideologies and pre-conceived notions of what reality means to them. What about the impact of social expectation and social norms or even the way our understanding of the world we live in is limited in accordance to the info we have of the  world?
What i'm trying to say is while you must to some degree discuss time, place and people the prompt is not limiting you from discussing other factors which influence someone's reality, or the way in which you split your ideas: e.g you may draw connections between time and place and the nature of reality in one paragraph and then in the second/ third para may focus on two different ideas regarding people.

good luck! hope this helps! :)
Title: Re: VCE English Question Thread
Post by: heids on June 04, 2015, 10:45:38 pm
Anyone ?
We'll get to it (I can try to tomorrow) but please don't bump like that after only an hour, people round here do their best to answer quickly but remember it's free help! (and I've got to go to bed -.- :P)
Title: Re: VCE English Question Thread
Post by: Vexna on June 05, 2015, 10:23:27 am
For an expository essay, do I have to actually mention the text that it is based around?

Thanks.
Title: Re: VCE English Question Thread
Post by: literally lauren on June 05, 2015, 11:44:15 am
For an expository essay, do I have to actually mention the text that it is based around?

Thanks.

Technically, no. Actually, yes.

The requirement is that you draw ideas from the text in order to aid your discussion; so long as you're dong that, you're fine. The trouble is that this criterion is stupidly vague and I doubt even VCAA could provide a concrete example of what they mean by this. So for safety's sake, I'd advise most people to use an overt link like

'Similar ideas are reflected in Barry Levinson's film 'Wag the Dog' when...' or 'As can be seen in Bertolt Brecht's play 'Life of Galileo'...'

especially for an expository essay where those sorts of links can be done smoothly, without disrupting the flow of something like a creative narrative of a speech.

If you're in the same position I was in that you don't want to write about the set text, or find other ideas waaaaaaaay more interesting, just get it out of the way really quickly. Unpack some key ideas from the text in your first paragraph, then you can move on to better territory afterwards :)
Title: Re: VCE English Question Thread
Post by: Vexna on June 05, 2015, 12:09:07 pm
Technically, no. Actually, yes.

The requirement is that you draw ideas from the text in order to aid your discussion; so long as you're dong that, you're fine. The trouble is that this criterion is stupidly vague and I doubt even VCAA could provide a concrete example of what they mean by this. So for safety's sake, I'd advise most people to use an overt link like

'Similar ideas are reflected in Barry Levinson's film 'Wag the Dog' when...' or 'As can be seen in Bertolt Brecht's play 'Life of Galileo'...'

especially for an expository essay where those sorts of links can be done smoothly, without disrupting the flow of something like a creative narrative of a speech.

If you're in the same position I was in that you don't want to write about the set text, or find other ideas waaaaaaaay more interesting, just get it out of the way really quickly. Unpack some key ideas from the text in your first paragraph, then you can move on to better territory afterwards :)


By first paragraph, do you mean the body paragraph or the introduction?  ;D
Title: Re: VCE English Question Thread
Post by: literally lauren on June 05, 2015, 01:03:29 pm
First body paragraph :)

Bur if you're doing something slightly creative, you're free to use the text in your intro as well. Many students like to adopt the voice of the author or a character and do a mini-imaginative piece for the intro and conclusion.

In 'traditional' expository pieces, the intro is just for opening up your ideas, but starting off with examples is a great way of capturing the reader's interest; it just depends what your strengths are and what you're willing to experiment with. :)
Title: Re: VCE English Question Thread
Post by: Vexna on June 05, 2015, 01:18:47 pm
First body paragraph :)

Bur if you're doing something slightly creative, you're free to use the text in your intro as well. Many students like to adopt the voice of the author or a character and do a mini-imaginative piece for the intro and conclusion.

In 'traditional' expository pieces, the intro is just for opening up your ideas, but starting off with examples is a great way of capturing the reader's interest; it just depends what your strengths are and what you're willing to experiment with. :)

Thank you very much!
Title: Re: VCE English Question Thread
Post by: Apink! on June 06, 2015, 07:38:48 am
Hi!

Do you know any real-life and current examples of Context- Whose Reality?
Also, is there any short stories or articles that are useful? I have an English exam in 3 days and I'm freaking out!
Please help me :P
Thank you :)
Title: Re: VCE English Question Thread
Post by: MathsNerd203 on June 06, 2015, 08:30:09 am
When I did my context SAC (Unit 1/2) a few weeks ago, I aced it and received full marks. I wrote it as an Imaginative/Expository TV interview, which worked really well for me.

However, on my English exam the other day, I wrote a standard expository essay, and I feel like it limited my ability to express my ideas; resulting in a poor performance.

As I progress through VCE English, should I stick with the TV interview format, or should I experiment around a little?

Also, if I were to stick with the interview/dialogue format, how should I start the piece. On my SAC I wrote a little paragraph at the start, something like:

The following is a television transcript from an interview conducted between (Person 1) and (Person 2), originally broadcast on the (Television Network) on (Current affairs program) at (Time) on the (Date):

This intro didn't feel right. Should I alter it, or just leave it out altogether?
Title: Re: VCE English Question Thread
Post by: JackSonSmith on June 06, 2015, 10:59:35 pm
In context, how much of our nominated text do we need to draw from?

I was very confused, do we need to have at least 25% of our writing to be based on our selected text?

Are a few references enough?

How much do we need to 'satisfy requirements' ?
Title: Re: VCE English Question Thread
Post by: StupidProdigy on June 09, 2015, 07:21:17 pm
When analysing and writing about the visual in language analysis, say I have 3 articles in the piece (one main one and two comments), what do i have to include? Like do I need to only write about the main article and it's relation to the image (since it would have been published to aid their argument/persuasion)? Thankyou!
Title: Re: VCE English Question Thread
Post by: HighTide on June 09, 2015, 09:16:30 pm
When I did my context SAC (Unit 1/2) a few weeks ago, I aced it and received full marks. I wrote it as an Imaginative/Expository TV interview, which worked really well for me.

However, on my English exam the other day, I wrote a standard expository essay, and I feel like it limited my ability to express my ideas; resulting in a poor performance.

As I progress through VCE English, should I stick with the TV interview format, or should I experiment around a little?

Also, if I were to stick with the interview/dialogue format, how should I start the piece. On my SAC I wrote a little paragraph at the start, something like:

The following is a television transcript from an interview conducted between (Person 1) and (Person 2), originally broadcast on the (Television Network) on (Current affairs program) at (Time) on the (Date):

This intro didn't feel right. Should I alter it, or just leave it out altogether?
Hi, I'm not overly sure about the introduction, but I would think that that is allowed as it does give the examiner insight to what you'll speak of since they don't have any prior information about it. I would personally have started mine of with dialogue in an interview transcript.
As for your other question, you are in year 11 so you have time to play around and find the writing style that suits you. However, saying that, if you're good at something in year 11, such as the imaginative transcript, I would think it would be better if you chose your style now and perfected it for year 12. By the way, considering you aced an interview transcript, I would personally practice it and stick with it till next year, as schools in year 12 rightfully tend to encourage students to pursue something other than the overused expository essays or articles.

In context, how much of our nominated text do we need to draw from?

I was very confused, do we need to have at least 25% of our writing to be based on our selected text?

Are a few references enough?

How much do we need to 'satisfy requirements' ?
It depends on your style of writing.
For imaginative pieces, you can take core-text elements, characters, themes and ideas and explore them within your piece. You don't need to make direct reference, but rather just exploring the same or similar ideas is fine.

For a persuasive or context pieces, you can make direct links to the text. However, I don't think that's a must. Instead you can also just explore the ideas. You can use quotes, history and evidence from the text.
Overall, you can't really define the extent to which they want you to refer to the text. You do have to, but as told by my own school, I think it's okay if you use indirect references, instead of saying "As seen in this book... blah blah".

When analysing and writing about the visual in language analysis, say I have 3 articles in the piece (one main one and two comments), what do i have to include? Like do I need to only write about the main article and it's relation to the image (since it would have been published to aid their argument/persuasion)? Thankyou!
You definitely have to include an analysis of the visual with respect to its accompanying article. As for the other two comments:
- If they seek to elicit the same feeling in parents, i.e. concern, then you can refer to the image. The image and the comment may have the same key argument, or position readers to feel or do something specific.
- If the comments contradict the ideas of the image, you can say "In stark contrast...." and compare. In this case, you can refer to the key arguments as they will contradict each other. Similarly, as such, the illustration and the writer of the comment would be seeking to position readers differently.
I'm not 100% sure about this, but I think you can and should compare the articles and images.
Hope this helps  :)
Title: Re: VCE English Question Thread
Post by: cosine on June 11, 2015, 09:04:20 pm
In an exam, is it wise to include quotes form the article, into the introduction of a language analysis essay?
Also how long does the english exam go for, and we need to write 3 essays right?
Title: Re: VCE English Question Thread
Post by: StupidProdigy on June 11, 2015, 09:10:21 pm
In an exam, is it wise to include quotes form the article, into the introduction of a language analysis essay?
Also how long does the english exam go for, and we need to write 3 essays right?
Don't have anything too big quoted in the intro, maybe one or two quoted words/phrases, but try to keep most stuff for the analysis.
The english exam goes for three hours and there is 15 mins reading time. Yep 3 essays, so people try to do one per hour, or one per 50mins so they have editting time or whatever time allocation works for them they use
Title: Re: VCE English Question Thread
Post by: heids on June 11, 2015, 09:10:46 pm
In an exam, is it wise to include quotes form the article, into the introduction of a language analysis essay?
Also how long does the english exam go for, and we need to write 3 essays right?
My teacher said to include a couple of quotes, I think Lauren says not to.  Take your pick :P (lol love the way I'm starting to quote Lauren at every turn).  In the end I think it's a matter of personal choice, but don't put in more than 1-2 and make sure they're integrated very smoothly.

15 mins reading time, 3 hours writing.  Yes - one TR, one context, and one LA.  Definitely check out a couple of past exams to get a feel for the layout and how it all works.

EDIT: beaten by SP :(
Title: Re: VCE English Question Thread
Post by: cosine on June 11, 2015, 09:54:21 pm
My teacher said to include a couple of quotes, I think Lauren says not to.  Take your pick :P (lol love the way I'm starting to quote Lauren at every turn).  In the end I think it's a matter of personal choice, but don't put in more than 1-2 and make sure they're integrated very smoothly.

15 mins reading time, 3 hours writing.  Yes - one TR, one context, and one LA.  Definitely check out a couple of past exams to get a feel for the layout and how it all works.

EDIT: beaten by SP :(

Thanks guys :D

How can I prepare myself for the english exam? What can I do now to ensure I can improve that when come exam time, or study periods, I don't panic (hopefully) and will feel optimistic about the exam?

Also do you recommend on doing the essay that im the weakest on during the exam first, or the one im best at?
Title: Re: VCE English Question Thread
Post by: literally lauren on June 12, 2015, 12:34:45 am
In an exam, is it wise to include quotes form the article, into the introduction of a language analysis essay?
Also how long does the english exam go for, and we need to write 3 essays right?
My logic is that the only time you should be quoting in L.A. is to analyse. If you're not analysing a quote, then it's there for summary, and summary is useless. (Note: not wrong, just useless. You don't lose marks for it, but you don't gain any either.) But you're not meant to be analysing anything in the intro, you're just introducing the piece and its contention. So if you were bringing up quotes and analysing them, then it's no longer an intro... more of a body paragraph.

And yes, Prodigy is right; the exam time averages to one essay per hour, but that doesn't mean you have to stick to the same time breakdown. If you only need 50 minutes to write a good Context piece, then you can give yourself an extra 10 minutes to deal with Language Analysis, if that was your weakness.

Thanks guys :D

How can I prepare myself for the english exam? What can I do now to ensure I can improve that when come exam time, or study periods, I don't panic (hopefully) and will feel optimistic about the exam?

Also do you recommend on doing the essay that im the weakest on during the exam first, or the one im best at?
Know what you're doing well and badly at the moment. Use teacher feedback, but also be your own critic. What do you find easiest or most difficult? And as always, be as specific as possible if you want to be efficient with your improvement.

Exam order should be either CAB or CBA (where Section A is T.R., B is Context, C is L.A.) I am yet to hear a convincing argument for any other order.
The majority of your reading time will be spent on the L.A. material, since you only have three other half-sentences to read: two T.R. prompts, of which you will choose one, and then the Context prompt. So if you spend ~13 minutes going through the L.A. material, and then write a T.R. essay first only to go back to L.A. second/third, you're essentially wasting time just transitioning between the different pieces. Get L.A. out of the way while it's all fresh in your mind; what you do after that is up to you. I usually advocate doing T.R. first since you have to use memorised quotes, whereas Context is a lot more fluid. AND if you're severely running out of time, you don't have to abide by any formal structure for Context, unlike in T.R. where you're expected to have 3 or 4 paragraphs + an intro and concl. But in the end, I went with CBA just because the Section B prompt was atrocious and I wanted to get it out of the way  ;D

Ideally you'll be able to get all three essay forms up to a stage where you feel confident enough with each of them, and then you can decide on the day which prompt you want to tackle first. But all the essays equally weighted, so there's no sense putting all your eggs in one basket :)
Title: Re: VCE English Question Thread
Post by: cosine on June 12, 2015, 07:24:49 am
My logic is that the only time you should be quoting in L.A. is to analyse. If you're not analysing a quote, then it's there for summary, and summary is useless. (Note: not wrong, just useless. You don't lose marks for it, but you don't gain any either.) But you're not meant to be analysing anything in the intro, you're just introducing the piece and its contention. So if you were bringing up quotes and analysing them, then it's no longer an intro... more of a body paragraph.

And yes, Prodigy is right; the exam time averages to one essay per hour, but that doesn't mean you have to stick to the same time breakdown. If you only need 50 minutes to write a good Context piece, then you can give yourself an extra 10 minutes to deal with Language Analysis, if that was your weakness.
Know what you're doing well and badly at the moment. Use teacher feedback, but also be your own critic. What do you find easiest or most difficult? And as always, be as specific as possible if you want to be efficient with your improvement.

Exam order should be either CAB or CBA (where Section A is T.R., B is Context, C is L.A.) I am yet to hear a convincing argument for any other order.
The majority of your reading time will be spent on the L.A. material, since you only have three other half-sentences to read: two T.R. prompts, of which you will choose one, and then the Context prompt. So if you spend ~13 minutes going through the L.A. material, and then write a T.R. essay first only to go back to L.A. second/third, you're essentially wasting time just transitioning between the different pieces. Get L.A. out of the way while it's all fresh in your mind; what you do after that is up to you. I usually advocate doing T.R. first since you have to use memorised quotes, whereas Context is a lot more fluid. AND if you're severely running out of time, you don't have to abide by any formal structure for Context, unlike in T.R. where you're expected to have 3 or 4 paragraphs + an intro and concl. But in the end, I went with CBA just because the Section B prompt was atrocious and I wanted to get it out of the way  ;D

Ideally you'll be able to get all three essay forms up to a stage where you feel confident enough with each of them, and then you can decide on the day which prompt you want to tackle first. But all the essays equally weighted, so there's no sense putting all your eggs in one basket :)

Wonderful, thanks!!

How do the assessors mark? Like say if you didnt have time to write the last essay, but like absolutely smashed the others as you devoted the 3 hours to it, would they really take off the whole 10 marks off the essay?

Also say you didn't have enough time to write up the conclusion of the last essay, what happens? In terms of marks :)
Title: Re: VCE English Question Thread
Post by: heids on June 12, 2015, 12:45:05 pm
How do the assessors mark? Like say if you didnt have time to write the last essay, but like absolutely smashed the others as you devoted the 3 hours to it, would they really take off the whole 10 marks off the essay?

Also say you didn't have enough time to write up the conclusion of the last essay, what happens? In terms of marks :)

Well, if you've written nothing, you've addressed none of the criteria for that essay - so you deserve 0 marks for that essay.  Simple.

About how long to devote to each - it depends on your situation and what works for you.  For me, I left my weakest to last and gave it very little time (~35 mins), in the hope that I'd wing it.  It meant I could maximise my marks for TR and LA, sections I was better prepared for.  If I'd spent more time on context, I wouldn't have done much better (let's face it, I was utterly hopelessly unprepared for it) and would have freaked out even more.  So while this seems ridiculous, it actually worked best for my circumstances.  Be flexible based on your needs.  Hopefully you won't let yourself get in that situation, though, so you can do approx 60-60-60.

I don't think I finished any of my pieces in the exam (actually maybe finished TR?) but got decent enough marks, better than deserved.  For a 10, finish; for a lower but still decent mark, I don't think you have to.  Better to write 4 decent paragraphs that impress the assessor and hit all the criteria, than 5 lower-quality paragraphs.

(All this is IM(ignorant)O).
Title: Re: VCE English Question Thread
Post by: chocolatecupcakes on June 15, 2015, 07:23:44 pm
How would I respond to a context prompt in an expository essay? I was told not to answer the prompt like a question (yes/no) but rather 'explore'.

For example using the 2012 conflict prompt "The experience of conflict changes people's priorities", I was told not to write my expository essay using 'the experience of conflict changes people's priorities and the experience does not change people's priorities' as my points because I'm responding to the prompt as if I'm answering a question. But how would I explore? 
Title: Re: VCE English Question Thread
Post by: MathsNerd203 on June 16, 2015, 06:47:31 pm
(I'm not sure if this is the best way of approaching the context prompt, but it works for me, so it may work for you too  ;) )

My teacher said to ask questions of the prompt, to help form 'big' ideas that relate to the set context. I will show some example questions for the prompt you mentioned: The experience of conflict changes people's priorities.

I like to use the words, who, what, when, where, why and how to ask questions.

1. Who's priorities are affected by conflict?

2. What about conflict changes people priorities?

3. When can one's experience with conflict influence their priorities?

4. Where does conflict occur that influences the priorities of people?

5. Why does conflict influence peoples' priorities?

(These are pretty rubbish questions, but I'm just using them as an example for what you should try and do)

Once you've come up with these questions, answer them, and use your answers as your big ideas for your essay. (Of course, for expository, make sure you ask questions that will allow you to fully explore all sides of the prompt, don't write a biased expository essay)

Anyway, this is my strategy, and it works well for me. I'm sure there are other strategies out there that work well for others.

I hope this has been of some assistance  :)
Title: Re: VCE English Question Thread
Post by: literally lauren on June 16, 2015, 08:30:40 pm
Wonderful, thanks!!

How do the assessors mark? Like say if you didnt have time to write the last essay, but like absolutely smashed the others as you devoted the 3 hours to it, would they really take off the whole 10 marks off the essay?

Also say you didn't have enough time to write up the conclusion of the last essay, what happens? In terms of marks :)

Your exam isn't just marked globally by one or two people. At the end of the year, you'll have (at least) six different assessors. None of them speak to each other; each essay is marked by two different people independently, then they add the two marks per essay together for all three essays to get a score out of 60.

For example:
Section A essay: {Assessor 1: 8/10} {Assessor 2: 9/10}
Section B essay: {Assessor 3: 7/10} {Assessor 4: 7/10}
Section C essay: {Assessor 5: 5/10} {Assessor 6: 9/10} --> this would be a special case. Since there is a disparity of three or more between the marks, this essay will be given to a third assessor to mark. If they said it was an 8/10, then they'd take the 8 and the 9 and add those together. But if they thought it was only a 6, then they'd take the 5 and the 6 since they're closer. In the rare event this third assessor also gives you a widely different mark, like a 2/10 or something, then your piece just goes to the Chief Assessor and he gets the final say.
So assuming that final piece got given an 8/10 by its third assessor, your overall exam score would equate to 48/60.


To answer your question, it doesn't matter how good a single essay is; if you've left a whole section blank, you can't get any marks for it. Don't think about it as 'taking 10 marks off;' you haven't done anything to earn those marks in the first place :p The assessors don't know how well you've done on your other essays - all they see is that one piece which might be brilliant or blank, and they'll mark it accordingly. I would think that if you do end up with vastly different scores for each one (eg. A:9+10/10, B:9+9/10, C:3+4/10) then the Chief Assessor might give things a look over just to make sure you weren't marked unfairly, but this is quite a frequent occurrence so they might not worry about it so much nowadays.

With regards to conclusions, they're structural requirements in T.R. and L.A. so if you leave them out, it's possible you could lose a mark, though usually only if you've done other things wrong as well (eg. vocab was mediocre, spelling was pretty bad, some parts were repetitive AND you were missing a conclusion)
Context has no formal structure, so if you're just writing a journal article or a speech or something, you don't have to have a specific paragraph that concludes your piece, but you should still be wrapping things up at the end. For the typical expository essay, not having a conclusion can sound a bit odd.

How would I respond to a context prompt in an expository essay? I was told not to answer the prompt like a question (yes/no) but rather 'explore'.

For example using the 2012 conflict prompt "The experience of conflict changes people's priorities", I was told not to write my expository essay using 'the experience of conflict changes people's priorities and the experience does not change people's priorities' as my points because I'm responding to the prompt as if I'm answering a question. But how would I explore? 

MathsNerd beat me to it, but questioning is definitely what I'd recommend too.

You never want to argue that something in Context is completely true or totally false; you can say something is often true or generally false however. Think about it - if you're reading an essay and a student writes: 'Conflict changes peoples priorities,' then it's as though they think that statement has no exceptions, which isn't necessarily the case (--> these exceptions are where we get the really interesting discussion! So what is it about a conflict or a person that determines whether or not the priorities change? Maybe we have to have an open mind, or be aware of our circumstances? Or maybe it is people who unintentionally become the victims in a conflict who are forced to alter their priorities?...) Likewise if someone says 'Conflict doesn't change priorities - ever,' you only have to think of one exception, and their whole argument falls apart. It's much safer to have a contention like 'Although conflict can bring about a change in our values, the degree to which this change affects us depends on much more...'
Title: Re: VCE English Question Thread
Post by: Apink! on June 25, 2015, 11:15:05 am
Hi Lauren (or anyone else)

I need help with this poem called "The owl and the pussycat" by Edward Lear, which is basically about a cat and an owl in love and getting married. Somehow I need to connect the poem (which is a pretty childish poem, I think) to the values of Victorian era such as marriage, courtship etc.

I'm completely blank. I was wondering if you could help me out?
Title: Re: VCE English Question Thread
Post by: Apink! on June 25, 2015, 11:25:19 am
These are the things I need to address:
- Consideration of the context of the poem: for example: era, gender or place.
*Understanding of the author’s message
*Analysis of how language is used to create meaning and the form of this particular poem
*Presentation skills: preparation, engagement of the audience, voice and eye contact
Title: Re: VCE English Question Thread
Post by: heids on June 25, 2015, 12:12:14 pm
Hi Lauren (or anyone else)

I need help with this poem called "The owl and the pussycat" by Edward Lear, which is basically about a cat and an owl in love and getting married. Somehow I need to connect the poem (which is a pretty childish poem, I think) to the values of Victorian era such as marriage, courtship etc.

I'm completely blank. I was wondering if you could help me out?

Aww, I love that poem!  (Yeah, it was written for kids, but it's so cute, right?).  Some ideas

Some ideas about the values of that era:
--> respect for each other is important in courtship
--> the point of courtship is the romance, love and joy they share, rather than a physical relationship
--> courtship and marriage were about becoming one and sharing together - unity and sharing were important, marriage wasn't there to fulfil their own individual needs, but to share together
--> courtship in that era was serious, not just a random date or two, and intended to end in marriage - they couldn't dance/hold hands/share completely until they were actually married

EDIT: didn't see your next post.  Message: emphasis on love, joy and sharing in relationships.  Just looked at the poem - this is most strongly shown in the bit
'And hand in hand, on the edge of the sand,
   They danced by the light of the moon,
             The moon,
             The moon,
They danced by the light of the moon.'

Sorry I can't give more, typing this in the middle of class when I'm kinda supposed to be reading about oedema ::)
Title: Re: VCE English Question Thread
Post by: Apink! on June 25, 2015, 12:23:17 pm
Thanks! (:
I think I have a clearer Idea now!
Title: Re: VCE English Question Thread
Post by: Apink! on June 25, 2015, 12:53:59 pm
Hi,

I have made some very good notes following your advice! Thanks!
However, I am having trouble figuring out the overall message/ contention that the poet was trying to say in this.

Could I have your help again?
Much appreciated!
Title: Re: VCE English Question Thread
Post by: heids on June 25, 2015, 02:10:59 pm
Hi,

I have made some very good notes following your advice! Thanks!
However, I am having trouble figuring out the overall message/ contention that the poet was trying to say in this.

Could I have your help again?
Much appreciated!

In reality, the author was writing a nice cute poem.  So your guess is as good as mine!
Stabbing randomly at it, I'd say something about love = unity, sharing and togetherness; love and relationships should bring joy, because of this love is worth waiting for.  Or something.  I'd draw this from the 'hand in hand on the edge of the sand they danced in the light of the moon' - like, 'hand in hand' suggests togetherness, 'danced' sounds beautiful  and joyful, 'in the light of the moon' sounds romantic, beautiful, shining etc.  Just think about how that line makes you feel!

(Can't be bothered thinking of good ways to express stuff so this is pretty basic, that's your job :P).

And, all that stuff about courtship I did draw (loosely) from the poem, I just didn't specify the links.
Title: Re: VCE English Question Thread
Post by: chocolatecupcakes on June 27, 2015, 02:13:42 pm
When researching external examples to use in my context essays what should I be looking for? My context is encountering conflict.
Title: Re: VCE English Question Thread
Post by: tashhhaaa on June 27, 2015, 10:42:15 pm
Hey Lauren (or anyone else),

I read some of your language analysis essays that I found on AN a while ago and I was in absolute awe of your amazing vocabulary.

How do I expand mine to get to even a tenth of yours? Even though I'm averaging A+, I don't write anything like that. I know you did incredibly well in English, but I don't want to write mediocre essays in the exam -- I'd really like to impress my assessors with the language I use lol.

Teachers and the like always say read to have a better vocabulary but I find that a little passive I don't really learn anything from it. Do you have any other tips to improve?
Title: Re: VCE English Question Thread
Post by: HighTide on June 28, 2015, 09:20:50 am
When researching external examples to use in my context essays what should I be looking for? My context is encountering conflict.
Look for stuff other than the standard war, physical brutality or the overused ones going around. You can use like any issue as a conflict but it depends on your text and the prompt given. Some ideas would be stuff like domestic violence, rivalries and arguments between individuals,societies or groups. Whatever you decide to use, remember that it does not have to effect wide scale.
I.e. You can have conflicts ranging from small scale to large scale. So like sibling rivalry, to the tension between countries.
Most people will research online to find conflicts. This would be good but, if you want to find smaller scale conflicts which give variety to your arguments, there's heaps in the newspapers.
Hey Lauren (or anyone else),

I read some of your language analysis essays that I found on AN a while ago and I was in absolute awe of your amazing vocabulary.

How do I expand mine to get to even a tenth of yours? Even though I'm averaging A+, I don't write anything like that. I know you did incredibly well in English, but I don't want to write mediocre essays in the exam -- I'd really like to impress my assessors with the language I use lol.

Teachers and the like always say read to have a better vocabulary but I find that a little passive I don't really learn anything from it. Do you have any other tips to improve?
Hi, not Lauren but I hope I can help.
I was in a similar situation at the start of the year. I needed to make my essays more sophisticated. It really helped to read the sample text responses, language analysis' and context pieces on both Atarnotes, VCAA past exams and on other websites. The best method of improving is through writing essays, trying out new words, and then asking your teachers or people on AN to read your essay.
However, saying that, in your attempt to impress the assessors by using new words and all that, it's probably not recommended to add new words just for the "flare" (as in, no thesaurus.com, but rather checking each and every word in a dictionary). Just consider, that by the time you do your exam, you want words that will stand out but also ensure that your arguments are short and concise.
Hope this helps  :)
Title: Re: VCE English Question Thread
Post by: anat0my on June 28, 2015, 01:34:35 pm
Hi what's the best way to go about reading/rereading a book?
Title: Re: VCE English Question Thread
Post by: tashhhaaa on June 28, 2015, 02:52:30 pm
Hi, not Lauren but I hope I can help.
I was in a similar situation at the start of the year. I needed to make my essays more sophisticated. It really helped to read the sample text responses, language analysis' and context pieces on both Atarnotes, VCAA past exams and on other websites. The best method of improving is through writing essays, trying out new words, and then asking your teachers or people on AN to read your essay.
However, saying that, in your attempt to impress the assessors by using new words and all that, it's probably not recommended to add new words just for the "flare" (as in, no thesaurus.com, but rather checking each and every word in a dictionary). Just consider, that by the time you do your exam, you want words that will stand out but also ensure that your arguments are short and concise.
Hope this helps  :)

thank you for your advice :) Do you know where I could find really good samples to download? I'm fine with going through the forums but it's really time consuming (and fuels my tendencies to procrastinate), especially having to read everything to find quality work :s
Title: Re: VCE English Question Thread
Post by: Coffee on June 28, 2015, 03:06:51 pm
Hi what's the best way to go about reading/rereading a book?
Read the book first so you can take everything in. Don't take notes/highlight/etc, just focus on the material. Everyone has a different idea as to how many times you should read a book (because it really depends on you) but I think the majority would agree that 2-3 times is sufficient. The second and third time should be for highlighting key points/quotes/etc.

thank you for your advice :) Do you know where I could find really good samples to download? I'm fine with going through the forums but it's really time consuming (and fuels my tendencies to procrastinate), especially having to read everything to find quality work :s
I'm not HighTide but I thought I might just pop in and help you while I'm at it.
Here are some essays from the VCAA Examination Reports: http://www.vcaa.vic.edu.au/Pages/vce/studies/english/englishexams.aspx
Sample High Scoring Responses: English Resources and Sample High Scoring Responses
There's quite a few essays there so hopefully it's enough to get you started. :)
Title: Re: VCE English Question Thread
Post by: tashhhaaa on June 28, 2015, 03:21:23 pm
Read the book first so you can take everything in. Don't take notes/highlight/etc, just focus on the material. Everyone has a different idea as to how many times you should read a book (because it really depends on you) but I think the majority would agree that 2-3 times is sufficient. The second and third time should be for highlighting key points/quotes/etc.
I'm not HighTide but I thought I might just pop in and help you while I'm at it.
Here are some essays from the VCAA Examination Reports: http://www.vcaa.vic.edu.au/Pages/vce/studies/english/englishexams.aspx
Sample High Scoring Responses: English Resources and Sample High Scoring Responses
There's quite a few essays there so hopefully it's enough to get you started. :)

thank you, you guys here are amazing  ;D
Title: Re: VCE English Question Thread
Post by: heids on June 29, 2015, 01:44:39 pm
Hey Lauren, I feel bad asking questions to help me out with tutoring... but I'm totally stuck on conclusions.  I used the foolproof method of never getting far enough in an essay to reach the conclusion, but can't exactly teach this to my students :P

People always say conflicting things:
> 'Don't just summarise - say something new and profound'
> 'Don't ever introduce new information'

When you question this, the response is impossibly vague: 'Just try to find the right balance'.  Since in general, I finish my intros with about the most 'profound' sentence I can come up with, the conclusion is a sad and sorry rehash as I have nothing left to say.  So any hints on conclusions - mainly TR, but context and LA would be nice too? :D
Title: Re: VCE English Question Thread
Post by: thaaanyan on June 29, 2015, 03:52:20 pm
Hey Lauren, I feel bad asking questions to help me out with tutoring... but I'm totally stuck on conclusions.  I used the foolproof method of never getting far enough in an essay to reach the conclusion, but can't exactly teach this to my students :P

People always say conflicting things:
> 'Don't just summarise - say something new and profound'
> 'Don't ever introduce new information'

When you question this, the response is impossibly vague: 'Just try to find the right balance'.  Since in general, I finish my intros with about the most 'profound' sentence I can come up with, the conclusion is a sad and sorry rehash as I have nothing left to say.  So any hints on conclusions - mainly TR, but context and LA would be nice too? :D

not Lauren, but maybe something i say will strike a chord and help :)
when i look at my conclusions they're often very similar to my introductions but theyre 'backward.' Like if you go for the standard intro of :
my conclusion will always end up starting with the third topic sentence and working its way back to the broader starting contention. i'd then sort of use the final statement to discuss my contention holistically, in reference to my broader interpretation of the text.

this is basically my default position, but honestly it depends on the prompt and the type of essay i want to write. some times i find it helpful not to bother with a recap and to instead examine different textual interpretations. other times i'm more interested in the central thematic concerns which pervade my essay. most of the time it's a combination of all these things. perhaps get your student to go with the basic model then try out different styles?? good-luck! hope this helps :)
Title: Re: VCE English Question Thread
Post by: heids on June 29, 2015, 04:39:41 pm
Thanks, much appreciated and definitely helpful, I forgive you for not being Lauren :D

One thing:

i'd then sort of use the final statement to discuss my contention holistically, in reference to my broader interpretation of the text.
I'm afraid I don't get what this actually means, in concrete terms.  That was my problem with English - you know, the teacher just waves their hands artistically, kinda abracadabra style, and they're like, 'So then just draw that to an holistic profound summation'.

Do you have any examples?  What do you mean exactly by 'holistically' - do you mean the crux of your overall argument (so, essentially repeating how I finished the intro)?  And 'broader interpretation'?
Title: Re: VCE English Question Thread
Post by: thaaanyan on June 30, 2015, 01:36:40 pm
Thanks, much appreciated and definitely helpful, I forgive you for not being Lauren :D

One thing:
I'm afraid I don't get what this actually means, in concrete terms.  That was my problem with English - you know, the teacher just waves their hands artistically, kinda abracadabra style, and they're like, 'So then just draw that to an holistic profound summation'.

Do you have any examples?  What do you mean exactly by 'holistically' - do you mean the crux of your overall argument (so, essentially repeating how I finished the intro)?  And 'broader interpretation'?

ok so i was sort of thinking of lit when i wrote that, it's a very literature thing to do, but sometimes i sort of do it in english as well. like, holistically is sort of mean look at the bigger picture - for example looking at the 2014 topic on mabo about pride mabe i would conclude through examining the deeper consequences of the idea. how is pride a intrinsic facet of human nature?
so like, my 'holistic' summation of the central argument in my essay would mainly focus on:
what the intricacies of eddie's personality reveal about pride/ his struggle

and then my conclusion (if i went with the 'broader interpretation' idea) would just briefly highlight something about the way pride is clear in the human condition, or just generally what i think pride reveals about people's intrinsic humanity.

i just sort of made the 'broader interpretation' model of conclusion up - it's not something i know other people to do.
my teacher's typically go with the 'general summary' model but i get tired of writing essays the same way over and over (though it's a perfectly valid method), mostly i just vary it however i would like to, though it also really depends on the stuff you've argued in your body paragraphs.
it's not so much 'mystical' as kind of made up on the spot, like you look at the flow of your paragraphs, see what your writing style is like and then conclude depending on how lazy your feeling or what you had for breakfast.
ultimately there's no right or wrong way to do a conclusion as long as you provide a sense of finality and closure in your essay. i do it by extending the scope of my argument, but you could still do it equally as well through following the "backward general summary" style.
hope this helpsss :)
thank - you for your forgiveness!
Title: Re: VCE English Question Thread
Post by: heids on June 30, 2015, 05:12:28 pm
hope this helpsss :)

Surrreee thing, I gotcha.  Thanks!

(what does this show about me, as a tutor, asking current yr 12s for help ::) that's why I'm tutoring people who are struggling, not people like you who are gonna get 50s, I'm soooo ignorant)
Title: Re: VCE English Question Thread
Post by: thaaanyan on June 30, 2015, 06:48:36 pm
Surrreee thing, I gotcha.  Thanks!

(what does this show about me, as a tutor, asking current yr 12s for help ::) that's why I'm tutoring people who are struggling, not people like you who are gonna get 50s, I'm soooo ignorant)

whether or not i get a 50 or a 20, i think it shows a level of humility and humbleness to approach others on behalf of your students needs. i think it highlights a sense of modesty and a lack of ego, which are likeable facets of any individual's personality. and i've read your posts, i have no doubt you're a very skilled english tutor. your students are very lucky. :)
Title: Re: VCE English Question Thread
Post by: Apink! on July 01, 2015, 10:33:33 am
Hi,

I just confused about connotation of some words. For example, in Language analysis, I often use words like "the author, [utilizes, employs, uses, apply, employ] technique a,b,c...
But what are the difference between these words (utilise, employ, use, ? I heard that used incorrectly, you can sound quite off and weird. I also read somewhere that it's better not to use "latter, former" kind of writing in my writing because there are so many ways to do it wrong. Is this true? Also what is the difference between Assume, presume and postulate? I always treated them as same things.

Also what the hell are doubled up auxiliary verbs...

Are there any common errors such as these, where students misuse words (because they don't understand the connotation of the words attached to it?)

Thank you in advance!
Title: Re: VCE English Question Thread
Post by: heids on July 01, 2015, 02:20:23 pm
To utilise, employ and use can be used completely interchangeably; apply is a bit different, like it just doesn't quite sound right to say, 'the author applies inclusive language to create a sense of…'.  I can’t quite explain why, it’s just that apply =/= use.
   
Presume and assume can be used interchangeably, but postulate is a bit different - it means more to present a new idea or suggestion – to pose, suggest, present.

Latter/former lends itself to complex, confusing and cumbersome sentence structures.  Avoid it, in general - especially you personally, because you have a bit of an issue with big long tangled unwieldy sentences.  And unless you're really comfortable with it (I am coz it's used commonly in the books I tend to read, older ones like Dickens) you really risk making mistakes.

No clue what doubled up auxiliary verbs are.  Google ‘em :)

Anyway, it's often just about what 'feels' right (though that's based on grammatical reasons generally).  Try this example that you'll easily get: 'mention' and 'refer'.  'The author mentions Jim Carey' sounds right; 'The author refers Jim Carey' you just know is wrong (I hope :P).  You know that you have to put a 'to' in there - refers to - because you've absorbed it.  But if you just learnt it off a list of synonyms, how would you know that they need to be used slightly differently?  Or, 'suggest' (in some senses) = 'present'. 'The author suggests that...' - fine, 'The author presents that...' - wrong.  You need 'the idea that...' or something for it to make grammatical sense.  This is why assessors often encourage people to stick to ‘small’ words they're comfortable with, because nothing grates more than a ‘big’ word crammed in incorrectly where it doesn't fit.

So.  You honestly can't sit and learn a list of what words to use exactly where, and how exactly they fit into the structure.  Never learn a new word from a vocab list without trying to find some examples of where it's used in proper sentences.  Read over a number of different sentences a number of times, you gradually get used to the 'feel' of words, exactly how they're used, and exactly how they make sense in a sentence.  This is why wide reading for the sake of it gets you a long way :))



... oh and thanks thaaanyan, encouraging :)
Title: Re: VCE English Question Thread
Post by: Apink! on July 01, 2015, 02:34:27 pm
Thanks bangali_lok!

Great help as always (: I guess the real solution to these kind of questions is just reading :P

It may be that English isn't my first language, but sometimes I have trouble identifying connotations in words :P
Thank you! (:

Title: Re: VCE English Question Thread
Post by: heids on July 01, 2015, 02:43:54 pm
No matter how long you've known a language for, still it takes time to get that 'feel' for any new words - like if I tried to use some of the words Lauren does...  Don't despair, the more you read you really will pick it up, and maybe ask anyone reading over your work to point out EVERY time you use a word slightly incorrectly, and what you should have used instead.  And as I said always look for sentences with the words in them - try Googling 'define _word_', it generally supplies a sentence or two, plus lots of synonyms to give you a feel for the word.  PM me if you ever want some sample sentences with specific words in them, or to check if you can use a certain word in a certain way. It's definitely harder as a second language learner, though :(
Title: Re: VCE English Question Thread
Post by: cosine on July 02, 2015, 06:11:57 pm
Urgent help needed.

I am sitting on a B+ average for english. For my school this is not bad, but for the state this is honestly around a low C. My concern is, every year the people who even average A+ at my school do not end up with anything above 37.. So I am a bit worried. I took on bangali_lok's and literally lauren's help, to just write and even if it doesnt make sense, just put pen to paper. This has helped me improve of course, but there is still a LOT more room for improvement. I will leave this here for anyone to answer so all of us can benefit. I really want to improve and am willing to work for it, so please anyone, I have a text response SAC in 5 weeks. I am GOING to start now, what should I do?

Thank you
Title: Re: VCE English Question Thread
Post by: iNT on July 02, 2015, 06:27:21 pm
Urgent help needed.

I am sitting on a B+ average for english. For my school this is not bad, but for the state this is honestly around a low C. My concern is, every year the people who even average A+ at my school do not end up with anything above 37.. So I am a bit worried. I took on bangali_lok's and literally lauren's help, to just write and even if it doesnt make sense, just put pen to paper. This has helped me improve of course, but there is still a LOT more room for improvement. I will leave this here for anyone to answer so all of us can benefit. I really want to improve and am willing to work for it, so please anyone, I have a text response SAC in 5 weeks. I am GOING to start now, what should I do?

Thank you

Apart from reading your text i recommend to read other books as well, this might help improve your vocabulary. Apart from that why dont u post one of your sample essays so i can look at it? I am no expert but maybe i can give u a tip or something
Title: Re: VCE English Question Thread
Post by: Coffee on July 02, 2015, 06:42:58 pm
Urgent help needed.

I am sitting on a B+ average for english. For my school this is not bad, but for the state this is honestly around a low C. My concern is, every year the people who even average A+ at my school do not end up with anything above 37.. So I am a bit worried. I took on bangali_lok's and literally lauren's help, to just write and even if it doesnt make sense, just put pen to paper. This has helped me improve of course, but there is still a LOT more room for improvement. I will leave this here for anyone to answer so all of us can benefit. I really want to improve and am willing to work for it, so please anyone, I have a text response SAC in 5 weeks. I am GOING to start now, what should I do?

Thank you

Hi, cosine!

What are you having trouble with in particular? Are you having trouble with formulating ideas or is it more the writing side of it (expression/sophistication)? If it's a case of the latter, this is something that takes time and practise but can easily be worked upon. It's important you can write well and express yourself well. So this is going to be your focus for the next few weeks. For now, I wouldn't worry about time or exam conditions. Keep your texts out, notes, thesaurus, etc and refer to this while writing. When you've perfected this and you're hitting that A+ standard, you can start writing under exam conditions. What you learn from writing without timed responses you will (hopefully) retain in exam conditions. Basically, learn to write well and then you can work this down into a shorter time frame.

If it's the former though, this boils down to not knowing your texts well enough. Re-assess your study techniques and we can help you from here. Alternatively, maybe you're fine with both of the above and I've gone on a tangent. :P Let us know how you get on.
Title: Re: VCE English Question Thread
Post by: cosine on July 02, 2015, 06:54:17 pm
Apart from reading your text i recommend to read other books as well, this might help improve your vocabulary. Apart from that why dont u post one of your sample essays so i can look at it? I am no expert but maybe i can give u a tip or something

Prompt: "Jason's decisions are based on reason and careful judgement; Medea's decisions are based on passion and selfishness"

Here is my introduction and first paragraph only from my first text response. I got a B+ (77%) for it. Just by reading this, any specific improvements can be made to the into structure and paragraph?

In the tragedy, 'Medea', Euripides distinctively explores the social differences between men and women in their era, particularly through the actions of the female protagonist, Medea. Throughout the play, she is constantly bewailing the disadvantages that women have because of men, imploring their 'wretched' existence. Euripides further portrays this view by representing the men as the logical ones, carefully analysing their decisions before they act, whereas the women are deeply are 'deeply concerned' with their reputation. However, through the Nurse and the Chorus, it is also evident that Jason was initially led by his desires for a more advantageous marriage, this abandoning his family.

In the early stages of the play, Jason's egotistic actions are made clear, as he is depicted to be the villain. Euripides thoroughly explores gender inequality as the play proceeds, and the main reason for the tragedies in 'Medea' are because of the social differences between men and women. As the Nurse alludes to the prior events that consequently lead to the conflict between Medea and Jason, she emphasises the fact that Jason dishonoured Medea, thus stands 'plainly convicted as a traitor to his friends.' Jason's irrational passion to live a 'royal' life disallows him to act morally, eventually abandoning his family to seek a 'royal match'. As Medea confronts him with a 'swarm of words', Jason claims that he left her to 'produce royal offspring', so that they could live a life without being in need, but later admits to having left for a better personal life. Jason's neglect of Medea's feelings and well-being emphasises Euripide's depiction towards gender antagonism, that women must be 'obedient to their husbands, and must not refuse a man his rights'. Jason's egocentric desires eventually lead him to commit irrational and regretful actions.
Title: Re: VCE English Question Thread
Post by: cosine on July 02, 2015, 06:55:27 pm
Hi, cosine!

What are you having trouble with in particular? Are you having trouble with formulating ideas or is it more the writing side of it (expression/sophistication)? If it's a case of the latter, this is something that takes time and practise but can easily be worked upon. It's important you can write well and express yourself well. So this is going to be your focus for the next few weeks. For now, I wouldn't worry about time or exam conditions. Keep your texts out, notes, thesaurus, etc and refer to this while writing. When you've perfected this and you're hitting that A+ standard, you can start writing under exam conditions. What you learn from writing without timed responses you will (hopefully) retain in exam conditions. Basically, learn to write well and then you can work this down into a shorter time frame.

If it's the former though, this boils down to not knowing your texts well enough. Re-assess your study techniques and we can help you from here. Alternatively, maybe you're fine with both of the above and I've gone on a tangent. :P Let us know how you get on.

I usually know the texts I read and enjoy reading them. I understand most/some of the themes that appear. My main problem I think is just interpretting it into an english sentence that actually 'expresses' the ideas i am trying to portray.
Title: Re: VCE English Question Thread
Post by: Coffee on July 02, 2015, 07:18:54 pm
Just read the above response. I studied Medea as well (and loved it!) That prompt was also on my sac. Did you by any chance have the option of "Medea claims that all women are afflicted with the most 'wretched' existence on earth. How is gender explored in the play?" (I wrote on this one) :P

I usually know the texts I read and enjoy reading them. I understand most/some of the themes that appear. My main problem I think is just interpretting it into an english sentence that actually 'expresses' the ideas i am trying to portray.
I would recommend gaining a better understanding of the themes first. Do you think you've covered all of them/the most important. I definitely found myself in trouble with this text when I was given a prompt on fate and destiny. Obviously I considered these themes and I did cover them, but not well enough. I was focused on other themes that I was sure would be on the exams/sacs but they didn't end up on it. So if you feel like you know 'some' or even 'most' of the themes, think about what you might have missed.

As for interpreting, I assume you mean you don't know how to say it/get it across to the audience. As I said previously, keep writing without exam conditions. Use all the resources you need and spend as much time as you need learning how to word things properly. Get feedback from teachers, peers, or from the AN community. It doesn't matter if it takes you 6 hours or even a day to write an A+ standard essay. Once you know how to write one and you can do this consistently, we can learn to cut down on the time it takes you.
Title: Re: VCE English Question Thread
Post by: cosine on July 02, 2015, 07:24:36 pm
Just read the above response. I studied Medea as well (and loved it!) That prompt was also on my sac. Did you by any chance have the option of "Medea claims that all women are afflicted with the most 'wretched' existence on earth. How is gender explored in the play?" (I wrote on this one) :P
I would recommend gaining a better understanding of the themes first. Do you think you've covered all of them/the most important. I definitely found myself in trouble with this text when I was given a prompt on fate and destiny. Obviously I considered these themes and I did cover them, but not well enough. I was focused on other themes that I was sure would be on the exams/sacs but they didn't end up on it. So if you feel like you know 'some' or even 'most' of the themes, think about what you might have missed.

As for interpreting, I assume you mean you don't know how to say it/get it across to the audience. As I said previously, keep writing without exam conditions. Use all the resources you need and spend as much time as you need learning how to word things properly. Get feedback from teachers, peers, or from the AN community. It doesn't matter if it takes you 6 hours or even a day to write an A+ standard essay. Once you know how to write one and you can do this consistently, we can learn to cut down on the time it takes you.

Thank you coffee xD

Nah the other prompt was something about Euripides.. doesn't even make sense..

I am doing 'The Complete Maus' now by Art Spiegleman, did you also read this comic book?
Title: Re: VCE English Question Thread
Post by: Coffee on July 02, 2015, 07:32:08 pm
Thank you coffee xD

Nah the other prompt was something about Euripides.. doesn't even make sense..

I am doing 'The Complete Maus' now by Art Spiegleman, did you also read this comic book?

I didn't actually. My school did 'War Poems & Others' by Wilfred Owen (much to the dismay of my classmates who disliked poetry :P).
Title: Re: VCE English Question Thread
Post by: iNT on July 02, 2015, 07:37:48 pm
I LOVE THIS TEXT :D 

In the tragedy, 'Medea', Euripides distinctively explores the social differences between men and women in their era, particularly through the actions of the female protagonist, Medea. The actions of Medea defy the conventions of that era, while you do explain that Euripides explores the social inequities between men and women you leave unclear whether or not Medea was trying to emancipate herself or exalt herself to a position where it was possible to commit an act of revenge/justice. This ties in with the prompt - were her actions based on passion? Keep in mind you can argue against the prompt, and in this case I personally would (Hint = there was careful planning from her) Throughout the play, she is constantly bewailing the disadvantages that women have because of men, imploring their 'wretched' existence. Euripides further portrays this view by representing the men as the logical ones, carefully analysing their decisions before they act, whereas the women are deeply are 'deeply concerned' with their reputation. Excellent use of a quote. Your contention is rather ambiguous; on one note you mention that Medea is constantly complaining, while this is true, it is best to go deeper, such as her being the spokeswomen for women, on the other note men are shown to make logical decisions... however did they allow themselves to be manipulated? (Hint - consider how she managed to get refuge)Keep in mind it is about Jason though.
However, through the Nurse and the Chorus, it is also evident that Jason was initially led by his desires for a more advantageous marriage, this abandoning his family. Its good to mention Euripedes here, maybe best starting the sentence with 'EURIPIDES however, through the USE the Nurse...' This shows that you understand what Euripides is doing. The word 'this' would be replaced with 'thus'.

So you provided an excellent quote. :) I think in order to improve your introduction has to be taken a notch higher. For me, its the most important part of the essay. Its the first thing the examiner will judge. Think of it as a first date; chances are you arent gonna get a 2nd date if you arrive unclean or misbehave during it. You want to impress in that date :) Just like an essay.. a well versed introduction will make the examiner NOT look for mistakes but the look for the things that emulate your excellent intro.

Your vocabulary is very good, word choice could be better though. Your not in a bad state though, it wont take much to get better at that aspect :)

Your contentions need to more clear. Think about your 3/4 paragraphs and try to summarize into a sentence or two to put in your intro. It is important to include Euripides as well, as you make the ready acknowledge that you understand what he is conveying across.

The structure isn't A+. You go around a point. As i mentioned its best to go directly with Euripides, or Medea/Jason did this blah blah :) 
Title: Re: VCE English Question Thread
Post by: cosine on July 02, 2015, 07:55:30 pm
I LOVE THIS TEXT :D 

In the tragedy, 'Medea', Euripides distinctively explores the social differences between men and women in their era, particularly through the actions of the female protagonist, Medea. The actions of Medea defy the conventions of that era, while you do explain that Euripides explores the social inequities between men and women you leave unclear whether or not Medea was trying to emancipate herself or exalt herself to a position where it was possible to commit an act of revenge/justice. This ties in with the prompt - were her actions based on passion? Keep in mind you can argue against the prompt, and in this case I personally would (Hint = there was careful planning from her) Throughout the play, she is constantly bewailing the disadvantages that women have because of men, imploring their 'wretched' existence. Euripides further portrays this view by representing the men as the logical ones, carefully analysing their decisions before they act, whereas the women are deeply are 'deeply concerned' with their reputation. Excellent use of a quote. Your contention is rather ambiguous; on one note you mention that Medea is constantly complaining, while this is true, it is best to go deeper, such as her being the spokeswomen for women, on the other note men are shown to make logical decisions... however did they allow themselves to be manipulated? (Hint - consider how she managed to get refuge)Keep in mind it is about Jason though.
However, through the Nurse and the Chorus, it is also evident that Jason was initially led by his desires for a more advantageous marriage, this abandoning his family. Its good to mention Euripedes here, maybe best starting the sentence with 'EURIPIDES however, through the USE the Nurse...' This shows that you understand what Euripides is doing. The word 'this' would be replaced with 'thus'.

So you provided an excellent quote. :) I think in order to improve your introduction has to be taken a notch higher. For me, its the most important part of the essay. Its the first thing the examiner will judge. Think of it as a first date; chances are you arent gonna get a 2nd date if you arrive unclean or misbehave during it. You want to impress in that date :) Just like an essay.. a well versed introduction will make the examiner NOT look for mistakes but the look for the things that emulate your excellent intro.

Your vocabulary is very good, word choice could be better though. Your not in a bad state though, it wont take much to get better at that aspect :)

Your contentions need to more clear. Think about your 3/4 paragraphs and try to summarize into a sentence or two to put in your intro. It is important to include Euripides as well, as you make the ready acknowledge that you understand what he is conveying across.

The structure isn't A+. You go around a point. As i mentioned its best to go directly with Euripides, or Medea/Jason did this blah blah :)

Alright so the introduction is the most important part, how can I improve it then? Like what is the basic structure to follow?
Title: Re: VCE English Question Thread
Post by: iNT on July 02, 2015, 08:06:16 pm
Alright so the introduction is the most important part, how can I improve it then? Like what is the basic structure to follow?

Most basic structure

1- author, text, (type) you mentioned in yours a 'tragedy'

2- background to topic - otherwise know as the ' big picture'

3- Your contention/argument about the topic - (smaller picture)
Has to be in 3 parts (depends if you write more or less than 3 paragraphs) . Each 'part' signifies one argument towards the prompt
 Part 1 will be what you will talk about in your 1st paragraph, Part 2 = 2nd paragraph Part 3 = 3rd Paragraph.
It is best to have 4 paragraphs that signify more depth in your SAC which will most likely be 2 periods.. in the exam you will be pushed for time so its your decision if your want 3 or 4.
Title: Re: VCE English Question Thread
Post by: Coffee on July 02, 2015, 08:19:16 pm
Alright so the introduction is the most important part, how can I improve it then? Like what is the basic structure to follow?
A basic structure is as follows:
You can deviate from this a little though. It doesn't have to be so formulaic. So something I wrote on one of the prompts I mentioned earlier was:
Euripides' play 'Medea' was first presented to the Ancient Athenian audience during the festival of Dionysus and tells the tale of a woman scorned. Despite it's reception it was ultimately unsuccessful and Euripides was the first Greek poet to suffer the fate of so many of the great modern writers. Rejected by his contemporaries, however now admired by many, modern audiences can learn from both the characters and themes as demonstrated in John Davie's translation. This explores the politics of gender within patriarchal Greek society and the power of patriarchal ideologies, constructs and conformity. Euripides enquires into the peril implicit with patriarchal views on women in Ancient Greek society.
^ By no means an amazing piece but it addresses the criteria/structure.
Title: Re: VCE English Question Thread
Post by: iNT on July 02, 2015, 09:10:23 pm
A basic structure is as follows:
  • Introduce the title of the text and the author in the first sentence.
  • Include background information on the text.
  • State your opinion on the topic and outline the main points that will contribute to your argument.
You can deviate from this a little though. It doesn't have to be so formulaic. So something I wrote on one of the prompts I mentioned earlier was:
Euripides' play 'Medea' was first presented to the Ancient Athenian audience during the festival of Dionysus and tells the tale of a woman scorned. Despite it's reception it was ultimately unsuccessful and Euripides was the first Greek poet to suffer the fate of so many of the great modern writers. Rejected by his contemporaries, however now admired by many, modern audiences can learn from both the characters and themes as demonstrated in John Davie's translation. This explores the politics of gender within patriarchal Greek society and the power of patriarchal ideologies, constructs and conformity. Euripides enquires into the peril implicit with patriarchal views on women in Ancient Greek society.
^ By no means an amazing piece but it addresses the criteria/structure.

I know but he asked for a basic structure. If one is struggling with structure then there is no reason to deviate from a basic structure that does its job. Once he gets better at it he may create his own structure, or a deviation just like u have :)
Title: Re: VCE English Question Thread
Post by: Coffee on July 02, 2015, 09:17:22 pm
I know but he asked for a basic structure. If one is struggling with structure then there is no reason to deviate from a basic structure that does its job. Once he gets better at it he may create his own structure, or a deviation just like u have :)
The above is a basic structure and the example deviates very little from what's listed above. And I wasn't trying to contradict what you posted. You just replied quicker than me. :P
Title: Re: VCE English Question Thread
Post by: iNT on July 02, 2015, 09:32:28 pm
The above is a basic structure and the example deviates very little from what's listed above. And I wasn't trying to contradict what you posted. You just replied quicker than me. :P

sorry if i sounded harsh :( my bad
Title: Re: VCE English Question Thread
Post by: heids on July 02, 2015, 09:34:45 pm
Here's a long discourse on your BP - note: it was totally solid and functional, I've just pulled it a bit to pieces.  Please don't stress, as I said, getting this solid stuff down is really important, and you seem to have the solid basics for a decent enough mark.

Prompt: "Jason's decisions are based on reason and careful judgement; Medea's decisions are based on passion and selfishness"

In the early stages of the play, Initially, Jason's egotistic actions are made clear, as he is depicted to be the villain. Euripides thoroughly explores gender inequality as the play proceeds, and the main reason for the tragedies in 'Medea' are because of the social differences between men and women. Look back at the prompt.  Now I know you can always stray a bit from the prompt a bit, but I can't see the link very well - you need to 100% spell out the relevance to me, even if it's crystal clear in your own head.  This sort of sentence is probably more for later in the essay though, as it's a conclusion you draw from the prompt about Euripides' views and values which he shows through how he presents these two characters.  As the Nurse alludes to the prior events that consequently lead to the conflict between Medea and Jason, she emphasises the fact that Jason dishonoured Medea, thus stands 'plainly convicted as a traitor to his friends.' Jason's irrational passion to live a 'royal' life disallows him to act morally, eventually abandoning his family to seek a 'royal match'. As Medea confronts him with a 'swarm of words', Jason claims that he left her to 'produce royal offspring', so that they could live a life without being in need, but later admits to having left for a better personal life.  All that underlined region is a bit too ‘story-telly’ – the last sentence especially.  Stuff like "As Medea confronts him with a ‘swarm of words’" feels like you put it there JUST to show that you have a quote available.  See how I've stripped it down a bit:
‘Nurse labels him a 'traitor' for dishonouring Medea.  Jason's irrational desire to live a 'royal' life is not compatible with maintaining his morality. While originally Jason claims that he left Medea to ‘produce royal offspring’, his later admission that he left for a better personal life REVEALS... '   
So, this cuts out unnecessary detail and story-telling (though it then doesn't flow well, I'm just showing how to cut out unnecessary description; avoid having a whole sentence that just describes what happens, the description should be a 'jumping off' point at the start of the sentence that leads on to some analysis).  The noun, 'his later admission', forces you to continue the sentence with some analysis.  I underlined the verb, because verbs like that bridge between description of the story and analysis of what that actually shows, what the whole point of your evidence is.  Try to use as many verbs like that as possible! reveals, conveys, demonstrates, exposes, highlights, underscores, illustrates etc.
  Jason's neglect of Medea's feelings and well-being emphasises Euripide's depiction towards gender antagonism, that women must be 'obedient to their husbands, and must not refuse a man his rights' Again, specify exactly what you mean.  Like, you haven't actually said exactly what Euripides thinks; you've only stated (I think) that he's against the notion that women must 'be obedient'.  Unpack it a bit more; explain to me how exactly this neglect shows Euripides' values. Jason's egocentric desires eventually lead him to commit irrational and regretful regrettable actions.  P.S. You should probably discuss a bit in this paragraph about how, through these actions, Jason shows that he ISN'T being rational/using careful judgment.  In what way does Euripides show him as rash?

Anyway, as Coffee said, it's about finding your specific issues and working on them.  Rather than saying, 'I'm going to sit down and improve my mark from a 7 to a 9', you should be going, OK, let's look over my work and see what some of my mistakes are.  What little tiny things am I getting wrong that I could focus on and fix up?  Maybe the best way of studying is searching through your work for your errors.  Just reading through feedback on other people's essays on the English Work Submission and Marking board can really help you to find out common errors that you might make too.  Once you know a really specific error you make, you can pay attention to fixing it up!

Feel free to throw a full piece in the marking board :D
Title: Re: VCE English Question Thread
Post by: Coffee on July 02, 2015, 09:37:10 pm
sorry if i sounded harsh :( my bad
No, no. I just didn't want you to think I was trying to contradict what you said. It's all good, man. :)
Title: Re: VCE English Question Thread
Post by: Annalise_jackson on July 03, 2015, 05:17:19 pm
Hi Lauren,
I am doing literature and was wondering how I should structure an adaptions and transformations SAC on amadeus?
 :D
Title: Re: VCE English Question Thread
Post by: heids on July 03, 2015, 05:26:26 pm
Hi Lauren,
I am doing literature and was wondering how I should structure an adaptions and transformations SAC on amadeus?
 :D

Hey Annalise!
Could you post any lit questions in Literature?  Thanks, and I hope you get heaps out of this site!
Title: Re: VCE English Question Thread
Post by: anat0my on July 03, 2015, 10:40:06 pm
Hi :)

I was wondering what is the best way to re-read a text?
Also how can I focus on the current area of study my school is focusing on and also study back other t.r texts, context and language analysis all at the same time?

Thanks!
Title: Re: VCE English Question Thread
Post by: lisax3 on July 05, 2015, 10:09:16 am
What would be the structure for an expository context essay body paragraph?
Title: Re: VCE English Question Thread
Post by: heids on July 06, 2015, 08:49:37 pm
I was wondering what is the best way to re-read a text?
Also how can I focus on the current area of study my school is focusing on and also study back other t.r texts, context and language analysis all at the same time?
If you have a pretty complete grasp of the plot, characters, and basic major themes, you're dissecting for the details here.  (If not, read like the first time, just reading it like a 'story').  Have a couple of Word documents open and add to them as you go through; mainly it's up to your intuition, but some pointers:

> Quote bank.  As you read, when you hit anything notable in some way, write out the quote, and a couple of brief notes on context/who said it if you think you need it.  Later, you can go through and organise it - by character or theme - and throw in any ideas about how you'd discuss/analyse them in essays.  This is the MOST important.
> Characters.  List any important events, turning points, quotes, etc. that show their characteristics and development.
> Themes.
> Random ideas.  When you hit on a metaphor you can use, or any insight, scribble it down.

Don't trust yourself to remember anything you stumble on, WRITE IT DOWN.  And don't just skim lightly without thinking analytically.

Doing what the class is doing and keeping up with other stuff is simply a general study skill/time management skill.  Maybe try compiling a list of ways you can study/things you can do for each of 3 essay types.  Then, if you're doing context in class, spend an hour doing stuff on that list for language analysis each week.  It's up to you, ultimately.

What would be the structure for an expository context essay body paragraph?
Hard question, because there are no rules and exact structure in context!

My guess, but look I don't know:
> broad, zoomed-out, wide opening couple of statements
> zoom in to nitty-gritty details and examples, repeating as often as you see fit:
------> assert or suggest some theoretical idea
------> give some evidence/example(s)
------> explain what you draw out of this evidence, the messages/theoretical ideas it proves
> zoom back out to another broad statement, the cumulation of what you've discussed in the paragraph rather than a rehash of the TS; stuff like what your discussion shows about the prompt/mankind/human condition
> link to next para (optional, but of course you want flow)

Try going through some sample high-scoring expositories (e.g. in English Resources and Sample High Scoring Responses) and deconstruct their paragraphs into brief dot-point sentences; that'll give you an idea of their structure and flow.
Title: Re: VCE English Question Thread
Post by: cosine on July 07, 2015, 05:31:15 pm
You guys always tell me to 'just' write, and so I have been following that and there has been improvements, so very grateful (bangali_lok youre the best...)

But it's time for some serious business. I don't just want 'slight' improvements, its time for seriousness. What do you guys recommend, writing in pencil or typing up practice essays, be honest with me please xD

Also would reading random books really improve my essays? And my teacher says I need to enhance my vocabulary, which ways can I take to ensure this is achieved?

Please offer serious help, and even if it sounds too hardcore, I am willing to follow the advice, be it reading the dictionary in my spare time :P
Title: Re: VCE English Question Thread
Post by: _fruitcake_ on July 07, 2015, 05:36:57 pm
You guys always tell me to 'just' write, and so I have been following that and there has been improvements, so very grateful (bangali_lok youre the best...)

But it's time for some serious business. I don't just want 'slight' improvements, its time for seriousness. What do you guys recommend, writing in pencil or typing up practice essays, be honest with me please xD

Also would reading random books really improve my essays? And my teacher says I need to enhance my vocabulary, which ways can I take to ensure this is achieved?

Please offer serious help, and even if it sounds too hardcore, I am willing to follow the advice, be it reading the dictionary in my spare time :P

Writing in pencil helps, first it improves your handwriting.. and it simulates what your sacs and exam will be like. Examiners loveee to see clear handwriting and good ideas after spending the last hour squinting at hard-to-read essays
Title: Re: VCE English Question Thread
Post by: heids on July 07, 2015, 05:56:55 pm
You guys always tell me to 'just' write, and so I have been following that and there has been improvements, so very grateful (bangali_lok youre the best...)

But it's time for some serious business. I don't just want 'slight' improvements, its time for seriousness. What do you guys recommend, writing in pencil or typing up practice essays, be honest with me please xD

Also would reading random books really improve my essays? And my teacher says I need to enhance my vocabulary, which ways can I take to ensure this is achieved?

Please offer serious help, and even if it sounds too hardcore, I am willing to follow the advice, be it reading the dictionary in my spare time :P

Do you have a list of your problems?  Do you know exactly what some of your issues are? Please, I haven't said 'just write' - I've said, write, then identify a problem, then write again fixing up that problem.  If you haven't got a list of your specific specific problems, then tomorrow, I assign you homework: sit down with a couple of your essays/feedbacks, and try to write a dot-point list of little things you can do to improve.  If you don't get anywhere, feel free to post an essay for feedback in the work submission forum; I'm willing to work on pointing out little specific issues and ways to improve them, if you tell me whether you want me to be harsh or gentle.

EDIT: Another thing is brainstorming and detailed plans - they save a lot of time since writing a whole essay is really time-consuming.  Then show the plans to your teacher to point out your overall flaws.  Or try just practice paragraphs, rather than whole essays.  You can generally find your issues even in just one paragraph.

Write in pen, ideally. :P

Vocab: (copy-pasted from a big post I'm compiling on vocab/expression in my worst procrastinating moments, stay tuned):
1.   Write something.
2.   Go through it, or get someone else to go through it, and list any ‘problem’ words – words that are commonly repeated (e.g. ‘the’… no not that), don’t quite express what you wanted, or are vague and generic (e.g. ‘good’).
3.   Thesaurus it.  I love verbising.
4.   From this, build a bank of synonyms.
5.   Similarly, read other people’s writing and steal any great words you see them use.
6.   Stick up sticky notes or mindmaps of good synonyms on your wall, in your locker, anywhere you’ll look at them.
7.   Practice writing the words in single (ideally analytical) sentences to get used to them.  Get other people to check them to make sure they make sense.
8.   When writing essays, have the bank there and refer to it as you go, trying to incorporate new words.  Or, write ‘closed-book’ essays, and go over them afterwards, replacing weaker words with stronger ones from your bank.
9.   If you have especial trouble with repeating one word, focus the next time you write an essay on NEVER using that word, and always trying to put in a synonym.
10.   Repeat.  Ad infinitum.

And yes, wider reading will help just with better expression.  Probably, at this point in the year, I'll break it to you that it won't have much impact on your essays because it's more a long-term gradual change.  BUT, it will really help you with EVERYTHING you ever write in the future, at uni or whatever.
Title: Re: VCE English Question Thread
Post by: cosine on July 07, 2015, 06:06:25 pm
Do you have a list of your problems?  Do you know exactly what some of your issues are? Please, I haven't said 'just write' - I've said, write, then identify a problem, then write again fixing up that problem.  If you haven't got a list of your specific specific problems, then tomorrow, I assign you homework: sit down with a couple of your essays/feedbacks, and try to write a dot-point list of little things you can do to improve.  If you don't get anywhere, feel free to post an essay for feedback in the work submission forum; I'm willing to work on pointing out little specific issues and ways to improve them, if you tell me whether you want me to be harsh or gentle.

EDIT: Another thing is brainstorming and detailed plans - they save a lot of time since writing a whole essay is really time-consuming.  Then show the plans to your teacher to point out your overall flaws.  Or try just practice paragraphs, rather than whole essays.  You can generally find your issues even in just one paragraph.

Write in pen, ideally. :P

Vocab: (copy-pasted from a big post I'm compiling on vocab/expression in my worst procrastinating moments, stay tuned):
1.   Write something.
2.   Go through it, or get someone else to go through it, and list any ‘problem’ words – words that are commonly repeated (e.g. ‘the’… no not that), don’t quite express what you wanted, or are vague and generic (e.g. ‘good’).
3.   Thesaurus it.  I love verbising.
4.   From this, build a bank of synonyms.
5.   Similarly, read other people’s writing and steal any great words you see them use.
6.   Stick up sticky notes or mindmaps of good synonyms on your wall, in your locker, anywhere you’ll look at them.
7.   Practice writing the words in single (ideally analytical) sentences to get used to them.  Get other people to check them to make sure they make sense.
8.   When writing essays, have the bank there and refer to it as you go, trying to incorporate new words.  Or, write ‘closed-book’ essays, and go over them afterwards, replacing weaker words with stronger ones from your bank.
9.   If you have especial trouble with repeating one word, focus the next time you write an essay on NEVER using that word, and always trying to put in a synonym.
10.   Repeat.  Ad infinitum.

And yes, wider reading will help just with better expression.  Probably, at this point in the year, I'll break it to you that it won't have much impact on your essays because it's more a long-term gradual change.  BUT, it will really help you with EVERYTHING you ever write in the future, at uni or whatever.

Cheers xD

I will write some paragraphs (extended answers) tomorrow and post them, but what if you haven't read the book (The Complete Maus) ?

I really love that synonym idea, will definitely do it! xD Can you start me off with a good word that I have used more than once in this interview with you (:P) and provide me with a synonym for it? No, not testing your vocab skills haha

But seriously, thank you for the constant help and encouragement!
Title: Re: VCE English Question Thread
Post by: heids on July 07, 2015, 06:14:13 pm
Y'welcome bro, it's called procrastination from doing paid/necessary work XD

Cheers xD

I will write some paragraphs (extended answers) tomorrow and post them, but what if you haven't read the book (The Complete Maus) ?
It won't be optimal, but I can still point out some stuff, probably.  I hain't read Medea, either.

Quote
Can you start me off with a good word that I have used more than once in this interview with you (:P) and provide me with a synonym for it?
Yes - 'synonym'. :P
'similar word' (can't think of anything better).

By 'write something', though, I meant, like write a language analysis, and go through and find that you're using the word 'positions' too often.  Or something.

Title: Re: VCE English Question Thread
Post by: literally lauren on July 07, 2015, 06:36:28 pm
Cheers xD

I will write some paragraphs (extended answers) tomorrow and post them, but what if you haven't read the book (The Complete Maus) ?
I'll happily jump in if there are any interpretative errors or comments about the text that you're not sure are completely accurate, but the majority of essay feedback will centre on things like relevance, clarity, and idea development (as well as grammar and general good expression) so an inside-out knowledge of the text isn't completely necessary.

I really love that synonym idea, will definitely do it! xD Can you start me off with a good word that I have used more than once in this interview with you (:P) and provide me with a synonym for it?

Start with 'said.' It's one of the worst and most overused words in English since it's just so generic. You never want to say 'the author says...' in your essays - there'll always be a way better option available.

General verbs about what the author is doing can also be a good starting point.
For instance, if you want to say the writer >makes us think good things< about a certain idea, then you could explore words like 'celebrates' / 'extols' / 'glorifies' / 'venerate' <-- each of which have their own subtleties in terms of when you'd use them.
Then you can explore the reverse, ie. the author 'condemns' / 'denounces' / 'rebukes' / 'inveighs' ...etc.

The best starting point is going to be your own writing though. Go back over old essays and look for the kinds of words you rely upon often, and then find some alternatives you can keep up your sleeve :)
Title: Re: VCE English Question Thread
Post by: angelrox00 on July 07, 2015, 10:15:05 pm
Hi everyone, How important is the length of essays in the english exam? Approximately, how many words do you think will be typical of a high scoring essay?

Thanks
May God Bless
~Angel Raju
Title: Re: VCE English Question Thread
Post by: Alter on July 07, 2015, 10:51:08 pm
I think I read on this forum previously that one would want to aim for ~1000 words, ideally. However, you could still get a 10/10 essay by going 900 or 1100 words, for example. As far as I know, there is no magic number. The only thing that's important is that you meet the criteria of each specific task A/B/C and aim to write up about 3-4 body paragraphs (very generalised rule) for your pieces. Overall, the cliche of 'quality over quantity' definitely applies, and you should always aim for clarity and sophistication over length for VCE English. Obviously, quality can be hampered if there is too little quantity, but this is where you just use common sense.
Title: Re: VCE English Question Thread
Post by: cosine on July 10, 2015, 06:19:03 pm
It's not an essay, so I don't think I should post it on the essay marking, but could someone have a look on the extended response question I answered, please?

Book: The Complete Maus

1. What makes someone a survivor like Vladek?

To become a survivor, much like Vladek, an individual must have endured, and essentially have overcome such challenging experiences. The events that Vladek went through can be seen as incomparable, the inhumane treatment, scarce supplies and the deadly diseases that the Jews had to witness overall, just to suffer another day. The conditions so bad and the food so scarce, that it was the perfect recipe to "die even more slowly". To give up and “to die, it is easy”, “But you have to struggle for life!” A person is classified as a survivor when they overcome situations where it is much easier to give up, then to persist and conquer. Surviving situations like this requires not only physical strength, but, more importantly, one must have the correct mindset and emotional stability to endure the conflict. That is what it takes, to become a survivor like Vladek.
Title: Re: VCE English Question Thread
Post by: TheAspiringDoc on July 10, 2015, 06:41:51 pm
Cheers xD

I will write some paragraphs (extended answers) tomorrow and post them, but what if you haven't read the book (The Complete Maus) ?

I really love that synonym idea, will definitely do it! xD Can you start me off with a good word that I have used more than once in this interview with you (:P) and provide me with a synonym for it? No, not testing your vocab skills haha

But seriously, thank you for the constant help and encouragement!
There's your word! :P
Title: Re: VCE English Question Thread
Post by: cosine on July 10, 2015, 08:29:34 pm
There's your word! :P

xD

It's not an essay, so I don't think I should post it on the essay marking, but could someone have a look on the extended response question I answered, please?

Book: The Complete Maus

1. What makes someone a survivor like Vladek?

To become a survivor, much like Vladek, an individual must have endured, and essentially have overcome such challenging experiences. The events that Vladek went through can be seen as incomparable, the inhumane treatment, scarce supplies and the deadly diseases that the Jews had to witness overall, just to suffer another day. The conditions so bad and the food so scarce, that it was the perfect recipe to "die even more slowly". To give up and “to die, it is easy”, “But you have to struggle for life!” A person is classified as a survivor when they overcome situations where it is much easier to give up, then to persist and conquer. Surviving situations like this requires not only physical strength, but, more importantly, one must have the correct mindset and emotional stability to endure the conflict. That is what it takes, to become a survivor like Vladek.

Anyone ?
Title: Re: VCE English Question Thread
Post by: _fruitcake_ on July 10, 2015, 08:37:57 pm
It's not an essay, so I don't think I should post it on the essay marking, but could someone have a look on the extended response question I answered, please?

Book: The Complete Maus

1. What makes someone a survivor like Vladek?

To become a survivor, much like Vladek, an individual must have endured, and essentially have overcome such challenging experiences. The events that Vladek went through can be seen as incomparable, the inhumane treatment, scarce supplies and the deadly diseases that the Jews had to witness overall, just to suffer another day. The conditions so bad and the food so scarce, that it was the perfect recipe to "die even more slowly". To give up and “to die, it is easy”, “But you have to struggle for life!” A person is classified as a survivor when they overcome situations where it is much easier to give up, then to persist and conquer. Surviving situations like this requires not only physical strength, but, more importantly, one must have the correct mindset and emotional stability to endure the conflict. That is what it takes, to become a survivor like Vladek.

not the best..i been doing homework since  9am.. just a different idea

The events that Vladek went through can be seen as incomparable, the inhumane treatment, scarce supplies and the deadly diseases that the Jews had to witness overall, just to suffer another day. The conditions so bad and the food so scarce, that it was the perfect recipe to "die even more slowly". Surviving situations like this requires not only physical strength, but, more importantly, one must have the correct mindset and emotional stability to endure the conflict. To become a survivor, much like Vladek, an individual must have endured, and essentially have overcome such challenging experiences. A person is classified a survivor when they have overcame situations where it is much easier to give up, than to persist and conquer.

Title: Re: VCE English Question Thread
Post by: cosine on July 11, 2015, 11:04:36 am
not the best..i been doing homework since  9am.. just a different idea

The events that Vladek went through can be seen as incomparable, the inhumane treatment, scarce supplies and the deadly diseases that the Jews had to witness overall, just to suffer another day. The conditions so bad and the food so scarce, that it was the perfect recipe to "die even more slowly". Surviving situations like this requires not only physical strength, but, more importantly, one must have the correct mindset and emotional stability to endure the conflict. To become a survivor, much like Vladek, an individual must have endured, and essentially have overcome such challenging experiences. A person is classified a survivor when they have overcame situations where it is much easier to give up, than to persist and conquer.

Cheers fruitcake, but do you have any specific improvements for me?

6. What changes do people experience, if any, after they have lived under tyranny for so long?

When people live under tyranny for so long, they start reform their lifestyles and habits, adjusting them just to survive the oppression. Vladek constantly stated that he remains a "strong man" throughout the Holocaust, “I was still strong, I could sit through the snow all night”. The war took a toll on Vladek, however, after the war had ended, Vladek had changed forever. The skills and ideas that Vladek developed during the war to survive became permanent, and he continued to express them in New York. The reason behind Vladek's substandard behaviours is because of the scarce conditions he endured, and the only way that he could preserver through the war was to cherish and hold on to anything he could get his hands on. Although the war made Vladek stronger and more respectable, it was slowly destroying Anja’s life. Anja was not as strong as Vladek, and so her post-war experience was a dark and depressing one. Anja fell victim to depression and was in a world of confusion, which eventually lead to her death. Anja committed suicide because she could not handle the stress of the war, the overflowing drama and paranoia.
Title: Re: VCE English Question Thread
Post by: _fruitcake_ on July 11, 2015, 11:52:02 am
Cheers fruitcake, but do you have any specific improvements for me?

6. What changes do people experience, if any, after they have lived under tyranny for so long?

When people live under tyranny for so long, they start to reform their lifestyles and habits, adjusting them just to survive the oppression. 'Adjusting themselves' would be a better phrase. When you  reform your lifestyle and habits, it means that you  are adapting to the situation. You change your mindset, personality..ect. So its best to summarise all this by 'Adjusting themselves'. It tackles the reform question through a holistic approach.

Vladek constantly stated that he remains a "strong man" throughout the Holocaust, “I was still strong, I could sit through the snow all night”. Good use of embedding quotes

The war took a toll on Vladek, however, after the war had ended, Vladek had changed forever. Very ambiguous sentence. Your vocabulary and structure is letting your ideas down. Its best to make the ideas clear then work on vocabulary. For example, a more clear way of expressing yourself here would be - The war took such a toll on Vladek, that it transformed him forever. Yes you get marked on vocabulary, but you lose way more if your ideas aren't clear. Keeping things short and clear allows you to write more, which means more ideas and concepts to talk about later.

The skills and ideas that Vladek developed during the war to survive became permanent, and he continued to express them in New York. The skills and ideas that Vladek had adopted to survive have become perpetual throughout his later life. The reoccurring problem with your answers is the subtle repetition of ideas. We know from your before sentences that he was in the Holocaust, which means that you dont have to explain later that 'during the war'. You still demonstrate solid ideas which is good.

The reason behind Vladek's substandard behaviours is due to  the scarce conditions he endured, and the only way that he could persevere throughout the war was to cherish and hold on to humanity. Although the war made Vladek stronger and more respectable, it was slowly destroying Anja’s life. Anja was not as strong as Vladek, and thus her post-war experience was dark and depressing. Anja fell victim to depression and was in a world of confusion, which ultimately lead to her death. Anja committed suicide because she could not handle the stress of the war, the overflowing drama and paranoia.Good ideas here... i feel like though your vocabulary is lacking in flair. This might just be me.. however it seems a bit dry. For example i bolded what could be change. This is just some ideas and by no means correct, its just me :) And by all means it could be wrong

What i would say of it is that its got very good content but vocabulary is at times lacking profoundness .
What I would recommend is to read complex-written books to expand on your vocabulary. Otherwise good effort. I did criticize heavily though and by no means all of what i wrote is correct, i am not an English teacher and i could be mistaken :P
Title: Re: VCE English Question Thread
Post by: cosine on July 11, 2015, 12:11:14 pm
What i would say of it is that its got very good content but vocabulary is at times lacking profoundness .
What I would recommend is to read complex-written books to expand on your vocabulary. Otherwise good effort. I did criticize heavily though and by no means all of what i wrote is correct, i am not an English teacher and i could be mistaken :P

Thank you so much _fruitcake_ that was actually very helpful, I will definitely be posting more soon!

Yah I know, my vocabulary needs improvements, my teachers tell me this too, so you're not mistaken :P
What books do you recommend me reading? Cheers.
Title: Re: VCE English Question Thread
Post by: _fruitcake_ on July 11, 2015, 01:02:59 pm
Thank you so much _fruitcake_ that was actually very helpful, I will definitely be posting more soon!

Yah I know, my vocabulary needs improvements, my teachers tell me this too, so you're not mistaken :P
What books do you recommend me reading? Cheers.

If you are doing conflict - Every man in this village is a liar is great one for conflict - very poetic as well in nature

- A People's Tragedy - very historical, its about the Russian revolution - a good book for revolution students but you can get good insight into language/conflict

In Cold Blood - complex themes and language in this novel

Charles Dickens - author

Frederick Forsyth - author

Title: Re: VCE English Question Thread
Post by: heids on July 11, 2015, 07:25:04 pm
1. What makes someone a survivor like Vladek?

To become a survivor, much like Vladek, an individual must have endured, and essentially have overcome such challenging experiences. The events that Vladek went through can be seen as incomparable, the inhumane treatment, scarce supplies and the deadly diseases that the Jews had to witness overall, just to suffer another day.  Rephrase: 'The inhumane treatment, scarce supplies and deadly diseases Vladek faced daily were incomparable in their brutality.' The conditions were [need a verb or else the sentence is incomplete/a fragment] so bad and the food so scarce, that it was the perfect recipe to "die even more slowly". To give up and “to die, it is easy”, “But you have to struggle for life!” Never write sentences that are solid quote.  Something more like: "While dying is presented as 'easy', [author] underscores the need to 'struggle for life'."  You are writing the sentence, not relying on quotes to write it for you; you should just pinch a word or phrase here and there and integrate them smoothly into your own sentence.  P.S. A person is classified as a survivor when they overcome situations where it is much easier to give up, then to persist and conquer. Surviving situations like this requires not only physical strength, but, more importantly, one must have the correct mindset and emotional stability to endure the conflict.  That is what it takes, to become a survivor like Vladek.

Notes:
> Tense.  Not important, but people often slip between tenses in essays which is bad, so practising writing ENTIRELY in present tense is helpful.

> Until the second-last sentence (which was your best sentence), you didn't actually fully address the question!  Boiling it down, you've said - to be a survivor, you must... er... survive.  You've more defined what a survivor is, than explained how one becomes a survivor, what characteristics someone must have to survive.  You should be delving more into what specific qualities the author is upholding, as qualities important for survival (be even more specific and use quotes/evidence/events to back it up).  Can you contrast him with another character who doesn't survive?  Just because it says 'like Vladek' doesn't mean you're restricted to solely discussing him, like you can say 'X character WASN'T a survivor like Vladek because, unlike Vladek, he...'

> Stick more closely to the text.  Extended responses aren't essays, sure, or phrased like essays (though, while Idk the text, there could actually possibly be an essay question based on this idea!), but you should still treat them like text response essays since that's what you're practising for.  In a text response, you CAN'T come to conclusions without referencing them through the lens of the text, like you can't say, 'surviving situations like this requires not only physical strength, but...' - you have to say 'Vladek's blah blah blah demonstrates that surviving situations...', or '[author] suggests that surviving situations...'.  This is why in text response those verbs, reveals, demonstrates, conveys, illustrates etc. are REALLY REALLY vital; they help you to move between evidence from the text, and what message that shows.  In context, sure, you're discussing the ideas, and the text is just a vehicle, but in text response, you can't ever make statements about life except in as far as the author or text demonstrates that.

> You need MORE specific textual evidence, exact examples where a certain quality of Vladek helps his survival or something.

> P.S. Picking on your quoting from the other ext response: Vladek constantly stated that he remains a "strong man" throughout the Holocaust, “I was still strong, I could sit through the snow all night”.  The 'strong man' quote is perfectly embedded, you see how it fits smoothly into your own sentence?  BUT, the second quote isn't embedded; to be grammatical, you'd need a semi-colon before the quote, but even then that's just tacking on a chunk without fitting it into your flow.  "Vladek's claim that he 'could sit through the snow all night', and constant statement that he remains a 'strong man'..."
Title: Re: VCE English Question Thread
Post by: cosine on July 11, 2015, 07:35:03 pm
Notes:
> Tense.  Not important, but people often slip between tenses in essays which is bad, so practising writing ENTIRELY in present tense is helpful.

> Until the second-last sentence (which was your best sentence), you didn't actually fully address the question!  Boiling it down, you've said - to be a survivor, you must... er... survive.  You've more defined what a survivor is, than explained how one becomes a survivor, what characteristics someone must have to survive.  You should be delving more into what specific qualities the author is upholding, as qualities important for survival (be even more specific and use quotes/evidence/events to back it up).  Can you contrast him with another character who doesn't survive?  Just because it says 'like Vladek' doesn't mean you're restricted to solely discussing him, like you can say 'X character WASN'T a survivor like Vladek because, unlike Vladek, he...'

> Stick more closely to the text.  Extended responses aren't essays, sure, or phrased like essays (though, while Idk the text, there could actually possibly be an essay question based on this idea!), but you should still treat them like text response essays since that's what you're practising for.  In a text response, you CAN'T come to conclusions without referencing them through the lens of the text, like you can't say, 'surviving situations like this requires not only physical strength, but...' - you have to say 'Vladek's blah blah blah demonstrates that surviving situations...', or '[author] suggests that surviving situations...'.  This is why in text response those verbs, reveals, demonstrates, conveys, illustrates etc. are REALLY REALLY vital; they help you to move between evidence from the text, and what message that shows.  In context, sure, you're discussing the ideas, and the text is just a vehicle, but in text response, you can't ever make statements about life except in as far as the author or text demonstrates that.

> You need MORE specific textual evidence, exact examples where a certain quality of Vladek helps his survival or something.

> P.S. Picking on your quoting from the other ext response: Vladek constantly stated that he remains a "strong man" throughout the Holocaust, “I was still strong, I could sit through the snow all night”.  The 'strong man' quote is perfectly embedded, you see how it fits smoothly into your own sentence?  BUT, the second quote isn't embedded; to be grammatical, you'd need a semi-colon before the quote, but even then that's just tacking on a chunk without fitting it into your flow.  "Vladek's claim that he 'could sit through the snow all night', and constant statement that he remains a 'strong man'..."

Thank you so much!! xD

Key things to work on:
- Embedding quotes into sentences, and ensuring they flow
- Vocabulary, mainly verbs to describe certain events
- Relate the ideas explored more to the text and use more examples from the text

Got it, cheers!
Title: Re: VCE English Question Thread
Post by: heids on July 11, 2015, 07:47:35 pm
Yes (love that you're listing it out, a brilliant exercise), and just delve deeper in to the topic in general.  Spend longer brainstorming the different implications, trying to be deeper and more specific and seeing it from different angles.  This one's harder to fix up, because it's a bit intangible - it's more about your THINKING than your writing.  (This was my comment about addressing the question more fully - not just saying what a survivor is, but how one becomes a survivor, what qualities they need.  And looking at other characters.)

Especially relate to the AUTHOR's perspective, btw - show awareness that the author presents the character Vladek in a specific way convey (verb <3) a message about what qualities one needs to be a survivor.
Title: Re: VCE English Question Thread
Post by: tashhhaaa on July 13, 2015, 12:19:26 am
Hey,

I feel like I'm all out of ideas for context (ID&B) and I want to shake things up a bit.

Are we allowed to discuss controversial or explicit material? For example (not that I will do this exactly), can we write a creative about a drug-affected drag queen? (totally random I know)

I go to a "religious" school so probably not in a SAC, but would this be ok for the exam? Or are examiners really conservative?
Title: Re: VCE English Question Thread
Post by: heids on July 13, 2015, 08:47:35 am
Hey,

I feel like I'm all out of ideas for context (ID&B) and I want to shake things up a bit.

Are we allowed to discuss controversial or explicit material? For example (not that I will do this exactly), can we write a creative about a drug-affected drag queen? (totally random I know)

I go to a "religious" school so probably not in a SAC, but would this be ok for the exam? Or are examiners really conservative?

I don't actually know, but I personally would say - do the bold, daring thing, rather than trying to play it safe (except, as you say, on SACs).  In general, I think examiners quite like something provocative, interesting and different because they're bored so sick of 'Identity is a key multidimensional aspect of our lives...'.  You want to wake them up and stand out.  Controversial is definitely fine; they've said before in exam reports that they can totally disagree with what you say as long as you write it well and it thoroughly relates to the prompt (avoid something super controversial like gay marriage though).  And most of them would be fine with explicit - you might by chance get someone conservative, but look, when are English teachers conservative?
Title: Re: VCE English Question Thread
Post by: tashhhaaa on July 13, 2015, 02:36:12 pm
I don't actually know, but I personally would say - do the bold, daring thing, rather than trying to play it safe (except, as you say, on SACs).  In general, I think examiners quite like something provocative, interesting and different because they're bored so sick of 'Identity is a key multidimensional aspect of our lives...'.  You want to wake them up and stand out.  Controversial is definitely fine; they've said before in exam reports that they can totally disagree with what you say as long as you write it well and it thoroughly relates to the prompt (avoid something super controversial like gay marriage though).  And most of them would be fine with explicit - you might by chance get someone conservative, but look, when are English teachers conservative?

I personally have a mix of teachers at my school: A couple who are very open minded (younger) and a couple who are ultra-conservative (do I need the hyphen lol) and biased...

I'd have to do a lot of research to write creatives or essays about things I've never experienced so I'm hoping it's worth it...
Title: Re: VCE English Question Thread
Post by: literally lauren on July 13, 2015, 08:09:16 pm
Hey,

I feel like I'm all out of ideas for context (ID&B) and I want to shake things up a bit.

Are we allowed to discuss controversial or explicit material? For example (not that I will do this exactly), can we write a creative about a drug-affected drag queen? (totally random I know)

I go to a "religious" school so probably not in a SAC, but would this be ok for the exam? Or are examiners really conservative?

I'm inclined to say no, because 'being controversial' just for the sake of being controversial doesn't get you any marks :p

If you had a valid reason for exploring a prompt from the perspective of a drugged-up transsexual, then by all means go for it. But if you're just writing a standard piece that shows how some people choose not to belong, or how sometimes our identity changes over time, then there's no real reason to be using that distinctive character or voice.

The question isn't really about whether the examiners will be on-board with more liberal ideas, or if they'll just faint at the mere mention of anything more scandalous than an ankle, but rather what your response has to gain by mentioning certain examples, or writing from a certain point of view! If you think you can offer a more profound insight by using an example that's a bit left-of-field, then I can't imagine too many assessors having a problem with it.

HOWEVER ( ;) ) the aim of the exam is to write what is safe. If you're like me and don't care too much about your score, then you can take some risks here and there (...pretty sure my Context piece contained more than a few silly puns) but ultimately, you don't know who will be reading your piece! I wrote in a completely different way for most of my SACs because I knew who my teacher was and what he was looking for. In the exam, though, you're writing what should suit the majority of teachers around the state.

This is why I generally advise against bringing up any religious or political examples unless they're purely expositional. The last thing you want is to write a piece about why Buddhism is a poor way of dealing with conflict, or why Julia Gillard had a weak sense of self... only to find out your assessor is the most hardcore Labour Buddhist in the state :p

The assessors are MEANT to put aside their biases when marking - and the good one's will - but they're only human, and it's hard for them not to be more critical if you've offended their sensibilities.

But it sounds like you're quite committed to fleshing out this idea and not doing what my friends did by thinking 'imma write from the point of view of a drunk hobo for the lolz' and provided you're doing the research and writing it in a sophisticated way, you should be absolutely fine.

Some optional reading, if you're looking for some different perspectives. Reading things like this can also help you get a sense of the 'voice' in a creative piece, which is just a fancy way of saying 'the pretend person you create as your writer.' Obviously you can't just open the story by saying 'Well as a drag queen junkie, I think identity is a fluid concept' because that would be ridiculous... so you have to find away of creating a character as well as telling a story or reflecting on certain ideas- that's the true challenge of imaginative writing.

Let us know if you've had any thoughts about how you'll go about this - I'd be interested to see how things turn out for this hypothetical cross-dressing drug addict (or whichever character you create) :P
Title: Re: VCE English Question Thread
Post by: heids on July 13, 2015, 08:30:59 pm
^ I always dive in and give it a go, figuring I'll assume Lauren's too busy to answer all questions and she'll contradict me when I'm wrong :P so take my answers with a teaspoon of salt
Thanks Lauren :D
Title: Re: VCE English Question Thread
Post by: tashhhaaa on July 13, 2015, 08:54:00 pm
I'm inclined to say no, because 'being controversial' just for the sake of being controversial doesn't get you any marks :p

If you had a valid reason for exploring a prompt from the perspective of a drugged-up transsexual, then by all means go for it. But if you're just writing a standard piece that shows how some people choose not to belong, or how sometimes our identity changes over time, then there's no real reason to be using that distinctive character or voice.

The question isn't really about whether the examiners will be on-board with more liberal ideas, or if they'll just faint at the mere mention of anything more scandalous than an ankle, but rather what your response has to gain by mentioning certain examples, or writing from a certain point of view! If you think you can offer a more profound insight by using an example that's a bit left-of-field, then I can't imagine too many assessors having a problem with it.

HOWEVER ( ;) ) the aim of the exam is to write what is safe. If you're like me and don't care too much about your score, then you can take some risks here and there (...pretty sure my Context piece contained more than a few silly puns) but ultimately, you don't know who will be reading your piece! I wrote in a completely different way for most of my SACs because I knew who my teacher was and what he was looking for. In the exam, though, you're writing what should suit the majority of teachers around the state.

This is why I generally advise against bringing up any religious or political examples unless they're purely expositional. The last thing you want is to write a piece about why Buddhism is a poor way of dealing with conflict, or why Julia Gillard had a weak sense of self... only to find out your assessor is the most hardcore Labour Buddhist in the state :p

The assessors are MEANT to put aside their biases when marking - and the good one's will - but they're only human, and it's hard for them not to be more critical if you've offended their sensibilities.

But it sounds like you're quite committed to fleshing out this idea and not doing what my friends did by thinking 'imma write from the point of view of a drunk hobo for the lolz' and provided you're doing the research and writing it in a sophisticated way, you should be absolutely fine.

Some optional reading, if you're looking for some different perspectives. Reading things like this can also help you get a sense of the 'voice' in a creative piece, which is just a fancy way of saying 'the pretend person you create as your writer.' Obviously you can't just open the story by saying 'Well as a drag queen junkie, I think identity is a fluid concept' because that would be ridiculous... so you have to find away of creating a character as well as telling a story or reflecting on certain ideas- that's the true challenge of imaginative writing.

Let us know if you've had any thoughts about how you'll go about this - I'd be interested to see how things turn out for this hypothetical cross-dressing drug addict (or whichever character you create) :P

Thank you so much Lauren, you're truly amazing

I almost died of laughter at this hahahah: "The question isn't really about whether the examiners will be on-board with more liberal ideas, or if they'll just faint at the mere mention of anything more scandalous than an ankle" & the Labour Buddhist part lolol

I've never written a creative in a SAC so that's another reason why I'd like to explore that idea. I'll definitely post or show you whatever I can come up with  ;)

I care about my score but I think I'm more afraid of boring my assessor than shocking them tbh...
Title: Re: VCE English Question Thread
Post by: tashhhaaa on July 13, 2015, 08:56:23 pm
^ I always dive in and give it a go, figuring I'll assume Lauren's too busy to answer all questions and she'll contradict me when I'm wrong :P so take my answers with a teaspoon of salt
Thanks Lauren :D

No your advice is just as valuable! Seriously, I look at your Guide to HHD almost religiously, and I have shamelessly stalked your posts to find answers to questions I have... (sorry)  8)
Title: Re: VCE English Question Thread
Post by: heids on July 13, 2015, 08:59:26 pm
No your advice is just as valuable! Seriously, I look at your Guide to HHD almost religiously, and I have shamelessly stalked your posts to find answers to questions I have... (sorry)  8)
Dw, I shamelessly stalk all Lauren's posts 8) 8) 8)
... and feel free to ask any questions directly :P !
Title: Re: VCE English Question Thread
Post by: cosine on July 13, 2015, 09:02:59 pm
After reading the book several times, and I have completed all comprehension questions, what do I do now? (text response essay in 3 weeks)
Title: Re: VCE English Question Thread
Post by: paper-back on July 13, 2015, 09:03:52 pm
How do you thoroughly explain and discuss evidence? I often find that my 'explanation' sentences are too short and when I do try to expand upon my explanations; my sentences become very repetitive

Thanks
Title: Re: VCE English Question Thread
Post by: tashhhaaa on July 13, 2015, 09:11:29 pm
Dw, I shamelessly stalk all Lauren's posts 8) 8) 8)
... and feel free to ask any questions directly :P !

will do next time, thank you!  :)
Title: Re: VCE English Question Thread
Post by: literally lauren on July 13, 2015, 10:53:17 pm
...she'll contradict me when I'm wrong...
There is no "wrong" in English :P

Well, there kind of is, but it's not like you're telling everyone to avoid reading their texts to avoid spoilers, or that they should spice up their essays by writing a paragraph in hieroglyphs or anything  ;D

In all honesty though, I'd absolutely agree that it's worth trying these sorts of ideas without too much fear of an assessor's reaction. You'd just have to be careful not to stick to stringently to a single method of writing when you know it's kinda risky.

Keeping Context essays interesting is definitely a worthy challenge though, so any attempts to invigorate things whilst still hitting all the important criteria will be much appreciated, I'm sure :)

I care about my score but I think I'm more afraid of boring my assessor than shocking them tbh...
It's probably better to fall on the side of caution than to deliberately "WOW" them with some never-before-seen insight into Identity and Belonging. That's not to say your piece shouldn't be interesting to read and write - if you can engage with the material and produce something that responds to the prompt in a creative and insightful way, then that's absolutely awesome.

But that's the trick - something that responds to the prompt - that's the core of the task that people often lose sight of in their quest to write what they want to write.

When it comes to practice, work on things from heaps of different angles. You can even go out of your way to choose a prompt that lets you construct the kind of piece you want to write. (Hell, I think I even wrote a few Conflict prompts in Yr 12 just so I could use certain examples or bookends ::) )

Depending on how easy you find creative writing, you might be fine going into the exam with a few options up your sleeve and a confidence in your ability to mould what you know to suit the prompt you're given. Or you may find that responding to a prompt in timed conditions is simply easier from an expository/hybrid perspective (...not that that's some indisputable fact; it's totally dependent on what kind of writer you are!) But planning or writing a piece now doesn't lock you into anything, so do all the experimentation you need before settling on a method with which you're comfortable :)

After reading the book several times, and I have completed all comprehension questions, what do I do now? (text response essay in 3 weeks)
If you feel like you need to do more reading of study guides or from the text in order to consolidate your knowledge, then do so. Otherwise, you're probably ready to start writing.

That doesn't mean you have to jump into writing essays, though. I usually recommend people just write some very general (almost non-Text Response-y) paragraphs about certain characters or themes. For instance, if you're studying Maus, (which I think you are, if I'm remembering a few pages of this thread ago correctly) then you could try writing a few hundred words about the character of Vladek. What are some of the important events that he experiences? What are his motivations or values? Do we sympathise with him? Next you might consider a certain theme, like 'suffering.' What does suffering look like in the text? Do characters suffer in different ways, or for different reasons? What is Spiegelman saying about the concept of suffering?

The writing that you'll be doing here isn't what you'll be writing in your SAC/the exam, per se; it's just to get your brain thinking about how you can connect important parts of the text. Later you'll start to do this in a more structured way, and eventually you'll be able to face the ultimate challenge - applying this knowledge to set prompts. :)

How do you thoroughly explain and discuss evidence? I often find that my 'explanation' sentences are too short and when I do try to expand upon my explanations; my sentences become very repetitive

Thanks
Which essay type(s) would this apply to? Or could you give us an example of the kinds of too short and/or repetitive sentences that you're writing? :)
Title: Re: VCE English Question Thread
Post by: paper-back on July 14, 2015, 11:00:40 pm
Text response

For example; the prompt: "Ordinary people with ordinary emotions, like the chorus, are often caught between sympathy towards, and horror at Medea's actions"

I'd get evidence that shows the chorus showing sympathy, but then I'd just be like "So this demonstrates that the chorus show's sympathy towards Medea's actions" without being able to thoroughly explain/discuss the significance of the quote and I end up with a paragraph that consists of the same re-phrased sentence throughout
Title: Re: VCE English Question Thread
Post by: lisax3 on July 15, 2015, 10:20:58 am
How would I improve my sentence structure? Whenever I'm writing essays my sentences seem to be too "clunky"?
Title: Re: VCE English Question Thread
Post by: cameotodd on July 15, 2015, 09:48:28 pm
Guys what is a 7/10 on an exam considered to be? (lower part of the upper range, mid range etc.)
and what study score do would three 7's net you in the end of year exam?
Title: Re: VCE English Question Thread
Post by: _fruitcake_ on July 15, 2015, 09:57:56 pm
Guys what is a 7/10 on an exam considered to be? (lower part of the upper range, mid range etc.)
and what study score do would three 7's net you in the end of year exam?

7/10 for exam is considered a 42/60 which is a low A.

GA1 - 70% = mid B+
GA2 - 70%= mid B+
GA3 - 70%= 42/60 in the exam is a low A

This would net u 34-37 depending on other factors.. if sacs scale down or up and rank as well
Title: Re: VCE English Question Thread
Post by: cameotodd on July 16, 2015, 03:52:41 pm
7/10 for exam is considered a 42/60 which is a low A.

GA1 - 70% = mid B+
GA2 - 70%= mid B+
GA3 - 70%= 42/60 in the exam is a low A

This would net u 34-37 depending on other factors.. if sacs scale down or up and rank as well

What about: GA1 - 89% (this is what I got in unit 3)
GA2 - 85% (prediction for unit 4)
Exam - 42/60
Title: Re: VCE English Question Thread
Post by: cosine on July 16, 2015, 04:03:04 pm
Guys how hard is it to get 8/10 in the exam for english on all three sections? What about 7/10?
Title: Re: VCE English Question Thread
Post by: appleandbee on July 16, 2015, 07:07:30 pm
Guys how hard is it to get 8/10 in the exam for english on all three sections? What about 7/10?

It hard to judge difficulty, as it varies from person to person. I'd say for an above average English student (as least in the context of VCE), 8/10 on all sections is pretty achievable (this judgement is based on the fact that I’m in an above average English cohort where around 70% of students get 40+), so you definitely don’t need to be a top student to get 8/10 essays. Your essays don’t need to be flawless either, as there is a significant difference between an 8/10 and 10/10 essay.  To put it in perspective though, 8-9% of the state will get an A+ on the English exam (total equivalent of 8/10 on all three sections. 7/10 is more achievable where around 20% of the state will get at least that.
Title: Re: VCE English Question Thread
Post by: heids on July 21, 2015, 01:32:24 pm
Text response

For example; the prompt: "Ordinary people with ordinary emotions, like the chorus, are often caught between sympathy towards, and horror at Medea's actions"

I'd get evidence that shows the chorus showing sympathy, but then I'd just be like "So this demonstrates that the chorus show's sympathy towards Medea's actions" without being able to thoroughly explain/discuss the significance of the quote and I end up with a paragraph that consists of the same re-phrased sentence throughout
Hoping Lauren will help me out here, a question I've often wanted to ask!

My best guess is that you need to spend some time developing more complex ideas.  If you have a paragraph with the sole argument of "the chorus shows sympathy towards Medea's actions", of course you're going to keep repeating it because you don't have anywhere further to go.

If you spend more time questioning the prompt, however, e.g.:
> what are 'ordinary people' and 'ordinary emotions'? in what way is the chorus 'ordinary'?
> are there other 'ordinary' characters? is the chorus 'ordinary' in contrast to other characters?
> is it different for other non/un/in-ordinary characters? do they have different feelings about her actions?
> do her actions evoke both sympathy and horror at once, or do some evoke sympathy and some other ones horror?
> ... etc.

then you might end up with paragraphs with more complex/broad focuses, rather than ones just trying to prove one narrow point.  If you have more than just one thing to say, you'll inevitably repeat yourself less.

And please don't put an apostrophe in 'shows', it hurts me :( :P

How would I improve my sentence structure? Whenever I'm writing essays my sentences seem to be too "clunky"?

Firstly: do you recognise their clunkiness, or need someone to point it out to you?

If you recognise it, great!  If you read a clunky sentence you've written, it's often just a matter of twisting and rearranging it in your mind until it sounds/feels 'right'.  Try asking yourself, 'What am I actually trying to say?'  Then, in really basic conversational English, verbally explain what your point is - that often makes it easier to make a good, clear, sophisticated, written sentences.  As long as you can pick your problems, you can generally sort them out (with some time and thought).

If you don't, get someone to sit down with you and highlight where you're going wrong with one of your essays.  If you don't recognise there's something wrong even when they point it out, you'll need them to explain the basic grammar rules.  Then, practise writing sentences of the same type, and get them to check that you've got the grammar right in those sentences too.

But most important: what exactly is 'clunky' about them?  Commas in the wrong spots?  Poor subject-verb agreement?  Run-on or fragmented sentences?  Using vocab incorrectly?  Short, choppy sentences?  Can you find types of sentences that you commonly make a mess off?  If you find specific weaknesses, then you can focus on avoiding them in all your prac pieces, or write lots of prac sentences of this type.

But remember, in the end, assessors realise that you're writing a 'first-draft' piece and don't expect a polished, beautiful piece.
Title: Re: VCE English Question Thread
Post by: tashhhaaa on July 21, 2015, 07:12:09 pm
hey y'all

are we allowed to use old VCAA texts as supplementary texts for context pieces?

I was thinking of writing an expository essay sometime soon using one of my school texts as well as an old Whose Reality text (however my context is ID&B)

if I can make it work would this be ok?
Title: Re: VCE English Question Thread
Post by: paper-back on July 22, 2015, 09:08:09 pm
Hoping Lauren will help me out here, a question I've often wanted to ask!

My best guess is that you need to spend some time developing more complex ideas.  If you have a paragraph with the sole argument of "the chorus shows sympathy towards Medea's actions", of course you're going to keep repeating it because you don't have anywhere further to go.

If you spend more time questioning the prompt, however, e.g.:
> what are 'ordinary people' and 'ordinary emotions'? in what way is the chorus 'ordinary'?
> are there other 'ordinary' characters? is the chorus 'ordinary' in contrast to other characters?
> is it different for other non/un/in-ordinary characters? do they have different feelings about her actions?
> do her actions evoke both sympathy and horror at once, or do some evoke sympathy and some other ones horror?
> ... etc.

then you might end up with paragraphs with more complex/broad focuses, rather than ones just trying to prove one narrow point.  If you have more than just one thing to say, you'll inevitably repeat yourself less.

And please don't put an apostrophe in 'shows', it hurts me :( :P

Thanks for the response bangali_lok! Sorry about the "shows", I've been unconsciously abusing the use of apostrophes in my sentences when typing since recently

Do I directly answer a question such as "what are 'ordinary people' and 'ordinary emotions'?" in a paragraph?
I'm worried that I might stray away from the prompt through asking/answering questions such as this
Title: Re: VCE English Question Thread
Post by: Adequace on July 22, 2015, 09:15:01 pm
Hey,

At school we're going to start learning how to write comparative essays to prepare for the upcoming study design changes. I'm quite ignorant on how to write a comparative essay, so if someone could enlighten me on the basics, that would be appreciated  :)
Title: Re: VCE English Question Thread
Post by: literally lauren on July 23, 2015, 05:30:38 pm
Text response

For example; the prompt: "Ordinary people with ordinary emotions, like the chorus, are often caught between sympathy towards, and horror at Medea's actions"

I'd get evidence that shows the chorus showing sympathy, but then I'd just be like "So this demonstrates that the chorus show's sympathy towards Medea's actions" without being able to thoroughly explain/discuss the significance of the quote and I end up with a paragraph that consists of the same re-phrased sentence throughout

To add to what bangali lok said, try asking these two questions:

   1. How do I know?
   2. Why is this significant?
(preferably in that order)
So in this case, if you have a piece of evidence that you think demonstrates the Chorus' sympathy towards Medea, start by outlining that evidence but then force yourself to be specific - how do you know they're demonstrating sympathy? It's not like they ever say 'Gee, poor old Medea, we sure do feel sympathetic towards her' and Euripides never includes a stage direction like [cue sympathetic sighs from the Chorus.] You've gotten that impression from the text somehow, so show your workings.
Then you start building back out again by asking what the point of this is. Why does this matter? What effect does it have on our view of the text? Or in this case - why do they feel sympathy? Saying 'they're sympathetic' without providing any reasons or interpretations can't get you far, so thinking about the bigger picture and 'zooming-out' can help immensely.

Assuming this was just one of the examples you'd use in your essay, the stuff mentioned above should be fine, but if you were trying to write a whole paragraph based around that one point, then you might be having a different problem. The sentence 'The Chorus feels sympathy towards Medea' is a point of evidence, not an idea - so if that was your focus for a whole paragraph, it's only natural for you to run out of things to say. So rather than breaking up a prompt into three/four different pieces of evidence, you could instead try to find three or four core ideas that are based on the ideas in the prompt, not just the key words it uses.

eg. the core of this prompt isn't just 'ordinary,' 'sympathy,' and 'horror' - it's that we cannot decide whether Medea is sympathetic or horrific. The point is that we're torn between those two extremes and don't know if her actions are justified. Yes, you'll talk about those key words in the bulk of your discussion, but the structure of your essay (the backbone of the skeleton, if you will) has to come from the core of the prompt. Only then can you 'flesh it out' to form the full body of your essay :)

hey y'all

are we allowed to use old VCAA texts as supplementary texts for context pieces?

I was thinking of writing an expository essay sometime soon using one of my school texts as well as an old Whose Reality text (however my context is ID&B)

if I can make it work would this be ok?
You can; just make absolutely sure that it's not on the current English list, or the current Lit. list either. If it is, the assessors are allowed to just cross out everything you've written on that text and only give you marks for what's left.

Even then, it can be worth researching some other examples to use instead since you'll have a chance to talk about something new in your piece, and assessor's who trek through hundreds of same-y essays usually appreciate that. But if you think a previous VCAA text demonstrates one of your key ideas really well, then you're free to use it.

Old school texts (eg. To Kill A Mockingbird or Macbeth or whatever people study in Year 9/10 etc.) are totally fine :)

Hey,

At school we're going to start learning how to write comparative essays to prepare for the upcoming study design changes. I'm quite ignorant on how to write a comparative essay, so if someone could enlighten me on the basics, that would be appreciated  :)

Ah, so this would be the entirely new essay style that VCAA has kindly given us very little information about :s
Usually I'm okay at decyphering the English Department's Study Designs, but I'm afraid until I've got a better idea what the actual task is about and what the criteria are, there's not much I can do to help, I'm afraid.

Based on what I know, you're studying two texts (right?) and then get given a prompt that you have to use to write about both of them?
eg. for Harry Potter and The Chronicles of Narnia
Prompt: Discuss the importance of friendship for the protagonists in the two texts.

I could be way off, so perhaps if you could let me know what your teacher/class are up to at the moment, I might be able to offer some advice as to how the comparison works. All I can say for the moment is it shouldn't be too separate... meaning you can't just have one paragraph on the first text, one on the second, and then a third para where you talk about them both. Instead, you'll be comparing them quite often, spending several sentences expanding on one idea and then transitioning to a relevant point in the other text.

At least that's how it is in other subjects with comparative tasks, and it's a format that I think enables you to write more sophisticated things, but for all I know, VCAA have gone in a totally different direction so...  :-\

Any details you've got would be greatly appreciated since very few schools I know have received any info at all :(
Title: Re: VCE English Question Thread
Post by: cosine on July 23, 2015, 05:31:56 pm
Hey guys

I forgot what to include in an introduction of a Text Response Essay. Also how/what is the best to start an intro?
May someone enlighten me please?

Thank you
Title: Re: VCE English Question Thread
Post by: Adequace on July 23, 2015, 08:59:35 pm
Thanks Lauren,

My teacher said that we'll comparing our text which is Romeo & Juilet to a modern film based off R&J, we'll be comparing how love is conveyed and portrayed between the two pieces.

For the structure, would I still have a contention in my intro and topic sentences in my body paragraphs?
Title: Re: VCE English Question Thread
Post by: heids on July 24, 2015, 11:32:10 am
Thanks Lauren, learnt much from your explanation :)) that was always an issue I struggled with last year.

I forgot what to include in an introduction of a Text Response Essay. Also how/what is the best to start an intro?
Read Ned Nerb's post and literally lauren's, they know more than me.

Different people like different intro structures and lengths, for instance my feeling is that Lauren prefers a briefer intro so you can dive into the 'real stuff'.  Pretty much all would agree that you must include:
> text/author name
> brief unpacking of the topic and what in general you're going to discuss throughout the essay
> your contention.

I tended towards a longer method like Ned Nerb does:
1.  Nice, memorised (but flexible) contextualising sentence, discussing the historical context of the text.
2.  Link this to text and topic (name and underline text and give author's full name) and discuss/unpack topic in whatever way you feel like, 1-2 sentences.
3.  Signpost arguments (subtly).  This doesn't mean 'write three sentences summing up your three paragraphs' (which was what I tended to do last year ::)), but still cover the main points you're going to discuss, the points that support your contention.
4.  Finish on an overall 'thesis statement', a 'profound' sentence that sums up your contention and mentions the author.
In year 12, I used two structures for this:
'Although X, ultimately [author] Y.' [Lauren taught this one too].
OR: 'Ultimately for [author], despite X, Y is the case.'

Get a bunch of intros from English Resources and Sample High Scoring Responses and any other high-scoring samples you get.  Break down their structure and see how they do things; write comments on them; compare how they do things differently, too.  Don't just read them, pull them to bits.

Here's a sample intro of mine - coincidentally, written exactly one year ago on July 24th! - to show you a decent-ish effort from someone at this time of the year.  But please go and find other, more skilled intros!!

Spoiler
'Despite being out of practice, it was a marvellous laugh.'  Humour is a central aspect of 'A Christmas Carol'.  Discuss.
By replacing agrarian lifestyles with machinery, the 1800s Industrial Revolution caused rapid population increase in Victorian London, exacerbating the disparity between rich and poor.  Against this backdrop of suffering and destitution, Charles Dickens in his novella 'A Christmas Carol' nonetheless paints a vibrant and family-oriented society powered by the festivities of Christmas. perhaps over-long in getting to the point For Dickens, this joy is fuelled by the ability to view the world as a child, using humour to transcend the bleakness of life.  Indeed, his narrative voice's rich word-plays engage the audience, enabling him to present the harshness of life in a nonetheless appealing way.  Yet perhaps the strongest role of humour in 'A Christmas Carol' is its very absence, as Dickens' darkening tone presents a harsh indictment of Victorian society heightened by the contrast interesting idea that challenges the prompt, but could be expressed better to make more sense.  Ultimately for Dickens, with the absence of a childlike appreciation of the world, society is in the thralls of 'Doom'.  This intro signposts too strongly.  It has three separate sentences that essentially state explicitly what each paragraph will be about – which can lead to repetitive topic sentences, a restricted focus, limited development through the essay, and the examiner getting bored of being told something too often.  There also doesn't seem to be one explicit thesis statement - for some reason I didn't follow the sentence structures I normally used in this intro.
Title: Re: VCE English Question Thread
Post by: cosine on July 24, 2015, 06:07:02 pm
Agh..

I don't know what to do. I have been sitting here trying to write something and I have come up with the most irrelevant introduction and a trash of a body paragraph. I read the book but I have no idea what to write... I know my topic/arguments but i cant put it to words :(
Title: Re: VCE English Question Thread
Post by: cameotodd on July 24, 2015, 07:10:56 pm
Hey guys when a questions says 'To what extent do you agree?' e.g (‘The women of In the Country of Men are totally powerless.’
To what extent do you agree?) Can you argue both ways, or do you have to show one side only?
Title: Re: VCE English Question Thread
Post by: _fruitcake_ on July 24, 2015, 07:30:33 pm
Hey guys when a questions says 'To what extent do you agree?' e.g (‘The women of In the Country of Men are totally powerless.’
To what extent do you agree?) Can you argue both ways, or do you have to show one side only?

You can certainly argue both ways. E.g if.. the women are totally powerless regarding (x) however (y). If you agree 80%, you talk about that 80% and you also talk about how u dont agree with the 20%
Title: Re: VCE English Question Thread
Post by: heids on July 25, 2015, 11:40:52 am
Agh..

I don't know what to do. I have been sitting here trying to write something and I have come up with the most irrelevant introduction and a trash of a body paragraph. I read the book but I have no idea what to write... I know my topic/arguments but i cant put it to words :(

Firstly, write out a plan; don't leave it in your head, write out a fairly detailed plan of exactly where you're heading, so you don't get lost in your ideas.  It's a bit like an anchor.  Then, explain to yourself in basic English what you're trying to say, like 'I'm trying to say that yeah, Medea does heaps of bad stuff, but actually Euripides is saying that we're not bad guys coz of the stuff we DO, but because of how we think of other people'.  (just making that up, don't have a clue of the play)

Then, write it down in more formal English ('While Medea does many bad actions, ultimately Euripides shows that we are not evil because of our actions, but rather because of how we think about other people'.)  Then, go over it and make your vocab more specific: like think about the word 'bad' - what sort of 'bad'? morally dubious, violent, selfish etc. (and hopefully you can finally be even more specific than those words).  What I wrote there was quite clunky ('... does many bad actions') and there's significant room for improvement because the ideas behind the sentence aren't expressed clearly, but you see how it was simply a more formal transcript of my thoughts?  That then gives me a jumping-off point to write a really GOOD sentence, with some time and thought.

Obviously in the exam you won't have the time to do this, but when you start panicking, you can do this process at triple-revs in your head.  Think, 'Okay, in basic words, what am I trying to say?'  Then think of a more formal way to say it, then clarify it more, then write.

Remember, if you can get your ideas out even in really basic English, that's better than not getting them out clearly in more sophisticated language, or worse, not writing them at all.

P.S. 'the most irrelevant introduction' sounds like you're letting your stress about how to write make you lose what to write.  DEFINITELY, write out a thorough plan, and go with the 'what am I trying to say' question first, writing out all your thoughts first and then going through the whole intro to make it better-written.  With practise, you'll learn to do the what and how together better, but for now get WHAT written down first and then stress about the HOW.  Remember, 'what' is more important!!
Title: Re: VCE English Question Thread
Post by: EspoirTron on July 25, 2015, 11:56:30 am
Hey guys when a questions says 'To what extent do you agree?' e.g (‘The women of In the Country of Men are totally powerless.’
To what extent do you agree?) Can you argue both ways, or do you have to show one side only?

Yes definitely go both ways. Incorporating both sides of the argument is crucial.
Title: Re: VCE English Question Thread
Post by: cameotodd on July 25, 2015, 03:19:11 pm
Yes definitely go both ways. Incorporating both sides of the argument is crucial.

Okay but then how do you conclude? Do you end with your own personal opinion about who holds more power or conclude in the sense that both genders hold power?
Title: Re: VCE English Question Thread
Post by: heids on July 25, 2015, 03:48:30 pm
Okay but then how do you conclude? Do you end with your own personal opinion about who holds more power or conclude in the sense that both genders hold power?
'Do you agree' is actually nothing to do with your own viewpoint; if you see a 'do you agree' prompt, rewrite it with 'discuss', because they mean exactly the same thing. 
Definitely don't have an essay discussing all viewpoints and then suddenly slam us at the end with 'well actually of the arguments I presented, this one's right and the others can all go to hell'.  If you present multiple viewpoints, that should be your contention, your argument throughout the whole essay: that in most situations X but in some situations Y.  Don't introduce something 'new', some new conclusion, at the end - it has to be consistent with your whole essay.

I shall now regurgitate slightly mutated slabs of what I've heard Lauren say, about challenging the prompt.  Lauren draws a spectrum that from memory looks something like this:

disagree | X ---------------------*---------------------X---------------------*---------------------X | agree

You have a prompt, and you have to sit somewhere on that spectrum, between 'I 100% agree that in ALL cases for ALL people this is definitely the case' and 'I 100% disagree, this is NEVER the case'.  The Xes are bad points to sit.  You don't want to fence-sit with 'yeah women don't have much power but actually men don't either and women do have some power, actually it's a bit of both', because it sounds wishy-washy and contradictory.  But you don't want to say 'yes all women are always, in every situation, totally powerless', because it's probably not true and as soon as someone can pick one example when women have power, your whole argument collapses.

So, you want to sit at one of the asterisk sweet spots, where you pretty much say, 'Yes, but... [in a few special cases we see...]' or 'No, but...'

'The women of ITCOM are powerless'.

So here, you'll want to think of different characters in the story, and different types of power (physical, power to make their own decisions, power to survive without men, power to influence others' lives, etc.).  So your contention may be that while in general women are relatively powerless, they do wield certain types of power in certain circumstances.  Plus you should go into the author's intentions and why they present it in a certain way: are they condemning the patriarchal society? are they suggesting that certain characteristics are important to achieve power? etc.  You could perhaps go off and talk a bit about how men are powerless, too, but that risks going off topic, because the topic is about women, not men.

Some good structures you could use in your intro or conclusion:
'Although X, ultimately Y.'
OR: 'Ultimately for [author], despite X, Y is the case.'

(Where 'X' is your 'but' in a 'Yes, but' or 'No, but' structure).
Title: Re: VCE English Question Thread
Post by: cameotodd on July 25, 2015, 06:13:04 pm
'Do you agree' is actually nothing to do with your own viewpoint; if you see a 'do you agree' prompt, rewrite it with 'discuss', because they mean exactly the same thing. 
Definitely don't have an essay discussing all viewpoints and then suddenly slam us at the end with 'well actually of the arguments I presented, this one's right and the others can all go to hell'.  If you present multiple viewpoints, that should be your contention, your argument throughout the whole essay: that in most situations X but in some situations Y.  Don't introduce something 'new', some new conclusion, at the end - it has to be consistent with your whole essay.

I shall now regurgitate slightly mutated slabs of what I've heard Lauren say, about challenging the prompt.  Lauren draws a spectrum that from memory looks something like this:

disagree | X ---------------------*---------------------X---------------------*---------------------X | agree

You have a prompt, and you have to sit somewhere on that spectrum, between 'I 100% agree that in ALL cases for ALL people this is definitely the case' and 'I 100% disagree, this is NEVER the case'.  The Xes are bad points to sit.  You don't want to fence-sit with 'yeah women don't have much power but actually men don't either and women do have some power, actually it's a bit of both', because it sounds wishy-washy and contradictory.  But you don't want to say 'yes all women are always, in every situation, totally powerless', because it's probably not true and as soon as someone can pick one example when women have power, your whole argument collapses.

So, you want to sit at one of the asterisk sweet spots, where you pretty much say, 'Yes, but... [in a few special cases we see...]' or 'No, but...'

'The women of ITCOM are powerless'.

So here, you'll want to think of different characters in the story, and different types of power (physical, power to make their own decisions, power to survive without men, power to influence others' lives, etc.).  So your contention may be that while in general women are relatively powerless, they do wield certain types of power in certain circumstances.  Plus you should go into the author's intentions and why they present it in a certain way: are they condemning the patriarchal society? are they suggesting that certain characteristics are important to achieve power? etc.  You could perhaps go off and talk a bit about how men are powerless, too, but that risks going off topic, because the topic is about women, not men.

Some good structures you could use in your intro or conclusion:
'Although X, ultimately Y.'
OR: 'Ultimately for [author], despite X, Y is the case.'

(Where 'X' is your 'but' in a 'Yes, but' or 'No, but' structure).

Wow this is great! Thank you so much for the pointers, they've helped me so much on how to attack 'discuss' and 'do you agree' prompts. Also, in the future, can I send you some of my essay for some feedback?

Thanks :D
Title: Re: VCE English Question Thread
Post by: heids on July 25, 2015, 07:19:56 pm
Wow this is great! Thank you so much for the pointers, they've helped me so much on how to attack 'discuss' and 'do you agree' prompts. Also, in the future, can I send you some of my essay for some feedback?

Thanks :D
Feel free to post essays in English Work Submission and Marking, I try to swing past all the essays there :) (slowly, still got one to do -.-)
Title: Re: VCE English Question Thread
Post by: chocolatecupcakes on July 25, 2015, 08:32:42 pm
I have to write an expository essay on the prompt "fear is at the heart of conflict". Am I allowed to explain other causes of conflict such as misunderstandings, and social hierarchies, or do I have to stick to fear?
Title: Re: VCE English Question Thread
Post by: Chang Feng on August 01, 2015, 10:00:06 pm
heyy, for context:
Does our external examples we use (for an expository essay) have to be based on the ideas of the set text or can they just be any ideas related to the prompt?
thanks.
Title: Re: VCE English Question Thread
Post by: Alter on August 01, 2015, 10:03:22 pm
Based on the prompt, imo. If they provide some ideas on the opposite end of the spectrum to your book, all the better. Your examples will be considered weaker if you can't connect them to the prompt or compare them to existing ideas you've already formed.
Title: Re: VCE English Question Thread
Post by: YellowTongue on August 03, 2015, 06:48:37 pm
I have a few questions regarding text response essays:

1. How much background information needs to be included in the essay. In a recent essay I wrote, I basically included a plot summary in the introduction. My teacher said that this was unnecessary, and that I should try and condense the amount of summarising. How can I condense this, and much much of the plot (if any) should be summarized throughout the essay?

2. Should "controversial" themes in the text be discussed in an essay? My teacher told me that a recent essay included some discussion that may offend certain readers, and that I should leave that sort of discussion out. Of course, in a SAC, I would do what the teacher prefers, however I fail to understand why this is wrong; as long as I am providing relevant references to the text.

3. When writing paragraphs, is it okay to break them up into smaller paragraphs for sub-ideas, or should this be avoided? I feel that some of my paragraphs are becoming too long, but my teacher has said to try and use one paragraph per "broad idea" from the text.

4. How to I write about different ways that the text can be interpreted? I know what to write about, I just on't know how to insert it into my essays without disrupting the flow.

Thank you in advance  ;D
Title: Re: VCE English Question Thread
Post by: thaaanyan on August 03, 2015, 08:21:29 pm
I have a few questions regarding text response essays:

1. How much background information needs to be included in the essay. In a recent essay I wrote, I basically included a plot summary in the introduction. My teacher said that this was unnecessary, and that I should try and condense the amount of summarising. How can I condense this, and much much of the plot (if any) should be summarized throughout the essay?

2. Should "controversial" themes in the text be discussed in an essay? My teacher told me that a recent essay included some discussion that may offend certain readers, and that I should leave that sort of discussion out. Of course, in a SAC, I would do what the teacher prefers, however I fail to understand why this is wrong; as long as I am providing relevant references to the text.

3. When writing paragraphs, is it okay to break them up into smaller paragraphs for sub-ideas, or should this be avoided? I feel that some of my paragraphs are becoming too long, but my teacher has said to try and use one paragraph per "broad idea" from the text.

4. How to I write about different ways that the text can be interpreted? I know what to write about, I just on't know how to insert it into my essays without disrupting the flow.

Thank you in advance  ;D


Hi!
1) You shouldn't be summarising at all in your essays, you should be providing evidence and quotes to back up you argument;
For example,
Topic sentence: Harry Potter's constant need to eat chocolate reveals the importance that food has in shaping our identity and sense of self. Harry's declaration, "It is chocolate! chocolate which feeds the soul" articulates the powerful presence of chocolate in providing not only physical nourishment but also food for the soul. Chocolate thus xyzxyzxyz
so like, there's no summary AT ALL it's just "quote" and then "how quote sheds light on topic sentence"
it's the same with intro: contention on topic, brief allusion to central ideas. no summary at all, though  you may have a brief contextualising sentence, like, "J.K Rowling's Harry Potter and the Chocolate explores the trials and tribulations of young baker, as his interaction with food through the saga acts to shape the landscape of his identity and the relationships he experiences. 

2) If the argument, though controversial, sheds light on the prompt in an insightful and clever manner, then you won't lose marks. cater to your teachers needs for the sac, then if your discussion fits the prompt, write what you'd like on the exam.

3) i'm not 100% sure what you mean? 1 paragraph = 1 idea + nuances of that idea for me. if your paragraph changes ideas then you should split paras them.

4) Erm, yeah, i'm not sure how to explain this. kind of like say your prompt is "Harry Potter is a hero"
and so in para 1 you go:
paragraph 1 = harry potter is a hero
but then in para 2 you say:
paragraph 2 = Harry's repeated indulgences in cake show his flaws as human being with selfish desires and therefore is a flawed hero
so i guess you've showed an alternate interpretation of his personality through addressing a different dimension of the prompt. this is a really kind of easy, broad way to do it; and you can manage it in the body paras and stuff through just explaining different ways things can be viewed.
i think it's also kinda nifty to end off on this note, through concluding on a broader/more expansive point. like, in my conclusion sometimes i'll say how the central understanding of the meaning of the text has changed through time in accordance to the reader/viewer's interpretation, and how modern interpretation differs from old interpretation.

so yeah! hope that helps. :) 
Title: Re: VCE English Question Thread
Post by: YellowTongue on August 03, 2015, 08:27:41 pm
Thank you  :)
Title: Re: VCE English Question Thread
Post by: Chang Feng on August 03, 2015, 10:10:24 pm
how do you write an engaging and interesting introduction and conclusion (for a standard expository essay for context)
Title: Re: VCE English Question Thread
Post by: heids on August 03, 2015, 10:28:14 pm
how do you write an engaging and interesting introduction and conclusion (for a standard expository essay for context)
See Re: English Q&A.
Ideas:
- start with an interesting metaphor sentence or two
(I'll go with an example from her lecture: the prompt was something like 'we can't stop change from happening' and her sample first sentence went like 'if you put a 10 foot high fence in the way of where people want to go all you'll really do is create a surge in the 11 foot ladder business'.)
- start with a brief 'creative' paragraph, eg inner monologue of one of your characters
- start by explaining an external example as 'interestingly' as possible (e.g. see Lauren's intro here for inspiration)
and then jump into a more proper intro afterwards.

And then could come back to these afterwards, it's called 'bookending'.
Title: Re: VCE English Question Thread
Post by: Chang Feng on August 03, 2015, 10:44:43 pm
for bookending; are we meant to signpost our body paragraph ideas through the utilization of ie. explaining external examples interestingly? or do you mean to first do that, and then continue on with the proper intro (of signposting paragraph ideas?)
thanks.
Title: Re: VCE English Question Thread
Post by: heids on August 03, 2015, 10:58:10 pm
for bookending; are we meant to signpost our body paragraph ideas through the utilization of ie. explaining external examples interestingly? or do you mean to first do that, and then continue on with the proper intro (of signposting paragraph ideas?)
thanks.
Either.  Context is 100% flexible, but it's easier to head into an intro-style paragraph after your more creative chunk, rather than trying to weave it in.  However it may be a briefer intro than you would otherwise have written.  Give a few a go (you don't have to write the whole essay to try out bookend + intro) and give them to your teacher for feedback!
Title: Re: VCE English Question Thread
Post by: Gogo14 on August 03, 2015, 11:11:20 pm
Hi everyone!!! >:( >:( :( :( :) :) :)

Im doing english 3/4 in 2 years time. I feel that my english vocabulary, comprehension and writing skills are poor. I need to get my foundation right so i can do well in 3/4

Any serious suggestions as to how to improve them before i take the 3/4?

P.s. I do not want a tutor
Title: Re: VCE English Question Thread
Post by: heids on August 11, 2015, 08:52:43 pm
Hi everyone!!! >:( >:( :( :( :) :) :)

Im doing english 3/4 in 2 years time. I feel that my english vocabulary, comprehension and writing skills are poor. I need to get my foundation right so i can do well in 3/4

Any serious suggestions as to how to improve them before i take the 3/4?

P.s. I do not want a tutor

So sorry for the slow reply, just missed this somehow!

Seriously, the main thing you can do is make yourself read, anything and everything, as often as can.  Search out books, magazines, the news, whatever you enjoy.

You could also go through any of the essays/pieces you write and find words that you use too much or don't quite express what you want; then use a thesaurus to find similar words, and google 'define >word<' to find sample sentences containing the words.  Through this make a word bank of new words, grouping them by theme.  Then try and use these words as much as possible!

Then you could try keeping a diary.  Sounds corny, but just writing more is the way to learn to write!  If possible, you could show your entries (or anything else you write) to someone, asking them to correct your expression and grammar.  I'm willing for you to send me some by PM. 

P.S. LOL'd at the last line :)
Title: Re: VCE English Question Thread
Post by: MorgnD on August 22, 2015, 05:12:07 pm
Just wondering if i can write my whose reality essay in an interview structure, also would this be considered as a creative piece or a hybrid? Thanks in advance.

Title: Re: VCE English Question Thread
Post by: pi on August 22, 2015, 05:24:02 pm
Just wondering if i can write my whose reality essay in an interview structure, also would this be considered as a creative piece or a hybrid? Thanks in advance.

I wrote my pieces in an interview style, which I consider to be hybrid.

Feel free to have a cheeky suss http://atarnotes.com/pages/?p=notes&a=feedback&id=748
Title: Re: VCE English Question Thread
Post by: MorgnD on August 22, 2015, 05:33:46 pm
I wrote my pieces in an interview style, which I consider to be hybrid.

Feel free to have a cheeky suss http://atarnotes.com/pages/?p=notes&a=feedback&id=748

Thanks, really helpful! Also are the people you put in the interview connected to the text you were studying or completely separate?
Title: Re: VCE English Question Thread
Post by: pi on August 22, 2015, 05:41:32 pm
Thanks, really helpful! Also are the people you put in the interview connected to the text you were studying or completely separate?

No worries!

Been a while now, but from memory:
#1 - fictional actress of fictional movie sequel to my text (text was a play that has a famous movie adaptation)
#2 - as above
#3 - real author's real daughter who was planning to really make that film
#4 - real author

Interviewer was fictional in all instances.

For #3 and #4 I did a *lot* of research and found all these interviews and transcripts from Frayn and his daughter, as well as another book by Frayn, to construct those interviews.
Title: Re: VCE English Question Thread
Post by: tashhhaaa on August 22, 2015, 10:32:45 pm
Hey guys,

Does anyone have any A+ pieces for Identity and Belonging?
Title: Re: VCE English Question Thread
Post by: paper-back on August 23, 2015, 03:54:26 pm
Is talking about terrorists in a context essay a risky thing to do?
Title: Re: VCE English Question Thread
Post by: thaaanyan on August 23, 2015, 04:18:13 pm
Is talking about terrorists in a context essay a risky thing to do?

???? no???? in what way??? (presuming this is for encountering conflict) if you're explaining that conflict may provoke extreme reactions from individuals, who feel politically and/or religiously threatened by differing ideologies and may therefore commit violent crimes, then it seems like a valid argument. i think the issue comes when you specifically stigmatise a minority group as the sole cause of terrorist behaviour - which isn't true, and likely isn't what your planning on arguing. so it's fine.
if you think it's risky because it's over done in context essays, then it kind of depends on how you argue it. old examples can be rewritten in a clever and insightful way. :) good-luck!
Title: Re: VCE English Question Thread
Post by: pi on August 23, 2015, 04:32:53 pm
Is talking about terrorists in a context essay a risky thing to do?

It's risky to be racist because you don't know who's reading your piece. If you're not conveying any racist messages, there shouldn't be an issue.
Title: Re: VCE English Question Thread
Post by: paper-back on August 23, 2015, 05:44:08 pm
It's risky to be racist because you don't know who's reading your piece. If you're not conveying any racist messages, there shouldn't be an issue.
???? no???? in what way??? (presuming this is for encountering conflict) if you're explaining that conflict may provoke extreme reactions from individuals, who feel politically and/or religiously threatened by differing ideologies and may therefore commit violent crimes, then it seems like a valid argument. i think the issue comes when you specifically stigmatise a minority group as the sole cause of terrorist behaviour - which isn't true, and likely isn't what your planning on arguing. so it's fine.
if you think it's risky because it's over done in context essays, then it kind of depends on how you argue it. old examples can be rewritten in a clever and insightful way. :) good-luck!

Thanks guys. I was planning to use it in Identity and belonging to show how the the notion of a sense of belonging can sometimes become detrimental (in regards to radicalization in Australia) - Is this even a good example?
No, I wasn't intending to use it in a racist/stereotypical manner
Just thought it might be a sensitive topic these days, I don't know why
Title: Re: VCE English Question Thread
Post by: thaaanyan on August 23, 2015, 10:35:34 pm
Thanks guys. I was planning to use it in Identity and belonging to show how the the notion of a sense of belonging can sometimes become detrimental (in regards to radicalization in Australia) - Is this even a good example?
No, I wasn't intending to use it in a racist/stereotypical manner
Just thought it might be a sensitive topic these days, I don't know why

Hey. I'm not 100% sure because i haven't studied this context at all; but reading that it's kind of like what your saying is the sense of belonging individuals feel to an extremist cause/group identity can cause them to partake in radical actions?? I guess you could consider it a motivator, though I would generally consider political oppression/ foreign intervention larger sources of extremist action.  You could look at the concept from a different perspective/add depth to this original idea: alienation and exclusion of identity; a lack of belonging due to the stigmatization and demonizing of ethnic and cultural groups results in minorities seeking to find endorsement and cultural acceptance in more radical places.

It is a plausible example, but there are tons of stuff you could use for  'the sense of belonging being detrimental.' From a global historical perspective you could talk about how majorities in power seek to enforce their understanding of identity for purposes of consolidating power. Like when nations take over other nations - settler sovereignty imposes sexuality, religion, language, skin colour, culture and jurisprudence in specific ways on First Peoples. Forcing people to conform to a specific identity can be seen as an extension of the genocidal apparatus of colonisation; as a more modern Australian example, think of the stolen generation, and how there was awful policy which aimed to 'breed out the colour' from Aboriginal First Peoples.
Or like,  the whole process of citizenship which initially revolved around the exclusion and expulsion of first peoples; making them 'other' or alien in their own nations, while transforming outside colonisers into exalted 'citizens.' Identity which is used to exclude and to marginalise is historically present throughout all global + historical imperial relations. I'm just throwing ideas out there and hoping they stick, but basically there's tons of scope here to have interesting and insightful discussion!

And while I recognise it was probably because you didn't want to alienate an examiner, i still would like to say thank- you for understanding and being wary of stereotypes. As someone who faces a lot of them on this issue specifically, I appreciate it  when people go out of their way to be more empathetic and understanding of what minority groups go through, or at least choose not proliferate some of the bigoted rhetoric that's prominent in the media these days. Thank-you to pi as well. :)

Hope this helps you, paper-back! :)
Title: Re: VCE English Question Thread
Post by: cosine on August 25, 2015, 09:45:48 pm
Is it a bad idea to use the same quote twice in a Language Analysis Essay? Obviously they will be at different points in the essay?
Title: Re: VCE English Question Thread
Post by: tashhhaaa on August 25, 2015, 09:50:37 pm
Is it a bad idea to use the same quote twice in a Language Analysis Essay? Obviously they will be at different points in the essay?

I wouldn't tbh, my teachers penalise this. However, if you're using different sections of the same quote it should be fine.

/but lets wait to see what the experts have to say ;)/
Title: Re: VCE English Question Thread
Post by: pi on August 25, 2015, 10:07:37 pm
Is it a bad idea to use the same quote twice in a Language Analysis Essay? Obviously they will be at different points in the essay?

I think it depends on how you structure your LA.

This is going way back, but from memory, the main two ways people structured an LA:
1) By "techniques"
2) By "arguments"

If you swear by method 1), which I think is an inferior method (but that's a post for another time and don't change now!), then maybe it would be fine to use the same quote twice. Although, it might sound clunky and disorganised, and hence, may not bode well with examiners.

If you swear by method 2), then you won't have this issue at all as that quote would probably be part of one particular argument, from which you can dissect the various "techniques" from it at once without it sounding clunky.

As above, keep waiting for the experts to have their say though :P
Title: Re: VCE English Question Thread
Post by: cosine on August 25, 2015, 10:18:33 pm
I think it depends on how you structure your LA.

This is going way back, but from memory, the main two ways people structured an LA:
1) By "techniques"
2) By "arguments"

If you swear by method 1), which I think is an inferior method (but that's a post for another time and don't change now!), then maybe it would be fine to use the same quote twice. Although, it might sound clunky and disorganised, and hence, may not bode well with examiners.

If you swear by method 2), then you won't have this issue at all as that quote would probably be part of one particular argument, from which you can dissect the various "techniques" from it at once without it sounding clunky.

As above, keep waiting for the experts to have their say though :P

Thank you guys, means a lot.

I always structure my LA by arguments, so I guess it's a bad idea.
Also for language analysis essays, how would you structure a certain paragraph?

For example, I use method 2 so topic sentence obviously addresses the argument. Then how can/should I structure the rest of the paragraph, including the link/last sentence?

Thank you
Title: Re: VCE English Question Thread
Post by: tashhhaaa on August 26, 2015, 01:11:58 am

I always structure my LA by arguments, so I guess it's a bad idea.

I think it depends on how you structure your LA.

This is going way back, but from memory, the main two ways people structured an LA:
1) By "techniques"
2) By "arguments"

If you swear by method 1), which I think is an inferior method (but that's a post for another time and don't change now!), then maybe it would be fine to use the same quote twice. Although, it might sound clunky and disorganised, and hence, may not bode well with examiners.

If you swear by method 2), then you won't have this issue at all as that quote would probably be part of one particular argument, from which you can dissect the various "techniques" from it at once without it sounding clunky.

As above, keep waiting for the experts to have their say though :P

Sorry for sounding like a noob but would structuring by techniques be writing a paragraph analysing all the uses of a particular technique, while by arguments gives you paragraphs that discuss multiple techniques?

I never even considered two ways of writing an LA, I just write depending on how I'm feeling...

thanks in advance & no worries cosine :p
Title: Re: VCE English Question Thread
Post by: pi on August 26, 2015, 09:47:52 am
^^Yup pretty much! :)
Title: Re: VCE English Question Thread
Post by: heids on August 26, 2015, 10:07:44 am
ALL PREPARE FOR A RIDICULOUSLY LONG SEMI-RELEVANT IDEAS DUMP POST because I have other stuff I have to do so I did this instead.

I wouldn't tbh, my teachers penalise this. However, if you're using different sections of the same quote it should be fine.

/but lets wait to see what the experts have to say ;)/
Absolutely no expert, but anyways... I don't see why you would, if you're structuring by argument (especially since I'd recommend 1-3 word quotes); however, if you draw different messages from the same quote, it'd be okay.  Do you have an example of where you've done it, Cosine?

Thank you guys, means a lot.

I always structure my LA by arguments, so I guess it's a bad idea.
Also for language analysis essays, how would you structure a certain paragraph?

For example, I use method 2 so topic sentence obviously addresses the argument. Then how can/should I structure the rest of the paragraph, including the link/last sentence?

Thank you
To structure:
TS - describe argument (and/or link to other paras, e.g. 'building on this fear, the author then...')
Then, repeat this cycle a few times with different quotes/techniques:
--- Pick one important 1-4 word quote (or technique, possibly)
--- Analyse the connotations or immediate effect - how does it make them feel/think?
--- Explain how this contributes to agreeing with the sub-argument
Link: draw back to overall article; how does agreeing with the sub-argument make the reader feel about the overall issue?  How does it make them agree with the author's main contention?

Will look at your samples and give you direct feedback today, btw :)

Sorry for sounding like a noob but would structuring by techniques be writing a paragraph analysing all the uses of a particular technique, while by arguments gives you paragraphs that discuss multiple techniques?

I never even considered two ways of writing an LA, I just write depending on how I'm feeling...
That's it :)

And, well, I kinda did a mixed version of these... it's pretty common that techniques and arguments overlap, like the techniques of a mocking tone, sarcasm and belittling the opposition = the argument that the opposition are pretty stupid so you shouldn't listen to them.  I called them 'approaches', which is kind of the broad strategies or methods they use to persuade; this is often a mixture of tone, general techniques, and the argument.  (Doesn't have to be chronological at all.)

EXAMPLE: an article I just invented on our need for high-speed trains in Australia, which goes something like this:
- anecdote: parents can't get to young daughter's interstate funeral when she died unexpectedly of car crash, because trains too slow (and for some reason can't get a plane?? lol)
- this would have been solved if only we had high-speed trains
- lists out economic/logical benefits of trains to society
- argues that it's only politicians with self-interest that are doing this
- say that we [authors] are doing everything for the benefits of the readers, we've researched thoroughly, here's our evidence, and more to the point we care about all you readers!

(note, by summarising/grouping it like this, it's already much easier to analyse - this can be a good approach when you get an article; think about how you'd summarise the article's general flow or methods to someone who hadn't read it)

There are also many other ways an author could argue on this point - like, rebutting all negative points, or appealing to patriotism and national pride (everyone else has got it, we can't stay behind the times!!).

Focus on the big picture (see the attachment); you want to show exactly how each technique or method of language usage makes the readers feel, and how this contributes to the wider sub-arguments, and how that contributes to the overall argument (plus show how arguments build on each other! it's all highly connected to make one big picture).  This way, you get a balance between the specifics (e.g. exactly how one individual word will make readers feel) and the overall ways the author is trying to persuade, and how all the individual words and techniques build up to these overall ways and thus the contention.  People often make the mistake of missing level 2, the subarguments, so they end up looking at a bunch of unconnected chronological techniques and trying to link each individual one back to the overall contention.

Anyway, here's kinda how I'd group analyse this article.

Well the post was looking massive
1.  Creating emotional pull to argument, arousing fear and concern about our current system.
- short, fast-paced, sharp, emotive sentences build up pressure and fear
- parents would feel 'what if it's my own child'?
- analyse words like 'devastated' or 'trapped' and how those specific words add to the impact and make us feel
- so now it's got us terrified about the current state of things, so we see the need for change
- provides huge relief by saying 'well here's the solution' - probably a change of tone, with one simple sentence after a para break saying 'look here's an easy solution' - got everyone so stressed they'll now grab at a solution (in theory)

2.  Listing logical benefits of high-speed trains to society's economy and wellbeing.
- complements first approach as readers realise it's not just emotional, it's also logical and in their own interests (targets both the thinking and the feeling members of the audience)
- discuss stuff like the stats used, how the author appeals to our need for income and holidays and how this will improve our quality of life and the country's economy etc.;
- simple, clear, formal sentences without really loaded adjectivey language, instead focus on stats and numbers and logical improvements - very clear tone shift to clearly and logically list out benefits and w/e

3.  Ridiculing opposition - attacking/undermining their credibility and thus the credibility of their views.
- look for the mocking/sarcastic/denigrating/belittling language - discuss how the author labels them in certain ways to make them look stupid or grasping
- by making the opposition look silly or selfish ('casting aspersions' on them), this subtly attacks or casts aspersions on their views; if such stupid/evil people are against high speed trains, then obviously not having high speed trains is bad; and also they're doing it for bad reasons so...
- NB analyse specific words (e.g.

4.  Contrastingly, building up credibility of author and thus of their views.
- could group with para 3 but would get too long, so make sure you directly link and contrast these methods
- last para - negatives of opposition/not having HS trains, this para - positives of author/having HS trains
- cites expert sources and their own knowledge and research - look reliable in their info, contrast to politicians who don't know a thing beyond self-interest
- also, through using second-person singular (e.g. 'we care about you') and shifting the focus to the reader, this contrasts with politicians who care only about themselves
- because the authors are nice and caring about the readers, the readers are more inclined to believe them
- also makes readers think that no high speed trains = no benefit to the reader, but high speed trains = benefits because they're not being implemented selfishly, but for the good of the readers.
And just to let you know, Lauren does it a slightly different way again. :P
Title: Re: VCE English Question Thread
Post by: tashhhaaa on August 26, 2015, 06:50:08 pm
ALL PREPARE FOR A RIDICULOUSLY LONG SEMI-RELEVANT IDEAS DUMP POST because I have other stuff I have to do so I did

wow, thank for you the awesome mini-guide! +1'ed!
Title: Re: VCE English Question Thread
Post by: paper-back on August 26, 2015, 07:30:12 pm
Hey. I'm not 100% sure because i haven't studied this context at all; but reading that it's kind of like what your saying is the sense of belonging individuals feel to an extremist cause/group identity can cause them to partake in radical actions?? I guess you could consider it a motivator, though I would generally consider political oppression/ foreign intervention larger sources of extremist action.  You could look at the concept from a different perspective/add depth to this original idea: alienation and exclusion of identity; a lack of belonging due to the stigmatization and demonizing of ethnic and cultural groups results in minorities seeking to find endorsement and cultural acceptance in more radical places.

It is a plausible example, but there are tons of stuff you could use for  'the sense of belonging being detrimental.' From a global historical perspective you could talk about how majorities in power seek to enforce their understanding of identity for purposes of consolidating power. Like when nations take over other nations - settler sovereignty imposes sexuality, religion, language, skin colour, culture and jurisprudence in specific ways on First Peoples. Forcing people to conform to a specific identity can be seen as an extension of the genocidal apparatus of colonisation; as a more modern Australian example, think of the stolen generation, and how there was awful policy which aimed to 'breed out the colour' from Aboriginal First Peoples.
Or like,  the whole process of citizenship which initially revolved around the exclusion and expulsion of first peoples; making them 'other' or alien in their own nations, while transforming outside colonisers into exalted 'citizens.' Identity which is used to exclude and to marginalise is historically present throughout all global + historical imperial relations. I'm just throwing ideas out there and hoping they stick, but basically there's tons of scope here to have interesting and insightful discussion!

And while I recognise it was probably because you didn't want to alienate an examiner, i still would like to say thank- you for understanding and being wary of stereotypes. As someone who faces a lot of them on this issue specifically, I appreciate it  when people go out of their way to be more empathetic and understanding of what minority groups go through, or at least choose not proliferate some of the bigoted rhetoric that's prominent in the media these days. Thank-you to pi as well. :)

Hope this helps you, paper-back! :)

Wow! You're a genius. I didn't even think of the stuff you've mentioned in your post before, although ironically enough I view these things occurring everyday
Thank you too, it helps a lot!
Title: Re: VCE English Question Thread
Post by: thaaanyan on August 27, 2015, 07:12:39 am
Wow! You're a genius. I didn't even think of the stuff you've mentioned in your post before, although ironically enough I view these things occurring everyday
Thank you too, it helps a lot!

Haha, really not a genius. Just passionate about social justice/ coming from a minority group that has faced struggle with trans-migrational identity -  grew up more sensitive to social structures and the like. Glad this info helps! I always felt the best thing about context is that it leaves a lot of lee way for exploration of examples which you're passionate about. :)
Title: Re: VCE English Question Thread
Post by: Maca 13 on August 29, 2015, 09:39:14 pm
Hey everyone,
In a context piece, if your tone changes from an analytical/scientific tone to a more personal/reflective tone, can you justify this in your explanatory piece by saying something like, 'the tone changes constantly because it represents the fluid nature of our identity, and how it cannot be fixed, just like the tone throughout the piece'? Or should I be safe and stick with one tone throughout the whole piece?
(Identity & Belonging Context)

Thanks! ;D
Title: Re: VCE English Question Thread
Post by: tashhhaaa on August 30, 2015, 01:02:27 am
Hey everyone,
In a context piece, if your tone changes from an analytical/scientific tone to a more personal/reflective tone, can you justify this in your explanatory piece by saying something like, 'the tone changes constantly because it represents the fluid nature of our identity, and how it cannot be fixed, just like the tone throughout the piece'? Or should I be safe and stick with one tone throughout the whole piece?
(Identity & Belonging Context)

Thanks! ;D

if you change the tone & language, you've got a hybrid :p
Title: Re: VCE English Question Thread
Post by: scarletmoon on August 31, 2015, 09:44:02 pm
I'm having trouble interpreting this prompt "In the film Mabo, the land plays such an important role it is like a character."
Specifically I am having trouble coming up with ideas to refute this as well as some complex ideas.
Title: Re: VCE English Question Thread
Post by: BlueHydrangea on September 06, 2015, 12:09:46 am
For Context (expository style) is it better to write about 1 idea in a paragraph and thoroughly explore it, or to have multiple ideas in a paragraph that are briefly explored?
Title: Re: VCE English Question Thread
Post by: Maca 13 on September 06, 2015, 11:24:58 am
For Context (expository style) is it better to write about 1 idea in a paragraph and thoroughly explore it, or to have multiple ideas in a paragraph that are briefly explored?
I think that it might be best to have one idea per paragraph that you can explore thoroughly because it'll show your teacher/assessor that you have a good understanding of the text and the context, and that you can express complex ideas that others might not have thought of (in my opinion, that would probably get you higher marks than having a bunch of unexplored ideas in the one paragraph).
That's what I would do, I hope this helps!  ;D
Title: Re: VCE English Question Thread
Post by: StupidProdigy on September 07, 2015, 08:47:18 pm
In the exam, I read that we can get another answer booklet. Can this be used as planning paper? It would just seem easier rather than flipping through pages to get to my rough plan on the blank pages...even though i probably should avoid wasting time writing a plan for the prompt...
Title: Re: VCE English Question Thread
Post by: frenchtom on September 12, 2015, 07:45:21 pm
Is anyone thinking of doing Medea for the text respond part of the exam?
I can't decide between Medea or the Thing Around your Neck.
Title: Re: VCE English Question Thread
Post by: thaaanyan on September 12, 2015, 08:43:56 pm
Is anyone thinking of doing Medea for the text respond part of the exam?
I can't decide between Medea or the Thing Around your Neck.

Yep! I'm doing Medea! I was gonna be totally biased an start extolling the virtues of Euripides, but then i just google Thing Around Your Neck and i found out it's by Chimamanda Ngozi Adichie who is an absolutely perfect human being. Americanah is the only thing i've read by her but omg. man. now i'm insanely jealous you got to learn both! (i have no advice btw i just wanted to say how flawless Adichie is)  pun unintended
Title: Re: VCE English Question Thread
Post by: tashhhaaa on September 12, 2015, 09:28:08 pm
Yep! I'm doing Medea! I was gonna be totally biased an start extolling the virtues of Euripides, but then i just google Thing Around Your Neck and i found out it's by Chimamanda Ngozi Adichie who is an absolutely perfect human being. Americanah is the only thing i've read by her but omg. man. now i'm insanely jealous you got to learn both! (i have no advice btw i just wanted to say how flawless Adichie is)  pun unintended

Is anyone doing The White Tiger??? Trying to decide between that and Medea
Title: Re: VCE English Question Thread
Post by: cosine on September 12, 2015, 10:11:25 pm
Is anyone doing The White Tiger??? Trying to decide between that and Medea

For anyone else doing Medea, I solely beg you, would you mind reading my Medea essays for feedback and improvements, and I can do the same for you too. Let me know.. :)
Title: Re: VCE English Question Thread
Post by: tashhhaaa on September 12, 2015, 10:16:05 pm
For anyone else doing Medea, I solely beg you, would you mind reading my Medea essays for feedback and improvements, and I can do the same for you too. Let me know.. :)

yeah sure! I have to write some first LOL but feel free to send through
Title: Re: VCE English Question Thread
Post by: literally lauren on September 13, 2015, 11:49:11 am
In the exam, I read that we can get another answer booklet. Can this be used as planning paper? It would just seem easier rather than flipping through pages to get to my rough plan on the blank pages...even though i probably should avoid wasting time writing a plan for the prompt...

Some invigilators will get snarky with you if you ask for an extra booklet before you need it. Hell, the ones in my Lit. exam wouldn't even give me one until I'd completely filled up the last one, and then I had to waste precious minutes while this woman slowly walked up to the front of the room and then sloooooowly walked back and handed me the booklet. I nearly took her eyes out with one of my 17 backup pens -.-

If you look through the writing booklets <http://www.vcaa.vic.edu.au/Documents/exams/english/2014/english%202014_answerbook.pdf> you'll notice you have a 'rough work only' page at the start of each Section. This should probably be sufficient, and if you're writing more than three pages for your plans, you're probably going overboard :p Alternatively, the actual question booklet is yours to keep and scribble on, so if you don't want to flip pages back and forth, you can just do your annotations and planning on the spare space in that booklet. Most people do this for L.A. anyway :)
~~~

Also, with regards to choosing your exam texts and working on them, some of you might find it helpful to start up a thread dedicated to the ones your studing. The Christmas Carol and All About Eve threads were quite popular last year, and it means you get a chance to air your ideas and build on your knowledge with one another. Obviously not all of them will take off if there aren't enough students willing to contribute, but the more popular texts (esp. Medea, TBL, HIV, All About Eve, and Wuthering Heights if this is anything to go by) should attract some attention. You could even bump a few of the ones from last year if you can find them - i think they're hovering around pages 10-13 on this board :p I know a lot of people head into this final stretch of the year in the mindset that they have to protect their precious ideas under lock and key lest they give anyone else the advantage, but ultimately, you'll gain so much from expressing your readings and bouncing off one another than you will if you just burrow down and stick your head in the sand for the next six weeks. Just from experience (my own, and my students') discussing things with people in the same boat as you can help immensely. Plus you might even get a few students from previous years dropping in to contribute their own understandings, as well as a bunch of tutors/teachers that float around these boards :)
Title: Re: VCE English Question Thread
Post by: duo0024 on September 13, 2015, 03:22:26 pm
Hey guys quick question. If you're given a prompt and it starts off with a quote, do you have to use that quote in your essay?
I've always thought the quote was to show that the prompt is applicable to the text/film.
Cheers  :)
Title: Re: VCE English Question Thread
Post by: Alter on September 13, 2015, 04:34:56 pm
You should definitely give reference to any quotes in TR prompts. They are there to guide you in a particular direction, and if you can pinpoint where from the text the quote comes from and what relevance it has to the question itself, the better off you will be. As a general rule of thumb, I try to do this before the end of the 1st body paragraph. If you do it later, it might seem like you crammed it in or you weren't really sure how it holds significance. Just my 2 cents.
Title: Re: VCE English Question Thread
Post by: StupidProdigy on September 15, 2015, 08:38:25 pm
For my context piece (essay), I'm thinking of having it structured with four pieces of evidence. Two of which are texts, one is a film (skin) and the other evidence will be personal stuff. Is four types too much or too little? And I'm also concerned about whether I shouldn't double up on using two texts as my supplementary sources? thanks
Title: Re: VCE English Question Thread
Post by: banaidil on September 17, 2015, 10:27:40 pm
Hi,
for a text response, how do you mention different interpretations in a sophisticated manner?
Title: Re: VCE English Question Thread
Post by: cosine on September 18, 2015, 09:31:37 pm
Can someone please mark my Language essay, the article is: Article 6 found: http://digeng.global2.vic.edu.au/files/2014/09/Using-Language-to-persuade-booklet-25ad3km.pdf

Title: Re: VCE English Question Thread
Post by: Photon on September 18, 2015, 10:18:38 pm
Hey, guys, I am a year 10 student and I am quite indecisive about what English subject I am going to be choosing next year. I am not sure what to choose between English Language or Standard English :(  I want to go into biomedicine and I still have not entirely made up my mind on my English subject for next year. Any tips?
Title: Re: VCE English Question Thread
Post by: odeaa on September 18, 2015, 10:25:35 pm
Hey, guys, I am a year 10 student and I am quite indecisive about what English subject I am going to be choosing next year. I am not sure what to choose between English Language or Standard English :(  I want to go into biomedicine and I still have not entirely made up my mind on my English subject for next year. Any tips?
Choose which one you enjoy, and are better at (usually one and the same)

Personally, I find English language much more relevant to the real world, and it's also much easier to study for
Title: Re: VCE English Question Thread
Post by: chemzy on September 19, 2015, 02:56:42 pm
Hey guys I'm an year 11 student this year and I was wondering how I should be preparing for next year English.

I feel that my language is pretty good, but I always stuff up because I'm not prepared for the topic given to us on the assessment task.

Should I be writing an essay each week? How do I improve my ability to understand texts and write for any prompt?

I also feel like I need a very good English tutor. If anyone knows an English Tutor in South eastern suburbs, could you please help me out? Thank you.
Title: Re: VCE English Question Thread
Post by: Coffee on September 19, 2015, 03:47:55 pm
Hey guys I'm an year 11 student this year and I was wondering how I should be preparing for next year English.

I feel that my language is pretty good, but I always stuff up because I'm not prepared for the topic given to us on the assessment task.

Should I be writing an essay each week? How do I improve my ability to understand texts and write for any prompt?

I also feel like I need a very good English tutor. If anyone knows an English Tutor in South eastern suburbs, could you please help me out? Thank you.
Why aren't you prepared for the topic? Is it simply a lack of knowledge and understanding about your texts? Are you exploring them in depth enough? Say you're studying Medea and you have a great understanding of passion/rage/revenge/etc and you know you can write anything on those topics, but then you get to the exam and it's asking you about how gender is explored in the play and you have no idea. Is it like this, or something else? If this is the case then you just need to work on understanding your texts better. Read them and reread them, use resources (mind maps, flash cards, etc) to connect and develop ideas. If you know your text well you should see improvement.

Writing an essay a week can't hurt and it would be helpful with any upcoming SACs and end of Year 11 exams. It might also be helpful though to write paragraphs, intros, conclusions (whatever you're struggling with). Don't feel like you need to write an entire essay just yet. Basically, whilst its great to employ these sort of study habits and whatnot early, you don't want to burn yourself out before Year 12 even starts. And obviously its important you have a good understanding of the novel and its themes/characters/etc before you start writing essays.

For tutoring, although I do think that you can do well enough without one if you and your teacher are willing to put in the work, if you absolutely feel like you need one then I'd recommend taking a look at the following websites:
http://www.gumtree.com.au/
http://www.tutorfinder.com.au/region/melbourne/
http://bettereducation.com.au/tutor/melbourne/tutors.aspx
ATARNotes actually does have its own section of awesome tutors but it doesn't look like its up at the moment. If you're willing to wait a bit though maybe keep an eye out.
Title: Re: VCE English Question Thread
Post by: bobisnotmyname on September 19, 2015, 05:45:56 pm
How many people do you think will be doing the white tiger this year, like a rough percentage
Title: Re: VCE English Question Thread
Post by: cosine on September 23, 2015, 08:54:48 am
Can someone please mark my Language essay, the article is: Article 6 found: http://digeng.global2.vic.edu.au/files/2014/09/Using-Language-to-persuade-booklet-25ad3km.pdf

Someone please..?
Title: Re: VCE English Question Thread
Post by: heids on September 23, 2015, 09:12:40 am
Someone please..?
I'd just started doing it just then :)

For my context piece (essay), I'm thinking of having it structured with four pieces of evidence. Two of which are texts, one is a film (skin) and the other evidence will be personal stuff. Is four types too much or too little? And I'm also concerned about whether I shouldn't double up on using two texts as my supplementary sources? thanks

Do you mean that you're structuring the essay ROUND the evidence?  Rather, your paragraphs should be structured round IDEAS about the prompt, and then you’d use relevant evidence to back that up.

It’d really be better to have a wider selection of examples (including some different types of examples, such as historical examples – you’re right, I don’t think you should use just texts).  Rather than saying ‘I will use these four pieces of evidence in my essay’, ideally you should say ‘I have these ten (or twenty) pieces of evidence I know, and then I’ll choose the four most relevant pieces’.
Title: Re: VCE English Question Thread
Post by: heids on September 23, 2015, 11:34:34 am
Feedback... hope it makes some sense, I got distracted doing other stuff all the way through so it was kinda disjointed marking :)
Title: Re: VCE English Question Thread
Post by: Escobar on September 23, 2015, 12:13:50 pm
roughly how long is a good essay?
my teacher says 600 words is enough at the end of the year but I feel like it is a bit short
Title: Re: VCE English Question Thread
Post by: heids on September 23, 2015, 12:18:13 pm
roughly how long is a good essay?
my teacher says 600 words is enough at the end of the year but I feel like it is a bit short

Yes, I'd say more like 1000 words (assuming you're thinking end of Unit 3/4) - but it depends on how concisely you write.  A concise 800 word essay may say as much as a waffly 1200 word essay (and will be better) - if you write really concisely, 8-900 could be fine.  When your essay is too short, you can't go into either enough breadth or depth (can't introduce as many examples and can't analyse as deeply and thoroughly), but you just don't have time to go on for like 1500 words.
Title: Re: VCE English Question Thread
Post by: paper-back on September 23, 2015, 12:44:49 pm
When doing Language analysis, I sometimes have trouble explaining 'HOW'/'WHY' an author uses a particular phrase/technique to persuade the reader even though I have a faint idea in my head, it's like I'm unable to express this idea in words though I know what it's trying to do. How do I fix this?
I also have trouble elaborating on my 'HOW' and 'WHY' sentences without them becoming too repetitive

EDIT: What are some 'synonyms' for: Through doing so/By doing so? (Need some L.A. sentence starters)
Title: Re: VCE English Question Thread
Post by: cameotodd on September 23, 2015, 01:35:10 pm
When doing Language analysis, I sometimes have trouble explaining 'HOW'/'WHY' an author uses a particular phrase/technique to persuade the reader even though I have a faint idea in my head, it's like I'm unable to express this idea in words though I know what it's trying to do. How do I fix this?
I also have trouble elaborating on my 'HOW' and 'WHY' sentences without them becoming too repetitive

I sometimes have this exact same problem :(
Title: Re: VCE English Question Thread
Post by: Splash-Tackle-Flail on September 23, 2015, 05:59:15 pm
For those of you guys who have done english, how many practice exams did you do? And how many practice essays (as in practice exam essays not prep for SACs or anything)? It's just so much harder to do with it being 3 hours and all..
Title: Re: VCE English Question Thread
Post by: appleandbee on September 23, 2015, 06:30:56 pm
For those of you guys who have done english, how many practice exams did you do? And how many practice essays (as in practice exam essays not prep for SACs or anything)? It's just so much harder to do with it being 3 hours and all..

My English tutor did three practice exams purely for the experience of writing for three hours straight (excluding the school trial). He didn't do more due practicality reasons like getting burnt out. He only did this a fortnight before the actual exam. In terms of practice essays, he did more plans than essays because he found them more useful (he wrote about 25 plans during the term 3 holidays and about 5-10 practice essays). He wrote timed and untimed essays simultaneously so he could get the standard of the timed up to the untimed.

If three hours is too long, you could just write two essays in a row or two essays with a half an hour break in between, which can still help you build up your stamina.
Title: Re: VCE English Question Thread
Post by: appleandbee on September 23, 2015, 09:20:41 pm
When doing Language analysis, I sometimes have trouble explaining 'HOW'/'WHY' an author uses a particular phrase/technique to persuade the reader even though I have a faint idea in my head, it's like I'm unable to express this idea in words though I know what it's trying to do. How do I fix this?
I also have trouble elaborating on my 'HOW' and 'WHY' sentences without them becoming too repetitive

EDIT: What are some 'synonyms' for: Through doing so/By doing so? (Need some L.A. sentence starters)

Can't help with the first part of the question, but for the 'synomyms', if you are linking the technique to the intended purpose/why, you could write something like 'ultimately, this is intended to...', 'the writer does (insert effect) by using...'or 'This is designed to... '. If you are trying to link the intended purpose and effect you could write something like 'this may act on the readers' sense of...', 'consequently, readers may harbour a sense of...', 'in effect, readers may experience...', 'Henceforth, (insert emotion) may evoke readers towards...', ''in addition, (insert emotion) may be aroused in the readers', or 'subsequently, readers are invited to consider/asked to question/confronted with/subjected to'.

Hope that this somewhat helps!

Title: Re: VCE English Question Thread
Post by: cosine on September 23, 2015, 11:28:47 pm
When doing Language analysis, I sometimes have trouble explaining 'HOW'/'WHY' an author uses a particular phrase/technique to persuade the reader even though I have a faint idea in my head, it's like I'm unable to express this idea in words though I know what it's trying to do. How do I fix this?
I also have trouble elaborating on my 'HOW' and 'WHY' sentences without them becoming too repetitive

EDIT: What are some 'synonyms' for: Through doing so/By doing so? (Need some L.A. sentence starters)

By asserting ____
By incorporating____
The incorporation of _____
The assertion of ______
The inclusion of______
By including_______
Mentioning that _______
Raising awareness of______
Doing so employs_____
Title: Re: VCE English Question Thread
Post by: literally lauren on September 24, 2015, 09:38:04 am
When doing Language analysis, I sometimes have trouble explaining 'HOW'/'WHY' an author uses a particular phrase/technique to persuade the reader even though I have a faint idea in my head, it's like I'm unable to express this idea in words though I know what it's trying to do. How do I fix this?
I also have trouble elaborating on my 'HOW' and 'WHY' sentences without them becoming too repetitive
I sometimes have this exact same problem :(

All you've gotta do is ask yourself the right questions at the right moment.

The what-how-why thing (explanation here for those who don't know what I'm talking about) is a strategy that's designed around being one step ahead of the assessors. Basically, if you fail to mention these areas in your analysis, then the assessors are going to be asking questions like 'but WHAT language is being used??' 'HOW are readers made to feel this way??' 'WHY is the author doing this??' ~So, the aim is to address these questions before they can even be asked.

You might find it helpful to flesh out each area a bit so that rather than sitting there thinking 'what do I put in the HOW section?' you can develop a formula that prevents you from getting lost or stuck. Each of the three components has a variety of sub-components, which I'll list below, but keep in mind that not all of them have to be mentioned every single time. Good paragraphs will focus primarily on the WHAT, a fair bit on the HOW, and then just a little of the WHY at the end of major points.

1. 'WHAT'
- What is the author trying to do?
- What devices or techniques are being employed? (note: this won't always be applicable; sometimes you're just talking about language more generally, ,which is fine)
- What language is there that provides evidence for this? (ie, quote here)
- What is the significance of this? What sense or idea is being created?
- What is the intended effect (note: this links into the HOW section to make things smoother)

2. 'HOW'
- How are the readers likely to think/feel?
- How are these thoughts/feelings/beliefs created through the language? (<<--IMPORTANT!!)
- How are certain audience members targeted? (note: only do this when the audience is given to you by the Background Information paragraph)
- How does the author want readers to view something as a result (eg. if the author is eliciting sadness through an anecdote, what is this connecting to? Where is that sadness directed? What key player is being manipulated in readers' minds?)
- How does this help the author argue their contention? (note: links to WHY)

3. 'WHY'
- Why does the author want audiences to think/feel/believe this?
- Why are the readers inclined to think/feel/believe this?
- Why does the author draw attention to certain aspects of a key player, or emphasise certain ideas?
- Why does the author's use of language create this effect? (<<--IMPORTANT!!)

Often, the lines will blur between each category, and it's entirely possible that you'll have sentences in your analysis that do a bit of both WHAT and HOW or HOW and WHY etc.

Now, in terms of specificity, you always want to focus on the language! Analysis like:

The author's use of inclusive language is designed to incorporate readers in his views, and thus compels them to agree with his contention

is waaaaay to superficial. WHAT language is being used? HOW does this language affect readers? And WHY is this language effective from the author's point of view?

A revised version might be something like:

The author's use of inclusive language in the phrase "we must act now" is designed to elicit a sense of collective responsibility and instill the notion that readers have a duty to do something to prevent the exacerbation of climate change. The fact that the author shifts this onus onto the collective "we" also engenders unity amongst the readership and even aligns the author himself alongside the audience, so his call to action creates a communal, unified sense of obligation, encouraging readers to do their part. This also allows the author to create a guilt-inducing scenario whereby those who do not act are inferred to be letting down the community; hence, the inclusivity establishes a greater amount of accountability with regards to climate action.

Yes, it's way longer, and you wouldn't have to do this every time, but that's the level of detail you can go to with just a few words to analyse. This gets easier when you have a whole article to deal with, and you can actually link the language together and comment on the overall effect (eg. with the above excerpt, I'd probably connect this to an instance where the author is outlining what actions can be taken, or where the author explicates the reasons why this is necessary.) Good analysis will tie together in logical places, and this is a characteristic of high-range essays because too many mid-range ones just run points of analysis together like:

'The author uses inclusive language which compels readers to act. Furthermore, the imperative language like "have to" and "should" makes readers more likely to take responsibility. The author also uses an aggressive tone. Moreover, there is a rhetorical question. Likewise, the dichotomy that is created here...' etc. etc.

Ignoring the fact that the analysis is really brief, you can see that the linking words like 'furthermore' and 'also' are doing nothing to unite the discussion. It reads like the student has just compiled a dot point list of things to analyse, and then stuck them in a random order in the paragraph, which you'd obviously want to avoid.

With regards to being more specific, don't use cop-outs like '...making them agree with the author.' or '....which reinforces the author's contention.' Try to clarify which part of the author's argument readers are being made to agree with, otherwise you'll end up zooming out too far, too quickly. Angle yourself towards sub-arguments instead; making connections from a quote to the overall contention isn't really efficient, and you'll likely descend into repetition quity quickly, so instead, link the language to a key player/sub-argument, and then at the end of the paragraph, link that sub-argument to the contention.

Once your theoretical approach is a little more steady, you just need to practise executing it until it becomes more automatic. Rather than doing whole/timed essays, maybe just focus on little sections for awhile until you get the hang of it. bangali_lok's weekly Letter to the Editor initiative is a great place to start :)

For those of you guys who have done english, how many practice exams did you do? And how many practice essays (as in practice exam essays not prep for SACs or anything)? It's just so much harder to do with it being 3 hours and all..
I realise I'm probably in a minority here, but I wouldn't have even made double digits with my practice essays in the last stretch of the year. I probably wrote three for Sec.A, three and a half for Sec.B and one for Sec.C - BUT they were the most efficient learning-tool essays I ever wrote. Churning wasn't how I learnt, so I'd write one piece, but then wring everything I could from it: getting feedback from my teacher or other sources, evaluating my own performance, editing anything I was unhappy with, considering any weak points or areas that I wasn't confident about, comparing it to other essays in terms of what was done better/worse, etc. In terms of full three hour sessions, don't force yourself into these unless you want to practice timing. You don't want to be doing huge three hour study sessions only to find you need to work on tiny things like quote integration or topic sentences. It's easy to be intimidated by the concept of writing for three hours straight, but for now, I think it'd be more helpful if you just think about it as 3 x 1 hour sessions rather than 1 x 3 hour one, so break it down first, and then you can concern yourself with getting through the whole 180 minutes once you've tidied up all the smaller stuff.

Also, appleandbee's point about gradually acclimatising yourself to the three hours is a great idea; enforcing full exam conditions out of the blue is unlikely to yield the desired results, so take it slow and try to maintain your quality of writing. That way, cutting down on time will feel easier and more gradual, as opposed to some insurmountable hurdle that holds you back :)
Title: Re: VCE English Question Thread
Post by: Aatif on September 24, 2015, 01:22:21 pm
What is a word/phrase I could use to describe the time period of King Henry IV Part 1? For example, you could describe the time period when Shakespeare wrote the play as Elizabethan.
Title: Re: VCE English Question Thread
Post by: literally lauren on September 24, 2015, 01:32:39 pm
What is a word/phrase I could use to describe the time period of King Henry IV Part 1? For example, you could describe the time period when Shakespeare wrote the play as Elizabethan.

Depends what kind of discussion you're going for. If the values of the time period or the idea of kingship come up in the prompt, then mentioning the Elizabethan period could be a good opening. But if you get a prompt like 'Hotspur's honesty is his undoing,' then you don't want to begin by bringing up some irrelevant point about Elizabethan times. It's not like you lose marks for having words like that in the intro, but they can come across as rote-learned and pointless if they don't connect with the overall discussion.

On a more general note, you could just refer to the political instability and uncertainty that shrouded England back in the day (-look up the end of Q Elizabeth I's reign if you don't know about this) or even just the tumultuous world that Shakespeare creates within the play. Other than that, delving into the time periods isn't really necessary; 'Elizabethan' would be the one I'd stick to, since that's what Shakespeare's plays are formally associated with (excluding the later Jacobean ones.) Plus, your analysis is more defined by the parts of the text you choose to comment on, and how you do so, rather than any extraneous commentary about the historical details.
Title: Re: VCE English Question Thread
Post by: tashhhaaa on September 24, 2015, 04:07:43 pm
guys,

what the hell are key players in language analysis? I've NEVER heard this until I saw it somewhere one day on a thread

edit: it happens to be that Lauren mentioned this on her above post, but I still have no idea lol  8)
Title: Re: VCE English Question Thread
Post by: literally lauren on September 24, 2015, 05:14:40 pm
guys,

what the hell are key players in language analysis? I've NEVER heard this until I saw it somewhere one day on a thread

edit: it happens to be that Lauren mentioned this on her above post, but I still have no idea lol  8)

Don't worry; it's kind of something I made up/ adapted from existing L.A. stuff.
Brief explanation here; just skip over the part about the orals. But the easiest way to think about it is that 'key players' are things which sub-arguments are based on.

In every piece, the author will have a variety of sub-arguments which support their overall contention, and in my opinion, the best way to structure an L.A. response is to focus on one sub-argument or 'key player' per paragraph.

Simply put: if the contention is 'People should read books instead of e-books' (see: 2012 exam) then one of the sub-arguments would be: 'Books are more permanent and reliable than e-books.' So the key player is: 'the permanence/reliability/trustworthiness of hard copies of books.' Then, one of your paragraphs can explore how the author establishes this idea of books being more reliable, and at the end of the paragraph, you'll link this sub-argument back to the overall contention.

Hopefully that clears things up; let me know if there's anything about this you don't understand :)
Title: Re: VCE English Question Thread
Post by: tashhhaaa on September 25, 2015, 12:38:46 am
Don't worry; it's kind of something I made up/ adapted from existing L.A. stuff.
Brief explanation here; just skip over the part about the orals. But the easiest way to think about it is that 'key players' are things which sub-arguments are based on.

In every piece, the author will have a variety of sub-arguments which support their overall contention, and in my opinion, the best way to structure an L.A. response is to focus on one sub-argument or 'key player' per paragraph.

Simply put: if the contention is 'People should read books instead of e-books' (see: 2012 exam) then one of the sub-arguments would be: 'Books are more permanent and reliable than e-books.' So the key player is: 'the permanence/reliability/trustworthiness of hard copies of books.' Then, one of your paragraphs can explore how the author establishes this idea of books being more reliable, and at the end of the paragraph, you'll link this sub-argument back to the overall contention.

Hopefully that clears things up; let me know if there's anything about this you don't understand :)

thank you for your explanation!

this does sound really sophisticated and I'm not sure if I can pull it off...
this whole year I've just used a chronological method, going through the article and analysing until I've run out of ideas... however, I realised that for comparatives, this isn't very efficient -- so if faced with 2 pieces to analyse, should I try to find similar key players in both and then compare and contrast each author's use of techniques etc to argue these mini-contentions? Or would you recommend doing a big comparison paragraph at the end of these essays?

also, do you know of any samples of essays or paragraphs that use the key players structure? I'm looking around but I'm having trouble identifying which ones do and don't :s

ty again :)
Title: Re: VCE English Question Thread
Post by: cosine on September 25, 2015, 11:08:56 am
Hey guys,

For the ones who are doing Medea for their end of year exam, attached is a document with 3000+ words of quotes and my own explanation/view of the quote. Enjoy xD
Title: Re: VCE English Question Thread
Post by: Cristiano on September 25, 2015, 11:51:51 am
Hey guys,

For the ones who are doing Medea for their end of year exam, attached is a document with 3000+ words of quotes and my own explanation/view of the quote. Enjoy xD

Don't think these are from the John Davie translation, but rather the James Morwood version. You would lose marks on the exam for writing these quotes.
Title: Re: VCE English Question Thread
Post by: duo0024 on September 25, 2015, 06:41:40 pm
I realized that one of my biggest weakness in writing a timed language analysis piece is that I'm not able to select the 'right' language to analyse. Normally, I write my first two paragraphs analysing maybe the first quarter of the article (with lots and lots of bullshit), then realizing that I have only 20 minutes left I tend to do a rushed analysis even though most of the good techniques are usually at the end. Do you guys have any tips to prevent this? Or tips on how to be more selective with your choice of devices? I think I subconsciously fear running out of stuff to analyse so I tend to analyse every little device and effect I see.
Cheers  :)
Title: Re: VCE English Question Thread
Post by: Escobar on September 25, 2015, 06:48:28 pm
Don't think these are from the John Davie translation, but rather the James Morwood version. You would lose marks on the exam for writing these quotes.
i was told by my teacher that the John Davie translation is only a recommendation
i think vcaa released a bulletin confirming this
Title: Re: VCE English Question Thread
Post by: S33667 on September 25, 2015, 07:13:42 pm
i was told by my teacher that the John Davie translation is only a recommendation
i think vcaa released a bulletin confirming this

You are correct it is ONLY the John Davie translation ..... Euripides, ‘Medea’, in Medea and Other Plays, John Davie (trans.), Penguin Classics, 2003 (1)
Title: Re: VCE English Question Thread
Post by: cosine on September 25, 2015, 07:27:52 pm
So is the Mede and other plays (A new translation by James Morwood) accepted?
Title: Re: VCE English Question Thread
Post by: Escobar on September 25, 2015, 07:48:54 pm
So is the Mede and other plays (A new translation by James Morwood) accepted?
http://www.vcaa.vic.edu.au/Documents/bulletin/2015/vcaabulletin005feb15.pdf
"The 2015 text list for English/EAL lists Medea by Euripides. The VCAA does not prescribe editions for VCE
English/EAL; any complete edition may be used. The bibliographic information in the text lists is provided to
assist teachers to obtain texts.
Therefore, any available edition of Medea may be used. This includes both the Davie and Vellacott
translations. "

so, yes
Title: Re: VCE English Question Thread
Post by: Cristiano on September 25, 2015, 09:24:29 pm
I would think the VCAA Examiner reading your essay would have studied the John Davie translation, so if you are using quotes from another version (in which there are many out there) there may be confusion. I highly doubt they would know quotes/have studied multiple translations, so to be safe you should use the Davie translation.
Title: Re: VCE English Question Thread
Post by: Splash-Tackle-Flail on September 25, 2015, 10:52:54 pm
I've got a few questions for Context:

1. Just out of curiosity, for those who have done the exam, or those with a clear idea on what you're going to write on, what form did you adopt for the exam (i.e creative, persuasive, hybrid etc)? Did it work out in the end?

2. For the whole year I've basically stuck to writing in a newspaper article/editorial kind of format, with basically a few threads (only got two at the moment :/ )- current affairs/issues to act as the foundation to my piece. However this brings about a few issues. What do I do if I get a prompt that is really specific and my 'threads' won't work very well (e.g. if they give a prompt only focusing on conflict between two people; I would genuinely be screwed if this happens)? Will they do this in an exam? I think my threads are fairly versatile and can be adapted to an extent.

3.If so, how can I prepare for this in case they do (like would you recommend writing individual paragraphs based on different issues, but exploring different ideas- 'cause then I need to find a way to bring these issues all together, because I'm writing a newspaper piece so I can't simply link something like domestic violence to the recent asylum seeker crisis? And how would you link these recent issues to something like A Separation, which (although the ideas can be linked) is mostly irrelevant. At the moment (and in past sacs) all I've been doing is saying something like "perhaps some important lessons can be found in Asghar Farhadi's ____.....", but is just seems so forced!
Title: Re: VCE English Question Thread
Post by: Escobar on September 26, 2015, 01:13:41 am
how closely related does a context piece have to be to the prompt?
eg if the prompt is ‘Our surroundings can be both threatening and comforting.’ can i just ignore the 'threatening' part
also, does the whole essay have to address the prompt or can i explore other ideas of the context (imaginative landscape)?
Title: Re: VCE English Question Thread
Post by: tashhhaaa on September 26, 2015, 08:23:12 am
I realized that one of my biggest weakness in writing a timed language analysis piece is that I'm not able to select the 'right' language to analyse. Normally, I write my first two paragraphs analysing maybe the first quarter of the article (with lots and lots of bullshit), then realizing that I have only 20 minutes left I tend to do a rushed analysis even though most of the good techniques are usually at the end. Do you guys have any tips to prevent this? Or tips on how to be more selective with your choice of devices? I think I subconsciously fear running out of stuff to analyse so I tend to analyse every little device and effect I see.
Cheers  :)

I usually go through the article and highlight a few things and tell myself that's ALL I'm going to analyse. ie. only write about the highlighted parts
Stops me from bullshitting too much
Title: Re: VCE English Question Thread
Post by: heids on September 26, 2015, 10:35:22 am
I've got a few questions for Context:

1. Just out of curiosity, for those who have done the exam, or those with a clear idea on what you're going to write on, what form did you adopt for the exam (i.e creative, persuasive, hybrid etc)? Did it work out in the end?

2. For the whole year I've basically stuck to writing in a newspaper article/editorial kind of format, with basically a few threads (only got two at the moment :/ )- current affairs/issues to act as the foundation to my piece. However this brings about a few issues. What do I do if I get a prompt that is really specific and my 'threads' won't work very well (e.g. if they give a prompt only focusing on conflict between two people; I would genuinely be screwed if this happens)? Will they do this in an exam? I think my threads are fairly versatile and can be adapted to an extent.

3.If so, how can I prepare for this in case they do (like would you recommend writing individual paragraphs based on different issues, but exploring different ideas- 'cause then I need to find a way to bring these issues all together, because I'm writing a newspaper piece so I can't simply link something like domestic violence to the recent asylum seeker crisis? And how would you link these recent issues to something like A Separation, which (although the ideas can be linked) is mostly irrelevant. At the moment (and in past sacs) all I've been doing is saying something like "perhaps some important lessons can be found in Asghar Farhadi's ____.....", but is just seems so forced!

1.  Expository.  (Spent like 35 mins on it though, finished only one body + started the other two, and... yeah.)

2.  Collate all the prompts you can find, and then try to invent as many more whacko prompts as you can.  Don't let yourself be scared by a nameless fear of 'what prompt could they invent', try to come up with as many prompts as you can. (e.g. you mentioned a prompt about conflict between two people; try to think of a few concrete one-sentence prompts like that, and then think about how you'd address those specific prompts.  If you can't think of one, chances are that they won't ask a prompt like that.)

Then go through the list one-by-one, really fast - like devote 1-5 min to each prompt, thinking out how you could adapt your current threads to meet the prompt.  If after 5 min you can't figure out how to do it, highlight it and come back to delving into that prompt later.

3.  Answered above.
I didn't do creative/hybrids, so have no practice; my guess is that you've gotta use transitional phrase, even if it's clunky.  Experiment with writing as many as you can until you hit some that sound less clunky; 'As I was lately reading Asghar Farhadi's ___, it really struck me that...', or 'we'd all do well to read ___, which shows...', or - well up to your inventiveness.

how closely related does a context piece have to be to the prompt?
eg if the prompt is ‘Our surroundings can be both threatening and comforting.’ can i just ignore the 'threatening' part
also, does the whole essay have to address the prompt or can i explore other ideas of the context (imaginative landscape)?
Very closely related.  That's the point of the piece - to discuss the implications of the prompt.  Take it from me, ensuring that your whole essay is relevant to the prompt will already give you a marks-boost.  It's your first, vital consideration in writing an essay.

NO YOU CANNOT.  YOU CANNOT.  YOU CANNOT.  Doing so will severely limit your marks.  You must answer the question, the whole question, and nothing (much) but the question.  The point of that prompt is that our surroundings can be both good and bad, both scary and nice, etc. - so you're not answering it if you just say 'they're nice', because you're not exploring how there can be multiple and even contradictory aspects/influences of the environment, which is the crux of the prompt.

The whole essay must address the prompt.  Okay, the prompt is a pretty wide playing field, and thus you'll find yourself discussing various ideas in the process - e.g. you might discuss the impact of changing your place, or how our surroundings define our identity, because they're related to the threatening/comforting nature of surroundings.  But unless it's closely linked to the prompt and your contention about the prompt, you can't just go talking about other unrelated ideas because you know them well.
Title: Re: VCE English Question Thread
Post by: Splash-Tackle-Flail on September 26, 2015, 12:12:20 pm
1.  Expository.  (Spent like 35 mins on it though, finished only one body + started the other two, and... yeah.)

2.  Collate all the prompts you can find, and then try to invent as many more whacko prompts as you can.  Don't let yourself be scared by a nameless fear of 'what prompt could they invent', try to come up with as many prompts as you can. (e.g. you mentioned a prompt about conflict between two people; try to think of a few concrete one-sentence prompts like that, and then think about how you'd address those specific prompts.  If you can't think of one, chances are that they won't ask a prompt like that.)

Then go through the list one-by-one, really fast - like devote 1-5 min to each prompt, thinking out how you could adapt your current threads to meet the prompt.  If after 5 min you can't figure out how to do it, highlight it and come back to delving into that prompt later.

3.  Answered above.
I didn't do creative/hybrids, so have no practice; my guess is that you've gotta use transitional phrase, even if it's clunky.  Experiment with writing as many as you can until you hit some that sound less clunky; 'As I was lately reading Asghar Farhadi's ___, it really struck me that...', or 'we'd all do well to read ___, which shows...', or - well up to your inventiveness.
Very closely related.  That's the point of the piece - to discuss the implications of the prompt.  Take it from me, ensuring that your whole essay is relevant to the prompt will already give you a marks-boost.  It's your first, vital consideration in writing an essay.


Thanks :) so you'd recommend quick plans, instead of full practice essays first??
Title: Re: VCE English Question Thread
Post by: heids on September 26, 2015, 12:18:53 pm
Thanks :) so you'd recommend quick plans, instead of full practice essays first??
A mixture.

Basically, here's the steps I was suggesting:
1.  Write, search for and collate prompts.
2.  Go through the list doing 1-5 min mental plans of how you'd adapt to each prompt.
3.  Any you can't do in such a short time, highlight.
4.  Come back later and plan until you figure out a way.  If you can't, either decide to take the risk or (if there are too many examples of this) it's time to research and rethink again.

In the meantime, you can and should also be doing other study, such as detailed 15min written plans on some of these prompts, or full 1-1.5 hour pieces.  The quick planning method helps you think under pressure and identify the areas of weakness that you need to study, and exposes you to a variety of ideas to think about.
Title: Re: VCE English Question Thread
Post by: tashhhaaa on September 26, 2015, 09:31:05 pm
should we be memorising our context pieces?
I know the assessors' report often says not to but I've heard from a few teachers and successful past students that we should???

I feel like this might be a good option for me since I struggle with conjuring up a context piece on the spot, but I'm also scared that I won't address the prompt and my mark will suffer  :'(
Title: Re: VCE English Question Thread
Post by: pi on September 26, 2015, 10:15:42 pm
should we be memorising our context pieces?
I know the assessors' report often says not to but I've heard from a few teachers and successful past students that we should???

I feel like this might be a good option for me since I struggle with conjuring up a context piece on the spot, but I'm also scared that I won't address the prompt and my mark will suffer  :'(

I memorised four adaptable pieces and managed to scrape an alright score. Like you, I highly doubted I would be able to come up with fresh ideas during the exam. I'd recommend it to anyone, even as a back-up plan if nothing else.
Title: Re: VCE English Question Thread
Post by: Splash-Tackle-Flail on September 26, 2015, 10:48:48 pm
I've been reading a few of the examiner's reports (thinking wow how does someone write a context piece like the second encountering conflict one in the 2014 report), and am pretty sure the foundation of their piece was "memorised"-i.e. a general outline of what they wanted to say, not word for word. I honestly doubt someone could come up with a 10/10 context piece from scratch unless they were shakespeare reincarnate..

On that note: four adaptable pieces seems like quite a lot, how different were they, and do you think the same thing could be done with say, two?

Another question here as well :) Whenever I'm writing, I seem to write really long body paragraphs, because it seems like the only way for me to explore ideas/analyse with enough depth to make me happy, and also means I don't have to come up with as many ideas so to speak. This means I can only churn out 3 body paragraphs in the given amount of time. However, upon reading all the sample pieces, it seems most people write more, shorter paragraphs, and I'm wondering if my longer paragraphs would be penalised in some way??
Title: Re: VCE English Question Thread
Post by: pi on September 26, 2015, 11:01:12 pm
On that note: four adaptable pieces seems like quite a lot, how different were they, and do you think the same thing could be done with say, two?

See for yourself! http://atarnotes.com/pages/?p=notes&a=feedback&id=748

I haven't read those essays in a few years, but pretty sure each was on a different theme in the context, with 2 essays loosely based around each book we studied. I ended up pretty much re-writing a piece I wrote for one of my SACs in the actual exam, which was awesome because I knew it was of a decent standard and I could smash it out in 30-40mins and save time for other things.

Could probably do two, I just did more because I had absolutely no faith in my English skills and wanted to cover all bases haha, so went for more rather than less :P Still have no idea how I got out of those 3 hours alive hahaha
Title: Re: VCE English Question Thread
Post by: Splash-Tackle-Flail on September 26, 2015, 11:19:31 pm
See for yourself! http://atarnotes.com/pages/?p=notes&a=feedback&id=748

I haven't read those essays in a few years, but pretty sure each was on a different theme in the context, with 2 essays loosely based around each book we studied. I ended up pretty much re-writing a piece I wrote for one of my SACs in the actual exam, which was awesome because I knew it was of a decent standard and I could smash it out in 30-40mins and save time for other things.

Could probably do two, I just did more because I had absolutely no faith in my English skills and wanted to cover all bases haha, so went for more rather than less :P Still have no idea how I got out of those 3 hours alive hahaha

Ah thanks, well by the looks of things (I'm not doing Whose Reality) it worked out for you haha. I'm not familiar enough with the ideas of your context (I'm doing Encountering Conflict, and have done Identity and Belonging for 1 2), but It's definitely reassuring that doing what you did (with your adaptable pieces) worked out for you-many thanks :).

Here's to praying the prompt this year is nice.
Title: Re: VCE English Question Thread
Post by: Escobar on September 27, 2015, 01:37:25 pm
You must answer the question, the whole question, and nothing (much) but the question.  The point of that prompt is that our surroundings can be both good and bad, both scary and nice, etc. - so you're not answering it if you just say 'they're nice', because you're not exploring how there can be multiple and even contradictory aspects/influences of the environment, which is the crux of the prompt.

The whole essay must address the prompt.  Okay, the prompt is a pretty wide playing field, and thus you'll find yourself discussing various ideas in the process - e.g. you might discuss the impact of changing your place, or how our surroundings define our identity, because they're related to the threatening/comforting nature of surroundings.  But unless it's closely linked to the prompt and your contention about the prompt, you can't just go talking about other unrelated ideas because you know them well.
thanks for the response, just wanna make sure:
can someone confirm that you must address a context prompt completely?

the teachers at our school have taught us that you do not need to use the whole prompt. eg in a handout they gave us "You must use the prompt in some way, but you do NOT have to use every part"
Title: Re: VCE English Question Thread
Post by: heids on September 28, 2015, 09:34:48 am
this does sound really sophisticated and I'm not sure if I can pull it off...
this whole year I've just used a chronological method, going through the article and analysing until I've run out of ideas... however, I realised that for comparatives, this isn't very efficient -- so if faced with 2 pieces to analyse, should I try to find similar key players in both and then compare and contrast each author's use of techniques etc to argue these mini-contentions? Or would you recommend doing a big comparison paragraph at the end of these essays?

It sounds really hard, but in reality it's not that hard (for most articles).  With a bit of practise, you'll be able to easily list things the author tries to present in a certain way: e.g. the author, the audience, the opposition, the government, dolphins, soft drinks, taxes, trees, whatever.  They use certain language to try and present these in certain ways, and you then analyse why they present them like this, and how it influences the reader's perception of the overall contention.

Couple of examples:

Spoiler
First piece in Weekly Letter-to-the-Editor LA Practise Club!, 'this ordinary bloke'.
Players include (the top two are important, the others minor and you probably wouldn't discuss them much/at all):
- the author (this is a huge one, how is he trying to present himself, and why?)
- politicians
- the people reporting back about the camps
- the people in the camps

For these, you then have to think:
1.  How does he present them?  What does he want these to look like?
2.  How does the language he uses contribute to this?
3.  Why does he want the readers to view them like this?
4.  How does this contribute to his overall point?

Spoiler
Read this

Here's where it's great for a comparative; for instance, imagine they both talk about the Government, so you have a paragraph on the govt.  You can contrast the language they use and show how this builds up different pictures of the govt - e.g. one might label the govt as 'meddlesome' and 'intefering', while another might label them as 'supportive' and 'innovative'.  And then at the end of the paragraph, having discussed the different ways they try to influence the reader, you'd show how these different ways contribute to their overall different contentions.

Whenever I'm writing, I seem to write really long body paragraphs, because it seems like the only way for me to explore ideas/analyse with enough depth to make me happy, and also means I don't have to come up with as many ideas so to speak. This means I can only churn out 3 body paragraphs in the given amount of time. However, upon reading all the sample pieces, it seems most people write more, shorter paragraphs, and I'm wondering if my longer paragraphs would be penalised in some way??
My writing was just like yours (except in LA where I'd do like 5 shorter bodies).  If you're still addressing the prompt thoroughly and exploring all your ideas, can't see why they'd penalise you - don't stress.  If you really want, you can generally split one paragraph into two similar, but slightly different ideas.

thanks for the response, just wanna make sure:
can someone confirm that you must address a context prompt completely?

the teachers at our school have taught us that you do not need to use the whole prompt. eg in a handout they gave us "You must use the prompt in some way, but you do NOT have to use every part"

I know you probably want someone else's proof, but I promise I've heard basically this from Lauren.

Quote from: VCAA 2014 Exam Report
Weaker scripts did not show critical thinking about the idea the prompt was communicating or ignored the prompt altogether.  The more successful responses explored the core ideas of the prompt instead of treating it like a text response.  Students need to be reminded that there is an important distinction between the Context they have studied and the task they have to complete in the
exam.  They are asked to ‘explore the idea’ that is represented in the prompt; students should not present prepared responses that relate to the concepts/issues central to their Context study and ignore the idea the prompt is communicating.  While the prompt can be seen as a springboard for effective writing, the ideas of the prompt must be explored.  It is important to demonstrate an understanding of the core of the prompt.

Okay, sure, from one prompt there are millions of possible ideas and you do have to be selective in what you cover - maybe that's what your teachers meant.  But what's most important is that you address the core of the prompt, the key ideas of the prompt, what it's all about.  Just jumping on a word you like doesn't address the implications of the whole sentence.

In the example you gave about 'threatening or comforting', if you simply addressed the comforting, you would have missed the core of the prompt!  The prompt doesn't just say 'here are two unrelated ideas: the landscape can be threatening, or it can be comforting - pick one and discuss'.  It says, the landscape can be BOTH.  In other words, what you have to discuss is how the landscape doesn't just affect us in one way, how it can have multiple and even contradictory influences - and why is this?  Just discussing how it's comforting misses that whole point.  Don't see the prompt as a string of words you can randomly choose or discard; it's a sentence with a core idea that you must address.
Title: Re: VCE English Question Thread
Post by: BanhBanh97 on October 02, 2015, 12:00:15 am
Can someone please share tips for writing TR conclusion?
Thanks  :D
Title: Re: VCE English Question Thread
Post by: paper-back on October 03, 2015, 07:49:52 pm
How do I smoothly incorporate evidence into my text response and tie several pieces of evidence together? When writing my essays I often just begin with (after writing my topic sentence) a sentence like: "John states that [insert quote] during the [insert scene]...this conveys that" and then go onto analysis, and then follow this up with another "Furthermore, John then states that...", so it seems to lack flow
Title: Re: VCE English Question Thread
Post by: YellowTongue on October 05, 2015, 07:40:54 am
 How can I avoid makin Language Analysis essays too long? Whenever I write one I analyse everything chronologically, but this takes a long time, so I end up analyzing the beginning very closely; whilst the end lacks a thorough analyse. How can I improve?

Thank you ;)
Title: Re: VCE English Question Thread
Post by: Coffee on October 05, 2015, 02:29:59 pm
How can I avoid makin Language Analysis essays too long? Whenever I write one I analyse everything chronologically, but this takes a long time, so I end up analyzing the beginning very closely; whilst the end lacks a thorough analyse. How can I improve?

Thank you ;)

1. Don't feel like you need to analyse every technique.

2. How are you structuring your paragraphs? Structuring based on argument or key players would be best I think. From there you would only be analysing the language and techniques that contribute to the argument. This should help you to cut down on anything unnecessary.
Title: Re: VCE English Question Thread
Post by: pi on October 05, 2015, 04:54:18 pm
In general, a chronological approach is an inferior one because it is going to take too long, and shows that you can't easily prioritise what is most important.
Title: Re: VCE English Question Thread
Post by: tashhhaaa on October 05, 2015, 06:17:07 pm
It sounds really hard, but in reality it's not that hard (for most articles).  With a bit of practise, you'll be able to easily list things the author tries to present in a certain way: e.g. the author, the audience, the opposition, the government, dolphins, soft drinks, taxes, trees, whatever.  They use certain language to try and present these in certain ways, and you then analyse why they present them like this, and how it influences the reader's perception of the overall contention.

...

thank you for the comprehensive explanation!

just another question for anyone

how do you actually structure an LA based on key players? It all seems good and well in theory (I've tried it out and my first attempts were a little pedestrian) As ridiculous as it sounds, I find that I get stuck on topic sentences because I'm so used to chronologically analysing random stuff. Obviously, the TS is going to set the tone for the whole paragraph but I struggle a lot with this, especially with comparatives. Is there any set 'formula' kind of thing that I can use here? Or can I just basically do whatever feels appropriate?
Title: Re: VCE English Question Thread
Post by: paper-back on October 08, 2015, 10:28:04 am
For a prompt like this one:
 "Everybody in the play must accept some responsibility for the tragedies which occur. Discuss."

How would you incorporate "everybody"?
I was going to do 1 paragraph on one character, 1 paragraph on another character and so forth... But then I wouldn't be including "everybody"

EDIT: Never-mind, I got it
BTW That essay prompt is for Medea for anyone who's doing the book
Title: Re: VCE English Question Thread
Post by: Cynbaz2 on October 08, 2015, 10:41:26 pm
 I'm going to write a speech for section B and i'm unsure if its a good idea for my score to make my purpose etc as explicit as this-Transcript of speech delievered by...
At the ....
On the...
The given title of the speech is …
 other than that, what else is a good way to elevate a speech and address different aspects of a prompt i.e. is it too simple to agree with a prompt and try to persuade the audience to that perspective or do I definitely need to go further. thanks
Title: Re: VCE English Question Thread
Post by: KYtho on October 09, 2015, 07:26:00 pm
hey everyone, how is everyone structuring their language analysis? i.e. by groups/people/things, or just doing it chronologically to save some time?
Title: Re: VCE English Question Thread
Post by: Cristiano on October 09, 2015, 07:58:09 pm
How do you incorporate an author's 'Bias' into the introduction? Like for the 2010 Biodiversity exam how is Lee biased, as he is a professor - he may contend etc.
How do you word this and smoothly weave it into the intro?

Any suggestions would be great as i am a bit lost :) Thanks
Title: Re: VCE English Question Thread
Post by: banaidil on October 09, 2015, 08:56:33 pm
I wanted to know that if I start a text response with a really long sentence is that a bad thing?
Here is an example:  " Sketching a tale of the harsh and gruelling conditions endured by the indigenous Australians during Australia’s Great Depression, Jack Davis’ play ‘No Sugar’ depicts how some minority groups act out in communal defiance against the oppressive environment they lived in." And if there is any way I could word this in a more sophisticated manner, I would like to know.

Thank you :)
Title: Re: VCE English Question Thread
Post by: duo0024 on October 09, 2015, 09:10:52 pm
So I'm planning to write an expository piece for section B of the exam, and my teacher has told me that my pieces do not show the FLAP (form language audience purpose). My expository writing adheres to the general expository writing format, with the formal writing, 3 paragraphed structure with different sources of evidence for each one. She recommends that I put 'Newspaper editorial' or some other thing like that at the beginning of the piece so that I can address the 'audience' part of the FLAP, however in seeing that many of the upper-range expository pieces just launch straight into it, I believe this could compromise some of my exam marks :\
Anyone got any ideas?
Title: Re: VCE English Question Thread
Post by: shivaji on October 09, 2015, 09:27:33 pm
Hey guys I'm kinda stuck- not sure which text response to do for the final exam.

I can do Henry IV, which I'm quite interested in though,but haven't really out much work in it, so will 2odd weeks be alright to study it?

Or the other book we did for unit 3, although I've done a lot of work on it, and don't want to waste it, it just doesn't seem that fun anymore (maybe because it was a long time ago)

Any advice please help?
Title: Re: VCE English Question Thread
Post by: duo0024 on October 09, 2015, 09:37:31 pm
Hey guys I'm kinda stuck- not sure which text response to do for the final exam.

I can do Henry IV, which I'm quite interested in though,but haven't really out much work in it, so will 2odd weeks be alright to study it?

Or the other book we did for unit 3, although I've done a lot of work on it, and don't want to waste it, it just doesn't seem that fun anymore (maybe because it was a long time ago)

Any advice please help?

I chose my text ( All About Eve ) purely because it was the one I did most recent and therefore was most fresh in my mind. If I chose my text in unit 3, I would have to dedicate time to reread that book and refresh it which would use up a lot of time. Although I did well on my text for unit 3, i believed the time required to get back to the standard that i wrote before wasn't worth it. my opinion tho.
Title: Re: VCE English Question Thread
Post by: Alter on October 09, 2015, 09:48:53 pm
Hey guys I'm kinda stuck- not sure which text response to do for the final exam.

I can do Henry IV, which I'm quite interested in though,but haven't really out much work in it, so will 2odd weeks be alright to study it?

Or the other book we did for unit 3, although I've done a lot of work on it, and don't want to waste it, it just doesn't seem that fun anymore (maybe because it was a long time ago)

Any advice please help?
Try to make your decision ASAP so you can put your full energy into learning one text instead of poorly learning two. For Section A there's always two prompts so it's not like you'll get 2 prompts you have no clue for compared to something like context where you only get one. My point here is that while I could see the value in learning two context books, learning two TR ones is basically useless.

I'd ask your teacher which text she/he thinks you're better at based on your tests and class discussions and then weigh that up with which text you enjoy the most. You're more likely to do well with texts that you really like to study than ones that simply get done by strong students.

You very likely have enough time to relearn a book from Unit 3, but I'd put a lot of effort into it if you want to have a strong control over author's craft. Best wishes.
Title: Re: VCE English Question Thread
Post by: tashhhaaa on October 09, 2015, 11:41:57 pm
Try to make your decision ASAP so you can put your full energy into learning one text instead of poorly learning two. For Section A there's always two prompts so it's not like you'll get 2 prompts you have no clue for compared to something like context where you only get one. My point here is that while I could see the value in learning two context books, learning two TR ones is basically useless.

I'd ask your teacher which text she/he thinks you're better at based on your tests and class discussions and then weigh that up with which text you enjoy the most. You're more likely to do well with texts that you really like to study than ones that simply get done by strong students.

You very likely have enough time to relearn a book from Unit 3, but I'd put a lot of effort into it if you want to have a strong control over author's craft. Best wishes.

Just to add to this, I haven't chosen by book either :s
I was pretty set on Medea but then I realised there are SO MANY people doing it.

Do you guys reckon it's better to do an unpopular book? (for me it's The White Tiger)
Title: Re: VCE English Question Thread
Post by: stockstamp on October 09, 2015, 11:59:00 pm
hi  :)

I am hoping for some advice on vocab in text response.

In the past, I've found that the most effective way of weaving fancier, less common words into text essays is to read scholarly articles, literary reviews and alike.
But what if there is nothing available? My text is Burial Rites, and as it is so new there is effectively nothing online to read about it, other than just a couple of brief reviews which don't really help much.

I understand that the likely response here is something along the lines of 'thought counts more than vocab', and while I agree that that is unquestionably true, I think you also have to be realistic. The essays approaching the highest end of the mark spectrum always have an impressive collection of phrases and terms fairly cleverly embedded, and to get the high marks that same approach needs to be taken.

I would say that naturally, my use of vocab is strong and expressive; it; it makes sense, but it doesn't have that wow factor unless I have quite a bit of time to 'craft' a piece, as opposed to pumping out 3 in as many hours. I would also like to make the point that I only use words if I am confident with them; I don't build my essays around language in the hope to impress, because I know that doesn't work.

Having said that, the language I use in my text response probably won't get my the marks I'm hoping for at the moment, and I was hoping for some advice on how to work past this.

Is it a good idea to 'craft' some perfect sentences/paragraphs and memorise them, instead of smashing out essays as often as possible?
What sort of stuff should I read to pick up vocab? Literary reviews of other books, extracting relevant ideas where necessary?
If I memorise work, am I at the risk of spending too much time on topics that could be totally different from what happens in the exam (JESUS ITS IN TWO WEEKS!!!!!!!!!!!!!!!!)?

sorry for the length of this......
I stress easily
Title: Re: VCE English Question Thread
Post by: S33667 on October 10, 2015, 09:07:20 am
Hey guys I'm kinda stuck- not sure which text response to do for the final exam.

I can do Henry IV, which I'm quite interested in though,but haven't really out much work in it, so will 2odd weeks be alright to study it?

Or the other book we did for unit 3, although I've done a lot of work on it, and don't want to waste it, it just doesn't seem that fun anymore (maybe because it was a long time ago)

Any advice please help?

I made my decision by picking up practice exams and looking at possible prompts for both of my texts - the decision was easy then.   Once decided don't second guess yourself - just start churning out the essays.
Title: Re: VCE English Question Thread
Post by: Burt Macklin on October 10, 2015, 02:41:28 pm
What do you guys think would be the optimal ratio between writing about your text and external examples in an expository essay for Context?
Title: Re: VCE English Question Thread
Post by: scottg15 on October 10, 2015, 03:02:00 pm
I have read somewhere on this forum to write on every second line in an essay so that you can have space for editing. Is this for in exams or just for SAC's and practice essays. Like, do the assessors mind???

Also, I have got a problem where in the context section of the exam I always write my piece in 15-20 min. Then I get another idea, which i think 'hey that will be a lot better' and then i write that one for the remaining 30 min. What do you think i should do??? It seems pointless writing two, but I never seem to be able to get the other idea unless i write the first one.
Title: Re: VCE English Question Thread
Post by: pi on October 10, 2015, 03:14:29 pm
I have read somewhere on this forum to write on every second line in an essay so that you can have space for editing. Is this for in exams or just for SAC's and practice essays. Like, do the assessors mind???

I know people who even double spaced in the exam. It's really whatever feels comfortable to you, assessors probably don't mind. I didn't space my lines whatsoever.
Title: Re: VCE English Question Thread
Post by: Splash-Tackle-Flail on October 10, 2015, 04:37:01 pm
Also, I have got a problem where in the context section of the exam I always write my piece in 15-20 min. Then I get another idea, which i think 'hey that will be a lot better' and then i write that one for the remaining 30 min. What do you think i should do??? It seems pointless writing two, but I never seem to be able to get the other idea unless i write the first one.

Hmm, I think this boils down to not being particularly sure of what you want to say (i.e. just writing the first related idea that springs to mind). The thing is, it takes fair amount of brainstorming to develop more complex, insightful ideas to respond to, perhaps like your second "a lot better" ideas. Although I don't really know what form your writing and what not, I would definitely recommend more extensive planning (for now, maybe start untimed, so you can do it well first), by planning, you should be doing similar cognitive thinking of sorts to what you were doing when you were writing on your first idea, but this way, you the idea expressed on paper will be the better one.

tl;dr do lots of planning! make sure you know what you want to say first!
Title: Re: VCE English Question Thread
Post by: cosine on October 11, 2015, 10:18:03 am
Someone able to read my Medea essay for me please. Any feedback is greatly appreciated.

Thank you
Title: Re: VCE English Question Thread
Post by: _fruitcake_ on October 11, 2015, 11:21:43 am
Someone able to read my Medea essay for me please. Any feedback is greatly appreciated.

Thank you

In the captivating play, ‘Medea’, Euripides explores the strife that may exist between husband and wife through actions that are primarily lead by passion and ‘fierce emotions’.  You are judging the play, it deviates from your academic tone that you showed throughout your essay. Good embedded quotes in your intro.
The heart of the play lies within Medea’s attempt at seeking her own form of justice in the male-dominated society of 480 BC Greece. The play charts Medea’s emotional transformation, a progression from suicidal despair to sadistic fury;and her devoted love towards Jason, to her unspeakable destruction of him. Medea, feeling obliged to some form of revenge for being dishonored, embarks on a rally of crime in which she believes is the ‘terrible but necessary’ thing to do; to redeem herself and bring justice and honor to the female sex. Nonetheless, the central characters of the play; the Nurse, Jason and the Chorus immensely condemn Medea’s expenditure (expenditure sounds like she went shopping, which is something a woman would do.. ouch pls dont kill me.. but a different word such as 'actions' or 'exertions' or 'efforts' would satisfy better in this context) and do not believe that it brought injustice to not only Jason, but also the innocent victims of Medea’s witchcraft. 

Some things from the intro. Try to not show a biased view of the text; such as mentioning it was captivating, for me i didn't connect with the novel, which is why i wont be talking about it on the exam. I changed a few of your commas to a semicolon. There is just a few words that dont apply in the context. However you do show a good knowledge of text and you use embedded quotes well :)

Jason’s abandonment and dishonoring of Medea serves to assist in her desires for justice and revenge. Euripides opens the play with the Nurse foreshadowing, and hence informing the audience, of the recent events that have lead led to the current conflict between Medea and Jason, which proves to encircle the majority of the play’s plot. Medea, ‘utterly destroyed’, has nothing to live for; hence she ‘surrenders her body to her sorrows, pining away in her tears’. Jason’s abandonment has struck Medea so unexpectedly and because she had sacrificed all, ‘her heart unhinged in her love for Jason’, Medea now has nothing, ‘no mother, no brother, no fatherland’, and at the beginning scenes of the play, Medea is at the outset of her vulnerability. Through the Nurse and the Chorus’ continuous downplay of Jason, the audience is encouraged to sympathize with Medea and understand her need for justice, because she is rightfully obliged to ‘exact revenge from Jason’. Ultimately, Medea’s unfortunate circumstances grant her the right to seek justice, for she was ‘wronged unprovoked’. 
Good paragraph just a few things i picked up. Its best to analyze what Medea did for Jason. From what i observe, is why should i sympathize with Medea? True Jason acted like she was his little bitch but what did she do? Maybe its best to mention how she helped Jason, and then got back stabbed. Such as helping him to get the Golden Fleece, and receiving no recognition. This would increase the level of sympathy that Medea receives from the reader. I also like the manipulation of your words, that Medea has nothing..'no brother'. Thats cos she killed him :'), I wouldn't include that she doesn't have a brother, because it sets her up as a murderer, cos there is no justification why she killed him.

Although Medea is entitled to some form of revenge, the audience, Jason, Chorus and the Nurse all condemn Medea’s sense and intentions of justice. Its either sense of justice, or intentions for justice. Intentions of justice doesnt sound the best Medea’s perception of revenge is far off from justice. The targeting of innocent characters (would most likely) evoke a sense of outrage in the audience of the play as well as the Nurse and Chorus, and upon discovering that Medea is planning to murder Glauce, King Creon and her two children stop here. The sympathy that Euripides initially creates for Medea fades into an underlying sense of disgust and shame. The Chorus and the Nurse initially sympathize with Medea and understand the fact that she had been cursed with a ‘sea of woes’ and miseries, and hence acknowledge the normal human nature of revenge. This sentences does not fit with the flow of the paragraph. Something like 'The Chorus and Nurse initially acknowledge Medea's 'seas of woes' and then continue with 'However, when Medea.. However, when Medea reveals her true intentions, the Corinthian women and the Nurse retaliate and censure her ‘lioness’ morality, as the princess, King Creon and the two children had ‘no role in their father’s (Jason) wrongdoing’, and did not deserve to pay the price of Medea’s violent and immoral nature.

Surprisingly, though, Medea seems to be completely satisfied and fulfilled at committing the most ‘unholiest of deeds’. Despite her actions being heavily censured, Medea’s ameliorated witchcraft and manipulation allowed allows her to fulfill the deed, and successfully murders the Princess, Creon and her two children. Medea now acknowledges that what she has done will haunt her for the rest of her life, but she dismisses this at the thought of ‘stinging’ Jason’s heart, as all her ‘sorrows are well repaid if you (Jason) cannot laugh at me’. Fucking good points Medea has officially proved a tragic hero, she herself feels accomplished and justified at her crimes, because her ‘fury against Jason is stronger than her counsels of softness’ and she brought ‘honor to the name of women’, that is, only in her eyes, as she flees the murder scene in a chariot handed down by her grandfather, the Sun-god, leaving Jason to vividly regret his abandonment. This is a deus ex machina. It would be best to include why Euripides allows her to escape. Something like 'Euripides use of Deus ex machina, as Medea escapes unpunished in a chariot drawn by dragons, supports Medea's cause and leaves Jason to vividly regret his abandonment

Good essay :) u should be fine for the exam. I would give this an 8.5/10
Title: Re: VCE English Question Thread
Post by: cosine on October 11, 2015, 11:37:19 am
In the captivating play, ‘Medea’, Euripides explores the strife that may exist between husband and wife through actions that are primarily lead by passion and ‘fierce emotions’.  You are judging the play, it deviates from your academic tone that you showed throughout your essay. Good embedded quotes in your intro.
The heart of the play lies within Medea’s attempt at seeking her own form of justice in the male-dominated society of 480 BC Greece. The play charts Medea’s emotional transformation, a progression from suicidal despair to sadistic fury;and her devoted love towards Jason, to her unspeakable destruction of him. Medea, feeling obliged to some form of revenge for being dishonored, embarks on a rally of crime in which she believes is the ‘terrible but necessary’ thing to do; to redeem herself and bring justice and honor to the female sex. Nonetheless, the central characters of the play; the Nurse, Jason and the Chorus immensely condemn Medea’s expenditure (expenditure sounds like she went shopping, which is something a woman would do.. ouch pls dont kill me.. but a different word such as 'actions' or 'exertions' or 'efforts' would satisfy better in this context) and do not believe that it brought injustice to not only Jason, but also the innocent victims of Medea’s witchcraft. 

Some things from the intro. Try to not show a biased view of the text; such as mentioning it was captivating, for me i didn't connect with the novel, which is why i wont be talking about it on the exam. I changed a few of your commas to a semicolon. There is just a few words that dont apply in the context. However you do show a good knowledge of text and you use embedded quotes well :)

Jason’s abandonment and dishonoring of Medea serves to assist in her desires for justice and revenge. Euripides opens the play with the Nurse foreshadowing, and hence informing the audience, of the recent events that have lead led to the current conflict between Medea and Jason, which proves to encircle the majority of the play’s plot. Medea, ‘utterly destroyed’, has nothing to live for; hence she ‘surrenders her body to her sorrows, pining away in her tears’. Jason’s abandonment has struck Medea so unexpectedly and because she had sacrificed all, ‘her heart unhinged in her love for Jason’, Medea now has nothing, ‘no mother, no brother, no fatherland’, and at the beginning scenes of the play, Medea is at the outset of her vulnerability. Through the Nurse and the Chorus’ continuous downplay of Jason, the audience is encouraged to sympathize with Medea and understand her need for justice, because she is rightfully obliged to ‘exact revenge from Jason’. Ultimately, Medea’s unfortunate circumstances grant her the right to seek justice, for she was ‘wronged unprovoked’. 
Good paragraph just a few things i picked up. Its best to analyze what Medea did for Jason. From what i observe, is why should i sympathize with Medea? True Jason acted like she was his little bitch but what did she do? Maybe its best to mention how she helped Jason, and then got back stabbed. Such as helping him to get the Golden Fleece, and receiving no recognition. This would increase the level of sympathy that Medea receives from the reader. I also like the manipulation of your words, that Medea has nothing..'no brother'. Thats cos she killed him :'), I wouldn't include that she doesn't have a brother, because it sets her up as a murderer, cos there is no justification why she killed him.

Although Medea is entitled to some form of revenge, the audience, Jason, Chorus and the Nurse all condemn Medea’s sense and intentions of justice. Its either sense of justice, or intentions for justice. Intentions of justice doesnt sound the best Medea’s perception of revenge is far off from justice. The targeting of innocent characters (would most likely) evoke a sense of outrage in the audience of the play as well as the Nurse and Chorus, and upon discovering that Medea is planning to murder Glauce, King Creon and her two children stop here. The sympathy that Euripides initially creates for Medea fades into an underlying sense of disgust and shame. The Chorus and the Nurse initially sympathize with Medea and understand the fact that she had been cursed with a ‘sea of woes’ and miseries, and hence acknowledge the normal human nature of revenge. This sentences does not fit with the flow of the paragraph. Something like 'The Chorus and Nurse initially acknowledge Medea's 'seas of woes' and then continue with 'However, when Medea.. However, when Medea reveals her true intentions, the Corinthian women and the Nurse retaliate and censure her ‘lioness’ morality, as the princess, King Creon and the two children had ‘no role in their father’s (Jason) wrongdoing’, and did not deserve to pay the price of Medea’s violent and immoral nature.

Surprisingly, though, Medea seems to be completely satisfied and fulfilled at committing the most ‘unholiest of deeds’. Despite her actions being heavily censured, Medea’s ameliorated witchcraft and manipulation allowed allows her to fulfill the deed, and successfully murders the Princess, Creon and her two children. Medea now acknowledges that what she has done will haunt her for the rest of her life, but she dismisses this at the thought of ‘stinging’ Jason’s heart, as all her ‘sorrows are well repaid if you (Jason) cannot laugh at me’. Fucking good points Medea has officially proved a tragic hero, she herself feels accomplished and justified at her crimes, because her ‘fury against Jason is stronger than her counsels of softness’ and she brought ‘honor to the name of women’, that is, only in her eyes, as she flees the murder scene in a chariot handed down by her grandfather, the Sun-god, leaving Jason to vividly regret his abandonment. This is a deus ex machina. It would be best to include why Euripides allows her to escape. Something like 'Euripides use of Deus ex machina, as Medea escapes unpunished in a chariot drawn by dragons, supports Medea's cause and leaves Jason to vividly regret his abandonment

Good essay :) u should be fine for the exam. I would give this an 8.5/10

Well wow.. This is good, very good. Thank you for the feedback I really loved how some parts you got enthusiastic about haha xD
About the 'captivating' part in the intro, I always start off the intro in the same way and decided to change things around, so no opinions, right? Obviously examiners will be reading heaps of these intros, and good ways to spice the first sentence a bit?

Thanks heaps fruitcake, really do appreciate your help and will definitely work to utilise your inputs! Which book are you writing about instead?
Title: Re: VCE English Question Thread
Post by: thaaanyan on October 11, 2015, 11:44:22 am
Someone able to read my Medea essay for me please. Any feedback is greatly appreciated.

Thank you

Hey,
I just finished marking this, so I figured I'd put it up! Guess you get double feedback! :) Hope this helps cosine

Thaaanyan
Title: Re: VCE English Question Thread
Post by: _fruitcake_ on October 11, 2015, 11:56:03 am
Well wow.. This is good, very good. Thank you for the feedback I really loved how some parts you got enthusiastic about haha xD
About the 'captivating' part in the intro, I always start off the intro in the same way and decided to change things around, so no opinions, right? Obviously examiners will be reading heaps of these intros, and good ways to spice the first sentence a bit?

Thanks heaps fruitcake, really do appreciate your help and will definitely work to utilise your inputs! Which book are you writing about instead?

No problem mate. I wanted to do Medea but i am already doing a book on women's rights and feminism in lit, so i wanted a different thing to talk about in English, so im doing This Boy's Life
Title: Re: VCE English Question Thread
Post by: cosine on October 11, 2015, 01:36:16 pm
Hey,
I just finished marking this, so I figured I'd put it up! Guess you get double feedback! :) Hope this helps cosine

Thaaanyan

Cheers for that, although you destroyed me xD I see that you say I go off topic a lot, I know this is the case, but how do I fix this?

Like for that prompt, there really was not much to talk about, so I had to fill in some spaces otherwise it would be very short. What are your tips to do when the prompt given is not your best friend? Is it a matter of mastering the text more or is it luck, depending on the prompt?

Cheers. Also anyone got some good introduction starters, besides 'In Euripides'...'

Also when we quote on the real essay, do we use " - " or ' - ' ? I am assuming " - " because someone is saying the quote in the play?

What about in LA essays, which one do we use?

Also also in LA essays, if we use the argument per paragraph approach, is it better to incorporate the visuals in the paragraph that supports the argument in the visual or should visuals ALWAYS be a paragraph of their own?

And for conclusions of LA essays, what needs to be stated? Is it necessary to restate the publication of the article etc ?
Title: Re: VCE English Question Thread
Post by: Cristiano on October 11, 2015, 01:57:02 pm


Cheers. Also anyone got some good introduction starters, besides 'In Euripides'...'

Also when we quote on the real essay, do we use " - " or ' - ' ? I am assuming " - " because someone is saying the quote in the play?

What about in LA essays, which one do we use?

Also also in LA essays, if we use the argument per paragraph approach, is it better to incorporate the visuals in the paragraph that supports the argument in the visual or should visuals ALWAYS be a paragraph of their own?

And for conclusions of LA essays, what needs to be stated? Is it necessary to restate the publication of the article etc ?
For the Medea intros - try use something like 'Euripides' Greek tragedy epitomizes.. or Silhoutted against the backdrops of Ancient Greece.. etc"

When you are quoting what somebody has said i believe it is "..." but for the title of the book it would be '..' - also the name of a newspaper or article so . John Abc from 'The Age' or the 'Herald Sun'. Also in LA it's ".." when you are quoting their use of language.

In LA essays, it is best to incorporate the visual inside a body paragraph and not to have it on its own as they are a method of persuasion and is not separate to the contention and does not need to be analysed separately.

In the conclusion you do not need to state where it has been published as we already know that if you have said that in the introduction.
You just need to sum up their overall use of language and how it has been used to validate their contention, while discussing the overall issue (not giving your opinion).

Hope this somewhat helps :)

Title: Re: VCE English Question Thread
Post by: thaaanyan on October 11, 2015, 02:35:33 pm
Cheers for that, although you destroyed me xD I see that you say I go off topic a lot, I know this is the case, but how do I fix this?

Like for that prompt, there really was not much to talk about, so I had to fill in some spaces otherwise it would be very short. What are your tips to do when the prompt given is not your best friend? Is it a matter of mastering the text more or is it luck, depending on the prompt?

Cheers. Also anyone got some good introduction starters, besides 'In Euripides'...'

Also when we quote on the real essay, do we use " - " or ' - ' ? I am assuming " - " because someone is saying the quote in the play?

What about in LA essays, which one do we use?

Also also in LA essays, if we use the argument per paragraph approach, is it better to incorporate the visuals in the paragraph that supports the argument in the visual or should visuals ALWAYS be a paragraph of their own?

And for conclusions of LA essays, what needs to be stated? Is it necessary to restate the publication of the article etc ?

No problem,
Glad I helped! :)
There was actually a lot to discuss with this prompt, the trick is reading in between the lines.
For example, if we map out your para 1 real quick:
BODY PARA 1: Medea is justified in believing her actions to be 'just' against Jason because Jason has broken his oath and committed a crime against her.
- you can discuss everything Medea has sacrificed for him
- you can discuss the nature of Medea's pain
- you can discuss that "the thing is common" (Chorus), and is a reflection of the way men use deceit to string women along
- you can talk about the greater social degradation of women and link that to Jason's crimes
- you can discuss the self interested nature of Jason's actions.

BOOM! Massive paragraph haha, as long as you link each of your ideas it's all good. Your main topic sentence is "Jason's crime against Medea" but all these other things connect to the crime of perjury: the consequences felt by Medea both as an exile and as a woman, the nature of Jason's betrayal highlighting his douchebag nature etc.

I believe it's a matter of understanding the text. I have been given some dodgy ass prompts but i know if you study hard enough and argue laterally enough it all works out haha...nah i just spend longer picking it apart at the start

But yeah I think Cristiano's advice pulls together a lot of my thoughts, but generally i have a rule of thumb for intros and that's don't put in any unnecessary bs. 
like if my essay prompt is on women only then will i write in my intro "Composed in a highly dichotomous society where the degradation of women was......." if my essay prompt is on revenge, i will start by talking about revenge. Anybody, and I mean anybody can shove in a pre-written intro with socio-historical context for the prompt. But i'm betting that not a lot of kids can make it relevant.
ultimately it's what makes you feel comfortable at the end of the day, if you feel like having a couple of back up stock phrases are going to make sure you feel confident and you can fluently mold the rest of your essay to the prompt then go for it. it's not a massive thing it won't hurt you, and people have written great essays with pre-written first lines so don't feel like you're obliged to do anything you're not cool with, but generally that's just my stance on 'em.

also for conclusion i was taught differently; to conclude by analysing how the author concludes their piece. i'm guessing there's no set way, just do what you've been taught or what sounds good.
Title: Re: VCE English Question Thread
Post by: cosine on October 11, 2015, 02:41:50 pm
No problem,
Glad I helped! :)
There was actually a lot to discuss with this prompt, the trick is reading in between the lines.
For example, if we map out your para 1 real quick:
BODY PARA 1: Medea is justified in believing her actions to be 'just' against Jason because Jason has broken his oath and committed a crime against her.
- you can discuss everything Medea has sacrificed for him
- you can discuss the nature of Medea's pain
- you can discuss that "the thing is common" (Chorus), and is a reflection of the way men use deceit to string women along
- you can talk about the greater social degradation of women and link that to Jason's crimes
- you can discuss the self interested nature of Jason's actions.

BOOM! Massive paragraph haha, as long as you link each of your ideas it's all good. Your main topic sentence is "Jason's crime against Medea" but all these other things connect to the crime of perjury: the consequences felt by Medea both as an exile and as a woman, the nature of Jason's betrayal highlighting his douchebag nature etc.

I believe it's a matter of understanding the text. I have been given some dodgy ass prompts but i know if you study hard enough and argue laterally enough it all works out haha...nah i just spend longer picking it apart at the start

But yeah I think Cristiano's advice pulls together a lot of my thoughts, but generally i have a rule of thumb for intros and that's don't put in any unnecessary bs. 
like if my essay prompt is on women only then will i write in my intro "Composed in a highly dichotomous society where the degradation of women was......." if my essay prompt is on revenge, i will start by talking about revenge. Anybody, and I mean anybody can shove in a pre-written intro with socio-historical context for the prompt. But i'm betting that not a lot of kids can make it relevant.
ultimately it's what makes you feel comfortable at the end of the day, if you feel like having a couple of back up stock phrases are going to make sure you feel confident and you can fluently mold the rest of your essay to the prompt then go for it. it's not a massive thing it won't hurt you, and people have written great essays with pre-written first lines so don't feel like you're obliged to do anything you're not cool with, but generally that's just my stance on 'em.

also for conclusion i was taught differently; to conclude by analysing how the author concludes their piece. i'm guessing there's no set way, just do what you've been taught or what sounds good.

What I meant about the prompt is the whole idea of it not just those paragraphs. I think this might be my main issue, ideas to discuss? I mean I have a pretty good knowledge of the play but I think linking things is my issue. How can I improve this, you seem very knowledgable about it, what did you do to be able to think outside the box to interlink the ideas together and manipulate it?

About the introduction sentence, what about just a quick historic background to provide some context for the assessor? Like just mentioning the era of Medea's production etc? And then getting straight into the prompt?

Cheers.

Title: Re: VCE English Question Thread
Post by: thaaanyan on October 11, 2015, 03:04:48 pm
What I meant about the prompt is the whole idea of it not just those paragraphs. I think this might be my main issue, ideas to discuss? I mean I have a pretty good knowledge of the play but I think linking things is my issue. How can I improve this, you seem very knowledgable about it, what did you do to be able to think outside the box to interlink the ideas together and manipulate it?

About the introduction sentence, what about just a quick historic background to provide some context for the assessor? Like just mentioning the era of Medea's production etc? And then getting straight into the prompt?

Cheers.



Ima reiterate the queen, literally lauren:
like let's go with "Medea's anger throughout the play is justified."
WHY YES JUSTIFIED?
- cos she's exiled
- cos Jason's an ass to her
- cos society treats her like crap
WHY NOT?
- she doesn't actually give a crap about women or anything, she's just manipulative - her anger isn't the anger of the oppressed it's the anger of a psycho
SO ARE HER ACTIONS JUSTIFIED?
- nah, she kills her kids and Glauce
OR
-yeah screw the kids and Glauce
OR
-kinda, i liked her till she put her anger into violent action

then put together key ideas to form a general contention= While Medea's anger and desire for justice throughout the tragedy are justified, the resulting consequence of her violent actions are not.
General contention done! Separate the above into separate key ideas to discuss which support the prompt.
That's just how i go about things, and how i learnt from the Queen.

But yeah, dude my word is not gospel here. Like, if you're comfortable with a quick socio-historical sentence before jumping into the prompt go for it. That was just my perspective, on what works for me/ what i consider important. there are exceptions for every rule!

Hope this helps!
Title: Re: VCE English Question Thread
Post by: _fruitcake_ on October 11, 2015, 03:37:01 pm
Im having trouble doing an LA piece. I am doing it on the 2014 LA part. There is the main article and a comment section.

So far i got
1 - intro for main article

2- Body Par 1 about main article

3 - Body Par 2 about main article

4- Body Par 3 about main article

Im stuck here, what should i do?

5- conclusion about main article?

6- intro for comment section?

7- Body par 1 for comment section?

8- conclusion for comment section?
Title: Re: VCE English Question Thread
Post by: Cristiano on October 11, 2015, 04:12:29 pm
I would have mentioned in the main intro that there was a response to the article.. so you would say something like "In response to abc, xyz contends that... in their comment"

Then talk about the main bodies for the first article, then after you're done you can say; in comparison to abc, xyz (comment section) says etc and analyse that

then just have one big conclusion which sums up both sides!  so you dont need 2 intros or 2 conclusions just one but tie the other article/comment in :)
Title: Re: VCE English Question Thread
Post by: _fruitcake_ on October 11, 2015, 04:36:54 pm
I would have mentioned in the main intro that there was a response to the article.. so you would say something like "In response to abc, xyz contends that... in their comment"

Then talk about the main bodies for the first article, then after you're done you can say; in comparison to abc, xyz (comment section) says etc and analyse that

then just have one big conclusion which sums up both sides!  so you dont need 2 intros or 2 conclusions just one but tie the other article/comment in :)

Ah thats cool thanks bro.
Title: Re: VCE English Question Thread
Post by: acm9 on October 12, 2015, 07:26:26 pm
if the language analysis article has no named author, will we get penalised for referring to them with gendered pronouns?
Also, I was wondering if I can get a 40 raw if my GA's are 90, 93, and 49/60 respectively.
Thanks!
Title: Re: VCE English Question Thread
Post by: Cristiano on October 12, 2015, 07:32:58 pm
if the language analysis article has no named author, will we get penalised for referring to them with gendered pronouns?
Also, I was wondering if I can get a 40 raw if my GA's are 90, 93, and 49/60 respectively.
Thanks!

On the Exam it will definitely have an author's name, but if a practice one does not then the writer or author is fine. (It's also usually assumed that its a 'he' if it does not say)

With your scores, you are looking at perhaps a 41 or 42 raw, maybe even higher :)
Title: Re: VCE English Question Thread
Post by: duo0024 on October 12, 2015, 10:14:14 pm
OKay so I've been trying to fix this problem of mine for a while now and I just keep doing it... I tend to overwrite in language analysis due to my innate fear of running out of things to analyse (I somehow feel bad if I don't analyse every good persuasive technique or connotations because it would feel like a waste to not analyse it-weird huh?) and because of this, I tend to spend too much time on L.A or spend half of my time analysing the first quarter of the text and the last doing a rushed analysis of the rest. Anyone have any tips? I tend to favour really large chunky paragraphs so maybe I should just start writing a lot of short paragraphs?
Cheers
Title: Re: VCE English Question Thread
Post by: jrx98 on October 13, 2015, 05:20:06 pm
Hi everyone,
Was wondering what the difference between a 9 and a 10 in an essay is? Is it just expression and cohesion?
Personally I've gotten 10s for text and 10s for context before, but keep getting 9s for section c :/ !!!!!
I've spoken to my teacher and she says I shouldn't be complaining LOL

Help would be much appreciated

Thanks
Title: Re: VCE English Question Thread
Post by: cosine on October 13, 2015, 08:13:05 pm
Is writing 3 body paragraphs in an exam dangerous? Should we aim for at least 4? (but with three paras, obviously the word limit will be reached)?
Title: Re: VCE English Question Thread
Post by: Cristiano on October 13, 2015, 09:12:52 pm
Is writing 3 body paragraphs in an exam dangerous? Should we aim for at least 4? (but with three paras, obviously the word limit will be reached)?

3 is fine and a lot of people do it, it's probably what you'll have time to write, that being said that each paragraph is complex and somewhat lengthy.
It is better to write 3 very good in depth paragraphs than 4 shorter mediocre paragraphs.
Title: Re: VCE English Question Thread
Post by: RJA7445 on October 15, 2015, 03:56:55 pm
Hi, which would be a better text to write on so I can get a higher score for text response: Stasiland or Medea?
Title: Re: VCE English Question Thread
Post by: heids on October 15, 2015, 04:05:14 pm
Hi, which would be a better text to write on so I can get a higher score for text response: Stasiland or Medea?

Neither text is inherently 'better' - it's about which one you know better or are most comfortable with.  Which one do you know more quotes from?  Which one do you have deeper notes or insights on?  Which one have you spent longer planning/analysing on before?  Which one do you like better?  If you still can't decide, grab a few (like maybe 5) prompts for each text, and do a really quick brainstorm; whichever one has more ideas flowing is better.

But if you're unsure, it won't make too much difference, pick a name out of a hat.  Once you've picked one, stick to it and exclusively study it.
Title: Re: VCE English Question Thread
Post by: cosine on October 15, 2015, 04:05:37 pm
Someone mind reading my Medea essay for me, please? Any feedback is greatly appreciated, many thanks.

Title: Re: VCE English Question Thread
Post by: heids on October 15, 2015, 04:06:23 pm
Someone mind reading my Medea essay for me, please? Any feedback is greatly appreciated, many thanks.

Will do.  Tomorrow, 95% sure.
Title: Re: VCE English Question Thread
Post by: dandunu on October 15, 2015, 10:30:58 pm
Hey guys

I just finished a practise language analysis and I was wondering whether anyone would be able to mark it and perhaps give it a score out of ten?

Also, for some context, the piece I analysed is in the last few pages of this PDF: https://proffpoet.files.wordpress.com/2015/09/vate-english-exam-2015.pdf

Thanks in advance and sorry if its dodgy (especially the second paragraph and conclusion, was struggling to think of things to analyse), haven't done language analysis since the term 1 sac  :P
Title: Re: VCE English Question Thread
Post by: cameotodd on October 15, 2015, 11:15:11 pm
In a context piece, for the film that we are studying, are we able to use the actors, director etc and make up like a talk show interview session or a Q and A session for a magazine. The information however would be all made up and may not accurately reflect their own real personal views. For example for the film skin, I could interview the director - Anthony Fabian - and ask him a question such as his decision for directing the film, and the response from him would be completely made up by me and not actually real and or as to why he chose to direct the film. Would this be allowed?

Thanks, I hope you all sorta understand what I'm trying to say  haha
Title: Re: VCE English Question Thread
Post by: sunshine98 on October 16, 2015, 09:24:58 am
Hey guys

I just finished a practise language analysis and I was wondering whether anyone would be able to mark it and perhaps give it a score out of ten?

Also, for some context, the piece I analysed is in the last few pages of this PDF: https://proffpoet.files.wordpress.com/2015/09/vate-english-exam-2015.pdf

Thanks in advance and sorry if its dodgy (especially the second paragraph and conclusion, was struggling to think of things to analyse), haven't done language analysis since the term 1 sac  :P

hey there ,
I looked through your piece and attached some feedback
Hope I've helped  :)
Title: Re: VCE English Question Thread
Post by: heids on October 16, 2015, 05:43:03 pm
Someone mind reading my Medea essay for me, please? Any feedback is greatly appreciated, many thanks.

Why can I just not concentrate now.  Moral: don't try to mentally work after way too many hours with dementia patients.  Nah, for some reason marking anything but LA has been mashing my brain for the last month or two and I'm never happy.  Apologies for the mediocre quality of feedback, but hope it's helpful nonetheless!


Title: Re: VCE English Question Thread
Post by: cosine on October 16, 2015, 07:24:51 pm
Why can I just not concentrate now.  Moral: don't try to mentally work after way too many hours with dementia patients.  Nah, for some reason marking anything but LA has been mashing my brain for the last month or two and I'm never happy.  Apologies for the mediocre quality of feedback, but hope it's helpful nonetheless!

Thank you Bangali_Lok, means a lot..

Attached is another essay of Medea, took on your feedback of including Euripides' view, however, is it alright if we make direct conclusions of his opinions? Like saying Euripides does this... Euripides thinks this and that..? Are we allowed to make conclusions like this? If you read my essay you will see what I am talking about. Once again, your feedback is greatly appreciated and I owe you heaps..

Anyone else is welcomed to leave feedback, the more the better. Cheers.
Title: Re: VCE English Question Thread
Post by: Chang Feng on October 17, 2015, 12:16:49 am
Heyy, for English Context- Standard Expository essay style. Can we have like a creative- personal story type of introduction that implicitly signposts the ideas later on in the essay? or is that changing the style of writing to much from the standard expository essay body paragraphs.
Title: Re: VCE English Question Thread
Post by: dandunu on October 17, 2015, 09:47:46 am
hey there ,
I looked through your piece and attached some feedback
Hope I've helped  :)

Thanks for the feedback :)
Title: Re: VCE English Question Thread
Post by: cosine on October 17, 2015, 11:01:49 am
On the exam, what if I can't come up with a third argument/paragraph? For example, for the prompt: 'Consider some key points of difference between men's and women's experiences of life in this play. Is Euripides a feminist?'

Arguments:
- Euripides acknowledges the patriarchal, male-dominated society of 431BC and this is evident through Medea's downplay by the men of the play.

- However, Euripides is a feminist, and this is highlighted through Medea's ascension and rally of crimes as she fights for equality of rights as men.

- ?

If this was the case during an exam, what would you do?
Title: Re: VCE English Question Thread
Post by: tashhhaaa on October 17, 2015, 12:13:55 pm
On the exam, what if I can't come up with a third argument/paragraph? For example, for the prompt: 'Consider some key points of difference between men's and women's experiences of life in this play. Is Euripides a feminist?'

Arguments:
- Euripides acknowledges the patriarchal, male-dominated society of 431BC and this is evident through Medea's downplay by the men of the play.

- However, Euripides is a feminist, and this is highlighted through Medea's ascension and rally of crimes as she fights for equality of rights as men.

- ?

If this was the case during an exam, what would you do?

if you're desperate, you could break down the first point?
Ie. WHICH men were affected by Medea? (one para for each -- Creon and Jason or whatever else you can come up with, which could lead to Euripides being a feminist as things didn't go well for these men)
that's fairly simplistic but better than nothing in my humble opinion
Title: Re: VCE English Question Thread
Post by: banaidil on October 17, 2015, 02:34:09 pm
I wanted to know that if I start a text response with a really long sentence is that a bad thing?
Here is an example:  " Sketching a tale of the harsh and gruelling conditions endured by the indigenous Australians during Australia’s Great Depression, Jack Davis’ play ‘No Sugar’ depicts how some minority groups act out in communal defiance against the oppressive environment they lived in." And if there is any way I could word this in a more sophisticated manner, I would like to know.

Thank you :)
hey guys can you please answer this, im kinda stuck :)
Title: Re: VCE English Question Thread
Post by: tashhhaaa on October 17, 2015, 02:45:45 pm
hey guys can you please answer this, im kinda stuck :)

depends on the prompt, that sounds like one of those rote learned sentences that's not really necessary

it still makes sense but you could try to shorten it by saying something like

Jack Davis' play 'No Sugar' depicts the harsh and gruelling conditions endured by the indigenous Australians during Australia's Great Depression. The oppressive environment endured by this minority group... blah blah blah
Title: Re: VCE English Question Thread
Post by: Cristiano on October 17, 2015, 02:48:09 pm
hey guys can you please answer this, im kinda stuck :)
Not doing the play but here:
As a construction of the gruelling conditions encountered by indigenous Australians during Australia’s Great Depression, the play ‘No Sugar’ composed by Jack Davis intricately explores the marginalisation of various minority groups in their attempts at subsistence.
Title: Re: VCE English Question Thread
Post by: bobisnotmyname on October 18, 2015, 03:37:46 pm
What do you guys think about memorizing essays. I'm really bad at English and will be happy with above 30, over the moon with 35. With the little time time in the exam, I'm thinking of taking a gamble and hoping that the prompts are similar to what I wrote a good essay on and modifying it with little effort compared to writing a completely new essay. What do you guys think?
Title: Re: VCE English Question Thread
Post by: elysian on October 18, 2015, 08:35:10 pm
How would you start revising for the exam , I have no idea where to start.
Title: Re: VCE English Question Thread
Post by: cameotodd on October 18, 2015, 09:34:27 pm
Anyone mind reviewing my text response for 'In the Country of Men' and giving it a mark out of 10?

cheers :)
Title: Re: VCE English Question Thread
Post by: thaaanyan on October 19, 2015, 08:11:02 am
Anyone mind reviewing my text response for 'In the Country of Men' and giving it a mark out of 10?

cheers :)

No problem.
Attached. :) Hope this helps.
Title: Re: VCE English Question Thread
Post by: thaaanyan on October 19, 2015, 08:18:45 am
Anyone mind reviewing my text response for 'In the Country of Men' and giving it a mark out of 10?

cheers :)

ALSO: Minor error on the attached document. It was meant to read "where male privilege is culturally codified between the sexes." Like, the first sentence in. I forgot the word codified, so the correction doesn't make much sense.
Anyway, yeah, this is everything. Even the mark out of 10. Hope it helps.
Title: Re: VCE English Question Thread
Post by: cameotodd on October 19, 2015, 10:49:57 am
No problem.
Attached. :) Hope this helps.

Thanks a lot thaaanyan! this is so helpful!
Title: Re: VCE English Question Thread
Post by: banaidil on October 19, 2015, 12:01:34 pm
hi guys, can you please read this practice essay on 'No Sugar' and rate it out of ten plus some constructive feedback? thanks :)
Title: Re: VCE English Question Thread
Post by: StupidProdigy on October 19, 2015, 12:25:48 pm
hi guys, can you please read this practice essay on 'No Sugar' and rate it out of ten plus some constructive feedback? thanks :)
Not attached!
Title: Re: VCE English Question Thread
Post by: banaidil on October 19, 2015, 12:39:16 pm
hi guys, can you please read this practice essay on 'No Sugar' and rate it out of ten plus some constructive feedback? thanks :)
sorry i thought i had :) here it is
Title: Re: VCE English Question Thread
Post by: heids on October 19, 2015, 12:49:50 pm
Can we please keep essays to the English Work Submission and Marking board?

If I see any more I'm going to split them into a new thread in that board (you should probably get better exposure in a new thread anyway), it just makes it easier for people to see if there's an essay they can mark.

And everyone, I want to say thanks for your recent contributions in essay marking - especially sunshine98 and thaaanyan since they're the ones I've noticed most!  Keep it up!
Title: Re: VCE English Question Thread
Post by: gaydrea on October 19, 2015, 04:54:23 pm
for the english exam how many body paragraphs do you think is best to write for each essay. My english teacher has told my class to do 2 body paragraphs to make sure we fit into the time constraints, however some other english teachers at my school have said to do 3 so i'm not quite sure how many i'm meant to do.
Title: Re: VCE English Question Thread
Post by: wombat123 on October 19, 2015, 05:07:59 pm
Hi! I was wondering if anyone could help me with this Medea prompt? I've got a few ideas but not enough to write a whole essay on, especially in an exam.
This is the prompt:
‘Characters display both cool logic and fiery emotion in Medea, but Euripides is supportive of neither.’ Discuss.

These are my ideas so far:
Cool logic -
Jason’s reasons for abandoning Medea - betrayal leaves her distraught. Euripides doesn’t approve of this

Fiery emotion -
Medea’s decision to kill 4 ppl leads to the end of Jason’s legacy and a kingdom with no royalty - he is left with nothing. Euripides doesn’t approve of this but approves of balance.

I want to have at the least 3 body paragraphs, maybe even 4. Can anyone give me some ideas??? It would be much appreciated  :)
Title: Re: VCE English Question Thread
Post by: cosine on October 19, 2015, 05:18:37 pm
Hi! I was wondering if anyone could help me with this Medea prompt? I've got a few ideas but not enough to write a whole essay on, especially in an exam.
This is the prompt:
‘Characters display both cool logic and fiery emotion in Medea, but Euripides is supportive of neither.’ Discuss.

These are my ideas so far:
Cool logic -
Jason’s reasons for abandoning Medea - betrayal leaves her distraught. Euripides doesn’t approve of this

Fiery emotion -
Medea’s decision to kill 4 ppl leads to the end of Jason’s legacy and a kingdom with no royalty - he is left with nothing. Euripides doesn’t approve of this but approves of balance.

I want to have at the least 3 body paragraphs, maybe even 4. Can anyone give me some ideas??? It would be much appreciated  :)

Cool logic:
- Jason's abandonment of Medea leaving her distraught, Jason was cool about it and thought it was the best choice for him. Euripides definitely does not approve of this as evident through Medea's final crime spree, and most importantly, departing the scene not only without punishment, but leaving on a dragon-led chariot handed down by her grandfather, the Sun-god.

- King Creon's decision to grant Medea one day in Corinth to plan her refuge, in which she instead uses to plan out her terrible crimes. Euripides once again does not approve of Creon's decision by having Medea destroy the city of Corinth as a result of Creon's decision, despite knowing her 'dangerous temperament' and he knew that he would regret the decision.

Fiery emotion:
- Medea's final actions serve to astound everyone - the audience, city of Corinth and Jason. The fact that, once again, Medea ditched the concluding scene unharmed directed on her dragon-led chariot, proves that Euripides agreed and supported Medea's actions, for ultimately standing up to the male-dominated patriarchy of 431BC and thus possessing natures of a tragic hero.

- Although Euripides supports Medea's fiery emotions, the characters of the play don't and detest Medea for her unnatural crimes.
Title: Re: VCE English Question Thread
Post by: wombat123 on October 19, 2015, 05:30:18 pm
- Although Euripides supports Medea's fiery emotions, the characters of the play don't and detest Medea for her unnatural crimes.

Thanks heaps for your help! Means a lot!  :D
Title: Re: VCE English Question Thread
Post by: heids on October 19, 2015, 05:58:40 pm
for the english exam how many body paragraphs do you think is best to write for each essay. My english teacher has told my class to do 2 body paragraphs to make sure we fit into the time constraints, however some other english teachers at my school have said to do 3 so i'm not quite sure how many i'm meant to do.

3-4 paragraphs is normal - 2 would suggest an unfinished essay and you wouldn't have enough different ideas.

Euripides doesn’t approve of this but approves of balance.

Don't know the text, but how about you discuss this in a separate paragraph?  Basically you've said he doesn't approve of either on their own (in your first two paras), and then you pull it together and build it up to say that neither of them are necessarily bad, if taken in moderation and balance.  Are there any other characters that could suggest this?  Don't know if this would work, just an idea :)

P.S. I'd like to alert you all to Re: English Guides, Sample Pieces, Tips and Resources if you haven't already seen it.
Title: Re: VCE English Question Thread
Post by: cosine on October 19, 2015, 06:23:46 pm
Hey bangali lok,

Is it alright if I make a conclusive statement about Euripides on the exam/essay? For example say that euripides thinks this... thought that... reveals this.... through major events in the play?

Like the essay I uploaded on the essay correction page, I had one paragraph talking about how euripides supported Medea's plight and how it was evident through allowing her to escape with haven.
Title: Re: VCE English Question Thread
Post by: Cristiano on October 19, 2015, 10:30:09 pm
Hey bangali lok,

Is it alright if I make a conclusive statement about Euripides on the exam/essay? For example say that euripides thinks this... thought that... reveals this.... through major events in the play?

Like the essay I uploaded on the essay correction page, I had one paragraph talking about how euripides supported Medea's plight and how it was evident through allowing her to escape with haven.
Hopefully someone else can provide another response but:

i think you're not supposed to say the [Author] DEFINITELY does something or not, as this is just your interpretation. To some it might be,or might not be, and we don't know the author's actual intentions.
It would best to say something like 'Through Medea, Euripides explores _____
Euripides exposes the true nature of a patriarchial society...
Euripides suggests that ___ can be ______
Euripides questions, advocates, intends to, constructs!
Title: Re: VCE English Question Thread
Post by: imaware on October 19, 2015, 11:46:25 pm
For language analysis, do we have to begin by introducing the issue and what sparked it or could we go straight into introducing the actual piece?
Title: Re: VCE English Question Thread
Post by: bvb09 on October 20, 2015, 08:51:41 pm
For prompts such as this : “Everything I said was taken from me and altered until the story wasn’t my own”. In what ways is Agnes disempowered in Burial Rites?"

Is it OK to talk about both disempowerment and empowerment? Or is it just asking for the ways in which Agnes is empowered all throughout?
Title: Re: VCE English Question Thread
Post by: Cristiano on October 20, 2015, 09:01:21 pm
For prompts such as this : “Everything I said was taken from me and altered until the story wasn’t my own”. In what ways is Agnes disempowered in Burial Rites?"

Is it OK to talk about both disempowerment and empowerment? Or is it just asking for the ways in which Agnes is empowered all throughout?

Depends i think, is it a 'Discuss' prompt? if so, then yes to your question.
But if a 'do you agree?' type question or 'to what extent?' then i would think no.
Title: Re: VCE English Question Thread
Post by: imaware on October 21, 2015, 08:24:11 pm
For context, is having external references a must? or can we provide just examples from the text/movie we choose to talk about

Secondly, since we study 2 context texts and choose to talk about one on the exam, if we talk about the 'other' text does that count as external reference or not?

Thanks
Title: Re: VCE English Question Thread
Post by: duo0024 on October 21, 2015, 11:02:44 pm
For context, is having external references a must? or can we provide just examples from the text/movie we choose to talk about

Secondly, since we study 2 context texts and choose to talk about one on the exam, if we talk about the 'other' text does that count as external reference or not?

Thanks

For context, having external references is necessary if you are planning to write an expository piece.

You can only write about one of your context piece. There is a box on top of Section B where you must write the title of the text you are planning about. IF you mention both texts that you studied, it's kinda like double dipping- you're telling the examiner how lazy you are at researching external pieces and consequently will be marked down because of it.
Title: Re: VCE English Question Thread
Post by: _fruitcake_ on October 21, 2015, 11:11:22 pm
For context, is having external references a must? or can we provide just examples from the text/movie we choose to talk about

Secondly, since we study 2 context texts and choose to talk about one on the exam, if we talk about the 'other' text does that count as external reference or not?

Thanks

Talking about one is the best. However u can mention your chosen text subtly throughout your piece. In the highest responses in the examiner reports there are essays where they mention their texts in most of their paragraphs.. with external sources as well.. kinda integrating both of them together, its very nice.
Title: Re: VCE English Question Thread
Post by: literally lauren on October 22, 2015, 12:17:48 am
Just clearing up some questions that haven't been answered - let me know if I've missed any or just repost yours if they're further than three pages back and I'll try and get around to them :)

+ blanket reminder off the back of bangali+lok's that ESSAYS GO HERE:  Submissions Board and this thread is for Questions only!! Way more people have started getting about the marking train, which is excellent - just keep your hands and feet inside the submissions board at all times; please and thank you :)

I'm going to write a speech for section B and i'm unsure if its a good idea for my score to make my purpose etc as explicit as this-Transcript of speech delievered by...
At the ....
On the...
The given title of the speech is …
 other than that, what else is a good way to elevate a speech and address different aspects of a prompt i.e. is it too simple to agree with a prompt and try to persuade the audience to that perspective or do I definitely need to go further. thanks
Re: making this explicit, you can integrate this within your piece by having the speaker say things like 'As the Chairperson of the Domestic Violence Prevention Committee, I would like to say...' or 'I would like to thank my fellow Members of Parliament for attending this afternoon's conference on...'
Alternatively you can just have an opening paragraph like 'The following is a transcript of a speech delivered by ____ following the recent ____ etc.' Both would be fine.
Regarding 'making speeches better,' try to do more than just completely agree or disagree. It's usually more effective if you can take a persuasive purpose, but a more expository contention. For instance, if you were doing a prompt like: 'Our identities are formed through conflict.'
And your purpose was: A British Labor politician addressing his party following their loss at the last election explaining why the defeat is actually an opportunity to change and grow
Then your contention could be: Sometimes conflicts can seem insurmountable, but in the end, they can be beneficial to our trajectory in life, and provide us with the chance to alter ourselves for the better.
-as opposed to-
Purpose: school captain saying students should keep strong in the face of bullying etc.
& Contention: Identities are formed through conflict.
^which would be too simplistic.

So long as you've got a sufficient exploration of ideas, the assessors won't mind whether you're mostly agreeing or mostly disagreeing - just try to say something interesting and more complex than yes/no.

How do you incorporate an author's 'Bias' into the introduction? Like for the 2010 Biodiversity exam how is Lee biased, as he is a professor - he may contend etc.
How do you word this and smoothly weave it into the intro?

Any suggestions would be great as i am a bit lost :) Thanks
Bias isn't a technique, and you shouldn't bring this up as it can seem evaluative. Think about it: if you're calling the author biased, then you're suggesting that his arguments aren't persuasive, or otherwise you're using the background information and saying something about his role in the issue, which isn't what the task is meant to be about.
(I know this is kind of annoying, because I remember being told to talk about 'bias' as a technique for years and years. It doesn't really fit at a VCE level though.)
What I think you might mean instead is the author's authority - ie. the fact that Prof. Lee was a professor is meant to elicit a sense of respect towards him, and his views by proxy. However, I'd still say to leave that out of the introduction and just bring it up in the body paragraphs so that you can analyse it. The intro is really just for a brief bit of background info, and then an outlining of the contention.

I wanted to know that if I start a text response with a really long sentence is that a bad thing?
Here is an example:  " Sketching a tale of the harsh and gruelling conditions endured by the indigenous Australians during Australia’s Great Depression, Jack Davis’ play ‘No Sugar’ depicts how some minority groups act out in communal defiance against the oppressive environment they lived in." And if there is any way I could word this in a more sophisticated manner, I would like to know.
Long sentences are fine, and I wouldn't even say that one was too long. So long as it's grammatical, the assessors have no problem with it. But I'd recommend having one or two of these malleable intro sentences instead of using the same one for every essay. Let's say you get a No Sugar prompt about the importance of survival, or of power and family dynamics - are you going to begin with the same line about communal defiance every single time? It's best to engage with the prompt from the outset, if possible. Other than that, this is worded fine though, and I'm sure it'd work for a significant number of prompts, so it'd be good to keep up your sleeve.

Hey guys I'm kinda stuck- not sure which text response to do for the final exam.

I can do Henry IV, which I'm quite interested in though,but haven't really out much work in it, so will 2odd weeks be alright to study it?

Or the other book we did for unit 3, although I've done a lot of work on it, and don't want to waste it, it just doesn't seem that fun anymore (maybe because it was a long time ago)

Any advice please help?
Hopefully you've chosen by now, but I would absolutely recommend King Henry - there's heaps to talk about, and it's one of the few interesting texts on the list this year. Happy to help with this one since that was my exam text back in the day. There was a thread dedicated to HIV1 discussions last year, so you could hunt around for that if you were looking for some resources or interpretations. Otherwise, just knowing the prompt really well and being able to talk about wach of the characters both individually and comparatively will help a lot. Also, familiarise yourself with as many themes and combinations of themes as you can. Some structural evidence wouldn't go astray either, since it's a fourth year text and you'll likely get some tricky prompts this year, so looking into the symbolism of time, the presence of order, manipulation of language etc. would be a good starting point.

hi  :)

I am hoping for some advice on vocab in text response.

In the past, I've found that the most effective way of weaving fancier, less common words into text essays is to read scholarly articles, literary reviews and alike.
But what if there is nothing available? My text is Burial Rites, and as it is so new there is effectively nothing online to read about it, other than just a couple of brief reviews which don't really help much.

I understand that the likely response here is something along the lines of 'thought counts more than vocab', and while I agree that that is unquestionably true, I think you also have to be realistic. The essays approaching the highest end of the mark spectrum always have an impressive collection of phrases and terms fairly cleverly embedded, and to get the high marks that same approach needs to be taken.

I would say that naturally, my use of vocab is strong and expressive; it; it makes sense, but it doesn't have that wow factor unless I have quite a bit of time to 'craft' a piece, as opposed to pumping out 3 in as many hours. I would also like to make the point that I only use words if I am confident with them; I don't build my essays around language in the hope to impress, because I know that doesn't work.

Having said that, the language I use in my text response probably won't get my the marks I'm hoping for at the moment, and I was hoping for some advice on how to work past this.

Is it a good idea to 'craft' some perfect sentences/paragraphs and memorise them, instead of smashing out essays as often as possible?
What sort of stuff should I read to pick up vocab? Literary reviews of other books, extracting relevant ideas where necessary?
I get where you're coming from, and I can't advocate going into the exam completely blind and just expecting that you'll be able to come up with the right words on the spot. In reality, there's a reason you study your texts all year (or half a year if it's your Sem. 2 text) and the assessors expect that you'll be familiar with certain elements.
By now you've probably realised that there are some points of evidence that will work for many (--not all--) prompts, so going over that analysis and refining it would be a good idea. You've probably also seen a significant amount of overlap in the prompts, and that the general message (or "core") of the text is ostensibly the same each time, so having some rote learned 'views and values' sentences like 'Kent explicates the dangers of succumbing to delusions and false impressions of justice' that you can wheel out wherever relevant.
So long as you're aware that relevance is the priority and that you should only use what best fits the prompt, not what objectively sounds best in isolation, you should be fine :)

What do you guys think would be the optimal ratio between writing about your text and external examples in an expository essay for Context?
My standard recommendation is to talk about the text in a fair amount of detail in your first body paragraph, and then reference it again in your ~3rd B.P. just for a sentence or two. ie say something like 'Just as >character in the set text< was forced to confront the significance of their actions, so too did... >lead to external evidence<'
Try not to have it in every paragraph, and ideally your external discussion will make up the bulk of your essay.
^ Having said all that, you could do the complete opposite and still score perfect marks. Context is funny like that, and I can't wait for this time next year when I'll finally be able to say goodbye to it forever. #1year&1week but who's counting

I have read somewhere on this forum to write on every second line in an essay so that you can have space for editing. Is this for in exams or just for SAC's and practice essays. Like, do the assessors mind???

Also, I have got a problem where in the context section of the exam I always write my piece in 15-20 min. Then I get another idea, which i think 'hey that will be a lot better' and then i write that one for the remaining 30 min. What do you think i should do??? It seems pointless writing two, but I never seem to be able to get the other idea unless i write the first one.
Re: skipping a line, yes, it's completely fine, and I'm actually recommending it to students who have big/small/messy handwriting. If you're writing's big, it lets you space it out across the page and makes things easier to read - if you're writing's small, it kind of forces you to write a bit bigger and more legibly by virtue of having more space to do so. And if you're a messy writer, it gives you the room on either side of a line to make edits and corrections where needed.
For context: try to account for this in planning time. Rather than spending 20 minutes writing a mediocre or sub-standard (based on your estimations of your ability?) piece, spend 5-10 minutes coming up with a solid plan so that you can come up with ideas you're happy with, and then start writing your piece. Also, is there any reason why you can't incorporate your new ideas into the one piece, or into the first one you write? Are you coming up with completely contradictory contentions, or are you writing creative pieces that can't really be tacked on to one another? If it's just an expository/ hybrid piece, I don't see any reason why you couldn't just extend your discussion to incorporate a different viewpoint, but let me know if the style/format is affecting your decision.

Im having trouble doing an LA piece. I am doing it on the 2014 LA part. There is the main article and a comment section.

So far i got
1 - intro for main article

2- Body Par 1 about main article

3 - Body Par 2 about main article

4- Body Par 3 about main article

Im stuck here, what should i do?

5- conclusion about main article?

6- intro for comment section?

7- Body par 1 for comment section?

8- conclusion for comment section?
Alternatively, here's a structure I'd recommend
~~and one others should read because the chances of getting an exam similar to the 2014 one (ie main piece with a comment) again this year is quite high, but you didn't hear that from me *nudge nudge*
---------------------------------
If you're getting one main article, two visuals, and a comment:
Paragraph 1: 3/4: main article + 1/4 visual (1)
Paragraph 2: 3/4: main article + 1/4 comment
Paragraph 3: 3/4: main article + 1/4 visual (2) + (comment - if not sufficiently covered in other para.)
---------------------------------
So each paragraph contains at least two 'texts' (ie. the main piece and one other; either visual or written) but no paragraph is burdened with having to deal with every single text all at once. This'll let you draw sufficient connections across the material and cover the whole scope of the thing without comparing at the expense of analysis.
That's how I'd do it anyway, and it probably suits a key player/ sub-argument method best, so if you have your own way of handling things, it might be best to stick with that at this point of the year. Ultimately the assessors care way more about the quality of your analysis anyway - the format and essay structure should be aspects that enhance (or at least don't hinder) your analysis anyway, so don't stress too much.

if the language analysis article has no named author, will we get penalised for referring to them with gendered pronouns?
Also, I was wondering if I can get a 40 raw if my GA's are 90, 93, and 49/60 respectively.
Thanks!
The 2013 VCAA exam had no author, and that annoyed me to no end because that was my exam and I'm pretty sure I alternated the phrases 'the writer'/'the author'/'the piece' so often that to this day my hand makes the motion of writing those words in my sleep.
It's possible you won't get an actual name, but it's unlikely if you get multiple authors since that'd get messy. I'd recommend just picking a gender and then being consistent with it - so go for either he or she and then just ensure you don't switch halfway through. They can't penalise you, but it could confuse them (again, especially if there are multiple authors.) Technically, even if you had a piece written by Brittany Girlson that was delivered at a Women's Rights Convention and you referred to the author as 'he,' they technically can't penalise you that much because you're being marked on your ability to analyse language :P

OKay so I've been trying to fix this problem of mine for a while now and I just keep doing it... I tend to overwrite in language analysis due to my innate fear of running out of things to analyse (I somehow feel bad if I don't analyse every good persuasive technique or connotations because it would feel like a waste to not analyse it-weird huh?) and because of this, I tend to spend too much time on L.A or spend half of my time analysing the first quarter of the text and the last doing a rushed analysis of the rest. Anyone have any tips? I tend to favour really large chunky paragraphs so maybe I should just start writing a lot of short paragraphs?
Cheers
Keep up with the longer paragraphs, as they tend to be stronger, but prioritise the stuff that you find most important, rather than just 'ooh! I can say something about this part!' Is it a worthwhile 'something?' If so, keep it. Or, if it just seems like a bit of filler analysis/ something you've already said before and better, cut it.
Also, forcing yourself not to do things chronologically can really help. Attempting some kind of sub-argument structure whereby each paragraph focuses on one particular part of the author's contention can be much more beneficial, because you won't be limited to the structure that the author has chosen.
Every year since 2009, VCAA have deliberately scattered the worthwhile language across the piece, but given you a significant amount to talk about at the start and end so that anyone doing things chronologically would be inclined to spend way too much time talking about the beginning and would neglect the ending. That separates the people who can analyse well from the people who can analyse well AND select the points to analyse and present them in a more orderly fashion.
Ultimately, it's a good thing that you're writing too much, but going over your old essays and being hyper-critical about what's actually necessary and what's just filler can help you break things down the core of what needs to be there. Keep in mind that VCAA are testing what you choose to analyse just as much as they are your analytical abilities, so prioritise the important/'meaty' stuff over assorted bits and pieces that don't really advance the author's point by much.

Hi everyone,
Was wondering what the difference between a 9 and a 10 in an essay is? Is it just expression and cohesion?
Personally I've gotten 10s for text and 10s for context before, but keep getting 9s for section c :/ !!!!!
I've spoken to my teacher and she says I shouldn't be complaining LOL

Help would be much appreciated

Thanks
I hate your teacher. I had a Lit. teacher like that in Year 12 and I'm still kind of bitter over the way she dismissed students who were doing 'well enough not to warrant any attention' -.-
If you want to post one of your L.A. pieces on the Submissions Board, someone (may or may not be me depending on how hectic the next week is) can give you some proper feedback on where you need to improve, but without having read your work, it's hard to tell.
A lot of things can separate 9s and 10s; it could be expression, but it could also be the quality (depth or breadth) of your analysis, or the scope of your discussion, or the quality of your visual analysis, or your essay structure, or your linking and flow... you see where I'm going with this? You might be able to find the problem yourself if you go through your own work really thoroughly and evaluate your own performance, but if you're not too certain about the criteria and don't know where you're going wrong, drop by the Markings Board and someone can send you in the right direction :)
#Atarnotes: making up for shitty teaching since 2007

Is writing 3 body paragraphs in an exam dangerous? Should we aim for at least 4? (but with three paras, obviously the word limit will be reached)?
Not dangerous at all. Three or four is totally fine, no less, and no more unless you're really prolific and quick. Aiming for five+ is usually deleterious to overall quality, but there's no inherent advantage or disadvantage to doing three instead of four.
Some prompts require more depth (ie. three would be better) than breadth (ie. four) so you can use your intuition. It might also depend on your writing style and speed, so don't stretch for an extra paragraph if you've only got five minutes left; just wrap up nicely and rest assured that three paragraphs is completely sufficient for a high score; (more so, in fact, than if you write three good paragraphs and then undo that good work at the end with a really slapdash paragraph.)

In a context piece, for the film that we are studying, are we able to use the actors, director etc and make up like a talk show interview session or a Q and A session for a magazine. The information however would be all made up and may not accurately reflect their own real personal views. For example for the film skin, I could interview the director - Anthony Fabian - and ask him a question such as his decision for directing the film, and the response from him would be completely made up by me and not actually real and or as to why he chose to direct the film. Would this be allowed?

Thanks, I hope you all sorta understand what I'm trying to say  haha
Absolutely - I had a few students attempt something similar over the course of this year. Just try to ensure that you move beyond the text, ie. don't have Fabian talking about the significance of all the stuff in Skin and nothing else, because that'd be more like an extensive text response from a weird perspective. Whereas, if you use his voice to talk about other things (eg. his inspiration, real world parallels, his own life- if relevant(?) or other points of interest) then it's totally fine. Just make sure whatever your doing comes across clearly to the assessor so they don't have to get 600 words in before realising 'oh I get it - he's the director!!' A brief bit of explanation at the start goes a long way :)

Heyy, for English Context- Standard Expository essay style. Can we have like a creative- personal story type of introduction that implicitly signposts the ideas later on in the essay? or is that changing the style of writing to much from the standard expository essay body paragraphs.
That's totally fine, provided it isn't too clunky ŕ la:
Prompt: The consequences of conflict are often unforeseen.
One time, on the bus home from school, I was sitting next to a kid whose girlfriend broke up with him over the phone. He totes flipped out and accidentally smacked me in the face, and I got a sore nose. So because his girlfriend broke up with him, I got a sore nose. Through this, we can conclude that the consequences of conflict are often unforeseen.

I'm so glad I don't have to write real assessor's reports... I'd be such a troll...

But so long as you're aware of the different styles you're using and make some effort to blend them from your intro into your first paragraph, you should be fine. Creative/hybrid 'hooks' can be great ways of making your piece more readable and interesting, so I think it's a very good option for the exam, provided it fits the prompt on the day.

What do you guys think about memorizing essays. I'm really bad at English and will be happy with above 30, over the moon with 35. With the little time time in the exam, I'm thinking of taking a gamble and hoping that the prompts are similar to what I wrote a good essay on and modifying it with little effort compared to writing a completely new essay. What do you guys think?
I've spoken at length about this... somewhere on this board... but in short: memorise parts of essays, not whole chunks of essays. You're setting yourself up for failure if you just commit paragraphs to memory and churn them out on the day, but if you have set points of analysis that you're familiar with, and a good understanding of the overall message of the text + some sentences to demonstrate this, there's nothing wrong with using what's familiar to you and fitting it to your exam prompt. Just be aware that relevance rules everything, and you shouldn't compromise the integrity of your piece by just using half-baked bits and pieces from other essays and stitching them together. That's what's known as an Ugly-Frankenstein-Essay... which I'm going to have to find another word for since Frankenstein is genuinely going to be on the text list next year  :-\

How would you start revising for the exam , I have no idea where to start.
Know what your weaknesses are, and start there. Once you know what those weaknesses are, then you can either work on them yourself if the path is obvious, or let us know in this thread what you feel you need to work on, and someone can help you along :)

for the english exam how many body paragraphs do you think is best to write for each essay. My english teacher has told my class to do 2 body paragraphs to make sure we fit into the time constraints, however some other english teachers at my school have said to do 3 so i'm not quite sure how many i'm meant to do.
At least three. ^See above advice in response to cosine's question

For language analysis, do we have to begin by introducing the issue and what sparked it or could we go straight into introducing the actual piece?
There's no requirement to include background info, but a lot of people do out of habit anyway. If you think it's relevant then go for it, but you're free to just jump in and outline the necessary biographical details (ie. author, title) and the contention if you want. If you are including the spark/issue though, be sure to keep it to one sentence or less - going overboard is typical of mid-range essays that have nothing better to do :P I'd favour the more direct route, but there's nothing wrong with a bit of 'Following the recent...' to get you started.

For prompts such as this : “Everything I said was taken from me and altered until the story wasn’t my own”. In what ways is Agnes disempowered in Burial Rites?"

Is it OK to talk about both disempowerment and empowerment? Or is it just asking for the ways in which Agnes is empowered all throughout?
Only use empowerment as a reference point for talking about disempowerment, which is the focus of the prompt (and the quote for that matter.) Having a whole paragraph devoted to empowerment would probably be too much, but you could bring it up in a few B.P.s just as a means of clarifying what you're talking about. Keep it close to the focus of the prompt though, and you should have plenty to talk about.

Also, just to clarify:
Depends i think, is it a 'Discuss' prompt? if so, then yes to your question.
But if a 'do you agree?' type question or 'to what extent?' then i would think no.
Different teachers will say different things, but there's not really much substantial different when it comes to the 'discuss'/'do you agree'/'to what extent' questions used in English prompts. Realistically, you're always 'discussing the extent to which you agree,' so it's just a matter of phrasing that they use to spice things up. The more definitive statements tend to get the 'Do you agree?' marking, but you can treat them as though they're the same.
This was pointed out to me long after I finished Year 12 by a teacher who said 'yeah, we basically just switch out the words if we feel like we've overused some on an exam - there's no difference' so although semantically they do mean different things, that doesn't translate to any real discrepancy in marks at the end of the year :P This subject is so strange sometimes... "PAY ATTENTION TO LANGUAGE... oh no, not that language... that language doesn't matter... you should've known that..."



Sorry I haven't been around much to field some of these, but I'll endeavour to drop by more often in the lead up to the exams. Hope the study is going well for all of you!
T-6 days #gethyped
...just me who's hyping
...okay, that's fine... I'll just sit in a corner with my party hat on... with all the other cool kids...
(http://i38.photobucket.com/albums/e127/mcollinknight/balloon_drop.gif)
Title: Re: VCE English Question Thread
Post by: Cristiano on October 22, 2015, 12:45:43 pm
chances of getting an exam similar to the 2014 one (ie main piece with a comment) again this year is quite high, but you didn't hear that from me *nudge nudge

Do you know something we don't ;) ?

Brace yourselves people, VCAA are coming  :o
Title: Re: VCE English Question Thread
Post by: literally lauren on October 22, 2015, 01:02:53 pm
Nothing concrete; just some patterns I've noticed and some rumours on the ol' grapevine.

I could be wrong, but it can't hurt to prepare for a comparative exam just in case :)
Title: Re: VCE English Question Thread
Post by: Lawyer on October 22, 2015, 01:08:16 pm
Nothing concrete; just some patterns I've noticed and some rumours on the ol' grapevine.

I could be wrong, but it can't hurt to prepare for a comparative exam just in case :)

How do u prepare for a comparative exam? The questions cant be the same.
Title: Re: VCE English Question Thread
Post by: Cristiano on October 22, 2015, 01:12:50 pm
Nothing wrong with a comparative, the trial exams for this year have been comparative - probably included as a result of last year's shock and downfall of some.

How do u prepare for a comparative exam? The questions cant be the same.

Practicing, it's fairly simple as you don't have to analyse the first article as you normally would and the response usually has contrasting views. If you've done the trial exams/2014 VCAA you'll know, if not then attempt the Section C of Lauren's 2015 free practice exams.
Title: Re: VCE English Question Thread
Post by: Lawyer on October 22, 2015, 01:31:17 pm
Nothing wrong with a comparative, the trial exams for this year have been comparative - probably included as a result of last year's shock and downfall of some.

Practicing, it's fairly simple as you don't have to analyse the first article as you normally would and the response usually has contrasting views. If you've done the trial exams/2014 VCAA you'll know, if not then attempt the Section C of Lauren's 2015 free practice exams.

So since VCAA creates the articles, they are usually on the 'repetitive' side? That's interesting. I have done the 2014 VCAA  and found section C a bit shallow; just the same use of persuasive techniques. If that's the same with this year then wonderful.
Title: Re: VCE English Question Thread
Post by: Rainbow19 on October 22, 2015, 10:50:30 pm
What's your advice on made up stories for context?
Title: Re: VCE English Question Thread
Post by: shivaji on October 23, 2015, 08:14:36 pm
In Henry IV, Hotspur gets furious by reading a letter outlining the dangers of the plans for the rebellion. (Act 2 Scene 3)
Might have misread, but does anyone know who wrote this letter?

Cheers
Title: Re: VCE English Question Thread
Post by: tashhhaaa on October 23, 2015, 09:00:02 pm
if we're given a comparative that has a lot going on eg. 3 comments, how do we address the comments?

do we state all of their contentions in the intro? do we analyse them all separately as though they were an article?
Title: Re: VCE English Question Thread
Post by: Cristiano on October 23, 2015, 09:07:42 pm
if we're given a comparative that has a lot going on eg. 3 comments, how do we address the comments?

do we state all of their contentions in the intro? do we analyse them all separately as though they were an article?
I would just quickly mention in the intro that there were three responding comments to the piece and then later on just analyse them one by one. (In contrast ABC contends...) Likewise, 123 argues .... and Similarly, 999 asserts that..) If one of the comments are supporting the original article then state that 'Supporting the views of the (Author), 999 asserts)

That's how i'd do it and have done it, not sure if it's entirely correct but someone can verify this hopefully :)
Title: Re: VCE English Question Thread
Post by: tashhhaaa on October 24, 2015, 01:55:11 pm
I would just quickly mention in the intro that there were three responding comments to the piece and then later on just analyse them one by one. (In contrast ABC contends...) Likewise, 123 argues .... and Similarly, 999 asserts that..) If one of the comments are supporting the original article then state that 'Supporting the views of the (Author), 999 asserts)

That's how i'd do it and have done it, not sure if it's entirely correct but someone can verify this hopefully :)


thank you!
Title: Re: VCE English Question Thread
Post by: 99.90 pls on October 24, 2015, 01:56:35 pm
Because of Wag the Dog is in this year's Whose Reality text list, would it be safe to assume that prompts pertaining to memories, shared experiences etc. will not appear this year? For instance, last year (the first year Wag the Dog appeared on the text list), the prompt was ‘Misrepresenting reality can have serious consequences', which fits perfectly with Wag the Dog, because the film is all about manipulating people's reality for political gain and the consequences which arise from doing so.

Surely the prompt must relate well to all four texts for Whose Reality? Or does VCAA expect you to know more than one Whose Reality text? Because we did Death of a Salesman as well, but I don't want to have to prepare for both texts for Context...
Title: Re: VCE English Question Thread
Post by: Cristiano on October 24, 2015, 02:06:10 pm
Because of Wag the Dog is in this year's Whose Reality text list, would it be safe to assume that prompts pertaining to memories, shared experiences etc. will not appear this year? For instance, last year (the first year Wag the Dog appeared on the text list), the prompt was ‘Misrepresenting reality can have serious consequences', which fits perfectly with Wag the Dog, because the film is all about manipulating people's reality for political gain and the consequences which arise from doing so.

Surely the prompt must relate well to all four texts for Whose Reality? Or does VCAA expect you to know more than one Whose Reality text? Because we did Death of a Salesman as well, but I don't want to have to prepare for both texts for Context...

The prompt may still include memories and shared experiences with the first year novel Foe appearing. This novel is not certainly based on memories, but certain parts you could draw upon are to do with past experiences. So don't completely rule this one out as we don't know what could appear, but ad least have a plan for what to do if it does. (most likely wont appear but yeah, be ready)
Title: Re: VCE English Question Thread
Post by: 99.90 pls on October 24, 2015, 02:10:46 pm
The prompt may still include memories and shared experiences with the first year novel Foe appearing. This novel is not certainly based on memories, but certain parts you could draw upon are to do with past experiences. So don't completely rule this one out as we don't know what could appear, but ad least have a plan for what to do if it does. (most likely wont appear but yeah, be ready)

I know that other texts would definitely pertain to shared experiences/past experiences, but I guess the point I'm trying to make is that Wag the Dog doesn't whatsoever, so surely VCAA wouldn't leave all Wag the Dog students up shit creek without a paddle? Or would they expect you to make do or know another text?
Title: Re: VCE English Question Thread
Post by: Lawyer on October 24, 2015, 02:17:55 pm
I know that other texts would definitely pertain to shared experiences/past experiences, but I guess the point I'm trying to make is that Wag the Dog doesn't whatsoever, so surely VCAA wouldn't leave all Wag the Dog students up shit creek without a paddle? Or would they expect you to make do or know another text?

Im pretty sure the context prompt should relate to all texts in some way. In my case for conflict- i have managed to talk about the position of women in my selected text in every single vcaa prompt since 2008.
Title: Re: VCE English Question Thread
Post by: Cristiano on October 24, 2015, 02:20:05 pm
I know that other texts would definitely pertain to shared experiences/past experiences, but I guess the point I'm trying to make is that Wag the Dog doesn't whatsoever, so surely VCAA wouldn't leave all Wag the Dog students up shit creek without a paddle? Or would they expect you to make do or know another text?

Oh, I see. Correct, i am doing "Foe" and "WTD" so
1) No, the prompt MUST be relatable to all texts, you won't be required to know another text as it should suit all 4
2) Brean and his team have constructed an imaginary war, which means (something on imagination could come in) and also the fact that their songs such as "303" permit those Americans in the way they remember the war..

Not sure, but it's highly unlikely there'd be one on memories as it is too specific and probably doesn't relate that well to all texts.

What type of prompt are you hoping for? I have a feeling one either on power (imposed reality) or subjective reality will be on it!

Title: Re: VCE English Question Thread
Post by: tashhhaaa on October 24, 2015, 02:50:59 pm
Because of Wag the Dog is in this year's Whose Reality text list, would it be safe to assume that prompts pertaining to memories, shared experiences etc. will not appear this year? For instance, last year (the first year Wag the Dog appeared on the text list), the prompt was ‘Misrepresenting reality can have serious consequences', which fits perfectly with Wag the Dog, because the film is all about manipulating people's reality for political gain and the consequences which arise from doing so.

Surely the prompt must relate well to all four texts for Whose Reality? Or does VCAA expect you to know more than one Whose Reality text? Because we did Death of a Salesman as well, but I don't want to have to prepare for both texts for Context...


Similar question:
Does anyone think there will be a prompt about change and personal growth for ID&B? My teacher fucked us over for our SAC and gave us 'Sometimes we need to accept change in order to grow' (VCAA 2013) when The Mind of a Thief wasn't even on the text list in 2013... (I think)

My other text is Skin and of course there are changes in MOAT and Skin but no one really grows because of it -- it ends up being destructive? Does anyone agree? (I really struggled with this prompt)

Title: Re: VCE English Question Thread
Post by: 99.90 pls on October 24, 2015, 02:56:21 pm
Oh, I see. Correct, i am doing "Foe" and "WTD" so
1) No, the prompt MUST be relatable to all texts, you won't be required to know another text as it should suit all 4
2) Brean and his team have constructed an imaginary war, which means (something on imagination could come in) and also the fact that their songs such as "303" permit those Americans in the way they remember the war..

Not sure, but it's highly unlikely there'd be one on memories as it is too specific and probably doesn't relate that well to all texts.

What type of prompt are you hoping for? I have a feeling one either on power (imposed reality) or subjective reality will be on it!

Oh okay, that's reassuring how they don't require you to know another text. As for WtD, the 'imagination' aspect is interesting and plausible (although a scary idea, because I would be direly fucked) but yeah, I agree that memories/past experiences can be safely ruled as impossible. The '303' aspect is a bit tangential, and is a very minor element of the film. I guess it's time to do some 'imagination' preparation!

I'm hoping for consequences of manipulating/constructing reality, the difficulty of trusting in (objective) reality or believing illusions over reality. I reckon it'll almost definitely be one of these three or subjective reality or imagination (the latter two I'm not prepared for)
Title: Re: VCE English Question Thread
Post by: Cristiano on October 24, 2015, 03:43:47 pm
I'm hoping for consequences of manipulating/constructing reality, the difficulty of trusting in (objective) reality or believing illusions over reality. I reckon it'll almost definitely be one of these three or subjective reality or imagination (the latter two I'm not prepared for)

For constructing reality, do you mind giving an example of consequences of this in WTD or /other examples?

I think i'd be fine with the other two, Subjective reality would be great :) though it might be something like 'those with power can construct reality....' etc.
Cheers :)
Title: Re: VCE English Question Thread
Post by: 99.90 pls on October 24, 2015, 04:37:43 pm
For constructing reality, do you mind giving an example of consequences of this in WTD or /other examples?

I think i'd be fine with the other two, Subjective reality would be great :) though it might be something like 'those with power can construct reality....' etc.
Cheers :)

Moral consequences for the propagandists, and also think about the people on the receiving end of the propaganda, how it would affect their perceptions of reality and the loss of freedom (since you're essentially manufacturing their consent).

Just wondering what you did for subjective reality? Even some a couple of points would kick-start me, thanks :)

UNRELATED: Does anyone feel like one hour is way too little to annotate one/two language analysis pieces AND write an 800-1000 word piece? Has anyone in the past tried an exam structure like:
Section A - 50 minutes
Section B - 50 minutes
Section C - 1 hour 20 minutes
Title: Re: VCE English Question Thread
Post by: Cristiano on October 24, 2015, 04:52:00 pm
Moral consequences for the propagandists, and also think about the people on the receiving end of the propaganda, how it would affect their perceptions of reality and the loss of freedom (since you're essentially manufacturing their consent).

Just wondering what you did for subjective reality? Even some a couple of points would kick-start me, thanks :)

UNRELATED: Does anyone feel like one hour is way too little to annotate one/two language analysis pieces AND write an 800-1000 word piece? Has anyone in the past tried an exam structure like:
Section A - 50 minutes
Section B - 50 minutes
Section C - 1 hour 20 minutes

Thanks, for subjective realities i based mine on the novel Foe but i would say for WTD:

a) the singular perspective the story is told from, we only see one side of the story- the reality manipulators
b) We know that it is false, whereas the american public dont (the way you see things)

with the second question.

yes, i have done 50,50, 1.20
This is achieved by:
Reading Time: Plan A,B , Read over C twice.
Writing time= Finish A in 50 mins, Finish 5 in 50 mins, devote the rest of the time to write as much as possible on section C.
I actually prefer this method apart from the fact that u need to quickly do the other two and may have to re-read the article :) let me know if theres anything else i can answer!
Title: Re: VCE English Question Thread
Post by: S33667 on October 24, 2015, 04:54:30 pm
I agree that memories/past experiences can be safely ruled as impossible.

I've also not done any work on this for the reasons you mention.
Title: Re: VCE English Question Thread
Post by: Alter on October 24, 2015, 04:57:10 pm
UNRELATED: Does anyone feel like one hour is way too little to annotate one/two language analysis pieces AND write an 800-1000 word piece? Has anyone in the past tried an exam structure like:
Section A - 50 minutes
Section B - 50 minutes
Section C - 1 hour 20 minutes
I do the English exam in CAB order and find that I always hit 60-70 minutes before I finished Section C. Personally, I find it is the one you have to devote the most amount of time because you can't take in as many ideas that you've already considered in the past. As a result, I don't spend as much time planning for section A/B, but this doesn't really hurt my work.

Contrary to what Cristiano is saying, I would absolutely always do section C first. You're going to be investing such a large proportion of your reading time to it that to change your focus for an hour or two would be, in my mind, to put your ideas in reading time straight into the bin. I would even go so far as to say I might not even read the section b prompt until I've finished writing it.
Title: Re: VCE English Question Thread
Post by: Lawyer on October 24, 2015, 05:00:10 pm
Moral consequences for the propagandists, and also think about the people on the receiving end of the propaganda, how it would affect their perceptions of reality and the loss of freedom (since you're essentially manufacturing their consent).

Just wondering what you did for subjective reality? Even some a couple of points would kick-start me, thanks :)

UNRELATED: Does anyone feel like one hour is way too little to annotate one/two language analysis pieces AND write an 800-1000 word piece? Has anyone in the past tried an exam structure like:
Section A - 50 minutes
Section B - 50 minutes
Section C - 1 hour 20 minutes
Thanks, for subjective realities i based mine on the novel Foe but i would say for WTD:

a) the singular perspective the story is told from, we only see one side of the story- the reality manipulators
b) We know that it is false, whereas the american public dont (the way you see things)

with the second question.

yes, i have done 50,50, 1.20
This is achieved by:
Reading Time: Plan A,B , Read over C twice.
Writing time= Finish A in 50 mins, Finish 5 in 50 mins, devote the rest of the time to write as much as possible on section C.
I actually prefer this method apart from the fact that u need to quickly do the other two and may have to re-read the article :) let me know if theres anything else i can answer!

I have tried exactly that structure and i got absolutely destroyed. All i was doing in Part A was thinking how i would deal with Part B and C, for me its nerve racking. My structure is:

1) Spend 15 min reading time going over C and analyzing, make points in the head- Write for 1 hour. Do not look at Section A + B
           2 hours + 15 min left at this stage.
2) Spend 2-3 min on structure/points of your text response and write for 1 hour. Again Do not look at Section B
             1 hour + around 10 min left
3) Spend 2-3 min on structuring points/essay for Section B. Write for 1 hour
             Around 5 min left
4) Go back and pray u didnt fuck anything up
Title: Re: VCE English Question Thread
Post by: pi on October 24, 2015, 05:03:25 pm
I looked at all the sections during reading time, focused more on LA and tried to read that a couple of times. Then I gave myself an hour for LA, then did my TR, then did Context (only needed like 30-40 mins thanks to some cheeky memorising of adaptable pieces). Then I came back to LA to finish it off (ended up just being a conclusion). Then I had a sigh of relief.

The best thing about English is when it's over haha.
Title: Re: VCE English Question Thread
Post by: Cristiano on October 24, 2015, 05:10:18 pm
I do the English exam in CAB order and find that I always hit 60-70 minutes before I finished Section C. Personally, I find it is the one you have to devote the most amount of time because you can't take in as many ideas that you've already considered in the past. As a result, I don't spend as much time planning for section A/B, but this doesn't really hurt my work.

The thing is with section A and section B, you need to memorise a ton of quotes, examples, ideas and (maybe paragraphs), so you would instantly be able to just write as much as you can once you see A and B at the start of the exam.
with section C, you can then basically write as much as you can instead of limiting your analysis to only 3 or 4 pages. However, if it is a comparative analysis, then remove all my ideas and messages and that one first

I've heard "the exam is in an order for a reason" and my sister who got a 43 did exactly ABC, so i guess it depends on the person and the given situation, prompts etc. if you dont really know how to tackle A or B then it would be best doing C first and then hopefully you can plan or have come up with something to say
Title: Re: VCE English Question Thread
Post by: literally lauren on October 24, 2015, 05:15:08 pm
What's your advice on made up stories for context?
There's no point telling the truth, since that'd just be dull, and it's not like the assessors are giving you marks on how accurate your story is (ie. if you write about a fight you had with your siblings but they somehow stalk your VTAC number and find out you don't have any siblings... they don''t care, so long as the ideas are good.)
So you can make up whatever you want; write about yourself*, write from the perspective of a character in your set text, adopt a voice of someone from history (eg. the inner monologue of Tony Abbot right after the libspill, or a POV narrative set in a post-nuclear future - anything) just so long as there's a valid reason for you making that choice. Try not to just rely on the plot or some novel aspect in order to get across your ideas. The reason most people opt for an expository style is because it's usually a more direct line of communication between you and the assessor, whereas creative pieces and stories tend to be more ambiguous, but that doesn't mean they can't be done well.

*Just don't make them "petty personal stories," as the examiner's reports used to say. The really tacky things like this example from the last page:
Prompt: The consequences of conflict are often unforeseen.
One time, on the bus home from school, I was sitting next to a kid whose girlfriend broke up with him over the phone. He totes flipped out and accidentally smacked me in the face, and I got a sore nose. So because his girlfriend broke up with him, I got a sore nose. Through this, we can conclude that the consequences of conflict are often unforeseen
...just come across as flimsy and insubstantial. Stories can be done well, but they take some work, so it just depends where your strengths lie.

In Henry IV, Hotspur gets furious by reading a letter outlining the dangers of the plans for the rebellion. (Act 2 Scene 3)
Might have misread, but does anyone know who wrote this letter?

Cheers
I think that letter is from the Archbishop of York, which really isn't made plain so I can't blame you for not knowing - it took me a few readings and I thought I knew that text well :P
It's a little tricky because he kind of contradicts himself by first claiming that the Archbishop is a hypocrite, then a coward, then a traitor, then an ally, then a traitor again {...perhaps an indication of his haphazard logic and impetuosity}
His character is technically listed at the start, and we see him chat with Sir Micheal in Act 4 Scene 4, but his only role is aptly summed up by lit charts:
     "Richard Scroop – The Archbishop of York, he is a rebel and friend of the Percy family who worries from a distance about their
      treason plot. He supports the rebels not with military might but with strategic advice and letters to potential allies."
There's not a lot to say about him, and the only thing I'd make of that 2.3 letter reading scene is the fact that right from the start, Hotspur exhibits a stubbornness and refusal to entertain the thought of retreat or loss, and the fact that he lists his supporters to be his father, uncle, Mortimer, Glendower, and Douglas - most of whom either end up dead, captured, or just never rock up in the first place. So by the end of the play, all the support Hotspur thought he had ends up abandoning him, whereas Hal rides into battle with his father, brother, and allies by his side, suggesting the tables have turned, and whatnot.

if we're given a comparative that has a lot going on eg. 3 comments, how do we address the comments?

do we state all of their contentions in the intro? do we analyse them all separately as though they were an article?
I would just quickly mention in the intro that there were three responding comments to the piece and then later on just analyse them one by one. (In contrast ABC contends...) Likewise, 123 argues .... and Similarly, 999 asserts that..) If one of the comments are supporting the original article then state that 'Supporting the views of the (Author), 999 asserts)

That's how i'd do it and have done it, not sure if it's entirely correct but someone can verify this hopefully :)
Verified :)
Exactly what Christiano said; a brief statement in the introduction like 'The piece was also accompanied by a variety of comments spanning different views from members of the public / of the school community / of the local barber's association / w/e' is sufficient, and then you can go into detail with each one when it's relevant to the discussion in your body paragraphs. The linking phrases in the quote above are also excellent ones to use in that situation.

If you're really desperate, you can just talk about them in a paragraph at the end, but it'd be better to try and integrate them, if possible.

Because of Wag the Dog is in this year's Whose Reality text list, would it be safe to assume that prompts pertaining to memories, shared experiences etc. will not appear this year? For instance, last year (the first year Wag the Dog appeared on the text list), the prompt was ‘Misrepresenting reality can have serious consequences', which fits perfectly with Wag the Dog, because the film is all about manipulating people's reality for political gain and the consequences which arise from doing so.

Surely the prompt must relate well to all four texts for Whose Reality? Or does VCAA expect you to know more than one Whose Reality text? Because we did Death of a Salesman as well, but I don't want to have to prepare for both texts for Context...
As mentioned, your context prompt will relate to all four texts, but some will fit better than others. Inevitably some texts have a natural advantage some years, but since it's what you do with those ideas that matters, you shouldn't have to worry too much. So long as you've endeavored to connect the core of the prompt with the core of the text somehow, you'll be fine.

However, the texts that are in their first year tend to have a very obvious link, whereas the others might be more peripheral, meaning that one text for each context:
[IL] - A Passage to India
[WR] - Foe
[EC] - The Lietenant & A Separation (because the person in charge of conflict can't count)
[ID&B] - Wild Cat Falling
will likely have a really clear prompt link. To some extent you can look back over previous years and see this - the '14 WR prompt was clearly angled towards Wag The Dog whilst the '14 EC prompt suited Every Man in this Village really well. It's not the be-all-end-all, and most competent writers can make their ideas fit a whole variety of prompts anyway, as exemplified by the fact that you can go back as far as 2008 and still consider connections even though your texts weren't even considered at that point.

You should be safe enough concentrating on a single text for context, but it can't hurt to go back over notes or old essays for your other one just in case.

Does anyone feel like one hour is way too little to annotate one/two language analysis pieces AND write an 800-1000 word piece? Has anyone in the past tried an exam structure like:
Section A - 50 minutes
Section B - 50 minutes
Section C - 1 hour 20 minutes
If you can't get Language Analysis done in under an hour, then you're probably doing a lot of unnecessary work, or demonstrating the same skill over and over to no effect. I'd argue L.A. is the one piece you want to finish in less than an hour, and Sec.A and B are the ones where you can spend more time fleshing out ideas. L.A. is ludicrously formulaic, and you shouldn't have to annotate them too extensively. Are you just finding persuasive devices? Or do you link them to effects and to other parts of the article too? Either way, try to cut your planning/annotating time down, and then be as efficient as possible when you're writing. 1000+ words is kind of excessive for L.A. hitting around there is pretty much necessary for T.R. and Context if you're aiming high (~though depending on your style for Context.)

Remember, they're testing your ability to be selective, and the assessors want to see that you know how to pick out the most important parts of the article to analyse, not just discuss absolutely everything you come across, so prioritise the important bits and try not to get caught up in the trap of overanalysing and wasting time.

I'd also advocate doing Section C first if you can get the timing under control, since having to revisit it after spending almost two hours doing other things can mean an unnecessary extra read-through or time wasted reacquainting yourself with the material, whereas if you spend the majority of reading time getting through the article(s,) jot down a couple of things for Parts A/B when writing time begins, but then get straight into L.A. and aim to finish in under an hour, you'll be in a better position to get more credit in the other two sections where the breadth of your coverage is more important.


+ in relation to various queries about what the prompts will be about, obvs. we won't know for sure until 9:00 on Wednesday, but be careful when ruling things out completely, as VCAA can throw you various kinds of curveballs (for reference, see the entirety of the 2011 exam) and one of their favourite curveballs is giving you a familiar concept wrapped up in unfamiliar terminology. So whilst the key areas that have come up in previous years are unlikely/basically impossible options for this year, that doesn't mean VCAA can't alter the wording just to mess with you.

eg. 2012 IL Prompt: The ways in which we understand a landscape and respond to it can change over time.
2014 IL Prompt: Imagination shapes our response to the landscape.

2009 WR Prompt: We do not see things as they are. We see them as we are.
2011 WR Prompt: Shared experience does not mean that people see things the same way.

2010 EC Prompt: It is difficult to remain a bystander in any situation of conflict.
2012 EC Prompt: The experience of conflict changes people’s priorities.

2008 ID&B Prompt: Our relationships with others help us to define who we are.
2011 ID&B Prompt: Without connection to others there is no me.

I'm not saying these prompts are identical, and your responses should differ depending on the exact wording, but notice how there's a lot of overlap between the kinds of concepts and arguments you'd use in these sets of situations? It's entirely possible VCAA will give you an entirely new bit of terminology that they haven't used before (... in fact that's quite likely,) but the core of it is still going to fit under that Context umbrella of ideas, and it will still hopefully be fairly familiar to you.

If you're pressed for time, by all means consider what's likely to come up and what you can fairly safely rule out, but make sure you're prepared for a difficult exam just in case. If it ends up being dead easy, then all those complex things you've developed can make your pieces even better. And if it is difficult, then you'll have an edge over the majority of the state who've only prepared for the standard stuff :)
Title: Re: VCE English Question Thread
Post by: literally lauren on October 24, 2015, 05:24:27 pm
Re: timing, the ABC order is actually completely arbitrary. It's a remnant from the previous (-2008) Study Design when they kept Text Response largely unchanged, but altered the other two tasks - there's no reason why it's structured like that, and most teacher's don't require you to go through it page by page, just like a lot of people will tackle short answer/ extended response questions in Maths/Science subjects first, and then do multi-choice last just in case they run out of time and can shade some random bubbles in the last few minutes :P This method also isn't compulsory, but it makes sense from a test-taking perspective.

1) Spend 15 min reading time going over C and analyzing, make points in the head- Write for 1 hour. Do not look at Section A + B
           2 hours + 15 min left at this stage.
2) Spend 2-3 min on structure/points of your text response and write for 1 hour. Again Do not look at Section B
             1 hour + around 10 min left
3) Spend 2-3 min on structuring points/essay for Section B. Write for 1 hour
             Around 5 min left
4) Go back and pray u didnt fuck anything up

This definitely works well if you find yourself overwhelmed by having to mentally plan all three tasks at once. Some people might prefer to let the prompts churn over in their heads, but it depends how you approach things. A good halfway point would be to plan one essay ahead at all times, so

Reading Time: go through C and look at the prompts for A - maybe choose one, but no stress.
First Hour: write C and contemplate A prompts
Second Hour: look at the B prompt, and then choose/write A
Third Hour write B

^Just another possibility; working out your own method by practicing is definitely valuable though :)

The best thing about English is when it's over haha.
Don't taunt them, man. They've still got three days left :P
Title: Re: VCE English Question Thread
Post by: Mc47 on October 24, 2015, 05:27:10 pm
How important is the word count exactly? e.g. If I write a 600 word language analysis and get a 7/10, could that have been much higher if I wrote 800+ words? Or is there a certain point where the examiners stop rewarding quantity and instead focus mainly on quality?
Title: Re: VCE English Question Thread
Post by: duo0024 on October 24, 2015, 05:36:43 pm

Similar question:
Does anyone think there will be a prompt about change and personal growth for ID&B? My teacher fucked us over for our SAC and gave us 'Sometimes we need to accept change in order to grow' (VCAA 2013) when The Mind of a Thief wasn't even on the text list in 2013... (I think)

My other text is Skin and of course there are changes in MOAT and Skin but no one really grows because of it -- it ends up being destructive? Does anyone agree? (I really struggled with this prompt)

IM DOING I&B TOO! Also I see what you mean. I studied Mind of a Thief too and could not write an essay about change (it was extremely difficult) so I wrote about Summer of the Seventeenth Doll instead, I find that for mind of thief, the prompts that are suited for it are mainly about culture and place, whereas the Doll is more about change. SO, hopefully to cater to all of the texts VCAA will make the I&B prompt really broad like last years :)
Title: Re: VCE English Question Thread
Post by: 99.90 pls on October 24, 2015, 05:49:46 pm
I'd argue L.A. is the one piece you want to finish in less than an hour. You shouldn't have to annotate them too extensively. Are you just finding persuasive devices? Either way, try to cut your planning/annotating time down, and then be as efficient as possible when you're writing. 1000+ words is kind of excessive for L.A.

What's a good amount of time to be spending annotating? During annotation, I usually write down the contention/s and highlight language devices and elements of any graphics. I use three colours to link the devices in three 'categories' (arguments, techniques etc. depending on the text) and then I start writing. Annotating takes me 15 minutes usually, or 20 minutes for comparative. Then I write for about an hour, in which I'll produce ~700 words :\

Sec A and B are the ones where you can spend more time fleshing out ideas. Hitting around 1000 words is pretty much necessary for T.R. and Context if you're aiming high (~though depending on your style for Context.)

I hit 1000 words for Context in 50-55 minutes because I know my piece/s pretty much back to front, but I'd definitely struggle to hit 1000 for Text Response. For my trial exam, it took me about 60 minutes to write ~700 words for This Boy's Life and the assessor gave me 19/20. Do you think I should be writing more (if I'm aiming for a very high score) and do you think maybe the assessor was being a bit too lenient then?
Title: Re: VCE English Question Thread
Post by: Mc47 on October 24, 2015, 05:56:24 pm
IM DOING I&B TOO! Also I see what you mean. I studied Mind of a Thief too and could not write an essay about change (it was extremely difficult) so I wrote about Summer of the Seventeenth Doll instead, I find that for mind of thief, the prompts that are suited for it are mainly about culture and place, whereas the Doll is more about change. SO, hopefully to cater to all of the texts VCAA will make the I&B prompt really broad like last years :)

I figured there could be something about expectations (of society/family/friends etc) and how they can influence us (fits well with the Doll and Skin I think)

Not sure how it fits into the other texts though
Title: Re: VCE English Question Thread
Post by: imaware on October 24, 2015, 06:00:23 pm

I hit 1000 words for Context in 50-55 minutes because I know my piece/s pretty much back to front, but I'd definitely struggle to hit 1000 for Text Response. For my trial exam, it took me about 60 minutes to write ~700 words for This Boy's Life and the assessor gave me 19/20. Do you think I should be writing more (if I'm aiming for a very high score) and do you think maybe the assessor was being a bit too lenient then?

Damn I'm doing This Boy's life too what the hell did you write to get a 19/20 in 700 words haha. Thats crazy good.
I usually have to hit 900+ words to get over 18/20 but thats just me
Title: Re: VCE English Question Thread
Post by: Cristiano on October 24, 2015, 06:17:23 pm
What's a good amount of time to be spending annotating? During annotation, I usually write down the contention/s and highlight language devices and elements of any graphics. I use three colours to link the devices in three 'categories' (arguments, techniques etc. depending on the text) and then I start writing. Annotating takes me 15 minutes usually, or 20 minutes for comparative. Then I write for about an hour, in which I'll produce ~700 words :\

I hit 1000 words for Context in 50-55 minutes because I know my piece/s pretty much back to front, but I'd definitely struggle to hit 1000 for Text Response. For my trial exam, it took me about 60 minutes to write ~700 words for This Boy's Life and the assessor gave me 19/20. Do you think I should be writing more (if I'm aiming for a very high score) and do you think maybe the assessor was being a bit too lenient then?

When doing a timed Section C, i don't waste time highlighting the page and annotating and instead just do this in my head. If i have to i will underline the words and just write about it. In that 20 mins you were planning and highlighting you could have been writing, and the assessors don't mark you on your highlighting skills - they don't even get to see it as you take the task book home!

So in reading time i would suggest just going over techniques and persuasive methods in your head and thinking of what you are going to write, rather than wasting valuable time as you could be up to 800-900
Title: Re: VCE English Question Thread
Post by: 99.90 pls on October 24, 2015, 07:12:53 pm
When doing a timed Section C, i don't waste time highlighting the page and annotating and instead just do this in my head. If i have to i will underline the words and just write about it. In that 20 mins you were planning and highlighting you could have been writing, and the assessors don't mark you on your highlighting skills - they don't even get to see it as you take the task book home!

So in reading time i would suggest just going over techniques and persuasive methods in your head and thinking of what you are going to write, rather than wasting valuable time as you could be up to 800-900

Thanks for the advice! I'm going to try that now, analysing during reading time and just highlighting only (usually I write a bunch of stuff next to the highlighted text) for maybe 5 minutes. I'll report back later tonight haha

And just wondering, if I'm aiming for very high 40's, how much should I be writing for Section C? I'm currently writing 650-700 words, which I feel is very inadequate.
Title: Re: VCE English Question Thread
Post by: Cristiano on October 24, 2015, 07:17:23 pm
It really depends. Quality over quality. One with 700 words but is written very concisely should score higher than a 1000 word one but is very waffly. However, i do think that if you want to get 45+ then you should be hitting the 1000 word mark for each section, even 900 words would suffice if it has good quality.
Title: Re: VCE English Question Thread
Post by: 99.90 pls on October 24, 2015, 07:21:47 pm
It really depends. Quality over quality. One with 700 words but is written very concisely should score higher than a 1000 word one but is very waffly. However, i do think that if you want to get 45+ then you should be hitting the 1000 word mark for each section, even 900 words would suffice if it has good quality.

900-1000 words for Language Analysis! I just spoke to someone who got 48 and they said Language Analysis should be the shortest of the three? He said even the best students probably wouldn't hit 1000 for Language Analysis and that they give a lot less paper to write LA because you're not meant to write as much?

Can anyone confirm or deny these allegations?
Title: Re: VCE English Question Thread
Post by: Cristiano on October 24, 2015, 07:25:47 pm
Well maybe not 1000, though it should be the shortest - if you are concise. 800ish words would score a 10/10 if it was of really good quality. (Last year's Yergon piece, in examiners report scored, the student scored 10/10 and was roughly 800-850 words.)
 1000 words might score a 7 or 8 if it didnt meet the criteria (Didnt analyse images) or maybe it was restating the contention every sentence and some parts were irrelevant.
Again. Quality > Quantity.

Edit: The 10/10 piece was 871 words (calm down, it was a comparative piece lol)
Title: Re: VCE English Question Thread
Post by: 99.90 pls on October 24, 2015, 07:50:14 pm
Well maybe not 1000, though it should be the shortest - if you are concise. 800ish words would score a 10/10 if it was of really good quality. (Last year's Yergon piece, in examiners report scored, the student scored 10/10 and was roughly 800-850 words.)
 1000 words might score a 7 or 8 if it didnt meet the criteria (Didnt analyse images) or maybe it was restating the contention every sentence and some parts were irrelevant.
Again. Quality > Quantity.

Edit: The 10/10 piece was 871 words (calm down, it was a comparative piece lol)

Thanks! It's reassuring to know that 871 words can score a 10/10. Thank god I don't have to waffle to 1000. And you mentioned comparative, which brings me to another question - would standalone pieces require less words than a comparative? Just so I can get a good guideline for Language Analysis (e.g. 900 words for Comparative, 800 words for standalone)
Title: Re: VCE English Question Thread
Post by: Cristiano on October 24, 2015, 07:57:37 pm
I would think so, some of the stuff is just introducing the piece and spitting out formula stuff. 800 would be fine for one piece. Even less, 700-750 if you're really running out of time.
Title: Re: VCE English Question Thread
Post by: 99.90 pls on October 25, 2015, 11:38:38 am
When doing a timed Section C, i don't waste time highlighting the page and annotating and instead just do this in my head. If i have to i will underline the words and just write about it. In that 20 mins you were planning and highlighting you could have been writing, and the assessors don't mark you on your highlighting skills - they don't even get to see it as you take the task book home!

So in reading time i would suggest just going over techniques and persuasive methods in your head and thinking of what you are going to write, rather than wasting valuable time as you could be up to 800-900

Update: Massive vouch for this method! I annotated for about 5 minutes instead of 15-20 and my word counts in an hour went from 650 -> 700 -> 750 -> 800 in just three practices! It just takes some time/practice to get used to, but it's fucken amazing :)

A side note, I got my hands on the 2011 VCAA Language Analysis (PM if anyone wants, because of copyright and shit), but I hear that it's really really shite, is it worth doing for practice?
Title: Re: VCE English Question Thread
Post by: Lawyer on October 25, 2015, 11:56:54 am
Update: Massive vouch for this method! I annotated for about 5 minutes instead of 15-20 and my word counts in an hour went from 650 -> 700 -> 750 -> 800 in just three practices! It just takes some time/practice to get used to, but it's fucken amazing :)

A side note, I got my hands on the 2011 VCAA Language Analysis (PM if anyone wants, because of copyright and shit), but I hear that it's really really shite, is it worth doing for practice?

Its kinda like literature close-analysis aye? I spend the least time on LA as i can pick up techniques they author used just like in close-analysis.
Title: Re: VCE English Question Thread
Post by: xeon88 on October 25, 2015, 04:09:17 pm
Hey guys, what kind of prompt do you think will most likely appear on this year's exam for Henry IV? What areas or themes?  :)
Title: Re: VCE English Question Thread
Post by: frenchtom on October 25, 2015, 05:07:19 pm
Hey do you get penalised for referencing two texts you have studied this year in your context piece?
I know that we are supposed to have one text that you have to focus most of your attention on but can you briefly include an example of the other text in a paragraph?
Title: Re: VCE English Question Thread
Post by: Cristiano on October 25, 2015, 05:09:39 pm
Yes you can include both texts and you wouldn't get penalised :)!
Title: Re: VCE English Question Thread
Post by: drmockingbird on October 25, 2015, 05:17:33 pm
Its kinda like literature close-analysis aye? I spend the least time on LA as i can pick up techniques they author used just like in close-analysis.

Doing Lit makes language analysis a lot more intuitive! :) You learn to not have to bother to mark every single technique on the page since if you did that in lit you'd be royally fked.
Title: Re: VCE English Question Thread
Post by: Mc47 on October 25, 2015, 05:34:59 pm
Hey do you get penalised for referencing two texts you have studied this year in your context piece?
I know that we are supposed to have one text that you have to focus most of your attention on but can you briefly include an example of the other text in a paragraph?

My teacher strictly told us not to do it
Title: Re: VCE English Question Thread
Post by: schooliskool on October 25, 2015, 05:47:47 pm
My teacher strictly told us not to do it
On the ABC Radio English exam panel earlier today the English examiners said that it is fine as long as the one you have noted as your main text is emphasised a lot more.
Not sure why your teacher said so :o
Title: Re: VCE English Question Thread
Post by: 99.90 pls on October 25, 2015, 05:51:19 pm
On the ABC Radio English exam panel earlier today the English examiners said that it is fine as long as the one you have noted as your main text is emphasised a lot more.
Not sure why your teacher said so :o

Did not realise this existed! I'm gonna give it a listen now, a nice way to take a break from cramming hahaha
http://blogs.abc.net.au/victoria/2015/10/sunday-school-2015-english-exam-.html for anyone else.

BTW, has anyone done 2011 VCAA Lang Analysis? (the plagiarised one) I've got a copy but not sure if it's worth it to do
Title: Re: VCE English Question Thread
Post by: tmso on October 25, 2015, 06:16:55 pm
Hey guys.
Does anybody know what the average mark out of 10 is for the English exam? I'm hoping to at least get 35 (pre req. for my course) but am worried!! I mostly get 7-8 on practice pieces.
Title: Re: VCE English Question Thread
Post by: Mc47 on October 25, 2015, 06:19:40 pm
On the ABC Radio English exam panel earlier today the English examiners said that it is fine as long as the one you have noted as your main text is emphasised a lot more.
Not sure why your teacher said so :o

Wouldn't surprise me

The guy's incredibly lazy and has no idea how to teach. Not exactly a winning combination

Thanks for clearing that up though  :)

Hey guys.
Does anybody know what the average mark out of 10 is for the English exam? I'm hoping to at least get 35 (pre req. for my course) but am worried!! I mostly get 7-8 on practice pieces.

Average is about 5.5 per piece. 7-8 should comfortably get you a 35
Title: Re: VCE English Question Thread
Post by: pi on October 25, 2015, 06:39:49 pm
BTW, has anyone done 2011 VCAA Lang Analysis? (the plagiarised one) I've got a copy but not sure if it's worth it to do

Worth doing. Found it to be quite different to anything I had looked over, especially because it was hardly persuasive hahaha. Traumatising af.
Title: Re: VCE English Question Thread
Post by: 99.90 pls on October 25, 2015, 10:04:16 pm
Worth doing. Found it to be quite different to anything I had looked over, especially because it was hardly persuasive hahaha. Traumatising af.

If there are comments in a Section C, do you have to state the contentions of all of them in the introduction?

I just had a crack at VCAA 2011 (10 mins reading + 60 mins writing on pen and paper); would anyone mind having a look at it? (it's in the spoiler below) I found it surprisingly okay, but a question I have is if a Section C has comments, do you have to discuss all of the comments?

VCAA 2011 EXAM (SECTION C INCLUDED): http://imgur.com/a/xT1Yh

Spoiler
Helen Day’s blog entry, ‘The Power of Ink’ (Friday 25 March, 2011), published in her blog ‘Street Beat’, contends in a nostalgic and later, irritated tone that tattoos have lost their meaning over time and become empty cosmetic adornments. In response, four people commented on her blog between March 25 and March 26, each expressing vehement opinions with a variety of tones.

Her use of “mega shopping centre” suggests that tattoos are now associated with consumerism and conformity, which she later emphasises with “tattoo artists set up shop in every Australian shopping strip”. “Suburban housewives” implies that tattoos can be worn by those who stereotypically do not have to endure extended hardship and are able to enjoy the comfort and safety of their home, persuading the readership that tattoos have lost their former association with resilience and subjugation. Her listing of “newsreaders, sitcom stars and upmarket shoppers” creates a sense of ubiquity, as readers feel that people with affluent statuses and occupations are also able to purchase tattoos, diminishing their former exclusivity to lower castes. “Even [Day]” has a tattoo and she “[sees it] as an ornament as ordinary as any other cosmetic quirk”, suggesting that even the wearers have lost respect for their tattoos, further lowering their value. The “Ta Moko” photo contrasts with Day’s disregard, depicting the importance of the “Ta Moho” to “the wearer”, as it “shows… status” and “family heritage”. This contrast accentuates Day’s disdain for tattoos, showing the immense extent to which tattoos have declined.

Kiwi’s comment supports Day’s desire for tattoos to become exclusive; Day is “profoundly [annoyed]” by the tattoos because tattoos’ spread diminishes wearers’ uniqueness while similarly, Kiwi claims, in an abhorred and irate tone, that to imitate a unique Ta Moko is “disgraceful and immoral”. Kiwi’s use of the second person: “…stole these from you” invites the reader to empathise with his derision of those who imitate “Ta Moko”; the referencing of familiar notions like “identity card” and “fingerprints” assist the reader in imagining the outrageous scenario which Kiwi speaks of, since even non-Maori readers would be familiar with these concepts.

Evoking different epochs: “From the earliest times… World War II… 18th century… 20th century” creates a temporal distinction between the days in which tattoos represented “trouble”, “deviance”, “horror”, “defiance” and “satire” and the present, in which “[t]he tattoo has been commodified”, supporting her overarching contention that “the power of ink has diminished” over time, “like, well, a tattoo.”

Day complains that tattoos once represented a daring willingness to revolt against gender stereotypes of “femininity”, but now they serve the contrary. For Day, “tattoos have become sexy”, suggesting that “young women” who wear them now are sexually objectifying themselves and thus, submitting to typical gender roles. As a result, the reader feels that tattoos have lost so much of their original meaning that they have begun to convey meanings opposite to their traditional purposes. Cleanskin plays upon this notion of subversion with his observation that “[so] many dolphin tattoos have faded and stretched into whales”, suggesting that tattoos once connoting grace and agility now connote the opposite – ugliness and torpidity. His proud proclamation that “[he is] a rebel and an individual” yet he has “no tattoos” shows that he believes that there are better ways to express individuality, offering an alternative solution to Day’s lamentations.

Because Day does express her desire for tattoos to regain their former meaning (“Personally, I find this profoundly annoying”), it is in her interests to dissuade readers from getting tattoos. The image of Sam de Brito’s quote and the tattoo on the single shoulder assist Day in achieving this, as the ellipse in “What I’ll tell my children…” connotes regret. This is exacerbated by the unsightly tattoo on his shoulder, which appears to be permanent. “before you’re thirty” suggests that those with sufficient life experience and maturity will not get a tattoo, since it does not make a difference at what age one gets a tattoo as they are lifelong and permanent. Dr AB’s comment further supports Day’s dissuasion, as he evokes trepidation in readers by listing undesirable diseases: “bleeding, hepatitis B, hepatitis C, tetanus”, as well as pain: “significant pain, and a pus-like drainage”; the reader is, as a result, instinctively repelled by tattoos because of the risk they pose.

Day contrasts the formerly “deviant” and “[defiant]” nature of tattoos with the conformist connotations of “fashion’s proprietary mark”, elucidating just how far tattoos have strayed from their meaning “[back] then”. Tash, however, argues that a tattoo can still “show off” “[uniqueness]”, as long as one invests creativity and effort into their tattoo. Her personal anecdote: “I didn’t just go to one of those sleazy places… I designed my own… [no] one else has one like mind” suggests to readers that they too can attain her “[uniqueness]”, especially since she uses very colloquial vernacular such as “nerd” and “talk” to create a casual mood. As a result, she encourages readers to feel that “unique” tattoos are accessible to everyone, regardless of their socioeconomic or educational circumstances.
Title: Re: VCE English Question Thread
Post by: dankfrank420 on October 26, 2015, 11:27:55 am
For language analysis, if there are two pieces (ie. a blog and a comment) then do we mention both in the introduction?

For context, what are your guys experiences with just memorising a piece and bringing it in?
Title: Re: VCE English Question Thread
Post by: Alter on October 26, 2015, 11:37:14 am
For language analysis, if there are two pieces (ie. a blog and a comment) then do we mention both in the introduction?
In my experience, yes. Obviously if there are 3 comments you wouldn't have to go one by one with the author, but it's nice to give them a mention. As a general rule, if I'm going to spend more than a sentence or two analysing it, it will get signposted in the introduction.

Quote
For context, what are your guys experiences with just memorising a piece and bringing it in?
Never done it, I use expository. Adaptable pieces are probably better though, or else a bad prompt will completely destroy your section B score.
Title: Re: VCE English Question Thread
Post by: literally lauren on October 26, 2015, 01:57:23 pm
Hey do you get penalised for referencing two texts you have studied this year in your context piece?
I know that we are supposed to have one text that you have to focus most of your attention on but can you briefly include an example of the other text in a paragraph?
Just to clarify here, you are allowed to mention both so long as you still have a 'primary text drawn upon.' However, since you're being marked on the quality of exploration, only using the set texts (or having one in each B.P. + one external example) might limit you somewhat. If none of your other evidence fits, then by all means use your set texts, but it'd be better if you were able to draw from different sources ideally.

If there are comments in a Section C, do you have to state the contentions of all of them in the introduction?

I just had a crack at VCAA 2011 (10 mins reading + 60 mins writing on pen and paper); would anyone mind having a look at it? (it's in the spoiler below) I found it surprisingly okay, but a question I have is if a Section C has comments, do you have to discuss all of the comments?
Re: the 2011 exam, it is worth looking over just to get a feel for how badly things could get in the exam, but I highly doubt VCAA would go down that route again.

There's no actual requirement to mention all of the comments, and so long as you are analysing both written and visual language, you're technically fulfilling the task instructions. However, if they're tossing up two pieces; one of which analyses the main article, one visual, and two comments really well, and the other which analyses everything really well without compromising quality, then it might be a 9-10 split. Much like the word count issue, quality>quantity, but my blanket recommendation is to say at least one thing about every text that you're given.

Plus, in the event you get stuck with that many comments, most of them will be quite one-dimensional (see: 'Tash' and 'Cleanskin's worthwhile contributions -.-)

For language analysis, if there are two pieces (ie. a blog and a comment) then do we mention both in the introduction?

For context, what are your guys experiences with just memorising a piece and bringing it in?
For L.A. if you get something like the 2011 exam, just say 'Day's piece was also accompanied by a variety of comments spanning different views from members of the public.'
If you get something like the 2014 exam, then I'd say it'd be worthwhile outlining Laikis' contention at the start. Otherwise, the first time you mention it in your body paragraphs, you'd have to spend a few sentences doing it, so best to get it out of the way early.

For Context, don't memorise a single piece; familiarise yourself with a range of ideas and examples, then apply the ones that suit the prompt on the day. The likelihood of you memorising a piece that fits perfectly is next to nothing, but if you prepare for the major concepts within your context, then having a malleable piece or two can work well.
Title: Re: VCE English Question Thread
Post by: coconut stripes on October 26, 2015, 02:34:48 pm
For context, are you required to quote your texts?
Title: Re: VCE English Question Thread
Post by: literally lauren on October 26, 2015, 02:39:16 pm
For context, are you required to quote your texts?

There's no requirement to do so. It might add to your piece, but it's not a necessity, so you can do so at your discretion. So long as there's an overt enough link for the assessors to see, you'll be fine.
Title: Re: VCE English Question Thread
Post by: coconut stripes on October 26, 2015, 02:45:08 pm
So you can just describe characters/situations whilst relating it back to your point?
(At my school, they ALWAYS tell us to quote, so I'm not sure how to go about linking the text otherwise, just in case I can't think of a quote that ties in on the day.)
Title: Re: VCE English Question Thread
Post by: literally lauren on October 26, 2015, 02:51:37 pm
So you can just describe characters/situations whilst relating it back to your point?
(At my school, they ALWAYS tell us to quote, so I'm not sure how to go about linking the text otherwise, just in case I can't think of a quote that ties in on the day.)
Relating characters/situations to the prompt = fine
Using quotes = fine
Doing both = fine
Doing neither = NOT FINE! :P

I suppose your school highly recommends quotes because it's one of the most direct ways to demonstrate textual knowledge, but you're right in that if the quotes you know don't fit the prompt, you shouldn't stick them in anyway.

The formal requirement is that you use the core of the text as a 'springboard', meaning you ask 'what does my set text have to say about this prompt,' and the incorporate the answer to that question in your piece. After that, you can move beyond it and explore your own ideas, but the underlying text link needs to be established (sooner rather than later, imo, so if you're writing an expository piece, try and have it in your 1st or 2nd paragraph.)

It's still possible to score well otherwise, but it's more risky, and you might get assessors who are too dumb to see a subtle link, or smart enough to see through a feeble, irrelevant connection, so aim for a tangible, worthwhile connection to be on the safe side.
Title: Re: VCE English Question Thread
Post by: Deshouka on October 26, 2015, 02:57:31 pm
Hi,
For language analysis would you lose marks if you made absolute statements? e.g. "instills fear within readers", "compels readers to..."
I've read many high-scoring essays and they seem to make such statements.
However, during my trial exam the examiner cautioned against this, advising me to say  "may instill fear" and "allay possible fears".
I feel that using 'may' and 'might' weakens my language analysis. So I'm quite confused. :S
Title: Re: VCE English Question Thread
Post by: literally lauren on October 26, 2015, 03:15:14 pm
Hi,
For language analysis would you lose marks if you made absolute statements? e.g. "instills fear within readers", "compels readers to..."
I've read many high-scoring essays and they seem to make such statements.
However, during my trial exam the examiner cautioned against this, advising me to say  "may instill fear" and "allay possible fears".
I feel that using 'may' and 'might' weakens my language analysis. So I'm quite confused. :S
Yeah, I've heard all sort of mutually exclusive recommendations about this. The best way to think of it is on a spectrum of:

0% - a blank page :p
25% - having no assessment of what the author's doing; just summary, basically
50% - sentences like 'perhaps this makes some readers feel ___ maybe'
75% - golden mean!
100% - super-definitive statements like 'all readers are made to feel completely ___'

so you're aiming for that sweet spot between 'seeming really unsure of yourself' and 'being way too confident that you're right.' The sentences you've mentioned ('this instills fear' and 'compels readers to') are both alright in my book, but the safest way of tackling this is to alter the sentence so that you're focusing on the authorial intent, or the language itself.

eg.
'Readers are made to feel afraid'
--> This is designed to elicit fear in readers.
'Thus, readers are likely to feel optimistic.'
--> Thus, the author intends to elicit a sense of optimism.
'Feelings of respect are created through the invocation of 'Professor Smarty-Pants''
--> The author's invocation of 'Professor Smarty-Pants' is included as a means of engendering respect'
etc.

That said, using the occasional 'perhaps' or 'possible' isn't going to disadvantage you. When you're sure of the analysis, don't bother, but if you're taking things a step further and aren't totally convinced you've got the right idea, those qualifiers can be good to balance things out.

Like most things, vary the vocab and the assessors won't mind. So long as your understanding of the effects is correct, and you're not going out on wild limbs by writing things like 'This encourages readers to pursue a career in journalism so that they too can write an article this awesome...' :P
Title: Re: VCE English Question Thread
Post by: 99.90 pls on October 26, 2015, 03:45:26 pm
What's a good way to conclude a Language Analysis? The general trend seems to be just to summarise the contention, tone and the nature of the techniques employed, but I feel as if that's just rehashing the intro and adding no value? Is it acceptable to finish with analysis or is waffling for a couple of lines at the end a necessary evil?
Title: Re: VCE English Question Thread
Post by: elysian on October 26, 2015, 03:48:24 pm
I was thinking of doing an interview with Sandra Laing from Skin, how would you do so without retelling the story but actually addressing the prompt and its big ideas?
Title: Re: VCE English Question Thread
Post by: literally lauren on October 26, 2015, 03:53:42 pm
What's a good way to conclude a Language Analysis? The general trend seems to be just to summarise the contention, tone and the nature of the techniques employed, but I feel as if that's just rehashing the intro and adding no value? Is it acceptable to finish with analysis or is waffling for a couple of lines at the end a necessary evil?
My conclusions were usually just two sentences. A good format is to begin with saying 'By >using some big overall technique, like a dichotomy or creating an implication< the author seeks to engender >&describe effect here.<' Then just revisit the contention, and you're done. It's really just a structural requirement, so there isn't as much weight put on it as there is in T.R. and Context where you're expected to reach a more satisfying conclusion. I remember being told 'just don't screw things up so badly in the concl. by writing something ungrammatical or repetitious that they'd be inclined to take a mark off.' Beyond that, you're usually fine to write whatever general summative statement seems appropriate.
Title: Re: VCE English Question Thread
Post by: cosine on October 26, 2015, 04:18:37 pm
Got a few qns:

1. Should you include character names in topic sentences? (TR)
2. For context, is it wise to include the text studied into the introduction? (if so, what should be stated about it?)
3. For context, can you weave in both the texts you studied as evidence to support?

Title: Re: VCE English Question Thread
Post by: Alter on October 26, 2015, 04:24:46 pm
1
I typically try to avoid using character names in TR questions unless the question relates very specifically to one character (this could happen in Mabo, for instance). Generally, it's not necessary and makes it seem like you're basing paragraphs on characters rather than ideas.

2/3
At the top of your essay there will be a box to fill in that asks which text you are using as your main reference. For regular expository essays, you should try not to put the title of your text in the introduction, nor any wider world examples you intend to use.

You are definitely allowed to use both texts (e.g. you could use both Every Man and The Lieutenant. However, if one does not work with the prompt, don't feel obliged to use it. You won't be getting any extra marks just because you want to use both. You're probably better off having examples from the real world that fit in well with the given prompt or your other text.

edit: my post got dongled by formatting
Title: Re: VCE English Question Thread
Post by: literally lauren on October 26, 2015, 04:29:23 pm
Got a few qns:

1. Should you include character names in topic sentences? (TR)
2. For context, is it wise to include the text studied into the introduction? (if so, what should be stated about it?)
3. For context, can you weave in both the texts you studied as evidence to support?
1. Try not to. If you get a character-based prompt, then it's fine, but ideally you want to be giving your assessor a general overview of your argument, rather than zooming in on your evidence. Easiest fix for this:
eg. for a prompt like Medea reveals the dangers of being ruled by passion.
and you want to write a paragraph about how the character of Medea lets her emotions get the better of her, then rather than saying: 'Through the character of Medea, Euripides shows that extreme passions that aren't tempered by logic can lead to serious consequences for others.' You can just write: 'Through the character of Medea, Euripides shows that extreme passions that aren't tempered by logic can lead to serious consequences for others.' -and that's your focus. Then, two or three sentences in, you can isolate the specific characters you want to discuss, and each paragraph should have more than one character as its focus anyway.

2. Not necessarily; for a conventional expository essay, the text should be in your first body paragraph, but there aren't formal rules in context, so you can use it in your intro if there's a purpose behind your link (or you're employing it as your hybrid-y 'hook.')

3. Yes, you can use both texts, but you need a 'primary text drawn upon,' and since you get marks for the exploration you're doing, it's usually best to refer more on your external evidence where possible.
Title: Re: VCE English Question Thread
Post by: literally lauren on October 26, 2015, 04:33:34 pm
Also, you guys:
http://blogs.abc.net.au/victoria/2015/10/sunday-school-2015-english-exam-.html

About 19:48 onwards when they're talking about Language Analysis:

"...[students]'ve got to look at the material as give, and the way it connects. Because it won't be coherent. It'll be in bits." {Other teacher: "Yes."} "So they've got to find a way to actually connect this; it's all on one issue - one concern, one controversy - and they've got to find a way through that."

This could just be a way of saying techniques will be scattered throughout the piece, but I'd take this as a very strong indication that the exam will be comparative.

That is all.


edit: lol@21:27 "I think [Context] is the most beautiful section on the exam"  ::)  ::)
Title: Re: VCE English Question Thread
Post by: Cristiano on October 26, 2015, 04:38:03 pm
Also, you guys:
http://blogs.abc.net.au/victoria/2015/10/sunday-school-2015-english-exam-.html

About 19:48 onwards when they're talking about Language Analysis:

"...[students]'ve got to look at the material as give, and the way it connects. Because it won't be coherent. It'll be in bits." {Other teacher: "Yes."} "So they've got to find a way to actually connect this; it's all on one issue - one concern, one controversy - and they've got to find a way through that."

This could just be a way of saying techniques will be scattered throughout the piece, but I'd take this as a very strong indication that the exam will be comparative.

That is all.


edit: lol@21:27 "I think [Context] is the most beautiful section on the exam"  ::)  ::)

Wait, are those guys actual VCAA examiners - who have seen the 2015 exam? I thought they were just normal English teachers
Title: Re: VCE English Question Thread
Post by: literally lauren on October 26, 2015, 04:39:50 pm
Wait, are those guys actual VCAA examiners - who have seen the 2015 exam? I thought they were just normal English teachers
Most of the prompts at the end of the year are written by various teachers, so some of them might have written sections. Also, gossip tends to get around, so they may have heard rumours. Still, it's nothing definitive - just another indicator that you should all be prepared for a comparative task just in case  ;)
Title: Re: VCE English Question Thread
Post by: GeniDoi on October 26, 2015, 04:42:54 pm
I'm writing a hybrid-ish piece for context (whose reality) and I want to quote some parts a book that isn't in the same genre as The Lot, but lends to similar issues and ideas. Is it okay if the quote is several lines long?
Title: Re: VCE English Question Thread
Post by: literally lauren on October 26, 2015, 04:46:22 pm
I'm writing a hybrid-ish piece for context (whose reality) and I want to quote some parts a book that isn't in the same genre as The Lot, but lends to similar issues and ideas. Is it okay if the quote is several lines long?

Define "several lines" :P ie. what's your handwriting like?
I can't imagine the assessors having a problem with it - a lot of pieces open with a quote from someone else, be it the author of their text, or just some related general concept - just make sure you're incorporating it in a valid way, and not sticking it there in place of your own discussion. Letting the examples do the work for you is a big no-no; you need to be able to explain it's relevance to the assessor as they won't make these connections on your behalf.

Maybe post the quote here, and I can give you some indication of how you could modify it so that it doesn't have to be too lengthy?
Title: Re: VCE English Question Thread
Post by: cosine on October 26, 2015, 06:17:55 pm
Can someone please give me a rundown on what to include in the intro of context? I am doing expository and identity and belonging
Title: Re: VCE English Question Thread
Post by: 99.90 pls on October 26, 2015, 06:26:57 pm
For language analysis, if a device you want to point out seems kind of obscure (e.g. citing "eight per cent" instead of "8%" hides the statistic since it blends in with the other words), should we scrap it? Or should we justify it by saying why we think it's significant, in brackets? For instance, "(figures are usually used in conjunction with percentages)"
Title: Re: VCE English Question Thread
Post by: Mc47 on October 26, 2015, 06:28:14 pm
Can someone please give me a rundown on what to include in the intro of context? I am doing expository and identity and belonging

Whatever you want really

I don't think it's as strict as in language analysis

I just note my main points, expand on them a little and then tie it all into the prompt
Title: Re: VCE English Question Thread
Post by: pi on October 26, 2015, 06:33:28 pm
For language analysis, if a device you want to point out seems kind of obscure (e.g. citing "eight per cent" instead of "8%" hides the statistic since it blends in with the other words), should we scrap it? Or should we justify it by saying why we think it's significant, in brackets? For instance, "(figures are usually used in conjunction with percentages)"

I think that's fine, I wouldn't justify it in brackets but just justify it as part of your analysis. Try to avoid brackets anywhere if you can (I have a bad habit of it too haha).
Title: Re: VCE English Question Thread
Post by: Adequace on October 26, 2015, 07:56:29 pm
Is there any benefit of bringing a dictionary in to any English SAC/exam, I have a pretty good dictionary but it's also a thesaurus so it won't be allowed. I'm just wondering if I should buy a new dictionary for VCE English...I'd rather not tbf  :P
Title: Re: VCE English Question Thread
Post by: literally lauren on October 26, 2015, 08:00:09 pm
Is there any benefit of bringing a dictionary in to any English SAC/exam, I have a pretty good dictionary but it's also a thesaurus so it won't be allowed. I'm just wondering if I should buy a new dictionary for VCE English...I'd rather not tbf  :P

YES 100% BRING A DICTIONARY!

In all likelihood, you won't need it, but imagine if you're staring at the Text Response prompts and there's a word you don't understand - that instantly stops you from being able to write on that prompt effectively. Worse still, what if it's in the Context prompt - then you have no choice.

Even for words that you 'know' but don't quite 'get' - a formal dictionary definition can be invaluable for making things super clear and allowing you to write your piece safely.

If you don't have a non-thesaurus version, maybe borrow one off a nearby friend or neighbour? Your school might have some too, but these tend to disappear on the morning of the English exam fairly quickly, so you probably won't be able to rely on them being there.
Title: Re: VCE English Question Thread
Post by: imaware on October 26, 2015, 08:04:14 pm
In regards to text response, how much "construction"or metalanguage do we need to employ for the 9-10 marks.
And in regards to in depth analysis of particular quotes and scenes, how much in detail are we meant to go?

Thanks
Title: Re: VCE English Question Thread
Post by: pi on October 26, 2015, 08:04:28 pm
Dictionaries can also be bought second hand from all those charity stores (St Vinnies, Salvos, etc.) for about $2. Although hopefully y'all have one by now :P
Title: Re: VCE English Question Thread
Post by: cameotodd on October 26, 2015, 08:05:46 pm
There's like a mini section in my dictionary in the middle which goes over grammar and stuff... would this be allowed? I've been using it the whole year for SACs and stuff but not sure about the exam.
Title: Re: VCE English Question Thread
Post by: pi on October 26, 2015, 09:11:51 pm
There's like a mini section in my dictionary in the middle which goes over grammar and stuff... would this be allowed? I've been using it the whole year for SACs and stuff but not sure about the exam.

Best to get something else if you can, although I doubt anyone well check or care.
Title: Re: VCE English Question Thread
Post by: Mc47 on October 26, 2015, 09:12:46 pm
There's like a mini section in my dictionary in the middle which goes over grammar and stuff... would this be allowed? I've been using it the whole year for SACs and stuff but not sure about the exam.

Just rip out those pages if you're unsure

Title: Re: VCE English Question Thread
Post by: djsbrks on October 26, 2015, 09:18:12 pm
How many outside sources at the least do we need for context? I've got 2 but my third one is a bit hazy still..
Title: Re: VCE English Question Thread
Post by: duo0024 on October 26, 2015, 09:39:44 pm
Is there some sort of trick to identify the 'key players' in L.A faster? I tend to spend along time during reading time trying to identify them and make links between the article, comments and visuals.
THANKS!
Title: Re: VCE English Question Thread
Post by: literally lauren on October 26, 2015, 11:10:49 pm
Can someone please give me a rundown on what to include in the intro of context? I am doing expository and identity and belonging

No definitive rules beyond 'make it interesting, and make it relevant.'
If it's just a regular expository introduction, then try to unpack the prompt and outline your ideas (in that order.) Alternatively, you could use an example /  the text / a quote / something to create a 'hook' to make it engaging, and there are no limits on how you could choose to begin. Probably best to just work out what you've been doing so far this year, and just refine that for the exam.

For language analysis, if a device you want to point out seems kind of obscure (e.g. citing "eight per cent" instead of "8%" hides the statistic since it blends in with the other words), should we scrap it? Or should we justify it by saying why we think it's significant, in brackets? For instance, "(figures are usually used in conjunction with percentages)"
Ideally you'll have enough language to talk about that you won't need to mention everything you come across, so if there are some points that are kind of dodgy or not as efficacious as others, definitely showcase your ability to be selective by leaving them out. Statistics tend to be kind of pedestrian techniques to point out anyways ('Author uses numbers because numbers are persuasion' tends to be the gist of most analyses) so unless you think it's especially pertinent, it'd be better to focus on the more important stuff.

In regards to text response, how much "construction"or metalanguage do we need to employ for the 9-10 marks.
And in regards to in depth analysis of particular quotes and scenes, how much in detail are we meant to go?
1) depends on the prompt, and the text come to think of it, but I'd say try to have a minimum of one structural thing in each paragraph ~that's really arbitrary though; you could easily flout that 'rule' and still get a perfect score. In terms of metalanguage, almost all your sentences should contain metalanguage because you'll be discussing and analysing the text.
2) You can assume that your assessor has read the text, but you should go into enough depth such that a clever five year old could follow your logic. When it comes to summative details (ie. This event happens following this character's revelation that this was a lie all along) you can be very brief, but in terms of interpretation, try to back yourself up wherever possible.

How many outside sources at the least do we need for context? I've got 2 but my third one is a bit hazy still..
I'd aim to have more than you need, so perhaps read up on some other ones tomorrow and give yourself four in total, just in case. If the three that you've got are incredibly flexible, then you might be okay, but you'd want to be as prepared as you can be.
As a preparation exercise, read through the prompts here and here. If you think your examples are sufficient for you to cover the majority of those prompts, then you're ready. If there are still a few key areas in your Context that you can't cover though, maybe do a bit of reading and research tomorrow just in case.

Is there some sort of trick to identify the 'key players' in L.A faster? I tend to spend along time during reading time trying to identify them and make links between the article, comments and visuals.
THANKS!
1. What is the contention? eg. space exploration = good
2. How does the author argue that^ to be true? eg. there are benefits for humanity; chasing dreams is good; exploration is cooperative and productive
3. You now have your three key players: 'the benefits of space exploration;' 'the advantage of pursuing dreams;' & 'the cooperative and productive nature of exploration'

If you end up with more than you need, try to group them so that you don't end up spending too much time on L.A. but ensuring that you cover the article in sufficient detail,
eg. 'financial gains to be made from exploring space' and 'global interest in attaining knowledge and other benefits (see: 2nd visual)' could easily be grouped into a big 'benefits for humanity' paragraph.

Likewise, if you end up only having two key players, try to split the bigger or more dominant one into two smaller arguments
eg. if you've only got 'benefits to humanity' and 'chasing dreams is good'
then perhaps turn 'benefits' into --> 'tangible improvements to technology and livelihood' + 'spiritual fulfillment at having pursued and accomplished something'

The other thing to keep in mind is that the assessors are never looking for a single, definitive breakdown. It may put your mind at ease to know that so long as your method is working for you, the assessors will approve. It's not as though they look at your essay and say 'that's not how I would've done it, so -4 marks instantly.' It's all about what you think are the most important parts of the article, and there's a lot of flexibility to move around within your body paragraphs, so don't stress. The fact that you're showing some awareness of the key players will already make you stand out amongst all the standard chronological/by-technique methods people tend to apply blindly.
Title: Re: VCE English Question Thread
Post by: Rainbow19 on October 26, 2015, 11:17:04 pm
Hi Lauren!
So with a comparative language analysis, which format is best??

1. Analyse main text
2. Linking sentence (ie: on the contrary ___ contends___ through.....)
3. Second text?

or

1. Analyse main text
2. Analyse second text
3. Comparative Paragraph

can you do it like this?
1. Analyse main text
2. Analyse second text weaving in comparisons/differences?
Title: Re: VCE English Question Thread
Post by: literally lauren on October 26, 2015, 11:22:50 pm
Hi Lauren!
So with a comparative language analysis, which format is best??

1. Analyse main text
2. Linking sentence (ie: on the contrary ___ contends___ through.....)
3. Second text?    <-- this is best!!

or

1. Analyse main text
2. Analyse second text
3. Comparative Paragraph  <-- if you were doing it this way, this paragraph would be unnecessary. You don't get any marks for comparison, so it's better if you can weave in the secondary texts together with your discussion of the main text, using a related idea as a pivot

can you do it like this?
1. Analyse main text
2. Analyse second text weaving in comparisons/differences? <-- yes, if you have to, but the first method would be preferable.

In other words:
Each paragraph would have:
  - 75% worth of analysis of the main piece
  - 25% of visual and/or comment analysis
very approximately, of course, the assessors aren't too fussed about the breakdown.

Having said that, if you're more familiar with a different method, then I'd recommend just refining that for the exam.
Title: Re: VCE English Question Thread
Post by: schooliskool on October 27, 2015, 08:48:53 am
In other words:
Each paragraph would have:
  - 75% worth of analysis of the main piece
  - 25% of visual and/or comment analysis
very approximately, of course, the assessors aren't too fussed about the breakdown.

Having said that, if you're more familiar with a different method, then I'd recommend just refining that for the exam.
Is it fine to have 2-3 paragraphs on the main article and then a separate for the sub-article? Or will that not allow for much comparative analysis?
Title: Re: VCE English Question Thread
Post by: cosine on October 27, 2015, 08:51:13 am
I need to develop some ideas; 'You women are all the same' To what extent are Jason's judgments validated?

So far I have:

- Euripides epitomises Medea's own personal beliefs encircling vengeance and selfishness
- On the contrary, the Nurse and Chorus' (also women) views and values differ from that of Medea's.
- ??

Any ideas/thoughts?
Title: Re: VCE English Question Thread
Post by: 99.90 pls on October 27, 2015, 08:57:11 am
Just a small grammatical question, would the film Wag the Dog be underlined or wrapped in quotation marks? Same goes for websites in Language Analysis (e.g. this opinion piece was published on the website Activism Australia)
Title: Re: VCE English Question Thread
Post by: appleandbee on October 27, 2015, 09:00:09 am
Just a small grammatical question, would the film Wag the Dog be underlined or wrapped in quotation marks? Same goes for websites in Language Analysis (e.g. this opinion piece was published on the website Activism Australia)

quotation marks

Also this is probably a stupid question, but are we allowed to use dictionaries during reading time?
Title: Re: VCE English Question Thread
Post by: 99.90 pls on October 27, 2015, 09:04:51 am
quotation marks

Also this is probably a stupid question, but are we allowed to use dictionaries during reading time?

For both? I've been underlining films my whole life and a website can be likened to an anthology of sorts, so wouldn't that be underlined too? Sorry for doubting, just very confused
Title: Re: VCE English Question Thread
Post by: appleandbee on October 27, 2015, 09:08:40 am
For both? I've been underlining films my whole life and a website can be likened to an anthology of sorts, so wouldn't that be underlined too? Sorry for doubting, just very confused

I'm not too sure about websites, it's just that I'm doing a film for text response and I use quotation marks .
Title: Re: VCE English Question Thread
Post by: Cristiano on October 27, 2015, 09:09:18 am
I don't think it makes a difference if you underline or use quotation marks
Title: Re: VCE English Question Thread
Post by: heids on October 27, 2015, 09:13:38 am
Don't believe it makes a difference, as Cristiano says; underlining is possibly better imo (since you'll be using quotation marks to quote parts of the text), but it won't make a millimetre of difference in your marks.

Yep, can use dictionaries in reading time.
Title: Re: VCE English Question Thread
Post by: literally lauren on October 27, 2015, 09:23:11 am
Is it fine to have 2-3 paragraphs on the main article and then a separate for the sub-article? Or will that not allow for much comparative analysis?
Yes, that's fine if you can't find a way to incorporate ideas from the comment(s) in to your main body paragraphs, but you really want to try and bring it up when it's relevant, rather than just stick it at the end.

eg. Para1: 'Yergon says dreams are good...' >analyse< 'Contrarily, Laikis suggests that dreams are a waste of time...' >analyse<
^that's all the comparison you need; a single sentence to transition from one to the other, making it clear that you are contrasting one author's portrayal of something with the other author's.

If those connections aren't possible for you, or you frequently run out of time with L.A. perhaps leaving it till the end would be best, but make sure you analyse it properly. Much like the visuals, not spending enough time on these elements can impact the mark a little, even if your discussion of the main piece is quite good.

Just a small grammatical question, would the film Wag the Dog be underlined or wrapped in quotation marks? Same goes for websites in Language Analysis (e.g. this opinion piece was published on the website Activism Australia)
Convention is to have the text in single quote marks, and things the characters say are in doubt quotes, so in 'Wag the Dog' the Connie says "A good plan today is better than a perfect plan tomorrow." But so long as you're consistent with whatever you're doing, the assessors won't mind.
A website would be considered a title in this case, so single quotes would be preferred, but again, it doesn't really matter. You wouldn't really even need to mention the specific website; it might be given as part of the background info, but that's unlikely to help your analysis much, especially if it's just a general audience, so you can leave out parts of the biographical details that don't seem all that relevant.
Title: Re: VCE English Question Thread
Post by: elysian on October 27, 2015, 10:08:39 am
In other words:
Each paragraph would have:
  - 75% worth of analysis of the main piece
  - 25% of visual and/or comment analysis
very approximately, of course, the assessors aren't too fussed about the breakdown.


Do you have an example of that? Would you need to compare the two or just analyse the visual as a single analysis?
Title: Re: VCE English Question Thread
Post by: GeniDoi on October 27, 2015, 10:09:31 am
Is the second topic of The Complete Maus always about the graphic novel and it's relationship to exploring the narrative?

I noticed Free Original Practice Exams - 2015 does for all four practice exams, as does the 2014 one from VCAA.

I seem to score better on these, and find it easier because you can use a lot of visual evidence, which I find easier to recall than quotes. It would be comforting to know that it will be an option on the exam.

EDIT: Also, an interesting but somewhat annoying thing about Maus quotes is that a lot of them are intentionally grammatically incorrect, which makes for an interesting thing to analyse but at the same time makes remembering quotes word for word a difficult task. If I provide a quote but accidentally correct its grammar, will I lose marks for "misquoting" the text?
Title: Re: VCE English Question Thread
Post by: imaware on October 27, 2015, 10:16:12 am
Can someone please explain how to effectively do tonal analysis in the bodyparagraphs?
how much depth do we need to go into?

Thanks
Title: Re: VCE English Question Thread
Post by: schooliskool on October 27, 2015, 10:23:13 am
What is a good method to conclude a comparative analysis?
Title: Re: VCE English Question Thread
Post by: dankfrank420 on October 27, 2015, 11:05:29 am
For context expository essays:

My plan is to do

Intro
text
external
external
conclusion

When I write my external paragraphs, should I link back to my main text?
Title: Re: VCE English Question Thread
Post by: Botopie on October 27, 2015, 01:02:07 pm
are 300-word paragraphs too long for context?
Title: Re: VCE English Question Thread
Post by: literally lauren on October 27, 2015, 01:07:05 pm
Do you have an example of that? Would you need to compare the two or just analyse the visual as a single analysis?
In the event you get something similar to last year's exam, you'd just have to focus on the main piece, integrate some visual analysis where relevant, and compare the main contention to that of the comment once or twice in order to analyse the comment.

Cope of an answer from a few pages back in this thread:
If you're getting one main article, two visuals, and a comment:
Paragraph 1: 3/4: main article + 1/4 visual (1)
Paragraph 2: 3/4: main article + 1/4 comment
Paragraph 3: 3/4: main article + 1/4 visual (2) + (comment - if not sufficiently covered in other para.)
---------------------------------
So each paragraph contains at least two 'texts' (ie. the main piece and one other; either visual or written) but no paragraph is burdened with having to deal with every single text all at once. This'll let you draw sufficient connections across the material and cover the whole scope of the thing without comparing at the expense of analysis.
That's how I'd do it anyway, and it probably suits a key player/ sub-argument method best, so if you have your own way of handling things, it might be best to stick with that at this point of the year. Ultimately the assessors care way more about the quality of your analysis anyway - the format and essay structure should be aspects that enhance (or at least don't hinder) your analysis anyway, so don't stress too much.

Is the second topic of The Complete Maus always about the graphic novel and it's relationship to exploring the narrative?

I noticed Free Original Practice Exams - 2015 does for all four practice exams, as does the 2014 one from VCAA.

I seem to score better on these, and find it easier because you can use a lot of visual evidence, which I find easier to recall than quotes. It would be comforting to know that it will be an option on the exam.

EDIT: Also, an interesting but somewhat annoying thing about Maus quotes is that a lot of them are intentionally grammatically incorrect, which makes for an interesting thing to analyse but at the same time makes remembering quotes word for word a difficult task. If I provide a quote but accidentally correct its grammar, will I lose marks for "misquoting" the text?
I wrote those 2015 prac. prompts because the 'Discuss the graphical features' structural prompts tend to be more difficult than the standard 'Discuss Artie and Vladek's relationship'-style character/thematic prompts, but if you're on top of analysing the visual elements then you should be fine for those ones.
There's no guarantee that VCAA will give you something like that, but in general, you tend to get one 'close' prompt that's zoomed in and provides a fairly narrow idea hinting at specific evidence, and then one 'big-picture' prompt that's about the author's intentions, the text as a whole, or decisions that the author makes. My suspicion is that VCAA just don't know what to do with a graphic novel since it's the first time one's ever been on the text list, so half of the prompts from them and by company papers are just like 'err... Discuss the pretty pictures!'
I could imagine something about the visual format and its effects, or perhaps the visual + textual combination coming up, but you should still prepare for other areas too.

With regards to the grammar of quotes, Vladek's broken English can make things tricky, but try to remember the quote in its original form. If you end up slightly altering things, you're not going to be penalised at all, especially not if you're just changing the grammar around. Most essays at the end of the year will have some degree of misquoting by virtue of it being a timed assessment task, so as long as the original meaning of the quote is preserved, you'll be fine.

Can someone please explain how to effectively do tonal analysis in the bodyparagraphs?
how much depth do we need to go into?
You can treat it like any other technique:
The author's optimistic tone, as evidenced by his proclamation that he is "so excited for the exam tomorrow!!!" coupled with the abundance of exclamation marks is designed to elicit...
Alternatively, you can use the tone as a point of transition, especially when there's a notable shift. Tone analysis can also be a single word (usually an adverb like aggressively, warmly, stoically, emphatically etc.) before the verb, so sentences like 'The author enthusiastically wishes students the best of luck for Wednesday' is incorporating tone analysis through that adverb 'enthusiastically.'
Just don't overdo it, but a bit of tone analysis (maybe once per paragraph?) can be a good thing.

What is a good method to conclude a comparative analysis?
Have one or two sentences about the main author's contention, and the intended effect on the audience. If there's a single comment, then you can do the same for that one and write something brief about the piece overall, but if there's multiple comments, I'd probably just focus on the main article rather than tick the boxes for every single text.

For context expository essays:

My plan is to do

Intro
text
external
external
conclusion

When I write my external paragraphs, should I link back to my main text?
That outline seems totally fine. There's no requirement to go back to the text, but the occasional reference can go a long way. Don't spend multiple sentences trying to tie things together, but you could have a quick 'Just as >character from set text< experienced a conflict of belonging (or w/e you're talking about for your Context,) so too did >this external example< involve a similar case of...'
That half-sentence parallel can make your piece feel more cogent overall, and less like three entirely separate discussions. In the event those links aren't possible, you'll still be safe enough writing about your external evidence in isolation though.

are 300-word paragraphs too long for context?

Nope! So long as you can write at least three body paragraphs within the time constraints, that's fine. But you might want to cut down and make your writing more concise if you're worried you might end up taking too long.

Most of my paragraphs in English were at least a page long, (and I think my lit ones were usually a page and a half,) but I was told if your focus is clear and you're not rambling, you're all good :)
Title: Re: VCE English Question Thread
Post by: Alter on October 27, 2015, 01:46:22 pm
The exam is obviously pretty close. Before we sit it, however, I just want to extend my gratitude to the English mod team and any others that have helped in the Q&A thread this year. On behalf of us all, I think it goes without saying that your help has been massively useful. Personally, my gratitude cannot be summed into just a few lines of text, so hopefully everything goes as planned tomorrow and we can put your lessons to action.

Going into 3/4 English, I had very little idea what to expect as I had never done 1/2 English at all. That being said, reading all of the guides over the summer holidays, asking (sometimes trying to answer!) questions, and analysing high-scoring responses has made me a whole lot more confident with English, and I feel like I can now say it's the subject I feel best about for exams.

So especially to Lauren, who is dedicating her weekdays to answering our last minute cramming questions, but also to others such as bangali: thank you so much!!!

We'll do you proud.
Title: Re: VCE English Question Thread
Post by: duo0024 on October 27, 2015, 01:58:11 pm
The exam is obviously pretty close. Before we sit it, however, I just want to extend my gratitude to the English mod team and any others that have helped in the Q&A thread this year. On behalf of us all, I think it goes without saying that your help has been massively useful. Personally, my gratitude cannot be summed into just a few lines of text, so hopefully everything goes as planned tomorrow and we can put your lessons to action.

Going into 3/4 English, I had very little idea what to expect as I had never done 1/2 English at all. That being said, reading all of the guides over the summer holidays, asking (sometimes trying to answer!) questions, and analysing high-scoring responses has made me a whole lot more confident with English, and I feel like I can now say it's the subject I feel best about for exams.

So especially to Lauren, who is dedicating her weekdays to answering our last minute cramming questions, but also to others such as bangali: thank you so much!!!

We'll do you proud.

Gentlemen, it has been a privilege playing with you tonight. *plays the violin*

Title: Re: VCE English Question Thread
Post by: CQcumber on October 27, 2015, 02:46:11 pm
what would be a reasonable length for a good essay?
Title: Re: VCE English Question Thread
Post by: literally lauren on October 27, 2015, 03:01:03 pm
what would be a reasonable length for a good essay?

Very approximately: 800 words.

Quality > quantity though, so if you feel like you've covered the prompts/ article(s) then that should do. I think there was a discussion about 4 or 5 pages back in this thread about how to tackle the unofficial word count, so you can scroll back if you're concerned, but I wouldn't worry too much.
Title: Re: VCE English Question Thread
Post by: literally lauren on October 27, 2015, 03:02:09 pm
We'll do you proud.
Gentlemen, it has been a privilege playing with you tonight. *plays the violin*

Awww <3 you guys

You'll do yourselves proud.
*cue the sappy melodramatic 80's ballad*
Title: Re: VCE English Question Thread
Post by: 99.90 pls on October 27, 2015, 05:03:16 pm
Gentlemen, it has been a privilege playing with you tonight. *plays the violin*

Wouldn't this imply impending death? :')
Title: Re: VCE English Question Thread
Post by: acm9 on October 27, 2015, 05:10:15 pm
if the LA article has comments, do you mention them briefly in the intro? And is it encouraged to begin LA with a contextualising statement?
Title: Re: VCE English Question Thread
Post by: Cristiano on October 27, 2015, 05:13:59 pm
if the LA article has comments, do you mention them briefly in the intro? And is it encouraged to begin LA with a contextualising statement?


Yeah you should mention in the introduction that there was contrasting views, and yes you can begin with one, though you don't have to, and you can get straight into it.
Title: Re: VCE English Question Thread
Post by: amanda2197 on October 27, 2015, 05:44:44 pm
What time would you advise to get to school prior to the exam? I usually get to school at like 8:30am on Wednesday's so I'm not sure whether to just go with my regular routine or just wake up, study and go to school?
Title: Re: VCE English Question Thread
Post by: Cristiano on October 27, 2015, 05:46:57 pm
Yea get there at 8:30, just treat it like a normal day, regular routine and try not to stress out :)

Good luck:)!
Title: Re: VCE English Question Thread
Post by: knightrider on October 27, 2015, 05:57:51 pm
Just wondering when we bring our equipment for tomorrow.

How are we meant to bring it(sorry if this sounds like a dopey question)

Like do we bring pens/ whatever we need tied together with an elastic band.

Also are we allowed to bring our equipment in a plastic folder that is see through?

Also how will we get our vcaa student number

Good luck to everyone for tomorrow!
Title: Re: VCE English Question Thread
Post by: Krakyn on October 27, 2015, 06:03:58 pm
Just wondering when we bring our equipment for tomorrow.

How are we meant to bring it(sorry if this sounds like a dopey question)

Like do we bring pens/ whatever we need tied together with an elastic band.

Also are we allowed to bring our equipment in a plastic folder that is see through?

Also how will we get our vcaa student number

Good luck to everyone for tomorrow!

Bring all of your materials in a clear bag. I guess a see through plastic folder would be alright- best thing to bring is one of those clear sandwich ziplock bags.

You need student ID to complete the exam, so bring your student ID card - your number should be on there.

If you don't have one, go to your school office and ask them to print one out (go a little early if this is the case). You will need ID for all your exams.

Also, bring a dictionary - which you can use in reading time, in case you need one.

Best of luck.
Title: Re: VCE English Question Thread
Post by: cosine on October 27, 2015, 06:52:43 pm
Have a few questions about context: (expository, ID+B)

- Is it alright if you include hardly/zero quotes, but actually include references.events from selected texts and other examples?
- How many references do we need to make to the text we studied? Once per paragraph, or?
- Can we include inclusive language like we/us/our?
- Anyone have any external sources for identity and belonging?
- Can you make up sources? Like make up a book and make up a story (this is all for expository)
Title: Re: VCE English Question Thread
Post by: literally lauren on October 27, 2015, 06:59:13 pm
Have a few questions about context: (expository, ID+B)

- Is it alright if you include hardly/zero quotes, but actually include references.events from selected texts and other examples?
- How many references do we need to make to the text we studied? Once per paragraph, or?
- Can we include inclusive language like we/us/our?
- Anyone have any external sources for identity and belonging?
- Can you make up sources? Like make up a book and make up a story (this is all for expository)
1) Totally fine to have no quotes at all from your Context text. Reference the ideas, and try to talk about what the text has to say about the prompt - that's all you need.

2) At least one overt, in-depth discussion, preferably in your first B.P. Everything after that is optional, but don't make the majority of your essay about the text, or it might feel limited. Longer explanation here

3) Yes, inclusive language is fine, and even recommended sometimes to avoid constantly having to say 'one' or 'people'

4) Use what you're familiar with, or consider some of these, but don't stress yourself out tonight trying to memorise a bunch of new stuff. Focus on refining what you know, and making that as good as it can be.

5) Technically yes, but it'd be  far better if you used the sources at your disposal. You won't actually be fact-checked, but it can get messy if you're trying to make up an example while you're analysing its connection to the ideas in the prompt. Actual real-world stuff will probably make for stronger discussion anyway.
Title: Re: VCE English Question Thread
Post by: cosine on October 27, 2015, 07:06:40 pm
Thank you, Lauren.

Can we also use generalised external sources, like the overall issue of refugees and how this questions one's true identity etc..?
Title: Re: VCE English Question Thread
Post by: literally lauren on October 27, 2015, 07:10:32 pm
Thank you, Lauren.

Can we also use generalised external sources, like the overall issue of refugees and how this questions one's true identity etc..?

Yep, that can work well. Just ensure you're familiar enough with the issue so that bringing up the example doesn't sound as bad as 'In Australia there are many people who are refugees and so this has an impact on their identity'  :P

Something I'd recommend as a good day-before-the-exam exercise; just go through all the evidence you have - textual or external - and practice summarising it in two sentences or less. When you're bringing it up in your body paragraph, it can often help to have one really concise statement of what the example is about first, and then you gradually unpack that over the course of your discussion. So doing this for your generalised examples might help give you and the assessor that extra bit of clarity needed.
Title: Re: VCE English Question Thread
Post by: jesterino on October 27, 2015, 07:17:19 pm
Hi just looking to clear some things up before the exam. My biggest problem this year is that I always feel lost when writing a Context (Identity + Belonging) piece. My approach so far has been to treat the prompt like a I would a text response then make 3 main points which supports my contention and provide examples from my text + external sources. I know that if I continue this way into the exam my section B would be marked quite low. Recently for my exam prep, I have been trying to write in a more "philosophical manner" for my context, exploring the implications and different sides to the prompt rather than simply trying to argue my stance. Is this a better approach? Honestly I still feel very lost as I do not have any solid ideas :S
Title: Re: VCE English Question Thread
Post by: elkxvii on October 27, 2015, 07:40:39 pm
What marks do I need to get a 40? I think I can pull 9/8/8, but would that only get me high 30s? What if I only get 8/8/8?
Title: Re: VCE English Question Thread
Post by: Krakyn on October 27, 2015, 07:47:03 pm
What marks do I need to get a 40? I think I can pull 9/8/8, but would that only get me high 30s? What if I only get 8/8/8?

I'm no expert in marking, but assuming your SAC scores have been a low A+ average, I think an 8/8/8 would give you a 40, if not, very high 30s.
Title: Re: VCE English Question Thread
Post by: tashhhaaa on October 27, 2015, 07:50:14 pm
hey guys,
when Medea says she's crying (line 900 p 74 in the John Davie translation) is it because she's thinking of the fact that she will lose her children or is she thinking of how Jason hurt her?

there is a stage direction stating that she's remembering Jason's treachery, but wouldn't that just make her angry at this stage?
Title: Re: VCE English Question Thread
Post by: fightingchance on October 27, 2015, 07:58:02 pm
How many persuasive techniques should we aim to discuss in our body paragraphs for language analysis? is around 3 enough?
Title: Re: VCE English Question Thread
Post by: Cristiano on October 27, 2015, 08:16:47 pm
hey guys,
when Medea says she's crying (line 900 p 74 in the John Davie translation) is it because she's thinking of the fact that she will lose her children or is she thinking of how Jason hurt her?

there is a stage direction stating that she's remembering Jason's treachery, but wouldn't that just make her angry at this stage?

At the thought of "Jason's treachery", Medea is reminded of the fact that, as a result of Jason's egocentricity, the children "must be killed" and she realizes that they will experience a horrific death, ultimately in order to fulfill her quest for vengeance. Thus, she weeps and laments that "What a wretched creature I am" and in turn, her eyes are "fill[ed] with tears" at this thought, which may be Euripides' attempt at displaying Medea's underlying passion for redemption.
Title: Re: VCE English Question Thread
Post by: YellowTongue on October 27, 2015, 08:24:01 pm
In this article: http://www.crikey.com.au/2013/04/12/high-speed-rail-just-doesnt-add-up-time-to-move-on/, the author says in the second paragraph: "For anyone who can add up, the high-speed rail phase 2 study released yesterday (or, if you were a newspaper journalist, Wednesday) should bring to an end the flirtation Australia’s polity is having with the idea of a high-speed rail network.".

I can see that he's trying to use humour (i.e. "flirting"), but when he says "or, if you were a newspaper journalist, Wednesday", what is he attempting to do through this? Is he making a joke of some sort?

Also, how would you describe what the author is trying to do when he says "For anyone who can add up"?

Thank you ;)
Title: Re: VCE English Question Thread
Post by: imaware on October 27, 2015, 08:25:21 pm
For text response if we structure our body paragraphs like below is that considered 'sitting on the fence' and not taking a stance or is it acceptable?

B1: agree
B2: agree
B3: disagree
B4: disagree

Thanks
Title: Re: VCE English Question Thread
Post by: Cristiano on October 27, 2015, 08:26:52 pm
Acceptable, just have an overall contention that discusses this change.
Title: Re: VCE English Question Thread
Post by: wombat123 on October 27, 2015, 08:29:36 pm
URGENT!!
I need ideas for this Medea prompt!!

"Jason's only flaw is his over-ambition. Discuss."
Title: Re: VCE English Question Thread
Post by: tashhhaaa on October 27, 2015, 08:51:53 pm
At the thought of "Jason's treachery", Medea is reminded of the fact that, as a result of Jason's egocentricity, the children "must be killed" and she realizes that they will experience a horrific death, ultimately in order to fulfill her quest for vengeance. Thus, she weeps and laments that "What a wretched creature I am" and in turn, her eyes are "fill[ed] with tears" at this thought, which may be Euripides' attempt at displaying Medea's underlying passion for redemption.

thank you!!!!
Title: Re: VCE English Question Thread
Post by: literally lauren on October 27, 2015, 09:52:55 pm
                    ~~~Just for the sake of visibility!!!
I'll be hanging around the forums all night and in the early hours of the morning tomorrow, so whilst I'd advocate getting a good night's sleep, if people want to drop their essays in on the forums someplace, I will get to everyone's, including anything that's slipped under the radar this week, so feel free to post a couple of things tonight, and then you'll hopefully get a chance to read through the recommendations tomorrow morning.

Best of luck everyone!
;D ;D
Hi just looking to clear some things up before the exam. My biggest problem this year is that I always feel lost when writing a Context (Identity + Belonging) piece. My approach so far has been to treat the prompt like a I would a text response then make 3 main points which supports my contention and provide examples from my text + external sources. I know that if I continue this way into the exam my section B would be marked quite low. Recently for my exam prep, I have been trying to write in a more "philosophical manner" for my context, exploring the implications and different sides to the prompt rather than simply trying to argue my stance. Is this a better approach? Honestly I still feel very lost as I do not have any solid ideas :S

The structure that you've outlined above isn't necessarily a bad one; I did something very similar in that I just wrote a mostly expository piece (with a creative intro + concl. from memory) and four body paragraphs, the first of which unpacked the set text, and the others based on external references.

The only way that could lead to you scoring really low marks would be if you talked about the text for too long, or in too much detail, as you're expected to move away from this over the course of your essay.

At this point, it's best to stick with what you're most familiar with though. Just make sure your examples are strong, and that you can link them to the prompt fluently and effectively, and there's nothing stopping you from getting a great mark.

How many persuasive techniques should we aim to discuss in our body paragraphs for language analysis? is around 3 enough?
Three would be sufficient, but try to aim for more if you get a chance. Again, stick with what you know, so if you're used to analysing three (give or take the visual and/or comment(s)) then that'll be perfectly fine.

In this article: http://www.crikey.com.au/2013/04/12/high-speed-rail-just-doesnt-add-up-time-to-move-on/, the author says in the second paragraph: "For anyone who can add up, the high-speed rail phase 2 study released yesterday (or, if you were a newspaper journalist, Wednesday) should bring to an end the flirtation Australia’s polity is having with the idea of a high-speed rail network.".

I can see that he's trying to use humour (i.e. "flirting"), but when he says "or, if you were a newspaper journalist, Wednesday", what is he attempting to do through this? Is he making a joke of some sort?

Also, how would you describe what the author is trying to do when he says "For anyone who can add up"?
I'm pretty sure the reference to Wednesday is just because journalists usually get an advanced copy of these things the day before to ensure they can report on them accurately. The idea of flirting is more like figurative language (implying Australian policy is some effete, silly thing) and the 'for anyone who can add up' is just an offhanded remark to elicit derision. Those articles don't tend to be very persuasive though. At least, not in the same way the VCAA exams are.

For text response if we structure our body paragraphs like below is that considered 'sitting on the fence' and not taking a stance or is it acceptable?

B1: agree
B2: agree
B3: disagree
B4: disagree

Thanks
This'd work for Context, but a safer method for T.R. would be to either cut one paragraph, or change one so that it does the opposite, and you have a 3-1 majority instead of a 2-2 split.

URGENT!!
I need ideas for this Medea prompt!!

"Jason's only flaw is his over-ambition. Discuss."
Tbh at this point, if you're not prepared for a prompt about Jason or the theme of ambition, you'd probably need to spend some time tonight or tomorrow going back to the text. Try to think about Jason's flaws in general; would you consider him a very flawed character, or is he a good person in a bad situation who is taken advantage of by other characters? And would you call him ambitious? Does he follow his ambitions and prioritise his own concerns above those of others? Just try to unpack the prompt a little bit until it's more familiar to you, and build on the understanding that you have, rather than try to find some elusive 'ideas.'
Title: Re: VCE English Question Thread
Post by: Rishi97 on October 27, 2015, 10:04:44 pm
                    ~~~Just for the sake of visibility!!!
I'll be hanging around the forums all night and in the early hours of the morning tomorrow, so whilst I'd advocate getting a good night's sleep, if people want to drop their essays in on the forums someplace, I will get to everyone's, including anything that's slipped under the radar this week, so feel free to post a couple of things tonight, and then you'll hopefully get a chance to read through the recommendations tomorrow morning.

Lauren you are absolutely amazing! So nice of you to help everyone out!!!!
Title: Re: VCE English Question Thread
Post by: cameotodd on October 27, 2015, 10:05:42 pm
                    ~~~Just for the sake of visibility!!!
I'll be hanging around the forums all night and in the early hours of the morning tomorrow, so whilst I'd advocate getting a good night's sleep, if people want to drop their essays in on the forums someplace, I will get to everyone's, including anything that's slipped under the radar this week, so feel free to post a couple of things tonight, and then you'll hopefully get a chance to read through the recommendations tomorrow morning.


Wow, you're so amazing! Legit would've been my dream to have a teacher with the same dedication as you!!!
Anyway, I'm just going to post this text response on In the Country of Men, and if you get the chance have a look :)
Title: Re: VCE English Question Thread
Post by: ladybug on October 27, 2015, 10:07:26 pm
My teacher advocates using 2 main texts for the conflict expository that i would write, is this fine and what text would i write in the nominated text?

Good luck tomorrow and we're all gonna make it
Title: Re: VCE English Question Thread
Post by: mnafady2006 on October 27, 2015, 10:08:29 pm
How can we incorporate views and values statements in our text response?
do we usually leave it to then end or? could you provide an example
Thanks
Title: Re: VCE English Question Thread
Post by: cosine on October 27, 2015, 10:17:54 pm
URGENT HELP LAUREN:

For text response, for example with Euripides' view are we allowed to use a whole paragraph just talking about his view on a certain prompt? Say for example it is about how everyone in the play is equally repugnant, can we discuss in the first two paragraphs about how each character displays their own levels of repugnancy and then on the last para, talk about Euripdies' ultimate view on which character is more repugnant, and include evidence to back this up? (or maybe not definitively say Euripides does this or does that, but make supportive statements like 'Euripides seems to present the notion that Medea's actions were indeed justified'.)

Title: Re: VCE English Question Thread
Post by: literally lauren on October 27, 2015, 10:33:22 pm
Lauren you are absolutely amazing! So nice of you to help everyone out!!!!

Amazing... mild insomniac... procrastinating from my own uni exams... these are all basically synonyms right?  ::)

Wow, you're so amazing! Legit would've been my dream to have a teacher with the same dedication as you!!!
Anyway, I'm just going to post this text response on In the Country of Men, and if you get the chance have a look :)
Will attach this essay along with your Id&b one in the Prac. Exam thread once I'm done so we can keep this for questions, but feel free to post more there or on the Submissions Board and I'll get to them :)

My teacher advocates using 2 main texts for the conflict expository that i would write, is this fine and what text would i write in the nominated text?
If you're writing three body paragraphs, then it'd be better to just use one text in your first para, and make your other two explorations based on external evidence. If you're writing four paragraphs, then you can use two texts, but perhaps use one of them in your intro, or reference it in another paragraph, and then you can state that one to be your main text drawn upon.

In general, both of the texts should fit the prompt tomorrow, but that doesn't mean you should use both of them. Your external examples will hopefully be more effective, and since you get marks for your quality of exploration, it's best to use those after you've made your one necessary text link unless you don't have any other option.

How can we incorporate views and values statements in our text response?
do we usually leave it to then end or? could you provide an example
Thanks
Views and values statements would be things like 'The author suggests that...' or 'The audience is made to feel sympathetic towards...' They're good for the intro, conclusion, and the ends of your body paragraphs.
So since you're doing This Boy's Life (I think?) you might have one paragraph discussing the portrayal of Rosemary as a mother, then get to the end and say something like 'Therefore, Wolff depicts Rosemary to be an incredibly well-intentioned parent, even though her actions may not always have entirely positive consequences.'

For text response, for example with Euripides' view are we allowed to use a whole paragraph just talking about his view on a certain prompt? Say for example it is about how everyone in the play is equally repugnant, can we discuss in the first two paragraphs about how each character displays their own levels of repugnancy and then on the last para, talk about Euripdies' ultimate view on which character is more repugnant, and include evidence to back this up? (or maybe not definitively say Euripides does this or does that, but make supportive statements like 'Euripides seems to present the notion that Medea's actions were indeed justified'.)
Yes, if you really wanted to, but I think you'd need to mention Euripides in your other paragraphs too. I don't know how you'd argue that one of the characters displays a trait without implying that the author is the one conveying this sense, so a better approach might be to examine certain characters' thoughts or intentions which strike you as notably harrowing or unjustified, then discuss some actions which might fall in to that category, and then finally consider whether any one character can be called 'completely' repugnant, or whether they all have some redeeming qualities.

But the paragraph breakdown doesn't instantly disqualify you from attaining high marks. The example I outlined above would be one possible way of doing things, but it's also likely that if you keep the task criteria in mind and are able to demonstrate a good textual understanding whilst unpacking the prompt, your approach could be equally efficacious :)
Title: Re: VCE English Question Thread
Post by: Adequace on November 01, 2015, 09:32:19 pm
Hi Lauren,

I'm studying "Little Miss Sunshine" at school currently and I'm having trouble embedding my quotes in to my statements.

We're supposed to analyse quotes and comment on what they tell us about the character's philosophy of success, pretty much writing a TR paragraph excluding the TS and LS. My teacher is encouraging me to embed the quote within the paragraph so it seems more cohesive and to prepare me for VCE English. My statements have usually been me stating the quote and then analysing it which seems pretty basic.

For example,

Quote - Richard: "Luck is the name losers give to their own failings. It's about wanting to win."

I wrote:
Richard suggests "luck" is an excuse that "losers give to their own failings", but achieving success is through your will "to win". This demonstrates that Richard believes achieving success or submitting to failure is solely dependent on the amount of effort you invest.

I'm not necessarily sure if this is correct or not, if you have time could you write up an example of how a pro would embed the quote in to their statement?

(I tried reading highly scoring TRs but I haven't read the novels and the language used is sophiscated as hell..)

Edit:
In a similar response to a similar quote, my teacher told me I just had to provide evidence then it would be okay. I thought the quote was the evidence?
Title: Re: VCE English Question Thread
Post by: literally lauren on November 05, 2015, 03:16:33 pm
I sure don't get much of a holiday with you diligent Year 11's floating about the place, huh? :P Jks jks, the remainder of this year and the summer holidays are great times to tidy up things in preparation for Year 12, and quote integration is a big one.

Although there are a few notable right and wrong ways to embed quotes, it can help to think of them on a spectrum from 'totally not part of the sentence' through to 'so smoothly blended it's like a delicious blended smoothie.'

On the 'wrong' end of the line you have things like
Richard idolises the idea of winning. "Luck is the name losers give to their own failings." This quote shows that he cares about winning.
Totally clunky, right?

Then you have things that are grammatically integrated, but the fact that they point out 'the character says: this' makes them a bit laborious. These can be okay in moderation though:
Richard idolises the idea of winning, as is evidenced by his declaration that: "luck is the name losers give to their own failings."
A little better, but you don't want to have to point things out that way every time. The above format can be really useful for Language Analysis, however; the fact that it allows you to comment on the language a bit (ie. his declaration that... his glorification of... his condemnation towards...) and there's a good opportunity at the end of this quote to start unpacking the words themselves.

Next, you have quotes that are fully integrated - the key difference b/n the orange and the green is that with this one, I could remove the quotation marks, and I wouldn't be able to tell what was being quoted. You can use a blend of these two styles if you want, but this next sort is usually the quickest and most efficient:
Richard has a quixotic faith in the promise of winning and he completely dismisses "luck" as a word which "losers give to their own failings."
This seems to be what you're doing at the moment, so this next option is probably going to be what you can start implementing.

To take your example:
Richard suggests "luck" is an excuse that "losers give to their own failings", but achieving success is through your will "to win". This demonstrates that Richard believes achieving success or submitting to failure is solely dependent on the amount of effort you invest.
This is getting across your point clearly, but notice how you've got one sentence that is just summarising what Richard says, and then the second sentence is where you analyse it by saying 'This demonstrates...'? This can easily be combined into a more efficient bit of analysis using the magic of NominalisationTM - now available in 4 easy payments of $19.99

Really, that's just a fancy word that means 'noun-ification' where you take a verb you're using in a summative sense, and turn it into a noun so you can analyse it.

So here you've used the verb 'suggests,' which we'll turn into 'suggestion' in order to get:
Richard's suggestion that "luck" is an excuse that "losers give to their own failings" demonstrates that he believes achieving success or submitting to failure is solely dependent on the amount of effort one invests.

Gradually, you'll find more and more ways of doing this; I was a fan of pointing out 'The fact that >something happens< is indicative of >something<' eg. 'The fact that Richard condemns "losers" is indicative of his own insecurities.' You can even shorten this to just 'That Richard condemns "losers" is indicative of his own insecurities,' which is a neat, creative way of changing things up, but it can sound a bit unnatural since it's kind of an archaic sentence structure.


So the two main things you should aim to do when embedding quotes:
1) make sure if flows grammatically with the sentence, and ideally integrate it to the point where you could remove punctuation and still have it be sensible. At the very least, aim for a 80/20 ratio of properly integrated green/blue ones to a few more overt orange types.
2) try to put the analysis within the same sentence as your quote. This enables you to more easily avoid summary, and can help make you more concise too.

(I tried reading highly scoring TRs but I haven't read the novels and the language used is sophiscated as hell..)
I can absolutely understand not being able to trek through some of the really verbose essays on these forums, but I'd say keep looking at those high scoring responses. They can take a while to grasp, but when you can start evaluating what they're doing right and wrong, you'll be able to improve your own writing too. Plus, all that hellishly sophisticated vocabulary will be stuff that you can write down, look up, and start using in your own writing where appropriate :)

Edit:
In a similar response to a similar quote, my teacher told me I just had to provide evidence then it would be okay. I thought the quote was the evidence?
Not quite sure what you're teacher is trying to say here, but based on what you've written here, perhaps s/he wants you to better spell out the connection between the evidence and your analysis. In other words, why is this quote evidence?
Break it down into stages:
1. Richard says luck is an excuse for losers.
2. He also says winning has to be something people want.
3. ? ? ?
4. Therefore he believes people are in control of their own lives, and that their success or failure is entirely their own fault.

There's a key link between the things that Richard says, and the estimation of his belief system, so try and find a midway point there. How do you know he invests so much faith in this belief? Why might his choice of words be particularly important? Think about the connotations of the word "excuse," or the way he divides/dichotomises the world so aggressively into 'winners' and 'losers,' then use this evidence as a means of substantiating your interpretation.

Hope that helps :)
Title: Re: VCE English Question Thread
Post by: Adequace on November 05, 2015, 04:40:57 pm
Extremely helpful reply, thank you so much!
Title: Re: VCE English Question Thread
Post by: argent on November 05, 2015, 07:08:02 pm
Hi Lauren,
Year 11 English exam coming up soon and I have a quick question.
I'm struggling quite a bit trying to form contentions for context prompts. I've been working on a few and been brainstorming, but the ideas just aren't seeming to come together. One of the ones I'm working on is:
"The need to belong can completely change who we are."
Of course there's the typical "does it always completely change our identity or only partially" and "what if people do not have a need to belong" etc.
So my question is, how would I tackle this? I really don't know how to put my ideas together to form a overall contention, and then split my ideas up into paragraphs without being to heavy on one side of the prompt as I'd like to cover all aspects evenly.
Thanks!
Title: Re: VCE English Question Thread
Post by: Shinkaze on November 07, 2015, 10:44:10 pm
Can anyone please link me to the attachment for the "useful language for language analysis", it just says attachment not found for me when I try to download it  :'(
Title: Re: VCE English Question Thread
Post by: YellowTongue on November 12, 2015, 06:11:13 am
Is it acceptable to change the form of a word when quoting in an essay?

For example, if I was quoting: "Fly, Mr Pelican, despite your broken wing. You know you can do this with the power of positivity!

Would it be acceptable to write something like this?: The author illustrates the "power" of approaching situations with a "positive" point of view by showing Mr Pelican "flying" with his "wings"; despite "breaking" one of them earlier in the novel.

Thanks for your help  ;D
Title: Re: VCE English Question Thread
Post by: heids on November 12, 2015, 07:56:54 pm
Dug round in downloads till I found that useful language document - attached :) (also lots of vocab if you scroll down in this post, nudge nudge ;))

Is it acceptable to change the form of a word when quoting in an essay?

For example, if I was quoting: "Fly, Mr Pelican, despite your broken wing. You know you can do this with the power of positivity!

Would it be acceptable to write something like this?: The author illustrates the "power" of approaching situations with a "positive" point of view by showing Mr Pelican "flying" with his "wings"; despite "breaking" one of them earlier in the novel.

Thanks for your help  ;D

It's more than acceptable, it's good - but you have to use square brackets to show that you're changing it; e.g. 'fly' in original quote would become 'fly[ing]'.  (Note: I think weaving in five words (4/5 of them being changed) separately in one sentence looks a bit clumsy; instead, you could try 1-3 phrases like 'power of positivity' and 'broken wing', which would feel nicer to me at least.)
Title: Re: VCE English Question Thread
Post by: YellowTongue on December 04, 2015, 01:14:55 pm
Is it acceptable to draw ideas from more than one text in Section B (providing that they're on the VCAA English text lost)?
Title: Re: VCE English Question Thread
Post by: thaaanyan on December 04, 2015, 01:45:36 pm
Is it acceptable to draw ideas from more than one text in Section B (providing that they're on the VCAA English text lost)?

Do you mean, that you've studied two context texts and you draw upon them both in an essay? I was specifically taught not to, one of my teachers told me it just appeared a bit lazy, and it was better to draw on external examples. She's an examiner so I trusted her judgement. But I guess, at the end of the day, if your teacher is fine with it then it's ok, but it's something to consider whether you'd like to do it in the exam or not,
Title: Re: VCE English Question Thread
Post by: YellowTongue on December 04, 2015, 02:08:43 pm
Do you mean, that you've studied two context texts and you draw upon them both in an essay?

No, I mean are you allowed to draw ideas from multiple texts from the same context?

Thank you for your help  ;D
Title: Re: VCE English Question Thread
Post by: appleandbee on December 04, 2015, 02:17:52 pm
No, I mean are you allowed to draw ideas from multiple texts from the same context?

Thank you for your help  ;D

Using multiple texts is fine, though personally I would avoid texts from different contexts or old texts list. This is for the exam at least. For SACs (depending on your school), you are only allowed to use one focus text from the list. Avoid being to text-dense in general because you want a balance between your selected texts and external ideas!
Title: Re: VCE English Question Thread
Post by: pi on December 04, 2015, 02:36:31 pm
In the exam booklet though you still have to have a "main text", which will be one of the two you studied throughout the year. That said, you can draw on other texts and other external resources in your piece, but one text will be more predominantly drawn upon (ie. your chosen "main text").
Title: Re: VCE English Question Thread
Post by: thaaanyan on December 04, 2015, 05:25:08 pm
No, I mean are you allowed to draw ideas from multiple texts from the same context?

Thank you for your help  ;D

Yep this is totally fine :) Though echoing apple-bee well rounded pieces tend to draw from a range of sources such as history, politics, philosophy etc.
Title: Re: VCE English Question Thread
Post by: Callum@1373 on December 25, 2015, 09:42:51 pm
This is for anyone who has read This Boy's Life, when do I refer to Tobias Wolff as Jack and when do I refer to him as Toby?
Title: Re: VCE English Question Thread
Post by: 99.90 pls on December 26, 2015, 05:23:38 pm
This is for anyone who has read This Boy's Life, when do I refer to Tobias Wolff as Jack and when do I refer to him as Toby?

When discussing the character, pick either Jack or Toby and stick with that name throughout your discussion. When referring to the author, use 'Wolff'. At the end of the day, it doesn't really matter which name you use to refer to the character, as long as you're consistent.
Title: Re: VCE English Question Thread
Post by: Callum@1373 on December 26, 2015, 05:48:22 pm
When discussing the character, pick either Jack or Toby and stick with that name throughout your discussion. When referring to the author, use 'Wolff'. At the end of the day, it doesn't really matter which name you use to refer to the character, as long as you're consistent.
Aha cheers
Title: Re: VCE English Question Thread
Post by: Callum@1373 on December 27, 2015, 07:43:16 pm
I have a question with drawing ideas from my text, This Boy's Life. I've noticed how Wolff comments on Chinook, and how due to the mountains light rarely reaches town. This kinda draws a similarity with his life in Chinook, he does not find any happiness with Dwight and feels miserable living with him and longs for his mother. In other words, physical environment <=> living with Dwight, there's a big reflection.

My problem is, if this were a fictional novel I could say how the author has chosen for the landscape to reflect the life living with Dwight. However, because This Boy's Life is a memoir, it actually HAPPENED that way, in other words, I cannot create meaning from this link between the environment and life with Dwight because that's how it actually physically was in real life. So how do I integrate that into my essays given what i've said above?
Title: Re: VCE English Question Thread
Post by: Alter on December 27, 2015, 08:15:13 pm
The fact that he chose to include it in his memoir is what makes it something you can mention. This is true for any non-fiction text. Just because it actually happened that way, doesn't necessarily mean you can't give reference to it as an author's choice in a text response. The author could have easily not mentioned the landscape in your text, but they chose to for specific reasons (or at least that's how you have to think as an English student).

Sorry for shortness, posting from phone. Lmk if you need further clarification of what I mean.
Title: Re: VCE English Question Thread
Post by: SlothPlays on December 27, 2015, 08:27:29 pm
I have a question with drawing ideas from my text, This Boy's Life. I've noticed how Wolff comments on Chinook, and how due to the mountains light rarely reaches town. This kinda draws a similarity with his life in Chinook, he does not find any happiness with Dwight and feels miserable living with him and longs for his mother. In other words, physical environment <=> living with Dwight, there's a big reflection.

My problem is, if this were a fictional novel I could say how the author has chosen for the landscape to reflect the life living with Dwight. However, because This Boy's Life is a memoir, it actually HAPPENED that way, in other words, I cannot create meaning from this link between the environment and life with Dwight because that's how it actually physically was in real life. So how do I integrate that into my essays given what i've said above?

A'lot of the time, the book is symbolic for a lot things. An instance of this in Page 62 after visiting Dwight's place in Chinook and Dwight shows him the salmon. In this section Wolff is metaphorically suggesting that Jack and Rosemary’s movement away from their home into the unknown is doomed. More broadly, he is examining the way that the constant journey looking for change and security is futile. More than that, the toll it takes is as visible as the ‘strips of flesh’ hanging from the body. You can find that environments + symbolism throughout the novel is quite significant.

Another the environment of the home in Chinook, when Dwight paints the whole house white before Rosemary arrives. In this instance it shows Dwight trying to literally paint over the inadequacy of what he has to offer to Rosemary. In doing so, in trying to make everything perfect, he makes the strangeness more pronounced and highlights his own inability to fix things.

It is important to recognize that the book is a memoir. However it is Wolff who decides what goes in and for what purpose, and most of the time its got a symbolic purpose, and you can talk alot about that.

Title: Re: VCE English Question Thread
Post by: literally lauren on December 27, 2015, 08:30:25 pm
I have a question with drawing ideas from my text, This Boy's Life. I've noticed how Wolff comments on Chinook, and how due to the mountains light rarely reaches town. This kinda draws a similarity with his life in Chinook, he does not find any happiness with Dwight and feels miserable living with him and longs for his mother. In other words, physical environment <=> living with Dwight, there's a big reflection.

My problem is, if this were a fictional novel I could say how the author has chosen for the landscape to reflect the life living with Dwight. However, because This Boy's Life is a memoir, it actually HAPPENED that way, in other words, I cannot create meaning from this link between the environment and life with Dwight because that's how it actually physically was in real life. So how do I integrate that into my essays given what i've said above?

Seconding what Alter and Slothplays have said; you can still comment on its inclusion in the memoir and discuss it's significance in that sense.

And if you wanted to push things even further into structural territory, you could think about how the author includes them, and the kinds of descriptions Wolff offers. There's a world of difference between:
a. 'There are mountains where I live. I think I'll have soup for dinner today'
and
b. 'The mountains outside blocked out the sun, so there was very little daylight to counteract my misery.'
In both instances, the fact that there are mountains is something that's presented as being true, but it's inclusion in the first case isn't really all that important. Whereas, in the second sentence, you could comment on the connection being made by the character, and discuss that metaphor/analogy/parallel in your essay.

You could also talk about it in relation to the retrospective nature of the text - I've found that to be a fairly big concern when dealing with TBL; how reliable is Wolff as a narrator? How much of this story is an accurate reflection of Jack's life, and how much if it is coloured by the advantage of Wolff's hindsight?
Title: Re: VCE English Question Thread
Post by: Callum@1373 on December 27, 2015, 08:41:42 pm
The fact that he chose to include it in his memoir is what makes it something you can mention. This is true for any non-fiction text. Just because it actually happened that way, doesn't necessarily mean you can't give reference to it as an author's choice in a text response. The author could have easily not mentioned the landscape in your text, but they chose to for specific reasons (or at least that's how you have to think as an English student).

Sorry for shortness, posting from phone. Lmk if you need further clarification of what I mean.
Cheers!

A'lot of the time, the book is symbolic for a lot things. An instance of this in Page 62 after visiting Dwight's place in Chinook and Dwight shows him the salmon. In this section Wolff is metaphorically suggesting that Jack and Rosemary’s movement away from their home into the unknown is doomed. More broadly, he is examining the way that the constant journey looking for change and security is futile. More than that, the toll it takes is as visible as the ‘strips of flesh’ hanging from the body. You can find that environments + symbolism throughout the novel is quite significant.

Another the environment of the home in Chinook, when Dwight paints the whole house white before Rosemary arrives. In this instance it shows Dwight trying to literally paint over the inadequacy of what he has to offer to Rosemary. In doing so, in trying to make everything perfect, he makes the strangeness more pronounced and highlights his own inability to fix things.

It is important to recognize that the book is a memoir. However it is Wolff who decides what goes in and for what purpose, and most of the time its got a symbolic purpose, and you can talk alot about that.


I love you for that haha. So basically, it's the fact that Wolff CHOSE to put it in the memoir, he has a wide array of things he can talk about, but it's the fact that he chooses to talk about this because he wants readers to realize what his life was like in Chinook through this analogy...

You could also talk about it in relation to the retrospective nature of the text - I've found that to be a fairly big concern when dealing with TBL; how reliable is Wolff as a narrator? How much of this story is an accurate reflection of Jack's life, and how much if it is coloured by the advantage of Wolff's hindsight?
Hey lauren, with the idea that you have mentioned above, where would I go with it? As in, how can I start putting that knowledge into practice? Like if I were to talk about that in an essay, what would I be saying?
Title: Re: VCE English Question Thread
Post by: literally lauren on December 27, 2015, 09:30:59 pm
Hey lauren, with the idea that you have mentioned above, where would I go with it? As in, how can I start putting that knowledge into practice? Like if I were to talk about that in an essay, what would I be saying?

Firstly, you can think of the text as existing on four main levels:

                    1. ---Views & Values Messages that the author expresses ---
                    2. ---Thematic Ideas and Concerns in the text---
                    3. ---Characters and their Journeys---
                    4. ---Structural and Language Features that convey meaning---

For the first part of your study (ie. reading the texts over the Summer and discussing them in class once school starts,) you'll mainly be concerned with levels two and three, since that's where most of the summative details are. So questions like 'Who is Arthur and why is he significant?' or 'What do Jack's experiences say about the nature of childhood?' are going to be the most valuable at that stage.

Once you've built a stable foundation there, you can move in either direction: up to level one (so zooming out of the text and thinking about bigger, broader ideas) or down to level four (a.k.a. zooming in to close textual details.) Otherwise, if you rush into thinking about huge concepts like authorial bias or how the setting affects our interpretation, you'll find it really hard to reconcile the different facets of the text, and you might even make things more complicated for yourself later down the track. Therefore, you shouldn't feel like you should address the biggest questions first, but rather document them until you are better equipped to explore them fully later.

But given where you're at now, you're probably starting to notice a few things like Jack's desire for self-determination in the way he tries to shape the narrative of his life, or even in smaller (~verging on 'level 4' structural) details like how he changes his name. Likewise, you might also read certain passages (especially the end of the text) where Wolff's narration becomes much more overt and wonder what the significance of those kinds of excerpts are. That's why it can be helpful to start considering the role of the author and his hindsight. You're not going to be able to comprehensively answer these questions just yet, but by acknowledging them, you'll broaden your understanding of the text in a way that'll be very advantageous later.

Secondly (...even though I made more than one point before, but whatevs...) embedding this in an essay is easier than it might seem. Sometimes you'll get a prompt that directly asks you to discuss such an idea, eg. Wolff's depiction of Jack in This Boy's Life is more idealised than honest. Do you agree? or In This Boy's Life, it is Tobias Wolff's narration that helps us justify Jack's behaviour and sympathise with his character. Discuss. in which case you'll obviously have to structure your discussion around that concern. But it's also possible for you to get a more 'level 2 or 3' prompt like Rosemary is the only positive role model in Jack's life. Discuss. and still be able to weave in discussions about authorial/narrative bias. For example, how do we know Rosemary is a positive role model? Does she seem that way to Jack, or to Wolff? Does Wolff perhaps look back on her parenting in a different way now that he's an adult, in contrast to how he would've seen it when he was a child? Are Jack's views and Wolff's views necessarily one and the same?

One way to explain this that I've found helps a lot for this text is to imagine you wanted to write a story about how drugs are bad, and you did drugs as a kid so you want to use that as a kind of moral reference point. But when you talk about yourself in hindsight, present-day-you is going to have a very different perspective, as well as different priorities, and perhaps even a different world view to drug-taking-you-from-the-past. So when you write from the point of view of your former self, you're not going to be accurately conveying yourself as you are in the present, because you're trying to capture how you were back when you were high on mescaline. Therefore, the values of your past-self as a character in your 'Drugs Are Bad' memoir are going to be different to the values you have as an author.

That's generally the kind of distinction you want to make when it comes to TBL; treat 'Jack/Toby' -- I prefer Jack, but I've seen prompts go both ways -- vs. 'Wolff' as separate entities and discuss them accordingly.

Then, when it comes to your analyses, you might have a point of discussion like:

'...which reinforces the idea of Jack's lies being damaging to his relationships. However, by virtue of Wolff's frankness and the candid honesty in his depiction of his former self, we are inclined to forgive this fault in Jack and see the memoir itself as a kind of penance for his wrongdoings. Furthermore, in the chapter where...'

or you might have a whole sub-argument structured around that kind of idea. It all depends on the prompt, but until you get to grips with the ideas in isolation, it can be tough to visualise how they'll all come together in the end.

That ended up being a lot longer than intended, but to answer your question more succinctly, the way you'd discuss this idea would hinge on what facet of this idea was relevant to the essay topic, and what you'd be saying would be whatever would strengthen your contention whilst showcasing your knowledge of the text. In some instances, that content might include a comment or two about the notion of an unreliable narrator or of authorial hindsight, but other times it might not, so be flexible with it :)

Title: Re: VCE English Question Thread
Post by: Callum@1373 on December 27, 2015, 10:49:19 pm
^ I've copied that into a word document for future reference haha  :)

Okay so essentially, I shouldn't be thinking about these 'big' ideas too soon before i've got my bundle of ideas together. I'm not trying to divulge too far into it yet, but could you give me a short example of where speaking about authorial bias/wolff's accuracy strengthens your contention just so I can get a rough idea of one of the ways I could integrate it into an essay? Thanks stacks you legend  :D

btw is mescaline available in broadmeadows xD
Title: Re: VCE English Question Thread
Post by: Hydraulix on January 01, 2016, 05:03:05 pm
I saw some people mention early last year that it was really important to read other books than our English books set for this year. Knowing that it is imperative to read our set texts multiple times and also do work for our other subjects, I cant see how people find the time to read other novels etc. Could someone please explain why reading other texts in important for year 12 english
Title: Re: VCE English Question Thread
Post by: pi on January 01, 2016, 05:11:49 pm
I haven't read that advice, but perhaps the idea would be to broaden their knowledge for their context area? Personally, I didn't read other books, didn't have the time nor the willpower haha.
Title: Re: VCE English Question Thread
Post by: heids on January 01, 2016, 05:15:54 pm
I saw some people mention early last year that it was really important to read other books than our English books set for this year. Knowing that it is imperative to read our set texts multiple times and also do work for our other subjects, I cant see how people find the time to read other novels etc. Could someone please explain why reading other texts in important for year 12 english

Reading more makes you write better.  It broadens your vocab and just makes good grammar and expression feel right and flow naturally - so you can convey your ideas much more clearly to the assessor (and in general whenever you have to write something in life).  Finding time was easy for me because reading a good book is my favourite form of relaxation, so I didn't see it as study but as a break.
Title: Re: VCE English Question Thread
Post by: pi on January 01, 2016, 05:24:00 pm
Reading more makes you write better.  It broadens your vocab and just makes good grammar and expression feel right and flow naturally - so you can convey your ideas much more clearly to the assessor (and in general whenever you have to write something in life).  Finding time was easy for me because reading a good book is my favourite form of relaxation, so I didn't see it as study but as a break.

Personally, I feel this advice would be much more applicable to those in more junior years of schooling, say years 7-10, whereby laying down foundations and doing the grunt-work at those less relatively stressful stages serves as excellent work for VCE. Other than my annual HP marathon, I don't think I'd have the motivation to read many other books, I'd stick to reading high quality essays and picking things up from them to improve what skills I already have. Granted, if reading is a favourable leisure activity then you're an exception :P

Just my thoughts on it.
Title: Re: VCE English Question Thread
Post by: heids on January 01, 2016, 05:31:20 pm
^ Agreed, entirely.
Title: Re: VCE English Question Thread
Post by: tashhhaaa on January 02, 2016, 12:14:29 am
just my 0.0005 cents but I think I only read 1 novel (not even, it was a play) apart from my set texts this year oops, last year

I found that it was easier to expand my vocabulary by reading newspaper-like articles about world issues etc, but I guess it depends what you're interested in. Also, this may seem like the wrong advice but I also learned a few new words from TV shows and movies (but again, it depends what you watch)

High quality essays (heaaaaaaaaaaps on AN) are invaluable in helping you improve your vocab and expression without even realising it.

I'm a bit of a closet nerd so something I picked up from my brother was to add unfamiliar words I heard or read to a list on the notes section of my phone, look them up, learn them and vow to use them at some point in my work. Never regretted it.
Title: Re: VCE English Question Thread
Post by: Callum@1373 on January 02, 2016, 05:02:57 pm
Another question for anyone who has studies this boy's life  :P

I was just analyzing the chapter where Rosemary goes for a walk with Jack and tells him about the letter she received from her brother in paris (about Jack moving there and studying while rosemary finds a job in chinook). After they both express their excitement, Rosemary says "Just don't say anything about the check [to Dwight]". Obviously, she is well aware that Dwight steals their money, particularly Jack's money from the paper rounds. But Rosemary doesn't confront Dwight about this? Is that most likely because she is scared Dwight will be aggressive to her, and she cannot withstand the abuse she will likely receive? Could that qualify her as an irresponsible parent because she is not defending her rights and not supporting Jack by ensuring his earnings are not spent by Dwight? Basically what was anyone else's analysis of this quote  :)
Title: Re: VCE English Question Thread
Post by: friedchromosome on January 07, 2016, 05:28:17 pm
In regard with English orals and chosen topics, whats a unique way to starting and grabbing the audience attention, aside from rhetorical questions or anecdotes?
Title: Re: VCE English Question Thread
Post by: heids on January 07, 2016, 06:07:20 pm
In regard with English orals and chosen topics, whats a unique way to starting and grabbing the audience attention, aside from rhetorical questions or anecdotes?

I'd say don't start with rhetorical questions unless it's an especially especially grabbing or unusual question - but I'd then recommend googling how to start speeches. There's HEAPS of speechwriting resources out there that you can find in ten seconds that are super relevant :D
Title: Re: VCE English Question Thread
Post by: HighTide on January 07, 2016, 08:16:06 pm
In regard with English orals and chosen topics, whats a unique way to starting and grabbing the audience attention, aside from rhetorical questions or anecdotes?
What's your speech about? and what's your persona? I reckon those both come into play haha. Take into account most people will tend to start with such flare that when they set out their arguments it seems like they've only perfected the start.
Title: Re: VCE English Question Thread
Post by: pi on January 07, 2016, 08:23:10 pm
In regard with English orals and chosen topics, whats a unique way to starting and grabbing the audience attention, aside from rhetorical questions or anecdotes?

For my English orals:
1) A quote, preferably from someone famous or relate-able:
Quote
Poet-actor, Stan Lee, once said: ‘with great powers comes great responsibilities’.

On the night of 21st of August, this power and its associated responsibilities, were put in the hands of five men - 4 independents and 1 Green, as we stared down the barrel of the first hung federal parliament since the early 1940s.

...

2) Paint a picture
Quote
Imagine a world where governments lie to their citizens; a world where governments secretly conduct illegal activities, and a world where these actions occur on the false premise that it is all for your own protection. Well, that’s what our world is turning into.

And all that’s between us, and the lies, hypocrisy and deception of those in power are ‘two’ things- the moral conscience, often non-existent in the political and corporate world, and Wikileaks.

...

Regardless of how you start, if your confidence is what matters most. If you're not confident even the best speech won't win you the audience. Other important things include appropriation of your voice (volume, tone, pace), use of hand gestures, not appearing to be reading the speech, etc.
Title: Re: VCE English Question Thread
Post by: friedchromosome on January 08, 2016, 12:06:34 pm
For my English orals:
1) A quote, preferably from someone famous or relate-able:
2) Paint a picture
Regardless of how you start, if your confidence is what matters most. If you're not confident even the best speech won't win you the audience. Other important things include appropriation of your voice (volume, tone, pace), use of hand gestures, not appearing to be reading the speech, etc.
Agreed, thanks. Would it also be okay to start with a real story that was featured in a recent article? For example displaying a photo and speaking of a patient that has recovered from medical marijuana recently, then proceed with introducing the topic and my stance?
Title: Re: VCE English Question Thread
Post by: Swagadaktal on January 08, 2016, 12:17:43 pm
Hey guys, I'm struggling to pick out an English oral topic.
I originally decided to do it on public school vs private schools, and the impact of families on a student's education... but after reaching near write-up stage I had to finally admit that the issue was not contentious enough.
So I'mback to stage 1.
I'm really knowledgeable on IS and terrorist organisations... but like that's too mainstream and I don't think it's a good topic for me to write an oral on (I'm predicting half my cohort will do this topic :P)
So... any ideas for topics? The issue im leaning towards is Indigenous deaths in custody.. but tbh I have no knowledge on this issue.
What would be the arguments on both sides for this issue?
Thanks :D 
Title: Re: VCE English Question Thread
Post by: Alter on January 08, 2016, 01:25:33 pm
Hey guys, I'm struggling to pick out an English oral topic.
I originally decided to do it on public school vs private schools, and the impact of families on a student's education... but after reaching near write-up stage I had to finally admit that the issue was not contentious enough.
So I'mback to stage 1.
I'm really knowledgeable on IS and terrorist organisations... but like that's too mainstream and I don't think it's a good topic for me to write an oral on (I'm predicting half my cohort will do this topic :P)
So... any ideas for topics? The issue im leaning towards is Indigenous deaths in custody.. but tbh I have no knowledge on this issue.
What would be the arguments on both sides for this issue?
Thanks :D 
It doesn't really matter if the context/issue of your oral is something that others do. You shouldn't let this influence your decision, as you can always stand out from the crowd. Sometimes, doing these topics is a better idea because there's more room for putting a unique spin on it. If you're interested in the topic and it has garnered enough media attention, try it out. You won't be disadvantaged for doing a similar topic to others because you can't control what other people do. It's just about how well you deliver your own one.
Title: Re: VCE English Question Thread
Post by: Swagadaktal on January 08, 2016, 01:30:24 pm
It doesn't really matter if the context/issue of your oral is something that others do. You shouldn't let this influence your decision, as you can always stand out from the crowd. Sometimes, doing these topics is a better idea because there's more room for putting a unique spin on it. If you're interested in the topic and it has garnered enough media attention, try it out. You won't be disadvantaged for doing a similar topic to others because you can't control what other people do. It's just about how well you deliver your own one.
yeah that's true... but the way I view it is that I can do a good job with the over-used topic and get an 8/10.. or I can do the same quality work on a unique issue, and because this issue isn't used often my ideas come out as especially unique and allows me to stand out more than I would with the other
Title: Re: VCE English Question Thread
Post by: tashhhaaa on January 08, 2016, 10:48:55 pm
yeah that's true... but the way I view it is that I can do a good job with the over-used topic and get an 8/10.. or I can do the same quality work on a unique issue, and because this issue isn't used often my ideas come out as especially unique and allows me to stand out more than I would with the other

you should do the topic you're most interested in -- you'll be naturally passionate about it which will shine through the delivery of your speech & it will also allow your writing to have a 'voice'
Title: Re: VCE English Question Thread
Post by: Bluesy123 on January 09, 2016, 12:43:34 am
you should do the topic you're most interested in -- you'll be naturally passionate about it which will shine through the delivery of your speech & it will also allow your writing to have a 'voice'

I agree with Swagadaktal. From experience, in public speaking competitions and last years oral SAC you don't necessarily have to argue a topic that you are passionate about. One of the fundamental aims of this SAC is to convince your audience to concur with your view - persuasion is a skill, a skill that can be learned. The more you practice a speech, practice intonation the more convincing you'll sound.
Even though teachers may say otherwise, the more a topic is used the harder it is to standout and engage your audience (unless it outshines the other speeches on the same topic) and subliminally even, the teacher will be not as intrigued in your arguments (most of which are likely to have been covered by other students) than if you chose a unique topic with fresh arguments and ideas.
Ultimately, no matter what topic you choose you should be able to imbue it with a 'voice' however vague and abstract that sounds.
In my opinion, choose a speech topic that your audience may not have even considered - get them thinking.
Good luck
Title: Re: VCE English Question Thread
Post by: tashhhaaa on January 09, 2016, 03:13:47 pm
I agree with Swagadaktal. From experience, in public speaking competitions and last years oral SAC you don't necessarily have to argue a topic that you are passionate about. One of the fundamental aims of this SAC is to convince your audience to concur with your view - persuasion is a skill, a skill that can be learned. The more you practice a speech, practice intonation the more convincing you'll sound.
Even though teachers may say otherwise, the more a topic is used the harder it is to standout and engage your audience (unless it outshines the other speeches on the same topic) and subliminally even, the teacher will be not as intrigued in your arguments (most of which are likely to have been covered by other students) than if you chose a unique topic with fresh arguments and ideas.
Ultimately, no matter what topic you choose you should be able to imbue it with a 'voice' however vague and abstract that sounds.
In my opinion, choose a speech topic that your audience may not have even considered - get them thinking.
Good luck

good point, however I was merely making a suggestion based on what worked for me in VCE and through high school. Surely Swagadaktal will make the right choice :)
Title: Re: VCE English Question Thread
Post by: Swagadaktal on January 09, 2016, 03:42:04 pm
ok guys, thanks for the responses (love you all xx) : an update, I am doing my oral on the way the government deals with indigenous disadvantages.
My contention: The government is not taking enough action to support the indigenous community. So far my body paragraphs go as follows: B1: indigenous deaths in custody - An aboriginal person is 20 times more likely to be imprisoned than non-aboriginal people, and the chances of them dying in custody is significant - The government has cut fundings to the Aboriginal Legal service which attributes to these deaths as they dont have proper representation.
B2: The government is only now considering to represent the indigenous in the constitution. - 2 points to this A) the government has waited too long to do this, B) It's not really achieving much
B3) Ways to improve treatment of our indigenous
Thoughts? ilys <3 you can pm me if you want to go more in-depth and stuff
thanks guys :D
Title: Re: VCE English Question Thread
Post by: tashhhaaa on January 09, 2016, 03:48:15 pm
ok guys, thanks for the responses (love you all xx) : an update, I am doing my oral on the way the government deals with indigenous disadvantages.
My contention: The government is not taking enough action to support the indigenous community. So far my body paragraphs go as follows: B1: indigenous deaths in custody - An aboriginal person is 20 times more likely to be imprisoned than non-aboriginal people, and the chances of them dying in custody is significant - The government has cut fundings to the Aboriginal Legal service which attributes to these deaths as they dont have proper representation.
B2: The government is only now considering to represent the indigenous in the constitution. - 2 points to this A) the government has waited too long to do this, B) It's not really achieving much
B3) Ways to improve treatment of our indigenous
Thoughts? ilys <3 you can pm me if you want to go more in-depth and stuff
thanks guys :D

sounds great!
definitely interested -- during all my years of doing and listening to oral presentations, I have never heard or presented a speech on this topic!
Title: Re: VCE English Question Thread
Post by: Swagadaktal on January 09, 2016, 03:53:50 pm
sounds great!
definitely interested -- during all my years of doing and listening to oral presentations, I have never heard or presented a speech on this topic!
aha that's really reassuring
thanks :D
Title: Re: VCE English Question Thread
Post by: Pawnpusher on January 09, 2016, 04:01:07 pm
I did my oral on footy helmets on why they shouldnt be allowed, i dont even know if that was a contentious issue.. meh teacher liked it...everyone else was asleep..
Title: Re: VCE English Question Thread
Post by: vor0005 on January 09, 2016, 04:07:09 pm
ok guys, thanks for the responses (love you all xx) : an update, I am doing my oral on the way the government deals with indigenous disadvantages.
My contention: The government is not taking enough action to support the indigenous community. So far my body paragraphs go as follows: B1: indigenous deaths in custody - An aboriginal person is 20 times more likely to be imprisoned than non-aboriginal people, and the chances of them dying in custody is significant - The government has cut fundings to the Aboriginal Legal service which attributes to these deaths as they dont have proper representation.
B2: The government is only now considering to represent the indigenous in the constitution. - 2 points to this A) the government has waited too long to do this, B) It's not really achieving much
B3) Ways to improve treatment of our indigenous
Thoughts? ilys <3 you can pm me if you want to go more in-depth and stuff
thanks guys :D

I'd save Body 3 for the "solutions" section of your oral (towards the end).

Instead, consider a rebuttal for your third body. For example: the government deems spending more money on the indigenous peoples as a waste - then, shut it down with evidence that justifies further expenditure on aboriginals (i.e. the irreversible hurt they incurred in the past, stolen generation, poverty gap etc.)
Title: Re: VCE English Question Thread
Post by: Swagadaktal on January 09, 2016, 06:50:33 pm
I'd save Body 3 for the "solutions" section of your oral (towards the end).

Instead, consider a rebuttal for your third body. For example: the government deems spending more money on the indigenous peoples as a waste - then, shut it down with evidence that justifies further expenditure on aboriginals (i.e. the irreversible hurt they incurred in the past, stolen generation, poverty gap etc.)
Um how about this, " Critics argue that plenty of government resources, time and effort has been placed in hopes of helping the indigenous community, but the community is not willing to change." - then rebut it using my solution.
Which is to give more jobs to indigenous community, and reflecting on methods proven by other governments (inbox me if you want the whole thing, not sure about how much detail you guys want in order to give me feedback :P)
Title: Re: VCE English Question Thread
Post by: Swagadaktal on January 09, 2016, 06:55:23 pm
Um how about this, " Critics argue that plenty of government resources, time and effort has been placed in hopes of helping the indigenous community, but the community is not willing to change." - then rebut it using my solution.
Which is to give more jobs to indigenous community, and reflecting on methods proven by other governments (inbox me if you want the whole thing, not sure about how much detail you guys want in order to give me feedback :P)
OOh a critical point I left out, is that although the government is providing resources, they are not utilising the resources provided effectively. We need to use the same resources in different ways
Title: Re: VCE English Question Thread
Post by: bts on January 11, 2016, 01:22:43 pm
hey i was wondering if anyone could give me a suggestion on how to improve this paragraph topic sentence. it seems quite dull for a topic sentence... its about using weapons especially in government for power.

'Despite the common use of weapons to settle disputes, there is a diverse range of other alternatives to combat problems and successfully govern. Weapons are often resorted to, primarily for the induced fear towards victims...'

Any help would be appreciated :)
Title: Re: VCE English Question Thread
Post by: Swagadaktal on January 11, 2016, 01:50:34 pm
hey i was wondering if anyone could give me a suggestion on how to improve this paragraph topic sentence. it seems quite dull for a topic sentence... its about using weapons especially in government for power.

'Despite the common use of weapons to settle disputes, there is a diverse range of other alternatives to combat problems and successfully govern. Weapons are often resorted to, primarily for the induced fear towards victims...'

Any help would be appreciated :)
Weapons are often prioritised by governments over the wide range of alternatives due to their coercive nature.
Does this work? I'm no English genius just had a go :P
Title: Re: VCE English Question Thread
Post by: bts on January 11, 2016, 01:57:47 pm
thank you Swagadaktal for your suggestion! i'll give it a shot and see if it fits :)
Title: Re: VCE English Question Thread
Post by: Champ101 on January 11, 2016, 02:07:52 pm
hey i was wondering if anyone could give me a suggestion on how to improve this paragraph topic sentence. it seems quite dull for a topic sentence... its about using weapons especially in government for power.

'Despite the common use of weapons to settle disputes, there is a diverse range of other alternatives to combat problems and successfully govern. Weapons are often resorted to, primarily for the induced fear towards victims...'

Any help would be appreciated :)

Weapons - cold, mechanical instruments designed to inflict harm by those who control them. Unfortunately, in many cases, the guise of defence conceals the responsibility of those who yield such instruments, responsibility which is ultimately abrogated in the pursuit of power, control and intimidation - a propensity which thrives off the innocence and acquiescence of those they're used to protect.
Title: Re: VCE English Question Thread
Post by: Swagadaktal on January 11, 2016, 02:14:07 pm
Weapons - cold, mechanical instruments designed to inflict harm by those who control them. Unfortunately, in many cases, the guise of defence is purported while the responsibility of those who yield such instruments is unnervingly abrogated in the pursuit of power, control and intimidation - a propensity which thrives off the innocence and acquiescence of those their used to protect.
I actually had to google 3 of those words.. damn
Title: Re: VCE English Question Thread
Post by: Pawnpusher on January 11, 2016, 02:19:40 pm
I actually had to google 3 of those words.. damn

Same :') i always got told that examiners dont like using their dictionaries.. cant argue with a 50 tho
Title: Re: VCE English Question Thread
Post by: Swagadaktal on January 11, 2016, 02:20:52 pm
Same :') i always got told that examiners dont like using their dictionaries.. cant argue with a 50 tho
is that what it takes to get a 50?
coz my vocab isnt that deep its just good enough for l.a and text responses :3
Title: Re: VCE English Question Thread
Post by: keltingmeith on January 11, 2016, 02:23:24 pm
is that what it takes to get a 50?
coz my vocab isnt that deep its just good enough for l.a and text responses :3

Man, don't stress - you have a whole year still to develop a deeper vocabulary.
Title: Re: VCE English Question Thread
Post by: Champ101 on January 11, 2016, 02:40:24 pm
is that what it takes to get a 50?
coz my vocab isnt that deep its just good enough for l.a and text responses :3

I wouldn't worry too much about your vocabulary, it is important but focus on the complexity of your ideas first and how clearly and succinctly you convey those ideas. Your vocabulary will improve naturally the more you read and the more you write. Just look out for 'uncommon' words and try use them in your pieces of writing. As Euler said it's only the start of the year.
Title: Re: VCE English Question Thread
Post by: appleandbee on January 11, 2016, 02:54:13 pm
is that what it takes to get a 50?
coz my vocab isnt that deep its just good enough for l.a and text responses :3

Your vocabulary doesn't need to be pretentious or overly deep, it justs needs to be sophisticated. Also LA vocalubary doesn't need to be that sophisticated, you just need to explain your points well and know your terminology. I think there is a stickied post by DJA on TR expression on the English boards.

As for text response vocabulary, read high-scoring essays and other stuff to learn new words (write the words in a notebook and learn how to use them in a sentence). Also read study guides (even as someone who detests text study guides, I must say VATE is really good and Insight is okay) and literary commentary (these are the best but they are more common for older texts like Wuthering Heights and Shakespeare, you may be able to find some on Academia.com and Paris Review) on your selected texts for specific vocab and terminogy that complements your text.

No 50er but I got 20/20 for text response in the exam
Title: Re: VCE English Question Thread
Post by: literally lauren on January 11, 2016, 02:59:29 pm
Hey guys, just letting you know that the first post in this thread has been updated to include more answers to common areas of concern, and they've now been sorted within each section, so you can browse through specific sub-categories within 'Text Response', 'Context' etc.

Hopefully this'll make things more navigable so that you can browse through past responses and then ask additional questions if you need. As always, if there's anything English-y that you need clarification with (for a pre-existing or entirely new question,) then drop it in this thread and someone'll help you out :)

(The English Q and A thread turns one year old today :'D they grow up so fast!)
Title: Re: VCE English Question Thread
Post by: heids on January 11, 2016, 03:40:15 pm
Meaning no offence to Champ101, I reckon the original TS was better, simply because it was simpler and clearer.  I wouldn't recommend trying to imitate that complex/wordy style because it sounds just a touch pretentious and confusing (I explain my reasoning in detail here).  But I got the 43 and they got the 50, so :P

I liked Swagadaktal's though - very clear, crisp and to the point. (y)
Title: Re: VCE English Question Thread
Post by: Swagadaktal on January 11, 2016, 03:42:30 pm
Meaning no offence to Champ101, I reckon the original TS was better, simply because it was simpler and clearer.  I wouldn't recommend trying to imitate that complex/wordy style because it sounds just a touch pretentious and confusing (I explain my reasoning in detail here).  But I got the 43 and they got the 50, so :P

I liked Swagadaktal's though - very clear, crisp and to the point. (y)
omg bangali stop that sweet talk of yours, if I was a diabetic I wouldn't be able to handle you
Title: Re: VCE English Question Thread
Post by: heids on January 11, 2016, 03:46:41 pm
omg bangali stop that sweet talk of yours, if I was a diabetic I wouldn't be able to handle you

Remember that I just criticised someone else (soz), so it's clearly not sugary flattery :P

(still undecided about whether you were complimenting me or not LOL)
Title: Re: VCE English Question Thread
Post by: Swagadaktal on January 11, 2016, 03:50:15 pm
Remember that I just criticised someone else (soz), so it's clearly not sugary flattery :P

(still undecided about whether you were complimenting me or not LOL)
It's a compliment boo :P
Title: Re: VCE English Question Thread
Post by: Splash-Tackle-Flail on January 11, 2016, 04:15:59 pm
I actually had to google 3 of those words.. damn

Don't worry I had to google 3 of them too!
Title: Re: VCE English Question Thread
Post by: huehue on January 11, 2016, 05:38:56 pm
Hello everyone! Can someone please tell me how language analysis is changing next year? I've been told that it's more argument focused but not sure what that means. Thanks :)
Title: Re: VCE English Question Thread
Post by: teacher28 on January 12, 2016, 11:38:09 am
I've been told that it's more argument focused but not sure what that means. Thanks :)


It's a subtle though significant change that the assessment commentary has been hinting at for a couple of years now making reference to aspects such as the logic or reason behind the argument. Remember that VCAA is a fickle creature!  What was enough last year or the year before may be simply the foundation in 2016.

This suggests that analysis is now more focused on analysing HOW the 'language' (words, visual texts, syntax, order of supporting points, for example) attempts to elicit THE AUDIENCE'S ACCEPTANCE of and or AGREEMENT with the SUPPORTING ARGUMENTS and therefore the CONTENTION.

This approach shifts attention away from simple investigation of what devices are being used, how  and why, and places the focus on a holistic view.

To achieve this come back to the over-riding point of view a couple of times through your analysis and make further reference to the supporting arguments that work to buttress the contention using phrases such as "highlights the point that...", "reinforces the main point of view expounded by ..." or "underlines the main argument that...".

Title: Re: VCE English Question Thread
Post by: teacher28 on January 12, 2016, 12:13:13 pm
Weapons - cold, mechanical instruments designed to inflict harm by those who control them. Unfortunately, in many cases, the guise of defence conceals the responsibility of those who yield such instruments, responsibility which is ultimately abrogated in the pursuit of power, control and intimidation - a propensity which thrives off the innocence and acquiescence of those their used to protect.

Like bangali_lok, I mean no offence either, Champ 101, but I agree with the point that you can propose a sophisticated idea without using too many BIG words. I also agree that while most teachers welcome the opportunity to expand their vocabulary, examiners do not want to be hitting the dictionary  every few seconds.

When so-called sophisticated word-use turns elegance into bling a strict paring down and tightening up is called for. If you were one of my students I would have edited thus:  

"...responsibility which is ultimately destroyed by the desire to control and intimidate -- a tendency which thrives on the weakness of the  innocent and submissive whom the powerful have a duty to protect."

(...those their used to protect.")

I do, however, like the innovative syntax of your introductory sentence!

"Weapons -- cold, mechanical instruments designed to inflict harm by those who control them."
Title: Re: VCE English Question Thread
Post by: superalexnguyen on January 22, 2016, 10:52:56 pm
for character based prompts in text reponse, If the prompt only mentions and focuses on one character, are we allowed to talk about other characters, or does it have to specifically answer the prompt only.
Title: Re: VCE English Question Thread
Post by: Alter on January 22, 2016, 10:56:10 pm
It depends a bit on the prompt, I suppose. However, it's normally perfectly fine to talk about other characters or else your discussion may be severely limited. For example, you might explain how a character's relationship or partnership with another's evolved over time, and what this tells us about the book. It could also depend on what you're studying, too. Some texts have few major characters (e.g. Mabo), whereas others are filled to the brim with many characters that are interrelated (such as Stasiland). This might mean you have more or less room to facilitate a discussion on more than just the character given in the prompt.
Title: Re: VCE English Question Thread
Post by: geminii on January 23, 2016, 01:28:27 am
Hey everyone,

For Year 12 English in 2017, I know that there will be no context essay for the end of year exam. Anyone have any idea what it is being replaced by for the exam?

Thanks :)
Title: Re: VCE English Question Thread
Post by: Cornrow Kenny on January 23, 2016, 01:57:35 am
Hey everyone,

For Year 12 English in 2017, I know that there will be no context essay for the end of year exam. Anyone have any idea what it is being replaced by for the exam?

Thanks :)
IIRC it will be some sort of creative writing task.
Title: Re: VCE English Question Thread
Post by: Alter on January 23, 2016, 07:16:55 am
In the English exam the context piece will be replaced by a currently unnamed task in which you have to compare two different texts and analyse them together eg Stasiland and 1984. It's a bit more reminiscent of a text response but you're looking at more than one book afaik.

Lauren made a post on it somewhere but I can't seem to find it at the moment. More information is also available on the VCAA website if you search in the English study designs.

There is also a compulsory creative SAC during the year, which is what the above poster may be thinking of.
Title: Re: VCE English Question Thread
Post by: superalexnguyen on January 23, 2016, 01:25:26 pm
Regarding the TEEL formula in text response, what exactly am i expanding/ explaining ?
Title: Re: VCE English Question Thread
Post by: Alter on January 23, 2016, 02:24:41 pm
hi superalexnguyen,

At this stage of your paragraph, you'll be explaining and expanding on the general idea or premise that you brought up in your topic sentence. It can also mean explaining the evidence preceding the sentence. This means that the "E" will be an extension of what you've already written; you're taking your logic a step further and explaining the impact or intricacies of your initial statement to the reader. Don't feel obliged to very strictly follow this format, however. English is very flexible and there's no definitive way of setting out every paragraph. That being said, I'll show you what I mean with an example, because such a structure is a good way of ensuring your essays are coherent.

Let's say I'm writing an essay on Stasiland and the prompt asks me to discuss the theme of absurdity. I plan out my essay and this is for the first body paragraph that I write.

Quote from: Topic Sentence
In Stasiland, the theme absurdity is used to highlight the ruthless and fundamentally sinister nature of the GDR's secret police.

Above is the topic sentence of my paragraph. I've got a central idea that Anna Funder employs a theme in order to make a particular point about what she thought. However, as you see it right now, I've only got a very limited amount of information. I haven't actually explained what this means and I haven't given evidence. Below is an example of what a sentence might look like explaining this idea.

Quote from: Explanation
Funder notes that, "despite his family history", Torsten was still sought after by the Stasi to see if "he would inform for them". This preposterous notion that a victim of the Stasi himself would agree to working for them illustrates how irrespective of an individual's experiences with the state, the Stasi desired total control. Ergo, they preyed on the most unlikely candidates to spy for them. In this way, Funder suggests that the Stasi were completely unforgiving when it came to achieving their set goals.

Using this explanation, I've given reason for why I've chosen my idea that was in the first sentence of the paragraph. At the start, I brought up my evidence, and then I've gone on to explain my evidence after that. Hope this makes sense.
Title: Re: VCE English Question Thread
Post by: superalexnguyen on January 23, 2016, 02:44:18 pm
hi superalexnguyen,

At this stage of your paragraph, you'll be explaining and expanding on the general idea or premise that you brought up in your topic sentence. It can also mean explaining the evidence preceding the sentence. This means that the "E" will be an extension of what you've already written; you're taking your logic a step further and explaining the impact or intricacies of your initial statement to the reader. Don't feel obliged to very strictly follow this format, however. English is very flexible and there's on definitive way of setting out every paragraph. That being said, I'll show you what I mean with an example, because such a structure is a good way of ensuring your essays are coherent.

Let's say I'm writing an essay on Stasiland and the prompt asks me to discuss the theme of absurdity. I plan out my essay and this is for the first body paragraph that I write.

Above is the topic sentence of my paragraph. I've got a central idea that Anna Funder employs a theme in order to make a particular point about what she thought. However, as you see it right now, I've only got a very limited amount of information. I haven't actually explained what this means and I haven't given evidence. Below is an example of what a sentence might look like explaining this idea.

Using this explanation, I've given reason for why I've chosen my idea that was in the first sentence of the paragraph. At the start, I brought up my evidence, and then I've gone on to explain my evidence after that. Hope this makes sense.


thank you!,
Title: Re: VCE English Question Thread
Post by: Cornrow Kenny on January 24, 2016, 12:18:32 am
There is also a compulsory creative SAC during the year, which is what the above poster may be thinking of.
You're probably right haha! Doesn't effect me really  :P
Title: Re: VCE English Question Thread
Post by: Sine on January 28, 2016, 03:59:45 pm
What intermediate steps should I take between reading the texts and writing essays?
Title: Re: VCE English Question Thread
Post by: Alter on January 28, 2016, 04:19:04 pm
What intermediate steps should I take between reading the texts and writing essays?
Build your own interpretation of the text
A lot of people seem to overlook the "response" part of text response. It is all too easy for people to view the task as simply regurgitating essays using ideas given by their teachers or those they stole from text guides. You should try to avoid this trap by building your own opinions and attitudes towards what happened in the text. This is particularly important for books where you're dealing with serious or typically taboo themes. In all honesty, this is something you may do subconsciously to a very great extent and you would have likely also done it while reading the text. Either way, by building your own interpretation, you begin forming ideas and conceptions about how the piece works as a whole to achieve a particularly meaning or meanings.

So why is this so important? Well, above all, by building your own interpretation of the text, you will ensure that the essays you write in the future will have coherent and consistent arguments. If you were to simply amalgamate a bunch of different ideas that your teacher or classmates have mentioned in passing, your response may seem like it lacks consistency in interpretation. On the other hand, if you have your own opinion on which characters are "good" or "bad" (for example), you can begin to formulate your own answers as to what views and values you think the author or director holds.

Construct a quote bank
This will be a massive help for you when writing essays. It goes without saying that you won't be able to write a great English Text Response without having evidence to back up your points. As a result, at one stage or another you'll need to learn quotes that you can use to support the ideas that you will argue for. Similarly, if you're doing a movie such as Mabo, you should gain an understanding of the filmic devices that occur, when they occur, and why they are used. This can go hand in hand with the next step (sorting quotes by themes), or you could even organise it by character or, at the very least, chronologically.

Establish the main themes of the text
Out of all of the prompts that can appear for your text, you'll more often than not be able to link the question with one or two major themes of the text. If you've already gone through all of these yourself, you're saving yourself a lot of trouble trying to decipher essay questions in the future. Once you've done this, you can ask yourself "what is the author of the text trying to say about theme xyz?"

Practise planning essays
Grab a huge list of potential questions that you can get for your TR and go through them one-by-one. I liked to highlight the ones I thought were simple in one colour and the more difficult ones in another. Try to pick some pieces of evidence that you could use for each question/prompt so that you will quickly be able to recall quotes etc. when you're doing the real task. Moreover, it's important you understand which questions you find difficult because it's likely that they will all share a common thread. If you can identify where the problem lies when you're doing prompts, you'll be happy that you didn't have to write out 5-6 paragraphs just to understand you don't get what's going in with a particular character.
Title: Re: VCE English Question Thread
Post by: tashhhaaa on January 29, 2016, 02:01:54 am
Build your own interpretation of the text

In my humble opinion, this is a must. No getting around this -- if you want to write well and do well, you need to think for yourself
Title: Re: VCE English Question Thread
Post by: Swagadaktal on February 01, 2016, 04:06:35 pm
Hey guys, need some feedback on my oral presentation, before i redraft my oral i want to get feedback on my current one.
It's on how the government deals with indigenous disadvantage
pm if you want to help me out :)
thanks
ilys
Title: Re: VCE English Question Thread
Post by: thepandaman on February 01, 2016, 05:48:45 pm
Hi guys i need some help with my oral regarding formulating some solid arguments to back up my contention. My topic is child vaccination laws and to give you some background info, the government is basically going to be restricting parents from receiving government child benefits if they do not get there children immunized.

My contention is that instead of penalizing parents, we should focus on the real roots of the issue which is the lack of education and misinformation people are fed when it comes to vaccinations. The part I am struggling with is to come up with 3 solid arguments.

So far I have something along the lines of 1. parents need more education so that there misconceptions can be corrected. 2. There needs to be a ban on over exaggerated stories of the rare side effects of vaccinations as this simply feeds more misinformation to the public. I am not sure what my third argument should be and also not sure how I can clearly distinguish my first two arguments from one another since they are quite similar. Would appreciate any help or even suggestions on bettering my first 2 arguments because I don't feel too confident in them :(
Title: Re: VCE English Question Thread
Post by: qazser on February 02, 2016, 10:13:38 pm
Your Thoughts on Prison Reform

Current ideas
-rehabilitation in prisons over punishment to reduce recidivism rates (Using Norway as an example comparing to US and Aust)
- Solitary confinement in prisons resulting in mental illnesses

Title: Re: VCE English Question Thread
Post by: literally lauren on February 04, 2016, 11:13:50 am
Hi guys i need some help with my oral regarding formulating some solid arguments to back up my contention. My topic is child vaccination laws and to give you some background info, the government is basically going to be restricting parents from receiving government child benefits if they do not get there children immunized.

My contention is that instead of penalizing parents, we should focus on the real roots of the issue which is the lack of education and misinformation people are fed when it comes to vaccinations. The part I am struggling with is to come up with 3 solid arguments.

So far I have something along the lines of 1. parents need more education so that there misconceptions can be corrected. 2. There needs to be a ban on over exaggerated stories of the rare side effects of vaccinations as this simply feeds more misinformation to the public. I am not sure what my third argument should be and also not sure how I can clearly distinguish my first two arguments from one another since they are quite similar. Would appreciate any help or even suggestions on bettering my first 2 arguments because I don't feel too confident in them :(
Your contention is solid, but I'd agree that you need to broaden your arguments a bit more.

The first is decent, but it might be a little bit too similar to your actual contention. For instance, if I was arguing that libraries should be open 24/7, and my first 'sub-argument' was that opening libraries 24/7 would be very beneficial for people... I'm not really being more specific there, I'm just reconfiguring my broader point.

So you've stated your contention is that a lack of education and the perpetuation of misinformation is the real problem surrounding vaccinations, and your first sub-argument is that people need to be educated to correct this misinformation. See how they're a bit too similar?

Also, I'm not sure your second argument is as strong as it could be. Arguing that the media should outright ban covering certain stories doesn't seem very justifiable - it'd probably be more effective to say that media coverage needs to be more balanced, and geared towards informing the public about the issues surrounding vaccination rather than alarming them. That way you can still use similar points, but your overall sub-contention will hopefully be a little less jarring than advocating for a less-than-free press.

In order to come up with sub-arguments, you're going to want to consider the 'key players' = people/things/ideas related to your issue that you want to position in a certain way.

For instance:
   Contention: there needs to be more education to ensure that the public is not misinformed about vaccinations, and that parents can make the right decision and vaccinate their children.

If I want you to believe that vaccinations are a good thing and that people who protest or refuse them are misinformed (excepting those who do so on religious or health grounds, of course) then I'd want to position the kinds of people who perpetuate myths and twisted facts as dangerous predators who exploit parents' love for their children. So to strengthen this contention, we need to construct a sub-argument surrounding how we want to portray this key player.

If our KP here is 'the people and groups who deliberately and maliciously misinform people about vaccinations, or who grossly overestimate the potential risks,' then that section of your speech would be devoted to portraying them as the most sinister, greedy, careless kinds of scum imaginable. Heap on the rhetorical devices and emphasis, and make your audience believe what you want them to believe about anti-vaccination groups and biased media sources.

Because if they believe what you say about the key player, it should make it much easier for you to convince them of your contention.

And you can consider the many other key players that might be relevant for this issue. For example, how do you want people to view:
- vaccinations and their benefits/risks
- parents and their duty to their children
- children's health and well-being
- the government's responsibility to protect its citizens
- the media and their accuracy or intentions
- the skepticism and denial leveled against vaccinations
- people's freedom to make their own decisions
- the value and power of education and information

etc.

You don't have to touch on all these key players, and you might end up grouping some into collective sub-arguments, but hopefully thinking about these strategic portrayals and depictions will give you a little bit more direction in your speech writing.

Having said all that, there's no real reason why you need to have three distinct sub-arguments unless that's something your teacher/school mandates. It's okay to just get up and talk about the issue, so long as what you say is persuasive. And it's fine if your arguments link together; in fact, they should do this! If your arguments are too disconnected then it makes it more difficult to tie things together on a contention-level.

The flow of your speech is more important than how clearly you demarcate your sub-arguments though, so see if you can just expand upon your ideas without the rigid framework of 'Argument 1 = ...' and so on. But if you do prefer/need to use the sub-argument structure, then have a think about the positioning of your key players to give you some direction.

Your Thoughts on Prison Reform

Current ideas
-rehabilitation in prisons over punishment to reduce recidivism rates (Using Norway as an example comparing to US and Aust)
- Solitary confinement in prisons resulting in mental illnesses
What kind of prison reform are you looking at exactly? Is this specifically in relation to Australian prisons, and if so, what proposed reform are you considering? US prisons are a horse of an entirely different and more problematic colour, so comparing the two is a little difficult.

Ultimately these debates tend to come down to what you believe prison should accomplish, and whether the priority should be the reform of the individual, or the safety of society, or the sense of justice in 'making the punishment fit the crime,' or the notion of 'setting an example' by doling out lengthier sentences (--> link to some of the recent terror threats and their sentencing,) or whether it's a mix of all of these and more.

But the first step is to be more specific if you're looking to iron out the details - it's one thing to say 'prisons need reform' but quite another to be able to stipulate how and why.
Title: Re: VCE English Question Thread
Post by: knightrider on February 04, 2016, 06:04:05 pm
Generally speaking how many quotes are we allowed to put per paragraph in a text response ?

Like my teacher says no more than 3 to 4 per paragraph.

What do VCAA examiners accept.

Thanks  :)
Title: Re: VCE English Question Thread
Post by: Alter on February 04, 2016, 06:59:48 pm
There's no perfect/set amount of quotes you need. About four sounds pretty good as a rule of thumb, but you could arguably fit a few more depending on how well you embed your quotes within your response. Just don't add quotes for the sake of adding them, and try to actually justify your mentioning of them in the essay. If you were to put a bunch of quotes throughout section A without ever explaining their significance whatsoever, you may as well not put any at all (which is a bad idea for obvious reasons).
Title: Re: VCE English Question Thread
Post by: Hasti on February 04, 2016, 10:14:07 pm
There's no one that can definitively tell you how many quotes to actually have in each paragraph! Generally, it's better to have more than one, but less than 6 (or depending on how big your paragraph is, more or less). Even still, you have complete agency over how many quotes you want to include. The point is that if you want one quote, you've got to analyse it soooooooo much that there's literally no more analysis left to be done. If you do this, you're proving how amazing you are at deep and insightful analysis - THIS IS GOOD.
But at the same time you want to show your knowledge of the text you're studying, so inherently you'd want to write a couple of techniques or quotes for each theme/paragraph you write. This is where you take control of your essay. Write as many quotes as you feel will prove whatever point you are trying to make.

BUT in saying this, if you've got 17 quotes (hypothetically), you have the physical space of about 5 words per quote to get your analysis done. If you cram your paragraph with quotes, you've got no room for deep analysis and explanation. Essentially, you must be the final judge and decide whether the amount of analysis and quotes per paragraph is enough to prove your argument - not just enough, but totally sufficient. Leave no room for doubt - GENERALLY, as a totally basic, average, 'guesstimation' of how many quotes students have per paragraph is 4 or 5. But like I said, you can stop at 1 quote if you feel that that one quote is the best piece of evidence for your argument and you've analysed it 100%.

Less is more... (if you can do it well).
Title: Re: VCE English Question Thread
Post by: knightrider on February 05, 2016, 08:29:33 am
There's no perfect/set amount of quotes you need. About four sounds pretty good as a rule of thumb, but you could arguably fit a few more depending on how well you embed your quotes within your response. Just don't add quotes for the sake of adding them, and try to actually justify your mentioning of them in the essay. If you were to put a bunch of quotes throughout section A without ever explaining their significance whatsoever, you may as well not put any at all (which is a bad idea for obvious reasons).

There's no one that can definitively tell you how many quotes to actually have in each paragraph! Generally, it's better to have more than one, but less than 6 (or depending on how big your paragraph is, more or less). Even still, you have complete agency over how many quotes you want to include. The point is that if you want one quote, you've got to analyse it soooooooo much that there's literally no more analysis left to be done. If you do this, you're proving how amazing you are at deep and insightful analysis - THIS IS GOOD.
But at the same time you want to show your knowledge of the text you're studying, so inherently you'd want to write a couple of techniques or quotes for each theme/paragraph you write. This is where you take control of your essay. Write as many quotes as you feel will prove whatever point you are trying to make.

BUT in saying this, if you've got 17 quotes (hypothetically), you have the physical space of about 5 words per quote to get your analysis done. If you cram your paragraph with quotes, you've got no room for deep analysis and explanation. Essentially, you must be the final judge and decide whether the amount of analysis and quotes per paragraph is enough to prove your argument - not just enough, but totally sufficient. Leave no room for doubt - GENERALLY, as a totally basic, average, 'guesstimation' of how many quotes students have per paragraph is 4 or 5. But like I said, you can stop at 1 quote if you feel that that one quote is the best piece of evidence for your argument and you've analysed it 100%.

Less is more... (if you can do it well).

Thanks so much for your help Alter  :) and Hasti  :)
Title: Re: VCE English Question Thread
Post by: kimmytaaa on February 12, 2016, 09:59:17 am
hi,
has anyone read the lieutenant by kate Grenville? if anyone has can you give me some external ideas that are relevant to the conflicts that arises in the novel?

Thanks
Title: Re: VCE English Question Thread
Post by: Splash-Tackle-Flail on February 12, 2016, 12:08:39 pm
hi,
has anyone read the lieutenant by kate Grenville? if anyone has can you give me some external ideas that are relevant to the conflicts that arises in the novel?

Thanks
Yea I did and used it in the exam. I can't remember everything that happens in it, but here's some ideas:

1. Sense of duty vs morality- Rooke and his fellow soldiers are embroiled in a land of rules, notions of honour and duty, and a rather emphasised hierarchy. Subordinates, such as Rooke and Gardiner, are required to follow higher orders unquestionably, and the consequences of "mutiny". These consequences also tie in ideas of fear- in the start of the book you should remember Rooke witnessing the execution of soldiers who betrayed the marine forces (I forgot what they did but you get the point).

2. It is often easier to remain a bystander- Rooke's slowly emerging moral scruples with how the governor treats not only his people, but also the Cardigal tribe are suppressed by the tempting ease of simply watching- he is not directly hurting the convict who attempted to steal potatoes, so it was much easier/less personal risk for him to just watch and make a presence. This is also quite related to the first idea- as if he encouraged dissent towards the lashings, he would certainly be punished. Contrast these first two ideas to Rooke's actions at the end of the novel, where he realises "if you were part of the machine, you were part of its wrong" (or something Idk the quote anymore-used it in the end of year though :P), and openly challenges the Governer's order to capture of slaughter 6 Aboriginal men.

3. Communication barriers- There is not only a language barrier between the settlers and the Cardigal tribe, but also a stark contrast in values and culture. This heightens the growing conflict between the two "sides" as genuine compromise and cooperation is hindered.

4. Understanding/lack of open-mindedness- the settlers in the novel are quite 'pushy' with imprinting their values on the Cardigal tribe, to the extent of kidnapping and forced immersion. They also have quite rigidly instilled prejudiced views against people of 'colour'. This creates difficulties, as there is no sense of mutual respect, instead, it's tainted cultural superiority, and a perceived justified subjugation of fellow human beings.

That's all I can think of at the moment, but hope it helps :)
Title: Re: VCE English Question Thread
Post by: kimmytaaa on February 12, 2016, 12:19:32 pm
Yea I did and used it in the exam. I can't remember everything that happens in it, but here's some ideas:

1. Sense of duty vs morality- Rooke and his fellow soldiers are embroiled in a land of rules, notions of honour and duty, and a rather emphasised hierarchy. Subordinates, such as Rooke and Gardiner, are required to follow higher orders unquestionably, and the consequences of "mutiny". These consequences also tie in ideas of fear- in the start of the book you should remember Rooke witnessing the execution of soldiers who betrayed the marine forces (I forgot what they did but you get the point).

2. It is often easier to remain a bystander- Rooke's slowly emerging moral scruples with how the governor treats not only his people, but also the Cardigal tribe are suppressed by the tempting ease of simply watching- he is not directly hurting the convict who attempted to steal potatoes, so it was much easier/less personal risk for him to just watch and make a presence. This is also quite related to the first idea- as if he encouraged dissent towards the lashings, he would certainly be punished. Contrast these first two ideas to Rooke's actions at the end of the novel, where he realises "if you were part of the machine, you were part of its wrong" (or something Idk the quote anymore-used it in the end of year though :P), and openly challenges the Governer's order to capture of slaughter 6 Aboriginal men.

3. Communication barriers- There is not only a language barrier between the settlers and the Cardigal tribe, but also a stark contrast in values and culture. This heightens the growing conflict between the two "sides" as genuine compromise and cooperation is hindered.

4. Understanding/lack of open-mindedness- the settlers in the novel are quite 'pushy' with imprinting their values on the Cardigal tribe, to the extent of kidnapping and forced immersion. They also have quite rigidly instilled prejudiced views against people of 'colour'. This creates difficulties, as there is no sense of mutual respect, instead, it's tainted cultural superiority, and a perceived justified subjugation of fellow human beings.

That's all I can think of at the moment, but hope it helps :)



thankyou
Title: Re: VCE English Question Thread
Post by: huehue on February 14, 2016, 09:52:56 am
Hello! Can someone please read over my introduction? I've tried something new here because my teacher said my introductions had way too much signposting, but I feel like it's a bit..random?? The prompt is below and my contention is that Achilles' actions represent the uncertainty of humanity in war. My two (couldn't think of three) supporting arguements are that morality is presented as lost in war and grief is presented as heightened by war.

Prompt: "The rage of Achilles and the defiant treatment of Hector's body express the essential loss of humanity in war". Discuss.

Written in the time of the Iliad, Malouf's Ransom could be seen as a modern day commentary on the effects of war on humanity. Malouf presents war as a herald for pain and anguish, with morality being lost in the chaos, but grief as only heightened and strengthened by war. The character Achilles' actions for example, represent his loss of morality, but shows the power grief ultimately has. However, his actions in the end reflect the uncertainty of humanity in war.

Thanks for all help in advance!  :) :)
Title: Re: VCE English Question Thread
Post by: jacurt on February 14, 2016, 10:14:27 am
Anyone doing Cloudstreet?
Title: Re: VCE English Question Thread
Post by: Swagadaktal on February 14, 2016, 01:37:10 pm
Hey guys, need help with All About Eve.
I'm struggling to remember the movie, so I'm going to keep watching it until I can hopefully remember it. But more importantly, how do I do more close scene analysis?
I'm trying to write an essay on the prompt "The only values to succeed in the theatre is egotism and ruthless ambition. Is this true?"
My contention is that these qualities are definitely important and can be used to be successful, however they are not the only qualities that can be used to succeed. My first argument is that other qualities such as loyalty is a requirement to succeed (also talk about what success means... coz is eve really successful?) - My second argument is that egotism is important, but only in moderation. and my third argument is that ruthless ambition alone will not make someone successful, but it must be used in conjunction with talent.

Now, I have some quotes here and there but I need to analyse more film techniques, and I need to do more close-scene analysis. But currently I'm struggling to find some...
Anyone got any ideas?
Title: Re: VCE English Question Thread
Post by: huehue on February 14, 2016, 04:01:46 pm
Hey guys, need help with All About Eve.
I'm struggling to remember the movie, so I'm going to keep watching it until I can hopefully remember it. But more importantly, how do I do more close scene analysis?
I'm trying to write an essay on the prompt "The only values to succeed in the theatre is egotism and ruthless ambition. Is this true?"
My contention is that these qualities are definitely important and can be used to be successful, however they are not the only qualities that can be used to succeed. My first argument is that other qualities such as loyalty is a requirement to succeed (also talk about what success means... coz is eve really successful?) - My second argument is that egotism is important, but only in moderation. and my third argument is that ruthless ambition alone will not make someone successful, but it must be used in conjunction with talent.

Now, I have some quotes here and there but I need to analyse more film techniques, and I need to do more close-scene analysis. But currently I'm struggling to find some...
Anyone got any ideas?

My advice would be to not jump ahead and start writing an essay, but rather analyse the film part by part first. This way you'll memorise and gain a better understanding of the film.
Title: Re: VCE English Question Thread
Post by: Sine on February 14, 2016, 04:08:28 pm
How should we structure our evidence/interpretation through close analysis of the text? By Themes or Characters or something else
Title: Re: VCE English Question Thread
Post by: Swagadaktal on February 14, 2016, 04:10:33 pm
My advice would be to not jump ahead and start writing an essay, but rather analyse the film part by part first. This way you'll memorise and gain a sound understanding of the film.
Yeah I see what you mean.
I've done a lot of reading about this film and I've been able to generate some sophisticated ideas, however I think my main problem is finding the right evidence due to me not knowing the film well enough..
But like tbh just re-watching the film over and over is waaaaaaaaaay too boring I need to write some stuff down, I need to see some tangible proof of my work. So like whilst im analysing the film part by part I find some quotes I write down with some scenes and then I go and write an essay on it. -- But the flaw in that is that im only using parts of the film in my essay and not the whole film
Title: Re: VCE English Question Thread
Post by: vor0005 on February 14, 2016, 05:13:57 pm
How should we structure our evidence/interpretation through close analysis of the text? By Themes or Characters or something else

Structure your paragraphs by arguments or ideas - then, use your evidence/interpretation to prove these arguments/ideas.
Title: Re: VCE English Question Thread
Post by: Sine on February 14, 2016, 05:16:33 pm
Structure your paragraphs by arguments or ideas - then, use your evidence/interpretation to prove these arguments/ideas.
sorry wasn't talking about essays just general notes. Thanks  :)
Title: Re: VCE English Question Thread
Post by: kimmytaaa on February 15, 2016, 10:17:23 am
hey
can someone who did 'the lieutenant'  give me with another idea for my prompt 'conflict creates a stronger relationship' ?

Title: Re: VCE English Question Thread
Post by: HopefulLawStudent on February 15, 2016, 05:28:27 pm
Heeeeelp.

What's the difference between anaphora and repetition??? Can you say anaphora is a language technique? Can someone give me an example of anaphora and how to use the word in a sentence?
Title: Re: VCE English Question Thread
Post by: Swagadaktal on February 15, 2016, 05:34:14 pm
Heeeeelp.

What's the difference between anaphora and repetition??? Can you say anaphora is a language technique? Can someone give me an example of anaphora and how to use the word in a sentence?
Anaphora is a type of repetition.
Google has the examples you need fam -- and you don't use the word anaphora in a sentence, you use the technique over a few sentences
Title: Re: VCE English Question Thread
Post by: literally lauren on February 15, 2016, 09:03:28 pm
hey
can someone who did 'the lieutenant'  give me with another idea for my prompt 'conflict creates a stronger relationship' ?
Which ideas have you explored already? And which relationships do you think you could talk about in the text? Obviously Rooke's dynamic with the other characters is worth talking about, but you could also examine the relationships between groups (eg. the white settlers and the indigenous people).

But I'd recommend going for the ideas first, and then zooming in to find textual evidence afterwards ie. once you've already got a good sense of what you want to convey in that paragraph.

Heeeeelp.

What's the difference between anaphora and repetition??? Can you say anaphora is a language technique? Can someone give me an example of anaphora and how to use the word in a sentence?
Repetition is the recurrence of a word, phrase, technique, or idea throughout a text.
Anaphora can be one or two things:

1. Grammatically, it's when the author uses a pronoun as a replacement for a noun.

For instance:
   'The Prime Minister has said that he doesn't want to invade the arctic tundra.'
- 'The Prime Minister' = Noun (technically Proper Noun, but whatevs)
- 'he' = Pronoun

This is in contrast to cataphora, which is where the pronoun comes before the noun
   'Following her recent interview in which she revealed that she hated puppies, the Vice Chancellor resigned in disgrace.'

It's basically a natural thing for English speakers to do, since otherwise you end up with sentences like:
  'The Prime Minister has said that the Prime Minister doesn't want to invade the arctic tundra'
and
   'Following the Vice Chancellor's recent interview in which the Vice Chancellor revealed that she hated puppies, the Vice Chancellor resigned in disgrace.'

So.. in a way... anaphora is a way for the author to avoid repetition by changing the phrasing around and assigning meaning to certain pronouns.

...but...

2. It also refers to repetition that occurs at the start of successive sentences.

For instance:
'I can't believe that I only scored a 19/20 on my last SAC. I can't believe my teacher could be so stupid. I can't believe I'm anything less than 100% perfect. And I can't believe that she wouldn't give me the extra mark when I begged on my hands and knees.'

In terms of its effect, it's quite similar to repetition, only calling it 'anaphora' when you see it can be a nice way to show off, and it allows you to comment on the way the author might want to create a link between those sentences.

eg.
'There's no justice for the teachers who have been fired. There's no justice for these innocent victims. There's no justice for good, honest people who are just trying to make the world a better place.'

^So in the above, I'm trying to make you view the teachers as innocent victims and good honest people, and the anaphora is helping me accomplish this be creating that rhetorical link in the phraseology.

In a sentence, you'd refer to it like you would any other technique,

eg.
'The author draws attention to the injustice of the issue through her use of anaphora in the phrase "there is no justice" which is reiterated throughout the piece. Not only does this amplify the sense of unfairness, but it also aids the author in establishing a link between the "good, honest" teachers and the notion of "victim[hood]"'

Hope that helps!
Title: Re: VCE English Question Thread
Post by: Gogo14 on February 15, 2016, 09:16:47 pm
Its probably been said one thousand times, but it wouldnt hurt to answer one more.

I'm in year 11 and how can I really improve my english to have strong foundations for year 12.Doing mainstream english btw.
Title: Re: VCE English Question Thread
Post by: vor0005 on February 15, 2016, 09:23:05 pm
Its probably been said one thousand times, but it wouldnt hurt to answer one more.

I'm in year 11 and how can I really improve my english to have strong foundations for year 12.Doing mainstream english btw.

Write essays. Get feedback (from a tutor/teacher etc.) Learn from the feedback. Repeat.
Title: Re: VCE English Question Thread
Post by: literally lauren on February 15, 2016, 09:24:43 pm
Its probably been said one thousand times, but it wouldnt hurt to answer one more.

I'm in year 11 and how can I really improve my english to have strong foundations for year 12.Doing mainstream english btw.
Write essays. Get feedback (from a tutor/teacher etc.) Learn from the feedback. Repeat.
^ +1

Most importantly: know exactly what you need to work on and where your weakness are at the moment.

Also read heaps. It's good for your brain in general, not just VCE English.

Also also, there's a tonne of advice if you look through the links in the first page of this thread, so there'll be some more specific stuff there.

Also also also, if you're in Year 11, you'll be dealing with this study design so familiarising yourself with that is a good place to start.

Title: Re: VCE English Question Thread
Post by: kimmytaaa on February 16, 2016, 10:36:47 am
Which ideas have you explored already? And which relationships do you think you could talk about in the text? Obviously Rooke's dynamic with the other characters is worth talking about, but you could also examine the relationships between groups (eg. the white settlers and the indigenous people).

But I'd recommend going for the ideas first, and then zooming in to find textual evidence afterwards ie. once you've already got a good sense of what you want to convey in that paragraph.
Repetition is the recurrence of a word, phrase, technique, or idea throughout a text.
Anaphora can be one or two things:

1. Grammatically, it's when the author uses a pronoun as a replacement for a noun.

For instance:
   'The Prime Minister has said that he doesn't want to invade the arctic tundra.'
- 'The Prime Minister' = Noun (technically Proper Noun, but whatevs)
- 'he' = Pronoun

This is in contrast to cataphora, which is where the pronoun comes before the noun
   'Following her recent interview in which she revealed that she hated puppies, the Vice Chancellor resigned in disgrace.'

It's basically a natural thing for English speakers to do, since otherwise you end up with sentences like:
  'The Prime Minister has said that the Prime Minister doesn't want to invade the arctic tundra'
and
   'Following the Vice Chancellor's recent interview in which the Vice Chancellor revealed that she hated puppies, the Vice Chancellor resigned in disgrace.'

So.. in a way... anaphora is a way for the author to avoid repetition by changing the phrasing around and assigning meaning to certain pronouns.

...but...

2. It also refers to repetition that occurs at the start of successive sentences.

For instance:
'I can't believe that I only scored a 19/20 on my last SAC. I can't believe my teacher could be so stupid. I can't believe I'm anything less than 100% perfect. And I can't believe that she wouldn't give me the extra mark when I begged on my hands and knees.'

In terms of its effect, it's quite similar to repetition, only calling it 'anaphora' when you see it can be a nice way to show off, and it allows you to comment on the way the author might want to create a link between those sentences.

eg.
'There's no justice for the teachers who have been fired. There's no justice for these innocent victims. There's no justice for good, honest people who are just trying to make the world a better place.'

^So in the above, I'm trying to make you view the teachers as innocent victims and good honest people, and the anaphora is helping me accomplish this be creating that rhetorical link in the phraseology.

In a sentence, you'd refer to it like you would any other technique,

eg.
'The author draws attention to the injustice of the issue through her use of anaphora in the phrase "there is no justice" which is reiterated throughout the piece. Not only does this amplify the sense of unfairness, but it also aids the author in establishing a link between the "good, honest" teachers and the notion of "victim[hood]"'

Hope that helps!

Hey Lauren,
I have explored the relationship between Tagaran and Rooke, how her arrival changed his perspective. Also, I have talked about what he did for the Indigenous people.
Title: Re: VCE English Question Thread
Post by: literally lauren on February 16, 2016, 11:22:29 am
Hey Lauren,
I have explored the relationship between Tagaran and Rooke, how her arrival changed his perspective. Also, I have talked about what he did for the Indigenous people.
If you're doing a standard expository essay, you don't necessarily have to use the text in every paragraph. In fact, your piece will likely be a lot stronger if you can integrate external evidence and stop things from being too 'text response-y.' So you could look into some real world parallels that pertain to the ideas in the text, eg.
• The Colonial History of Australia
• The Stolen Generation (possibly a bit overused, but still interesting if explored well)
• Kevin Rudd's Sorry Speech
• The 'Closing the Gap' Campaign
• Charity Programs like AIME
This Debate (started by a radio shock-jock who said we should have another stolen generation for their own good because Indigenous culture is rife with alcohol abuse and low SES living conditions -.-)
• Other examples of racial tension, especially between native and colonial cultures.

Alternatively, if you have to use another example from the text for some reason, consider using it to explore the prompt in some other capacity so that you're not just proving it right. For instance:
'Conflict creates stronger relationships.'
- Why does it create them?
- Do we try to create stronger relationships, or do they happen automatically?
- Is it the conflict that creates these relationships, or our response to the conflict?
- How are these relationships 'stronger?' What does stronger mean? Is stronger better?
- Is this always true? Are there some relationships that are damaged by conflict?
- Why are these relationships important?

Hopefully that'll help get you started :)
Title: Re: VCE English Question Thread
Post by: kimmytaaa on February 16, 2016, 01:49:12 pm
If you're doing a standard expository essay, you don't necessarily have to use the text in every paragraph. In fact, your piece will likely be a lot stronger if you can integrate external evidence and stop things from being too 'text response-y.' So you could look into some real world parallels that pertain to the ideas in the text, eg.
• The Colonial History of Australia
• The Stolen Generation (possibly a bit overused, but still interesting if explored well)
• Kevin Rudd's Sorry Speech
• The 'Closing the Gap' Campaign
• Charity Programs like AIME
This Debate (started by a radio shock-jock who said we should have another stolen generation for their own good because Indigenous culture is rife with alcohol abuse and low SES living conditions -.-)
• Other examples of racial tension, especially between native and colonial cultures.

Alternatively, if you have to use another example from the text for some reason, consider using it to explore the prompt in some other capacity so that you're not just proving it right. For instance:
'Conflict creates stronger relationships.'
- Why does it create them?
- Do we try to create stronger relationships, or do they happen automatically?
- Is it the conflict that creates these relationships, or our response to the conflict?
- How are these relationships 'stronger?' What does stronger mean? Is stronger better?
- Is this always true? Are there some relationships that are damaged by conflict?
- Why are these relationships important?

Hopefully that'll help get you started :)


hey Lauren
are you able to read my essay if you are free? Ill message you
Title: Re: VCE English Question Thread
Post by: literally lauren on February 16, 2016, 06:37:30 pm

hey Lauren
are you able to read my essay if you are free? Ill message you
Will do; I'm happy to give general tips via PM, but if you want full corrections/feedback, post it here and I'll get back to you :)
Title: Re: VCE English Question Thread
Post by: Gogo14 on February 16, 2016, 07:12:22 pm
If i decided to write an essay each week, what could I write about and how can I motivate myself to continue this routine throughout the year as I often slack off.  :-\
Title: Re: VCE English Question Thread
Post by: literally lauren on February 16, 2016, 07:33:01 pm
If i decided to write an essay each week, what could I write about and how can I motivate myself to continue this routine throughout the year as I often slack off.  :-\
If you're the kind of student who doesn't stick to timetables and study regimes normally, then committing to an essay per week is probably just going to burn you out and leave you detesting English by the end of the year.

Instead of angling for numerical goals, try and work on your qualitative weaknesses instead. Make a big list of everything you'd like to change about your current ability level, or perhaps a list of all the things you wished you knew how to do... for instance:
- get better at writing topic sentences
- be able to think of the right evidence when writing in time conditions
- learn how to do visual analysis for L.A.
- be able to remember more varied quotes and use them in sentences
- stop myself from repeating words in the prompts over and over again in essays
- be able to express ideas more efficiently

..etc.
and you can just target one or two of those goals each week. So your starting point might be to post here and/or ask your teacher about some area of assistance (e.g. remembering a greater variety of quotes and implementing them in your analysis --> 'yo teacher, can you show me how to do this?' + 'sup ATAR Notes, lend me your wisdom.') Then from there, you have to write, say a paragraph where you test this out. You can mix and match here; maybe it's a paragraph one week, an essay plan the next, and a full essay the week after that - it's up to you - whatever you think will be most effective for that specific point of weakness. After that, you can post what you've done or give it to your teacher to check, use that feedback to refine what you've done, and then move on to the next point.

I guarantee doing this^ will be far more efficient than just churning out an essay every week and hoping that improvement happens automatically, and it'll make you feel better since there'll be a tangible sense of progress you're making (i.e. 'On Monday, I didn't know how to write conclusions for Text Response; now it's Friday, and I do. I am an English god!') as opposed to having a pile of essays at the end of the year with nothing to show for it.
Title: Re: VCE English Question Thread
Post by: eth-dog on February 17, 2016, 10:01:00 pm
Hey, i don't know if this is a bit of an obvious question but how would you introduce an argument into an oral. Would you just go straight in for example ' a common argument is...' or would you introduce the argument somewhat secretly and very smooth?

Thanks in advance  :) :)
Title: Re: VCE English Question Thread
Post by: qazser on February 17, 2016, 10:33:42 pm
Hey, i don't know if this is a bit of an obvious question but how would you introduce an argument into an oral. Would you just go straight in for example ' a common argument is...' or would you introduce the argument somewhat secretly and very smooth?

Thanks in advance  :) :)

IMO Try not to start with
"good afternoon Superman and fellow classmates, my name is Batman. The issue which i will put forward today is..."

Try
-Anecdotes(stories)
-Rhetorical Question
-Video(esp shocking pics, immediately attack's reader's appeal for sympathy etc.)
-Stats

My opinion might not be right, see what the other AN fellows say :)
Title: Re: VCE English Question Thread
Post by: literally lauren on February 18, 2016, 10:48:45 am
Hey, i don't know if this is a bit of an obvious question but how would you introduce an argument into an oral. Would you just go straight in for example ' a common argument is...' or would you introduce the argument somewhat secretly and very smooth?

Thanks in advance  :) :)
That depends on whether this is an argument, or your argument. Obviously if it's your contention then you don't want to introduce it as something "common," and if it's a counterpoint that you want to disagree with, then you should characterise it as such.

Consider:

1. The reason why students are at a disadvantage nowadays is because the ATAR system is confusing and broken - something must be done to fix this for the sake of our country's future!

vs.

2. Some people may think that the ATAR system is confusing or broken, but this is just product of their own misunderstanding.

Notice how, in the first example, I'm presenting this point in a persuasive way ("something must be done") in order to frame it as something I am endorsing or agreeing with? Whereas in the second case, the same point is presented in a "some people may think this, but they're wrong" sort of way :P

So if you're bringing up a point you agree with, there's no reason to preface it with "The following point I am about to make is an argument. Here is the argument: _____" but if you want to challenge this point or actually discuss whether it has any merit, then you can go for the more explicit markers like "some people may believe" or "it is widely known that..." etc.
Title: Re: VCE English Question Thread
Post by: HopefulLawStudent on February 19, 2016, 05:59:16 pm
I'm studying Medea for Reading and Responding.

Medea argues that she would never "think of leaving sons of mine... for those who hate me to insult", thereby validating (to some degree) the infanticide that closes the play as an act of love, performed to save her sons from becoming a vehicle through which others may seek to hurt her.

What I don't get is this: Why doesn't Medea just take her sons with her to Athens?
Title: Re: VCE English Question Thread
Post by: Darth_Pepe on February 19, 2016, 06:23:41 pm
I'm studying Medea for Reading and Responding.

Medea argues that she would never "think of leaving sons of mine... for those who hate me to insult", thereby validating (to some degree) the infanticide that closes the play as an act of love, performed to save her sons from becoming a vehicle through which others may seek to hurt her.

What I don't get is this: Why doesn't Medea just take her sons with her to Athens?

Revenge against her husband. Back then it was a big deal to have a family line and if she killed her two sons Jason would literally have nothing left and no generation to pass anything onto.
Title: Re: VCE English Question Thread
Post by: Coffee on February 19, 2016, 06:31:27 pm
What I don't get is this: Why doesn't Medea just take her sons with her to Athens?

She kills her children as an act of revenge against Jason. When he leaves her to marry Glauce she is absolutely devastated. In order for Medea to inflict the same amount of pain he did to her she decides that he must be left with nothing. This is done through her murder (poisoning) of the Princess and incidentally Creon too, and finally the murder of their children.

If you take a look around page 59/60 you can find her reasoning for this:
"But my pain's a fair price, to take away your smile."
"I can stab too: your sons are dead! ... To haunt your life with vengeance."
And probably most importantly:
"Jason: Dear - and your murdered them?
Medea: "Yes, Jason, to break your heart."
Title: Re: VCE English Question Thread
Post by: Swagadaktal on February 19, 2016, 06:48:02 pm
I'm studying Medea for Reading and Responding.

Medea argues that she would never "think of leaving sons of mine... for those who hate me to insult", thereby validating (to some degree) the infanticide that closes the play as an act of love, performed to save her sons from becoming a vehicle through which others may seek to hurt her.

What I don't get is this: Why doesn't Medea just take her sons with her to Athens?
I've just read the book and I haven't done much on it (doing it in term 3 sooooo)
But from what I've gathered, her son's would live a life in the shadows? They would live alone in a foreign city, being raised by a single mother. At those times, single mothers had it tough and their lives simply would not of been worth living (like this is what the crazy bitch is thinking right?) - coz they'd always be second-rate citizens, and their royal siblings will always have first preference of everything if they were to ever reunite with Jason's new fam --
Can anyone agree? I've only read this book to a shallow degree and I havent read externally so my opinions aren't founded
Title: Re: VCE English Question Thread
Post by: HopefulLawStudent on February 19, 2016, 07:52:42 pm
Huh.

You guys have given me some fodder to mull over.
Title: Re: VCE English Question Thread
Post by: Swagadaktal on February 21, 2016, 01:32:03 pm
Ok guys, I need help structuring my bodies (ik its year 12 i should already be good at this shit, but I think what I've learnt in year 11 isn't effective in year 12) -- I structure my bodies as: T.s - evidence 1 - explain evidence 1 - relate evidence 1 to argument. Then, evidence 2 - explain evidence 2 - relate evidence 2 to argument - link evidence 1 and 2 to the argument - link evidence 1 and 2 to a greater idea (eitehr a different perspective, or a social/historical context or really just bring home the values of the author) - but I find that i always end up re-telling the plot and it's just simply not effective. How do I structure my bodies so that I have continuous embedded quotations and have better flow throughout the whole body?
Like I write my topic sentence.. and then what? Do i go into my argument and then use quotes whilst simultaneously explaining what the author is doing to show that?
Title: Re: VCE English Question Thread
Post by: Swagadaktal on February 21, 2016, 02:17:59 pm
Ok guys, I need help structuring my bodies (ik its year 12 i should already be good at this shit, but I think what I've learnt in year 11 isn't effective in year 12) -- I structure my bodies as: T.s - evidence 1 - explain evidence 1 - relate evidence 1 to argument. Then, evidence 2 - explain evidence 2 - relate evidence 2 to argument - link evidence 1 and 2 to the argument - link evidence 1 and 2 to a greater idea (eitehr a different perspective, or a social/historical context or really just bring home the values of the author) - but I find that i always end up re-telling the plot and it's just simply not effective. How do I structure my bodies so that I have continuous embedded quotations and have better flow throughout the whole body?
Like I write my topic sentence.. and then what? Do i go into my argument and then use quotes whilst simultaneously explaining what the author is doing to show that?
ok update: I understand the integration of quotes and development of ideas...
I just need to know where to go from a topic sentence? Do i go strraight into evidence and yolo it from there? What's your first sentence after the topic sentence?
Title: Re: VCE English Question Thread
Post by: JewHunter on February 21, 2016, 02:50:15 pm
Hello, was a straight D student last year. somehow got A+ for the oral. now im doing a film "All about Eve" how do i get A+ on this. SACs in 4 weeks :D
pl
Title: Re: VCE English Question Thread
Post by: heids on February 21, 2016, 03:00:04 pm
I'm gonna leave it to Lauren to discourse on body structures, but wanted to throw in a comment:

ik its year 12 i should already be good at this shit, but I think what I've learnt in year 11 isn't effective in year 12

Worry not, that's exactly how I felt.  I constantly felt that I 'should know' how to write and structure an essay by now... but because of string of dodgy teachers from yr 9-11, I felt clueless (still do haha :P).  It's actually very, very normal among 95% of the population, so no stress.

But yes, I think you just dive straight from your overall 'idea' into evidence to support/prove/build on that idea, as long as the transition is smooth rather than feeling like a sudden 'jerk' into a different topic.

If you're not sure, a great thing to do can be deconstructing sample essays (we've got heaps by different people) into dot-points or flow-charts or highlighted colour-coding - even if they're not on texts you've done, you can still see exactly how they structure their essay and transition between evidence, analysis, and bigger ideas.

Hello, was a straight D student last year. somehow got A+ for the oral. now im doing a film "All about Eve" how do i get A+ on this. SACs in 4 weeks :D
pl

Firstly, congrats on that A+ - must have been so satisfying, well done!

When you read one of your straight-D essays, can you figure out some stuff that you didn't do so well on?  What made you lose those marks?  What bits could you focus on improving?

The way to improve is to:
1.  Pick out a small weakness (e.g. my quoting isn't integrated smoothly, or, my topic sentences focus on evidence rather than ideas etc.)
2.  Try to find out how to do that well - search for examples and ask questions about it.
3.  Practise that skill specifically a few times, and get feedback, until you're on top of it.

Do that with enough specific things, and that's the path to A+!

So, when you've found a couple of specific weaknesses, feel free to drop us a question about how to improve in those specific areas :D
Title: Re: VCE English Question Thread
Post by: Stewart98 on February 21, 2016, 06:51:29 pm
Hey Guys,
I'm in need of some help. So I've got a SAC coming up real soon, which is a text response, and I haven't really learnt how to write a proper essay. I've tried writing a couple and I just can't think of any ideas to write after the first paragraph. Not only that, I also don't know how to find arguments and how to elaborate on them properly.
Please Help! :)
Title: Re: VCE English Question Thread
Post by: Apink! on February 23, 2016, 10:02:27 am
Hi,
I'm having a massive problem developing a complex, well-developed contention for a prompt:
For example, if I had received this prompt: The strength of Wolff's memoir is his readiness to cast himself in the worst possible light
I would have no idea what to write, and would probably end up discussing things that's really really basic and shallow.  Or I could try to come up with a complex contention and waste my writing time. How can I develop a complex contention? Thank you so much :)

edit: I study This Boy's Life
Title: Re: VCE English Question Thread
Post by: molecular. on February 23, 2016, 02:16:18 pm
Hey guys,

I am struggling to get my writing to an A+ level, which is the required level to get the score I am after. Currently, I write one essay a week, hand it to my teacher, get feedback 4-5 days later, re-read and check what I need to improve on - sometimes I rewrite and improve on the mistakes but due to time constraints this rarely occurs.

The issue is that although my teacher has said that I have improved, I still cannot score past a B+ (I was a B+ from start of year 11 and am still a B+ in year 12). I do not understand how I have improved so much, which I have felt and can see in my writing if I am still getting the same score.

Overall, I need to get my essays up to an A+ but I have run out of ideas. The text I am studying right now is ‘In the Country of Men’ for text response.

Any responses will be appreciated.


Title: Re: VCE English Question Thread
Post by: literally lauren on February 24, 2016, 01:48:11 pm
Ok guys, I need help structuring my bodies (ik its year 12 i should already be good at this shit, but I think what I've learnt in year 11 isn't effective in year 12) -- I structure my bodies as: T.s - evidence 1 - explain evidence 1 - relate evidence 1 to argument. Then, evidence 2 - explain evidence 2 - relate evidence 2 to argument - link evidence 1 and 2 to the argument - link evidence 1 and 2 to a greater idea (eitehr a different perspective, or a social/historical context or really just bring home the values of the author) - but I find that i always end up re-telling the plot and it's just simply not effective. How do I structure my bodies so that I have continuous embedded quotations and have better flow throughout the whole body?
Like I write my topic sentence.. and then what? Do i go into my argument and then use quotes whilst simultaneously explaining what the author is doing to show that?
ok update: I understand the integration of quotes and development of ideas...
I just need to know where to go from a topic sentence? Do i go strraight into evidence and yolo it from there? What's your first sentence after the topic sentence?
Okay, I could write thousands of words about this but in the interests of keeping things concise, your B.P.s for T.R. should look something like:

1. Topic Sentence that outlines a sub-argument or key concern you're going to explore
2. Evidence and analysis
3. Statement linking this discussion back to your main contention and the prompt.

Some people are fans of the 'gradual zooming in' sentence after the T.S. which basically lets you explain your points in a little more detail before jumping into the evidence, and I think that's an excellent idea.

I'd actually advocate for more of a 'yolo' approach when it comes to stage two though. Your previous structure seems a bit too narrow, and restricting yourself to just two pieces of evidence per paragraph is not a good idea. Think of it like building a case in a courtroom or even building a stack of lego bricks - you can't just say two things or put two bricks down and call it a stable structure. You need sufficient coverage of the text to get marks, and limiting your evidence base kind of sets you up to fall short.

Instead, perhaps structure your body paragraphs around the 'explain it to a 5 year old' strategy whereby you pretend to take someone (ideally a small child, as this will encourage you to spell out your logic more clearly) through your thought process from the opening idea through to the closing link to your argument. You're allowed to assume that this 5 year old is very intelligent and will understand anything you say so long as the logic is explained clearly.

Regarding retelling the story, think of it this way. Imagine you had a quote from your text, and someone was asking you where that quote was from. If you were explaining it to a parent, or to your friend at a different school, you'd want to be giving them quite a bit of background info, right? You couldn't just say 'oh, Susan said it after the fire' because they'd be like... who's Susan? What fire? Wtf?? But if you're talking to a friend in your English class, you're not going to give them  a huge speech like 'well this quote was delivered by Susan who is Neil's wife and the protagonist of the novel, following her dramatic escape from the hospital which burned down after Neil accidentally set some curtains on fire' because your friend is going to be like... yeah, I know... I've read the book too... you could've just said 'it was Susan after the fire.'

When you write T.R. essays, you need to assume that your assessor has read the text. That doesn't mean you can jump all over the place with no context to your discussion, but it does mean that when you introduce points of evidence, you can just say 'Following her confrontation with the fireman, Susan...' and go on to analyse from there.

Hey Guys,
I'm in need of some help. So I've got a SAC coming up real soon, which is a text response, and I haven't really learnt how to write a proper essay. I've tried writing a couple and I just can't think of any ideas to write after the first paragraph. Not only that, I also don't know how to find arguments and how to elaborate on them properly.
Please Help! :)
Have a read through the links on the first page of this thread under the 'Text Response' heading. Hopefully that'll give you some foundation for understanding, and just let us know if there's anything you're still unsure of :)

Hi,
I'm having a massive problem developing a complex, well-developed contention for a prompt:
For example, if I had received this prompt: The strength of Wolff's memoir is his readiness to cast himself in the worst possible light
I would have no idea what to write, and would probably end up discussing things that's really really basic and shallow.  Or I could try to come up with a complex contention and waste my writing time. How can I develop a complex contention? Thank you so much :)

edit: I study This Boy's Life
See above; the first page of this thread has some good links to discussions about unpacking prompts and formulating contentions, so have a read through those first. Also, consider whether this is a problem with you feeling as though you don't know the text well enough, or just not being able to word your ideas in a more complex way. Simply put: do you not know what to say, or do you not know how to say it, because those are two very different issues that require fairly different approaches.

Hey guys,

I am struggling to get my writing to an A+ level, which is the required level to get the score I am after. Currently, I write one essay a week, hand it to my teacher, get feedback 4-5 days later, re-read and check what I need to improve on - sometimes I rewrite and improve on the mistakes but due to time constraints this rarely occurs.

The issue is that although my teacher has said that I have improved, I still cannot score past a B+ (I was a B+ from start of year 11 and am still a B+ in year 12). I do not understand how I have improved so much, which I have felt and can see in my writing if I am still getting the same score.

Overall, I need to get my essays up to an A+ but I have run out of ideas. The text I am studying right now is ‘In the Country of Men’ for text response.

Any responses will be appreciated.
Okay, so you're getting a B+. But why are you getting a B+? <-- if you can answer that question, then improvement will be easy, but if you get stuck in the mindset of 'I want to be scoring A+s and I'm not,' then it'll be really hard to facilitate qualitative improvement.

Are you making the same mistakes every time, or are you fixing some mistakes and them making others every time you write? Is your teacher able to go through some of your pieces with you and explain what you need to do to improve? And by that I don't mean having someone say 'your Topic Sentences need work,' but rather, someone who will sit down with you and explain why your T.S.s are flawed, what good ones look like, why they're good, and how you can go about modifying your writing style in order to turn yours into those good versions.

Try to put thoughts of numerical scores or letter grades out of your head for now, and see if you can come up with a personalised list of 'do's and don't's' for Text Response.

eg.
DO:
Ensure your contention is clear by the end of the introduction
Include at least five quotes in every paragraph
Use the author's name when 'zooming out' and talking about the text as a whole
etc.

DON'T
Summarise all your arguments in the introduction and conclusion
Stick quotes into your sentences without integrating them
Reuse words from the prompt without variation or expansion
etc.

...and this will give you a much more coherent plan for improvement :)

Let me know if you have any questions!
Title: Re: VCE English Question Thread
Post by: molecular. on February 24, 2016, 06:54:49 pm
Thanks for your advice Lauren. I will get working!
Title: Re: VCE English Question Thread
Post by: molecular. on February 28, 2016, 01:30:08 pm

Hey guys,

Can someone mark my essay? I have a sac very soon.

Any response will be helpful.

Thanks.
Title: Re: VCE English Question Thread
Post by: literally lauren on February 28, 2016, 04:21:06 pm
Got your PM, I will get there eventually :) Essay marking related stuff usually goes here for future reference.

Also, to the people sending me some less-than-patient PMs at the moment (not at all directed at you molecular - this just a general request to others) please be aware that the times when you're dealing with SACs are often times when the rest of the state are dealing with SACs too... meaning that at several stages throughout the year, a lot of people are seeking help all at once. Since the start of Feb, I've been getting on average thirty essays a day from students either through the forums or elsewhere, and the stuff you see me marking publically is probably about 2% of my workload right now :-\

In general I try and give priority to people posting on the submissions boards since that has the potential to benefit others too, so if you're messaging me with an essay expecting feedback the evening before your homework is due or your SAC begins, you're unlikely to get a response -- not because I'm ignoring you but because if I don't have a system for dealing with the tidal waves of essays to mark then I'll end up unfairly disadvantaging everybody. I'll happily give comments to people if they're PMing their work, and I'm by no means trying to discourage people from seeking feedback, but please be patient with this poor little workhorse - there are only so many hours in a day, and I'm told I have to spend at least some of them sleeping.

So post on the submissions board for a quicker and more in-depth reply, otherwise be prepared to wait a few days for me to get around to things.
Title: Re: VCE English Question Thread
Post by: molecular. on February 28, 2016, 06:11:55 pm
Hey Lauren,

I completely understand were you are coming from. Sorry for any misunderstanding - you must be sick of essays.

Peace :)
Title: Re: VCE English Question Thread
Post by: literally lauren on February 28, 2016, 06:59:59 pm
Hey Lauren,

I completely understand were you are coming from. Sorry for any misunderstanding - you must be sick of essays.

Peace :)
All good man, the above wasn't even slightly directed at you; it's more the people spamming me with four or five essays and then demanding an explanation as to why I haven't responded within 24 hours  ::) - I'll try and get some comments back to you later tonight :)
Title: Re: VCE English Question Thread
Post by: Swagadaktal on February 28, 2016, 07:06:12 pm
All good man, the above wasn't even slightly directed at you; it's more the people spamming me with four or five essays and then demanding an explanation as to why I haven't responded within 24 hours  ::) - I'll try and get some comments back to you later tonight :)
Wait is this an invitation to send you an essay :O - Like once every while (not too often, maybe 1 or 2 a term?)
Title: Re: VCE English Question Thread
Post by: literally lauren on February 28, 2016, 07:19:02 pm
Wait is this an invitation to send you an essay :O - Like once every while (not too often, maybe 1 or 2 a term?)
This is a... polite reminder that there are hundreds of other people sending me essays so if you do intend to send me things, you shouldn't expect instantaneous replies :P I'd much prefer you post on the submissions board since you have no idea how frustrating it is to have like twenty people send you essays on a certain text and ask you for sample essays about that text since if they'd all put theirs up on the forums, they could all benefit from one another's work. Alas, I know people labour under the delusion of not wanting to give an advantage to anyone in VCE, so if you do want to message me, just know that I'm usually tackling half a dozen other pieces on any given day, so your patience will be appreciated. Not trying to come across as standoffish, but I just wanted to say something here about how essays are welcome(-ish) but the 'y u no mark my essay yet!?' messages... not so much... Also this is just a bit of a catch-all for some of the PMs I've neglected recently - I will get there eventually :P
Title: Re: VCE English Question Thread
Post by: Apink! on February 29, 2016, 03:48:34 pm
Hello everyone,
I was wondering if I could receive assistance on how I can move away from very simple ideas per paragraph. I'm studying This Boy's Life at the moment. If I got this prompt:
"I didn't come to Utah to be the same boy I'd been before. I had my own dreams of transformation" Ultimately, the text demonstrates that everyone covers up or transforms in some way.

Immediately seeing this, this is how I would structure it (and it's bad)
Par 1: Examples of people who covers up (to agree with prompt)
Par2: Examples of people who transform (to agree with prompt)
Par3: Examples of people who don't cover up or transform (to disagree with prompt)
I know I'm doing this wrong. How can I come up with insightful ideas to discuss (quickly). Please help! :) :)
And if you get a really simple 2 mark text response prompt, HOW can you discuss more in dept ideas when the prompt is asking you something fairly simple?
Title: Re: VCE English Question Thread
Post by: Swagadaktal on February 29, 2016, 07:30:45 pm
Wassup my peeps,

I was wondering about what we have to do if there is a quote in the prompt? Do we have to use it? I draw ideas out from it but do I have to acknowledge the quote itself in my essay?
I couldn't find anything on the QnA front page, if it's there link me there pls

thx
:)
Title: Re: VCE English Question Thread
Post by: Alter on February 29, 2016, 08:09:05 pm
If there is a quote in the prompt, you should always address it in one form or another. Quotes are often placed to steer you in a certain direction and allow you to build off an idea that is only in its foundations. Typically, I'd address the quote in the first paragraph, or perhaps the introduction if it is possible or the quote is very minor. Doing so in the first body paragraph shows that you've thought about your response before you've jumped head-first into the question and allows you to extend off the quote given into some more complex ideas. This will prove to your examiner that you know your text well and you can respond to the actual task given, both of which are things that won't hurt you.
Title: Re: VCE English Question Thread
Post by: GradReady on February 29, 2016, 11:29:43 pm
Hello everyone,
I was wondering if I could receive assistance on how I can move away from very simple ideas per paragraph. I'm studying This Boy's Life at the moment. If I got this prompt:
"I didn't come to Utah to be the same boy I'd been before. I had my own dreams of transformation" Ultimately, the text demonstrates that everyone covers up or transforms in some way.

Immediately seeing this, this is how I would structure it (and it's bad)
Par 1: Examples of people who covers up (to agree with prompt)
Par2: Examples of people who transform (to agree with prompt)
Par3: Examples of people who don't cover up or transform (to disagree with prompt)
I know I'm doing this wrong. How can I come up with insightful ideas to discuss (quickly). Please help! :) :)
And if you get a really simple 2 mark text response prompt, HOW can you discuss more in dept ideas when the prompt is asking you something fairly simple?

Sometimes you're simply bound to the prompt, and its simplicity can be creatively prohibitive. Your primary concern is addressing the prompt. Resorting to several broad and simple point does not spell doom. In fact, it can be refreshing and very powerful if it is persuasively argued.

On the other hand, if the prompt enables nuanced arguments to be made, locating those arguments comes with practice. Spend lots of time thinking about potential prompts before an exam/SAC, and think deeply about the issues those prompts raise. Come exam/SAC time, you should have an itinerary ready to adapt to the specific prompt to which you're asked to respond.

Hope that's of some assistance.
Title: Re: VCE English Question Thread
Post by: Swagadaktal on March 01, 2016, 03:48:18 pm
Sometimes you're simply bound to the prompt, and its simplicity can be creatively prohibitive. Your primary concern is addressing the prompt. Resorting to several broad and simple point does not spell doom. In fact, it can be refreshing and very powerful if it is persuasively argued.

On the other hand, if the prompt enables nuanced arguments to be made, locating those arguments comes with practice. Spend lots of time thinking about potential prompts before an exam/SAC, and think deeply about the issues those prompts raise. Come exam/SAC time, you should have an itinerary ready to adapt to the specific prompt to which you're asked to respond.

Hope that's of some assistance.
On that point, do our contentions need to be really mind blowing? As well as our topic sentences/ arguments. Or could they just be insightful body paragraphs that have good use of evidence and relate it to the author's ideas and what not?
Title: Re: VCE English Question Thread
Post by: literally lauren on March 03, 2016, 01:32:20 pm
Hello everyone,
I was wondering if I could receive assistance on how I can move away from very simple ideas per paragraph. I'm studying This Boy's Life at the moment. If I got this prompt:
"I didn't come to Utah to be the same boy I'd been before. I had my own dreams of transformation" Ultimately, the text demonstrates that everyone covers up or transforms in some way.

Immediately seeing this, this is how I would structure it (and it's bad)
Par 1: Examples of people who covers up (to agree with prompt)
Par2: Examples of people who transform (to agree with prompt)
Par3: Examples of people who don't cover up or transform (to disagree with prompt)
I know I'm doing this wrong. How can I come up with insightful ideas to discuss (quickly). Please help! :) :)
And if you get a really simple 2 mark text response prompt, HOW can you discuss more in dept ideas when the prompt is asking you something fairly simple?
Just going to break this up a bit...

Firstly:
I was wondering if I could receive assistance on how I can move away from very simple ideas per paragraph. I'm studying This Boy's Life at the moment. If I got this prompt:
"I didn't come to Utah to be the same boy I'd been before. I had my own dreams of transformation" Ultimately, the text demonstrates that everyone covers up or transforms in some way.
The best way to go from simple ideas to complex ones is to create a sophisticated contention. The way I explain this makes more sense visually when I can make erratic hand gestures, but you basically want to funnel the text through the prompt, then funnel that through your contention, then distribute what's left into three body paragraphs.

Crude approximation:

(http://i.imgur.com/qXJYABC.png)

You can also have four, (or, I suppose, five sub-arguments) but that's hard to format so let's assume you're sticking with three for argument's sake. So you're never writing on 'the text' or 'the prompt;' you're writing on your contention! This means that your first thought should be turning this prompt into a viable argument, since if you bypass that step and try to come up with sub-arguments before you've come up with... well... an argument... things get a bit tricky.

For example:
"I didn't come to Utah to be the same boy I'd been before. I had my own dreams of transformation"
Ultimately, the text demonstrates that everyone covers up or transforms in some way.

Alter's already covered what to do when a quote is involved in the prompt, so let's put that to the side for now. The core of this prompt is:
All of the characters in TBL cover up or transform in some way.
Next, do you mostly agree or mostly disagree with that sentiment?
--> If you mostly agree, then consider how you might still challenge the prompt somehow.
--> If you mostly disagree, then consider how the prompt might still be somewhat true.

Let's assume you're mostly agreeing, since that seems to be the more logical line of inquiry here.
We want to argue that the characters do cover up or transform, but we don't want to be too definitive. The prompt states that everyone covers up/transforms, and we shouldn't just take that absolute word without questioning it. Perhaps our challenge could come in the form of examining characters that don't really cover up/transform. But that's not really an 'argument' kind of challenge since listing some evidence that doesn't conform to our rule isn't a very substantive discussion point. A lot of students will write a 'challenge' or 'rebuttal' paragraph where they examine the 'other side' of the prompt, but too often this will occur in the form of them just listing evidence that goes against their points.

For instance, if you were to have:
Para 1: the characters who cover up
Para 2: the characters who transform
Para 3: the characters who DON'T cover up or transform
...then you won't have argued anything other than 'some characters cover up/transform and some don't.'

This seems to be kind of where you're at right now, but I'll unpack your sample structure later. For now, because we're concentrating on ways of formulating an overall argument, let's revisit our paraphrasing of the prompt:
All of the characters in TBL cover up or transform in some way.
If I'm agreeing with the underlying principle here, then I want to be arguing that many of the characters cover up, transform, or both. (Note the use of the word 'many,' which serves to pull my contention back from the cliffs of definitiveness - I'm not saying 'all,' because I don't want to seem aggressively 100% certain about my interpretation.)

As a possible challenge, I might consider:
...that the characters cover up/transform for different reasons
...that the characters cover up/transform in different ways and to different extents
...that the characters cover up/transform, but their individual journeys are nevertheless unique because of who they were before and after these cover ups/transformations
...that the characters cover up/transform, but they differ in terms of their own self-awareness

^see how each of these is giving me a more substantial argument than simply saying ...but some of the characters don't cover up/transform?

So our contention might look something like:
Many of the characters in TBL cover up/transform, but they do so for different reasons, which brings about different consequences.

Or, in more academic, essay-ish terms:
Although the characters in This Boy's Life often have vastly different motivations for their actions, Wolff unites many of them through demonstrating their capacity to deceive others by subverting expectations, as well as their propensity to conform to the society or circumstances around them.

Notice here that I've also unpacked that terminology of 'covering-up' (=deceiving others and subverting expectations) and 'transforming' (=conforming to their society/circumstances) which up until now, we've just been treating as self-evident ideas. This is something that will need to be evident in your essay because you don't want to get through the whole thing using words like 'cover up' and 'transform' without ever clarifying what you're talking about. For the record, I'd probably use the bulk of my introduction in this case to flesh out these terms and broaden the definitions beyond the bit of clarifying I've done here, so it's not like we're replacing the idea of 'covering-up' with the idea of 'deceiving others by subverting expectations' - that's just one of many avenues to be explored.

Immediately seeing this, this is how I would structure it (and it's bad)
Par 1: Examples of people who covers up (to agree with prompt)
Par2: Examples of people who transform (to agree with prompt)
Par3: Examples of people who don't cover up or transform (to disagree with prompt)
I know I'm doing this wrong. How can I come up with insightful ideas to discuss (quickly). Please help! :) :)
Okay, so in each of those paragraphs, you've got evidence but no argument. It is possible to structure an argument around the evidence that you've presented, but you can't go into a paragraph thinking 'I'm going to write about the characters who transform in the text' because all you'll do is talk about those characters, and there won't be any broader point that helps your contention.

Imagine if we're arguing about the best city to live in, and you say 'Melbourne's the best because it's got a diverse culture, plenty of attractions, and it has won awards for its livability.' Then I say, 'yeah, well, there's a really nice house in Adelaide. So there.'

^One piece of evidence does not make a viable argument, so when you're structuring your essays, you need to prioritise actual sub-contentions rather than just jumping to ways of sorting out evidence.

This is also why structuring paragraphs by characters is a fairly weak approach.
eg. This Boy's Life shows that lies can be damaging. Discuss.
Para 1: Jack's lies and their damage
Para 2: Rosemary's lies and their damage
Para 3: Dwight's lies and their damage.

^Again, I haven't said anything substantial about these different points, I'm just proving the prompt right in three different ways... which isn't exactly valuable and won't earn any credit for exploration/idea discussion.

Instead, going for thematic breakdowns where each paragraph has a proper sentence as its focus makes for a much stronger piece overall. And in terms of doing it quickly... practice makes perfect :)
And if you get a really simple 2 mark text response prompt, HOW can you discuss more in dept ideas when the prompt is asking you something fairly simple?
Not sure what you mean by a '2 mark text response prompt,' but in the event you get a really simple prompt like: This Boy's Life is about accepting change. Discuss. then it's up to you to make things more expansive. Question things in order to broaden your discussion and try to construct your own arguments rather than relying solely on the prompt to provide three obvious talking points.


On that point, do our contentions need to be really mind blowing? As well as our topic sentences/ arguments. Or could they just be insightful body paragraphs that have good use of evidence and relate it to the author's ideas and what not?
If you can manage to write something 'mind blowing,' then go for it, but ultimately your teachers will be reading hundreds of essays (and might have been doing so for multiple years) so the chance of you stumbling onto a never-before-seen amazing argument that's totally unique and utterly impressive is fairly low :P Most people who try to write 'mind blowing' stuff end up doing irrelevant stuff with really tenuous links to evidence, but it's more than sufficient to be insightful. A competent essay that takes a safe line of argument but does all the important structural stuff well is going to be leagues ahead of someone who goes out of the way to have a special snowflake contention at the expense of actually hitting the criteria :)
Title: Re: VCE English Question Thread
Post by: Apink! on March 03, 2016, 02:19:00 pm
Thank you Alter, GradReady and Lauren.

Lauren, what you have said just made me have a light bulb moment. Thank you  :)
I would just like to ask you one more question - the quote.
Alter has already mentioned that I should discuss them, as the quote reveals a direction of the prompt.
Using the previous prompt, could you please help me see HOW the quote reveals a direction to my argument? Because to me, that just sounds like any other quote about transformation. Is this, by any chance a cue for me to discuss different motivations of transformations and covering up of different characters? (... I had my own dreams of transformation. Indicating individual, unique motivation behind transformations). Also, how can I show that I have thought about the quote? Should I just embed it somewhere and analyse it?

Thank you so much for helping me. I have a SAC soon and I have been very stressed. Thank you :)
Title: Re: VCE English Question Thread
Post by: literally lauren on March 03, 2016, 02:29:21 pm
I would just like to ask you one more question - the quote.
Alter has already mentioned that I should discuss them, as the quote reveals a direction of the prompt.
Using the previous prompt, could you please help me see HOW the quote reveals a direction to my argument? Because to me, that just sounds like any other quote about transformation. Is this, by any chance a cue for me to discuss different motivations of transformations and covering up of different characters? (... I had my own dreams of transformation. Indicating individual, unique motivation behind transformations). Also, how can I show that I have thought about the quote? Should I just embed it somewhere and analyse it?

So:
"I didn't come to Utah to be the same boy I'd been before. I had my own dreams of transformation"
Ultimately, the text demonstrates that everyone covers up or transforms in some way.


The prompt is asserting that all the characters cover up or transform somehow.

The quote is from Jack, and reveals his desire to be different and to transform because he doesn't want to be the "same boy" he was before Utah.

So the quote hints at the idea of the characters deliberately wanting to be different and transform. Therefore, one of the core concepts you should explore is whether the characters are able to engineer their own transformations, and how deliberate these changes are. You might even read something into that bit where he says "I had my own dreams of transformation," implying that other people had dreams too. That seems to be what you're hinting at above, so if you wanted to, you could definitely follow that train of thought.

Basically, any time there's a quote in a prompt, the assessors are giving you some indication of the kind of evidence you should cover, but consider it a starting point rather than something your whole argument should revolve around. I'd recommend using the quote within the first paragraph too, as Alter mentioned.
Title: Re: VCE English Question Thread
Post by: Apink! on March 03, 2016, 03:03:18 pm
Thank you Lauren,
That clarified a lot for me :) :)
Title: Re: VCE English Question Thread
Post by: Evan C on March 06, 2016, 09:05:54 pm
PLS help, SAC in 2 days.

http://www.abc.net.au/news/2015-05-15/mcmahon-doggone-it,-depp!-time-to-obey-some-local-customs/6472194

This was given to our class as a bit of analysis practise and i've tried to plan my analysis by identifying key players, as outlined in some of the guides up here.
Here's what I got for key players:
The wealthy- and how they're given extra/special treatment by everyone
Depp- Depp's recent actions illustrate the entitlement the wealthy have
The common person- and how we yearn for wealth yet despise those who have it out of envy
The air travel business- who allow this disparity to continue by providing special benefits to the wealthy and largely disregarding common people. But also their ineptitude for failing to properly check Depp when he entered the country.

I really struggled to find these, took 2-3 readings, have I found the key players correctly? Are there any that I missed? And is my statement about how the author manipulates them ok?
Title: Re: VCE English Question Thread
Post by: Apink! on March 07, 2016, 12:09:46 pm
Hello,
I'm in need of some quick advice :) I would appreciate it if you could help me!
I wrote a contention and topic sentences to a prompt. Please let me know if there is anything wrong about it. Because I put in the characters' name in my topic sentences, it sounds like I'm just listing evidence. Please help me if you can! :)

Jack has no positive role models in his life, adult or otherwise. Discuss

Contention: While many exert power over Jack and stump his path to self-actualisation and emotional growth, it is by some individuals that Jack realises the goodness of people, and the possibility to have a life characterised by fostering love and pursuing dreams.
Par 1: Dwight and Marian stump Jack’s own formation of his identity, by forcing Jack to accept their negative idea of him onto himself as well. 
Par3: However, Jack learns that those with power do not necessarily abuse their power (Like Dwight and Roy) through Sister James and Mr Howard.
Par2:  Furthermore, the unconditional love Rosemary showers Jack, despite his frequent rebellious behaviour, teaches Jack the possibility of love, and the need to assume the role of the protector towards the weak.
Title: Re: VCE English Question Thread
Post by: HopefulLawStudent on March 07, 2016, 04:53:48 pm
URGENT


I have my English SAC tomorrow and I'm stressed so I'm just casually reading the dictionary in the hopes of relieving some of my stress.

My English teacher's one of those savage VCAA assessors that's been marking probably since dinosaurs were around and he's insanely into contextually correct use of words. *cries*

How do I use the word machinations in a sentence?

More specific: Can I refer to the Medea's plot to kill her sons as a machination? No idea how to use it in a sentence. Would love it if you could provide an example specific to Medea because it's how I'd best be able to understand it.
Title: Re: VCE English Question Thread
Post by: Swagadaktal on March 07, 2016, 05:15:33 pm

 I'm stressed so I'm just casually reading the dictionary in the hopes of relieving some of my stress.
That's an interesting coping strategy :P - extremely nerdy but extremely efficient because you're so proficient with your vocab!
And for machinations, I would use it in a sentence as follows: "Medea' machinations were of an estranged and isolated house wife" - not sure how to relate it to your context though but I would compare (A) machinations to (B)
So you compare to someone (btw I've only read the text once sooo don't judge me"
Title: Re: VCE English Question Thread
Post by: literally lauren on March 07, 2016, 09:37:22 pm
This was given to our class as a bit of analysis practise and i've tried to plan my analysis by identifying key players, as outlined in some of the guides up here.
Here's what I got for key players:
The wealthy- and how they're given extra/special treatment by everyone
Depp- Depp's recent actions illustrate the entitlement the wealthy have
The common person- and how we yearn for wealth yet despise those who have it out of envy
The air travel business- who allow this disparity to continue by providing special benefits to the wealthy and largely disregarding common people. But also their ineptitude for failing to properly check Depp when he entered the country.
These seem perfectly fine to me! :)
Remember there are no completely right answers when it comes to key players - it's all about how well you present your case. If you can go through the piece and find these major ideas that
a) are supported by language features and persuasive devices, and
b) contribute to the author's overall argument
... then you're all good!

It's still worth asking these questions though - just so you know you're on the right track. I'd say perhaps your third K.P. might need refining as that's probably the weakest of the four (i.e. I'm not sure the author is portraying commoners as envious, or at least, he's not doing that often enough for it to be a really substantive sub-argument) but just because it's not what I'd personally focus on, that doesn't make it wrong. You could definitely defend this, and provided your analysis was decent, the assessors wouldn't even think twice about which "key players" you'd've chosen. Partially because "key players" is a thing I kind of made up, but also because they're going to be concentrating on the quality of your analysis; not so much on the structure/formatting :)

I wrote a contention and topic sentences to a prompt. Please let me know if there is anything wrong about it. Because I put in the characters' name in my topic sentences, it sounds like I'm just listing evidence. Please help me if you can! :)

Jack has no positive role models in his life, adult or otherwise. Discuss

Contention: While many exert power over Jack and stump his path to self-actualisation and emotional growth, it is by some individuals that Jack realises other individuals enable Jack to realise the goodness of people, and the possibility to have a life characterised by fostering love and pursuing dreams.
Par 1: Dwight and Marian stump Jack’s own formation of his identity, by forcing Jack to accept their negative idea of him onto himself as well.  --> Wolff shows how negativity and abuse can be transmitted and warp the mindset of an impressionable youth.
Par3: However, Jack learns that those with power do not necessarily abuse their power (Like Dwight and Roy) through Sister James and Mr Howard.   --> However, the text reveals that those with power do not necessarily always abuse their power.
Par2:  Furthermore, the unconditional love Rosemary showers Jack, despite his frequent rebellious behaviour, teaches Jack the possibility of love, and the need to assume the role of the protector towards the weak. --> Furthermore, the loving relationships in the text provide a more optimistic example for Jack.
^edited these a bit to make them more general and topic sentence-y. Notice how even in the last one where I've used Jack's name, I'm still talking about the text as a whole and am focusing more so on broader points as opposed to specific evidence? The stuff that you've got here is a good indication of where your direction should go after the T.S. but you want to make sure you don't start off by saying: "Sister James and Mr Howard are really nice to Jack!" ...that's obviously an exaggeration and your current examples are nowhere near that bad, but you can see what that sentence would be a bad introduction to a discussion, right? Your job in each paragraph is to discuss a point that aids your contention, but beginning that point and making it too narrow means it's harder for you to then zoom out at the end and think about the bigger picture.

Hope that makes sense :)

URGENT


I have my English SAC tomorrow and I'm stressed so I'm just casually reading the dictionary in the hopes of relieving some of my stress.

My English teacher's one of those savage VCAA assessors that's been marking probably since dinosaurs were around and he's insanely into contextually correct use of words. *cries*

How do I use the word machinations in a sentence?

More specific: Can I refer to the Medea's plot to kill her sons as a machination? No idea how to use it in a sentence. Would love it if you could provide an example specific to Medea because it's how I'd best be able to understand it.
Machinations: noun the inner workings or plottings that happen in a person's mind; usually connoting some sinister motives or propensity for calculated schemes

Example:
- Medea's machinations had tragic consequences.
- The machinations of Medea had ramifications for many different characters.
- Euripides shows how Medea's inner machinations brought about damaging consequences for innocent individuals.
- The play revolves around Medea's machination to take revenge (<--though the non-plural usage is less common and sounds a bit weird, so I'd stick with 'machinations' wherever possible. That also opens things up to talk about her motives in a more multifaceted sense, rather than just saying 'yep, she had one machination and that's where her intentions begin and end.')

So, grammatically, you can use it like the word 'thoughts' or 'plans,' but semantically, think of it as referring to the clockwork in her brain that is churning over the situation and helping her decide what to do next.
Title: Re: VCE English Question Thread
Post by: HopefulLawStudent on March 07, 2016, 09:41:07 pm
So machinations is a direct synonym to schemes/plots/whatever? Or is that not on?

My English teacher is going to MURDER me if I can't use my words in the right context. He legit took away 6 marks in a practice essay because he was feeling grumpy and didn't like how often I was screwing up my word usage. *cries*
Title: Re: VCE English Question Thread
Post by: literally lauren on March 07, 2016, 09:48:22 pm
So machinations is a direct synonym to schemes/plots/whatever? Or is that not on?
Yes, but don't use it as a verb or things get muddled. But it comes from an old word for 'contrive' that means 'to bring into being' or 'to bring a plan to fruition,' so it is closely related to the idea of scheming. If it helps, think of it as being like a 'mind-machine' ie. 'Medea's mind-machine (/machinations) are evident in her use of the words...' etc.

My English teacher is going to MURDER me if I can't use my words in the right context. He legit took away 6 marks in a practice essay because he was feeling grumpy and didn't like how often I was screwing up my word usage. *cries*
He sounds fun :P It can be kind of advantageous to have a really brutal marker though; better than someone who gives out full marks like they're tictacs and offers you no constructive criticism :P
Title: Re: VCE English Question Thread
Post by: YellowTongue on March 07, 2016, 10:29:15 pm
What is the best way to approach  a 'how does the author...' essay question?

E.g. "How does Jack Davis use stage directions to explore hope in No Sugar?

I generally find my response to such questions to be very restricted, in that I'm only discussing stage directions and hope. How would I challenge such a prompt?
Title: Re: VCE English Question Thread
Post by: Apink! on March 08, 2016, 09:52:45 am
Hello,
Could I have some assistance? Please check over if I've done good ;D

"I was tempted by the idea of belonging to a conventional family." To what extent was the lack of a conventional family responsible for Jack's issues?

Contention:
Rather than the mere lack of a conventional family, it is Jack’s feverish wish to belong to a conventional family and its subsequent bitter disappointments that are responsible for Jack’s issues concerning his inconsistent identity
Par 1:Despite the glorified depiction of the conventional family in the ‘50s, Wolff asserts that happiness can be pursued without one; thus positioning Jack’s issues as not sourcing from a mere lack of a conventional family.
Par2:  Wolff demonstrates that having a conventional family sometimes only serve to appease the society’s expectation but often does not foster real connection between family members.
Par 3: Ultimately, Wolff shows that the bitterness sourcing from inability to belong to one’s family is what causes Jack to adopt various poses (issue)

Thank you :) :)
Title: Re: VCE English Question Thread
Post by: HopefulLawStudent on March 08, 2016, 12:56:33 pm
Yes, but don't use it as a verb or things get muddled. But it comes from an old word for 'contrive' that means 'to bring into being' or 'to bring a plan to fruition,' so it is closely related to the idea of scheming. If it helps, think of it as being like a 'mind-machine' ie. 'Medea's mind-machine (/machinations) are evident in her use of the words...' etc.
He sounds fun :P It can be kind of advantageous to have a really brutal marker though; better than someone who gives out full marks like they're tictacs and offers you no constructive criticism :P

But I like tictacs...

Thanks Lauren, for your help! You have my eternal gratitude. I didn't end up chucking in machinations. But I survived by English SAC (I think) so I count that as a major success.
Title: Re: VCE English Question Thread
Post by: kimmytaaa on March 08, 2016, 02:37:18 pm
urgent
are there any other words I can use instead of conflict? Whenever I write I end up using conflict, like I use it everytime I have a chance to because if I try another in it doesn't make any sense at all?
Title: Re: VCE English Question Thread
Post by: Callum@1373 on March 08, 2016, 02:42:27 pm
Hello,
Could I have some assistance? Please check over if I've done good ;D

"I was tempted by the idea of belonging to a conventional family." To what extent was the lack of a conventional family responsible for Jack's issues?

Contention:
Rather than the mere lack of a conventional family, it is Jack’s feverish wish to belong to a conventional family and its subsequent bitter disappointments that are responsible for Jack’s issues concerning his inconsistent identity
Par 1:Despite the glorified depiction of the conventional family in the ‘50s, Wolff asserts that happiness can be pursued without one; thus positioning Jack’s issues as not sourcing from a mere lack of a conventional family.
Par2:  Wolff demonstrates that having a conventional family sometimes only serve to appease the society’s expectation but often does not foster real connection between family members.
Par 3: Ultimately, Wolff shows that the bitterness sourcing from inability to belong to one’s family is what causes Jack to adopt various poses (issue)

Thank you :) :)
One thought is whether you should branch out a bit more than just his identity as an issue? Maybe not.

One thing does confuse me a little tho. In Body paragraph one, you've said that it's not the lack of a conventional family that causes his issues because it is possible to be happy without one. However, in body paragraph 3 you're contradicting that by saying 'inability to belong to one's family' i.e by Jack not having a conventional family he generates issues.

Probably a bit harsh but just my 2c  :)
Title: Re: VCE English Question Thread
Post by: HopefulLawStudent on March 08, 2016, 04:34:37 pm
urgent
are there any other words I can use instead of conflict? Whenever I write I end up using conflict, like I use it everytime I have a chance to because if I try another in it doesn't make any sense at all?

http://www.thesaurus.com/browse/conflict

BEWARE: If you use a thesaurus, make sure you understand what the word means (not all words are direct synonyms) and make sure you know how it's used (best way is to google "______ in a sentence" and you get a whole tonne of ways you can use the sentence). Keep in mind, also, that sometimes, the best way to convey an idea is sometimes the simplest way.
Title: Re: VCE English Question Thread
Post by: Apink! on March 08, 2016, 05:10:41 pm
One thought is whether you should branch out a bit more than just his identity as an issue? Maybe not.

One thing does confuse me a little tho. In Body paragraph one, you've said that it's not the lack of a conventional family that causes his issues because it is possible to be happy without one. However, in body paragraph 3 you're contradicting that by saying 'inability to belong to one's family' i.e by Jack not having a conventional family he generates issues.

Probably a bit harsh but just my 2c  :)

Thanks Callum :) I also think I should have branched out a bit more, because Jack's issues aren't only on his identity! Thank you for that, I will try and come up with an idea that will incorporate more of his issues in my essay.

For the contradicting paragraph thing, I meant to say that, Jack's issues are NOT from lacking a conventional family because we see that he is a happy boy even with just Rosemary. But I realise that I have not made my topic sentences clearly enough :) I will try to improve that too.

Thanks! aha it wasn't harsh at all. Thanks for the constructive feedback ;D
Title: Re: VCE English Question Thread
Post by: kimmytaaa on March 08, 2016, 10:13:46 pm
http://www.thesaurus.com/browse/conflict

BEWARE: If you use a thesaurus, make sure you understand what the word means (not all words are direct synonyms) and make sure you know how it's used (best way is to google "______ in a sentence" and you get a whole tonne of ways you can use the sentence). Keep in mind, also, that sometimes, the best way to convey an idea is sometimes the simplest way.

Thanks :)
Title: Re: VCE English Question Thread
Post by: Apink! on March 11, 2016, 08:35:05 am
Hi,
Might be a really stupid question, but I noticed that in This Boy's Life, there is a lot of mention about hamburgers:
-When Rosemary tells Gil and Judd that Jack likes hamburgers
-When Dwight tells Jack, 'are you afraid of a hamburger?' when he is told to pick up the dead beaver but doesn't
-After killing the beaver, Dwight buys Jack a hamburger.

Why so many hamburgers in one book?
Any ideas? (This might be pointless but.. it's fun!) :)
Title: Re: VCE English Question Thread
Post by: Callum@1373 on March 11, 2016, 06:03:40 pm
Hi,
Might be a really stupid question, but I noticed that in This Boy's Life, there is a lot of mention about hamburgers:
-When Rosemary tells Gil and Judd that Jack likes hamburgers
-When Dwight tells Jack, 'are you afraid of a hamburger?' when he is told to pick up the dead beaver but doesn't
-After killing the beaver, Dwight buys Jack a hamburger.

Why so many hamburgers in one book?
Any ideas? (This might be pointless but.. it's fun!) :)
Omg thanks for the observation that is so damn good! I'm stealing this idea no matter what sorry not sorry  :P

Hamburgers --> meat, and meat is probably representative of masculinity, toughness, strength. Wolff i'd imagine includes the fact that Dwight bought him a hamburger as a suggestion to readers that Dwight is trying to 'man' Jack up. Similar to the way that Dwight teaches him the fight moves perhaps...?
Title: Re: VCE English Question Thread
Post by: Apink! on March 11, 2016, 09:16:34 pm
Well Callum,
If I make the observations, you make the interpretations. I'm stealing this too #sorrynotsorry
 ;D ;D :P
Title: Re: VCE English Question Thread
Post by: Callum@1373 on March 11, 2016, 09:51:42 pm
Well Callum,
If I make the observations, you make the interpretations. I'm stealing this too #sorrynotsorry
 ;D ;D :P
Give me $100,000 and I'll refrain from calling the armed forces to pull you out of the SAC on Wednesday  ;)

I'm serious
Title: Re: VCE English Question Thread
Post by: Apink! on March 12, 2016, 08:32:28 am
Give me $100,000 and I'll refrain from calling the armed forces to pull you out of the SAC on Wednesday  ;)

I'm serious

Armed forces? It would take more than just armed forces to take me out on the English SAC ;D
Title: Re: VCE English Question Thread
Post by: huehue on March 13, 2016, 11:59:38 am
Hello! Does anyone have any text response practice prompts for the play Bombshells? If you do, mind posting them? Thanks in advance :)
Title: Re: VCE English Question Thread
Post by: HopefulLawStudent on March 13, 2016, 03:13:09 pm
Hey guys!

I want to do something about the refugee/asylum seeker stuff that's been going on for my oral. But that's about all I've decided. Can I talk about the refugee stuff that's going on in Europe or does it have to be specifically to do with Australia? I want to talk about the whole mass migration stuff from the Middle East (?) and why developed countries need to offer more help as a collective. Is that an okay contention that fits all of the criteria for our oral topics (after September 1, published in AUS media, etc).

I'd ask my English teacher but he scares me.
Title: Re: VCE English Question Thread
Post by: tashhhaaa on March 13, 2016, 10:38:59 pm
Hey guys!

I want to do something about the refugee/asylum seeker stuff that's been going on for my oral. But that's about all I've decided. Can I talk about the refugee stuff that's going on in Europe or does it have to be specifically to do with Australia? I want to talk about the whole mass migration stuff from the Middle East (?) and why developed countries need to offer more help as a collective. Is that an okay contention that fits all of the criteria for our oral topics (after September 1, published in AUS media, etc).

I'd ask my English teacher but he scares me.

Both. Focus on Australia but it's all relevant

& seems good to me
so your contention is essentially "Developed countries need to do more to resolve the refugee crisis"?
Title: Re: VCE English Question Thread
Post by: molecular. on March 18, 2016, 07:40:41 pm
Hey guys my school has just started section B. This year we are doing Invictus. I am going to be doing an expository, most likely a hybrid/expository. Could anyone offer some advice on potential ideas of what to include. Also, how to start or plan this piece.

My SAC is first day back from holidays.

Thanks guys
Title: Re: VCE English Question Thread
Post by: literally lauren on March 19, 2016, 08:47:38 pm
Hey guys my school has just started section B. This year we are doing Invictus. I am going to be doing an expository, most likely a hybrid/expository. Could anyone offer some advice on potential ideas of what to include. Also, how to start or plan this piece.

My SAC is first day back from holidays.

Thanks guys
Firstly, there's this from ages ago that contains a bunch of potential external examples for your perusal, and there are a bunch of links to previous questions regarding Context on page one of this thread. I can also recommend this if you don't know much about the Context criteria (and I can't blame you because the VCAA way of explaining it is nonsensical :P)

But to answer your question, I'd have to know what kind of hybrid piece you were planning on doing since that could be anything from a slightly creative essay to a full blown imaginative feature article or something. The hybrid style is kind of like the grey area in between the black and white of the expository and imaginative/persuasive styles, so what kind of form were you thinking of choosing?

Starting and planning should always be dependent on a prompt though, even if you're just choosing a very general one. I'd recommend something like: 'Strong identities can give us a sense of purpose' or 'Belonging to a group gives one a sense of identity.'
(list here if you want to select your own)

It's really hard to construct a piece without a prompt to base it all around, and talking about 'identity and belonging' in general is really vague, so start with a prompt and work out :)
Title: Re: VCE English Question Thread
Post by: molecular. on March 19, 2016, 10:35:27 pm
Thanks for the guidance Lauren. I was thinking about doing a persuasive expository. This was the prompt I chose from the list - 'society often creates unrealistic and unattainable expectations of individuals.' It sounded quite interesting in correlation to 'Invictus.'

I will start brainstorming ideas.

 ;D
Title: Re: VCE English Question Thread
Post by: rosemaria20 on March 23, 2016, 04:33:22 pm
Hi,
Just a general question.
I got Bs for my first two sacs. (Was expecting higher but anyway) What scores should I aim to get if i want to get a study score of over 35 in English (if its still possible) regardless of how my class goes.

Thanks in advance  :)
Title: Re: VCE English Question Thread
Post by: HopefulLawStudent on March 23, 2016, 09:58:49 pm
It's really hard to tell. It depends on your cohort, your ranking and your exam performance. After all, your SAC scores are subject to moderation from your exam! There's no way of telling you for sure "hey, you MUST get x/30 if you want a 35" or really predicting your end SS so early on in the year imo. At this stage, anything is still possible in theory.
Title: Re: VCE English Question Thread
Post by: tashhhaaa on March 24, 2016, 12:06:41 am
Hi,
Just a general question.
I got Bs for my first two sacs. (Was expecting higher but anyway) What scores should I aim to get if i want to get a study score of over 35 in English (if its still possible) regardless of how my class goes.

Thanks in advance  :)

smash the exam and you should be fine
Title: Re: VCE English Question Thread
Post by: rosemaria20 on March 24, 2016, 03:43:54 pm
It's really hard to tell. It depends on your cohort, your ranking and your exam performance. After all, your SAC scores are subject to moderation from your exam! There's no way of telling you for sure "hey, you MUST get x/30 if you want a 35" or really predicting your end SS so early on in the year imo. At this stage, anything is still possible in theory.

Thank you for the advice, i feel better now
Title: Re: VCE English Question Thread
Post by: girlz_love123 on March 24, 2016, 07:54:46 pm
Hey Guys,
Lauren actually I'm new to all of this but I would like to improve my English skills
so if u could link me to stuff that has language analysis, text response and
comparative essay resources and samples I would really appreciate it.
Thanx
Title: Re: VCE English Question Thread
Post by: HopefulLawStudent on March 25, 2016, 11:27:30 am
So I'm not Lauren (clearly).

BUT...

English resources for can be found here, having briefly skimmed through the AN English board:
English Resources and Sample Essays
ASHA Lectures FREE VCE English Resources Thread

However, as you are a 2017 kid, please check with the new study design to ensure you aren't doing anything that is outside of the course because the study design IS changing.

Unfortunately, the resources exclusively for 2017 kids and beyond are at present pretty lacking (English Resources for 2017 and Beyond), however, if you keep checking in throughout the year, there should be more resources available that are guaranteed to be 100% relevant to you.
Title: Re: VCE English Question Thread
Post by: girlz_love123 on March 25, 2016, 10:04:12 pm
Thanx HopefulLawStudent for replying this was a great help.
Title: Re: VCE English Question Thread
Post by: literally lauren on March 28, 2016, 10:39:36 am
Hey Guys,
Lauren actually I'm new to all of this but I would like to improve my English skills
so if u could link me to stuff that has language analysis, text response and
comparative essay resources and samples I would really appreciate it.
Thanx

Adding on to the above, reading this is probably the best thing you can do since the SD itself is rather confusing. Also, if you go back to the first page in this thread, there's a whole bunch of hyperlinks to previous questions + advice, so if you've got any known troublespots, have a look through that list and see if you can clarify any uncertainties. Sample essays can be found in that resource thread HLS linked, though you should ignore all the Context stuff since that'll be gone by the end of this year.

Let us know if you've got any specific questions regarding the new SD or just general English stuff though, and someone on this thread should be able to help you out :)
Title: Re: VCE English Question Thread
Post by: HopefulLawStudent on March 28, 2016, 06:45:40 pm
RIP to Context. You will be missed.
Title: Re: VCE English Question Thread
Post by: Swagadaktal on March 28, 2016, 07:22:36 pm
RIP to Context. You will be missed.
noo I hate context :(
Title: Re: VCE English Question Thread
Post by: molecular. on March 29, 2016, 06:27:33 pm
Hey guys,

I need some help figuring out some points or different perspectives for this context prompt. I do not want to entirely agree with the prompt because that results in a pretty boring discussion so I was wondering about some different ways I could attack it.

'Belonging can be fulfilling and difficult at the same time'

Thanks!!
Title: Re: VCE English Question Thread
Post by: Cornrow Kenny on March 29, 2016, 07:47:05 pm
For Text Response, how many quotes per body paragraph should you aim for?

I'm aiming for 4/5 (that I am properly analysing), but sometimes I struggle to find one that supports my argument (leaving me with 3), and am left with a shortish essay.

Any other tips for getting up the word count? I am struggling to make it to 800/900 words  :-\

Title: Re: VCE English Question Thread
Post by: Muchos Help on March 29, 2016, 08:20:06 pm
For Text Response, how many quotes per body paragraph should you aim for?

I'm aiming for 4/5 (that I am properly analysing), but sometimes I struggle to find one that supports my argument (leaving me with 3), and am left with a shortish essay.

Any other tips for getting up the word count? I am struggling to make it to 800/900 words  :-\
Enough quotes to justify why your interpretation is 'correct', shouldn't limit yourself to the number of quotes.
Title: Re: VCE English Question Thread
Post by: HopefulLawStudent on March 29, 2016, 09:31:02 pm
For Text Response, how many quotes per body paragraph should you aim for?

I'm aiming for 4/5 (that I am properly analysing), but sometimes I struggle to find one that supports my argument (leaving me with 3), and am left with a shortish essay.

Any other tips for getting up the word count? I am struggling to make it to 800/900 words  :-\



Quantity < Quality

If you're lacking in evidence to support your point, you should consider reviewing the text. If it's just like name-dropping type of quotes (e.g. The "frightening woman" that is Medea employs the middle way to manipulate others) then I'd say it isn't that big of a deal.
Title: Re: VCE English Question Thread
Post by: huehue on March 31, 2016, 07:48:55 pm
Can I use the term "white person" in an essay or is it too colloquial?
Title: Re: VCE English Question Thread
Post by: HopefulLawStudent on April 01, 2016, 09:24:06 pm
Wouldn't it depend on the context in which you're using it?
Title: Re: VCE English Question Thread
Post by: tashhhaaa on April 03, 2016, 11:05:09 pm
Can I use the term "white person" in an essay or is it too colloquial?

I used it in context (ID&B) and that was fine, but I would probably say Caucasian or something in a text response if needed.
Title: Re: VCE English Question Thread
Post by: blacksanta62 on April 04, 2016, 05:24:02 pm
Hey guys, planning my oral response with the criteria sheet in mind and I seem to be confused by one of the requirements in the 17-20 mark area. What does it mean by: "In an oral response, the skillful use of highly appropriate oral language conventions to engage an audience"

I will probably post my ideas on the topic I have chosen after I do more research about it so I can get some feedback from the AN community (someone other than my teacher, mixing it up a bit :D) since I want it to be "logical", "complex" ad "highly appropriate" :).

Thanks!!
Title: Re: VCE English Question Thread
Post by: literally lauren on April 06, 2016, 03:20:29 pm
Hey guys, planning my oral response with the criteria sheet in mind and I seem to be confused by one of the requirements in the 17-20 mark area. What does it mean by: "In an oral response, the skillful use of highly appropriate oral language conventions to engage an audience"

The main thing to worry about here is your use of "oral language conventions," which could be interpreted in two ways. On the one hand you have general language conventions (eg. points that flow into one another logically, good grammar, varied vocabulary (+ rhetorical devices, since this is a persuasive piece) and so on) but you also have features that are specific to oral presentations (eg. tone of voice/inflection, stress/emphasis, volume, posture, eye contact, hand gestures etc.)

My guess is that your teacher wants you to consider both of these sub-categories, but I'd recommend paying particular attention to the latter if you're looking to engage your audience :)
Title: Re: VCE English Question Thread
Post by: One Step at a Time on April 07, 2016, 11:04:05 am
Hi everyone!

Topic for Text Response essay: It is individual acts of defiance that makes Stasiland so engaging. Discuss.

My brain's completely stuck on how to form a contention and topic sentences for this  :-\

Any ideas would be greatly appreciate. Thanks  ;D
Title: Re: VCE English Question Thread
Post by: One Step at a Time on April 07, 2016, 11:05:34 am
*appreciated. Sorry about the typo!  :)
Title: Re: VCE English Question Thread
Post by: literally lauren on April 07, 2016, 11:35:54 am
It is individual acts of defiance that makes Stasiland so engaging. Discuss.

For starters, what does 'defiance' look like in Stasiland? It takes a couple of different forms, so think about the most prominent types/examples of this. Now think about the effect this defiance has on the audience, or, if that's too much of a leap, think about what Funder is saying about these acts of defiance and then think about what she's conveying to readers.

Also, since there's an emphasis on 'individual acts,' you'll need to flesh out specific examples of single characters doing something defiant (which shouldn't be too hard to find since the majority of the text is made up of Funder recounting those individuals' stories). From there you can think about what ties this sort of defiance together; eg. is it always directed at the same ideology? Is it always beneficial/harmful? Is it always admirable/unexpected?

Avoid having a character-by-character breakdown where each paragraph just focusses on one individual's resistance, if possible, since that's a surefire way to make your analysis really superficial. It'd be way better to concentrate on one thematic concern per paragraph, and then substantiate your argument that way.

Hopefully that'll open up your discussion a bit; let me know if you're still having trouble with topic sentences and I'll see if I can help :)
Title: Re: VCE English Question Thread
Post by: One Step at a Time on April 07, 2016, 01:16:23 pm
Thank you so much Lauren, you always appear with your advice when I'm struggling hehe  ;D

I tried to come up with 4 topic sentences and I'd love to hear your opinion on what I have written  :D
1. The play shows how the individual vignettes of defiance against authority have profound and lingering effects on the perpetrators.
2. Throughout the text, individuals have to come to terms with the consequences of their acts of resistance against society's morals.
3. Ultimately, the characters who follow through with their acts of defiance and harness it to fuel their aspirations are rewarded despite the struggles faced.
4. However, this topic neglects the myriad of sources that permeate the narrative which make it engaging.

Some worries below hahah...
I'm not sure if I'm challenging the prompt correctly though?
Do I need to mention the "engaging" part of the prompt? (Tried to do this for the 4th topic sentence but would I be going off topic by doing so?)
Is it clear that the first two topic sentences are basically the same but differentiate between the acts of defiance against different groups?

Can't thank you enough Lauren  ;D ;D
Title: Re: VCE English Question Thread
Post by: One Step at a Time on April 07, 2016, 01:23:06 pm
Just wanted to add that if anyone has any ideas or advice, I'd love it if you could add to this post!! Please help a struggling student out  ;D
Title: Re: VCE English Question Thread
Post by: literally lauren on April 07, 2016, 01:30:00 pm
Thank you so much Lauren, you always appear with your advice when I'm struggling hehe  ;D
I am indeed the Beetlejuice of ATAR Notes. Say 'essay' three times and my spectral essence will be summoned.

I tried to come up with 4 topic sentences and I'd love to hear your opinion on what I have written  :D
1. The play shows how the individual vignettes of defiance against authority have profound and lingering effects on the perpetrators.
2. Throughout the text, individuals have to come to terms with the consequences of their acts of resistance against society's morals.
3. Ultimately, the characters who follow through with their acts of defiance and harness it to fuel their aspirations are rewarded despite the struggles faced.
4. However, this topic neglects the myriad of sources that permeate the narrative which make it engaging.
1. That these vignettes affect the perpetrators doesn't seem wholly relevant, but if you swap out the end of that sentence with something more audience-based, you should be fine. eg. 'The text (not play, btw) shows how the individual vignettes of defiance against authority can be both inspiring, and harrowing.' (<-- this will let you look at the effect on the perpetrators/ other characters (including Anna, if you want, though that might make this paragraph a bit too long) as well as the audience.)
2. Good, and just make sure that by the end of that para, you've touched on how the audience responds to this depiction of coming to terms with things.
3. Ditto^
4. I'd probably advise against having a challenge paragraph that, broadly speaking, says 'yeah, but also here's some other stuff.' It'd be more effective if you could look at how the acts of defiance also make the text somewhat challenging in that it forces the audience to confront the greyness of morality, or how the defiance humanises the characters and exposes readers to their darker, more selfish sides. Ultimately, this still makes the text 'engaging,' but it is engaging in different ways, and for different reasons. <-- that's a safer challenge, imo.

Hopefully that addresses your worries, but let me know if any of that was unclear :)
Title: Re: VCE English Question Thread
Post by: Syndicate on April 07, 2016, 04:19:55 pm
Hey,

Are there any certain factors assessors look for while marking creative pieces (like the use of metaphorical language)?

Thanks
Title: Re: VCE English Question Thread
Post by: HopefulLawStudent on April 07, 2016, 04:35:53 pm
I think they look at pieces hollistically. Instead of just going "oooh you didn't have metaphorical language. -10 points."
Title: Re: VCE English Question Thread
Post by: literally lauren on April 07, 2016, 04:57:25 pm
Are there any certain factors assessors look for while marking creative pieces (like the use of metaphorical language)?

It's all relative. Some pieces would be strengthened by the use of metaphors or language devices, but it's by no means a requirement.

The overall factor they'd be looking for (for Context pieces and the Year 12s of 2016) is your ability to construct a creative piece that responds to the prompt, relates to the text, and expresses interesting ideas about the Context. How you do that is up to you.
(And for the Creative Response to Texts on the new SD for 2017 Year 12s, they'll be looking for the ways in which you can extrapolate ideas and structural devices from one text and implement them yourself in your own creative piece - so a slightly different focus there.)

On the whole, it's not going to hinder you if you chose to employ metaphors and the like, but you shouldn't do so for the sake of raking up extra marks because they're more concerned with what your piece sets out to accomplish instead of the little decisions you make along the way.
Title: Re: VCE English Question Thread
Post by: One Step at a Time on April 07, 2016, 08:45:36 pm
Sorry to interrupt all the context discussion but I just wanted to quickly follow up with what we were talking about above  :D

Seriously a legend Lauren, thank you so much for helping me out with all my questions! I'll definitely keep that in mind- have to say "essay, essay, essay..." to catch your attention hahah  :P

I'm a bit confused about how you would challenge throughout the essay instead of having a challenge paragraph. Would this plan work instead? I've listed what could be explored in each para.

Para 1: How individual acts of defiance are engaging
Para 2: How the greyness of morality is engaging
Para 3: How defiance humanises the characters and exposes readers to their darker, more selfish sides which makes it more engaging

Some new worries...
Does having all challenge paragraphs but 1 work? So in this case, 2 challenge paragraphs. (I've tried to base them on how the text can be engaging for different reasons)
I'm scared I'm going off topic with 3 challenge paragraphs!  :(



Title: Re: VCE English Question Thread
Post by: HopefulLawStudent on April 08, 2016, 11:32:18 am
Think of it this way.

When you answer a question, you can't exactly go:
Yes I agree. (contention)
And here are 3 reasons/paragraphs on why I don't agree and 1 paragraph on why I do agree.

So what we do instead is (from my understanding) have 3 paragraphs why you do agree, for example but within these paragraphs have like little bits where you acknowledge the other side has some good points.

Does that make sense? I tried to make it sound simple and understandable but idk... What sounds simple to me often sounds convoluted and difficult to others. :P
Title: Re: VCE English Question Thread
Post by: melissaromeo on April 09, 2016, 12:51:16 am
can anyone suggest any topics that I can do my oral presentation on? I wanted to do something that would interest my class therefore I'm trying to avoid the common topics that come up every year like lowering the driving age, vaccinations etc.
Thanks!
Title: Re: VCE English Question Thread
Post by: One Step at a Time on April 09, 2016, 01:08:57 am
Hi HLS   :D

Thanks for your reply! I don't quite understand where you're coming from though- and it's definitely me, not you! English is not my forte ;D

Actually, I didn't want my contention to simply be "Yes I agree." I wanted to be something along the lines of this : "Although individual acts of defiance make the text engaging, ultimately there are also other factors which render the text engaging in different ways."

I've tried to have the 1 paragraph agreeing and the others disagreeing to stick with the contention, but as you said, I don't think it's the right approach...  :-\

What ideas could the paragraphs be based on instead of what I listed in my last post?  :D

Title: Re: VCE English Question Thread
Post by: HopefulLawStudent on April 09, 2016, 07:50:57 am
Sorry... I didn't realise this was like a follow up question to a previous one. I think I've screwed up. Ignore me...
Title: Re: VCE English Question Thread
Post by: One Step at a Time on April 09, 2016, 02:49:31 pm
Hahah all good HLS, thanks for your help anyway  :D

If anyone has any tips on what to do about the topic sentences (starts a few posts back), I would be so so grateful if you could leave a post  :P
Title: Re: VCE English Question Thread
Post by: literally lauren on April 10, 2016, 01:52:44 pm
"essay, essay, essay..."

(http://www.animalanswers.co.uk/wp-content/uploads/2014/11/giphy-9.gif)

I'm a bit confused about how you would challenge throughout the essay instead of having a challenge paragraph. Would this plan work instead? I've listed what could be explored in each para.

Para 1: How individual acts of defiance are engaging
Para 2: How the greyness of morality is engaging
Para 3: How defiance humanises the characters and exposes readers to their darker, more selfish sides which makes it more engaging

Some new worries...
Does having all challenge paragraphs but 1 work? So in this case, 2 challenge paragraphs. (I've tried to base them on how the text can be engaging for different reasons)
I'm scared I'm going off topic with 3 challenge paragraphs!  :(

Okay, you seem a few steps ahead of the average student, so forgive me if this is just reinforcing what you already know but I figured I may as well go through the whole process for the benefit of others.

Let's backtrack a bit and return to the prompt for a second.

'It is individual acts of defiance that makes Stasiland so engaging. Discuss.'

What you definitely shouldn't do is completely agree or completely disagree with no challenging whatsoever. I'm guessing you probably already know this, but just to reinforce it... an essay structure like:
   P1: Miriam's defiance is very engaging to readers
   P2: The defiance characters show against the Stasi is also engaging
   P3: Funder engages readers by showing how characters can be defiant against laws and expectations
...or something like that would be incredibly flawed and weak. All of these "arguments" are basically boiling down to just 'YES. PROMPT IS RIGHT. LET ME REPEAT THAT AGAIN USING SLIGHTLY DIFFERENT WORDS.' And that's... not a valid argument, let alone the kind of strong, well-supported points you're expected to construct at a VCE level. As such, complete agreement is a bad idea, and completely disagreeing is, if anything, even worse. For a prompt like this, it's really hard to argue 'No. Individual acts of resistance don't make Stasiland engaging' because how the hell do you show something doesn't lead to something else? What will invariably happen is that you'd end up 'topic dodging' by saying either:
       'No, it's the sense of camaraderie and perseverance that makes Stasiland engaging.'
or
       'No, the individual acts of defiance actually make Stasiland unrelatable because of how confronting they can be.
And in either case, you'd be neglecting to talk about a key element of the prompt because you'd've supplemented your own focus and effectively 'dodged' the topic. 100% disagreeing would mean that you'd have to stick with those key terms the whole way through and just continually talk about how they don't connect with one another, and that's a pretty unrealistic approach.

So we're left with the choice to either mostly agree, or mostly disagree. And let's assume we're mostly agreeing for this prompt seeing as that's probably the easiest interpretation to argue for in this case.

Here's what most students in the state would do:
   P1: Yes, the individual acts of resistance make the text engaging, and here's an example of that.
   P2: Yes, the individual acts of resistance make the text engaging, and here's another example of that.
   P3: However, the individual acts of resistance don't always make the text engaging, and here's an example of that.

But what are you left with at the end? Your conclusion has to try and pick up the pieces and tie things together into some semblance of a sensible interpretation, and if all you've done is let examples drive the discussion, you'll have very little chance to impress the assessors on the basis of your ideas.

My recommendation: you devise a contention that's more holistic, meaning that you have an overarching interpretation that contains a bit of a challenge but still expresses a clear point of view in response to the prompt.

The best way to do this is to use the following format:
'Although    X   , ultimately    Y   '
...whereby X = some form of challenge, and Y = your primary line of argumentation.

For that prompt above, for example, you might argue that:
Although the instances of defiance depicted in Stasiland complicate the text's portrayal of morality, ultimately these actions are an integral part of Funder's attempts to communicate her fascination with the engaging stories of those affected by the Wall.

Then, within each body paragraph, you explore how this is achieved. It's kind of like each individual B.P. will contain an element of 'challenging' the prompt, but you're also doing more than just blatantly agreeing/supporting it. When I was in Year 12, I used to like thinking of it as reconfiguring the prompt into a more substantive interpretive point - kind of like the assessors had provided this vague outline of an idea, and then I'd dismantled and rebuilt it into a proper contention :P

Now let's look at where your contention is at:
I didn't want my contention to simply be "Yes I agree." I wanted to be something along the lines of this : "Although individual acts of defiance make the text engaging, ultimately there are also other factors which render the text engaging in different ways."

Structurally, you've got the right idea, but if you want to mostly disagree, you need to do more than just say 'there's other stuff that makes the text engaging' because your argument hinges on EXAMPLES, not IDEAS!

Compare the following:
Defiance does make the text engaging, but it also complicates the notions of morality and justice throughout.
vs.
Defiance does make the text engaging, but defiance does other stuff too.

or
Defiance does make the text engaging, but this is primarily because of the context of that defiance and the fact that the characters being defiant do so for valid and justifiable reasons.
vs.
Defiance does make the text engaging, but there's other stuff that makes it engaging too.

I'm not saying your intro/contention has to be highly specific in terms of which evidence you'll explore; rather that you need to give us a sense of where your ideas are going. The challenge you've got is a good start, but if you are going to disagree, be careful not to turn the discussion into a topic dodge by saying 'no, and here's some different stuff.'

So the reason you may be a bit unsure of your topic sentences is that your contention runs the risk of veering into less than relevant territory. That's why, for the original T.S.s you listed, you'd be best to modify your second and third one to ensure you're still looking at the ideas of defiance and engaging-ness because every paragraph has to touch on both of those two ideas to some extent - even if you're disagreeing.

For these versions:
Para 1: How individual acts of defiance are engaging
Para 2: How the greyness of morality is engaging
Para 3: How defiance humanises the characters and exposes readers to their darker, more selfish sides which makes it more engaging
Few questions/notes:
   - How would paragraphs 1 and 3 differ; what would you talk about in 1 that wouldn't be covered by 3? It seems like there's a bit of overlap at the moment since 3 is almost like a subset of 1, so making 1 more specific might help avoid that potential redundancy
   - What's the connection between the greyness of morality and the notion of defiance? I know it exists, you know it exists, but if it's not in your T.S. then it's tough to judge what you've written as being accurate. (And there was probably a lot of thinking and planning in your head that you didn't explicitly give away in your T.S. which is totally fine, but I'm just making sure that you know what the assessors would need to see.)
   - It seems like you're agreeing more so than disagreeing here(?) though because you've listed general ideas rather than argumentative points (which is also totally fine for the sake of your own planning - I actually preferred doing it that way sometimes) I'm not 100% sure.
   - What you've got for 3 is a pretty strong point, but it doesn't really align with your current contention, so you'll have to make a decision as to which one you're going to alter.

And to address these concerns:
Does having all challenge paragraphs but 1 work? So in this case, 2 challenge paragraphs. (I've tried to base them on how the text can be engaging for different reasons)
I'm scared I'm going off topic with 3 challenge paragraphs!  :(
You can mostly challenge/disagree with prompts, but not all prompts would let you do this. You don't have to just agree as your default option because there will be other prompts that actively invite disagreement (eg. 'None of the characters in Stasiland are sympathetic. Discuss.')

But I think it will be more helpful if you go from thinking about essays in terms of:
- Contention: disagreeing with the prompt
- P1: agreeing with the prompt
- P2: challenging the prompt
- P3: challenging the prompt

...to...
- P1: backing up contention
- P2: backing up contention
- P3: backing up contention

The assessors want to see you mount a case for your interpretation, and simply agreeing/disagreeing is only the starting point for doing that, so if you're able to shift your mindset from thinking about 'prompt-->paragraphs' to 'prompt-->contention-->paragraphs,' you'll be in a much better position.

Let me know if any of that didn't make sense! :)
Title: Re: VCE English Question Thread
Post by: literally lauren on April 10, 2016, 03:28:23 pm
can anyone suggest any topics that I can do my oral presentation on? I wanted to do something that would interest my class therefore I'm trying to avoid the common topics that come up every year like lowering the driving age, vaccinations etc.
Thanks!

See: here.

It can be good to avoid the really overdone topics that teachers are just utterly sick of, but you don't necessarily have to dodge anything that's remotely popular. Common topics can still be presented upon in an interesting and persuasive, manner if you're interested in the subject matter :)
Title: Re: VCE English Question Thread
Post by: knightrider on April 11, 2016, 02:29:28 am
what would be some good things to analyse in this image?(issue is about the guilty verdict of drug taking and subsequent 12 month banning of essendon players and whether this is fair or not )

http://cdn.newsapi.com.au/image/v1/36391f6e4fb4c38801fa9bd100d4e747?width=650

Title: Re: VCE English Question Thread
Post by: Cornrow Kenny on April 11, 2016, 09:20:22 pm
what would be some good things to analyse in this image?(issue is about the guilty verdict of drug taking and subsequent 12 month banning of essendon players and whether this is fair or not )

http://cdn.newsapi.com.au/image/v1/36391f6e4fb4c38801fa9bd100d4e747?width=650
Personally I think the illustrator is implying how the punishment was not harsh enough - The AFL are saying Essendon are "cleared for take off" (they can still play) despite ASADA in possession of evidence (the briefcases), that proves they were guilty of doping.
Title: Re: VCE English Question Thread
Post by: kimmytaaa on April 12, 2016, 09:16:36 am
need help with my oral
Title: Re: VCE English Question Thread
Post by: literally lauren on April 12, 2016, 02:20:59 pm
what would be some good things to analyse in this image?(issue is about the guilty verdict of drug taking and subsequent 12 month banning of essendon players and whether this is fair or not )

http://cdn.newsapi.com.au/image/v1/36391f6e4fb4c38801fa9bd100d4e747?width=650

Disclaimer: I know nothing about football and have been deliberately ignoring all these drug sagas because sportspeople on performance enhancing drugs are way less interesting than sportspeople on hallucinogenic drugs, so I don't care. Hence, this may be a radical misinterpretation of whatever is going on at the moment...

I get the sense the cartoonist is mocking the AFL somewhat since the guy in the control tower is like 'yep, you're all good' and yet we can see that the plane is full of holes and isn't really in any state to fly (i.e. the team's legal defence is 'full of holes' because of how inadequate it is; thus the team is in no state to 'fly'/ play footy.) But whether this reflects worse on the team or the AFL... I'll leave that for you to decide since I'm not entirely sure where the culpability lies in this case. As for the ASADA rep. in the foreground, I definitely agree with Cornrow in that the stuff spilling out of his briefcase would seem to indicate the fact that there's so much evidence against Essendon that it literally can't be contained by ASADA. You might even analyse how that figure looks somewhat incredulous at the idea of the plane being able to take off despite the mountain of evidence he has that suggests they shouldn't be flying/playing. That said, if the efficacy of ASADA is at all relevant to this issue, you may also be able to critique how this guy is rocking up late with briefcases spilling everywhere as an indictment of his own ineptitude --> ∴ Essendon being allowed to play is partially the fault of ASADA's inability to mount a strong enough case (???)

^ignore anything that seems wrong or unsubstantiated here - can't stress enough just how little I know about this story. Seriously, I thought the Fitzroy team still existed until, like, two weeks ago :3

need help with my oral
What exactly do you need help with?
Title: Re: VCE English Question Thread
Post by: knightrider on April 13, 2016, 03:34:01 am
Personally I think the illustrator is implying how the punishment was not harsh enough - The AFL are saying Essendon are "cleared for take off" (they can still play) despite ASADA in possession of evidence (the briefcases), that proves they were guilty of doping.

Disclaimer: I know nothing about football and have been deliberately ignoring all these drug sagas because sportspeople on performance enhancing drugs are way less interesting than sportspeople on hallucinogenic drugs, so I don't care. Hence, this may be a radical misinterpretation of whatever is going on at the moment...

I get the sense the cartoonist is mocking the AFL somewhat since the guy in the control tower is like 'yep, you're all good' and yet we can see that the plane is full of holes and isn't really in any state to fly (i.e. the team's legal defence is 'full of holes' because of how inadequate it is; thus the team is in no state to 'fly'/ play footy.) But whether this reflects worse on the team or the AFL... I'll leave that for you to decide since I'm not entirely sure where the culpability lies in this case. As for the ASADA rep. in the foreground, I definitely agree with Cornrow in that the stuff spilling out of his briefcase would seem to indicate the fact that there's so much evidence against Essendon that it literally can't be contained by ASADA. You might even analyse how that figure looks somewhat incredulous at the idea of the plane being able to take off despite the mountain of evidence he has that suggests they shouldn't be flying/playing. That said, if the efficacy of ASADA is at all relevant to this issue, you may also be able to critique how this guy is rocking up late with briefcases spilling everywhere as an indictment of his own ineptitude --> ∴ Essendon being allowed to play is partially the fault of ASADA's inability to mount a strong enough case (???)

^ignore anything that seems wrong or unsubstantiated here - can't stress enough just how little I know about this story. Seriously, I thought the Fitzroy team still existed until, like, two weeks ago :3
What exactly do you need help with?

Thanks so much literally lauren  :) and Cornrow Kenny  :). Really appreciate it !!
Title: Re: VCE English Question Thread
Post by: kimmytaaa on April 13, 2016, 08:33:27 am
Disclaimer: I know nothing about football and have been deliberately ignoring all these drug sagas because sportspeople on performance enhancing drugs are way less interesting than sportspeople on hallucinogenic drugs, so I don't care. Hence, this may be a radical misinterpretation of whatever is going on at the moment...

I get the sense the cartoonist is mocking the AFL somewhat since the guy in the control tower is like 'yep, you're all good' and yet we can see that the plane is full of holes and isn't really in any state to fly (i.e. the team's legal defence is 'full of holes' because of how inadequate it is; thus the team is in no state to 'fly'/ play footy.) But whether this reflects worse on the team or the AFL... I'll leave that for you to decide since I'm not entirely sure where the culpability lies in this case. As for the ASADA rep. in the foreground, I definitely agree with Cornrow in that the stuff spilling out of his briefcase would seem to indicate the fact that there's so much evidence against Essendon that it literally can't be contained by ASADA. You might even analyse how that figure looks somewhat incredulous at the idea of the plane being able to take off despite the mountain of evidence he has that suggests they shouldn't be flying/playing. That said, if the efficacy of ASADA is at all relevant to this issue, you may also be able to critique how this guy is rocking up late with briefcases spilling everywhere as an indictment of his own ineptitude --> ∴ Essendon being allowed to play is partially the fault of ASADA's inability to mount a strong enough case (???)

^ignore anything that seems wrong or unsubstantiated here - can't stress enough just how little I know about this story. Seriously, I thought the Fitzroy team still existed until, like, two weeks ago :3
What exactly do you need help with?
Hi Lauren
ill PM you
Title: Re: VCE English Question Thread
Post by: One Step at a Time on April 13, 2016, 02:21:18 pm
You are incredible Lauren, thank you so much! I think I understand the gist of what you're saying and hopefully that'll carry through to my writing  :D

So it'd be wiser to have the contention itself as a sort of challenge, rather than having challenge paragraphs for less topic dodging and potential contradiction?  :P

But now I'm confused about how to change my contention so that my 3rd topic sentence can stay as a topic sentence! From there, how would you divide the contention into the 3 arguments/ form 3 topic sentences so that there is a new idea in each paragraph. Basically, I just want to say that by making the contention a challenge, how do I go about finding 3 topic sentences which support it?

Thanks once again Lauren  :D One step closer hahah

Title: Re: VCE English Question Thread
Post by: knightrider on April 13, 2016, 08:18:00 pm
In a language analysis is it still possible to achieve a 9 or 10 without mentioning tone at all?
Title: Re: VCE English Question Thread
Post by: Champ101 on April 14, 2016, 07:38:01 am
In a language analysis is it still possible to achieve a 9 or 10 without mentioning tone at all?

Hey knightrider, there is no prerequisite to mention tone - examiners expect you to reflect and express your understanding on the methods, meta-language and techniques used to persuade the reader to share his point of view and why he uses specific language. For instance a shift in tone is just one of the many techniques.
 Incidentally, I would say that it's not worth commenting on tone and plucking out a memorised word unless it's relevant, however, it's still important to give those nitpicky assessors an opportunity to give you marks, so if you find the piece isn't overtly or clearly Vitriolic or lacking in loaded language, just comment on a few instances here and there. Also, to avoid your piece from appearing formulaic i.e. 'in an opinion piece entitle ... John smith asserts in a didactic tone...' try and turn your tonal words into adverbs - for example - 'John smith didactically emphasises the importance of... and then continue with how he tries to position his readership - 'evoking feelings of guilt to corall the reader into...
Title: Re: VCE English Question Thread
Post by: literally lauren on April 14, 2016, 08:23:30 am
So it'd be wiser to have the contention itself as a sort of challenge, rather than having challenge paragraphs for less topic dodging and potential contradiction?  :P
YES!!! ^THIS x 107%

But now I'm confused about how to change my contention so that my 3rd topic sentence can stay as a topic sentence! From there, how would you divide the contention into the 3 arguments/ form 3 topic sentences so that there is a new idea in each paragraph. Basically, I just want to say that by making the contention a challenge, how do I go about finding 3 topic sentences which support it?
K, so it's a bit of a shift in thinking, but look at it this way:
Once you turn the prompt into your contention, you don't need the prompt anymore!
From that point on, your whole essay (esp. your T.S.s) are geared towards supporting that contention.


So it's kind of like your T.S.s will 'agree' with your contention, if you want to think of it that way. And you never 'disagree' with your contention because that would be silly. You can have the occasional line within your B.P.s like 'Though one may interpret Character X's actions as a form of weakness, the author seems to celebrate his unique strength in the form of resilience in seemingly impossible circumstances...' where you acknowledge potential alternate interpretations, but the main gist of your argument will always be geared towards your primary point.

General rule: to turn your contention into three (or more) sub-arguments, you need to make three (or more) assertions that support this main point.

For example:
Prompt: In Stasiland, no character is immune from the effects of the Wall. Discuss.
Contention: Although the characters in Stasiland are irrefutably altered by the social climate of Germany and the sense of division that pervaded the nation, ultimately, these effects manifest themselves in vastly different ways, and it is this variation in how they respond to their circumstances which Funder propounds to be a true test of character.
Paragraph 1: The Wall's physical presence inhibits characters' freedom and right to self-determination, and even after the Wall is gone, the sense of a lingering divide is palpable.
Paragraph 2: However, the Wall also takes a psychological toll, particularly for those characters who
Paragraph 3: Moreover, Funder reveals how the Wall became a metonym for national division and the source of the characters' suffering, making it virtually impossible to escape.
Conclusion: Hence, by associating the Wall as an enforcer of isolation and detachment, Funder implies that it is only once characters are able to reconcile with one another, as well as their pasts, that they can hope to overcome the ramifications it had on their lives.

Note this is by no means the only way to break down this contention, nor is this contention the only plausible way to argue this prompt. Plus, as I kind of mentioned before, it can be difficult to get a sense of someone's rationale for their argument when you're only being given glimpses into parts of their essay, but as another general rule:
If you read the {intro + topic sentences + concluding sentences + conclusion} of someone's essay, you should know exactly what they're arguing, and there should be no glaring contraditions. You don't have to know why they're arguing it (i.e. if you can still say 'prove it!' at the end of every T.S. that's a good thing because it'll be the goal that paragraph needs to accomplish by using evidence.) If, however, you do read the whole paragraph and feel as though the points haven't been substantiated, that's where the B.P. discussion has failed in what it set out to do.

Basically, the above breakdown makes sense to me, but that's at least partially because I wrote it :P So if it doesn't make total sense to you, don't stress; the bulk of the discussion is where I'd need to justify everything I've outlined, and so long as you've got a sense for the direction of the argument, you should be okay :)

Thanks once again Lauren  :D One step closer hahah
That's the spirit!

In a language analysis is it still possible to achieve a 9 or 10 without mentioning tone at all?
As Champ mentioned, you absolutely can. But you shouldn't plan to leave out tone because it's 'unimportant.' It can still be helpful in scoring highly, especially if you're able to combine tonal analysis with some other analysis of language (i.e. the author uses an aspirational tone coupled with superlative language in describing that "Australians [could be] the best in the world" at fighting corruption, thus encouraging readers to strive for this "best" case scenario wherein their country is renowned for its strong stance against fraud and malfeasance.)
Title: Re: VCE English Question Thread
Post by: knightrider on April 15, 2016, 12:49:26 am
Hey knightrider, there is no prerequisite to mention tone - examiners expect you to reflect and express your understanding on the methods, meta-language and techniques used to persuade the reader to share his point of view and why he uses specific language. For instance a shift in tone is just one of the many techniques.
 Incidentally, I would say that it's not worth commenting on tone and plucking out a memorised word unless it's relevant, however, it's still important to give those nitpicky assessors an opportunity to give you marks, so if you find the piece isn't overtly or clearly Vitriolic or lacking in loaded language, just comment on a few instances here and there. Also, to avoid your piece from appearing formulaic i.e. 'in an opinion piece entitle ... John smith asserts in a didactic tone...' try and turn your tonal words into adverbs - for example - 'John smith didactically emphasises the importance of... and then continue with how he tries to position his readership - 'evoking feelings of guilt to corall the reader into...


As Champ mentioned, you absolutely can. But you shouldn't plan to leave out tone because it's 'unimportant.' It can still be helpful in scoring highly, especially if you're able to combine tonal analysis with some other analysis of language (i.e. the author uses an aspirational tone coupled with superlative language in describing that "Australians [could be] the best in the world" at fighting corruption, thus encouraging readers to strive for this "best" case scenario wherein their country is renowned for its strong stance against fraud and malfeasance.)

Thanks so much Champ101 and literally lauren for your repsonses !! :)
Title: Re: VCE English Question Thread
Post by: Yakooza123 on April 15, 2016, 04:15:46 pm
Need help tackling this prompt: "When confronted by Conflict, it is always better to take a side". Any advice would be of much appreciation.
Title: Re: VCE English Question Thread
Post by: One Step at a Time on April 15, 2016, 04:39:22 pm
Hi Lauren  ;D

Thank you so much, I think I understand where you are coming from.

If I'm not wrong, the contention itself is the challenge and all the TS are the arguments stemming from the contention.

I wrote the contention and 4 topic sentences as practice for Gattaca, which I studied last year, to the topic below.
It is individual acts of defiance that makes Gattaca so engaging. Discuss.

Contention: Although the vignettes of individual defiance depicted complicate the film's portrayal of morality, ultimately these actions are an integral part of Niccol's attempts to communicate his fascination with the engaging stories of individuals who are affected by the defiance of social construct.
1. Niccol delves into the way in which the omnipresence of defiance against authority renders Gattaca engaging.
2. Coalesced with a fixation on the ubiquity of defiance to oppose authority, viewers are exposed to individuals coming to terms with the corollary effects of their acts of resistance against society’s morals in an engaging manner.
3. Whilst the struggles faced in satiating pursuits are inevitable in a controlled society, Niccol may foster audience engagement in crafting those who harness such defiance to fuel their aspirations, as ultimately being rewarded.
4. Despite the engaging nature of the defiance perpetrated by individuals, viewers are also confronted with the conflicting notions of defiance and morality which permeate the plot, hence divulging Gattaca as engaging in a different manner.

Would love to hear your feedback! I feel like I'm getting the gist of what to do when given the topic  :D
Title: Re: VCE English Question Thread
Post by: Syndicate on April 16, 2016, 05:40:26 pm
Hey,

Just have a quick question. Is it good to use colloquial language in creative pieces?

For example: My family will pull off another embarrassing moment.

I am just somewhat confused as my criteria states:
Spoiler
Highly expressive, fluent and coherent written language that employs the skilful and accurate use of appropriate conventions for stylistic effect. <--any recommendations?

Thanks,
Syndicate
Title: Re: VCE English Question Thread
Post by: Swagadaktal on April 16, 2016, 09:50:33 pm
Hey,

Just have a quick question. Is it good to use colloquial language in creative pieces?

For example: My family will pull off another embarrassing moment.

I am just somewhat confused as my criteria states:
Spoiler
Highly expressive, fluent and coherent written language that employs the skilful and accurate use of appropriate conventions for stylistic effect. <--any recommendations?

Thanks,
Syndicate
Haven't done context in a while so I'm a bit rusty here - Lauren (or anyone else) could probs give you a more in-depth answer, but a part of the context criteria is something along the lines of "using language that is specific to the form and audience" (maybe something else but i forgot). So if colloquial language relates to your form or your audience then sure. I.E a personal letter to a loved one doesn't have to be formal (it'd be quite weird if you used formal analytical language to your lover) - yeah this is about the extent of my understanding soo

hope this helps
Title: Re: VCE English Question Thread
Post by: tashhhaaa on April 16, 2016, 10:48:11 pm
Hey,

Just have a quick question. Is it good to use colloquial language in creative pieces?

For example: My family will pull off another embarrassing moment.

I am just somewhat confused as my criteria states:
Spoiler
Highly expressive, fluent and coherent written language that employs the skilful and accurate use of appropriate conventions for stylistic effect. <--any recommendations?

Thanks,
Syndicate

to my understanding it's fine since I wasn't ever penalised for it (obviously in the SAC you can do an explanatory piece/statement of intention/whatever so you can explain why you're using that language)

basically

So if colloquial language relates to your form or your audience then sure. I.E a personal letter to a loved one doesn't have to be formal (it'd be quite weird if you used formal analytical language to your lover)

I'd argue that colloquialisms could make your writing more realistic since we don't all use proper syntax in conversation. Also, your characters might be more interesting to read about if they don't all sound like they're reading a textbook :p
Don't overthink it

Need help tackling this prompt: "When confronted by Conflict, it is always better to take a side". Any advice would be of much appreciation.

The way I always broke down prompts was a bit basic... Nevertheless, the following method worked for me:
1. What are the 'key words' of the prompt? Also note any absolutes/qualifiers like 'never' and 'always', as well as words like 'better', 'damaging', 'harmful', whatever
In this case we've got confronted, better and side as keywords while always is an absolute
2. Think about the key words again -- what does 'confronted' mean in this prompt? What could it mean? eg. is the conflict a challenge to confront? A burden? Something to deal with or handle? Or is it just a synonym for 'faced'? o.O What it means is up to you though ;)
3. Note any connections between the words. For me, confronted and side call to mind an argument, disagreement etc, but it could for everyone else too -- any ideas are welcome.
A few ideas I just had (I actually did identity and belonging so I hope this is still relevant):
- Conflict between who someone is (eg. culture, sexual orientation) and who they 'should' be (expectations), herein the side being what/who they choose to be
- A scenario where taking sides is/isn't advantageous
- An expository essay with some examples of real life stuff when confronting conflict did or did not work (yes this is very superficial but hopefully you catch my drift)
4. Keep thinking, probably try to challenge the prompt at this stage -- something like "What if I don't want to confront conflicts? What if confrontation is bad? What are sides anyway? What if the best way to approach conflict is by doing your own thing and not taking any side? susefoiewifguiesiues"

not sure if I confused you even more but try out my weird thinking pathway and you might get some ideas ;)

someone plz pull me up if I give dodgy advice lol
Title: Re: VCE English Question Thread
Post by: literally lauren on April 17, 2016, 10:38:55 am
Need help tackling this prompt: "When confronted by Conflict, it is always better to take a side". Any advice would be of much appreciation.
Tasha covered this nicely, so all I have to add is ASK "WHY?" It's not the most extensive of processes, but if you're ever lost in Context, look at the prompt or the last sentence you've written and ask "but why??"

So, apparently it's always better to take sides in Conflict? --> WHY?
Or, if you're mostly disagreeing, then I guess it's not always a good idea to pick a side. --> WHY?

Then, if you can come up for an answer for that question and get stuck again, repeat the process. Just keep asking "why" until you get to a point where you're like '...because...that's just how things are!!@!' At that point, you've gone far enough. Everything up till that point is stuff you can include in your essays though, because the assessors like to see students following their train of thought rather than making random, unsubstantiated claims about conflict and the way the world is.

Can be kind of frustrating at first, but once you're used to it, this is a fairly foolproof method for unpacking your ideas :)

If I'm not wrong, the contention itself is the challenge and all the TS are the arguments stemming from the contention.
Yep!
I wrote the contention and 4 topic sentences as practice for Gattaca, which I studied last year, to the topic below.
It is individual acts of defiance that makes Gattaca so engaging. Discuss.
Hmm... prompt sounds familiar :P
Contention: Although the vignettes of individual defiance depicted complicate the film's portrayal of morality, ultimately these actions are an integral part of Niccol's attempts to communicate his fascination with the engaging stories of individuals who are affected by the defiance of social construct.
1. Niccol delves into the way in which the omnipresence of defiance against authority renders Gattaca engaging.
Wording is a little bogged down here, but I get what you're going for. The text shows how omnipresent defiance is, which makes it engaging, yeah? It sounds a bit odd to say 'Niccol delves into how X renders the film engaging' since that's kind of like saying 'the author explores how readers enjoy the novel.' And be careful with how you're using 'vignettes.' I'd say it's appropriate in Stasiland, but Gattaca has a more straightforward plot. Aside from those little concerns, this is all good.
2. Coalesced with a fixation on the ubiquity of defiance to oppose authority, viewers are exposed to individuals coming to terms with the corollary effects of their acts of resistance against society’s morals in an engaging manner.
Ditto regarding your word choices; 'coalesced' doesn't quite work here, but this is also fine in terms of relevance.
3. Whilst the struggles faced in satiating pursuits are inevitable in a controlled society, Niccol may foster audience engagement in crafting those who harness such defiance to fuel their aspirations, as ultimately being rewarded.
Here, 'satiating' is the odd word, and be careful not to talk about what the author 'may' be doing. Same rule goes for Language Analysis - just assume the author is successful and talk about what they intended to achieve - don't evaluate whether or not it works.
4. Despite the engaging nature of the defiance perpetrated by individuals, viewers are also confronted with the conflicting notions of defiance and morality which permeate the plot, hence divulging Gattaca as engaging in a different manner.
And this time 'divulging' is your weird word. Divulging has to be used in the sense of 'X divulges Y to Z' as in, 'He divulges his secrets to me.' You seem to be using it as a synonym for 'suggests' or 'positioning' which isn't 100% right even if most people would know what you meant.
I feel like I'm getting the gist of what to do when given the topic  :D
Good to hear! If you do get a bit frustrated with this process - as most students will at some point - just know that the fact that you're able to work through these problems and reformulate your arguments is putting you way above the vast majority of the state who just blindly follow the 'agree+agree+disagree' or 'TEEL' methods without even thinking :P

Hey,

Just have a quick question. Is it good to use colloquial language in creative pieces?

For example: My family will pull off another embarrassing moment.

I am just somewhat confused as my criteria states:
Spoiler
Highly expressive, fluent and coherent written language that employs the skilful and accurate use of appropriate conventions for stylistic effect. <--any recommendations?

Thanks,
Syndicate
Believability matters. If your character would believable use this phrase, then you can use it in your writing.

In fact, it's often worse to stick with a ridiculously formal/ 'essay-ish' tone if you're writing a creative or hybrid piece. Otherwise, you end up with clunky lines like: 'My family are indelibly flawed, and as a consequence of their somewhat neglectful parenting, I am left with an insufficient understanding of the world.' No one talks like that, right? So you shouldn't write like that if you're trying to encapsulate a believable voice for your character :)
Title: Re: VCE English Question Thread
Post by: Syndicate on April 17, 2016, 12:21:59 pm
Thank you all for your valuable responses :)
Title: Re: VCE English Question Thread
Post by: michael leahcim on April 17, 2016, 07:22:09 pm
Hi guys, I was wondering how you might go about doing an encountering conflict repository for revision. I'm a little stuck on how I should outline my evidences and how I might organise it, so that by the end of the year it wouldn't be so much of a hassle.

Thanks!
Title: Re: VCE English Question Thread
Post by: literally lauren on April 17, 2016, 08:56:30 pm
Hi guys, I was wondering how you might go about doing an encountering conflict repository for revision. I'm a little stuck on how I should outline my evidences and how I might organise it, so that by the end of the year it wouldn't be so much of a hassle.

Thanks!

Most of my students have found a simple word doc that goes through each example to be sufficient. Then under each one, you'll have a brief description (in case you come back to one after a few months and think wtf was I on about??) and a few key dot points for discussion purposes. I'd also recommend linking each example to some of the major 'themes' within your Context (e.g. for Conflict, you might have areas like 'cause vs. consequence,' 'fear/negative effects' 'strength/growth/positive effects,' 'our response to conflict and our true values' etc.) Basically go through as many prompts as you can find (& there's a fairly conclusive list here unless your school has a bunch of weird ones) and collate all the similar words and ideas you come across. That way, if you get a SAC/exam prompt like 'Conflict brings out the best in people' you go to your mental happy place and think 'k, cool, so I can use examples X, Y, and Z because they relate to our response to conflict and the notion of positive consequences.'

You'll usually have to end up merging categories since it's rare that exam prompts correspond with a single concept/theme, but this should enable you to find which examples are most flexible, and which have more narrow applicability.

Since this is a resource that's basically just for you, though, you should work out what things are going to be most helpful to you. So if connecting your examples with the prompt is an issue, then the stuff I outlined above will probably be your best bet. But if you struggle to adequately explain the connections between your examples and your ideas, then that should be your priority. Or, if you always struggle to summarise the example quickly, then write a nice, efficient one/two word statement in your repository which you can wheel out in your essays. Or, if you can never find links between your examples, then add a section to each piece of evidence where you try and forge connections to other stuff in your list. etc. etc. Structure it based on whatever you believe your weaknesses to be. And you can always add/change stuff as you go :)

Brilliant work on starting to compile a repository right now though - a lot of students will wait till waaaay later on the year and they'll miss out on that more gradual collection/documentation process, so you're definitely taking some of the strain off your Semester 2 self :)
Title: Re: VCE English Question Thread
Post by: tashhhaaa on April 17, 2016, 10:05:03 pm
Most of my students have found a simple word doc that goes through each example to be sufficient. Then under each one, you'll have a brief description (in case you come back to one after a few months and think wtf was I on about??) and a few key dot points for discussion purposes. I'd also recommend linking each example to some of the major 'themes' within your Context (e.g. for Conflict, you might have areas like 'cause vs. consequence,' 'fear/negative effects' 'strength/growth/positive effects,' 'our response to conflict and our true values' etc.) Basically go through as many prompts as you can find (& there's a fairly conclusive list here unless your school has a bunch of weird ones) and collate all the similar words and ideas you come across. That way, if you get a SAC/exam prompt like 'Conflict brings out the best in people' you go to your mental happy place and think 'k, cool, so I can use examples X, Y, and Z because they relate to our response to conflict and the notion of positive consequences.'

You'll usually have to end up merging categories since it's rare that exam prompts correspond with a single concept/theme, but this should enable you to find which examples are most flexible, and which have more narrow applicability.

Since this is a resource that's basically just for you, though, you should work out what things are going to be most helpful to you. So if connecting your examples with the prompt is an issue, then the stuff I outlined above will probably be your best bet. But if you struggle to adequately explain the connections between your examples and your ideas, then that should be your priority. Or, if you always struggle to summarise the example quickly, then write a nice, efficient one/two word statement in your repository which you can wheel out in your essays. Or, if you can never find links between your examples, then add a section to each piece of evidence where you try and forge connections to other stuff in your list. etc. etc. Structure it based on whatever you believe your weaknesses to be. And you can always add/change stuff as you go :)

Brilliant work on starting to compile a repository right now though - a lot of students will wait till waaaay later on the year and they'll miss out on that more gradual collection/documentation process, so you're definitely taking some of the strain off your Semester 2 self :)

can confirm that I did this (although I was one of the people who rushed it/did it before exams and thus didn't gain the full benefit) & it was extremely helpful, definitely worth your time.
Title: Re: VCE English Question Thread
Post by: kimmytaaa on April 18, 2016, 11:01:01 am
Does anyone have notes or summary on Brooklyn?
Title: Re: VCE English Question Thread
Post by: literally lauren on April 18, 2016, 12:33:47 pm
Does anyone have notes or summary on Brooklyn?

Some stuff here
Title: Re: VCE English Question Thread
Post by: HasibA on April 18, 2016, 03:42:28 pm
how do i go about improving my confidence for english? i know this is a bit of a random question, but even if i'm gettign really high scores for essays and such, i always feel like there's something wrong with my essays (lack of coherency, stylistic vocab, analysis etc.)
idk if this is the right place to even post such a question :o haha i mean , i do the assigned homework and write essays to the best of my ability, but i just don't feel quite 'there' yet
thank you!
Title: Re: VCE English Question Thread
Post by: Swagadaktal on April 18, 2016, 03:54:02 pm
how do i go about improving my confidence for english? i know this is a bit of a random question, but even if i'm gettign really high scores for essays and such, i always feel like there's something wrong with my essays (lack of coherency, stylistic vocab, analysis etc.)
idk if this is the right place to even post such a question :o haha i mean , i do the assigned homework and write essays to the best of my ability, but i just don't feel quite 'there' yet
thank you!
TBH i dont think you want to feel quite there. As long as you keep feeling like you aren't 'there' you'll keep striving to improve. I guess you'll need your confidence in tact during the end of year exam, but till then it's an extremely healthy thing to have.
Title: Re: VCE English Question Thread
Post by: HopefulLawStudent on April 18, 2016, 04:22:19 pm
(it'd be quite weird if you used formal analytical language to your lover)

Psh. Fairly sure I'd be the sort of person who'd write uber formal letters to my boyfriend just for giggles. Full disclosure: This might be one of the reasons I don't date... I'm too crazy. :P

how do i go about improving my confidence for english? i know this is a bit of a random question, but even if i'm gettign really high scores for essays and such, i always feel like there's something wrong with my essays (lack of coherency, stylistic vocab, analysis etc.)
idk if this is the right place to even post such a question :o haha i mean , i do the assigned homework and write essays to the best of my ability, but i just don't feel quite 'there' yet
thank you!

Honestly? I think you're never gonna be quite "there". I have the same feeling re: english. I could write 10 practice essays before a SAC (Have actually done this before, don't recommend doing it) and do okay in them and I'd still obsess over the "what ifs" (e.g. what if the topic we get in the SAC/exam is something I've never ever ever seen? What if my brain quits on me just before? What if I don't know what to write?!). I have never gone into a SAC thinking "I'm gonna smash this". I've always been a nervous wreck before and after (more after than before which makes no sense but whatevs). I feel like it's the sort of thing you just have to deal with unfortunately (if there's some secret to instantly feeling confident, please share). At the end of the day, just trust that you've done everything you possibly could and more than likely, your hard work will be converted in your essay and in your grade.
Title: Re: VCE English Question Thread
Post by: HasibA on April 18, 2016, 04:26:14 pm
yeah , i see where you guys are coming from! thanks for the input- i think i'm going to just try do as much as i can to the best of my ability to prepare for English- no other subjects make me feel this inadequate other than English T_T *sigh*
thanks!! :)))
Title: Re: VCE English Question Thread
Post by: nadiaaa on April 19, 2016, 05:43:52 pm
Hi guys,
For creating & presenting, my sister told when she did year 12 (in 2014) her tutor told her that you dont mention language, structures & features in your expository essays -- you just use the themes and mainly explore the themes through external examples. Is this correct??
Thanks guys  :)
Title: Re: VCE English Question Thread
Post by: blacksanta62 on April 19, 2016, 08:08:10 pm
The linked article supports my POV on the Safe Schools program. In this article Judith Ireland says activities in the Safe Schools program are "touching" and "G-rated".

I quoted this in my speech but my teacher says to briefly mention what she (Ireland) bases this on. What does my teacher want me to do exactly? Can I just briefly explain one of the activities. I'm confused with what she's recommending. Thank you :))

Edit: Forgot to link the article  :P

The link: http://www.smh.com.au/federal-politics/political-opinion/there-is-nothing-dangerous-about-the-safe-schools-coalition-20160224-gn2yrn.html
Title: Re: VCE English Question Thread
Post by: japanese on April 20, 2016, 08:08:33 pm
hi,
for my oral i'm doing gender selection. I was wondering if someone can help me in explaining the argument for gender selection which states that it can help with balancing the gender of the family.
Thankyou
Title: Re: VCE English Question Thread
Post by: literally lauren on April 21, 2016, 11:19:55 am
Hi guys,
For creating & presenting, my sister told when she did year 12 (in 2014) her tutor told her that you dont mention language, structures & features in your expository essays -- you just use the themes and mainly explore the themes through external examples. Is this correct??
Thanks guys  :)
Yep, totally right. You can delve into language features or quotes if you want to, but you're under no obligation to do so for Context pieces. Ideas are way more important :)

The linked article supports my POV on the Safe Schools program. In this article Judith Ireland says activities in the Safe Schools program are "touching" and "G-rated".

I quoted this in my speech but my teacher says to briefly mention what she (Ireland) bases this on. What does my teacher want me to do exactly? Can I just briefly explain one of the activities. I'm confused with what she's recommending. Thank you :))

Edit: Forgot to link the article  :P

The link: http://www.smh.com.au/federal-politics/political-opinion/there-is-nothing-dangerous-about-the-safe-schools-coalition-20160224-gn2yrn.html
Okay so (correct me if I'm wrong) you're arguing that the Safe Schools Program is a good thing that'll be beneficial for students? And you're citing Ireland as someone who believes the program to be "touching" and "G-rated." I'm guessing your teacher just wants you to substantiate these points by discussion how and why the program is "G-rated"/harmless/suitable for children rather than letting that quote do the work for you. So discussing one of the activities might help, but it seems like she just wants you to unpack the rationale behind the program being safer than others in the media may be portraying it.

hi,
for my oral i'm doing gender selection. I was wondering if someone can help me in explaining the argument for gender selection which states that it can help with balancing the gender of the family.
Thankyou
Not entirely sure what you're wanting to argue here? Is it that people should have the right to choose the gender of their children? If so, why do you think this is important? You mention 'balancing the gender of the family' - why is this a good thing? Not saying you're wrong; just trying to flesh out this line of argument a bit. What kind of advantages are there in being able to have this choice, and what disadvantages are there if that choice isn't available? You may also find it useful to read up on some opinion pieces about this to get a feel for what common arguments you could add to or rebut.
Title: Re: VCE English Question Thread
Post by: NerdyPi on April 21, 2016, 04:17:31 pm
Hi guys, so I have my context SAC next week (On Foe, for Whose Reality), and I was just wondering if this example would be enough of a link back to Foe:

There's a character in Foe named "Friday", who starts dancing around the house in the novel, and becomes completely unresponsive when doing this. The protagonist Susan eventually concludes that he does this to escape reality. In my piece, (if I get a prompt that has something to do with coping with reality or creating illusions), I was thinking of writing about a young autistic boy, named "Freddy", being bullied at school (from the mother's perceptive), and talking about how he's started dancing around the grass at lunch and recess, which results in him being able to block out the bullies, and hence becoming significantly happier.

Obviously I would need other examples in my piece to get to 900+ words, but would this example alone be enough of a link back to Foe?  Thanks everyone :)
Title: Re: VCE English Question Thread
Post by: HopefulLawStudent on April 21, 2016, 09:46:11 pm
This suggests these people did not feel the devotion the reader would typically associate with "owner[ s ]" and their pets. The implication being that the deprivation these animals endure is not only physical but also psychological in nature. This has the effect of emphasising how great this cruelty was.

The bolded bit isn't a sentence according to some feedback I got from my teacher.
My question is this:
1) Why isn't it a sentence?
2) How do I make it into a sentence? I want to keep the word "implication" but I have no idea how to actually use it in a sentence apparently.
Title: Re: VCE English Question Thread
Post by: HopefulLawStudent on April 21, 2016, 10:09:33 pm
ALSO how do I combat tone irregularities? In my LA pieces what's started to happen is I start slipping into past tense. I'm guessing it's because my issue (greyhound racing) is one that is by and large a 2015 issue and so in my mind, I keep moving to speak of the issue in past tense because it's already happened and so it makes sense to me to have past tense. What can I do to resolve my tense issue?

My English teacher told me to just "be conscious" of it and it'd resolve itself but it hasn't and I don't know what to do. Like legit, I went through a piece I previously wrote and scrutinised every. single. word. and somehow, there are still substantial tense irregularities (I oscillate between present and past tense throughout my entire essay) that I didn't pick up at all and I'm only just now seeing because my English teacher has pointed them out. Being conscious of it didn't work, editing clearly won't work either... Does anyone have any solution?
Title: Re: VCE English Question Thread
Post by: tashhhaaa on April 21, 2016, 10:51:29 pm
ALSO how do I combat tone irregularities? In my LA pieces what's started to happen is I start slipping into past tense. I'm guessing it's because my issue (greyhound racing) is one that is by and large a 2015 issue and so in my mind, I keep moving to speak of the issue in past tense because it's already happened and so it makes sense to me to have past tense. What can I do to resolve my tense issue?

My English teacher told me to just "be conscious" of it and it'd resolve itself but it hasn't and I don't know what to do. Like legit, I went through a piece I previously wrote and scrutinised every. single. word. and somehow, there are still substantial tense irregularities (I oscillate between present and past tense throughout my entire essay) that I didn't pick up at all and I'm only just now seeing because my English teacher has pointed them out. Being conscious of it didn't work, editing clearly won't work either... Does anyone have any solution?

I had the exact same problem, oddly enough with an LA on greyhound racing haha
I used to switch tense a lot without realising and the only thing that helped me overcome it was being really anal and meticulously editing my work immediately. I noticed that I made a lot of silly mistakes under pressure so I'd suggest writing an essay at home without time constraints and editing as you go (as soon as you finish a sentence, read over it and fix any errors.  Repeat and continue this until you finish) You will become faster at this trust me and soon it will be second nature to you. Once I had formed this habit I also never needed to allocate extra time at the end to edit. (If I had spare time I would use it, but I usually didn't)

Something else that I found helpful was a lot of self talk as I was writing I know I sound like a maniac shh Little reminders like "remember that X author encourages not encouraged Z reader to reconsider..." every time you're about to flesh something out could work for you

Basically, you're only going to see a mistake in two situations: as soon as you've made it, or several minutes/hours/days later. In the interim (when you're writing the rest of your essay) you probably won't see it so you have to actively try and avoid small mistakes

sorry if this is something you've already heard, but this pretty much resolved my tense issues
Title: Re: VCE English Question Thread
Post by: kimmytaaa on April 22, 2016, 09:57:29 am
Hi literally lauren
I pm you about some English confusions, are you able to help me out?
thanks 
Title: Re: VCE English Question Thread
Post by: literally lauren on April 22, 2016, 10:39:02 am
Hi guys, so I have my context SAC next week (On Foe, for Whose Reality), and I was just wondering if this example would be enough of a link back to Foe:

There's a character in Foe named "Friday", who starts dancing around the house in the novel, and becomes completely unresponsive when doing this. The protagonist Susan eventually concludes that he does this to escape reality. In my piece, (if I get a prompt that has something to do with coping with reality or creating illusions), I was thinking of writing about a young autistic boy, named "Freddy", being bullied at school (from the mother's perceptive), and talking about how he's started dancing around the grass at lunch and recess, which results in him being able to block out the bullies, and hence becoming significantly happier.

Obviously I would need other examples in my piece to get to 900+ words, but would this example alone be enough of a link back to Foe?  Thanks everyone :)
That sounds really cool, actually!

First thing's first: for your Context SACs you'll have the chance to write a Statement of Intention/ Written Explanation in which you justify the choices that you've made and flesh out the links between your piece and the context/ prompt/ set text. It's a tricky thing to talk about because each school (and sometimes even each teacher) handles it differently, so you might be told that the S.O.I. is hugely important and determines your overall mark, or that it doesn't even matter and you just have to do it to satisfy the requirement. There are even some schools that don't make students do it at all (which is against VCAA's regulations but I ain't namin' no names cause I ain't no snitch...)

Point being: with the advantage of a S.O.I. where you can drop the pretense of a story at the end and just say 'I've written an imaginative narrative that takes inspiration from Foe's character Friday in order to show...' - there'll be no doubt what you were trying to say. HOWEVER:
a) your teacher may not give much credence to the S.O.I. and will prefer it if they can just read your story and understand the links. i.e. if they need to read the S.O.I. to get the connection, then your piece has failed in some regard because...
b) you won't get the chance to write an S.O.I. in the exam! This doesn't mean you have to ditch this narrative completely, but it might mean that you'll need to adapt your story so that the connections are extremely obvious, just in case.

The two tips I have for you:
- make sure that's not all your piece is exploring. The fact that you've noted you'll "need other examples" tells me you probably already know this, but the creative stories that just retell the plot of the set text by changing a few details around whilst conveying exactly the same message don't tend to be very impressive.

It's kind of like how, if you were studying The Titanic, and you wrote a piece with identical-ish characters only it's set on a plane in the year 2012 instead of 1912. You're not really doing anything different with the ideas of that text. Contrast this with a piece that follows The Titanic's narrative but adopts the POV of one of those selfish characters who pushes others out of the way to get onto a lifeboat. Then the story continues from their perspective as they have to contemplate the enormity of their actions and the fact that they prioritised their own lives above others'. Originally, the film explores an act of selflessness, so by looking at the flip side of that, you're furthering the discussion rather than limiting yourself to the ideas evident in the film.

- & this may or may not suit you, but something that can be very effective if well executed is using some of the language of the text in your own writing. You can do this verbatim just to ensure some fairly obvious links to the set text, or you can modify and recontextualise them to add depth to your story. eg. the line "He does not know what freedom is" which is fairly central in Foe could be said by Freddy's mother in your story while she watches him dance after dropping him off at school that day. Or you might even change it to "He knows what freedom is" if you wanted to put a different spin on it.

^That kind of stuff is great to explore in your S.O.I. too, if you so desire.

This suggests these people did not feel the devotion the reader would typically associate with "owner[ s ]" and their pets. The implication being that the deprivation these animals endure is not only physical but also psychological in nature. This has the effect of emphasising how great this cruelty was.

The bolded bit isn't a sentence according to some feedback I got from my teacher.
My question is this:
1) Why isn't it a sentence?
2) How do I make it into a sentence? I want to keep the word "implication" but I have no idea how to actually use it in a sentence apparently.
Fair warning, I have written the equivalent of a postgrad thesis on the question 'what is a sentence?' at uni, and it's the kind of surprisingly open-ended question that keeps academic linguists up at night, but I'll try and break this down:

The implication being that the deprivation these animals endure is not only physical but also psychological in nature.

The golden rule (in English and about 98% of documented languages worldwide) is that a sentence must have two things: a noun phrase and a verb phrase. Or, in less jargon-y terms: a thing and stuff that happens to that thing.

For example:
She is a talented bellydancer.
He laughs a lot.
The Czech Republic recently changed its name to Czechia.
Through this, the author highlights the brutality of mankind.

Red = Noun Phrases / 'things'
Green = Verb Phrases / 'stuff that happens'

All of the sentences above are totally grammatical. The first two are more straightforward since we've got our 'thing' (she/he respectively) that the whole sentence is focussing on. In the first sentence we need to include the stuff that comes after the verb 'is' (because we can't just say 'She is' unless we're trying to say 'She exists' but we're trying to say 'She is a bellydancer' so we need that other information). In the second sentence, though, 'a lot' is more omitable since we could still convey the main gist of the sentence 'He laughs' without it.

The third example is similar to the first in that we need the information that comes after the verb 'changed,' but we don't really need 'recently.' So we can include stuff in between the noun phrase and the verb phrase if we want to. Finally, the last sentence has some optional information at the start of the sentence (i.e. 'Through this,') in the form of a linking phrase.

Compare these sentences to:
She a bellydancer.
Laughs he a lot.
The Czech recently changed Republic its name to Czechia.
Through this, highlighting the brutality of mankind.

NONE of these are grammatical! The first is missing a verb phrase. The second has the verb phrase occurring before the noun phrase. The third has a verb phrase interrupting the noun phrase, and the fourth is missing a noun phrase.

So the rules are:
- Every sentence must have a 'thing'/NP and 'stuff that happens to the thing'/VP
- The NP must come BEFORE the VP
- There can be information BETWEEN the NP and the VP but not WITHIN them.
- Other additional and omittable information can occur before or after the NP and VP.

Now let's consider your sentence:

The implication being (?) that the deprivation these animals endure is not only physical but also psychological in nature.

'The implication' is fine as an NP so you don't have to change that. The problem comes with your choice of VP.

The verb 'being' can't occur on its own; it needs another verb.

Try and use 'being' in a sentence as a verb. You'll likely come up with something like:
- I hate being so forgetful.
- I am used to being left out.
- My pen is being very difficult right now.
- You are just being silly.

In every one of these sentences, there's another verb involved.
Spoiler
- I hate being so forgetful.
- I am used to being left out.
- My pen is being very difficult right now.
- You are just being silly.

'Being' as a verb is something that adds additional information to a verb (and basically emphasises the present tense, but we won't get into that). So using 'being' without another verb is kind of like using the word 'the' without anything after it.

The is that the deprivation these animals endure is not only physical but also psychological in nature.

Sounds weird, right?

The easiest change we can make to this sentence would be:
The implication being is that the deprivation these animals endure is not only physical but also psychological in nature.

...though there are a few other possible alternatives that could work, so feel free to run them by me if you're unsure.

tl;dr:
Your original version isn't a sentence because the verb isn't right. It would be okay if it were tacked onto a previous statement like: The author suggests that their suffering is "manifold;" the implication being that the deprivation these animals endure is not only physical but also psychological in nature. But as a standalone sentence, it is incomplete.

I can't tell if that was too vague or too extensive but please let me know if any of that didn't make sense and I'll do my best to explain things. I know it can be really tough when you're told your intuitions are wrong and you can't work out why, and correcting false impressions is one of the hardest things to fix, but good on you for isolating this concern. Hopefully that process above will help you work out where you're going wrong.

Hi literally lauren
I pm you about some English confusions, are you able to help me out?
thanks 
Post on the forums and I should get back to you sooner :)
Title: Re: VCE English Question Thread
Post by: YellowTongue on April 25, 2016, 07:40:27 am
What would you say is the intended effect of the author in the opening paragraph of this article?:

http://m.smh.com.au/comment/bring-back-the-nightlife-lockout-laws-require-a-rethink-20160214-gmtl6v.html

"I wouldn’t hit the Kings Cross night clubs if they were opened til 6am or shut promptly on the dot at 6pm. After all, nothing will make you feel old like seeing hordes of party people who weren’t even born when you could take your first legal drink."
Title: Re: VCE English Question Thread
Post by: tashhhaaa on April 25, 2016, 04:28:55 pm
What would you say is the intended effect of the author in the opening paragraph of this article?:

http://m.smh.com.au/comment/bring-back-the-nightlife-lockout-laws-require-a-rethink-20160214-gmtl6v.html

"I wouldn’t hit the Kings Cross night clubs if they were opened til 6am or shut promptly on the dot at 6pm. After all, nothing will make you feel old like seeing hordes of party people who weren’t even born when you could take your first legal drink."

He's trying to make a joke by saying he's too old to be clubbing anyway, and therefore is unaffected by the lockout laws. I'd say he wants to appear likeable to readers by opening with this line because if you hadn't seen the title of the piece one might assume that he endorses the lockout laws and that was just a bitter rant rather than a joke

However we realise that it's not because he then states "Despite this I think it’s time for a rethink of the state’s alcohol lockout laws because it is undeniable that a large part of Sydney’s night life is closing down, perhaps never to reappear... the lockout laws in Sydney are squeezing the life out of Sydney’s social scene" and the rest of his piece laments the loss of Sydney's nightlife because he sees it as a positive thing. Note his use of language such as "previously vibrant metropolitan streetscapes" "squeezing the life out" and that the Premier "killed the promised review of the laws"... I might be going off on a bit of a tangent but these words are all associated with life/death, so it links to how he thinks "parts of the city are now a wasteland"

Also there's a huge picture of him before the text (he doesn't look that old) and he has a modern-ish haircut which further reinforces the idea that he is relatable and not some ancient politician complaining about Sydney's club scene
Title: Re: VCE English Question Thread
Post by: zxcvbnm18 on April 25, 2016, 06:08:30 pm
ENGLISH Q&A AND PAST QUESTIONS DATABASE

What is this thread for?
If you have general questions about the VCE English (or EAL) course about what the tasks entail or how to improve in certain areas, this is the place to ask!

However, if you're looking for text-specific advice (eg. 'How could I talk about character X from text Y' or 'Is my interpretation of Z correct?') then please make a new thread on the English page. This section is designed to be an information and advice resource, so keep your questions fairly general, if possible.

If you're looking for essay marking and feedback, go to the English Work Submission and Marking


Who can/ will answer questions?
Everyone is welcome to contribute; even if you're unsure of yourself, providing different perspectives is incredibly valuable in English, - since we all know there's rarely one "right" answer ;)

Please don't be dissuaded by the fact that you haven't finished Year 12, or didn't score as highly as others, or you're advice contradicts something else you've seen on this thread, or w/e; none of this disqualifies you from helping others. And if you're worried you do have some sort of misconception, put it out there and someone else can clarify and modify your understanding! 

I will endeavour to help wherever I can, but there'll be a whole bunch of other high-scoring students with their own wealths of wisdom to share with you. So you may even get multiple answers from different people offering their insights.


Which questions get included in the database?
The questions in the tabs below are ones deemed helpful to multiple people (ie. dealing with common pitfalls and struggles.) That is not to say other questions are less valid, but these are just the most common areas that students need help with, so please have a read through some before asking your own questions. If there are points that haven't been covered yet, or you'd like a more detailed explanation for an existing question, post here and we can update the database for everyone.

There's also a list of useful Language Analysis vocab attached to this post :)


TEXT RESPONSE
Preparation
What to discuss in a T.R. essay + picking out evidence? (helpful for early in the year)
The 'goal' of T.R. essays (scroll down to second response)
Using background information
Understanding the text

Planning
Planning T.R. essays (scroll down to second response)

Essay Structure
Teachers "recommending" structures

Introductions
What should be included in the intro? 
Structuring introductions
How to make introductions more sophisticated 
What makes an introductions stand out
Using quotes in the introduction (2)

Body Paragraphs
Wording of topic sentences
Choosing evidence from the text
Finding paragraph weaknesses? (scroll down to third response)
Appropriate explanation of evidence in essays (2)

Ideas
Coming up with original ideas
The difference between 'surface level' themes and 'in-depth' themes
Making simple ideas sophisticated
Memorising ideas
Exploring the impact of major/minor characters

Conclusions
Structuring conclusions (2)

Contentions
Strengthening contentions

Areas of Study
The most important criterion
Layers of Text Response texts
Learning about Views and Values with Harry Potter
Addressing views and values (ft. Paddington Bear)
Branching out from the text and explaining the marking process

Prompts
What are 'implications?'
How to discuss implications (2)
Dealing with difficult prompts
Why 'How...' questions aren't as scary as you think
Addressing 'Do you agree?' prompts

Quotes
Embedding quotes
Shortening and integrating quotes
Memorising and categorising quotes
Memorising quotes for the exam

Exam Preparation
Choosing your exam text before the exam (2)
Making the most of rereading texts
Planning in the exam + choosing exam text

CONTEXT
Criteria
A translation of VCAA's criteria?
Why memorisation is a massive pitfall
How to ensure relevance
Explaining the target audience
Originality in context pieces
How to approach Context studying

Intoductions
Why you shouldn't mention Hitler ;) and the importance of originality in context + my sample intro
What should be in an introduction?

Body Paragraphs
Structuring body paragraphs (2) (3)

General Stylistic Advice
Forms and styles: playing it safe
A basic overview of the three styles
Equal opportunity writing styles? + waxing poetic about The American Dream
Choosing a style

Expository Writing
Balancing examples with discussion + responding to prompts + expository structure?
Abstract discussion in expository essays (with pretty colours because Lauren learnt how to format)
Tips for expository writing
What to include in expository pieces - ft. cake

Imaginative/Creative Writing
Benefits of the Imaginative style? (scroll down to second response)
An explanation of the 'show, don't tell' rule for creative writing
Made-up stories

Persuasive Writing
Structuring a persuasive piece

Ideas
Generating ideas (2)
Managing different ideas at once
Going beyond the text: The Very Hungry Caterpillar and Year 12 English
Ratio of examples to discussion + questioning the prompts?
Using close analysis (ie. symbols, motifs) in a Context piece?

Set Texts
Extent of set text usage (2) (3) (4)
Quoting the set text
'Jumping around' within the text, and why chronological exposition isn't worth much (scroll up a bit for an example involving a headless chicken)
Using texts from previous years

External Examples
Finding external examples
Tying in examples, and zooming in/out?
Trying to find examples (esp. for Encountering Conflict)
Categorising examples
Linking between examples
Sopistication of examples (scroll to third response) (2) (3)
Using risky examples (2)

Prompts
Questioning the prompts - with example (scroll to second response)
Dissecting prompts
Simplifying prompts - using the 2013 conflict prompt as an example
Unpacking prompts (2)
The importance of prompts


MISCELLANEOUS
Studying for English
Advice for Year 12s (2)
Improving in English
'Thinking' in English, with a chicken-flavoured analogy
Fascilitating self-improvement
Going beyond practice essays
Using high-scoring responses
Frequency of study
Typing or handwriting essays? (scroll down to second response)
Is a tutor necessary? (Spoilers! The answer is no.)
SAC Preparation (2)

Using Study Guides
Using study guides (2) (3)

Reading
Wider Reading
Reading academic journals?

Vocabulary
Acquiring new vocabulary (2)
Improving vocabulary (2) (3) (4) (5)
Circumlocating verbosity (2)
Vocab: evoke/provoke/invoke
Self-editing for expression
Improving clunky sentences (scroll down to second response)

General Writing Tips
Overcoming writer's block (2) (3) (4) (4) (5) (6)
Quality vs. Quantity: recommended word count?
Writing with clarity

Time Constraints
Dealing with time esp. in the exam (2) (3) (4)
Planning under timed conditions?
Timing of SACs
Cost/benefit analysis of planning under test conditions
How to prepare for time constraints
Concerns about timing + checklist to go through before a SAC (scroll down to second response)

Advice for non-Year 12s and other English subjects
Advice for Year 9/10 English?
General advice for year 11
English/EAL differences
English 3/4 without 1/2 (with Lit 1/2)


Exam Preparation
When to start studying for the exam
Exam Details (scroll down to second response)
Exam writing booklets
What to bring to the exam (2)
Assorted pre-exam questions 2015 (2)
How exam marking works
Day before the exam?

In the Exam
Order of essays (scroll down to second response) (2)
Timing in the exam

Anyways to improve my fluency in essays?
Title: Re: VCE English Question Thread
Post by: Callum@1373 on April 25, 2016, 06:16:48 pm
Does anyone have any good examples for identity and belonging which I could use to talk about how sometimes belonging leads to the development of identity? E.g 'The best way to find yourself is to lose yourself in the service of others.' ~ Mahatma Gandhi
Title: Re: VCE English Question Thread
Post by: HopefulLawStudent on April 25, 2016, 06:28:03 pm
(http://cache.reelz.com/assets/content/repFrame/43136/43136_01_med.jpg)

How do I describe her facial expression? In this scene, she's sorta staring off into the distance dreaming about a life she can never have because it's so wildly beyond her means. She's singing about how it'd be lovely to have chocolate and warmth and what have you but she and the audience know that such things would have been beyond her means. I have no idea how to describe her facial expression though.

I know this isn't English (actually for Lit but the Lit boards are dead) and I'm desperate...  please?
Title: Re: VCE English Question Thread
Post by: One Step at a Time on April 25, 2016, 06:57:17 pm
HLS, it looks like reality is sinking in for her  :P

For anyone on this board, I have a question about creative writing for you!

The topic is: Explore an idea, issue or theme from the original text.

One theme is conflict in the text and I want to write about conflict generally. I'm scared I'll go off topic and my teacher will penalise me though...

Please HELP! :-\
Title: Re: VCE English Question Thread
Post by: literally lauren on April 25, 2016, 07:17:32 pm
Anyways to improve my fluency in essays?
What do you mean by fluency? Is it the way each sentence is constructed (as in, you need individual sentences to make more sense because at the moment they're a bit bogged down or ungrammatical) or the way the sentences are strung together (as in, you need the ideas to flow together and build on one another.) 'Fluency' can mean anything from overall essay structure down to individual word choices, so the more specific you can be, the more we can help you :)

Does anyone have any good examples for identity and belonging which I could use to talk about how sometimes belonging leads to the development of identity? E.g 'The best way to find yourself is to lose yourself in the service of others.' ~ Mahatma Gandhi
You could look at how certain groups add to one's character (e.g. volunteering at a charity and becoming a more compassionate person.) And that, in turn, can make the group stronger ŕ la this lovely story. Or perhaps look into some justice reform programs/ the idea of compulsory community service as a sentence for juvenile offenders as opposed to just prison time.
& if you want to extend that to a discussion of how we can define or redefine groups based on our identities and shared values, consider: "#YouAin'tNoMuslimBruv"

(http://cache.reelz.com/assets/content/repFrame/43136/43136_01_med.jpg)

How do I describe her facial expression? In this scene, she's sorta staring off into the distance dreaming about a life she can never have because it's so wildly beyond her means. She's singing about how it'd be lovely to have chocolate and warmth and what have you but she and the audience know that such things would have been beyond her means. I have no idea how to describe her facial expression though.

I know this isn't English (actually for Lit but the Lit boards are dead) and I'm desperate...  please?
"Lost in reverie" would be the term I'd use there.

& I will swing by the Lit boards when people post there. It's funny cause I'll get like 10 messages in a day saying 'hey, here's a Lit thing; I would post it, but no one goes on the Lit boards' :P

Looks like reality is sinking in for HLS  :P

For anyone on this board, I have a question about creative writing for you!

The topic is: Explore an idea, issue or theme from the original text.

One theme is conflict in the text and I want to write about conflict generally. I'm scared I'll go off topic and my teacher will penalise me though...

Please HELP! :-\
I'm a bit lost as to what you're being asked to do here - it sounds like you've just isolated a theme you want to discuss, and the task is telling you to explore that theme, so in what way would you be going 'off topic?' ???
Title: Re: VCE English Question Thread
Post by: One Step at a Time on April 25, 2016, 07:36:38 pm
Thanks Lauren for your reply  ;D

Not too sure to be honest, but I just kept in mind that my teacher said that I must stay to the themes, ideas and spirit of the text, as well as incorporating bits of how the author writes e.g. linking seasons with feeling. Didn't want to go against what she said hahah :)

So do you think that writing about the idea of control (which is a theme in the text) for my creative piece would be alright? The only thing in common with the text and be essay would be overarching idea of control  :D
Title: Re: VCE English Question Thread
Post by: HopefulLawStudent on April 25, 2016, 07:57:33 pm
Hey Lauren:

Is this a sentence?

She wore a tie which would have been archetypal for the men of the period.

It sounds funny. I want to use "typical" or something like that but trying to expand my vocab sooooo... for the text I'm studying, it would have been expected for men to wear pants not women (apparently women wearing pants was socially unacceptable or something like that). That's basically what I'm trying to say but just not in so many words. Apparently conciseness in any English subject is much appreciated at a VCE level.

I know... I should be studying for lit and rereading my text or something but I'm stressed so I may or may not be just casually reading the dictionary because y'know... that's what I do when I'm stressed. :P
Title: Re: VCE English Question Thread
Post by: Swagadaktal on April 25, 2016, 08:06:32 pm
Hey Lauren:

Is this a sentence?

She wore a tie which would have been archetypal for the men of the period.

It sounds funny. I want to use "typical" or something like that but trying to expand my vocab sooooo... for the text I'm studying, it would have been expected for men to wear pants not women (apparently women wearing pants was socially unacceptable or something like that). That's basically what I'm trying to say but just not in so many words. Apparently conciseness in any English subject is much appreciated at a VCE level.

I know... I should be studying for lit and rereading my text or something but I'm stressed so I may or may not be just casually reading the dictionary because y'know... that's what I do when I'm stressed. :P
I think you're teacher is dissing your sentences because there is no point to them (maybe?) I dont see how he can just disenfranchise a sentence, like there are no rules in tact which determines what a sentence is. Maybe you go on further to explain the significance of the hoe not wearing pants, but in that 'sentence' it does not have any purpose?  (i disagree with your teacher but i think that's what he's saying?)
And I just stopped judging you from the last time you commented that you read the dictionary when you're stressed why you gotta bring it up again :>
im kidding please don't send me mean messages

im actually not hehe  8)
Title: Re: VCE English Question Thread
Post by: literally lauren on April 25, 2016, 08:12:57 pm
Hey Lauren:

Is this a sentence?

She wore a tie which would have been archetypal for the men of the period.

It sounds funny. I want to use "typical" or something like that but trying to expand my vocab sooooo... for the text I'm studying, it would have been expected for men to wear pants not women (apparently women wearing pants was socially unacceptable or something like that). That's basically what I'm trying to say but just not in so many words. Apparently conciseness in any English subject is much appreciated at a VCE level.

I know... I should be studying for lit and rereading my text or something but I'm stressed so I may or may not be just casually reading the dictionary because y'know... that's what I do when I'm stressed. :P
Grammatically, it's fine. But in terms of your word choice, I'd agree that 'archetypal' is a bit off.

...like there are no rules in tact which determines what a sentence is.
THERE ARE MANY RULES, PAL. MANY RULES. O.o
But I do totally agree with your underlying point that it's the ideas/sentiment behind your discussion that matters more than the words and syntax you choose to express those points.

However, improper grammar or vocabulary can sway one's mark a little, especially if it's a SAC being marked out of 30 or 50 where the difference between a 28 and a 29 or a 46 and a 47 could just come down to a few of these little quibbles.

She wore a tie which would have been archetypal for the men of the period.

Think of an 'archetype' as being like a 'blueprint.' So the archetypal 1950's housewife was one who cooked and cleaned and wore aprons and stayed home to look after the children, etc. Therefore, you could not say that aprons were 'archetypal' of 1950's women, because the 'archetype' is like the whole package.

'Typical' works much better because it can apply to smaller elements. So an apron might be a typical garment of an archetypal 1950's house frau - that's fine, albeit a little repetitive.


Fun fact: the word 'archetype' comes from the Greek word 'arkhetupon' which meant a basic/primitive (arkhe-) model or mould (tupos). Whereas 'typical' is thought to have Latin roots that come from 'typicalis' meaning 'to be symbolic of something.'

And before you ask, yes, I am great fun at parties.
Title: Re: VCE English Question Thread
Post by: HopefulLawStudent on April 25, 2016, 08:28:57 pm
I think you're teacher is dissing your sentences because there is no point to them (maybe?) I dont see how he can just disenfranchise a sentence, like there are no rules in tact which determines what a sentence is. Maybe you go on further to explain the significance of the hoe not wearing pants, but in that 'sentence' it does not have any purpose?  (i disagree with your teacher but i think that's what he's saying?)
And I just stopped judging you from the last time you commented that you read the dictionary when you're stressed why you gotta bring it up again :>
im kidding please don't send me mean messages

im actually not hehe  8)

This isn't English related. This is me flipping through the dictionary and making up random sentences because I'm stressed about tomorrow's literature SAC. With my English teacher and how savage he's been re: marking, if I started randomly babbling about pants, I think he might just murder me.

PS: Stress-reading the dictionary is good for stress... for me anyway.

Think of an 'archetype' as being like a 'blueprint.' So the archetypal 1950's housewife was one who cooked and cleaned and wore aprons and stayed home to look after the children, etc. Therefore, you could not say that aprons were 'archetypal' of 1950's women, because the 'archetype' is like the whole package.

'Typical' works much better because it can apply to smaller elements. So an apron might be a typical garment of an archetypal 1950's house frau - that's fine, albeit a little repetitive.


Fun fact: the word 'archetype' comes from the Greek word 'arkhetupon' which meant a basic/primitive (arkhe-) model or mould (tupos). Whereas 'typical' is thought to have Latin roots that come from 'typicalis' meaning 'to be symbolic of something.'

And before you ask, yes, I am great fun at parties.

Huh. Okay. I knew I'd used it wrong because it sounded funny but I couldn't figure out why. Thanks Lauren! :D
Title: Re: VCE English Question Thread
Post by: HopefulLawStudent on April 25, 2016, 08:32:58 pm
Pleasedon'tmurdermeforspammingthisthreadwithmyrandomquestions.

Am I using the bolded word correctly:

She recognised the flower girl would ultimately be nothing more than an ersatz image of a duchess.
Title: Re: VCE English Question Thread
Post by: literally lauren on April 25, 2016, 08:42:14 pm
Thanks Lauren for your reply  ;D

Not too sure to be honest, but I just kept in mind that my teacher said that I must stay to the themes, ideas and spirit of the text, as well as incorporating bits of how the author writes e.g. linking seasons with feeling. Didn't want to go against what she said hahah :)

So do you think that writing about the idea of control (which is a theme in the text) for my creative piece would be alright? The only thing in common with the text and be essay would be overarching idea of control  :D
Just realised you're a 2017 grad and that this is a part of the new SD :P

Okay, so you're going to want to compose a creative piece that works on these two levels:
- delivering a similar message about the kinds of ideas in your set text
- using language features and devices that are present in your set text

The best advice I can give you would be to be more specific about what ideas you're angling for. Or, to put it another way: turn those themes into views and values statements. So rather than saying 'I'm going to write about control (=theme,)' say 'I'm going to write about how being controlled by others can impede one's sense of self, and this can lead to a lack of self-control which brings about destructive consequences (=V&V statement.)'

This should help you target those ideas with more precision, since otherwise the broadness of 'conflict' and 'control' might be a little hard to mirror in your piece.

Let me know if any of that doesn't make sense!

Am I using the bolded word correctly:

She recognised the flower girl would ultimately be nothing more than an ersatz image of a duchess.
Yep, ersatz is an adjective used to refer to something that's artificial or eerily dissimilar from an original, so is fine in this context.

It's also from German because of course it is. ERSATZ!!!
& one of my fav books in the Lemony Snicket series was called The Ersatz Elevator  ;D

Pleasedon'tmurdermeforspammingthisthreadwithmyrandomquestions.
For every random word-related question, I promise to retort with one random-unsolicited-piece-of-arcane-etymology  8)
Title: Re: VCE English Question Thread
Post by: HopefulLawStudent on April 25, 2016, 08:46:34 pm
Off-topic but: Could ersatz be used as a direct synonym for fake or to describe an imitation of the real thing?
Title: Re: VCE English Question Thread
Post by: literally lauren on April 25, 2016, 09:04:33 pm
Off-topic but: Could ersatz be used as a direct synonym for fake or to describe an imitation of the real thing?
You can't use it as a noun, and 'ersatz' kind of connotes deficiency or inferiority whereas 'imitation' is a bit more neutral in that it implies the copy is more accurate. It's not a direct synonym, but it's pretty close.
Title: Re: VCE English Question Thread
Post by: One Step at a Time on April 25, 2016, 09:21:03 pm
Thanks so much Lauren, think I understand where you're coming from! And sorry I didn't make the task clearer :)

The text is the Wife of Martin Guerre. Although the idea of control isn't prominent, I want to show that it in fact is! e.g. Being controlled/ restricted in a society. I was hoping to base the creative piece on rowing (heaps of control needed there  :P) and the rower seeing someone take his/ her life which obviously has a huge impact. I want to make the connection between my piece and the text strong but I can't express myself clearly though, especially in the written statement. Or should I write about conflict? Conflict doesn't fit my creative piece idea though...

Totally lost and appreciate your guidance  ;D

Title: Re: VCE English Question Thread
Post by: kimmytaaa on April 26, 2016, 01:57:33 pm
Hi
Just a general question about text response, do we need to include the authorial voice because originally I thought we were meant to write about the novel not actually including what the author is trying to express? I'm so lost and my sac is tomorrow :(
Title: Re: VCE English Question Thread
Post by: Callum@1373 on April 26, 2016, 08:33:53 pm
This is gonna be one of them 'it depends' questions I feel but in general, what is better with context: one-two sources of evidence/example (e.g a film) to be used throughout an expository essay or many smaller examples? I've been predominantly going with the numerous small examples but from reading over my work it seems like it is preventing me from going into depth with the discussion; i.e using the example to point out the obvious part of the idea I am exploring.
Title: Re: VCE English Question Thread
Post by: kimmytaaa on April 27, 2016, 08:38:48 am
This is gonna be one of them 'it depends' questions I feel but in general, what is better with context: one-two sources of evidence/example (e.g a film) to be used throughout an expository essay or many smaller examples? I've been predominantly going with the numerous small examples but from reading over my work it seems like it is preventing me from going into depth with the discussion; i.e using the example to point out the obvious part of the idea I am exploring.
Ah thanks
Title: Re: VCE English Question Thread
Post by: Elizawei on April 27, 2016, 08:08:43 pm
Hey, got a quick question.
Which method of language analysis is the "safest" way? For example we could write them in chronological order, order by arguments, order by persuasive devices, etc.

Title: Re: VCE English Question Thread
Post by: HopefulLawStudent on April 27, 2016, 08:34:02 pm
DON'T DO IT IN CHRONOLOGICAL ORDER.
Title: Re: VCE English Question Thread
Post by: vor0005 on April 27, 2016, 08:47:56 pm
Hey, got a quick question.
Which method of language analysis is the "safest" way? For example we could write them in chronological order, order by arguments, order by persuasive devices, etc.

Arguments are by far the best way to structure.

Why?
It helps you analyse the intended effect precisely as you just link how the technique/device positions audiences to adopt the particular argument your paragraph is based on (with a bit more specificity to the technique and quote, of course).

Additionally, you know exactly where you will analyse each visual image - simply analyse the image in the argument (paragraph) that it matches. Same thing goes with comments or anything else that VCAA throws at you. Just put it in the argument that it fits.

Good luck.
Title: Re: VCE English Question Thread
Post by: rosecookiie on April 27, 2016, 09:54:36 pm
I'm in Year 11 and I just can't seem to finish my essays in the 100 minutes we have for the SAC. For both my text response and Language analysis pieces I didn't finish, barely starting my conclusion. Any tips for improving? Time management is my biggest weakness when it comes to English
Title: Re: VCE English Question Thread
Post by: Ameliajanem on April 28, 2016, 12:05:32 pm
My teacher wants me to write an essay on the prompt

"In stasiland, funder exposes a world both cruel and absurd" discuss

I can't think of any counter argument for this? Can someone help
Title: Re: VCE English Question Thread
Post by: upandgo on April 28, 2016, 05:24:56 pm
My teacher wants me to write an essay on the prompt

"In stasiland, funder exposes a world both cruel and absurd" discuss

I can't think of any counter argument for this? Can someone help

hey! im studying stasiland too and i recall writing on a prompt similar to this  :) as a counter argument, maybe you can argue that funder does reveal a cruel and absurd world to readers, however the impacts of these cruelties and absurdities on its people (eg, frau paul, miriam, the ex-stasi operatives, etc) ultimately position readers to conclude that funder exposes a world that is damaged and broken despite the regime's collapse. if that doesnt make sense let me know and i'll explain in more detail!
Title: Re: VCE English Question Thread
Post by: Elizawei on April 28, 2016, 10:18:18 pm
Haha thanks HLS and vor0005 :D
Lovin' that huge font  ;D :P
Title: Re: VCE English Question Thread
Post by: literally lauren on April 29, 2016, 10:25:11 am
Thanks so much Lauren, think I understand where you're coming from! And sorry I didn't make the task clearer :)

The text is the Wife of Martin Guerre. Although the idea of control isn't prominent, I want to show that it in fact is! e.g. Being controlled/ restricted in a society. I was hoping to base the creative piece on rowing (heaps of control needed there  :P) and the rower seeing someone take his/ her life which obviously has a huge impact. I want to make the connection between my piece and the text strong but I can't express myself clearly though, especially in the written statement. Or should I write about conflict? Conflict doesn't fit my creative piece idea though...

Totally lost and appreciate your guidance  ;D
That seems alright to me, but I'd say try to align messages and not just themes. The idea of 'being controlled' might be a bit too broad, which is why you may be struggling to make the connections to the text clear. So consider what Lewis is saying about control, and then either reinforce, build up, or critique that notion. I'm not entirely sure which aspect of 'control' in the text you're looking at, but the more specific you can be, the easier you'll find it to forge a connection.

Hi
Just a general question about text response, do we need to include the authorial voice because originally I thought we were meant to write about the novel not actually including what the author is trying to express? I'm so lost and my sac is tomorrow :(
Both are crucial. You are mainly looking at the details in the text, but the whole point of doing so is to consider what the author is trying to say, meaning that you should aim to comment on the authorial voice within your paragraphs at some point (or preferably at the end.)

This is gonna be one of them 'it depends' questions I feel but in general, what is better with context: one-two sources of evidence/example (e.g a film) to be used throughout an expository essay or many smaller examples? I've been predominantly going with the numerous small examples but from reading over my work it seems like it is preventing me from going into depth with the discussion; i.e using the example to point out the obvious part of the idea I am exploring.
I'm on the 'fewer examples but more depth' side of the fence, but yeah, it depends :P

But I can be a bit more specific: it depends on:
a) your own strengths
b) the nature of your examples
c) the prompt's breadth

Hey, got a quick question.
Which method of language analysis is the "safest" way? For example we could write them in chronological order, order by arguments, order by persuasive devices, etc.
What do you mean by safest? Because the chronological method is objectively simplest (i.e. you don't have to put any effort into thinking about where to start; you can kick of with analysis straight away). However, because it's so dependent on the material, you might end up with something as horrific as the 2011 Exam (or some of the samples I wrote last year :P) and you'll likely have a tough time compiling it all neatly. Also, if you look through the most recent VCAA exams, you'll notice that the authors' focus shifts throughout the piece such that what's being discussed at the start of page 1 is the same as the middle of page 2. And the way they conclude has links to the stuff mentioned in their third paragraph. So if you're going through totally chronologically, you either have to repeat yourself, or ditch potential analysis opportunities to avoid repeating yourself.

The chronological approach may have sufficed in earlier years ('08-'13, excluding '11) but nowadays, I kind of see it as a recipe for disaster.

And by disaster, I mean a mid-range mark :P So if you're only looking to get yourself across the line in English, or need an easy method to suit your strengths, then it might do. But I tend to advocate against it by default.

It's still a good last resort; it just shouldn't be your first resort!

Structuring by devices just seems bizarre to me; I don't know how anyone can do that sensibly without turning their piece into a jumbled mess.

Structuring by arguments/ key players is my best advice (mainly for the reasons vor has mentioned above) but also because it's kind of what the assessors are looking for nowadays. When you read the assessor's report, you see that they emphasise, amongst other things:
Quote
Quote from: VCAA 2014
• There was no expected manner in which the pieces were to be analysed and most students began by analysing the first piece then analysing the second. The more successful responses made insightful analytical comparisons, focusing on such things as tone, structure and, most importantly, the language used by the two writers.

• Successful responses were able to show the inherent connection between the general ideas of each piece and the language used to present those views.

• Some students offered topic sentences that suggested they were searching for particular ‘techniques’, with little regard to the instructions for this section: ‘How is written and visual language used in the newspaper article and the letter to attempt to persuade readers to share the points of view presented in them?’ Simply identifying techniques demonstrates a limited approach to this task.


Quote from: VCAA 2015
• It is important for students to be aware of the intentions of speakers in respect to the contention

• High-scoring students found an appropriate balance between the two speeches and between analysis of the visual language and written language.

I'm in Year 11 and I just can't seem to finish my essays in the 100 minutes we have for the SAC. For both my text response and Language analysis pieces I didn't finish, barely starting my conclusion. Any tips for improving? Time management is my biggest weakness when it comes to English

Some hints here :)
Title: Re: VCE English Question Thread
Post by: Seńor on April 29, 2016, 10:43:42 am
I dont know if this is the right board or not. Im answering an essay question right now and the question is a 'to what extent' type of question. I have forgotten how to properly answer this haha. Can you say to a high or low extent? How do you explain the between of high and low? So basically how do you say 50% type of extent :')
Title: Re: VCE English Question Thread
Post by: literally lauren on April 29, 2016, 11:59:45 am
I dont know if this is the right board or not. Im answering an essay question right now and the question is a 'to what extent' type of question. I have forgotten how to properly answer this haha. Can you say to a high or low extent? How do you explain the between of high and low? So basically how do you say 50% type of extent :')
(Assuming this is VCE English related?) if it's asking 'to what extent' you agree, then it's just a standard prompt.

E.g. 'The novel suggests betrayal is impossible to truly overcome. To what extent do you agree?'

But it'd be no different if it said Discuss. or Do you agree. And if you want to be on the safe side, you should never be 100% agreeing or 100% disagreeing, OR 50% both ways. The latter implies that you don't know the text well enough to make a judgement call on how true that statement is, and it can make your essay feel a bit directionless if you say 'umm... it's true to some extent, but also not really' :P


Though since you graduated last year, I'm guessing this is a uni-related question, in which case the answer will depend on what kind of subject you're taking and what the task requirements are :)
Title: Re: VCE English Question Thread
Post by: Elizawei on April 29, 2016, 12:13:56 pm
Thanks Lauren!
By "key players", do you mean the main components of the article? Like for example in last year's LA the key players would have been Bigsplash, volunteering, and the Tradies without borders?
Title: Re: VCE English Question Thread
Post by: literally lauren on April 29, 2016, 12:23:29 pm
Thanks Lauren!
By "key players", do you mean the main components of the article? Like for example in last year's LA the key players would have been Bigsplash, volunteering, and the Tradies without borders?
Yes, though sometimes you can go for more 'abstract' key players that are based on ideas rather than tangible groups or people.

See: the key on these sample annotations

Also, there's no 'correct' 3 or 4 key players you're required to find; it's all about what you believe to be the most important components of the author's argument.
Title: Re: VCE English Question Thread
Post by: Elizawei on April 29, 2016, 06:07:31 pm
Yes, though sometimes you can go for more 'abstract' key players that are based on ideas rather than tangible groups or people.

See: the key on these sample annotations

Also, there's no 'correct' 3 or 4 key players you're required to find; it's all about what you believe to be the most important components of the author's argument.

Thanks Lauren! One last question haha  :-X :
are we allowed to use personal anecdotes for our context essay? I read an example where someone talked about her grandparents living under the rule of Pol Pot, and weaving it into her context essay for encountering conflict. So in that case, are we allowed to use personal pronouns?

Thanks again  ;D
Title: Re: VCE English Question Thread
Post by: literally lauren on April 29, 2016, 06:15:25 pm
Thanks Lauren! One last question haha  :-X :
are we allowed to use personal anecdotes for our context essay? I read an example where someone talked about her grandparents living under the rule of Pol Pot, and weaving it into her context essay for encountering conflict. So in that case, are we allowed to use personal pronouns?

Thanks again  ;D
You can use personal anecdotes, so long as they're not "petty personal anecdotes" ŕ la:
'My mate and me had a fight the other day but now we're mates again. This shows how conflict can be resolved sometimes.' :P

But you can definitely use personal pronouns, regardless of the evidence you're using. Often it's just easier to say 'we' and 'us' rather than 'people' or 'one,' and you're totally free to do that in Context; just not in Text Response or Language Analysis.
Title: Re: VCE English Question Thread
Post by: Elizawei on April 29, 2016, 08:34:50 pm
Thanks Lauren!!!! <3  ;D
Title: Re: VCE English Question Thread
Post by: JellyBeanz on May 01, 2016, 12:14:54 pm
Hey guys quick question here, It's a pretty vague question but how can i increase the overall quality of the introduction and conclusion of an expository when writing a context piece on identity and belonging?

My teacher says that the ideas and examples i bring in to the piece is fine but to increase the overall quality of my expository i need to write a better intro and conclusion.

For example on the prompt "Sometimes we need to accept change in order to grow". Could someone please give me pointers :P And sorry i know it's an incredibly vague question. \

Thanks.
Title: Re: VCE English Question Thread
Post by: TheWatto on May 01, 2016, 03:07:22 pm
Could someone please help me identify the contention of this article?

http://www.smh.com.au/sport/cage-fight-lived-up-to-the-hype-20151119-gl2xay.html
Title: Re: VCE English Question Thread
Post by: vor0005 on May 01, 2016, 03:16:04 pm
Hey guys quick question here, It's a pretty vague question but how can i increase the overall quality of the introduction and conclusion of an expository when writing a context piece on identity and belonging?

My teacher says that the ideas and examples i bring in to the piece is fine but to increase the overall quality of my expository i need to write a better intro and conclusion.

For example on the prompt "Sometimes we need to accept change in order to grow". Could someone please give me pointers :P And sorry i know it's an incredibly vague question. \

Thanks.

Maybe go for more of a hybrid piece that incorporates both an expository and personal aspect. In particular, you can start your piece with a personal anecdote that relates to your contention in some way. This will help provide authenticity and credibility to your piece and also gives you a unique voice (within your own piece).

Whatever you do, don't write your Context piece as you would a text response essay (the greatest sin!). Remember, the prompt is there to act as a "moral of the story", and you should treat it accordingly.

Make it something that you'd like to see published in a newspaper. I think that should be the goal with any context piece.
Title: Re: VCE English Question Thread
Post by: literally lauren on May 01, 2016, 03:19:57 pm
Quote from: article
Now, at a time when political correctness is more iron-clad than ever and some young men feel the need to identify themselves as feminists...
Omg that tone of condescension is ridiculous -.-

How bitter do you think the Sydney Morning Herald are now that youth culture has coopted their acronym to 'so much hate'?  8) 8)


He's just basically extolling the virtues of boxing and cage fighting. Aside from that brief bit at the start where he semi-attacks the PC brigade, he doesn't seem to have a contention beyond 'fighting is cool.' I'd probably choose a different piece to analyse, if you can; this one's not very persuasive :-\
Title: Re: VCE English Question Thread
Post by: TheWatto on May 01, 2016, 03:50:25 pm
Omg that tone of condescension is ridiculous -.-

How bitter do you think the Sydney Morning Herald are now that youth culture has coopted their acronym to 'so much hate'?  8) 8)


He's just basically extolling the virtues of boxing and cage fighting. Aside from that brief bit at the start where he semi-attacks the PC brigade, he doesn't seem to have a contention beyond 'fighting is cool.' I'd probably choose a different piece to analyse, if you can; this one's not very persuasive :-\
Thanks, yeah I thought that is sounded a bit vague.

Anyway, for this one
http://www.smh.com.au/sport/ufc-193-ronda-rousey-not-the-only-one-knocked-out-by-sundays-events--how-has-it-come-to-this-20151115-gkzpqe.html

I said that the contention of this one is that the Brutality and un-regulation of UFC fighting cannot be fantasized and because of this has no place to stay in Australia.
Would you agree?
Title: Re: VCE English Question Thread
Post by: Champ101 on May 02, 2016, 03:57:44 am
Thanks, yeah I thought that is sounded a bit vague.

Anyway, for this one
http://www.smh.com.au/sport/ufc-193-ronda-rousey-not-the-only-one-knocked-out-by-sundays-events--how-has-it-come-to-this-20151115-gkzpqe.html

I said that the contention of this one is that the Brutality and un-regulation of UFC fighting cannot be fantasized and because of this has no place to stay in Australia.
Would you agree?

This is how i would start off my language analysis. It includes target audience and contention.

In response to the growing acceptance of UFC on the worldwide sporting stage, Peter FitzSimons in an impassioned opinion piece for the Age, didactically asserts that UFC glorifies brutality and condones violence, unequivocally urging Australian authorities to denounce the sport.
Title: Re: VCE English Question Thread
Post by: upandgo on May 02, 2016, 07:40:38 pm
how badly would i be penalised for not concluding my language analysis? i had a comparative SAC today and managed to write everything but the conclusion. would the quality of my piece be more important than whether or not i finished? really didn't want to drop a single mark in english... oh well  :-\
Title: Re: VCE English Question Thread
Post by: Swagadaktal on May 02, 2016, 08:37:59 pm
how badly would i be penalised for not concluding my language analysis? i had a comparative SAC today and managed to write everything but the conclusion. would the quality of my piece be more important than whether or not i finished? really didn't want to drop a single mark in english... oh well  :-\
Erm it shouldn't really tbh, my teachers say there are no marks to be gained in the conclusion so I don't see why there are marks to be lost - right? Especially because this is a comparative. If you had done the task required then I don't see why they should penalise you.

And you're trying to not lose 1 mark the whole year omg r u a wizard (pls halp me)
Title: Re: VCE English Question Thread
Post by: HopefulLawStudent on May 02, 2016, 09:04:17 pm
URGENT. Have an English SAC tomorrow. I swore I'd get an early night tonight but instead I'm cramming as much info as I can before my self-imposed bedtime.

What's the technique called when someone does something like this:

If we do this, A will happen. If A happens B will happen. If B happens then C will happen.

Where like you're sorta being driven down this path of logic and you aren't really left with room to go like "If A happens, D could happen instead of B"

What's that technique called?

I've just been calling it appealing to reason and logic which is wrong.
Title: Re: VCE English Question Thread
Post by: upandgo on May 02, 2016, 09:27:01 pm
Erm it shouldn't really tbh, my teachers say there are no marks to be gained in the conclusion so I don't see why there are marks to be lost - right? Especially because this is a comparative. If you had done the task required then I don't see why they should penalise you.

And you're trying to not lose 1 mark the whole year omg r u a wizard (pls halp me)

ahh really?! that makes me feel so relieved, thanks swag  :P

What's the technique called when someone does something like this:

If we do this, A will happen. If A happens B will happen. If B happens then C will happen.


cascade of events? im terrible at these things haha
Title: Re: VCE English Question Thread
Post by: HopefulLawStudent on May 02, 2016, 09:34:05 pm
cascade of events? im terrible at these things haha

No. There's an actual name for the technique but I can't remember what it is. arghhh...
Title: Re: VCE English Question Thread
Post by: Champ101 on May 02, 2016, 10:34:35 pm
URGENT. Have an English SAC tomorrow. I swore I'd get an early night tonight but instead I'm cramming as much info as I can before my self-imposed bedtime.

What's the technique called when someone does something like this:

If we do this, A will happen. If A happens B will happen. If B happens then C will happen.

Where like you're sorta being driven down this path of logic and you aren't really left with room to go like "If A happens, D could happen instead of B"

What's that technique called?

I've just been calling it appealing to reason and logic which is wrong.

The slippery slope fallacy. It is suggests that a  relatively insignificant first event is suggested to lead to a more significant event, which in turn leads to a more significant event, and so on, until some ultimate, significant event is reached, where the connection of each event is not only unwarranted, but with each step it becomes increasingly improbable. 

For Example: Lost your pen= No pen
No pen= No notes
No notes= No study
No study= Fail
Fail= No diploma
No diploma= No work
No work= No money
No money= No food
No food= You get skinny
You get skinny= You get ugly
You get ugly= No love
No love= No marriage
No marriage= No children
No children= alone
alone= Depression
Depression= Sickness
Sickness= Death


Moral of the story, don't lose your pen!

Hoperfully not  ;D
Title: Re: VCE English Question Thread
Post by: literally lauren on May 03, 2016, 08:30:42 am
how badly would i be penalised for not concluding my language analysis? i had a comparative SAC today and managed to write everything but the conclusion. would the quality of my piece be more important than whether or not i finished? really didn't want to drop a single mark in english... oh well  :-\
Not at all, unless your teacher is a real stickler for essay structure. Even then, the quality of your analysis is where, like, 98% of the marks are. Little things like the intro + concl. are just there to make a good first and last impression, but if your analysis was good enough not to need that last impression, you'd be fine.

Not having a conclusion is unlikely to cost you a whole mark on its own (esp. in the exam when it's marked out of 10, but even for your SAC where it's out of 20) but it might feed into the marking scheme somehow.

So if you were lacking a conclusion AND your paragraph structure was imbalanced and hard to follow AND there was a bit of repetition in your analysis AND some word choice issues, then you might lose a mark or two.

I wouldn't stress about losing a mark even if that does end up happening though. It's great that you've set your standards so high, but don't let a 19/20 discourage you :)

URGENT. Have an English SAC tomorrow. I swore I'd get an early night tonight but instead I'm cramming as much info as I can before my self-imposed bedtime.

What's the technique called when someone does something like this:

If we do this, A will happen. If A happens B will happen. If B happens then C will happen.

Where like you're sorta being driven down this path of logic and you aren't really left with room to go like "If A happens, D could happen instead of B"

What's that technique called?

I've just been calling it appealing to reason and logic which is wrong.
I'd call it leading logic or cumulative logic. Technically, the formal name for it would be the slippery slope fallacy, but calling it a 'fallacy' sounds a bit evaluative. Most teachers probably wouldn't mind, but I could understand some taking issue with you using words that are critiquing their arguments instead of metalanguage that discusses how the author is being persuasive.
Title: Re: VCE English Question Thread
Post by: teacher28 on May 04, 2016, 06:11:55 pm
No. There's an actual name for the technique but I can't remember what it is. arghhh...

Most of my students have been told to identify it as"cause and effect", although most of those other appellations sound more sophisticated!
Title: Re: VCE English Question Thread
Post by: Swagadaktal on May 04, 2016, 07:54:44 pm
Hey Champ I sent you a message about your essay marking service - did you get it?
Title: Re: VCE English Question Thread
Post by: Callum@1373 on May 04, 2016, 10:09:52 pm
In regards to identity and belonging, does anyone know any people who have been in jail, promised to be good after a court hearing and then gotten back to crimes e.g murder after they are released? This is to talk about how often when people seem to have changed their identity it is just their public, displayed identity but not who they truly are on the inside  :D
Title: Re: VCE English Question Thread
Post by: Ameliajanem on May 05, 2016, 01:44:55 pm
Hey guys, i have a text response sac on next week and i was wondering how to approach structural prompts in terms of what ideas to base a paragraph on? like how do i do paragraphs for structural prompts?
Title: Re: VCE English Question Thread
Post by: Champ101 on May 05, 2016, 03:54:42 pm
Hey Champ I sent you a message about your essay marking service - did you get it?

Hey Swagadaktal,
I did indeed receive your email, however, where I am the Internet connection is very poor - will get back to you shortly
Title: Re: VCE English Question Thread
Post by: NerdyPi on May 06, 2016, 09:13:28 pm
Hi guys, for the visual aspect of language analysis, I'm really struggling with this picture. The article is basically contending that although girls achieve fantastic high school results, this doesn't translate into workplace success. I'm guessing there's a glass wall / reference to glass ceiling, but I'm really unsure as there appears to be women on both the top and bottom of the staircase??? Any ideas/help would be appreciated :)
Title: Re: VCE English Question Thread
Post by: vor0005 on May 06, 2016, 10:19:52 pm
Hi guys, for the visual aspect of language analysis, I'm really struggling with this picture. The article is basically contending that although girls achieve fantastic high school results, this doesn't translate into workplace success. I'm guessing there's a glass wall / reference to glass ceiling, but I'm really unsure as there appears to be women on both the top and bottom of the staircase??? Any ideas/help would be appreciated :)

It's just saying that men control the workforce and female positions in top jobs are at the male's discretion. The reason why there are women at the top is to highlight how the only way to make it to the top is by male approval (as indicated by the male who guards the stairs). Hope that makes sense.
Title: Re: VCE English Question Thread
Post by: NerdyPi on May 06, 2016, 10:57:32 pm
Thanks :) It definately cleared it up
Title: Re: VCE English Question Thread
Post by: Rob16 on May 07, 2016, 01:24:51 pm
I seem to always be using "positioning the reader to..........." and other repetetive conjuctive words (furthermore, etc) in my language analysis and need to vary things up a bit. Does anyone have some kind of resource with a bunch of different vocabulary that they can share with me?
Title: Re: VCE English Question Thread
Post by: literally lauren on May 08, 2016, 10:28:25 am
In regards to identity and belonging, does anyone know any people who have been in jail, promised to be good after a court hearing and then gotten back to crimes e.g murder after they are released? This is to talk about how often when people seem to have changed their identity it is just their public, displayed identity but not who they truly are on the inside  :D
This isn't exactly what you're looking for, but Robert Durst (about whom the awesome TV documentary 'The Jinx' was made) might be worth considering as an example of someone whose external/ projected identity was designed to mislead and gain the trust of others.

There's also Gary Gilmore who was a notorious repeat offender, and Jack Abbott who committed some minor offences, seemingly turned his life around, but then went back to prison after committing an even worse crime.

I'd be careful about letting an argument hinge on these individuals, though. Especially because most of these^ people were found by psychiatrists to be mentally unstable in some way. It's still fine to use them to support your discussion; just make sure you're not saying 'here's someone whose identity was a facade. Therefore, people's identities are often facades.' Perhaps use these individual cases to launch into a discussion of recidivism or the justice system if you need another facet to explore.

Hey guys, i have a text response sac on next week and i was wondering how to approach structural prompts in terms of what ideas to base a paragraph on? like how do i do paragraphs for structural prompts?
Turn that structural prompt into a non-structural prompt, and go from there :)

Eg. 'How does the author show the dangers of rebelling against authority in the text?'
--> The author suggests that rebelling against authority is dangerous in the text. Discuss.

...then each paragraph is centred around a relevant thematic/ V&V concern.

Some prompts are a little less malleable, especially if they're mentioning specific devices, but try to see structural features as evidence and not the basis for your ideas. So if you had something like

'How does the language of the text reveal the differences between men and women?'

...you would NOT want a paragraph breakdown like:
P1: here's how the metaphors reveal the differences between men and women
P2: here's how the epithets reveal the differences between men and women
P3: here's how the dialogue reveals the differences between men and women

Instead, use those features as supporting evidence within those body paragraphs and extrapolate ideas from them.

For example, if, in P1 above, I wanted to talk about how the metaphors in the text are used by male characters as a means of subtly reminding female characters that they are inferior, I would take that idea as my focus.

i.e.
P1: Throughout the text, many of the female characters are left disenfranchised and subjugated by their male counterparts.
^with that topic sentence outlining a clear thematic focus, I could then use the metaphor stuff as the supporting evidence.

Hope that makes sense; let me know if there's a specific structural prompt that's confusing you and I can try and outline some strategies :)

I seem to always be using "positioning the reader to..........." and other repetetive conjuctive words (furthermore, etc) in my language analysis and need to vary things up a bit. Does anyone have some kind of resource with a bunch of different vocabulary that they can share with me?

Yarp. See attachment. It's by no means conclusive, so I'd highly recommend adding to it as you go so you can create your own repository of good words/ sentences/ sentence structures.
Title: Re: VCE English Question Thread
Post by: japanese on May 08, 2016, 10:04:10 pm
Hi everyone!

What psychology areas would be suitable for Identity and Belonging?
Such as social psychology and within that conformity.

Thanks!~
Title: Re: VCE English Question Thread
Post by: HopefulLawStudent on May 09, 2016, 09:06:46 pm
Heyyyy.

Quick question.

If someone refers to someone by the wrong name (like calling someone "Joan" instead of "Jane"), is that a misnomer? Like does that count? I get what misnomers are in general but I'm just iffy as to whether this would be an example of a misnomer...
Title: Re: VCE English Question Thread
Post by: kimmytaaa on May 10, 2016, 11:11:01 am
Hi
I have a problem with language analysis, for language analysis I thought we write it similar to year 11 hence I am a bit confused . My teacher wrote this on the board analyzing Andrew Bolt's article 'it's a smokescreen', but I don't understand what he is trying to say. I showed it to my tutor (a current teacher as well) and he said this is full of bullshit and that my teacher should have made it more simply so that the whole class understands in which he guarantees that no one understands this. My tutor said that my teacher is only teaching 1 or 2 students in class, because this is only used for elite students (A+ students) not ordinary students like me. Tbh I am finding the vocabs a bit difficult. What do you guys think? All opinions are welcome

My teacher's analysis of Andrew Bolt's
The issue on decriminalising recreational drug use, motivated by social and international trends, has been recently debated in media circles. One piece, Andrew Bolt’s IT’S A SMOKESCREEN (Herald Sun, April 5th, 2012), contends that if drugs are legalised, ‘more people are likely to use them.’ The editor persuades in an enthusiastic style that adopts foremost sarcasm, ridicule and patronising flavour. Bolt’s craft appeals to both individuals with insight towards current government policies and initiatives towards drug use as well as possible users of drugs. Titles opening contraction ‘IT’S’, an atmosphere of conversation is established. This, passed with the smokescreen pun, offers an absolution that aligns with his contention on drug decriminalisation. The title collectively engages the reader and invites them to consult his text with its central and symbolic marijuana leaf.

Bolt’s piece mocks the “eminent Australians” by sarcastically aligning them with recreational drug use. He maintains this voice of bemusement through the repeated ‘experts’, thereby decreeing the Australia21 group. The reader’s trust in this ‘think tank’ is challenged and further by connotative ‘absurdly’, ‘simplistic’ and 'backpedaling.’ The effect is fundamentally cemented as the reader is presented with a cyclical piece that opens with an attack and ends with similar concern with ‘dangerous’. This metalanguage driven by Bolt is thwarted with casual undertones included the contraction ‘you’d’ An atmosphere of inclusion is therefore establish and, when paired with the writer’s sharp wit, enhances his position on drug. When considering the iconic marijuana leaf central to the Editorial the readership aligns this masterful and sharp use of the English language with the razor edges of the leaf.
Title: Re: VCE English Question Thread
Post by: kimmytaaa on May 10, 2016, 02:45:23 pm
Hi
Does this sound like an speech?
Hello Ladies and gentleman,
It is an honour and pleasure to be here talking about banning gambling ads during sporting events.  Gambling ads has encouraged gambling and even worse, it educates the young to accept gambling as normal. During the first game of the AFL season, “Crown Bet”, who by the way are partners of the AFL, accounted for half the gambling ads during broadcast of the game. Of the 200 ads, 34 were gambling ads. This is an attempt to “normalise” gambling to make it appear as natural as watching a game.The gambling organisation sees this as an opportunity to groom a new generation of gamblers, ensuring the continued success of their business. The basic law of gambling is that most gamblers will lose, meaning that those who accepts bets will always come out on top.

Victoria Responsible Gambling Association Chief executive, Serge Sardo, said there was widespread corner about the relationship between sport and betting. The AFL also owns or controls many pokie outlets in Victoria. These are “cash cows” for the clubs and rely on the misery and despair of these who lose money, to finance the club. Some clubs are increasingly concerned at this development and believe that the AFL should cut all contacts with gambling. This is also concern about criminal elements hijacking the competition, luring players to not play at their best, so that the criminals can win large sums when they know the outcome of a game before it has even commenced. International cricket has been a target along with tennis. Both sports have suffered because of gambling.

In Europe it was suspected that as many as 380 soccer matches were suspected of being manipulated according to the European Law Enforcement Agencies. The matches involved 15 countries and over 400 players and officials. Over 50 were arrested and charged. Do we really want to see this happening here in Australia? Is it too late?? Evidence already suggests that criminal elements have already infiltrated Australian Sports. By accepting sponsorship from gambling organisations, the League must acknowledge their support. Thus there is potential for sponsors to pressure the League because of this financial arrangement. In the past, the League has acted swiftly and decisively against players who have gambled, which they are not permitted to do under their contract. They rightly received substantial fines and lengthy suspensions. Yet to many it was only a matter of time before such transgressions occurred.

They argue that it is hypocritical permit saturation advertising of gambling ads then slap extreme punishments when players decide to bet. Many find it difficult to resist a bet and yet they are encouraged by the AFL and the advertising campaign to do so. There are many who complain that if gambling sponsorship is made illegal, the game will suffer. However not too long ago, tobacco companies were major sponsors of Australian Sport. Many insisted that the sport would die if there were no tobacco sponsorship. Yet sport has thrived without their sponsorship and more importantly the number of smokers in the community has been reduced considerably and therefore fewer deaths. It is time for the AFL to wake up and remove this scourge from sport and protect our youth.
Title: Re: VCE English Question Thread
Post by: TheLlama on May 10, 2016, 08:39:32 pm
Quote
Hi
Does this sound like an speech?

I would probably say that it doesn't. My reasoning would be that, if you take away the introduction, the "hello, ladies and gentlemen", what you're really left with is just an essay that you're reading out.

If you're looking to transform it into a speech, I would suggest that you start by establishing a sense of context. Why are you giving this speech? Who are these ladies and gentlemen? Make it clear why this speech is relevant to this particular audience, whoever they are.

Besides that, you might want to consider some stylistic changes in shifting from a written essay to a spoken piece. Have you thought of using a little bit of signposting at the outset? Or perhaps think of rewriting it while considering some rhetorical devices. Vary your sentence length. Add in some punchy language. Even if it's to be submitted as a written piece, you want to show that you've considered how it sounds. :)
Title: Re: VCE English Question Thread
Post by: josh92012 on May 10, 2016, 10:13:36 pm
I an doing a year 10 argument analysis tomorrow and i don't really understand english all to well. I was wondering how you can find out some embrace points within a certain text piece and embed them with you embrace paragraph? Also what is an embrace point within the text?
Title: Re: VCE English Question Thread
Post by: vor0005 on May 11, 2016, 12:48:04 am
Hi
I have a problem with language analysis, for language analysis I thought we write it similar to year 11 hence I am a bit confused . My teacher wrote this on the board analyzing Andrew Bolt's article 'it's a smokescreen', but I don't understand what he is trying to say. I showed it to my tutor (a current teacher as well) and he said this is full of bullshit and that my teacher should have made it more simply so that the whole class understands in which he guarantees that no one understands this. My tutor said that my teacher is only teaching 1 or 2 students in class, because this is only used for elite students (A+ students) not ordinary students like me. Tbh I am finding the vocabs a bit difficult. What do you guys think? All opinions are welcome

My teacher's analysis of Andrew Bolt's
The issue on decriminalising recreational drug use, motivated by social and international trends, has been recently debated in media circles. ✓ One piece Thus, Andrew Bolt’s pragmatic opinion piece "IT’S A SMOKESCREEN" (published in the Herald Sun, April 5th, 2012), contends (avoid dull and lifeless words like "contends" - they don't tell us anything about the tone of the piece) is vehement that if drugs are legalised, ‘more people are likely to use them.’ (I'd probably avoid quoting in the introduction; even decoratively) The editor (since when is he an editor? just refer to him as Bolt/the writer/he etc.) persuades (another dull and lifeless word. Avoid.) in an enthusiastic style that adopts foremost sarcasm, ridicule and patronising flavour. ✓ Bolt’s craft appeals to both individuals with insight towards current government policies and initiatives towards drug use as well as possible users of drugs. Titles opening contraction ‘IT’S’, an atmosphere of conversation is established. This, passed with the smokescreen pun, offers an absolution that aligns with his contention on drug decriminalisation. The title collectively engages the reader and invites them to consult his text with its central and symbolic marijuana leaf. ((Okay, you probs want to avoid analysing in the introduction of your analysis. All that an intro needs is: trigger event/problem, introduce article + sources, writer's qualified contention and introduce any visuals + their contentions very broadly. That's it...)

Bolt’s piece mocks the “eminent Australians” by sarcastically aligning them with recreational drug use. ✓ He maintains this voice of bemusement through the repeated ‘experts’, thereby decreeing the Australia21 group. The reader’s trust in this ‘think tank’ is challenged and further by connotative ‘absurdly’, ‘simplistic’ and 'backpedaling.’ The effect is fundamentally cemented as the reader is presented with a cyclical piece that opens with an attack and ends with similar concern with ‘dangerous’. This metalanguage driven by Bolt is thwarted with casual undertones included the contraction ‘you’d’ An atmosphere of inclusion is therefore establish and, when paired with the writer’s sharp wit, enhances his position on drug. When considering the iconic marijuana leaf central to the Editorial the readership aligns this masterful and sharp use of the English language with the razor edges of the leaf. (I'll leave one comment on this paragraph: Without even reading it (just by glancing over it), I have realised that your teacher has started two sentences in a row with the word 'The'. It's little things like this - not varying your sentence structure - which can lose you marks in an essay. It's blatantly obvious and very easy for the marker to spot.)

I'd have to agree 100% with your tutor here. Your teacher's analysis essay lacks any recognisable structure, and in turn, serves to confuse students.

For your intro, all you need is:
1) Generic/Broad sentence which contextualises the problem being discussed or its trigger event
2) Introduce writer, article name & type (is it an opinion piece, editorial, letter to the editor, speech etc.) & sources
3) What is the qualified contention?
4) Introduce images and their respective contentions

THAT IS IT.

Body paragraphs:
1) Topic sentence that exposes one of the writer's arguments/ideas
2) Highlight a Technique --> Quote it --> Discuss specific intended effect (relating back to the specific technique and the argument (1)
3) Repeat 2) as many times as is necessary until paragraph looks decent
4) Concluding sentence which highlights the overall effect of the techniques on the audiences (i.e. what actions/feelings/beliefs do they encourage?)

It only just hit me, but your teacher tends to use the passive voice rather than the active voice. Avoid, avoid, avoid!!!
This is a huge difference between high scoring pieces and mid-low range pieces. You won't see the passive voice in high scoring essays...

For example:
Metalanguage driven by Bolt (passive) --> Bolt's metalanguage (active)
An atmosphere of inclusion (passive) --> An inclusive atmosphere (active)

Additionally:
"readership aligns this masterful and sharp use of the English language" --> is he/she taking the piss? This is language analysis - not some sort of patronising commentary regarding the writer's ability to use English.
Title: Re: VCE English Question Thread
Post by: kimmytaaa on May 11, 2016, 09:48:11 am
I would probably say that it doesn't. My reasoning would be that, if you take away the introduction, the "hello, ladies and gentlemen", what you're really left with is just an essay that you're reading out.

If you're looking to transform it into a speech, I would suggest that you start by establishing a sense of context. Why are you giving this speech? Who are these ladies and gentlemen? Make it clear why this speech is relevant to this particular audience, whoever they are.

Besides that, you might want to consider some stylistic changes in shifting from a written essay to a spoken piece. Have you thought of using a little bit of signposting at the outset? Or perhaps think of rewriting it while considering some rhetorical devices. Vary your sentence length. Add in some punchy language. Even if it's to be submitted as a written piece, you want to show that you've considered how it sounds. :)

Thanks for your opinion
Title: Re: VCE English Question Thread
Post by: kimmytaaa on May 11, 2016, 09:52:19 am
I'd have to agree 100% with your tutor here. Your teacher's analysis essay lacks any recognisable structure, and in turn, serves to confuse students.

For your intro, all you need is:
1) Generic/Broad sentence which contextualises the problem being discussed or its trigger event
2) Introduce writer, article name & type (is it an opinion piece, editorial, letter to the editor, speech etc.) & sources
3) What is the qualified contention?
4) Introduce images and their respective contentions

THAT IS IT.

Body paragraphs:
1) Topic sentence that exposes one of the writer's arguments/ideas
2) Highlight a Technique --> Quote it --> Discuss specific intended effect (relating back to the specific technique and the argument (1)
3) Repeat 2) as many times as is necessary until paragraph looks decent
4) Concluding sentence which highlights the overall effect of the techniques on the audiences (i.e. what actions/feelings/beliefs do they encourage?)

It only just hit me, but your teacher tends to use the passive voice rather than the active voice. Avoid, avoid, avoid!!!
This is a huge difference between high scoring pieces and mid-low range pieces. You won't see the passive voice in high scoring essays...

For example:
Metalanguage driven by Bolt (passive) --> Bolt's metalanguage (active)
An atmosphere of inclusion (passive) --> An inclusive atmosphere (active)

Additionally:
"readership aligns this masterful and sharp use of the English language" --> is he/she taking the piss? This is language analysis - not some sort of patronising commentary regarding the writer's ability to use English.
Hi
What do you mean by "taking the piss"? Should I go and confront my teacher that this isn't a language analysis piece or just leave it? Also are you able to explain again about passive and active because I still don't understand that.
Title: Re: VCE English Question Thread
Post by: TheLlama on May 11, 2016, 11:41:32 am
Quote
What do you mean by "taking the piss"? Should I go and confront my teacher that this isn't a language analysis piece or just leave it? Also are you able to explain again about passive and active because I still don't understand that.

Confronting your English teacher isn't likely going to lead to a good outcome. If anything, you might want to actually just approach your teach calmly and thoughtfully, explaining that you're a little bit unsure about what's required.
The thing about language analysis: there isn't actually a "correct" structure that you need, so long as you're analysing how and why the text's creator is making specific choices about language in order to persuade the reader. That being said...

Quote
This metalanguage driven by Bolt is thwarted with casual undertones included the contraction ‘you’d’

It feels as if your teacher doesn't have the best sense of what's required of this task - there are some weird things included, and their word choice doesn't actually make... um... sense. It's the type of response you sometimes see in Year 12 writing where the student wants to sound fancy, but is really disconnected from the task. It's going to mislead students, even the very able ones. You might want to consider talking to other students to get a feel for what's going on? Seems a little odd.

The difference between active/passive:
Active: Bolt adopts a more casual tone in order to...
Passive: A more casual tone is adopted by Bolt

Typically in English, we order sentences subject-verb-object; a passive sentence flips this around.
Title: Re: VCE English Question Thread
Post by: literally lauren on May 12, 2016, 02:48:54 pm
Hi everyone!

What psychology areas would be suitable for Identity and Belonging?
Such as social psychology and within that conformity.

Thanks!~

See: here. Anything can be made suitable or relevant; it all depends on the quality of your discussion, but I'd recommend perusing some of the theories in the link above as a starting point.

Heyyyy.

Quick question.

If someone refers to someone by the wrong name (like calling someone "Joan" instead of "Jane"), is that a misnomer? Like does that count? I get what misnomers are in general but I'm just iffy as to whether this would be an example of a misnomer...

Misnomers tend to apply to more established definitions/names rather than just people. Best example of this was in the Lemony Snicket books where The Incredibly Deadly Viper was actually a sweet and cuddly snake who wouldn't hurt a fly. Or, like, The Great Northern Hotel being on the southmost part of an island, or a big rock being called The Smiling Rock when it actually looks like it has an angry frown.

Basically, a misnomer will have some kind of contradiction between the name of a thing, and the thing itself. There's nothing about the name 'Joan' that would contradict 'Jane,' but there is something in 'The Great Northern Hotel' which would contradict it being in the south.

Hope that makes sense!

My teacher's analysis of Andrew Bolt's
The issue on of decriminalising recreational drug use, motivated by social and international trends, has been recently debated in media circles just a filler statement. One piece, Andrew Bolt’s IT’S A SMOKESCREEN (Herald Sun, April 5th, 2012), contends that if drugs are legalised, ‘more people are likely to use them.’ it'd be better to actually explain the contention rather than stick a quote in and have that do all the work in this context. The editor persuades in an enthusiastic style clunky expression that adopts foremost this word doesn't belong here sarcasm, ridicule and patronising flavour umm? Bolt’s craft appeals to both individuals with insight towards current government policies and initiatives towards drug use as well as possible users of drugs specifying the audience isn't really necessary here. Titles opening contraction ‘IT’S’, an atmosphere of conversation is established Firstly, a single contraction doesn't create a conversational tone, and secondly, analysis doesn't belong in the intro!. This, passed ?? with the smokescreen pun, offers an absolution how? I'm not following the logic here -  how is the pun giving us an 'absolution?' that aligns with his contention on drug decriminalisation waaaay too general. If you're going to analyse, you want to be specific about what the effect/intention is. The title collectively this word doesn't belong engages the reader and invites them to consult this doesn't work either his text it'd be more accurate to call it an opinion piece or article with its central and symbolic how is it symbolic? What is it a symbol of? marijuana leaf.

Bolt’s piece mocks the “eminent Australians” by sarcastically aligning them with recreational drug use. Okay, this topic sentence is actually pretty decent. He maintains this voice of bemusement what voice of bemusement? This doesn't flow on from the previous sentence at all? through the repeated ‘experts’, thereby decreeing either this word doesn't belong, or this sentence is incomplete the Australia21 group. The reader’s trust in this ‘think tank’ is challenged and further by I'm guessing there are words missing here? connotative ‘absurdly’, ‘simplistic’ and 'backpedaling.’ Awful lot of quotes so far with very little actual analysis. The effect is fundamentally cemented this is another generic filler statement that could easily be cut as the reader is presented with a cyclical piece that opens with an attack and ends with similar concern with ‘dangerous’. This metalanguage driven more weird word choices by Bolt is thwarted ditto; this is actually a bit evaluative since it seems like this piece is criticising Bolt for 'thwarting' his own arguments with casual undertones included the contraction ‘you’d’ An atmosphere of inclusion is therefore how? You can't just say 'therefore' and have that be the explanation of how we got from A to B established and, when paired with the writer’s sharp wit really no need to flatter him -.-, enhances his position on drug. When considering the iconic marijuana leaf central to the editorial the readership aligns this masterful and sharp use of the English language again, pandering to the author and talking about how smart and wonderful he is just comes across as a waste of time and words with the razor edges of the leaf. huh??
I'm with TheLlama - this is nonsense :P There are parts that aren't too bad, but on the whole, the underlying analysis and the expression here really aren't good examples of the requirements of Language Analysis.

Don't "confront" your teacher in a very aggressive "hey sir/miss, you're wrong!" kind of way, but it might be worth sitting down with your teacher and working out what they expect of you. We can help you when it comes to the exam criteria (and vor's breakdown of the intro + body paragraph requirements is definitely a great framework to use) but ultimately, you want to be catering to your teacher for your SAC.

After that SAC is done, though, you can concentrate on writing pieces from an exam standpoint, which is where we and your tutor can be of more use to you :)

Title: Re: VCE English Question Thread
Post by: HopefulLawStudent on May 12, 2016, 03:57:43 pm

Misnomers tend to apply to more established definitions/names rather than just people. Best example of this was in the Lemony Snicket books where The Incredibly Deadly Viper was actually a sweet and cuddly snake who wouldn't hurt a fly. Or, like, The Great Northern Hotel being on the southmost part of an island, or a big rock being called The Smiling Rock when it actually looks like it has an angry frown.

Basically, a misnomer will have some kind of contradiction between the name of a thing, and the thing itself. There's nothing about the name 'Joan' that would contradict 'Jane,' but there is something in 'The Great Northern Hotel' which would contradict it being in the south.

Thanks Lauren! :)
Title: Re: VCE English Question Thread
Post by: kimmytaaa on May 13, 2016, 11:55:10 am
Quote
Hope that makes sense!
I'm with TheLlama - this is nonsense :P There are parts that aren't too bad, but on the whole, the underlying analysis and the expression here really aren't good examples of the requirements of Language Analysis.

Don't "confront" your teacher in a very aggressive "hey sir/miss, you're wrong!" kind of way, but it might be worth sitting down with your teacher and working out what they expect of you. We can help you when it comes to the exam criteria (and vor's breakdown of the intro + body paragraph requirements is definitely a great framework to use) but ultimately, you want to be catering to your teacher for your SAC.

After that SAC is done, though, you can concentrate on writing pieces from an exam standpoint, which is where we and your tutor can be of more use to you :)
Yeah I made one or two spelling errors, but when it comes with the sac, do I have to carter it similar with what the teacher expects (like what he has there) or can I write it the way I am comfortable with?
Title: Re: VCE English Question Thread
Post by: vor0005 on May 13, 2016, 02:46:34 pm
Yeah I made one or two spelling errors, but when it comes with the sac, do I have to carter it similar with what the teacher expects (like what he has there) or can I write it the way I am comfortable with?

To determine what your teacher expects of you, it's best to just hand in practice paragraphs or practice essays - this is the only way you can truly receive feedback that will help you in your SAC.

Experiment with the advice we have given you and see what he/she says. Then, incorporate her feedback into the drafts that follow. I know it's a very back and forth process, but this is probably the best way to improve in English.

Hope that helps.

Title: Re: VCE English Question Thread
Post by: Swagadaktal on May 14, 2016, 01:42:45 pm
Hey guys, I'm seeking help on context - I'm pretty lost

We're studying encountering conflict (the lieutenant),

I want to write an expository piece - I was thinking about something along the lines of a speech but tbh I need more ideas of which forms I can write in.

And can anyone recommend me some good books/documentaries/movies where I can gain some ideas from?

I was thinking about just spewing out an essay but that wouldn't be very good in terms of form and language choice - because I don't have anything in mind.

Any ideas on how to approach this? Sorry for the vague questions I just don't have any specific questions in mind at the moment.
Title: Re: VCE English Question Thread
Post by: vor0005 on May 14, 2016, 03:51:22 pm
Hey guys, I'm seeking help on context - I'm pretty lost

We're studying encountering conflict (the lieutenant),

I want to write an expository piece - I was thinking about something along the lines of a speech but tbh I need more ideas of which forms I can write in.

And can anyone recommend me some good books/documentaries/movies where I can gain some ideas from?

I was thinking about just spewing out an essay but that wouldn't be very good in terms of form and language choice - because I don't have anything in mind.

Any ideas on how to approach this? Sorry for the vague questions I just don't have any specific questions in mind at the moment.

First and foremost, you must use the ideas from your assigned text for Encountering Conflict. Hence, a good place to start would be by reading/watching those texts and jotting down the ideas that emanate from them.

Secondly, once you have all of these ideas written down somewhere: try to think of a focus or particular topic that you are passionate about which encompasses ideas from those texts, but also, is relevant to Encountering Conflict.

Once you have picked a topic or something that you are passionate about - and they both are relevant to the Context (Encountering Conflict) and the ideas from your assigned text - then take some sort of a stance on that topic.

I understand that you'd like to do an expository pieces but the best pieces tend to incorporate features of different modes of writing (persuasive and personal); a hybrid. This allows you to use your own voice throughout your piece which provides it with that extra bit of authenticity. Remember: the worst thing you can do in context is write a boring text-responsy type of essay.

Oh and lastly: before you begin writing, take a look at the Encountering Conflict prompts from the past 8 years. Whilst writing your context piece draft, try to ensure your piece encompasses as many ideas from those prompts as possible so that you can easily mould your piece to a prompt on the actual day. (most of the prompts are very very very similar so your piece is likely to fit many of them - just make sure that your topic isn't TOO specific so that you don't restrict yourself with prompts)

Hope that makes sense.

Title: Re: VCE English Question Thread
Post by: Swagadaktal on May 14, 2016, 06:27:28 pm
First and foremost, you must use the ideas from your assigned text for Encountering Conflict. Hence, a good place to start would be by reading/watching those texts and jotting down the ideas that emanate from them.

Secondly, once you have all of these ideas written down somewhere: try to think of a focus or particular topic that you are passionate about which encompasses ideas from those texts, but also, is relevant to Encountering Conflict.

Once you have picked a topic or something that you are passionate about - and they both are relevant to the Context (Encountering Conflict) and the ideas from your assigned text - then take some sort of a stance on that topic.

I understand that you'd like to do an expository pieces but the best pieces tend to incorporate features of different modes of writing (persuasive and personal); a hybrid. This allows you to use your own voice throughout your piece which provides it with that extra bit of authenticity. Remember: the worst thing you can do in context is write a boring text-responsy type of essay.

Oh and lastly: before you begin writing, take a look at the Encountering Conflict prompts from the past 8 years. Whilst writing your context piece draft, try to ensure your piece encompasses as many ideas from those prompts as possible so that you can easily mould your piece to a prompt on the actual day. (most of the prompts are very very very similar so your piece is likely to fit many of them - just make sure that your topic isn't TOO specific so that you don't restrict yourself with prompts)

Hope that makes sense.
Thanks for the advice. Have a few questions though.

What do you mean by writing a piece that can be moulded to different prompts? A personal/persuasive hybrid - wouldn't you have different arguments throughout? And when you say personal do you mean using stuff like "I" or do you go as far as writing a creative personal essay?
If I manage to get the structure of a speech down tight I think I might be able to write different pieces, but I don't see how I'll be able to write a general piece which encompasses the ideas in so many prompts where if I change a paragraph it'll suddenly fit.
Title: Re: VCE English Question Thread
Post by: oooo on May 14, 2016, 08:18:46 pm
question: with topic sentences, can i have topic sentences? as in multiple sentences to serve as a 'topic sentence'. i remember being told that i can by one of my teachers, but only if cramming everything into one sentence makes it confusing. can someone confirm this for me? thanks  :) :)
Title: Re: VCE English Question Thread
Post by: vor0005 on May 14, 2016, 11:10:54 pm
Thanks for the advice. Have a few questions though.

What do you mean by writing a piece that can be moulded to different prompts? A personal/persuasive hybrid - wouldn't you have different arguments throughout? And when you say personal do you mean using stuff like "I" or do you go as far as writing a creative personal essay?
If I manage to get the structure of a speech down tight I think I might be able to write different pieces, but I don't see how I'll be able to write a general piece which encompasses the ideas in so many prompts where if I change a paragraph it'll suddenly fit.


Essentially, avoid writing a piece that homes in on a really specific and restrictive issue. Instead, write a piece with a broad contention and then use a variety of evidence/specific arguments to prove that broad contention.

Yes, you would have different arguments - but they should all ultimately lead to the one conclusion. And yes, when I say 'personal', using the first person 'I' would be a good way to go about it (but do so scarcely!)

Lastly, with the prompts: they are all very similar - they have to be! Notice how they're all so broad and adaptable. Indeed, some may be more difficult than others and may require some more moulding (i.e. changing a few sentences or altering your word choices). However, some of them you won't have to change anything at all...

Either way: it is completely unreasonable for a student to produce a high quality piece on the day under timed conditions without having a pre-prepared response (to a large extent). That being said, though: don't take the prompt for granted - make sure that it is addressed. But keep in mind, the prompt is more of a 'moral of the story' rather than a text response prompt, so treat it accordingly.

Title: Re: VCE English Question Thread
Post by: literally lauren on May 15, 2016, 11:59:29 am
Yeah I made one or two spelling errors, but when it comes with the sac, do I have to carter it similar with what the teacher expects (like what he has there) or can I write it the way I am comfortable with?
Cater to your teacher for SACs; cater to the examiners in the exam.

If your teacher is open to different approaches and is willing to be flexible, then you might be okay. Otherwise, it's best that you learn to modify your writing style and focus so you can get decent SAC marks.

E.g. if your teacher wants you to discuss whether or not techniques are persuasive in L.A. (which you're not meant to do,) just do it for your SAC anyway. Then, when you get to the exam, you can just fulfil the task criteria and ditch the evaluation.

question: with topic sentences, can i have topic sentences? as in multiple sentences to serve as a 'topic sentence'. i remember being told that i can by one of my teachers, but only if cramming everything into one sentence makes it confusing. can someone confirm this for me? thanks  :) :)

You can have multiple sentences, especially if you're unpacking an idea that's especially complex. However, if you find yourself always needing 2-4 sentences at the start of your paragraphs, perhaps consider making your expression a bit more concise. It's possible your writing is just a little convoluted, but you're absolutely allowed to spend a little longer explaining your points before you start delving into your examples.

Either way: it is completely unreasonable for a student to produce a high quality piece on the day under timed conditions without having a pre-prepared response (to a large extent). That being said, though: don't take the prompt for granted - make sure that it is addressed. But keep in mind, the prompt is more of a 'moral of the story' rather than a text response prompt, so treat it accordingly.

One thing I'd challenge here: it's a much safer idea to have various responses (or even better, bits and pieces of responses) which is then adapted and reassembled based on the prompt vs. having a 'swiss cheese' piece that tries to cater to every possible prompt by switching a few words or examples around. I definitely agree with your point that students can't be expected to come into the exam with their brains as blank slates and just have all these spontaneous epiphanies about the nature of conflict, but I think those who have multiple options up their sleeve are in a better position than those who invest all their hopes in one "malleable" piece. There are students who get lucky are able to make this work, but they're few and far between.

Yes, you could conceivably pick out ~10 prompts at random and have them touch on a similar thematic concern (e.g. 'the way people respond to conflict tells us something about our values' is a very common one) and maybe you'll end up with an exam prompt that relates to this same idea. But if you want to feel prepared for Context, you're going to need to actually prepare for a whole host of potential prompts.

No one expected that hideous 'conflicts of conscience' prompt in my year level, and if all you'd been preparing for were the ideas from past years' exams and practice prompts that were available at the time...
(i.e. 2008: extraordinary responses from ordinary people
2009: victims show us what's important
2010: remaining a bystander is difficult
2011: compromise is important
2012: conflict changes our priorities)

...then you'd be left high and dry trying to cobble together a response to the core of the 2013 prompt.

I tend to be an advocate for what I call 'frameworks' that you can mould and reconfigure to suit material, as opposed to memorised responses. I know a lot of teachers/ tutors/ students who'd disagree with me here (and tbh, I'm eagerly awaiting 2017 when we don't have to worry about this awful AOS anymore :P) but if you are wanting to commit to a memorised piece, just be aware of the risks.
Title: Re: VCE English Question Thread
Post by: Swagadaktal on May 15, 2016, 12:10:09 pm
Cater to your teacher for SACs; cater to the examiners in the exam.

If your teacher is open to different approaches and is willing to be flexible, then you might be okay. Otherwise, it's best that you learn to modify your writing style and focus so you can get decent SAC marks.

E.g. if your teacher wants you to discuss whether or not techniques are persuasive in L.A. (which you're not meant to do,) just do it for your SAC anyway. Then, when you get to the exam, you can just fulfil the task criteria and ditch the evaluation.

You can have multiple sentences, especially if you're unpacking an idea that's especially complex. However, if you find yourself always needing 2-4 sentences at the start of your paragraphs, perhaps consider making your expression a bit more concise. It's possible your writing is just a little convoluted, but you're absolutely allowed to spend a little longer explaining your points before you start delving into your examples.

One thing I'd challenge here: it's a much safer idea to have various responses (or even better, bits and pieces of responses) which is then adapted and reassembled based on the prompt vs. having a 'swiss cheese' piece that tries to cater to every possible prompt by switching a few words or examples around. I definitely agree with your point that students can't be expected to come into the exam with their brains as blank slates and just have all these spontaneous epiphanies about the nature of conflict, but I think those who have multiple options up their sleeve are in a better position than those who invest all their hopes in one "malleable" piece. There are students who get lucky are able to make this work, but they're few and far between.

Yes, you could conceivably pick out ~10 prompts at random and have them touch on a similar thematic concern (e.g. 'the way people respond to conflict tells us something about our values' is a very common one) and maybe you'll end up with an exam prompt that relates to this same idea. But if you want to feel prepared for Context, you're going to need to actually prepare for a whole host of potential prompts.

No one expected that hideous 'conflicts of conscience' prompt in my year level, and if all you'd been preparing for were the ideas from past years' exams and practice prompts that were available at the time...
(i.e. 2008: extraordinary responses from ordinary people
2009: victims show us what's important
2010: remaining a bystander is difficult
2011: compromise is important
2012: conflict changes our priorities)

...then you'd be left high and dry trying to cobble together a response to the core of the 2013 prompt.

I tend to be an advocate for what I call 'frameworks' that you can mould and reconfigure to suit material, as opposed to memorised responses. I know a lot of teachers/ tutors/ students who'd disagree with me here (and tbh, I'm eagerly awaiting 2017 when we don't have to worry about this awful AOS anymore :P) but if you are wanting to commit to a memorised piece, just be aware of the risks.
Given that we have 2 different context sacs and I'm planning on writing up 2 different malleable pieces - would I be lulled into a false sense of security into believing that at least 1 of my pieces would be able to apply to the prompt?

And I'm thinking about writing up an editorial which covers indigenous disadvantage - I've done plenty of research on this topic.
I'd be able to transform my piece by focusing on different issues within indigenous disadvantage - and since there are like 4 issues i'd cover under the 1 umbrella term "indigenous disadvantage" i'd have plenty of different views to look from.

However, my concern is that I won't be able to right a high marking essay because an editorial is kinda eh. - But I hope I'm wrong. What is your experience with editorials?
Title: Re: VCE English Question Thread
Post by: TheLlama on May 15, 2016, 01:12:35 pm
Given that we have 2 different context sacs and I'm planning on writing up 2 different malleable pieces - would I be lulled into a false sense of security into believing that at least 1 of my pieces would be able to apply to the prompt?

And I'm thinking about writing up an editorial which covers indigenous disadvantage - I've done plenty of research on this topic.
I'd be able to transform my piece by focusing on different issues within indigenous disadvantage - and since there are like 4 issues i'd cover under the 1 umbrella term "indigenous disadvantage" i'd have plenty of different views to look from.

However, my concern is that I won't be able to right a high marking essay because an editorial is kinda eh. - But I hope I'm wrong. What is your experience with editorials?

One of the constant warnings is to avoid coming in with a prepared piece. It depends how malleable they are, but you really need to be able to show that you've considered the prompt. Far better, as Lauren suggested, to think in terms of frameworks or elements. Even though the prompts tend to be quite broad (for the exam, they have to allow access to all students, and have to fit in with four different texts), they tend to be relatively focused on a central idea.

It's all about the quality of the individual piece rather than the particular form it takes. It's ultimately up to what you're able to do with one. I would suggest that you think about where your strengths are as a writer, the particular approach that allows you to show off! If you're going down the editorial path, you might want to look at some international newspapers (The Guardian, NYTimes) that tend to have especially strong editorials. Though an op-ed/opinion piece might give you a little bit more flexibility as it will allow you to step into the first person, weave in anecdotes and play around a little bit more.

I would encourage you to try a few different approaches; until you've written a few and compared them, you won't know what suits you and where your strengths are!
Title: Re: VCE English Question Thread
Post by: Swagadaktal on May 15, 2016, 02:26:02 pm
One of the constant warnings is to avoid coming in with a prepared piece. It depends how malleable they are, but you really need to be able to show that you've considered the prompt.
Yeah I was intending to write a piece which covers multiple issues - so I have my general frame work there but I won't come in prepared with a line by line recital of my prepared piece - rather I have the structure of my whole piece pre planned, and maybe my title + intro.

When you say the word "considered" - how much discussion do we have to have of the prompt? I'm still uncertain on how much I have to relate my piece to the prompt and to the texts - do I need to show a sophisticated understanding of the text? Like I can extract ideas and include them in my piece but I'm not dealing with the different layers of conflict that closely...

Title: Re: VCE English Question Thread
Post by: TheLlama on May 15, 2016, 03:53:26 pm
When you say the word "considered" - how much discussion do we have to have of the prompt? I'm still uncertain on how much I have to relate my piece to the prompt and to the texts - do I need to show a sophisticated understanding of the text? Like I can extract ideas and include them in my piece but I'm not dealing with the different layers of conflict that closely...

Yes! If you haven't already, make sure you look at the exam criteria on the VCAA website.
Basically, the prompt is there for a reason. If it wasn't, you could come in and write any response. You need to be relevant to it and ideally show that you recognise some of the complex ideas it contains. Your response has to relate to it rather than avoid it. By dealing with the different layers of conflict - by exploring the context and text in light of the prompt - you'll do better. The stuff it sounds like you're not planning to do is probably what you should be doing!  :D
Title: Re: VCE English Question Thread
Post by: kimmytaaa on May 15, 2016, 05:53:40 pm
Cater to your teacher for SACs; cater to the examiners in the exam.

If your teacher is open to different approaches and is willing to be flexible, then you might be okay. Otherwise, it's best that you learn to modify your writing style and focus so you can get decent SAC marks.

E.g. if your teacher wants you to discuss whether or not techniques are persuasive in L.A. (which you're not meant to do,) just do it for your SAC anyway. Then, when you get to the exam, you can just fulfil the task criteria and ditch the evaluation.

You can have multiple sentences, especially if you're unpacking an idea that's especially complex. However, if you find yourself always needing 2-4 sentences at the start of your paragraphs, perhaps consider making your expression a bit more concise. It's possible your writing is just a little convoluted, but you're absolutely allowed to spend a little longer explaining your points before you start delving into your examples.

One thing I'd challenge here: it's a much safer idea to have various responses (or even better, bits and pieces of responses) which is then adapted and reassembled based on the prompt vs. having a 'swiss cheese' piece that tries to cater to every possible prompt by switching a few words or examples around. I definitely agree with your point that students can't be expected to come into the exam with their brains as blank slates and just have all these spontaneous epiphanies about the nature of conflict, but I think those who have multiple options up their sleeve are in a better position than those who invest all their hopes in one "malleable" piece. There are students who get lucky are able to make this work, but they're few and far between.

Yes, you could conceivably pick out ~10 prompts at random and have them touch on a similar thematic concern (e.g. 'the way people respond to conflict tells us something about our values' is a very common one) and maybe you'll end up with an exam prompt that relates to this same idea. But if you want to feel prepared for Context, you're going to need to actually prepare for a whole host of potential prompts.

No one expected that hideous 'conflicts of conscience' prompt in my year level, and if all you'd been preparing for were the ideas from past years' exams and practice prompts that were available at the time...
(i.e. 2008: extraordinary responses from ordinary people
2009: victims show us what's important
2010: remaining a bystander is difficult
2011: compromise is important
2012: conflict changes our priorities)

...then you'd be left high and dry trying to cobble together a response to the core of the 2013 prompt.

I tend to be an advocate for what I call 'frameworks' that you can mould and reconfigure to suit material, as opposed to memorised responses. I know a lot of teachers/ tutors/ students who'd disagree with me here (and tbh, I'm eagerly awaiting 2017 when we don't have to worry about this awful AOS anymore :P) but if you are wanting to commit to a memorised piece, just be aware of the risks.
Thankyou Lauren
Title: Re: VCE English Question Thread
Post by: HayleyGrimshaw on May 16, 2016, 05:52:53 pm
Hello..... i am writing a creative piece at the moment about encountering conflict in The Rugmaker..... and i don't know what to do for it any ideas?????
Title: Re: VCE English Question Thread
Post by: literally lauren on May 17, 2016, 01:19:31 pm
Hello..... i am writing a creative piece at the moment about encountering conflict in The Rugmaker..... and i don't know what to do for it any ideas?????
1. Find a/some conflict(s) in Rugmaker.
2. Think about what Mazari is saying about this conflict.
3. Come up with a contention statement that augments/ challenges this idea.
4. Devise a creative format that demonstrates your contention
5. Embed this creative piece with structural features and/or references to Rugmaker.

For example:
1. the internal conflict Najaf had to deal with even after leaving Afghanistan
2. Mazari suggests that even after a conflict is resolved/ concluded, its effects can still have a profound impact on the individuals involved.
3. Our own actions in times of conflict can affect us just as much as the events and circumstances around us.
4. Short story about a person who committed atrocities in the Middle East in order to survive, and is haunted by what they did.
5. Including references to the text (e.g. perhaps your protagonist could be one of the men responsible for torturing Najaf, but was only following orders and feared for his own safety) and modifying certain quotes (e.g."All I wanted to do was to stand up on the soil of a land where rockets did not land on my house in the middle of the night'" --> "All I wanted to do was to stand up on the soil of a land where I did not have to fire rockets and people's houses in the middle of the night.")

This'll also depend on what sort of creative piece you intend to write. The above outline would be for a kind of POV narrative/ short story, but if you wanted to do a feature article or some letter writing, you could do that as well. I'd also recommend getting your teacher's input since creative writing is really subjective, so checking to see what they're expecting of you would be a good place to start if you're really lost :)
Title: Re: VCE English Question Thread
Post by: Callum@1373 on May 19, 2016, 09:52:17 pm
We're studying the 60 minutes Sally faulkner child abduction case, what's a good word I can use in my essays that describes journalists when they are believed to only be pursuing a story for the financial benefit and are not taking morals into account?
Title: Re: VCE English Question Thread
Post by: literally lauren on May 22, 2016, 08:54:23 pm
We're studying the 60 minutes Sally faulkner child abduction case, what's a good word I can use in my essays that describes journalists when they are believed to only be pursuing a story for the financial benefit and are not taking morals into account?

Words for people who disregard morals and are only in it for the money:
- avaricious
- mercenary
- amoral/ immoral
- rapacious
- unscrupulous
- iniquitous (might be a bit strong :P)
- reprobate
- shameless

Choose whatever suits your writing voice :) You may also find it worthwhile to spell out the fact that these people pursued their own selfish ends at the expense of the greater moral good or w/e if that's a central point in your discussion. Having a neat word to sum it up is good, but you might need to flesh this idea out a bit too.
Title: Re: VCE English Question Thread
Post by: IDK on May 27, 2016, 12:35:43 am
Hey Lauren, given the use of passive voice in vce is discouraged by some, perhaps disliked by many, what would you say is a good balance between active and passive or should it depend solely on what I feel works for me? I feel that my writing looks much neater using passive.
Title: Re: VCE English Question Thread
Post by: kimmytaaa on May 29, 2016, 07:28:12 pm
Hi
I need a bit of clarification here, so my teacher said that at my school we don't do comparing language analysis for both sacs and exams so does that mean for my final exams do I write separate pieces of the language analysis?
Title: Re: VCE English Question Thread
Post by: Beast10 on May 29, 2016, 08:21:50 pm
Hi,
Does anyone know which stories from The Lot: in words talk about non-conformity?
Also does anyone know how to start an essay about Michael Leunig being a non-conformist?
Thanks!
 ;D
Title: Re: VCE English Question Thread
Post by: Beast10 on May 29, 2016, 08:26:30 pm
Hi
I need a bit of clarification here, so my teacher said that at my school we don't do comparing language analysis for both sacs and exams so does that mean for my final exams do I write separate pieces of the language analysis?
The final exam for this year is gonna be just text responses so if you are in year 12, your final exam will only consist of text responses (3) and language analysis will only be assessed via sacs.
Title: Re: VCE English Question Thread
Post by: TheLlama on May 29, 2016, 08:54:34 pm
The final exam for this year is gonna be just text responses so if you are in year 12, your final exam will only consist of text responses (3) and language analysis will only be assessed via sacs.
The final exam this year will be based on the same structure as last year's, meaning that you will do a text response, a contexts piece and a language analysis.
Not sure where you're getting your information o.O
Title: Re: VCE English Question Thread
Post by: Beast10 on May 29, 2016, 09:06:44 pm
The final exam this year will be based on the same structure as last year's, meaning that you will do a text response, a contexts piece and a language analysis.
Not sure where you're getting your information o.O

Didn't the study design change? that's what the teachers from our school told us...  :-\
Title: Re: VCE English Question Thread
Post by: literally lauren on May 29, 2016, 09:17:55 pm
Hey Lauren, given the use of passive voice in vce is discouraged by some, perhaps disliked by many, what would you say is a good balance between active and passive or should it depend solely on what I feel works for me? I feel that my writing looks much neater using passive.
I'm yet to hear a good argument for why the active voice is always inherently better :-\

In some circumstances (esp. for Language Analysis and to some extent Text Response) when you're trying to discuss what the author is doing, it often makes more sense to say 'The author verbs this idea' than 'this idea is verbed by the author,' but sentence variation is important, so I don't really get why some teachers say the passive voice is never justifiable.

Try not to 'rely' on it too much, but in circumstances where the passive voice seems more natural or efficient, I'd say just go for it.

Hi
I need a bit of clarification here, so my teacher said that at my school we don't do comparing language analysis for both sacs and exams so does that mean for my final exams do I write separate pieces of the language analysis?
I'm not sure what your teacher is trying to say, but you will do one Language Analysis SAC in semester 1 (for Unit 3,) and one in the exam. They'll probably both be comparative, but that will depend on your school and what the examiners end up writing for the end of the year.
What do you mean by writing 'separate pieces of the language analysis?' As in, you write two separate essays for the different written material? Because you'd definitely only need to write a single piece ???

The final exam for this year is gonna be just text responses so if you are in year 12, your final exam will only consist of text responses (3) and language analysis will only be assessed via sacs.
There was a study design change, but it won't affect the current Year 12s as it's coming into effect for next year's (2017) exam. But that exam doesn't consist of three Text Response tasks either, so I have no idea what your teacher would be referring to.
Full explanation here. And if you want to know what this year's exam will look at, just check out the 2015 version as it'll essentially be the same, albeit with a few new texts replacing old ones.

My guess is that your teacher was talking about the fact that the new SD for next year will contain one Text Response task, and one Comparative Task (kind of like T.R. but with two texts) ... and they just added those up to three Text Response tasks for some reason? Though Language Analysis is definitely still a part of the course, so idk. Maybe talk to your teacher; they're certainly screwing you over if you're in Year 12 and they're telling you to only prepare for a triple Text Response exam  :-\  :-\
Title: Re: VCE English Question Thread
Post by: kimmytaaa on May 30, 2016, 09:02:55 am
What my teacher meant is that we are writing 2 separate pieces of the language analysis instead of one piece altogether. So I'm a bit confused with that. :(
Title: Re: VCE English Question Thread
Post by: Elizawei on May 30, 2016, 06:09:21 pm
Hey guys,

does these sentences make sense? Like is there any syntax/grammatical errors here?

Quote
By unleashing its power, the ubiquitous Church is able to silence Galileo. Ostensibly, this may appear as the resolution of the conflict, however the internal conflict within Galileo is never resolved.

Title: Re: VCE English Question Thread
Post by: reachfortheatars on May 30, 2016, 06:19:54 pm
Hey guys,

does these sentences make sense? Like is there any syntax/grammatical errors here?



It seems like you have a tautology with "ostensibly"/"this may appear". I'd say leave out "ostensibly" - the sentence still sounds fine. I'm not sure about the comma after "by unleashing its ubiquitous power", but I think it'll be fine if you leave it.
Title: Re: VCE English Question Thread
Post by: Elizawei on May 30, 2016, 06:48:34 pm
It seems like you have a tautology with "ostensibly"/"this may appear". I'd say leave out "ostensibly" - the sentence still sounds fine. I'm not sure about the comma after "by unleashing its ubiquitous power", but I think it'll be fine if you leave it.

Thanks! :D
Title: Re: VCE English Question Thread
Post by: reachfortheatars on May 30, 2016, 07:26:49 pm
no worries  :)
Title: Re: VCE English Question Thread
Post by: kimmytaaa on May 31, 2016, 02:30:32 pm
Hi Guys
what type of article is this? is it a editorial, opinion piece or something?
http://www.dailytelegraph.com.au/lifestyle/health/body-soul-daily/bree-warren-australian-model-sparks-plussize-debate-over-what-is-normal/news-story/97f2d6cc893ab7daa129bb84235e8618
Title: Re: VCE English Question Thread
Post by: rosemaria20 on May 31, 2016, 05:38:32 pm
Hi Guys, :)
I'm studying Every man in this village is a liar by Megan Stack
"Individuals who are faced with conflict are forever shaped by their experience” is an essay topic we were given.
If I was to write an expository essay what ideas could I use?
Thanks in advance

Title: Re: VCE English Question Thread
Post by: kimmytaaa on June 02, 2016, 02:05:55 pm
Hi
I have a question, just had my language analysis sac today session 2 today and the article that was given was dated in 2012 but my school changed it to 2015, are they allowed to do that?
Title: Re: VCE English Question Thread
Post by: Cowboy on June 02, 2016, 02:35:17 pm
Hi
I have a question, just had my language analysis sac today session 2 today and the article that was given was dated in 2012 but my school changed it to 2015, are they allowed to do that?

Yeah. The article can be manipulated, created, changed.

I heard that VCAA creates its own articles so they dont have problems with copyright. If exams can have 'made up' articles then i dont see why a school cant just change a date on an actual article.
Title: Re: VCE English Question Thread
Post by: NerdyPi on June 02, 2016, 03:56:12 pm
Hey guys, so I had my language analysis SAC today, and after I got home I (stupidly) looked up the article to reread/attempt to predict my score. Anyway, I realized that I completely misunderstood the contention of one of the comments :( (4 were included in the SAC). The only thing I really said about this particular comment was that it supported and added to the original articles contention... Does anyone know how badly this will affect my score, or how many marks (out of 20) I'd lose for this :( If it helps, I had 4 main body paragraphs, 3 of which focused solely on the article, and the last about the comments (but my main analysis was for a different comment).

Thanks for any ideas...
Title: Re: VCE English Question Thread
Post by: HopefulLawStudent on June 02, 2016, 04:12:42 pm
Hi
I have a question, just had my language analysis sac today session 2 today and the article that was given was dated in 2012 but my school changed it to 2015, are they allowed to do that?

My school did this for one of our practices. It was so glaringly obvious too because it referred to an event that happened in 2013 as something that happened "last week". I have no idea if it's okay for an actual SAC though...

Hey guys, so I had my language analysis SAC today, and after I got home I (stupidly) looked up the article to reread/attempt to predict my score. Anyway, I realized that I completely misunderstood the contention of one of the comments :( (4 were included in the SAC). The only thing I really said about this particular comment was that it supported and added to the original articles contention... Does anyone know how badly this will affect my score, or how many marks (out of 20) I'd lose for this :( If it helps, I had 4 main body paragraphs, 3 of which focused solely on the article, and the last about the comments (but my main analysis was for a different comment).

How badly did you misconstrue it? Was it like the comment was actually completely mocking the original article and you said it was supporting the article or....? Ultimately, it's up to your marker (it is, after all, an internal assessment)... just don't do it for the exam. If it's any consolation: your essay will be marked holistically so if the rest of your essay is a-okay than you screwing up the contention of one eensy comment shouldn't adversely affect your mark from like a 19 or 20 out of 20 to a 1 out of 20.
Title: Re: VCE English Question Thread
Post by: Swagadaktal on June 02, 2016, 04:28:17 pm
Yeah. The article can be manipulated, created, changed.

I heard that VCAA creates its own articles so they dont have problems with copyright. If exams can have 'made up' articles then i dont see why a school cant just change a date on an actual article.
Actually yes they do.
They once modified a piece to make it worse than it actually was and the person who wrote the piece complained (I think there was some legal action but I don't know with certainty so i wont say there were)
Title: Re: VCE English Question Thread
Post by: Alter on June 02, 2016, 04:48:19 pm
^My brother sat that exam, lol. It was the 2011 English exam with some article about tattoos which you could find with some googling. I don't think you can even access the LA part of it from the VCAA website, though.
Title: Re: VCE English Question Thread
Post by: literally lauren on June 02, 2016, 04:50:53 pm
Hi Guys
what type of article is this? is it a editorial, opinion piece or something?
http://www.dailytelegraph.com.au/lifestyle/health/body-soul-daily/bree-warren-australian-model-sparks-plussize-debate-over-what-is-normal/news-story/97f2d6cc893ab7daa129bb84235e8618
I'd probably call this an opinion piece, since editorials have to be written by the editor of a publication, and this is written by someone called Lizza Gebilagin, who isn't the editor of The Daily Tele.

Hi Guys, :)
I'm studying Every man in this village is a liar by Megan Stack
"Individuals who are faced with conflict are forever shaped by their experience” is an essay topic we were given.
If I was to write an expository essay what ideas could I use?
Thanks in advance

Are you looking for examples from your set text, or external evidence?

Hi
I have a question, just had my language analysis sac today session 2 today and the article that was given was dated in 2012 but my school changed it to 2015, are they allowed to do that?
Technically they're not meant to do this...

From the SD:
(http://i.imgur.com/8plwEQB.png)


However, it is common practice at a lot of schools, (and even more technically, that excerpt of the SD is structurally ambiguous, since it might be saying 'schools have to choose articles that have been in the media since Sept. 1' OR 'schools have to choose articles about an issue, and that issue has to have been in the media since Sept. 1') so I doubt there's much you can do. Unless it was about an issue that was totally out of date and therefore confusing, hopefully it won't have mattered too much anyway.

Actually yes they do.
They once modified a piece to make it worse than it actually was and the person who wrote the piece complained (I think there was some legal action but I don't know with certainty so i wont say there were)
Yeah, they totally got sued back in 2011, which is why you can't view the Section C of that year, and they didn't even publish any samples in the Assessor's Report :P :P

(you can view it here though, if you want to see how atrocious it was. Awfully condescending tone they used, and the contention is all over the place imo)

Long story short, yes, they modified an existing article by a lady called Helen Razer, changed her name to Helen Day, and thought that'd be enough to cover their tracks. But a bunch of bitter kids from that year level found the original article and sent a bunch of hate mail to the author (WHY'D YOU WRITE SUCH A DIFFICULT PIECE YOU @#S@#%$) and made a fb page called 'Getting a tattoo just to spite Helen' or something along those lines, which I actually found quite funny. The author then contacted VCAA to ask why so many grumpy Year 12s were taking out their frustrations on her, and they had to issue an apology.

But since that whole debacle, they've just written original pieces, as Cowboy pointed out.
(...or maybe they've just been more covert in their plagiarism :P)
Title: Re: VCE English Question Thread
Post by: kimmytaaa on June 02, 2016, 06:15:46 pm
I'd probably call this an opinion piece, since editorials have to be written by the editor of a publication, and this is written by someone called Lizza Gebilagin, who isn't the editor of The Daily Tele.

Are you looking for examples from your set text, or external evidence?
Technically they're not meant to do this...

From the SD:
(http://i.imgur.com/8plwEQB.png)


However, it is common practice at a lot of schools, (and even more technically, that excerpt of the SD is structurally ambiguous, since it might be saying 'schools have to choose articles that have been in the media since Sept. 1' OR 'schools have to choose articles about an issue, and that issue has to have been in the media since Sept. 1') so I doubt there's much you can do. Unless it was about an issue that was totally out of date and therefore confusing, hopefully it won't have mattered too much anyway.
Yeah, they totally got sued back in 2011, which is why you can't view the Section C of that year, and they didn't even publish any samples in the Assessor's Report :P :P

(you can view it here though, if you want to see how atrocious it was. Awfully condescending tone they used, and the contention is all over the place imo)

Long story short, yes, they modified an existing article by a lady called Helen Razer, changed her name to Helen Day, and thought that'd be enough to cover their tracks. But a bunch of bitter kids from that year level found the original article and sent a bunch of hate mail to the author (WHY'D YOU WRITE SUCH A DIFFICULT PIECE YOU @#S@#%$) and made a fb page called 'Getting a tattoo just to spite Helen' or something along those lines, which I actually found quite funny. The author then contacted VCAA to ask why so many grumpy Year 12s were taking out their frustrations on her, and they had to issue an apology.

But since that whole debacle, they've just written original pieces, as Cowboy pointed out.
(...or maybe they've just been more covert in their plagiarism :P)
Oh I didn't know about this, but Thankyou everyone for the clarification because I went home and searched up the article and I was like why is it dated 2015 when the actually article was written in 2012. However it does relate to article A which was ' baby's gender should be a choice' by Rita Panahi. Also another question,is this an Australian article or not because this was article B?
http://www.patheos.com/blogs/ellenpainterdollar/2012/09/why-allowing-parents-to-choose-their-babys-gender-is-wrong/
Title: Re: VCE English Question Thread
Post by: Gogo14 on June 03, 2016, 12:29:45 pm
Confused on how to structure the body paragraphs for lang analysis. How do you organise the paragraphs? Do you split each by grouping the metalanguage?Chronological progression?tone changes?

Also, would you include the analysis of the image as a separate paragraph or incorporate bits of them into other body paragraphs?
Thnx
Title: Re: VCE English Question Thread
Post by: oooo on June 03, 2016, 08:54:13 pm
Confused on how to structure the body paragraphs for lang analysis. How do you organise the paragraphs? Do you split each by grouping the metalanguage?Chronological progression?tone changes?

Also, would you include the analysis of the image as a separate paragraph or incorporate bits of them into other body paragraphs?
Thnx

This is the structure I've been taught, and recommended to by, but it is just one way at approaching arguement analysis (I'm assuming you're doing English under the new study design?):

** Just a note, I had a dodgy teacher teach me this so please don't take my word for it :P

- Start with your standard language analysis introduction (context, author, audience, contetion etc..)
- A paragraph analysing how the arguement is presented (stuff like; why is X argument said before Y argument? Why does author make Z assertion before presenting X argument? Why is the contention stated at the end?)
- two or more body paragraphs with each analysing one argument (here you analyse the arguement technique used and how language is used to support the argument)
- finish with a conclusion if there is time

Also, regarding the images I believe it is best to weave its analysis into your discussion of language techniques. It just shows you have a more holistic and detailed understanding of the piece. So to answer your question yes, you should incorporate bits of it into your other body paragraphs.





Title: Re: VCE English Question Thread
Post by: HopefulLawStudent on June 03, 2016, 08:57:39 pm
Disclaimer: You're a 2017 kid and I have no idea what changes they're making but I'm running on the assumption LA isn't changing too much next year. I'm giving you advice from a 2016 standpoint though so I may be wrong.

However... Assuming LA isn't changing too much in 2017:

Chronological progression is still a no go zone.

Images need to be incorporated into your body paragraphs.

And paragraphing depends. My school teaches the grouping the sub-arguments which is similar to Lauren's key player method. Explanation below, copy and pasted from one of Lauren's previous posts that I saved at one point to show to my teacher for some reason I can't rlly remember anymore.

Quote
It's a simple enough concept, but can make for a sophisticated way of structuring your analyses. A 'player' is basically a party involved in the issue, eg. for the 2012 Exam paper, the players included: schoolchildren, books/ebooks (players don't have to be sentient), parents/teachers, and the author herself. It's easy enough to identify who or what the key players are, but the nuance is in determining the author's portrayal/ positioning of them. (eg. ebooks are something overpriced and pale in comparison to the thrill of genuine books) and thus how the readership is made to view the issue through this player.
Instead of dealing with articles chronologically, the player approach is much more flexible and highly recommended. I used the chronological method for a while, but found I had to keep jumping around either because I wanted to link to a similar point later on, or because I'd forgotten something earlier, meaning my essay lacked cohesion. Grouping paragraphs by players is also beneficial when dealing with multiple articles, as it forces you to compare and contrast, instead of dealing with each text as a separate construct. The exam pieces usually involve a single text and multiple visuals, so this can be a much more practical way of setting up your essay.
For the aforementioned example, you might chose to dedicate one paragraph to the treatment of books as opposed to ebooks (though generally the core player needs discussing throughout your essay,) one to children/students, and one to parents and teachers. For an issue with many players, these can often be grouped together in relatively easy ways. Otherwise, if there are a lot of minor players, a few mentions here and there are sufficient.
This approach is not foolproof, and there are situations where it can turn otherwise solid analysis into a convoluted and nonsensical piece of writing, but overall it's the most reliable method I've found.
Title: Re: VCE English Question Thread
Post by: oooo on June 03, 2016, 09:08:05 pm
I'd have to agree 100% with your tutor here. Your teacher's analysis essay lacks any recognisable structure, and in turn, serves to confuse students.

For your intro, all you need is:
1) Generic/Broad sentence which contextualises the problem being discussed or its trigger event
2) Introduce writer, article name & type (is it an opinion piece, editorial, letter to the editor, speech etc.) & sources
3) What is the qualified contention?
4) Introduce images and their respective contentions

THAT IS IT.

Body paragraphs:
1) Topic sentence that exposes one of the writer's arguments/ideas
2) Highlight a Technique --> Quote it --> Discuss specific intended effect (relating back to the specific technique and the argument (1)
3) Repeat 2) as many times as is necessary until paragraph looks decent
4) Concluding sentence which highlights the overall effect of the techniques on the audiences (i.e. what actions/feelings/beliefs do they encourage?)

It only just hit me, but your teacher tends to use the passive voice rather than the active voice. Avoid, avoid, avoid!!!
This is a huge difference between high scoring pieces and mid-low range pieces. You won't see the passive voice in high scoring essays...

For example:
Metalanguage driven by Bolt (passive) --> Bolt's metalanguage (active)
An atmosphere of inclusion (passive) --> An inclusive atmosphere (active)

Additionally:
"readership aligns this masterful and sharp use of the English language" --> is he/she taking the piss? This is language analysis - not some sort of patronising commentary regarding the writer's ability to use English.

Hello! Just wondering, what do you mean by images and their respective contentions? As in what the images are saying? For example, if the image is a bald smoker with rotten teeth, would the 'respective contention' be that smoking is bad or something along those lines?

Also, a more general question: are contractions considered informal or formal language? Can I use them in language analysis essay?

Thanks in advance for all help : ))
Title: Re: VCE English Question Thread
Post by: vor0005 on June 04, 2016, 05:42:14 pm
Hello! Just wondering, what do you mean by images and their respective contentions? As in what the images are saying? For example, if the image is a bald smoker with rotten teeth, would the 'respective contention' be that smoking is bad or something along those lines?

Also, a more general question: are contractions considered informal or formal language? Can I use them in language analysis essay?

Thanks in advance for all help : ))

Image's contention = what broad stance/opinion does it take on the issue?
E.g. Accompanying the piece is a pessimistic image which warns of the harmful ramifications that smoking poses for our society.

Then in your body para, you do a much more specific analysis:
E.g. The dreary and dull facial expression of the smoker coupled with the dark and ominous background exposes the miserable circumstances which these individuals must endure on a daily basis. Consequently, a sense of fear is aroused in smokers who are enticed to recognise just how sorrowful their lives may become if they allow their addictions with smoking to thrive.

And with contractions, you can use them but only if they don't mar your clarity - so make sure they fit/sound like they should be there. Sometimes, contractions can ruin flow. Otherwise, they're fine :)

Hope that helps :D
Title: Re: VCE English Question Thread
Post by: oooo on June 04, 2016, 09:14:55 pm
Image's contention = what broad stance/opinion does it take on the issue?
E.g. Accompanying the piece is a pessimistic image which warns of the harmful ramifications that smoking poses for our society.

Then in your body para, you do a much more specific analysis:
E.g. The dreary and dull facial expression of the smoker coupled with the dark and ominous background exposes the miserable circumstances which these individuals must endure on a daily basis. Consequently, a sense of fear is aroused in smokers who are enticed to recognise just how sorrowful their lives may become if they allow their addictions with smoking to thrive.

And with contractions, you can use them but only if they don't mar your clarity - so make sure they fit/sound like they should be there. Sometimes, contractions can ruin flow. Otherwise, they're fine :)

Hope that helps :D

Thanks for the help! :) If you don't mind, can you please give me an example of contractions ruining the flow and where it is used correctly. It's just that I've always been taught that contractions are a big no no in essays and I'm still a bit confused.
Title: Re: VCE English Question Thread
Post by: Beast10 on June 05, 2016, 12:55:06 am
Can someone send me sample text response and language analysis piece plzzzzzzzzzz as I'm having trouble obtaining high marks.
also I follow the structure of TEEL which right now is not helping me obtain high marks
thanks! ;D
Title: Re: VCE English Question Thread
Post by: upandgo on June 05, 2016, 01:47:48 am
i know in the english exam you can't use a particular text in both sections A and B, but why do they have this rule? and would you get an automatic 0 on both pieces for doing so?
Title: Re: VCE English Question Thread
Post by: Apink! on June 06, 2016, 12:53:10 pm
I have a GAT question.

For the second writing task, where you're given a general prompt, can I write my response in a Context-y expository style (so exploring all facets of the issue) Or is my aim to convince someone?

Thank you so much! :)
Title: Re: VCE English Question Thread
Post by: oooo on June 06, 2016, 04:23:50 pm
Another GAT question; for the first writing task can I write creatively? As in still present the information but weave it into a story. For example, the 2015 GAT information was all about the future. In this case, would it be okay if I wrote a story on the future whilst still presenting the information?
Title: Re: VCE English Question Thread
Post by: upandgo on June 06, 2016, 06:22:27 pm
Another GAT question; for the first writing task can I write creatively? As in still present the information but weave it into a story. For example, the 2015 GAT information was all about the future. In this case, would it be okay if I wrote a story on the future whilst still presenting the information?

yep im pretty sure you can  :) i did last year and there was no issue
Title: Re: VCE English Question Thread
Post by: Gogo14 on June 06, 2016, 07:33:14 pm
In the GAT for writing task 1, do you have to present all of the information or just most?thnx
Title: Re: VCE English Question Thread
Post by: literally lauren on June 06, 2016, 07:38:49 pm
Confused on how to structure the body paragraphs for lang analysis. How do you organise the paragraphs? Do you split each by grouping the metalanguage?Chronological progression?tone changes?

Also, would you include the analysis of the image as a separate paragraph or incorporate bits of them into other body paragraphs?
Thnx
As others have mentioned, I would highly recommend structuring by sub-arguments (as you're in Year 11, this is even more important for you guys) as it will show your assessor that you have more of an understanding of the arguments. The SD changes aren't that major for Language Analysis, but basically there's going to be a shift away from pointing out metalanguage towards your grasp of the author's contention and sub-contention, so a structure that has the same focus will likely work to your advantage.

Also, better to incorporate the visual rather than deal with it on its own :)

Can someone send me sample text response and language analysis piece plzzzzzzzzzz as I'm having trouble obtaining high marks.
also I follow the structure of TEEL which right now is not helping me obtain high marks
thanks! ;D
Check out the resources and samples here :)

i know in the english exam you can't use a particular text in both sections A and B, but why do they have this rule? and would you get an automatic 0 on both pieces for doing so?
My understanding is that if you use a Section A text in Section B (even if it's not the Section A text you studied,) they cross out anything in your Section B piece that pertains to that Section A text. So if you write a whole body paragraph on Identity and Belonging (which is a Sec. B Context AOS) about Tobias Wolff's memoir 'This Boy's Life' (which is a Sec. A text,) they'd just cross out that whole paragraph and mark what was left. And if you wrote a creative piece that was from the perspective of the main character in a Section A text, you'd be in even bigger trouble, and would probably be looking at a flat 0... maybe a 2/10 if they were generous.

I believe the rationale here is that they don't want it to be possible for a student to get through Year 12 English using a single text, and they they force you to study different stuff throughout the year to make you as well read as possible (lol, 4 books) without straining the curriculum too much. But don't worry - once I take over VCAA and establish my reich, I'll be sure to get rid of all those pesky nonsense rules.

Also, a question from myself, are conclusions needed for arguement analysis essays (the 2017 onward 'language analysis')?
Conclusions aren't technically 'needed' at the moment, but they are *definitely* good to have, and a really strict marker might dock a point or two if you didn't have one (though they can usually only justify this if there are some other flaws in your essay too - an otherwise perfect piece with awesome analytical skills is highly unlikely to get a 9/10 just because it lacked a conclusion.)

My tentative answer is yes, even though I only have a grasp of what the new SD looks like in theory and not in practice, so my answer is subject to change pending VCAA actually releasing more information about the new and updated areas of study :)

I have a GAT question.

For the second writing task, where you're given a general prompt, can I write my response in a Context-y expository style (so exploring all facets of the issue) Or is my aim to convince someone?

Thank you so much! :)
Your primary aim is to be persuasive. If you want to do that by exploring different facets of arguments you can, but you need to use one of the prompts as your CONTENTION! Don't just use them all as a general starting point and talk about the issue/idea they're raising - formulate a strong argument, and you'll do much better than those who are just writing vague stuff about the topic.

Another GAT question; for the first writing task can I write creatively? As in still present the information but weave it into a story. For example, the 2015 GAT information was all about the future. In this case, would it be okay if I wrote a story on the future whilst still presenting the information?
Yes, you absolutely can. Just don't let your creativity get in the way of the task itself, which is to present the information you're given in a neat and orderly fashion. Slightly imaginative pieces can be really interesting, but because you're not technically marked on your creative ability for WT1, there's no requirement to do this unless you think it'd suit your style or be much easier for you.

In the GAT for writing task 1, do you have to present all of the information or just most?thnx
Definitely most, but certainly not all. For example, the material is about the harvesting of wheat and one if the images is a graph of the top ten wheat-harvesting countries, mentioning all ten would be too excessive. Instead, picking out maybe the top three and then some other generalisation (e.g. 'seven of the world's top ten wheat-harvesting countries are located in Asia' or w/e) would be fine.

I remember being told to aim to include at least 80% of the material - I doubt that's the exact figure they use, but hopefully it gives you some idea of what warrants mentioning and what's less important :)
Title: Re: VCE English Question Thread
Post by: HasibA on June 06, 2016, 09:03:07 pm
does anyone have additional criteria for the writing tasks in the GAT tmr? the ones given are vague and i want to score highly so i can avoid dodgy sac scaling ahha
further, does anyone have tips for scoring really well on the writing tasks? like what should i ensure / cater for to please the assessors?
thanks friends! peace and love
Title: Re: VCE English Question Thread
Post by: Alter on June 06, 2016, 09:15:53 pm
You can't avoid 'dodgy SAC scaling' by doing well in the GAT. It only comes into play if you need a derived score or something similar.

The tasks my appear vague, but that's just because they're simple and open ended. Just do what they say in whatever form seems suitable. And don't stress -- chances are that you probably won't need the GAT. It's like a security blanket.
Title: Re: VCE English Question Thread
Post by: HasibA on June 06, 2016, 09:16:54 pm
You can't avoid 'dodgy SAC scaling' by doing well in the GAT. It only comes into play if you need a derived score or something similar.

The tasks my appear vague, but that's just because they're simple and open ended. Just do what they say in whatever form seems suitable. And don't stress -- chances are that you probably won't need the GAT. It's like a security blanket.
hmm, i see! Thanks for your insight, Alter :)
Title: Re: VCE English Question Thread
Post by: Yakooza123 on June 06, 2016, 09:33:51 pm
Need help with this prompt for No Sugar
"How does Jack Davies use dramatic elements to explore racism, dispossession and identity in No Sugar?

I understand the elements of racism, dispossession and identity in the play, however, I can't quite get my head around what 'dramatic elements' mean? Any help would be appreciated
Title: Re: VCE English Question Thread
Post by: HopefulLawStudent on June 06, 2016, 09:43:04 pm
I know the GAT doesn't matter; hell, I've been that person who in the weeks leading up to tomorrow who's been telling all my friends AN users and practically anyone who cared that it didn't matter but I'm curious (okay, I lied, I'm actually just asking for a friend who's just a little stressed but is too gun-shy to post this herself):

For writing task 1:

Do we need to have an intro and/or a conclusion for the GAT? The friend who I'm asking for just does has this subargument approach and clusters them so that it's essentially like three separate pieces of writing so like she did a practice for the 2014 chess writing task 1 and basically had 3 paragraphs that went something like this:

1. How chess has changed over time
2. The popularity of chess
3. The advantages of chess

Does she need something to tie all three of these paragraphs into one?
Title: Re: VCE English Question Thread
Post by: literally lauren on June 07, 2016, 09:15:28 am
Need help with this prompt for No Sugar
"How does Jack Davies use dramatic elements to explore racism, dispossession and identity in No Sugar?

I understand the elements of racism, dispossession and identity in the play, however, I can't quite get my head around what 'dramatic elements' mean? Any help would be appreciated
'Dramatic elements' would just be the kinds of structural things that Davis employs in the play (e.g. stage directions, language, dramatic irony, symbolism, etc.) Most of your essay will still be focusing on those three key themes, but when you're dealing with textual evidence, just make sure you're highlighting some of these 'elements' using metalanguage :)
I know the GAT doesn't matter; hell, I've been that person who in the weeks leading up to tomorrow who's been telling all my friends AN users and practically anyone who cared that it didn't matter but I'm curious (okay, I lied, I'm actually just asking for a friend who's just a little stressed but is too gun-shy to post this herself):

For writing task 1:

Do we need to have an intro and/or a conclusion for the GAT? The friend who I'm asking for just does has this subargument approach and clusters them so that it's essentially like three separate pieces of writing so like she did a practice for the 2014 chess writing task 1 and basically had 3 paragraphs that went something like this:

1. How chess has changed over time
2. The popularity of chess
3. The advantages of chess

Does she need something to tie all three of these paragraphs into one?
Both approaches would probably be fine. I'd try and write an intro + concl. just for the sake of having a nice structure, but (like a Lit essay) if the start of your first paragraph is a bit introduction-y and the end of your last is a bit conclusion-y, that'd probably suffice too. So long as you're not actually making arguments or extrapolating from/analysing the material and are just summarising the information that's been provided, you and your friend should be fine :)
Title: Re: VCE English Question Thread
Post by: Apink! on June 07, 2016, 02:12:31 pm
Could someone please help me with identifying the contention in this writing piece?

Gone in 60 Minutes... Doug's Blog posted on April 14, 2016

Gosh, what can you say about the sorry saga of Sally Faulkner, Tara Brown, the 60 Minutes crew and those boneheads who claim to be “experts” in returning children that have allegedly been abducted across international borders? (If you’ve missed it, here’s the latest summary.)

I know nothing about the law in Lebanon, but I hope it is gracious and forgiving. These are not serious criminals who need to be locked away for a very long time. Not even the boneheads.

It seems likely that they have almost certainly broken some laws. Significant laws too, not the trivial kind that normally get a rap on the knuckles. But perhaps the mother’s love for her children will mitigate the outrage and perhaps the father will be prepared at least to reduce whatever charges he brings against anyone. Perhaps.

But even if the law is lenient, the consequences ought to be far reaching:

It’s difficult to see how any court in Australia or Lebanon can now support full custody rights for the mother. If she comes out of this with generous visitation rights, she should be grateful to her ex-husband and the law.

For the 60 Minutes crew, only time will tell where the truth really lies but on the surface of it, it looks like this has been a classic case of a TV documentary team trying to make the news, or at least to be part of it, to be a player rather than a reporter. If there is any truth to that, then many more people than just the crew who are currently in jail ought to be sacked.

And for the boneheads, one would hope that their business model (and any others like it) will now be completely discredited and they never again get the chance to take advantage of a distraught and vulnerable mother.

Rev Douglas R. Robertson is the Senior Minister, The Scots' Church Melbourne, Australia
Title: Re: VCE English Question Thread
Post by: HopefulLawStudent on June 07, 2016, 03:01:14 pm
Could someone please help me with identifying the contention in this writing piece?

Gone in 60 Minutes... Doug's Blog posted on April 14, 2016

Gosh, what can you say about the sorry saga of Sally Faulkner, Tara Brown, the 60 Minutes crew and those boneheads who claim to be “experts” in returning children that have allegedly been abducted across international borders? (If you’ve missed it, here’s the latest summary.)

I know nothing about the law in Lebanon, but I hope it is gracious and forgiving. These are not serious criminals who need to be locked away for a very long time. Not even the boneheads.

It seems likely that they have almost certainly broken some laws. Significant laws too, not the trivial kind that normally get a rap on the knuckles. But perhaps the mother’s love for her children will mitigate the outrage and perhaps the father will be prepared at least to reduce whatever charges he brings against anyone. Perhaps.

But even if the law is lenient, the consequences ought to be far reaching:

It’s difficult to see how any court in Australia or Lebanon can now support full custody rights for the mother. If she comes out of this with generous visitation rights, she should be grateful to her ex-husband and the law.

For the 60 Minutes crew, only time will tell where the truth really lies but on the surface of it, it looks like this has been a classic case of a TV documentary team trying to make the news, or at least to be part of it, to be a player rather than a reporter. If there is any truth to that, then many more people than just the crew who are currently in jail ought to be sacked.

And for the boneheads, one would hope that their business model (and any others like it) will now be completely discredited and they never again get the chance to take advantage of a distraught and vulnerable mother.

Rev Douglas R. Robertson is the Senior Minister, The Scots' Church Melbourne, Australia

While those responsible for the failed kidnapping attempt may have erred and broken international law - an act that would normally bring with it serious ramifications - their cases must be dealt with in a lenient and compassionate manner because the backlash from this failed attempt have been punishment enough.
Title: Re: VCE English Question Thread
Post by: oooo on June 08, 2016, 09:41:16 am
Hello! A few questions:

- In orals, how much sub-arguments should I have? I've been told by some teachers to just include 2 (to get to the point quickly) and some 3. Can someone please confirm which is the best approach to take? :(
- In Arguement analysis essays (new L.A), does the introduction need an audience and tone? Or can I just include context, contention, piece details (author, title, publication) and images?


Title: Re: VCE English Question Thread
Post by: fonetik on June 08, 2016, 08:53:05 pm
Hi there oooo(d)

1) I always found my orals to be me just talking like John Green (WELCOMETOCRASHCOURSEHISTORY) as I tried to emphisise all my points.  If you're anything like me, and your orals have a tendency to run overtime rather than under - I would suggest you trim the argumentative fat.  This would also help keep your central message clear, but keep in mind that your oral should be a singular pathway towards your proposed solution/contention, and if your sub-arguments don't *build* to your point, then there isn't any point in having them.  Flesh out your oral with events, stories and information that keep the momentum of your speech going.

2) This is just hearsay, but Argument analysis essays are apparently directed (and marked) with respect to author intent as opposed to affect on audience.  It had always been the common practice at my school to consider tone its own paragraph, but really, authors will adopt various tones that support a point made, and a well structured essay is knowing when to link these tones to other specific examples of language and/or constructed depictions of the subjects in a piece.  I would argue tonality is usually better utilized in your body paragraphs, and your introduction should be the preamble beforehand.  It should be good preamble, but it's still preamble, and less rewardable than devoting more time to your body paragraphs.  I think setting up the audience is a good idea however, but don't jam it in if it's not fitting. 

If its sounding like there's no concrete rules for this stuff, then congratulations!  All the structure taught is a rubric for success,
but really, its about writing the best piece linguistically and analytically.  Hooray for ambiguity!
Title: Re: VCE English Question Thread
Post by: HopefulLawStudent on June 08, 2016, 09:20:21 pm
Hello! A few questions:

- In orals, how much sub-arguments should I have? I've been told by some teachers to just include 2 (to get to the point quickly) and some 3. Can someone please confirm which is the best approach to take? :(
- In Arguement analysis essays (new L.A), does the introduction need an audience and tone? Or can I just include context, contention, piece details (author, title, publication) and images?

As orals are internal assessments, I would personally advise you do whatever your teacher slash whoever's marking your oral tells you to do.
Title: Re: VCE English Question Thread
Post by: Apink! on June 09, 2016, 02:23:21 pm
Can someone tell me a list of possible audiences for LA?
For example, what if the piece is just aimed at the general public? Could I say, progressive readers within the general public? What else is there?

Also, could someone please mark my LA in the essay marking thread.  :'(
Title: Re: VCE English Question Thread
Post by: Apink! on June 09, 2016, 04:24:58 pm
If you're doing a comparative LA, and the one article turns out to be a lot smaller than the other one, how can I structure the comparative analysis?
Title: Re: VCE English Question Thread
Post by: HopefulLawStudent on June 09, 2016, 04:38:23 pm
Don't mention the general public... ever. It's like a big no-go.

It's a bit hard to list possible audiences for LA because they tend to vary so much; you sorta have to just deal with them on a article to article basis imo.

Focus more heavily on the longer article. The spread of your analysis should match the spread of the articles. So like (someone correct me if I'm wrong) if one article takes up 30% of the page(s) you have to analyse, it should take up about 30% of your essay.
Title: Re: VCE English Question Thread
Post by: vor0005 on June 10, 2016, 01:47:16 pm
Can someone tell me a list of possible audiences for LA?
For example, what if the piece is just aimed at the general public? Could I say, progressive readers within the general public? What else is there?

Also, could someone please mark my LA in the essay marking thread.  :'(
Audiences/readers/citizens/society

And then if it's talking about a particular topic such as: The proliferation of e-books in society and their benefits

Audience could be:
Those with a vested interested in new technology are positioned to....
Those who starkly oppose the uprise of e-books are positioned to..

So you can kind of turn the argument/idea into an audience.

Hope that makes sense :)
Title: Re: VCE English Question Thread
Post by: wingsofadragon on June 11, 2016, 10:17:37 pm
Hmm not sure if this is the right place to ask, but I got an average of A for unit 3 in English, do I still have a chance of getting 40 at the end of the year if I do really well on the exam?
My cohort is quite small but our top ranked are very strong (I think I'm ranked within the top 1/3rd atm)

Thankssss
Title: Re: VCE English Question Thread
Post by: Sine on June 11, 2016, 10:30:33 pm
Hmm not sure if this is the right place to ask, but I got an average of A for unit 3 in English, do I still have a chance of getting 40 at the end of the year if I do really well on the exam?
My cohort is quite small but our top ranked are very strong (I think I'm ranked within the top 1/3rd atm)

Thankssss
Definitely still possible to get 40+  :)

I know that this will vary from between every student but what are the "general" things that differ between a 8/10 essay and a 9-10/10 essay?

Title: Re: VCE English Question Thread
Post by: Apink! on June 12, 2016, 02:51:31 pm
Just a quick question.

For comparative LA, let's say I get 1 long article and 1 short article (let's say a comment) wouldn't it be more  reasonable to do a block analysis? (I want to do integrated whenever I can, but is that possible)

If I get 2 similar length pieces, I can defs do integrated.

Can someone clarify that for me?
Much thanks :) :)
Title: Re: VCE English Question Thread
Post by: Alter on June 12, 2016, 03:07:27 pm
Just a quick question.

For comparative LA, let's say I get 1 long article and 1 short article (let's say a comment) wouldn't it be more  reasonable to do a block analysis? (I want to do integrated whenever I can, but is that possible)

If I get 2 similar length pieces, I can defs do integrated.

Can someone clarify that for me?
Much thanks :) :)
Yeah, of course. There's no clear-cut rule to say that you need to dedicate a paragraph solely to separate pieces. If you can intertwine your analysis so that you group similar ideas together, it could have the potential to improve your piece. Don't limit yourself with paragraph structures if you know you can meet the aim of the task.
Title: Re: VCE English Question Thread
Post by: HopefulLawStudent on June 12, 2016, 04:21:15 pm
I'm pro-integration but block analysis is a good fall-back.
Title: Re: VCE English Question Thread
Post by: Apink! on June 12, 2016, 04:25:58 pm
Thank you Alter and Hopefullylawstudent!

That's what I thought too. Thanks!
Also, do you know whether I can write only 2 paragraphs and get away with it for integrated style?
The paragraphs tend to be really long, and I saw my teacher write only 2 and some pieces floating around AN just 2.
All these years, I thought you need 3 body paragraphs, but is 2 okay for LA? Would I lose marks for just wrting 2? (probs need to check with my Eng teacher but ehh)
Title: Re: VCE English Question Thread
Post by: HopefulLawStudent on June 12, 2016, 07:51:19 pm
It's not so much "how many paragraphs you write" but rather, "do you analyse an even spread of the material?"

Tbh it doesn't rlly matter how many paragraphs, so long as you have more than one body para in your essay and your paragraphs aren't so long they're going for like 6 pages each.
Title: Re: VCE English Question Thread
Post by: chenay0123 on June 12, 2016, 09:19:50 pm
Hi, can someone please explain what the difference between Themes and Views & Values for Text Response is? And how your approach to prompts would differ

thanks :)
Title: Re: VCE English Question Thread
Post by: literally lauren on June 12, 2016, 09:53:48 pm
Hi, can someone please explain what the difference between Themes and Views & Values for Text Response is? And how your approach to prompts would differ
Theme: family.
Views & values statement: family can help us persist through trauma

Theme: the horrors of war
Views & values statement: war can have profound and lingering impacts on people long after it has ended

Theme: compassion
Views & values statement: compassion is a good thing

Make sense? :) Basically...

Theme: a thing.
Views & values statement: an idea that says something about a thing.


So in terms of how you'd reflect this in your essay: if you're given a thematic prompt, then you'll lean slightly towards unpacking that theme and how it's evident in the text, only really rounding out to a V&V assertion at the end of your paragraphs. But for a V&V prompt, you'll dedicate a little more time to this broader message or idea, so maybe the last three sentences in each paragraph will be dedicated to this more general concern.

Ultimately, you can't talk about a V&V idea without dealing with the themes it implicates, and you shouldn't talk about themes without zooming out to contemplate the views and values by the ends of your paragraphs anyway, so both are pretty much equally important :)
Title: Re: VCE English Question Thread
Post by: blacksanta62 on June 13, 2016, 04:51:14 pm
Can anyone suggest some reading for my context: Identity and belonging?? I have a SAC next week and haven't had time (SACs!!! amirite?? :P) to read around. We've watched Invictus but that's not going to be enough for a top band response (teachers made it very clear from the beginning that this isn't a text response). Things more recent in the news would be great :D or things which aren't overused e.g. MLK ( 8)) and Nelson Mandela

I'll keep searching and reading but someones else's guidance would be greatly appreciated :) Thank you
Title: Re: VCE English Question Thread
Post by: HopefulLawStudent on June 13, 2016, 05:38:22 pm
Try here: ~*Context External Examples Guide*~
Title: Re: VCE English Question Thread
Post by: HopefulLawStudent on June 13, 2016, 08:03:59 pm
Sorry to burst your bubble but... it all depends on how long your articles are. That sorta set out may work for two articles of equal length, but what happens if you get one article that is 4 pages long and one article that is less than a page long?
Title: Re: VCE English Question Thread
Post by: HopefulLawStudent on June 13, 2016, 09:29:38 pm
I know I'm usually the one answering questions but I have a question (hopefully someone will answer it for me). For create and present, I've decided I need a Plan B just in case Plan A doesn't work for the upcoming SAC. Anyway, my question is, if I have a creative starter, how do I differentiate between where my creative bit ends and where my expository starts? Like, would I leave a whole bunch of lines blank or....? I feel like without the clear distinction, my entire piece would fall apart because there'd be this one part where it looks like I just had a huuuuuge change of heart and took my essay in a completely new direction leaving the assessor to wonder "what is going on?". We have the written explanation, I know, but what would I do in an exam situation when we don't have the written explanation?
Title: Re: VCE English Question Thread
Post by: oooo on June 14, 2016, 08:21:42 am
I know I'm usually the one answering questions but I have a question (hopefully someone will answer it for me). For create and present, I've decided I need a Plan B just in case Plan A doesn't work for the upcoming SAC. Anyway, my question is, if I have a creative starter, how do I differentiate between where my creative bit ends and where my expository starts? Like, would I leave a whole bunch of lines blank or....? I feel like without the clear distinction, my entire piece would fall apart because there'd be this one part where it looks like I just had a huuuuuge change of heart and took my essay in a completely new direction leaving the assessor to wonder "what is going on?". We have the written explanation, I know, but what would I do in an exam situation when we don't have the written explanation?

Not sure, but wouldn't asterisks do the job?
Title: Re: VCE English Question Thread
Post by: Callum@1373 on June 14, 2016, 06:34:07 pm
Is using the words 'white privilege' too informal for a language analysis? If so, what are some substitutes?
Thanks
Title: Re: VCE English Question Thread
Post by: Swagadaktal on June 14, 2016, 07:09:35 pm
Is using the words 'white privilege' too informal for a language analysis? If so, what are some substitutes?
Thanks
Depends in what context you use it - if the author of a piece implies something similar to it in a similar tone then it would fit in nicely - wouldn't seem awkward and informal. However if the piece you're analysing is super formal (i.e an editorial) that doesn't have explicit references to 'white privilege' then i dont think it'd suit it.
Title: Re: VCE English Question Thread
Post by: HasibA on June 15, 2016, 04:33:27 pm
can anyone help me with these prompts for brooklyn? i've got some ideas, but im still struggling to create a few more/ get textual details from the novel to explain my ideas :( thank you (im thinking of going with the second prompt for my essay, though)
Brooklyn by Colm Tóibín
i. ‘Eilis’ sense of duty dictates all of her decisions.’ To what extent do you agree?

OR

ii. How does Tóibín explore the complex nature of love in Brooklyn?
Title: Re: VCE English Question Thread
Post by: Photon on June 16, 2016, 08:15:27 pm
I heard that when writing an introduction in text response essays, the very first sentence has to directly address/acknowledge the essay topic.
I'm a scrub at English so please excuse my lack of knowledge :p

Thanks.

Title: Re: VCE English Question Thread
Post by: Alter on June 16, 2016, 09:05:38 pm
I heard that when writing an introduction in text response essays, the very first sentence has to directly address/acknowledge the essay topic.
I'm a scrub at English so please excuse my lack of knowledge :p

Thanks.
What's your actual question here, sorry? Are you just checking to confirm if your understanding of how to write an introduction is correct?

If so, you're definitely on the money. You'll be wanting to break down the prompt using the introduction and signpost the ideas that you will late expand on. If your introduction does not immediately link to the prompt, the marker of your paper may assume you either don't understand what's going on, or that you're trying to churn out a pre-written essay.

Hope this helps - let me know if you had a more specific question you forgot to jot down.
Title: Re: VCE English Question Thread
Post by: Photon on June 16, 2016, 09:41:32 pm
What's your actual question here, sorry? Are you just checking to confirm if your understanding of how to write an introduction is correct?

If so, you're definitely on the money. You'll be wanting to break down the prompt using the introduction and signpost the ideas that you will late expand on. If your introduction does not immediately link to the prompt, the marker of your paper may assume you either don't understand what's going on, or that you're trying to churn out a pre-written essay.

Hope this helps - let me know if you had a more specific question you forgot to jot down.

That was really helpful  :) You pretty much answered my question haha.

But as an example there is a topic for Macbeth: "Fair is foul and foul is fair". How does the equivocation play a vital role in bringin about Macbeth's destruction.

Would it be okay to start off with "William Shakespeare's play Macbeth examines the impacts of equivocal language."
Title: Re: VCE English Question Thread
Post by: Swagadaktal on June 16, 2016, 09:57:12 pm
That was really helpful  :) You pretty much answered my question haha.

But as an example there is a topic for Macbeth: "Fair is foul and foul is fair". How does the equivocation play a vital role in bringin about Macbeth's destruction.

Would it be okay to start off with "William Shakespeare's play Macbeth examines the impacts of equivocal language."
Yeah I think that's fine because you directly tackle the topic.

A lot of people are like "macbeth was set in the (fancy word)  era and it has had a (fancy word) impact which demonstrates the threats of unbridled ambition and the anarchy that spells afterwards.

And the examiners are like ???? bitch wat this gotta do with equivocation and destruction you hoe you can't just be writing your pre memorised intro that has nothing to do with this u know?

but if you directly tackle your examiner and teacher is like
(https://www.colourbox.com/preview/4058146-woman-with-double-thumbs-up.jpg)
Title: Re: VCE English Question Thread
Post by: HopefulLawStudent on June 17, 2016, 01:39:09 pm
Wordy question: Is it possible to posit a question?
Title: Re: VCE English Question Thread
Post by: upandgo on June 18, 2016, 01:55:08 am
hi everyone  :D in regards to a question i posted recently about using the same text in both sections A and B... is it safe to discuss the east german stasi in my context response even though i've written about the novel 'stasiland' in section A?
Title: Re: VCE English Question Thread
Post by: HopefulLawStudent on June 18, 2016, 09:12:25 am
hi everyone  :D in regards to a question i posted recently about using the same text in both sections A and B... is it safe to discuss the east german stasi in my context response even though i've written about the novel 'stasiland' in section A?

You can't talk about/refer to a Section A text for Section B. Ever. - My English teacher at the beginning of every single class we have because apparently last year, some students made that mistake in their exam even though they'd been explicitly warned weeks/months before NOT to do it.

You could, however, probably get away with talking about the East German Stasi so long as you don't refer to stasiland.
Title: Re: VCE English Question Thread
Post by: upandgo on June 18, 2016, 04:32:55 pm
You can't talk about/refer to a Section A text for Section B. Ever. - My English teacher at the beginning of every single class we have because apparently last year, some students made that mistake in their exam even though they'd been explicitly warned weeks/months before NOT to do it.

You could, however, probably get away with talking about the East German Stasi so long as you don't refer to stasiland.

thanks for the clarification! perhaps i should stay away from it in both practice and real exams just to be safe  :P.
Title: Re: VCE English Question Thread
Post by: Photon on June 18, 2016, 08:01:34 pm
Yeah I think that's fine because you directly tackle the topic.

A lot of people are like "macbeth was set in the (fancy word)  era and it has had a (fancy word) impact which demonstrates the threats of unbridled ambition and the anarchy that spells afterwards.

And the examiners are like ???? bitch wat this gotta do with equivocation and destruction you hoe you can't just be writing your pre memorised intro that has nothing to do with this u know?

but if you directly tackle your examiner and teacher is like
(https://www.colourbox.com/preview/4058146-woman-with-double-thumbs-up.jpg)

Thanks Swag. Great Job. Keep it up.
Title: Re: VCE English Question Thread
Post by: Photon on June 18, 2016, 08:04:54 pm
Dumb question but if I have to write a text response in an hour should I be aiming for 10 minutes per paragraph (including introduction and conclusion)? I'm talking about end-of-year year 12 exam. Unless I shouldn't be dedicating an hour to each of three essays.

xoxoxo

~Photon
Title: Re: VCE English Question Thread
Post by: HopefulLawStudent on June 18, 2016, 08:30:39 pm
Thereabouts, keeping in mind that you also need to leave some time at the end for editing.
Title: Re: VCE English Question Thread
Post by: upandgo on June 18, 2016, 09:46:54 pm
Dumb question but if I have to write a text response in an hour should I be aiming for 10 minutes per paragraph (including introduction and conclusion)? I'm talking about end-of-year year 12 exam. Unless I shouldn't be dedicating an hour to each of three essays.

xoxoxo

~Photon

thats what i do and it generally works out to be 1 hr for me (10 mins for intro, 3BP, concl.) along with 5 mins to plan and another 5 to proofread, but i'm clueless as to whether thats recommended  ::)
Title: Re: VCE English Question Thread
Post by: Alter on June 18, 2016, 09:53:03 pm
It also depends on which section you're best at, I suppose. For some, planning a response in Section A might take a bit longer because you need to recall specific evidence/quotes from the text.

Similarly, you might find that you save time on the L.A (section C) because you should be spending a lot of your reading time planning out how you'll respond to that. I think it goes without saying that if you feel like you're spending too much time on a section, you should try to wrap it up ASAP to move onto another part or you'll find yourself running out of time. Three 8/10 essays is better than two 10/10s and a 3.

For these reasons I think it's crucial that you try to sit multiple English exams under REAL timed conditions before the exam (maybe 2-3 in real conditions). This is arguably one of the best ways to prepare for the exam overall tbh.
Title: Re: VCE English Question Thread
Post by: literally lauren on June 19, 2016, 10:30:47 pm
Is using the words 'white privilege' too informal for a language analysis? If so, what are some substitutes?
Thanks
Unless you're quoting, this is probably outside the scope of the task. Hard to suggest substitutes without knowing the context, but is the author making some kind of point based on racial intolerance or something? Try to just use general vocabulary in that case, as anything as specific or pejorative as 'white privilege' is likely to not suit the analysis. That said, it's highly unlikely you'll get anything pertaining to such ideas on the exam as they try and steer away from controversial or potentially upsetting topics (i.e. the L.A. material isn't going to be about cancer patients or the road toll because that might unfairly impact students who've been affected by such things - that's why the material is usually on dull/safe stuff like gardens and e-books) :P

can anyone help me with these prompts for brooklyn? i've got some ideas, but im still struggling to create a few more/ get textual details from the novel to explain my ideas :( thank you (im thinking of going with the second prompt for my essay, though)
Brooklyn by Colm Tóibín
i. ‘Eilis’ sense of duty dictates all of her decisions.’ To what extent do you agree?

OR

ii. How does Tóibín explore the complex nature of love in Brooklyn?
I'll mainly focus on this second one because it's a structural prompt and therefore more difficult (for most students at least.) First step is to reword this into something you can build an essay around, e.g. a statement like 'Toibin portrays love as a complex emotion in Brooklyn. Discuss.' Now ask yourself in what ways love is complex (remember that you can talk about both romantic love, like that between Eilis and Tony/Jim, as well as 'storge'/familial/filial love between Eilis and her family.) Then, think about why love is complex (e.g. because it's unrequited? because of distance? because people don't love each other equally? etc.) The tricky part comes when you have to return to the 'how' part of this question and start considering the structural features Toibin employs in order to create these ideas. Brooklyn (and most novels on the VCE lists, actually) aren't all that rife with structural nuance, but you could look at some of Eilis' narration, the symbolism of things like journeys and deaths, or particular quotes about her relationships.

In general, if you're coming across prompts that you feel you don't have enough textual evidence for, going back to the text or your notes can be a good revision exercise, especially when a prompt is foregrounding ideas you haven't thought of before. In this case, we're dealing with 'the complexity of love,' which probably wasn't a key theme you found on a first reading (because it's not that much of a key theme anyway :P) but when you go back and look at certain passages explicitly looking for examples of the complexity of love, it becomes much easier to find relevant scenes and quotes :)

(https://www.colourbox.com/preview/4058146-woman-with-double-thumbs-up.jpg)
Umm... where did you get that picture of me? You think there's something funny about my trendy cardigans?? -.-

Wordy question: Is it possible to posit a question?
Yep; that's one of the very few things you can 'posit.' To 'pose a question' would probably be the more conventional expression, but 'posit' is also acceptable in that case.

Dumb question but if I have to write a text response in an hour should I be aiming for 10 minutes per paragraph (including introduction and conclusion)? I'm talking about end-of-year year 12 exam. Unless I shouldn't be dedicating an hour to each of three essays.
For three body paragraphs, I'd probably go fifteen for each B.P. and then five for the intro and concl, giving you five minutes extra whenever you need it. But this is an end-of-year exam standard, and if you're not at that stage yet (and there's no real need to be because getting to grips with the content is more important) then don't stress. Also, this will depend on your own strengths and weaknesses (e.g. you might be someone who can churn out a first B.P. in five minutes flat, but struggles to write a third even in 25 minutes because you burn out too quickly, or you might be reeeeeally slow to start and need ten whole minutes of planning time, but after that, you can write consistently quickly for 50 minutes straight) and will vary from essay to essay too, as Alter said.

Better to write a great essay slowly, then get quicker, than to be writing average essays really quickly and then hoping you're essays will spontaneously get better :)

Three 8/10 essays is better than two 10/10s and a 3.
YEEEEESSS!!! I was struggling to put this succinctly to someone the other day who was asking why they couldn't just rely on one Area of Study to pull up their grades - the fact that you Year 12's have to write all three at the end of the year and that they're equally weighted should factor into your study somehow, so if you haven't touched Text Response in a couple of months, or if you've forgotten how to do a Language Analysis, or if you're like me and loathe writing Context pieces because you know you're bad at them - START THERE. Work on your weakest thing so that you can get everything up to a decent standard and you will be in a tangibly & mathematically better position than someone who's just a 'one trick pony' on the exam.
Title: Re: VCE English Question Thread
Post by: Photon on June 20, 2016, 08:44:53 pm
Hello. So for my creative writing SAC my teacher had us write a statement of intention to explain all the techniques that were used in the actual piece itself. I was wondering if that's an actual thing that we'll be asked to write in year 12 or if that's just something my teacher does.

Thanks.
Title: Re: VCE English Question Thread
Post by: oooo on June 20, 2016, 08:51:28 pm
Hello. So for my creative writing SAC my teacher had us write a statement of intention to explain all the techniques that were used in the actual piece itself. I was wondering if that's an actual thing that we'll be asked to write in year 12 or if that's just something my teacher does.

Thanks.

Yup, it's an actual thing.   :)
Title: Re: VCE English Question Thread
Post by: Photon on June 20, 2016, 08:57:27 pm
Yup, it's an actual thing.   :)

Cheers.
Title: Re: VCE English Question Thread
Post by: blacksanta62 on June 20, 2016, 09:04:58 pm
Yo, when writing a context piece, are we expected to have quotes from our wider reading? Or can we just use them as real world examples which get our ideas across (i.e. elaboration more than quoting Mr X)? I believe that since our text is a set text we will be expected to have quotes for it though. Am I correct?

Thank you  :)
Title: Re: VCE English Question Thread
Post by: Alter on June 20, 2016, 09:18:08 pm
Yo, when writing a context piece, are we expected to have quotes from our wider reading? Or can we just use them as real world examples which get our ideas across (i.e. elaboration more than quoting Mr X)? I believe that since our text is a set text we will be expected to have quotes for it though. Am I correct?

Thank you  :)
You technically don't need any quotes for Section B. I didn't use quotes at all and it was of no detriment to my expository pieces. Obviously it'd be even harder to use them effectively in a hybrid or purely creative piece. I generally used my set text (Every Man) in the first body paragraph, and then filled in the gaps of the prompt/used countering ideas for my other body paragraphs without ever giving direct analysis of the book itself.

That being said, general knowledge of world history and current issues may help strengthen your ideas, and you should ask your teacher what they prefer, because they will be marking your SACs, not us. Moreover, I'm not saying you can't use quotes at all. If used correctly, they are amazing. However, you must be careful in making sure it doesn't look like you're writing a text response, which is a common pitfall for people in section B.

tl;dr: Quotes aren't mandatory for any section B piece, but ask your teacher what they want to see for your SACs.

Hope this clarifies. Have a nice day.
Title: Re: VCE English Question Thread
Post by: blacksanta62 on June 20, 2016, 09:27:58 pm
You technically don't need any quotes for Section B. I didn't use quotes at all and it was of no detriment to my expository pieces. Obviously it'd be even harder to use them effectively in a hybrid or purely creative piece. I generally used my set text (Every Man) in the first body paragraph, and then filled in the gaps of the prompt/used countering ideas for my other body paragraphs without ever giving direct analysis of the book itself.

That being said, general knowledge of world history and current issues may help strengthen your ideas, and you should ask your teacher what they prefer, because they will be marking your SACs, not us. Moreover, I'm not saying you can't use quotes at all. If used correctly, they are amazing. However, you must be careful in making sure it doesn't look like you're writing a text response, which is a common pitfall for people in section B.

tl;dr: Quotes aren't mandatory for any section B piece, but ask your teacher what they want to see for your SACs.

Hope this clarifies. Have a nice day.

Thanks Alter! Clarified things really well. I'll ask my teacher so that my SAC response is tailored to her liking.
Title: Re: VCE English Question Thread
Post by: Swagadaktal on June 20, 2016, 09:31:18 pm
Yo, when writing a context piece, are we expected to have quotes from our wider reading? Or can we just use them as real world examples which get our ideas across (i.e. elaboration more than quoting Mr X)? I believe that since our text is a set text we will be expected to have quotes for it though. Am I correct?

Thank you  :)
I love using one liners from the text which encapsulate main ideas of the text in my pieces. The marker will be familiar to them so there's an explicit reference and it's not text responsey if you execute it correctly.
Title: Re: VCE English Question Thread
Post by: upandgo on June 21, 2016, 01:08:15 am
hi guys  :) i'm studying a play for section B (life of galileo). in my context response, would it be necessary to briefly mention/refer to play techniques, ie. stage directions? or just focus on the ideas emanating from galileo?

thanks in advance!
Title: Re: VCE English Question Thread
Post by: kimmytaaa on June 21, 2016, 09:17:45 am
does anyone have any useful/handy resources on Every man in this village is a liar?
Thanks
Title: Re: VCE English Question Thread
Post by: HopefulLawStudent on June 21, 2016, 01:23:38 pm
hi guys  :) i'm studying a play for section B (life of galileo). in my context response, would it be necessary to briefly mention/refer to play techniques, ie. stage directions? or just focus on the ideas emanating from galileo?

thanks in advance!

As in analyse the stage directions to further your discussion? No. If you mean like quoting the stage directions, maybe, but preferably not. Remember, this isn't a text response so be v. careful it doesn't become one.

does anyone have any useful/handy resources on Every man in this village is a liar?
Thanks

Have you tried here (Text Response Resources
Title: Re: VCE English Question Thread
Post by: Alter on June 21, 2016, 01:32:31 pm
Have you tried here (Text Response Resources
Every Man is a section B text, actually :P

@kimmytaaa: This might provide some extra insights.
Title: Re: VCE English Question Thread
Post by: HopefulLawStudent on June 21, 2016, 03:46:28 pm
Whoops. :P
Title: Re: VCE English Question Thread
Post by: blacksanta62 on June 21, 2016, 05:11:55 pm
In a context response (expository) would I be allowed to use inclusive language such as we and our when referring to a sense of identity and belonging. E.g. at times who we believe we are, our sense of self, may be challenged etc, or would that not be acceptable for expository?

Another question: Will prompts given by VCAA for section B give us the ability to challenge them? The prompts I've be practicing with (only 2 so far :P) are so hard to disagree with because there just so true!

And finally, what does the ability to: "Skillfully shape ideas and arguments..." and a "sophisticated understanding of complex ideas/arguments relevant to chosen context" mean in plain English??

Thank you
Title: Re: VCE English Question Thread
Post by: HopefulLawStudent on June 21, 2016, 05:52:28 pm
Another question: Will prompts given by VCAA for section B give us the ability to challenge them? The prompts I've be practicing with (only 2 so far :P) are so hard to disagree with because there just so true!

I feel like you can explore the prompts VCAA gives but it's a bit hard to disagree with them.

For example from VCAA 2015:

Quote
‘An individual’s sense of identity and belonging changes throughout life.’

It's a bit hard to be like "nah, bruv. your identity and belonging never ever changes."
Title: Re: VCE English Question Thread
Post by: upandgo on June 21, 2016, 06:08:09 pm
In a context response (expository) would I be allowed to use inclusive language such as we and our when referring to a sense of identity and belonging. E.g. at times who we believe we are, our sense of self, may be challenged etc, or would that not be acceptable for expository?

ive read through some context responses in the assessment reports and they use inclusive language, so i think you'd be able to  :)

Another question: Will prompts given by VCAA for section B give us the ability to challenge them? The prompts I've be practicing with (only 2 so far :P) are so hard to disagree with because there just so true!
Thank you

of course! i agree, some prompts are easier to simply agree/disagree with, but there's definitely prompts out there that can be challenged  :P for example, the conflict prompt  'when conflict arises, everyone has a role', you can contend that each individual does play a role, however the actions taken by these individuals in enacting their role results in consequences that ultimately exacerbates the conflict.

And finally, what does the ability to: "Skillfully shape ideas and arguments..." and a "sophisticated understanding of complex ideas/arguments relevant to chosen context" mean in plain English??

Thank you

'skilfully shape ideas and arguments'- formulating unique and fresh ideas/arguments. so thats basically discerning themes and values in the text/film that are less obvious and linking them to the prompt you're responding to  :) for example, in the novel stasiland, stasi victim frau paul is portrayed as 'spic and span' -> perhaps one may argue that her nature to be neat and tidy may have arisen from a compulsive disorder educed by the stasi's brutality on her mindset

"sophisticated understanding of complex ideas/arguments relevant to chosen context" - linking the type of ideas mentioned above back to the prompt you're responding to

if im wrong someone please correct me!
Title: Re: VCE English Question Thread
Post by: blacksanta62 on June 21, 2016, 06:39:04 pm
I feel like you can explore the prompts VCAA gives but it's a bit hard to disagree with them.

For example from VCAA 2015:

It's a bit hard to be like "nah, bruv. your identity and belonging never ever changes."

I was just looking at that prompt on the VCAA website and I was like "Bruh" *facepalm* hahaha
Thanks HLS

ive read through some context responses in the assessment reports and they use inclusive language, so i think you'd be able to  :)

of course! i agree, some prompts are easier to simply agree/disagree with, but there's definitely prompts out there that can be challenged  :P for example, the conflict prompt  'when conflict arises, everyone has a role', you can contend that each individual does play a role, however the actions taken by these individuals in enacting their role results in consequences that ultimately exacerbates the conflict.

'skilfully shape ideas and arguments'- formulating unique and fresh ideas/arguments. so thats basically discerning themes and values in the text/film that are less obvious and linking them to the prompt you're responding to  :) for example, in the novel stasiland, stasi victim frau paul is portrayed as 'spic and span' -> perhaps one may argue that her nature to be neat and tidy may have arisen from a compulsive disorder educed by the stasi's brutality on her mindset

"sophisticated understanding of complex ideas/arguments relevant to chosen context" - linking the type of ideas mentioned above back to the prompt you're responding to

if im wrong someone please correct me!

Thanks UpandGo. Cleared things up :D
Title: Re: VCE English Question Thread
Post by: blacksanta62 on June 21, 2016, 07:15:11 pm
3 posts in one day :O
Anyway, I just wanted input from someone other than myself. When writing an expository essay (yep, it's context again :P) someone suggested making a link to my text and making a link to a world example for every body paragraph. In a SAC, would that be expected? I'll ask my teacher after finishing my 2nd practice essay (2nd time I've done this) for her opinion but I'm still not finished so AN was the next best thing.

What I've been doing is having one solid example a paragraph and elaborating on that. And so far, it's been easier to refer to world examples than text examples. If that is better, in terms f length of paras, increased complexity I'm willing and will probs rewatch the film just to have it fresh in my mind.

recommendations??

Thank you
Title: Re: VCE English Question Thread
Post by: upandgo on June 21, 2016, 07:31:35 pm
3 posts in one day :O
Anyway, I just wanted input from someone other than myself. When writing an expository essay (yep, it's context again :P) someone suggested making a link to my text and making a link to a world example for every body paragraph. In a SAC, would that be expected? I'll ask my teacher after finishing my 2nd practice essay (2nd time I've done this) for her opinion but I'm still not finished so AN was the next best thing.

What I've been doing is having one solid example a paragraph and elaborating on that. And so far, it's been easier to refer to world examples than text examples. If that is better, in terms f length of paras, increased complexity I'm willing and will probs rewatch the film just to have it fresh in my mind.

recommendations??

Thank you


in my opinion any amount of external examples is fine as long as its relevant to the point you're making. the fact that you're including them is good because if you don't your piece will sound like a text response (the mistake i made when first writing context  :P). perhaps it'd be beneficial to balance them out in your essay, in that you have an equal discussion of both external examples and textual evidence. that way your piece isnt filled with too many external examples and insufficient discussion of the text/film.

for SAC purposes it'd be best to go along with what your teacher says since he/she will be marking your piece!
Title: Re: VCE English Question Thread
Post by: upandgo on June 22, 2016, 02:34:36 pm
hi everyone  ;D in regards to this prompt (stasiland):

'how does anna funder use visual imagery to describe the cheerlessness of life in the GDR?'

is it okay to occasionally incorporate the effect-on-reader approach from LA in my response? for example, in one paragraph i mentioned funder's description of 'monstrous grey buildings' in alexanderplatz, that 'made people feel small'. is it alright if i said that this description aligns us readers to liken the great size of the buildings to the large size of the stasi, and imagine the east german citizens to feel small and powerless compared to the stasi, thereby contributing to the absence of happiness and joy in the GDR? 
 
Title: Re: VCE English Question Thread
Post by: oooo on June 29, 2016, 04:43:00 pm
Hola AtarNotes. I've been told to not mention audience in a language analysis intro (just contention, text info, images and context) but rather in the body paragraphs. How would I approach this :( ?
Title: Re: VCE English Question Thread
Post by: HopefulLawStudent on June 29, 2016, 05:49:40 pm
For your SAC, your teacher's word is law. Basically for your SACs, if your teacher says jump, you jump instead of asking too many how and why questions. If they're telling you no audience, then no audience it is.

For your exam, it's up to you as to whether or not you follow your teacher and ultimately you have to make a judgement.

I personally don't believe there is a set "you must/must not mention the audience in the intro". And it's not like VCAA examiners are cackling every time they see an essay that does/does not refer to the audience and deducting 10 marks. If you flick back to the language analysis essays that are featured in the examiner's reports there isn't much consistency re: audience imo.

From the examiner's reports:
2014 - featured an "upper range response" that completely neglected to mention the audience in the intro.
2013 - implied the audience was  "households in the surrounding area", also an upper-range
2012 - explicitly referred to an "audience of teachers, librarians and senior school students"
Title: Re: VCE English Question Thread
Post by: Swagadaktal on June 29, 2016, 07:33:21 pm
For your SAC, your teacher's word is law. Basically for your SACs, if your teacher says jump, you jump instead of asking too many how and why questions. If they're telling you no audience, then no audience it is.

For your exam, it's up to you as to whether or not you follow your teacher and ultimately you have to make a judgement.

I personally don't believe there is a set "you must/must not mention the audience in the intro". And it's not like VCAA examiners are cackling every time they see an essay that does/does not refer to the audience and deducting 10 marks. If you flick back to the language analysis essays that are featured in the examiner's reports there isn't much consistency re: audience imo.

From the examiner's reports:
2014 - featured an "upper range response" that completely neglected to mention the audience in the intro.
2013 - implied the audience was  "households in the surrounding area", also an upper-range
2012 - explicitly referred to an "audience of teachers, librarians and senior school students"
Also need to keep in mind that sometimes pieces have more than one audiences. There will be some aspects of a piece that intend to resonate with one audience but not with another. So by mentioning it in the introduction it could limit you from having a sophisticated analysis of language use and its intended audience.

But if it's a specific event where only one type of audience is present then i'd suggest mentioning it. But if it's the age for instance, which has a wide range of audiences then it's not possible to limit it to one.

*can someone vouch for this? I don't see this happening much but I think it's pretty good stuff to have in ur L.A essays
Title: Re: VCE English Question Thread
Post by: EvangelionZeta on June 30, 2016, 02:47:18 pm
Examiners mark according to the exam criteria. The exam criteria does not explicitly state you have to mention the audience in the intro. I encourage my students not to include the audience in the intro, and generally they have gotten high marks for language analysis.

The only reason IMO you should mention the intended audience is if you can make some kind of use of it, eg. by commenting on how a specific language technique might create different kinds of effects for different kinds of audiences. That way, it's being used to show the examiner that you have a more nuanced appreciation of the persuasive effects of language (which is what they're really marking you on).
Title: Re: VCE English Question Thread
Post by: NerdyPi on July 01, 2016, 08:27:12 am
Short question:
For a Medea text response essay, to what extent are we allowed to use as evidence/refer to the story of Jason and the Argonaunts?

Detailed version:
I'm current attempting to write a practice essay on the topic (from the AN sample prompts forum) " "You woman are all the same". To what extent are Jason's judgments validated in Medea? " , and I was going to write a paragraph about how Medea defined the societal expectations of women by choosing to follow/help Jason, and therefore chose her own husband (when this was usually done by husbands or fathers). However, as the prompt clear says "in Medea", I'm a bit worried about if I would be allowed to use this point, which although was referred to a lot in Medea, didn't actually occur in the play. Would this be ok, or should I stick exclusively to events that actually occur in the play? Thanks
Title: Re: VCE English Question Thread
Post by: oooo on July 01, 2016, 09:39:13 am
Hello! Does anyone have any Bladerunner (Ridley Scott film) text response prompts they are willing to share??  ;) ;)
Title: Re: VCE English Question Thread
Post by: HopefulLawStudent on July 01, 2016, 12:10:27 pm
Short question:
For a Medea text response essay, to what extent are we allowed to use as evidence/refer to the story of Jason and the Argonaunts?

Detailed version:
I'm current attempting to write a practice essay on the topic (from the AN sample prompts forum) " "You woman are all the same". To what extent are Jason's judgments validated in Medea? " , and I was going to write a paragraph about how Medea defined the societal expectations of women by choosing to follow/help Jason, and therefore chose her own husband (when this was usually done by husbands or fathers). However, as the prompt clear says "in Medea", I'm a bit worried about if I would be allowed to use this point, which although was referred to a lot in Medea, didn't actually occur in the play. Would this be ok, or should I stick exclusively to events that actually occur in the play? Thanks

Personally, I would avoid going too much into the whole "Jason and the argonauts" thing. The focus of your essay should be your text and while the text is based on the "Jason and the argonauts" myth, going on a huge spiel about it could be a bit of a risk. I would just stick exclusively to things that happen in the play.
Title: Re: VCE English Question Thread
Post by: P.GUAN on July 04, 2016, 11:36:58 pm
Hi,
I loved the  lecture today and I found it very helpful!! Just a question, what marks should I be aiming for my SACS and the exam to get a 30+ study score?

Thank you
Title: Re: VCE English Question Thread
Post by: blacksanta62 on July 07, 2016, 06:47:11 pm
Hi,
I loved the  lecture today and I found it very helpful!! Just a question, what marks should I be aiming for my SACS and the exam to get a 30+ study score?

Thank you
Can't say I know for sure, you'd want a good rank and a exam score to compliment. This is probably best answered on the VTAC score discussion side of the forum which can be found here: Victorian Technical Score Discussion
What I will stress is that if you give English your best go and ask for advice and comments from your teacher and do well on the exam as a result of the effort you put in, you'll defs get a higher SS than 30 (wouldn't that be nice :) )

Now my question: On the last day of term we did our context SAC (worth 30 marks) and I feel as if I can already improve before my next context SAC which is about 4-5 weeks away (maybe...:P) and this ones out of 50! Does writing expository limit my marks in anyway or can I still score well based on the exploration of the prompt? Secondly, does going under the word limit (1000) mean you lose a mark? Or do teachers go, "uuhh, 3 pages worth of writing, close enough"? Or do they actually count every word?

Thank you
Title: Re: VCE English Question Thread
Post by: HopefulLawStudent on July 07, 2016, 06:55:58 pm

Now my question: On the last day of term we did our context SAC (worth 30 marks) and I feel as if I can already improve before my next context SAC which is about 4-5 weeks away (maybe...:P) and this ones out of 50! Does writing expository limit my marks in anyway or can I still score well based on the exploration of the prompt? Secondly, does going under the word limit (1000) mean you lose a mark? Or do teachers go, "uuhh, 3 pages worth of writing, close enough"? Or do they actually count every word?

Thank you

The style you choose to write in should not compromise your ability to do well. You are marked on your exploration of the prompt, your use of the text, etc, not on what style you chose.

You have a word limit? O.o

No one's gonna sit there and count every word  to make sure you're within the word limit. English teachers are v. busy people (a fact that my Year 11 teacher went to great extents to highlight to her class at around SAC + Exam time) and their time is better spent doing other stuff. As long as it looks about 1000 words you should be a-okay. (i.e. longer than a page but not 100 pages long)
Title: Re: VCE English Question Thread
Post by: upandgo on July 07, 2016, 07:01:37 pm
Now my question: On the last day of term we did our context SAC (worth 30 marks) and I feel as if I can already improve before my next context SAC which is about 4-5 weeks away (maybe...:P) and this ones out of 50! Does writing expository limit my marks in anyway or can I still score well based on the exploration of the prompt? Secondly, does going under the word limit (1000) mean you lose a mark? Or do teachers go, "uuhh, 3 pages worth of writing, close enough"? Or do they actually count every word?

Thank you

i reckon you can still score high with expository! not sure if this is correct (this is from my school) but most marks come from the exploration of the prompt  :) and with word count i highly doubt they count each word but i reckon it should be fine unless its obvious that you havent written much (eg, <2 pages).
Title: Re: VCE English Question Thread
Post by: Sine on July 07, 2016, 07:06:33 pm
I know that this will vary from between every student but what are the "general" things that differ between a 8/10 essay and a 9-10/10 essay?
^
Title: Re: VCE English Question Thread
Post by: blacksanta62 on July 07, 2016, 07:18:28 pm
I'll answer your question first Sine: My teacher tells me that things like fluency and word choice (i.e. don't make it too clunky by using larger words which sound great but actually shouldn't be there) are important. It needs to "roll off the tongue".
She also says that correct punctuation and spelling (happens on my essays sometimes) are never a bad thing. All these factors could make assessors believe a mark should be deducted or added. And there's more important things like ideas used when exploring the prompt in context (the assessor will most likely have seen MLK  8) 100+ times and could be a bit bored of reading the same thing), good analysis of language ("don't just signpost persuasive techniques, I already know what they are and how they work"-My English teacher) and use of quotes and all the other jazz needed in a text response (I'm sorry, I haven't written a text response for ages and have forgotten  what makes a good text response great :P) 

i reckon you can still score high with expository! not sure if this is correct (this is from my school) but most marks come from the exploration of the prompt  :) and with word count i highly doubt they count each word but i reckon it should be fine unless its obvious that you havent written much (eg, <2 pages).
Thanks upandgo, hoping to get >22 out of 30, I would be over the moon with that
The style you choose to write in should not compromise your ability to do well. You are marked on your exploration of the prompt, your use of the text, etc, not on what style you chose.

You have a word limit? O.o

No one's gonna sit there and count every word  to make sure you're within the word limit. English teachers are v. busy people (a fact that my Year 11 teacher went to great extents to highlight to her class at around SAC + Exam time) and their time is better spent doing other stuff. As long as it looks about 1000 words you should be a-okay. (i.e. longer than a page but not 100 pages long)
The SAC sheet said we need at least 1000 words so I was weary. Thank you :)
Title: Re: VCE English Question Thread
Post by: Dem16 on July 08, 2016, 04:29:17 pm
I am about to write a context piece on Life of Galileo.....anyone got ideas?
Title: Re: VCE English Question Thread
Post by: HopefulLawStudent on July 09, 2016, 07:34:17 am
I am about to write a context piece on Life of Galileo.....anyone got ideas?

A little vague, perhaps? Some specificity would enable us to better help you out. :)
Title: Re: VCE English Question Thread
Post by: NerdyPi on July 19, 2016, 04:38:57 pm
Hi guys, so I'm trying to practice analysis the wording/implications of the text for text response (for Medea), and I was wondering to what degree this is subjective, and if so, if what I've done is ok..?

My example so far are:

"when the dice of life fall badly" - implies that as a woman/wife, Medea was simply a pawn in Jason's game, who is now on the receiving end of misfortune due to both Jason's actions and chance

"old ties of affection give way to new" - implies that any feelings Jason had for Medea were simply replaceable/a passing phrase, reinforcing the idea that he is an "unfeeling monster"

If someone could tell me if this is ok, (or if not what would be the correct approach) it would be appreciated
Thanks :)
Title: Re: VCE English Question Thread
Post by: Jakeybaby on July 21, 2016, 12:12:37 am
I'm just wondering if anyone has read/analysed Harwood's Ad Orientem poem? I am currently analysing this poem for an oral which I need to complete, I'm working my way through it, but it'd be awesome if someone would be able to assist in the finer details of the poem.

Thanks :)
Title: Re: VCE English Question Thread
Post by: literally lauren on July 21, 2016, 10:30:48 am
Hi guys, so I'm trying to practice analysis the wording/implications of the text for text response (for Medea), and I was wondering to what degree this is subjective, and if so, if what I've done is ok..?

My example so far are:

"when the dice of life fall badly" - implies that as a woman/wife, Medea was simply a pawn in Jason's game, who is now on the receiving end of misfortune due to both Jason's actions and chance yes, though you may want to consider the extent to which the fate of the characters comes down to their own actions vs. the actions of others vs. chance. This quote has more to do with chance than Jason's actions (unless you wanted to argue that the 'dice of life' also pertains to how the Gods have determined Jason's character, but that's a bit of a messy argument.)

"old ties of affection give way to new" - implies that any feelings Jason had for Medea were simply replaceable/a passing phrase, good reinforcing the idea that he is an "unfeeling monster" I think this second point is more debatable. What is monstrous about Jason's actions, exactly? He's definitely cruel from Medea's perspective as her feelings were less fleeting, but he's not really "unfeeling" just because his loyalties/priorities have shifted. That quote implies he has new affections, not that he has no affections whatsoever. If you want to see him as a more callous/manipulative character, you definitely can, but you can also see him as just a bit of a fool who wasn't as aware or cautious of Medea's feelings as he should have been.
These are some great quotes to use in essays, though they do relate to some rather debatable elements of the text; so long as you can justify the interpretations you're delving into here, you'll be solid :)

I'm just wondering if anyone has read/analysed Harwood's Ad Orientem poem? I am currently analysing this poem for an oral which I need to complete, I'm working my way through it, but it'd be awesome if someone would be able to assist in the finer details of the poem.

Thanks :)
'For some reason I can't find this poem online at all; if you have a copy do you want to type it up or scan it, and then I can help you analyse it? :) If you're looking for some general analysis there's a decent discussion of some key themes (esp. the role of women) here though I'm not sure how relevant that is for this particular poem.

Title: Re: VCE English Question Thread
Post by: Jakeybaby on July 21, 2016, 03:59:30 pm
'For some reason I can't find this poem online at all; if you have a copy do you want to type it up or scan it, and then I can help you analyse it? :) If you're looking for some general analysis there's a decent discussion of some key themes (esp. the role of women) here though I'm not sure how relevant that is for this particular poem.
Here's the poem Lauren :)

All night in hospital I law
with my intemperate lover, pain.
A sister, calm in her serene
indifferent charity, sat to pray
for me, as for all suffering men,
beside my bed. Darkness and fear
rose up between me and her prayer

eclipsing the rich world of love
in which I grew, and richer trust
of lives that with my life were nourished.
And still my violator thrust
hard, hard, into a throbbing wound.
Fast in that grip I felt all cherished
images of good dissolve.

A cock crowed, and the sister rose,
yawning, to raise the window blind--
a row of cypress, scalpel-clean,
crowning a near-by hill, incised
the first of morning. Clear between
each cypress and its neighbour glowed
a winecup space of light defined

by the dark trees. Those winecups brimmed
with a new day's untasted wine:
I drank, until my spirit leapt
high on the morning's tranquil crest
and drunken sang its hope, and blessed
all suffering and rejoicing men.
I slipped from pain's embrace, and slept.
Title: Re: VCE English Question Thread
Post by: Dem16 on July 21, 2016, 06:30:41 pm
A little vague, perhaps? Some specificity would enable us to better help you out. :)

I am looking at doing imaginative but i need some ideas to respond to the prompt - 'The strengths and weaknesses of an individual surfaces when they encounter conflict'
I prefer to use the characters and ideas in the book, Life of Galileo, rather than an external example.
Any ideas?
Title: Re: VCE English Question Thread
Post by: oooo on July 22, 2016, 09:13:37 pm
Question: if I'm given a prompt asking me to discuss 2 characters specifically, should I expand it so I talk about all characters, or just stick with the 2? The prompt I need help with is based on 1984; 'How is Winston different from Julia?'.
Title: Re: VCE English Question Thread
Post by: Maz on July 22, 2016, 09:36:39 pm
Question: if I'm given a prompt asking me to discuss 2 characters specifically, should I expand it so I talk about all characters, or just stick with the 2? The prompt I need help with is based on 1984; 'How is Winston different from Julia?'.
Hey,
hopefully, I can provide some assistance  :)
I would really stick to the two characters specifically. this is kind of like haveing a question based on setting, but then doing a paragraph on language with no link to setting. You can definitely talk about these two characters and then branch it to others through their relationship, however, don't have your focus on the other character; stay close to the character you are asked about  :)
This is probably the best way to maximise the 'answering the question' criteria. Take special care that when linking, keep say 85-90% of the focus on the two characters you are talked about, and only say 10% on the others.
I'm sorry, I am not familiar with your text. However here is an example i just made up to give you an idea. This is about The Great Gatsby

Furthermore, Fitzgerald here again combines the generic conventions of characterization with point of view to convey Nick’s moral judgement of tort law in regards to the negligent behaviour of the upper class; who seem to only give shallow thought towards any idea that does not have a direct influence upon them. As Nick ‘shake(s) hands’ he condemns Tom’s brutish indifference, which pertained a likeness to negligence. Thus, Tom’s calamitous recklessness is analogised to that which would be felt when ‘talking to a child’. Nick concludes that it would be ‘silly not to’ shake hands with him, portraying Tom as decrepitude, exactly like ‘a child’ would be. Thus conveying the minimal attention society, during the 1920s, placed upon anything unrelated to materialism and wealth.

Analysing this, you can kind of see that Tom is the focus. I don't ever discuss Nick's behaviour or the 'hows' and 'why' relating to him. However, I do use him and his point of view as an aid to convey Tom's characterisation, and how he being wealthy makes him be portrayed in the eyes of the narrator :)
I hope this clarified something for you. If not feel free to inquire  :)
Title: Re: VCE English Question Thread
Post by: blacksanta62 on July 24, 2016, 11:47:09 am
Hey guys, I'm writing a context (I&B) expository essay with the following prompt:
"Sometimes we need to accept change in order to grow"

I'm trying to give the essay more depth by finding ideas where it might not be true. I'm thinking that sometimes "people have the ability to develop and grow without the need for change" i.e. change can be avoided and still result in growth. However, I can't think of any examples where this is true. Can anyone lend some examples from novels you've read or examples that you can think of? Thank you
Title: Re: VCE English Question Thread
Post by: Callum@1373 on July 25, 2016, 09:19:59 am
Question, I only received a 19/20 for my oral yet that contradicts my feedback:

"Callum, you carefully put together an excellent speech, incorporating all the features of oral language that were required in a seamless way. Your use of hand gestures was subtle but appropriate, your anecdotes and rhetoric were not overdone and your facial expressions showed your interest without over-acting. It was admirable that you didn’t need to rely too heavily on your cue cards. The content of your speech was well researched and strongly structured. Overall, your audience was thoroughly won over to your point of view. If it were up to me, I would be funding science immediately!"

Is this worth asking for a remark or is there something I am missing?
Title: Re: VCE English Question Thread
Post by: kimmytaaa on July 25, 2016, 12:27:36 pm
Hi
I am doing Every Man in this village is a liar, just wanted to know if what my teacher wrote is correct. The prompt is 'Complete survival is rare in times of conflict'
1.   Physical survival
2.   Emotional/social survival
3.   Benefits

Stack’s experiences in the plethora of actions across the Middle East highlight her exposure to gross cordnits? (I have no idea what this word was?) of physical violence.. While every man in this village is a liar offers the uncontrollable difficulties that herald in war and violence that ensues, more recent fears that have spread to Europe further this position on violence and its in-discriminatory efforts.
Extending the expression of physical unrest that plagues EMITVIAL, emotional and social scars further exemplify how absolute resolution is rare in times of conflict.
Localised in a more Australian context, the wide-reaching social and emotional harrowing consequences of domestic violence compounds the examples offered in EMITVIAL.

Thanks
Title: Re: VCE English Question Thread
Post by: Alter on July 25, 2016, 02:37:00 pm
Question, I only received a 19/20 for my oral yet that contradicts my feedback:

"Callum, you carefully put together an excellent speech, incorporating all the features of oral language that were required in a seamless way. Your use of hand gestures was subtle but appropriate, your anecdotes and rhetoric were not overdone and your facial expressions showed your interest without over-acting. It was admirable that you didn’t need to rely too heavily on your cue cards. The content of your speech was well researched and strongly structured. Overall, your audience was thoroughly won over to your point of view. If it were up to me, I would be funding science immediately!"

Is this worth asking for a remark or is there something I am missing?
A 19/20 is an amazing score. There doesn't need to be anything inherently wrong with your oral for you to not get a perfect score. In fact, basically any perfect score in a piece of English assessment should rarely be attainable, because it implies the work could not be improved in any way. I wouldn't take your result so harshly, because it really is a good score.

I can't make a fair judgement because I haven't actually heard what you presented. Maybe it was the best speech to ever go down in history, but your teacher probably just felt is was 'really good' as opposed to being flawless. Who knows? Maybe it will even be the highest score within your cohort. If it's really bugging you, ask your teacher what you could've done to move up to a 20, but don't enter the conversation with the expectation of moving your score up or arguing you did what she mentions in response, as it won't be healthy for your future work!

Just my 2 cents. Keep up the good work.
Title: Re: VCE English Question Thread
Post by: HasibA on July 28, 2016, 07:16:55 pm
it is reasonable to analyse the font choice in a language analysis, or should i steer clear of it? is there reall a hierarchy of language techniques ? for example, i hear rhetorical questions are far too commonly used, so its preferred to vary the techniques analysed.

what is highest mark you can get in a text response if you have very limited references/quotes to your text? more so, if you have like 1 quote per paragraph, but your essay rocks, can you still get a 10 or nah?

with context- if the assessors can't find the link between the prompt and your story, but you incorporate nice ideas/language choices etc., are you destined to receive a far lower mark than if the link was mroe clearer?

thank you- sorry if the questions ive asked have been unclear! :)
Title: Re: VCE English Question Thread
Post by: Swagadaktal on July 28, 2016, 07:25:45 pm
it is reasonable to analyse the font choice in a language analysis, or should i steer clear of it? is there reall a hierarchy of language techniques ? for example, i hear rhetorical questions are far too commonly used, so its preferred to vary the techniques analysed.

what is highest mark you can get in a text response if you have very limited references/quotes to your text? more so, if you have like 1 quote per paragraph, but your essay rocks, can you still get a 10 or nah?

with context- if the assessors can't find the link between the prompt and your story, but you incorporate nice ideas/language choices etc., are you destined to receive a far lower mark than if the link was mroe clearer?

thank you- sorry if the questions ive asked have been unclear! :)
Um you should probs wait for a big baws to answer this (HLS or Lauren) - but ima give you this piece of advice.

It has to flow naturally - if your piece calls for a rhetorical question then you put in that rhetorical question. If ur text response begs for an overused quote coz it fits so nicely and it proves ur point - then use it...
There is no quote limit but obviously u dont wanna under quote - you just need to put as many as needed. There's no number to give you. You just gotta see what works and what doesnt...

Soz this is pretty vague but this is the answer you need to understand imo

- if HLS, Lauren or any other big baws says otherwise then I'd probs follow their advice coz they're way better than i :P
Title: Re: VCE English Question Thread
Post by: Maz on July 28, 2016, 07:53:50 pm
it is reasonable to analyse the font choice in a language analysis, or should i steer clear of it? is there reall a hierarchy of language techniques ? for example, i hear rhetorical questions are far too commonly used, so its preferred to vary the techniques analysed.

what is highest mark you can get in a text response if you have very limited references/quotes to your text? more so, if you have like 1 quote per paragraph, but your essay rocks, can you still get a 10 or nah?

with context- if the assessors can't find the link between the prompt and your story, but you incorporate nice ideas/language choices etc., are you destined to receive a far lower mark than if the link was mroe clearer?

thank you- sorry if the questions ive asked have been unclear! :)

Hey
Hopefully, I can be of assistance...
I would honestly steer clear from font choice...I just feal as though it's a little risky. Especially since font choice varies across texts and according to who the publisher is and where it was published. You can 'mention' it briefly, but I wouldn't make this a big part of your essay/point.
Here is a little website I found that you might want to check out...it has a few different language techniques:
http://www.tickingmind.com.au/uploads/40084/ufiles/Revision/Language_Analysis_-_Study_Notes.pdf

I don't do English, I do Litt. However, in terms of evidence I would imagine they would be about similar. I really think that one quote per paragraph is underquoting. From personal experience, there was 5 possible marks to get for evidence, and I had 65 quotes in a 3000-word essay, but I only got 4/5. So I really would suggest to use more evidence. Remember, pretty much everything you say should be validated in the text, and you need to back yourself up- quotes are one of the best ways to do this!  So even is your essay 'rocks', it would 'rock' more with evidence. It's probably better to over-quote rather than underquote (overquoting is still a bad idea, but better)

The essay is about context. That is the key syllabus point. Kind of like with chem and physics, if you don't answer the question, you won't get the marks for it. In English you are probably going to only reach around 70% max I would say. So remember- ANSWER THE QUESTION. You really need to spend time making sure the link is clear. This happened to one of my friends, he wrote the essay brilliantly, but couldn't get the core idea across and only managed a 72%. So yeah, I think you would attain a lower mark, than say if you managed to convey the point more adroitly. Which makes sense, I mean you won't get marks for a question, if you don't answer it.

I hope this helped!  :)
Title: Re: VCE English Question Thread
Post by: cooldude123 on July 28, 2016, 09:45:40 pm
with context- if the assessors can't find the link between the prompt and your story, but you incorporate nice ideas/language choices etc., are you destined to receive a far lower mark than if the link was mroe clearer?

One of the main criteria for a high mark (9-10 mark response) is
Quote
Demonstrates an insightful grasp of the implications of the prompt/stimulus material, and perceptively explores its conceptual complexity using an appropriate strategy for dealing with it
while a criteria for a lower mark response is
Quote
Demonstrates a limited awareness of the prompt/stimulus material
so it's essential that you address the prompt- running the risk of appearing like you've just memorised a piece and copied it out in the exam is very high if you don't explore the prompt in sufficient detail, and this is very frowned upon by assessors. There is also an onus on you to make your prompt links clear - especially since there's no statement of intent like in SACs.

For reference VCAA examination reports:
"Weaker responses, on the other hand, also seemed to have pre-prepared introductions, which were obvious as they were
often generic discussions of the Contexts... followed by a passing recognition of the prompt" - VCAA 2008
"...students must respond to the ideas in the prompt. They cannot write a prepared response that may have worked in class...ideas
related to the prompt must be the focus of the writing." - VCAA 2012
"...students must make the connections between text and prompt clear."- VCAA 2014




Title: Re: VCE English Question Thread
Post by: HopefulLawStudent on July 29, 2016, 05:17:53 pm
it is reasonable to analyse the font choice in a language analysis, or should i steer clear of it? is there reall a hierarchy of language techniques ? for example, i hear rhetorical questions are far too commonly used, so its preferred to vary the techniques analysed.

what is highest mark you can get in a text response if you have very limited references/quotes to your text? more so, if you have like 1 quote per paragraph, but your essay rocks, can you still get a 10 or nah?

with context- if the assessors can't find the link between the prompt and your story, but you incorporate nice ideas/language choices etc., are you destined to receive a far lower mark than if the link was mroe clearer?

thank you- sorry if the questions ive asked have been unclear! :)

Agree with what Swag has said but I'll just extend on it.

Maybe steer clear of font choices. I know everything is technically free game but analysing fonts could be a bit contentious. There's so much other more interesting aspects of the text that you could be analysing imo. I've read some atrocious font-analysis stuff and some hella awesome font-analysis but honestly I wouldn't risk it.

Just because a technique is commonly used doesn't mean that you shouldn't analyse it.

Wouldn't be an insta-fail as your essay is marked holistically but use of quotes demonstrates textual knowledge, which is one of the key criteria, you feel?
Title: Re: VCE English Question Thread
Post by: HasibA on July 29, 2016, 09:59:41 pm
thanks for your replies guys! :)
Title: Re: VCE English Question Thread
Post by: FallingStar on August 01, 2016, 08:46:51 pm
Hi everyone,
I think this may be a stupid question, and I am unsure if it had been asked before.
In text response, what is the different between the "discuss" prompts and the "Do you agree?" prompts.

I'm just really confused here, and the teachers have been giving conflicting answers.

Thanks.
Title: Re: VCE English Question Thread
Post by: oooo on August 01, 2016, 10:34:07 pm
Hi everyone,
I think this may be a stupid question, and I am unsure if it had been asked before.
In text response, what is the different between the "discuss" prompts and the "Do you agree?" prompts.

I'm just really confused here, and the teachers have been giving conflicting answers.

Thanks.

'Do you agree?' prompts are usually just simply looking for a yes/no answer* and an expansion on your reasons. Discuss questions however aren't looking for anything in particular, so you have free reign over what you want to talk about.

* This shouldn't just be your contention though
Title: Re: VCE English Question Thread
Post by: Alter on August 01, 2016, 11:17:25 pm
I agree with the above, more or less. The difference between 'Discuss' and 'Do you agree?' prompts is pretty minute.

I just skimmed through the prompts in the 2015 exam and mentally replaced each 'Discuss' with 'To what extent/do you agree?' and vice versa. Honestly, I think that they can almost be used interchangeably, and you shouldn't drastically change your arguments based on which one shows up. Just my 2c.
Title: Re: VCE English Question Thread
Post by: Maawa OSMAN on August 02, 2016, 11:56:36 am
Im really lost for english!!!  :-X Does anyone know how i can get a contention from the prompt : "the heart of conflict is fear"???
Title: Re: VCE English Question Thread
Post by: literally lauren on August 03, 2016, 11:42:50 am
Here's the poem Lauren :)
I totally forgot to come back to this after you posted it, so here's a stupidly detailed poetic dissection by way of apology!
Spoiler
All night in hospital I law
with my intemperate lover, odd word choice to describe a person - lover connotes emotional closeness and affection, and yet 'intemperate' is a mild slight, so right from the outset we get the sense that this is a somewhat fraught relationship pain.
A sister
= enjambment - continuing a train of thought or a phrase across lines of poetry. So there's an association between 'pain' and 'a sister' here, but it's an estranged relationship as displayed through the enjambment, calm in her serene
indifferent charity
love that turn of phrase; that stacking of adjectives: 'serene + indifferent' is curious in that it implies she's awe-worthy, calm, and at peace (='serene') but also detached, impassive, and pococurante (=indifferent). It's kind of like the difference between 'care-free' - which has positive connotations - and 'careless' - which has negative ones. You could focus on either (or both) here, depending on your interpretation, sat to pray
for me, as for all suffering men getting some clarification as to who the speaker might be,
beside my bed. Darkness and fear =caesura - a break within a line of poetry. So here, the 'darkness and fear' is threateningly close to 'my bed,' but hasn't yet engulfed it; there's still some separation, but its presence is nevertheless felt.
rose up between me and her prayer interesting implied physicality of 'her prayer'... and of 'darkness and fear' too for that matter. It's as though the speaker is attempting to be close to 'her prayer' but the 'darkness and fear' is intercepting, or acting as an obstacle that neither the speaker, nor the prayer can overcome.

eclipsing the rich world of love new stanza for 'the rich world of love,' but there's some enjambment here since the 'darkness and fear' is 'eclipsing' the positive things. So you could look at the symbolism of light and dark here
in which I grew, and richer trust
of lives that with my life were nourished
this part... I'm not sure about. My guess is that the speaker is acknowledging that she wasn't alone in this 'rich world,' and that there was a kind of 'trust(fund)' of other happy lovely lives/people who 'nourished' one another. Hence, there's a key thematic juxtaposition from the loneliness in the first stanza to this lamentation of halcyon days here.
And still my violator thrust
hard, hard,
O.O into a throbbing wound much more visceral imagery going on here all of a sudden; the cheerful reminiscing is punctured by these violent, aggressive verbs and adverbs.
Fast in that grip I felt all cherished
images
again, the enjambment separates 'cherished' from 'images,' suggesting that the memories have become removed from the happy emotions once associated with them of good dissolve great word to unpack the connotations of - dissolution = melting, deliquescence, gradually fading away.

A cock crowed, and the sister rose, temporal and tonal shift back to where the poem began
yawning, to raise the window blind--
a row of cypress, scalpel-clean,
'scalpel-clean' trees 'incised' into the landscape - also rather clinical, unnatural imagery
crowning a near-by hill, incised
the first of morning. Clear between
each cypress and its neighbour glowed
a winecup space of light defined

by the dark trees
picking up on the light/dark metaphor from earlier, the darkness of the trees is interpolated with the defined, 'glow[ing]' light between them. The 'winecup space' bit confuses me a little, Maybe I'm being too literal, but this is what I'm imagining: (why yes, I am an artist, thank you for noticing). Those winecups brimmed
with a new day's untasted wine:
I drank
translation: the speaker delights in optimism - she drinks the wine of the light between the trees rather than losing her psyche in the dark shadows cast by them, until my spirit leapt
high on the morning's tranquil crest
and drunken sang its hope so focusing on the sunny-side of things has this intoxicating effect; for better or for worse, it's certainly changing her state of mind, and blessed
all suffering and rejoicing men contrast to just 'suffering men' earlier - now it's more inclusive, and in her bliss, she feels like her spirit blesses others whether they're happy or not.
I slipped from pain's embrace, and slept. nice conclusory final line here that implies some relief from this emotional rollercoaster. But notice how she 'slip[s ] from pain's embrace' and THEN 'slept?' So going to sleep isn't some cop-out or means of evading her emotional troubles - instead, she resolves herself to centring on the good in life before she's able to attain a sense of serenity sufficient for sleep. Sleep is her reward, but it's not the solution - psychological rationalisation and altering one's viewpoint is the solution.


tl;dr of this poem.
Note that this is by no means a conclusive reading; I've tried to isolate the bits of this that substantiate the interpretation I've taken, but there are probably a bunch of other ways you could extrapolate a feminist interpretation or something completely different. Plus, I'm approaching this as an outsider who's only moderately familiar with Harwood's poems (and I'd never read this one until you posted it) so it's likely there are some thematic ties I haven't picked up on. But I hope this helps you make sense of it - let me know if you have any questions :)

Hey guys, I'm writing a context (I&B) expository essay with the following prompt:
"Sometimes we need to accept change in order to grow"

I'm trying to give the essay more depth by finding ideas where it might not be true. I'm thinking that sometimes "people have the ability to develop and grow without the need for change" i.e. change can be avoided and still result in growth. However, I can't think of any examples where this is true. Can anyone lend some examples from novels you've read or examples that you can think of? Thank you
Consider the fact that sometimes resisting change can provide us with a sense of self - like, if everyone I know is playing Pokemon and I'm not, I might define myself as existing outside of that phenomenon and become one of those old cynics saying 'this is such a dumb game omg.' Plus, imagine how difficult it would be to accept all kinds and instances of change. We can't let everything shape us, lest we become too malleable and lack any core identity whatsoever. By way of examples, you could look at how throughout history, there have been groups that have been opposed to the tides of change and have refused to 'grow' as a result. (e.g.)

There's also the line of argument you seemed to be taking, which was that we don't always have to accept change in order to grow. In that case, I think you'd want to look at how we can fight against change, or even create our own change in order to forge a sense of self (e.g. 'be the change you wish to see in the world' ~ if you don't like the way things are, then do something about it... rather than accepting a change you don't like and allowing it to shape you, change the change and grow the way you want to instead). That might then lead you to a discussion of famous civil rights leaders or activist groups who campaigned against the inequalities they observed, which will hopefully tie in neatly with your id&b text(s) :)

Hi
I am doing Every Man in this village is a liar, just wanted to know if what my teacher wrote is correct. The prompt is 'Complete survival is rare in times of conflict'
1.   Physical survival
2.   Emotional/social survival
3.   Benefits

Stack’s experiences in the plethora of actions across the Middle East highlight her exposure to gross cordnits? 'conflicts,' I'm guessing? (I have no idea what this word was?) of physical violence.. While every man in this village is a liar offers the uncontrollable difficulties that herald in war and violence that ensues, more recent fears that have spread to Europe further this position on violence and its in-discriminatory efforts. okay, this sentence makes no sense... is this word for word what your teacher wrote because I'm struggling to work out what's going on here.
Extending the expression of physical unrest that plagues EMITVIAL, emotional and social scars further exemplify how absolute resolution is rare in times of conflict. this is a little bit clearer, although saying 'emotional scars show that resolution is rare' is a bit of a stretch. Provided you could back this up in your body paragraph, you should be alright with this one though.
Localised in a more Australian context, the wide-reaching social and emotional harrowing consequences of domestic violence compounds the examples offered in EMITVIAL. this one's also a bit strange - 'domestic violence in Australia has an effect on the examples in EMITVIAL...???' Not sure where your teacher is going with this, but if this is what they want you to do for your SAC, then go for it.

it is reasonable to analyse the font choice in a language analysis, or should i steer clear of it? (1) is there really a hierarchy of language techniques ? (2) for example, i hear rhetorical questions are far too commonly used, so its preferred to vary the techniques analysed. (3)

what is highest mark you can get in a text response if you have very limited references/quotes to your text? more so, if you have like 1 quote per paragraph, but your essay rocks, can you still get a 10 or nah? (4)

with context- if the assessors can't find the link between the prompt and your story, but you incorporate nice ideas/language choices etc., are you destined to receive a far lower mark than if the link was mroe clearer? (5)
(1) As others have mentioned, probably best to steer away from this. You can analyse emboldened / italicised / underlined words or phrases, though they're rarely in exams. Also the exam material tends to use the same font the whole way through (usually just a generic times new roman or similar) so idk what you could realistically say about it unless you're like
In an attempt to burn the retinas of readers, the author employs comic sans as a means of conveying her sadism and disregard for basic human decency.

(2) I actually like the idea of 'hierarchy of techniques,' though I think of it more as a ranking of 'this is a really redundant point' to 'this is an excellent, well-substantiated point of analysis.' So if you're analysing alliteration in the title of a piece, you're probably down the lower end of the scale because saying the author repeats a certain sound isn't really aiding you in getting to the heart of the argument. Whereas, if you're unpacking some connotations, implications, or repeated phrases, there's a greater chance you'll stumble upon something really impressive. That said, mentioning the odd 'lower' technique isn't disastrous, and there are still decent things to say about commonplace things like rhetorical questions or inclusive language, but if you're able to push yourself beyond the basics, you'll likely reap the rewards mark-wise.

(3) It's absolutely necessary to vary your techniques, but that doesn't mean you can't comment on rhetorical techniques et. al. Rather, if you do find a rhetorical technique, comment on it, analyse it effectively, and then move on to something else. So long as you're not picking up on three or four different rhetorical questions and discussing them in the exact same way, there shouldn't be any problem.

(4) You couldn't get a 10 if you only had a single quote in each paragraph. I doubt you'd break the mid-range bandwidth (5-7). It is possible to conduct decent analysis without using direct quotes (esp. for visual novels and films) but the assessors are still on the lookout for your discussion of language. Plus, you should see quotes as helpful things rather than hindrances - they're the buttresses of your essay - your contention won't be able to stand up without them!

(5) Links to the set text is a core part of the Context marking scheme, so if there's nothing there for the assessors to find, they'll get snarky. I remember hearing that it's impossible to score about a 6/10 with no text link whatsoever - I don't know if that's true, but there are definitely some assessors who believe it, so make of that what you will. You could still be alright if you didn't have an overt link, i.e. if you were referencing the ideas of the text clearly, but didn't use the words 'In text title the main character experiences...' HOWEVER, this is still quite risky. I'm an advocate for at least one explicit connection just to be on the safe side. For your SAC, stick with whatever your teacher says, but for the exam, it's best to err on the side of caution.

Hi everyone,
I think this may be a stupid question, and I am unsure if it had been asked before.
In text response, what is the different between the "discuss" prompts and the "Do you agree?" prompts.

I'm just really confused here, and the teachers have been giving conflicting answers.
As oooo and Alter have noted, there's no difference :P Arguably you could say that 'do you agree' tend to be a little more close-ended, whereas 'discuss' gives you more of an opportunity to take the core of the prompt in other directions, but the difference is so arbitrary that it shouldn't feed into your planning/contention at all, really.

Im really lost for english!!!  :-X Does anyone know how i can get a contention from the prompt : "the heart of conflict is fear"???
Steps for getting a contention:
1. Understand the prompt; put it into your own words like 'wtf is this actually asking me?' --> e.g. 'fear causes conflict.'
2. Ask 'why might this be true?' --> e.g. 'fear can cause us to react impulsively or instinctively without thought for potential consequences; so it's easier to cause conflict when we're in a state of panic or uncertainty.'
3. Ask 'why might this not be completely true?' --> e.g. if we can overcome this fear, it can actually help us to resolve conflict, so it's not as simple as saying fear always leads to conflict.
4. Put the above two observations into an 'Although (3), ultimately (2)' style sentence --> e.g. 'Although fear can bring about positive changes in people and circumstances, often emotions stemming from panic or uncertainty makes it easier for conflict to spark and flourish.'

Then in your essay, you just reinforce that 'Although...ultimately...' sentence as your contention, mainly focusing on the latter part, but also occasionally acknowledging the former (e.g. once per paragraph, you might say something like 'although, in this example, there were other factors that also contributed to the conflict, it was ____'s fear of ___ which had the most impact...' just as a bit of a challenge.)

Make sense? Let me know if you have any questions :)
Title: Re: VCE English Question Thread
Post by: studybuddy7777 on August 03, 2016, 05:50:35 pm
Also the exam material tends to use the same font the whole way through (usually just a generic times new roman or similar) so idk what you could realistically say about it unless you're like
In an attempt to burn the retinas of readers, the author employs comic sans as a means of conveying her sadism and disregard for basic human decency.

(Off-topic)
I dont go on VCE english threads much simply because im doing the HSC but this has been the best thing ive heard all day!! So true but, especially in cities/on advertisements where they are trying to attract their attention. Literally laughed out loud when I read this!
Title: Re: VCE English Question Thread
Post by: kimmytaaa on August 03, 2016, 05:56:29 pm
snip
Yeah I sometimes don't get what my teacher means :(
How would I go about answering this prompt 'the consequences of conflict will always be resonate'?
Title: Re: VCE English Question Thread
Post by: literally lauren on August 03, 2016, 06:46:25 pm
Yeah I sometimes don't get what my teacher means :(
How would I go about answering this prompt 'the consequences of conflict will always be resonate'?

This prompt isn't really worded right... is it meant to be 'the consequences of conflict will always resonate' or 'the consequences of conflict will always be resolved?' Or something else entirely? Sorry, I can't be of much help until I know what the question is, but if you go back to some of the past advice in this thread and others, you should be able to apply the same process when it comes to breaking apart a prompt and forming a contention :)
Title: Re: VCE English Question Thread
Post by: YellowTongue on August 03, 2016, 08:41:28 pm
In an expository essay for context, how often do I need to refer to the text? Is it enough to draw one example from the text and the remainder from the "real world"?
Title: Re: VCE English Question Thread
Post by: FallingStar on August 03, 2016, 09:22:24 pm
In an expository essay for context, how often do I need to refer to the text? Is it enough to draw one example from the text and the remainder from the "real world"?

That really depends on the teacher and your own preferences. You will need to draw on it enough to make it clear that you understand the particular text in terms of the ideas involved, but not too much as to make it sound text-responsy (which is when the examiners would really get annoyed at you).

As a general rule of thumb, you should have roughly 2/3 real world or other textual examples and 1/3 form the text you studied. But as long as you make you references clear, and make it clear that you are understanding and drawing ideas form your text, you should be fine. (though Lauren can give a better answer to this)
Title: Re: VCE English Question Thread
Post by: Alter on August 03, 2016, 09:33:51 pm
In an expository essay I used to use this structure:

[introduction, without referring to any examples or the text. mostly just breaking down the prompt]
[body paragraph 1: exploring the prompt referring to my primary text (EMITVIAL)]
[body paragraph 2: what my primary text fails to explain re: the prompt, and a historical/social/political/whatever example of something that does]
[body paragraph 3, same as above but with a new example]
[conclusion]

Sometimes body paragraph 2 and 3 might have very minor references to the text for the sake of flow or if it was appropriate to do so, but I'd rarely dedicate much more than that. Obviously there are many different structures you can use, and the one I did is by no means 'the right one'. At the end of the day, it's more important to address the key areas of the prompt than to show off your knowledge of the text.
Title: Re: VCE English Question Thread
Post by: YellowTongue on August 04, 2016, 03:15:28 pm
I had my context SAC this afternoon, and I used the word "confragrative" in my piece. I now realise that what I meant to use was "confragrant", as I'm not entirely sure if "confragrative" is actually a word. How screwed am I???  :'(
Title: Re: VCE English Question Thread
Post by: studybuddy7777 on August 04, 2016, 04:08:44 pm
I had my context SAC this afternoon, and I used the word "confragrative" in my piece. I now realise that what I meant to use was "confragrant", as I'm not entirely sure if "confragrative" is actually a word. How screwed am I???  :'(

I'm sure that theyll let one word slip as long as the sentence still makes sense. I put "relevancies" in my exam and im pretty sure that isnt a word :P
So dont stress about it, it'll probably get circled but no marks deducted :)
Title: Re: VCE English Question Thread
Post by: HopefulLawStudent on August 05, 2016, 01:54:20 pm
You can always tell when a SAC is coming up with me... I may or may not have started reading the dictionary again... :P

Quote
He had hoped beyond hope that the world he had known growing up was an accurate microcosm of what the world beyond his house was like.

Have I used "microcosm" correctly?
Title: Re: VCE English Question Thread
Post by: oooo on August 05, 2016, 02:50:57 pm
Conclusions and linking sentences; how do I write them? :( For conclusions, I've been taught to 'go backwards'; starting with my 3rd argument, 2nd argument, 1st argument, contention then the "big idea". I don't get this method, can someone please explain it to me (with examples please if not too much of a hassle), or if possible, a better approach. Also, with the linking sentences, do I link it more directly to the prompt or my contention? 
Title: Re: VCE English Question Thread
Post by: Maz on August 05, 2016, 06:48:47 pm
Conclusions and linking sentences; how do I write them? :( For conclusions, I've been taught to 'go backwards'; starting with my 3rd argument, 2nd argument, 1st argument, contention then the "big idea". I don't get this method, can someone please explain it to me (with examples please if not too much of a hassle), or if possible, a better approach. Also, with the linking sentences, do I link it more directly to the prompt or my contention? 
Hey,
Hopefully, I can help a bit
You can go backwards or forwards, I would just recommend that it is in an order. I generally go forwards, like I re-state first body paragraph, then second, then third, fourth etc. Conclusions are the opposite of introductions; so in an introduction, you would start broad then narrow in on what you are going to be talking about, in the conclusion start narrow (this is where you re-hash your essay) then build up to the broad part towards the ending.
Here is one of my essays about generic conventions and The Great Gatsby

The Intro:
Set during the post World War 1 economic boom of the 1920’s, F. Scott Fitzgerald’s novel, The Great Gatsby (1926), is often referred to as a chronicle of the American Dream, exploring a point of the nation’s history when capitalism and economic opportunity for all was at it’s peak. The Great American Dream incorporated the belief that anyone, regardless of where they were born, could obtain prosperity and success. Deep-rooted in the Declaration of Independence, the American Dream was the proclamation that ‘all men are equal’, with full right to ‘life, liberty and the pursuit of happiness.’ Perhaps one the best works of literature, representing the American Dream is The Great Gatsby. Throughout the novel, Fitzgerald contextualizes a gripping antithesis regarding the Great American Dream, through a contrast and disclosure of societies key value systems, including the emphasis upon materialism and commodification. The Great Gatsby juxtaposes characters born in the upper echelons of society, with Myrtle and Gatsby, who attempt to transcend the class boundaries but ultimately become ‘incoherent failures’. The novel displays ‘how dreaming can be tainted by reality, and that if you don’t compromise, you may suffer’(Azar Nifisi). Fitzgerald’s text exposes the key value systems prevalent within society through an adaption of generic conventions, reflecting the misery of a modern capitalist society.

The essay question was Examine the ways in which writers shape and adapt generic conventions to reflect and expose particular value systems. In your response, you must make reference to at least one literary text. So I start out just talking about the novel, where it was set, and just establish the context of the value system I'm going to talk about...It is only at the very ending sentence that I introduce the idea of generic conventions. So start out broad, give the reader some background. Always remember that your essay and more importantly the into should be structured in a way that someone who has never read the book can understand it...so your intro needs a lot of context...
then the conclusion for it was:
Fitzgerald exposes, through an accentuation of the generic conventions of characterisation, language, setting and dialogue; that American society within the 20th century was manifested with the value system of commodification, driven by the want for a materialistic lifestyle. Fitzgerald highlights this through a ridicule of the values of the American Dream and the resultant effects upon those who followed its incorporated concepts. Both Myrtle and Gatsby, the two characters’ striving for the attainment of the American dream suffered the severe consequence of death. In the end, Gatsby never attains ‘the green light, and the orgastic future’, despite believing in it even when it ‘eluded’ and ‘receded before [him]’. Thus, through the medium of The Great Gatsby, Fitzgerald reflects upon the American dream as unattainable and an empty promise of happiness. Despite this, society within and beyond the 20th century, places great emphasis upon materialistic wealth and commodification, a value system that is to this day, timeless. ‘That is part of the beauty of all literature. You discover that your longings are universal longings, that you are not lonely and isolated from anyone. You belong.’ (Francis Scott Key Fitzgerald).   

With this, unlike the intro, I went straight into what was being discussed in the essay and addressed the question. I really think the topic sentence of your introduction needs to address the question and answer it. So I re-stated all of the generic conventions that I discussed. Then I restate my thesis statement/essay question...and gradually build up to a really broad idea...going from talking about one text and at the end getting a quote upon literary texts as a whole from the author :)
That is kind of what they mean by go backwards or forwards, the first generic convention I mention in my conclusion was characterisation, which was my first body paragraph. Then the second I mention was language, which was my second body paragraph...and so on and so forth...

With linking sentences, I would say both...Your contention is pretty much the prompt isn't in..just a disagreement. Since you are arguing your contention, I would focus more on that. So really the last thing you should be leaving your audience with is what you are arguing :) So the thesis statement :)

I hope this helped :)
If not, just let me know and I'll expand some more
Title: Re: VCE English Question Thread
Post by: clarke54321 on August 06, 2016, 10:03:39 pm
Hi everyone,

For the following essay prompt:

'What does 12 Angry Men and The Crucible suggest about the connection between fear and prejudice?'

How do I create a contention out of this? The texts explore this idea in multiple ways, therefore I am unsure about how I am supposed to construct my argument.

Any advice would be greatly appreciated!  :)
Title: Re: VCE English Question Thread
Post by: oooo on August 06, 2016, 10:22:20 pm
In text response essays, can I just plonk in a quote and not analyse it?
Title: Re: VCE English Question Thread
Post by: Maz on August 06, 2016, 11:31:51 pm
In text response essays, can I just plonk in a quote and not analyse it?
I would advise against that, like you can use it for evidence, but it doesn't really work if you don't explain it :)
Title: Re: VCE English Question Thread
Post by: FallingStar on August 07, 2016, 07:59:21 am
In text response essays, can I just plonk in a quote and not analyse it?

Don't. There are very specific cases where this may be allowed (such as the conclusion written by mq123) but for the most part, this is not a good idea as it says to the assessors that you are just putting in example and can't analyse them in terms of the characterisation and themes of your text.

Instead, format it as quote/evidence, then followed by your analysis, which would show the assessors that you can use evidence to substantiate your contention or main idea of your text response essay.

Hope that helps.
Title: Re: VCE English Question Thread
Post by: oooo on August 09, 2016, 06:26:24 pm
Another question (sorry :p): my teacher keeps on saying that my topic sentences sound more like headings/contentions than actual topic sentences. How can I fix this?
Title: Re: VCE English Question Thread
Post by: FallingStar on August 09, 2016, 08:09:50 pm
Another question (sorry :p): my teacher keeps on saying that my topic sentences sound more like headings/contentions than actual topic sentences. How can I fix this?

First of all, determine what your contention is. Your contention is generally about whether you agree with the topic or not. Do not talk about any sub-arguments here in your contention.

Now that you've determined whether you agree with the topic or not, now, you need to say outline each of your 3 - 4 reasons into one sentence each. These are your actual topic sentences.

You contention sentence is you broader argument. That is, whether you agree with the topic for not. State what you think of the topic bluntly. Make sure you adhere to the prompt (or else lose marks).

However, can you please post your specific sentences which your teacher has commented about? It would really make it easier for us to help you, as I am finding it quite difficult to give a specific answer to your question. That being said, just take my words above as I had done my best to answer your question, which I hope helps you.
Title: Re: VCE English Question Thread
Post by: oooo on August 09, 2016, 08:19:08 pm
First of all, determine what your contention is. Your contention is generally about whether you agree with the topic or not. Do not talk about any sub-arguments here in your contention.

Now that you've determined whether you agree with the topic or not, now, you need to say outline each of your 3 - 4 reasons into one sentence each. These are your actual topic sentences.

You contention sentence is you broader argument. That is, whether you agree with the topic for not. State what you think of the topic bluntly. Make sure you adhere to the prompt (or else lose marks).

However, can you please post your specific sentences which your teacher has commented about? It would really make it easier for us to help you, as I am finding it quite difficult to give a specific answer to your question. That being said, just take my words above as I had done my best to answer your question, which I hope helps you.

Here's an example: Malouf presents stories with an ability to transcend both time and space. I think my teacher has a problem with how I express it, it seems a bit blunt and like a single contention on its own. But then again I'm not too sure..
Title: Re: VCE English Question Thread
Post by: FallingStar on August 09, 2016, 09:05:45 pm
Here's an example: Malouf presents stories with an ability to transcend both time and space. I think my teacher has a problem with how I express it, it seems a bit blunt and like a single contention on its own. But then again I'm not too sure..

Actually, I do agree with your teacher. That sentence of "Malouf presents stories with an ability to transcends both time and space." is too broad, such that it can be a contention on it own. Instead of that, you may use a method (in summary, the detail should be in the subsequence sentences, not your topic sentences). 

I suggest you use:
"Malouf presents stories with an ability to transcend both time and space by _______." Or "By ________ Malouf presents stories with an ability to transcend both time and space."

All you have to do is to add in the method in which this is done. However, I am aware there are other ways, in which this can be done, but I am personally not familiar with them. So I think it's better to leave to others here. You just have to narrow down, and be more specific in this particular sentence.
Title: Re: VCE English Question Thread
Post by: blacksanta62 on August 15, 2016, 09:40:57 pm
Yo guys, I have a 3 hour practice English exam tomorrow and I've forgotten how to deconstruct prompts for a text response. It was sprung on us last week on Thurs so there hasn't been much time for practice.

Could anyone refresh my memory (it's been a good 6 months) on how I should attack the prompt and what the point of a text response is. I know it sounds like a joke but this is a serious question.

Thank you :)
Title: Re: VCE English Question Thread
Post by: FallingStar on August 16, 2016, 11:00:52 am
Yo guys, I have a 3 hour practice English exam tomorrow and I've forgotten how to deconstruct prompts for a text response. It was sprung on us last week on Thurs so there hasn't been much time for practice.

Could anyone refresh my memory (it's been a good 6 months) on how I should attack the prompt and what the point of a text response is. I know it sounds like a joke but this is a serious question.

Thank you :)

To answer your question, have you downloaded Lauren's Lecture slides? The most helpful pages in terms of answering prompts are from pages 4 - 12 of these notes. If you can't get these notes then:

1. Make sure you understand the prompt. Perhaps reword the prompt so that you do understand it.
2. Pick out the key words of the prompt. The prompt "This text is about Agnes being restored to humanity. Discuss" has the key words of Humanity and the concept of being restored.
3. Define your Key words. Ask "what do you mean by humanity?"
4. Ask questions. Take the same prompt I took about Burial Rites. You could ask "How is Agnes being restored to humanity?, and "What does the author say about humanity?"
5. Construct a stance. Do not either fully agree or fully disagree with the prompt. And do not sit on the fence. Just construct a mostly agree or a mostly disagree stance.

A bit long, but I hope that helps. And the lecture slides should help too.  ;)
Title: Re: VCE English Question Thread
Post by: blacksanta62 on August 16, 2016, 05:26:28 pm
Thank you
I'll have a look at the lecture slides
More questions:
1) I done the practice essay today and I didn't finish all my sections. What's weird is that I started with section c and didn't manage to finish it in an hour, and I didn't want to go over. I was trying to analyse the whole piece but couldn't finish in time. Are we expected to analyse the whole piece or just aspects we think are important espically the accompanying image. What parts of a piece/how much should I analyse (please, I'm not expecting the perfect response, just a rough guide and then I can practice and find what works for me :)) I finished section a and b however. My question is if I start doing practice exams (3 hour sessions, but not too much that it kills me) can I increase my speed? I really want my exams to be of good quality but not take forever to write
2) What are the features of a well written essay in the exam? General stuff will do because I know they're not expecting perfection with the sentence structure etc.
3) In LA, when we analyse the authors use of language, will everyone have a different effect on the reader? Nothing too extreme though. Also, will people have slightly different tones?

Thank you
Title: Re: VCE English Question Thread
Post by: FallingStar on August 17, 2016, 01:14:16 pm
Thank you
I'll have a look at the lecture slides
More questions:
1) I done the practice essay today and I didn't finish all my sections. What's weird is that I started with section c and didn't manage to finish it in an hour, and I didn't want to go over. I was trying to analyse the whole piece but couldn't finish in time. Are we expected to analyse the whole piece or just aspects we think are important espically the accompanying image. What parts of a piece/how much should I analyse (please, I'm not expecting the perfect response, just a rough guide and then I can practice and find what works for me :)) I finished section a and b however. My question is if I start doing practice exams (3 hour sessions, but not too much that it kills me) can I increase my speed? I really want my exams to be of good quality but not take forever to write.
2) What are the features of a well written essay in the exam? General stuff will do because I know they're not expecting perfection with the sentence structure etc.
3) In LA, when we analyse the authors use of language, will everyone have a different effect on the reader? Nothing too extreme though. Also, will people have slightly different tones?

Thank you

1) This is a good question, but some parts of the this questions has touched on my weakness in terms of doing English essays (timed writing), which I actually cannot answer as of yet. In terms of analysing the LA pieces, I don't think you are expected to analyse every single technique you can spot. You just pick the ones you think are important, or ones you can write the most on in terms of convincing the audience of the contention.
2) Check the examiner's criteria. Also see pages 35, 36 and 37 of the lecture slides which I had linked in my above post. A well written essay is one which actually adheres to these criteria, and you must make it obvious that you are adhering to these criteria.
3) Can you please clarify this question? I don't know what you mean.
Title: Re: VCE English Question Thread
Post by: blacksanta62 on August 17, 2016, 05:56:33 pm
1) This is a good question, but some parts of the this questions has touched on my weakness in terms of doing English essays (timed writing), which I actually cannot answer as of yet. In terms of analysing the LA pieces, I don't think you are expected to analyse every single technique you can spot. You just pick the ones you think are important, or ones you can write the most on in terms of convincing the audience of the contention.
That makes things so much more bearable :)
2) Check the examiner's criteria. Also see pages 35, 36 and 37 of the lecture slides which I had linked in my above post. A well written essay is one which actually adheres to these criteria, and you must make it obvious that you are adhering to these criteria.
Would you be able to link me to the criteria? Or is that just another way of referring to the examiners reports? Just need clarification
3) Can you please clarify this question? I don't know what you mean.
Of course,so when assessors are marking section c, will they see a variety of ideas on the tones employed by the author and slightly different explanations of the intended impact on the audience. My reasoning is that we will all take the article in a slightly different way but the contention will still be the same (obvs!). Hope my question is clearer  :)
 

And anyone who can comment on this: "if I start doing practice exams (3 hour sessions, but not too much that it kills me) can I increase my speed? I really want my exams to be of good quality but not take forever to write"

Thanks Star, abs fire!!
Title: Re: VCE English Question Thread
Post by: FallingStar on August 17, 2016, 06:59:35 pm
Examination criteria from 2009. Still very relevant as the study design haven't changed.
Please note:
Quote
Assessors mark holistically, relating student performance to the published criteria and ranking students over the full range of marks available. Determination of the mark is assisted by descriptors of Expected Qualities for the Mark Range; these have been written to reflect the level of achievement expected at a particular mark or mark range.

This means that a weakness in one area may pull down your whole writing. Conversely, a strength in one area would pull up your whole writing. And visa versa. Just remember that although I did say that you do not have to analyse every single technique, you still have to analyse the techniques you have chosen well, in the "what?, How? Why?" format.

About your 3rd question, you have to get the contention right or else you get you whole LA wrong. But in terms of the different takes in technique, the examiners will use discretion here. (Don't know if I'm right, Lauren) As long as your explanation is reasonable and the examiners can see where you are coming from, then you should be fine. What they're looking at is if you can understand and analyse the effects of language upon the audience, and if you show that you can do that in the exam, then you should be fine.
Title: Re: VCE English Question Thread
Post by: literally lauren on August 19, 2016, 10:22:26 am
1) I done the practice essay today and I didn't finish all my sections. What's weird is that I started with section c and didn't manage to finish it in an hour, and I didn't want to go over. I was trying to analyse the whole piece but couldn't finish in time. Are we expected to analyse the whole piece or just aspects we think are important espically the accompanying image. What parts of a piece/how much should I analyse (please, I'm not expecting the perfect response, just a rough guide and then I can practice and find what works for me :)) I finished section a and b however.
Okay, there are a whole bunch of sub-concerns here, so I'll try and break this down.

For starters, not finishing essays at this stage of the year is totally fine. You wouldn't expect to be able to sit a Maths or Science end-of-year exam in August and score as highly as possible, right? Of course not - you wouldn't have covered much of the Unit 4 content, and you probably would have been focusing more so on SACs than exam revision. Assuming you came close to finishing the L.A. piece (like - you'd written at least two and a half decent body paragraphs by the one hour mark) then you should be fine. Timing is only a huge concern at this stage of the year if it takes you a whole hour just to plan, write an intro, and do one body paragraph  :-\

But you did the right thing moving on to the other sections - a lot of students screw up in the exam by spending way too long on a single section (- I nearly did this for Text Response, and a lot of people in my year level did so for L.A.) to the detriment of their other two pieces.Keeping yourself to relatively strict one hour cut-offs for each section is a pretty good plan as it's better to have three essays that are really high quality, but missing the end of a body para or a conclusion, as opposed to two complete, polished essays and one mangled half essay. The assessors marking your work know nothing of how well you did in other sections, so there's no chance of things 'balancing out' just because one essay was brilliant and the other one was barely an essay.

With regards to whether you need to cover everything in Language Analysis... short answer: no.
Long answer: no, because they're testing your ability to be selective, and they don't actually want students to comment on EVERY SINGLE TECHNIQUE they find because that would get really tedious. More to the point - that's not the focus of the task! Your job is to comment on how language is used to persuade readers. Pointing out seven different rhetorical questions, nine examples of inclusive language and six instances of appeals to fear doesn't help you do that. You might analyse some of those things, but you have to contextualise them by linking them to what the author intends for readers to think or feel. VCAA are way more concerned with your capacity to demonstrate an understanding of arguments than how many bits of 'emotive language' you can point out.

Assessors do not have a list of features you must mention - there's no one particular quote or technique that must be mentioned - it's all up to you to select what you believe to be most important in the construction of the author's argument.

The only exception to ^that rule is that you do have to analyse the visual material at least once. Rumour has it that you can't score about a 7/10 without doing this, and whilst I don't know how stringent the assessors are about this, you're at a definite disadvantage if you don't even acknowledge or pick apart any of the images. (& if you really want to be sure, link the visual to the written material! <--That's usually something only high-range essays are able to do, so forging those connections can help bump up your mark in the assessor's mind, albeit only slightly... the quality of the rest of your analysis plays a bigger role, obvs.)

Simply put:
- Student A, who can only find 5 language features to comment on out of the whole material, has to analyse all 5, and is probably going to have a fairly weak essay lacking in breadth.
- Student B, who can find 25 language features and feels the need to comment on all of them, is probably going to have an essay that's choppy and unfocused with too much breadth and not enough quality analysis.
- Student C, who can also find 25 features but selects the 17 BEST and MOST RELEVANT ones to include and dissect, is going to have a much more precise, much more impressive piece.

(note: 17 should not be your ultimate goal, that was a very arbitrary choice for the sake of this example. In fact, you shouldn't really know how many language features you're commenting upon; it should be second nature to you over the course of your analysis. If you can very quickly tell how many techniques you're dealing with in each paragraph or over the course of your essay, it probably means your piece isn't as well integrated as it could be.)

So be like Student C and ensure you're whittling down the material to find the most essential and analysable components! The best way to do this is to practice annotating and planning essays - go through some exam material and highlight absolutely everything you think you COULD analyse. Then, go back through and isolate the stuff you think you SHOULD analyse.

My question is if I start doing practice exams (3 hour sessions, but not too much that it kills me) can I increase my speed? I really want my exams to be of good quality but not take forever to write.
That's a very valid concern; you don't want to focus so much on timing that you end up getting your average down to 55 minutes (yaay!) but the quality has dropped down into the D+ range ( :-\) Instead, try to gradually decrease your time! Let's say it takes you 90 minutes to write a good Text Response piece right now. The next one you write has to be done in 80. Then 75. Then 70. Then 65. You get the idea.

You may run into a hurdle at some point with a particular Area of Study (e.g. I could never manage to write a Context piece in under 65 minutes) which means you'll then have to work on strengthening your other essays even more (e.g. get L.A. under an hour to give you extra time to work on Context, or vice versa).

But it's way better to go for this gradual approach that ensures you maintain the same level of quality than to go 'alright, one essay in one hour, no notes, exam conditions, cold turkey, let's go!'

2) What are the features of a well written essay in the exam? General stuff will do because I know they're not expecting perfection with the sentence structure etc.
This is a bit tough to answer outside of just saying that a well-written essay is one that fulfils the criteria for each section, as FallingStar has already noted. If you're talking about what level of imperfection they're willing to accept (as in, how many ungrammatical sentences, word choice errors, spelling mistakes, etc.) then the answer is: so long as your mistakes don't impact the clarity of your work, you'll be fine. They're not going to instantly penalise you for every word you misspell - they're aware that your exam pieces are unpolished first drafts done under time constraints with unseen material - that's the point! But they do have to split the state somehow, and imposing time limits is often the easiest way for them to break apart those who can write decent essays from those who can do so quickly.

However, if your sentences are so wacky that they have no clue what you're trying to say, or if your handwriting is all over the place and they'd need a rosetta stone to decode it... you may not get the mark you're capable of. If some of these small things are worrying you, then it's worth spending some time fixing them sooner rather than later.

3) In LA, when we analyse the authors use of language, will everyone have a different effect on the reader? Nothing too extreme though. Also, will people have slightly different tones?
Not every tone/technique needs to be linked explicitly to the reader. It's good to do this often, but your focus is always on the author's intention, remember. That said, of course different students will comment on different tones, or use slightly different vocabulary in order to do so. Again, it's not like the assessors have a conclusive list of 'stuff you're allowed to talk about' or that if you don't say the tone is 'aggressive' and instead say it's 'hostile' or 'antagonistic,' they won't give you the credit. The way you actually analyse tone is far more important. There is some leeway with interpretation (ŕ la 'if you can justify your answer, then it's right') but if you've radically misunderstood the argument or the tone, then you run the risk of assessors not recognising the underlying analysis skills you're demonstrating. Try to play it safe, and if identifying/analysing tone or linking it to the author/audience is something you're unsure of, then that'd be a good place to start with your revision :)

Title: Re: VCE English Question Thread
Post by: oooo on August 21, 2016, 06:47:17 pm
For a prompt, can my contention be essentially the prompt but with one of the words changed. For example, if the prompt was Orwell suggests that fear destroys Winston. Discuss, can my contention be Orwell suggests that fear contributes to Winston's destruction.
Title: Re: VCE English Question Thread
Post by: avince on August 22, 2016, 06:38:49 pm
Hi,
Are there any practice essays/ resources (other than websites like sparknotes or shmoop) for the play Measure for Measure by William Shakespeare, this is for my text response sac two weeks from now

Thanks
Title: Re: VCE English Question Thread
Post by: FallingStar on August 22, 2016, 08:21:05 pm
Hi,
Are there any practice essays/ resources (other than websites like sparknotes or shmoop) for the play Measure for Measure by William Shakespeare, this is for my text response sac two weeks from now

Thanks

This: Text Response Resources

A full list of text response materials. Perhaps you have looked at this already. If so, try and find your own and look at buying commercial study guides.
Title: Re: VCE English Question Thread
Post by: HasibA on August 22, 2016, 08:35:35 pm
couple of q's this time:
1) do assessors really care about what type of evidence you use in a text response? i know its mainly quotes vs actual examples, but is there ever really a preference? not sure if i worded this correctly.
2) Are assessors looking to give marks, as opposed to dock marks? if so, how they determine what differentiates a 9 vs 10 :P
3) similar to 1st q- if you decided you like using actual examples in your TR essays, rather than integrating quotes- are you actually penalised? like if i have a 1 or 2 direct quotes but a lot of examples im drawing on and analysed, would they dock marks off for that? again, dont know if i worded it correctly :P
4) how do you deal with teacher feedback being vague and insufficient ? haha im trying to cater to my teachers style for sacs, but after writing an essay all i get is 'be more specfic about what you are arguing' or 'bit more coherence'. again, normally id sent it through atarnotes to check, but unfortunately my teachers style is a bit unique and hard to cater for.. and ive had convos with her to no avail :( (bit random, but any help would be appreciated)

thanks guys :)
Title: Re: VCE English Question Thread
Post by: blacksanta62 on August 23, 2016, 05:24:13 pm
Examination criteria from 2009. Still very relevant as the study design haven't changed.
Please note:
This means that a weakness in one area may pull down your whole writing. Conversely, a strength in one area would pull up your whole writing. And visa versa. Just remember that although I did say that you do not have to analyse every single technique, you still have to analyse the techniques you have chosen well, in the "what?, How? Why?" format.

About your 3rd question, you have to get the contention right or else you get you whole LA wrong. But in terms of the different takes in technique, the examiners will use discretion here. (Don't know if I'm right, Lauren) As long as your explanation is reasonable and the examiners can see where you are coming from, then you should be fine. What they're looking at is if you can understand and analyse the effects of language upon the audience, and if you show that you can do that in the exam, then you should be fine.
Sorry for the late reply and my extended absence (bar the slight posts) from AN.
Thanks for this star, really cleared things up for me :)
Okay, there are a whole bunch of sub-concerns here, so I'll try and break this down.

For starters, not finishing essays at this stage of the year is totally fine. You wouldn't expect to be able to sit a Maths or Science end-of-year exam in August and score as highly as possible, right? Of course not - you wouldn't have covered much of the Unit 4 content, and you probably would have been focusing more so on SACs than exam revision. Assuming you came close to finishing the L.A. piece (like - you'd written at least two and a half decent body paragraphs by the one hour mark) then you should be fine. Timing is only a huge concern at this stage of the year if it takes you a whole hour just to plan, write an intro, and do one body paragraph  :-\

But you did the right thing moving on to the other sections - a lot of students screw up in the exam by spending way too long on a single section (- I nearly did this for Text Response, and a lot of people in my year level did so for L.A.) to the detriment of their other two pieces.Keeping yourself to relatively strict one hour cut-offs for each section is a pretty good plan as it's better to have three essays that are really high quality, but missing the end of a body para or a conclusion, as opposed to two complete, polished essays and one mangled half essay. The assessors marking your work know nothing of how well you did in other sections, so there's no chance of things 'balancing out' just because one essay was brilliant and the other one was barely an essay.

With regards to whether you need to cover everything in Language Analysis... short answer: no.
Long answer: no, because they're testing your ability to be selective, and they don't actually want students to comment on EVERY SINGLE TECHNIQUE they find because that would get really tedious. More to the point - that's not the focus of the task! Your job is to comment on how language is used to persuade readers. Pointing out seven different rhetorical questions, nine examples of inclusive language and six instances of appeals to fear doesn't help you do that. You might analyse some of those things, but you have to contextualise them by linking them to what the author intends for readers to think or feel. VCAA are way more concerned with your capacity to demonstrate an understanding of arguments than how many bits of 'emotive language' you can point out.

Assessors do not have a list of features you must mention - there's no one particular quote or technique that must be mentioned - it's all up to you to select what you believe to be most important in the construction of the author's argument.

The only exception to ^that rule is that you do have to analyse the visual material at least once. Rumour has it that you can't score about a 7/10 without doing this, and whilst I don't know how stringent the assessors are about this, you're at a definite disadvantage if you don't even acknowledge or pick apart any of the images. (& if you really want to be sure, link the visual to the written material! <--That's usually something only high-range essays are able to do, so forging those connections can help bump up your mark in the assessor's mind, albeit only slightly... the quality of the rest of your analysis plays a bigger role, obvs.)

Simply put:
- Student A, who can only find 5 language features to comment on out of the whole material, has to analyse all 5, and is probably going to have a fairly weak essay lacking in breadth.
- Student B, who can find 25 language features and feels the need to comment on all of them, is probably going to have an essay that's choppy and unfocused with too much breadth and not enough quality analysis.
- Student C, who can also find 25 features but selects the 17 BEST and MOST RELEVANT ones to include and dissect, is going to have a much more precise, much more impressive piece.

(note: 17 should not be your ultimate goal, that was a very arbitrary choice for the sake of this example. In fact, you shouldn't really know how many language features you're commenting upon; it should be second nature to you over the course of your analysis. If you can very quickly tell how many techniques you're dealing with in each paragraph or over the course of your essay, it probably means your piece isn't as well integrated as it could be.)

So be like Student C and ensure you're whittling down the material to find the most essential and analysable components! The best way to do this is to practice annotating and planning essays - go through some exam material and highlight absolutely everything you think you COULD analyse. Then, go back through and isolate the stuff you think you SHOULD analyse.
That's a very valid concern; you don't want to focus so much on timing that you end up getting your average down to 55 minutes (yaay!) but the quality has dropped down into the D+ range ( :-\) Instead, try to gradually decrease your time! Let's say it takes you 90 minutes to write a good Text Response piece right now. The next one you write has to be done in 80. Then 75. Then 70. Then 65. You get the idea.

You may run into a hurdle at some point with a particular Area of Study (e.g. I could never manage to write a Context piece in under 65 minutes) which means you'll then have to work on strengthening your other essays even more (e.g. get L.A. under an hour to give you extra time to work on Context, or vice versa).

But it's way better to go for this gradual approach that ensures you maintain the same level of quality than to go 'alright, one essay in one hour, no notes, exam conditions, cold turkey, let's go!'
This is a bit tough to answer outside of just saying that a well-written essay is one that fulfils the criteria for each section, as FallingStar has already noted. If you're talking about what level of imperfection they're willing to accept (as in, how many ungrammatical sentences, word choice errors, spelling mistakes, etc.) then the answer is: so long as your mistakes don't impact the clarity of your work, you'll be fine. They're not going to instantly penalise you for every word you misspell - they're aware that your exam pieces are unpolished first drafts done under time constraints with unseen material - that's the point! But they do have to split the state somehow, and imposing time limits is often the easiest way for them to break apart those who can write decent essays from those who can do so quickly.

However, if your sentences are so wacky that they have no clue what you're trying to say, or if your handwriting is all over the place and they'd need a rosetta stone to decode it... you may not get the mark you're capable of. If some of these small things are worrying you, then it's worth spending some time fixing them sooner rather than later.
Not every tone/technique needs to be linked explicitly to the reader. It's good to do this often, but your focus is always on the author's intention, remember. That said, of course different students will comment on different tones, or use slightly different vocabulary in order to do so. Again, it's not like the assessors have a conclusive list of 'stuff you're allowed to talk about' or that if you don't say the tone is 'aggressive' and instead say it's 'hostile' or 'antagonistic,' they won't give you the credit. The way you actually analyse tone is far more important. There is some leeway with interpretation (ŕ la 'if you can justify your answer, then it's right') but if you've radically misunderstood the argument or the tone, then you run the risk of assessors not recognising the underlying analysis skills you're demonstrating. Try to play it safe, and if identifying/analysing tone or linking it to the author/audience is something you're unsure of, then that'd be a good place to start with your revision :)



Lots of goodies in this and stars posts, I've given it a good read through and have taken it on board.
Title: Re: VCE English Question Thread
Post by: HopefulLawStudent on August 23, 2016, 06:07:09 pm
couple of q's this time:
1) do assessors really care about what type of evidence you use in a text response? i know its mainly quotes vs actual examples, but is there ever really a preference? not sure if i worded this correctly.
2) Are assessors looking to give marks, as opposed to dock marks? if so, how they determine what differentiates a 9 vs 10 :P
3) similar to 1st q- if you decided you like using actual examples in your TR essays, rather than integrating quotes- are you actually penalised? like if i have a 1 or 2 direct quotes but a lot of examples im drawing on and analysed, would they dock marks off for that? again, dont know if i worded it correctly :P
4) how do you deal with teacher feedback being vague and insufficient ? haha im trying to cater to my teachers style for sacs, but after writing an essay all i get is 'be more specfic about what you are arguing' or 'bit more coherence'. again, normally id sent it through atarnotes to check, but unfortunately my teachers style is a bit unique and hard to cater for.. and ive had convos with her to no avail :( (bit random, but any help would be appreciated)

thanks guys :)

1) From my understanding, assessors only care so far as does your evidence support your argument? If you go on a whole spiel about chocolate in your text response essay, it does matter how good that spiel is unless it's connected to your text and the topic you're writing on.

2) They're looking for the things you're doing right, not what you're doing wrong. They're not sitting there in their evil dens post October 26 cackling over every mistake a student makes. Even tho it's hard for us VCE kids to remember sometimes, assessors are by and large genuinely good people (if not a little overworked but then again what Year 12 teacher isn't come exam time?) and not evil masterminds; as such, they're trying to give you the highest mark they can but you gotta give them something to work with.

I may be wrong but I was told that English teachers mark hollistically. i.e. they read your entire essay and after a certain point, your essay *feels* like a certain mark and past that point everything you do should just confirm to them that your essay = this mark out of 10. So when they finish your essay, they go, that essay was an x out of 10, imma write down x out of 10 and move on.

4) After your SACs are over, all you have to worry about is your exam. So the English submissions board will defs be your new BFF cos there are a million hella crazy talented people hovering over there who will be able to give you in-depth and very useful feedback. Apart from that, try sitting down with her and asking her specific questions like...

"How could I have made it more specific?"
"How would you have done that?"
"What did you mean by this piece of feedback? Where in my piece was I not specific enough/incoherent/whatever?"
etc.
Title: Re: VCE English Question Thread
Post by: Gogo14 on August 23, 2016, 10:09:39 pm
Any debaters here? How would you run the argument of preserving human rights? (E.g. Detention centres infringe on the rights of asylum seekers).
Title: Re: VCE English Question Thread
Post by: FallingStar on August 23, 2016, 10:37:29 pm
Any debaters here? How would you run the argument of preserving human rights? (E.g. Detention centres infringe on the rights of asylum seekers).

Please post your specific debate topic and wether you are affirmative or negative. It is really difficult for us to come up with arguments without us knowing what you topic is.  :D
Title: Re: VCE English Question Thread
Post by: purplegiraffe on August 24, 2016, 12:25:36 pm
I need help with writing a text response on This Boy's Life.
I was reading somewhere that when writing, I need to understand and point out that the characters of a text are constructed for the purpose of helping to present the authors idea/opinion.
However, in the case of This Boy's Life, the characters are real people (as the book is a memoir).

Also, any tips in distinguishing between Tobias the character and Tobias the author?

Thank you :D
Title: Re: VCE English Question Thread
Post by: HasibA on August 24, 2016, 07:53:53 pm
does anyone have a good list of techniques used in lang analysis? like rhetoric question, anaphora,etc. An extensive list would be great, but any would do (cant find in notes section)- thanks guys :)
Title: Re: VCE English Question Thread
Post by: FallingStar on August 25, 2016, 12:44:37 pm
does anyone have a good list of techniques used in lang analysis? like rhetoric question, anaphora,etc. An extensive list would be great, but any would do (cant find in notes section)- thanks guys :)

I've found 3. Don't know if anyone else can find more:
http://resources.mhs.vic.edu.au/englishenrichment/analysisoflanguage.htm
http://wiki.engageeducation.org.au/english/language-analysis/
http://www.vcestudyguides.com/wp-content/uploads/downloads/2010/09/Persuasive-Techniques-Insight-Outcomes1.pdf

Hope you're okay with online resources.
Title: Re: VCE English Question Thread
Post by: HasibA on August 25, 2016, 02:16:32 pm
i know this is vague, but what are lets say the 3,4,5 most common mistakes students make in Language Analysis?
again, what the are most common mistakes for context and text response? thank you
edit: more q's
1) again , really vague, but with a quote bank, roughly how many quotes should one have? a rough number would be nice, i know its vague , but thanks :)
Title: Re: VCE English Question Thread
Post by: HopefulLawStudent on August 25, 2016, 03:58:39 pm
i know this is vague, but what are lets say the 3,4,5 most common mistakes students make in Language Analysis?
again, what the are most common mistakes for context and text response? thank you
edit: more q's
1) again , really vague, but with a quote bank, roughly how many quotes should one have? a rough number would be nice, i know its vague , but thanks :)

Language Analysis: "The writer does X to make the reader interested" and my all time favourite of all time "There is a question mark, therefore it is a rhetorical question and this is supposed to make the reader think." These are all quotes from essays I've marked/read over for friends and classmates.
Title: Re: VCE English Question Thread
Post by: Gogo14 on August 25, 2016, 06:57:41 pm

The government should Ban the current Australian policy on asylum seekers(or something like that. I'm aff, and just need help on the argument for human rights. Don't know how to explain why rights are important.
Please post your specific debate topic and wether you are affirmative or negative. It is really difficult for us to come up with arguments without us knowing what you topic is.  :D
Title: Re: VCE English Question Thread
Post by: HasibA on August 29, 2016, 10:16:09 am
anyone go over the main differences between style and tone? examples? thanks guys :)
edit: my teacher says generally examiners do not like diary entries in context pieces, is this true?
Title: Re: VCE English Question Thread
Post by: maylovesgelati on August 29, 2016, 11:25:48 am
When writing a text response, should we write our body paragraphs in order of importance or in reverse order of importance? As in write our best ideas first or write them last?
Title: Re: VCE English Question Thread
Post by: Maz on August 29, 2016, 02:04:38 pm
When writing a text response, should we write our body paragraphs in order of importance or in reverse order of importance? As in write our best ideas first or write them last?
In all honesty I don't think it really matters much; either way, it's going to get written and marked. Try and get it in an order that flows; so if you talk about point of view first for example; it kind of ties into everything and will lead to other things like how the characterisation of x character is (which is influenced by the point of view). Just make sure you actually finish the essay (if in doubt of this happening do the good ones first). You could have a really good paragraph fist, the worst one second and another really good one last- could be one way it could be structured.
Title: Re: VCE English Question Thread
Post by: FallingStar on August 29, 2016, 06:58:06 pm
I'm not Lauren but I shall answer your questions to the best of my abilities.

When writing a text response, should we write our body paragraphs in order of importance or in reverse order of importance? As in write our best ideas first or write them last?
It doesn't really matter that much. As mq123 has said, you should write in the order that flows the most. Say your body paragraph is about these:
(assuming a thematic prompt)
1. Major characters and how they show a certain theme.
2. Minor character and how they show a certain theme.
3. Symbols and how they show a theme.
4. Events and how they show a theme.

You can order your essay in 1,2,4,3 or 4,1,2,3. Just as long as the order makes sense, and as long as you can link your paragraphs to the next, then you should be fine.

i know this is vague, but what are lets say the 3,4,5 most common mistakes students make in Language Analysis?
again, what the are most common mistakes for context and text response? thank you
edit: more q's
1) again , really vague, but with a quote bank, roughly how many quotes should one have? a rough number would be nice, i know its vague , but thanks :)

Common mistakes in Language Analysis (In my opinion)
Technique spotting
This is when in a language analysis, all you do is to spot out the techniques, but you do not analyse the effects, as in the how and the why. Eg. The writer uses a mired of questions. Then, he uses a personal anecdote. And so on... This is called language analysis for a reason. Your aim is to analyse the language intentions, not to write every technique you can think of down.

Missing the "how" or the "why"
This is very similar to the above, but is more about the lack of analysis instead of none at all. Eg. There is a personal anecdote, encouraging the readers to feel sympathy for the writer.

Being repetitive
With something like language analysis, it is very common for people to be repetitive as it is a very structured piece. Sometimes, this can be due to a lack of vocabulary. Other times, the structure of the essay is formulaic, which are responses that examiner are already sick of. (I've been told off for this)

Evaluating rather than analysing
Pretty much what is says. Sometimes, this is done unintentionally but it is nonetheless a mistake I've seen people make (and I am an offender too). Eg. Piece A is more persuasive than piece B, this piece is very persuasive, this piece is not very persuasive overall.

About your quotes question...
An examiner talk about context in my school quite recently, and for context, he said that you need to memories 10-15 quotes about your context. That means that for text response, you may need ~20 quotes. On the contrary, it is better to memories 15 really good quotes you can apply to any essay topic than to memories 30 quotes that you are rarely use in your text response essays. Do not let these numbers be definitive as you should focus on memorising quality quotes as opposed to many really not-so-good quotes.

The government should Ban the current Australian policy on asylum seekers(or something like that. I'm aff, and just need help on the argument for human rights. Don't know how to explain why rights are important.
Think about the UN declaration of human rights. Read them carefully. They had came after the world wars where governments were very cruel to the minorities. Also, we've been roasted by the UN for a policy as such. You could argue that it makes us a cruel nation in terms of how we treat refugees and that does not make for a good rep. overseas. Like what would other countries think of us. (Of course when you are actually debating, don't put it in my words as this is too casual for DAV). As I probably explained it badly, I suggest ask anyone doing global politics 3/4 about this issue. They probably know better than me.
Title: Re: VCE English Question Thread
Post by: One Step at a Time on August 30, 2016, 04:59:09 pm
Hi everyone  ;D

Pretty general question about text response topics:

How do you answer questions that ask how the text shows the importance of a theme?

How do you tackle this kind of prompt? Totally lost as to what the topics sentences/ contention could be... I can't base each paragraph on a theme since the essay itself is on a theme...

e.g. How does Gattaca show the importance of social class?
Title: Re: VCE English Question Thread
Post by: HopefulLawStudent on August 30, 2016, 05:08:09 pm
Hi everyone  ;D

Pretty general question about text response topics:

How do you answer questions that ask how the text shows the importance of a theme?

How do you tackle this kind of prompt? Totally lost as to what the topics sentences/ contention could be... I can't base each paragraph on a theme since the essay itself is on a theme...

e.g. How does Gattaca show the importance of social class?

I'm well aware of the fact that my approach is not necessarily the most... erm... conventional when it comes to these prompts.

But I tackle these prompts by basically centering my discussion on elements of the text that showcase the theme and the different aspects of the theme that the text explores.
Title: Re: VCE English Question Thread
Post by: One Step at a Time on August 30, 2016, 07:01:23 pm
Hi HopefulLawStudent!

Thanks for your reply :D

What are you referring to when you said "elements of the text" and "the different aspects of the film"?


Title: Re: VCE English Question Thread
Post by: One Step at a Time on August 30, 2016, 07:17:42 pm
Did some thinking and came up with ideas for 2 of the paragraphs for the topic "How does Gattaca show the importance of class division?"

Para 1: The society of Gattaca is based on class division.
-Talk about the nature of this society and how it come about
-Talk about why the citizens were wiling to accept this

Para 2: Some characters challenge the class system though...

Is what I'm talking about for each paragraph on track? I'm so confused as to how to break the prompt down into topic sentences and the contention  :(

Title: Re: VCE English Question Thread
Post by: taylorcrivari on August 31, 2016, 02:33:50 pm
I have my last English SAC coming up, which is a text response to In The Country of Men. The prompt we've been given to write a draft on is, "How does the dominance of men affect Suleiman?". Can someone please help me in working out how to structure the piece and what topics to discuss? I'm lost  :-[
Title: Re: VCE English Question Thread
Post by: HopefulLawStudent on September 01, 2016, 09:27:09 am
Hi HopefulLawStudent!

Thanks for your reply :D

What are you referring to when you said "elements of the text" and "the different aspects of the theme"?

So like what is the director saying about the theme? So like for the prompt you've supplied... What is Niccol saying about class division? How does he/the film demonstrate that it's important? Are there instances in the film wherein Niccol is arguing that class division ISN'T important? All stuff that I would personally want to explore in an essay.

I have my last English SAC coming up, which is a text response to In The Country of Men. The prompt we've been given to write a draft on is, "How does the dominance of men affect Suleiman?". Can someone please help me in working out how to structure the piece and what topics to discuss? I'm lost  :-[

Well, what've you got so far and what are you stuck on? No one on AN will ever do your work for you; we will endeavour to help you as much as we can but you need to give us something to work with too. :)
Title: Re: VCE English Question Thread
Post by: One Step at a Time on September 01, 2016, 07:22:51 pm
Thanks so much HopefulLawStudent!

How would I centre my paragraphs on what Niccol is saying about class division (element of the text) as you said?

Title: Re: VCE English Question Thread
Post by: larissaaa_ on September 02, 2016, 08:12:26 pm
Is anyone studying This Boy's Life and can give me some tips on how to structure an essay around it? Like is it better to structure paragraphs according to characters or thematically? And what are some of the most common examples to structure essays around? I'm averaging A+'s but now my teacher has really high expectations for our last text response SAC on This Boy's Life on Wednesday but I feel like I am absolutely gonna screw it up, all my practice pieces have been horrible and I can't meet the time limit!

Title: Re: VCE English Question Thread
Post by: Elizawei on September 03, 2016, 09:29:27 pm
Hey guys, does anyone have examples of social class conflict that has happened in Australia? Looking for external sources for my context essay, but finding it hard to find real world examples :)

Thanks in advance :)
Title: Re: VCE English Question Thread
Post by: Adequace on September 03, 2016, 09:35:06 pm
Hey guys, does anyone have examples of social class conflict that has happened in Australia? Looking for external sources for my context essay, but finding it hard to find real world examples :)

Thanks in advance :)
Don Dale Youth Detention Centre.
Title: Re: VCE English Question Thread
Post by: Elizawei on September 03, 2016, 09:44:00 pm
Don Dale Youth Detention Centre.

Oh yeah! Thanks so much :D
Title: Re: VCE English Question Thread
Post by: Sine on September 04, 2016, 12:30:09 am
What sort of holiday routine would one have for studying for the english exam? Would it be something like an essay a day?
Title: Re: VCE English Question Thread
Post by: HopefulLawStudent on September 04, 2016, 11:47:16 am
What sort of holiday routine would one have for studying for the english exam? Would it be something like an essay a day?

Hell no.

If you do an essay a day, I can guarantee that you'll burn out and in the long run that'll be worse. Aim for one/two essays a week on top of like other English stuff that you do re:consolidating notes or whatever. If you aim too high, you'll burn and crash before the English exam and no one wants that.
Title: Re: VCE English Question Thread
Post by: larissaaa_ on September 04, 2016, 04:00:44 pm
I've read when doing text response it's bad to structure your paragraph in the agree-agree-disagree order. does that mean if we are challenging the prompt and agreeing to it but also disagreeing in some way in each of our paragraphs we have to talk about why we agree and use examples AND why we also disagree and use examples? How do you do that without fence-sitting?
Title: Re: VCE English Question Thread
Post by: HopefulLawStudent on September 04, 2016, 04:43:33 pm
I've read when doing text response it's bad to structure your paragraph in the agree-agree-disagree order. does that mean if we are challenging the prompt and agreeing to it but also disagreeing in some way in each of our paragraphs we have to talk about why we agree and use examples AND why we also disagree and use examples? How do you do that without fence-sitting?

Pick a side.

I honestly can't think of a way where you can argue a case for agree and then turn around and argue a case for disagree and not get chewed out by an assessor... unless you were a wizard (m'kay, that was a bad joke). seriously tho it's not cohesive enough and reads like you have no idea what you're doing so you're throwing everything at the assessor with the hopes that they'll work it out and give you a 10/10 for trying (doesn't work that way, unfortunately).
Title: Re: VCE English Question Thread
Post by: FallingStar on September 04, 2016, 05:05:05 pm
I've read when doing text response it's bad to structure your paragraph in the agree-agree-disagree order. does that mean if we are challenging the prompt and agreeing to it but also disagreeing in some way in each of our paragraphs we have to talk about why we agree and use examples AND why we also disagree and use examples? How do you do that without fence-sitting?

Lauren explained what to do in this post: Re: I'm not scared essay help
Just that the title may be deceptive, as it is a text that a someone else is doing. Basically, you mostly agree or disagree with the prompt, with you contention being an "Although _______ ultimately ________ " statement.
Eg. Although the central female characters in the film are restricted by other things, ultimately they are limited by their gender.

As that would be the contention already establishes a mostly agree stance, they we need to put some challenge statements. eg. Despite Eve's rise to fame through her defiance of limitations placed upon women, most of the characters in the film are seen to be limited by their gender (probably not the best example). Now, you will need to integrate this sentence into your body paragraphs, as opposed to placing all of your "challenge statements" into one paragraph. Just do one of these per paragraph.
Title: Re: VCE English Question Thread
Post by: larissaaa_ on September 04, 2016, 05:37:42 pm
Lauren explained what to do in this post: Re: I'm not scared essay help
Just that the title may be deceptive, as it is a text that a someone else is doing. Basically, you mostly agree or disagree with the prompt, with you contention being an "Although _______ ultimately ________ " statement.
Eg. Although the central female characters in the film are restricted by other things, ultimately they are limited by their gender.

As that would be the contention already establishes a mostly agree stance, they we need to put some challenge statements. eg. Despite Eve's rise to fame through her defiance of limitations placed upon women, most of the characters in the film are seen to be limited by their gender (probably not the best example). Now, you will need to integrate this sentence into your body paragraphs, as opposed to placing all of your "challenge statements" into one paragraph. Just do one of these per paragraph.

Thankyou! So basically, in each paragraph we just need a sentence or two challenging the prompt, but do we need examples from the text also challenging the prompt? So for example, if the prompt was "men and violence are inextricably linked in This Boy's Life" I would have my contention as something like: Although Wolff (the author) presents some of his male characters as somewhat benevolent and compassionate, ultimately men are shown to be linked to violence as they strive to achieve the American Dream. Then in each of my paragraphs I'll explain how the men are violent and give examples, then have like a few sentences here and there reminding the reader that not all of the men in the novel are violent? Or do I need to also need some examples in each of my paragraphs as to why not all of them are violent? I feel like I'm still contradicting myself
Title: Re: VCE English Question Thread
Post by: Sine on September 04, 2016, 09:20:38 pm
Hell no.

If you do an essay a day, I can guarantee that you'll burn out and in the long run that'll be worse. Aim for one/two essays a week on top of like other English stuff that you do re:consolidating notes or whatever. If you aim too high, you'll burn and crash before the English exam and no one wants that.
What would you or anyone else here suggest?

Any examples from people who have completed english previously?
Title: Re: VCE English Question Thread
Post by: blacksanta62 on September 05, 2016, 09:01:53 pm
Yo guys, this may be personal preference, but how do I write a killer introduction for text response? It's been 6 months since I've written a text response and I have a SAC next Thurs. Would you recommend embedding quotes into intro?

Thanks :)
Title: Re: VCE English Question Thread
Post by: goodbye on September 10, 2016, 11:30:10 am
So I've been reading a lot Ianguage analysis essays lately and a few of the high scoring ones used contractions; isn't this like one of the greatest no-no's of essay writing? Or is it just a language analysis thing cause it's not really an essay?
Title: Re: VCE English Question Thread
Post by: blacksanta62 on September 10, 2016, 04:50:40 pm
So I've been reading a lot Ianguage analysis essays lately and a few of the high scoring ones used contractions; isn't this like one of the greatest no-no's of essay writing? Or is it just a language analysis thing cause it's not really an essay?
Really? Yeah, my English teacher has warned us that using informal language will change the quality of a piece. Could you provide just a snippet please? Maybe the analysis of the language used overshadowed it. I personally got a 17/20 for language analysis back in term 2 and I too read high range responses.

That mark brings me to my question: How can I improve my expression in my essay's? I think some of my sentences read awkwardly and when the teachers are cross marking, my mark is reduced as a result. Any recommendations would be appreciated.

Also, when reading a novel, what things should I be annotating in pencil? Key quotes (short and sweet), important scenes etc. The novel I am reading is In the county of men by Hisham Matar.

Thank you
Title: Re: VCE English Question Thread
Post by: goodbye on September 10, 2016, 07:55:41 pm
Really? Yeah, my English teacher has warned us that using informal language will change the quality of a piece. Could you provide just a snippet please? Maybe the analysis of the language used overshadowed it. I personally got a 17/20 for language analysis back in term 2 and I too read high range responses.

That mark brings me to my question: How can I improve my expression in my essay's? I think some of my sentences read awkwardly and when the teachers are cross marking, my mark is reduced as a result. Any recommendations would be appreciated.

Also, when reading a novel, what things should I be annotating in pencil? Key quotes (short and sweet), important scenes etc. The novel I am reading is In the county of men by Hisham Matar.

Thank you

Um, this isn't really one of the high scoring ones, but it's from Lauren's (so 'high scoring' in essence  :P) blogpost on language analysis structure:

"...praise was welcome, it shouldn't be an expected part of..." (from her sample intro)

Also, I've done a bit more digging and I've found that a similar question was asked a few pages back. Someone answered that it's okay as along as it doesn't "mar your clarity". Can someone please second this? Also, is it just a L.A thing?

Title: Re: VCE English Question Thread
Post by: HopefulLawStudent on September 11, 2016, 12:07:49 pm
Um, this isn't really one of the high scoring ones, but it's from Lauren's (so 'high scoring' in essence  :P) blogpost on language analysis structure:

"...praise was welcome, it shouldn't be an expected part of..." (from her sample intro)

Also, I've done a bit more digging and I've found that a similar question was asked a few pages back. Someone answered that it's okay as along as it doesn't "mar your clarity". Can someone please second this? Also, is it just a L.A thing?

Your exam is marked HOLISTICALLY.

No assessor is ever gonna be like "ooooh that wrote 'isn't' instead of 'is not'. This essay would have been a 10/10 if you had but since you didn't... I'll give you a 3/10." So long as it does not impede clarity and it isnt dead obvious (i.e. you aren't using a colloquialism/informal word for every three words you write), you should be fine.

Also, Lauren will probably second this whenever she sees this but no essay is perfect irrespective of what the person who wrote it ended up getting in terms of study scores. As such, while I'm sure as hell that her essay is hella awesome (cos y'know Lauren is just hella awesome full stop), it will still have it's good points and bad points so don't take her essay (or anyone's essay really) as a "THIS IS A PERFECT ESSAY. EVERYTHING SHE DOES MUST BE 100% CORRECT."
Title: Re: VCE English Question Thread
Post by: Hydraulix on September 11, 2016, 04:04:35 pm
Hey peeps



"While we deplore Medea's actions, we find little sympathy for Jason? Do you agree

For the prompt above are we supposed to accept the "we deplore Medea's actions" part and write our text response regarding the second part. or are we allowed to argue the first part

Thankssss
Title: Re: VCE English Question Thread
Post by: goodbye on September 11, 2016, 04:51:51 pm
Hey peeps



"While we deplore Medea's actions, we find little sympathy for Jason? Do you agree

For the prompt above are we supposed to accept the "we deplore Medea's actions" part and write our text response regarding the second part. or are we allowed to argue the first part

Thankssss

Nope, you can disagree with the we deplore part. Kind of treat the "while we...sympathy for Jason" part as a statement, if that makes it easier.
Title: Re: VCE English Question Thread
Post by: Hydraulix on September 11, 2016, 06:14:55 pm
Two more questions for text response:

Can I use "we" in regard to how the text makes us feel?

Should I instead be saying that "Euripides positions the audience to...."

Thanks guys
(and thanks goodbye for the speedy response)
Title: Re: VCE English Question Thread
Post by: Maz on September 11, 2016, 06:17:49 pm
Two more questions for text response:

Can I use "we" in regard to how the text makes us feel?

Should I instead be saying that "Euripides positions the audience to...."

Thanks guys
(and thanks goodbye for the speedy response)
I would say write 'positions the audience to...', there is nothing 'wrong' with saying we, but when discussing the positioning of people in general,it sounds better to say audience. When it asks for your reader response, use I.
Just my opinion though, both are fine  :)
Title: Re: VCE English Question Thread
Post by: goodbye on September 13, 2016, 02:37:34 pm
 Hello! I have a question about the new study design, specifically regarding the new language analysis. Within the study design it says "how this may strengthen or detract from the intended impact". Does this essentially mean we have to say if a text is persuasive or not? One of the key skills is also to "critique ideas presented in the arguments". Also, since there's a new emphasis on author's intent, should it be wise to include this within introductions?
Title: Re: VCE English Question Thread
Post by: larissaaa_ on September 13, 2016, 11:35:40 pm
"we are never without hope even in the most difficult conflicts"

I'm really struggling to think of three sub-arguments to base my paragraphs around, does anyone have any suggestions? My text is Every man in this village is a liar but any help with historical examples and stuff I could use would be much appreciated  :)
Title: Re: VCE English Question Thread
Post by: HopefulLawStudent on September 17, 2016, 01:34:27 pm
"we are never without hope even in the most difficult conflicts"

I'm really struggling to think of three sub-arguments to base my paragraphs around, does anyone have any suggestions? My text is Every man in this village is a liar but any help with historical examples and stuff I could use would be much appreciated  :)

Well, what have you got so far?
Title: Re: VCE English Question Thread
Post by: EmileeSmith on September 17, 2016, 02:42:41 pm
how do you talk about context in an essay
Title: Re: VCE English Question Thread
Post by: FallingStar on September 18, 2016, 05:22:04 pm
how do you talk about context in an essay

I have to assume you're talking about Text Response.
It depends. You will need to mention it in your introduction somewhere. And perhaps in you body paragraphs. I know the answer is very general and because this is an online forum and you do need to give us more specific information so that we can give you a better answer.
Title: Re: VCE English Question Thread
Post by: EmileeSmith on September 18, 2016, 05:30:54 pm
my teacher says to give context when introducing a text in an essay, she explained to me about what context is but I still don't understand? like do you talk about the history of the novel or author? or do you talk about the location of the set scene in the text???

and also in an essay, my teacher tells us to talk about what happens before a quote and the scene for example in Away, my teacher tells us to write something like 'In Away,  Act 3, Scene 2, Coral goes up to Tom and says, "quote", but isn't this retelling? should I use this or not
Title: Re: VCE English Question Thread
Post by: Coffee on September 18, 2016, 07:07:12 pm
my teacher says to give context when introducing a text in an essay, she explained to me about what context is but I still don't understand? like do you talk about the history of the novel or author? or do you talk about the location of the set scene in the text???

and also in an essay, my teacher tells us to talk about what happens before a quote and the scene for example in Away, my teacher tells us to write something like 'In Away,  Act 3, Scene 2, Coral goes up to Tom and says, "quote", but isn't this retelling? should I use this or not

Don't talk about the history of the novel or author, it isn't necessary and it won't gain you any marks. Context can be helpful but it should be brief and to the point. For example, if you're talking about Medea, you might want to mention that it is set within a patriarchal society and make a distinction between the ancient Greek audience and modern audiences (and how this affects the interpretation of the text). Another good example can be found in the 2014 English examination report with the example of Stasiland which writes: "In Anna Funder's exploration into the oppressive and restrictive reign of communism in what was the GDR...", or something along the lines of 'Set against the backdrop of...

As for the example of retelling, I would think this is fine so long as its followed up by analysis which should constitute the majority of your essay. You definitely shouldn't be writing extensively about what happens before a quote but the example you gave seems fine.
Title: Re: VCE English Question Thread
Post by: tim.wells on September 18, 2016, 09:22:41 pm
my teacher says to give context when introducing a text in an essay, she explained to me about what context is but I still don't understand? like do you talk about the history of the novel or author? or do you talk about the location of the set scene in the text???

and also in an essay, my teacher tells us to talk about what happens before a quote and the scene for example in Away, my teacher tells us to write something like 'In Away,  Act 3, Scene 2, Coral goes up to Tom and says, "quote", but isn't this retelling? should I use this or not

I agree with Coffee here, you don't want to get too caught up in the minutia of the background of your text, however you want to breifly set up the context for further discussion — this is especially important if the context is an important feature of the text you are studying. For instance, a previous English text was Ransom by David Malouf, which was derived from Homer's Iliad, this had a lot of bearing on further discussion of the novel and that's why it would be resourceful to include references in the introduction touching on this. To see this in action, have a read of the shortened introduction and body paragraphs below:

"David Malouf has explored many concepts concerning courage in order to fully encapsulate its meaning within his poetic retell of Homer’s Iliad , Ransom. To Malouf, courage is determined through acts of valour and bravery; it also involves the notion of humility, a recurring motif within Ransom. Juxtaposing the classical mythology of the Iliad, Malouf’s understanding of courage is not centred on feats of military prowess but rather explores courage as an expression of humanity. This contrast is a central focus throughout the text as we witness those who would once be considered to possess courage portrayed as the most flawed individuals within Ransom ..."

And then you can use these references as touchstones throughout your essay, for example:

"Achilles, hero of the Iliad and quintessential embodiment of power and the ‘warrior spirit’ is introduced to readers within Ransom ‘hunker[ing] down… shoulders hunched’. Initially he is presented as a man who is ‘listening for the voice of his mother tasting the sharp salt’ of his reality without her on ‘his lip’. Hardly an impenetrable hero he is ‘darkly divided’. In the aftermath of the loss of his ‘soul mate’, Patroclus, Achilles is inconsolable, indeed we witness ‘his soul change colour’ ... Similarly, Heracles, a figure from Priam’s early history and renowned as “the labourer of the gods” in other texts, too is a “traditional” hero. Yet ‘the whole terrible machinery of the man’ is just ‘rank meatiness’ and is unable to comprehend Hesione’s act of compassion – rather he thought she would ‘choose some gaudy trinket’ than save her brother. He is made to look both ‘brutish’ and ‘foolish’ before Hesione and her great act of courage. It is through this exchange that Malouf, unlike his predecessor, suggests that acts of bravery do not always belong to the physically commanding and powerful but also reside with the meek and uncelebrated."

So I think the main takeaway is that context can be a useful tool, as long as it is done subtly and you never stray too far from your contention. Examiners are on the lookout for pre-prepared responses. So be careful of this, they want originality, and I think this will occur if you always keep the prompt in the forefront of your mind when including context. For instance, in the above example, the prompt is clearly about courage, and therefore context can be used to illustrate how courage is perceived in our society, as compared to what it was when the Iliad was first written.
Title: Re: VCE English Question Thread
Post by: EmileeSmith on September 18, 2016, 09:33:57 pm
ohh that makes sense, but what if you cant make the context link to the question ??
Title: Re: VCE English Question Thread
Post by: tim.wells on September 19, 2016, 08:27:32 am
ohh that makes sense, but what if you cant make the context link to the question ??

I think it's a skill that you just need to practise, I found when I practised writing text responses I would break them down into different components and just focus on getting that one aspect right for a period of time. So for instance here, this would mean looking at as many practise questions as possible and finding/writing connections to the context.

But after doing this, if you still feel unsure. Then I recommend including a sentence regarding context in your intro
Title: Re: VCE English Question Thread
Post by: Hydraulix on September 19, 2016, 10:59:10 am
For the prompt (Medea by Euripides)

‘Ordinary people with ordinary emotions, as represented by the Chorus, are caught between sympathy for Medea and horror at her actions.’ Discuss

Is it okay if I take this approach to writing it. Do you guys think I'm not addressing the prompt well or if there are better ways to do it. Is the way that I have seperated my arguments okay?

CONTENTION
There is a constant battle between sympathy and horror for Medea but ultimately when Medea has the cold resolve to kill her innocent children only horror if felt for Medea.

PLAN

Para 1 – They have sympathy for Medea at the start. Fear is present but only slightly as they fear what she may do in act of revenge (especially the Nurse). Creon solidifies this fear when he decides to exile her. The chorus mainly feel sympathy towards her for the way she has been treated.

Para 2 – She names the people that she plans to kill and here the chorus is horrified. The chorus still do not believe she will be able to "steel" her heart so they maintain sympathy for her condition

Para 3 – Why is sympathy never lost… because of her language and the way the characters around her talk about her and treat her (Aegeus is compassionate, .  ( This paragraph is about the language Medea uses to maintain sympathy from the Chorus

Para 4 – Ultimately horror prevails and we have no sympathy for a woman who has murdered and gotten away with it. She is literally and figuratively above the people of Corinth and leaves the city in ruins. She put her emotions above other people’s lives and the Chorus attempt however to soften the blame on Medea by placing it on the unexpected god for making it possible


Thanks so much for taking the time <3

Title: Re: VCE English Question Thread
Post by: tim.wells on September 19, 2016, 12:56:40 pm
Personally I think this is a good set-up, there is a nice progression of feelings for Medea before you ultimately come to a conclusion as to the sentiment of the Chorus.

Just be careful with this question as it is split into two parts: ordinary people; and the balance of sympathy and horror. You have clearly addressed the second component of the question very well. But just be mindful to continually reference 'ordinary people with ordinary emotions' throughout your response.

I think the first way to do this would be to mention why the chorus does represent ordinary people [I would do this in your intro, and then refer back to it throughout the essay].

I also think another way to do this would be to contrast how 'ordinary people' feel these emotions with how royalty/divinity feels these emotions. This will deepen your analysis as you can refer to what separates the ordinary from the divine and how this affects their perception of the world around them.

I haven't studied Medea in depth, but I think you could particularly compare the way Aegeus perceives Medea as compared to the chorus because he is an outsider and has recently come into contact with a quasi-divine being [the Oracle at Delphi].

But just remember that the focus of such comparisons are to strengthen your analysis of how ordinary people are caught between sympathy and horror.
Title: Re: VCE English Question Thread
Post by: FallingStar on September 19, 2016, 06:24:12 pm
Hi,
Can someone please interpret this comments by my teacher please (for a context SAC) and put it in simple forms.

" --Insert name here-- , You have developed a strong conceptual idea, and been consistent in following the potential possibilities that can be gleaned. The structure enables the response to cite a variety of ways in which realities can be 'distorted.'

To really enable yourself to develop and finesse you writing techniques, it is essential you discard unnecessary addition elements in providing an explaination. If you don't construct simple, clear sentences, strong ideas will wither amid the flood of unnecessary surround sentence elements."

Also, are there any tips and to mitigating this issue in terms of revising for the exams?

Thanks guys.
Title: Re: VCE English Question Thread
Post by: TheLlama on September 19, 2016, 07:19:08 pm
Hi,
Can someone please interpret this comments by my teacher please (for a context SAC) and put it in simple forms.

" --Insert name here-- , You have developed a strong conceptual idea, and been consistent in following the potential possibilities that can be gleaned. The structure enables the response to cite a variety of ways in which realities can be 'distorted.'

To really enable yourself to develop and finesse you writing techniques, it is essential you discard unnecessary addition elements in providing an explaination. If you don't construct simple, clear sentences, strong ideas will wither amid the flood of unnecessary surround sentence elements."

Also, are there any tips and to mitigating this issue in terms of revising for the exams?

Thanks guys.

Ah, somewhat ironic. The suggestion for improvement seems to be twofold:

As for mitigating the issue, it really depends on what your writing actually looks like - in isolation, the feedback is a little generic and could mean many things (including that the giver of the feedback likes to make themselves sound intelligent ;)
Title: Re: VCE English Question Thread
Post by: Elizawei on September 19, 2016, 08:23:04 pm
Hey guys, got a general question (kinda)

In introductions of text responses/context, is it better to directly quote the prompt? Or reword the prompt instead of copying it?

Thanksss
Title: Re: VCE English Question Thread
Post by: FallingStar on September 19, 2016, 08:33:05 pm
Hey guys, got a general question (kinda)

In introductions of text responses/context, is it better to directly quote the prompt? Or reword the prompt instead of copying it?

Thanksss

Just reword the prompt. Make sure you understand what the prompt is actually saying before you do so though. Just use Lauren's technique of distilling the prompt down to a "Basically ______." Then reword the prompt in the intro. The sooner you get to the prompt, the better (as Lauren pointed out if you've been to the lecture).

Also, use the prompt words throughout your context piece, which would make it clear to the examiner that you are addressing the prompt. And use synonyms too, such that your essay does not sound repetitive (a trap I constantly fall into).
Title: Re: VCE English Question Thread
Post by: HopefulLawStudent on September 20, 2016, 12:27:56 pm
Hi,
Can someone please interpret this comments by my teacher please (for a context SAC) and put it in simple forms.

" --Insert name here-- , You have developed a strong conceptual idea, and been consistent in following the potential possibilities that can be gleaned. The structure enables the response to cite a variety of ways in which realities can be 'distorted.'

To really enable yourself to develop and finesse you writing techniques, it is essential you discard unnecessary addition elements in providing an explaination. If you don't construct simple, clear sentences, strong ideas will wither amid the flood of unnecessary surround sentence elements."

Also, are there any tips and to mitigating this issue in terms of revising for the exams?

Thanks guys.

Ah, somewhat ironic. The suggestion for improvement seems to be twofold:
  • The first part seems to imply that you've gone into depth when responding to the prompt, that you've seen an opportunity to raise interesting and thoughtful ideas. You've also selected a structure that allows you to consider the prompt / context in multiple ways.
  • That your ideas are lacking clarity because you're providing additional, seemingly unrelated points in the same sentence. Without seeing your writing, what this means is speculation. It could be that you're combining two ideas in the same sentence and not developing each. Alternatively, it might be that you're working on developing a particular style that detracts from the essence of meaning, for this reader. Or it could be that your reader feels that you're overloading a sentence with too many phrases.

As for mitigating the issue, it really depends on what your writing actually looks like - in isolation, the feedback is a little generic and could mean many things (including that the giver of the feedback likes to make themselves sound intelligent ;)

I agree with TheLlama. Sounds like your teacher is saying that your ideas are lacking clarity.

This sentence is interesting though:

Quote
If you don't construct simple, clear sentences, strong ideas will wither amid the flood of unnecessary surround sentence elements.

Is verbosity a big problem for you? Check sentence length from that SAC (if they go on for like a million lines, you need to cut back).

Bit hard to give advice re:mitigating the issue without knowing exactly what the issue is. Have you talked to your teacher about your SAC? They tend to be pretty good at explaining their comments. :)
Title: Re: VCE English Question Thread
Post by: MightyBeh on September 26, 2016, 12:42:16 pm
Hey family. I've been entered into some essay writing competition (exhilarating, I know ::) ) but it has a very english style prompt and I don't even know how to approach it. I can count on one hand how many times in the last two years I've had to write from a prompt so any pointers would be super cool. ty :-*
Title: Re: VCE English Question Thread
Post by: HasibA on September 26, 2016, 12:47:48 pm
When re-analysing a film for text response, what features should i be looking for that are normally missed/ forgotten. i.e for Mabo, looking at Perkins' cinematic techniques, camera angles and how they relate to the film is common, but im finding it difficult to find 'other' film related things to go over for it! thanks :)
Title: Re: VCE English Question Thread
Post by: NerdyPi on September 26, 2016, 04:11:43 pm
Hi guys, so I've been getting contradictory info was teacher(s)/tutor so I'm wondering if AN can clarify these points for me in relation to the English exam...?

-In language analysis, you're not required to comment on /predict the audience's reaction to the persuasion (pretty sure examiners report said this too)

-If doing expository for context, you're only meant to discuss the text in one paragraph

Also, for an expos piece in context, is basing each paragraph (aiming for 4) on an idea related to the prompt and then bringing in 1 relevant example that demonstrates this idea ok??? My teacher said 1 example / paragraph is fine, but my tutor said 2 is needed...
Thanks
Title: Re: VCE English Question Thread
Post by: HopefulLawStudent on September 26, 2016, 04:46:04 pm
Hi guys, so I've been getting contradictory info was teacher(s)/tutor so I'm wondering if AN can clarify these points for me in relation to the English exam...?

-In language analysis, you're not required to comment on /predict the audience's reaction to the persuasion (pretty sure examiners report said this too)

-If doing expository for context, you're only meant to discuss the text in one paragraph

Also, for an expos piece in context, is basing each paragraph (aiming for 4) on an idea related to the prompt and then bringing in 1 relevant example that demonstrates this idea ok??? My teacher said 1 example / paragraph is fine, but my tutor said 2 is needed...
Thanks

Re:LA -- none of my teachers have ever said anything about needing to predict the audience's reaction to the persuasion. Someone else confirm but I think you just assume that the writer is being persuasive and roll with that.
Title: Re: VCE English Question Thread
Post by: FallingStar on September 26, 2016, 05:21:49 pm
Re:LA -- none of my teachers have ever said anything about needing to predict the audience's reaction to the persuasion. Someone else confirm but I think you just assume that the writer is being persuasive and roll with that.

Confirmed. Always assume that the writer is being persuasive, even if you think it's not. In terms of predicting audience's reaction, you are not supposed to do that, but you are instead to write on the intended reaction. This means you are talking about what the writer intended for the audience/readers/viewers/listeners (depending on text type) to react.
Title: Re: VCE English Question Thread
Post by: tim.wells on September 27, 2016, 05:54:07 pm
Hi guys, so I've been getting contradictory info was teacher(s)/tutor so I'm wondering if AN can clarify these points for me in relation to the English exam...?

-In language analysis, you're not required to comment on /predict the audience's reaction to the persuasion (pretty sure examiners report said this too)

-If doing expository for context, you're only meant to discuss the text in one paragraph

Also, for an expos piece in context, is basing each paragraph (aiming for 4) on an idea related to the prompt and then bringing in 1 relevant example that demonstrates this idea ok??? My teacher said 1 example / paragraph is fine, but my tutor said 2 is needed...
Thanks

For an expository piece there is no requirement for you to only discuss the text in one paragraph. If you find doing that helps you keep your ideas and your writing structured then go with that. But personally, I wrote an expository piece and I referred to the text in every paragraph throughout the piece, rather than doing a discrete 'text-based' paragraph. I felt that this allowed me to express my ideas in a more nuanced manner as I could use the text as an example of the idea that I was discussing in a particular paragraph. However, if you are going to opt for this method, you need to know your text inside-out as it is much harder to 'pre-prepare' a response.

I also think that one example per paragraph is perfectly acceptable provided that you discuss it in adequate depth. Personally, I have a preference for discussing fewer examples but really delving into them. This shows the examiner a greater engagement with your context and allows you to really flesh out your ideas and tie them to the prompt.
Title: Re: VCE English Question Thread
Post by: rhosemary on September 28, 2016, 10:33:50 pm
My question is... probably a stupid one. I'm going to ask it regardless.
Am I allowed to write about a text on the English exam that we haven't studied in class? I don't like either of our texts- "This Boy's Life" and "Burial Rites". I can WRITE about them, sure, I've gotten high marks, but I don't have any IDEAS about them other than understanding them academically. I look at the exam prompts and mostly go "yeah, I guess you could say that". It would be boring.
But, two of the texts on the list are some that I've loved before this year: "Wuthering Heights" and "All About Eve". I have literally written about both of these in my spare time for the internet (because I'm a fun person  ::) ) and I have, I think, a thorough understanding of them both. The Examiner's Report for 2015 exams states:
Students should be encouraged to have confidence in their own reading and demonstrate a
personal understanding of their text, rather than relying exclusively on commercially produced
material. All texts are complex works of art with a wealth of opportunity for exploration. There are
no „expected‟ responses to topics, and the highest-scoring pieces were those that were thoughtful
and fresh.

Hard to do that if you aren't interested in the texts. As is probably obvious, I genuinely love literature and I want to do my best in the exam- I think I will almost definitely have more thoughtful and detailed responses to "Heights" or "Eve". So the question is: a) Am I allowed? and b) Do you think it's suicide-by-essay?
Wow, sorry to ramble.
Title: Re: VCE English Question Thread
Post by: FallingStar on September 29, 2016, 01:44:16 pm
My question is... probably a stupid one. I'm going to ask it regardless.
Am I allowed to write about a text on the English exam that we haven't studied in class? I don't like either of our texts- "This Boy's Life" and "Burial Rites". I can WRITE about them, sure, I've gotten high marks, but I don't have any IDEAS about them other than understanding them academically. I look at the exam prompts and mostly go "yeah, I guess you could say that". It would be boring.
But, two of the texts on the list are some that I've loved before this year: "Wuthering Heights" and "All About Eve". I have literally written about both of these in my spare time for the internet (because I'm a fun person  ::) ) and I have, I think, a thorough understanding of them both. The Examiner's Report for 2015 exams states:
Students should be encouraged to have confidence in their own reading and demonstrate a
personal understanding of their text, rather than relying exclusively on commercially produced
material. All texts are complex works of art with a wealth of opportunity for exploration. There are
no „expected‟ responses to topics, and the highest-scoring pieces were those that were thoughtful
and fresh.

Hard to do that if you aren't interested in the texts. As is probably obvious, I genuinely love literature and I want to do my best in the exam- I think I will almost definitely have more thoughtful and detailed responses to "Heights" or "Eve". So the question is: a) Am I allowed? and b) Do you think it's suicide-by-essay?
Wow, sorry to ramble.

This isn't a stupid question. This is a tough one actually.

How ironic is it that I come from a school that literally does the two texts you wanted to do?  :P
 
Have you done any VCAA style prompts on either All About Eve or Wuthering Heights?

I would like to say though that examiners don't actually know the text that you are doing at school. In fact, they blind mark your exam paper (ie. no VCAA number or any of your details). And it gets marked by two examiners, who don't even teach in your school. They are also marked independently of other student at your school, so it's not like one examiner marks papers for one school.

So theoretically, you can do a text/context outside of you school. In practise, it is usually preferred that you do the text that your school's doing since your teachers are not guaranteed to know the texts in the list that you want to do, or at least know it well. So if you do choose to do a text outside of school, then you may be better off asking for external help. Perhaps get in a study group with others studying the particular text that your're doing.

Whilst you are to demonstrate a personal understanding of the text that you are doing, you can still use study guides as an aid in your studies. And since you are doing a text outside what you school is doing, you may as well use them. A list can be found in Text Response Resources for all your texts.

Focus on either All About Eve or Wuthering Heights but not both. If you choose to do All About Eve, then you could get HopefulLawStudent to mark you essay. However, if you like Wuthering Heights more, than perhaps someone else can mark you essay when you post it in the English Work Submission and Marking thread. But all in all, you will need to get help from a teacher in terms of the text since the submissions thread is a peer marking system, and you may need to take their marking as a grain of salt (not saying it's bad, if you mark other's essays, it can help you too!  :) And if you essay gets marked, then the student that marked it does help you get used to different styles of marking). Thereby I will ask you to do the following steps:

1. Speak to your teacher about that. If s/he knows either of these texts well and can discern VCAA prompts from these texts, you're in luck. If not, take the next step.
2. Look for another teacher (in your school or outside) that is doing your texts. In the best case, you will find one within your school. If you can't find a teacher, a student who scored highly in the English exam and can teach/tutor such as one of the National Moderators of this forum can be asked too (although the person I linked would be a really good teacher anyway  ;) ).
3. Get into a study group of students who are actually doing the text you pick , and have done it at school.

Although: I would like a second opinion on this as I am not 100% sure about it. So I advise you to take this as a guide.
Title: Re: VCE English Question Thread
Post by: HasibA on September 29, 2016, 05:59:02 pm
alright guys , just a few q's
1) with context, ive got a whole piece set out and memorised. Im wary of writing verbatim on the exam and linking to the prompt, but as a lot of people mentioned on AN, how about do i go adding 'malleable' fragments and ideas to the piece, or even start thinking of a new approach for my context 😭 :'(
i have one relatively malleable idea based piece, but i feel as if i need more, but i dont really know what else i can cover in the Imaginative/Hybrid medium 😭 my context is encountering conflict.

2) if you're not aiming for a particularly high english score, is it pragmatic to just read study guides for each section, particularly section A- Text reponse? esp. if your short on time etc. ? thoughts? i really dont need that high to get into my course, but i still want to try my best etc. so impretty conflicted
Title: Re: VCE English Question Thread
Post by: HopefulLawStudent on September 30, 2016, 11:53:20 am
If you choose to do All About Eve, then you could get HopefulLawStudent to mark you essay.

Gotten a few PM's asking re: this. So just gonna clarify it all here to save myself the time. Yes, I am actually marking AAE essays for any AN users who ask (esp if you throw in a please cos I tend to be more willing to help people with manners :P), no I'm not crazy, no there is absolutely no catch and I can have essays marked within a week generally.
Title: Re: VCE English Question Thread
Post by: FallingStar on October 01, 2016, 09:33:24 am
I have been given conflicting information about this:

Is it better to do timed full essays, or to do timed paragraphs? Like from my experience, some teachers say timed paragraphs well others say paragraphs or say: an intro and a paragraph.

Can you please explain which one is more effective in terms of studying for the English exam? Thanks.  :)
Title: Re: VCE English Question Thread
Post by: Swagadaktal on October 01, 2016, 10:51:33 am
I have been given conflicting information about this:

Is it better to do timed full essays, or to do timed paragraphs? Like from my experience, some teachers say timed paragraphs well others say paragraphs or say: an intro and a paragraph.

Can you please explain which one is more effective in terms of studying for the English exam? Thanks.  :)
Both are excellent!

What are you looking for at this current moment? Is your content fine but your timing isn't? Then Timed conditions it is!
However, if you're not satisfied with the content then don't do timed - no point in writing paragraphs for practice that aren't up to standard :)
Title: Re: VCE English Question Thread
Post by: Hydraulix on October 02, 2016, 09:16:26 am
Hiyaa!!

For Language Analysis where there are multiple pieces to analyse are we supposed to compare them or analyse them separately? I'm familiar with writing comparatively as that is what we had to do for our Language Analysis SAC.

I've heard conflicting information regarding this so I was hoping one of you peeps could clear it up for me.

Thankkk youuu
Title: Re: VCE English Question Thread
Post by: tim.wells on October 02, 2016, 02:45:57 pm
Hiyaa!!

For Language Analysis where there are multiple pieces to analyse are we supposed to compare them or analyse them separately? I'm familiar with writing comparatively as that is what we had to do for our Language Analysis SAC.

I've heard conflicting information regarding this so I was hoping one of you peeps could clear it up for me.

Thankkk youuu


The structure for this is up to your individual taste, but I was taught two different methods.

1. Analyse the pieces by grouping arguments, comparing the articles throughout your response.

2. Analyse the pieces separately and then write 1 - 2 comparative paragraphs at the end.

The first method probably allows for a more sophisticated analysis, and you will have more opportunities to juxtapose/contrast the articles side by side.

Whereas the second method is easier, and probably much easier for an examiner to identify/allocate marks.

You can do excellently with either method, try practising both and then pick the method that you find easiest to execute!
Title: Re: VCE English Question Thread
Post by: Elizawei on October 02, 2016, 03:23:36 pm
Quick question,
Is it better to do 3 huge body paragraphs or 4 smaller body paragraphs?
4 I can explore more ideas but 3 I can go into them in greater detail :P

Any thoughts?

Thanks in advance!
Title: Re: VCE English Question Thread
Post by: MightyBeh on October 02, 2016, 03:35:32 pm
Quick question,
Is it better to do 3 huge body paragraphs or 4 smaller body paragraphs?
4 I can explore more ideas but 3 I can go into them in greater detail :P

Any thoughts?

Thanks in advance!
I think three excellent paragraphs are much better than four regular paragraphs. I'd be picking my three best ideas and writing a lot on those. :)
Title: Re: VCE English Question Thread
Post by: Choco Chops on October 02, 2016, 04:07:00 pm
Hi, this is my first time asking a question on ATAR notes,

I just wanted to know, with textual topics that contain a quote in it such as, "I am knifed to the hilt with fate." What does Burial Rites say about the burden of destiny?, does the quote need to be addressed in the essay or should it be ignored? I've been told both things by my teachers so I'm not quite sure what the examiners expect. Could someone help clarify this for me?

Thanks in advance :)
Title: Re: VCE English Question Thread
Post by: FallingStar on October 02, 2016, 05:04:06 pm
Hi, this is my first time asking a question on ATAR notes,

I just wanted to know, with textual topics that contain a quote in it such as, "I am knifed to the hilt with fate." What does Burial Rites say about the burden of destiny?, does the quote need to be addressed in the essay or should it be ignored? I've been told both things by my teachers so I'm not quite sure what the examiners expect. Could someone help clarify this for me?

Thanks in advance :)

With topics containing quotes, they must be embedded in you essays, and should not be ignored. Also, it is not just a free quote you can put in your essays anytime.  you need to also show an understanding of the context in which the quote is used in you essay, and what that particular quote means, and what is means for the character, themes and views and values of the text.
Title: Re: VCE English Question Thread
Post by: kimmytaaa on October 04, 2016, 02:00:48 pm
Hi
i have a question my teacher said that we won't get awarded if we compare the language analysis in the exam, is that correct?
Title: Re: VCE English Question Thread
Post by: Choco Chops on October 04, 2016, 07:34:25 pm
With topics containing quotes, they must be embedded in you essays, and should not be ignored. Also, it is not just a free quote you can put in your essays anytime.  you need to also show an understanding of the context in which the quote is used in you essay, and what that particular quote means, and what is means for the character, themes and views and values of the text.

Thanks FallingStar,

Sorry for the delayed reply but could you possibly explain what you mean in more simpler terms? So, do I have to directly show my knowledge of where the quote comes from and use it in one of my paragraphs in its entirety? If you could please provide an example of what you mean as well, that would be great.

Thanks
Title: Re: VCE English Question Thread
Post by: FallingStar on October 05, 2016, 03:35:21 pm
Thanks FallingStar,

Sorry for the delayed reply but could you possibly explain what you mean in more simpler terms? So, do I have to directly show my knowledge of where the quote comes from and use it in one of my paragraphs in its entirety? If you could please provide an example of what you mean as well, that would be great.

Thanks

Basically:
  - Must put the quote in the essay
  - Must know the context of the quote.
  - Must understand how the quote fits in with the text.

(Since I haven't done Burial Rites, I will do All About Eve)
Say if a prompt said Margo says "funny business, a woman's career." All About Eve is about the roles that women must play. Discuss. Then I can say:

Topic Sentence of paragraph extract: In All About Eve, it is evident that women who have careers must eventually marry and give their career up.

By confessing in the car that a "woman's career" is "funny business," Margo realise that she eventually need to succumb to the predefined roles as  a women. She defines these roles as being a wife where she "look up just before bed" to see the man she loves, who is seen to bring her comfort throughout the film. This confession implies that women must eventually marry and become a wife to the man that she loves most, signifying that women are to marry no matter how long they had their careers for, or how many careers they ever has.

Well, I'm not too good at English  :o <gasps> but at least that should give you an idea.
Title: Re: VCE English Question Thread
Post by: TheLlama on October 05, 2016, 03:55:48 pm
Basically:
  - Must put the quote in the essay
  - Must know the context of the quote.
  - Must understand how the quote fits in with the text.

(Since I haven't done Burial Rites, I will do All About Eve)
Say if a prompt said Margo says "funny business, a woman's career." All About Eve is about the roles that women must play. Discuss. Then I can say:

Topic Sentence of paragraph extract: In All About Eve, it is evident that women who have careers must eventually marry and give their career up.

By confessing in the car that a "woman's career" is "funny business," Margo realise that she eventually need to succumb to the predefined roles as  a women. She defines these roles as being a wife where she "look up just before bed" to see the man she loves, who is seen to bring her comfort throughout the film. This confession implies that women must eventually marry and become a wife to the man that she loves most, signifying that women are to marry no matter how long they had their careers for, or how many careers they ever has.

Well, I'm not too good at English  :o <gasps> but at least that should give you an idea.

Largely, the aim is to show your engagement with the quote - you don't have to specifically quote it back to your reader (although doing so can show you're analysing the ideas).

If you take the quote you've just mentioned, a strong response would recognise what happens around that quote and how it connects to the second part of the prompt. Here, the remainder of the quote goes to the idea of a woman losing or shedding parts of herself in order to ascend into the show-business stratosphere. So what's crucial is that becoming a star entails sacrificing your femininity. That is, if women need to return to the "role" of being a wife, it's because of the cost to their personal lives and well-being: they cannot hold onto both in this "funny business." If you're able to get to that idea, it will allow you to create a much stronger response to the prompt!

As for Burial Rites, if you got a topic including the "knifed to the hilt with fate" quote (it's one of those really, really quoted ones), then you would ideally deal with the concepts it suggests. That is, Agnes believes that her tragic situation is the product of divine forces and remains outside her control. Find the quote and you'll see she goes a little into religion! It's useful and relevant to deal with the consequences of her belief: the suggestion of violence, and the fact that she believes the knife has been plunged all the way in. Put differently, the suggestion is that she has been metaphorically stabbed: eventually the knife will have to be removed, leading to her inevitable demise. If you focus on the consequences of the quote, and what it tells us more broadly about the text, then you'll be writing a solid, relevant response.

Hope that helps. :)
Title: Re: VCE English Question Thread
Post by: Choco Chops on October 05, 2016, 06:36:27 pm
Yes, they help.

Thanks FallingStar (again) and TheLlama ;D
Title: Re: VCE English Question Thread
Post by: Daliaradosevic on October 05, 2016, 07:03:30 pm
Could someone please explain to me the differences of syntax sentence structure) eg. polysndenton syntax, asyndeton etc?? :)
Title: Re: VCE English Question Thread
Post by: Rob16 on October 05, 2016, 08:37:52 pm
I know there is no uniform essay structure in language analysis but if there are two articles given, have u guys been taught to always have one paragraph comparing the two articles or is it okay to have for example paragraph 1: Article A Analysis, Paragraph 2: Article B analysis, Paragraph 3: Image analysis + link to article
Title: Re: VCE English Question Thread
Post by: taylorjj57 on October 06, 2016, 08:26:49 am
Hi
i have a question my teacher said that we won't get awarded if we compare the language analysis in the exam, is that correct?

I'm of the understanding that if there are multiple pieces in the LA then you have to mention them all otherwise you will lose marks. I've also been told that you should comment on how the reader is swayed after reading everythig at the end of your response so yoyou sstill hahave to compare in some mode .

There are two ways you can go, either compare throughout each body or compare iin a fifinal body. Markers still reward each way as long as you have explained how techniques influence the reader. I'm pretty sure I've been told by an exam marker that as long as you do this for each piece you're  pretty good.

That being said including comparative phrases like 'in contrast', 'however' and 'alternatively' when moving between pieces will deepen your response and hence gain higher marks
Title: Re: VCE English Question Thread
Post by: nadiaaa on October 06, 2016, 02:45:43 pm
Hiii just have some questions for context
Im writing about 4-4 and a half pages in a hour, is that normal? or too short? How long is everyone elses?
Also do we have to talk about our chosen text the same number of times as our external examples?
SO like in my essay i have like 3 external and 2 invictus (my chosen text) ideas - is that alright?
Thanks guys :D
Title: Re: VCE English Question Thread
Post by: HopefulLawStudent on October 06, 2016, 04:05:30 pm
Hiii just have some questions for context
Im writing about 4-4 and a half pages in a hour, is that normal? or too short? How long is everyone elses?
Also do we have to talk about our chosen text the same number of times as our external examples?
SO like in my essay i have like 3 external and 2 invictus (my chosen text) ideas - is that alright?
Thanks guys :D

Perfectly normal. Think quality of your writing over quantity.* i.e. 4 hella good pages are waaaay better than 8 half-assed idk-what-I'm-doing pages.

Think less "ratio of text to external" and think more about how you're using the text.
Title: Re: VCE English Question Thread
Post by: nadiaaa on October 07, 2016, 01:23:16 pm
Perfectly normal. Think quality of your writing over quantity.* i.e. 4 hella good pages are waaaay better than 8 half-assed idk-what-I'm-doing pages.

Think less "ratio of text to external" and think more about how you're using the text.
Sweet, thanks heaps :D
Title: Re: VCE English Question Thread
Post by: dylan862 on October 07, 2016, 01:59:41 pm
Is a context paragraph important to include or can I go without it? I feel it's not relevant but I've been told I need one
Title: Re: VCE English Question Thread
Post by: Coffee on October 07, 2016, 06:51:48 pm
I know there is no uniform essay structure in language analysis but if there are two articles given, have u guys been taught to always have one paragraph comparing the two articles or is it okay to have for example paragraph 1: Article A Analysis, Paragraph 2: Article B analysis, Paragraph 3: Image analysis + link to article
Hi Rob16 :)

You might find this helpful: Re: Comparative Language Analysis - Essay Structure It outlines a few different ways you can approach structuring a comparative language analysis essay. Take a look and see if that helps and if you still have questions, let us know. :)
Title: Re: VCE English Question Thread
Post by: YellowTongue on October 07, 2016, 10:32:54 pm
In Burial Rites, Hannah Kent has included an 'author's note' at the conclusion of the text. Is this something that I can refer to in my essays?

Also, this may be a ridiculous observation, but I think their are some elements of a Shakespearean tragedy contained within the novel. That is, Agnes is "cursed" by a mysterious "traveller" as a young maid, and she attempts to "escape" her "destiny" but ultimately succumbs to her inevitable "fate".

Would it be okay to comment on this as a structural element in my essays? If so, how would I refer to it?
Title: Re: VCE English Question Thread
Post by: IDK on October 10, 2016, 02:59:25 am
Doing Burial Rites on the exam. Obviously considering last year was the first year it was on the TR list, we have two prompts to look at. Anyone doing this text will know that the first prompt last year on the exam was about 'Life and Death' and the second about Agnes 'freedom and internal thoughts'.

How big a risk am I taking in assuming that the next two prompts would likely encompass two completely different sets of themes or concerns in the novel. The novel has a vast array of themes and concerns, and the next two logically appropriate and most important themes to me are the Landscape and Patriarchal Society.

How much am I gambling by directing my focus primarily to these two themes. I operated on this assumption on my sac and trial exam as well, these two themes are just so large in the book and I would almost argue that they are as big as the two themes in the prompts for last year.

Anyone comments on the reward for risk here?

I'm honestly of the opinion that prompt luck plays a massive role in exam scores, especially Text Response, so it may be worth it.

Title: Re: VCE English Question Thread
Post by: molecular. on October 10, 2016, 05:13:26 pm
Doing Burial Rites on the exam. Obviously considering last year was the first year it was on the TR list, we have two prompts to look at. Anyone doing this text will know that the first prompt last year on the exam was about 'Life and Death' and the second about Agnes 'freedom and internal thoughts'.

How big a risk am I taking in assuming that the next two prompts would likely encompass two completely different sets of themes or concerns in the novel. The novel has a vast array of themes and concerns, and the next two logically appropriate and most important themes to me are the Landscape and Patriarchal Society.

How much am I gambling by directing my focus primarily to these two themes. I operated on this assumption on my sac and trial exam as well, these two themes are just so large in the book and I would almost argue that they are as big as the two themes in the prompts for last year.

Anyone comments on the reward for risk here?

I'm honestly of the opinion that prompt luck plays a massive role in exam scores, especially Text Response, so it may be worth it.

Hey mate,

I am doing the same thing for 'Medea'. Last year, the biggest theme was on there. And I'm guessing that this year their will be the other key theme and essentially, if I get it, I'll do well OR If it doesn't come up, I'm screwed. SO thats that risk I think. But I'm so sure its going to be on there; I just have a feeling.
Title: Re: VCE English Question Thread
Post by: Sine on October 10, 2016, 10:34:52 pm
Should I revise a little for a second text response text just in case the prompts for my preferred texts aren't what i expected?
Title: Re: VCE English Question Thread
Post by: IDK on October 11, 2016, 01:15:44 pm
Hey mate,

I am doing the same thing for 'Medea'. Last year, the biggest theme was on there. And I'm guessing that this year their will be the other key theme and essentially, if I get it, I'll do well OR If it doesn't come up, I'm screwed. SO thats that risk I think. But I'm so sure its going to be on there; I just have a feeling.

There are never gurantees though in reality. I've heard that the VCAA prompts are essentially randomly generated, but I'm not 100%, can anyone clarify?

My text has a lot of different themes which does mean I'm running quite a big risk too but I'm not sure. Perhaps it will be enough to just have enough knowledge of the whole text to write a decent response if it doesn't go my way.
Title: Re: VCE English Question Thread
Post by: HopefulLawStudent on October 11, 2016, 04:33:34 pm
There are never gurantees though in reality. I've heard that the VCAA prompts are essentially randomly generated, but I'm not 100%, can anyone clarify?

My text has a lot of different themes which does mean I'm running quite a big risk too but I'm not sure. Perhaps it will be enough to just have enough knowledge of the whole text to write a decent response if it doesn't go my way.

VCAA prompts are not randomly generated.

There isn't some magic VCAA machine that pumps out two prompts per text every year. From my understanding, there are a bunch of people who design the prompts every year.

My two cents:

-

It's Oct 26. You've been studying the theme of oranges for a text that deals with the themes apples, oranges, pears, tomatoes and zucchini for your Section A text. The main themes from this text are apples and oranges; apples was assessed last year so obv this year, it MUST be oranges. You're so ready; you're gonna write a killer text response essay on oranges because that's definitely gonna be what's on the exam. You're gonna write this hella awesome piece and then you're gonna get that 10/10 and that 50SS and become a millionaire selling oranges. You are so sorted.

Then you open the booklet.

Two prompts.

One is on zucchini, that thing you looked at once all year.
The other is on celery; how is celery in the text?!

You have two choices:
1. Write a piece on oranges when they asked for zucchini and be penalised for lack of relevance.
2. Write a piece on zucchini/celery/whatever with half-assed points and ideas because you didn't look over it and haven't written a single essay dealing with that all year.

"But I want to get my 50SS and become a millionaire selling oranges!", you say?

Well then, here's my advice:

Prepare for everything. Don't make assumptions. Focus on a particular theme if you want to take the risk but for christ's sake practice other ones. Don't assume anything. You can hope and pray for certain things but that shouldn't give you the freedom to not at least look at other things. If there's anything I learnt from failing methods and spec last year it's this: prepare for the worst case scenario while wishing for the best case scenario.

tl;dr: Oranges > Zucchini and Celery any day.
Title: Re: VCE English Question Thread
Post by: TheLlama on October 11, 2016, 07:49:17 pm
VCAA prompts are not randomly generated.

There isn't some magic VCAA machine that pumps out two prompts per text every year. From my understanding, there are a bunch of people who design the prompts every year.

Indeed: the idea is that the prompts are deliberately designed. More than that, they're meant to work in a few specific ways. For one, they are typically meant to be broad enough in nature that they provide you with an opportunity to show what you know about the text. Besides that, they need to be accessible so that the vast majority of students (ideally all of them) would be able to write some response.

So you'll often see prompts that look like: "Text is about big idea/theme" or "A key conflict/relationship/takeaway of the text is ____". The surprising or unexpected part of the prompt is often the specific quote that's selected, if one is used. Equally, you might find a prompt directing you more to specific characters and their relationship, which tends to narrow your focus. But...

Quote
Prepare for everything. Don't make assumptions. Focus on a particular theme if you want to take the risk but for christ's sake practice other ones. Don't assume anything. You can hope and pray for certain things but that shouldn't give you the freedom to not at least look at other things. If there's anything I learnt from failing methods and spec last year it's this: prepare for the worst case scenario while wishing for the best case scenario.

I would encourage you to think in different terms. Trying to prepare for everything is often a surefire way to stretch yourself thin in English. Instead, you might want to think about the interconnections between ideas. What are the main conflicts within your texts? What are the central themes its author is drawing to our attention? In terms of preparing, it's often useful to consider the ways in which you might adapt and alter smaller prepared pieces given a number of different topics. Given you've got a relatively short time left, it's often wiser focusing on flexibility.

Equally, remember the essay is an opportunity to show off. You don't so much want to show that you know a lot of details about everything. Instead, you're aiming to reveal some insight into really fascinating ideas. Or, to paraphrase HLS a little: you're looking to do more than spot the vegetables in a question. If they suggestion is, "in what ways is this text about zucchini?", and you can reveal the dichotomy or link between zucchinis and oranges, then you're likely creating a response that's relevant to the prompt. :)
Title: Re: VCE English Question Thread
Post by: nadiaaa on October 12, 2016, 07:17:17 am
Guys i begin my expository piece with a quote that i touch on the intro and then in my body paragraph i expand on it, my teacher has never made a comment on it but now she is saying im not allowed to begin a paragraph with a quote, is this true?
Title: Re: VCE English Question Thread
Post by: literally lauren on October 12, 2016, 11:22:47 am
Could someone please explain to me the differences of syntax sentence structure) eg. polysndenton syntax, asyndeton etc?? :)
Okay, polysyndeton and asyndeton aren't really to do with sentence structure - it's about how you list things.
Polysyndeton means having lots of 'and's or conjunctions (since 'poly' = many)
Asyndeton means having no 'and's or conjunctions (since 'a-' = without; as in, 'asymmetrical' = without symmetry, etc.)

So an example of polysyndeton would be if I were to say 'I went to the shops because I had to buy chips and drinks and lollies and chocolate.'
Asyndeton would be: 'I went to the shops and bought chips, drinks, lollies, chocolate.'

Usually, the effect is the same (assuming you're analysing this in L.A.) as the author is typically emphasising the excessive nature of whatever he's listing.

I'm not sure if your question was more about sentence structure though, so let me know if I've misinterpreted anything! :)

Is a context paragraph important to include or can I go without it? I feel it's not relevant but I've been told I need one
Would this be for Text Repsonse or Language Analysis? Either way, my answer's the same - you probably don't need to include it. My best guess is that this is an L.A. thing where you have an opening paragraph about the issue and what sparked it or what people's response has been, which you definitely don't need to do in the exam. Just get to the author's contention ASAP in your intro, and then you can start earning marks for your analysis in your body paragraphs. Everything else is just peripheral window dressing :P

In Burial Rites, Hannah Kent has included an 'author's note' at the conclusion of the text. Is this something that I can refer to in my essays?

Also, this may be a ridiculous observation, but I think their are some elements of a Shakespearean tragedy contained within the novel. That is, Agnes is "cursed" by a mysterious "traveller" as a young maid, and she attempts to "escape" her "destiny" but ultimately succumbs to her inevitable "fate".

Would it be okay to comment on this as a structural element in my essays? If so, how would I refer to it?
You can refer to the author's note if you want to, but to me it seems like the kind of thing that's not going to be wholly useful for many prompts, so I'd prioritise the contents of the actual novel above that.

Re: Shakespearean allusions... I'm not sure how strong the connection is there. Fate and destiny are age-old themes that many texts use as plot devices or key themes, so whilst you could definitely refer to those features in B.R. I don't think it's worth comparing them to Shakespeare or any other text unless you feel like it's directly contributing to your analysis.

Should I revise a little for a second text response text just in case the prompts for my preferred texts aren't what i expected?

Personally, I wouldn't bother because I'm of the opinion that you'd be better off spending that time revising your chosen text and trying to cover all bases than trying to re-learn an entirely different plot/bunch of quotes 'just in case' :/

Think of it this way: if you double down and devote all your T.R. time to your preferred text, it means that virtually everything you do is going to directly help you in the exam. Whereas, if you split your efforts between two texts, you're doing twice the amount of work for only half the reward. What's more - there are very few things VCAA could give you that you wouldn't be able to handle if you prepare adequately, and I say this as someone who got a killer prompt for my text back in 2013. No joke, I looked at it and went '...well there's an entire theme I never knew existed until just now. K, let's do this' and just used what I knew and applied it to the bizarrely left-of-field prompt I was dealt. (There was another option, but that was boring ::) ) So I tend to recommend just focusing on one text to make things easier for you, but if you really want a back up plan and you feel like it wouldn't be too taxing to devote some time to it, I guess it wouldn't hurt to give your secondary text a quick read through or write an essay or two.


& on a related note... just to pick up on the delightful food metaphors going on above...
Two prompts.

One is on zucchini, that thing you looked at once all year.
The other is on celery; how is celery in the text?!
Totally agree with HLS and TheLlama here; the worst thing you can do in cases like this is to say that
   a) Yes, zucchini is a thing in the text, as are tomatoes... here's an essay about tomatoes.
   b) Yes, celery is in the text, just like oranges... here's an essay on oranges.

You can work in related themes and ideas if they are relevant, but you have to make them relevant! To use TheLlama's example:
you're looking to do more than spot the vegetables in a question. If they suggestion is, "in what ways is this text about zucchini?", and you can reveal the dichotomy or link between zucchinis and oranges
Connections between themes and ideas are an awesome way to feel more prepared, and VCAA will often try to throw you off balance by giving you a prompt like 'The text is about courgettes. Discuss' so that half the state goes 'oh shit, wtf is a courgette??' ...but if you're smart and brought a dictionary in, then you'll be able to work out...
(http://i.imgur.com/YiJwe4p.png)
...oh look, it's the same major theme and they're just using a close synonym!

But to return to the previous analogy, you NEVER want a contention like 'yep, courgettes are a thing, BUT SO ARE ORANGES' because that's going to be seen as a massive topic dodge. Even having a whole body paragraph exclusively devoted to oranges would be too much, in my opinion. What you can do, however, is have a paragraph about the way courgettes and oranges go great together in a salad... or something... k, imma move away from the food metaphors and go for an actual prompt so this doesn't get too weird...

e.g. In All About Eve, the characters struggle to overcome their own egos. Discuss.

If you look at that and think, yeah, they do struggle to overcome their own egos, but they also struggle to overcome social expectations... be very careful! You can't just talk about social expectations INSTEAD of their egos... but you can talk about how social expectations shape their egos and sense of self, for instance, of how their egos create social expectations for themselves and one another that can be hard to overcome.

In short: add stuff to the prompt, if you can and if you need to, but never replace stuff in the prompt with something you prepared earlier, as you'll likely be heavily penalised for trying.

Guys i begin my expository piece with a quote that i touch on the intro and then in my body paragraph i expand on it, my teacher has never made a comment on it but now she is saying im not allowed to begin a paragraph with a quote, is this true?
Somewhat frustratingly, you can do whatever you want in an expository essay. You can begin a paragraph with a quote if you use it and explain it well, but perhaps your teacher is telling you not to do that because at the moment, the quote doesn't feel like it's being integrated? You never want to make it seem like the quote is doing the work for you - if you're able to explain its significance to your discussion/contention/prompt though, that's a perfectly fine way to begin.

Defs don't do it in T.R. or L.A. though! :)
Title: Re: VCE English Question Thread
Post by: Manige123 on October 14, 2016, 09:09:23 pm
How does the structure of an article persuade readers?
I have a generally good understanding of doing language analysis', except my teacher said I need to focus on the structure used by the writer in attempt to persuade. I didn't get a chance to ask my teacher, so hopefully someone on here can help : )
Title: Re: VCE English Question Thread
Post by: YellowTongue on October 16, 2016, 07:04:09 am
For text response, my teacher has told me that I must not refer to any specific characters in topic sentences; only discussing what the author says through the text. How can I use this approach to respond to character specific prompts?

For instance (for Burial Rites):

"What role does absence play in Agnes' life?" Discuss
Title: Re: VCE English Question Thread
Post by: Daniellac on October 16, 2016, 02:47:18 pm
Hey,

I was just wondering if there is another word for 'readership' for language analysis, I saw 'circulation" for like newspapers and magazines and stuff could I use that instead so I don't sound so mundane?

Thank you!  :)
Title: Re: VCE English Question Thread
Post by: FatimaEl on October 16, 2016, 04:59:07 pm
Hey,

I was just wondering if there is another word for 'readership' for language analysis, I saw 'circulation" for like newspapers and magazines and stuff could I use that instead so I don't sound so mundane?

Thank you!  :)

I could be wrong, and do correct me if i am, but i would say that circulation more so refers to how widely distributed a newspaper or magazine is, and is not a very close synonym to readership, could you give me the sentence you are writing so i can give you a more appropriate word? 
I would say the word 'demographic' is a more suited replacement :) hope that helped x
Title: Re: VCE English Question Thread
Post by: FatimaEl on October 16, 2016, 05:32:17 pm
This may be a very obvious question, but i was just wondering are we allowed to use brackets in essays? Ive always avoided using them for some reason, and i do not mean these [ ] types of brackets that can be used when modifying a quote to grammatically suit your sentence, but rather these ( ) type of brackets to include additional information rather than using commas, same goes with - this -, are we allowed to use dashes for extra information? or must we always use commas?
Title: Re: VCE English Question Thread
Post by: tim.wells on October 16, 2016, 08:42:18 pm
This may be a very obvious question, but i was just wondering are we allowed to use brackets in essays? Ive always avoided using them for some reason, and i do not mean these [ ] types of brackets that can be used when modifying a quote to grammatically suit your sentence, but rather these ( ) type of brackets to include additional information rather than using commas, same goes with - this -, are we allowed to use dashes for extra information? or must we always use commas?

I don't think there are any hard and fast rules regarding this, personally I have used both and there have never been any issues. I think it can be a useful way to add ancillary information without detracting from the focus of your discussion. I have found them particularly useful during language analysis.
Title: Re: VCE English Question Thread
Post by: FatimaEl on October 16, 2016, 08:58:50 pm
I don't think there are any hard and fast rules regarding this, personally I have used both and there have never been any issues. I think it can be a useful way to add ancillary information without detracting from the focus of your discussion. I have found them particularly useful during language analysis.

thank you Tim! and yes i was mainly wondering this cause i found it would be rather useful to use during my language analysis essays :)
Title: Re: VCE English Question Thread
Post by: Rob16 on October 16, 2016, 10:30:12 pm
Can anyone help me think of some synonyms for moral compass? my teacher said its too overused by other people and am having trouble thinking of what to replace it with
Title: Re: VCE English Question Thread
Post by: FatimaEl on October 16, 2016, 11:03:24 pm
Can anyone help me think of some synonyms for moral compass? my teacher said its too overused by other people and am having trouble thinking of what to replace it with
damn :/ didnt know that was overused, i like to use it myself i guess i should be more cautious from now on, you could use 'code of ethics' or 'moral standing',
Title: Re: VCE English Question Thread
Post by: literally lauren on October 18, 2016, 12:43:15 pm
How does the structure of an article persuade readers?
I have a generally good understanding of doing language analysis', except my teacher said I need to focus on the structure used by the writer in attempt to persuade. I didn't get a chance to ask my teacher, so hopefully someone on here can help : )

Hey there!

It's a bit tough to say, since the structure can change from piece to piece, but in general, you can almost always analyse the order of information in the material. For instance, if an author spends, like, five paragraphs talking about how the new pizza shapes are literally the worst thing to ever happen ever, and then says "so what can we do about this gross injustice?" before going on to talk about people should all write letters complaining to Arnotts, then you can analyse his presentation of a problem + solution. This kind of ties in with things like tonal shifts too.

If you get some kind of unconventional style like an interview or a transcript of a podcast or something weird like that (which I think might be likely this year) - you may have to discuss how different speakers interrupt one another, or how one author subtly attempts to dichotomise readers through their structure. But I can't imagine any scenario where that would be a requirement. More likely, it'll be an opportunity for you to analyse, but you could just as easily ignore that, focus on others, and still get a great mark :)

For text response, my teacher has told me that I must not refer to any specific characters in topic sentences; only discussing what the author says through the text. How can I use this approach to respond to character specific prompts?

For instance (for Burial Rites):

"What role does absence play in Agnes' life?" Discuss
Note: you can have characters' names in topic sentences if you wish, though tbh I'm with your teacher on this one; your points will be stronger and more holistic if you focus on key ideas to start off with instead.

e.g. 'Throughout the novel, Kent draws attention to the ways in which the absence of trust and closeness can negatively impact one's state of mind. For instance, when Agnes...' <-- and from there, you can zoom in to a specific character or chunk of evidence :)

Hey,

I was just wondering if there is another word for 'readership' for language analysis, I saw 'circulation" for like newspapers and magazines and stuff could I use that instead so I don't sound so mundane?

Thank you!  :)

I could be wrong, and do correct me if i am, but i would say that circulation more so refers to how widely distributed a newspaper or magazine is, and is not a very close synonym to readership, could you give me the sentence you are writing so i can give you a more appropriate word? 
I would say the word 'demographic' is a more suited replacement :) hope that helped x
Yeah, 'circulation' is a little weird; 'demographic' is good; 'readers,' 'readership,' 'the audience,' 'the target audience,' and 'his/her audience of ____' (e.g. 'his audience of parents,' 'her audience of scientistis' etc.) can all be used interchangeably, and the material may offer you a more specific alternative. For instance, if the exam piece is a special interest magazine aimed at people looking at getting into the property market, you could talk about how an an author is attempting to persuade 'potential first home buyers' instead of just using the generic term 'readers' all the time.

This may be a very obvious question, but i was just wondering are we allowed to use brackets in essays? Ive always avoided using them for some reason, and i do not mean these [ ] types of brackets that can be used when modifying a quote to grammatically suit your sentence, but rather these ( ) type of brackets to include additional information rather than using commas, same goes with - this -, are we allowed to use dashes for extra information? or must we always use commas?
For reasons that I won't go into (because I think they're dumb) the use of brackets is considered somewhat informal in T.R. and L.A. essays. Anything goes in Context, though ::)

If you want to include parenthetical information - like this - then the use of hyphens is preferred. But you'd have to get a reeeeeally fussy assessor to ever get pulled up for this. It'd only be a problem if there were other odd word choices or colloquial phrases scattered throughout your piece, in which case you might lose a mark or two for the quality of your expression. But if you're including necessary/useful information, I doubt any assessor would mind :)

Can anyone help me think of some synonyms for moral compass? my teacher said its too overused by other people and am having trouble thinking of what to replace it with
'sense of morality' 'judgement' 'integrity' or 'probity' might work, depending on the context. But it might also help to be more specific (i.e. are you referring to what's right and wrong, or what's ethically justifiable, or what a person is prioritising, etc.)
Title: Re: VCE English Question Thread
Post by: FallingStar on October 18, 2016, 04:26:53 pm
Hi everyone:

I know this is late for asking this question but for timed writing, do we time up (stopwatch) or time down (countdown timer)?
Title: Re: VCE English Question Thread
Post by: literally lauren on October 18, 2016, 06:34:37 pm
Hi everyone:

I know this is late for asking this question but for timed writing, do we time up (stopwatch) or time down (countdown timer)?
I have it on good authority that 1 hour = 1 hour no matter whether you count up or down ;)

Personally, I'd go for counting up because a countdown clock would probably freak me out and is a little too reminiscent of some kind of impending nuclear meltdown for my taste (T minus 20 minutes till the end of my essay ohmygodOHMYGOD).

Timing 'upwards' also lets you be more aware of intervals (e.g. "at the 20 minute mark, I had my intro and first body paragraph done, but then paragraph two took me 25 minutes :/ --> might need to plan so I know what to cover in each section" or "it took me 15 minutes just to get through the intro, but after that I could write all the B.P.s in under 15 minutes each --> need to work on ways to break down the prompt and get started ASAP.")
Title: Re: VCE English Question Thread
Post by: larissaaa_ on October 18, 2016, 07:08:16 pm
Does anyone have any tips on getting timing right?? I have always struggled with timing; give me 3 hours and I can pull out a killer essay but give me 60 minutes and it all goes downhill. Unless my school marks really easily, I'm not actually bad at english. I've gotten A+ on all SAC's this year, but under the stress and time limits of exam conditions my mind just goes blank and I can never finish a full essay. Tips please?
Title: Re: VCE English Question Thread
Post by: FatimaEl on October 18, 2016, 08:27:47 pm
Does anyone have any tips on getting timing right?? I have always struggled with timing; give me 3 hours and I can pull out a killer essay but give me 60 minutes and it all goes downhill. Unless my school marks really easily, I'm not actually bad at english. I've gotten A+ on all SAC's this year, but under the stress and time limits of exam conditions my mind just goes blank and I can never finish a full essay. Tips please?

idk if that helped, hope it did  :)
Title: Re: VCE English Question Thread
Post by: FatimaEl on October 18, 2016, 08:34:07 pm
hey guys can we ask questions in a text response essay then answer them or is that too informal?
Title: Re: VCE English Question Thread
Post by: goodbye on October 18, 2016, 10:08:58 pm
hey guys can we ask questions in a text response essay then answer them or is that too informal?

As like a rhetorical question sort of thing? If so, then no. You want to be succinct as possible when writing.
Title: Re: VCE English Question Thread
Post by: FatimaEl on October 18, 2016, 10:23:35 pm
As like a rhetorical question sort of thing? If so, then no. You want to be succinct as possible when writing.
Not a rhetorical question, as that sorta answers itself, but a legitimate question that audiences may ask, then answer it? Personally i think it might sound weird but ive seen people in my class do it and theyre high scorers so idk
Title: Re: VCE English Question Thread
Post by: Coffee on October 19, 2016, 07:41:09 am
Not a rhetorical question, as that sorta answers itself, but a legitimate question that audiences may ask, then answer it? Personally i think it might sound weird but ive seen people in my class do it and theyre high scorers so idk

I would say no question marks is the key thing when talking about questions raised in the text. So rather than writing:

[Question]? [Author] suggests...

You would write:

This raises the question of [question]. [Author] suggests...

Which might look like:
This raises the question of whether we are ever truly free from the remnants of our past. [Author] suggests...

Does that make sense? :)
Title: Re: VCE English Question Thread
Post by: michael leahcim on October 19, 2016, 07:54:57 am
Hey I know this is really late to be asking this, but how would you recommend going about doing a text response for poetry? I'm going to do John Donne's poems and I was wondering how much I should talk about his life and times.

Thanks!  :)
Title: Re: VCE English Question Thread
Post by: FatimaEl on October 19, 2016, 09:47:36 am
I would say no question marks is the key thing when talking about questions raised in the text. So rather than writing:

[Question]? [Author] suggests...

You would write:

This raises the question of [question]. [Author] suggests...

Which might look like:
This raises the question of whether we are ever truly free from the remnants of our past. [Author] suggests...

Does that make sense? :)
Yes that makes sense, thank you :)
Title: Re: VCE English Question Thread
Post by: Rob16 on October 19, 2016, 03:12:50 pm
do topic sentences always have to be in one sentence? I'm finding that in a lot of my essays the topic sentence is usually two sentences because I can't seem to succinctly explain my paragraph in one sentence
Title: Re: VCE English Question Thread
Post by: katk11 on October 19, 2016, 09:07:56 pm
Quick question, I've been hearing different things and I am unsure which way to go, but with context, is it best to mention at least one text in every paragraph with it relating to an external example? Or can you write one paragraph on one text, then another on an external example, and so on? Thanks!
Title: Re: VCE English Question Thread
Post by: Daniellac on October 21, 2016, 03:29:58 pm
Hey :))

I was wondering if there are any visual analysis examples for Lang Analysis?

Thank you!! x
Title: Re: VCE English Question Thread
Post by: HopefulLawStudent on October 21, 2016, 04:39:30 pm
Hey :))

I was wondering if there are any visual analysis examples for Lang Analysis?

Thank you!! x

English Resources and Sample Essays > Samples > Language Analysis Samples
Title: Re: VCE English Question Thread
Post by: HopefulLawStudent on October 22, 2016, 12:59:30 pm
Could someone please confirm the following:

- The LA visuals will be black and white, right? And they'll be clear, right? Had a practice exam a few days ago and the printer the teachers used to print it out was legit so low quality that it was a real struggle to make out what the visual even was... (seeking peace of mind more than anything really)

- I hope this isn't the case but god forbid I hit the first hour and have like a paragraph to finish and a conclusion to do. Do I cut it short and work on the others and come back to it if I have the time or do I finish it? Is it better to have a complete essay and compromise the quality of another or an incomplete essay and not compromise the quality of the other two essays?
Title: Re: VCE English Question Thread
Post by: anthonyttran on October 22, 2016, 02:12:33 pm
do topic sentences always have to be in one sentence? I'm finding that in a lot of my essays the topic sentence is usually two sentences because I can't seem to succinctly explain my paragraph in one sentence

I find that it is best to try and succinctly outline the body paragraph idea in one sentence. Although it is not too strict. If you decide on using two sentences just make sure you are not being too wordy and explaining unnecessary ideas.
Title: Re: VCE English Question Thread
Post by: KDB on October 22, 2016, 02:21:09 pm
Could someone please confirm the following:

- The LA visuals will be black and white, right? And they'll be clear, right? Had a practice exam a few days ago and the printer the teachers used to print it out was legit so low quality that it was a real struggle to make out what the visual even was... (seeking peace of mind more than anything really)

- I hope this isn't the case but god forbid I hit the first hour and have like a paragraph to finish and a conclusion to do. Do I cut it short and work on the others and come back to it if I have the time or do I finish it? Is it better to have a complete essay and compromise the quality of another or an incomplete essay and not compromise the quality of the other two essays?

Yes, the L.A. visual(s) will be in black & white, it will be clear and easy to see.

If i assume you'll be doing L.A. first, then you should probably go ahead and finish it as it needs a conclusion (just incase there's no time to go back to it at the end). If it was context, an overall conclusion isn't necessary (you can sort of end them anywhere, but not too randomly) although for Text Response and L.A. they kind of are needed. Just try and aim for a balance between all three sections, keeping an eye on the time!
Title: Re: VCE English Question Thread
Post by: literally lauren on October 22, 2016, 02:26:23 pm
Hey I know this is really late to be asking this, but how would you recommend going about doing a text response for poetry? I'm going to do John Donne's poems and I was wondering how much I should talk about his life and times.

Thanks!  :)
For starters, never limit a paragraph to a single poem if you can avoid it. The big challenge with poetry and story stories is to unite/consolidate the text and talk about how things connect together; conducting three or four separate analyses of three or four poems/stories is more typical of mid-range responses.

Thematic connections are awesome, and if the prompt allows, I'd recommend breaking down the topic into three or four core concepts that you can explore using a variety of poems in each para. As for the author's life and times... try to minimise that stuff. If it's relevant, you can include it, but harking on and on about historical background info kind of just distracts from your focus on the text. But if it is actually aiding your analysis AND you can integrate it with textual analysis, then go for it! (Use the half sentence rule - e.g. 'Donne's preoccupation with metaphysical conceits can be seen in the imagery of 'Valediction' where...' as opposed to: 'Donne was preoccupied with metaphysical conceits. This can be seen in the imagery of 'Valediction' where...' - so your use of background info is visibly connected to some other part of your discussion, rather than having a whole sentence like the underlined one which isn't as relevant to the task).

do topic sentences always have to be in one sentence? I'm finding that in a lot of my essays the topic sentence is usually two sentences because I can't seem to succinctly explain my paragraph in one sentence
You can stretch it out over two, and it can be good to explore your key idea before launching into evidence, though it may also be worth practising your conciseness just so you have the option of quicker, sharper T.S.s if you need them :)

Quick question, I've been hearing different things and I am unsure which way to go, but with context, is it best to mention at least one text in every paragraph with it relating to an external example? Or can you write one paragraph on one text, then another on an external example, and so on? Thanks!

Either of those would be fine! :) It depends how well your set text works with your external examples, but for either scenario, you want to make sure your piece ties together nicely. For instance, you should try and avoid having one paragraph on the text, and then two or three other paragraphs that are entirely unrelated (e.g. 1. 'here's a discussion of a book' 2. 'here's something that happened in the Australian media this year' 3. 'here's something that happened in WWII.' 4. 'here's an anecdote about my experiences.') Ideally, you want some kind of link that ties them together, or some kind of common thread running through your piece, no matter your text+example combination. (e.g. 1. 'here's a discussion of how this novel addresses dealing with change' 2. 'here's a significant change that was going on at the time the text was set/written' 3. 'here are some of the consequences of those changes for today's society' etc.)

Could someone please confirm the following:

- The LA visuals will be black and white, right? And they'll be clear, right? Had a practice exam a few days ago and the printer the teachers used to print it out was legit so low quality that it was a real struggle to make out what the visual even was... (seeking peace of mind more than anything really)

- I hope this isn't the case but god forbid I hit the first hour and have like a paragraph to finish and a conclusion to do. Do I cut it short and work on the others and come back to it if I have the time or do I finish it? Is it better to have a complete essay and compromise the quality of another or an incomplete essay and not compromise the quality of the other two essays?
Visuals will absolutely 100% be black and white unless VCAA splurge on a colour printer this year :P For some reason, the GAT is in colour, but the English exams never are :'( <-- visual art student tears
Printing quality will also be good; the scans on VCAA's site are sometimes kind of dodgy, but the printed ones are always clear enough.

Re: sticking with one essay or jumping to the others, that's up to you. If it's a whole other paragraph you've got to do, I'd maybe go to the others and just ensure you can write enough for each of them, but maybe try and speed things up to ensure you get an extra ten or so minutes at the end. Basically, it's better to have three essays that are worth 9/10 and are all a bit imperfect because you cut each of them short than one that's worth a 10/10 and is utterly immaculate and complete with two others that are only worth 7s because you sacrificed time you should've devoted to those ones. But on the flip side, you might be able to race through the end of a paragraph and a conclusion in about five minutes to save yourself the trouble of coming back and re-acquainting yourself with the material, so you might also decide that's a better option in the heat of the moment. Either would be fine, so don't stress about picking the wrong choice if you're in that situation :)

edit: beaten by Jason :P
Title: Re: VCE English Question Thread
Post by: HopefulLawStudent on October 22, 2016, 02:29:20 pm
Thanks guys! :D
Title: Re: VCE English Question Thread
Post by: megt on October 22, 2016, 02:31:48 pm
Quick Question -
I was wondering if it's considered okay in a Text Response to use inverted commas when it's not a quote? For example, if I want to discuss the author's handling of the construct of 'being' and how it's seen in many different forms, can I speak about the concept of the word and use the ''- or is this too likely to be mistaken for a poorly chosen quote?
Thanks
Title: Re: VCE English Question Thread
Post by: Sine on October 22, 2016, 02:35:10 pm
Does language analysis really need a conclusion? I was under the impression that you could just tie it all up and the end of your last paragraph is it isn't too long.
Title: Re: VCE English Question Thread
Post by: literally lauren on October 22, 2016, 02:38:55 pm
Quick Question -
I was wondering if it's considered okay in a Text Response to use inverted commas when it's not a quote? For example, if I want to discuss the author's handling of the construct of 'being' and how it's seen in many different forms, can I speak about the concept of the word and use the ''- or is this too likely to be mistaken for a poorly chosen quote?
Thanks
Hmm, this is a tricky one. Is there a better word you could use to side step the ambiguity? Like, 'the author's construction of the characters' sense of self' or 'the author's depiction of consciousness as a construct'?

Alternatively, ensure you're always using "double quotation marks" when actually quoting, and you should be fine to use 'single ones' for demarcating things like this :)

Does language analysis really need a conclusion? I was under the impression that you could just tie it all up and the end of your last paragraph is it isn't too long.
It's a "structural requirement," meaning that it's better to include one to be on the safe side. It's not as necessary as it is in a T.R. essay, but conclusions in L.A. only take about two or three sentences, and it means you'll win over the super fussy assessors, so if you do a cost-benefit analysis, the extra two minutes it takes you to whip a quick concl. together probably outweighs the potential for a marker to dock a point for a missing conclusion.

Though what's more likely is that if you were:
a) missing a conclusion
b) repeating some vocab
c) only doing the bare minimum of visual analysis
...then you'd lose a mark. If everything else was perfect, I highly doubt they'd give an L.A. piece a 9 instead of a 10 just because it didn't have a conclusion. But if there were other faults that they'd picked up on, a missing conclusion might be what bumps you down a point.
Title: Re: VCE English Question Thread
Post by: HopefulLawStudent on October 22, 2016, 02:42:07 pm
Hmm, this is a tricky one. Is there a better word you could use to side step the ambiguity? Like, 'the author's construction of the characters' sense of self' or 'the author's depiction of consciousness as a construct'?

Alternatively, ensure you're always using "double quotation marks" when actually quoting, and you should be fine to use 'single ones' for demarcating things like this :)
It's a "structural requirement," meaning that it's better to include one to be on the safe side. It's not as necessary as it is in a T.R. essay, but conclusions in L.A. only take about two or three sentences, and it means you'll win over the super fussy assessors, so if you do a cost-benefit analysis, the extra two minutes it takes you to whip a quick concl. together probably outweighs the potential for a marker to dock a point for a missing conclusion.

Though what's more likely is that if you were:
a) missing a conclusion
b) repeating some vocab
c) only doing the bare minimum of visual analysis
...then you'd lose a mark. If everything else was perfect, I highly doubt they'd give an L.A. piece a 9 instead of a 10 just because it didn't have a conclusion. But if there were other faults that they'd picked up on, a missing conclusion might be what bumps you down a point.

Should we be analysing stuff in our conclusions?
Title: Re: VCE English Question Thread
Post by: literally lauren on October 22, 2016, 02:45:25 pm
Should we be analysing stuff in our conclusions?
No need; a quick summation of the overarching contentions and intentions of the author(s) and the effect on readers or the consequences they're seeking to bring about would be sufficient. You can do a really brief 'By doing X, the author seeks to Y' sentence at the start, but even that's more summative than analytical :)
Title: Re: VCE English Question Thread
Post by: HopefulLawStudent on October 22, 2016, 02:47:34 pm
No need; a quick summation of the overarching contentions and intentions of the author(s) and the effect on readers or the consequences they're seeking to bring about would be sufficient. You can do a really brief 'By doing X, the author seeks to Y' sentence at the start, but even that's more summative than analytical :)

Cheers Lauren! :D
Title: Re: VCE English Question Thread
Post by: scandin9 on October 22, 2016, 02:57:41 pm
Hi Lauren,

For a creative context piece what would typically distinguish an 8 from a 9 or 10 if your language is already very sophisticated?
Title: Re: VCE English Question Thread
Post by: literally lauren on October 22, 2016, 03:14:58 pm
Hi Lauren,

For a creative context piece what would typically distinguish an 8 from a 9 or 10 if your language is already very sophisticated?
Creative piece scoring in the 7-8 range are usually the ones that take the prompt (e.g. 'Our identity depends on where we choose to belong') and then construct a narrative or imaginary scenario around that idea (e.g. here's a short story about someone whose identity depends on where they choose to belong). And they'll do that pretty effectively, but they won't be saying anything about the prompt other than, yes and here's an example of that.

The 9s and 10s are the ones that actually explore the implications of a prompt (e.g. our identity can depend on where we choose to belong because we can consciously shape who we are through the power of our decisions, but only if we have the courage of our convictions and stick by those decisions, and the groups that we belong to can play a big role in this) and structure their story around an actual contention, not just the core of the prompt. Beyond that, language can sometimes be a separating factor, but that's a more minor one. The quality of ideas, and how 'multi-dimensional' your creative piece is will usually determine the mark. (i.e. are you only suggesting a single concept over the course of the whole narrative, or are you fleshing out a variety of ideas and questions?)
Title: Re: VCE English Question Thread
Post by: michael leahcim on October 22, 2016, 04:14:44 pm
For starters, never limit a paragraph to a single poem if you can avoid it. The big challenge with poetry and story stories is to unite/consolidate the text and talk about how things connect together; conducting three or four separate analyses of three or four poems/stories is more typical of mid-range responses.

Thematic connections are awesome, and if the prompt allows, I'd recommend breaking down the topic into three or four core concepts that you can explore using a variety of poems in each para. As for the author's life and times... try to minimise that stuff. If it's relevant, you can include it, but harking on and on about historical background info kind of just distracts from your focus on the text. But if it is actually aiding your analysis AND you can integrate it with textual analysis, then go for it! (Use the half sentence rule - e.g. 'Donne's preoccupation with metaphysical conceits can be seen in the imagery of 'Valediction' where...' as opposed to: 'Donne was preoccupied with metaphysical conceits. This can be seen in the imagery of 'Valediction' where...' - so your use of background info is visibly connected to some other part of your discussion, rather than having a whole sentence like the underlined one which isn't as relevant to the task).
You can stretch it out over two, and it can be good to explore your key idea before launching into evidence, though it may also be worth practising your conciseness just so you have the option of quicker, sharper T.S.s if you need them :)

Hi Lauren! Thanks for getting back to me :) I have one more question concerning poems, I have a little trouble with discerning between ideas and splitting them into mini-contentions for my body paragraphs. For example in the prompt 'How does Donne convey an appreciation of what love can do in his Selected Poems?', my intro would look something like this:

In his collection Selected Poems, John Donne conveys the concept of love with an extraordinary sensitivity that breathes life into it, exploring its "infinite" dimensions. In his world, Donne views love as a transformative and powerful force that overcomes all physical and rational boundaries. He displays love as a liberating force as well as a transcending kind of power. However, he also shows love as a source of pain, restriction and sorrow. But it is in Donne’s world of love, a world so interconnected, that his powerful descriptions of love seem to capture an essence of life, brimming so full of substance, mystery and beauty.

I've come up with some topic sentences but they seem to be like a lit essay and I'm not sure how to recify it :S i.e.

BP1: Through showing love as a transformative force, Donne conveys love's ever-changing state. --> eg. love connects you and therefore liberates
BP2: Donne's vision of love as an anomaly highlights his preoccupation with love's many dimensions and realities. --> eg. love is mysterious; it is fleeting and opens up a world of infinite possibilities.
BP3: Throughout his poems, Donne shows love as a force which brings together many of the facets of life's uncertainties, and propounds an interconnected cosmos brimming with beauty and profound meaning. ---> eg. life is unpredictable and love is unrehearsed -- he paints this with extraordinary vision/imagery.

But they kinda all blend together and it makes it hard for me to split them up without repeating myself. How would you recommend splitting it up so that I would have some straight-forward topic sentences? Sorry if this is a long question, but my school is terrible (cry) and I'm so stressed and have nobody :'(

Thanks in advance. You're the best!
Title: Re: VCE English Question Thread
Post by: NerdyPi on October 22, 2016, 04:19:15 pm
Hi guys, this is a very text specific question, as I'm trying to prepare for all possible Medea prompt.

I'm not completely sure why Jason is described as weak. My best ideas are

-He's desperate for power and social status
-He cannot/does not protect his family as a father/husband should

Any other ideas would be appreciated, thanks :)
Title: Re: VCE English Question Thread
Post by: Swagadaktal on October 22, 2016, 05:06:56 pm
Hi guys, this is a very text specific question, as I'm trying to prepare for all possible Medea prompt.

I'm not completely sure why Jason is described as weak. My best ideas are

-He's desperate for power and social status
-He cannot/does not protect his family as a father/husband should

Any other ideas would be appreciated, thanks :)
He becomes a little bitch is emasculated at the end of the play because of his desire to bury and weep over his children - this was considered extremely unmanly  and weak coz men were not supposed to feel that attached to them (they viewed children as a way to pass on their legacy and not  actual love)

He can also be considered weak because he can't even use sophistry right to persuade others. He failed utterly and he's a complete moron (mostly in the eyes of Medea but this sentiment is shared to an extent)

Title: Re: VCE English Question Thread
Post by: MandhreeE on October 22, 2016, 05:53:43 pm
Hi guys,

I just really need some clarification on Language analysis. i have some teachers telling me to use labels for specific technique in LA and then i have other teacher telling me to avoid them completely and instead use phrases that refer to the technique without explicitly saying it. I'm really confused and i thought the examiner's  report said that students who do not rely on labels generally do better.
So for example one teacher says that in the analysis if there is an appeal to a sense of justice i have to say "the writer appeals to readers sense of justice to encourage them to feel...." and another teacher tells me to say "the writer uses readers desire for people to be treated equally and fairly to encourage them to feel...". Which one should i do?
thank you!!!
Title: Re: VCE English Question Thread
Post by: BLTCC on October 22, 2016, 06:25:39 pm
Hi, I hope I'm asking this question in the right place and if I am not sorry in advance and sorry if this question has already been asked. With text response, is it okay to write character based paragraphs, for example, with Medea, one paragraph be on Medea herself then Jason, or does this limit the sophistication of the piece?? Thank you  :)
Title: Re: VCE English Question Thread
Post by: literally lauren on October 22, 2016, 06:51:58 pm
Hi Lauren! Thanks for getting back to me :) I have one more question concerning poems, I have a little trouble with discerning between ideas and splitting them into mini-contentions for my body paragraphs. For example in the prompt 'How does Donne convey an appreciation of what love can do in his Selected Poems?', my intro would look something like this:

In his collection Selected Poems, John Donne conveys the concept of love with an extraordinary sensitivity that breathes life into it, exploring its "infinite" dimensions. In his world, Donne views love as a transformative and powerful force that overcomes all physical and rational boundaries. He displays love as a liberating force as well as a transcending kind of power. However, he also shows love as a source of pain, restriction and sorrow. But it is in Donne’s world of love, a world so interconnected, that his powerful descriptions of love seem to capture an essence of life, brimming so full of substance, mystery and beauty.

I've come up with some topic sentences but they seem to be like a lit essay and I'm not sure how to recify it :S i.e.

BP1: Through showing love as a transformative force, Donne conveys love's ever-changing state. --> eg. love connects you and therefore liberates
BP2: Donne's vision of love as an anomaly highlights his preoccupation with love's many dimensions and realities. --> eg. love is mysterious; it is fleeting and opens up a world of infinite possibilities.
BP3: Throughout his poems, Donne shows love as a force which brings together many of the facets of life's uncertainties, and propounds an interconnected cosmos brimming with beauty and profound meaning. ---> eg. life is unpredictable and love is unrehearsed -- he paints this with extraordinary vision/imagery.

But they kinda all blend together and it makes it hard for me to split them up without repeating myself. How would you recommend splitting it up so that I would have some straight-forward topic sentences? Sorry if this is a long question, but my school is terrible (cry) and I'm so stressed and have nobody :'(

Thanks in advance. You're the best!

Those topic sentences seem excellent to me - is your concern that you'll end up repeating yourself within your body paragraphs, or that the topic sentences themselves are too repetitious? Because I certainly don't think the latter is the case (-when a prompt is as broad as 'Discuss love,' it can be kind of hard to move away from that central concern-) and the former could easily be avoided by ensuring you're delving into a wide variety of different poems in each para.

In general, so long as each paragraph has a self-contained point, as well as linking together and forming an overarching contention, you should be all good :)

Hi guys,

I just really need some clarification on Language analysis. i have some teachers telling me to use labels for specific technique in LA and then i have other teacher telling me to avoid them completely and instead use phrases that refer to the technique without explicitly saying it. I'm really confused and i thought the examiner's  report said that students who do not rely on labels generally do better.
So for example one teacher says that in the analysis if there is an appeal to a sense of justice i have to say "the writer appeals to readers sense of justice to encourage them to feel...." and another teacher tells me to say "the writer uses readers desire for people to be treated equally and fairly to encourage them to feel...". Which one should i do?
thank you!!!
This is kind of tricky, but I'd say you should endeavour to use those labels and then explain their meaning/significance e.g. 'the writer's appeal to justice is evident in his use of the phrase "everyone deserves a second chance," suggesting that people should be treated equally and fairly in order to encourage readers to feel...'

The 'don't use labels' rule stems from the fact that so many low/mid-range essays will simply start and end their analysis by pointing out a device (e.g. the author uses a rhetorical question "how can they do this?" He also uses inclusive language like "we" and emotive language to persuade readers of his contention) <-- stuff like that is waaaay too generic. If you can zoom in on particular language features and comment on how they're being used to persuade, then the technique labelling is a secondary concern. If it helps, think of it like a stepping stone for your analysis - it's often a good place to start, but it's only ever a starting point.

But there's also no reason why you'd have to do the exact same things every time. Rather, you could mention three or four official 'techniques' in a paragraph but intersperse these with other points of analysis where you're just unpacking connotations or talking about the general persuasiveness of certain words/phrases :)

Hi, I hope I'm asking this question in the right place and if I am not sorry in advance and sorry if this question has already been asked. With text response, is it okay to write character based paragraphs, for example, with Medea, one paragraph be on Medea herself then Jason, or does this limit the sophistication of the piece?? Thank you  :)
You're absolutely in the right place! :)

Character-based paragraphs are permissible, and it's what I tend to recommend as a last-resort/back-up option for really difficult prompts or if you're short on time and feeling stuck. But yes, imo it does limit sophistication because the whole piece invariably turns into 'here's how the prompt relates to Character X. And Y. And Z. The end.'

Instead, aim for a thematic breakdown

For example, if your prompt was 'Medea shows the dangers of desire.'

A bad essay would just be like:
- Jason desires power.
- Medea desires revenge
- The chorus desire justice (~debatable, but w/e)

A good essay might look like:
- the characters become blinded by their own desires and lose sight of potential ramifications
- obsessing over desires can obfuscate the intentions of others and open characters up to exploitation
- even desires that are unfulfilled can corrupt characters' minds

^that's by no means the only breakdown here - any kind of separation of core concepts would do!

Hope that helps :)
Title: Re: VCE English Question Thread
Post by: MandhreeE on October 22, 2016, 07:19:04 pm
Hi Lauren,

Thank you so much for your prompt reply!! I hope you don't mind me asking but i'm also getting mixed marks for my context pieces. I know this is a huge ask but i was wondering if i could perhaps send you one of my context pieces for you to mark. it's totally fine if you can't i know that this time of year you are super busy.

Thank you once again.
Title: Re: VCE English Question Thread
Post by: BLTCC on October 22, 2016, 07:39:50 pm
Thank you so much Lauren for your help really appreciate it!!!!!
Title: Re: VCE English Question Thread
Post by: blacksanta62 on October 22, 2016, 08:02:23 pm
Hey guys, if writing a context piece (expository and I&B) and I can only weed out 2 main ideas that I want to explore can I still get a 7-8/10?
So in total that would be an introduction, 2 body paragraphs and a conclusion. Or is it really recommended that the 3 paragraphs minimum rule be followed?
Thank you
Title: Re: VCE English Question Thread
Post by: Koipond11111111111 on October 22, 2016, 09:27:32 pm
How would you structure an essay on this Medea practice exam prompt "How does Medea deceive other characters in the play?"
Do we incorporate a discussion on themes into essay or is it strictly about how the text was constructed?

Do you think VCAA will put more than two different written pieces for LA like in your practice exams? I am having trouble analyzing  the comments in these pieces (e.g. The 3 comments on the blog in Exam 4) in LA, should I put the contention of every comment in the introduction?
Title: Re: VCE English Question Thread
Post by: literally lauren on October 23, 2016, 12:10:34 pm
Hi Lauren,

Thank you so much for your prompt reply!! I hope you don't mind me asking but i'm also getting mixed marks for my context pieces. I know this is a huge ask but i was wondering if i could perhaps send you one of my context pieces for you to mark. it's totally fine if you can't i know that this time of year you are super busy.

Thank you once again.

No worries :) Feel free to chuck your piece up on the Submissions Board if you want really detailed feedback; otherwise, you can PM me and I'll give you some general advice :)

Hey guys, if writing a context piece (expository and I&B) and I can only weed out 2 main ideas that I want to explore can I still get a 7-8/10?
So in total that would be an introduction, 2 body paragraphs and a conclusion. Or is it really recommended that the 3 paragraphs minimum rule be followed?
Thank you
"2 ideas" doesn't necessarily have to be two body paragraphs though. Even if you can only extract two main concepts from the prompt, you could still break them up into three or more paragraphs.

Three paragraphs would be the ideal minimum, though the rules are more flexible for Context, so if you only had two paras but your discussion was excellent, you could easily score in the 7-8 region, though it might be hard to bump it to a 9-10.

How would you structure an essay on this Medea practice exam prompt "How does Medea deceive other characters in the play?"
Do we incorporate a discussion on themes into essay or is it strictly about how the text was constructed?

Do you think VCAA will put more than two different written pieces for LA like in your practice exams? I am having trouble analyzing  the comments in these pieces (e.g. The 3 comments on the blog in Exam 4) in LA, should I put the contention of every comment in the introduction?
Basically, your body paragraphs should always be structured around themes (if possible... otherwise just go for a character breakdown as a last resort if you need to) and then within those paragraphs, you can delve into the construction of the text for 'How...' prompts.

I think a comparative exam is likely this year, but I reckon there's also a high chance of them giving you something weird like an interview or a transcript of some description, which is why I tend to make the practice exams comparative out of habit. Maybe they'll be nice and just give you a single, straightforward piece this year, but I think they'll be looking to challenge you :P In the even you get given a bunch of different bits and pieces, you wouldn't have to go through them all in the intro. For instance, for Exam 3, I'd just have 2-3 sentences at the start talking about how the newsletter published pieces from a variety of teachers about the different language options (rather than running through all four pieces.) For Exam 4, on the other hand, your intro should contain the contention of the main piece since that's taking up around 3/4 of the material. The comments are more optional though, and it'd be more efficient to just leave them for your body paragraphs where you can delve into them properly. If you want, you can just have a quick sentence at the end of your intro like 'The piece was also accompanied by a variety of comments spanning different views from members of the forum community' and leave it at that :)
Title: Re: VCE English Question Thread
Post by: blacksanta62 on October 23, 2016, 03:13:37 pm
"2 ideas" doesn't necessarily have to be two body paragraphs though. Even if you can only extract two main concepts from the prompt, you could still break them up into three or more paragraphs.

Three paragraphs would be the ideal minimum, though the rules are more flexible for Context, so if you only had two paras but your discussion was excellent, you could easily score in the 7-8 region, though it might be hard to bump it to a 9-10.
How would I do that?
And this doesn't happen with VCAA prompts only with prompts from company papers which are worded difficulty.

Also, when doing a LA, does the audience have to be mentioned in the introduction?

Thank you
Title: Re: VCE English Question Thread
Post by: Hydraulix on October 23, 2016, 07:06:51 pm
For those prompts that are like: This Text is about something

like 2014 VCAA prompt for This Boy's Life
This Boy's Life is not only about hardship; it is also about determination and resourcefulness.

How do you go about your paragraphs. To me it sounds bad to just say yeah it is about that because of [example]. What is the good way to approach one of these prompts?

Thank youu!
Title: Re: VCE English Question Thread
Post by: HasibA on October 23, 2016, 07:59:56 pm
quick q
with the time remaining, would it be pragmatic to do the VCAA lang analyses and look through there prompts, or should i look through Lauren's exams? leaning over to lauren's bc VCAA probs wont repeat themselves, and its better to expose myself to new content, but still, not sure
thanks :)
Title: Re: VCE English Question Thread
Post by: FallingStar on October 23, 2016, 08:03:48 pm
How would I do that?
And this doesn't happen with VCAA prompts only with prompts from company papers which are worded difficulty.

Also, when doing a LA, does the audience have to be mentioned in the introduction?

Thank you

Method 1: Say you can only do two ideas from a prompt, which are apples and bananas. But the text may say two different things about Apples and two different things about oranges. Write a paragraph about what the text both the different things said about Apples, and then 2 different things about what the text says oranges. You now have 4 paragraphs.

Method 2: You have a prompt, which you only have 2 ideas for. Each of these two ideas can be split into categories. Back to the apples and oranges analogy, your apple idea has green apple and red apple.  Your oranges idea has navel orange and blood oranges. Now write about green apple, red apple, navel oranges and blood oranges. You now have 4 paragraphs.

And yes, you do have to mention the audience. And you should do it in your introduction. Note: The audience has to be specific and never "the general public."

For those prompts that are like: This Text is about something

like 2014 VCAA prompt for This Boy's Life
This Boy's Life is not only about hardship; it is also about determination and resourcefulness.

How do you go about your paragraphs. To me it sounds bad to just say yeah it is about that because of [example]. What is the good way to approach one of these prompts?

Thank youu!

Don't. If you want to give a mostly yes answer, talk about what it means for the author, and why the author argues that. For example, (using All About Eve) If a prompt say that "All About Eve is All About Ambition in the theatre," then I could use an all yes answer, then talk about what the text actually says about ambition. Eg. I could say in one paragraph that the film criticises ambition for being destructive.

For your particular prompt: you have three key words. It is very common that a students like us would fall in the trap of only talking about one or two without talking about the third. May sure you address all of the words. And since they are grouped together, you will also need to talk about the link between the themes in the text. Like how they interconnect.

quick q
with the time remaining, would it be pragmatic to do the VCAA lang analyses and look through there prompts, or should i look through Lauren's exams? leaning over to lauren's bc VCAA probs wont repeat themselves, and its better to expose myself to new content, but still, not sure
thanks :)

Do the VCAA ones to get used to their style of them. Although I doubt they will repeat themselves, they do have a general pattern or style in which they run. And it's not like those commercial papers are going to be able to predict what VCAA would give you either, so I think its best to do a range of language analysis stuff. On the other hand, Lauren knows what she's on about but her exams (if you've seen them) are really tough (they're meant to be). In fact, some high range students in my school found them hard so you may find them really hard. But they are worth doing if you have the time.

I hope these help you all. (sorry that I'm not Lauren  :P)
Title: Re: VCE English Question Thread
Post by: Koipond11111111111 on October 23, 2016, 08:38:08 pm
Can I write a context piece for a text that we did not study in school but is still a VCAA text for Whose reality?
Title: Re: VCE English Question Thread
Post by: FallingStar on October 23, 2016, 09:22:31 pm
Can I write a context piece for a text that we did not study in school but is still a VCAA text for Whose reality?

Technically, you can. Examiner don't know what text you're doing in school. However, they will not count as external resources and you do need external resources. However I don't know if that's a good idea unless you can twist your text into any prompt. :)
Title: Re: VCE English Question Thread
Post by: HasibA on October 23, 2016, 10:52:44 pm
with last yrs prompt for MABO, what other ideas could i generate? 
‘Eddie Mabo is not the only hero of this fi lm.’
 Do you agree?
1) obviously, i could talk about bonita, but the rest of my ideas seem either shallow as hell, or irrelevant.
2) i was thinking the whole murray community is, for dealing with the oppression and the very ignorant views and values in the 1900s
3) Mabos lawyers that assisted him in his case, as well as the relatively unbiased HIgh Court judges?
and then i had some deep thought, and was contemplating ideas like
-maybe MABO is the only true hero, and what it means to be a hero etc.

any help will be lovely, <3
Title: Re: VCE English Question Thread
Post by: Sine on October 23, 2016, 11:22:31 pm
Do essays get remarked if one examiner gives you for example a 7 and the other gives your a 9? (difference of 2)

EDIT: mistakenly quoted  :P
Title: Re: VCE English Question Thread
Post by: Koipond11111111111 on October 24, 2016, 03:46:53 am
Technically, you can. Examiner don't know what text you're doing in school. However, they will not count as external resources and you do need external resources. However I don't know if that's a good idea unless you can twist your text into any prompt. :)
So if I have a really good piece on the American dream related to The Death of A Salesman but we studied Wag the Dog and Foe can I still write it and not get penalized?

Do essays get remarked if one examiner gives you for example a 7 and the other gives your a 9? (difference of 2)

EDIT: mistakenly quoted  :P
Check this out: Re: English Q&A
Title: Re: VCE English Question Thread
Post by: Hydraulix on October 24, 2016, 08:46:48 am
Hiiii

I want to use quotes from my text for my context piece but I do not ever explicitly mention my text. Should I put quotation marks: "like this" when I quote so it is clear that it came from my text? Will the exam markers be cool with it?

Thank youuu
Title: Re: VCE English Question Thread
Post by: kimmytaaa on October 24, 2016, 09:12:48 am
Hi
For language analysis, I asked my teacher if we are comparing it because throughout the whole year we have been on writing one pieces. So when I asked my teacher do we have to compare our pieces if there was 2 article, my teacher said 'it all depends on what is presented in the exam....nobody knows. You do not have to compare only offer a linking sentence (if more than one piece is offered) like we practiced in class.' Is this correct or wrong?
Title: Re: VCE English Question Thread
Post by: HopefulLawStudent on October 24, 2016, 09:43:11 am
Hi
For language analysis, I asked my teacher if we are comparing it because throughout the whole year we have been on writing one pieces. So when I asked my teacher do we have to compare our pieces if there was 2 article, my teacher said 'it all depends on what is presented in the exam....nobody knows. You do not have to compare only offer a linking sentence (if more than one piece is offered) like we practiced in class.' Is this correct or wrong?

No marks directly awarded for comparison/there isn't a criteria that says you need to compare. We just tend to do it every now and then because it allows for a more coherent piece.

tl;dr: You don't need to.
Title: Re: VCE English Question Thread
Post by: FallingStar on October 24, 2016, 10:43:03 am
Hiiii

I want to use quotes from my text for my context piece but I do not ever explicitly mention my text. Should I put quotation marks: "like this" when I quote so it is clear that it came from my text? Will the exam markers be cool with it?

Thank youuu

Yes you should. Examiners do look for quotation marks to ensure that you are putting your quotes in. However, you do need to make obvious that your are reffing to the text as quotation marks can refer to any other quote you've learn't throughout the text. Eg. Biff calls Willy a "fake" and "phoney." (note that I haven't explicitly said it was from Death of a Salesman but have made it obvious that it's from that text. That's what you need to do).
Title: Re: VCE English Question Thread
Post by: HopefulLawStudent on October 24, 2016, 11:01:28 am
Eve Harrington is egotistical. Discuss.

a) This is waaaay too specific to be VCAA-esque, right?
b) How is Eve egotistical anyway?

Quote
How do women in the film defy/adhere to the 1950s feminine idea?

How would I qualify/challenge this prompt? How would I even break down this prompt?

Quote
In order to succeed in the theatre, the only values that count are egotism and ruthless ambition.

If I broke it down like this:
1. Egotism is a trait of the theatre; can lead to success.
2. But it doesn't necessarily = success
3. Ambition is also a thing in the theatre; can lead to success.
4. But it doesn't necessarily = success either.
Conclusion: Ultimately, egotism and ambition don't necessarily = success; they are not the "only values that count". Interpersonal relationships count more, as shown through M

^^Would that count as sufficient exploration of the prompt?
Title: Re: VCE English Question Thread
Post by: FallingStar on October 24, 2016, 11:27:39 am
So if I have a really good piece on the American dream related to The Death of A Salesman but we studied Wag the Dog and Foe can I still write it and not get penalized?

You can. As I said, you never know which examiner your gonna get, and they don't even know which school you are from, nor do they know what text you are studying for school. So if they don't know that, you can't get penalised for it. Please make an effort to know Death of a Salesman very well though - you should be able to twist the prompt you get into Death of a Salesman, but be able to do so to Wag the Dog and Foe too.
Title: Re: VCE English Question Thread
Post by: Hydraulix on October 24, 2016, 11:52:13 am
Yes you should. Examiners do look for quotation marks to ensure that you are putting your quotes in. However, you do need to make obvious that your are reffing to the text as quotation marks can refer to any other quote you've learn't throughout the text. Eg. Biff calls Willy a "fake" and "phoney." (note that I haven't explicitly said it was from Death of a Salesman but have made it obvious that it's from that text. That's what you need to do).

Hmm wait I'm a bit confused. I'll let you know my example and you can tell me if it's not obvious:

The piece that I write on the exam may be a speech by someone that grew up in South Africa apartheid (for Whose Reality - Text: Foe)

I wouldn't use any of the characters or mention the novel as that would seem random. But within the speech where I am discussing the prompt can I drop a key quote of the text like: "their trade is in books, not in truth" as if my speaker is saying them. If this was a true speech it would be weird to put quotation marks but I want to so it is obvious.

I don't get what you mean when you say

Quote
that your are reffing to the text as quotation marks can refer to any other quote you've learn't throughout the text.

Thankkssss youuuu for the helpp :)
Title: Re: VCE English Question Thread
Post by: Hydraulix on October 24, 2016, 01:20:26 pm
Okay another question. Eek!

For the prompt: Medea is the true victim in Euripides' play. Do you agree?
I want to do these body paragraphs

BP1 - From the outset of the play Medea is portrayed as the victim

BP2 - As the play progressed Medea's manipulative nature towards the people that care for her, namedly the Nurse and the Chorus, show that Medea is not a true victim

BP3 - She goes on to use her "evil skills to make her enemies a victim of her wrath

BP4 - Thus the true victim of the play cannot be Medea but is in fact the sons

What do you guys think of this approach? I'm more worried about the last paragraph... will that be considered off topic?

Thanks peeps!!
Title: Re: VCE English Question Thread
Post by: HopefulLawStudent on October 24, 2016, 01:33:17 pm
Okay another question. Eek!

For the prompt: Medea is the true victim in Euripides' play. Do you agree?
I want to do these body paragraphs

BP1 - From the outset of the play Medea is portrayed as the victim

BP2 - As the play progressed Medea's manipulative nature towards the people that care for her, namedly the Nurse and the Chorus, show that Medea is not a true victim

BP3 - She goes on to use her "evil skill[ s] to make her enemies a victim of her wrath"

BP4 - Thus the true victim of the play cannot be Medea but is in fact the sons

What do you guys think of this approach? I'm more worried about the last paragraph... will that be considered off topic?

Thanks peeps!!

Warning: I haven't touched Medea since Unit 3.

Be careful though cos it sounds like you're essentially planning on writing an essay that goes:

Medea is a victim.
Medea is a victim.
Wait no she isn't a victim.
Actually her sons are the victims.

^^ That wouldn't be coherent at all and you'd essentially just be contradicting yourself.**

**Could just be how you've worded it tho.

Medea is the true victim in Euripides' play. Do you agree?

I'd probably do something more like:
1. Medea presents herself as the victim at the beginning of the play.
2. And to some extent, yeah, she is the victim, due to the actions of Jason and the patriarchal world she lives in.
3. But her status as a victim is undermined by her actions over the course of the play.
4. Thus, ultimately, the true victim of the play is not Medea, it is her sons.

Can you see the difference?
Title: Re: VCE English Question Thread
Post by: Hydraulix on October 24, 2016, 01:39:30 pm
Warning: I haven't touched Medea since Unit 3.

Be careful though cos it sounds like you're essentially planning on writing an essay that goes:

Medea is a victim.
Medea is a victim.
Wait no she isn't a victim.
Actually her sons are the victims.

^^ That wouldn't be coherent at all and you'd essentially just be contradicting yourself.**

**Could just be how you've worded it tho.

Medea is the true victim in Euripides' play. Do you agree?

I'd probably do something more like:
1. Medea presents herself as the victim at the beginning of the play.
2. And to some extent, yeah, she is the victim, due to the actions of Jason and the patriarchal world she lives in.
3. But her status as a victim is undermined by her actions over the course of the play.
4. Thus, ultimately, the true victim of the play is not Medea, it is her sons.

Can you see the difference?

Oh wow that is beautiful
My miind is blown thank you thank you!!!
Title: Re: VCE English Question Thread
Post by: HasibA on October 24, 2016, 01:42:27 pm
with regards to films in text response, it is advisable to prefer/use more cinematic , film based techniques that the author used (not being prompt specific) or stick with regular examples/quotes? ive been told its better to focus on the film techniques for films.. but i am not sure.. ty!
Title: Re: VCE English Question Thread
Post by: Swagadaktal on October 24, 2016, 01:43:16 pm
Oh wow that is beautiful
My miind is blown thank you thank you!!!
Yo Just keep in mind that you can't just base your essay on the main bitch. You got other hoes you gotta discuss

Title: Re: VCE English Question Thread
Post by: blacksanta62 on October 24, 2016, 02:04:50 pm
Can someone provide a strong way to integrate the author's views and values on certain themes?
Thank you :)
Title: Re: VCE English Question Thread
Post by: Hydraulix on October 24, 2016, 02:07:23 pm
Yo Just keep in mind that you can't just base your essay on the main bitch. You got other hoes you gotta discuss

I like how you put things haha

But I was worried that if I talk about other hoes then it would look like I'm going off topic. How do I discuss other hoes while remaining on topic? Orrr am I just wrong about the whole off topic thing

Thank youu

Title: Re: VCE English Question Thread
Post by: HopefulLawStudent on October 24, 2016, 02:11:55 pm
Yo Just keep in mind that you can't just base your essay on the main bitch. You got other hoes you gotta discuss

+100

 ;D ;D ;D
Title: Re: VCE English Question Thread
Post by: HopefulLawStudent on October 24, 2016, 02:18:47 pm
I like how you put things haha

But I was worried that if I talk about other hoes then it would look like I'm going off topic. How do I discuss other hoes while remaining on topic? Orrr am I just wrong about the whole off topic thing

Thank youu

You have more leeway for prompts like that.

But I think, so long as you mention the other characters you should be set.

e.g.:

1. Medea presents herself as the victim at the beginning of the play.
wailing to the Chorus

2. And to some extent, yeah, she is the victim, due to the actions of Jason and the patriarchal world she lives in.
Talk about Jason's actions here, plus maybe the actions of Aegeus to exile her despite the fact that she's done nothing wrong

3. But her status as a victim is undermined by her actions over the course of the play.
By her actions onto Jason, Glauce, Aegeus

4. Thus, ultimately, the true victim of the play is not Medea, it is her sons.
Talk about M and sons

^^ You wouldn't just be talking about Medea in 1000 words using that.

I think that that counts as "not basing your essay on the main bitch" ahaha

Title: Re: VCE English Question Thread
Post by: Swagadaktal on October 24, 2016, 02:57:23 pm
You have more leeway for prompts like that.

But I think, so long as you mention the other characters you should be set.

e.g.:

1. Medea presents herself as the victim at the beginning of the play.
wailing to the Chorus

2. And to some extent, yeah, she is the victim, due to the actions of Jason and the patriarchal world she lives in.
Talk about Jason's actions here, plus maybe the actions of Aegeus to exile her despite the fact that she's done nothing wrong

3. But her status as a victim is undermined by her actions over the course of the play.
By her actions onto Jason, Glauce, Aegeus

4. Thus, ultimately, the true victim of the play is not Medea, it is her sons.
Talk about M and sons

^^ You wouldn't just be talking about Medea in 1000 words using that.

I think that that counts as "not basing your essay on the main bitch" ahaha
Aha, yeah that's one way to do it.
I'd frame your arguments in a diff way though, coz the topic is a "do you agree" so you can challenge it.
i'd go 1. Medea suffers at the hands of men and their pursuit of reputation or w/e
2. As a result, Medea is corners and ends up causing suffering upon others (my T.S would be broader than that)
3. Ultimately, It's the men who made Medea suffer who ended up being the victims(worded more nicely)

Do you see how this allows for a discussion about a different person entirely? So all the side hoes get a mention, - coz all though they aint the main bitch they are still valuable hoes to the main hoe you feel? Like if i had to class them I'd go with 'partial main hoes' coz they are significant in the play.
Title: Re: VCE English Question Thread
Post by: literally lauren on October 24, 2016, 03:09:31 pm
Eve Harrington is egotistical. Discuss.

a) This is waaaay too specific to be VCAA-esque, right?
b) How is Eve egotistical anyway?

How would I qualify/challenge this prompt? How would I even break down this prompt?

If I broke it down like this:
1. Egotism is a trait of the theatre; can lead to success.
2. But it doesn't necessarily = success
3. Ambition is also a thing in the theatre; can lead to success.
4. But it doesn't necessarily = success either.
Conclusion: Ultimately, egotism and ambition don't necessarily = success; they are not the "only values that count". Interpersonal relationships count more, as shown through M

^^Would that count as sufficient exploration of the prompt?
I'd say they'd be more likely to give you something like 'We cannot sympathise with Even because she is so egotistical. Discuss.' but that's basically paraphrasing the 2014 i. prompt: i. It is Eve's ruthless pursuit of ambition that leads us to dislike her. Do you agree?

So that specific concept/ combination of character+theme+message is unlikely to crop up again. Nevertheless, the breakdown that you've done here is really solid, and would definitely suffice provided that the majority of your discussion centred around Eve since she's obviously meant to be the focus in this case.

with regards to films in text response, it is advisable to prefer/use more cinematic , film based techniques that the author used (not being prompt specific) or stick with regular examples/quotes? ive been told its better to focus on the film techniques for films.. but i am not sure.. ty!
The most impressive thing you can do would be to combine your discussion of cinematic devices WITH 'regular examples' and quotes. Blending the two is something only high range essays tend to showcase, so it's a great way of standing out. Otherwise, if you're only ever talking about quotes or camera angles but can't comment on how they work in tandem to create ideas, it can be tough for the assessors to piece things together for you :P

Yo Just keep in mind that you can't just base your essay on the main bitch. You got other hoes you gotta discuss
Excuse you. Glauce is a nice lady. How dare you besmirch her name.

+100

 ;D ;D ;D

*101
(http://i.imgur.com/ACflDl8.png)

Now let's see if we can crack 100,000 views by Wednesday ;)
Title: Re: VCE English Question Thread
Post by: HopefulLawStudent on October 24, 2016, 03:53:10 pm
Quote
AAE highlights the neverending judgement on what it means to be a woman. Discuss.

How would you break down this topic? Legit, I have no idea what to do with it...

EDIT: On second thought, what's this topic even saying??
Title: Re: VCE English Question Thread
Post by: HasibA on October 24, 2016, 04:07:46 pm
'The most impressive thing you can do would be to combine your discussion of cinematic devices WITH 'regular examples' and quotes. Blending the two is something only high range essays tend to showcase, so it's a great way of standing out. Otherwise, if you're only ever talking about quotes or camera angles but can't comment on how they work in tandem to create ideas, it can be tough for the assessors to piece things together for you :P'
how do i really do that effectively xD :/
i was thinking, very vaguely- 'Perkins' use of extreme close-up on Eddie's facial features demonstrates the anger and disappointment he experienced following the result of the initial Supreme Court trial, as a result of his passion, pride and desperation to maintain cultural ties to both himself and his people, in endeavouring to provide a substantiated argument against the rigid, ever present racism and discrimination present in the film, forcing Eddie to be nothing but "a trouble maker.." - allowing Perkins' to instill the erratic decisions Eddie had to make...

does that even make sense :'(

also- my teacher practically yelled at me for not having an 'obvious' resolution to one of my conflict pieces (the piece called 'The Journey' on the submissions thread)- but in that particular essay (prompt: there are no winners on conflict or something like that) , i thought i addressed the criterion for the essay :/
excerpt of what my teacher said- You need resolution at the end even if it means giving up and facing death. What you wrote  seems confusing. What do you have and will you hang on to that expecting nothing more out of life? Because that is all you can do ?
Is this short term resolution?
What about long term?

>criticizes my essay for their vague nature
>gives me vague feedback in response to like 3 essays *cries*

edit: context piece i was talking about haha! Conflict piece- advice would be appreciated :)
THANK YOU SO MUCH LAUREN (or whoever else replies <3)
Title: Re: VCE English Question Thread
Post by: FallingStar on October 24, 2016, 04:10:32 pm

"How do women in the film defy/adhere to the 1950s feminine idea?"
How would I qualify/challenge this prompt? How would I even break down this prompt?


This is my opinion, but I'll go for different approaches:
Approach one: Literally a method per paragraph:
 - Adhere by Becoming housewives
 - Adhere by being obedient to men
 - Defy by having a career outside the house
(You get the idea. even though they're not my best ideas)
 
Approach two: Grouped methods
Para 1: How women adhere to the 1950s ideals
Para 2: How women defy the 1950s ideals
Para 3: What does the film say about these ideals.

Bear in mind that characters of All About Eve are multidimensional, in which case some may adhere or defy more than others, yet they are mostly a combination of adherence and defiance. It may seem that Karen is adhering to the ideals but keep in mind she is very well educated (went to Radcliffe Univeristy) which does defy the 1950s ideals (only 1.2% of women went to uni). And Miss Casswell has most of the ideals apart from being married.
Title: Re: VCE English Question Thread
Post by: Manige123 on October 24, 2016, 04:11:40 pm
Can someone please help!! When looking at conflict, what are some external examples to look at when looking at intellectual/internal conflict?
Title: Re: VCE English Question Thread
Post by: literally lauren on October 24, 2016, 05:18:19 pm
Quote
AAE highlights the neverending judgement on what it means to be a woman. Discuss.
How would you break down this prompt? Legit, I have no idea what to do with it...

"neverending" ??? ???

I'd simplify this to 'Discuss the judgement that the women in AAE face' or something like that... idk how the hell you'd argue it's 'neverending,' especially because the film kind of hints at the idea that the characters can break free of this... unless you read into the cyclical Margo-->Eve--->Phoebe thing and see that as indicative of the systemic judgement and 'shelf-life' of women in the public eye. But yeah. treating this as a prompt about the role of women in relation to judgement/prejudice would suffice.

Quote
The most impressive thing you can do would be to combine your discussion of cinematic devices WITH 'regular examples' and quotes. Blending the two is something only high range essays tend to showcase, so it's a great way of standing out. Otherwise, if you're only ever talking about quotes or camera angles but can't comment on how they work in tandem to create ideas, it can be tough for the assessors to piece things together for you :P'
how do i really do that effectively xD :/
blah blah blah here's a discussion about a point in the film where the character says "I really want to fight against injustice." The fact that this is accompanied by the inspirational digetic sound of other characters cheering, and a slow panning shot of the smiles of the crowd amplifies the message that... blah blah blah link to a bigger idea.
i was thinking, very vaguely- 'Perkins' use of extreme close-up on Eddie's facial features demonstrates the anger and disappointment he experienced following the result of the initial Supreme Court trial, as a result of his passion, pride and desperation to maintain cultural ties to both himself and his people, in endeavouring to provide a substantiated argument against the rigid, ever present racism and discrimination present in the film, forcing Eddie to be nothing but "a trouble maker.." - allowing Perkins' to instill the erratic decisions Eddie had to make...
This is also excellent; either order (quote + technique, or technique + quote) would be fine.

also- my teacher practically yelled at me for not having an 'obvious' resolution to one of my conflict pieces (the piece called 'The Journey' on the submissions thread)- but in that particular essay (prompt: there are no winners on conflict or something like that) , i thought i addressed the criterion for the essay :/
excerpt of what my teacher said- You need resolution at the end even if it means giving up and facing death. What you wrote  seems confusing. What do you have and will you hang on to that expecting nothing more out of life? Because that is all you can do ?
Is this short term resolution?
What about long term?

>criticizes my essay for their vague nature
>gives me vague feedback in response to like 3 essays *cries* hahaha, oh this brings back memories :'D

edit: context piece i was talking about haha! Conflict piece- advice would be appreciated :)
THANK YOU SO MUCH LAUREN (or whoever else replies <3)
Will get round to all the as-yet-unmarked pieces on the Subs board soon!
Title: Re: VCE English Question Thread
Post by: HasibA on October 24, 2016, 05:26:27 pm
thanks a tonne lauren <3
my saviour in english tbh !!
Title: Re: VCE English Question Thread
Post by: nadiaaa on October 24, 2016, 05:29:27 pm
Reading my corrections for my context essay by my teacher and all she said was 'Go deeper with the ideas you're discussing'
Like um lady its like 1 day before the exam and youre telling me this now.. Guys how do we go deeper? Like what should i  do to make my examples more complex? #Stressed
Title: Re: VCE English Question Thread
Post by: MaeveHuxtable on October 24, 2016, 05:34:40 pm
Hey Lauren!
I'm wondering with an expository essay, should I signpost what external texts I'm going to talk about in my body paragraphs, in my introduction? I've received heaps of mixed messages about this. Some people say have the intro very broad and not mention the body paragraphs and then have my body paragraphs start off as "In Plato's allegory of the cave...".
How do you suggest I structure my intro?
Thanks!
Title: Re: VCE English Question Thread
Post by: literally lauren on October 24, 2016, 05:44:03 pm
Reading my corrections for my context essay by my teacher and all she said was 'Go deeper with the ideas you're discussing'
Like um lady its like 1 day before the exam and youre telling me this now.. Guys how do we go deeper? Like what should i  do to make my examples more complex? #Stressed
Hahaha, it kind of depends what sort of 'deeper' she's looking for. Did she say anything more than that? It might be an issue with your explanations (e.g. 'the character says "I can't believe this" which reveals their longing for home' ... that might be right, but there's not really enough discussion to warrant that conclusion, so that student would need to 'go deeper' with their explanations.) Or it might be that the discussion itself is lacking, so you've got an awesome collection of evidence in your essay, but you don't link it to the prompt effectively enough? Like, if you're ending paragraphs with examples, that's usually a hint that the connection to the Context hasn't yet been made clear.

As a general rule of thumb: think of it like writing for a five year old. They can follow your logic when you explain things to them, but they're not going to come to grand conclusions about reality/conflict/identity/etc. all on their own. YOU have to make that stuff clear, and take them through your train of thought step-by-step! :)

Hey Lauren!
I'm wondering with an expository essay, should I signpost what external texts I'm going to talk about in my body paragraphs, in my introduction? I've received heaps of mixed messages about this. Some people say have the intro very broad and not mention the body paragraphs and then have my body paragraphs start off as "In Plato's allegory of the cave...".
How do you suggest I structure my intro?
Thanks!
Signposting is completely optional in all essays. Some students like it because it gives them the clarity they need to get started with writing. Some teachers like it because it gives them an idea of what the piece will cover. Some students don't like it because they feel like it 'locks' them into talking about certain ideas/examples, and they might want the freedom to change their minds or their approach later. Some teachers don't like it because they think it's simplistic. So it's really up to you!

For Context, I'd say general introductions are best because it'll help you avoid the pitfall of writing very clunky signposts like 'Firstly... Secondly... Thirdly...' This can also let you do a bit of general discussion at the start of each paragraph so that you don't have to begin every single one with 'As this[/i] piece of evidence shows us...' Sometimes going back to the prompt and revisiting a general concern can be a useful way of launching yourself into your piece. Some of the samples here (scroll down!) might be worth a read if you're really unsure :)
Title: Re: VCE English Question Thread
Post by: pra96 on October 24, 2016, 05:49:42 pm
Hi guys,

Could someone please read my Medea essay. Thanks

Prompt: ‘Medea has no choice but to seek revenge.’ Do you agree?

Euripides’ ancient Greek tragedy Medea, follows the journey of its titular character as she seeks vengeance upon her husband. This desire for revenge stems from Medea’s hubristic and passionate nature, which dominates her actions. Further, the social injustice Medea receives as a “barbarous” woman forces her to seek justice by her own means. Similarly, Medea receives divine approval in her fight for justice as the Gods support her actions.

Medea’s emotional and excessively proud nature dominates her actions throughout the play. The Nurse highlights the sorrow that Medea suffers as she “dissolve[es] the long hours in tears”. Jason’s betrayal has “hurt her where her affection runs deepest”, which is her love for Jason. Thus her melancholic reaction to the news of Jason’s decision demonstrates her passionate nature. This sorrow transmutes into “fierce and intractable” hatred for Jason. Hence she has “fierc[e] anger… rag[ing] in place of dearest love” for Jason. Further, it is this transformation of affection that precipitates her plan to “deal Jason the deepest wound” by “reach[ing his] heart”. Therefore Medea passionate nature drives her to avenge her betrayal by killing the royal family and her own sons. Similarly, Medea’s hubris causes her to seek revenge. Having “the mind of a queen”, Medea doesn’t wish to face “the laughter of [her] enemies”. She cannot bear to be belittled by other people, even wishing death on her sons rather than to “leave sons of [hers] behind for [her] enemies to insult”. Thus Medea has excessive pride, for which reason she cannot let her betrayal be forgotten easily. Subsequently, she is compelled to “punish [her] enemies” in order to defend this pride. Thus her actions are controlled by her emotion and pride. Therefore Medea’s hubristic and passionate nature causes her to seek revenge on Jason.

Society’s treatment of Medea forces her to deal justice on her own. The patriarchal society that Euripides portrays in Medea gives little heed to her problems. Society views women with intelligence as “ignorant”, disliking such a trait in women. Thus Medea is viewed as an “evil woman” because of her intelligence and her skill in magic. Creon views Medea as a threat for these reasons, which is why he decides to exile her from Corinth. Furthermore, women in ancient Greek society have no rights in marriage as they are “forced to look to one man only”, expected to be obedient and “yield to” their husbands. Jason’s misogynistic opinion that “without women, human life would be rid of its miseries” echoes the misogyny in ancient Greek society. Thus Medea is not taken seriously, for which reason she is unable to find justice in the eyes of society. Medea instead proves herself as “of a different kind”, contrary to the social perception that women are “weak… humble [and] passive”. She wishes to stand up for herself, as well as for the other oppressed women in Greek society. Similarly, Medea’s position as a “barbarous” woman leads to her unfair treatment in Corinth. Jason rebukes Medea after the death of his sons, believing that a “Hellene woman” would not have done as she did. Medea is viewed as an outsider by Greek society, for which reason Creon exiles her without protest. She therefore has fewer rights than a Greek citizen. Thus the perception that society has of Medea prevents her from receiving justice. Hence Medea is forced to deal justice herself.

The gods approve of Medea’s actions, supposedly sympathizing with her as they help her in her pursuit of justice. Medea constantly calls on the gods to help her in the midst of her troubles. She “invokes Themis” and “Zeus who is named keeper of men’s oaths”. Medea calls on “the mighty Sun” her grandfather. Thus she seeks divine justice while she may not have social justice. As events unfold, Medea has her desires fulfilled as “the will of heaven brings down on Jason justice and calamity”. Thus it is evident that the gods approve of Medea’s actions, especially as they do not protect those she seeks to harm. This is further evident in the end as Helios sends Medea a dragon chariot to “keep [her] from the hands of her enemies”. Hence the gods, being absolute symbols and dispensers of justice, vindicate Medea. Their approval of her fight for justice does not give her a choice; it merely encourages her course of action, as she must ultimately deal it herself. It is thus that her actions were pre-ordained, “many are the fates which Zeus on Olympus ordains”, giving her no choice in her fate. Thus it is ultimately not up to Medea to seek revenge, for the gods had “set her course” beforehand. Therefore Medea’s battle for revenge was pre-ordained by the gods, giving her no other choice.

The hubristic and emotional nature of Medea causes her to seek vengeance for her betrayal by Jason. Similarly, the societal injustice that she receives, stemming from society’s perceptions of her, gives Medea no choice but to seek revenge. Further, Medea’s journey to seek revenge on Jason was pre-ordained by Zeus, thus not giving her a choice.
Title: Re: VCE English Question Thread
Post by: jacquic on October 24, 2016, 05:52:46 pm
Hello,
Could someone please help me with this Brooklyn prompt?

Toibin resists offering readers a simply uplifting story but provides a more complex portrait of his protagonist, Eilis Lacey. Discuss.

What paragraphs would you do for a prompt like this?
Thanks heaps
Title: Re: VCE English Question Thread
Post by: literally lauren on October 24, 2016, 06:00:40 pm
Hey pra96,

I've moved your essay to the Submissions Board and I'm working through those pieces now, so should get to yours soon! :)

Hello,
Could someone please help me with this Brooklyn prompt?

Toibin resists offering readers a simply uplifting story but provides a more complex portrait of his protagonist, Eilis Lacey. Discuss.

What paragraphs would you do for a prompt like this?
Thanks heaps
Hey jacquic,

There's no definitive right answer when it comes to paragraph structure; it's all about breaking down the prompt in a way that lets you discuss the text effectively. So if we were to simplify this prompt down to 'Brooklyn isn't a basic, happy story, and Eilis is portrayed as a complex character' does that seem any easier?

Either think ahead to the kinds of evidence you might bring up (e.g. Rosy dying = not uplifting; Eilis' relationship with Ireland/America = complex) so that you can start grouping your examples into paragraphs... OR... start with the prompt itself and think about how you could link it to your interpretation of the novel and what you believe Toibin is saying overall.

If that still doesn't make sense, let me know what initial thoughts you had about the prompt or what about it you found challenging and I might be able to offer more assistance! :)
Title: Re: VCE English Question Thread
Post by: blacksanta62 on October 24, 2016, 06:08:19 pm
Can someone provide a strong way to integrate the author's views and values on certain themes?
Thank you :)
Bump :)
Title: Re: VCE English Question Thread
Post by: literally lauren on October 24, 2016, 06:12:45 pm
Can someone provide a strong way to integrate the author's views and values on certain themes?
Thank you :)
Ah sorry - missed one!

Easiest way is to, at the ends of your paragraphs, have sentences like:
 • Therefore, the author highlights...
 • Thus, the text showcases...
 • To this end, The Text emphasises...
 • Ultimately, this forms part of the author's critique/celebration of...
 • Hence, the author suggests that...

^whereby you force yourself to zoom out and consider the implications. This is the case for any prompt type too - no matter how narrow or general it is, being about to get to the end of a discussion and say 'so basically, the author is conveying this idea' is immensely valuable!
Title: Re: VCE English Question Thread
Post by: purplegiraffe on October 24, 2016, 06:14:34 pm
Hi :)
I'm really having trouble with preparing for context.
I've written creative pieces all year so I don't want to change 2 days before the exam.
Like should i have an idea of what I'll write about before i go into the exam? English isn't my strong suit.

I'm doing Encountering Conflict and my text is Life of Galileo
Title: Re: VCE English Question Thread
Post by: jacquic on October 24, 2016, 06:21:22 pm

Hey jacquic,
There's no definitive right answer when it comes to paragraph structure; it's all about breaking down the prompt in a way that lets you discuss the text effectively. So if we were to simplify this prompt down to 'Brooklyn isn't a basic, happy story, and Eilis is portrayed as a complex character' does that seem any easier?

Either think ahead to the kinds of evidence you might bring up (e.g. Rosy dying = not uplifting; Eilis' relationship with Ireland/America = complex) so that you can start grouping your examples into paragraphs... OR... start with the prompt itself and think about how you could link it to your interpretation of the novel and what you believe Toibin is saying overall.
If that still doesn't make sense, let me know what initial thoughts you had about the prompt or what about it you found challenging and I might be able to offer more assistance! :)
thanks lauren
I was mostly confused about what the prompt is asking. So is it basically asking why Toibin gives such deep descriptions and discussions about Eilis rather than an exhilarating novel? I wondered if you could discuss how Eilis transformations after migrating to Brooklyn and then all the sorrows that she suffers and the effects it has on her and finally the complexity of love in the novel?
Title: Re: VCE English Question Thread
Post by: literally lauren on October 24, 2016, 06:23:01 pm
Hi :)
I'm really having trouble with preparing for context.
I've written creative pieces all year so I don't want to change 2 days before the exam.
Like should i have an idea of what I'll write about before i go into the exam? English isn't my strong suit.

I'm doing Encountering Conflict and my text is Life of Galileo
Hi there!

If you go to the first page of this thread and click on the 'Context' tab, there'll be a bunch of links to previous questions and bits and pieces of advice that might help you out (sorry - your question was a little vague so I don't really know where to start!)

If you're writing creatively - I'm guessing some kind of narrative? - then having a rough idea of plot points and your narrative arc can be a good idea. Then, you just mould this to suit the prompt.

For example, if you were writing a short story about a scientist who makes a monumental discovery, but then experiences inner conflict because he/she is very apprehensive about the potential ramifications that this discovery will have on society (--> hence, link to LoG), then...
 • if the prompt is about the causes of conflict, then you'd spend more time talking about how the discovery affects your main character and why they are feeling so conflicted
 • if the prompt is about our response to conflict, then you'd focus more so on your character's reaction and deliberation
 • if the prompt is about the consequences of conflict, then you'd either have your character wonder what the consequences might be, or maybe actually show those consequences if you wanted to
 • if the prompt is about the resolution of conflict, then you'd spend more time towards the end of the narrative journey and flesh out how, if at all, your character tried to resolve that turmoil/clash of ideas etc.

So each time, you have the same rough idea of what needs to be covered, but you mix and match different focal points to suit the prompt.

Going through lists of past VCAA prompts (and just as many prompts as you can) would also be a good practice exercise for you to do tomorrow :)
Title: Re: VCE English Question Thread
Post by: thetaromochi on October 24, 2016, 06:26:05 pm
Hi guys,

I'm having a lot of trouble with this prompt for Whose Reality:
"An entertaining story is far more believable than the truth."

I can't seem to think of many external sources since I don't really agree with the prompt but I'm afraid that I might go off topic.

Any advice would be appreciated, thanks in advance!
Title: Re: VCE English Question Thread
Post by: sophieharper on October 24, 2016, 06:29:42 pm
Hey guys last minute questions and I'm sure its irrelevant, but if we are faced with section c piece that has multiple comments even some one liners, should we analyse all of the comments or is just one enough???
thankssssss x
Title: Re: VCE English Question Thread
Post by: blacksanta62 on October 24, 2016, 06:36:00 pm
Ah sorry - missed one!

Easiest way is to, at the ends of your paragraphs, have sentences like:
 • Therefore, the author highlights...
 • Thus, the text showcases...
 • To this end, The Text emphasises...
 • Ultimately, this forms part of the author's critique/celebration of...
 • Hence, the author suggests that...

^whereby you force yourself to zoom out and consider the implications. This is the case for any prompt type too - no matter how narrow or general it is, being about to get to the end of a discussion and say 'so basically, the author is conveying this idea' is immensely valuable!
Amazing!
Would you (or anyone) have any suggestions for what to do now and tomorrow? I feel like writing any essays whatsoever would just add unecessary stress because they seem to require immense focus (more than any other subject imo)
Thank you :)
Title: Re: VCE English Question Thread
Post by: FatimaEl on October 24, 2016, 06:38:22 pm
Hi Lauren, i have a difficult time knowing whether i am retelling the story or analysing, here is an extract of my No Sugar essay,
"Davis’ choice to focus the title on sugar, and not the other rations denied, is not one with no basis. The audience is immediately introduced, act one, scene one, to the character’s attachment to sugar; “Sam Millimurra prepares mugs of tea, lacing them generously with sugar”. However the sugar ration is cut, “No sugar in our tea”, upon arrival at the Moore River Native Settlement. This signifies that the “sweet” and comfortable life the Millimurra family relished back home at Northam would end as a result of the dispossession to the settlement. In the final scene, scene ten, sugar is mentioned once more; “Milly gives Mary a sugar bag”. This mention is significant as Mary is leaving the Moore River settlement, along with her departure returns the saccharine life she was once accustomed to. Through the emphasis on ‘sugar’, Davis is able to shed light on the polarity of the two locations, and on how disagreeable the settlement is with the lifestyle of the Millimurra family"
Is this a good analysis or am i regurgitating the novel? (also feel free to add any extra feedback on this paragraph)
Thank you!
(this is a repost btw need to bump this, would really appreciate some last minute feedback)
Title: Re: VCE English Question Thread
Post by: HasibA on October 24, 2016, 06:39:10 pm
is there a list of 'MUST-HAVES' for an introduction of a lang analysis?
i.e so far, i've got the authors contention, and accompanying pieces contentions, as well as any major tone/style incorporated in the piece(s)- and any image's contention/purpose, and main audience/readers - anything else i can add?

what about for lang analysis conclusions- all ive got is : summarise the main techniques author used, how they affected the audience, any major technique- if any- that was used and a subtle rehash of the author(s) main contentions.
anything else? ty guys <3
Title: Re: VCE English Question Thread
Post by: literally lauren on October 24, 2016, 06:40:32 pm
Hi Lauren, i have a difficult time knowing whether i am retelling the story or analysing, here is an extract of my No Sugar essay,
"Davis’ choice to focus the title on sugar, and not the other rations denied, is not one with no basis. double negative is a bit clunky here The audience is immediately introduced, act one, scene one, to the character’s attachment to sugar; “Sam Millimurra prepares mugs of tea, lacing them generously with sugar”. <-- this quote isn't really integrated However the sugar ration is cut, “No sugar in our tea”, upon arrival at the Moore River Native Settlement. This signifies that the “sweet” and comfortable life the Millimurra family relished back home at Northam would end as a result of the dispossession to the settlement. In the final scene, scene ten, sugar is mentioned once more; “Milly gives Mary a sugar bag”. This mention is significant as Mary is leaving the Moore River settlement, along with her departure returns the saccharine life she was once accustomed to. Through the emphasis on ‘sugar’, Davis is able to shed light on the polarity of the two locations, and on how disagreeable the settlement is with the lifestyle of the Millimurra family"
Is this a good analysis or am i regurgitating the novel? (also feel free to add any extra feedback on this paragraph)
Thank you!
Blue stuff is analytical, so I don't think there's too much summary, but I do think it'd be a lot stronger if your quotes were integrated (i.e. if your quotes fit your sentences like: 'In the final scene, this motif is seen once again when Milly gives Mary "a sugar bag" to symbolise the saccharine life she was once accustomed to...') That way, your slightly summative (non-blue) sentences can be made more analytical, and thus worth more marks! :)

thanks lauren
I was mostly confused about what the prompt is asking. So is it basically asking why Toibin gives such deep descriptions and discussions about Eilis rather than an exhilarating novel? I wondered if you could discuss how Eilis transformations after migrating to Brooklyn and then all the sorrows that she suffers and the effects it has on her and finally the complexity of love in the novel?
It's more like it's asking about why Eilis is emotionally complicated as opposed to constructing a novel where she's a beautiful flawless heroine who undergoes no difficulties and is totally 100% happy at the end. So why does Eilis have to/benefit from experiencing difficulty in her life, and what is Toibin saying about challenges/complexity etc.? So yes, you could absolutely talk about transformations, suffering, and relationships as a potential paragraph breakdown for this prompt :)

Hey guys last minute questions and I'm sure its irrelevant, but if we are faced with section c piece that has multiple comments even some one liners, should we analyse all of the comments or is just one enough???
thankssssss x
It's not at all irrelevant! And it's awesome to see people thinking ahead to potentially weird material that could crop up this year!

Golden rule re: comparative tasks or just plain weirdly structures:
Let the spread of your analysis reflect the spread of the material.

In other words, if you get, like 20 little comments in the exam and you only talk about, like 14, you'd probably  be okay. But if you got something like Exam 3 here and you completely ignored one of those authors, that might be a problem. If there's a part of the material or even an entire piece/comment that's only taking up a few lines, it's no big deal, but if it takes up nearly a whole page (like the 'comment' in 2015) then you'd definitely be expected to address it and you'd definitely be penalised if you didn't because you'd be ignoring half the material. Obvs aiming to say at least one thing about each text + visual would be great, but that may not be possible/realistic, so trust your intuitions and the golden rule above :)

Amazing!
Would you (or anyone) have any suggestions for what to do now and tomorrow? I feel like writing any essays whatsoever would just add unecessary stress because they seem to require immense focus (more than any other subject imo)
Thank you :)
See: Boosting your mark in 48 hours :)

I might put together a little 'stuff to do on Tuesday' guide tonight too, just for all you diligent English kids out there ::)
Title: Re: VCE English Question Thread
Post by: FatimaEl on October 24, 2016, 06:47:47 pm
Blue stuff is analytical, so I don't think there's too much summary, but I do think it'd be a lot stronger if your quotes were integrated (i.e. if your quotes fit your sentences like: 'In the final scene, this motif is seen once again when Milly gives Mary "a sugar bag" to symbolise the saccharine life she was once accustomed to...') That way, your slightly summative (non-blue) sentences can be made more analytical, and thus worth more marks! :)
Thank you Lauren! that was very helpful!! I will try to integrate the quotes better in the actual exam (fingers crossed)
Title: Re: VCE English Question Thread
Post by: literally lauren on October 24, 2016, 06:47:55 pm
is there a list of 'MUST-HAVES' for an introduction of a lang analysis?
i.e so far, i've got the authors contention, and accompanying pieces contentions, as well as any major tone/style incorporated in the piece(s)- and any image's contention/purpose, and main audience/readers - anything else i can add?

what about for lang analysis conclusions- all ive got is : summarise the main techniques author used, how they affected the audience, any major technique- if any- that was used and a subtle rehash of the author(s) main contentions.
anything else? ty guys <3
Intro:
1. CONTENTION(s) <--most important!
2. spark/issue <-- can make for a good opening sentence or half-sentence (e.g. 'Following the recent...' or 'In response to...')
3. tone, target audience <-- pretty optional, since it's more important to bring these up in your body paragraphs, and you don't have to do these for every piece if you don't want to

Concl:
1. Say something about the authors' use of language in general (e.g. 'By appealing to readers' sense of fear, the author seeks to... --> 2.')
2. Link this to the overall intended effect or the consequence the author hopes to bring about
3. Recap contentions, and end on a high note :)

That's pretty much it; there's a lot of optional peripheral stuff I haven't mentioned (like signposting sub-arguments) cause I reckon it's a waste of time. Cover ^those bases, and you should be all good!
Title: Re: VCE English Question Thread
Post by: knightrider on October 24, 2016, 06:52:45 pm
Hey lauren just wanted your opinion on something.

for LA in my intros, i have just usually been doing a brief context sentence, followed by the contentions of the authors with their tones occasionally.

Then i move straight into my body paras is this fine ?

Thanks  :)
Title: Re: VCE English Question Thread
Post by: scandin9 on October 24, 2016, 06:52:57 pm
Hi Lauren,


My teacher says that my creative context pieces do not meet the criteria for a link to the ideas of text; however, they fulfill all the other criterion to a 9 0r 10 standard; despite the fact that included 5 quotes from the text and discussed the ideas of the text and used them as a framework for the piece? She says that the ideas from the text are not discussed  in depth. Is my teacher correct or  is  the incorporation of quotes and a link to the ideas sufficient in terms of a link to the text?
Title: Re: VCE English Question Thread
Post by: mayonnaise on October 24, 2016, 06:55:23 pm
Hey!
So I was looking at some context essays on the submissions board and got super jealous of them cool titles they got there   8)
I know that it is completely optional, but is there any particular benefits to having a title for a creative piece ?
Title: Re: VCE English Question Thread
Post by: mayonnaise on October 24, 2016, 06:58:07 pm
Hey lauren just wanted your opinion on something.

for LA in my intros, i have just usually been doing a brief context sentence, followed by the contentions of the authors with their tones occasionally.

Then i move straight into my body paras is this fine ?

Thanks  :)

Not Lauren but this follows the guidelines she just set out so it seems completely fine :D
If it's important you might want to add the audience, but you've covered all the basics pretty much
Title: Re: VCE English Question Thread
Post by: studentofthenow on October 24, 2016, 07:01:52 pm
Hey Lauren.

For expository essays, do you recommend knowing one really well and adapting it? Do you recommend having spare paragraphs or ideas as backup?
Title: Re: VCE English Question Thread
Post by: Hydraulix on October 24, 2016, 07:26:37 pm
Hey guys!!

For this prompt for Medea: Are the characters in Medea motivated more so by reason or emotion?

I seem to be unable to formulate arguments that aren't character-based for each paragraph.
How would you approach this and how do I avoid doing this ugghh.

Thank youu sahhh much
Title: Re: VCE English Question Thread
Post by: pra96 on October 24, 2016, 07:42:10 pm
Hey guys!!

For this prompt for Medea: Are the characters in Medea motivated more so by reason or emotion?

I seem to be unable to formulate arguments that aren't character-based for each paragraph.
How would you approach this and how do I avoid doing this ugghh.

Thank youu sahhh much

Hi Hydraulix,

I'm not the best at english so take my advice with a grain of salt.  :D

I would have three paragraphs for this prompt:
- how some characters are motivated by reason (ie. Jason, maybe Creon to a certain extent who decides to exile Medea for this reason, Aegeus who takes a rational approach to dealing with Medea)
- how some characters are motivated by emotion/passion (ie. Medea, also Creon as he "is a father" and sympathises with Medea's situation to some degree, Glauce who falls for the gifts of Medea and her love for Jason)
- how some characters (ie. the Chorus + Nurse) are guided by a combination of the two, embodying sophrosyne

Hope this helps. :)

PS: always nice to see a fellow MHS student
Title: Re: VCE English Question Thread
Post by: HasibA on October 24, 2016, 07:44:19 pm
Hey!
So I was looking at some context essays on the submissions board and got super jealous of them cool titles they got there   8)
I know that it is completely optional, but is there any particular benefits to having a title for a creative piece ?
no real benefit, but i hear they like having a title for creative pieces :)
Title: Re: VCE English Question Thread
Post by: Dem16 on October 24, 2016, 07:53:09 pm
Can someone please help me with this Brooklyn prompt?
How does Toibin's understated, economical writing style influence the reader's understanding of characters and settings of Brooklyn?
What paragraphs do you do for these types of prompts asking about the language used??
Title: Re: VCE English Question Thread
Post by: cookb111 on October 24, 2016, 08:07:29 pm
Hey just i general question , ive put alot of work in but im still worried im gonna get under 25 , because im not even close to better than half the state. so how hard is it really to get a 25 ?
Title: Re: VCE English Question Thread
Post by: Biology24123 on October 24, 2016, 08:12:12 pm
Hey just i general question , ive put alot of work in but im still worried im gonna get under 25 , because im not even close to better than half the state. so how hard is it really to get a 25 ?

Getting 3 5's should be  25-30
Title: Re: VCE English Question Thread
Post by: pra96 on October 24, 2016, 08:17:35 pm
Hey just i general question , ive put alot of work in but im still worried im gonna get under 25 , because im not even close to better than half the state. so how hard is it really to get a 25 ?

Using the normal distribution stuff:

- A study score of 25 (top 78%) would be 26/60 on the exam. So that would be getting two 4s and a 5 out of 10.
- A study score of 30 (top 53%) would be 33/60 on the exam. So you would need around two 6s and a 5 out of 10.

Hope this helps.  :D
Title: Re: VCE English Question Thread
Post by: FallingStar on October 24, 2016, 08:19:57 pm
Hey Lauren.

For expository essays, do you recommend knowing one really well and adapting it? Do you recommend having spare paragraphs or ideas as backup?

Know your ideas and examples really well. And no, I don not recommend you memorising a paragraph unless you are able to memories and adapt too. Which can actually get very clunky when you try to adapt it to the prompt if memorised. I have four main categories of examples which I would use in the exam. And practise planning prompts at this stage.

Can someone please help me with this Brooklyn prompt?
How does Toibin's understated, economical writing style influence the reader's understanding of characters and settings of Brooklyn?
What paragraphs do you do for these types of prompts asking about the language used??

Basically, you sort your paragraphs by the type of language in the text. Perhaps look at metalanguage words would help you. Eg. Metaphors and similes, imagery, etc.
Title: Re: VCE English Question Thread
Post by: P.GUAN on October 24, 2016, 08:55:14 pm
Hello,
I am really worried about the text response. For Brooklyn do you have to write what techniques Toibin uses to support the prompt?

Thanks in advance!!
Title: Re: VCE English Question Thread
Post by: Biology24123 on October 24, 2016, 08:55:32 pm
Are vcaa prompts for TR usually very general and easy to respond to?
Title: Re: VCE English Question Thread
Post by: Alter on October 24, 2016, 08:58:47 pm
Are vcaa prompts for TR usually very general and easy to respond to?
They're usually open and accessible and at least one of the topics VCAA gives for a given text will be about an idea you've at least thought about if you've put in any effort into English. It's a waste of time for VCAA to distinguish students with overly-difficult prompts if they can already differentiate them by the quality of their responses.

At any rate, there is no correct answer to your question. One prompt might be very easy for one person and hard for another student. I'd highly recommend that you look at past exams on the VCAA website and make an appraisal for yourself. After all, we don't know what you know about your book. Hope this helps.
Title: Re: VCE English Question Thread
Post by: JellyBeanz on October 24, 2016, 08:59:35 pm
Are vcaa prompts for TR usually very general and easy to respond to?

Definitely more straight forward than prompts you may see on commercial exams. Just have a look at past VCAA exam prompts and gauge for yourself.
Title: Re: VCE English Question Thread
Post by: happymeal on October 24, 2016, 09:00:03 pm
I'm having some trouble trying to make my context expository pieces not sound like a text response essay when I'm trying to link back to my set text. What are some ways to avoid this?
Title: Re: VCE English Question Thread
Post by: Biology24123 on October 24, 2016, 09:04:16 pm
I'm having some trouble trying to make my context expository pieces not sound like a text response essay when I'm trying to link back to my set text. What are some ways to avoid this?

In mine I don't mention specific moments in the book but rather the overarching themes
Title: Re: VCE English Question Thread
Post by: Alter on October 24, 2016, 09:06:54 pm
I'm having some trouble trying to make my context expository pieces not sound like a text response essay when I'm trying to link back to my set text. What are some ways to avoid this?
Perhaps you're referring to a lot of specific evidence from the text. While it's fine to refer to your text, you mainly just want to cover the ideas from it that are relevant to the prompt itself. You don't need to be quoting tons of characters or explaining the significance of cobblestone around Klaus's house in Stasiland. Save all of that for TR. Similarly, you shouldn't need to be referring to the views and values of the author in a context piece.

For the most part, you'll be covering concepts and ideas from your text at a broad and holistic level, and then linking them into other examples that you know of (e.g. historical or contemporary events in society). In a text response, your discussion is confined to the text itself, rather than its significance in a greater scheme of things.

I also avoided referring to any texts or examples in my introduction and conclusion. This is a good way of avoiding the trap of writing a text response as you force yourself to pilot your discussion at a broad level. To this end, the reference text will act as a supplement to your discussion, not the main component.

Hope this helps.
Title: Re: VCE English Question Thread
Post by: lxX_Mercer_lXxl on October 24, 2016, 09:16:43 pm
I can't remember quotes. What is the best way to find remember quotes for the exam? WHICH IS IN 2 DAYS OMG
Title: Re: VCE English Question Thread
Post by: HasibA on October 24, 2016, 09:44:52 pm
I can't remember quotes. What is the best way to find remember quotes for the exam? WHICH IS IN 2 DAYS OMG
is ur text a film or book? if its a film like mine, i dont think word by word quotes are super important, but rather directors angles, techniques etc.
if its a book, can someone correct me, having a few quotes with just straight up examples like 'when jason did this... this blah . which the author conveys' - wouldnt that be fine? ahha
Title: Re: VCE English Question Thread
Post by: lxX_Mercer_lXxl on October 24, 2016, 09:58:16 pm
I am doing 2 texts: Every Man In This Village Is A Liar by Megan Stack and Media.
Title: Re: VCE English Question Thread
Post by: Alter on October 24, 2016, 10:01:52 pm
I am doing 2 texts: Every Man In This Village Is A Liar by Megan Stack and Media.
No need to stress; there's still time to learn.

The good news is that you don't need to memorise quotes for Every Man as it's a Section B text. I mean, technically, you can if you really think they're applicable, but they're not a prerequisite.

For learning quotes for Medea, try to learn them with respect to the major themes and use cue cards/sticky notes to test yourself. If you do enough immersion, you'll be able to remember as many as you want come exam time.

Best of luck!
Title: Re: VCE English Question Thread
Post by: lxX_Mercer_lXxl on October 24, 2016, 10:06:18 pm

The good news is that you don't need to memorise quotes for Every Man as it's a Section B text. I mean, technically, you can if you really think they're applicable, but they're not a prerequisite.

I was told quotes are a MUST?? If I do not use quotes, how do i use examples from the text to support my ideas?
Title: Re: VCE English Question Thread
Post by: XD10602 on October 24, 2016, 10:12:42 pm
Hey guys!
I was fleshing out this prompt for Stasiland: 'Stasiland suggests that the more difficult to walls to break down are the ones that remain in the minds of those that lived through East Germany's history. ' But I couldn't think of any counter argument. Does anyone have any ideas or do I not need to include one for this prompt?
Thanks in advance!
Title: Re: VCE English Question Thread
Post by: klippo on October 24, 2016, 10:13:57 pm
Hi there,

I was just wondering how important it is to have 4 body paragraphs? I've been hearing from a couple teachers that having 4 bps is essential to getting to that 9-10 range. Can anyone confirm/debunk this claim?

Most of the time, I can only think of 3 main ideas and usually can only write 3 bps within the time limit, unless I'm super familiar with the topic, then 4 is possible, but a little risky.

Thanks in advance!!
Title: Re: VCE English Question Thread
Post by: klippo on October 24, 2016, 10:25:52 pm
Also, wondering whether anyone could critique these topic sentences? And possibly ways to make it more complex? Also not sure if it's fully unpacking the prompt:

Burial Rites is about Agnes being restored to humanity. Discuss.

Contention: Agnes' humanity is irrefutable, however is unacknowledged in the eyes of the society. However, through the regaining of her personal autonomy by being able to tell her own story, her humanity is accepted by those who listen to her. Thus, Burial Rites is about the acknowledgement of Agnes' humanity, rather than her restoration to humanity.

1. Kent asserts the existence of Agnes' humanity through demonstrating the extremities of emotions that Agnes experiences.
2. However, Kent illustrates how her humanity is unacknowledged by the unfeeling and prejudicial society in which she inhabits.
3. Nevertheless, Kent elucidates how her humanity is ultimately recognized by those who are the audience to her life's narrative.

Thank you!!
Title: Re: VCE English Question Thread
Post by: BNard on October 24, 2016, 10:28:25 pm
So my plan of attack for context (my worst section) at this stage is to go for a creative expository hybrid kind of thing. I do a 200 ish word introduction, usually from the perspective of Galileo (my text is obviously LoG) or Brecht, and then go into an expository, and then have a quick creative conclusion

With that kind of structure, would you say it's better to have a more free flowing discussion of ideas (throwing up examples and contrasting/comparing with examples from text plus a bit of loose conceptual discussion) or try and still have three distinct ideas (paragraphs) about the prompt in an obvious, topic sentency kind of way.

Ps does anyone else feel nauseous just thinking about Wednesday....
Title: Re: VCE English Question Thread
Post by: Swagadaktal on October 24, 2016, 10:48:56 pm
Ps does anyone else feel nauseous just thinking about Wednesday....
Yes, very.

Like that's the exact word I'd describe it.
Title: Re: VCE English Question Thread
Post by: Alter on October 24, 2016, 11:28:07 pm
I was told quotes are a MUST?? If I do not use quotes, how do i use examples from the text to support my ideas?
Quotes are essentially non-negotiable in Section A (Text Response). However, they are not needed at all to get a good/perfect mark in Section B. There is no checklist of criteria that assessors read off when looking at your Section B piece. Further, it wouldn't make much sense to throw in quotes from the book if you had an imaginative piece, would it?

If your teacher is telling you that quotes are mandatory for Section B, then maybe that's just the way they prefer you to do it in SACs/on the exam. I can't undermine whoever has told you this, as there may be a reason. However, it's entirely possible to write a 20/20 piece without quoting the book; it's just an option if you choose to do an expository essay.

You can refer to examples from the text by simply just mentioning, much the same way that you would if you were using another historical/social example in your essay. Quotes aren't needed if I try to bring up 20th century American politics, so why should they be here? Refer to the main ideas and overarching messages that you can ascertain from a text or external source and work those into your writing.

Hey guys!
I was fleshing out this prompt for Stasiland: 'Stasiland suggests that the more difficult to walls to break down are the ones that remain in the minds of those that lived through East Germany's history. ' But I couldn't think of any counter argument. Does anyone have any ideas or do I not need to include one for this prompt?
Thanks in advance!
You don't necessarily need a 'counter-argument' to address more than one aspect of the prompt. Similarly, it's unrealistic and virtually impossible to cover all aspects of a prompt, so don't concern yourself with that.

In this case, it doesn't really make sense to talk about the converse of the prompt, or else you'd be running the argument that physical walls are the centrepiece of the text, and that their structural integrity is really relevant to Funder's message.

Hint: Try refer to the idea of ''Mauer im Kopf'' (literally 'head in the wall') or Ostalgie. It's also worth considering the prompt from the perspective of both victims and perpetrators.

Hi there,

I was just wondering how important it is to have 4 body paragraphs? I've been hearing from a couple teachers that having 4 bps is essential to getting to that 9-10 range. Can anyone confirm/debunk this claim?

Most of the time, I can only think of 3 main ideas and usually can only write 3 bps within the time limit, unless I'm super familiar with the topic, then 4 is possible, but a little risky.

Thanks in advance!!
Not vital at all. It's better to have three excellent body paragraphs that all work together than to have four slightly above average ones. Many students who achieve great scores use only three body paragraphs, but still have enough quantity of a given quality in their writing to make it work in this manner.

Keep in mind that just because someone is writing four paragraphs doesn't mean all of them are good. Just try to aim for the three best ideas you can, and if you find you have time for one more great one, go for the forth. The only problem you'll have here is when your number drops below three, as that might be indicative of you not writing enough or having paragraphs that are too long.

Hope this helps.
Title: Re: VCE English Question Thread
Post by: FatimaEl on October 24, 2016, 11:36:02 pm
Hi there,

I was just wondering how important it is to have 4 body paragraphs? I've been hearing from a couple teachers that having 4 bps is essential to getting to that 9-10 range. Can anyone confirm/debunk this claim?

Most of the time, I can only think of 3 main ideas and usually can only write 3 bps within the time limit, unless I'm super familiar with the topic, then 4 is possible, but a little risky.

Thanks in advance!!

I think we can only think of 3 because weve been taught that way for a long time, but honestly all you got to do is write a general introduction that doesnt explore your ideas in too much detail just in case you end up having enough time to write 4 bps, as im pretty sure the assessors would like to see more ideas if you're good with generating them and explaining them of course.

The upper-range essays that are found in the examination reports normally have 4+ paragraphs, so i'd say that kinda confirms that claim.

But of course, writing well beats all  ;) ;D
Title: Re: VCE English Question Thread
Post by: Mirna.Kula on October 24, 2016, 11:54:10 pm
In the language analysis section of the exam, roughly how many persuasive techniques do we need to include for each article?
Title: Re: VCE English Question Thread
Post by: flickybree123 on October 25, 2016, 01:12:23 am
*This is probably the wrong place to ask but this is the first time I've used this site sorry*

Can someone please tell me where I can find or send me moderatley good to pretty decent "The White Tiger" text responses? I can't find ANY.

Plz im really struggling.
Title: Re: VCE English Question Thread
Post by: call.me.blueberry on October 25, 2016, 03:21:08 am
Hey guys,

some questions about
* Section C:
1/ TONE: In the introduction when I identify the predominant tone, do I have to analyse it? And how to analyse tone (both the overall tone and the tone that shifts/adopts during the body paragraphs too), like how it is taken to persuade the reader?
2/ Can anyone please give me an example of xenophobic or scapegoating tone? Sometimes I find the writer/speaker alienate a particular group of audience by portraying them as [... some negative stuff... that contradicts...] but I'm not sure what tone are they adopting.

* Section A:
I've been searching throughout the AN English board about unpacking prompts for weeks now. But I'm still not really what is the difference between approaching a "zoom-in" prompt (character, structure (?)) and a "big-picture" prompt (v-v, themes). Can sb please give me a quick recap on this?

Thank you so much guys and good luck tmr :)
Title: Re: VCE English Question Thread
Post by: literally lauren on October 25, 2016, 07:47:20 am
Hey lauren just wanted your opinion on something.

for LA in my intros, i have just usually been doing a brief context sentence, followed by the contentions of the authors with their tones occasionally.

Then i move straight into my body paras is this fine ?

Thanks  :)
Yep, that's absolutely fine! :)

Hi Lauren,


My teacher says that my creative context pieces do not meet the criteria for a link to the ideas of text; however, they fulfill all the other criterion to a 9 0r 10 standard; despite the fact that included 5 quotes from the text and discussed the ideas of the text and used them as a framework for the piece? She says that the ideas from the text are not discussed  in depth. Is my teacher correct or  is  the incorporation of quotes and a link to the ideas sufficient in terms of a link to the text?
You'd probably be ticking the box for a basic text link, but part of the expectation for a 10/10 is that you somehow (i.e. explicitly for expository pieces or implicitly for creatives/hybrids) cover what the text has to say about the prompt. So incorporating your answer to that question (i.e. 'what does the set text say about this prompt?') should help you hit that 10/10 territory :)

Hey!
So I was looking at some context essays on the submissions board and got super jealous of them cool titles they got there   8)
I know that it is completely optional, but is there any particular benefits to having a title for a creative piece ?
Yep, totally optional, but it can be useful to make a good first impression on your assessor as a nice, snappy, relevant-to-the-prompt title might make a decent starting point for you to set up the direction of your ideas. Some assessors are partial to it too, but if you can't come up with a good one on the spot, it's by no means expected/compulsory :)

Hey Lauren.

For expository essays, do you recommend knowing one really well and adapting it? Do you recommend having spare paragraphs or ideas as backup?
I'm a big fan of back up options just in case; in fact, rather than rote learning a single piece, if you familiarise yourself with a bunch of potential sub-arguments and potential examples, then you can just cherry pick the best ones and apply them to whatever prompt you're dealt :)

Can someone please help me with this Brooklyn prompt?
How does Toibin's understated, economical writing style influence the reader's understanding of characters and settings of Brooklyn?
What paragraphs do you do for these types of prompts asking about the language used??
This kind of prompt would be very unlikely as it's not really in keeping with VCAA' style, but if you do get something structural or about how Toibin does something (e.g. 'How does Toibin show the differences between Enniscorthy and Brooklyn in the novel?') then you can reword it into a prompt that's a bit easier to handle. Then, within your body paragraphs, you bring up evidence that stems from the language (or the metaphors, or the setting, or the juxtaposition, or w/e structural feature is in the prompt).

Hello,
I am really worried about the text response. For Brooklyn do you have to write what techniques Toibin uses to support the prompt?

Thanks in advance!!
Referencing techniques can help, but it's not essential. You need to use textual evidence to support your contention, which will be based on the prompt. But you don't analyse techniques in the same way as you would for Language Analysis. Using metalanguage to describe what Toibin does can be useful though, so it depends which evidence you're drawing from :)

I can't remember quotes. What is the best way to find remember quotes for the exam? WHICH IS IN 2 DAYS OMG

I am doing 2 texts: Every Man In This Village Is A Liar by Megan Stack and Media.
You don't need quote for Every Man, so don't worry about that one. Talking about the ideas in that text in terms of conflict is sufficient. But for Medea, you definitely need quotes. Luckily, that play's quite short, so it shouldn't take you long to go through the ~40 page text and pick out what's important. And as Alter said, aim to find quotes that can be grouped under a list of core themes so that you can handle a variety of prompts (e.g. revenge, justice/justification, emotion, etc.)

Hey guys!
I was fleshing out this prompt for Stasiland: 'Stasiland suggests that the more difficult to walls to break down are the ones that remain in the minds of those that lived through East Germany's history. ' But I couldn't think of any counter argument. Does anyone have any ideas or do I not need to include one for this prompt?
Thanks in advance!
You don't really need counter-arguments. Instead, you can question the prompt by asking 'why is this the case?' or 'what does this tell us about ____' (then fill in the blank with a key idea from the prompt, e.g. 'what does this tell us about breaking down walls' or 'what does this tell us about the past?') That would let you discuss how, though mental barriers may have derived from physical ones, ultimately it's these psychological remnants that have the greater effect on people.

Hi there,

I was just wondering how important it is to have 4 body paragraphs? I've been hearing from a couple teachers that having 4 bps is essential to getting to that 9-10 range. Can anyone confirm/debunk this claim?

Most of the time, I can only think of 3 main ideas and usually can only write 3 bps within the time limit, unless I'm super familiar with the topic, then 4 is possible, but a little risky.

Thanks in advance!!
(http://colleendilen.com/wp-content/uploads/2013/11/Screen-Shot-2013-11-20-at-8.07.15-AM.png)

3 decent paragraphs > 4 mediocre ones any day!
So if it's easier for you to come up with three strong ideas, or you find yourself stretched for time and can't commit to a fourth one, just focus on making those three as good as you can - provided you're hitting a minimum of three, you wouldn't be automatically excluded from certain marks/bandwidths at all :)

Also, wondering whether anyone could critique these topic sentences? And possibly ways to make it more complex? Also not sure if it's fully unpacking the prompt:

Burial Rites is about Agnes being restored to humanity. Discuss.

Contention: Agnes' humanity is irrefutable, however is unacknowledged in the eyes of the society. However, through the regaining of her personal autonomy by being able to tell her own story, her humanity is accepted by those who listen to her. Thus, Burial Rites is about the acknowledgement of Agnes' humanity, rather than her restoration to humanity.

1. Kent asserts the existence of Agnes' humanity through demonstrating the extremities of emotions that Agnes experiences.
2. However, Kent illustrates how her humanity is unacknowledged by the unfeeling and prejudicial society in which she inhabits.
3. Nevertheless, Kent elucidates how her humanity is ultimately recognized by those who are the audience to her life's narrative.

Thank you!!
Those are excellent topic sentences! The contention is worded a little strangely w/ some repetition in phrasing, but overall it's very clear. And the three concepts you've chosen to unpack here are definitely relevant. I like the use of linking words too - it seems like you should be pretty confident for Text Response :)

So my plan of attack for context (my worst section) at this stage is to go for a creative expository hybrid kind of thing. I do a 200 ish word introduction, usually from the perspective of Galileo (my text is obviously LoG) or Brecht, and then go into an expository, and then have a quick creative conclusion

With that kind of structure, would you say it's better to have a more free flowing discussion of ideas (throwing up examples and contrasting/comparing with examples from text plus a bit of loose conceptual discussion) or try and still have three distinct ideas (paragraphs) about the prompt in an obvious, topic sentency kind of way.

Ps does anyone else feel nauseous just thinking about Wednesday....
Totally up to your preferred writing style. You're under no obligation to conform to the standard 3-4 body paragraph rules for context pieces, so you could just conduct a more general discussion and see where that takes you. Or, perhaps the structure will give you some sense of direction/clarity, so you could use it if you wanted to. I'd probably lean more towards a free-flow discussion just to make your piece stand out from all the typical essays, but again, do whatever suits your writing style!

In the language analysis section of the exam, roughly how many persuasive techniques do we need to include for each article?
It depends on the material. If you look over the VCAA past exams or some of this year's practice exams, you'll see that there are all sorts of possible combinations of articles/visuals. In general, if you have fewer than ~4 in each of your body paragraphs, assessors will probably notice. But remember that you can always unpack things like connotations or suggestions if you don't want to use a specific technique as your starting point. And remember to go beyond techniques as well - the assessors don't just want to see you point them out, they want to see you analyse them!

*This is probably the wrong place to ask but this is the first time I've used this site sorry*

Can someone please tell me where I can find or send me moderatley good to pretty decent "The White Tiger" text responses? I can't find ANY.

Plz im really struggling.
Here's one! We don't have many since it's not studied at very many schools - sorry! Maybe message some of your peers and trade essays today?

Hey guys,

some questions about
* Section C:
1/ TONE: In the introduction when I identify the predominant tone, do I have to analyse it? And how to analyse tone (both the overall tone and the tone that shifts/adopts during the body paragraphs too), like how it is taken to persuade the reader?
2/ Can anyone please give me an example of xenophobic or scapegoating tone? Sometimes I find the writer/speaker alienate a particular group of audience by portraying them as [... some negative stuff... that contradicts...] but I'm not sure what tone are they adopting.

* Section A:
I've been searching throughout the AN English board about unpacking prompts for weeks now. But I'm still not really what is the difference between approaching a "zoom-in" prompt (character, structure (?)) and a "big-picture" prompt (v-v, themes). Can sb please give me a quick recap on this?

Thank you so much guys and good luck tmr :)
1. Addressing the tone in your intro can be good, but it's not really worth any marks. Whereas, if you bring it up in body paragraphs and treat it like any other language features (i.e. analyse it, provide evidence, talk about its effect) then you can get marks, so try and do that at least once in your essay.
2. I'm not sure the writer is ever looking to segregate or disparage the readers - he's trying to persuade them, right? So he might be trying to demonise a certain group of people, but he'd be doing that to encourage the audience to reject that group, not to yell at the audience and say 'you people are dumb idiots!' I've attached a sheet to this post that has some tone vocab that might be useful for you :)
3. If a prompt seems quite narrow in its focus or mentions specific characters, then it's a ''zoomed in'' one, e.g. 'Jason is responsible for Medea's suffering. Discuss.' If it's really broad and uses the author's name or the text title, it's probably a views and values one, e.g. 'Euripides' Medea explores what it means to feel betrayed. Discuss.' This can change your focus slightly, but if you don;t find the distinction helpful then don't worry about it! :)
Title: Re: VCE English Question Thread
Post by: Coffee on October 25, 2016, 07:49:50 am
I know I'm going to need to edit my essays and I'm planning to do this in the last 20 minutes of the exam, but is it worth it if I don't manage to finish my last essay (Section C- most likely). I timed myself last night and I didn't manage to get in a conclusion within the 60 minutes although I averaged about 770 words. So I guess my question is, is it better to have three polished but unfinished essays, or three finished essays that could do with some cleaning up concerning vocabulary, minor expression issues, consistency, etc?
Title: Re: VCE English Question Thread
Post by: literally lauren on October 25, 2016, 07:53:38 am
I know I'm going to need to edit my essays and I'm planning to do this in the last 20 minutes of the exam, but is it worth it if I don't manage to finish my last essay (Section C- most likely). I timed myself last night and I didn't manage to get in a conclusion within the 60 minutes although I averaged about 770 words. So I guess my question is, is it better to have three polished but unfinished essays, or three finished essays that could do with some cleaning up concerning vocabulary, minor expression issues, consistency, etc?
Hmm, I'd probably lean more so to three polished-but-unfinished ones since ultimately, what the assessors are looking for is an insight into your skillset, and that's going to put you in better stead than three essays that are complete, but very flawed. That said, how much editing do your pieces need?? 20 minutes seems excessive to me, so perhaps while writing you could take note of any vocab/expression issues and try and fix some as you go? If you know editing time benefits you, then go for it, but if it means finishing everything way too early and potentially compromising quality, then you might be better off admitting that maybe the expression wasn't 100% perfect, but at least you communicated all your ideas effectively :)

edit: also, "last essay (Section C - most likely)" ...are you sure? :/ I'd advise starting with that one for efficiency's sake!
Title: Re: VCE English Question Thread
Post by: MandhreeE on October 25, 2016, 08:16:03 am
Hi Lauren,
I know you're probably really busy but i was just wondering if you got my Context piece that i PM to you a couple of days ago. It's totally fine if you are too busy i just needed a second opinion on it that's all.
Thank you. :)
Title: Re: VCE English Question Thread
Post by: literally lauren on October 25, 2016, 08:18:37 am
Hi Lauren,
I know you're probably really busy but i was just wondering if you got my Context piece that i PM to you a couple of days ago. It's totally fine if you are too busy i just needed a second opinion on it that's all.
Thank you. :)
Yep; I'll get there! Prioritising essays and questions on the forums at the moment, but I'll get to messages soon! :)
Title: Re: VCE English Question Thread
Post by: HasibA on October 25, 2016, 08:42:50 am
Super simple q; but I seem to forget a lot of the word by word quotes in my text (it's a film, Mabo) - would I be penalized if  most, or all, my textual evidence is in the form of example and straight techniques etc. Eg instead of mentioning a specific quote like 'perkins use of the traditional Meriam music blah blah does this, instilling "eddis is nothing but a trouble maker" - would I be penalized for saying something like - 'perkins selectively uses the traditional Meriam music, alongside a close up of Mabos facial features when Mabo displays indignation as a result of not being successful in the initial supreme court trial. Etc. And then analysing like so ?
I find I mainly talk about 'perkins and how she does things for most prompts, or if it's a really specific prompt - just my reasons supporting / not supporting the prompt I.e in a do you agree prompt?
Ive pretty much done this use of 'non specific evidence' all year and my teachers have been okay with it - but now I'm a little worried haha. I'm aiming for a mid range piece for text, would it still be possible if I only had like 1-2 quotes per paragraph, but most of my evidence was a film technique, discussion of a specific scene?

Hope I made sense- thanks guys !!
Title: Re: VCE English Question Thread
Post by: blacksanta62 on October 25, 2016, 08:59:47 am
Yep, that's absolutely fine! :)
You'd probably be ticking the box for a basic text link, but part of the expectation for a 10/10 is that you somehow (i.e. explicitly for expository pieces or implicitly for creatives/hybrids) cover what the text has to say about the prompt. So incorporating your answer to that question (i.e. 'what does the set text say about this prompt?') should help you hit that 10/10 territory :)
Hey Lauren, how do we do this? I'm having some trouble trying to apply it and was hoping you could just give an example. My context is I&B and texts are Summer of the Seventeenth Doll and Invictus.
Also, in my essays this year I've been quoting the book i.e. "In the Country of Men" instead of underlining. My teacher said that this was accecptable. Can I keep this in the exam or should I underline?
Thank you
Title: Re: VCE English Question Thread
Post by: HopefulLawStudent on October 25, 2016, 09:16:37 am
Would this sort of prompt breakdown work?

"We all have abnormality in common. We're a breed apart form the rest of humanity, we theatre folk; we are the original displaced personalities."
What is it that links the characters of All About Eve?


- Their role playing links all the characters in AAE.

- Their shared ambition to “be somebody” other than who they are links them

- However, it is how they enact these ambitions that ultimately divide and distinguish them.

Hence, Mankiewicz compels his viewer to recognise that the thespians in the film are both “a breed apart” and similar. Through this, he ultimately delivers a commentary on the relationship between the “Ivory Green Room… call[ed] the Theatre” and the broader macrocosm, indicating that like the characters in the film, these two entities are both a faithful reflection of one another and a means of escapism.
Title: Re: VCE English Question Thread
Post by: HopefulLawStudent on October 25, 2016, 09:34:21 am
The society portrayed in All About Eve is depicted as fundamentally superficial and driven purely by the self-interest of members.
To what extent do you agree?


1. Superficial in that the theatre and the world of the thespians is ultimately merely an illusion, a pretense and a deception.

2. It is a superficiality that is at times maintained by these characters out of sheer self-interest.

3. And yet… it’s not. This superficiality is driven by this 1950s patriarchal society that forced women to adhere to a certain gendered role and did not allow them the freedom to be themselves.

Conclusion: Hence, at its heart, the society portrayed in AAE is driven to maintain an illusion out of not just the self-interest of its members but also the rigidity of this social structure. The inflexibility of this society means that “a woman’s career” is confined to the confines of an apartment even if her “native habitat” and her “heart” was in the “Ivory Green Room… call[ed] the Theatre”. To this end, Mankiewicz ultimately seeks to challenge and repudiate these restrictive gendered roles, advocating for a reevaluation in order to proffer women liberty from the restrictive traditional gender roles and discover for themselves what it means to “be… a woman.”

Would ^ conclusion be okay? Would that last sentence be considered too clumsily put? How could I make it more crisp/concise??
Title: Re: VCE English Question Thread
Post by: GillyAnt on October 25, 2016, 09:41:28 am
Hi Lauren, do you reckon there's a chance there'll be a prompt about family in identity in belonging? The trend seems to be steering more to change and self-growth since Summer of the Seventeenth Doll came in, and I feel as though as Summer doesn't really link to the idea of family well. But I suppose it's worth preparing for curveballs?
Title: Re: VCE English Question Thread
Post by: Manige123 on October 25, 2016, 09:46:20 am
Hey Lauren, I'm not sure if you'll be able to help, but what external examples would you recommend looking at for internal/intellectual conflict for context: encountering conflict.
Title: Re: VCE English Question Thread
Post by: HopefulLawStudent on October 25, 2016, 09:51:25 am
Sorry for the question spam but...

Quote
The tensions between the theatre and Hollywood mirror the conflicts between characters throughout All About Eve. Discuss.

How would I deal with ^ prompt?? I don't know how to break it down/structure an essay around it...

Also, how do I use desist in a sentence? Can desist and stop/cease be used as direct synonyms or nah?
Title: Re: VCE English Question Thread
Post by: P.GUAN on October 25, 2016, 10:04:56 am
Hello guys and girls,

What actually happens if you get a study score of below 20 for English? Does that mean you have failed VCE and you need to take it again?

Thanks
Title: Re: VCE English Question Thread
Post by: literally lauren on October 25, 2016, 10:26:06 am
Super simple q; but I seem to forget a lot of the word by word quotes in my text (it's a film, Mabo) - would I be penalized if  most, or all, my textual evidence is in the form of example and straight techniques etc. Eg instead of mentioning a specific quote like 'perkins use of the traditional Meriam music blah blah does this, instilling "eddis is nothing but a trouble maker" - would I be penalized for saying something like - 'perkins selectively uses the traditional Meriam music, alongside a close up of Mabos facial features when Mabo displays indignation as a result of not being successful in the initial supreme court trial. Etc. And then analysing like so ?
I find I mainly talk about 'perkins and how she does things for most prompts, or if it's a really specific prompt - just my reasons supporting / not supporting the prompt I.e in a do you agree prompt?
Ive pretty much done this use of 'non specific evidence' all year and my teachers have been okay with it - but now I'm a little worried haha. I'm aiming for a mid range piece for text, would it still be possible if I only had like 1-2 quotes per paragraph, but most of my evidence was a film technique, discussion of a specific scene?

Hope I made sense- thanks guys !!
Try to memorise a few direct quotes; you need to use them alongside your discussion of moments in the film or certain cinematic devices, so rote-learning quotes can be your revision exercise for the day! (Watching the film again might also help get them to stick in your head!) The discussion you've got there would be really good, but assessors will pick up on the fact that you have very few quotes, and the stringent ones might penalise you for it, so try and focus on memorising a couple just to be on the safe side.

Hey Lauren, how do we do this? I'm having some trouble trying to apply it and was hoping you could just give an example. My context is I&B and texts are Summer of the Seventeenth Doll and Invictus.
Also, in my essays this year I've been quoting the book i.e. "In the Country of Men" instead of underlining. My teacher said that this was accecptable. Can I keep this in the exam or should I underline?
Thank you
To work this out, take an Id&b prompt and ask 'what does SotSD say about this prompt?' e.g. if the prompt is 'our identity comes from those around us,' does the text support or challenge this idea? How? Why? etc. <-- embed that idea in your piece, and you should be fine.

Think of it like the text offering you some kind of starting point for your contention; you shouldn't be arguing against what your set text is saying about the context/prompt - you should be using it as your springboard for other ideas! :)

And underlining the text title for T.R. is the standard convention, so I'd stick with that in the exam. Save the "double quote marks" for actual quotes from the novel :)

Would this sort of prompt breakdown work?

"We all have abnormality in common. We're a breed apart form the rest of humanity, we theatre folk; we are the original displaced personalities."
What is it that links the characters of All About Eve?


- Their role playing links all the characters in AAE.

- Their shared ambition to “be somebody” other than who they are links them

- However, it is how they enact these ambitions that ultimately divide and distinguish them.

Hence, Mankiewicz compels his viewer to recognise that the thespians in the film are both “a breed apart” and similar. Through this, he ultimately delivers a commentary on the relationship between the “Ivory Green Room… call[ed] the Theatre” and the broader macrocosm, indicating that like the characters in the film, these two entities are both a faithful reflection of one another and a means of escapism.

This is absolutely fine. Might be good to hint at the idea you're bringing up in P3 in your intro if that is to be your overarching interpretation, but that breakdown seems solid :)

The society portrayed in All About Eve is depicted as fundamentally superficial and driven purely by the self-interest of members.
To what extent do you agree?


1. Superficial in that the theatre and the world of the thespians is ultimately merely an illusion, a pretense and a deception.

2. It is a superficiality that is at times <-- what do you mean by this exactly? maintained by these characters out of sheer self-interest.

3. And yet… it’s not. This superficiality is driven by this 1950s patriarchal society that forced women to adhere to a certain gendered role and did not allow them the freedom to be themselves.

Conclusion: Hence, at its heart, the society portrayed in AAE is driven to maintain an illusion out of not just the self-interest of its members but also the rigidity of this social structure. The inflexibility of this society means that “a woman’s career” is confined to the confines of an apartment even if her “native habitat” and her “heart” was in the “Ivory Green Room… call[ed] the Theatre”. To this end, Mankiewicz ultimately seeks to challenge and repudiate these restrictive gendered roles, advocating for a reevaluation in order to proffer women liberty from the restrictive traditional gender roles and discover for themselves what it means to “be… a woman.”

Would ^ conclusion be okay? Would that last sentence be considered too clumsily put? How could I make it more crisp/concise??
All good! Be careful with the 'P3 twist' where you pivot to your wider point as you don't want this to seem like you're doubling back on what you've said earlier, but rather that you're building upon it to say something more complex. Conclusion and concluding line are also fine; I'd probably cut the "advocating for a reevaluation" part just because I'm not sure you could state that this is definitely what Mankie's angling for, but the use of the quote is totally fine :)

Hi Lauren, do you reckon there's a chance there'll be a prompt about family in identity in belonging? The trend seems to be steering more to change and self-growth since Summer of the Seventeenth Doll came in, and I feel as though as Summer doesn't really link to the idea of family well. But I suppose it's worth preparing for curveballs?
Family's way too narrow. Some of the other texts  wouldn't work for that at all, so I highly doubt they'd be that mean. But you might get something about 'people closest to us' or 'the people in our lives,' which would let you talk about family if you wanted to :)

Hey Lauren, I'm not sure if you'll be able to help, but what external examples would you recommend looking at for internal/intellectual conflict for context: encountering conflict.
What do you mean by 'intellectual' conflict, exactly? For internal conflict, a nice generic one would be to look at cognitive dissonance, though I'm hesitant to suggest any more specific ones since I don't know which other examples you're using and I don't want to give you one that clashes or doesn't fit. Are there any that you're considering at the moment that I could maybe help refine or clarify? :)

Sorry for the question spam but...
The tensions between the theatre and Hollywood mirror the conflicts between characters throughout All About Eve. Discuss.
How would I deal with ^ prompt?? I don't know how to break it down/structure an essay around it...

Also, how do I use desist in a sentence? Can desist and stop/cease be used as direct synonyms or nah?
What are the tensions between the theatre and Hollywood (hint: old vs. new) and what are the tensions between the characters (see previous hint)? Probably safest to mostly agree here and consider how these tensions are similar, since this is a slightly structural prompt. Then zoom out to the broader point of 'what is Mank. saying about these tensions?' Does that help?

'Desist' is pretty much interchangeable with 'stop' and 'cease' - I'm trying to think of exceptions but they seem like pretty close synonyms to me. But words like 'prevent' or 'refrain' might let you be more specific, if you wanted some alternatives :)

Hello guys and girls,

What actually happens if you get a study score of below 20 for English? Does that mean you have failed VCE and you need to take it again?

Thanks
You need a 25 to pass, so failing to get a 25 would mean you would not receive a VTAC offer for any courses that listed a 25 in English as a prerequisite. But if you're applying for courses that don't require this, or you're not looking to go on to tertiary study next year, you don't have to repeat Year 12. You can also achieve English qualifications at other institutions if you'd prefer. Though I believe that you'll still be grated a high school completion certificate even if you end up with a <30 ATAR.

But let's hope you don't have to worry about any of that!
Title: Re: VCE English Question Thread
Post by: cookb111 on October 25, 2016, 10:28:05 am
"To what extent is the creature in Frankenstein a reflection of Victor himself?"

Reflection mean what in this context ? opposite (like a graph in maths) or the same (like a mirror image) 
Title: Re: VCE English Question Thread
Post by: literally lauren on October 25, 2016, 10:32:35 am
"To what extent is the creature in Frankenstein a reflection of Victor himself?"

Reflection mean what in this context ? opposite (like a graph in maths) or the same (like a mirror image)
Very good question! You could talk about both, but I think the latter would be the expected interpretation; as in, the creature exhibits many of the same desires and flaws as Victor does. Definitely worth coming up with some evidence for that one if you haven't written about it already, as it's a very central concern of the novel and a great overarching reading to build towards :)
Title: Re: VCE English Question Thread
Post by: HasibA on October 25, 2016, 10:37:49 am
thanks heaps lauren! i seem to have about only 5-10 memorised, and im aimin for like 20ish for today- would roughly 3ish direct quotes and a lot of 'non specific evidence' been plenty, or do i still need more ? :/ kinda annoyed my teachers didn't emphasise this.
Title: Re: VCE English Question Thread
Post by: HasibA on October 25, 2016, 10:40:25 am
sorry to double post. but i forgot to ask- what is the rule about the 'multimodal text' for section A/B? im using the film MABO for text, and every man in this village is a liar for context- is that ok? my context will be a creative piece
Title: Re: VCE English Question Thread
Post by: Dem16 on October 25, 2016, 10:43:48 am
What structure would you use for a Language Analysis if you are asked to analyse an interview?
Title: Re: VCE English Question Thread
Post by: GillyAnt on October 25, 2016, 11:01:50 am
Also, one last thing.

Do you feel like it's a bit difficult to interpret Mankiewicz's views and values for All About Eve? He holds up the injustices females suffer in the industry and in society, but you could hardly say he challenges this view.

Plus, it's hard to tell with the whole Hollywood v Theatre debate, he obviously was fascinated with the theatre, but do you think he was challenging the elitism commonly seen?
Title: Re: VCE English Question Thread
Post by: n.a on October 25, 2016, 11:17:16 am
This is a Burial Rites related question:

What is the symbolism behind Kent's usage of wind and the sense of being underwater as a motif?

So for example, "only the wind speaks and it will not talk sense", "a cold wind passes through you as though you are not there..." and "there are ravens in the sky but what bird flies underwater", "we are all underwater and i cannot swim", "if i spesk, it will bein bubbles of air".

What is the significance of this?
Title: Re: VCE English Question Thread
Post by: HopefulLawStudent on October 25, 2016, 11:19:59 am
Thanks Lauren!

ii. How does the film show the barriers between men and women in the theatre?

^^ How would you tackle this prompt and break it down??? I don't know how I could structure an essay around this...
Title: Re: VCE English Question Thread
Post by: HopefulLawStudent on October 25, 2016, 11:21:03 am
Also

In All About Eve, there are many different ways for a relationship to fracture. Do you agree?

How many different ways would you explore in your essay?
Title: Re: VCE English Question Thread
Post by: Bumpstead on October 25, 2016, 11:28:33 am
Does any one know where i could find writing paper the same as we get in the english exam?
Title: Re: VCE English Question Thread
Post by: HopefulLawStudent on October 25, 2016, 11:31:01 am
Every conflict involves both an internal and an external struggle.

How would I springboard from this prompt and broaden it out???

Does any one know where i could find writing paper the same as we get in the english exam?

http://www.vcaa.vic.edu.au/Documents/exams/english/2016/18762-VCE%20English%202016_FINAL_12072016.pdf
Title: Re: VCE English Question Thread
Post by: powergab on October 25, 2016, 11:32:17 am
Hi!
I was just wondering for a prompt that talks about how relationships impact on your identity, like  'without connection to others, there is no me' would these three paragraphs for an expository essay be okay

1. connection to parents, they are around in our most formative years and our values and beliefs are largely gained from them
2. our identity continues to change throughout our lives through relationships we gain and lose in a lifetime
3. however there are some integral parts of our identity that can not be changed in anyway by those around us, such as race.

My teacher isn't sure about the third paragraph and said it doesn't really keep to the prompt but I just wanted to check whether other people think that too, I talk about white privilege etc and how these aspects of identity that are not altered by relationships are a large influence on your life and a really big impact on your identity and who you are, and then just talk about how Duggan is affected by his race in regards to his identity and views etc. because my book is Wild Cat Falling.

Thanks!!
Title: Re: VCE English Question Thread
Post by: HopefulLawStudent on October 25, 2016, 11:37:46 am
Conflict and injustice arise from the human rive for power over others.

What's this prompt saying?
Title: Re: VCE English Question Thread
Post by: HasibA on October 25, 2016, 11:42:59 am
Conflict and injustice arise from the human rive for power over others.

What's this prompt saying?
not sure what rive could mean in this context- but i think its conflict occurs due to humans having a predisposition so split themselves/ segregate themselves in order to become powerful/ maintain power?
sorry if i didnt help :/ hard prompt ahha
Title: Re: VCE English Question Thread
Post by: HopefulLawStudent on October 25, 2016, 11:45:25 am
Maybe. Rive is throwing me off a bit. :P

The most significant divisions in any community are not ethical or moral, but personal animosities, insecurities and greed.

^^There are so many aspects in this prompt... and not enough time to fully hash them all out. Could I sorta just treat ethical = moral ad personal animosities = insecurities = greed? Or would that be too simplistic?
Title: Re: VCE English Question Thread
Post by: HopefulLawStudent on October 25, 2016, 11:48:00 am
Conflict can only be resolved when there is an agreed version of events.

and

Conflict is inevitable, the way we deal with it is not.

^^ What big ideas are these two prompts dealing with? What's the second prompt saying?
Title: Re: VCE English Question Thread
Post by: happymeal on October 25, 2016, 01:25:20 pm
Just wondering if its okay to draw upon two of the set texts I've learnt as examples in a context expository essay, or whether I should just stay focused on one text throughout.
e.g. In one paragraph I might use the ideas from text A
In the next I might use ideas from text B
Title: Re: VCE English Question Thread
Post by: HasibA on October 25, 2016, 01:31:36 pm
Just wondering if its okay to draw upon two of the set texts I've learnt as examples in a context expository essay, or whether I should just stay focused on one text throughout.
e.g. In one paragraph I might use the ideas from text A
In the next I might use ideas from text B
i dont think this is allowed- on the front of your answer booklet, it says something not to use a 'multimodal' text for section b, that was used in section 'a'- so im not sure!
Title: Re: VCE English Question Thread
Post by: happymeal on October 25, 2016, 01:35:38 pm
i dont think this is allowed- on the front of your answer booklet, it says something not to use a 'multimodal' text for section b, that was used in section 'a'- so im not sure!

Oops, I meant to mean, would it be okay to use ideas from both Foe and Wag the Dog (not from the actual Section A)
Sorry for not being clear
Title: Re: VCE English Question Thread
Post by: HasibA on October 25, 2016, 01:38:21 pm
Oops, I meant to mean, would it be okay to use ideas from both Foe and Wag the Dog (not from the actual Section A)
Sorry for not being clear
i dont think (not sure)  u can use ideas from both, bc you have to write your 'main text' on the top, so try not to use ideas evenly between the two; pretty sure yo have to maintain one main text- but double check w/ others on here :)
Title: Re: VCE English Question Thread
Post by: melissaromeo on October 25, 2016, 01:51:36 pm
For encountering conflict, I'm writing a persuasive opinion piece, is it okay to focus it all on one big broad idea such as divorce or the ferguson riots case and then extract my ideas from that? and then throughout link it to a separation?
Title: Re: VCE English Question Thread
Post by: FatimaEl on October 25, 2016, 01:52:18 pm
Just wondering if its okay to draw upon two of the set texts I've learnt as examples in a context expository essay, or whether I should just stay focused on one text throughout.
i dont think (not sure)  u can use ideas from both, bc you have to write your 'main text' on the top, so try not to use ideas evenly between the two; pretty sure yo have to maintain one main text- but double check w/ others on here :)
You are allowed to use both texts, just one has to be the main and written at the top of section B while the other can be used a little bit throughout your piece
"Your writing must draw directly from at least one selected text that you have studied for this Context" (from VCAA)

However, if one of your texts from section A is already multimodal (mabo, AAE) then you cant write on a multimodal text in section B (for example Invictus).
"If a student writes on a multimodal text in Section A of the examination, they must not write on a multimodal text as their main text in
Section B. Films are multimodal texts."
(also from VCAA)

Title: Re: VCE English Question Thread
Post by: Mellyboo on October 25, 2016, 01:52:21 pm
Hi!
I was just wondering for a prompt that talks about how relationships impact on your identity, like  'without connection to others, there is no me' would these three paragraphs for an expository essay be okay

1. connection to parents, they are around in our most formative years and our values and beliefs are largely gained from them
2. our identity continues to change throughout our lives through relationships we gain and lose in a lifetime
3. however there are some integral parts of our identity that can not be changed in anyway by those around us, such as race.

My teacher isn't sure about the third paragraph and said it doesn't really keep to the prompt but I just wanted to check whether other people think that too, I talk about white privilege etc and how these aspects of identity that are not altered by relationships are a large influence on your life and a really big impact on your identity and who you are, and then just talk about how Duggan is affected by his race in regards to his identity and views etc. because my book is Wild Cat Falling.

Thanks!!

I'm also not sure about the last one... While your actual race i.e the colour of your skin might not be changed, you can still identify with a race other than your own. A prime example of this is in the film 'skin'. The protagonist constantly changed the race they identified with...
You are right though, SOME things might not be influenced. But I don't think the race you identify with is one of them.
Title: Re: VCE English Question Thread
Post by: dhvani on October 25, 2016, 01:54:02 pm
For those who are doing 'Medea', What can you talk about in the theam Manipulation?
Thanks heaps
Title: Re: VCE English Question Thread
Post by: literally lauren on October 25, 2016, 02:03:15 pm
thanks heaps lauren! i seem to have about only 5-10 memorised, and im aimin for like 20ish for today- would roughly 3ish direct quotes and a lot of 'non specific evidence' been plenty, or do i still need more ? :/ kinda annoyed my teachers didn't emphasise this.
3 in each paragraph might get you over the line, but try and memorise a wide variety for a bunch of different themes just to ensure you have your bases covered.

sorry to double post. but i forgot to ask- what is the rule about the 'multimodal text' for section A/B? im using the film MABO for text, and every man in this village is a liar for context- is that ok? my context will be a creative piece
Yep, you're all good. You're just not allowed to write on films for both Sec. A and B (e.g. All About Eve + A Separation or Mabo + Invictus)

What structure would you use for a Language Analysis if you are asked to analyse an interview?
Same structure I'd recommend for any material; group by arguments and transition between the different authors' depiction of certain ideas :)

Also, one last thing.

Do you feel like it's a bit difficult to interpret Mankiewicz's views and values for All About Eve? He holds up the injustices females suffer in the industry and in society, but you could hardly say he challenges this view.

Plus, it's hard to tell with the whole Hollywood v Theatre debate, he obviously was fascinated with the theatre, but do you think he was challenging the elitism commonly seen?
Yeah, AAE has some surprisingly complex views and values, but I think you can argue things either way, and you may even get a prompt revolving around what things Mank. is condoning/condemning. I don't really want to give you one interpretation because I'd rather not panic a bunch of people by inferring that this'd be the only way to argue things, so instead, I'd recommend coming to some conclusion either way and ensuring you can substantiate your position - that should be sufficient to get you through any potential V&V prompts :)

This is a Burial Rites related question:

What is the symbolism behind Kent's usage of wind and the sense of being underwater as a motif?

So for example, "only the wind speaks and it will not talk sense", "a cold wind passes through you as though you are not there..." and "there are ravens in the sky but what bird flies underwater", "we are all underwater and i cannot swim", "if i spesk, it will bein bubbles of air".

What is the significance of this?
Significance changes with context, but in general, BR has a lot of examples of pathetic fallacies (something that this essay handles really well) so consider how some of these natural symbols/metaphors could be used as indictments of the characters' state of mind. Also, ideas of submergence or feelings of deprivation and abandonment that are present in the quotes you've outlined are pretty clear links to Agnes' turmoil, so that's probably the easiest way to interpret things.

Thanks Lauren!

ii. How does the film show the barriers between men and women in the theatre?

^^ How would you tackle this prompt and break it down??? I don't know how I could structure an essay around this...
What are the barriers between men and women in the theatre in AAE? Start there. If in doubt, jump for evidence first and try to work backwards. Are there things that the men are capable of (or that they can do without having to suffer many social consequences) that the women can't get away with? Are there things that the men can acquire or assume that the women can't? Are there barriers that the women have to try and surmount that the men don't have to worry about? And are there gaps between the lives/values/intentions/desires of the men and women in the film? Hopefully that'll give you some starting points here :)

Also

In All About Eve, there are many different ways for a relationship to fracture. Do you agree?

How many different ways would you explore in your essay?
As many as you want ;)

Sorry for the vague response, but it really does depend on what ground you want to cover and whether you'd rather go for depth v. breadth. But within each different 'way' for fracturing to occur, you could talk about multiple relationships, so maybe three or four different 'ways' (i.e. one per paragraph) with multiple examples for each?

Every conflict involves both an internal and an external struggle.

How would I springboard from this prompt and broaden it out???
Can internal struggles affect external ones? Or vice versa? Or both? (Hint: it's both!) What does this say about conflict? Is the internal struggle the source of external ones? Which one affects us more? Which one is more telling/meaningful/important/impactful?

Just keep questionin'! :)

Hi!
I was just wondering for a prompt that talks about how relationships impact on your identity, like  'without connection to others, there is no me' would these three paragraphs for an expository essay be okay

1. connection to parents, they are around in our most formative years and our values and beliefs are largely gained from them
2. our identity continues to change throughout our lives through relationships we gain and lose in a lifetime
3. however there are some integral parts of our identity that can not be changed in anyway by those around us, such as race.

My teacher isn't sure about the third paragraph and said it doesn't really keep to the prompt but I just wanted to check whether other people think that too, I talk about white privilege etc and how these aspects of identity that are not altered by relationships are a large influence on your life and a really big impact on your identity and who you are, and then just talk about how Duggan is affected by his race in regards to his identity and views etc. because my book is Wild Cat Falling.

Thanks!!
Be careful not to base your arguments solely on one example (e.g. 'yes, because family' or 'no, because race'). The ideas you've outlined here seem solid, but you might find it useful to spend the first couple of sentences in each paragraph just fleshing out your idea before delving into examples to support your stance :)

Conflict and injustice arise from the human rive for power over others.

What's this prompt saying?

The most significant divisions in any community are not ethical or moral, but personal animosities, insecurities and greed.

^^There are so many aspects in this prompt... and not enough time to fully hash them all out. Could I sorta just treat ethical = moral ad personal animosities = insecurities = greed? Or would that be too simplistic?
People want power, therefore conflict and injustice. 'Rive' is a really arcane word to use in this context though :P And that second prompt would probably be too specific for VCAA since it doesn't work for all four texts. But if you get overwhelmed by key terms, then boiling them down to their essence is a smart way to approach things.

Conflict can only be resolved when there is an agreed version of events.

and

Conflict is inevitable, the way we deal with it is not.

^^ What big ideas are these two prompts dealing with? What's the second prompt saying?
1. We can't resolve conflict if people don't see things the same way. Hence, conflict stems from a clash of perspectives, and will persist unless these perspectives can be unified/'agreed upon.'

2. The way we respond to conflict is more important than why it happens.

Just wondering if its okay to draw upon two of the set texts I've learnt as examples in a context expository essay, or whether I should just stay focused on one text throughout.
e.g. In one paragraph I might use the ideas from text A
In the next I might use ideas from text B
You can use both, but you still have to nominate a 'main text drawn upon,' so try to concentrate on one for the most part rather than talking about them both in equal measure.

For encountering conflict, I'm writing a persuasive opinion piece, is it okay to focus it all on one big broad idea such as divorce or the michael ferguson case and then extract my ideas from that? and then throughout link it to a separation?
Absolutely! Just make sure you can create at least one or two overt links to the set text, and maybe use that spark/example as a way into talking about other things (e.g. from divorce to the cultural significance of marriage and tradition, or the Michael Ferguson to racism and the justice system more broadly, etc.)

For those who are doing 'Medea', What can you talk about in the theam Manipulation?
Thanks heaps
Start by thinking about examples of manipulation in the text. For example, would you say Jason manipulates Medea? How about Medea manipulating Aeges? From there, you can build out to thinking about what the author is saying about this theme[/b] (i.e. what is Euripides saying about manipulation based on this evidence?) :)
Title: Re: VCE English Question Thread
Post by: JellyBeanz on October 25, 2016, 02:11:55 pm
Is it necessary to mention the visual aids accompanying a language analysis piece in the intro?
Title: Re: VCE English Question Thread
Post by: literally lauren on October 25, 2016, 02:16:30 pm
Is it necessary to mention the visual aids accompanying a language analysis piece in the intro?
Nope! Totally optional. You can acknowledge them briefly if you want; some people even give an overview of what they're depicting/suggesting/contending, but it's totally up to you :)

MAKE SURE YOU ANALYSE THE VISUALS IN YOUR BODY PARAGRAPHS THOUGH!!!
Title: Re: VCE English Question Thread
Post by: oliviacaon on October 25, 2016, 02:21:07 pm
What are some outside sources for my context essay for whose reality related to death of a salesman?
Title: Re: VCE English Question Thread
Post by: literally lauren on October 25, 2016, 02:26:03 pm
What are some outside sources for my context essay for whose reality related to death of a salesman?
The idea of the American Dream is a good place to start; it's a little bit well-worn as a lot of students will discuss this, but it relates to a whole plethora of other concepts like ambition, hope, expectation etc. which in turn can be used as a way into other examples. I also reckon Donald Trump's entire campaign is like one big manipulation of people's realities - like, he's unwittingly done VCE WR students a massive favour by running for present this year because the past year has been like one big case study of clashing subjective realities :P (this vid explains it quite well)

But it depends on what you're using so far - if all your examples are very philosophical/psychological, then you may prefer to delve more so into those theories or concepts, whereas if you prefer historical/political stuff, then that should be what you focus on. If you want, let me know what you've started exploring and I might be able to give you some more specific recommendations :)
Title: Re: VCE English Question Thread
Post by: HopefulLawStudent on October 25, 2016, 02:26:59 pm
Should we throw in textual references early for context?
Title: Re: VCE English Question Thread
Post by: literally lauren on October 25, 2016, 02:33:36 pm
Should we throw in textual references early for context?
Sooner rather than later is preferred, just so that the assessors can see that you're starting from the text (as a "springboard") and working out to bigger, broader ideas, vs. them waiting till the end to discern the connection :)
Title: Re: VCE English Question Thread
Post by: Daniellac on October 25, 2016, 02:38:41 pm
NEED HELP!

I'm always saying, in Language Analysis, the author APPEALS to... or ATTEMPTS to ....

Are there any other ways of saying what they are trying to do as I know I can't say they MAKE the reader feel something!!

Thankyou in advance!
Title: Re: VCE English Question Thread
Post by: FatimaEl on October 25, 2016, 02:39:28 pm
Somewhat irrelevant to the discussions happening at the moment, but this forum has reached
100k reads!!!
Right before the english exam  ;D
Title: Re: VCE English Question Thread
Post by: KDB on October 25, 2016, 02:44:09 pm
Somewhat irrelevant to the discussions happening at the moment, but this forum has reached
100k reads!!!
Right before the english exam  ;D

1k of them are from me tbh
Title: Re: VCE English Question Thread
Post by: literally lauren on October 25, 2016, 02:45:52 pm
NEED HELP!

I'm always saying, in Language Analysis, the author APPEALS to... or ATTEMPTS to ....

Are there any other ways of saying what they are trying to do as I know I can't say they MAKE the reader feel something!!

Thankyou in advance!
See attachment!

Also, it might be worth listing a bunch of alternate words and phrases - look them up in a thesaurus now so you have them at your disposal tomorrow!

Somewhat irrelevant to the discussions happening at the moment, but this forum has reached
100k reads!!!
Right before the english exam  ;D

(https://m.popkey.co/cf7623/Q8VDJ.gif)

1k of them are from me tbh
Dw, I'm at least 50k myself :'D
Title: Re: VCE English Question Thread
Post by: FatimaEl on October 25, 2016, 03:01:01 pm
Hi Lauren!
In your 10 things to do before the exam guide you mention listing themes. do i just briefly list them, or do i write in detail with prompts and all?
Title: Re: VCE English Question Thread
Post by: GillyAnt on October 25, 2016, 03:01:54 pm
Yeah, AAE has some surprisingly complex views and values, but I think you can argue things either way, and you may even get a prompt revolving around what things Mank. is condoning/condemning. I don't really want to give you one interpretation because I'd rather not panic a bunch of people by inferring that this'd be the only way to argue things, so instead, I'd recommend coming to some conclusion either way and ensuring you can substantiate your position - that should be sufficient to get you through any potential V&V prompts :)
You're a gun! Cheers! I know you don't want to give too much of your opinion but would you agree that Mankiewicz does give merit to different forms of medium (Going off Bill's long monologue about the 'theatuh').
Title: Re: VCE English Question Thread
Post by: literally lauren on October 25, 2016, 03:05:53 pm
Hi Lauren!
In your 10 things to do before the exam guide you mention listing themes. do i just briefly list them, or do i write in detail with prompts and all?
Listing single words would be okay as a brainstorm, or you can turn them into bigger categories with synonyms for each theme and lists of prompts and quotes/examples too - depends what you feel like you need to work on :)

You're a gun! Cheers! I know you don't want to give too much of your opinion but would you agree that Mankiewicz does give merit to different forms of medium (Going off Bill's long monologue about the 'theatuh').
Absolutely; and if you got a theatre-based prompt (which hasn't directly cropped up in the past two years, so could be a possibility this time?) that'd be a great way of teasing out some more complex views and values assertions :)
Title: Re: VCE English Question Thread
Post by: Elizawei on October 25, 2016, 03:13:51 pm
Hey Lauren,
What's a good/recommended number of quotes per paragraph? You said the bare minimum was around 3, is there a threshold where it becomes too excessive?


Thanks
Title: Re: VCE English Question Thread
Post by: smarkov8 on October 25, 2016, 03:22:49 pm
Hey, just a quick question for the english exam tomorrow would you be able to write a newsletter in Section B however using columns etc and not get marked down?
Title: Re: VCE English Question Thread
Post by: literally lauren on October 25, 2016, 03:24:37 pm
Hey Lauren,
What's a good/recommended number of quotes per paragraph? You said the bare minimum was around 3, is there a threshold where it becomes too excessive?


Thanks
Three would definitely be the bare minimum; probably aim for around five if you want to be safe. There's not really a maximum, though obvs if you're just inserting multiple quotes in every single sentence without analysing them, you'll probably end up compromising your ideas, and thus compromising your mark. But if you're mixing up the use of examples with actual discussion, you should be fine.

Hey, just a quick question for the english exam tomorrow would you be able to write a newsletter in Section B however using columns etc and not get marked down?
Yep, you definitely can, I've had a couple of students do this in the past and they never ran into any problems with it. Though make sure it's for a good reason and you're not just doing it for the 'novelty' factor or anything :P
Title: Re: VCE English Question Thread
Post by: larissaaa_ on October 25, 2016, 03:33:05 pm
"The worst affront to Medea is the desecration of her pride" Do you agree?

"Conflict stretches back into the past as well as into the future".

Can someone please help break these prompts down into arguments for me? How do I get three arguments for each, I've literally been avoiding these prompts for so long because they were so difficult.
Title: Re: VCE English Question Thread
Post by: Biology24123 on October 25, 2016, 03:39:13 pm
In TR, is it better to have 1 example with through discussion or two examples that are more brief
Title: Re: VCE English Question Thread
Post by: HasibA on October 25, 2016, 03:44:45 pm
being pragmatic here- i dont think i can remember a huge amount of quotes for MABO by tmr, lets say a write an essay of like 6-7/10, but do not have a huge selection of quotes/even modify-almost make up- a quote or two, and i have a pretty strict/smart assessor who catches on to that- how much would i be penalised? im thinking maybe a drop of a mark. Most of my evidence will be film technique based/example based.
thanks to whoever replies <3
Title: Re: VCE English Question Thread
Post by: Mhysa on October 25, 2016, 04:47:33 pm
Hi guys :)
Can someone tells me what quotes I can insert in this paragraph relating to how photographs serve as a means to authenticate the events depicted in MAUS, since I heard it is vital to have more than two quotes in each body paragraph.

Photographs serve as an authentication of the story as they illustrate that the Holocaust did indeed happen and that the characters within ‘The Complete MAUS’ are real people. The inclusion of three “human” photographs authenticates the story   by offering “analogical images of ‘real’ human beings. These photographs corroborate the truth of the story being told. The second photo in the graphic novel is of Richieu, Art’s older brother. To see a picture of a small child who was killed during the war add even more to the seriousness of the events depicted. Even though Richieu appears early in the narrative drawn by Spiegalman as a small mouse, the actual photograph of Richieu reinforces and illustrates how real the events of 1930s are. Furthermore, the last photograph in ‘The Complete MAUS’, Vladek’s souvenir photo illustrates the reality of the holocaust as it depicts Vladek wearing a camp uniform. The “human” Vladek appears shockingly real, in contrast to his minimalist ‘mouse form throughout the text. The photographs serve as reminders of the reality of Vladek’s story, pulling the reader back from any fantasy world that reading Maus might have created.
Title: Re: VCE English Question Thread
Post by: sarah2403 on October 25, 2016, 05:21:11 pm
heyy so with discuss prompts like Medea is a cautionary tale about being consumed by emotion. Discuss. would it be ok to use the following ideas for an essay
- The play Medea warns against allowing emotions to consume you
   - eg. character of Medea - emotion clouds reason - kills children
- The tale warns against allowing emotions to rule your decisions
   - eg. character of Jason - ambition rules decision - loses everything
- Medea also serves as a warning against inflicting patriarchal views
   - eg. views of women allow Medea to succeed bc she can manipulate men
   - eg. Medea's anger at role fuels her desire for revenge

thanks :)
Title: Re: VCE English Question Thread
Post by: literally lauren on October 25, 2016, 06:29:00 pm
"The worst affront to Medea is the desecration of her pride" Do you agree?

"Conflict stretches back into the past as well as into the future".

Can someone please help break these prompts down into arguments for me? How do I get three arguments for each, I've literally been avoiding these prompts for so long because they were so difficult.
What exactly were you having trouble with?
And don't worry, they were meant to be super hard, and even the simple act of thinking about them puts you ahead of a vast number of people around the state who only ever even think about the easy stuff!

In TR, is it better to have 1 example with through discussion or two examples that are more brief
Try to mix it up between the two. It can depend on the prompt as well; sometimes a 'depth-y' approach is more useful than a 'breadth-y' one.

being pragmatic here- i dont think i can remember a huge amount of quotes for MABO by tmr, lets say a write an essay of like 6-7/10, but do not have a huge selection of quotes/even modify-almost make up- a quote or two, and i have a pretty strict/smart assessor who catches on to that- how much would i be penalised? im thinking maybe a drop of a mark. Most of my evidence will be film technique based/example based.
thanks to whoever replies <3
It's okay to draw from techniques too, but try and mix it up with at least a couple of quotes per paragraph - hopefully that's achievable no matter what the prompt is.

Hi guys :)
Can someone tells me what quotes I can insert in this paragraph relating to how photographs serve as a means to authenticate the events depicted in MAUS, since I heard it is vital to have more than two quotes in each body paragraph.
Try to go back to the text and find quotes that support your assertions; there's not much point in me suggesting random bits of evidence to you! Ask yourself how do I know when you're making those assertions about what the text means or what the characters think/feel, and that should help you find some quotes to draw from :)

heyy so with discuss prompts like Medea is a cautionary tale about being consumed by emotion. Discuss. would it be ok to use the following ideas for an essay
- The play Medea warns against allowing emotions to consume you
   - eg. character of Medea - emotion clouds reason - kills children
- The tale warns against allowing emotions to rule your decisions
   - eg. character of Jason - ambition rules decision - loses everything
- Medea also serves as a warning against inflicting patriarchal views
   - eg. views of women allow Medea to succeed bc she can manipulate men
   - eg. Medea's anger at role fuels her desire for revenge
thanks :)
Definitely! Just be careful with your final paragraph as that might come across as a bit off-topic unless you can work in the idea of emotion in there too. But overall, that should give you a nice breadth of points to discuss! :)
Title: Re: VCE English Question Thread
Post by: HopefulLawStudent on October 25, 2016, 07:58:09 pm
Aaaaand I'm reading the dictionary again.  ::) ::) ::)

Could you please check I'm using the underlined words correctly?

For the 2014 VCAA:

For Laikis' contention: Could we have said that Laikis is seeking to negate the support Yergon may have engendered by derisively attack her and her support for space exploration as delusional?
Title: Re: VCE English Question Thread
Post by: literally lauren on October 25, 2016, 08:01:59 pm
Aaaaand I'm reading the dictionary again.  ::) ::) ::)

Could you please check I'm using the underlined words correctly?

For the 2014 VCAA:

For Laikis' contention: Could we have said that Laikis is seeking to negate the support Yergon may have engendered by derisively attacking her and her support for space exploration as delusional?
- undermines, challenging, or invalidating would also work as synonyms for 'negate' in this context
- derisively is a great word to characterise Laikis' tone in this case! :)
Title: Re: VCE English Question Thread
Post by: HopefulLawStudent on October 25, 2016, 08:04:41 pm
Thanks Lauren

Also:

Would "prevaricate" be too evaluative for LA?
Title: Re: VCE English Question Thread
Post by: P.GUAN on October 25, 2016, 08:07:14 pm
Hi Lauren,
For Section C should I use 2 language techniques per paragraph?

Thanks!!
Title: Re: VCE English Question Thread
Post by: literally lauren on October 25, 2016, 08:09:51 pm
Thanks Lauren

Also:

Would "prevaricate" be too evaluative for LA?
I swear I've used it once or twice, but in hindsight, it probably came across as evaluative, yeah. Maybe stick with a safer verb like 'suggests' and just use a tonal adverb to modify it if you find something that seems like an example of prevarication. Unless you get a piece where the author is condemning someone else for prevaricating, in which case you can use it :P Best to stick with safe alternatives for main verbs though; always play to your strengths in the exam! :)

Hi Lauren,
For Section C should I use 2 language techniques per paragraph?

Thanks!!
Aim for more than two, if possible. 2 per paragraph would only give you, like six overall, unless you plan on writing a heap of body paragraphs? And you'd need to incorporate the visual too, which also counts as analysis.

Remember that you can talk about other things like CONNOTATIONS too! :)
Title: Re: VCE English Question Thread
Post by: knightrider on October 25, 2016, 08:10:10 pm
This question is being asked for a friend.

first he asked me i havent done any timed pieces at all for english, but he said he knows the things from his text/ context.
He asked me how worried he should be about time?
i said if he knows everything good enough he should be able to handle the time constraints fine.

He wanted other peoples opinion on this what do you guys think ?
Title: Re: VCE English Question Thread
Post by: EllaD on October 25, 2016, 08:12:04 pm
hey guys,

so i just came across something on vce discussion space that said you shouldn't talk about 2 texts in context? i have been doing this all year and have been getting a+'s so i'm kind of confused, do assessors really care if you use one or two texts ?????

thanks :)
Title: Re: VCE English Question Thread
Post by: P.GUAN on October 25, 2016, 08:14:39 pm
Hi Lauren,

So if I am writing about connotations in section C,  can I dedicate one whole body paragraph to connotations?

Thanks again!!
Title: Re: VCE English Question Thread
Post by: HopefulLawStudent on October 25, 2016, 08:17:59 pm
Lauren

For language analysis

HOW: language technique
WHY: author's intention
SO: effect on reader

Right?

What's the diff between WHY and SO?
Title: Re: VCE English Question Thread
Post by: literally lauren on October 25, 2016, 08:20:56 pm
This question is being asked for a friend.

first he asked me i havent done any timed pieces at all for english, but he said he knows the things from his text/ context.
He asked me how worried he should be about time?
i said if he knows everything good enough he should be able to handle the time constraints fine.

He wanted other peoples opinion on this what do you guys think ?


Tbh, not having done any time trials whatsoever might put your friend at a slight disadvantage as he potentially won't have found certain trouble-spots that might affect him in the exam. But you shouldn't tell him anything that's going to freak him out and make him think that he should do a whole 3 hour exam tonight or anything dumb like that :P Maybe encourage him to do a quick 15 min. reading time exercise just to put his mind at ease?

hey guys,

so i just came across something on vce discussion space that said you shouldn't talk about 2 texts in context? i have been doing this all year and have been getting a+'s so i'm kind of confused, do assessors really care if you use one or two texts ?????

thanks :)
You need to nominate a "main text drawn upon," so whilst it's totally fine if you're using two, you need you need to have one, official 'primary' text :)

Hi Lauren,

So if I am writing about connotations in section C,  can I dedicate one whole body paragraph to connotations?

Thanks again!!
It's best to talk about connotations alongside the rest of your analysis. Maybe try and have one bit of connotative discussion + three techniques in each paragraph, and possibly a bit of the visual if you really need a formula.

Lauren

For language analysis

HOW: language technique
WHY: author's intention
SO: effect on reader

Right?

What's the diff between WHY and SO?
I'd say

WHAT: language technique
HOW: effect on reader
WHY: link to argument // SO: link to purpose <-- these are basically one and the same

"Why" = 'why would the author want to position readers in this way, or portray this idea as ____?'
"So" = 'so... why would the author...' etc. :P

Just different terminology that different teachers employ sometimes to make our lives difficult ::)
Title: Re: VCE English Question Thread
Post by: larissaaa_ on October 25, 2016, 08:21:52 pm
What exactly were you having trouble with?
And don't worry, they were meant to be super hard, and even the simple act of thinking about them puts you ahead of a vast number of people around the state who only ever even think about the easy stuff!

So for the Medea one, I've literally been staring at it for days and cannot come up with three arguments for it. I've reworded it to "the worst 'insult' to Medea is the 'violation' of her pride", I don't know if that's right but I still can't seem to think of three paragraphs for it!

And the conflict one I'm assuming it's talking about how the history of the conflict can also shape any future conflicts experienced?
Title: Re: VCE English Question Thread
Post by: HopefulLawStudent on October 25, 2016, 08:26:42 pm

Tbh, not having done any time trials whatsoever might put your friend at a slight disadvantage as he potentially won't have found certain trouble-spots that might affect him in the exam. But you shouldn't tell him anything that's going to freak him out and make him think that he should do a whole 3 hour exam tonight or anything dumb like that :P Maybe encourage him to do a quick 15 min. reading time exercise just to put his mind at ease?
You need to nominate a "main text drawn upon," so whilst it's totally fine if you're using two, you need you need to have one, official 'primary' text :)
It's best to talk about connotations alongside the rest of your analysis. Maybe try and have one bit of connotative discussion + three techniques in each paragraph, and possibly a bit of the visual if you really need a formula.
I'd say

WHAT: language technique
HOW: effect on reader
WHY: link to argument // SO: link to purpose <-- these are basically one and the same

"Why" = 'why would the author want to position readers in this way, or portray this idea as ____?'
"So" = 'so... why would the author...' etc. :P

Just different terminology that different teachers employ sometimes to make our lives difficult ::)

What do you mean by link to argument???

I've been doing LA intuitively all year but apparently exam stress is screwing up my intuition... :(
Title: Re: VCE English Question Thread
Post by: HopefulLawStudent on October 25, 2016, 08:29:51 pm
Also: can exhorts and encourage be used interchangeably??
Title: Re: VCE English Question Thread
Post by: Just4Funzies on October 25, 2016, 08:31:23 pm
How would you structure the interview in the practise exam 2? I'm having trouble figuring out what would be analysed
Title: Re: VCE English Question Thread
Post by: Kimahso on October 25, 2016, 08:40:18 pm
For LA, would I lose marks for not using paragraphs at all (Unless it's two pieces in which case I would use two)? I discuss the techniques chronologically as they are in the piece and sometimes one technique can appear in different sections of the piece (for example in the e-book LA from 2012 the author emphasises that she's open to change both at the beginning and at the end of the piece).
Title: Re: VCE English Question Thread
Post by: jessamyh16 on October 25, 2016, 08:42:47 pm
could someone please tell me what exactly will the examiners be looking for in a context (imaginative)
Title: Re: VCE English Question Thread
Post by: literally lauren on October 25, 2016, 08:49:05 pm
So for the Medea one, I've literally been staring at it for days and cannot come up with three arguments for it. I've reworded it to "the worst 'insult' to Medea is the 'violation' of her pride", I don't know if that's right but I still can't seem to think of three paragraphs for it!

And the conflict one I'm assuming it's talking about how the history of the conflict can also shape any future conflicts experienced?
That's alright, the Medea one was quite left-of-field; consider what you could say about the different kinds of challenges Medea faces and the different wrongs that she endures. The prompt kind of hints that they can all be traced back to her pride (e.g. the reason she's so upset that Jason betrayed her was that it undermined her sense of self, etc.) so that should give you somewhere to start.

For the conflict one, yes, definitely, you could look at it from a variety of perspectives. Though I don't think anything like that would crop up in the exam since it wouldn't really suit the set texts.

What do you mean by link to argument???

I've been doing LA intuitively all year but apparently exam stress is screwing up my intuition... :(
As in, why would the author do this and how would it help their argument? (like linking things back to the key player, though it's more like an optional step - you don't have to do this for every single point of analysis)

Also: can exhorts and encourage be used interchangeably??
'exhorts' is a little stronger; 'exhorts' is more like 'implores,' whereas 'encourages' would be a more generic, mild version.

How would you structure the interview in the practise exam 2? I'm having trouble figuring out what would be analysed
The host of the show didn't really have a contention, so you can leave him out. But you'd want to deal with both of the guests and their contentions, so the easiest structure would probably be to pick out three core concepts that they both talk about (e.g. the respectfulness of young people) and then talk about how they both position that concept (approx half a paragraph on each) - does that make sense? :)

For LA, would I lose marks for not using paragraphs at all (Unless it's two pieces in which case I would use two)? I discuss the techniques chronologically as they are in the piece and sometimes one technique can appear in different sections of the piece (for example in the e-book LA from 2012 the author emphasises that she's open to change both at the beginning and at the end of the piece).
Your analysis will probably be stronger if you can find some sub-arguments to talk about.
See: this guide

Assessors wouldn't be keen on a wall of text with no paragraphs or breaks though, so try and divy up the material if you can. Also, the fact that one technique can appear multiple times is kind of a reason why you should AVOID tackling things chronologically since you may end up repeating yourself. But that's a good back-up option in case you get some really tough material and just want to try and work through the analysis :)

could someone please tell me what exactly will the examiners be looking for in a context (imaginative)

That's a pretty general question... is there anything in particular you're unsure of or struggling with? :) There's some broad advice if you click on the links in the 'Context' section in this post - hope that helps!
Title: Re: VCE English Question Thread
Post by: Just4Funzies on October 25, 2016, 08:59:40 pm
cheers lauren :)
Title: Re: VCE English Question Thread
Post by: powergab on October 25, 2016, 09:33:10 pm

If the language analysis piece has a big article with a few comments or something in response, how do we structure the piece? I usually lean to sorting my paragraphs through the writers arguments, but if there are just a few little comments in response to the bigger article and they only really have one main contention would you just do one paragraph each of the comments? And then stick to 3 paragraphs based on arguments for the bigger article?

Thanks!!
Title: Re: VCE English Question Thread
Post by: literally lauren on October 25, 2016, 09:37:24 pm
If the language analysis piece has a big article with a few comments or something in response, how do we structure the piece? I usually lean to sorting my paragraphs through the writers arguments, but if there are just a few little comments in response to the bigger article and they only really have one main contention would you just do one paragraph each of the comments? And then stick to 3 paragraphs based on arguments for the bigger article?

Thanks!!
See: here

tl;dr: find some common arguments across the material (e.g. maybe the main piece + two of the comments are dealing with the concept of free speech; some for, some against, so group them together) and then discuss what each author is saying about that concept (e.g. 'Author X contends that free speech is a core tenet of democracy that must be upheld at all cost *insert analysis here*. Contrarily, Author Y challenges this assertion by implying that whilst free speech has its merits, it is also responsible for a great deal of suffering and should therefore be regulated... *insert analysis here*') <-- that way, you can just 'compare' by transitioning between the pieces, whilst still ensuring that analysis is your priority & that you're dealing with the material in sufficient depth :)
Title: Re: VCE English Question Thread
Post by: HasibA on October 25, 2016, 10:54:39 pm
quick checklist
- is it okay if my dictionary has my name on it? on the front page, and written on the side of the dictionary (in fairly big blue writing), is a new dictionary required? ahah
- not expected to know our student numbers, are we?
-clear water bottles up until 1.5 litres?
- do we need our student I.Ds if the invigilators can recognise us (ill be in uniform haha )
?
thank u
Title: Re: VCE English Question Thread
Post by: Alter on October 25, 2016, 11:12:58 pm
quick checklist
- is it okay if my dictionary has my name on it? on the front page, and written on the side of the dictionary (in fairly big blue writing), is a new dictionary required? ahah
Should be fine.
Quote
- not expected to know our student numbers, are we?
Nope.
Quote
-clear water bottles up until 1.5 litres?
Yep, unlabelled, clear and up to 1.5L.
Quote
- do we need our student I.Ds if the invigilators can recognise us (ill be in uniform haha )
Nah that isn't necessary.

If you're sitting the exam tomorrow: best of luck. Actually, if you're reading this, get some sleep! :P Last minute cramming won't help, but sleep will.
Title: Re: VCE English Question Thread
Post by: thuynh on November 01, 2016, 10:19:53 pm
Are there any resources for writing and sample comparative essays? Thanks!
Title: Re: VCE English Question Thread
Post by: Syndicate on November 05, 2016, 02:14:00 pm
Hey guys,

So... I have a 3-hour English unit 1/2 exam in the forthcoming week, and was wondering if someone can give me a general outline of how many pages I should aim to write in 1 hour (considering that I write between 8-12 words per sentence).

Thanks  :)
Title: Re: VCE English Question Thread
Post by: upandgo on November 05, 2016, 03:18:42 pm
Hey guys,

So... I have a 3-hour English unit 1/2 exam in the forthcoming week, and was wondering if someone can give me a general outline of how many pages I should aim to write in 1 hour (considering that I write between 8-12 words per sentence).

Thanks  :)

id say focus on the quality of your writing rather than the amount of pages you write  :) having written 1 or 1 + a 1/2 pages of an essay that is insightful and substantial (despite it being incomplete) would score much higher than writing 3-4 pages of an essay that's rushed and lacks detail. i ended up leaving my text response incomplete for a externally assessed practice exam at my school and received a 9 from an examiner- so i reckon the quality of what you write is more important than the amount of pages you complete.
Title: Re: VCE English Question Thread
Post by: HopefulLawStudent on November 05, 2016, 04:18:41 pm
Hey guys,

So... I have a 3-hour English unit 1/2 exam in the forthcoming week, and was wondering if someone can give me a general outline of how many pages I should aim to write in 1 hour (considering that I write between 8-12 words per sentence).

Thanks  :)

Enough to get your point across.**

**So long as your definition of "enough" is like 700+ words.
Title: Re: VCE English Question Thread
Post by: sqluxy on November 07, 2016, 08:34:20 pm
Hi,
Can an EAL student study English Language this subject instead of EAL?
I've studied Units 1/2 for EAL, can I change into Units 3/4 English Language next year? Will there be a lot of challenges?
Thanks xD
Title: Re: VCE English Question Thread
Post by: Alter on November 07, 2016, 08:45:19 pm
Hi,
Can an EAL student study English Language this subject instead of EAL?
I've studied Units 1/2 for EAL, can I change into Units 3/4 English Language next year? Will there be a lot of challenges?
Thanks xD

Unfortunately, it is assumed that English is your primary/first language if you're doing English Language, so the same leniency and understanding in marking that may be seen in EAL won't necessarily apply here. As a result, I've never really heard of anyone doing EL if they have the option to do EAL. I don't want to totally discourage you, because your English skills might be up to scratch or you could enjoy EL a lot more. I encourage you to chat with your English teacher or the head of English at your school. Best of luck!
Title: Re: VCE English Question Thread
Post by: drepwns on November 09, 2016, 07:56:00 pm
Hey guys,

So for attendance reasons, I have received an N for Unit 2 English; although averaging 90% for my SACs all year. I've been getting real worried as to the effect an N in english can have on my record - will it have much of an affect if at all ANY affect on my study score for English Units 3/4 next year or applications for Uni? Thanks
Title: Re: VCE English Question Thread
Post by: upandgo on November 09, 2016, 08:03:51 pm
Hey guys,

So for attendance reasons, I have received an N for Unit 2 English; although averaging 90% for my SACs all year. I've been getting real worried as to the effect an N in english can have on my record - will it have much of an affect if at all ANY affect on my study score for English Units 3/4 next year or applications for Uni? Thanks

i highly doubt it, i failed unit 2 methods and it had no impact on my study in units 3/4 (not sure if thats just for math) but i heard from my school that you can afford to fail either unit 1 or 2 of english and it wont adversely affect you in year 12  :)
Title: Re: VCE English Question Thread
Post by: Muchos Help on November 10, 2016, 06:43:02 pm
Is any good English student willing to mark my language analysis.

My (new) replacement teacher who had to fill in for the rest of my year for my sick teacher gave me a C for our LA analysis outcome. I've been averaging B+/A the entire year. The people who aren't good at english in my class got higher than me, but I had the same teacher in Sem 1 Year 10 when I was pretty bad at English. The new teache appears biased against me.

It also appears that she only looked at my intro and BP1 and gave me a mark since she wrote nothing for the rest of my essay (3 pages). Whereas she annotated all over the essays for the people who she marked properly.
Title: Re: VCE English Question Thread
Post by: HopefulLawStudent on November 10, 2016, 06:45:16 pm
Is any good English student willing to mark my language analysis.

My (new) replacement teacher who had to fill in for the rest of my year for my sick teacher gave me a C for our LA analysis outcome. I've been averaging B+/A the entire year. The people who aren't good at english in my class got higher than me, but I had the same teacher in Sem 1 Year 10 when I was pretty bad at English. The new teache appears biased against me.

It also appears that she only looked at my intro and BP1 and gave me a mark since she wrote nothing for the rest of my essay (3 pages). Whereas she annotated all over the essays for the people who she marked properly.

Sure. I'm willing; idk if I count as a "good English student" though considering the fact that I still have no credentials or study score to my name. :P

Is it on the submissions board??
Title: Re: VCE English Question Thread
Post by: hola on November 11, 2016, 02:17:54 pm
Hello! For the new study design, it there much change to the language analysis section? I'm assuming there is since they changed the name, but my teacher seems to be teaching it like the old design (i.e impact of language on audiences). If I'm correct, shouldn't there a more holistic approach to it; considering both argument and language?
Title: Re: VCE English Question Thread
Post by: Gogo14 on November 11, 2016, 09:55:39 pm
Hello! For the new study design, it there much change to the language analysis section? I'm assuming there is since they changed the name, but my teacher seems to be teaching it like the old design (i.e impact of language on audiences). If I'm correct, shouldn't there a more holistic approach to it; considering both argument and language?
Also just to add, is there any change for context as well?
Title: Re: VCE English Question Thread
Post by: hola on November 11, 2016, 10:19:36 pm
Also just to add, is there any change for context as well?

That's been replaced with comparative writing.
Title: Re: VCE English Question Thread
Post by: LPadlan on November 12, 2016, 08:03:55 pm
Hello, I am currently struggling with language analysis. My question is, when do we use argument techniques and when do we use persuasive techniques?
Title: Re: VCE English Question Thread
Post by: qazser on November 12, 2016, 08:44:00 pm
Hello, I am currently struggling with language analysis. My question is, when do we use argument techniques and when do we use persuasive techniques?

Would be lovely if you could post a sample essay or elaborate on your questions. Typically in a language analysis essay, you identify the technique and relate it back to how it persuades the audience and what it does to the audience.  Your essay does not involve you trying to persuade your examiner. Hope that clears things up.
Title: Re: VCE English Question Thread
Post by: thuynh on November 16, 2016, 08:33:17 pm
I am planning to read my English texts for next year now. Just wondering if I should be annotating or taking notes during my first read.
Title: Re: VCE English Question Thread
Post by: HopefulLawStudent on November 16, 2016, 08:43:38 pm
I am planning to read my English texts for next year now. Just wondering if I should be annotating or taking notes during my first read.

Do a dry reading (preferably in one sitting) where you just sit, read the text and get to know the plot points, the characters, etc. Then, if you're feeling up to it, start making annotations on key themes and other key details that you feel might be useful for future essays.
Title: Re: VCE English Question Thread
Post by: clarke54321 on November 16, 2016, 09:44:45 pm
Hi all,

This may seem like a straight forward question, but is it possible to explore complex ideas (well) in English with mid-high vocabulary skills.

Often I receive feedback from my teachers that while I discuss complex ideas and go into depth with themes, I lack a high range vocabulary to thoroughly execute this.

Thanks!
Title: Re: VCE English Question Thread
Post by: michael leahcim on November 18, 2016, 01:25:09 pm
Hi all,

This may seem like a straight forward question, but is it possible to explore complex ideas (well) in English with mid-high vocabulary skills.

Often I receive feedback from my teachers that while I discuss complex ideas and go into depth with themes, I lack a high range vocabulary to thoroughly execute this.

Thanks!

While it's good to have some good vocab under your belt, it's totally fine to explore complex ideas with an average vocabulary. Just be weary about your tone, and use simple but neatly compact sentences -- and you should be on your way to a forty. Not saying English is easy like that, but if you can explain those ideas without convoluted sentences and strange expression - I don't see why you can't do well.

Also on a side note with complex ideas: I think it's an issue a lot of students face when they write English essays that their ideas become too complex for the prompt. And examiners pick up on that, to the point where it seems like they're running off-track and regurgitating essays they've already previously written. This goes for literature as well, but for English (with particular emphasis), keep it simple, smart and sharp. You can pretty much write on the most basic ideas and pull off a 9/10 with just the way you express it.

A way to break down your ideas would be to just form links, since these links build up to the contention. If you consolidate those links, writing on any prompt should be easy breezy. Be very careful though. Vocabulary is good but from my own experience of trying to expand my vocab -- you might fall into the trap of using flamboyant flamingos in favour of the easy-to-understand good stuff. But definitely look into other ways of saying 'positions the reader' etc., because that stuff will help you with essay-writing.

Focus on the meat and how you can best prepare it before putting it on platter for the nasty examiners. Ideas are great, but only when they can be understood, and this goes for whatever vocabulary you have at your disposal.
Title: Re: VCE English Question Thread
Post by: Gogo14 on November 21, 2016, 02:33:14 pm
Anyone recommend any good workbooks for english next year? E.g. Checkpoints
Title: Re: VCE English Question Thread
Post by: Buddster on November 21, 2016, 03:53:27 pm
Anyone recommend any good workbooks for english next year? E.g. Checkpoints

as far as I know there isn't many workbooks as English is really only essay writing. There's some really great study material out there though. For your text analysis defs grab a study guide companion for your novel, like MetaMaus for Maus or whatever
Title: Re: VCE English Question Thread
Post by: hola on December 16, 2016, 09:22:21 am
Hi all,

I have a question about how I should approach this Medea prompt: "Medea is the true victim of Euphrides' play". Do you agree?

My contention is "Medea is a victim but not the only one of the play". My body paragraphs will each discuss why Medea, Jason and the children are victims.

My questions are:
- Do I address the prompt sufficiently? I'm a bit iffy about the "true" part since my discussion is based solely on one side of the story where the characters are victims, completely disregarding their actions that could make them seem more like villans than victims.
- Should I change the topics of my body paragraphs? I know it's generally advised to steer away from character-based paragraphs, but I feel like it would be okay in this case since the prompt itself is character based.
- If I should change my topics, would the following be okay:


Thanks in advance for all help :)
Title: Re: VCE English Question Thread
Post by: HopefulLawStudent on December 16, 2016, 09:45:36 am
Hi all,

I have a question about how I should approach this Medea prompt: "Medea is the true victim of Euphrides' play". Do you agree?

My contention is "Medea is a victim but not the only one of the play". My body paragraphs will each discuss why Medea, Jason and the children are victims.

My questions are:
- Do I address the prompt sufficiently? I'm a bit iffy about the "true" part since my discussion is based solely on one side of the story where the characters are victims, completely disregarding their actions that could make them seem more like villans than victims.
- Should I change the topics of my body paragraphs? I know it's generally advised to steer away from character-based paragraphs, but I feel like it would be okay in this case since the prompt itself is character based.
- If I should change my topics, would the following be okay:

  • Medea is a victim but she has victims of her own
  • All characters suffer at the hands of others (should I make this more specific by talking about the innocence of some characters?)
  • All characters fall prey to societal pressures

Thanks in advance for all help :)

I'm a little biased but I tend to not like the whole character by character paragraph breakdown so much which is what you seem to be gearing towards. However, your BP breakdown would probably fine (like my school actively encouraged the use of a character by character breakdown fr the simple reason of it was easier); personally, I'd want to throw in something about whether Medea really is a victim instead of basing your discussion "solely on the side of the story where the characters are victims". I mean there's nothing there to say that you can't talk about their actions.

I'm a bigger fan of the thematic or idea based paragraphs largely because I personally feel that it allows for a more cohesive and structured argument (for me personally, character based paras always ended in 3 - 4 paragraphs where I just reiterated the same thing idea over again whereas thematic based paras allowed for more exploration of the prompt).

I prefer what you're trying to achieve via the second prompt breakdown:

Quote
Medea is a victim but she has victims of her own
All characters suffer at the hands of others (should I make this more specific by talking about the innocence of some characters? Anything relevant is free game and can be explored in your response.)
All characters fall prey to societal pressures So? The connection between this idea and the prompt needs to be clarified so that no one can turn around and argue that it's irrelevant to the prompt.

Your questions are quite legitimate concerns, however, and it's so good that you're asking these questions now. Tbh I don't think I started asking these questions until like midyear so good on you! :)

If you feel any more questions, please feel free to ask. :)
Title: Re: VCE English Question Thread
Post by: hola on December 18, 2016, 10:48:57 am
I'm a little biased but I tend to not like the whole character by character paragraph breakdown so much which is what you seem to be gearing towards. However, your BP breakdown would probably fine (like my school actively encouraged the use of a character by character breakdown fr the simple reason of it was easier); personally, I'd want to throw in something about whether Medea really is a victim instead of basing your discussion "solely on the side of the story where the characters are victims". I mean there's nothing there to say that you can't talk about their actions.

I'm a bigger fan of the thematic or idea based paragraphs largely because I personally feel that it allows for a more cohesive and structured argument (for me personally, character based paras always ended in 3 - 4 paragraphs where I just reiterated the same thing idea over again whereas thematic based paras allowed for more exploration of the prompt).

I prefer what you're trying to achieve via the second prompt breakdown:

Your questions are quite legitimate concerns, however, and it's so good that you're asking these questions now. Tbh I don't think I started asking these questions until like midyear so good on you! :)

If you feel any more questions, please feel free to ask. :)

Hey HLS,

Thanks for the help. I redid my plan and is it okay if you look over it (and answer some questions?);

Contention: Euphrides is ambiguous in which character he suggests is the victim. (Is this okay? Or would something like "Within Medea, Euphrides explores the ideas of being a victim, and innocence".)

Argument 1:All characters suffer to some extent at the hands of others.
- For this argument, I would talk about how Medea suffers due to Jason's betrayal, but also due to her own extreme emotions (leading to moral repercussions). I would then talk about how Jason suffers due to the stuff expected of him (social pressures).

Argument 2: All characters however aren't completely innocent.
- Is this a strong argument? It's just that I could already come up with a counter argument for it, and that is the children; they're completely innocent.
- For this paragraph, I would essentially discuss the bad things the characters did, making them more villainous than victims.

- Would just these 2 body paragraphs suffice for an essay?

Thanks for all help.  :) :)
Title: Re: VCE English Question Thread
Post by: HopefulLawStudent on December 18, 2016, 01:34:33 pm
Hey HLS,

Thanks for the help. I redid my plan and is it okay if you look over it (and answer some questions?); I'll try -- if I mess up, fingers crossed Lauren or someone more knowledgeable pulls me up on it. :p

Contention: Euphrides is ambiguous in which character he suggests is the victim. (Is this okay? Or would something like "Within Medea, Euphrides explores the ideas of being a victim, and innocence"I quite like the original contention but you have to be careful to remain within the confines of the topic with your contention. Like I feel like the second one "Within Medea..." doesn't as clearly address the topic?.)

Argument 1:All characters suffer to some extent at the hands of others.
- For this argument, I would talk about how Medea suffers due to Jason's betrayal, but also due to her own extreme emotions (leading to moral repercussions). I would then talk about how Jason suffers due to the stuff expected of him (social pressures).

Argument 2: All characters however aren't completely innocent.
- Is this a strong argument? It's just that I could already come up with a counter argument for it, and that is the children; they're completely innocent. Would probs be stronger if you ditched the absolute "All characters". "Some characters" would probs work better considering that you are correct in that the children are completely innocent. -- Perhaps the children being innocent could form a third paragraph?
- For this paragraph, I would essentially discuss the bad things the characters did, making them more villainous than victims.

- Would just these 2 body paragraphs suffice for an essay? Probably not. 3 BP's is preferred and typically considered the min number of paragraphs though ofc 2 good body paragraphs is better than 2 good paragraphs and 1 dodgy paragraphs or worse, 3 dodgy paragraphs.

Thanks for all help.  :) :)

Please please please remember the topic: Medea is the true victim of Euphrides' play. As Medea is directly referred to, she needs to form a key aspect of your argument (though obviously, you must include other characters in your discussion!!)

From memory, when I did this my prompt breakdown looked something like this:

Spoiler
1. Medea is, to some extent, a victim due to the actions of those around her.
2. However, her vengeful actions preclude us from casting her as the "true victim"
3. Similarly, there are characters in the play that are not purely "villains" or "victims" (Jason, Glauce's father whose name I can't remember)
4. Ultimately, it is Medea's sons who are the "true" victims due to their innocence blah blah blah.

See how ^ is more Medea centric whilst still incorporating discussion of other characters? Your plan is good but it could be improved through a stronger emphasis on Medea due to the direct reference to Medea in the prompt.

(Sharing this with you because I know for me personally, it always helped to see how other people would have approached the topic and explanations never *really* worked for me unless they were supplemented with an example.   :P )
Title: Re: VCE English Question Thread
Post by: hola on December 18, 2016, 02:57:52 pm
Please please please remember the topic: Medea is the true victim of Euphrides' play. As Medea is directly referred to, she needs to form a key aspect of your argument (though obviously, you must include other characters in your discussion!!)

From memory, when I did this my prompt breakdown looked something like this:

Spoiler
1. Medea is, to some extent, a victim due to the actions of those around her.
2. However, her vengeful actions preclude us from casting her as the "true victim"
3. Similarly, there are characters in the play that are not purely "villains" or "victims" (Jason, Glauce's father whose name I can't remember)
4. Ultimately, it is Medea's sons who are the "true" victims due to their innocence blah blah blah.

See how ^ is more Medea centric whilst still incorporating discussion of other characters? Your plan is good but it could be improved through a stronger emphasis on Medea due to the direct reference to Medea in the prompt.

(Sharing this with you because I know for me personally, it always helped to see how other people would have approached the topic and explanations never *really* worked for me unless they were supplemented with an example.   :P )

Hey HLS,

Thanks for the quick reply and help. I have some more questions (hehe sorry  :P):
- Does my contention has to be centred around Medea?
- Is is okay to have paragraphs based solely on one character (like your 1st one) and bring in discussion about other characters, but link them back to that one character?
- In text response, what does it mean by "views and values"? In Medea, does it means something like "Athenian audience would see Medea's actions as..." or "A modern audience would condemn...".
- To score high, I've been told that you had to discuss the "implications of the topic". Is that like what the author is saying about the topic? (If not too much of a hassle, could you give an example for this topic?)

Thanks for all help. 

Title: Re: VCE English Question Thread
Post by: HopefulLawStudent on December 18, 2016, 03:53:00 pm
Hey HLS,

Thanks for the quick reply and help. I have some more questions (hehe sorry  :P): Welcomes. Tbh Im not doing anything w my time anyway so at least this way someone is hopefully gaining something from my free time that would have otherwise been wasted.
- Does my contention has to be centred around Medea? I think so largely due to the nature of the prompt. However, Medea can't be the only aspect of your argument, you still have to incorporate discussion re: other characters.
- Is is okay to have paragraphs based solely on one character (like your 1st one) and bring in discussion about other characters, but link them back to that one character? Yep. I quite liked to do this (esp for prompts like this) largely because I found that linking them like that made everything feel more cohesive. And my teacher never seemed to complain about it so I guess it's fine?
- In text response, what does it mean by "views and values"? In Medea, does it means something like "Athenian audience would see Medea's actions as..." or "A modern audience would condemn...". Ugh. My Literature teacher did a full 45 minute class on views and values and the nuanced differences between the two and I could never quite grasp onto what it meant; I still can't confidently answer that question from a Literature standpoint. In English however, I think view and values are just treated sorta like "themes" (though from memory, using the word "themes"  is blasphemy) or like the main "ideas".

Like for Medea, I think views and values would be stuff pertaining to:
- Reason v. Passion
- "The Middle Way"
- Role of women
- Family
- Role of men

(Haven't touched Medea since like mid Term 1 though so I'm a big vague on the v&v from Medea.)

- To score high, I've been told that you had to discuss the "implications of the topic". Is that like what the author is saying about the topic? (If not too much of a hassle, could you give an example for this topic?) So implications is looking at stuff like "if this is true, then what?". Basically its what the prompt is suggesting and it's the sort of stuff that you can structure your arguments around. I know a bunch of my friends see implications as statements but for some reason my brain always jumps towards a bunch of questions.  ::)

Example:

"In AAE, unmarried women are unhappy." (Yes, this was an actual prompt that was thrown at me as a "what if I get this??" the day before the exam -- I freaked out horribly about it and as such this is the only AAE prompt I can remember. :p)

Implications: these women are unhappy because they are unmarried/due to their relationships --> relationships and female happiness are interconnected --> this only applies to women? --> Are the men unhappy? --> are relationships and male happiness interconnected? --> is this the only factor that affects the happiness of these women?

Or like for this prompt:

Medea is the true victim of Euripides'  play.

Implications: Medea is a true victim --> Do we always see her as a victim? --> Do we ever see her as a villain? --> Why do we see her as a victim/villain --> Is she the only character we see as a victim? --> Is there someone that is a truer victim than her?



Thanks for all help.
Title: Re: VCE English Question Thread
Post by: hola on December 18, 2016, 04:47:00 pm


Question, how would I structure my arguments around the implications of the Medea prompt?
Title: Re: VCE English Question Thread
Post by: HopefulLawStudent on December 18, 2016, 05:11:43 pm
Question, how would I structure my arguments around the implications of the Medea prompt?

I tend to do one implication/question per paragraph because the implications I tended to find were typically pretty general with like one overarching idea/contention.
Title: Re: VCE English Question Thread
Post by: deStudent on December 18, 2016, 06:14:43 pm
Is it normal when reading a book for the first time that you don't full understand what's going on? Sometimes when I read a page and then go on to the next, I can't even remember what I just read??
Title: Re: VCE English Question Thread
Post by: HopefulLawStudent on December 18, 2016, 06:19:41 pm
Is it normal when reading a book for the first time that you don't full understand what's going on? Sometimes when I read a page and then go on to the next, I can't even remember what I just read??

What book are you reading? This does happen sometimes from my personal experience (looking atchu Wuthering Heights -- still haven't finished the entire book yet lol -- almost done though!!). It also sometimes happens due to the structure of the text or complexities in the author's writing style. I find that the easiest way to combat this is to first do a quick wikipedia or cliffsnotes search of the general plot if you don't mind the spoilers and then go back and read it; things make a whole lot more sense when you have some background to go with it. Alternatively, you can do what my best friend does and read it twice, the first time being a cursory "wtf am I reading" read and the second time being a "ohhhh so this is what was happening" read.
Title: Re: VCE English Question Thread
Post by: deStudent on December 18, 2016, 07:37:21 pm
What book are you reading? This does happen sometimes from my personal experience (looking atchu Wuthering Heights -- still haven't finished the entire book yet lol -- almost done though!!). It also sometimes happens due to the structure of the text or complexities in the author's writing style. I find that the easiest way to combat this is to first do a quick wikipedia or cliffsnotes search of the general plot if you don't mind the spoilers and then go back and read it; things make a whole lot more sense when you have some background to go with it. Alternatively, you can do what my best friend does and read it twice, the first time being a cursory "wtf am I reading" read and the second time being a "ohhhh so this is what was happening" read.
Thanks. Yeah, I usually look over the plot/summaries on those websites, I did this Medea which worked but then again Medea was pretty short.

This problem is occurring when reading a new book called 'The Golden Age' by Joan London. There isn't much resources on this since it's fairly new and by an Australian author.
Title: Re: VCE English Question Thread
Post by: Coffee on December 18, 2016, 07:46:29 pm
Thanks. Yeah, I usually look over the plot/summaries on those websites, I did this Medea which worked but then again Medea was pretty short.

This problem is occurring when reading a new book called 'The Golden Age' by Joan London. There isn't much resources on this since it's fairly new and by an Australian author.
Have you had a look at this PDF for The Golden Age? http://static.booktopia.com.au/pdf/9780857989000-1.pdf It's a brief overview which you might find helpful. Particularly, I think it would be good for you to have a think about the reading questions to help you think critically about the book.

Title: Re: VCE English Question Thread
Post by: HopefulLawStudent on December 18, 2016, 08:23:16 pm
Thanks. Yeah, I usually look over the plot/summaries on those websites, I did this Medea which worked but then again Medea was pretty short.

This problem is occurring when reading a new book called 'The Golden Age' by Joan London. There isn't much resources on this since it's fairly new and by an Australian author.

I'd recommend reading reviews for new books for which there tend to be less resources.

For example:
https://www.australianbookreview.com.au/abr-online/current-issue/122-september-2014-no-364/2103-joan-london-s-new-novel
https://www.themonthly.com.au/issue/2014/november/1414760400/brenda-walker/golden-age-joan-london
http://sydneyreviewofbooks.com/golden-age-joan-london/

Largely because you're just looking for some background/context to enable you to better understand what's going on in the text.

Good luck! :D
Title: Re: VCE English Question Thread
Post by: Gogo14 on December 18, 2016, 08:40:54 pm
hey guys just some qs with medea:
1. at teh end of the book she rides on some dragon chariot. In some resources, it says that the sun god or something gave her the chariots (can't remember) but in the text, is this true? Cant find any references in my text
2. is the chorus actual characters or is it just like a narrator's voice?
Title: Re: VCE English Question Thread
Post by: HopefulLawStudent on December 18, 2016, 08:52:18 pm
hey guys just some qs with medea:
1. at teh end of the book she rides on some dragon chariot. In some resources, it says that the sun god or something gave her the chariots (can't remember) but in the text, is this true? Cant find any references in my text
2. is the chorus actual characters or is it just like a narrator's voice?

There are a bunch of different translations of the text; in some it says dragon chariot in says there's no mention of one. If there isn't a mention of one in your translation, don't refer to one... ever.

Chorus from memory are the Corinthian women so I guess to this end they're actual characters; however they do form a narrator's voice of sorts. If anything, they act more like a removed third party or moral compass.

Hope that helped! :)
Title: Re: VCE English Question Thread
Post by: hola on December 18, 2016, 08:56:36 pm
hey guys just some qs with medea:
1. at teh end of the book she rides on some dragon chariot. In some resources, it says that the sun god or something gave her the chariots (can't remember) but in the text, is this true? Cant find any references in my text
2. is the chorus actual characters or is it just like a narrator's voice?

1) In John Davie's translation, Medea says that the chariot was given to her by "her father's father" in line 1321. This is referring to Helios (Ancient Greek sun god), her grandfather.
2) The chorus is made out of Corinthian women and were sort of narrators? If I recall correctly, they were used (back then) to commentate on what was happening on stage, adding a complexity to whatever was being performed.
Title: Re: VCE English Question Thread
Post by: Mooshkat on December 26, 2016, 02:33:42 pm
Hi,
I'm feeling really lost in terms of how to prepare for English next year.
My novels are medea, Nineteen Eighty Four, Stasiland and I for Isobel.
I've read my novels and feel somewhat familiar with them, but I don't know where to go from here, and not doing anything is making me nervous  :-[
Does anyone have any tips as to what to do between now and the start of the year?
Title: Re: VCE English Question Thread
Post by: clarke54321 on December 26, 2016, 02:55:04 pm
Hi,
I'm feeling really lost in terms of how to prepare for English next year.
My novels are medea, Nineteen Eighty Four, Stasiland and I for Isobel.
I've read my novels and feel somewhat familiar with them, but I don't know where to go from here, and not doing anything is making me nervous  :-[
Does anyone have any tips as to what to do between now and the start of the year?

Hi Mooshkat,

That's great that you've already read all of your texts.

I'm also studying 'Medea' for English. Since the 'Medea' SAC will be held in the first term, I am spending more time with this particular text. What I've done, which I've found useful, is annotate the individual lines of 'Medea.' While this can become quite tedious, it helps to generate your ideas on the text and assists in building up an interpretation.

As well as this, I've been reading some practice essays of 'Medea' on AN, which has been a great help. If you're feeling really keen, you may want to look at an essay prompt and plan a response (finding relevant quotes, building up arguments, etc.).

Also remember to enjoy your holidays, as I'm sure year 12 will be a very busy year!
All the best!!  :)
Title: Re: VCE English Question Thread
Post by: www on December 26, 2016, 06:03:07 pm
Hi,
I'm feeling really lost in terms of how to prepare for English next year.
My novels are medea, Nineteen Eighty Four, Stasiland and I for Isobel.
I've read my novels and feel somewhat familiar with them, but I don't know where to go from here, and not doing anything is making me nervous  :-[
Does anyone have any tips as to what to do between now and the start of the year?

Something you could do is begin creating a document with various themes/motifs you've picked up when reading as headings, and then going through the text(s) and typing in the quote, who said it, what page, and any other contextual information required if it doesn't make much sense on its own. Multiple quotes can fit multiple themes too. This can assist when preparing for SACs, the actual SACs, and the exam because you can later pick out the relevant headings to utilise according to the prompt you're dealing with (to memorise, I recommend the method of Loci). It isn't limited to quotes either, a sentence or two summing up an event or sentences you craft that you think you could use are just as good. Also a section for key words to assist in your expression right at the top won't hurt.

Overwhelming, but don't overkill it this break. It's a feat in itself to have read all your set texts already. Work progressively and refine as you go. Collaborate with friends if that's possible so you can cover more ground. Most importantly, be sure to enjoy the time you have off! Stay focused, know when to relax, and good luck!
Title: Re: VCE English Question Thread
Post by: uuuu on January 03, 2017, 04:40:42 pm
Hey all.

Regarding the creative SAC, are you supposed to replicate the text's ideas (to show your understanding) or extend on them (i.e add your own ideas)?
Title: Re: VCE English Question Thread
Post by: HopefulLawStudent on January 04, 2017, 11:48:20 am
Hey all.

Regarding the creative SAC, are you supposed to replicate the text's ideas (to show your understanding) or extend on them (i.e add your own ideas)?

Hey! Welcome to AN! :)

I'm leaning towards extending it in some way though if your teacher happens to say someth to the contrary then ignore me. The task feels a lot like the creative task in Lit and for that one the point of it was imo to rework or extend the text and its core ideas which is what I think they're looking for in the Eng creative. And for that creative task you were heavily penalised if you just replicated the text and its main ideas and didnt extend beyond the text.

Like if I was doing Id sort of approach the task like I was borrowing key ideas/symbols and giving them new meaning or putting it in a new context.

Hope that helped?
Title: Re: VCE English Question Thread
Post by: uuuu on January 04, 2017, 12:32:36 pm
Hey! Welcome to AN! :)

I'm leaning towards extending it in some way though if your teacher happens to say someth to the contrary then ignore me. The task feels a lot like the creative task in Lit and for that one the point of it was imo to rework or extend the text and its core ideas which is what I think they're looking for in the Eng creative. And for that creative task you were heavily penalised if you just replicated the text and its main ideas and didnt extend beyond the text.

Like if I was doing Id sort of approach the task like I was borrowing key ideas/symbols and giving them new meaning or putting it in a new context.

Hope that helped?

Hello! Thanks for the reply. The text I'm studying is The Lieutenant by Kate Greenville, where one of its messages is essentially "unquestioning acceptance of authority has its consequences". My idea for the creative is to somewhat extend on this and discuss the consequences of complete acceptance of authority (as she explores the idea through a bystander of sorts, rather than someone who is totally a part of it). Is this okay, or is it deviating from the task too much?

Also a few more questions (if not too much of a hassle):
- Do you have to explore all ideas of the text? Or do you choose one in particular and go in depth?
- I've read that you should consider audience and purpose. Could this be something like "modern audiences" and "social commentary"?
- I have a second plan which explores another of the text's ideas; which is "self discovery is only successful with risks". If my plan above doesn't work, could I write a creative piece saying "self discovery requires risks", but in a different way to the author?

Thanks for all help  :) :)
Title: Re: VCE English Question Thread
Post by: HopefulLawStudent on January 04, 2017, 04:03:25 pm
Hello! Thanks for the reply. The text I'm studying is The Lieutenant by Kate Greenville, where one of its messages is essentially "unquestioning acceptance of authority has its consequences". My idea for the creative is to somewhat extend on this and discuss the consequences of complete acceptance of authority (as she explores the idea through a bystander of sorts, rather than someone who is totally a part of it). Is this okay, or is it deviating from the task too much? Looks okay to me.

Also a few more questions (if not too much of a hassle):absoluely no hassle :)
- Do you have to explore all ideas of the text? Or do you choose one in particular and go in depth? if you try to cover all ideas you'll end up in a knot. Choose one or two to explore and go into depth.
- I've read that you should consider audience and purpose. Could this be something like "modern audiences" and "social commentary"? think so yeah. Ultimately if your teacher says no tho then its a no cos theyrw the one marking your sac but it looks fine to me. :)
- I have a second plan which explores another of the text's ideas; which is "self discovery is only successful with risks". If my plan above doesn't work, could I write a creative piece saying "self discovery requires risks", but in a different way to the author? depends by what you mean by different way tbh

Thanks for all help  :) :)

Hoped that helped. Feel free to ask if yoi have any other questions. :)
Title: Re: VCE English Question Thread
Post by: uuuu on January 04, 2017, 05:57:46 pm
Hoped that helped. Feel free to ask if yoi have any other questions. :)

Thanks for the help! :) One more question though; do I have to justify why I chose to extend on one of the author's ideas? Can I just say "cos she didn't do it" if so?
Title: Re: VCE English Question Thread
Post by: HopefulLawStudent on January 04, 2017, 08:34:58 pm
Thanks for the help! :) One more question though; do I have to justify why I chose to extend on one of the author's ideas? Can I just say "cos she didn't do it" if so?

There's some written explanation thing that goes along with the creative I think and with that I think you'd have to justify why; I feel like "cos she didn't do it" might be  considered a bit insufficient at VCE English level (unfortunately).
Title: Re: VCE English Question Thread
Post by: Gogo14 on January 05, 2017, 04:29:02 pm
I've been trying to improve my english by posting in the language analysis thread, but the thread is kinda dead :-\
I was wondering if  someone would be kind enough to correct my essays or just give feedback on how to improve
Thanks!!
Title: Re: VCE English Question Thread
Post by: FallingStar on January 05, 2017, 05:43:27 pm
I've been trying to improve my english by posting in the language analysis thread, but the thread is kinda dead :-\
I was wondering if  someone would be kind enough to correct my essays or just give feedback on how to improve
Thanks!!
Hi Gogo14,

The English Language Analysis thread was more or less targeted at the now graduated class of 2016, and some of them have not been acitve since they've finished their exams. I believe that they may be running another Language Analysis club for the class of 2017, more so tailored to new study design currently being introduced.  (Although not sure about it to be honest).

That being said, if you want us to correct your essays, please post them in the English Work Submission and Marking board, in which people can mark you essay there. It is visible to everyone though,

I'm sure HopefulLawStudent won't mind (provided that you ask nicely) marking your essays and I'm pretty happy to do it too. So good luck with you endeavours though.
Title: Re: VCE English Question Thread
Post by: Gogo14 on January 05, 2017, 06:34:33 pm
Hi Gogo14,

The English Language Analysis thread was more or less targeted at the now graduated class of 2016, and some of them have not been acitve since they've finished their exams. I believe that they may be running another Language Analysis club for the class of 2017, more so tailored to new study design currently being introduced.  (Although not sure about it to be honest).

That being said, if you want us to correct your essays, please post them in the English Work Submission and Marking board, in which people can mark you essay there. It is visible to everyone though,

I'm sure HopefulLawStudent won't mind (provided that you ask nicely) marking your essays and I'm pretty happy to do it too. So good luck with you endeavours though.
oh ok Thanks so much!I'll check it out
Title: Re: VCE English Question Thread
Post by: BlinkieBill on January 20, 2017, 02:05:47 pm
Hey y'all

Could someone please give me some examples of trigger sentences for Language Analysis introductions THANKS
Title: Re: VCE English Question Thread
Post by: HopefulLawStudent on January 20, 2017, 02:19:33 pm
Hey y'all

Could someone please give me some examples of trigger sentences for Language Analysis introductions THANKS

What do you mean by "trigger sentences"?

If you're looking for sample language analysis introductions, you can find some here: English Resources and Sample Essays
Title: Re: VCE English Question Thread
Post by: BlinkieBill on January 20, 2017, 04:09:16 pm
Trigger sentences as in what triggers the persuaive piece.
Like when you start off the intro with describing the issue or event that prompted the author to write the piece.
Title: Re: VCE English Question Thread
Post by: helloeveryone on January 22, 2017, 03:35:59 pm
Pretty noob question but can someone please explain the concept of 'irony' to me please lol. Thanks.
Title: Re: VCE English Question Thread
Post by: zhen on January 22, 2017, 04:05:56 pm
Pretty noob question but can someone please explain the concept of 'irony' to me please lol. Thanks.
Check this site out. http://www.dailywritingtips.com/what-is-irony-with-examples/   It has a pretty good explanation of irony.
Title: Re: VCE English Question Thread
Post by: DHP123 on January 24, 2017, 04:38:15 pm
Hi, I was just wondering how hard it is to get a 35 in EAL because I realised you need 35 raw to get into most courses. What average do u need for each sac? and like how good do you have to be? and any study tips for EAL would also be appreciated :)
Title: Re: VCE English Question Thread
Post by: helloeveryone on January 25, 2017, 12:01:07 am
This is highly unlikely, but does anyone have any The Lieutenant essay prompts that they don't mind sharing? I know it's a first year text (well, for text response that is), but any prompts will be greatly appreciated. Or even better, people who've studied the text, is it okay if you write me one? Doesn't have to be fancy or anything, just something so that I can start practicing. Thanks in advance! 
Title: Re: VCE English Question Thread
Post by: HopefulLawStudent on January 27, 2017, 07:02:24 pm
Hi, I was just wondering how hard it is to get a 35 in EAL because I realised you need 35 raw to get into most courses. What average do u need for each sac? and like how good do you have to be? and any study tips for EAL would also be appreciated :)

This may be of some help to you re: tips for EAL: 50 in English (as an additional language)

To be honest, I'd avoid playing the numbers games largely because it's so hard to predict cos there are so many variables at play; just aim to do the very best you can do as opposed to obsessing over what you need to be getting because in my opinion, obsessing over numbers is just counterproductive. Like some of my friends decided to procrasti-obsess over numbers and got their predictions waaaaay off (which isn't all that hard to do tbh).

This is highly unlikely, but does anyone have any The Lieutenant essay prompts that they don't mind sharing? I know it's a first year text (well, for text response that is), but any prompts will be greatly appreciated. Or even better, people who've studied the text, is it okay if you write me one? Doesn't have to be fancy or anything, just something so that I can start practicing. Thanks in advance! 

Unfortunately, I'm not sure anyone will be able to help you there because as far as I know, TL hasn't been used for text response so there aren't too many resources for it as a text response text yet ( :( ).

If you're just looking for what a good text response looks like though, please feel free to check out this: English Resources for 2017 and Beyond and English Resources and Sample Essays
Title: Re: VCE English Question Thread
Post by: minerva on January 30, 2017, 07:58:02 pm
Hey guys, are GAT scores generally a good indication for what you would get as a study score in English? I did the GAT last year in Year 11 and got a 46 for Written Communication and I'm really hoping to hit low-mid 40's with English this year.
Title: Re: VCE English Question Thread
Post by: HopefulLawStudent on January 30, 2017, 08:00:13 pm
Hey guys, are GAT scores generally a good indication for what you would get as a study score in English? I did the GAT last year in Year 11 and got a 46 for Written Communication and I'm really hoping to hit low-mid 40's with English this year.

I got a 39 for Written Communication in 2016 and like a 42 for Written Communication in 2015 so I wouldn't trust GAT results all that much.
Title: Re: VCE English Question Thread
Post by: smiley123 on February 02, 2017, 11:56:16 am
Hi, can someone please teach me how to write an evaluative essay? Also, is an evaluative essay the same as an analytical essay?
Title: Re: VCE English Question Thread
Post by: atar.notes.user on February 08, 2017, 09:22:30 pm
Hi, I am doing This Boy's Life in school. For the essay question 'why do readers sympathise with Jack?' Is it recommended that I refer to Toby as Jack in the entire essay because the question specified? Or am I just looking too deep into it?
Title: Re: VCE English Question Thread
Post by: anthonyttran on February 11, 2017, 09:41:38 am
Hi, i was wondering what types of prompts we would get for the creative response SAC? I know schools will be different but can someone give me a general idea?
Title: Re: VCE English Question Thread
Post by: Syndicate on February 11, 2017, 10:32:49 am
Hi, i was wondering what types of prompts we would get for the creative response SAC? I know schools will be different but can someone give me a general idea?

You can write whatever you want, as long as you can relate it to the text(isn't that the point of a creative SAC?). However, the students usually tend to write:
- a monologue from a character that didn't get to express their points quite well (or wasn't exactly given a voice)
- a scene (using the writer's style) that continues from any point in the play- usually the end, the beginning or after something dramatic had occurred
- newspaper article
- a piece whilst acting as an omniscient narrator
- a piece where you act as the author of the text, and express his/her views on his/her protagonist/ tragedy that occurred
Title: Re: VCE English Question Thread
Post by: HopefulLawStudent on February 11, 2017, 10:46:52 am
Hi, i was wondering what types of prompts we would get for the creative response SAC? I know schools will be different but can someone give me a general idea?

Unfortunately, there is no consistent approach between schools re: the creative SAC (not so far as I have seen anyway). Like in some schools, the prompt is basically just "write a creative response to the text" and in other schools it's been like "write a creative response in this form, from the perspective of this character, looking at these two scenes."

Hi, I am doing This Boy's Life in school. For the essay question 'why do readers sympathise with Jack?' Is it recommended that I refer to Toby as Jack in the entire essay because the question specified? Or am I just looking too deep into it?

I haven't read TBL (on my to read list still) but personally, I'd lean towards referring to toby as Jack cos that's what the prompt has done; with that said though, I don't think it matters all that much because any assessor reading your essay will know that Jack = Toby cos they'll have read the text so it should be fine either way. Don't worry too much about it, I think.

Hi, can someone please teach me how to write an evaluative essay? Also, is an evaluative essay the same as an analytical essay?

What do you mean by evaluative essay? + What exactly are you struggling with?
Title: Re: VCE English Question Thread
Post by: Adammurad on February 13, 2017, 12:28:07 am
I need a bit of help I got a oral presentation/seminar that's due a bit later in the month. I got the quote of my choice which was "An eye for an eye will make the whole world blind" by Gandhi and I need to find two texts that relate to the core idea of this quote and compare them. Any suggestions of texts?
Title: Re: VCE English Question Thread
Post by: arlynl99 on February 13, 2017, 02:02:59 pm
hi i am study 'the Lieutenant' and need some help with my essay topic.
'the lieutenant is a novel about overcoming the familiar' Discuss
any help would be appreciated
thanks
Title: Re: VCE English Question Thread
Post by: TheCommando on February 14, 2017, 09:38:13 am
Lauren. Im llike a spone soaking up information so as a result im somewhat confused.
What is the best way from your pov to prepare for my first sac which is a text response.
So far i reckon its just just re reading and annotating the book, writing chapter summuraries and then doing prompt writings.
Im also trying to improve my vocab
Title: Re: VCE English Question Thread
Post by: TheCommando on February 14, 2017, 09:48:10 am
Oh no sac is in week 5 and its week 3
What do i do
Its a test response based on i for isobel 😢😢😢😱😱😨
Title: Re: VCE English Question Thread
Post by: Gogo14 on February 18, 2017, 11:53:46 am
what is the metalanguage called when the author tries to acknowledge opposing arguments and tries to seek a compromise?
Title: Re: VCE English Question Thread
Post by: kupus on February 19, 2017, 12:34:30 pm
Hi everyone,
My Creative Response SAC is tomorrow, and there is a limit of 600-800 words.
Do you think that I will get penalised if mine is much longer? Say 1200 words or so.
Thanks
Title: Re: VCE English Question Thread
Post by: seth on February 19, 2017, 12:53:22 pm
Hi everyone,
My Creative Response SAC is tomorrow, and there is a limit of 600-800 words.
Do you think that I will get penalised if mine is much longer? Say 1200 words or so.
Thanks

Depends on your school, but most have a 10% rule where you're allowed to go over or under the word limit by 10% without penalty.
Title: Re: VCE English Question Thread
Post by: clarke54321 on February 24, 2017, 07:17:18 am
Hi all,

Is there a preferable number of body paragraphs one should have in their text response essays? I've always been used to writing 3 body paragraphs, however some year 12 teachers are recommending at least 4.

Any clarification would be appreciated.
Title: Re: VCE English Question Thread
Post by: seth on February 24, 2017, 08:27:16 am
Hi all,

Is there a preferable number of body paragraphs one should have in their text response essays? I've always been used to writing 3 body paragraphs, however some year 12 teachers are recommending at least 4.

Any clarification would be appreciated.

Any number is fine as long as you're able to sufficiently argue your contention. Just don't always think that you have to write 3; sometimes only 2 long paragraphs are needed, but some contentions require at least 4 or 5.
Title: Re: VCE English Question Thread
Post by: HopefulLawStudent on February 24, 2017, 09:08:03 am
hi i am study 'the Lieutenant' and need some help with my essay topic.
'the lieutenant is a novel about overcoming the familiar' Discuss
any help would be appreciated
thanks

Well... what are you stuck on, exactly? Some specificity re: what you're struggling with exactly would help us give you the help that you need. :)

what is the metalanguage called when the author tries to acknowledge opposing arguments and tries to seek a compromise?

Bump -- anyone know? I have no idea here. Sorry!

Lauren. Im llike a spone soaking up information so as a result im somewhat confused.
What is the best way from your pov to prepare for my first sac which is a text response.
So far i reckon its just just re reading and annotating the book, writing chapter summuraries and then doing prompt writings.
Im also trying to improve my vocab

Not Lauren but:

Identify your problem areas -- i.e. the ones that you struggle with the most about text response. So like is it timing, breaking down the prompt, unpacking evidence, topic sentences. And from there, branch out and focus on improving on those problem areas.

Some nifty vocab stuff can be found here: http://atarnotes.com/forum/index.php?topic=164878.0

Hi everyone,
My Creative Response SAC is tomorrow, and there is a limit of 600-800 words.
Do you think that I will get penalised if mine is much longer? Say 1200 words or so.
Thanks

Ouch. Word limits suck. I realise that it's been a while since your post so just gonna say: I hope your SAC went well!

Hi all,

Is there a preferable number of body paragraphs one should have in their text response essays? I've always been used to writing 3 body paragraphs, however some year 12 teachers are recommending at least 4.

Any clarification would be appreciated.

seth's pretty much answered your question but basically, there isn't a set number of body paragraphs that'll score you a 50 study score/perfect marks/whatever (unfortunately). Just aim to have enough body paragraphs so that you're confident that you've sufficiently explored the topic and argued your contention.

And to seth: I have two words for you.

(https://s-media-cache-ak0.pinimg.com/originals/f2/b2/92/f2b292e38373964cf12a0820be6deb13.gif)

(You - and your answers - are awesome!)
Title: Re: VCE English Question Thread
Post by: seth on February 24, 2017, 11:11:43 am
And to seth: I have two words for you.

(https://s-media-cache-ak0.pinimg.com/originals/f2/b2/92/f2b292e38373964cf12a0820be6deb13.gif)

(You - and your answers - are awesome!)

Hehe ty  ;D ::) ::)

hi i am study 'the Lieutenant' and need some help with my essay topic.
'the lieutenant is a novel about overcoming the familiar' Discuss
any help would be appreciated
thanks

Here's some thoughts:

- Who overcomes the familiar? Is it just Rooke, or perhaps all the characters?
- Rooke eventually realizes the limitations of his knowledge, something he relied on to understand the unfamiliar
- Language is the unfamiliarity that divides the Colonists and Indigenous
- There is little room for individuality in a military environment (or even authoritative)
- The Gun could be considered a one-way conversation of sorts
- His Majesty's service is centered around self interests with little consideration for conscience. Does Rooke overcome this in the end?
- If so, what is Grenville say thing through this? Is overcoming the familiar good or bad? In every circumstance, or only some?
- Relationships are founded on mutual understanding of one another. How does this affect the Colonists' relationship with the Indigenous, people who they perceive as "savages"? Does this change?
- What does Grenville suggest as necessary in overcoming the familiar?
- Rooke only truly discovers himself after rejecting the 'norm'. In previous attempts of self discovery, he had always accepted the judgements of others as truth.
- Is there anything in the novel about sticking to the familiar? Unwillingness to accept change?
Title: Re: VCE English Question Thread
Post by: arlynl99 on February 24, 2017, 11:22:57 am
Hehe ty  ;D ::) ::)

Here's some thoughts:

- Who overcomes the familiar? Is it just Rooke, or perhaps all the characters?
- Rooke eventually realizes the limitations of his knowledge, something he relied on to understand the unfamiliar
- Language is the unfamiliarity that divides the Colonists and Indigenous
- There is little room for individuality in a military environment (or even authoritative)
- The Gun could be considered a one-way conversation of sorts
- His Majesty's service is centered around self interests with little consideration for conscience. Does Rooke overcome this in the end?
- If so, what is Grenville say thing through this? Is overcoming the familiar good or bad? In every circumstance, or only some?
- Relationships are founded on mutual understanding of one another
- What does Grenville suggest as necessary in overcoming the familiar?
- Rooke only truly discovers himself after rejecting the 'norm'. In previous attempts of self discovery, he had always accepted the judgements of others as truth.
- Is there anything in the novel about sticking to the familiar? Unwillingness to accept change?

thanks heaps.........
got another topic as well
in the lieutenant conflict is the product of the failure to understand each other. 
got some conflicts
english vs natives
natives vs english
silk vs rooke????? - don't know any more big conflicts that i could include........
thanks agin
Title: Re: VCE English Question Thread
Post by: BlinkieBill on February 25, 2017, 09:51:36 am
Hey - for texts like Medea
I'm not sure how or even to talk about modern vs ancient greek audiences
Or
Relevance of the play today??
Title: Re: VCE English Question Thread
Post by: seth on February 25, 2017, 10:18:48 am
thanks heaps.........
got another topic as well
in the lieutenant conflict is the product of the failure to understand each other. 
got some conflicts
english vs natives
natives vs english
silk vs rooke????? - don't know any more big conflicts that i could include........
thanks agin

Try and think of characters as not just characters, but rather vehicles in which the author deliver their ideas. For example, what does the English represent? Silk? Rooke? From here, you'll be able to identify larger conflicts within the play. Look at the page where Rooke says "I can't be part of this" out loud if you're still stuck; Grenville makes reference to one of the biggest conflicts in the novel here.
Title: Re: VCE English Question Thread
Post by: BlinkieBill on February 25, 2017, 10:51:09 am
Hey yoooooo
Peeps are always talking abt "defining themes" in theme based essays.
Is this as in to literally define the key word and explain it??
Can someone give me an example?!!!!!!!!!!!!!!!!!?!?!??!?
Title: Re: VCE English Question Thread
Post by: TheCommando on February 25, 2017, 08:23:01 pm
Well... what are you stuck on, exactly? Some specificity re: what you're struggling with exactly would help us give you the help that you need. :)

Bump -- anyone know? I have no idea here. Sorry!

Not Lauren but:

Identify your problem areas -- i.e. the ones that you struggle with the most about text response. So like is it timing, breaking down the prompt, unpacking evidence, topic sentences. And from there, branch out and focus on improving on those problem areas.

Some nifty vocab stuff can be found here: http://atarnotes.com/forum/index.php?topic=164878.0

Ouch. Word limits suck. I realise that it's been a while since your post so just gonna say: I hope your SAC went well!

seth's pretty much answered your question but basically, there isn't a set number of body paragraphs that'll score you a 50 study score/perfect marks/whatever (unfortunately). Just aim to have enough body paragraphs so that you're confident that you've sufficiently explored the topic and argued your contention.

And to seth: I have two words for you.

(https://s-media-cache-ak0.pinimg.com/originals/f2/b2/92/f2b292e38373964cf12a0820be6deb13.gif)

(You - and your answers - are awesome!)
Thank you
How can i structure words better? I now know how to implement qoutes but structuring words inca scetence i dont really know.
Title: Re: VCE English Question Thread
Post by: literally lauren on February 27, 2017, 10:15:11 am
Hey yoooooo
Peeps are always talking abt "defining themes" in theme based essays.
Is this as in to literally define the key word and explain it??
Can someone give me an example?!!!!!!!!!!!!!!!!!?!?!??!?

Easiest way to do this is to use the "theme and synonym" method. So if the key theme you're dealing with is 'power,' you could have a sentence in your introduction like:

There are many different forms of power and authority that are seen in the text...
or
Throughout the novel, Ringo displays a great deal of power and self-control...
or
Although the characters initially struggle to come to terms with the oppressive power structures and societal pressures ultimately they are able to...

That way, you're explaining that key theme by linking it to another closely related term and clarify what kind of 'power' you're dealing with :)

Thank you
How can i structure words better? I now know how to implement qoutes but structuring words inca scetence i dont really know.
Read through some of the tips in this guide that HLS linked - that should help you get started!
Title: Re: VCE English Question Thread
Post by: pha0015 on February 27, 2017, 10:44:30 pm
Hi,

How would one practice for English?

Also, when it comes to essays, if I have a few hours to do it, then I can produce an 800 word A+ piece easily. But when they're timed e.g. 1 hour, I get a huge writer's block and produce around 500 word B+ pieces. Do you have any suggestions? I also don't really make a plan beforehand, so should I start?

And when it comes to my main arguments in essays, it's hard for me to not go off on a tangent.

I feel like I have the potential to do better, but don't know where to start.
Title: Re: VCE English Question Thread
Post by: Gogo14 on February 28, 2017, 10:30:26 am
Apparently language analysis has been changed to argument analysis, and my school has a major focus on the argument. However, I have trouble identifying and "analysing" argument, and have no clue as to how to analyse it and compare it. Could someone give me a detailed example plz?
Title: Re: VCE English Question Thread
Post by: clarke54321 on March 04, 2017, 10:26:10 am
Hi,

Do I need a qualifier for this essay topic:

'Medea' is about the extremes of human emotion. Discuss.

With this prompt, I don't feel the need to disagree.



Title: Re: VCE English Question Thread
Post by: HopefulLawStudent on March 04, 2017, 10:56:16 am
Hi,

How would one practice for English?

Also, when it comes to essays, if I have a few hours to do it, then I can produce an 800 word A+ piece easily. But when they're timed e.g. 1 hour, I get a huge writer's block and produce around 500 word B+ pieces. Do you have any suggestions? I also don't really make a plan beforehand, so should I start?

And when it comes to my main arguments in essays, it's hard for me to not go off on a tangent.

I feel like I have the potential to do better, but don't know where to start.

Writer's block is horrible. I feel like planning beforehand, even if it's just a couple of words that signpost your arguments would help you combat it and ensure that you don't go off on a tangent.

And it doesn't even have to be something that makes sense to anyone other than you because you're the only one who will be reading it!

For example, for one of my text response SACs my plan literally went along the lines of:
- M + J
- G + C
- crown
- pickles

I don't even like pickles and like looking back I still have no idea what I meant by pickles but at the time when I was doing the SAC, it helped to trigger some argument that I had and it made sense to me, I guess.

Hi,

Do I need a qualifier for this essay topic:

'Medea' is about the extremes of human emotion. Discuss.

With this prompt, I don't feel the need to disagree.


Generally, you should avoid agreeing completely; qualifying helps in that it shows that you've thought through the topic whereas completely dis/agreeing could make your argument seem more shallow and less carefully thought out, I guess? Like is there any aspect of that prompt that you *could* challenge/expand upon further?

Apparently language analysis has been changed to argument analysis, and my school has a major focus on the argument. However, I have trouble identifying and "analysing" argument, and have no clue as to how to analyse it and compare it. Could someone give me a detailed example plz?

Hey! Have you checked out the sample essays that are available here: https://atarnotes.com/forum/index.php?topic=8375.0 ?
Title: Re: VCE English Question Thread
Post by: seth on March 04, 2017, 11:05:21 am
Hi,

Do I need a qualifier for this essay topic:

'Medea' is about the extremes of human emotion. Discuss.

With this prompt, I don't feel the need to disagree.

Hehe me again. Generally, you shouldn't ever fully agree with a topic as it shows that all you've done is read it once without giving it much thought. For this prompt, you could disagree with the fact that Medea at time shows a moderation of emotion, being driven both by logic (i.e no emotion) and passion (i.e very emotional). You could also weave in discussion about the parts where the Chorus debates with itself; this embodying the mid-way between different sides of human emotion.

edit: beaten by HLS
Title: Re: VCE English Question Thread
Post by: clarke54321 on March 04, 2017, 11:15:38 am
Thanks for the advice HLS and Seth! Will check out these resources  :)

I suppose I'm just worried about over-complicating the essay. The only way I would challenge this prompt is by talking about the way Medea is capable of exhibiting logic and reason, as well as excessive passion. But at the same time, I don't think that she achieves moderation- which is getting towards my MC (that the principle of sophrosyne ought to be followed to prevent self-destruction).  How can I make these arguments without contradicting myself?

Or could I just say that:

Yes: 'Medea' is about the extremes of human emotions
But: The play is also a warning to find a balance between two extremes

Would that satisfy a qualifier? I'm not entirely sure.
Title: Re: VCE English Question Thread
Post by: TheCommando on March 05, 2017, 12:55:00 pm
How can i stop myself from reppetition in text response essays?
Sample is attached below
Title: Re: VCE English Question Thread
Post by: geminii on March 07, 2017, 08:21:24 pm
Hey! Does anyone have any ideas for a Medea creative writing piece? It's our next SAC and I'm trying to think of something unique. We need to write on a part/scene in the text that wasn't thoroughly explored, or write in someone's point of view, for example. Does anyone have some parts of the play that they think would make a good scene for a creative piece?
Thanks heaps!!! :D
Title: Re: VCE English Question Thread
Post by: clarke54321 on March 08, 2017, 11:24:48 am
Hi all,

For this prompt:

'In Medea, there are no heroes, only victims.' Do. you agree.

My contention is:

Despite the absence of a clearly defined hero, Euripides 'Medea' presents characters who exhibit heroic qualities but are ultimately victims to their own fatal flaws.

Could someone please give me some advice.
Title: Re: VCE English Question Thread
Post by: NAT0003 on March 10, 2017, 04:15:24 pm
Do you really need to read your texts more than once in unit 1/2 english
Title: Re: VCE English Question Thread
Post by: ish708 on March 10, 2017, 07:11:00 pm
Do you really need to read your texts more than once in unit 1/2 english

I liked to read once, then read and annotate. That's all I did in 1/2. It's personal preference how you think works best. I do more in 3/4 but that was my 1/2 strategy.
Title: Re: VCE English Question Thread
Post by: Syndicate on March 10, 2017, 07:45:21 pm
Thanks for the advice HLS and Seth! Will check out these resources  :)

I suppose I'm just worried about over-complicating the essay. The only way I would challenge this prompt is by talking about the way Medea is capable of exhibiting logic and reason, as well as excessive passion. But at the same time, I don't think that she achieves moderation- which is getting towards my MC (that the principle of sophrosyne ought to be followed to prevent self-destruction).  How can I make these arguments without contradicting myself?

Or could I just say that:

Yes: 'Medea' is about the extremes of human emotions
But: The play is also a warning to find a balance between two extremes

Would that satisfy a qualifier? I'm not entirely sure.

Personally, I think that would be perfectly fine. You just need to provide enough evidence to support your arguments. Medea, believe it or not, does exhibit reasoning as she, in her moral anguish, justifies the premature demise of her children (you can take this the other way around as well! Remember: you are never wrong in English).  I don't really think you would be contradicting yourself if you don't remain quite black and white throughout your piece (I can be wrong here, if so, please do specify why you think you might be contradicting yourself).

How can i stop myself from reppetition in text response essays?
Sample is attached below

I skimmed over your piece (do note that I have never read the text before), and found that you are repeating Isobel's childhood and the theme of isolation a lot. What I would suggest is breaking your prompt into key ideas, and then explore them by analysing your text multiple times. Even if your prompt focuses on one key theme, you can still create several different ideas from it (try to think differently from everyone, and view the text from a different perspective). 

Also, using higher standard vocabulary may eliminate such repetition.

Hi all,

For this prompt:

'In Medea, there are no heroes, only victims.' Do. you agree.

My contention is:

Despite the absence of a clearly defined hero, Euripides 'Medea' presents characters who exhibit heroic qualities but are ultimately victims to their own fatal flaws.

Could someone please give me some advice.

From following after your contention, I believe you can go on talking about how no one in the end was really were satisfied (especially Jason and Medea), as everyone did pay the price (more Jason than Medea) for their misdeeds (Medea didn't really get punished, but she did lose a lot in the end). You should also prove why there aren't any heroes in 'Medea', rather than just exploring how each of the characters were victim to their own political ambition/ passion (etc...), as you should be addressing every part of the prompt (summary of prompt: first part: why there aren't any heroes in 'Medea', second part: why are there victims). I also think you can also explore how Medea's innocent children were victim to such a horrible filicide, as it would show that you understand the text quite thoroughly. To further this, you can also talk about Euripides insertion of the theme of patriarchal abandonment, as Jason abandons his children to fulfil his ambition (do make sure you relate this back to the prompt, otherwise you can easily go on a tangent and lose marks for this), and then relate all this to how both Jason and Medea can be classified as villains rather than heroes. 

Do you really need to read your texts more than once in unit 1/2 english

Well, if you want to get different ideas from the rest of your cohort, I would suggest reading and then analysing your text to find helpful quotes, metalanguage etc...
Title: Re: VCE English Question Thread
Post by: zhen on March 10, 2017, 08:18:34 pm
Do you really need to read your texts more than once in unit 1/2 english
To add onto the previous responses. I think that reading your texts multiple times is a way to gaining a more in depth knowledge of the text. I think that the first time I read my text, I just get the general gist of what's going on without really analysing the themes and symbols. However, during my second reading I can better identify the important quotes and themes, because I already know what the text is about and what I should be looking for.
Title: Re: VCE English Question Thread
Post by: lovebiology on March 11, 2017, 11:40:43 am
What will happen to the marks if i didnt finish the written explanation of my creative sac? will it drag down my marks by a lot?
Title: Re: VCE English Question Thread
Post by: seth on March 11, 2017, 08:15:54 pm
What will happen to the marks if i didnt finish the written explanation of my creative sac? will it drag down my marks by a lot?

The written explanation shouldn't significantly drag down your mark as it only makes up a small portion of the overall SAC. It's more of a support for your actual piece as it explain your choices, so if your creative piece is clear in its direction and ideas, then you shouldn't worry too much about marks lost. Also, do note that English pieces are usually marked holistically, meaning that if from what you have demonstrates an insightful and detailed explanation of your choices, then there's no reason for marks to be deducted.
Title: Re: VCE English Question Thread
Post by: NAT0003 on March 12, 2017, 10:22:05 am
What do i need to do from now until october 2018 to ensure a raw 25+ study score for english. Please help!!!! :) :) :) :)
Title: Re: VCE English Question Thread
Post by: Quantum44 on March 12, 2017, 10:39:19 am
What do i need to do from now until october 2018 to ensure a raw 25+ study score for english. Please help!!!! :) :) :) :)

Read lots of quality literature to improve writing style and turn of phrase. Practice text response, language analysis and comparative essays to understand the best way to structure your responses for those formats. Read the texts the year 12s are studying this year multiple times as they will likely be your texts for next year. Regularly read newspaper articles and think about how the author is using language to persuade. This will improve your language analysis and will expose you to issues you may want to speak about for your oral SAC next year.
Title: Re: VCE English Question Thread
Post by: NAT0003 on March 12, 2017, 11:03:32 am
Read lots of quality literature to improve writing style and turn of phrase. Practice text response, language analysis and comparative essays to understand the best way to structure your responses for those formats. Read the texts the year 12s are studying this year multiple times as they will likely be your texts for next year. Regularly read newspaper articles and think about how the author is using language to persuade. This will improve your language analysis and will expose you to issues you may want to speak about for your oral SAC next year.
Thank so you much :D
Title: Re: VCE English Question Thread
Post by: undefined on March 16, 2017, 10:36:44 pm
hey, probably an obvious question, but is there a place where we can get some feedback on our general english essays? or is it only available for hsc students?
Title: Re: VCE English Question Thread
Post by: seth on March 16, 2017, 11:12:12 pm
hey, probably an obvious question, but is there a place where we can get some feedback on our general english essays? or is it only available for hsc students?

https://atarnotes.com/forum/index.php?board=406.0
Title: Re: VCE English Question Thread
Post by: Gogo14 on March 17, 2017, 09:55:50 am
I have to do an oral creative for this boys life. I am really stumped on how to present it, and what I can do. I want to recreate a scene from the novel, but dont know how to present it.
Title: Re: VCE English Question Thread
Post by: hello friends on March 19, 2017, 07:10:59 pm
Hello! I have a question regarding the use of the word 'contends' in argument analysis; should I use it? I know some teachers are against it since it's a pretty bland word by definition, but I also see the merit in it that it says "this is the contention" quite clearly. So, opinions?  :o

(my teacher is fine with me using either contends or a more specific alternative)

Thanks.
Title: Re: VCE English Question Thread
Post by: illuminatisky on March 19, 2017, 07:12:40 pm
Will you lose a lot of marks if you wrote 200 words for a 600-800 word essay
Title: Re: VCE English Question Thread
Post by: Syndicate on March 19, 2017, 07:47:20 pm
Hello! I have a question regarding the use of the word 'contends' in argument analysis; should I use it? I know some teachers are against it since it's a pretty bland word by definition, but I also see the merit in it that it says "this is the contention" quite clearly. So, opinions?  :o

(my teacher is fine with me using either contends or a more specific alternative)

Thanks.


You would be better off using words that are more descriptive and alludes to the writer's emotions (such as [tone] condemns, castigates, accentuates, decries etc...) as these words are more specific than contends, which is a really general words (From what I know, a lot of students use "contends" or such generic terms in their LA exam) .

Do make sure that you are using the right words in the right context though.
Will you lose a lot of marks if you wrote 200 words for a 600-800 word essay

If you have written a sophisticated piece in 200 words, that explains everything really well, then no. However, if your piece lacks analysis, then you would most likely lose marks. Generally, English teacher marks your piece based on your vocabulary/ grammar, understanding of the text and how well you remain on point through out your essay (so like not going off on tangents).

With a 200-word essay, you are likely to lose a lot marks, as I believe it will not be enough.
Title: Re: VCE English Question Thread
Post by: literally lauren on March 21, 2017, 10:03:47 am
I have to do an oral creative for this boys life. I am really stumped on how to present it, and what I can do. I want to recreate a scene from the novel, but dont know how to present it.
Could you give us a bit more info on the SAC details? Do you have to deliver an oral from the point of view of one of the characters or something? Is this an individual task or a group one? Is there anything in particular your teacher wants you to address, or anything/anyone you'd be interested in exploring? :)

Hello! I have a question regarding the use of the word 'contends' in argument analysis; should I use it? I know some teachers are against it since it's a pretty bland word by definition, but I also see the merit in it that it says "this is the contention" quite clearly. So, opinions?  :o

(my teacher is fine with me using either contends or a more specific alternative)

Thanks.

Defs agree with Syndicate in that if there's a more descriptive word you can use - go for it! However, if you can't think of one, you can bundle the word 'contends' with an adverb describing tone e.g. (the author enthusiastically contends that... the author bitterly contends that... the author sympathetically contends that...)

Will you lose a lot of marks if you wrote 200 words for a 600-800 word essay
It's not that you'd *lose* marks... it's just that you wouldn't be able to gain many since you wouldn't have demonstrated sufficient skills. It's kind of like if someone asked you to judge their cooking abilities when all they've made for you is half a toasted cheese sandwich - maybe it's a great sandwich, but it's not much to go on.

If this is just a hypothetical you're worried about, then suffice it to say that writing 200 words is definitely not enough if you're looking to score more than 3/10. But if this has happened to you in a SAC or a practice essay, consider why you found it difficult to write any more - did you end up wasting time because you didn't know how to word your ideas properly, or did you not understand the text/material in enough detail to talk about it? Hopefully you'll be able to wring more than 200 words out of and prompts/material you get given, but if not, talk to your teacher or let us know if there's anything we can do to help :)
Title: Re: VCE English Question Thread
Post by: Gogo14 on March 22, 2017, 09:27:49 am
Could you give us a bit more info on the SAC details? Do you have to deliver an oral from the point of view of one of the characters or something? Is this an individual task or a group one? Is there anything in particular your teacher wants you to address, or anything/anyone you'd be interested in exploring? :)
Defs agree with Syndicate in that if there's a more descriptive word you can use - go for it! However, if you can't think of one, you can bundle the word 'contends' with an adverb describing tone e.g. (the author enthusiastically contends that... the author bitterly contends that... the author sympathetically contends that...)
It's not that you'd *lose* marks... it's just that you wouldn't be able to gain many since you wouldn't have demonstrated sufficient skills. It's kind of like if someone asked you to judge their cooking abilities when all they've made for you is half a toasted cheese sandwich - maybe it's a great sandwich, but it's not much to go on.

If this is just a hypothetical you're worried about, then suffice it to say that writing 200 words is definitely not enough if you're looking to score more than 3/10. But if this has happened to you in a SAC or a practice essay, consider why you found it difficult to write any more - did you end up wasting time because you didn't know how to word your ideas properly, or did you not understand the text/material in enough detail to talk about it? Hopefully you'll be able to wring more than 200 words out of and prompts/material you get given, but if not, talk to your teacher or let us know if there's anything we can do to help :)
The sac is open ended and I can pretty much present anything. It can also be a group presentation if I want, also my teacher really wants us to explore and extend on the themes in the book. My idea is:
Having Toby hitckhike with a stranger. I want to explore themes of family, identity, and escapism
The problem is that I don't know how to present it
Title: Re: VCE English Question Thread
Post by: OOOOO on March 25, 2017, 06:33:48 pm
Hello VCE friends, I have a question; how are you guys being taught to structure argument analysis essays?

My school is teaching contradicting stuff, on one hand they're claiming that the new course in essence is the old language analysis with just a relabel and a few clarifications, but on the other hand, they're telling me to focus on arguments and start all body paragraphs with a sub-argument. This however is obviously flawed when it comes to comparison with pieces that have 5-6 points. To solve this problem, they've taught to discuss piece 1 solely in the first few paragraphs, ignoring less important arguments if necessary, then do that with piece 2 but with comparisons of piece 1 woven within. I'm a bit iffy with this method though since at its core, we're being assessed on how we compare so it's a bit unwise in my opinion to start doing so only half way through. So, are your schools teaching something different, or the same method as mine?  :o

Thanks for all help.  :) :)

P.S I know I should follow whatever my teachers say in SACs, but I got pretty cool ones that are pretty open minded provided that I can argue the merit of another structure.
Title: Re: VCE English Question Thread
Post by: clarke54321 on March 25, 2017, 06:46:41 pm
Hello VCE friends, I have a question; how are you guys being taught to structure argument analysis essays?

My school is teaching contradicting stuff, on one hand they're claiming that the new course in essence is the old language analysis with just a relabel and a few clarifications, but on the other hand, they're telling me to focus on arguments and start all body paragraphs with a sub-argument. This however is obviously flawed when it comes to comparison with pieces that have 5-6 points. To solve this problem, they've taught to discuss piece 1 solely in the first few paragraphs, ignoring less important arguments if necessary, then do that with piece 2 but with comparisons of piece 1 woven within. I'm a bit iffy with this method though since at its core, we're being assessed on how we compare so it's a bit unwise in my opinion to start doing so only half way through. So, are your schools teaching something different, or the same method as mine?  :o

Thanks for all help.  :) :)

P.S I know I should follow whatever my teachers say in SACs, but I got pretty cool ones that are pretty open minded provided that I can argue the merit of another structure.

Hey OOOOO,

My school is also doing your method. I suppose that you analyse the first piece in isolation, as this will generally be the biggest piece and the one which the comparative is based around. For example, an article (the bulk) and then a smaller write in from a reader (in relation to the bulk piece).

And yes, this new addition is, in essence, the old language analysis. VCAA are just trying to formalise the way students set out their work. For example, instead of just having paragraphs centred around single language devices, VCAA are trying to encourage students to take a holistic approach and explain how these language devices support the arguments throughout.

Good luck!  :)
Title: Re: VCE English Question Thread
Post by: Gogo14 on March 27, 2017, 11:25:41 am
How do you write a written explanation for a creative SAC? Dont know how to structure it and what to include. Thanks
Title: Re: VCE English Question Thread
Post by: TheCommando on March 29, 2017, 09:38:53 pm
Tips on creative writing.
Apart from practising and gaining feedback what can i do?
Title: Re: VCE English Question Thread
Post by: TheCommando on March 29, 2017, 09:59:15 pm
So a method if improving your English is to read your essay and high scoring essays line by line and trying to dissect what makes it good?
How on earth do i do this excatly? Rubric? But it seems vague and hard to understand. Please help
Illy ❤️
Title: Re: VCE English Question Thread
Post by: waterangel82 on April 01, 2017, 10:33:36 am
Hi guys,

I have posted one of my practice pieces for Argument Analysis. Is it alright if someone can mark it and give me some feedback? We have a SAC on this the second week of next term, and I feel like though I'm very much screwed. By the way, I have attached the two articles. Thanks!

So here's my piece:

The issue of the legislation by the Queensland Government to deregister vehicles with offensive advertisements has recently caused much debate, in particular towards those who are concerned that their ‘freedom of speech’ is being undermined. The newly formed legislation seeks to target such companies which uses vehicles with indecent slogans, such as the well-known ‘Wicked Campers’. The online blog ‘Wicked Games’ by journalist Clementine Ford contends that the Queensland Government’s legislation to prevent companies from emblazoning such slogans onto their vehicles is the right choice. She argues critically that society cannot tolerate slogans that normalise violence and social segregation, mainly aiming to target an audience of female rights supporters. However, Victorian Greens MP Nina Springle’s Huffington Post Blog ‘Other States Must Buy-In To Queensland’s Wicked New Legislation’, published on the 29th July 2016, extends on Ford’s contention, and seeks to target people with left-wing ideologies. She addresses that while the Queensland Government’s legislation is indeed an ‘ingenious solution’, other states, including Victoria, must also act against such companies that have vehicles which advertise such slogans.

Like many opinion pieces in blogs, Ford immediately establishes her frustrated and outraged tone in her piece. She argues that although companies such as the Wicked Campers may believe their offensive slogans to be a form of ‘comedy’, such ‘jokes’ are not acceptable because they only seek to undermine those who are being targeted. She opens her piece with the pun in the headline ‘Wicked Games’, derived from the name of the ‘Wicked’ Campers, seeking to make readers to become aware of her stance that the promotion of the inappropriate advertisements is more than cruel, but a ‘wicked’ act that normalises violence in society. She continues to build upon her argument through the rhetorical question ‘So you think the Wicked Campers ruling is a win for the wowsers?’. By addressing her readers directly with the word ‘you’, Ford encourages them to feel personally invested in the issue of the legislation against offensive advertisements on vehicles in an accusatory manner, positioning them to feel guilty if they oppose her stance, hence encouraging them to accept her argument. Through descriptions companies such as the Wicked Campers as through the use of emotive words such as ‘feral’, she paints these companies in a negative manner by contributing to an impression that they uneducated and lack humanity, positioning her readers to feel disgusted by the actions of these companies and recognise the importance of the Government’s new legislation. Although Ford initially agrees that although there may be a limit in ‘freedom of speech’ and an ‘apparent death of ‘comedy’’ that comes with the legislation in a sarcastic manner, she argues that such jokes are not ‘new or interesting’, but are instead ‘hackneyed tropes’ that seeks to victimise those who are in ‘less power’. By painting such companies and people who make offensive comments as bullies, Ford positions her readers to feel an alarming sense of guilt if they have made such ‘jokes’ themselves. She reasons that people make such offensive comments because the violence ‘will never form a reality in their own lives’ and by providing examples that often women, Indigenous people and people with disabilities are often at the of these ‘jokes’. As such, Ford conveys that the impacts that such comments can have are very pervasive, positioning her readers to agree with her stance for the legislation.  By doing so, Ford reinforces her argument that the decision to reinforce the new legislation is very important as society can no longer tolerate companies advertising such offensive slogans that promote violence and undermine those considered with less power.

Ford continues to build upon her case by providing other examples, primarily to do with the misogynistic power of the slogans promoted by such companies. She argues that the advertisements made by the Wicked Campers are not only misogynistic, but also seeks to normalise misogyny, resulting in women’s personal safety being undermined from men’s misuse of their power. Through deductive reasoning that men have ‘nothing to fear from women’ Ford seeks to establish a negative image of men by painting them as having absolute control over women, positioning her female readers to feel a sense of sympathy towards her stance. By tying this image with the exaggeration that women ‘are beaten, raped and killed in huge numbers’ and the use of inclusive language ‘us’, she argues that women are nearly always the centre of the violence inflicted by men, encouraging her female readers to feel a sense of alarm and fear as they are characterised as easy victims of such violence. By undermining men with such an overstatement, Ford seeks to directly attack the Wicked Campers company as the misogynistic slogans are made by men, positioning her female readers to share her sense of outrage towards the actions of such companies. She continues to reinforce this argument by providing numerous other generalised examples the violence that men have inflicted towards women throughout time, employing highly emotive language such as ‘sex slaves’ and ‘murdered’, depicting all women as being victims of violence arising from misogyny. Hence, by positioning her readers to feel a sense of anger towards the misogyny of the advertisements made by the Wicked Campers, Ford once again supports her contention that the legislation is necessary, as it helps to stop such misogyny.

Similar to Ford’s opinion piece, Nina Springle’s opinion piece in the Huffington Post Blog also stresses the importance of the Queensland Government’s new legislation. In a more reasoned tone the Ford, she argues that inciting violence in the form of a joke is not ‘freedom of speech.’ Through the use of logic by highlighting that there is no clear definition of what is considered as freedom of speech and that Australia is deemed to be a ‘multicultural’ society with ‘equal respect’ between both genders, Springle not only seeks to undermine those who are against her stance, but also positions her readers to realise that there is no valid reason to allow such slogans as they only seek to promote violence and hatred. However, unlike Ford, Springle emphasises the detrimental effects the offensive slogans could have on children. By appealing to family values through the descriptions that children will ‘repeat them [the offensive slogans] in the playground’ and parents having to ‘field uncomfortable questions’, she seeks to target parent’s natural desire to protect their children. Furthermore, through the photograph depicting two Wicked Campers vehicles, with one having a slogan ‘If you can’t do it naked, it’s not worth doing!’, Springle provides credibility towards her argument as readers are made aware that such slogans do exist. By connecting the appeal with the photograph, Springle positions readers to agree with her stance on the new legislation being an appropriate solution, reinforcing her argument that offensive advertisements and jokes are not ‘freedom of speech’. 

However, whilst Ford emphasises that the new legislation is necessary to prevent the Wicked Campers company from promoting misogyny, Springle argues that all states also need to establish a similar legislation to Queensland. Through the use of an anecdote that she once received a response letter from the Victorian Minister for the Prevention of Family Violence, who ‘could not address the issue’ because the Advertising Standard Board is in charge of such issues, she provides a real life example of the lack of responsibility by the Victorian Government towards preventing companies from advertising such slogans. This positions the readers to feel a sense of urgency towards the need to establish a similar legislation in Victoria, and also a sense of anger as they are made aware of the Government’s lack of action to tackle such concerning issues. She continues to build upon her argument by attacking the Advertising Standards Board with the use of the metaphor that it ‘is essentially a toothless tiger’, suggesting that the Board has almost no power when it comes to deciding companies that have breached its rules, as the Wicked Campers will ‘ensure that its vehicles are registered interstate’. This dramatic comparison seeks to evoke a sense of urgency amongst readers, inclining them to agree with the Springle that there must be another alternative to stop the Wicked Campers from advertising their slogans in Victoria. By doing so, Springle reinforces her argument that other states must consider a similar legislation to Queensland to stop the Wicked Campers from promoting its offensive slogans. 

Both Ford and Springle agree that the Queensland Government’s new legislation to deregister vehicles with offensive advertisement is a necessary act. Nevertheless, each piece comes to the debate from a different perspective and presents a different overall contention. Ford takes on a more critical approach towards the importance of the legislation, arguing that society can no longer tolerate offensive advertisements that promote violence and misogyny. However, Springle, in a more reasoned manner, extends on this view, arguing that all other states and territories must also introduce a similar legislation to prevent companies such as the Wicked Campers from advertising their slogans more efficiently.
Title: Re: VCE English Question Thread
Post by: lovebiology on April 05, 2017, 06:14:31 pm
Hi, I am working through past VCAA essay prompts on Medea. I really don't know how to start the topic "Medea is about extremes of human emotion. Discuss." Like where should I start?
Title: Re: VCE English Question Thread
Post by: Moist on April 05, 2017, 10:47:31 pm
How do you write a written explanation for a creative SAC? Dont know how to structure it and what to include. Thanks

Structure I was taught was to begin with the author's idea that your creative piece extends on. From here, you offer your perspective on the this idea, follow with a link to audience and purpose then an explanation of your choices, including things like why you chosen a specific form or used a literary device. Number one rule when writing the explanation though is to always link back to your idea as, ideally, your piece should be similarly centered around it too.   

Hi, I am working through past VCAA essay prompts on Medea. I really don't know how to start the topic "Medea is about extremes of human emotion. Discuss." Like where should I start?

We haven't studied Medea yet at my school but first thing I would do is to define "emotions" in the context of the play. Ask yourself questions like: what are the emotions displayed? What happens when you display no emotion? What happens when you display too much emotion? From here, you'll eventually segue into asking yourself "what is Euripides saying about this?", wherein your answer will form the basis of your essay.

Question of my own, that is more particularly directed at Lauren, but I was just reading the sample argument analysis posted on the Facebook page and I was a bit confused about how it was structured. Could someone explain it to me, and also, would it be okay to use such structure for comparative essays?

Thanks in advance for all help.  :) :)
Title: Re: VCE English Question Thread
Post by: literally lauren on April 06, 2017, 07:35:23 am
Question of my own, that is more particularly directed at Lauren, but I was just reading the sample argument analysis posted on the Facebook page and I was a bit confused about how it was structured. Could someone explain it to me, and also, would it be okay to use such structure for comparative essays?

Thanks in advance for all help.  :) :)
Hey hey!

I've always had a very fluid/malleable approach to essay structure since I prefer essays that flow from one point to the next over ones that stick to anything too rigid or repetitive, but my recommendation for argument analysis is that you go by sub-arguments! So each of your paragraphs is centred around an idea or 'thing' (what I call a 'key player' = something the author positions in a certain way to strengthen their contention) and all you have to do is discuss the various language features that support that sub-argument.

So for the sample piece on the facebook page, I had:
   - Paragraph 1: the importance of biodiversity and why it is vulnerable and in need of preservation (+ 1st visual, since it deals with this same sentiment)
   - Paragraph 2: the dangers of inaction and the damaging repercussions this can have
   - Paragraph 3: the need for honest reappraisal of the situation and for a genuine commitment to tangible goals (+ 2nd visual, since it can be linked to this idea too)

Each of these things bolsters the author's contention in some way, so splitting the piece up like this has always helped me feel like I'm achieving a better coverage of the piece instead of just going through it chronologically or structuring things via techniques/tone etc.

That said, this is by no means the only way of doing things! If you look through previous years' assessment reports, you'll find a whole range of methods that students have used to score highly. It's all about how well you execute whatever strategy you're implementing :)
Title: Re: VCE English Question Thread
Post by: Moist on April 06, 2017, 07:51:43 am
Hey hey!

I've always had a very fluid/malleable approach to essay structure since I prefer essays that flow from one point to the next over ones that stick to anything too rigid or repetitive, but my recommendation for argument analysis is that you go by sub-arguments! So each of your paragraphs is centred around an idea or 'thing' (what I call a 'key player' = something the author positions in a certain way to strengthen their contention) and all you have to do is discuss the various language features that support that sub-argument.

So for the sample piece on the facebook page, I had:
   - Paragraph 1: the importance of biodiversity and why it is vulnerable and in need of preservation (+ 1st visual, since it deals with this same sentiment)
   - Paragraph 2: the dangers of inaction and the damaging repercussions this can have
   - Paragraph 3: the need for honest reappraisal of the situation and for a genuine commitment to tangible goals (+ 2nd visual, since it can be linked to this idea too)

Each of these things bolsters the author's contention in some way, so splitting the piece up like this has always helped me feel like I'm achieving a better coverage of the piece instead of just going through it chronologically or structuring things via techniques/tone etc.

That said, this is by no means the only way of doing things! If you look through previous years' assessment reports, you'll find a whole range of methods that students have used to score highly. It's all about how well you execute whatever strategy you're implementing :)

Hey Lauren, thanks for the reply. I have one more question: what structure would you recommend for comparing articles? Since won't it be too clunky to use this sub-arguments approach given that they'll most likely won't be the same for both pieces?

Again, thanks for all help.  :) :D


Title: Re: VCE English Question Thread
Post by: literally lauren on April 06, 2017, 07:57:51 am
Hey Lauren, thanks for the reply. I have one more question: what structure would you recommend for comparing articles? Since won't it be too clunky to use this sub-arguments approach given that they'll most likely won't be the same for both pieces?

Again, thanks for all help.  :) :D
Oh yeah! Totally forgot to address your actual question ::)

Yes, the reason I mentioned incorporating the visuals is because this approach allows you to blend the material waaaay easier than a chronological approach. As you've said, the sub-arguments (or rather, what the sub-arguments are about) will likely be the same across multiple texts, since the authors will be discussing the same issue. Though you might end up with something like a paragraph on 'the detrimental consequences of the proposal' where one author discusses environmental concerns and another one discusses economic or ethical implications. But all you'd have to do would be to analyse the first one and then say 'By contrast, Author B bitterly argues that the more serious consequences are fiscal ones...' to create a link.

Hope that makes sense!

Here's a somewhat messy example I did of the 2015 exam :)
Title: Re: VCE English Question Thread
Post by: Moist on April 06, 2017, 08:22:33 am
Oh yeah! Totally forgot to address your actual question ::)

Yes, the reason I mentioned incorporating the visuals is because this approach allows you to blend the material waaaay easier than a chronological approach. As you've said, the sub-arguments (or rather, what the sub-arguments are about) will likely be the same across multiple texts, since the authors will be discussing the same issue. Though you might end up with something like a paragraph on 'the detrimental consequences of the proposal' where one author discusses environmental concerns and another one discusses economic or ethical implications. But all you'd have to do would be to analyse the first one and then say 'By contrast, Author B bitterly argues that the more serious consequences are fiscal ones...' to create a link.

Hope that makes sense!

Here's a somewhat messy example I did of the 2015 exam :)

Oh whoopsies, sort of phrased my question wrong (sorry); I was going to ask about how would structure an analysis where the sub-arguments differ entirely from each other but still address the same issue? For example, still going with the climate change topic, what would I do if there's an article that argues that more action needs to taken otherwise everyone'll die, and another that goes off on a tangent and asserts that current passivity in addressing the issue demonstrates this generation's selfishness? Since won't it be essentially impossible to group them by sub-arguments given that they have entirely different contentions?

Anyways, thanks for your answer, it helped clear up a few of my other questions :)
Title: Re: VCE English Question Thread
Post by: literally lauren on April 06, 2017, 08:37:22 am
Oh whoopsies, sort of phrased my question wrong (sorry); I was going to ask about how would structure an analysis where the sub-arguments differ entirely from each other but still address the same issue? For example, still going with the climate change topic, what would I do if there's an article that argues that more action needs to taken otherwise everyone'll die, and another that goes off on a tangent and asserts that current passivity in addressing the issue demonstrates this generation's selfishness? Since won't it be essentially impossible to group them by sub-arguments given that they have entirely different contentions?

Anyways, thanks for your answer, it helped clear up a few of my other questions :)
Good question - VCAA will rarely give you anything that's too disparate since it makes it really hard to analyse, but there'll usually be some kind of connections to be made. To take your example, you could write a paragraph about 'why our response to global warming is important' (since the first article would be saying it's an urgent, life-or-death matter, and the second is saying that our response reflects our inner character and we should be more conscious of the consequences of our actions) and then one paragraph on 'personal responsibility' (with the first article presumably saying that everyone needs to do their part to avert the impending doom of climate change, and the second talking about why integrity and global awareness are vital traits for future generations to come to terms with), and maybe one on the idea of 'opportunity' (with the first saying 'this is our last opportunity to fix things!' and the second saying 'this is our opportunity to rise to this challenge and shake off that passivity/inaction) etc.

Importantly, though, if you're ever in a situation where you can't find 3 or 4 tenable sub-arguments to discuss, it's worth having another approach as a back-up option. When I had to analyse the 2011 exam piece for homework once, I couldn't find a realistic way to deal with the material (one blog post, two visuals, and four blog comments - each about entirely separate matters ::) ) so I just separated it into three roughly equal chunks and tried to just analyse whatever I could.

The sub-argument approach is a great first resort, but if you get dealt exceptionally difficult material, by all means find another more accessible way to break stuff down :)
Title: Re: VCE English Question Thread
Post by: Moist on April 06, 2017, 08:50:10 am
Good question - VCAA will rarely give you anything that's too disparate since it makes it really hard to analyse, but there'll usually be some kind of connections to be made. To take your example, you could write a paragraph about 'why our response to global warming is important' (since the first article would be saying it's an urgent, life-or-death matter, and the second is saying that our response reflects our inner character and we should be more conscious of the consequences of our actions) and then one paragraph on 'personal responsibility' (with the first article presumably saying that everyone needs to do their part to avert the impending doom of climate change, and the second talking about why integrity and global awareness are vital traits for future generations to come to terms with), and maybe one on the idea of 'opportunity' (with the first saying 'this is our last opportunity to fix things!' and the second saying 'this is our opportunity to rise to this challenge and shake off that passivity/inaction) etc.

Importantly, though, if you're ever in a situation where you can't find 3 or 4 tenable sub-arguments to discuss, it's worth having another approach as a back-up option. When I had to analyse the 2011 exam piece for homework once, I couldn't find a realistic way to deal with the material (one blog post, two visuals, and four blog comments - each about entirely separate matters ::) ) so I just separated it into three roughly equal chunks and tried to just analyse whatever I could.

The sub-argument approach is a great first resort, but if you get dealt exceptionally difficult material, by all means find another more accessible way to break stuff down :)

Thank you!! One more last last question: what do you mean by "three roughly equal chunks" - did you have one paragraph for the blogpost, another for the visual and the last one for the comments? Or was it something like: 1) blog post + a bit of relevant stuff from the visual 2) comments + some woven-in analysis of the post 3) visuals + comments?
Title: Re: VCE English Question Thread
Post by: literally lauren on April 06, 2017, 08:57:14 am
Thank you!! One more last last question: what do you mean by "three roughly equal chunks" - did you have one paragraph for the blogpost, another for the visual and the last one for the comments? Or was it something like: 1) blog post + a bit of relevant stuff from the visual 2) comments + some woven-in analysis of the post 3) visuals + comments?
The latter! :) It would've been tough to cram the whole blog post into just one paragraph whilst also stretching out analysis of the visual for one paragraph, and then erratically jumping between four different comments for the last paragraph.

Golden rule: let the spread of your analysis reflect the spread of the material!

The blog post that year was about a page long, the images were two tiny little rectangles, and the comments took up about a third of a page's worth of space. So spending one paragraph on each 'part' would've made the essay feel very imbalanced... which you can lose marks for. So making some effort to integrate your analysis is usually a huge advantage - even if all you do is talk about the written and visual texts in the same paragraph, you'll be in better stead than those who just leave the visual for the end and talk about it in isolation. The assessors will be looking for your ability to make some connections across the stuff they've given you - not at the expense of analysis, but certainly once or twice in an average paragraph :)
Title: Re: VCE English Question Thread
Post by: pixonman on April 07, 2017, 12:48:41 pm
How do you write a good short story in english
Title: Re: VCE English Question Thread
Post by: r3my on April 07, 2017, 02:21:50 pm
How do you write a good short story in english

All good creative writing, whether it be monologue, play or in your case short story, begins with a central idea or an overall message that you're trying to convey. All your choices such as literary techniques used, chosen form or plot should be geared towards expressing this idea. Don't aim for creative or writing flair as often you'll end up with a piece that lacks direction and seems confused. Needless to say though, also avoid making it totally dry and boring as you still have to keep your marker somewhat interested. :P

(this is at least what I've been taught - your teacher could expect something entirely different)
Title: Re: VCE English Question Thread
Post by: clarke54321 on April 07, 2017, 07:41:28 pm
For a comparative language/argument analysis, should I be analysing all three letters to the editor, for example? Or should I just analyse one or two with the article given?

Thanks  :)
Title: Re: VCE English Question Thread
Post by: zxcvbnm18 on April 08, 2017, 11:07:31 pm
How do i incoprate argument techniques ( ad hominon/ strawman's fallacy etc) into my language analysis essay. It would be great if someone could provide an exmaple and help me out as i am really struggling to get my head around this. Thnx in advance
Title: Re: VCE English Question Thread
Post by: HopefulLawStudent on April 09, 2017, 09:57:38 am
For a comparative language/argument analysis, should I be analysing all three letters to the editor, for example? Or should I just analyse one or two with the article given?

Thanks  :)

To be safe, you should be analysing all three.

How do i incoprate argument techniques ( ad hominon/ strawman's fallacy etc) into my language analysis essay. It would be great if someone could provide an exmaple and help me out as i am really struggling to get my head around this. Thnx in advance

Tbh don't bother; fancy argument techniques will only get you so far. At the end of the day, it's your elaboration and explanation that'll get you marks as an assessor doesn't really mark you based on how fancy your techniques are.

(I know, it's been a month since I last posted, where the heck have I been? Drowning in uni assignments  :'( )
Title: Re: VCE English Question Thread
Post by: zxcvbnm18 on April 09, 2017, 12:44:23 pm
To be safe, you should be analysing all three.

Tbh don't bother; fancy argument techniques will only get you so far. At the end of the day, it's your elaboration and explanation that'll get you marks as an assessor doesn't really mark you based on how fancy your techniques are.

(I know, it's been a month since I last posted, where the heck have I been? Drowning in uni assignments  :'( )

Really i got told that the new study design wants us to focus more on how the arguments are made/presented rather than the persuasive techniques?
Title: Re: VCE English Question Thread
Post by: TheCommando on April 09, 2017, 11:21:50 pm
How do u write a creative wirting peice for my next sac
Title: Re: VCE English Question Thread
Post by: ImmaculateJeff on April 16, 2017, 09:23:41 pm
How would schools turn Frankenstein into creative? like what would we have to do for its sac and shit?
Title: Re: VCE English Question Thread
Post by: zhen on April 16, 2017, 09:35:19 pm
How would schools turn Frankenstein into creative? like what would we have to do for its sac and shit?
My school gives us pretty open ended essay topics. Potential topics includes stuff like creating a new chapter inside the story or doing a creative on the stuff before or after the events in the book. Other topics include stuff like retelling parts of the book from another person's perspective. For creative I'm pretty sure that most schools will let you do whatever you like or give you topics where you can do pretty much anything.
Title: Re: VCE English Question Thread
Post by: ImmaculateJeff on April 16, 2017, 09:44:11 pm
My school gives us pretty open ended essay topics. Potential topics includes stuff like creating a new chapter inside the story or doing a creative on the stuff before or after the events in the book. Other topics include stuff like retelling parts of the book from another person's perspective. For creative I'm pretty sure that most schools will let you do whatever you like or give you topics where you can do pretty much anything.

ohhk thanks! :)
Title: Re: VCE English Question Thread
Post by: Max Kawasakii on April 16, 2017, 10:04:48 pm
I need someone to objective and honest when answering my question, lets just say 'hypothetically' on my language analysis sac I scored 7/10, hypothetically... If had aspirations of achieving a 40+ SS, should I just kick those hopes to the curb and strap my self in for a depressing outcome? Or by the lords is my dream still as realistic as I wish it to be? I'm genuinely distraught, I did so many practice essays, was getting great scores 36/40 on average, some higher! Then the actual sac came, I stressed, ran out of time, cut my analysis to the response short, did a whole lot of wrongs, and my score reflected this.

TLDR: Got 7/10 on my language analysis, have I rooted my dream of a 40+ SS? Thanks in advance to my harbinger of good news..
Title: Re: VCE English Question Thread
Post by: zhen on April 16, 2017, 10:17:58 pm
I need someone to objective and honest when answering my question, lets just say 'hypothetically' on my language analysis sac I scored 7/10, hypothetically... If had aspirations of achieving a 40+ SS, should I just kick those hopes to the curb and strap my self in for a depressing outcome? Or by the lords is my dream still as realistic as I wish it to be? I'm genuinely distraught, I did so many practice essays, was getting great scores 36/40 on average, some higher! Then the actual sac came, I stressed, ran out of time, cut my analysis to the response short, did a whole lot of wrongs, and my score reflected this.

TLDR: Got 7/10 on my language analysis, have I rooted my dream of a 40+ SS? Thanks in advance to my harbinger of good news..
I've heard someone at my school get a 50 with one SAC as a B and another person getting a 47 with one C+ SAC, but these are people who aced the rest of their SACs and the exam. Basically, one single SAC will not determine your whole study score and as long as you try hard and ace the rest of your SACs (ranking highly by the end of the year in your cohort in your SACs) and ace the exam, then 40+ is definitely achievable. I'd say 45+ and maybe even 50 is still achievable provided you ace everything else.
Title: Re: VCE English Question Thread
Post by: vcestressed on April 16, 2017, 10:53:19 pm
Hey Guys,
So I have a persuasive speech coming up really soon and my topic is surrogacy. My aim is to persuade the audience that surrogacy should be allowed in Australia. I've written the speech and I am happy with everything except my introduction. In school debates, I start with a really important and attention grabbing phrase, but with a topic like surrogacy, I am really stuck. I want to make the first thing I say conspicuous. Right now, my introduction is really boring  as it just states what surrogacy is and its problems etc etc and I really need some ideas. Thanks in advance! :'')
Title: Re: VCE English Question Thread
Post by: zhen on April 17, 2017, 04:08:15 pm
Hey Guys,
So I have a persuasive speech coming up really soon and my topic is surrogacy. My aim is to persuade the audience that surrogacy should be allowed in Australia. I've written the speech and I am happy with everything except my introduction. In school debates, I start with a really important and attention grabbing phrase, but with a topic like surrogacy, I am really stuck. I want to make the first thing I say conspicuous. Right now, my introduction is really boring  as it just states what surrogacy is and its problems etc etc and I really need some ideas. Thanks in advance! :'')
I'm really not sure what to say either, but maybe start with a statistic like one in something people are not able to have children and then proceed to elicit sympathy from the audience to those people. Or you could start with the stereotypical close your eyes and imagine if you were in their situation.
Title: Re: VCE English Question Thread
Post by: vcestressed on April 17, 2017, 04:42:53 pm
I'm really not sure what to say either, but maybe start with a statistic like one in something people are not able to have children and then proceed to elicit sympathy from the audience to those people. Or you could start with the stereotypical close your eyes and imagine if you were in their situation.
i think the 'close your eyes and imagine. . . ' would work perfectly! Thank you zhen!!
Title: Re: VCE English Question Thread
Post by: pmmenotes on April 22, 2017, 12:39:52 pm
Hey ATARNOTES so I have a language analysis sac on Monday and I've done about 4 Practice Sacs in preperation. With the grades I've received being, 80%, 70%, 60% and 70% again. My problem is that in my old school, I moved this year so this is my first and last year at this new school I always got 90%+ on my lang analysis and have always been a good english student. This isnt because the school i went to was bad. It has a higher ranking then the school I go to currently as it was a top private school. Anyways the teacher doesnt think Im a good student and has basically told me to just focus on the next sac (text response) so basically "give up" on this. The biggest problem with this is i recieved a 57% on my creative piece so I really REALLY can't afford to drop off again. I really don't know what to do as I know english is extremely important and my ranking is already super low. The SAC is worth 40 marks so its super important :(
Title: Re: VCE English Question Thread
Post by: lovebiology on April 22, 2017, 04:09:41 pm
Hi, for the topic ‘The play Medea is more about justice than revenge.’ Discuss. How should i start it? i am so confused... Thanks!
Title: Re: VCE English Question Thread
Post by: zhen on April 22, 2017, 05:40:26 pm
Hi, for the topic ‘The play Medea is more about justice than revenge.’ Discuss. How should i start it? i am so confused... Thanks!
I think for this topic the main idea is to discuss whether or not you think that Medea's actions are an act of justice or revenge. To start a text response essay, it's always a good idea to introduce the book and the ideas you're exploring. A stereotypical starting sentence is Set against the backdrop of ...... Euripides' Medea explores the notion of....
Title: Re: VCE English Question Thread
Post by: Max Kawasakii on April 22, 2017, 07:05:05 pm
I think for this topic the main idea is to discuss whether or not you think that Medea's actions are an act of justice or revenge. To start a text response essay, it's always a good idea to introduce the book and the ideas you're exploring. A stereotypical starting sentence is Set against the backdrop of ...... Euripides' Medea explores the notion of....

Further developing Zhen's response;

I'm presuming it's an analytical essay, so it's asking for your interpretation of the text in short.

Do you;
a) Belive Medea's actions were all for a cause of 'justice'?

b) Belive she was taking 'revenge' upon those who had wronged her?

Typically you chose 3 main 'arguments' to support what you belive, then you support your arguments with the use of quotes.

It's all about what you will find easier to write about, and how you interpreted the text.
Title: Re: VCE English Question Thread
Post by: lovebiology on April 23, 2017, 02:47:49 pm
I think for this topic the main idea is to discuss whether or not you think that Medea's actions are an act of justice or revenge. To start a text response essay, it's always a good idea to introduce the book and the ideas you're exploring. A stereotypical starting sentence is Set against the backdrop of ...... Euripides' Medea explores the notion of....

Thank you for the advice!
Title: Re: VCE English Question Thread
Post by: patriciarose on April 23, 2017, 09:18:55 pm
Hey ATARNOTES so I have a language analysis sac on Monday and I've done about 4 Practice Sacs in preperation. With the grades I've received being, 80%, 70%, 60% and 70% again. My problem is that in my old school, I moved this year so this is my first and last year at this new school I always got 90%+ on my lang analysis and have always been a good english student. This isnt because the school i went to was bad. It has a higher ranking then the school I go to currently as it was a top private school. Anyways the teacher doesnt think Im a good student and has basically told me to just focus on the next sac (text response) so basically "give up" on this. The biggest problem with this is i recieved a 57% on my creative piece so I really REALLY can't afford to drop off again. I really don't know what to do as I know english is extremely important and my ranking is already super low. The SAC is worth 40 marks so its super important :(

okay, so firstly, that's kind of awful that your teacher is like that :/ since you said monday, which is probably not far enough away for you to do another practice sac, i'd recommend you check out the language analysis section of the english board because there are articles there which you could maybe do a plan for? they posted heaps in 2016! give yourself ten/twenty minutes and do a detailed plan for one, then for the next cut the time down a bit, etc. also there's heaps of examples of really good language analysis there, so you can try to work out what they do well, which might be different from the way you're doing it currently.

if all else fails, there are heaps more sacs for you to do well on, and anyway your sac will be (probably? i think every school does it) crossmarked by another teacher at your school, who might be less harsh. you said you're averaging around 70, which isn't a bad score haha. just try to remember to discuss the way language contributes to argument and remember the exam will moderate your sac performance however well you do (or don't do) (:
Title: Re: VCE English Question Thread
Post by: zhen on April 23, 2017, 09:47:10 pm
Hey ATARNOTES so I have a language analysis sac on Monday and I've done about 4 Practice Sacs in preperation. With the grades I've received being, 80%, 70%, 60% and 70% again. My problem is that in my old school, I moved this year so this is my first and last year at this new school I always got 90%+ on my lang analysis and have always been a good english student. This isnt because the school i went to was bad. It has a higher ranking then the school I go to currently as it was a top private school. Anyways the teacher doesnt think Im a good student and has basically told me to just focus on the next sac (text response) so basically "give up" on this. The biggest problem with this is i recieved a 57% on my creative piece so I really REALLY can't afford to drop off again. I really don't know what to do as I know english is extremely important and my ranking is already super low. The SAC is worth 40 marks so its super important :(
Having a bad English teacher really sucks. I can empathise with you completely because that's how I feel with my English teacher. For me, I deal with my English teacher by constantly asking questions to see what he wants in my essay and to tailor my essay to his tastes. I'd also recommend asking your teacher why they gave that mark for your essay and what they expect you to do to get a higher mark. Also, if you think your English teacher is giving you bad marks because they think you're bad at English, try to prove them wrong and get on their good side. I'm pretty sure my English teacher originally thought I was stupid, but I presented some good pieces of writing and I'm pretty sure he changed his opinion of me. So, basically find out why your teacher is giving you those marks and ask them how you can improve. Also, try and get on their good side and make your teacher change their opinion if you.
Title: Re: VCE English Question Thread
Post by: TheCommando on April 23, 2017, 10:02:39 pm
Hey ATARNOTES so I have a language analysis sac on Monday and I've done about 4 Practice Sacs in preperation. With the grades I've received being, 80%, 70%, 60% and 70% again. My problem is that in my old school, I moved this year so this is my first and last year at this new school I always got 90%+ on my lang analysis and have always been a good english student. This isnt because the school i went to was bad. It has a higher ranking then the school I go to currently as it was a top private school. Anyways the teacher doesnt think Im a good student and has basically told me to just focus on the next sac (text response) so basically "give up" on this. The biggest problem with this is i recieved a 57% on my creative piece so I really REALLY can't afford to drop off again. I really don't know what to do as I know english is extremely important and my ranking is already super low. The SAC is worth 40 marks so its super important :(
You should feel confident that you've done 4 practise sacs leading into your language analysis sac. Not many people would have done this. It's important you get these peices graded and figure what you could have improved. For etc If your flow of structuring words and expression is poor doing simply things practising reading opinion articles from the herald sun (which is highly reccomended as the language used is sophisticated) can help alot. You may want to have a checklist of things you could have improved on and at the end of writing another essay check to see if your peice has sucessfully satisifed the criteria of the things u could have improved. For eg. If you need to i prove your expression having better vocab is one way, therefore check to see if your use of vocab was better. Like instead of saying show (which you shouldnt) say articulates, conveys etc
Title: Re: VCE English Question Thread
Post by: Max Kawasakii on April 23, 2017, 11:15:10 pm

One thing so many people skip over is the following; You get your marks for your analysis NOT identification! Let me elaborate slightly, you could identify every technique and persuasive device under the sun and not get full marks. It's all about identifying, analyzing and then discussing what the intended effect was.

Do everything you would normally for the introduction, then try to focus on how the writer has constructed their argument.

Example, The article has an opening, 3 main body paragraphs, then conclusion. Your analysis should have an introduction, three paragraphs of analysis (One key argument = 1 body paragraph) then your conclusion.

In each paragraph Identify the argument technique (What was used constantly throughout the paragraph)  (e.g Rhetoric, = Identify, analyze, then intended effect.)

Example body paragraph : "Alliteration was  predominantly throughout the first paragraph, "example of alliteration", the use of the word 'x' and 'y', both highlight and enable the readers to be consciously aware of the severity of their actions, in doing so readers are positioned to feel a strong sense of justice. The use of Rhetoric "Example of rhetoric", "Analysis here", in doing so "intended effect here.

Do you see the pattern yet?
In 1 body paragraph there should be;
1. 1x Argument technique (what's supporting the argument, and used frequently), 3x Persuasive devices; You then included an example, analyze it then state the intended effect.

Doing the above x3 will get you a very solid analytical essay!

This might be a bit confusing so please ask any questions! I really hope this is helpful in someway, but remember most people lose marks by forgetting the intended effect which is the biggest part!


Title: Re: VCE English Question Thread
Post by: HopefulLawStudent on April 24, 2017, 04:23:02 pm
How do u write a creative wirting peice for my next sac

Hey!

Would you be able to please specify what exactly it is that you're struggling with?? That way we can be of more assistance. :)

Hi, for the topic ‘The play Medea is more about justice than revenge.’ Discuss. How should i start it? i am so confused... Thanks!

Building on from the high quality answers from zhen and Max Kawasakii...

You'd want to consider the similarities and differences between the concepts of 'justice' and 'revenge' and the extent to which the concepts are evidenced in Medea. Moreover, the prompt seems to imply that justice and revenge are (to some extent?) mutually exclusive and that you can't have justice without revenge and vice versa so you'd defs want to unpack the extent to which this is true/false. Basically, branch out from the topic, considering what it is fundamentally saying (i.e. the core of the prompt) and from there, the implications of the prompt and the extent to which you agree with the cores/implications.
Title: Re: VCE English Question Thread
Post by: TheCommando on April 24, 2017, 07:06:41 pm
Hey!

Would you be able to please specify what exactly it is that you're struggling with?? That way we can be of more assistance. :)

Building on from the high quality answers from zhen and Max Kawasakii...

You'd want to consider the similarities and differences between the concepts of 'justice' and 'revenge' and the extent to which the concepts are evidenced in Medea. Moreover, the prompt seems to imply that justice and revenge are (to some extent?) mutually exclusive and that you can't have justice without revenge and vice versa so you'd defs want to unpack the extent to which this is true/false. Basically, branch out from the topic, considering what it is fundamentally saying (i.e. the core of the prompt) and from there, the implications of the prompt and the extent to which you agree with the cores/implications.
How would i best improve my creative writing
Apart from the geral tips like practise it get feedback and reading
Title: Re: VCE English Question Thread
Post by: clarke54321 on April 24, 2017, 07:18:13 pm
Could jaded be a tone for language analysis? The author opens his piece by preceding the first 9 sentences with, 'I could.....'

To me, this indicates a feeling of tiredness or exhaustion.
Title: Re: VCE English Question Thread
Post by: TheCommando on April 24, 2017, 08:15:22 pm
Could jaded be a tone for language analysis? The author opens his piece by preceding the first 9 sentences with, 'I could.....'

To me, this indicates a feeling of tiredness or exhaustion.
It can i think. However you should always if this is language analysis remmeber to analysis a technique in terms of its effects on the reader, how it positions the reader to feel etc. this should happen throughout your peice
Title: Re: VCE English Question Thread
Post by: n__n on April 24, 2017, 08:19:49 pm
Could jaded be a tone for language analysis? The author opens his piece by preceding the first 9 sentences with, 'I could.....'

To me, this indicates a feeling of tiredness or exhaustion.

Yup, but be careful though because jaded is sometimes used as slang for "I've had enough of this shit", so it might come off as a bit colloquial to some teachers.
Title: Re: VCE English Question Thread
Post by: clarke54321 on April 24, 2017, 08:34:31 pm
Yup, but be careful though because jaded is sometimes used as slang for "I've had enough of this shit", so it might come off as a bit colloquial to some teachers.

Yes, that's true. I'm just trying to find the right tone word to capture the feeling of an author, whose writing comes across as tired and sick of current situations. The way he constantly says 'I could' is almost like, I could do X, Y and Z, but my message would never get across. It's almost like he has given up. I'm just finding it difficult to pinpoint all of this.
Title: Re: VCE English Question Thread
Post by: patriciarose on April 24, 2017, 08:43:04 pm
Yes, that's true. I'm just trying to find the right tone word to capture the feeling of an author, whose writing comes across as tired and sick of current situations. The way he constantly says 'I could' is almost like, I could do X, Y and Z, but my message would never get across. It's almost like he has given up. I'm just finding it difficult to pinpoint all of this.

jaded does work in that context. i'm struggling to think of synonyms that would exactly match it in this context, but i suppose cynical, or even at a stretch sardonic (or exhausted? kind of basic but hey it'd get your point across), are the closest adjectives that'd also work? (since if you're cynical that suggests you've seen a lot of the same shit lmao. idk the context exactly but if they're mocking something they're tired of then sardonic is your word). but please anyone feel free to contradict this because i'm sure i've missed something haha. (:
Title: Re: VCE English Question Thread
Post by: n__n on April 24, 2017, 08:51:52 pm
Yes, that's true. I'm just trying to find the right tone word to capture the feeling of an author, whose writing comes across as tired and sick of current situations. The way he constantly says 'I could' is almost like, I could do X, Y and Z, but my message would never get across. It's almost like he has given up. I'm just finding it difficult to pinpoint all of this.

If I'm interpreting you correctly, as in you're saying the author is going  "I could do this, and I could do this, and this and this, but noooo, I'm surrounded by idiots", I would probably describe it as an almost-sarcastic (maybe even satirical or cynical) and disgruntled tone. Not sure though, but hope that helps? :-\
Title: Re: VCE English Question Thread
Post by: clarke54321 on April 24, 2017, 08:58:09 pm
If I'm interpreting you correctly, as in you're saying the author is going  "I could do this, and I could do this, and this and this, but noooo, I'm surrounded by idiots", I would probably describe it as an almost-sarcastic (maybe even satirical) and disgruntled tone. Not sure though, but hope that helps? :-\

Here is a snippet of the article from the Age:

"I could remind them that for all the passion the game inspires, it is only a game. I could urge them to remember that a footballer is also a human being, as sensitive as any other. I could ask them if they would like to be taunted for being who and what they are: fat, for example."

Ultimately, he comes to the conclusion that it wouldn't be worth doing all of this because no one would listen. Sorry, I should have given more context. I'm just trying to find a formal tone word to encapsulate feelings of defeat or tiredness. Thanks for all the help!
Title: Re: VCE English Question Thread
Post by: clarke54321 on April 24, 2017, 09:00:32 pm
jaded does work in that context. i'm struggling to think of synonyms that would exactly match it in this context, but i suppose cynical, or even at a stretch sardonic (or exhausted? kind of basic but hey it'd get your point across), are the closest adjectives that'd also work? (since if you're cynical that suggests you've seen a lot of the same shit lmao. idk the context exactly but if they're mocking something they're tired of then sardonic is your word). but please anyone feel free to contradict this because i'm sure i've missed something haha. (:

Thanks for the advice! Yes, I think exhausted is a good one. I can't find any other synonymous which accurately pinpoint the tone.
Title: Re: VCE English Question Thread
Post by: n__n on April 24, 2017, 11:23:48 pm
Here is a snippet of the article from the Age:

"I could remind them that for all the passion the game inspires, it is only a game. I could urge them to remember that a footballer is also a human being, as sensitive as any other. I could ask them if they would like to be taunted for being who and what they are: fat, for example."

Ultimately, he comes to the conclusion that it wouldn't be worth doing all of this because no one would listen. Sorry, I should have given more context. I'm just trying to find a formal tone word to encapsulate feelings of defeat or tiredness. Thanks for all the help!

Describing the tone as defeated is pretty much spot on imo, but maybe you could also add in something to indicate a sense of implied disappointment?
Title: Re: VCE English Question Thread
Post by: johnhalo on April 25, 2017, 12:04:04 pm
With prompts that end with a 'Do you agree?' is it advised to only partially agree so both sides can be discussed?

Or can you fully agree/disagree?
Title: Re: VCE English Question Thread
Post by: Quantum44 on April 25, 2017, 12:13:34 pm
With prompts that end with a 'Do you agree?' is it advised to only partially agree so both sides can be discussed?

Or can you fully agree/disagree?

Yes, the best responses generally interrogate the topic so you can explore both sides of the argument, giving greater depth to your response and allowing you to show more knowledge of the text.
Title: Re: VCE English Question Thread
Post by: zhen on April 25, 2017, 12:48:07 pm
With prompts that end with a 'Do you agree?' is it advised to only partially agree so both sides can be discussed?

Or can you fully agree/disagree?
To add onto quantum44's response, I agree that the best responses always explore both sides of the argument, but I think that it's good to have a main point or argument in your essay, so basically take a side to whether you agree or disagree, however acknowledge the opposing argument and suggest that you agree/disagree, but not completely since there are exceptions. I think this allows you to explore both sides of the argument and shows your complexity of ideas.
Title: Re: VCE English Question Thread
Post by: zxcvbnm18 on April 25, 2017, 01:57:33 pm
Hi guys
I was wondering if for language anaylsis we need to talk about 'argument techniques' i.e ad hominen, strawman's fallacy etc if its present or just concentrate on the language used by the writer to persuade their audience.
Thnx
Title: Re: VCE English Question Thread
Post by: zhen on April 25, 2017, 02:12:40 pm
Hi guys
I was wondering if for language anaylsis we need to talk about 'argument techniques' i.e ad hominen, strawman's fallacy etc if its present or just concentrate on the language used by the writer to persuade their audience.
Thnx
I think that argument analysis is focussing on how argument positions the reader and how the language used supports the argument and positions the reader. So, I don't think you need to talk about argument techniques, instead you should be focussing on the content of the argument and its purpose. I'm not too sure about this, so someone should correct me if I'm wrong.
Title: Re: VCE English Question Thread
Post by: patriciarose on April 25, 2017, 02:20:32 pm
With prompts that end with a 'Do you agree?' is it advised to only partially agree so both sides can be discussed?

Or can you fully agree/disagree?

discussing both sides means you run the risk of sitting on the fence, which is bad. fully agreeing or fully disagreeing is also generally not the best because you limit your argument to the exact parameters of the question (although sometimes you get a question/statement that is just totally wrong/right so obviously don't try and create conditions for it that don't exist lmao). generally i think the easiest way to do it well is to agree or disagree with a but, because that lets you add a little more depth to your answer than you'd otherwise get.
so for the record, yes, you CAN fully agree or disagree and still do well, but agreeing/disagreeing with conditions tends to make it easier to score highly.
Title: Re: VCE English Question Thread
Post by: clarke54321 on April 25, 2017, 02:27:58 pm
I think that argument analysis is focusing on how argument positions the reader and how the language used supports the argument and positions the reader. So, I don't think you need to talk about argument techniques, instead you should be focussing on the content of the argument and its purpose. I'm not too sure about this, so someone should correct me if I'm wrong.

Yes, I agree with this.

You want to take a more holistic approach to the piece. Focusing too much on the argument techniques can make your analysis too 'listy' and detract from your understanding of the piece as a whole. However, if you can identify argument techniques prevalent in the piece, which will assist your analysis or explanation, then include it by all means. I'm just saying that a constant identification of techniques can weaken your analysis.

Hope this helps!
Title: Re: VCE English Question Thread
Post by: patriciarose on April 25, 2017, 03:01:18 pm
Hi guys
I was wondering if for language anaylsis we need to talk about 'argument techniques' i.e ad hominen, strawman's fallacy etc if its present or just concentrate on the language used by the writer to persuade their audience.
Thnx

there's nothing that says you CAN'T talk about techniques but don't focus on them too much. vcaa literally named the essay 'argument analysis' so mention the techniques if they're contributing to the overall development of the argument but don't focus on listing etc. techniques are quite useful though because if you can identify them, you generally then know why they'd being used and thus the effect they have, which leads naturally into argument analysis territory. so they're nice to know and it doesn't hurt to briefly mention them but make sure they're contributing to your analysis, not detracting from it or wasting your time, if that makes sense? (:
Title: Re: VCE English Question Thread
Post by: zxcvbnm18 on April 25, 2017, 03:21:36 pm
Thnx everyone! Yall made it much more clear
Title: Re: VCE English Question Thread
Post by: j.wang on April 25, 2017, 03:36:36 pm
hellooo

so i'm doing creative and i always hear that we should challenge the text's ideas/ themes and not just reuse symbols

how exactly do we challenge the ideas??
Title: Re: VCE English Question Thread
Post by: n__n on April 25, 2017, 07:09:19 pm
hellooo

so i'm doing creative and i always hear that we should challenge the text's ideas/ themes and not just reuse symbols

how exactly do we challenge the ideas??


To challenge the author's ideas, you just extend on them. For instance, let's say the author's idea is "war is bad because people die"; you can agree with this to a degree, but not 100% as otherwise you'll just be copying the author (nor can you 100% disagree). Instead, you can for example say "war is bad because people die BUT it is good in certain situations for population control" (probably not the most morally correct example, but whatever). See how the addition of the last part extends on the original idea, making it your own and essentially challenging it?

Some questions of my own;
- How I do write quicker under SAC conditions? I do this thing during English SACs where I just procrastinate (?) by continually rereading over what I've already written and staying there doing nothing. It's not because I don't know what to write next, I do, it's just that I don't want to move on because I feel like what I've written is either not good enough (or even in some cases too good - I just sit there impressed at my own writing. I know, I'm humble), or what I'm going to write next is too hard. Any tips or advice to get out of this mindset?
- Can someone confirm the contention for this piece for me? I've identified it as something along the lines of "The changes to the citizenship test does not help in ensuring that potential citizens abide by Australian values, and the decision to revise it is a political stunt designed to regain dwindling Coalition support by playing to growing populist views of xenophobia and racism."

Thanks in advance for all help!!  :) :)
Title: Re: VCE English Question Thread
Post by: patriciarose on April 26, 2017, 01:12:12 am
Some questions of my own;
- How I do write quicker under SAC conditions? I do this thing during English SACs where I just procrastinate (?) by continually rereading over what I've already written and staying there doing nothing. It's not because I don't know what to write next, I do, it's just that I don't want to move on because I feel like what I've written is either not good enough (or even in some cases too good - I just sit there impressed at my own writing. I know, I'm humble), or what I'm going to write next is too hard. Any tips or advice to get out of this mindset?
- Can someone confirm the contention for this piece for me? I've identified it as something along the lines of "The changes to the citizenship test does not help in ensuring that potential citizens abide by Australian values, and the decision to revise it is a political stunt designed to regain dwindling Coalition support by playing to growing populist views of xenophobia and racism."

Thanks in advance for all help!!  :) :)

1. i feel you on the rereading part, haha. i don't think i finished an in class essay until half way through year eleven because i was so focused on perfection. i think if that's your problem, a good way to look at it is that a half finished good essay will do worse than an averagely written finished essay. just don't reread until the end. personally, my english sacs are eighty minutes and i write five paragraphs, so i give myself fifteen minutes per paragraph and fifteen combined for the intro/conclusion, and force myself to move to the next one if i hit the time limit (although once you have one you seem to instinctively write faster, which is nice). then at the end i go back and edit the heck out of everything i've written, which is better anyway because it gives me time to forget my phrasing, which means that when i go back if anything sounds clunky i can edit it out, whereas if you edit as you go along it's more likely you'll skim over something that sounded good in the moment and really bad later (: if you can't think of how to start the next paragraph, honestly just start. pick the first thing that comes into your head and go with it. ideally you'll have a rough plan already so that should keep your ideas linear, but if not, hey, you'll lose a mark for clarity, and that's a whole lot better than losing several for not finishing because you spent twenty minutes on an opening sentence. (:

2. yup, that contention basically works for me. i'm too sleepy to put it in great english, but yeah she's basically saying that it's not a bad thing to reform the citizenship test, and that in fact she believes it should happen bc migrants don't know our values, but the /way/ it's going to be done is pandering to xenophobic groups and won't assist or benefit those actually taking it. but please don't write it like that omg i'm v messy.
Title: Re: VCE English Question Thread
Post by: Mariamnourine on April 26, 2017, 02:16:28 pm
How many essays should I write a day to improve my English?
Title: Re: VCE English Question Thread
Post by: patriciarose on April 26, 2017, 02:33:48 pm
How many essays should I write a day to improve my English?

idk if that's the best way to improve your english? if it's vocabulary you're after i'd suggest building word banks or honestly just plain old reading? but i guess if you're after improving clarity and set on essays, don't do more than one a day unless you're studying for a sac! you'll just get stale and bored of it, or burn out. tbh i try to write a paragraph/plan an essay every two days and that's just enough that i don't actively resent the subject (which i'd hate to do since i'll have to spend so much time on it soon). more than one essay a day honestly sounds like madness to me. but someone else might have a different perspective?
Title: Re: VCE English Question Thread
Post by: zhen on April 26, 2017, 05:55:48 pm
How many essays should I write a day to improve my English?
I'd say on average an essay a week or maybe a bit less is good enough. Churning out essays does not necessarily improve your English ability if you don't improve and change how you write. When I write an essay, I generally hand it to someone to correct and then make a list of mistakes I made in that essay. Then I either write a new essay and try to avoid those mistakes or rewrite the essay fixing the mistakes made. I feel like by doing this I actively improve my skills rather than just writing heaps of essays and not making any progress.
Title: Re: VCE English Question Thread
Post by: zxcvbnm18 on April 26, 2017, 08:44:59 pm
What if an english subject is not in your top 4? How does this affect your atar?
Title: Re: VCE English Question Thread
Post by: vcestressed on April 26, 2017, 08:49:17 pm
What if an english subject is not in your top 4? How does this affect your atar?
English is in your top 4 no matter what score you achieve. Its a very sad life.
Title: Re: VCE English Question Thread
Post by: MattBro on April 26, 2017, 10:23:58 pm
Hi,
Does anyone know of any way to compare corruption to a game of cards?
Title: Re: VCE English Question Thread
Post by: zxcvbnm18 on April 26, 2017, 10:25:52 pm
English is in your top 4 no matter what score you achieve. Its a very sad life.
Oh nooooooo
Title: Re: VCE English Question Thread
Post by: zhen on April 26, 2017, 10:38:10 pm
Hi,
Does anyone know of any way to compare corruption to a game of cards?

I think maybe talking about cheating in card games or stuff like card counting, which allows people to gain an unfair advantage could be comparable to corruption.
Title: Re: VCE English Question Thread
Post by: vcestressed on April 29, 2017, 09:43:09 pm
Approx how many quotes should a text response essay have?
Title: Re: VCE English Question Thread
Post by: Quantum44 on April 29, 2017, 09:48:16 pm
Approx how many quotes should a text response essay have?

I usually aim for 6 per body paragraph although I don't really bother counting, just add em in where it fits.
Title: Re: VCE English Question Thread
Post by: zhen on April 29, 2017, 09:53:34 pm
Approx how many quotes should a text response essay have?
I'm pretty sure that you should aim to have at least one quote for every 2 to 3 sentences in a body paragraph of your text response essay. However, this is probably something you should ask your teacher, since different teachers might want different number of quotes and also since I'm no expert on these types of things.
Title: Re: VCE English Question Thread
Post by: n__n on April 30, 2017, 02:33:42 pm
Question: In Medea, would it be okay to comment that Euripides condemns extremity through the fact that the prejudices (i.e sexism and xenophobia) of Medea's society makes conventional justice impossible for her? Or is incorrect to label the prejudices as 'extremities'?
Title: Re: VCE English Question Thread
Post by: pmmenotes on April 30, 2017, 08:23:46 pm
Thoughts on my Medea essay? any improvements etc
What was your reaction to Medea and Jason by the end of the play? For whom did you feel sympathy and why?

Euripides’ tragedy, Medea is an exploration of the two main characters antipodal natures in the face of  conflict—passion and reason, with passion being found in Medea and reason found in the Jason, which becomes more and more evident as the play progresses. Throughout the play Medea, the titular character is a frightening and passionate woman who once after finding out her husband and father of her children participated in adultery, lets her reasoning and logic disappear. With Medea by the end of the play performing one of the most atrocious acts a mother can do, murdering her children to spite her husband, Jason, a man not completely innocent in his acts. Towards the end of the play it is evident that Jason's manipulative actions led to his children demise by the hands of his scorned wife. Whilst Medea although being wronged by Jason still does not garner the audiences sympathy due to her committing the terrible crime of filicide even acknowledging herself  that ‘my passion is master of my reason’, Euripides makes sure with this sentence that the readers be inclined to feel more sympathy towards Jason as the crime committed against him, the murder of his children being more severe and heartbreaking, then his abandonment of Medea.


Euripides character and the original ‘antagonist’ of the play, Jason commits many wrongs against Medea including infidelity with the princess and later on arranging a marriage with her. This marriage to Medea and the audience at the time can be perceived as a huge betrayal in which he claims to Medea as being a union, being both beneficial to him and his “family”, referring to the two sons he and Medea share. His acts against Medea although selfish and conniving are still outweighed by Medea's ultimate betrayal of murdering there children in order to spite and seek vengeance against Jason nullifying any initial sympathy readers may have had for her. Euripides also portrays the character of Jason as a sexist male character somewhat symbolic of the very patriarchal greek society. With phrases such as “You woman are all the same” after Medea angrily reacts to his new marriage and other sayings such as "What we poor males really need is a way of having babies on our own – no females, please. Then the world would be completely trouble free” validating Medea's feelings on her being ‘a woman’ highlighting his sexist nature in regard to woman. During these moments, sympathy for Jason is hard to find and easy to feel for Medea. Jason's seemingly sexist nature and reasoning for his marriage to Creon's daughter the princess is understandable. Jason tells Medea that "I was not […] tired of your attractions […] it was simply that I wanted above all to let us live in comfort, not be poor" . The use of the word ‘us’ when speaking to Medea about his upcoming marriage to Glauke (the princess) shows that despite his sexist nature and his insensitive actions to Medea, he still had Medea's best interest in heart, stating his marriage to Glauke as not only being beneficial to Jason and his kids but “us” Medea included in that equation, rescuing both himself and Medea from poverty and assuring their sons a place in society. Medea's banishment was not  part of the plan, reminding the audience that Medea bought the punishment down on herself and her children. Medea chose revenge against Jason to be more important than protecting her own children, whilst Jason always had his children's interest at heart once again proving that the love Jason has for there children is more than the love Medea posses for them, once again forcing the audience to sympathise with the somewhat ignorant character that is Jason.


In the beginning of the play before Medea spirals out of control, the character in which the audience’s sympathies are extended towards is Medea, a choice which seems like an obvious one. With a prominent scene before Medea is even introduced, the Nurse explains the pitiful situation She explains how Medea and Jason once had a wonderful relationship. Speaking of Medea's current state, she says, “…she will neither eat nor drink, except her own tears, She turns her face towards the earth, remembering her father’s house and her native land, which she abandoned for the love of this man: who now despises her,” showing both how depressed she is and the ‘hate’ Jason presumably has for her. With statements like these and Medea's crying the audience is also more likely to sympathise with Medea rather than Jason in the beginning of the play because they have yet to meet the true nature of her character and see her determination for vengeance as well as only bing subjected to hearing about Jason's wrong doings from the nurse and the chorus. Medea is once again sympathised with when she is banished from Corinth because her children will also be included in that banishment, characters which the audience can view as the only true innocents in the play. Medea’s misery continues to be  evident in her “lamentations” that “it’s death [she wants]” and that Jason’s actions “torment [her] heart”. She also states that she herself that her logic has been skewed by her intense feelings of contempt for Jason and that her ‘[my] passion is master of my [her] reason’. It is her fierce pride which clouds her judgement and leads her to commit filicide, as she “[would] not tolerate” being captured by her enemies and does not want to be thought of as “weak and feeble”. However, it is notable that Medea has moments of clarity where she concedes that she is “well aware” of the “terrible crime” she is to commit, highlighting how even in such a drastic state of mind unlike Jason, Medea is aware of the harm she is inducing yet does not care as long as she gets her revenge on Jason, enticing sympathy to Jason and none for her.


Ultimately Euripides tragic tale of Medea although does invoke feelings along the way of sympathy for both of the main characters, the titular character of Medea, the scorned wife and the ignorant Jason ultimately forces the readers to pick one of the characters as the most sympathetic, Jason. After making Medea perform the heinous act of filicide one even she cannot come back from any feelings of sympathy for her come character is eradicated, proving that even one of the most ignorant characters Jason can be the most sympathy worthy character over a abandoned wife.
Title: Re: VCE English Question Thread
Post by: zhen on April 30, 2017, 09:15:34 pm
Thoughts on my Medea essay? any improvements etc
What was your reaction to Medea and Jason by the end of the play? For whom did you feel sympathy and why?

Euripides’ tragedy, Medea is an exploration of the two main characters antipodal natures in the face of  conflict—passion and reason, with passion being found in Medea and reason found in the Jason, which becomes more and more evident as the play progresses. Nice start to the essay, which properly introduces the themes and the text. Maybe try to link this to the topic more, since right now the link isn't obvious. Throughout the play Medea, the titular character is a frightening and passionate woman who once after finding out her husband and father of her children participated in adultery, lets her reasoning and logic disappear. With Medea by the end of the play performing one of the most atrocious acts a mother can do, murdering her children to spite her husband, Jason, a man not completely innocent in his acts. This is just retelling the story, which is something you should avoid Towards the end of the play it is evident that Jason's manipulative actions led to his children demise by the hands of his scorned wife. Whilst Medea although being wronged by Jason still does not garner the audiences sympathy due to her committing the terrible crime of filicide even acknowledging herself  that ‘my passion is master of my reason’, Euripides makes sure with this sentence that the readers be inclined to feel more sympathy towards Jason as the crime committed against him Be careful of analysing quotes in the introduction, since the introduction isn't supposed to be where you analyse the text, but when you introduce your arguments and the text, the murder of his children being more severe and heartbreaking, then his abandonment of Medea.

Euripides character and the original ‘antagonist’ of the play, Jason commits many wrongs The expression here is weird against Medea including infidelity with the princess and later on arranging a marriage with her Again be careful about retelling the story. This marriage to Medea and the audience at the time can be perceived as a huge betrayal in which he claims to Medea as being a union, being both beneficial to him and his “family”, referring to the two sons he and Medea share. His acts against Medea although selfish and conniving are still outweighed by Medea's ultimate betrayal of murdering there children in order to spite and seek vengeance against Jason nullifying any initial sympathy readers may have had for her.  Solid points here.  Euripides also portrays the character of Jason as a sexist male character somewhat symbolic of the very patriarchal greek society. With phrases such as  Although this is ok, try to imbed the quotes into your analysis more, rather than introducing the quote with these kinds of sentences. “You woman are all the same” after Medea angrily reacts to his new marriage  There is no need to put the context of the quote in here, since it doesn't add anything to your argument.  and other sayings such as "What we poor males really need is a way of having babies on our own – no females, please. Then the world would be completely trouble free” validating Medea's feelings on her being ‘a woman’ highlighting his sexist nature in regard to woman. During these moments, sympathy for Jason is hard to find and easy to feel for Medea. Jason's seemingly sexist nature and reasoning for his marriage to Creon's daughter the princess is understandable. I feel like this sentence contradicts your previous sentences, since you're saying that at this point in the play, Jason doesn't have the sympathy of the reader, so I'd think that his actions would not be understandable. Jason tells Again you don't want to say Jason tells or mentions, instead you should try to imbed your quotes. Something like Euripides highlights that Jason's actions were motivated by the his desire to "live in comfort" would be better in my opinion Medea that "I was not … tired of your attractions … it was simply that I wanted above all to let us live in comfort, not be poor" . The use of the word ‘us’ when speaking to Medea about his upcoming marriage to Glauke (the princess) shows that despite his sexist nature and his insensitive actions to Medea, he still had Medea's best interest in heart, stating his marriage to Glauke as not only being beneficial to Jason and his kids but “us” Medea included in that equation, rescuing both himself and Medea from poverty and assuring their sons a place in society. I really like the deep analysis here. Medea's banishment was not  part of the plan, reminding the audience that Medea brought the punishment down on herself and her children. Medea chose revenge against Jason to be more important than protecting her own children, whilst Jason always had his children's interest at heart once again proving that the love Jason has for there children is more than the love Medea posses for them, once again forcing the audience to sympathise with the somewhat ignorant character that is Jason I feel like this isn't needed and detracts from your argument.


In the beginning of the play before Medea spirals out of control, the character in which the audience’s sympathies are extended towards is Medea, a choice which seems like an obvious one. With a prominent scene before Medea is even introduced, the Nurse explains the pitiful situation She explains how Medea and Jason once had a wonderful relationship. Speaking of Medea's current state, she says, “…she will neither eat nor drink, except her own tears, She turns her face towards the earth, remembering her father’s house and her native land, which she abandoned for the love of this man: who now despises her,” showing both how depressed she is and the ‘hate’ Jason presumably has for her. Try to avoid retelling the story. With statements like these and Medea's crying the audience is also more likely to sympathise with Medea rather than Jason in the beginning of the play because they have yet to meet the true nature of her character and see her determination for vengeance as well as only bing subjected to hearing about Jason's wrong doings from the nurse and the chorus.  Something minor but this sentence is really long, making it really convoluted. Medea is once again sympathised with when she is banished from Corinth because her children will also be included in that banishment, characters which the audience can view as the only true innocents in the play. Medea’s misery continues to be  evident in her “lamentations” that “it’s death [she wants]” and that Jason’s actions “torment [her] heart”.  Good imbedding of quotes. She also states that she herself that her logic has been skewed by her intense feelings of contempt for Jason and that her ‘passion is [the] master of [her] reason’. It is her fierce pride which clouds her judgement and leads her to commit filicide, as she “[would] not tolerate” being captured by her enemies and does not want to be thought of as “weak and feeble”.  Good analysis here. However, it is notable that Medea has moments of clarity where she concedes that she is “well aware” of the “terrible crime” she is to commit, highlighting how even in such a drastic state of mind unlike Jason, Medea is aware of the harm she is inducing yet does not care as long as she gets her revenge on Jason, enticing sympathy to Jason and none for her. Some good analysis here


Ultimately Euripides tragic tale of Medea although does invoke feelings along the way of sympathy for both of the main characters, the titular character of Medea, the scorned wife and the ignorant Jason I don't think it's necessary to reintroduce the characters, since you've already introduced them ultimately forces the readers to pick one of the characters as the most sympathetic, Jason. After making Medea perform the heinous act of filicide one even she cannot come back from any feelings of sympathy for her come character is eradicated, proving that even one of the most ignorant characters Jason can be the most sympathy worthy character over a abandoned wife.

You should post your essays here next time. https://atarnotes.com/forum/index.php?board=406.0
Take my advice with a grain of salt, since I'm in year 12 this year. Overall it's a solid essay with good analysis, but generally the biggest problem is you retell the story too much. Anyway good job and keep up the good work.  :D

Edit: Forgot to mention that blue is the stuff I think is good, red is the stuff I think needs improvement, whereas green is my comments. Also, I feel like you should have at least 3 body paragraphs for your essay.
Title: Re: VCE English Question Thread
Post by: pmmenotes on May 01, 2017, 06:09:42 pm
You should post your essays here next time. https://atarnotes.com/forum/index.php?board=406.0
Take my advice with a grain of salt, since I'm in year 12 this year. Overall it's a solid essay with good analysis, but generally the biggest problem is you retell the story too much. Anyway good job and keep up the good work.  :D

Edit: Forgot to mention that blue is the stuff I think is good, red is the stuff I think needs improvement, whereas green is my comments. Also, I feel like you should have at least 3 body paragraphs for your essay.

Thank you so much Zhen! literally helped me so much! I see all the points you made and will make those changes and ill also post on there from now on lolll
Title: Re: VCE English Question Thread
Post by: Ashjames on May 03, 2017, 12:27:48 pm
My SAC's have been like this 83%' 76% and  65% I know I have a lot of gooddamn potential but I'm just not practising enough and also make up excuses. My teacher always reassures my that I have a lot of potential and have the capability of getting 40+ . Some one help me to achieve this with my current sac scores and give me hope. I've promised myself to try harder for my next sacs.  :'( :-\
Title: Re: VCE English Question Thread
Post by: pmmenotes on May 03, 2017, 01:50:27 pm
current english grades are 57% (creative) and 70% (lang analysis) do i have any chance of getting a 35 study score 😥 keep in mind im good at Medea essays/text responses have scored 8/10, 9/10 on practice ones
Title: Re: VCE English Question Thread
Post by: zhen on May 03, 2017, 05:08:57 pm
My SAC's have been like this 83%' 76% and  65% I know I have a lot of gooddamn potential but I'm just not practising enough and also make up excuses. My teacher always reassures my that I have a lot of potential and have the capability of getting 40+ . Some one help me to achieve this with my current sac scores and give me hope. I've promised myself to try harder for my next sacs.  :'( :-\
Percentages and exact SAC scores don't matter. What matters is your SAC rank. So, if you're ranked highly, then these SAC percentages don't really matter. If your school is like mine, we only have 6 SACs throughout the year, meaning that you've finished half your SACs. Assuming that your rank is above average but not excellent scores (which would be standard for those types of marks), then it's definitely still possible to get 40+. As long as you ace the rest of your SACs and the exam it's definitely possible. That said, it'll still be very difficult, but it's definitely doable.

current english grades are 57% (creative) and 70% (lang analysis) do i have any chance of getting a 35 study score 😥 keep in mind im good at Medea essays/text responses have scored 8/10, 9/10 on practice ones
Like I said above SAC ranking is what matters, not percentage. Right now it looks like you're getting average SAC marks (That's about average at my school, so I'm assuming that it's the average at your school). But you've still got 4 more SACs to improve your ranking and the exam. So, I'd say 35+ is definitely doable as long as you put in the work. It looks like you're gradually improving your marks right now. So, if you keep working hard, It's definitely possible to get 35+.

Note that I've heard a person get C+ for one of their SACs at my school and go on go get a 47 and I've heard a person get 50 with one of their SACs as a C, but these people aced the rest of their SACs and the exam. So, as long as you work hard, then most scores are still achievable.
Title: Re: VCE English Question Thread
Post by: deStudent on May 03, 2017, 05:20:23 pm
For everyone who is complaining about their scores, if you're getting 80%+ you are in highest band possible, ie A+. If your cohort is any good at English you are likely to get 40+ if you keep getting similar marks.
Title: Re: VCE English Question Thread
Post by: nicnia on May 04, 2017, 08:08:28 pm
Would someone be able to link or show me a high scoring text response essay for Frankenstein? Doesn't have to be a 10/10, just anything from 8/10-10/10
Title: Re: VCE English Question Thread
Post by: TheCommando on May 07, 2017, 03:57:45 pm
Is there any annotations rescources or sample top scoring essays for the section c language analysis of last years exams?
Title: Re: VCE English Question Thread
Post by: snoz11 on May 07, 2017, 07:48:13 pm
creative writing doing me head in, does anyone have a sample written explanation/statement of intention for a news report?
Title: Re: VCE English Question Thread
Post by: clarke54321 on May 07, 2017, 09:35:20 pm
How could I refer to the repeated phrase of 'we want' in an article. The writer continually reinstates that 'we want students to be exposed to new experiences' and that 'we want them to experience challenge.' Could I refer to it as a dictate? I don't think this really works, but can't find a better word to describe it.

Does anyone have suggestions?
Title: Re: VCE English Question Thread
Post by: Gogo14 on May 08, 2017, 07:14:44 pm
How could I refer to the repeated phrase of 'we want' in an article. The writer continually reinstates that 'we want students to be exposed to new experiences' and that 'we want them to experience challenge.' Could I refer to it as a dictate? I don't think this really works, but can't find a better word to describe it.

Does anyone have suggestions?
You could replace "we" or "want" individually with synonyms. e.g. The audience want... or we desire
Title: Re: VCE English Question Thread
Post by: patriciarose on May 09, 2017, 12:49:41 pm
How could I refer to the repeated phrase of 'we want' in an article. The writer continually reinstates that 'we want students to be exposed to new experiences' and that 'we want them to experience challenge.' Could I refer to it as a dictate? I don't think this really works, but can't find a better word to describe it.

Does anyone have suggestions?

if you're asking for the technical term, i have no idea. if you want to know how to refer to it, though, that sounds to me like a kind of a constant demand, so: demand, command that, perfunctorily request, call for, enjoin (a stretch oops), adjure (the last one is a maybe though because not quite sure of the tone of your piece haha). dictate grammatically might work, but i'd agree that it doesn't exactly sound /right/. dictate their demands, maybe? idk. this is probably too late for you to use but i'll post it just in case (:
Title: Re: VCE English Question Thread
Post by: peterpiper on May 09, 2017, 06:23:13 pm
How could I refer to the repeated phrase of 'we want' in an article. The writer continually reinstates that 'we want students to be exposed to new experiences' and that 'we want them to experience challenge.' Could I refer to it as a dictate? I don't think this really works, but can't find a better word to describe it.

Does anyone have suggestions?

Don't use dictate. That doesn't really make sense. The word you're better off using is something like 'demanding' or 'protest'. Best not to linger on what it sounds like. An approximation is as good as any word that will contribute and expand your thesis.
Title: Re: VCE English Question Thread
Post by: clarke54321 on May 09, 2017, 06:24:14 pm
Thanks for the input everyone!  :)
Title: Re: VCE English Question Thread
Post by: Joseph41 on May 11, 2017, 03:29:47 pm
(http://i.imgur.com/NyIZuuk.png)

Welcome to the English Questions thread! ;D

To post a question or response, you'll first need to make a free ATAR Notes account. It should take about four seconds! Then, simply scroll down to the bottom of this thread and type in the "Quick Reply" box, as shown below!

(http://i.imgur.com/pvuRqvX.png)

Alternatively, feel free to browse 123 pages of previous questions and answers! Navigate the thread with these page number buttons, found at the top and bottom of each page.

(http://i.imgur.com/NdzYc0v.png)

All the best! :)
Title: Re: VCE English Question Thread
Post by: beverlycheeeee on May 11, 2017, 04:39:45 pm
Hi! I am in a language analysis crisis now  :'( like my teacher keeps saying my body paragraph is too long- its like 286 words- she did'nt even read my work and she told me to rewrite it because its too long  :-\ could any of you guys help?

Bantick claims that the homeless are mocking Melbourne by ruining the aesthetic of Melbourne. Starting off with a tired and disgusted tone, Bantick states that he is “over with the homelessness in Melbourne”, and questions about why he needs to “show special understanding to the homelessness who trash the city pavements” implying that the homeless are disrespecting Melbourne and that they do not deserve any sympathy for their behaviour. The author employs emotive language such as “over” and “trash” to antagonise the homeless and positions the audience to feel disgusted about the actions of the homeless and address the issue of the homeless disrespecting Melbourne. Further to this, Bantick also highlights the fact that homeless are littering around Melbourne, causing Melbourne to appear as unsanitary by using descriptive language such as “fast food containers, alcohol bottles, urination” to emphasise how the homeless are trashing Melbourne. This prompts the readers to be appalled by the immoral actions of the homeless and support his argument of how the homeless are mocking Melbourne. In contrary, Rizvi acknowledges how the makeshift camps of the homeless is a “collective shame”, however, she argues that the public is just ashamed about the makeshift camps ruining Melbourne’s aesthetic because the famous landmarks are not “shiny and perfect” but covered with makeshift camps. Her acknowledgement of the makeshift camps improves the reader’s opinion on the author. In addition, her use of emotive language such as “collective shame” and “shiny and perfect” encourages the readers to reflect upon their opinion about homelessness. Thus, Bantick’s article highlights that people do not need to sympathise with the homelessness who are trashing Melbourne’s aesthetic, and Rizvi argues that the wellbeing of the homeless are more important that the aesthetic of Melbourne.

+++ I don't know how to start second paragraphs ;-; help?
Title: Re: VCE English Question Thread
Post by: littlemouse397 on May 12, 2017, 03:18:17 pm
Can anyone please tell me if my introduction is good? What range do you think it is? (low), (mid) or (high) and how can i improve?

The recent upbeat proposal in response to the onset growth of Melbourne’s population, has prompted measured debates concerning whether or not Melbourne’s Metro Rail Project would be affective. One measured response to this issue has been the informative post, which proposes confidently, to the rationality of his readers, that in order to maintain Melbourne liveable status and provide for its increasing population, improvements in relation to transport services via the construction of an underground railway system should be implemented. Accompanying the reproving article is a multiple of appealing visuals which concur the reality of improved metro that result in ease congestion and maintain prosperity. Contrary to his view, an experienced practitioner, Rusty Eggerton in a comment published shortly after, voices his disagreement critically, by inferring that proposed tunnels in Melbourne’s CBD will not address any of problems or pose any benefits. That they will only make matters worse.

(NOTE: there's no title, author name, date of publication, form etc..)
PS- what is the best structure? This article has 3 visuals relating to the main, and a comment. Do i have to mention the visuals each or can i do a general statement for all 3. PS- if there's more than 3 articles, do i have to mention the authors contention etc.. for all 3. (contrary to this view 3x+??) THANKS
Title: Re: VCE English Question Thread
Post by: clarke54321 on May 12, 2017, 05:35:50 pm
Can anyone please tell me if my introduction is good? What range do you think it is? (low), (mid) or (high) and how can i improve?

The recent upbeat  I'm just being picky. But I probably wouldn't use the word upbeat.  proposal in response to the onset growth of Melbourne’s population, has prompted measured  I'd leave it at debates only and omit any adjective, like measured  debates concerning whether or not Melbourne’s Metro Rail Project would be effective. One measured response to this issue has been the informative post, which proposes confidently  Switch around: confidently proposes , to the rationality of his readers  It sounds a bit unclear. You can't exactly propose something to someone's rationality , that in order to maintain Melbourne liveable status and provide for its increasing population, improvements in relation to transport services via the construction of an underground railway system should be implemented  This sentence is too long . Accompanying the reproving  Try not to be subjective and place adjectives before a noun, unless they are from the piece itself.  article is a multiple of appealing visuals  Are multiple visuals... which concur the reality of improved metro that  ----> the wording is a bit off  result in ease congestion and maintain prosperity. Contrary to his view, an experienced practitioner, Rusty Eggerton in a comment published shortly after, voices his disagreement critically, by inferring  Not the right word. Suggesting/Intimating  that proposed tunnels in Melbourne’s CBD will not address any problems or pose any benefits.  That they will only make matters worse <----- Omit this .


(NOTE: there's no title, author name, date of publication, form etc..)
PS- what is the best structure? This article has 3 visuals relating to the main, and a comment. Do i have to mention the visuals each or can i do a general statement for all 3. PS- if there's more than 3 articles, do i have to mention the authors contention etc.. for all 3. (contrary to this view 3x+??) THANKS

In this situation, I would state the main contention of the article and that of the comment (Link these with a comparative phrase. For example, does the comment echo the sentiments of the main article or is it in opposition to it?). Personally, I don't make mention of the images in my introduction, unless they are separate from the main pieces (ie. Last year's exam had a visual in the form of a cartoon, which was separated from both articles and had a contention of its own). If you have multiple letters to the editor, which are complementing the main piece, then it is okay to just make a brief reference to them in your intro. You don't have to outline all of their separate contentions. 

I'm also in year 12, so am not the best to judge the level of your work. However, I would probably give you a mid for this intro. To improve, I think you should be careful of your expression and the wordiness of your sentences. Also, ensure that you have a clear link between the contentions of the main piece and that of the comment.

If you have any questions, please feel free to ask! All the best.  :)
Title: Re: VCE English Question Thread
Post by: littlemouse397 on May 12, 2017, 06:06:24 pm
Thankyou soo much for replying! will you be able to write an introduction for this article? so i can learn and compare the differences? DM if you havent read the article
Title: Re: VCE English Question Thread
Post by: clarke54321 on May 12, 2017, 06:08:21 pm
Thankyou soo much for replying! will you be able to write an introduction for this article? so i can learn and compare the differences? DM if you havent read the article

Do you have a copy of the article that I can look at?
Title: Re: VCE English Question Thread
Post by: zhen on May 12, 2017, 09:00:21 pm
Hi! I am in a language analysis crisis now  :'( like my teacher keeps saying my body paragraph is too long- its like 286 words- she did'nt even read my work and she told me to rewrite it because its too long  :-\ could any of you guys help?

Bantick claims that the homeless are mocking Melbourne by ruining the aesthetic of Melbourne. The expression is unusual because of the repitition of the word Melbourne.  Starting off with a tired and disgusted tone, Bantick states that he is “over with the homelessness in Melbourne”, and questions about why he needs to “show special understanding to the homelessness who trash the city pavements” Good integration of quotes, but you should try to condense the quotes and pick out the important bits.  implying that the homeless are disrespecting Melbourne and that they do not deserve any sympathy for their behaviour.  This is a good place to talk about the intended effect on readers.  The author employs emotive language such as “over” and “trash” Good attempt at unpacking quotes, but it's a bit repetitive using the same quote twice. So, try to cut that out. Also, be more picky about the quotes you choose to use. I think that "trash" is a good quote but the word "over" isn't really the best word to analyse. to antagonise the homeless and positions the audience to feel disgusted about the actions of the homeless and address the issue of the homeless disrespecting Melbourne. Further to this Weird expression, Bantick also highlights the fact that homeless are littering around Melbourne, causing Melbourne to appear as unsanitary by using descriptive language You can cut out all this, since it's just retelling stuff, without providing evidence.  such as “fast food containers, alcohol bottles, urination” to emphasise how the homeless are trashing Melbourne. This prompts the readers to be appalled by the immoral actions of the homeless and support his argument of how the homeless are mocking Repitition of the word mocking Melbourne. In contrary Not too sure about this, but it sounds weird , Rizvi acknowledges how the makeshift camps of the homeless is a “collective shame”, however, she argues that the public is just ashamed about the makeshift camps ruining Melbourne’s aesthetic because the famous landmarks are not “shiny and perfect” but covered with makeshift camps. Her acknowledgement of the makeshift camps improves the reader’s opinion on the author. In addition, her use of emotive language such as “collective shame” and “shiny and perfect” encourages the readers to reflect upon their opinion about homelessness. This can be shortened down so much. Be more concise with your analysis and more selective on what you analyse. Also try to avoid generic stuff like makes the reader reflect on...  Thus, Bantick’s article highlights that people do not need to sympathise with the homelessness who are trashing Melbourne’s aesthetic, and Rizvi argues that the wellbeing of the homeless are more important that the aesthetic of Melbourne. I feel like this last sentence can be made more concise, since it's so long.

+++ I don't know how to start second paragraphs ;-; help?
I've pointed out areas where you can cut down above. Take my advice with a grain of salt, since I'm in year 12 this year. Also, I realised my comments seem a bit negative, since I just pointed out flaws and places where you need to cut down, but I'd say that you have a good grasp of how language positions the reader and the main arguments of each piece. So, good job and keep up the good work.  :)
Title: Re: VCE English Question Thread
Post by: ImmaculateJeff on May 17, 2017, 08:22:24 pm
If you don't agree with a mark your teacher gives you for a SAC, how can you take action?
Title: Re: VCE English Question Thread
Post by: deStudent on May 17, 2017, 08:53:12 pm
If you don't agree with a mark your teacher gives you for a SAC, how can you take action?
Lawsuit?

Or approach her.
Title: Re: VCE English Question Thread
Post by: zhen on May 17, 2017, 09:07:36 pm
If you don't agree with a mark your teacher gives you for a SAC, how can you take action?
I'd say consult with another teacher (preferably the coordinator of the subject) and ask them to see whether they think your SAC deserves a higher mark. If they agree that you deserved a higher mark then you could ask them to consult with your teacher and maybe change the mark.
Title: Re: VCE English Question Thread
Post by: bravado on May 19, 2017, 12:37:42 am
Hey Lauren!

This year, I am tutoring VCE English and I wanted to get your opinion on how the study design has changed for Section C (Language Analysis). Some teachers of my students are saying it is now argument-focused and more centred on how the contention is supported by the arguments. I've read the study design and, indeed, it does seem more focused on how argument is developed by logic and reason. Do you have any tips to tackle this new task or perhaps a sample essay/resources? How would you update your previous LA guides to suit this?

Thanks.
Title: Re: VCE English Question Thread
Post by: Ashjames on May 21, 2017, 03:15:11 pm
can someone tell me what they got on their individual SAC marks and exam scores for English?
Also was your school a strong or weak cohort when it comes to English?
Title: Re: VCE English Question Thread
Post by: clarke54321 on May 29, 2017, 05:37:30 pm
Hi all,

Since our essays at the end of the year are marked out of 10, is there a grade equivalent to the number? Eg. does a 9-10 equal an A+?
Is it even calculated in this way?

Thanks!
Title: Re: VCE English Question Thread
Post by: zhen on May 29, 2017, 05:42:34 pm
Hi all,

Since our essays at the end of the year are marked out of 10, is there a grade equivalent to the number? Eg. does a 9-10 equal an A+?
Is it even calculated in this way?

Thanks!
Here are the grade distributions for English. An A+ is 47/60, which is about an 8/10.
Title: Re: VCE English Question Thread
Post by: pmmenotes on May 30, 2017, 06:39:31 pm
Hey guys I have the Medea SAC on thursday and my teacher told us it would be a combination or something similar to the two out of three practice papers we've done. First one was; "What is difficult to understand about Medea is not her desire  for revenge but her means of achieving it" and the second was "Jason's decisions are based on reason and careful judgement while Medea's decisions are based on passion and reason' Anyone done any sacs which are similar to these two like a 'combo' or can guess what it would be? sorry know this is vague/confusing just trying to work out the question for the SAC ahah
Title: Re: VCE English Question Thread
Post by: clarke54321 on May 30, 2017, 06:50:43 pm
Hey guys I have the Medea SAC on thursday and my teacher told us it would be a combination or something similar to the two out of three practice papers we've done. First one was; "What is difficult to understand about Medea is not her desire  for revenge but her means of achieving it" and the second was "Jason's decisions are based on reason and careful judgement while Medea's decisions are based on passion and reason' Anyone done any sacs which are similar to these two like a 'combo' or can guess what it would be? sorry know this is vague/confusing just trying to work out the question for the SAC ahah

Hey!

So if we break it down:

Question (1):

-Medea is complex because we cannot understand her thinking (consequently her means. For example killing her children).
-We can't understand her thinking because she often blurs the concepts of passion and reason.
-Her desire for justice is also a key to this question. Her desire also blurs these concepts.

Question (2):

-Medea's decisions based on passion and reason

You can therefore see how the questions relate. At their core, they are ultimately trying to decipher what kind of character Medea is. It is dealing with the extremes of human emotions. Does she think and act with only passion or reason? Does she combine both?

So questions to ask are:

Are Medea's acts motivated by pure reason or pure passion?
Is she ever able to embrace the quality of sophrosyne and possess moderation?
Does passion coerce her to commit murderous acts?
Is Medea complex because her self-control fluctuates throughout the play?

I hope this helps you. Good luck on Thursday!  :)

EDIT: Sorry I didn't make reference to Jason. Given that the first question was exclusive to Medea, I thought I wouldn't bother mentioning him. It would broaden the possibility of potential questions.

Title: Re: VCE English Question Thread
Post by: pmmenotes on May 30, 2017, 07:33:02 pm
Hey!

So if we break it down:

Question (1):

-Medea is complex because we cannot understand her thinking (consequently her means. For example killing her children).
-We can't understand her thinking because she often blurs the concepts of passion and reason.
-Her desire for justice is also a key to this question. Her desire also blurs these concepts.

Question (2):

-Medea's decisions based on passion and reason

You can therefore see how the questions relate. At their core, they are ultimately trying to decipher what kind of character Medea is. It is dealing with the extremes of human emotions. Does she think and act with only passion or reason? Does she combine both?

So questions to ask are:

Are Medea's acts motivated by pure reason or pure passion?
Is she ever able to embrace the quality of sophrosyne and possess moderation?
Does passion coerce her to commit murderous acts?
Is Medea complex because her self-control fluctuates throughout the play?

I hope this helps you. Good luck on Thursday!  :)

EDIT: Sorry I didn't make reference to Jason. Given that the first question was exclusive to Medea, I thought I wouldn't bother mentioning him. It would broaden the possibility of potential questions.



Thanks so much! really helpful btw do you have any tips to prepare for my SAC? I've already done practice pieces and have had them marked but I don't know what to memorise? I'm just compiling a list currently with lines throughout my pieces my teacher has liked and I can try reuse. As well as looking at the quote sheet we got given and trying to memorise those. Other than that idk what else to do? Any advice would be appreciated
Title: Re: VCE English Question Thread
Post by: clarke54321 on May 30, 2017, 07:46:55 pm
Thanks so much! really helpful btw do you have any tips to prepare for my SAC? I've already done practice pieces and have had them marked but I don't know what to memorise? I'm just compiling a list currently with lines throughout my pieces my teacher has liked and I can try reuse. As well as looking at the quote sheet we got given and trying to memorise those. Other than that idk what else to do? Any advice would be appreciated

Well at my school, we were given 10 possible prompts. Therefore, I planned each of those and found that I could use similar arguments for quite a few of them.

But I think that going through past essays is the best idea. This is especially good if you have written essays, which have covered multiple themes. This will make it easier to develop arguments and incorporate relevant quotes. Perhaps re-work some paragraphs that weren't as great as you had hoped. It may also be worth compiling a list of 'Medea' prompts and determining a contention and sub-arguments for each. This may help you work faster in the SAC, as you will already have opinions on certain topics and would have thought through your own understanding.
Title: Re: VCE English Question Thread
Post by: scanz_ on May 30, 2017, 09:05:08 pm
Hey guys.
I have a desperate question. What kind of SAC scores should I be aiming for to get 35 in mainstream english?
I'm in year 11 right now and im averaging at a B for each of my SACs (I know right.) I want to start preparing now, but I need to know how much time I should dedicate towards writing practise essays etc.

Thanks!
Title: Re: VCE English Question Thread
Post by: Ashjames on June 01, 2017, 09:46:32 am
Hey guys,

I have an oral presentation topic in a week. My topic is "should ebooks be banned and should the use of traditional, hard copy books be implemented? I have to present 3 arguments,  what would be really good argument that will engage my audience.
Title: Re: VCE English Question Thread
Post by: clarke54321 on June 01, 2017, 01:01:37 pm
Hey guys,

I have an oral presentation topic in a week. My topic is "should ebooks be banned and should the use of traditional, hard copy books be implemented? I have to present 3 arguments,  what would be really good argument that will engage my audience.

This is a funny suggestion, but take a look at the 2012 VCAA paper section C. The article discusses the advantages and disadvantages of eBooks!
Title: Re: VCE English Question Thread
Post by: xghostariax on June 07, 2017, 07:33:37 pm
Hey, so I have mid-term exams soon and English is my first exam. Analytical essay for Medea and a language analysis is the task I have to complete for the exam. One of the topics that was explored in my class was about sympathy, such as this prompt, 'It is Jason, not Medea who gains the audience's sympathy. Do you agree?'
Could anyone give a brief explanation on how to break down the prompt and then in paragraphs. My teacher said that I should add a rebuttal for my last third body paragraph but I'm kinda on the fence about it. Any advice??
Thanks :)
Title: Re: VCE English Question Thread
Post by: clarke54321 on June 07, 2017, 08:49:54 pm
Hey, so I have mid-term exams soon and English is my first exam. Analytical essay for Medea and a language analysis is the task I have to complete for the exam. One of the topics that was explored in my class was about sympathy, such as this prompt, 'It is Jason, not Medea who gains the audience's sympathy. Do you agree?'
Could anyone give a brief explanation on how to break down the prompt and then in paragraphs. My teacher said that I should add a rebuttal for my last third body paragraph but I'm kinda on the fence about it. Any advice??
Thanks :)
Hi!

So~

'It is Jason, not Medea who gains the audience's sympathy. Do you agree?'

First, it is important that you explore the ways both Jason and Medea gain audience sympathy and the ways they don't. In this way, you'll be better able to reach your own conclusion (ie. work out whether Jason does in fact gain the audiences' sympathy, whether it is Medea who does, or whether it is both).

So in very basic terms, the prompt is saying that Jason obtains all of the audiences' sympathy, while Medea gains none. To develop a sophisticated response to this prompt, you must include a qualifier. For example, While Jason gains audience sympathy, so too does Medea. This is obviously quite simplified, but it challenges the prompt in some way.

Once you have established your evidence and sorted out all your ideas, it will be easier for you to clarify your own ideas and understanding. Sorry about my rushed response (SAC week  :'( ) If you have more questions please feel free to ask!

Title: Re: VCE English Question Thread
Post by: xghostariax on June 08, 2017, 08:38:12 am
Hi!

So~

'It is Jason, not Medea who gains the audience's sympathy. Do you agree?'

First, it is important that you explore the ways both Jason and Medea gain audience sympathy and the ways they don't. In this way, you'll be better able to reach your own conclusion (ie. work out whether Jason does in fact gain the audiences' sympathy, whether it is Medea who does, or whether it is both).

So in very basic terms, the prompt is saying that Jason obtains all of the audiences' sympathy, while Medea gains none. To develop a sophisticated response to this prompt, you must include a qualifier. For example, While Jason gains audience sympathy, so too does Medea. This is obviously quite simplified, but it challenges the prompt in some way.

Once you have established your evidence and sorted out all your ideas, it will be easier for you to clarify your own ideas and understanding. Sorry about my rushed response (SAC week  :'( ) If you have more questions please feel free to ask!
Ohhh okay, I totally understand where your coming from. Could I alternate between sides with my body paragraphs? How many body paragraphs would suffice in this kind of prompt?
Thank you for your advice by the way, really appreciate it! :)
Title: Re: VCE English Question Thread
Post by: clarke54321 on June 08, 2017, 08:58:40 am
Ohhh okay, I totally understand where your coming from. Could I alternate between sides with my body paragraphs? How many body paragraphs would suffice in this kind of prompt?
Thank you for your advice by the way, really appreciate it! :)

No problems!

It's difficult to say exactly how many body paragraphs you need. It really depends on how many ideas you have and what will make your essay flow better.

This prompt does necessitate a fair bit of oscillation, though. So, you may find that you have 2 or 3 body paragraphs that are in opposition to the prompt, given that it is quite narrow. With sympathy in 'Medea,' it typically tends to develop chronologically for characters.

For example:

-Medea scorned/shamed, Chorus and Nurse support
-Speaks with Creon banished, left double exile (worst punishment) again Chorus sympathy
-Agon with Jason (he displays little emotion regarding their former relationship, blunt with her regarding her position)
-Speaks with Aegeus (male voice) offers male perspective, again intending to win over sympathy for audience. (It is really important that you consider structural features of 'Medea.' This was suggested in last years examiner's report. In doing so, you are gaining more of an insight into Euripides choices and the reasons for them. For example, why is the Chorus all women, why does he include Aegeus).
-Medea decides to kill children/manipulates/cunning revealed (perhaps more inclined to sympathise with Jason; especially since it was considered quite normal for a male to have multiple families at this time. Therefore, he may see no flaws in what he is doing).
-Medea suffers a psychological breakdown (may be worthy speaking about her psychomania?)
-Killing Glance, Creon and the kids (descriptions of these deaths and ruining Jason's lineage, how are male audiences supposed to react to this? Lineage very important aspect. Basically a man's life.

Hopefully these ideas help you arrange your body paragraphs in some ways. While it is important to always centre body paragraphs around ideas (ideas in relation to characters, given that this is a character prompt), I don't think order will matter too much with this prompt. I actually think it would flow better if you examined it chronologically, given the way sympathy shifts throughout.

All the best  :)

Title: Re: VCE English Question Thread
Post by: xghostariax on June 08, 2017, 01:40:49 pm
No problems!

It's difficult to say exactly how many body paragraphs you need. It really depends on how many ideas you have and what will make your essay flow better.

This prompt does necessitate a fair bit of oscillation, though. So, you may find that you have 2 or 3 body paragraphs that are in opposition to the prompt, given that it is quite narrow. With sympathy in 'Medea,' it typically tends to develop chronologically for characters.

For example:

-Medea scorned/shamed, Chorus and Nurse support
-Speaks with Creon banished, left double exile (worst punishment) again Chorus sympathy
-Agon with Jason (he displays little emotion regarding their former relationship, blunt with her regarding her position)
-Speaks with Aegeus (male voice) offers male perspective, again intending to win over sympathy for audience. (It is really important that you consider structural features of 'Medea.' This was suggested in last years examiner's report. In doing so, you are gaining more of an insight into Euripides choices and the reasons for them. For example, why is the Chorus all women, why does he include Aegeus).
-Medea decides to kill children/manipulates/cunning revealed (perhaps more inclined to sympathise with Jason; especially since it was considered quite normal for a male to have multiple families at this time. Therefore, he may see no flaws in what he is doing).
-Medea suffers a psychological breakdown (may be worthy speaking about her psychomania?)
-Killing Glance, Creon and the kids (descriptions of these deaths and ruining Jason's lineage, how are male audiences supposed to react to this? Lineage very important aspect. Basically a man's life.

Hopefully these ideas help you arrange your body paragraphs in some ways. While it is important to always centre body paragraphs around ideas (ideas in relation to characters, given that this is a character prompt), I don't think order will matter too much with this prompt. I actually think it would flow better if you examined it chronologically, given the way sympathy shifts throughout.

All the best  :)
Great!! Thank you so much!! You've just given more ideas to discuss in my essay!
I've also decided to do another essay but with a different prompt. The prompt that i will be doing is, 'Are Medea's actions justifiable? Discuss.'
What are some pointers to consider in this topic, are her actions unjustified? if so, why?
Thanks again for help!! :) :)
Title: Re: VCE English Question Thread
Post by: clarke54321 on June 08, 2017, 03:30:00 pm
Great!! Thank you so much!! You've just given more ideas to discuss in my essay!
I've also decided to do another essay but with a different prompt. The prompt that i will be doing is, 'Are Medea's actions justifiable? Discuss.'
What are some pointers to consider in this topic, are her actions unjustified? if so, why?
Thanks again for help!! :) :)

Well, the questions you have just asked are exactly what the prompt is asking. I think it is important that you gather up your own ideas and evidence first and come to your own conclusion/interpretation. While my ideas may be helpful to you, you are the one who must ultimately come up with your own interpretation. Only then will your essay be really strong and better still, you will be able to respond to a wider range of prompts.

If you can develop your own contention and body paragraph ideas, I will definitely be able to help you out! :)
Title: Re: VCE English Question Thread
Post by: xghostariax on June 08, 2017, 03:55:12 pm
Well, the questions you have just asked are exactly what the prompt is asking. I think it is important that you gather up your own ideas and evidence first and come to your own conclusion/interpretation. While my ideas may be helpful to you, you are the one who must ultimately come up with your own interpretation. Only then will your essay be really strong and better still, you will be able to respond to a wider range of prompts.

If you can develop your own contention and body paragraph ideas, I will definitely be able to help you out! :)
I've actually just finished up writing the essay on that prompt, care to take a look? :)
Title: Re: VCE English Question Thread
Post by: clarke54321 on June 08, 2017, 04:02:24 pm
I've actually just finished up writing the essay on that prompt, care to take a look? :)

Fantastic! Yeah, I'll be happy to look over it for you :)
Title: Re: VCE English Question Thread
Post by: zxcvbnm18 on June 11, 2017, 12:09:28 pm
Internal monologue structure and example??
Title: Re: VCE English Question Thread
Post by: beverlycheeeee on June 11, 2017, 01:20:45 pm
I've pointed out areas where you can cut down above. Take my advice with a grain of salt, since I'm in year 12 this year. Also, I realised my comments seem a bit negative, since I just pointed out flaws and places where you need to cut down, but I'd say that you have a good grasp of how language positions the reader and the main arguments of each piece. So, good job and keep up the good work.  :)

gah this is so long ago but thank you so much :)))
Title: Re: VCE English Question Thread
Post by: TooLazy on June 11, 2017, 03:13:57 pm
Does anyone know where I can find articles and corresponding high language analysis essays?
Title: Re: VCE English Question Thread
Post by: clarke54321 on June 11, 2017, 03:45:41 pm
Does anyone know where I can find articles and corresponding high language analysis essays?


Hey!

I'd suggest that you read through some past examiner's report on the VCAA website: http://www.vcaa.vic.edu.au/Pages/vce/studies/english/englishexams.aspx
Here, examiners make note of what differentiates high/medium/low range responses by providing examples (note that not every year provides full essays, but shows only sections).

Another good place to find high-scoring essays is right here on AN. Take a look at this page where sample essays have been compiled: https://atarnotes.com/forum/index.php?topic=8375.0

Hope this helps!
Title: Re: VCE English Question Thread
Post by: Ashjames on June 12, 2017, 07:20:10 pm
Hello everyone!!!

I'm currently ranked 2 in English with only 3 marks between me and rank 1. For GA1 I achieved an overall score of 80/100. I was wondering if I get A+'s on the other outcomes what SS would I be looking at?
Title: Re: VCE English Question Thread
Post by: zhen on June 12, 2017, 07:44:44 pm
Hello everyone!!!

I'm currently ranked 2 in English with only 3 marks between me and rank 1. For GA1 I achieved an overall score of 80/100. I was wondering if I get A+'s on the other outcomes what SS would I be looking at?
It depends on your cohort a bit, but if you can get A+ for the exam and your semester 2 SACs, then you'll probably be set for 40+, provided your cohort is half decent so your first semester SACs are A+. But to really answer this, I need to know the strength of your cohort, since cohort strength varies massively (some schools might get like 50 scores above 40 and others might not get any) and this will give a better indication of where you're at and what that ranking translates to in terms of a VCAA letter grade.
Title: Re: VCE English Question Thread
Post by: Mariamnourine on June 12, 2017, 09:48:46 pm
It depends on your cohort a bit, but if you can get A+ for the exam and your semester 2 SACs, then you'll probably be set for 40+, provided your cohort is half decent so your first semester SACs are A+. But to really answer this, I need to know the strength of your cohort, since cohort strength varies massively (some schools might get like 50 scores above 40 and others might not get any) and this will give a better indication of where you're at and what that ranking translates to in terms of a VCAA letter grade.


To be honest with you, I'd say my cohort is weak, with only 8 people getting decent marks for English
Title: Re: VCE English Question Thread
Post by: zhen on June 12, 2017, 10:32:22 pm

To be honest with you, I'd say my cohort is weak, with only 8 people getting decent marks for English
It might depend on rank 1. If rank 1 aces the exam and gets an A+ on the exam and you also maintain your rank 2 and get an A+ on the exam, then 40+ is what you'd probably be getting unless they're really low A+. Basically it depends on what you and rank 1 get on your exams, if you maintain that ranking. Since you'd be getting the second highest exam mark for your SAC mark and rank 1 would be getting the highest exam mark for their SAC mark. So, you want at least 2 people in your cohort including yourself to do well in the exams.
Title: Re: VCE English Question Thread
Post by: ardria on June 12, 2017, 10:35:52 pm
It might depend on rank 1. If rank 1 aces the exam and gets an A+ on the exam and you also maintain your rank 2 and get an A+ on the exam, then 40+ is what you'd probably be getting unless they're really low A+. Basically it depends on what you and rank 1 get on your exams, if you maintain that ranking. Since you'd be getting the second highest exam mark for your SAC mark and rank 1 would be getting the highest exam mark for their SAC mark. So, you want at least 2 people in your cohort including yourself to do well in the exams.

Do you know how much your school's ranking matters? Or is it just your cohort (year level)?

E.g. if the school was ranked much, much more poorly than usual in 2016, but your cohort (class of 2017) is a strong (high-achieving, competitive and hard-working) one...?
Title: Re: VCE English Question Thread
Post by: zhen on June 12, 2017, 10:47:05 pm
Do you know how much your school's ranking matters? Or is it just your cohort (year level)?

E.g. if the school was ranked much, much more poorly than usual in 2016, but your cohort (class of 2017) is a strong (high-achieving, competitive and hard-working) one...?
It's cohort that matters and how your classmates do on the exam.
Title: Re: VCE English Question Thread
Post by: maria.ali on June 15, 2017, 10:56:44 pm
hi everyone,
so my prompt for text response for All About Eve is 'Eve is an outsider in the film. She never truly belongs. Discuss'
I have trouble coming up with ideas and struggle with suitable topic sentences. Can someone help me come up with some?
Thanks
Title: Re: VCE English Question Thread
Post by: Syndicate on June 17, 2017, 09:28:29 pm
hi everyone,
so my prompt for text response for All About Eve is 'Eve is an outsider in the film. She never truly belongs. Discuss'
I have trouble coming up with ideas and struggle with suitable topic sentences. Can someone help me come up with some?
Thanks


Hi,

A few suggestions you may wish to talk about in your essay:
- Women have never truly been part of the theatrical world- control is only given to men.
- 1950's is a claustrophobic world. Only the elite are allowed to enter (mention the use of doors). Women either need to employ manipulation, or be a "part of the theatre by marriage".
- Eve brings chaos into the lives of theatrical characters. She ruined careers, relationship, trusts etc...
- Although Eve did get a role in the theatre, she never truly understood the importance of it (ie she doesn't care about the SSA in the end). She is often found secluded from other characters.


Title: Re: VCE English Question Thread
Post by: Gogo14 on July 01, 2017, 10:00:27 pm
our oral presentation is at the start of next term and I want to choose an issue to do....but I can't decide.
I want to do something political/ international relations that can go for 6 minutes. Would like an issue that's juicy and has principle arguments.
currently thinking about whether we should repeal obama care,  but want to see what are your thoughts and other issues out there.
Title: Re: VCE English Question Thread
Post by: noregret on July 02, 2017, 10:44:39 pm
What is the difference between language and structural devices in a film, a book and a short story?
Title: Re: VCE English Question Thread
Post by: Ashjames on July 03, 2017, 05:37:20 pm
Rank 1/2 for English, )unit 3 and 4 respectively)- Now What?
Title: Re: VCE English Question Thread
Post by: clarke54321 on July 03, 2017, 06:02:03 pm
Rank 1/2 for English, )unit 3 and 4 respectively)- Now What?

Hey!

Congrats on holding such a wonderful ranking for English. What do you mean by, 'now what?' :)

If you're suggesting that you've already reached your peak, there is definitely always more you can do! If you haven't done so already, ensure that you have a thorough understanding of your comparative texts. Make extensive mind maps on how they are similar/different with supporting quotes, etc. At this stage of the year, it is also very important to re-visit your text response material. Perhaps your perception of the text has changed. If so, you may be able to refine initial ideas or add further depth to ideas. In this way, you may be able to bring your essays to a whole new standard.

Title: Re: VCE English Question Thread
Post by: Ashjames on July 03, 2017, 08:04:52 pm
Hey!

Congrats on holding such a wonderful ranking for English. What do you mean by, 'now what?' :)

If you're suggesting that you've already reached your peak, there is definitely always more you can do! If you haven't done so already, ensure that you have a thorough understanding of your comparative texts. Make extensive mind maps on how they are similar/different with supporting quotes, etc. At this stage of the year, it is also very important to re-visit your text response material. Perhaps your perception of the text has changed. If so, you may be able to refine initial ideas or add further depth to ideas. In this way, you may be able to bring your essays to a whole new standard.

Yeah, that was a bit to vague- my apologies. What I mean by that is what do I do to ensure the best SS I can achieve? (and thanks!! I didn't think I was ranked that high until someone told me)
Title: Re: VCE English Question Thread
Post by: zhen on July 04, 2017, 12:06:04 pm
Rank 1/2 for English, )unit 3 and 4 respectively)- Now What?
I'd say continue what you're already doing, but maybe ask your teacher in areas you can improve upon. Unless you're getting full marks for everything and maybe even if you get full marks there are always small things you can tweak in your essays to make them better. So, whenever you lose a mark on something find out why. In my opinion getting feedback and improving your writing based on the feedback is a great way to improve.
Title: Re: VCE English Question Thread
Post by: Lavar Big BBB Balls on July 04, 2017, 02:14:32 pm
Just curious has anyone managed to write any types of essays within an hour yet? Anywhere close?
Title: Re: VCE English Question Thread
Post by: zhen on July 04, 2017, 04:02:55 pm
Just curious has anyone managed to write any types of essays within an hour yet? Anywhere close?
I haven't gotten anywhere near being able to write an essay in an hour, because we get one and a half hours to write essays for our SACs. Anyway, I feel like we should be able to do it with a bit of practice.
Title: Re: VCE English Question Thread
Post by: clarke54321 on July 04, 2017, 04:14:35 pm
Just curious has anyone managed to write any types of essays within an hour yet? Anywhere close?

I also haven't started timing myself yet. At the moment, I'm more focused on bettering the quality of my writing material. After this, it should be easy to craft a high-scoring essay under timed conditions. I'll probably start restricting myself more and more towards the term 3 holidays :)
Title: Re: VCE English Question Thread
Post by: TheCommando on July 04, 2017, 07:54:44 pm
How Do i plan my oral presention
Ive researched about my issue already
Title: Re: VCE English Question Thread
Post by: zhen on July 04, 2017, 08:03:21 pm
How Do i plan my oral presention
Ive researched about my issue already
For my plans I just compile all the evidence I plan to use and dot point my arguments.
Title: Re: VCE English Question Thread
Post by: Coffee on July 04, 2017, 08:41:23 pm
Just curious has anyone managed to write any types of essays within an hour yet? Anywhere close?
I also haven't started timing myself yet. At the moment, I'm more focused on bettering the quality of my writing material. After this, it should be easy to craft a high-scoring essay under timed conditions. I'll probably start restricting myself more and more towards the term 3 holidays :)
+1

Focus on improving the quality of your essays. Once you know how to write a good essay, you can focus on exam technique and minimising the time it takes you to write an essay.

How Do i plan my oral presention
Ive researched about my issue already
Everyone plans differently, but what I found really useful was planning by argument. So if your overall argument is 'We should have a three day school week' and your sub-argument is 'It will give students more time to complete work outside of school', then you'll want to note down any evidence or additional points that you plan to use in your oral to support your argument. Does that kind of make sense? :)

If you're struggling, it can be helpful to refer to the age-old PEEL structure: ::)
Point - What's your argument?
Evidence - What facts and statistics are you going to use to support your argument?
Explain - How does your evidence support your argument? Why should we accept your argument?
Link - Link back to your overall argument.
Title: Re: VCE English Question Thread
Post by: Lavar Big BBB Balls on July 05, 2017, 11:09:57 pm
Hi,

So I'm wondering for text response, how do I improve/find unique examples/improve my understanding of the text?

I'm doing 'Medea' so it's a short text and I've already made a pretty comprehensive quote bank but I'm not sure what else to do? It feels like I'm using the same examples over and over again, I don't think they're bad but there could be some better that I could be doing?

Thanks
Title: Re: VCE English Question Thread
Post by: clarke54321 on July 06, 2017, 07:37:32 am
Hi,

So I'm wondering for text response, how do I improve/find unique examples/improve my understanding of the text?

I'm doing 'Medea' so it's a short text and I've already made a pretty comprehensive quote bank but I'm not sure what else to do? It feels like I'm using the same examples over and over again, I don't think they're bad but there could be some better that I could be doing?

Thanks

Hey!

I definitely understand where you're coming from. I'm also studying Medea for the text response and feel as though my ideas have run out of zing.
What has helped me, however, is reading the play again. It's been about 4 months since I studied it in class, so refreshing your knowledge can help. For example, perhaps you never picked up on the significance of a quote or are now able to clarify and develop ideas to a greater extent. Have you also tried annotating the text? This may sound pointless if you've already put together important quotes, but it can sometimes reveal unusual things. Unusual in a good way! For example, a single word alone can reveal volumes about a certain character.  It just adds some further originality.

But apart from the above suggestions, it may be beneficial to read others' essays. This may provide you with a new perspective on ideas or interpretations. You can find plenty of Medea essays on AN. I'd also suggest, if you can get access, to read some scholar reports. These are fantastic ways to broaden your understanding of a text and vocabulary!

One more suggestion! If you haven't done so already, try and unpack about 10 different Medea prompts covering different themes. In this way, your forced to consider different arguments and ideas. It will also test how effective your quotes are in relating to a wide range of themes. If you find they're not that effective, look through the play again and perhaps you'll find some more specific ones.

Hopefully this helps :)
Title: Re: VCE English Question Thread
Post by: Lavar Big BBB Balls on July 06, 2017, 02:58:42 pm
^^^ thanks for the help  :) :)
Title: Re: VCE English Question Thread
Post by: Ashjames on July 06, 2017, 07:41:09 pm
Hey everyone!!

I was just wondering how many paragraphs are necessary in a persuasive analysis (vcaa standard)
Title: Re: VCE English Question Thread
Post by: amigos on July 06, 2017, 08:18:24 pm
Hey everyone!!

I was just wondering how many paragraphs are necessary in a persuasive analysis (vcaa standard)

There's no set number, just write enough to show your understanding of the material and task. This might take 2, the typical yet totally arbitrary 3, or 5.

Thattttt being said, advice from VCAA examiners at my school is to have 1000+ words for each essay, so enough paragraphs to write that much? 
Title: Re: VCE English Question Thread
Post by: patriciarose on July 06, 2017, 08:22:18 pm
Hey everyone!!

I was just wondering how many paragraphs are necessary in a persuasive analysis (vcaa standard)

if you mean the argument analysis (if you don't, ignore me, i'm super exhausted right now oops), i think people are getting different advice about it tbh. i'd be aiming for at least introduction, three paragraphs and conclusion (or conclude in your third paragraph) but i know a lot of people who are somehow just doing two large body paragraphs. but three or four (body) seems safest to me, otherwise you either miss out on a lot of content or compact it down a little too much and that'll simplify the standard of your analysis. if you're doing any more than four, you're either a super fast writer (and in that case damn am i jealous) or you're skimming over too much and your paragraphs will be too short. so better to aim for three or four unless there's a desperate need for more. (: but vcaa never actually sets a limit so realistically, you're fine to do whatever you're comfortable with, providing you get all the necessary parts in haha.
Title: Re: VCE English Question Thread
Post by: Sine on July 07, 2017, 04:31:52 am
Hey everyone!!

I was just wondering how many paragraphs are necessary in a persuasive analysis (vcaa standard)
everyone is different and you won't be inherently marked down for choosing a particular number of paragraphs.

I think most people do an introduction and 3 or 4 body paragraphs (really depends how you want to tackle the persuasive piece, chronologically, by argument, etc) and no conclusion.
Title: Re: VCE English Question Thread
Post by: TheCommando on July 08, 2017, 11:48:51 pm
Is context past study design right?
Title: Re: VCE English Question Thread
Post by: amigos on July 08, 2017, 11:57:01 pm
Is context past study design right?

Yup.
Title: Re: VCE English Question Thread
Post by: Ashjames on July 11, 2017, 04:22:56 pm
Hey guys,

I was just wondering how do you incorporate the history of the novel into your text response??
Title: Re: VCE English Question Thread
Post by: patriciarose on July 11, 2017, 04:57:53 pm
Hey guys,

I was just wondering how do you incorporate the history of the novel into your text response??

literally anywhere tbh. i want to say it generally fits really well into the intro – ie. A Doll's House is set within the strict constraints of Victorian society – but briefly, because otherwise you're writing a historical report instead of a text response. if there's nowhere it really fits but you know your marker will be looking for it (which is honestly the worst), the introduction's the best place for it because it'll be least out of place there (: i wouldn't recommend concluding with anything like that because it's likely to sound like an afterthought (:
Title: Re: VCE English Question Thread
Post by: Quantum44 on July 11, 2017, 05:24:56 pm
Hey guys,

I was just wondering how do you incorporate the history of the novel into your text response??

As patriciarose said, the introduction is usually the best place for sociopolitical/historical context of the novel as it's always nice to drop some knowledge before getting into the groove of your essay.

Being able to discuss the context further in your essay without sounding awkward really depends on the prompt. Often the prompt invites a cultural interpretation of the novel and you can use a paragraph to support your contention by linking themes, characters or structural features of the text to the sociopolitical landscape of the time period. But sometimes that is simply not possible and you just have to accept it.
Title: Re: VCE English Question Thread
Post by: Sine on July 12, 2017, 12:58:28 am
Hey guys,

I was just wondering how do you incorporate the history of the novel into your text response??
definitely have something explicit about the historical significance in the introduction and I feel if you want to discuss it later on it'll have to be more implicit or related to your prompt
Title: Re: VCE English Question Thread
Post by: agnesbenjamin on July 16, 2017, 01:13:40 pm
Figuring how to work this website but that's beside the point. I was just wondering if you could tell me if we need a conclusion in the language analysis. In one of the lectures I attended, the lecturer said you don't even need one for that section.
Title: Re: VCE English Question Thread
Post by: clarke54321 on July 16, 2017, 01:26:42 pm
Figuring how to work this website but that's beside the point. I was just wondering if you could tell me if we need a conclusion in the language analysis. In one of the lectures I attended, the lecturer said you don't even need one for that section.

Hey!

First of all, welcome to AN :)

For the argument analysis, the conclusion is really only a structural feature. It's a nice way to wrap things up. But in saying that, I know of people who have scored a 10 for this section without even including it. Nearly all of your marks for this section will come from your body paragraphs and the analysis itself.

Therefore to answer your question, I'd say yes to be safe. However, if you run out of time and have to leave it out, don't stress!
Title: Re: VCE English Question Thread
Post by: patriciarose on July 16, 2017, 01:27:01 pm
Figuring how to work this website but that's beside the point. I was just wondering if you could tell me if we need a conclusion in the language analysis. In one of the lectures I attended, the lecturer said you don't even need one for that section.

so far as i know, there are no marks designated for a conclusion! but since just cutting your analysis off abruptly isn't the best way to finish, it makes sense either to have a conclusion of a sentence or two, or to draw everything to a close in your last paragraph. but definitely don't waste ten minutes writing three or four sentences for a separate conclusion! there's no point in that because there aren't marks given out for it, so all you need is for your essay to be clearly 'finished' so that overall it looks great when the marker is reading it (: but i'm only a year twelve student this year so take my advice with a grain spoonful of salt haha.
Title: Re: VCE English Question Thread
Post by: littledreamer on July 16, 2017, 05:37:45 pm
just wanted to know when is a good time to start timing 1 hour essays? I've been doing a few essays these holidays and can never make the one hour cut off :( I'm getting worried but wanted to know what you guys think and how you guys are going with the 1 hour time limit?

Thanks :)
Title: Re: VCE English Question Thread
Post by: patriciarose on July 16, 2017, 06:24:14 pm
i don't have an answer to the question part of that, but if it makes you feel better, i'm in the same boat (and i think we're sharing it with half of victoria lmao) :( i can manage it when i have reading time but otherwise, no. i feel like it'll probably improve naturally for us though because we'll get better at writing the essay type and should gradually speed up. we do have a fair amount of time to get there though! (:
Title: Re: VCE English Question Thread
Post by: zhen on July 16, 2017, 06:52:39 pm
just wanted to know when is a good time to start timing 1 hour essays? I've been doing a few essays these holidays and can never make the one hour cut off :( I'm getting worried but wanted to know what you guys think and how you guys are going with the 1 hour time limit?

Thanks :)
Today, I did an argument analysis essay for tutoring in an hour with 15 minutes reading time and I came out thinking that it was the worst essay I wrote all year long. It was completely different from my SAC where I 90 minutes writing time (I think) and also got reading time. For my SAC I could afford to just stop and think of ways to appropriately express myself and analyse the piece in depth, but for my timed essay if I stopped I felt like I was falling behind. I feel like coming out of that, I realise that I should start doing more timed essays, since it felt like I was just rushing through it and sacrificing heaps of depth so that I could finish my essay. I also missed out on chunks of the text which I thought could have been analysed really well. Overall, I think I'm in the same boat as everyone, since right now I can't even make a half decent essay in timed conditions. So, I guess don't feel too bad about not being able to do an essay in an hour right now since most year 12 students should be in the same situation (I hope). Personally, I plan on doing more timed essays after my UMAT which is coming up really soon. Sorry for the long rant, I needed to vent out my frustration. :P
Title: Re: VCE English Question Thread
Post by: scout on July 16, 2017, 07:03:20 pm
Today, I did an argument analysis essay for tutoring in an hour with 15 minutes reading time and I came out thinking that it was the worst essay I wrote all year long. It was completely different from my SAC where I 90 minutes writing time (I think) and also got reading time. For my SAC I could afford to just stop and think of ways to appropriately express myself and analyse the piece in depth, but for my timed essay if I stopped I felt like I was falling behind. I feel like coming out of that, I realise that I should start doing more timed essays, since it felt like I was just rushing through it and sacrificing heaps of depth so that I could finish my essay. I also missed out on chunks of the text which I thought could have been analysed really well. Overall, I think I'm in the same boat as everyone, since right now I can't even make a half decent essay in timed conditions. So, I guess don't feel too bad about not being able to do an essay in an hour right now since most year 12 students should be in the same situation (I hope). Personally, I plan on doing more timed essays after my UMAT which is coming up really soon. Sorry for the long rant, I needed to vent out my frustration. :P

100% agree.

Best of luck for the UMAT, zhen. 
Title: Re: VCE English Question Thread
Post by: Quantum44 on July 16, 2017, 07:23:43 pm
just wanted to know when is a good time to start timing 1 hour essays? I've been doing a few essays these holidays and can never make the one hour cut off :( I'm getting worried but wanted to know what you guys think and how you guys are going with the 1 hour time limit?

Thanks :)

I haven't been brave enough to try any 1 hour essays yet. I struggle putting out 3 pages in 90 mins, I can't imagine doing that in an hour. I probably wouldn't worry too much about it now, I'm planning on starting 60 min essays after the comparative SAC.
Title: Re: VCE English Question Thread
Post by: TheCommando on July 16, 2017, 09:24:43 pm
Where can i find compartive resources on here
How do i write a A+ peice and what can i do to get an A+ etc

As well how do i discuss complexities and nuances of the topic.
Title: Re: VCE English Question Thread
Post by: lilyrosee on July 17, 2017, 04:10:48 pm
Hey guys,

I just did my oral presentation which was meant to be in the range of 5-6 minutes. My oral got 6 minutes and 22 seconds. Will I get marked down for going over the limit??
Title: Re: VCE English Question Thread
Post by: Sine on July 17, 2017, 04:18:45 pm
Hey guys,

I just did my oral presentation which was meant to be in the range of 5-6 minutes. My oral got 6 minutes and 22 seconds. Will I get marked down for going over the limit??
Personally i dont think that should be an  issue. I think teachers will start getting concerned if its too short e.g below 3 minutes or 8+ mins. However i don't know about your teacher.

But i really dont think 22s is a big deal

Definitly the content is much more important!! :)
Title: Re: VCE English Question Thread
Post by: patriciarose on July 17, 2017, 04:20:28 pm
good luck for the umat zhen and anyone else sitting it!

Hey guys,
I just did my oral presentation which was meant to be in the range of 5-6 minutes. My oral got 6 minutes and 22 seconds. Will I get marked down for going over the limit??

it's subjective tbh. my friend's school did mark them down for that, mine just stopped marking at the cutoff, so anything we said afterwards wouldn't be counted, but i know another school that was super lenient with their timeframe. you'd need to ask your teacher to know (: but marking you down seems pretty unfair, normally 30+ seconds would be when they start getting iffy about timing.
Title: Re: VCE English Question Thread
Post by: lilyrosee on July 17, 2017, 05:48:42 pm
Personally i dont think that should be an  issue. I think teachers will start getting concerned if its too short e.g below 3 minutes or 8+ mins. However i don't know about your teacher.

But i really dont think 22s is a big deal

Definitly the content is much more important!! :)

Thanks for the reassurance. I think my content was good and a few people walking out said they would be surprised if I lose any marks for it so I am hoping that the teachers do not deduct any marks just because of 22 seconds.

good luck for the umat zhen and anyone else sitting it!

it's subjective tbh. my friend's school did mark them down for that, mine just stopped marking at the cutoff, so anything we said afterwards wouldn't be counted, but i know another school that was super lenient with their timeframe. you'd need to ask your teacher to know (: but marking you down seems pretty unfair, normally 30+ seconds would be when they start getting iffy about timing.

At my school they have one of the students timing each person on their phone so they didn't know what time it was until afterwards. I hope they lenient because they wouldn't know when the student started timing and when they stopped e.g. they could have started the timer a few seconds before my speech or stopped it a few seconds afterwards.
Title: Re: VCE English Question Thread
Post by: HopefulLawStudent on July 22, 2017, 06:46:48 pm
Where can i find compartive resources on here
How do i write a A+ peice and what can i do to get an A+ etc

As well how do i discuss complexities and nuances of the topic.

Check the notes section for comparative resources (specifically, I'm gonna direct you to the July lecture slides which I know for sure have stuff on comparing texts having spent forever slaving over those slaves - they should serve as a good starting point).

With regards to writing an A+ piece, are we talking in general or for a certain section?

To discuss complexities and nuances of the topic, you wanna be considering the cores and implications of the topic.

Core = what the prompt is fundamentally asking
Implications = if the core of the prompt is true, then what? What is implied by the topic, essentially.

Here is an example of a prompt breakdown.

Prompt: In Stasiland, no character is immune from the effects of the Wall. Discuss.


Contention: Although the characters in Stasiland are irrefutably altered by the social climate of Germany and the sense of division that pervaded the nation, ultimately, these effects manifest themselves in vastly different ways, and it is this variation in how they respond to their circumstances which Funder propounds to be a true test of character.

Paragraph 1: The Wall's physical presence inhibits characters' freedom and right to self-determination, and even after the Wall is gone, the sense of a lingering divide is palpable.

Paragraph 2: However, the Wall also takes a psychological toll, particularly for those characters who

Paragraph 3: Moreover, Funder reveals how the Wall became a metonym for national division and the source of the characters' suffering, making it virtually impossible to escape.

Conclusion: Hence, by associating the Wall as an enforcer of isolation and detachment, Funder implies that it is only once characters are able to reconcile with one another, as well as their pasts, that they can hope to overcome the ramifications it had on their lives.

So you've got an overall contention, and you're trying to examine different facets of the prompt/the extent to which you agree/disagree.
Title: Re: VCE English Question Thread
Post by: kaii on July 23, 2017, 12:38:06 am
Hey guys, I'm currently on the writing phase of my oral and would love everyone's opinions on how I would need to convey my contention to the audience  Do I say it on the get-go or ..? I'm confused please help!
My topic is should there be a national ban on single-use plastic bags?
And I think yes!


Here's an extract from my intro:
Australia. We’re one of the trashiest countries in the developed world. In fact, we’re the 2nd highest producers of rubbish (only US is trashier ><). As a nation, we use over 4 billion plastic bags every year, that’s over 10 million new bags every day!
We dump over 7000 recyclable plastics every minute which all end up into landfill!
[pause]
Now, let me ask you something…does anybody know Australia’s current population?
⎝   YES : Respond to audience, “Yes, that’s correct…/ close but...”
⎝   NO: “Well we have a growing population of 24 million and our waste is growing twice as fast
If 48 million of these plastic bags remained in the environment and made into a single plastic sheet, it would be big enough to cover the whole of Melbourne’s CBD.
With this rate, how are we as a nation meant to tackle this issue of plastic bags? ……………………… MAIN CONTENtION??...............

 
blah blah body paragraph blah
Title: Re: VCE English Question Thread
Post by: HopefulLawStudent on July 23, 2017, 10:13:53 am
Hey guys, I'm currently on the writing phase of my oral and would love everyone's opinions on how I would need to convey my contention to the audience  Do I say it on the get-go or ..? I'm confused please help!
My topic is should there be a national ban on single-use plastic bags?
And I think yes!


Here's an extract from my intro:
Australia. We’re one of the trashiest countries in the developed world. In fact, we’re the 2nd highest producers of rubbish (only US is trashier ><). As a nation, we use over 4 billion plastic bags every year, that’s over 10 million new bags every day!
We dump over 7000 recyclable plastics every minute which all end up into landfill!
[pause]
Now, let me ask you something…does anybody know Australia’s current population?
⎝   YES : Respond to audience, “Yes, that’s correct…/ close but...”
⎝   NO: “Well we have a growing population of 24 million and our waste is growing twice as fast
If 48 million of these plastic bags remained in the environment and made into a single plastic sheet, it would be big enough to cover the whole of Melbourne’s CBD.
With this rate, how are we as a nation meant to tackle this issue of plastic bags? ……………………… MAIN CONTENtION??...............

 
blah blah body paragraph blah

To be honest, in terms of conveying your contention, you have a fair bit of leeway (as with most things in the oral tbh). Like some of my friends never expressly mentioned what their contention was, instead preferring to allow their listener to make an informed inference as to what their stance on the issue was. With that said, I always preferred to flag my contention at the end of my intro just to be safe and ensure that I never got pulled up for not having a clear stance on the issue.

Consider:

1.   Some people may think that plastic bags serve a very real purpose and their removal from our supermarkets would precipitate an unwanted inconvenience. However, it is my contention that this view is just a product of their own misunderstanding.

vs.

2.   For the sake of our children, our country and our future, something must be done. The government MUST implement the proposed ban if we are to insure our environment against these plastic menaces that lurk our streets.

In both instances, I’m presenting my contention from the get go. However, notice how, in the second example, I'm presenting my contention in a persuasive way ("something must be done") to frame it as something I am endorsing or agreeing with? Whereas in the first case, the same point is presented in a "some people may think this, but they're wrong" sort of way and it lacks the persuasive punch that the second example has because it has been prefaced with “The following point I am about to make is an argument. Here is the argument: _____".

tl;dr – you can give your contention from the get go but try to be persuasive while doing it.
Title: Re: VCE English Question Thread
Post by: clarke54321 on July 23, 2017, 12:34:26 pm
Is it unreasonable to start asking my English teacher to mark my work out of 10? Do other people have their teachers mark their practice essays as so?

Thanks  :)
Title: Re: VCE English Question Thread
Post by: Sine on July 23, 2017, 12:39:01 pm
Is it unreasonable to start asking my English teacher to mark my work out of 10? Do other people have their teachers mark their practice essays as so?

Thanks  :)
Not that unreasonable really depends on the teacher. My teacher last year would only tell me where I could improve and then on my last essay before a sac give me a score (out of 30 since that is what sacs were out of).

However closer to the exam I was given scores for essays but not exact scores. Moreso something along the lines of "this is around an 8" but it wouldve been a 9 if you did"x" and "y".
Title: Re: VCE English Question Thread
Post by: HopefulLawStudent on July 23, 2017, 12:47:14 pm
Is it unreasonable to start asking my English teacher to mark my work out of 10? Do other people have their teachers mark their practice essays as so?

Thanks  :)

It's worth asking, I suppose. You'll find that as you near the exam, teachers will end up marking that way anyway but it depends upon your teacher as to when they want to start. Don't try to pressure them into it if they don't want and you can always ask them to verbally give you an approximate mark if it means that much to you and they outright refuse to at this stage in the year.

However, with that said, I'd like to caution you against falling into the trap that is the numbers game where you allow the numbers to define your essay. Focus less on getting that number and in making consistent improvement in each essay that you write. My approach was that I liked to summarise feedback I'd gotten into three "key" areas for improvement and in the next essay I wrote, after I'd been given feedback, I'd follow up on those previous areas of improvement I'd identified and be like "do you think problem X from the last essay I wrote was still evident here or nah?" to my teacher.
Title: Re: VCE English Question Thread
Post by: kaii on July 24, 2017, 01:41:55 pm
To be honest, in terms of conveying your contention, you have a fair bit of leeway (as with most things in the oral tbh). Like some of my friends never expressly mentioned what their contention was, instead preferring to allow their listener to make an informed inference as to what their stance on the issue was. With that said, I always preferred to flag my contention at the end of my intro just to be safe and ensure that I never got pulled up for not having a clear stance on the issue.

Consider:

1.   Some people may think that plastic bags serve a very real purpose and their removal from our supermarkets would precipitate an unwanted inconvenience. However, it is my contention that this view is just a product of their own misunderstanding.

vs.

2.   For the sake of our children, our country and our future, something must be done. The government MUST implement the proposed ban if we are to insure our environment against these plastic menaces that lurk our streets.

In both instances, I’m presenting my contention from the get go. However, notice how, in the second example, I'm presenting my contention in a persuasive way ("something must be done") to frame it as something I am endorsing or agreeing with? Whereas in the first case, the same point is presented in a "some people may think this, but they're wrong" sort of way and it lacks the persuasive punch that the second example has because it has been prefaced with “The following point I am about to make is an argument. Here is the argument: _____".

tl;dr – you can give your contention from the get go but try to be persuasive while doing it.

Oh okay yep that makes so much sense and thank you for going into details because that cleared up all the confusion my teacher didn't bother discussing lol
Title: Re: VCE English Question Thread
Post by: kaii on July 24, 2017, 09:23:20 pm
Hello again! I'm having trouble with how to finish my oral presentation strongly. I mean what I have now is pretty good to me but I'd just like to take it up one more level and I'd love your help ! Also, if you find any grammar mistakes and/or have advice for other ways to say this or that please tell me. Thank youu, your help is much appreciated :)

So what can we do to resolve this issue? Is it too late??!?
Luckily, the answer is no. Just a week ago, major supermarkets such as Woolworths and Coles have announced that they were going to get rid of single-use plastic bags over the next twelve months. This will mostly affect VIC,NSW and WA since other states and territories have lead the way and banned the plastic bags already.

A nationwide plastic bag ban is the main solution to this problem but if we don’t work together now and give the state premiers a push for a ban, nothing will change. You and I will keep using plastic bags. But I believe we can change this. You can change this.
For now, we can start small and it’s never too late!
First, we can reduce the amount of plastic bags we take home and instead use those green re-usable bags, and so those grey plastic bags we could recycle to the supermarkets (I recommend Coles!).

Banning plastic bags will be like an education tool for our population to remind ourselves about the environmental impacts of our own human actions. Banning single-use plastic bags will have valuable consequences in reducing plastic waste in the world’s oceans and waterways and it may also reduce their occurrence in the landfills.
If people start thinking about the wider problems caused by plastic wastes in the ocean and on land, I’m sure things will turn themselves around and I believe it will be sooner than later. So join me and demand a national plastic bag ban.

Title: Re: VCE English Question Thread
Post by: clarke54321 on July 27, 2017, 04:05:26 pm
Just to clarify, can I include evidence from the post-script of a novel in my essay? Also, if there are photos in the middle, can they be used for analysis?

Thanks in advance  :D
Title: Re: VCE English Question Thread
Post by: amigos on July 27, 2017, 04:30:06 pm
Just to clarify, can I include evidence from the post-script of a novel in my essay? Also, if there are photos in the middle, can they be used for analysis?

Thanks in advance  :D

Is this for Tracks? If so, my teacher said it was fine to mention both but also cautioned against relying too much on the photos.
 




Title: Re: VCE English Question Thread
Post by: clarke54321 on July 27, 2017, 04:36:01 pm
Is this for Tracks? If so, my teacher said it was fine to mention both but also cautioned against relying too much on the photos.

Aah... thanks very much! And yes, it is for Tracks :)
Title: Re: VCE English Question Thread
Post by: Lavar Big BBB Balls on July 31, 2017, 10:41:12 pm
For LA, I'm just wondering should there be more of an emphasis on the effect/intended effect on the reader or on the reason why the readers feel a certain way to evidence/words used?

Does it really matter or should I not be viewing these two things as seperate 'steps'? Should these be naturally combined together?
Title: Re: VCE English Question Thread
Post by: clarke54321 on August 01, 2017, 08:02:09 am
For LA, I'm just wondering should there be more of an emphasis on the effect/intended effect on the reader or on the reason why the readers feel a certain way to evidence/words used?

Does it really matter or should I not be viewing these two things as seperate 'steps'? Should these be naturally combined together?

Hey!

This is a rough kind of structure I use:

Within body paragraphs (not including 'topic sentence')-

Sentence #1: Stating evidence for first half of sentence and then moving onto some kind of analysis
Sentence #2: Expanding on that analysis further
Sentence #3: Bringing it back to the reader and what the author is trying to achieve
Sentence #4: This feeling of/ notion that/ is further bolstered/fortified by...... (Next piece of evidence---- these linking words demonstrate your understanding of cumulative effects)

This kind of structure ensures that you are getting enough analysis in as well as acknowledging reader effect. So I don't necessarily think there should be more of an emphasis on reader effect, but only a consistent reference back to them throughout the piece.

Hopefully this helps you out  :)
Title: Re: VCE English Question Thread
Post by: Lavar Big BBB Balls on August 01, 2017, 08:47:16 am
^thanks!
Title: Re: VCE English Question Thread
Post by: -elcee on August 02, 2017, 09:43:26 pm
Hey All,
So recently I've had a talk with my English and Literature teachers about the quality of my essays.
Their main criticism, along with other teachers who have cross marked my sacs, is that I tend to ramble on in my sentences. ie; it takes a while for me to actually reach my point. I find this occurs in my everyday speech as well; I'm not really an effective communicator unless given time and a pen. When I write my essays, (often in timed conditions), I write whatever is in my mind, so it's not completely coherent and concise as it could be.
My knowledge of the content is good, but my teachers commented that this rambling is what is dividing me from that A+.
How can I improve on my coherence and conciseness in my essays ? My English teacher told me to read high scoring responses but since this is a new study design, there isn't many available apart from those in the text guides.
Thanks !
Title: Re: VCE English Question Thread
Post by: MightyBeh on August 02, 2017, 10:18:01 pm
Hey All,
So recently I've had a talk with my English and Literature teachers about the quality of my essays.
Their main criticism, along with other teachers who have cross marked my sacs, is that I tend to ramble on in my sentences. ie; it takes a while for me to actually reach my point. I find this occurs in my everyday speech as well; I'm not really an effective communicator unless given time and a pen. When I write my essays, (often in timed conditions), I write whatever is in my mind, so it's not completely coherent and concise as it could be.
My knowledge of the content is good, but my teachers commented that this rambling is what is dividing me from that A+.
How can I improve on my coherence and conciseness in my essays ? My English teacher told me to read high scoring responses but since this is a new study design, there isn't many available apart from those in the text guides.
Thanks !
This is a really good question and I think a problem for a lot of students. I was a serial rambler but managed to get over it (mostly ::) ). There's nothing wrong with writing whatever is in your mind, but it needs filtering to lock your writing together perfectly. By taking ten (or 20) minutes to write out an essay plan in dot points, you could get everything out and only pick the cream of the crop for your essay. You do have to get pretty specific though - I tended to list my ideas, evidence, and the logical conclusions so that writing the essay was less about the thinking and more about just making it cover everything with minimal fluff. Obviously that's not going to magically make you a more concise writer but it's definitely a start.

Try practicing without a time limit - take a few hours if you can spare it and just write slowly. You said you have some trouble in timed conditions so it coulod be helpful for you to start by writing a few 'perfect' essays with no pressure and then incrimentally adding more time restrictions. Just dipping your toe in, so to speak.

Maybe starting your sentences with evidence or a strong point could help too. I'm personally of the opinion that essays don't need a lot of fluff to be good. I don't know many people that actually enjoy reading handwritten english essays so being blunt and less artsy is a positive attribute in my mind; 'masterful inclusion of unique and abstract alliteration' is probably over doing it, for example ::). Not to say that you can't have fun and show off a bit, but being frugal with your flair can really spice things up a bit.

Heidi wrote a bit about being concise here, she has some other tips that you might find to be worth a read. :)

Edit because I forgot to mention it: For conciseness, the actual content of the old essays don't matter. I don't know if any of the texts you're studying right now have relevant stuff to look at, but what you're looking for is sentence structure. It's worth printing some (or just reading them on the computer or whatever) and highlighting sentences that sound really, really cool to you. Maybe it's a bit nerdy to think there are cool sounding sentences in essays, but there's some really great ones that come up in examiner's reports and such that I'm pretty sure it's impossible to find any faults with at all. Definitely worth a look at old essays even if you don't know anything about what they'e actually about.
Title: Re: VCE English Question Thread
Post by: TheCommando on August 03, 2017, 07:33:44 pm
Whats a thematic prompt
Title: Re: VCE English Question Thread
Post by: Ashjames on August 03, 2017, 08:08:59 pm
Hey everyone!!

I was just wondering, for English I am rank 2 [there was only 2 points between me and rank 1] and for Unit 3 I got an A [80], what scores would I be looking at for exams?  {unit 4 so far, I got a 35/40]. Can I still get at least a 35, my cohort is moderate strength, like 8 people get good marks for English, the rest absolutely loathe English [I am being sooo serious]
Title: Re: VCE English Question Thread
Post by: sonnyangel on August 03, 2017, 08:23:53 pm
Whats a thematic prompt

A thematic prompt is one based on themes/key concepts in your text.
E.g (for the text Medea)
-Medea is a play about the extremes of emotion. Discuss.
-Medea is more about justice than revenge. Discuss.
-Medea is an examination of the difference between reason and passion. Discuss.

Title: Re: VCE English Question Thread
Post by: clarke54321 on August 03, 2017, 09:23:28 pm
Hey everyone!!

I was just wondering, for English I am rank 2 [there was only 2 points between me and rank 1] and for Unit 3 I got an A [80], what scores would I be looking at for exams?  {unit 4 so far, I got a 35/40]. Can I still get at least a 35, my cohort is moderate strength, like 8 people get good marks for English, the rest absolutely loathe English [I am being sooo serious]

Hey!

Unfortunately it is quite difficult to put a number on your potential study score. While a good rank is important, it is more important to refine your skills and ace the final exam. At the end of the day, it is the exam that will standardise your SAC scores and determine your ultimate study score.

Good luck :)
Title: Re: VCE English Question Thread
Post by: Spinxho on August 03, 2017, 10:27:10 pm
Hi everyone,
I am currently in year 9 and in English, I have been racking up A's across the entire year. I olan to take up Literature in year 10 as one my electives.
Is there a grade average I have to achieve in English/Literature in year 10 to take up 3/4 English/Literature in year 11? If so, what is it.
Many thanks
Title: Re: VCE English Question Thread
Post by: Coffee on August 03, 2017, 10:36:43 pm
Hi everyone,
I am currently in year 9 and in English, I have been racking up A's across the entire year. I olan to take up Literature in year 10 as one my electives.
Is there a grade average I have to achieve in English/Literature in year 10 to take up 3/4 English/Literature in year 11? If so, what is it.
Many thanks
There's no official rule or grade you need to achieve, but your school might have certain restrictions. It's best to speak with a careers counsellor from your school. :)

Just a word of warning, but you might want to leave English and Literature to Year 12, rather than completing them early, as students tend to perform better with an extra year's practise under their belt, as well as that, your writing tends to just 'mature' in general. It is, of course, up to you. I'd highly recommend completing both English and Literature in Year 12 though if you enjoy them, I did, and Literature was one of my favourite subjects! ;D
Title: Re: VCE English Question Thread
Post by: Spinxho on August 03, 2017, 10:44:23 pm
There's no official rule or grade you need to achieve, but your school might have certain restrictions. It's best to speak with a careers counsellor from your school. :)

Just a word of warning, but you might want to leave English and Literature to Year 12, rather than completing them early, as students tend to perform better with an extra year's practise under their belt, as well as that, your writing tends to just 'mature' in general. It is, of course, up to you. I'd highly recommend completing both English and Literature in Year 12 though if you enjoy them, I did, and Literature was one of my favourite subjects! ;D
Haha I see. Thank you very much! I have only thought about taking English/Literature in 3/4 early as I am unsure about the other subjects that I am choosing/have chose and their availability to me as a 3/4 subject in Year 11.
Title: Re: VCE English Question Thread
Post by: Coffee on August 03, 2017, 10:50:07 pm
Haha I see. Thank you very much! I have only thought about taking English/Literature in 3/4 early as I am unsure about the other subjects that I am choosing/have chose and their availability to me as a 3/4 subject in Year 11.
No worries, let us know how you get on. :)

If you're feeling stuck, or unsure about your subject selection, don't be afraid to post in the Victorian Education Discussion forums. There are plenty of people willing to help you out. :)
Title: Re: VCE English Question Thread
Post by: Spinxho on August 03, 2017, 10:55:31 pm
No worries, let us know how you get on. :)

If you're feeling stuck, or unsure about your subject selection, don't be afraid to post in the Victorian Education Discussion forums. There are plenty of people willing to help you out. :)
Of course :). Thank you very much for your advice!
Title: Re: VCE English Question Thread
Post by: HopefulLawStudent on August 06, 2017, 02:37:38 pm
To add on to the gr8 answers everyone else has already given:

Hey All,
So recently I've had a talk with my English and Literature teachers about the quality of my essays.
Their main criticism, along with other teachers who have cross marked my sacs, is that I tend to ramble on in my sentences. ie; it takes a while for me to actually reach my point. I find this occurs in my everyday speech as well; I'm not really an effective communicator unless given time and a pen. When I write my essays, (often in timed conditions), I write whatever is in my mind, so it's not completely coherent and concise as it could be.
My knowledge of the content is good, but my teachers commented that this rambling is what is dividing me from that A+.
How can I improve on my coherence and conciseness in my essays ? My English teacher told me to read high scoring responses but since this is a new study design, there isn't many available apart from those in the text guides.
Thanks !

Ways to Avoid Rambling in Writing


1.   Read your own writing.

2.   Avoid super long sentences. If your sentence is more than 2 lines when hand-written out, then consider shortening it.

3.   Ask yourself what the main point of each sentence is. Each sentence should only have one main point. Therefore, if you find that a sentence has more than one main point, divide the sentence accordingly.

4.   If you have heaps of conjunctions in one sentence (and, but, or, nor, because, although, when, for, so), start breaking up that sentence.

Just to clarify, can I include evidence from the post-script of a novel in my essay? Also, if there are photos in the middle, can they be used for analysis?

Thanks in advance  :D

If it's in your text, you could prob get away with it, but don't do it too much.

For LA, I'm just wondering should there be more of an emphasis on the effect/intended effect on the reader or on the reason why the readers feel a certain way to evidence/words used?

Does it really matter or should I not be viewing these two things as seperate 'steps'? Should these be naturally combined together?

Truth be told, I always conflated them together in my head cos I found that they naturally combined together really well in my head so I never saw them as two separate steps. So long as you're discussing both though, you should be allg.
Title: Re: VCE English Question Thread
Post by: TheCommando on August 06, 2017, 05:21:54 pm
Hey,could someone explain how u would structure a prompt for year of wonders and the crucible
Then prompt can be for eg What do these texts suggest about human behaviour in a crisis?
Title: Re: VCE English Question Thread
Post by: HopefulLawStudent on August 06, 2017, 05:37:39 pm
As in structure an essay?

If so, please check this out: https://atarnotes.com/forum/index.php?topic=172569.0
Title: Re: VCE English Question Thread
Post by: chantelle.salisbury on August 08, 2017, 08:47:39 pm
Hello,
i was just wondering whether i could get some feedback on my introduction on a q on 'Ransom'?
anything would be greatly appreciated ;)

topic-   ‘Ransom’ suggests that revenge is not the answer to grief. Discuss.
Whilst the narrative ‘Ransom’ is based on epic ancient historic stories from ‘The Iliad’, David Malouf discusses the impacts of human reactions and ways which individuals cope with emotions no matter of the time, era or world which we are part of. Malouf exhibits that revenge is a vicious never-ending cycle of destruction, both within persons and within entire groups, and the principle of vengeance is just as relevant today as it was in the 1200 BC. Revenge becomes the exterior of characters, tying them with rage and anger, making them unable to deal with the natural response of intensely personal sorrow and remorse generated by the loss of a loved one.  Part 1 of ‘Ransom’ seeks to establish a platform for the audience to discern a world which is stricken with intense grief, focusing on Achilles’ and his self-serving way to escape and externalise his anger. It is through Achilles that Malouf illustrates the ultimate burden of revenge and the damaging, antagonistic effects it has on one’s own character and those in the vicinity. In contrast, whilst Priam would be somewhat justified to seek revenge on Achilles -for the slaying and treatment of his son Hector- his approach is much more honourable. For Priam revenge is useless; it is unable to overcome the grief or restore the sense of loss, and therefore to bring about the peace for the Trojans’, he and his role ultimately changes. Similarly, the carter Somax finds no benefit in vindictiveness for the death of his children, but learns to cope with grief in an entirely different way; to share it.
Title: Re: VCE English Question Thread
Post by: lilyrosee on August 09, 2017, 06:19:35 pm
Hello,
i was just wondering whether i could get some feedback on my introduction on a q on 'Ransom'?
anything would be greatly appreciated ;)

topic-   ‘Ransom’ suggests that revenge is not the answer to grief. Discuss.
Whilst the narrative ‘Ransom’ is based on epic ancient historic stories from ‘The Iliad’, David Malouf discusses the impacts of human reactions and ways which individuals cope with emotions no matter of the time, era or world which we are part of. Malouf exhibits that revenge is a vicious never-ending cycle of destruction, both within persons and within entire groups, and the principle of vengeance is just as relevant today as it was in the 1200 BC. Revenge becomes the exterior of characters, tying them with rage and anger, making them unable to deal with the natural response of intensely personal sorrow and remorse generated by the loss of a loved one.  Part 1 of ‘Ransom’ seeks to establish a platform for the audience to discern a world which is stricken with intense grief, focusing on Achilles’ and his self-serving way to escape and externalise his anger. It is through Achilles that Malouf illustrates the ultimate burden of revenge and the damaging, antagonistic effects it has on one’s own character and those in the vicinity. In contrast, whilst Priam would be somewhat justified to seek revenge on Achilles -for the slaying and treatment of his son Hector- his approach is much more honourable. For Priam revenge is useless; it is unable to overcome the grief or restore the sense of loss, and therefore to bring about the peace for the Trojans’, he and his role ultimately changes. Similarly, the carter Somax finds no benefit in vindictiveness for the death of his children, but learns to cope with grief in an entirely different way; to share it.


I don't have time to closely go through it but overall the length could be shorter. Try being more clear and concise because introductions are not where the bulk of your marks come from. Also, it is worth noting that for the exam you will have to include both Ransom and Invictus in your introduction so try and keep it short and sweet. Some teachers want you to go a little into the background of the text but others do not mind and just want you to focus on answering the prompt, so you could possibly not go into the wider context e.g. how Malouf based the text on the Illiad - but it does vary from teacher to teacher.

Hope this helps :)
Title: Re: VCE English Question Thread
Post by: chantelle.salisbury on August 09, 2017, 10:10:27 pm
I don't have time to closely go through it but overall the length could be shorter. Try being more clear and concise because introductions are not where the bulk of your marks come from. Also, it is worth noting that for the exam you will have to include both Ransom and Invictus in your introduction so try and keep it short and sweet. Some teachers want you to go a little into the background of the text but others do not mind and just want you to focus on answering the prompt, so you could possibly not go into the wider context e.g. how Malouf based the text on the Illiad - but it does vary from teacher to teacher.

Hope this helps :)

yup thats awesome :).... on reflection it was well long so thanks heaps for that
i have issues where my writing tends to 'ramble' and i can never seem to master succinct writing.. i'll keep pursuing tho ;)
thanks heaps again
Title: Re: VCE English Question Thread
Post by: lilyrosee on August 09, 2017, 10:21:33 pm
yup thats awesome :).... on reflection it was well long so thanks heaps for that
i have issues where my writing tends to 'ramble' and i can never seem to master succinct writing.. i'll keep pursuing tho ;)
thanks heaps again

Happy to help.

I am studying Ransom too and I find reading/marking other people's work really helps to develop my understanding of a text.
Title: Re: VCE English Question Thread
Post by: TSEtuition on August 10, 2017, 10:20:49 am
Hello,
i was just wondering whether i could get some feedback on my introduction on a q on 'Ransom'?
anything would be greatly appreciated ;)

topic-   ‘Ransom’ suggests that revenge is not the answer to grief. Discuss.
Whilst the narrative ‘Ransom’ is based on epic ancient historic stories from ‘The Iliad’, David Malouf discusses the impacts of human reactions and ways which individuals cope with emotions no matter of the time, era or world which we are part of. Malouf exhibits that revenge is a vicious never-ending cycle of destruction, both within persons and within entire groups, and the principle of vengeance is just as relevant today as it was in the 1200 BC. Revenge becomes the exterior of characters, tying them with rage and anger, making them unable to deal with the natural response of intensely personal sorrow and remorse generated by the loss of a loved one.  Part 1 of ‘Ransom’ seeks to establish a platform for the audience to discern a world which is stricken with intense grief, focusing on Achilles’ and his self-serving way to escape and externalise his anger. It is through Achilles that Malouf illustrates the ultimate burden of revenge and the damaging, antagonistic effects it has on one’s own character and those in the vicinity. In contrast, whilst Priam would be somewhat justified to seek revenge on Achilles -for the slaying and treatment of his son Hector- his approach is much more honourable. For Priam revenge is useless; it is unable to overcome the grief or restore the sense of loss, and therefore to bring about the peace for the Trojans’, he and his role ultimately changes. Similarly, the carter Somax finds no benefit in vindictiveness for the death of his children, but learns to cope with grief in an entirely different way; to share it.

You certainly know your text! This is a common problem amongst students (I'm a full time English tutor) - you're packing too much into your intro. Many of these points should be in your body paragraphs - remember the intro gives your reader a taste of what you're going to say. I recently posted a video about how to structure an intro (video is on my FB page) but the overall gist is that you should treat it like you're asking someone out on a date - just enough info to keep your reader interested in reading more, but not so much that you'll scare them off. Hope that helps! =)
Title: Re: VCE English Question Thread
Post by: chantelle.salisbury on August 10, 2017, 04:54:01 pm
Happy to help.

I am studying Ransom too and I find reading/marking other people's work really helps to develop my understanding of a text.

yes its a great idea... i like sharing resources!
Title: Re: VCE English Question Thread
Post by: chantelle.salisbury on August 10, 2017, 04:58:54 pm
You certainly know your text! This is a common problem amongst students (I'm a full time English tutor) - you're packing too much into your intro. Many of these points should be in your body paragraphs - remember the intro gives your reader a taste of what you're going to say. I recently posted a video about how to structure an intro (video is on my FB page) but the overall gist is that you should treat it like you're asking someone out on a date - just enough info to keep your reader interested in reading more, but not so much that you'll scare them off. Hope that helps! =)

ok sweet... ill keep that in mind. my intro's have never really been 'short'. and whilst i know there is no word amount count, can i ask, how many words would a good short succinct intro be?
thanks :)
Title: Re: VCE English Question Thread
Post by: brady_price on August 12, 2017, 10:12:18 am
ok sweet... ill keep that in mind. my intro's have never really been 'short'. and whilst i know there is no word amount count, can i ask, how many words would a good short succinct intro be?
thanks :)

Intros should be roughly 10% of your word count. You basically just want to get your contention in there very succinctly, your signposting in, and any contextual information in there as quickly as possible, so only around 4 sentences I would probably recommend, but even then these sentences shouldn't be too lengthy. That said it does depend on your topic and also the type of essay, but this is a general rule to go by :)
Title: Re: VCE English Question Thread
Post by: TheCommando on August 12, 2017, 10:25:40 am
As in structure an essay?

If so, please check this out: https://atarnotes.com/forum/index.php?topic=172569.0
nah, i read that but im not quite sure what to say or plan exactly for 'What do these texts suggest about human behaviour in a crisis?' for year of wonders and the crucible
Title: Re: VCE English Question Thread
Post by: Syndicate on August 12, 2017, 05:49:20 pm
Hey,

What does it mean by "implications of the topic" in order to achieve a 9/10 for Section A. Can someone please give me any examples.

Thanks,
Syndicate
Title: Re: VCE English Question Thread
Post by: clarke54321 on August 12, 2017, 09:21:36 pm
Hey,

What does it mean by "implications of the topic" in order to achieve a 9/10 for Section A. Can someone please give me any examples.

Thanks,
Syndicate

Hey!

'Implications of the topic' basically means, what assumptions or suggestions are being made through the prompt? Markers want you to effectively tease out these ideas. Using skilful evidence, they want you to challenge these assumption and debate them, until you can come to a well justified, coherent and sustained interpretation. So put simply, it is 'read between the lines.'

Example: 'An important theme in a Christmas Carol is that actions have enduring consequences.'

A mid-range essay would probably talk about actions having consequences. But they would ignore the implication/assumption that they have enduring ones. A high-range essay would tease this idea out. Do all actions have enduring consequences? Are there specific types of actions that have enduring consequences? The term 'enduring' is pertinent to this topic and is an example of an implication.

Therefore, the easiest way of determining an implication would be to underline key words and boil the prompt down to a really basic idea. In every-day, English language, what is the prompt saying?

I hope this helps in some way :)
Title: Re: VCE English Question Thread
Post by: HopefulLawStudent on August 13, 2017, 10:26:29 am
Hello,
i was just wondering whether i could get some feedback on my introduction on a q on 'Ransom'?
anything would be greatly appreciated ;)

topic-   ‘Ransom’ suggests that revenge is not the answer to grief. Discuss.
Whilst the narrative ‘Ransom’ is based on epic ancient historic stories from ‘The Iliad’, David Malouf discusses the impacts of human reactions and ways which individuals cope with emotions no matter of the time, era or world which we are part of. Malouf exhibits Misused exhibits methinks that revenge is a vicious never-ending cycle of destruction, both within persons and within entire groups, and the principle of vengeance is just as relevant today as it was in the 1200 BC. Relevance to the topic?  Revenge becomes the exterior of characters, What do you mean by this?  tying them with rage and anger, making them unable to deal with the natural response of intensely personal sorrow and remorse generated by the loss of a loved one.  Part 1 of ‘Ransom’ seeks to establish a platform for the audience to discern a world which is stricken with intense grief, focusing on Achilles’ and his self-serving way to escape and externalise his anger. It is through Achilles that Malouf illustrates the ultimate burden of revenge and the damaging, antagonistic effects it has on one’s own character and those in the vicinity. In contrast, whilst Priam would be somewhat justified to seek revenge on Achilles -for the slaying and treatment of his son Hector- his approach is much more honourable. For Priam revenge is useless; it is unable to overcome the grief or restore the sense of loss, and therefore to bring about the peace for the Trojans’, he and his role ultimately changes. Similarly, the carter Somax finds no benefit in vindictiveness for the death of his children, but learns to cope with grief in an entirely different way; to share it. This stuff would be better suited in your body paragraphs than your intro. Your intro should be inherently brief.

ok sweet... ill keep that in mind. my intro's have never really been 'short'. and whilst i know there is no word amount count, can i ask, how many words would a good short succinct intro be?
thanks :)

3 to 4 sentences for your intro is the general rule.

nah, i read that but im not quite sure what to say or plan exactly for 'What do these texts suggest about human behaviour in a crisis?' for year of wonders and the crucible

Weeeell, what have you got so far by way of a plan/what you're thinking you might say?

Hey,

What does it mean by "implications of the topic" in order to achieve a 9/10 for Section A. Can someone please give me any examples.

Thanks,
Syndicate

To add on to clarke54321's fantastic advice:

Think of implications almost as you asking the question of: If the core of the prompt is true, then what?

You're almost like poking holes at the prompt and considering the extent to which you agree with certain sections of the prompt with relation to the text at hand and the more questions you posit, the more complex your discussion, the less superficial your argument is. Unpacking the implications of the topic is more typical of high range responses so don't stress too much if you're having difficulty with finding implications - just keep working at it. :)
Title: Re: VCE English Question Thread
Post by: dx1sy on August 13, 2017, 12:31:48 pm
Hi! I was wondering for the comparative topic "Compare and contrast how ‘The Crucible’ and ‘Year of Wonders’ explore how individuals cope in times of crisis." whether it would be better to structure the body paragraphs by focusing on the different crisis that are present in both texts and using those as topic sentences and then explaining how different individuals respond to them within the body paragraphs or whether it would be better to structure the body paragraphs by using the different ways that individuals cope in times of crisis that are present in both texts as topic sentences.  :)
Title: Re: VCE English Question Thread
Post by: clarke54321 on August 13, 2017, 12:45:46 pm
Hi! I was wondering for the comparative topic "Compare and contrast how ‘The Crucible’ and ‘Year of Wonders’ explore how individuals cope in times of crisis." whether it would be better to structure the body paragraphs by focusing on the different crisis that are present in both texts and using those as topic sentences and then explaining how different individuals respond to them within the body paragraphs or whether it would be better to structure the body paragraphs by using the different ways that individuals cope in times of crisis that are present in both texts as topic sentences.  :)


Hey!

I'd probably be more inclined to base topics sentences around the different ways characters cope with crises. This has a more direct link to the prompt (an exploration of how individuals cope). A focus on different crises is straying too far away from this central notion. I've only studied the Crucible, but I suspect that within both texts, a character's response to a crisis is dependent on the type of crisis. Therefore your two ideas are definitely interrelated.

So to clarify, develop your topic sentences in a way where you can maintain the essence of the prompt!

Hope this helps :)

Title: Re: VCE English Question Thread
Post by: HopefulLawStudent on August 13, 2017, 12:54:05 pm
Hi! I was wondering for the comparative topic "Compare and contrast how ‘The Crucible’ and ‘Year of Wonders’ explore how individuals cope in times of crisis." whether it would be better to structure the body paragraphs by focusing on the different crisis that are present in both texts and using those as topic sentences and then explaining how different individuals respond to them within the body paragraphs or whether it would be better to structure the body paragraphs by using the different ways that individuals cope in times of crisis that are present in both texts as topic sentences.  :)

I agree with clarke12345: the similarities and differences in the ways ways characters cope with would prob make for a stronger essay than here a bunch of different crisis you work it out how this relates to the prompt.
Title: Re: VCE English Question Thread
Post by: arlynl99 on August 13, 2017, 08:42:07 pm
hello  :) :)
currently studying 'Ransom' and 'Invictus'.
my q. is Compare the ways in which ‘Ransom’ and ‘Invictus’ highlight the futility of revenge.
just seeking some help how to spit the paragraphs up. i was thinking one of Invictus and another on Ransom. Just wondering what to do for the other one.
Thanks
Any help is much appreciated  ;D ;D ;D
Title: Re: VCE English Question Thread
Post by: chantelle.salisbury on August 13, 2017, 09:45:32 pm
hello  :) :)
currently studying 'Ransom' and 'Invictus'.
my q. is Compare the ways in which ‘Ransom’ and ‘Invictus’ highlight the futility of revenge.
just seeking some help how to spit the paragraphs up. i was thinking one of Invictus and another on Ransom. Just wondering what to do for the other one.
Thanks
Any help is much appreciated  ;D ;D ;D

i think comparison essays are best to compare continually throughout the essay, over against doing separate paragraphs on each. (depending on the teacher of course, as some prefer it that way)

well i would be inclined to suggest;
para1- the uselessness of revenge, whilst revenge is self-serving it isnt satisfying but instead creates more damage, grief, sorrow, remorse within the revengeful character
para 2- the vicious ongoing downward spiral of revenge, it doesnt enhance lives, but rather one individual's actions can cause grief to entire groups/society's. it is fruitless and pointless. revenge only breaks down relationships rather than enhancing them.
para 3- reconciliation and humanity is a much greater, more honourable, respectful answer or response to grief and sorrow
whilst readers may feel Priam and Mandela could be somewhat justified in seeking revenge they show a more honourable approach. and despite reproach (from hecuba/the princes and Mandela's family and security team) the leaders do what is best for their nation/tribe/groups rather than satisfy personal feelings, which ultimately brings them peace, happiness, respect, legacy

just suggestions.. so dont feel like you have to go with that.!  ;)
 :)

Title: Re: VCE English Question Thread
Post by: heids on August 14, 2017, 10:31:56 am
hello  :) :)
currently studying 'Ransom' and 'Invictus'.
my q. is Compare the ways in which ‘Ransom’ and ‘Invictus’ highlight the futility of revenge.
just seeking some help how to spit the paragraphs up. i was thinking one of Invictus and another on Ransom. Just wondering what to do for the other one.
Thanks
Any help is much appreciated  ;D ;D ;D

This post deals with this in detail, worth a thorough read. :)
Title: Re: VCE English Question Thread
Post by: arlynl99 on August 14, 2017, 06:42:45 pm
This post deals with this in detail, worth a thorough read. :)
thanks heaps very helpful and applicable ;D ;D ;D ;D

i think comparison essays are best to compare continually throughout the essay, over against doing separate paragraphs on each. (depending on the teacher of course, as some prefer it that way)

well i would be inclined to suggest;
para1- the uselessness of revenge, whilst revenge is self-serving it isnt satisfying but instead creates more damage, grief, sorrow, remorse within the revengeful character
para 2- the vicious ongoing downward spiral of revenge, it doesnt enhance lives, but rather one individual's actions can cause grief to entire groups/society's. it is fruitless and pointless. revenge only breaks down relationships rather than enhancing them.
para 3- reconciliation and humanity is a much greater, more honourable, respectful answer or response to grief and sorrow
whilst readers may feel Priam and Mandela could be somewhat justified in seeking revenge they show a more honourable approach. and despite reproach (from hecuba/the princes and Mandela's family and security team) the leaders do what is best for their nation/tribe/groups rather than satisfy personal feelings, which ultimately brings them peace, happiness, respect, legacy

just suggestions.. so dont feel like you have to go with that.!  ;)
 :)


 
ok thanks heaps for that....discussed in class today and that is essentially the same what we covered......
Thanks heaps
Title: Re: VCE English Question Thread
Post by: TheCommando on August 20, 2017, 02:10:37 pm
Okay, I know the posts above are at least partly in jest, but I'm going to answer these seriously because I think there's an underlying problem here that's unique to English subjects.

Most subjects consist of a fairly straightforward series of tasks: sit down, do exercises 1-5, Qs. a-g; memorise vocab list from chapter 4; do a practice exam, that sort of thing. Many people would argue you can treat English the same way, and that it's just a matter of finding the right activity to work on.

This approach isn't invalid. Plenty of people have scored very highly by simply understanding the system, writing heaps (/"spamming" essays as I'm told da kidz are calling it) and memorising chunks of whatever works.
I feel like this is perpetuated by a lot of schools, tutors, and even professional companies because it's a comfort. Being told you can write a certain number of essays and be certain of a reasonably good mark is nice to hear. And whilst it's not totally untrue, I do think it's contributing to some serious misconceptions regarding the subject.

The best kept secret, I've found, is that although you may score well if you know what to write, you will definitely score well if you know how to think.

This is an unpopular view for good reasons; it's not like a teacher with a class of 30 can feasibly teach students how to think within 8 months, ~4 hours a week, whilst also conforming to a syllabus and the pressure of exams. I'll admit I've caved to this pressure as well with a few of my students, and end up just focusing on criteria and assessment because of time restrictions or other issues.

But English is a subject that rewards thinking.

You can rote-learn. You can know your texts inside-out and study high-scoring responses and annoy your classroom teacher until they give you so much feedback you can predict what they're going to say. What's more, you're going to feel good about this, because you're physically doing tangible work that your brain associates with progress. Even if you're hitting some mental blockades with the content, it'll still feel like improvement because you're working at it.

For some people this is the only way they learn. Quite a few people in my year level would be writing essays from day one. They were atrocious essays, and were in no way indicative of their abilities, but for them, it was a way of consolidating their knowledge.

But the only reason why this works is because of the (often unintentional) tangential benefits.

Doing the 'class-assigned' kinds of activities is a slow-but-effective way to better your thinking. So why do this when you could go for a fast-but-effective method?

Well, partly because it's going to feel slower. You'll be doing unfamiliar things, and for a long time, you'll probably be doing them badly. However, as someone who's seen the system from both sides now, I've concluded this is a much better way of tackling the course.

For starters, let's clarify what's meant by 'thought.' We all know what it means, but strangely it's not a word that gets tossed around in English classes quite so much as 'juxtaposition' or 'inclusive language' might. This can be attributed to it's abstract-ness: your teacher can't see you thinking, save for seeing the result of the process in essay-form.

So when you're getting essay feedback, you're receiving criticisms for the product of your thinking, right? (-Excluding handwriting issues or minor structural things that you maybe weren't aware of.) Here's where problems start to arise...

For anyone who's worked in retail/ hospitality, you'll probably be familiar with copping blame for things totally out of your control. I used to work in a chicken and chip shop, and I'd have customers who would come in and complain about everything under the sun; chips aren't cooked right, the salt is to salty, the chickens are too small, why does this salad have lettuce in it? etc. etc.
I was but a mere server-girl, and so, realistically, if these bitterly displeased customers actually wanted their problems solved, they would have addressed the root of the problem and not hurled abuse and/or utensils at me.

Your English teachers, in this somewhat tortured metaphor, are akin to my old chicken shop patrons. They're not trained to consider the source of their discomfort, be that a misunderstanding of the text or the fact that our shop was drastically understaffed most nights; instead, their natural inclination is to blame what is made apparent:  the wrong words in a body paragraph, or a slightly dirty fork.

When you get comments on your essay like 'needs development' or 'I'm not sure about this,' what your teachers are actually-sort-of-kinda-but-not-really-trying-to-say is change your thinking! But that sounds weird because it's easier for them to focus on your essay itself, and it's less strenuous for students to obsess over numerical scores or criteria than it is to consider the possibility of mind-altering-drugs-study instead.

Now prepare yourselves for
Lauren's foolproof guide for How to Think Good
To demonstrate this we're going to look at a textual excerpt. You don't have to know anything about it, in fact it's better if you don't. I'm adapting this from an Andrew Bovell play called 'Speaking in Tongues' if anyone's interested.
         VALERIE: [answering machine] John, it's me... Valerie. I wish you'd let me do the message.
                       You sound so... I don't know... distant.

To learn how to think properly for English (/Lit, which is what I usually use this example for, but tomaito tomahto) all you have to do is answer this question: What do you know about Valerie and John?
That's all. But fair warning, my answer to this question is over 2000 words long, and that's all without reference to the play this came from.

Most people will fumble for a starting point at first, like 'well, we know she's talking to John on an answering machine, and that she wants to do the message instead of him.' Later, once you get past the basic, denotative stuff, you'll end up in 'assumptions' territory, eg. maybe they're not getting along, and that's why he sounds distant and she's not allowed to do the message. Keep building on this, and eventually you'll reach full blown implications: John is trying to maintain some semblance of power in their marriage by exerting control over petty things like which of them record an answering machine message. Meanwhile, Valerie is able to undermine his authority through criticism; she is still able to voice her objections, meaning he does not have complete command over her.
That's not to say there is a 'right' answer. You could go in a completely different direction, eg. The fact that the two are communicating via an answering machine - an innately indirect form of conversation - suggests they are not able to engage with one another on any level. Both John and Valerie are "distant," and without artificial conduits like answering machines between them, their relationship has very little holding it together.

Evidently what I'm talking about here is more like overthinking than just thinking, but perhaps that's appropriate.
Let me be clear: this will not directly help you. You should not spend 200 words in an English body paragraph analysing two sentences from the text/article. This is not about a subject-specific skillset, this is about rewiring your brain to look at things differently.

In the above exercise, I extrapolated from two lines of dialogue and concluded that the couple had a serious communication barrier between them, and were likely in the midst of some confusion regarding the power balance between them. I could be wrong, but that's not the point. The point is that I can justify my thinking.

I had a teacher who conducted a similar exercise in class and ended it with 'of course you couldn't say something ridiculous like 'this excerpt suggests John wants to grow a beard' or anything.' But I disagree. 
John's lack of control over Valerie signifies his emasculation ,which is exacerbated by Valerie's implied criticism. She is able to express her wants in no uncertain terms, and her power is marked by a stereotypically feminine "I wish you'd let me" brand of passive aggression. Thus, it seems logical then for John to gravitate towards physicality as a means of reasserting himself with something equally gender codified; perhaps a handlebar mustache, or even a proper, fully-fledged man-beard - an ideogram of his patient but firm dominance.
Note: I would never seriously write that in an essay. This was an exercise in thinking, and taking my analysis further than the surface level. Do this often enough, and you begin to get a feel for what actually belongs in an essay, and what's just conjecture.

I know this is quite text-heavy, but a willingness to read is just important as a willingness to write for English :)

Yes, it's frustrating when you're not making obvious numerical improvements, and yes, working out what constitutes as "useful" study is a lot tough in English than it is in other subjects, but it ultimately boils down to your willingness to engage with the material.

Think about stuff, and learn how to demonstrate this thinking in the best way possible. No magic required :)


Sooooooo bloody good

So just keep tryingg to gather as much information from a scentence or two from any text which improves ur thinking
Also im not 100% what being able to think implies, is it your ability to gatther ideas and interpret passagges from the text
Title: Re: VCE English Question Thread
Post by: chrissy111 on August 20, 2017, 05:20:49 pm
Hi Lauren :)

I'm struggling to find resources for 'The lieutenant' which is my text response text.
Do you know where I could find some? (quotes, analysis etc)

And for comparative, my teachers are stressing on the 'idea' the paragraph is based on and talking more about the idea rather than just texts and evidence. Almost 10 lines are for 'commenting on the idea'. Do you think this is a good way to go about comparative? I thought it was more important to compare the texts. (I'm doing Invictus and Ransom btw!)
Title: Re: VCE English Question Thread
Post by: literally lauren on August 29, 2017, 04:16:37 pm
Hi Lauren :)

I'm struggling to find resources for 'The lieutenant' which is my text response text.
Do you know where I could find some? (quotes, analysis etc)

And for comparative, my teachers are stressing on the 'idea' the paragraph is based on and talking more about the idea rather than just texts and evidence. Almost 10 lines are for 'commenting on the idea'. Do you think this is a good way to go about comparative? I thought it was more important to compare the texts. (I'm doing Invictus and Ransom btw!)


Hi chrissy!

There's a good article here if you're looking for a general overview of key ideas. There's also some decent stuff here. Unfortunately The Lieutenant is in it's first year as a Text Response text, so there may not be a heap of stuff online, but google around and see what you can find :)

Regarding the comparative task, I would advise against spending too long talking about general ideas - you can certainly discuss them or have topic sentences / connecting sentences / linking sentences that focus more on general commonalities than on specific textual details, but the VCAA criteria seems to put more emphasis on close textual discussion (of both texts) than on your ability to discuss bigger concerns.

Exam marking scheme for your comparative essay is as follows:

(http://i.imgur.com/BtTKnE4.png)

So there's nothing wrong with having a few sentences like about key themes like honour, tolerance, community, strength, memory, etc. but I think spending 10 lines per paragraph just on that stuff would be quite risky.

But as always, do what your teacher suggests for your SAC, and do what the examiners suggest for your exam! ;)
Title: Re: VCE English Question Thread
Post by: S200 on August 30, 2017, 02:04:52 pm
Randomly...
Would it be right to call 'Animal Farm' a lampoon against socialism?

(Lampoon... that awkward moment when you literally only know a word because of studying the 'Mabo' documentary... :D
#Thanks2blackfellafilms)
Title: Re: VCE English Question Thread
Post by: chrissy111 on August 31, 2017, 01:52:18 pm
Thank you so much!!  :)
Title: Re: VCE English Question Thread
Post by: TSEtuition on August 31, 2017, 03:39:01 pm
ok sweet... ill keep that in mind. my intro's have never really been 'short'. and whilst i know there is no word amount count, can i ask, how many words would a good short succinct intro be?
thanks :)

My recommendation is somewhere between 4-6 sentences. It depends on how long your sentences are! Generally the guideline is: 1 or 2 to intro the text, 1 for contention, 1 or 2 to intro body paragraphs. Whatever you write, your intro must be shorter than your shortest body paragraph.

~Shirlaine from TSEtuition
Title: Re: VCE English Question Thread
Post by: humblepie on August 31, 2017, 08:53:46 pm
Hi! Just wondering if someone could help me with this prompt regarding Ransom and Invictus: "compare how the texts explore the competing impulses of duty and self-interest". I have a few basic ideas for each text, but have no clue on how to structure the three body paragraphs (I tend to use the idea-based method rather than the 'one text per paragraph' structure). I would really appreciate some help :)
Title: Re: VCE English Question Thread
Post by: tinagranger on September 01, 2017, 06:28:28 pm
Hi Lauren :) I have a few questions regarding the structure of a language analysis essay for the exam when there are 2 or more texts. I have been trying to paragraph by argument, and within the same paragraph, talk about different texts (i.e. 1st article --> linking word --> 2nd article), but I find it very difficult to make quick decisions on how to split my paragraphs when often, the arguments in each written text don't link up nicely with each other.

1. Could I still get 10/10 if I decide to make this my go-to structure:
Separate each text into main arguments/'themes'. Then find 2-3 links for each image in different sections of the essay - i.e. talk about the image in the same paragraphs as the written material.
BUT go through the written texts chronologically - eg: talk about 1st text (blocking that text off by argument) and find 2-3 links to image, then talk about 2nd text (blocking that text off by argument) and then find 2-3 links to image.

This would personally be a lot less stressful for me, because in the past I have sat there for a long time just figuring out how to link the 2 texts into the same paragraph! Would this structure always work?

2. However, if I use this method, how would I compare the two texts? Should my rule of thumb be that the 1st sentence of each paragraph of my 2nd text should be a comparative sentence? Also, is comparison part of the marking criteria? Is a simple linking word such as 'similarly' or 'on the other hand' sufficient, or should my comparative sentence at the start of the paragraphs for my 2nd text be comparing the argument? Or should I be aiming to make comparative comments throughout my whole piece?

3. Also, my concern with this method is that my piece will be imbalanced because the 2nd text is often shorter than the 1st text, and 1 paragraph wouldn't be enough to talk about the 2nd text.

Thus, at the moment, i think this would be my most likely structure, for 2 texts and 2 visuals:
1. Text 1 + visual 1
2. Text 1 + visual 1
3. Text 2 + visual 2
4. Text 2 + visual 2

This would mean that my 3rd and 4th paragraphs would be SIGNIFICANTLY shorter than my 1st and 2nd. Is this structure okay, or could you think of a better way to split paragraphs?

4. For example, is this a good structure for the 2015 LA (on bigsplash):
1. Para Text 1 - praise for bigsplash company --> shift to extolling Australian volunteers and arguing that they enrich and are essential to society.
+ 2 references to the 1st visual linked with text analysis

Para 2. Text 1 - generating guilt in Australians by arguing that we have undervalued and underappreciated volunteers
+ 1 reference to the 1st visual linked with text analysis

Para 3. Text 2 - Mathew Nguyen building praise for his organisation Tradespeople Without Borders and casting himself as a humble, approachable individual, thereby humanising himself in Australian's eyes and gaining the reader's trust.
+ 2 references to the 2nd visual linked with text analysis
(1. Presenting himself as hospitable and casual --> casual clothing of the people joining hands. Lol don't know if this is okay to talk about?
2. By emphasising his desire to 'make a difference' and 'help Australia's reputation as a caring country', Nguyen appeals to the patriotic instincts of his audience, and strives to generate a sense of solidarity and unity amongst Australians who have a fundamental sense of responsibility to help those around him. This notion of cohesion and harmony is given visual reinforcement by the accompanying image, which depicts....)

Para 4. Text 2 - Nguyen humbly proclaiming that though recognition and praise of volunteers is welcomed, it is not necessary and the true reward of volunteering is the delight of helping others.

(My 3rd and 4th paragraphs would be like half the length of my 1st and 2nd paragraphs though, which is what I am worried about!)

5. Would you ever make links to the visual when talking about a text that the visual is NOT accompanied with? Is this common/recommended? Eg: The 2nd visual of the 2015 piece feeding into Bennett's proclamation that humanitarian work 'stitches together the social fabric of our nation'

6. Edit: When the 2nd text is significantly shorter than the 1st text, could I do (using the 2016/2014 exams as an example):
Para 1. 1st text + 1 link to visual
Para 2. 1st text + 1 link to visual
Para 3. 1st text + 1 link to visual
Para 4. 2nd text + 1 link to visual

So could I make my rule of thumb -
if the 2nd text is a decent length, like in 2015, allocate the last 2 paragraphs to it. But if the 2nd text is significantly shorter, like in 2015, allocate only the last paragraph to it? (Again with the concern about uneven paragraph lengths - my last paragraph would most likely be very long. But if I split that paragraph into 2, they would both be very short. Sigh...)

Sorry, I know there is a lot of detail in these questions, but I am really determined to do well on the exam! Thank you so much for your help :D
Title: Re: VCE English Question Thread
Post by: bluebubbles on September 02, 2017, 11:35:34 am
Hi there,
I am struggling to fit my text response and comparative essays in 1 hour, so I am wondering if you think it is a 'turn off' (so-to-speak) when an examiner sees an essay with 2 body paragraphs compared to an essay with 3 body paragraphs? Because I am finding it easier to write 2 longer body paragraphs that include more depth, than 3 bodies.
Thanks
Title: Re: VCE English Question Thread
Post by: Sine on September 02, 2017, 05:52:47 pm
Hi there,
I am struggling to fit my text response and comparative essays in 1 hour, so I am wondering if you think it is a 'turn off' (so-to-speak) when an examiner sees an essay with 2 body paragraphs compared to an essay with 3 body paragraphs? Because I am finding it easier to write 2 longer body paragraphs that include more depth, than 3 bodies.
Thanks
Do you think you could find a way to split a body paragraph up? Technically the examiner shouldn't have a problem but unconsciously I feel it may impact your score.

How long are your paragraphs? Because I was always told to never have them too long :)

I think most people would say 4 body paragraphs is ideal and the min I would go to is 3 long ones.
Title: Re: VCE English Question Thread
Post by: lovelyperson on September 04, 2017, 11:36:33 am
Question about the oral: would it be too weird or out of place to use past historical events to support a argument about a contemporary issue?
Title: Re: VCE English Question Thread
Post by: Sine on September 04, 2017, 03:48:44 pm
Question about the oral: would it be too weird or out of place to use past historical events to support a argument about a contemporary issue?
depends the specific case you are talking about.  :)

I remember I used very old historic events for my oral presentation.
Title: Re: VCE English Question Thread
Post by: HopefulLawStudent on September 04, 2017, 06:40:23 pm
Randomly...
Would it be right to call 'Animal Farm' a lampoon against socialism?

(Lampoon... that awkward moment when you literally only know a word because of studying the 'Mabo' documentary... :D
#Thanks2blackfellafilms)

Considering lampoon means:

Quote
lampoon /lamˈpuːn/
verb
1. publicly criticize (someone or something) by using ridicule, irony, or sarcasm.
"the actor was lampooned by the press"

2. a speech or text lampooning someone or something.
"the magazine fired at God, Royalty, and politicians, using cartoons and lampoons"

I think it could work.

My recommendation is somewhere between 4-6 sentences. It depends on how long your sentences are! Generally the guideline is: 1 or 2 to intro the text, 1 for contention, 1 or 2 to intro body paragraphs. Whatever you write, your intro must be shorter than your shortest body paragraph.

~Shirlaine from TSEtuition

TBH I'd argue that 6 sentences especially is potentially too long. I was always taught 3 - 4 sentences but with that said, everyone has a different preference and it does ultimately depend on how long your sentences are. :)

Hi there,
I am struggling to fit my text response and comparative essays in 1 hour, so I am wondering if you think it is a 'turn off' (so-to-speak) when an examiner sees an essay with 2 body paragraphs compared to an essay with 3 body paragraphs? Because I am finding it easier to write 2 longer body paragraphs that include more depth, than 3 bodies.
Thanks

If they're two hella awesome body paragraphs you might get away with it but the problem with 2 (and why we often recommend 3) is because it's hard to demonstrate depth of knowledge and argument in two body paragraphs. Instead of going with only 2 BPs, try and consider why you're struggling to fit your essay into an hour and work on addressing those issues because 3 in-depth BPs > 2 in-depth BPs any day.

Question about the oral: would it be too weird or out of place to use past historical events to support a argument about a contemporary issue?

All about how you use it. There is no hard and fast rule about that sort of stuff.

Hi Lauren :) I have a few questions regarding the structure of a language analysis essay for the exam when there are 2 or more texts. I have been trying to paragraph by argument, and within the same paragraph, talk about different texts (i.e. 1st article --> linking word --> 2nd article), but I find it very difficult to make quick decisions on how to split my paragraphs when often, the arguments in each written text don't link up nicely with each other.

1. Could I still get 10/10 if I decide to make this my go-to structure:
Separate each text into main arguments/'themes'. Then find 2-3 links for each image in different sections of the essay - i.e. talk about the image in the same paragraphs as the written material.
BUT go through the written texts chronologically - eg: talk about 1st text (blocking that text off by argument) and find 2-3 links to image, then talk about 2nd text (blocking that text off by argument) and then find 2-3 links to image.

This would personally be a lot less stressful for me, because in the past I have sat there for a long time just figuring out how to link the 2 texts into the same paragraph! Would this structure always work?

2. However, if I use this method, how would I compare the two texts? Should my rule of thumb be that the 1st sentence of each paragraph of my 2nd text should be a comparative sentence? Also, is comparison part of the marking criteria? Is a simple linking word such as 'similarly' or 'on the other hand' sufficient, or should my comparative sentence at the start of the paragraphs for my 2nd text be comparing the argument? Or should I be aiming to make comparative comments throughout my whole piece?

3. Also, my concern with this method is that my piece will be imbalanced because the 2nd text is often shorter than the 1st text, and 1 paragraph wouldn't be enough to talk about the 2nd text.

Thus, at the moment, i think this would be my most likely structure, for 2 texts and 2 visuals:
1. Text 1 + visual 1
2. Text 1 + visual 1
3. Text 2 + visual 2
4. Text 2 + visual 2

This would mean that my 3rd and 4th paragraphs would be SIGNIFICANTLY shorter than my 1st and 2nd. Is this structure okay, or could you think of a better way to split paragraphs?

4. For example, is this a good structure for the 2015 LA (on bigsplash):
1. Para Text 1 - praise for bigsplash company --> shift to extolling Australian volunteers and arguing that they enrich and are essential to society.
+ 2 references to the 1st visual linked with text analysis

Para 2. Text 1 - generating guilt in Australians by arguing that we have undervalued and underappreciated volunteers
+ 1 reference to the 1st visual linked with text analysis

Para 3. Text 2 - Mathew Nguyen building praise for his organisation Tradespeople Without Borders and casting himself as a humble, approachable individual, thereby humanising himself in Australian's eyes and gaining the reader's trust.
+ 2 references to the 2nd visual linked with text analysis
(1. Presenting himself as hospitable and casual --> casual clothing of the people joining hands. Lol don't know if this is okay to talk about?
2. By emphasising his desire to 'make a difference' and 'help Australia's reputation as a caring country', Nguyen appeals to the patriotic instincts of his audience, and strives to generate a sense of solidarity and unity amongst Australians who have a fundamental sense of responsibility to help those around him. This notion of cohesion and harmony is given visual reinforcement by the accompanying image, which depicts....)

Para 4. Text 2 - Nguyen humbly proclaiming that though recognition and praise of volunteers is welcomed, it is not necessary and the true reward of volunteering is the delight of helping others.

(My 3rd and 4th paragraphs would be like half the length of my 1st and 2nd paragraphs though, which is what I am worried about!)

5. Would you ever make links to the visual when talking about a text that the visual is NOT accompanied with? Is this common/recommended? Eg: The 2nd visual of the 2015 piece feeding into Bennett's proclamation that humanitarian work 'stitches together the social fabric of our nation'

6. Edit: When the 2nd text is significantly shorter than the 1st text, could I do (using the 2016/2014 exams as an example):
Para 1. 1st text + 1 link to visual
Para 2. 1st text + 1 link to visual
Para 3. 1st text + 1 link to visual
Para 4. 2nd text + 1 link to visual

So could I make my rule of thumb -
if the 2nd text is a decent length, like in 2015, allocate the last 2 paragraphs to it. But if the 2nd text is significantly shorter, like in 2015, allocate only the last paragraph to it? (Again with the concern about uneven paragraph lengths - my last paragraph would most likely be very long. But if I split that paragraph into 2, they would both be very short. Sigh...)

Sorry, I know there is a lot of detail in these questions, but I am really determined to do well on the exam! Thank you so much for your help :D

Hey! So I'm not Lauren obv but thought I'd chime in.

Tbh the breakdown that you had for the 2015 piece looks solid though I feel like your first BP argument for Bennett's piece has too many ideas. And I like your theoretical breakdown for where it's not an even split. Don't let yourself get too bogged down in comparison. Keep in mind that comparison is not technically a criteria for Analysing Argument (from my understanding, anyway); we generally just do it so as to facilitate a smoother transition between the pieces. :)
Title: Re: VCE English Question Thread
Post by: S200 on September 04, 2017, 11:49:18 pm
Thanks Hopeful/Actual.LawStudent... :D
Title: Re: VCE English Question Thread
Post by: pha0015 on September 05, 2017, 05:36:36 pm
If in the exam you don't get to finish the conclusion, how many marks will be deducted, and what might your study score be pushed down to, if the rest of the piece was good enough to get around a 40 study score?
Title: Re: VCE English Question Thread
Post by: KANYEWEST on September 05, 2017, 06:42:20 pm
hey y'all,

has anyone here written a comparative essay for Invictus/ Ransom? i have my SAC tomorrow and would love to read what people have written!! it would be very appriciated !

thank you in advance x
Title: Re: VCE English Question Thread
Post by: Willba99 on September 05, 2017, 07:21:43 pm
hey y'all,

has anyone here written a comparative essay for Invictus/ Ransom? i have my SAC tomorrow and would love to read what people have written!! it would be very appriciated !

thank you in advance x

Have look in the "VCE English Work Submitted for Marking" section. I noticed three or four!
Title: Re: VCE English Question Thread
Post by: KANYEWEST on September 05, 2017, 07:53:22 pm
Have look in the "VCE English Work Submitted for Marking" section. I noticed three or four!

awesome, ill check it out!
would you able to link it onto this, im not too sure where it is!
Title: Re: VCE English Question Thread
Post by: patriciarose on September 05, 2017, 07:58:49 pm
https://atarnotes.com/forum/index.php?board=406.0 that's the board

https://atarnotes.com/forum/index.php?topic=173111.0 and this is the first essay for ransom/invictus that i saw (:
Title: Re: VCE English Question Thread
Post by: Willba99 on September 05, 2017, 07:59:20 pm
awesome, ill check it out!
would you able to link it onto this, im not too sure where it is!

to be honest I'm not sure how to! its one of the child boards under VCE english
Title: Re: VCE English Question Thread
Post by: Willba99 on September 05, 2017, 07:59:50 pm
https://atarnotes.com/forum/index.php?board=406.0 that's the board

https://atarnotes.com/forum/index.php?topic=173111.0 and this is the first essay for ransom/invictus that i saw (:

perfect timing lol
Title: Re: VCE English Question Thread
Post by: KANYEWEST on September 05, 2017, 08:01:35 pm
thank you so much to both of you guys!!!!
appreciate it alot!!
Title: Re: VCE English Question Thread
Post by: TheCommando on September 06, 2017, 12:11:04 am
Sooooooo bloody good

So just keep tryingg to gather as much information from a scentence or two from any text which improves ur thinking
Also im not 100% what being able to think implies, is it your ability to gatther ideas and interpret passagges from the text
Just reasking to hopefully get it answered
Title: Re: VCE English Question Thread
Post by: HopefulLawStudent on September 06, 2017, 02:54:05 pm
If in the exam you don't get to finish the conclusion, how many marks will be deducted, and what might your study score be pushed down to, if the rest of the piece was good enough to get around a 40 study score?

Your essay is marked holistically. We like conclusions because they make for a nice well-rounded complete essay but it's not like vcaa are sitting there ticking boxes and you have to get all those boxes ticked to get a certain mark. In marking, they also keep in mind that you were under timed constraints. Aim to have a finished conclusion though if possible by way of practicing time management because an unfinished essay is not ideal.

hey y'all,

has anyone here written a comparative essay for Invictus/ Ransom? i have my SAC tomorrow and would love to read what people have written!! it would be very appriciated !

thank you in advance x

Hope the SAC went well!

Thank you everyone who helped out by linking KANYEWEST to some essays. We are, after all, a community and the more everyone puts in, the more we get out of this great community/resource/whatever so kudos to you all.

Just reasking to hopefully get it answered

What was the question? If it is to do with what it means to think, unfortunately, I'm going to have to pass on that question and leave it to someone more knowledgeable. :)
Title: Re: VCE English Question Thread
Post by: KANYEWEST on September 06, 2017, 07:18:28 pm
Your essay is marked holistically. We like conclusions because they make for a nice well-rounded complete essay but it's not like vcaa are sitting there ticking boxes and you have to get all those boxes ticked to get a certain mark. In marking, they also keep in mind that you were under timed constraints. Aim to have a finished conclusion though if possible by way of practicing time management because an unfinished essay is not ideal.

Hope the SAC went well!

Thank you everyone who helped out by linking KANYEWEST to some essays. We are, after all, a community and the more everyone puts in, the more we get out of this great community/resource/whatever so kudos to you all.

What was the question? If it is to do with what it means to think, unfortunately, I'm going to have to pass on that question and leave it to someone more knowledgeable. :)

Thank you, and yes, my SAC went well, the prompt was challenging but once it was broken into simpler words it was very easy to discuss, so i am feeling somewhat confident for what i had written!
Title: Re: VCE English Question Thread
Post by: Willba99 on September 07, 2017, 06:19:44 pm
Is anyone else doing their comparative SAC on Into the Wild and Tracks? If so, do you want to swap notes/practice essays? My English class is pretty small so it'd be nice to get some new viewpoints!
Title: Re: VCE English Question Thread
Post by: simrat99 on September 09, 2017, 08:26:56 pm
Hi,
For the prompt "The Longest Memory and Black Diggers show us that there are short term and long term ramifications of racism. Discuss", would you advise doing one paragraph on short term ramifications of racism in both texts and then one on the long term ramifications?
Thank you
Title: Re: VCE English Question Thread
Post by: Willba99 on September 10, 2017, 09:11:03 am
Hi,
For the prompt "The Longest Memory and Black Diggers show us that there are short term and long term ramifications of racism. Discuss", would you advise doing one paragraph on short term ramifications of racism in both texts and then one on the long term ramifications?
Thank you

I haven't read the texts, but as a rule of thumb I'd suggest having at least 3 body paragraphs. I don't think its actually stated in the study design, but I think if you're only doing two body paragraphs then you're at risk of not demonstrating a breadth of ideas!
Title: Re: VCE English Question Thread
Post by: captkirk on September 19, 2017, 05:09:15 pm
Hey everyone,

For the Year of wonders and The crucible what is the authors trying to say about resistance and death.

Any help is appreciated :)
Title: Re: VCE English Question Thread
Post by: lilyrosee on September 20, 2017, 10:14:03 pm
Hi,

could someone please fix up my topic sentence for a comparative essay on Ransom/Invictus

'Leadership that is supported in society is admirable and commended for sacrifices and selflessness'.

It sounds a bit odd and I do not know how to reword it
Title: Re: VCE English Question Thread
Post by: sonnyangel on September 20, 2017, 11:10:14 pm
Hi!
I'm not doing Ransom/Invictus but you could try:
Commended for the selflessness involved and the sacrifices one must make, endorsed leadership is viewed as admirable and meritorious (by the society).
Hope that helps!
Title: Re: VCE English Question Thread
Post by: lilyrosee on September 21, 2017, 06:02:48 am
Hi!
I'm not doing Ransom/Invictus but you could try:
Commended for the selflessness involved and the sacrifices one must make, endorsed leadership is viewed as admirable and meritorious (by the society).
Hope that helps!

Thanks :)
Title: Re: VCE English Question Thread
Post by: waterangel82 on September 21, 2017, 08:20:20 pm
Hi,

I'm doing Tracks and Into the Wild for comparative this year. However, I am quite concerned with answering essay topics, particularly after my english teacher noted in the comments after my comparative SAC that I 'needed to respond the the topic more fully'. I haven't gotten the sac back yet.

The topic that I chose for my essay was on identity, and it was something like this (but can't remember the two quotes exactly):

'Compare how both Tracks and Into the Wild explore the construction of identity'

I don't know what I did wrong, but I remember my ideas being something like this:

1.) Both texts suggests that the need to abandon societal conventions and expectations and seek an existence in the natural world is necessary for the development of one's true identity as they able to seek a freedom unrestrained by other's expectations. Here I talked about how Robyn abandons the identity being imposed onto her by Western society, escaping materialistic world and finding a sense of belonging in the natural world. I said pretty much the same thing for Chris.

2.) Both texts suggest that identity is more than simply belonging, but developing a sense of identity also comes from recognising one's weaknesses. Here I talked about the camel and moose scene - and how especially for Chris, his inability to kill the moose results in him to realise that his true identity within the natural world is not what he believes it to be.

3.) However, most importantly, both texts suggests that a sense of identity is mainly constructed from human relationships. Here I talked about importance of human relationships. Especially for Robyn, I talked about how her relationship with Mr Eddie allows her to truly find a sense of belonging in the Australian landscape and form an identity where 'the self becomes more like the desert.'

I know my ideas really aren't great, but the head of English at my school said that I needed to talk about things like Chris Mccandless creating his own alter ego of Alexander Supertramp, etc. Overall though, my teacher said that I have a 'sound understanding of the texts and an ability to analyse and how they convey ideas, issues and themes.' He also said that 'You made some relevant connections, but could have tackled the topic more consistently and directly throughout the response.'

I'm really confused right now :(.  Can someone (like literally lauren) or anyone doing Tracks and Into the Wild please help me? I don't want to fall into the same trap during the actual exam :(.
Title: Re: VCE English Question Thread
Post by: Perryman on September 25, 2017, 11:05:59 am
Hello,

What are the most important things to do before an exam? I have a practise exam tomorrow and have only finished the course work this past Friday. Is it best just to do practise essays/plans and re-read notes of issues/themes about the books? Any help will be great!

Thanks,
Title: Re: VCE English Question Thread
Post by: clarke54321 on September 25, 2017, 11:39:00 am
Hello,

What are the most important things to do before an exam? I have a practise exam tomorrow and have only finished the course work this past Friday. Is it best just to do practise essays/plans and re-read notes of issues/themes about the books? Any help will be great!

Thanks,


Hey!

Given that your practice exam is tomorrow, I'd recommend that you read through past essays and SACs. This should help refresh your knowledge on important quotes, ideas and interpretations, which you've formulated throughout the year. Perhaps also take note of the areas that could do with some improvement. I wouldn't worry too much about not finishing complete, polished essays tomorrow. There's still quite a bit of time to get used to constraint! :)

What you do after the practice exam is completely up to you. English is very much a personal subject, in terms of how you can prepare. But if you're looking for recommendations, here's what I'd suggest.

-Instead of writing a copious number of essays, re-write the essays that you've completed throughout the year. You would have already formed the basis of your interpretation and can just fix up expression, relevance to prompt, clarity of ideas, etc.

-Time yourself every now and then, but ensure that you first have the quality down pat. There is no point writing essay after essay if you are making the same mistake and aren't getting enough depth in your writing.

-Like you say, it's good to have a re-read over notes and themes. Something additional to this is selecting small passages from your texts and then annotating them in detail. Annotating is a really great skill, which allows you to recognise the small nuances of the text and pick up on connections, which you previously did not see.

-Contribute to the AA Club on AN, where a new article is added each fortnight. Here, others can mark your work and you too will have the opportunity to mark the work of others. Marking is a fantastic way to better your own skills, as it forces you to identify what is strong in a piece, and what can be improved.

Hopefully these ideas give you a starting point for revision. All the best :D


Title: Re: VCE English Question Thread
Post by: Perryman on September 25, 2017, 12:33:23 pm
Hey!

Given that your practice exam is tomorrow, I'd recommend that you read through past essays and SACs. This should help refresh your knowledge on important quotes, ideas and interpretations, which you've formulated throughout the year. Perhaps also take note of the areas that could do with some improvement. I wouldn't worry too much about not finishing complete, polished essays tomorrow. There's still quite a bit of time to get used to constraint! :)

What you do after the practice exam is completely up to you. English is very much a personal subject, in terms of how you can prepare. But if you're looking for recommendations, here's what I'd suggest.

-Instead of writing a copious number of essays, re-write the essays that you've completed throughout the year. You would have already formed the basis of your interpretation and can just fix up expression, relevance to prompt, clarity of ideas, etc.

-Time yourself every now and then, but ensure that you first have the quality down pat. There is no point writing essay after essay if you are making the same mistake and aren't getting enough depth in your writing.

-Like you say, it's good to have a re-read over notes and themes. Something additional to this is selecting small passages from your texts and then annotating them in detail. Annotating is a really great skill, which allows you to recognise the small nuances of the text and pick up on connections, which you previously did not see.

-Contribute to the AA Club on AN, where a new article is added each fortnight. Here, others can mark your work and you too will have the opportunity to mark the work of others. Marking is a fantastic way to better your own skills, as it forces you to identify what is strong in a piece, and what can be improved.

Hopefully these ideas give you a starting point for revision. All the best :D



Thanks heaps for the advice, when you say, "I wouldn't worry too much about not finishing complete, polished essays tomorrow.", do you mean it is better to have unfinished good quality essays in the actual practise exam since it is just a practice one??
Thanks again,
Title: Re: VCE English Question Thread
Post by: clarke54321 on September 25, 2017, 02:33:35 pm
Thanks heaps for the advice, when you say, "I wouldn't worry too much about not finishing complete, polished essays tomorrow.", do you mean it is better to have unfinished good quality essays in the actual practise exam since it is just a practice one??
Thanks again,

No worries! Yeah, basically don't bother yourself by stressing too much. It is just a practice, which will indicate areas of strength and weakness. By all means try and finish the essays, but if you can't, you've still got plenty of time to work on this :)
Title: Re: VCE English Question Thread
Post by: princessofpersia on October 04, 2017, 12:30:05 pm
hey guys, just wanted to ask a quick question.

With questions such as this 'Both Victor and the Creature are motivated by revenge. Discuss.' how do you relate back to the author and not make it a  character- based essay?
Title: Re: VCE English Question Thread
Post by: boooom on October 04, 2017, 01:20:30 pm
hey guys, just wanted to ask a quick question.

With questions such as this 'Both Victor and the Creature are motivated by revenge. Discuss.' how do you relate back to the author and not make it a  character- based essay?

Think about what the author is saying through your answer to the essay question. For instance, if they are both motivated by revenge, is Shelley saying that it is good or bad thing? To what degree and why so?
Title: Re: VCE English Question Thread
Post by: princessofpersia on October 04, 2017, 02:48:48 pm
Think about what the author is saying through your answer to the essay question. For instance, if they are both motivated by revenge, is Shelley saying that it is good or bad thing? To what degree and why so?


ohh thanks
Title: Re: VCE English Question Thread
Post by: Sigma on October 06, 2017, 10:17:51 am
Help!!! Year 11 English Unit 2, Area of Study 2: Presenting Argument

Need urgent help.

Topic:  Homelessness in Melbourne is a complex social issue that needs addressing.

Could someone give me a few arguments and dot points, contention and also the structure?

Thanks. Sigma.


Title: Re: VCE English Question Thread
Post by: K888 on October 06, 2017, 01:40:59 pm
Help!!! Year 11 English Unit 2, Area of Study 2: Presenting Argument

Need urgent help.

Topic:  Homelessness in Melbourne is a complex social issue that needs addressing.

Could someone give me a few arguments and dot points, contention and also the structure?

Thanks. Sigma.
What is it about the topic specifically that's got you stuck?
I think the thing with presenting an argument is that for it to be properly effective, it needs to be your own arguments! After all, how can you argue about a topic properly if you don't engage with what you're meant to be arguing about? :)

It might help to break it down:
- Why do you think homelessness is an issue that needs to be addressed? What are the benefits for addressing it?
- Why is it a complex social issue? -> maybe from here, you can look at the risk factors and things that lead to homelessness, barriers that prevent people from changing, you could also look at perceptions of homeless people, what we as a society do to help homeless people, etc.

Your first port of call should be doing some research. Just do a quick google, find out a bit more about the issue as a whole so you understand it better - maybe read some news articles, etc. then you can start forming some arguments :)
I'd also recommend watching ABC's "You Can't Ask That" episode where they interview people who are homeless or have been homeless in the past. You can watch it on iview here, and it'll take less than 30 minutes! :)

I hope that this gives you a few ideas and somewhere to start. All the best!
Title: Re: VCE English Question Thread
Post by: simrat99 on October 06, 2017, 02:58:43 pm
Hey guys,
In for Section C, if there are many comments accompanied by the main piece, do we have to analyse every single one of them? Are we allowed to leave one or two out?
Thanks :)
Title: Re: VCE English Question Thread
Post by: K888 on October 06, 2017, 04:20:48 pm
Hey guys,
In for Section C, if there are many comments accompanied by the main piece, do we have to analyse every single one of them? Are we allowed to leave one or two out?
Thanks :)
You should analyse everything - obviously, if they're just small comments, you don't need to do a large analysis, but you need to show the examiner that you've read and considered everything :) If you don't mention every comment, then your analysis won't be as comprehensive, and you won't have shown that you've considered all the sides to the story (if that's what you want to call it), so you'll lose marks.

Hope this helped! :)
Title: Re: VCE English Question Thread
Post by: Sine on October 06, 2017, 07:01:18 pm
Hey guys,
In for Section C, if there are many comments accompanied by the main piece, do we h ave to analyse every single one of them? Are we allowed to leave one or two out?
Thanks :)
if there are comments I would best try to analyse them on it's own (e.g. a seperate paragraph) and then try to show and differences or similarities between them and the main piece of writing.
Title: Re: VCE English Question Thread
Post by: simrat99 on October 06, 2017, 07:15:29 pm
You should analyse everything - obviously, if they're just small comments, you don't need to do a large analysis, but you need to show the examiner that you've read and considered everything :) If you don't mention every comment, then your analysis won't be as comprehensive, and you won't have shown that you've considered all the sides to the story (if that's what you want to call it), so you'll lose marks.

Hope this helped! :)
if there are comments I would best try to analyse them on it's own (e.g. a seperate paragraph) and then try to show and differences or similarities between them and the main piece of writing.
Thanks a lot!
Title: Re: VCE English Question Thread
Post by: pmmenotes on October 15, 2017, 05:04:41 pm
Hey guys so i've written a couple really good (according to teachers/tutor lol)  english essays for Medea and i want to memorise one of them that im hoping to regurgitate in the exam if the essay question is similar enough (revenge/justice etc). Anyways my question is what are you guys doing? Is anyone else memorising there pieces?
Title: Re: VCE English Question Thread
Post by: LifeisaConstantStruggle on October 15, 2017, 05:16:53 pm
Hey guys so i've written a couple really good (according to teachers/tutor lol)  english essays for Medea and i want to memorise one of them that im hoping to regurgitate in the exam if the essay question is similar enough (revenge/justice etc). Anyways my question is what are you guys doing? Is anyone else memorising there pieces?

BRUH that's brave, considering the variation of prompts on Medea, some questions might look similar, but can be very different if you dissect it, and tbh regurgitating the same essay just because there's some similar parts wouldn't be ideal. I have a friend who did exactly that in his practice exam (memorised comparative and Medea) and got an 8 (comparative)/4 (Medea)/8 (AA) because even though the topics are fairly general (passion-reason), answering the question should be more important. (if you struggle with spontaneously constructing sentences then yeah you can memorise, but if you don't modify your memorised sentences in the exam it's gonna work against you, just don't overdo it).

I do regular plans on Medea, and save it all in a word document, so I'd refer to it regularly, and it actually helps me form complex arguments and think of heaps of examples quickly, so yeah.
Title: Re: VCE English Question Thread
Post by: clarke54321 on October 15, 2017, 06:04:34 pm
Hey guys so i've written a couple really good (according to teachers/tutor lol)  english essays for Medea and i want to memorise one of them that im hoping to regurgitate in the exam if the essay question is similar enough (revenge/justice etc). Anyways my question is what are you guys doing? Is anyone else memorising there pieces?

I agree with what LifeisaConstantStruggle said. Know how to manipulate memorised essays. In all honesty, one can't go into the exam without having ideas/quotes/arguments memorised to some degree. The best essays just know how to break down and fit this pre-planned knowledge to the relevant prompt.
Title: Re: VCE English Question Thread
Post by: halo on October 16, 2017, 07:36:51 pm
Hi everyone. So recently, halfway through writing an essay (just finished body para 1 and starting on second one) I got "stuck" because I realised that 2 of my ideas were quite similar, and that I'd essentially be essentially repeating myself in body para 2 and 3. My instincts were to plan again and rewrite my essay (or at least my intro), but I probably won't have the time to do that in the exam, so I was wondering does anyone have any advice on what I should do in such a situation? Should I just keep writing, leaving my essay at two body paras, or try to desperately differentiate my remaining ideas?

Also, I guess this shows that I absolutely suck at planning. So does anyone have a foolproof method of planning they're willing to share? I especially want to know how to make sure you come up with 3 DIFFERENT ideas that are relevant to the prompt (and aren't topic dodges or anything).

Thank you so much :3
Title: Re: VCE English Question Thread
Post by: chantelle.salisbury on October 17, 2017, 08:43:46 pm
i was wondering how to address the theme of justice in the novel 'The Lieutenant'?
a sample topic is 16.   How is the importance of justice addressed in the novel ‘The Lieutenant’?
how would i set out my paragraphs and can you include relationships and how people treat each other, or is this straying too far from the topic?

thankyou
Title: Re: VCE English Question Thread
Post by: dec.hargreaves on October 19, 2017, 09:59:44 am
This is probably a long shot, but does anyone know how to address a 'how' question in section B. i.e. compare the portrayals of family in Tracks and Into the Wild. i can get an alright score for it (60-65%) but wouldn't mind bumping that up a bit. thanks in advance
Title: Re: VCE English Question Thread
Post by: clarke54321 on October 19, 2017, 10:18:04 am
This is probably a long shot, but does anyone know how to address a 'how' question in section B. i.e. compare the portrayals of family in Tracks and Into the Wild. i can get an alright score for it (60-65%) but wouldn't mind bumping that up a bit. thanks in advance

Hey!

Well a "how" question is essentially focused on the texts' construction. This can range from film shot, narrative structure, relationship types, dialogue, post scripts and a whole lot more. Every essay, regardless of the question type (ie. to what extent, do you agree) should all incorporate ideas on how the authors/directors have constructed elements to emphasise certain ideas/messages.

So for Tracks and Into the Wild- "Compare the portrayals of family."
-How do the texts portray family? The rigid definition of a biological family is certainly challenged throughout the texts (more so in Into the Wild)
-Is a family necessary? If yes, how is this expressed? If no, how is this expressed?
-Are families toxic or needed for constant guidance? Again, directors/authors choose varying structures/dialogue/film shots/images (in Tracks) to articulate this.

Hopefully this gives you a starting point  :)
Title: Re: VCE English Question Thread
Post by: lovelyperson on October 19, 2017, 10:58:42 am
i was wondering how to address the theme of justice in the novel 'The Lieutenant'?
a sample topic is 16.   How is the importance of justice addressed in the novel ‘The Lieutenant’?
how would i set out my paragraphs and can you include relationships and how people treat each other, or is this straying too far from the topic?

thankyou

Haven't done The Lieutenant in agesss, but you could talk about Rooke's relationship with Targaran and how it affects his sense of justice.

Other things about justice you could consider are:
- Does the self-serving nature of the colony prevent them from being just and fair towards the Indigenous population?
- The public whipping of the convicts who stole the food; why did they have to punished and what is the signifance of ???'s (forgot his name, starts with 'w' I think?) response to this form of justice?
- How does this compare to Rooke's punishment?

This is probably a long shot, but does anyone know how to address a 'how' question in section B. i.e. compare the portrayals of family in Tracks and Into the Wild. i can get an alright score for it (60-65%) but wouldn't mind bumping that up a bit. thanks in advance

Generally in English, prompt words like those don't matter because you should be answering to all of them in your essay regardless. But with regards to your prompt, it's essentially asking: "what are the texts saying about family? Good, bad and to what extent?"
Title: Re: VCE English Question Thread
Post by: S200 on October 19, 2017, 02:42:28 pm
Could some grammatical person help here plz??

Is polemical refusal better english than polemic refusal?
Title: Re: VCE English Question Thread
Post by: zhen on October 19, 2017, 02:49:26 pm
Recently I lost a ton of marks for saying stuff like the writer compels the reader to feel a certain way instead of the writer attempts to compel the reader to feel a certain way. Does this really matter, cause adding the words attempts to or endeavours to really doesn’t add anything to the content of a piece. I know that we don’t really know how the reader will react so we do this and maybe I should just do it just in case, but should something that adds virtually nothing to the content of an essay really contribute to a person’s mark? What do you guys think about it?
Title: Re: VCE English Question Thread
Post by: dec.hargreaves on October 19, 2017, 02:52:03 pm
Hey!

Well a "how" question is essentially focused on the texts' construction. This can range from film shot, narrative structure, relationship types, dialogue, post scripts and a whole lot more. Every essay, regardless of the question type (ie. to what extent, do you agree) should all incorporate ideas on how the authors/directors have constructed elements to emphasise certain ideas/messages.

So for Tracks and Into the Wild- "Compare the portrayals of family."
-How do the texts portray family? The rigid definition of a biological family is certainly challenged throughout the texts (more so in Into the Wild)
-Is a family necessary? If yes, how is this expressed? If no, how is this expressed?
-Are families toxic or needed for constant guidance? Again, directors/authors choose varying structures/dialogue/film shots/images (in Tracks) to articulate this.

Hopefully this gives you a starting point  :)

thanks will give that a shot
Title: Re: VCE English Question Thread
Post by: clarke54321 on October 19, 2017, 03:09:34 pm
Recently I lost a ton of marks for saying stuff like the writer compels the reader to feel a certain way instead of the writer attempts to compel the reader to feel a certain way. Does this really matter, cause adding the words attempts to or endeavours to really doesn’t add anything to the content of a piece. I know that we don’t really know how the reader will react so we do this and maybe I should just do it just in case, but should something that adds virtually nothing to the content of an essay really contribute to a person’s mark? What do you guys think about it?

I've also been criticized for this. Sometimes there's such a subtle difference that it seems like a petty correction. But I suppose the addition of endeavours/aims/strives/seeks and all of those verb types, places a greater emphasis on authorial intent- the primary purpose of the task.

Here is an excerpt from the 2015 VCAA examiners report: Note that the task focuses on the intent („attempt to persuade‟), not on the result.

So to be safe, I'd stick with the addition of these phrases  :)
Title: Re: VCE English Question Thread
Post by: lovelyperson on October 19, 2017, 03:10:42 pm
Recently I lost a ton of marks for saying stuff like the writer compels the reader to feel a certain way instead of the writer attempts to compel the reader to feel a certain way. Does this really matter, cause adding the words attempts to or endeavours to really doesn’t add anything to the content of a piece. I know that we don’t really know how the reader will react so we do this and maybe I should just do it just in case, but should something that adds virtually nothing to the content of an essay really contribute to a person’s mark? What do you guys think about it?

Shouldn't matter that much in the grand scheme of things, but it's erroneous to say that a "reader is compelled" to do something when you can't be entirely sure of it. Plus, it's a small issue by itself but if you do it throughout it becomes a jarring problem that will likely lose you marks.

And it shouldn't be an issue in the first place as you're analysing the material as if it's successful with its persuasive techniques. So you don't have to continually say "attempts" or "endeavours" - just phrase it another way like " author depicts X as Y" or "suggests".
Title: Re: VCE English Question Thread
Post by: Cassidyhogi on October 19, 2017, 08:39:18 pm
Hello
With language analysis I have been finding it difficult to only write as much as I need to.
I find myself writing six pages of analysis, and I don't even analyse every single technique I see.
I know there is no perfect answer, but how many techniques do you guys usually mention within one paragraph (one paragraph being devoted to one argument)?
Or any advice on cutting down the length of my essays would be nice  ;D
Thank youuuuu
Title: Re: VCE English Question Thread
Post by: LifeisaConstantStruggle on October 19, 2017, 09:02:09 pm
Hello
With language analysis I have been finding it difficult to only write as much as I need to.
I find myself writing six pages of analysis, and I don't even analyse every single technique I see.
I know there is no perfect answer, but how many techniques do you guys usually mention within one paragraph (one paragraph being devoted to one argument)?
Or any advice on cutting down the length of my essays would be nice  ;D
Thank youuuuu

It very much depends on your essays, how many key arguments, pieces, key players etc etc.
For a typical 3 key argument single piece essay I only include 3-4 rhetorical devices, but analyse on the what how why to its detailed bits. At times one example could have a few techniques being used, and I tend to blend them together.
The most important thing to do here (timed conditions) is to be selective, and choose the real obvious ones (the ones that ACTUALLY contribute to the argument being presented, eg: stuff like inclusive language can be omitted if it's not really important/already used, or any form of complex techniques and connotative analyses that don't contribute much to the persuasiveness of an argument can generally be ignored).
I used to write more than 6 per argument, but I find that by practising to look for the important PLTs I can cut down on a lot of unnecessary analyses and get to the point.
Again, it depends on one's style, and I can't vouch that my approach is the best approach.
Title: Re: VCE English Question Thread
Post by: gnaf on October 20, 2017, 12:12:30 pm
Are there any quotes for how Hecuba and her son are resistant to change?
Title: Re: VCE English Question Thread
Post by: halo on October 20, 2017, 12:57:39 pm
Are there any quotes for how Hecuba and her son are resistant to change?

"But to Hecuba the image is a shocking one – she is more tied to convention than she believes – and as Priam warms to his subject she grows more and more disturbed. " (after Priam tells her about his vision) pg55
"But the danger of what he is determined on fills her with alarm. She will need all her wiles, all her powers of firm but calm persuasion,to lead him back from it."pg58
"Chance? She looks up quickly. Surely she has misheard... She wishes she had misheard." pg61 (embracing the idea of chance would bring change, and she is reluctant to accept it)
Title: Re: VCE English Question Thread
Post by: atar.notes.user on October 21, 2017, 11:14:39 pm
how long should i spend in each section ideally?
also, whats the average word limit for text response? (im nervous bc i write <1000 usually under timed conditions)
Title: Re: VCE English Question Thread
Post by: ellellen on October 22, 2017, 12:00:30 am
how long should i spend in each section ideally?
also, whats the average word limit for text response? (im nervous bc i write <1000 usually under timed conditions)

You should try and spend one hour on each section. Start with Language Analysis, because you can use the reading time to read the article and have it fresh in your mind when you start writing, rather than having to go back and reread it all again much later in the exam, wasting time. Then do sections A and B in whatever order you prefer.

For each section, you should probably write 800-1000 words. Try not to go over 1000 words. If you write too much, you won't finish in time, but if you write under 800 you won't be able to fully flesh out your ideas. Just try and practice writing text responses in timed conditions and you'll get better and better at it!
Title: Re: VCE English Question Thread
Post by: simrat99 on October 22, 2017, 10:27:10 am
Hi,
For language analysis articles with more than one written text, is it acceptable to talk about each text in different paragraphs? Because when I talk about two or more in the same paragraph, often my paragraphs get too long  :-\
Title: Re: VCE English Question Thread
Post by: halo on October 22, 2017, 10:36:53 am
Hi,
For language analysis articles with more than one written text, is it acceptable to talk about each text in different paragraphs? Because when I talk about two or more in the same paragraph, often my paragraphs get too long  :-\

Yes, it's fine to do so. Just curious, when you have two or more in the same paragraph, are you comparing them?
Title: Re: VCE English Question Thread
Post by: simrat99 on October 22, 2017, 11:04:03 am
Yes, it's fine to do so. Just curious, when you have two or more in the same paragraph, are you comparing them?
Thank you for your reply. Yup, I compare the arguments when I have two or more texts in a paragraph, but they tend to get very long :)

Another question, can I do the same with visuals? Can I have a separate paragraph of just the visual?
Title: Re: VCE English Question Thread
Post by: **** on October 22, 2017, 11:09:41 am
Thank you for your reply. Yup, I compare the arguments when I have two or more texts in a paragraph, but they tend to get very long :)

Another question, can I do the same with visuals? Can I have a separate paragraph of just the visual?

Just saying, you don't have to compare for the exam, even if they give you multiple texts. Best way to approach the task is to analyse A on its own then move on to text B - the task is not to compare unlike the SAC so you won't be penalised for not comparing.
Title: Re: VCE English Question Thread
Post by: halo on October 22, 2017, 11:21:01 am
Thank you for your reply. Yup, I compare the arguments when I have two or more texts in a paragraph, but they tend to get very long :)

Another question, can I do the same with visuals? Can I have a separate paragraph of just the visual?
Right, makes sense. Do you know if we're required to compare texts in section C? My understanding is that there are no marks allocated to comparisons as such, so I haven't really been focusing on it.

I don't think it is advisable to analyse the visual separately. Of course there is no set rule, but you'll probably score higher if you can link the visual to the accompanying text (and show how they work together to convey certain idea(s)) in the same paragraph. If the visual is a piece on it's own (is not accompanying any other text), then it would be a good idea to have your analysis of it as a separate para.
Title: Re: VCE English Question Thread
Post by: shiba woof woof on October 22, 2017, 12:03:07 pm
For section C we do not need to compare, that being said we can still use comparison to aid our analysis. But comparison should not be the focus of our analysis. I've always talked about text 1, then moved onto to text 2, sometimes making a few comparisons whilst analysing text 2.

And for the analysis of the visual, i think rule of thumb is. If it's embedded in the text, analyse as if it was an argument (so no need to base a paragraph around it, just pop it into your body paragraphs). But if it's a separate visual (eg 2016's VCAA watermelon cartoon), then you would base a paragraph around it.
Title: Re: VCE English Question Thread
Post by: christinett_ on October 22, 2017, 01:08:59 pm
Hi, I am studying Medea for Text Response, however am struggling with revising / preparing for the exam. I have gathered quotes and themes and prompts but it hasn't really helped me. What can i do?

Thank you
Title: Re: VCE English Question Thread
Post by: Ashjames on October 22, 2017, 01:13:50 pm
Hi, I am studying Medea for Text Response, however am struggling with revising / preparing for the exam. I have gathered quotes and themes and prompts but it hasn't really helped me. What can i do?

Thank you

What you can do at this stage is try to reread Medea as much as possible. Because the play is relatively short, you can read half of it everyday. While you're reading try to pay close attention to quotes so that you are able to remember them in the exams [trust me they will come to you]. Another thing you can do is apply your knowledge of the text, which basically means writing text responses based on different themes, that way you'll be ready for exams!
Title: Re: VCE English Question Thread
Post by: christinett_ on October 22, 2017, 01:21:11 pm
What you can do at this stage is try to reread Medea as much as possible. Because the play is relatively short, you can read half of it everyday. While you're reading try to pay close attention to quotes so that you are able to remember them in the exams [trust me they will come to you]. Another thing you can do is apply your knowledge of the text, which basically means writing text responses based on different themes, that way you'll be ready for exams!
Thanks for the advice but I have reread Medea and trying to complete text responses but keep getting stuck on prompts and what i should be writing for each pargraph
Title: Re: VCE English Question Thread
Post by: halo on October 22, 2017, 01:26:54 pm
Thanks for the advice but I have reread Medea and trying to complete text responses but keep getting stuck on prompts and what i should be writing for each pargraph

Have you tried reading any guides? They might have some new ideas you can use in your essays. You could also look at sample essays (heaps here on atarnotes)
Title: Re: VCE English Question Thread
Post by: Ashjames on October 22, 2017, 01:36:23 pm
Thanks for the advice but I have reread Medea and trying to complete text responses but keep getting stuck on prompts and what i should be writing for each pargraph

Sounds like you need to be working on breaking down prompts: for example if the prompt was ís medea hte only victim in the play?", ou can try to do something like this:

PARA 1: Medea is the victim as the play commences. Talk about how chorus even sympathies with her and give her their 'loyal friendship'. Talk about WHY she is the victim [betrayal e.g.]

PARA 2: talk about how other characters were victimized by Medea in order to seek revenge on Jason. : Glauce was killed harshly, children were killed despite their innocence.

PARA 3: talk about hoe Medea absolved herself from being the 'victim' of the play when she committed murders. Talk about how chorus recoil in horror at the act and how Jason is left standing in a weak position, which ultimately makes him the victim.


You really need to just work on breaking the prompt down, it does not necessarily mean you have to write essays, but even if it means sitting for half an hour and breaking a point down and thinking of relevant an valid points -this could really help you.
Title: Re: VCE English Question Thread
Post by: zofromuxo on October 22, 2017, 01:57:42 pm
Thanks for the advice but I have reread Medea and trying to complete text responses but keep getting stuck on prompts and what i should be writing for each pargraph
What prompts are you struggling with in particular or is it more putting idea onto paper?
As you stated you have reread Medea, so I'm assuming you have enough notes on quotes, themes and have a solid understanding of the text.

I agree with Ashjames that breaking down prompts would help you as opposed to going to write a full text response.
You can also just write a paragraph for a prompt as opposed to a full text response essay. This can help because you can start seeing other ideas you can use for other paragraphs.

Title: Re: VCE English Question Thread
Post by: atar.notes.user on October 22, 2017, 03:52:47 pm
You should try and spend one hour on each section. Start with Language Analysis, because you can use the reading time to read the article and have it fresh in your mind when you start writing, rather than having to go back and reread it all again much later in the exam, wasting time. Then do sections A and B in whatever order you prefer.

For each section, you should probably write 800-1000 words. Try not to go over 1000 words. If you write too much, you won't finish in time, but if you write under 800 you won't be able to fully flesh out your ideas. Just try and practice writing text responses in timed conditions and you'll get better and better at it!
and what about comparative? how long should it be
Title: Re: VCE English Question Thread
Post by: atar.notes.user on October 23, 2017, 10:01:28 am
hi guys, im struggling with writing more in text response. i seem to finish on time but my essay is always between the 775-850 word range, which is pretty less

how should i improve this in 9 days till the exam? also do examiners take off marks if u write less?
Title: Re: VCE English Question Thread
Post by: Sine on October 23, 2017, 10:04:15 am
hi guys, im struggling with writing more in text response. i seem to finish on time but my essay is always between the 775-850 word range, which is pretty less

how should i improve this in 9 days till the exam? also do examiners take off marks if u write less?
they won't inherently mark you down for being on the lower side of the word count but it could impact you indirectly. A 1200 word essay of X quality will always beat out a 800 word essay of X (same) quality.
Title: Re: VCE English Question Thread
Post by: chantelle.salisbury on October 24, 2017, 09:49:23 am
hello...
i have been told to write out and memorise 1/2 to 3/4 of my intro for my text responses for on the exam and then to moderate them according to the question...
is this a good idea becuase the last thing i want to do is not 'answer' the question or prompt? or shall i do it more for a 'go to' if i rrly get stuck with the prompt in front of me?
glad of opinions and help :)
Title: Re: VCE English Question Thread
Post by: atar.notes.user on October 24, 2017, 01:17:07 pm
what are some example of unity in The Crucbile

all i can think of is the idea of uniting based on christianity, does anyone have any other examples they'd like to share?
Title: Re: VCE English Question Thread
Post by: zofromuxo on October 24, 2017, 02:41:54 pm
hello...
i have been told to write out and memorise 1/2 to 3/4 of my intro for my text responses for on the exam and then to moderate them according to the question...
is this a good idea becuase the last thing i want to do is not 'answer' the question or prompt? or shall i do it more for a 'go to' if i rrly get stuck with the prompt in front of me?
glad of opinions and help :)
I would have a basic framework/structure for the introduction as opposed to having a word for word memorised introduction that you use.
Such as I would make reference to the prompt and what ideas I would discuss in my body paragraph, but that was about it.

If you find that memorisation helps you. Then go for it.
Title: Re: VCE English Question Thread
Post by: SweetJack on October 24, 2017, 03:38:57 pm
Hi there,
Im really struggling with this Medea prompt and im not sure how to approach it, should i discuss the deceptive nature of different characters in each paragraph eg. Jason, Medea, Creon

'There are no trustworthy characters in Medea'. Discuss
pls help
Title: Re: VCE English Question Thread
Post by: zofromuxo on October 24, 2017, 03:49:38 pm
Hi there,
Im really struggling with this Medea prompt and im not sure how to approach it, should i discuss the deceptive nature of different characters in each paragraph eg. Jason, Medea, Creon

'There are no trustworthy characters in Medea'. Discuss
pls help

You can certainly discuss the deceptive nature of the different characters.
I would if you can, write a paragraph of characters that are trustworthy such as Aegus, Nurse and Tutor.
As later on after the events of the play, Aegus does in fact keep his promise to Medea with dire consequences as well. But this isn't relevant. Point being you don't only have to talk about the deceptiveness at play by Jason, Medea and Creon.

Title: Re: VCE English Question Thread
Post by: clarke54321 on October 24, 2017, 03:59:02 pm
what are some example of unity in The Crucbile

all i can think of is the idea of uniting based on christianity, does anyone have any other examples they'd like to share?

Hi!

I studied The Crucible last year, so my ideas may be a bit scratchy. First establish a definition for unity. What is your understanding of the concept. Is unity inherently positive? It doesn't have to be.

So some ideas may be:
-Unity for a malicious, deceptive cause. For this film especially, it is important that you consider the context of McCarthyism. How does injustice prevail? Does it do so because of blind support? I'm thinking specifically of Abigail and the other girls.
-Unity between the judges. Another example of evil collaboration.
-Unity could also include single relationships (John Proctor/Elizabeth----> predicated upon trust? Disloyalty? Misunderstand?)

Hope this helps  :)


Hi there,
Im really struggling with this Medea prompt and im not sure how to approach it, should i discuss the deceptive nature of different characters in each paragraph eg. Jason, Medea, Creon

'There are no trustworthy characters in Medea'. Discuss
pls help


Hey!

I too found this prompt to be a struggle. It may be a good idea to first define trustworthy (or what your own idea of the concept is). Some synonyms: reliable, dependable, principled, etc.

From this list, you may be able to expand on ideas (by brainstorming) and develop a contention. By establishing the contention, the rest of the essay becomes much easier; in terms of delineating argument, and outlining a logical progression.

Some more general ideas, which you may like to consider:

-Appearance of moral righteousness (how they are perceived by others? Or how do they want other characters to perceive them?
-Are the characters trustworthy to themselves? That is, do they follow through with their intentions (you might want to consider internal conflicts for this one)
-Think about the ultimate bodies of righteousness- The Chorus. Here you could evaluate the extent to which ordained bodies of morality fulfil their purpose? Are they biased? Do they maintain a sense of conviction?
-Something extra: If your contention moves towards the realm of, characters cannot be trusted, ask yourself why? Think about emotion especially.

I hope this helps  :)
Title: Re: VCE English Question Thread
Post by: **** on October 24, 2017, 04:15:49 pm
Hi there,
Im really struggling with this Medea prompt and im not sure how to approach it, should i discuss the deceptive nature of different characters in each paragraph eg. Jason, Medea, Creon

'There are no trustworthy characters in Medea'. Discuss
pls help

For this prompt, you don't need to (and preferably shouldn't) have a clear cut contention. There is no character in the play that is entirely honest nor completely untrustworthy. Some things to consider:

- Medea's relationship with Aegeus and the Chorus. They're like her friends - is she truthful to them? Compare this to how she interacts with her 'enemies', so to speak: Jason, Creon etc.
- Women and foreigners' are displayed as shifty and dodgy (e.g Medea and the Chorus of Corinthian women colluding all the male characters' back, Medea herself scheming behind everyone else's)
- What prevent the characters from telling the truth? Self-desires? Being consumed by emotions?
- Is honesty situational?
- Is lying used to avoid difficult moral choices?
Title: Re: VCE English Question Thread
Post by: atar.notes.user on October 25, 2017, 01:02:17 pm
hi guys, just a few doubts to be cleared

- do i have to write down my prompt for section A and B or can i just write the number (like ii or something)
- i heard that examiners dont like paragraphs that go on for more than a page (hopefully not bc my handwriting is big so i tend to go over a page esp for lang analysis)
- should i highlight my quotes to make it easier for the examiner or not

thank you!!
Title: Re: VCE English Question Thread
Post by: simrat99 on October 25, 2017, 02:25:20 pm
Hi,
For prompts that ask about characters ( such as "All characters are selfish") can we base our paragraphs on certain characters or do we have to base them around ideas?
Thanks :)
Title: Re: VCE English Question Thread
Post by: K888 on October 25, 2017, 02:37:13 pm
hi guys, just a few doubts to be cleared

- do i have to write down my prompt for section A and B or can i just write the number (like ii or something)
- i heard that examiners dont like paragraphs that go on for more than a page (hopefully not bc my handwriting is big so i tend to go over a page esp for lang analysis)
- should i highlight my quotes to make it easier for the examiner or not

thank you!!
I assume you're talking about the exam, in which iirc you'll have a box somewhere near the top of the paper where you write down or tick (or something similar) the number of the question you've selected - you don't have to write out the prompt.
RE: paragraph length - I don't think they'd care that much about the length (particularly if it's just because of the size of your handwriting!), as long as you: successfully make your point in the paragraph, don't write excessive amounts, and meet the general criteria for the essay.
Personally, I wouldn't highlight quotes in your essay. The whole point of integrating your quotes in your essay is to make it smooth and help it flow, so I'd worry that highlighting the quotes you include might make the examiners subconsciously focus a bit more on the bits that are jumping off the page, rather than the overall essay and its quality (after all - having a strong essay that gels well and is smooth and all that is in my opinion, more important than showing you can integrate 20 quotes into your essay). Plus, the examiner is going to be reading your whole essay anyway - they're not going to miss the quotes.

Hope this helped :)
Title: Re: VCE English Question Thread
Post by: atar.notes.user on October 25, 2017, 06:44:15 pm
I assume you're talking about the exam, in which iirc you'll have a box somewhere near the top of the paper where you write down or tick (or something similar) the number of the question you've selected - you don't have to write out the prompt.
RE: paragraph length - I don't think they'd care that much about the length (particularly if it's just because of the size of your handwriting!), as long as you: successfully make your point in the paragraph, don't write excessive amounts, and meet the general criteria for the essay.
Personally, I wouldn't highlight quotes in your essay. The whole point of integrating your quotes in your essay is to make it smooth and help it flow, so I'd worry that highlighting the quotes you include might make the examiners subconsciously focus a bit more on the bits that are jumping off the page, rather than the overall essay and its quality (after all - having a strong essay that gels well and is smooth and all that is in my opinion, more important than showing you can integrate 20 quotes into your essay). Plus, the examiner is going to be reading your whole essay anyway - they're not going to miss the quotes.

Hope this helped :)
thank you! :)
Title: Re: VCE English Question Thread
Post by: abbeyrose on October 25, 2017, 08:52:55 pm
I don't know if this is the correct place to ask this, so for that I apologise, but are we allowed to bring our own dictionary into the exam?
Title: Re: VCE English Question Thread
Post by: zofromuxo on October 25, 2017, 08:55:22 pm
I don't know if this is the correct place to ask this, so for that I apologise, but are we allowed to bring our own dictionary into the exam?
Yes you can as long it doesn't violate these rules.

Dictionaries must not contain any highlighting, annotation or tabs not part of the original publication.

A thesaurus or a dictionary that contains a thesaurus is NOT permitted in any examination.

Electronic dictionaries are NOT permitted.
Title: Re: VCE English Question Thread
Post by: LifeisaConstantStruggle on October 25, 2017, 09:00:20 pm
Yes you can as long it doesn't violate these rules.

Dictionaries must not contain any highlighting, annotation or tabs not part of the original publication.

A thesaurus or a dictionary that contains a thesaurus is NOT permitted in any examination.

Electronic dictionaries are NOT permitted.

Depends on what tab parts/annotations you have.
I went into my Indonesian exam last year with 2 dictionaries I got from the library 1 day ago, and it had some markings on it (mostly just library record and stuff), and it was allowed so yeah.
Title: Re: VCE English Question Thread
Post by: LPadlan on October 26, 2017, 02:04:18 pm
Hey, I know its QUALITY OVER QUANTITY but how many words would you recommend writing on the exam for example: text response? The general word count I've heard is 800 words, currently i only ever manage to get 500-600. Can anyone give me advice on how to be more thorough or expand on my topics? The general formula for text response is usually 3 contentions, should i provide 4 contentions? Any advice is greatly appreciated, thanks!
Title: Re: VCE English Question Thread
Post by: clarke54321 on October 26, 2017, 02:19:27 pm
Hey, I know its QUALITY OVER QUANTITY but how many words would you recommend writing on the exam for example: text response? The general word count I've heard is 800 words, currently i only ever manage to get 500-600. Can anyone give me advice on how to be more thorough or expand on my topics? The general formula for text response is usually 3 contentions, should i provide 4 contentions? Any advice is greatly appreciated, thanks!

Hey!

I'd recommend 800-1000. You want to get enough depth in your essay and develop a nunanced discussion of the prompt. In terms of expansion, I'd recommend that you really try and challenge the prompt by asking yourself a whole range of questions. In this sense, you're more likely to tease out any latent/subtle implications.

Eg. 'Medea' is a play about pride.
- Is it about pride for all or only one?
- Can pride and justice ever coexist?
- Why is pride necessary? --->relevant context necessary
- How does one achieve pride?
- Does pride necessarily fulfil one? Do people still yearn for something more?
- Is one's search for pride selfish?

^^ These are only some of the possible questions you could ask yourself. But as you can see, there are multiple directions that you can take a prompt after brainstorming some simple questions. I'd also recommend that you look up synonyms for key words in a prompt. This may spark a new idea or route.

When you say 3 contentions, do you mean topic sentences? Definitely devise one contention only! You don't want contradictions. Topic sentences should only justify your overarching contention.
Title: Re: VCE English Question Thread
Post by: plsbegentle on October 26, 2017, 02:28:20 pm
Hey, I know its QUALITY OVER QUANTITY but how many words would you recommend writing on the exam for example: text response? The general word count I've heard is 800 words, currently i only ever manage to get 500-600. Can anyone give me advice on how to be more thorough or expand on my topics? The general formula for text response is usually 3 contentions, should i provide 4 contentions? Any advice is greatly appreciated, thanks!
You should really be aiming for 1000 words.... i would say 800 words is the bare minimum, if you write less than that, you are most likely looking at a low to mid range essay.
If you are writing around 500-600 word essays, i'm assuming each body paragraph is around ~150 words, which i way i see it, your paragraphs probably consists of bunch of story telling from your text with no analysis/evidence. If you are having trouble with coming up ideas, re-read your novel, or look at your text guides (insight ones, i find are extremely helpful), or even just read other peoples high scoring essays, that way it gives  you give a feel for what the length and quality of writing that is expected.
Title: Re: VCE English Question Thread
Post by: chantelle.salisbury on October 26, 2017, 04:37:06 pm
not sure this is the right place to post this..
but... just wondering if anyone had any prompts that they could share on 'The Lieutenant' and or comparison one on 'ransom' & 'invictus'?
would be greatly appreciated! :)
Title: Re: VCE English Question Thread
Post by: gnaf on October 26, 2017, 06:37:28 pm
in medea, is there a quote/ stage direction for how the children are close to Medea when she's crying over the thought of killing them? we went over it in class but i can't find it!!
Title: Re: VCE English Question Thread
Post by: clarke54321 on October 26, 2017, 06:50:12 pm
in medea, is there a quote/ stage direction for how the children are close to Medea when she's crying over the thought of killing them? we went over it in class but i can't find it!!

If you have the Vellacott addition, Medea breaks down in a sudden flood of weeping on pg.44 and weeps on pg.48 and 49.
Title: Re: VCE English Question Thread
Post by: gnaf on October 26, 2017, 07:40:31 pm
thanks  :)

there's no stage direction that the children are close to her though, right? or did I just miss it
Title: Re: VCE English Question Thread
Post by: clarke54321 on October 26, 2017, 08:01:57 pm
thanks  :)

there's no stage direction that the children are close to her though, right? or did I just miss it

Hmm... on pg.48- 'the children come to Medea'

On pg.49 'the children go from her a little'

That's a really interesting contrast that I previously never picked up on!
Title: Re: VCE English Question Thread
Post by: atar.notes.user on October 27, 2017, 11:24:15 am
hi guys,

what do u usually write in ur conclusions for lang analysis?
Title: Re: VCE English Question Thread
Post by: Willba99 on October 27, 2017, 11:47:13 am
hi guys,

what do u usually write in ur conclusions for lang analysis?

I'd talk about the likely overall effect that all pieces (including visuals and comments) are likely to have on a reader, and also a maybe a quick summary of which arguments are most likely to have caused this overall effect
Title: Re: VCE English Question Thread
Post by: clarke54321 on October 27, 2017, 11:53:27 am
hi guys,

what do u usually write in ur conclusions for lang analysis?

Hey!

Usually I make reference to a difference in tone and manner between the two authors (if there is more than one article), and how this works to bolster the main contention. Some people decide to do it by technique. Given that the conclusion is not a necessity in an AA essay, I wouldn't worry too much about it :)
Title: Re: VCE English Question Thread
Post by: jonwil on October 27, 2017, 04:05:33 pm
Hey I have this really important question to ask.

Can you use one quote to explain or conclude your points about both texts in comparative?
For example "Forgiveness liberates the soul,... that is why it is such a powerful weapon"
This is a quote from Invictus that you can use to justify the nature of Mandela's forgiveness, more over it can be used as evidence to how Mandela could achieve what he could. But I also wanted to use this quote to explain as well as magnify the same idea from Ransom.
Priam is willing to forgive Achilles who has violated Hector's body when no one else could. (Hecuba calls him a "jackal" the council see this as "folly" yadda ya). So if I were to conclude a paragraph on the power of forgiveness, or if were to make a point about each text and link them in one sentence. Is it okay to use ONE QUOTE from ONE TEXT to justify your idea about BOTH TEXTS?
Thanks :D
Title: Re: VCE English Question Thread
Post by: zhen on October 27, 2017, 04:20:10 pm
Hey I have this really important question to ask.

Can you use one quote to explain or conclude your points about both texts in comparative?
For example "Forgiveness liberates the soul,... that is why it is such a powerful weapon"
This is a quote from Invictus that you can use to justify the nature of Mandela's forgiveness, more over it can be used as evidence to how Mandela could achieve what he could. But I also wanted to use this quote to explain as well as magnify the same idea from Ransom.
Priam is willing to forgive Achilles who has violated Hector's body when no one else could. (Hecuba calls him a "jackal" the council see this as "folly" yadda ya). So if I were to conclude a paragraph on the power of forgiveness, or if were to make a point about each text and link them in one sentence. Is it okay to use ONE QUOTE from ONE TEXT to justify your idea about BOTH TEXTS?
Thanks :D
Wait for someone else to reply but I would not recommend using a quote from one text to justify ideas in both texts. You can say that the ideas are similar in both texts, but in my opinion you need to provide a quote for each text to really justify your idea and provide evidence that it’s true for both texts. Also if you use a quote from one text to justify your idea in both texts, you probably won’t be able to analyse the slight differences in ideas between texts cause you’re only quoting one text.
Title: Re: VCE English Question Thread
Post by: superficious on October 28, 2017, 12:41:35 pm
In your opinion, to anyone reading this, would the Romantic Era be a specific theme in the Frankenstein Text Response Prompts? I was kind of hoping for the themes of Nature Vs. Nurture or Innocence...
Title: Re: VCE English Question Thread
Post by: kaii on October 28, 2017, 12:46:42 pm
Hi,

It's exam season and I have left it till the last minute as always. Can someone please tell me how to study for my English exam?
I don't really know where to start because I'm so overwhelmed :(

P.S.
Besides rereading/rewatching please give me tips on how to begin revision. Thanks!
Title: Re: VCE English Question Thread
Post by: Mariamnourine on October 28, 2017, 12:54:44 pm
Hi,

It's exam season and I have left it till the last minute as always. Can someone please tell me how to study for my English exam?
I don't really know where to start because I'm so overwhelmed :(

P.S.
Besides rereading/rewatching please give me tips on how to begin revision. Thanks!

Hey, don't be discouraged by that, you've still got a chance at it and remember there are heaps of VCE students in your position right now.

I suggest for language analysis you try and do 2 from the Vcaa website, only because you can then read sample responses and correct your own work and see where you went wrong.

For text response, it is easier to just read themes online because they usually incorporate quotes AND you don;t have to reread your bboks. Thisis a better otion becaseu you are more likely to remeber what points to talk about in your essay id a praticialur theme comes up!

Regarding comparative, I advise you to reread your texts and have them fresh in mind for the exam as it is new to the study design and there isn't much resources out there.if your teacher supplied you with summaries and stuff, I advise you to go over them.

I believe in you, don;t be the one that has to feel overwhelmed, just remember there are many people in your position right now and use them as a force of motivation to push you through these last few days. if you don;t stress and get on with it, you will be fine, because if you haven't realized you have been studying these texts all year, so you're not in  a bad position at ALL!!!!

Hope that helped you and Good luck!


Title: Re: VCE English Question Thread
Post by: K888 on October 28, 2017, 12:59:58 pm
Hi,

It's exam season and I have left it till the last minute as always. Can someone please tell me how to study for my English exam?
I don't really know where to start because I'm so overwhelmed :(

P.S.
Besides rereading/rewatching please give me tips on how to begin revision. Thanks!
Hi there! Just going to add on to what Mariamnourine has said.

How do you feel writing essays - are you confident with them? How do you feel with timing?

I think some important things to do between now and Wednesday are:
- Consolidate your quotes - make sure you have enough short and sweet (but useful) quotes to take into your exam.

- Plan! This was what most of my revision consisted of last year - get all the topics you can for your text(s) (look through past VCAA papers, and resources you've been given over the year from your teacher) and smash out some plans! Doesn't have to be complex - just brainstorm your thoughts and ideas, some arguments you could use, evidence you'd use, and maybe about how the author's message links in with it all. Planning is great because it'll help you come up with something quick when you get into the exam, and you'll be less likely to be taken off guard by the topics you've been given, because you'll already have arguments from heaps of topics that you can pick and choose to make a great essay on the day!

- Read some articles - you can just grab them from past VCAA papers, or use stuff you've been given in class - and highlight and annotate them, decide what you'd analyse if you were to write an essay, and make sure you have figuring out the contention and issue downpat :) Then, maybe try writing a few paragraphs of analysis, then work up to a full analysis!

- Decide what your weakest part is, and maybe dedicate a bit more time to that - that might mean doing a few more essay plans, or maybe trying to write some language analysis essays or even just read, highlight and annotate some articles.

If you're really not confident with your timing - I'd maybe recommend trying to do some essays under timed conditions, or even trying sitting a practice exam.
Another great thing to do would be email your teacher for some ideas, and maybe see if you can have a chat with them between now and the exam - if for nothing else other than a pep talk!

All the best, I hope this has helped a bit :)
Title: Re: VCE English Question Thread
Post by: usernameincorrect on October 28, 2017, 07:32:44 pm
Hello there! Just wondering what people think about the text response section of the exam for The Complete Maus. Considering that this is its last year on the study design, and that for the past few years there have been prompts relating to conventions, do you guys think that convention prompts will still be on this year? Also taking into account it is a graphic novel after all.

Anddd, for the comparative section, whats the chances of the prompts being similar to those of the sample ones? I know its close to impossible to tell, but is it extremely unlikely? Or possible. Thoughts?

Thanks!!
Title: Re: VCE English Question Thread
Post by: Willba99 on October 28, 2017, 07:45:03 pm
Anddd, for the comparative section, whats the chances of the prompts being similar to those of the sample ones? I know its close to impossible to tell, but is it extremely unlikely? Or possible. Thoughts?


I'd say its reasonably likely that there will be at least one question/prompt focusing on the obvious, major themes of a text. For example, I'd say its likely that at least one of the Tracks & Into the Wild q's will be about society or personal development/self-discovery, so if you've studied both of these themes then you should be alright. Impossible to say for sure though
Title: Re: VCE English Question Thread
Post by: pmmenotes on October 28, 2017, 07:47:29 pm
bit of a random question but in the exam what order are you guys doing everything? idk if i should do lang analysis first (my weakest), then comparative and then medea or another way
Title: Re: VCE English Question Thread
Post by: Willba99 on October 28, 2017, 07:51:10 pm
bit of a random question but in the exam what order are you guys doing everything? idk if i should do lang analysis first (my weakest), then comparative and then medea or another way

Personally, I'm going to do LA first, as I'll spend most of my reading time on it so I don't want to have to reread it in writing time. Additionally, doing LA first will (hopefully) let the questions for the other two essays bubble away in the back of my mind. Next I'll do This Boy's Life, as its my weakest and I want to smash it out before I get too tired, hopefully leaving me ~65 minutes to write a good comparative essay. Just my opinion though! I think its definitely popular to do LA first
Title: Re: VCE English Question Thread
Post by: clarke54321 on October 28, 2017, 07:52:01 pm
bit of a random question but in the exam what order are you guys doing everything? idk if i should do lang analysis first (my weakest), then comparative and then medea or another way

Hey!

Usually I do argument analysis first, given that I would have just read it during reading time. I also find it the most stressful, so want it to be out of the way. Then I go text response and comparative. All the best for exams :)
Title: Re: VCE English Question Thread
Post by: eloise1231 on October 28, 2017, 09:45:25 pm
hey all! just a quick question- when an essay prompt has a quote in it, if it's are fairly long (i.e. a sentence or more), do you have to embed the entire quote in your essay? Or will fragments of the quote do?
Title: Re: VCE English Question Thread
Post by: clarke54321 on October 28, 2017, 09:51:35 pm
hey all! just a quick question- when an essay prompt has a quote in it, if it's are fairly long (i.e. a sentence or more), do you have to embed the entire quote in your essay? Or will fragments of the quote do?

Hey!

Fragments will be fine :) If anything, fragments demonstrate a stronger understanding of the quote and its implications (ie. you can craftily integrate them and use them to bolster your main points).
Title: Re: VCE English Question Thread
Post by: LPadlan on October 29, 2017, 11:23:11 am
Am i allowed to post a Youtube link for my teacher channel? Theres solid summaries and analysis for texts such as Crucible and Year of Wonders.
Title: Re: VCE English Question Thread
Post by: halo on October 29, 2017, 02:42:43 pm
For Medea, do we have to refer to both the traditional and modern audience in our essays? Especially for a prompt like the one below, I feel as if it is important to make that distinction as it is very likely that our responses to Medea and her plight are different to what the traditional male audience might feel. I would assume that, given the strictly patriarchal structure of society, they would be more inclined to support Jason and might be unable to relate to the group of Corinthian women or Medea?

Prompt: “The audience's sympathy remains aligned with the Chorus throughout the play”. Do you agree?
Title: Re: VCE English Question Thread
Post by: LifeisaConstantStruggle on October 29, 2017, 02:51:46 pm
For Medea, do we have to refer to both the traditional and modern audience in our essays? Especially for a prompt like the one below, I feel as if it is important to make that distinction as it is very likely that our responses to Medea and her plight are different to what the traditional male audience might feel. I would assume that, given the strictly patriarchal structure of society, they would be more inclined to support Jason and might be unable to relate to the group of Corinthian women or Medea?

Prompt: “The audience's sympathy remains aligned with the Chorus throughout the play”. Do you agree?

The Ancient Athenian male audience are far more important than the modern audience imo. (this is solely due to the fact that the play is written for a specific religious event, and not for the general public like you and me) Also, it's harder to elaborate on the psychological impact of the play in terms of the modern context as well, unless if you want to go off topic and relate it to social issues then yeah, but in relation to VCE English it's not really required.
Title: Re: VCE English Question Thread
Post by: LPadlan on October 29, 2017, 04:49:36 pm
For language analysis is it actually required to state the persuasive techniques? (eg. Through the use of powerful imagery, the author....) As i'm reading a LA that got 19/20 and i cannot see any sentences that contain a statement of the persuasive technique used.
Title: Re: VCE English Question Thread
Post by: clarke54321 on October 29, 2017, 04:58:47 pm
For language analysis is it actually required to state the persuasive techniques? (eg. Through the use of powerful imagery, the author....) As i'm reading a LA that got 19/20 and i cannot see any sentences that contain a statement of the persuasive technique used.

Metalanguage is good to use when you can identify it. But it is by no means necessary to writing a good analysis. Assessors are more concerned with your ability to work closely with your selected evidence- pulling out all intimations/implications/suggestions. This should be your main priority.

In a different vein, is anyone feeling slightly concerned about a hero prompt coming out for Medea. I have an inkling that it may arise  :-\ However, in all honesty, I have not spent enough time familiarising myself (either in class or in my own study) with the context of a Greek hero. I find the whole issue way too subjective, and difficult to build into my existing essays. What's everyone's opinion on this?
Title: Re: VCE English Question Thread
Post by: LifeisaConstantStruggle on October 29, 2017, 06:32:45 pm
In a different vein, is anyone feeling slightly concerned about a hero prompt coming out for Medea. I have an inkling that it may arise  :-\ However, in all honesty, I have not spent enough time familiarising myself (either in class or in my own study) with the context of a Greek hero. I find the whole issue way too subjective, and difficult to build into my existing essays. What's everyone's opinion on this?

Hey hey :) I have read about the concerns of people on this topic and heard my friends talking about it. I have an essay on this which touches on the notion of a tragic hero, and I'll upload it if my internet is back up (probs in a few hours) so yeah. :) I might be able to type up some stuff on the features of Greek tragedies, and an archetypal tragic hero, things like that.
It will be alright tho tbh I don't think they will put something that complex into our exam, but anyways I hope that I can give you guys some preparations to this as well.

Do give my essay a mark tho, if you want to that'd be great :)
Title: Re: VCE English Question Thread
Post by: gnaf on October 29, 2017, 07:20:31 pm
anyone have an example of how to analyse tone in language analysis? I usually just mention it in the intro then never talk about it again  :P

Title: Re: VCE English Question Thread
Post by: rnrn on October 29, 2017, 07:35:33 pm
In a different vein, is anyone feeling slightly concerned about a hero prompt coming out for Medea. I have an inkling that it may arise  :-\ However, in all honesty, I have not spent enough time familiarising myself (either in class or in my own study) with the context of a Greek hero. I find the whole issue way too subjective, and difficult to build into my existing essays. What's everyone's opinion on this?


Didn't they do a hero one already? I feel like there's going to be one on pride - it's one of the obvious themes that they have yet to touch on.
Title: Re: VCE English Question Thread
Post by: clarke54321 on October 29, 2017, 07:45:18 pm


Didn't they do a hero one already? I feel like there's going to be one on pride - it's one of the obvious themes that they have yet to touch on.

Yes, there's already been one on heroism (2001?), which makes me sceptical. True. Pride could come up.

anyone have an example of how to analyse tone in language analysis? I usually just mention it in the intro then never talk about it again  :P

I usually merge tone analysis with the written language. Ie. The vehement undertones of the phrase X, exemplifies/emphasises/underscores Y (more explanation/analysis).

It is a good spring board :)
Title: Re: VCE English Question Thread
Post by: Mariamnourine on October 29, 2017, 07:56:38 pm


Didn't they do a hero one already? I feel like there's going to be one on pride - it's one of the obvious themes that they have yet to touch on.
[/quote]


I feel like they haven't touched on selfishness in Medea yet. So there may be a prompt about selfishness.
Title: Re: VCE English Question Thread
Post by: zofromuxo on October 29, 2017, 08:01:14 pm
Yes, there's already been one on heroism (2001?), which makes me sceptical. True. Pride could come up.
If you referring to VCCA 2001's prompts, there was nothing on heroes.
It could be on the exam.
Although looking at the 2001-02 and 2015-16 VCCA prompts, I think it could be on emotions again and a character based one.
I wouldn't worry about predicting what it would be and instead be ready for any prompt they throw at you.
Who knows it could be the year they give prompts based on play devices like the Chorus' role.

If you guys have any quick question on Medea, my PMs are open. I'm not marking work on it though.
Title: Re: VCE English Question Thread
Post by: Rieko Ioane on October 29, 2017, 09:30:16 pm
Well, VCAA needs to adhere to all skill levels across the entire state so even if they give us a dodgy prompt, it's almost guaranteed the other one won't be.

The trend seems to be 1 theme based prompt and 1 character based prompt, I don't think it's that unlikely they'll give us a hero based prompt for Medea. You don't have to write it in regards to what the paradigm of a Greek hero is anyway. It's English you can write whatever the fuck you want lol.
Title: Re: VCE English Question Thread
Post by: LPadlan on October 29, 2017, 09:47:30 pm
How are you guys memorizing quotes? Eg. Writing it out over and over again etc.  i have a decent quote bank for Medea, but YOW and The crucible is another story...
Title: Re: VCE English Question Thread
Post by: rosecookiie on October 29, 2017, 10:14:00 pm
How are you guys memorizing quotes? Eg. Writing it out over and over again etc.  i have a decent quote bank for Medea, but YOW and The crucible is another story...

I'm doing YOW and The Crucible as well! Try brainstorming the quotes under theme branches and keep doing this until you can pretty much write down all the quotes from memory.
Title: Re: VCE English Question Thread
Post by: lyoko on October 30, 2017, 12:21:54 am
How are you guys memorizing quotes? Eg. Writing it out over and over again etc.  i have a decent quote bank for Medea, but YOW and The crucible is another story...

Yup basically that, writing it over again and like rosecookiie  said, sort of cluster in groups.

I'm doing YOW and The Crucible as well! Try brainstorming the quotes under theme branches and keep doing this until you can pretty much write down all the quotes from memory.

I'm also doing YOW and TC as well :D
Title: Re: VCE English Question Thread
Post by: atar.notes.user on October 30, 2017, 10:03:48 am
what happens if i forgot  'the' or 'a' from a quote in section a and/or b? will i be penalised bc im not using the right quote

also for AA, i tend to always just write 'therefore, the readers will agree with the writer's argument'
how can i force myslef to execute specific responses from the audience?
Title: Re: VCE English Question Thread
Post by: K888 on October 30, 2017, 11:57:39 am
what happens if i forgot  'the' or 'a' from a quote in section a and/or b? will i be penalised bc im not using the right quote

also for AA, i tend to always just write 'therefore, the readers will agree with the writer's argument'
how can i force myslef to execute specific responses from the audience?
I don't think it should matter *too* much if you miss a word or two in your quote - as long as the idea of the quote is still intact, and it still makes sense to have it in your essay, you should be fine. :) Remember that the people marking the English exams have to be able to mark every text - so they'll have read them all, but they won't necessarily know all texts that well. My teacher told me that you can usually get away with a bit, because a) the examiner may not be super acquainted with the text, so as long as the quote fits the picture you'll be fine and b) they're not going to go look through their book to see if that quote matches perfectly - they've got tonnes of other essays to mark! :)

In terms of article analysis, I think a) try to expand your vocabulary in terms of the way you express the effect on the readers, and b) make sure you say why they're being influenced to agree or whatever. That second part is super important - it's not enough to just analyse a snippet of the article, and chuck on a "therefore readers will agree with the author" at the end - you need to elaborate on why they're agreeing. :) I think a lot of people struggle with this to some extent when they do English. Just be really conscious when you're writing that you're actually talking about how (and why) the certain part of the article you're analysing is important - what's it actually there for in the article? What purpose does it serve? It's your job to analyse that :)

Hope I didn't go too far off track with my answer. All the best!
Title: Re: VCE English Question Thread
Post by: atar.notes.user on October 30, 2017, 12:51:05 pm
I don't think it should matter *too* much if you miss a word or two in your quote - as long as the idea of the quote is still intact, and it still makes sense to have it in your essay, you should be fine. :) Remember that the people marking the English exams have to be able to mark every text - so they'll have read them all, but they won't necessarily know all texts that well. My teacher told me that you can usually get away with a bit, because a) the examiner may not be super acquainted with the text, so as long as the quote fits the picture you'll be fine and b) they're not going to go look through their book to see if that quote matches perfectly - they've got tonnes of other essays to mark! :)

In terms of article analysis, I think a) try to expand your vocabulary in terms of the way you express the effect on the readers, and b) make sure you say why they're being influenced to agree or whatever. That second part is super important - it's not enough to just analyse a snippet of the article, and chuck on a "therefore readers will agree with the author" at the end - you need to elaborate on why they're agreeing. :) I think a lot of people struggle with this to some extent when they do English. Just be really conscious when you're writing that you're actually talking about how (and why) the certain part of the article you're analysing is important - what's it actually there for in the article? What purpose does it serve? It's your job to analyse that :)

Hope I didn't go too far off track with my answer. All the best!
thank you!!!
Title: Re: VCE English Question Thread
Post by: LPadlan on October 30, 2017, 04:47:30 pm
Is there any comparative essays on Year of Wonders and The Crucible here? I cant find any
Title: Re: VCE English Question Thread
Post by: hmdeadas on October 30, 2017, 05:04:03 pm
Hey, can i use the word "conclusively" , to start off my conclusion? If not are there other terms?
Also what do I do if for section C one of the texts have no Author/writer? How would i refer to the text piece?
Thank You
Title: Re: VCE English Question Thread
Post by: zxcvbnm18 on October 30, 2017, 07:28:30 pm
anyone got any answers for the sample aural task given in VCAA?
Title: Re: VCE English Question Thread
Post by: LPadlan on October 30, 2017, 07:59:56 pm
Would i get penalized if i misused a quote for a character? eg. It was due to Jason's "proud, impassioned soul" that...
Since i'm sure(i may be mistaken) that quote was used to describe Medea, not Jason
Title: Re: VCE English Question Thread
Post by: clarke54321 on October 30, 2017, 08:02:33 pm
Hey, can i use the word "conclusively" , to start off my conclusion? If not are there other terms?
Also what do I do if for section C one of the texts have no Author/writer? How would i refer to the text piece?
Thank You

You can use "conclusively" to start conclusions. However, it does give off a kind of generic feel. Even the simple "Thus, Hence or Indeed" can solve this problem.

If there is no designated author/writer, I would quite simply use one of these terms!

Would i get penalized if i misused a quote for a character? eg. It was due to Jason's "proud, impassioned soul" that...
Since i'm sure(i may be mistaken) that quote was used to describe Medea, not Jason

I don't think it would be a huge issue. If you continually mixed up characters and evidence, then perhaps examiners would start to question your knowledge of the play. But one quote wouldn't have too great an impact  :)
Title: Re: VCE English Question Thread
Post by: zxcvbnm18 on October 30, 2017, 11:59:11 pm
In what ways is medea a hero?
Title: Re: VCE English Question Thread
Post by: hmdeadas on October 31, 2017, 12:12:46 am
Does anyone have any resources/tactics for identifying the text type/form of the texts we are given in section C?
Like identifying whether is an editorial / article etc?
THANK YOU!
Title: Re: VCE English Question Thread
Post by: K888 on October 31, 2017, 12:49:56 am
Does anyone have any resources/tactics for identifying the text type/form of the texts we are given in section C?
Like identifying whether is an editorial / article etc?
THANK YOU!

- The most important thing I can say is read the snippet of context/background info that you're given at the start of section C carefully! It might say it outright then and there (like it did in 2012, 2013, 2014 and other years no doubt haha), or it'll at least give hints.
- If it doesn't tell you in the background info, it might tell you in the text itself! Last year it said "the local newspaper of Lawton publishes a weekly column written by the mayor" or something like that, and then was addressed as "From the Mayor". They also gave you the form of the other texts. In 2015, it had these sort of stage direction things that complemented the format it was written in to really clue you into what it was.
- Failing that, read the title of the text and look for the author - editorials usually won't have a specific author's name, they'll probably be published under the name of the newspaper. Likewise, if there is the name of the author there - see if there's any official titles - maybe it's the Prime Minister, and the text form is an address from them?
- Check the tone - maybe it's a satirical piece? If they're using "I" or referring to themselves lots - maybe it's an opinion piece
- If its a visual - is it a cartoon? Is it a photo? Is it a drawing? Etc, etc.

I remember finding it much easier to find the type or form of text in VCAA exams than in any articles, etc. that we got given in class, so don't stress! If you get flustered, just take a step back, take a deep breath, and get back into the zone. :)
All the best!
Title: Re: VCE English Question Thread
Post by: zofromuxo on October 31, 2017, 07:29:04 am
In what ways is medea a hero?
If your thinking of making Medea a "traditional" hero then I would say not to do that. As I believe Medea isn't a hero, but a Tragic Hero.
-She completes a heroic deed with Jason in obtaining the golden fleece
-She had a noble position as the Daughter of King Aeethes and the Granddaughter of the sun god Helios
-You could argue that she had free will in the decisions she made (You can argue that she was fated by the gods in her decision as well)
-She has a hamartia of hubris
-You could say she had reached some understanding of the breaking of marriage of Jason due to his greed for power and even knew this while committing sophistry against the male characters.

In a modern sense, she is characterised as an anti-hero.
Title: Re: VCE English Question Thread
Post by: zxcvbnm18 on October 31, 2017, 09:51:33 am
If your thinking of making Medea a "traditional" hero then I would say not to do that. As I believe Medea isn't a hero, but a Tragic Hero.
-She completes a heroic deed with Jason in obtaining the golden fleece
-She had a noble position as the Daughter of King Aeethes and the Granddaughter of the sun god Helios
-You could argue that she had free will in the decisions she made (You can argue that she was fated by the gods in her decision as well)
-She has a hamartia of hubris
-You could say she had reached some understanding of the breaking of marriage of Jason due to his greed for power and even knew this while committing sophistry against the male characters.

In a modern sense, she is characterised as an anti-hero.
[/quote
Thank you!
Title: Re: VCE English Question Thread
Post by: tinagranger on October 31, 2017, 01:29:35 pm
Hi, I have a comparative question for The Crucible and Year of Wonders: For my conclusion, if I want a general difference about religion in both texts, I am not sure what to write - could someone help me word something? Thank you :)
Title: Re: VCE English Question Thread
Post by: rpapa on October 31, 2017, 01:43:38 pm
Hi, I have a comparative question for The Crucible and Year of Wonders: For my conclusion, if I want a general difference about religion in both texts, I am not sure what to write - could someone help me word something? Thank you :)
Hi! So what I have been told to do by my teacher is to incorporate the purposes of each writer in your conclusion. Basically, it's the 'so what?' of your entire essay. Why have Brooks and Miller bothered to write about the plague and Salem witch trials? What do they want us to take away from this?
For religion, I would probably say something along the lines of this (depending on what I actually include in my essay ofc):
Both Brooks and Miller warn us against literal interpretations of religious texts. The theocracies founded in both Eyam and Salem are rife with binary thinking and punitive justice as a result. Miller ultimately uses religion as a didactic tool to illustrate the dire consequences of these political structures. However, Brooks is more concerned with demonstrating the impact of religion on individuals rather than political structures. The loss of faith and movement towards an age of reason is made markedly apparent in 'Year of Wonders' etc...
Hopefully this helps!
Feel free to add anything guys
Title: Re: VCE English Question Thread
Post by: hmdeadas on October 31, 2017, 02:25:36 pm
Hey, So for section B(Comparative), how long should the introduction be? Mine go for over half a page, as i introduce both texts / authors / style / context / and the three points i will be making. Is that too much information?
Title: Re: VCE English Question Thread
Post by: clarke54321 on October 31, 2017, 02:32:13 pm
Hey, So for section B(Comparative), how long should the introduction be? Mine go for over half a page, as i introduce both texts / authors / style / context / and the three points i will be making. Is that too much information?

Depending on your hand-writing size, more than half a page may or may not be a bit too much. With section B, I've always condensed my introduction to 3 large sentences (multiple clauses). However, this is definitely personal preference. My only reasoning is that, with the comparative, the bulk of your essay will generally be greater than the text response; given the comparison between two different texts. A good idea might be to go through your essay with a highlighter, and highlight words or phrases that are key to context or interpretation. After having done this, see if you can re-write a shorter, and maybe more concise, introduction. Hopefully this helps :)
Title: Re: VCE English Question Thread
Post by: plsbegentle on October 31, 2017, 02:38:27 pm
any phrases or ways to incorporate tone in body paragraphs for LA? Rarely do i ever mention about tone after the introduction.
Thanks
Title: Re: VCE English Question Thread
Post by: rpapa on October 31, 2017, 02:39:14 pm
Hey, So for section B(Comparative), how long should the introduction be? Mine go for over half a page, as i introduce both texts / authors / style / context / and the three points i will be making. Is that too much information?
Depending on your hand-writing size, more than half a page may or may not be a bit too much. With section B, I've always condensed my introduction to 3 large sentences (multiple clauses). However, this is definitely personal preference. My only reasoning is that, with the comparative, the bulk of your essay will generally be greater than the text response; given the comparison between two different texts. A good idea might be to go through your essay with a highlighter, and highlight words or phrases that are key to context or interpretation. After having done this, see if you can re-write a shorter, and maybe more concise, introduction. Hopefully this helps :)

Hey! Adding to clarke54321's wonderful advice, I would try to cut out the style and context in your introduction. I am assuming you are including things such as the setting of the texts, when they were written, what genre they are in etc. (please correct me if i'm wrong). Unless they are directly related to the three points you are making, I wouldn't include them. Otherwise, everything else you are including seems fine! Maybe just condense them a bit!
Hopefully this helps!
Title: Re: VCE English Question Thread
Post by: clarke54321 on October 31, 2017, 02:43:22 pm
any phrases or ways to incorporate tone in body paragraphs for LA? Rarely do i ever mention about tone after the introduction.
Thanks

I think I answered a similar question in a previous post. To incorporate tone into analysis, I like to connect it to textual evidence. Eg. "The undertones of (tone X- sarcasm, distress), which riddle/consume the phrase Y, seeks to stress/emphasise/foreground."

I also like to preface new paragraphs with, "Shifting to a more Z tone, author P ....."
Title: Re: VCE English Question Thread
Post by: rpapa on October 31, 2017, 02:48:54 pm
any phrases or ways to incorporate tone in body paragraphs for LA? Rarely do i ever mention about tone after the introduction.
Thanks
I think I answered a similar question in a previous post. To incorporate tone into analysis, I like to connect it to textual evidence. Eg. "The undertones of (tone X- sarcasm, distress), which riddle/consume the phrase Y, seeks to stress/emphasise/foreground."

I also like to preface new paragraphs with, "Shifting to a more Z tone, author P ....."
Hi! Adding to clarke54321 again (sorry man), explaining the impact of tonal shifts on the target audience could also be used. It provides you with another level of complexity to your analysis.  For example, if you note that the author moves from an aggressive tone to a more calmer one, you could talk about how they would seem more reasonable to their audience etc.
Hopefully that makes sense! Please feel free to add anything!
Title: Re: VCE English Question Thread
Post by: LPadlan on October 31, 2017, 03:18:34 pm
How many quotes are you looking to memorize for each text(Medea, YOW and The Crucible)
Title: Re: VCE English Question Thread
Post by: zofromuxo on October 31, 2017, 03:29:17 pm
How many quotes are you looking to memorize for each text(Medea, YOW and The Crucible)
I can't say for the other texts besides Medea since I didn't do comparative last year.
I would say going into the exam I memorise ~20-25 quotes most likely more then that if I'm honest.
With Medea, then are a lot of good short quotes that hold a lot of information such as "tuscan syclla", "lioness guarding her cubs", "everybody loves himself more than his neighbour", "I fear you", "to punish Jason will be just", "...child killer", "vile murderess!" and many more.

I would remember the key ones that everyone puts in their essay and maybe some specific ones like the animalistic quotes to show Medea is an outsider or ones that show Medea is different to other women.
Title: Re: VCE English Question Thread
Post by: AceVentura on October 31, 2017, 04:53:27 pm
How many quotes would you recommend to memeorise for each of sections A and B? Should I memorise a quote for each theme?

And what make would I need to get a 36? A 7/10 for each essay or...?

Title: Re: VCE English Question Thread
Post by: BlinkieBill on October 31, 2017, 04:54:35 pm
Hi, for comparative should you make comparisons between texts throughout the paragraph? and take a similar approach to comparative language analysis?
I usually structure my paras by talking about one text then i have a transitional comparative statemnt and start talking about the next text, but should i include more comparison throughout???
Title: Re: VCE English Question Thread
Post by: LPadlan on October 31, 2017, 05:03:32 pm
Does anyone have any key quotes for Crucible and YOW?
Title: Re: VCE English Question Thread
Post by: Pinkydoo on October 31, 2017, 05:06:36 pm
Hey guys, does anyone know if we'll be marked down if we don't have a conclusion for Section C? I've heard some people say that's it's not that important... so could someone please confirm this? Thx  ;)
Title: Re: VCE English Question Thread
Post by: zhen on October 31, 2017, 05:13:13 pm
Hi, for comparative should you make comparisons between texts throughout the paragraph? and take a similar approach to comparative language analysis?
I usually structure my paras by talking about one text then i have a transitional comparative statemnt and start talking about the next text, but should i include more comparison throughout???

Ideally you’d want more than one transitional comparative statement. So you should include more comparison throughout, cause comparison is a major criteria.

Hey guys, does anyone know if we'll be marked down if we don't have a conclusion for Section C? I've heard some people say that's it's not that important... so could someone please confirm this? Thx  ;)
My impression is that you should at least chuck in a 2 sentence conclusion to make sure you have it. It only takes like a minute for me to write one. This is just to be safe. But, I think that if you don’t have time for a conclusion, then it’s not the biggest deal and you shouldn’t stress about it.
Title: Re: VCE English Question Thread
Post by: manisha99 on October 31, 2017, 05:19:56 pm
For the burial rites and medea section, are we allowed to write about both?? I've been told that we need to pick one book only and write an essay on that using the prompts given but what if i find burial rites too difficult in the exam, am i allowed to write about both?? Obviously on different papers
Title: Re: VCE English Question Thread
Post by: kjade on October 31, 2017, 05:21:51 pm
Anyone have a simple, easy acronym for language analysis structure? Like what to include in the intro and how to lay out each body paragraph?
Title: Re: VCE English Question Thread
Post by: zofromuxo on October 31, 2017, 05:27:41 pm
Anyone have a simple, easy acronym for language analysis structure? Like what to include in the intro and how to lay out each body paragraph?
For body paragraph,
Argument
Persuasive Technique
Example of it
Explanation of how it affects the reader
Persuasive technique 
Example of it
Explanation of how it affects the reader
Title: Re: VCE English Question Thread
Post by: zofromuxo on October 31, 2017, 05:30:13 pm
For the burial rites and medea section, are we allowed to write about both?? I've been told that we need to pick one book only and write an essay on that using the prompts given but what if i find burial rites too difficult in the exam, am i allowed to write about both?? Obviously on different papers
You are only able to write about one text for Section B A on the English exam. You can't write about both on the exam.
EDIT: K888 has informed me Section B is now comparative and not text response in the previous exams.
Title: Re: VCE English Question Thread
Post by: K888 on October 31, 2017, 05:35:15 pm
How many quotes would you recommend to memeorise for each of sections A and B? Should I memorise a quote for each theme?

And what make would I need to get a 36? A 7/10 for each essay or...?
Really up to you and what you feel comfortable with, I'd have a few quotes for each theme and a handful of quotes you could use for anything :) Having short (even 2 or 3 word) quotes up your sleeve is great and because they're short you can remember more!
You'll surprise yourself by how many quotes you remember when you're in the exam.

Not sure about scores, it would depend on how you went in SACs as well. I got A and A+ for GA1 & GA2 respectively, got 51/60 (8s and 9s) in the exam and got a 41. My advice would be, just aim to smash it out of the park in the exam and not worry yourself with scores :)

Hi, for comparative should you make comparisons between texts throughout the paragraph? and take a similar approach to comparative language analysis?
I usually structure my paras by talking about one text then i have a transitional comparative statemnt and start talking about the next text, but should i include more comparison throughout???

I can't comment on the comparative, but for language analysis - I don't think you're *actually* required to compare. My school and a lot of other schools have taught it this way though, and I think the idea is that it helps you understand the effect of language on the reader - and being able to show how the language impacts the reader differently from text to text probably reflects well on your language analysis skills. Just my thoughts, though.
I think I ended up putting in the analysis of the second text at the bottom of my paragraph analysing the first text, and combined my analysis so that I didn't have heaps of separate little paragraphs. It wasn't *that* much of a comparison, though, I just chucked in a few howevers, maybe compared different effects a bit, but your main aim is analysing the texts and the effect on the reader, not comparing the texts. :)

What you described about talking about one text, then having a transitional statement or two, and then launching into analysing the second text seems absolutely fine to me. You could also completely separate them, if you wanted to. I always preferred a similar method to the one you described, but it's horses for courses, really.
My advice would be don't change your structure dramatically given it's so close to the exam - stick to your guns! :)

Hey guys, does anyone know if we'll be marked down if we don't have a conclusion for Section C? I've heard some people say that's it's not that important... so could someone please confirm this? Thx  ;)
Nah, shouldn't really affect you at all. It's better to have complete body paragraphs and no conclusion rather than incomplete body paragraphs and a full conclusion. I'd recommend even just chucking a short one in (only has to be a few sentences - I think mine was maybe 2 or 3 and written in about 30 seconds haha) for completeness, but definitely not the end of the world if you don't have one. :)

Ideally you’d want more than one transitional comparative statement. So you should include more comparison throughout, cause comparison is a major criteria.
Is it, though? I think there was a debate on this somewhere in this board.
Here's the criteria from the 2017 sample exam:
Section C will be assessed against the following criteria:
• understanding of the argument(s) presented and point(s) of view expressed
• analysis of ways in which language and visual features are used to present an argument and to
persuade
• control and effectiveness of language use, as appropriate to the task

Anyone have a simple, easy acronym for language analysis structure? Like what to include in the intro and how to lay out each body paragraph?
I personally used ITAPTACTV for my intro, and just sort of went with the flow for my body paras. I know some people use "Support, Condemn, Do" and others use "People, Groups, Things" to order paragraphs, but I always just sort of analysed things that fit together or complemented each other in my body paras.
My general structure for each bit of analysis was
- the quote/strategy
- explain it - what's the author doing in this?
- what's the effect on the reader? what does it make them think/feel/do? how and why does it make them support or agree with the author?

You are only able to write about one text for Section B on the English exam. You can't write about both on the exam.
Actually, Section B is the comparative section - so they do need to compare texts. Section A is where you only write about one text - it's the traditional text response.
Assuming that manisha99 is asking about Section A, will provide an answer in the next post.
 
Title: Re: VCE English Question Thread
Post by: K888 on October 31, 2017, 05:42:41 pm
Double posting because of the length of my post.

For the burial rites and medea section, are we allowed to write about both?? I've been told that we need to pick one book only and write an essay on that using the prompts given but what if i find burial rites too difficult in the exam, am i allowed to write about both?? Obviously on different papers
So, I assume we're talking about Section A here.

You have to pick one book, pick one prompt, and write an essay on it. You can only write one essay for Section A. If you find the prompts for Burial Rites too difficult in the exam, then respond to one of the Medea prompts. :)

As it is said in the exam instructions:
"Section A requires students to write an analytical interpretation of a selected text in response to one topic (either i. or ii.) on one text."

Hope this has cleared things up :)
Title: Re: VCE English Question Thread
Post by: manisha99 on October 31, 2017, 06:06:59 pm
Double posting because of the length of my post.
So, I assume we're talking about Section A here.

You have to pick one book, pick one prompt, and write an essay on it. You can only write one essay for Section A. If you find the prompts for Burial Rites too difficult in the exam, then respond to one of the Medea prompts. :)

As it is said in the exam instructions:
"Section A requires students to write an analytical interpretation of a selected text in response to one topic (either i. or ii.) on one text."

Hope this has cleared things up :)

Okay thanks for clearing that up. Is there anything you recommend me to do to get a good study score?
I have looked at a few themes in Burial Rites, Medea, 1984 and Stasiland and characters and quotes
Title: Re: VCE English Question Thread
Post by: lilyrosee on October 31, 2017, 06:28:53 pm
How is everyone feeling for tomorrow?

Good luck to you all  :)
Title: Re: VCE English Question Thread
Post by: zxcvbnm18 on October 31, 2017, 06:43:39 pm
GOOD LUCK EVERYONE
Title: Re: VCE English Question Thread
Post by: K888 on October 31, 2017, 06:45:52 pm
Okay thanks for clearing that up. Is there anything you recommend me to do to get a good study score?
I have looked at a few themes in Burial Rites, Medea, 1984 and Stasiland and characters and quotes
Such an ambiguous question haha.

Just believe in yourself, know your themes/quotes/texts in general, know what the examiners are looking for (read the criteria!), and let your writing do the talking! :)



Good luck everyone! You'll smash it :) Can't wait to hear how it goes once I've finished my exam tomorrow afternoon! Make sure you come back on and share your thoughts in the exam discussion board. :)
Title: Re: VCE English Question Thread
Post by: kiki. on October 31, 2017, 07:06:44 pm
Good luck everyone!! :)

I'm nervous hahaha. How is the spacing of everyone's exams? I have so much time between each one which is both a blessing and a curse.
Title: Re: VCE English Question Thread
Post by: lyoko on October 31, 2017, 09:52:46 pm
GOOD LUCK EVERYONE! Just think, by this time tomorrow, it will all be over!
Title: Re: VCE English Question Thread
Post by: pmmenotes on October 31, 2017, 10:43:45 pm
Just wanted to say good luck to everyone here and Thanks so much for getting me through english this year!!! I’m sure we will all kill it tomorrow good night and good luck !
Title: Re: VCE English Question Thread
Post by: appleandbee on October 31, 2017, 10:44:53 pm
Good luck for tomorrow!

Hopefully the odds are in your favor for section A and B prompts. For lang analysis, given that many of you are aiming for high scores and worked hard, if you are disappointed or found the pieces difficult, remember that most people probably feel the same way. If you are disappointed with the exam overall a) it may just feel that way because you worked hard and aimed high, hopefully it turns out better than you first thought, b) in the worst scenario, a few points difference in your English SS (42 as opposed to a 45 or a 35 as opposed to a 38) won't make a significant difference to your Atar unless you are aiming for the top end (99+) where the margin for error is really small, the deficit can be made up in your other subjects so keep your head up. If venting or discussing the exam gives you the peace of mind, use the exam discussion boards. If you think doing so may make you feel more anxious or distract you from your upcoming exams, it's probably best to stay away from there.

A few last minutes bits of advice, for lang analysis, pay attention to the form of the pieces (whether it's a speech or a televised one, a letter, proposal etc.) as various unique stylistic devices may be included.  As long as you discuss the author/speaker's intentions or what the technique is designed to do, followed by how the reader is positioned to react you will be perfectly fine. Lang analysis pieces don't have to be read beautifully.

For text analysis stuff, focus on the way in which stylistic/linguistic features of the text is used to create meaning on various themes, characters and events. Try to reach the essence of the text (yes, it's philosophical) and question how the question or thing the prompt is discussing is central to unlocking the meaning in the text.

Hopefully most of you are getting some rest, but I'm be around to answer last minute questions, essay feedback or All About Eve tidbits.  :)
Title: Re: VCE English Question Thread
Post by: Joseph41 on November 01, 2017, 06:56:32 am
A huge good luck to everybody sitting the exam today. Can't wait to hear your thoughts this afternoon!
Title: Re: VCE English Question Thread
Post by: Mariamnourine on November 01, 2017, 07:00:42 am
A huge good luck to everybody sitting the exam today. Can't wait to hear your thoughts this afternoon!

Thank you so much. I have to admit I am terrified of those examiners [and the exam obvs]. Let's hope all goes well
Title: Re: VCE English Question Thread
Post by: Joseph41 on November 01, 2017, 07:02:11 am
Thank you so much. I have to admit I am terrified of those examiners [and the exam obvs]. Let's hope all goes well

I'm sure it will - the exam'll be sweet once you get stuck in. :)
Title: Re: VCE English Question Thread
Post by: Joseph41 on November 01, 2017, 12:17:20 pm
CONGRATULATIONS FOR FINISHING!

For all post-exam discussion and questions, please head to this thread here. Can't wait to hear how you all went! :D
Title: Re: VCE English Question Thread
Post by: superficious on November 01, 2017, 02:19:00 pm
So I walked in my exam, sat down and waited patiently for the reading time to begin. When I opened my paper I literally started laughing, every single theme I’ve studied and focused on was there;
Forgiveness/Revenge for the Comparison
Nature for the text response

I honestly spaced out an hour for each essay and with 55 mins to go, I started the language analysis. I blanked out and ended up writing 2 pages of bullshii, way to go am I right?
Title: Re: VCE English Question Thread
Post by: Mariamnourine on November 01, 2017, 03:15:26 pm
I felt exactly the same way. I panicked when I entered the exam room because I was expecting something bizarre and outta this world, Then when I looked at the prompts I wished I never stressed about English. The prompts for comparative and Text response were soooooooo easy. And can we talk about that section C article? way tooo easy, it was short and bursting with techniques. I guess I should stop overreacting about exams. All in all, happy that I am satisfied with my performance AND ecstatic that I will NEVER have t sit a 3 hour English exam in my life! [Btw did any one else finish like 15 minutes early, because I was freaked out that I wouldn't finish on time, then I was shocked when I realized that there was plenty of time remaining to proof read my essays}
Title: Re: VCE English Question Thread
Post by: c_niloo on November 05, 2017, 09:19:18 pm
Hi big question here, (year 10 student)
In school we've been doing text response essays all year as well as recently starting comparitive but I can't for some reason improve in creating more ideas or analysis in my explanation. I have a braindead teacher and I've searched and tried all the tips on this forum I can't get that writing mentality so I've decided to ask the question myself. Specifically, I find my self just reiterating the same point throughout my body paragraphs or sounding unneccesarily verbose. For example in a normal tr essay for Macbeth with a prompt asking to discuss ambition all I find myself doing is just saying- ambition is a dangerous quality capable of wracking a persons mind if not tamed>>>shakespeare exemplifies through Macbeth the unsuspecting nature of unbridled ambition, highlighting a heavy lust for power and murderous tendency as symptomatic of its atrocious ability.  See how they both kinda make the same point, thats what the majority of my paragraphs end up repeating for any prompt. I want to go further than just saying ambition is dangerous and makes someone wreck havoc.  Thanks and appreciate any answers to help me :D
Title: Re: VCE English Question Thread
Post by: Willba99 on November 11, 2017, 06:23:23 pm
Hi big question here, (year 10 student)
In school we've been doing text response essays all year as well as recently starting comparitive but I can't for some reason improve in creating more ideas or analysis in my explanation. I have a braindead teacher and I've searched and tried all the tips on this forum I can't get that writing mentality so I've decided to ask the question myself. Specifically, I find my self just reiterating the same point throughout my body paragraphs or sounding unneccesarily verbose. For example in a normal tr essay for Macbeth with a prompt asking to discuss ambition all I find myself doing is just saying- ambition is a dangerous quality capable of wracking a persons mind if not tamed>>>shakespeare exemplifies through Macbeth the unsuspecting nature of unbridled ambition, highlighting a heavy lust for power and murderous tendency as symptomatic of its atrocious ability.  See how they both kinda make the same point, thats what the majority of my paragraphs end up repeating for any prompt. I want to go further than just saying ambition is dangerous and makes someone wreck havoc.  Thanks and appreciate any answers to help me :D

I struggled with the same issue all year so, even though I wouldn't consider myself a gun at English, I'll tell you how I dealt with it.

I found that the best way to deal with these sort of problems was to have a really diverse knowledge of the text. That means, yeah still talk about the big flagship themes and characters, but don't be afraid to dedicate a paragraph to a lesser character or idea. I think that, especially in Year 12 exams, having an exam thats a bit different to the tens of thousands of others can only be a bonus that will work in your favour. To try and accomplish this, my advice would be to a) read as many high-level responses from different schools/sources as possible, so you get ideas from multiple sources, not just the ones readily available at your school, and b) try and be a bit adventurous in your ideas.

You mentioned that you sometimes have problems due to being too "verbose". I think that a really good way to deal with this is to look back at an essay after you've read it and try to cut down as many words as possible. Try and turn a 1200 word response into a 600 word one by cutting out unnecessary fluff and replacing it with concise verbs and adjectives. In this way, you can show off your vocabulary but still be direct, but you will also discover if you have enough concepts for an entire essay. Heids wrote the best AN article ever on this topic (https://atarnotes.com/forum/index.php?topic=162777.0) and it included a link to a really interesting essay by George Orwell (https://www.npr.org/blogs/ombudsman/Politics_and_the_English_Language-1.pdf) on the same topic. They're both definitely worth a read.

Good luck with VCE!
Title: Re: VCE English Question Thread
Post by: Sigma on November 12, 2017, 12:05:56 pm
Exam topic for practice English exam. Compare the impact of surveillance on the lives of citizens in both Nineteen Eighty-Four
and Stasiland.

Could someone please give me three themes and example for each. Thanks.
Title: Re: VCE English Question Thread
Post by: zhen on November 12, 2017, 12:35:22 pm
Exam topic for practice English exam. Compare the impact of surveillance on the lives of citizens in both Nineteen Eighty-Four
and Stasiland.

Could someone please give me three themes and example for each. Thanks.
I’m procrastinating right now, so I might as well help you out. My body paragraphs for this were 1) Physical and psychological ramifications of surveillance. I discussed Julia and Winston and how surveillance influenced their mind and spirit. 2) Surveillance and the effect upon relationships. I discussed how surveillance destroyed trust between people and discussed internal emigration in Stasiland and the how informers, which is a method of surveillance destroys trust. Also, here would be a great place to discuss Mrs Parsons and her children. 3) The nature of surveillance to restrict individual freedom.
Title: Re: VCE English Question Thread
Post by: Sigma on November 12, 2017, 02:19:50 pm
Thanks for your response. Zhen.
Title: Re: VCE English Question Thread
Post by: snowisawesome on November 30, 2017, 02:48:46 pm
What sac marks and exam marks do you need to get to get a 30 raw in English?
Thanks
Is memorising essays a good idea to get a 30 study score raw in english?
Title: Re: VCE English Question Thread
Post by: snowisawesome on December 07, 2017, 07:12:59 pm
Is there a way to understand the English texts you're reading? I've just finished the first 2 chapters of stasiland and I'm struggling to remember what has happened so far (first 2 chapters) and ended up having to use study guides for chapter summaries
Title: Re: VCE English Question Thread
Post by: clarke54321 on December 07, 2017, 08:44:10 pm
Is there a way to understand the English texts you're reading? I've just finished the first 2 chapters of stasiland and I'm struggling to remember what has happened so far (first 2 chapters) and ended up having to use study guides for chapter summaries

Hello!

That can be quite a common problem when studying more difficult texts. I'd suggest that you try and annotate the text as you are reading it. That is, take notes beside certain phrases or words. This ensures that you are fully engaging with all the information on the page- not just skimming it through without thought. You may also choose to write your own little summary after each chapter. However, what you're doing at the moment (reading study guide chapter summaries) is by no means a bad thing! I often did this for both English and Literature. It meant that I could join latent dots and continue reading the text with certainty.

I've personally never studied Stasiland, only read it. Much of the text is reliant on contextual knowledge. So you may find that doing some research on Germany during the relevant time period may help you out.

All the best  :)
Title: Re: VCE English Question Thread
Post by: abbeyrose on December 07, 2017, 09:47:15 pm
Is memorising essays a good idea to get a 30 study score raw in english?

Unless you mean for a sac where you've been given the prompts beforehand (which in that case then sure, go ahead), frankly I don't think memorising essays is ever a good idea. The examiners can tell when someone has memorised an essay and they express their disapproval of the practice quite frequently in the examination reports. Examiners look to reward people who offer insightful interpretations and engage with the prompt. Through rewriting an essay that you've previously written, even if the prompts are similar in the themes or characters they discuss, you run at high risk of not engaging with the prompt and offering an insightful analysis. I think it would be easier to study to understand the texts and develop a repertoire of your own interpretations that you can use and adapt to a prompt, than studying to try and memorise 800 or so words (if psych taught me anything it's that elaborative rehearsal > maintenance rehearsal ;) ). Sure, maybe memorise the way you discuss some themes or narrative conventions and use it if it relates to your contention and deepens your analysis, but definitely don't look at a prompt and think "yeah, I wrote an essay about this theme/prompt type before" and then rewrite that essay. You have more chance of getting a good score if the examiner can see that you've thought about and engaged with the prompt, rather than memorising an essay :)
Title: Re: VCE English Question Thread
Post by: snowisawesome on December 23, 2017, 08:24:44 pm
How many words should we write for english sacs in year 12 (comparitive essays, language analysis, short stories, etc)?
Title: Re: VCE English Question Thread
Post by: LifeisaConstantStruggle on December 23, 2017, 08:35:18 pm
How many words should we write for english sacs in year 12 (comparitive essays, language analysis, short stories, etc)?

It really depends on your ability to convey yourself and the task you are given. Normally in my school we are instructed to write 800-1000 words regardless of the SAC we do, but high scoring students normally write 1000-1200 words so that's usually where they target. Then again some people might need more words to say what they are trying to say so this is a really inaccurate scale to measure quality.
Title: Re: VCE English Question Thread
Post by: snowisawesome on December 24, 2017, 10:10:07 am
It really depends on your ability to convey yourself and the task you are given. Normally in my school we are instructed to write 800-1000 words regardless of the SAC we do, but high scoring students normally write 1000-1200 words so that's usually where they target. Then again some people might need more words to say what they are trying to say so this is a really inaccurate scale to measure quality.
Thank you :)
Title: Re: VCE English Question Thread
Post by: Dr. Nick on January 07, 2018, 05:46:27 pm
Does spelling influence the mark you get in vce english?
Its my greatest weakness... I would consider myself a fairly good writer in terms of essays and stuff but I suck at spelling, will this bring my marks down by a lot?
Title: Re: VCE English Question Thread
Post by: clarke54321 on January 07, 2018, 06:05:30 pm
Does spelling influence the mark you get in vce english?
Its my greatest weakness... I would consider myself a fairly good writer in terms of essays and stuff but I suck at spelling, will this bring my marks down by a lot?

Hello Dr. Nick,

If you're making the occasional spelling error, then it won't adversely influence your marks. However, if inaccurate spelling becomes a frequent occurrence in your essay, then yes, you're most likely to lose marks. Precise and accurate language application is part of the criteria. And while assessors do not rigidly mark essays with a checklist beside them, they can, from a holistic overview of the essay, gauge your skill in this area.

Here are some links, which outline criteria:

http://www.vcaa.vic.edu.au/Documents/exams/english/english-crit-descriptors-w.pdf
http://www.vcaa.vic.edu.au/Pages/vce/studies/english/index.aspx#H2N10088 (under the subtitle "support material" click on "advice for teachers")

All the very best :)
Title: Re: VCE English Question Thread
Post by: snowisawesome on January 08, 2018, 05:09:56 pm
Does anyone know what it means when we have to give a brief explanation of a quote? Like what do we need to specifically talk about etc?
Title: Re: VCE English Question Thread
Post by: clarke54321 on January 08, 2018, 05:26:45 pm
Does anyone know what it means when we have to give a brief explanation of a quote? Like what do we need to specifically talk about etc?

Hello!

When you say ‘brief explanation of a quote,’ do you mean in terms of an essay? Or is this a homework task?

If this is a homework task, perhaps your teacher is looking for you to pull out the main view/value of the evidence snippet. What is the main idea that is coming through? In an essay, a brief explanation may mean providing some context to the evidence.

We may be able to help you out more if you can clarify your question :)
Title: Re: VCE English Question Thread
Post by: snowisawesome on January 08, 2018, 05:29:24 pm
Hello!

When you say ‘brief explanation of a quote,’ do you mean in terms of an essay? Or is this a homework task?

If this is a homework task, perhaps your teacher is looking for you to pull out the main view/value of the evidence snippet. What is the main idea that is coming through? In an essay, a brief explanation may mean providing some context to the evidence.

We may be able to help you out more if you can clarify your question :)
It's a homework task, so would that make a difference?
Thanks
Title: Re: VCE English Question Thread
Post by: Sine on January 08, 2018, 05:32:49 pm
homework task would mean less of a context in contrast to using it within an essay.
For your task try to explain the possible interpretations of the quote and any ideas you can synthesise from it.
Title: Re: VCE English Question Thread
Post by: ilikenoodlez on January 08, 2018, 10:16:55 pm
Is it necessary to read your english books? I've finished reading the Crucible and 1/3 of the way with Left Hand of Darkness and I honestly can't tell if my understanding of the texts has improved compared to when i only read the online summaries. I'm also a slow reader and it takes me ages to finish a book, and english is really boring for me even though i try to enjoy it
Title: Re: VCE English Question Thread
Post by: Sine on January 08, 2018, 10:33:47 pm
How is it possible to gain a deeper understanding of texts without the proper resources? For example, I'm going to have to write a creative piece on 'Like a House on Fire' by Cate Kennedy, and I haven't come across any resources online that could help me gain a better understanding about the short stories in her novel.

Also, on a different note, how important is it to include historical context in essays? I've started to do this as a habit, but is it always required?

Thanks.
As for historical context it's quite useful to show the asessor that the piece of work was not done in isolation and obviously influenced by exterior sources. I used to try to include a little bit of history whilst I introduced the text. (text response)
Title: Re: VCE English Question Thread
Post by: lovelyperson on January 08, 2018, 11:01:55 pm
How is it possible to gain a deeper understanding of texts without the proper resources? For example, I'm going to have to write a creative piece on 'Like a House on Fire' by Cate Kennedy, and I haven't come across any resources online that could help me gain a better understanding about the short stories in her novel.

Also, on a different note, how important is it to include historical context in essays? I've started to do this as a habit, but is it always required?

Thanks.

to get a better understanding, talk about it with other people. challenge each other's interpretation, question your own. just keep asking yourself why? how? what if? what does this mean? resources will only get you so far; your own thinking and interpretation will get you there.

historical context is only important when relevant. don't just plonk it in for the sake of it.
Title: Re: VCE English Question Thread
Post by: clarke54321 on January 09, 2018, 06:35:00 am
Is it necessary to read your english books? I've finished reading the Crucible and 1/3 of the way with Left Hand of Darkness and I honestly can't tell if my understanding of the texts has improved compared to when i only read the online summaries. I'm also a slow reader and it takes me ages to finish a book, and english is really boring for me even though i try to enjoy it

Hello!

Unfortunately, reading texts in English is a necessity. This can be especially difficult when a text is boring, or is written in an odd style. Like yourself, I’ve always found that the online summaries make a text clearer (on the first reading). But this is generally in terms of plot only. All originality and nuanced ideas will stem from your own close reading of a text, meaning that it is fundamental that you read them yourself. Of course there are analyses available online, relating to symbolism/imagery/plot analysis. However, half of the state will be privy to this information, making it very difficult for you to set yourself apart.

To ensure that you don’t grow bored too quickly, try and set yourself allocated page numbers each day. Eg. Today I will read 20 pages of text X. And when you allocate yourself these pages, commit to being fully absorbed in the text. Annotate it, ask yourself questions about it, or write some notes beside yourself. This should ensure full engagement, and make it easier for you to understand the text.

All the best :)

How is it possible to gain a deeper understanding of texts without the proper resources? For example, I'm going to have to write a creative piece on 'Like a House on Fire' by Cate Kennedy, and I haven't come across any resources online that could help me gain a better understanding about the short stories in her novel.

Also, on a different note, how important is it to include historical context in essays? I've started to do this as a habit, but is it always required?

Thanks.

Hello!

Lovelyperson and Sine have already provided some fantastic advice, but I’ll just add some further ideas!

To gain a deeper understanding, I really recommend that you go through each short story and highlight important (only important) phrases or words that you think are central to either a character or certain theme. Once you’ve done this, transfer these highlighted segments onto a word document, where beneath each of them, you can carry out a 2-3 sentence analysis. Now, this analysis does not need to be in full academic English. It can be as simple as you finding the main view/value, asking questions, unearthing strange traits of a character. Although this task can become tedious at times, it will elevate your perspicacity by 10. You’ll start finding links that you’ve previously never come across, illuminating aspects of characters that were never clear, or establishing a deeper understanding of the author’s intent/style (a critical element for the creative response).

In relation to historical context, it’s always nice to weave elements throughout your essay. But as lovelyperson and Sine have mentioned, don’t turn this into a history lesson for the examiner. They are aware of a text’s background. All they want to know is that you can effectively link textual elements to context, and analyse the significance of this.

Hope this helps :)
Title: Re: VCE English Question Thread
Post by: snowisawesome on January 09, 2018, 12:05:04 pm
https://www.shmoop.com/1984/book-2-chapter-9-quotes-2.html


In Oceania at the present day, Science, in the old sense, has almost ceased to exist. In Newspeak there is no word for "Science." The empirical method of thought, on which all the scientific achievements of the past were founded, is opposed to the most fundamental principles of Ingsoc. And even technological progress only happens when its products can in some way be used for the diminution of human liberty.

I was getting quotes from 1984 and the website says that the above is a quote, but in the book it's not in inverted commas, so is it really a quote?
Title: Re: VCE English Question Thread
Post by: Sine on January 09, 2018, 12:10:24 pm
https://www.shmoop.com/1984/book-2-chapter-9-quotes-2.html


In Oceania at the present day, Science, in the old sense, has almost ceased to exist. In Newspeak there is no word for "Science." The empirical method of thought, on which all the scientific achievements of the past were founded, is opposed to the most fundamental principles of Ingsoc. And even technological progress only happens when its products can in some way be used for the diminution of human liberty.

I was getting quotes from 1984 and the website says that the above is a quote, also in the book it's not in inverted commas, so is it really a quote?
you can pick out anything from the book and use as a quote. When you are quoting something you yourself input the inverted commas to show that it is not yours originally.
Title: Re: VCE English Question Thread
Post by: snowisawesome on January 09, 2018, 12:14:26 pm
you can pick out anything from the book and use as a quote. When you are quoting something you yourself input the inverted commas to show that it is not yours originally.
But isn't there a difference between picking out anything from the book as a quote and someone speaking in the book as a quote?
Title: Re: VCE English Question Thread
Post by: clarke54321 on January 09, 2018, 01:22:26 pm
But isn't there a difference between picking out anything from the book as a quote and someone speaking in the book as a quote?

If you’re quoting something that has already been quoted in the text (meta), then add double marks, “ “, within the normal single marks, ‘ ‘. Here is a fantastic resource from UniMelb, which clarifies all these quotation concerns: http://services.unimelb.edu.au/__data/assets/pdf_file/0009/529776/Using_quotation_marks_Update_051112.pdf
Title: Re: VCE English Question Thread
Post by: snowisawesome on January 12, 2018, 03:25:05 pm
Is it generally possible to memorise essays and then fit it to the specific topic/prompt on a sac?
Title: Re: VCE English Question Thread
Post by: Lear on January 12, 2018, 03:34:55 pm
I read some where on atarnotes that someone wrote very general, big paragraphs on each theme with evidence and then just morphed that paragraph to fit any prompt in the exam. I find this to be a very smart thing to do but also wonder if it is the ‘correct’ way of going about writing well.
Title: Re: VCE English Question Thread
Post by: Calebark on January 12, 2018, 03:35:41 pm
Is it generally possible to memorise essays and then fit it to the specific topic/prompt on a sac?

I'd say it's possible. But I'd also say the essay would probably end up a bit on the shitty side. Your ideas aren't going to flow and it's probably going to feel like you're forcing the essay to move in an uncomfortable direction. I wouldn't recommend memorising essays.

A better approach would be to brainstorm a wide range of ideas, like the most common themes, symbols, and characters. This way allows you to be prepared for a wide variety of prompts while staying on topic.
Title: Re: VCE English Question Thread
Post by: clarke54321 on January 12, 2018, 03:40:46 pm
Is it generally possible to memorise essays and then fit it to the specific topic/prompt on a sac?

Hello!

Memorisation is a very controversial topic in English, and one that is received differently by each person.

In my experience, I've been able to mould pre-prepared and well defined paragraphs around essay prompts, in both SACs and the exam. Memorising an entire essay is somewhat fruitless. Memorising arguments and sections of paragraphs is, however, not a bad idea at all. Just ensure that you can manipulate these already existing ideas in a way that is authentic and natural.

Elements of essays that cannot be memorised are the introductions and conclusions. From the outset, you must alert the examiner to the fact that you are responding to the question before you, not the one you desire.

Hopefully this helps :)
Title: Re: VCE English Question Thread
Post by: Yertle the Turtle on January 12, 2018, 03:55:11 pm
Is it generally possible to memorise essays and then fit it to the specific topic/prompt on a sac?
Possible, but not necessarily a good idea. We had a SAC last year where we had to write a poem, memorize it and write it out in class, before analysing it from a literary point of view.
(Funny story about that, one of my friends used his name as the author in the analysis, and the substitute teacher we had corrected, incorrectly, his spelling of his own name!)
It is possible, therefore, to do this, but in general this will not fit well any given question, and therefore is not necessarily a good idea.
Title: Re: VCE English Question Thread
Post by: sophomania on January 12, 2018, 08:29:01 pm
Is it generally possible to memorise essays and then fit it to the specific topic/prompt on a sac?

It is possible, however some may argue that it is not a good idea. The problem with memorising essays is that you might be in danger of not answering the essay prompt correctly.

Personally, I never memorised my essays. I kind of roughly memorised specific examples and the way I wrote about them, and other chunks of writing. However, it wasn’t like I sat down and tried to actively memorise them. It was just that I was using specific examples and ideas so often in my practice essays that I knew exactly how I was going to write about them.
Title: Re: VCE English Question Thread
Post by: Sunset.T on January 23, 2018, 01:28:53 pm
Hello
So for the holidays I’m supposed to be going through a booklet of articles about the topic of increasing the legal smoking age and annotating it.
I’m really struggling. I’m finding all the arguments first but it’s so hard for me to identify the beginning and end of each argument and it feels like the entire piece is just one argument.

Then when I do find an argument I feel like every piece has the same arguments and it’s annoying as. And every piece seems to have 3 arguments but I’m not sure if they actually do have 3 arguments or if I’m just assuming they all have 3 so that they match. Idk
Title: Re: VCE English Question Thread
Post by: clarke54321 on January 23, 2018, 01:42:56 pm
Hello
So for the holidays I’m supposed to be going through a booklet of articles about the topic of increasing the legal smoking age and annotating it.
I’m really struggling. I’m finding all the arguments first but it’s so hard for me to identify the beginning and end of each argument and it feels like the entire piece is just one argument.

Then when I do find an argument I feel like every piece has the same arguments and it’s annoying as. And every piece seems to have 3 arguments but I’m not sure if they actually do have 3 arguments or if I’m just assuming they all have 3 so that they match. Idk

Hello!

I can understand where you're coming from entirely. Identifying arguments can be extremely difficult when there is little variation in content/techniques. Despite this, there are some handy tips, which can improve your "argument locating" ability. Here are some:


-Find the main contention. In your case, this is likely to be, we should increase the legal smoking age. After you've done this, try and locate reasons why the age should be increased. These will be your arguments. They may include health benefits, economic benefits or social benefits.

-Analyse the way the article is constructed in terms of paragraphs. Generally when there is a new paragraph, there is a new train of thought/new direction that the author wants to explore. Think about it- whenever you write a text response or oral presentation, your paragraphs cover new material (further arguments that will support your main contention).

-Sense shifts in tone. Like paragraph, a shift in tone can signify a new pathway (in terms of argument). If the author suddenly takes on an encouraging/optimistic tone, they may be postulating the benefits of some kind of future action or benefit. If they decide to take on a more exigent voice, they may attempting to stress future urgency.


Hopefully these tips help you out  :)
Title: Re: VCE English Question Thread
Post by: MissSmiley on January 23, 2018, 01:47:21 pm
Hello
So for the holidays I’m supposed to be going through a booklet of articles about the topic of increasing the legal smoking age and annotating it.
I’m really struggling. I’m finding all the arguments first but it’s so hard for me to identify the beginning and end of each argument and it feels like the entire piece is just one argument.

Then when I do find an argument I feel like every piece has the same arguments and it’s annoying as. And every piece seems to have 3 arguments but I’m not sure if they actually do have 3 arguments or if I’m just assuming they all have 3 so that they match. Idk
Hello! :)
Firstly, is this argument analysis or are you preparing for your persuasive oral?

If the entire piece just feels like one big argument, then that's probably the contention of the writer.
There are subtle nuances that you need to consider to distinguish arguments.
Often, if the author is calling for action (in this case maybe to encourage increasing the legal smoking age), they would make 'arguments' on perhaps how exactly to increase it, the consequences of not increasing it or even the benefits of increasing it.
So if you think about it, every argument doesn't necessarily have to be persuasive (I'm only saying this because you're having a hard time finding them).
Sometimes the writer may refer to another study or someone else's opinions and provide a rebuttal or agree with it - that is sort of an 'argument.'
The writer may even compare Australia's legal age with say a different country and then talk about if an increased age in that country has benefited them, and so should happen in Australia so we can have a healthier environment.

Hope this helps just to guide you perhaps to analysise more closely and really dig deep into it! :)

(It'll be great if you could attach one of the articles, or if you can't then post the link of that article (a newspaper link) so we can read it and understand the arguments) :)
Or you can just take a photo of the article and just post it! It'll be really helpful! :)
Title: Re: VCE English Question Thread
Post by: Lear on January 23, 2018, 01:53:24 pm
It'll be great if you could attack one of the articles
Please don't attack one of the articles! :P
Anyway, I've been taught to look for choices in language first before trying to link them together to form an argument. More often than not I find arguments can be spread out through the piece and aren't necessarily in each paragraph. I find it easier to connect choices in language/techniques throughout to form arguments. Is this a viable method of doing it?
Title: Re: VCE English Question Thread
Post by: MissSmiley on January 23, 2018, 02:24:40 pm
Please don't attack one of the articles! :P
Anyway, I've been taught to look for choices in language first before trying to link them together to form an argument. More often than not I find arguments can be spread out through the piece and aren't necessarily in each paragraph. I find it easier to connect choices in language/techniques throughout to form arguments. Is this a viable method of doing it?
Hahahahahahaha!!!  ;D ;D ;D
I just can't stop laughing, Lear!!!! I meant to say 'attach' not 'attack'  ;D ;D :P

I've changed it now, so you're right, no need to attack an article!!  ;D ;D

And yes, I really like where you're coming from with understanding the purposes of the language devices and then coming to an argument.
I've never gone with this method, because I sometimes fall into the trap of generalising an argument. So I tend to stick to the paragraphing method of separating arguments, regardless of how the paragraphs and their arguments may be all over the place.

Thanks for making me laugh!!!!!  ;D ;D
Title: Re: VCE English Question Thread
Post by: Lear on January 23, 2018, 03:01:06 pm
Gave me quite a giggle too
Title: Re: VCE English Question Thread
Post by: Lear on January 23, 2018, 09:26:30 pm
I'm sorry if this has been asked before but how would I go about incorporating specific words into my writing? I feel there are three types of words for me. Ones I know and automatically use in writing, ones I know but never occur to me when writing and words I don't know. Say I have a list of words which I hope to include in my writing, what kind of techniques would I use to have them come naturally to me?
Title: Re: VCE English Question Thread
Post by: clarke54321 on January 23, 2018, 09:36:12 pm
I'm sorry if this has been asked before but how would I go about incorporating specific words into my writing? I feel there are three types of words for me. Ones I know and automatically use in writing, ones I know but never occur to me when writing and words I don't know. Say I have a list of words which I hope to include in my writing, what kind of techniques would I use to have them come naturally to me?


Hello!

Expanding vocabulary, in a conscious sense, can be very difficult in English. To make it feel more natural, I'd recommend that you construct text specific vocabulary lists. For each word, try and incorporate it into a sentence or two (that relates to your text). By actually applying the word, and not blindly writing it down, you will be more likely to remember it and its applicable usage when writing.

Hopefully this helps :)
Title: Re: VCE English Question Thread
Post by: Lear on January 23, 2018, 09:46:41 pm
Thanks for your response Clarke.
I am trying to incorporate some words into my Argument Analysis essays. I will try to have my vocab book open next to me when writing my next essay to see if it helps :)
Title: Re: VCE English Question Thread
Post by: skrt skrt on January 29, 2018, 11:05:24 pm
Can someone please give me a general direction on where to start on these creative responses.
The topic are -Write a piece about a place and time where both beauty and hardship exist
                      -Routines and rituals help maintain connections and security. Reflect on your own daily practices and ceremonies

Ive been sitting here for the past 30 minutes just staring into the abyss and as each minute goes by I question the importance of English(this is bad)
And also are there any general techniques on tackling these sort of questions or do you just write straight from the mind.
Thankss
Title: Re: VCE English Question Thread
Post by: Opengangs on January 29, 2018, 11:26:20 pm
Write a piece about a place and time where both beauty and hardship exist
Beauty in itself is very broad; it encompasses more than just complexions and the idea of "beautiful people". It's a characteristic associated to an object that encapsulates the feeling of pleasure and satisfaction. The perception of "ideal beauty" then can be translated into hardship really compactly in the sense where someone tries to capture "ideal beauty", yet their hardship comes when he realises and discovers that they always fall short of this characteristic. You can then place heavy emphasis to a time of the modern era, whereby society collectively views this "ideal beauty" as acceptable. You can choose to have your main character dislike their "societal" title; a lot of hardships can happen because of this. Just something to work with. :)

Routines and rituals help maintain connections and security. Reflect on your own daily practices and ceremonies
I sense a strong spiritual connection here; routines and rituals are very much of the "spiritual realm" so definitely have that vibe throughout your piece. Connections and security, again, give off a sense of inclusion and the will to act freely without the underlying fear of judgement, so in a way, the prompt entails that keeping up with your spiritual well being (routines and rituals) allows you to connect and understand deeper ideas about spirituality in a way where without spiritualism, there is materialism and judgement.

To reflect means to think deeper about something, so consider your own experiences with spiritualism. In what way does it give off a sense of inclusivity? And if so, how does it make you feel safe from judgement? A few ideas to get you started.

Though do take my ideas with a grain of salt; I'm from the HSC boards, but thought I'd drop by and chip in when I can! :)
Title: Re: VCE English Question Thread
Post by: 011116 on February 06, 2018, 09:21:15 pm
im currently struggling starting a text response which kind of leads to the question, what is the best way to start an introduction cos legit im completely blank  ;D
Title: Re: VCE English Question Thread
Post by: Sine on February 06, 2018, 09:22:25 pm
im currently struggling starting a text response which kind of leads to the question, what is the best way to start an introduction cos legit im completely blank  ;D
You need to know what you will talk about in your body paragraphs before you start your introduction so make sure you plan your essay :)

EDIT: if you still aren't quite sure many others and myself would be happy to help
Title: Re: VCE English Question Thread
Post by: clarke54321 on February 06, 2018, 09:36:20 pm
im currently struggling starting a text response which kind of leads to the question, what is the best way to start an introduction cos legit im completely blank  ;D

Hello!

The way you choose to start a text response is purely personal preference. Some like to start with a quote, some like to start with their contention, and others like to contextualise. After trialling a few of these, I felt that the best approach was the contextualisation one. Here is an example in response to the play, 'Medea'~

 Euripides’ tragedy 'Medea' seeks to examine the psyche of a woman betrayed by her husband’s infidelity.

By opening your essay with the verbs like, examines + explores, you are immediately pinpointing a crucial tension in the text. You can then flesh out this tension/conflict in the rest of your introduction (eg. arguments that support your own interpretation of the issue at hand).

But like Sine said, you must have a firm idea about your arguments/contention before you start writing.

All the best  :)
Title: Re: VCE English Question Thread
Post by: skrt skrt on February 07, 2018, 06:36:35 pm
Heyo, at my school we're studying Old/New poems and for my creative response sac I'm thinking about writing my personnel experience on migrating from my original country to Australia. The person who is receiving the letters is someone close to me so i can describe my emotions and thoughts with them on the migration experience.

Is this a good start for my sac??

Thanks
Title: Re: VCE English Question Thread
Post by: lovelyperson on February 07, 2018, 08:40:06 pm
Heyo, at my school we're studying Old/New poems and for my creative response sac I'm thinking about writing my personnel experience on migrating from my original country to Australia. The person who is receiving the letters is someone close to me so i can describe my emotions and thoughts with them on the migration experience.

Is this a good start for my sac??

Thanks

Tbh, you should check with your teacher - their word over anyone else's here, especially since creative writing is only a SAC and not an exam component. But, in general, one of the criteria for this unit is how well you capture stylistic features of the author (in this case poet); it'll be difficult for you to do this in letter form (although not impossible). Also, how will the letters explore the poems' ideas? Cause remember, your job is to creatively respond to a text, not just to write creatively whilst paying lip service to the set text's ideas and concerns.
Title: Re: VCE English Question Thread
Post by: MissSmiley on February 07, 2018, 08:58:13 pm
Check with your teacher - their word over anyone else's advice that is provided here. But, in general, one of the criteria for this unit is how well you capture stylistic features of the author (in this case poet); it'll be difficult for you to do this in letter form (although not impossible). Also, how will the letters explore your poems' ideas? Cause remember, your job is to creatively respond to a next, not just to write creatively and pay lip service to the set text's ideas and concerns.
Could I just ask whether writing a series of letters for the creative SAC is a good idea or not?
Stemming from what you said lovelyperson, does a letter limit the ability to write with insight or you know like the typical style of any narrative?

So would you be marked down for the simplistic language that you would use in a letter? (obviously you can't really get complex with your vocab too much in a letter, as you would in a narrative or like in a novel-style genre, but is that bad if you use simple language?)

Would it matter at all if you still incorporate the original text's values and central ideas?
Afterall, that's what counts doesn't it?

But is it in general that in a letter you get trapped into more of "tell" rather than "show"?

Sorry! so many questions !!

But would be really great if anyone helps out !! :)


Title: Re: VCE English Question Thread
Post by: lovelyperson on February 07, 2018, 09:10:36 pm
Could I just ask whether writing a series of letters for the creative SAC is a good idea or not?
Stemming from what you said lovelyperson, does a letter limit the ability to write with insight or you know like the typical style of any narrative?

So would you be marked down for the simplistic language that you would use in a letter? (obviously you can't really get complex with your vocab too much in a letter, as you would in a narrative or like in a novel-style genre, but is that bad if you use simple language?)

Would it matter at all if you still incorporate the original text's values and central ideas?
Afterall, that's what counts doesn't it?

But is it in general that in a letter you get trapped into more of "tell" rather than "show"?

Sorry! so many questions !!

But would be really great if anyone helps out !! :)

Not necessarily, but there are certain stylistic features that are unique to certain forms, which you might be expected to incorporate into your own work by, accordingly, writing in such forms. Key word here is might though; the creative sac doesn't appear on the exam so schools will more than likely be doing it different from another. So check with your teacher first - the advice I provide is only coming from the experience I had at my school.

And no - responding to your set text's views and values is only one part of the criteria. Imitating the stylistic features is another one. Also, you're not bound by the normal conventions of a form, so you don't necessarily have to write using simple language in a letter. This is where some of the creative licence lies; you can write a letter in however way you like as long as you're able to clearly justify it in your statement of intention.
Title: Re: VCE English Question Thread
Post by: Opengangs on February 07, 2018, 09:12:31 pm
Could I just ask whether writing a series of letters for the creative SAC is a good idea or not?
Stemming from what you said lovelyperson, does a letter limit the ability to write with insight or you know like the typical style of any narrative?

So would you be marked down for the simplistic language that you would use in a letter? (obviously you can't really get complex with your vocab too much in a letter, as you would in a narrative or like in a novel-style genre, but is that bad if you use simple language?)

Would it matter at all if you still incorporate the original text's values and central ideas?
Afterall, that's what counts doesn't it?

But is it in general that in a letter you get trapped into more of "tell" rather than "show"?

Sorry! so many questions !!

But would be really great if anyone helps out !! :)



Hey,
A creative wouldn't get marked down based on the form or narrative style you're writing in. I would believe that markers are looking for a diversified range of textual style, so if you're able to write in letter style, I think it would stand out to the thousands of students you're competing against. :) Even though it doesn't matter what style of writing you choose, your marks are depended on your manipulation of form and narrative structure. That is, are you able to compose an insightful and meaningful story within the allocated time in letter form?

Personally, I think a letter is a good choice in terms of new insights and building a personal relationship with the audience - but again, that depends on your word choice. To be effective doesn't mean it has to be filled with verbosity. Instead, your focus should be based around clarity. Your ideas need to be the foundation to your story; the rest will follow when you have a strong foundation to begin with. Having a wide vocabulary does help, but it's not necessary to be scoring high.

In terms of 'tell' and not 'show', yes - writing in a letter style is more prone to tell and not show - but again, it's up to how you interpret it. I think a letter will open up a more emotional attachment because you're essentially writing with emotion. To show means to use these emotions and to really captivate the reader into experiencing the journeys you as the composer makes. There are many elements within a letter that you could definitely employ within your writing.

Hopefully, this sheds some light and to encourage you with your letter styled creative. Definitely a good idea, but make sure you are able to do so. :)
Title: Re: VCE English Question Thread
Post by: MissSmiley on February 07, 2018, 09:46:54 pm
Thanks so much lovelyperson and Opengangs! :)
I really appreciate it! :)

Would be great to hear some more views and even better if any of last year's students who wrote a letter for their Creative SAC shared their thoughts about their SAC!!  ;D

Thanks guys!! :)
Title: Re: VCE English Question Thread
Post by: Gogo14 on February 08, 2018, 09:56:17 pm
Thanks so much lovelyperson and Opengangs! :)
I really appreciate it! :)

Would be great to hear some more views and even better if any of last year's students who wrote a letter for their Creative SAC shared their thoughts about their SAC!!  ;D

Thanks guys!! :)

Hey,
So last year our text was "This Boy's Life" and a lot of my friends did letters for their creative sac... but it was also an oral as well. So a lot of students did a presentation where they were reading out the letter on stage, or even talking about their thoughts whilst writing the letter. Many people who did the sac in a letter format did really well (acing the sac), so don't be afraid to experiment with different forms! Don't be afraid to get creative!

You won't get marked down if you did a letter, as teachers are really just assessing 1. the content of your piece (how it explores themes etc) 2. writing (how well your piece is written/presented) 3. relevance/understanding (how you demonstrate knowledge of the book). Those are main criteria I can think of right now, but best be sure to check with your teacher. Also, I think doing it in the form of letters gives you lots of opportunity to open up new and creative perspectives in the book so you can really flesh out themes/characters/ideas. In my opinion the major downside is that if you are planning to write a bunch of consecutive letters between characters (repetitively going back and forth),  you have to be careful not to make it repetitive; if you are planning something like a dialogue, then I suggest writing it as a play script.
Title: Re: VCE English Question Thread
Post by: Guideme on February 11, 2018, 10:07:51 am
What is the difference between invoke and evoke

Thx in Advance!
I know it is a completely random question but I am actually confused and curious haha :)
Title: Re: VCE English Question Thread
Post by: MissSmiley on February 11, 2018, 10:32:56 am
What is the difference between invoke and evoke

Thx in Advance!
I know it is a completely random question but I am actually confused and curious haha :)
Bit hard to pinpoint the difference, isn't it?
To me,
invoke = calling on, appealing to someone to take action, or to support your argument. If you invoke something, it'll be more effective to reduce the social distance (sorry for using the Eng Lang metalanguage, but still useful here :) )between you and the reader.
for e.g. If you invoke a law, you state that you are taking a particular action because that law allows or tells you to.

evoke = to make someone bring or recall a memory, feeling or image to their conscious mind. Can you see how evoking something is more of the 'on the surface approach'? Because you can't be certain that your readers will actually evoke those emotions or whatever in their minds...
for e.g. The silent garden evoked sounds of birds.

Do you see how invoke is more like 'dynamic'? as it prompts more to do with actions? whereas evoke is just the feelings behind those actions, if you like.

Sorry if I've confused you!! But hopefully you understand!  :D
Title: Re: VCE English Question Thread
Post by: sophiayim on February 12, 2018, 05:36:46 pm
Anyone studying the The Lieutenant by Kate Grenville?? Can anyone help me in terms of Silk's perception of reality compared to Rookes'
Thank you in advance
I know it is alot but if anyone can help me further I would really like help in regards to chapter and character summaries....
Title: Re: VCE English Question Thread
Post by: lovelyperson on February 12, 2018, 07:03:41 pm
Anyone studying the The Lieutenant by Kate Grenville?? Can anyone help me in terms of Silk's perception of reality compared to Rookes'
Thank you in advance
I know it is alot but if anyone can help me further I would really like help in regards to chapter and character summaries....

Haven't touched the Lieutenant in ageessssssss but ...

The most important thing to consider is the character's different values. Both Rooke and Silke place differing levels of importance on different things, and this affects how they view the world around them, and most importantly, their actions as white settlers trying to colonise the 'newly discovered' land. For instance, Silk is concerned mainly about his own self-interests (evident in how he writes the book to make the $$), whilst Rooke couldn't care less about himself and is willing to go as far as mutiny to do the right thing. Moreover, Rooke is an outsider (this is verrryyy !!!!!!important!!!!! Grenville did not spend an entire chunk of the novel talking about how Rooke was a loner for most of his life for no reason) whilst Silk is some preppy rich white boy.

There's so many differences in their values that consequently affect their perception of reality - you could even talk about their military rank really and how perceived power influence situations involving people that are supposedly 'inferior'. Hopefully the two I gave you is helpful and a starting point for you!

With regards to chapter and character summaries, what exactly are you having trouble with? 
Title: Re: VCE English Question Thread
Post by: vceme on February 12, 2018, 07:41:58 pm
Hey!
I was wondering if anyone could give me any advice on writing a creative piece based on a film, such as Rear Window? Also, if anyone has any advice on adopting a similar voice to the characters that would be great! Please keep in mind, i dont get the sac topics until the day (rip)
( also any advice on writing under pressure would be great)
Thank you in advance  :) ;D
Title: Re: VCE English Question Thread
Post by: clarke54321 on February 12, 2018, 08:26:44 pm
Hey!
I was wondering if anyone could give me any advice on writing a creative piece based on a film, such as Rear Window? Also, if anyone has any advice on adopting a similar voice to the characters that would be great! Please keep in mind, i dont get the sac topics until the day (rip)
( also any advice on writing under pressure would be great)
Thank you in advance  :) ;D

Hello vceme :)

Writing a creative piece based on a film, can be tackled in the same way that you might respond to either a novel or play. Take note of certain gaps in the plot or characterisation of characters. If you choose to go down the plot path, think to yourself, what might have happened prior to this moment or what might happen after this moment. Ambiguity is something to look out for with creative pieces, because it gives you the perfect opportunity to implement your own original insight.

It's excellent to see that you appreciate the significance of character voice! This is an incredibly important facet to any piece of creative writing (especially in adaptation format). To establish a voice consistent with that of your main character, I'd encourage you to go through your text and note 5-10 quotes/film techniques, which you think are pertinent to the construction of your character. Beneath each piece of evidence, spend some time brainstorming what each reveal/imply/intimate/suggest. You'll be surprised by the wealth of hidden knowledge that will emerge. If you need any further help with the creative, check out some of my guide tips.

And as for writing under pressure, this unfortunately comes with experience. But with an assessment like the creative, you can loosely form your piece before the SAC (even if the prompts aren't released at an earlier stage). All you'll have to do is manipulate elements of your response, so as to adequately engage with the relevant prompt.

Hopefully this helps!
Title: Re: VCE English Question Thread
Post by: 011116 on February 12, 2018, 09:56:26 pm
tragedian-playwriter Euripides' play Medea highlights the struggles that were faced by women in a society where women were seen as equals to slaves, in order to depict medea as a  heroic figure more than a villain. written to criticise the mistreatment of women during 431 BC, euripides shows the damage tht can occur when a women is disregarded and mistreated.

can someone tell where to improve on in this introduction
prompt: was medea a hero or villain, discuss?
Title: Re: VCE English Question Thread
Post by: Yertle the Turtle on February 12, 2018, 10:06:58 pm
tragedian-playwriter Euripides' play Medea highlights the struggles that were faced by women in a society where women were seen as equals to slaves, in order to depict medea as a  heroic figure more than a villain. written to criticise the mistreatment of women during 431 BC, euripides shows the damage tht can occur when a women is disregarded and mistreated.

can someone tell where to improve on in this introduction
prompt: was medea a hero or villain, discuss?
Looks pretty good, maybe make mention of your three prongs (three main ideas and arguments). I'm doing Medea as well, and I really had difficulty reading it because it was just such an awkward book to read and was boring.

Anyone else doing Skzynecki's poetry? I'm not finding it too bad... It's pretty easy to learn, but it's pretty hard to write about since it is so easy to interpret in any way, but there's no real substance to put into an essay.
Title: Re: VCE English Question Thread
Post by: MissSmiley on February 12, 2018, 10:30:18 pm
tragedian-playwriter Euripides' play Medea highlights the struggles that were faced by women in a society where women were seen as equals to slaves, in order to depict medea as a  heroic figure more than a villain. written to criticise the mistreatment of women during 431 BC, euripides shows the damage tht can occur when a women is disregarded and mistreated.

can someone tell where to improve on in this introduction
prompt: was medea a hero or villain, discuss?
Hi vceme ! :)

Tragediany-playwriter playwright Euripides, through his play Medea you know what? you could even say "Through his tragic play Medea,Euripides...just to sound simpler :)highlights the struggles that were faced by women in a society where women they were seen as equals to slaves, really? I think this is a bit of exaggerating. Like I hear your disapproval in their treatment, :) but could you say "where women were down played" maybe? in order to depict medea as a  heroic figure more than a villain. what makes Medea a heroic figure? you sound like you're saying that just the fact that women were looked down upon, made Medea a heroic figure. You need to mention something like because she attempted to challenge the patriarchy, that can suggest she's a heroine. . One of the purposes in writing the play,(just so you're acknowledging that there were many purposes in writing the play, not just criticising mistreatment of women) :)  Euripides criticises the mistreatment of women during 431 BC which shows the damage that can occur when a womenan is disregarded and mistreated. add more to this sentence to link this back to the prompt. So maybe something like Euripides, through Medea's revengeful persona warns of the damages that can occur when women are disregarded and mistreated, in this case, overthrowing one's status in society to appear godly (deus ex machina event).  And then as Marvin K. Mooney said, you'd need to signpost a bit. :)

Hope this helps ! :)
Title: Re: VCE English Question Thread
Post by: 011116 on February 12, 2018, 10:41:46 pm
Hi vceme ! :)

Tragediany-playwriter playwright Euripides, through his play Medea you know what? you could even say "Through his tragic play Medea,Euripides...just to sound simpler :)highlights the struggles that were faced by women in a society where women they were seen as equals to slaves, really? I think this is a bit of exaggerating. Like I hear your disapproval in their treatment, :) but could you say "where women were down played" maybe? in order to depict medea as a  heroic figure more than a villain. what makes Medea a heroic figure? you sound like you're saying that just the fact that women were looked down upon, made Medea a heroic figure. You need to mention something like because she attempted to challenge the patriarchy, that can suggest she's a heroine. . One of the purposes in writing the play,(just so you're acknowledging that there were many purposes in writing the play, not just criticising mistreatment of women) :)  Euripides criticises the mistreatment of women during 431 BC which shows the damage that can occur when a womenan is disregarded and mistreated. add more to this sentence to link this back to the prompt. So maybe something like Euripides, through Medea's revengeful persona warns of the damages that can occur when women are disregarded and mistreated, in this case, overthrowing one's status in society to appear godly (deus ex machina event).  And then as Marvin K. Mooney said, you'd need to signpost a bit. :)

Hope this helps ! :)
Thank you for the feedback, but u know how u said "where women were played down" i dont get what u mean by that  :) thanks again
Title: Re: VCE English Question Thread
Post by: 011116 on February 12, 2018, 11:03:49 pm
Through his tragic play Medea, Euripides highlights the struggles that women faced, in a patriarchal society, in order to portray Medea as a heroic figure who defends the right of women. Furthermore, Euripides criticizes the mistreatment of women and emphasizes the damage that a patriarchal society can do to a woman when she attempts to break free from societal norms

how is this?  ;D ;D ;D
Title: Re: VCE English Question Thread
Post by: MissSmiley on February 12, 2018, 11:22:59 pm
Thank you for the feedback, but u know how u said "where women were played down" i dont get what u mean by that  :) thanks again
;D ;D Yeah so I said "down played" :)
Does that clear it up?

So they were given lesser importance than males, treated like they are had no value, no importance.

Is that all right?
Title: Re: VCE English Question Thread
Post by: lilyrosee on February 12, 2018, 11:26:12 pm
Through his tragic play Medea (too wordy - try Euripides' tragedy highlights...), Euripides highlights the struggles (find a stronger word here) that women faced, in a patriarchal society, (too many commas halts the fluidity of your writing - just say 'women in a patriarchal society') in order to portray Medea as a heroic figure who defends the right of women (you need to be more specifc than just the 'rights of women', any one can mention that - what specfically does Medea do that creates a postive/negative consequence for women?). Furthermore, Euripides (Assessors do not actually like 'furthermore' as much as many people believe, as I got rid of your use of 'play' before, here you could say 'The playwright criticizes' to signal that you are discussing a play') criticizes the mistreatment of women and emphasizes the damage that a patriarchal society can do to (have on) a woman when she attempts to break free from societal norms  (societal norms is a very generalised statement - try to be more specific about what she breaks free from)
Title: Re: VCE English Question Thread
Post by: MissSmiley on February 12, 2018, 11:26:28 pm
Through his tragic play Medea, Euripides highlights the struggles that women faced, in a patriarchal society, in order to portray Medea as a heroic figure who defends the right of women. Furthermore, Euripides criticizes the mistreatment of women and emphasizes the damage that a patriarchal society can do to a woman when she attempts to break free from societal norms

how is this?  ;D ;D ;D
Yeah great ! Sounds even better now! :)

Once the signposting gets into this, it'll be a powerful, short and sweet intro (like it should be!!) :)
Title: Re: VCE English Question Thread
Post by: Yertle the Turtle on February 13, 2018, 09:27:03 am
Furthermore, Euripides criticizes the mistreatment of women and emphasizes the damage that a patriarchal society can do to (have on) a woman when she attempts to break free from societal norms  
Damage that a patriarchal society can "have on"? That doesn't work in a sentence, 'do to' would be better in this case imo
Title: Re: VCE English Question Thread
Post by: lilyrosee on February 13, 2018, 09:38:56 am
Damage that a patriarchal society can "have on"? That doesn't work in a sentence, 'do to' would be better in this case imo

If you change the 'when' to an 'as' it works, 'do it' sounds a bit informal imo but that is just me :)
Title: Re: VCE English Question Thread
Post by: Yertle the Turtle on February 13, 2018, 09:45:40 am
If you change the 'when' to an 'as' it works, 'do it' sounds a bit informal imo but that is just me :)
I would have thought that you can't have damage on something, but maybe that's just me...
Title: Re: VCE English Question Thread
Post by: nice! on February 13, 2018, 03:23:02 pm
Quote
emphasizes the damage that a patriarchal society can do to (have on) a woman when she attempts to break free from societal norms  

Or you could change it to:
... emphasises the damaging effects that a patriarchal society can inflict upon a woman when she attempts to break free from societal norms.
Title: Re: VCE English Question Thread
Post by: Yertle the Turtle on February 13, 2018, 05:01:27 pm
Or you could change it to:
... emphasises the damaging effects that a patriarchal society can inflict upon a woman when she attempts to break free from societal norms.
YES!! Perfecto! You are a virtuoso!! :D Thanks nice!
Title: Re: VCE English Question Thread
Post by: Uch.95+ on February 16, 2018, 10:05:20 am
Could you please help me improve on my introduction? Any contrsutive criticism would be much appreciated!! Thank you :)

Alfred Hitchcock’s 1954 film, ‘Rear Window’ endorses how easy it is to be deceived by appearances. Given that the entirety of the film is observed from the limited perspective of protagonist, L.B. Jefferies, the audience becomes privy to only a fraction of the lives of a number of characters at the apartment complex at Greenwich Village. On the surface, the audience is presented with Jefferies’ perceptions, judgements and assumptions of characters based on his own personal prejudices and beliefs. However, by the conclusion of the film, a broader picture of the complexity of people’s lives is presented and such we learn how easy it is to be deceived by appearances.
Title: Re: VCE English Question Thread
Post by: clarke54321 on February 16, 2018, 11:48:01 am
Could you please help me improve on my introduction? Any contrsutive criticism would be much appreciated!! Thank you :)

Alfred Hitchcock’s 1954 film, ‘Rear Window’ endorsesbit of an odd word choice. Perhaps demonstrates/depicts how easy it is to be deceived by appearances. Given that the entirety of the film is observed from the limited perspective of protagonist, L.B. Jefferies, the audience becomes privy to only a fraction of the lives of a a little crowded with the preposition, 'of.' number of characters at the apartment complex at Greenwich Village where is your argument? This is more analysis.. On the surface, the audience is presented with Jefferies’ perceptions, judgements and assumptions of characters based on his own personal prejudices and beliefs <--endeavour to make these points more specific.. However, by the conclusion of the film, a broader picture of the complexity of people’s lives be specific is presented and such we learn how easy it is to be deceived by appearances.

Hello!

Although you write with a lovely expression and nice, coherent sentences, I found it difficult to pinpoint your main arguments. Why is it so easy to be deceived by appearances (your ultimate contention)? You need to answer this question with 3-4 arguments, which ought to be signposted in the introduction.

Please feel free to ask any further questions :)
Title: Re: VCE English Question Thread
Post by: Uch.95+ on February 16, 2018, 11:29:03 pm
Hello!

Although you write with a lovely expression and nice, coherent sentences, I found it difficult to pinpoint your main arguments. Why is it so easy to be deceived by appearances (your ultimate contention)? You need to answer this question with 3-4 arguments, which ought to be signposted in the introduction.

Please feel free to ask any further questions :)

Thankyou, i will work on this further and i may post again! Thanks again!
Title: Re: VCE English Question Thread
Post by: msminzy on February 20, 2018, 05:44:49 pm
Where can I find rear window essay prompts to practise with?
I'Ve done all the ones school gave us and want to do more, but am having trouble finding them.
thaaankss :)
Title: Re: VCE English Question Thread
Post by: clarke54321 on February 20, 2018, 06:42:31 pm
Where can I find rear window essay prompts to practise with?
I'Ve done all the ones school gave us and want to do more, but am having trouble finding them.
thaaankss :)

Hello msminzy!

Firstly, welcome to the AN forums  :D

Given that 2018 is the first year that 'Rear Window' has been included in the VCE English study design, finding prompts is likely to be a difficult endeavour. Aside from the prompts that your teacher has provided you with, and the ones that you may find in a relevant study guide, I'd encourage you to create your own prompts! While this may sound laborious or mundane, it is a surprisingly stimulating task that will pay off in the long run.

A really effective way that you can construct prompts is by looking at VCAA's summary of your particular text. On this page, you can find it for 'Rear Window.'
Try and pick out certain elements of this description, and link them to an appropriate theme. For example, the line, frustrated and bored in the summer heat, he begins watching his neighbours across the courtyard. Boredom turns to suspicion when he believes that one of his neighbours has been murdered, implies that the most dreary of circumstances can lead to the most enlightening of revelations. Although I have not read this text, I assume that such a thematic idea would be quite prominent.

Who knows, by embarking on an activity like this, you may be able to foresee VCAA's direction with the next 'Rear Window' essay prompts  ;)
Title: Re: VCE English Question Thread
Post by: snowisawesome on February 20, 2018, 06:53:00 pm
What are some examples of metalanguage?
Title: Re: VCE English Question Thread
Post by: clarke54321 on February 20, 2018, 07:16:32 pm
What are some examples of metalanguage?

By definition, 'meta' = creative work referring to itself. Hence, this all depends on the context of that particular creative work. For example, in a film, metalanguage may includes things like the film's score, a long shot, or the lighting. In a play, you'd consider elements such as stage directions, characterisation, or dialogue.

Basically, metalanguage is a zoomed out articulation of a creative work's functioning.
Title: Re: VCE English Question Thread
Post by: MissSmiley on February 21, 2018, 06:11:16 pm
Hi everyone!

Could I just ask with text response essays and comparative essays, does your very first sentence of your introduction have to start answering the prompt? Or can it be like a general sentence that summaries the whole plot, its themes, character motivations, etc? (that perhaps might not relate to the prompt?)

Some teachers are saying get into starting the essay straight away (first sentence still covers the plot, themes, etc, but it has to relate to the prompt), and other teachers are saying it's ok to start your intro with a broad, overarching sentence.

Which one's the best? (Though I kinda think answering the prompt in your very first sentence will leave a good impression :), but still wanted to hear some of your thoughts!)

Thanks guys! :)
Title: Re: VCE English Question Thread
Post by: clarke54321 on February 21, 2018, 07:07:36 pm
Hi everyone!

Could I just ask with text response essays and comparative essays, does your very first sentence of your introduction have to start answering the prompt? Or can it be like a general sentence that summaries the whole plot, its themes, character motivations, etc? (that perhaps might not relate to the prompt?)

Some teachers are saying get into starting the essay straight away (first sentence still covers the plot, themes, etc, but it has to relate to the prompt), and other teachers are saying it's ok to start your intro with a broad, overarching sentence.

Which one's the best? (Though I kinda think answering the prompt in your very first sentence will leave a good impression :), but still wanted to hear some of your thoughts!)

Thanks guys! :)


Hello MissSmiley :)

The way you choose to start an essay in VCE English is entirely personal preference. For me, I found that an early contextualisation of the text's/texts' main concerns was the best way to begin. It established a natural pathway to my ultimate contention. Just ensure that these main concerns relate to the prompt in some manner. Eg. If you are given a prompt about the power of familial love, you don't want to begin your essay by reeling off ideas about isolation. To better illustrate this approach, I'll post one of my previous comparative essays on my English guide!

But just to be clear, any introduction structure has its merit, so long as it makes for plausible and coherent content. 
Title: Re: VCE English Question Thread
Post by: Yertle the Turtle on February 21, 2018, 07:13:16 pm
Hi everyone!

Could I just ask with text response essays and comparative essays, does your very first sentence of your introduction have to start answering the prompt? Or can it be like a general sentence that summaries the whole plot, its themes, character motivations, etc? (that perhaps might not relate to the prompt?)

Some teachers are saying get into starting the essay straight away (first sentence still covers the plot, themes, etc, but it has to relate to the prompt), and other teachers are saying it's ok to start your intro with a broad, overarching sentence.

Which one's the best? (Though I kinda think answering the prompt in your very first sentence will leave a good impression :), but still wanted to hear some of your thoughts!)

Thanks guys! :)

In general I tend to make an opening statement that shows the text and it's main themes. This can then lead in easily into the point of the prompt. Very early on discuss the overarching picture of the text, and then bring the focus in tight onto the theme of the prompt. That's just my method, it doesn't work for everyone.
Title: Re: VCE English Question Thread
Post by: Sine on February 21, 2018, 07:15:26 pm
Hi everyone!

Could I just ask with text response essays and comparative essays, does your very first sentence of your introduction have to start answering the prompt? Or can it be like a general sentence that summaries the whole plot, its themes, character motivations, etc? (that perhaps might not relate to the prompt?)

Some teachers are saying get into starting the essay straight away (first sentence still covers the plot, themes, etc, but it has to relate to the prompt), and other teachers are saying it's ok to start your intro with a broad, overarching sentence.

Which one's the best? (Though I kinda think answering the prompt in your very first sentence will leave a good impression :), but still wanted to hear some of your thoughts!)

Thanks guys! :)
As clarke4321 has already said it won't matter too much in the grand scheme of things for the essay - I personally preffered to have a broad introductory statement which often alluded to the historical context of the text, however still relevant to the prompt and then I was able to link that towards a sentence which directly relates to the prompt.
Title: Re: VCE English Question Thread
Post by: MissSmiley on February 21, 2018, 07:41:00 pm
Thanks so much clarke54321, Marvin K.Mooney and Sine !!
You've helped heaps :)

Yes!! How awesome if clarke54321 posted one of her comparative essays!! Yes please!!

Can't wait to read it!!!  ;D

Thanks a lot guys!
Title: Re: VCE English Question Thread
Post by: madhu04 on February 24, 2018, 01:13:08 pm
Hey guys,

I am EAL student.
I am currently studying Invictus and soon have a creative writing SAC on it. 


I was wondering if I can get some thorough feedback on a creative piece I did.  It's a letter from Brenda to Jason. 

20/6/1994                                                                                                                    From: Brenda

Dear Jason,

Good evening Jason! Thank you for protecting our Madiba in the rugby match today, you did a great job. I sometimes get agitated as he is carless and doesn’t worry about himself at all. I don’t understand why he is so obsessed with rugby and the Springbok’s? Can you believe he literally left his meeting with the minster of environment to watch the news footage of the rugby coaching clinic? He is just unbelievable! He is trying to reconcile the white and the black, which of course already sounds ridiculous and on top of that he is utilising Springboks to achieve this, even if he is aware that it symbolises apartheid and segregation to us. During the years of apartheid, the white dominated  us! Snatched our rights to be humans and treated us like dogs. And that apartheid flag! Ugh! They are a disgrace, I feel furious when I see it, it must be removed from the constitution as soon as possible! Don’t you think so as well?

So many Africans still live in immense poverty and that unprivileged shanty towns! Trust me, if the Afrikaners see the living conditions of people who live in shanty towns, they will be glad that they don’t live over there. That ‘s exactly why, I wish Mandel focuses on these aspects rather than that irrelevant idea of reconciliation and forgiveness. Moreover, you know what really confuses me? Madiba’s whole idea of “forgiveness”, “reconciliation” and what not! Does he not remember the past, his past! He was part of the ANC, he was convicted of treason and as you also know he protested regarding the pass book law imposed by the white! He did all this, and he wanted revenge on them for treating us in a impartial manner. But now, when I tell him about the problems we are facing in regards with the food, jobs and much more that is all result of the apartheid regime, so that he stops interrupting affairs of state to placate the white minority; he told me that that is a “human calculation to address these issues! I was furious at him Jason, all this he is doing now, will not change anything! I hope he also realizes that soon, soon before he increases his risk of losing his presidency. 

With regards,
Brenda
Title: Re: VCE English Question Thread
Post by: emmawatsonlover990 on February 26, 2018, 08:00:55 pm
Hi,

How do you write a really good Introduction for an essay?
Title: Re: VCE English Question Thread
Post by: emmawatsonlover990 on February 26, 2018, 08:09:59 pm
How do you write a contention in an essay that isn't an opinion essay or an language analysis?
Title: Re: VCE English Question Thread
Post by: Yertle the Turtle on February 26, 2018, 08:18:52 pm
Hi,

How do you write a really good Introduction for an essay?
That depends on the type of essay. For LA, many teachers use the CCTAP method: Context, Contention, Tone, Audience, Purpose. In other words, begin by a broad statement about the context of the piece, its subject matter. Then discuss what side of this issue the author argues, and how, and then who they are targeting with their piece and why.

For TR, in general I like to have a wide statement about the author and main themes of the text, before delving in to the key focus of the question. Have the three main points you will be arguing in there, and then express an overarching reason for the theme, or an explanation of the theme in question.

How do you write a contention in an essay that isn't an opinion essay or an language analysis?
Only write about a contention in a language analysis piece. You don't need to talk about a contention in a TR or Comparative, just in the LA.
Title: Re: VCE English Question Thread
Post by: emmawatsonlover990 on February 26, 2018, 08:23:34 pm
My teacher told me to write a contention in a TR. I am really confused as I thought that it was just for LA. Thank you for your help.  ;D ;) :-\
Title: Re: VCE English Question Thread
Post by: sdfg on February 26, 2018, 08:29:57 pm
My teacher told me to write a contention in a TR. I am really confused as I thought that it was just for LA. Thank you for your help.  ;D ;) :-\

A 'contention', by definition, is nothing more than your 'main argument'. In L.A (or argument analysis, whatever you want to call it), the contention you state in your introduction isn't actually your's but rather of the writer's of the text in which you're analysing. In TR, you give your own contention/argument, which provides your viewpoint or answer to the essay question you're given.
Title: Re: VCE English Question Thread
Post by: abbeyrose on February 26, 2018, 09:38:04 pm
How do you write a contention in an essay that isn't an opinion essay or an language analysis?

Are you meaning for essays such as those you would write for Sections A and B?  :)

Essentially with the essays that you would write for both types of text responses, you are arguing your own interpretation/point in response to a prompt and thus you can think of it as an opinion essay. Your contention will serve as your overall argument in response to the given prompt, and which the topic sentences in each of your body paragraphs should tie back to and support. When formulating your contention, just simply think of what your own response to the prompt is and what you are overall trying to argue through your response. If an example would help (however, this is by no means the most incredible example of a contention and is merely a contention I pulled from one of the few essays still on my computer) here is one in response to a prompt to I for Isobel:

" 'How does writing shape Isobel's journey to realising her identity?'
Through writing and realising that she belongs as ‘a writer’, Isobel is able to overcome the limitations of the upbringing and conservative society that prevented her from uncovering her identity. "

It's something that becomes easier with practise, particularly as you become more familiar with responding to prompts. An exercise that I always found helpful when planning an essay and generating a contention was to write my prompt in the middle of a piece of paper, identifying what themes/character(s) it related to, and then write down and link the points that come to mind (e.g. for the above contention I would write something like 'Isobel's creativity and imagination form her identity' and then link it with 'The mistreatment of her childhood hindered her from embracing her creativity'). Once you have done this, you can then see what arguments your interpretation creates, and then you can go about writing a contention. I also found that it also allows you to plan some topic sentences that flow well.

I hope this makes sense and helped a little. :)
Title: Re: VCE English Question Thread
Post by: Poet on February 27, 2018, 11:51:13 am
Hi friends,

On the topic of I for Isobel, I'm struggling to find good sample essays to pick apart and get ideas from. Does anybody have an essay (highly scored, please) they're willing to share?

Thank you!
Poet.  :-)
Title: Re: VCE English Question Thread
Post by: madhu04 on March 02, 2018, 07:24:29 pm
Hey guys, 

This is not really a question related to English,  but I didn't know where else to ask.  Sorry

I was wondering whether anyone what the range of marks are?  Like there is very low, low... High very high... High excellent...that stuff!!

Does anyone what all the categories are and the associated marks?


Thank you
Title: Re: VCE English Question Thread
Post by: Sine on March 02, 2018, 08:04:30 pm
Hey guys, 

This is not really a question related to English,  but I didn't know where else to ask.  Sorry

I was wondering whether anyone what the range of marks are?  Like there is very low, low... High very high... High excellent...that stuff!!

Does anyone what all the categories are and the associated marks?


Thank you
A lot of schools do give ranges of marks to students (including mine) although the actual range should very between each school since it is not standerdised by VCAA. It could be worthwhile asking your teacher if they don't make it clear after the first sac.
Title: Re: VCE English Question Thread
Post by: clarke54321 on March 03, 2018, 09:00:35 am
Hey guys, 

This is not really a question related to English,  but I didn't know where else to ask.  Sorry

I was wondering whether anyone what the range of marks are?  Like there is very low, low... High very high... High excellent...that stuff!!

Does anyone what all the categories are and the associated marks?


Thank you

In addition to the info provided by Sine, note that VCAA (as opposed to your school's individual system) marks your essays according to a number scheme. The highest you can achieve is a 10. Here is a more detailed breakdown.
Title: Re: VCE English Question Thread
Post by: snowisawesome on March 04, 2018, 09:00:54 pm
Few questions
1. What is the best way to prepare for an english text response sac on Medea?
2. How do you memorise quotes?
3. How many quotes should we be using in essays generally? How about for text response?
4. How many words should text response essays be?
5. How do you improve your essay writing skills?

Thanks ;D
Title: Re: VCE English Question Thread
Post by: Willba99 on March 04, 2018, 09:33:35 pm
Few questions
1. What is the best way to prepare for an english text response sac on Medea?
2. How do you memorise quotes?
3. How many quotes should we be using in essays generally? How about for text response?
4. How many words should text response essays be?
5. How do you improve your essay writing skills?

Thanks ;D
1. I haven't done Medea, but from past experience the best way to practice is to pump out practice essays in the same time that you'll have on the day of your SAC (no use being able to write a great essay in 4 hours if your sac is only an hour long). People will disagree and say that too many practice essays is a waste of time, but I honestly think that as long as you know the text well then practice essays are the way to go. I probably did 5 practice essays for my text response sac
2. Once you've found the quotes you're going to want to use, practice using them, whether in small sentences, or in practice essays. I found that any quotes i used tented to be remembered, while I struggled to remember quotes I'd never practiced with.
3. I'd say 3-5 per para is a good base. They don't have to be long.
4. My English teacher always used to say "It's not about how big the pie is, its about how good it tastes". Regardless, you want to be writing 600 words minimum and preferably 800+.
5. Practice and Feedback. The English Work Submission forum is great for getting feedback!
Title: Re: VCE English Question Thread
Post by: clarke54321 on March 04, 2018, 09:33:50 pm
Few questions
1. What is the best way to prepare for an english text response sac on Medea?
2. How do you memorise quotes?
3. How many quotes should we be using in essays generally? How about for text response?
4. How many words should text response essays be?
5. How do you improve your essay writing skills?

Thanks ;D

1. This is an extremely broad question, which is best answered by the individual preparing for the text response. Which thematic ideas do you struggle with the most? Do you have a solid understanding of the play's relevant context? Have you developed coherent interpretations? These are the types of questions that I'd be asking myself before sitting a text response SAC. They will help you narrow down your focus, and make the task of preparing seem less overwhelming.

2. Again, this is a matter of personal preference. Some students like to make flash cards, others read them aloud, some place them on sheets around the house, and others keep using them in practice essays. I've always gravitated towards the last approach. The more you use the quotes in context the more likely it'll be that you'll remember them, because they serve some kind of relevance.

3. Enough to justify your points. On average, I had about 14 quotes in each body paragraph. However, a majority of these were only 2-4 words long.

4. 800-1000, but as always quality>quantity.

5. By unearthing your own specific weaknesses. This comes about through processes such as teacher feedback, careful editing and repetitive redrafting. Once you've identified your shortcomings, have a read through high-scoring essays and note how other students have overcome your weaknesses. The more essays you read, the more tools you'll have to counteract your writing concerns.

Hopefully these answers have provided you with some guidance :)
Title: Re: VCE English Question Thread
Post by: Sine on March 05, 2018, 09:41:52 pm
Few questions
1. What is the best way to prepare for an english text response sac on Medea?
2. How do you memorise quotes?
3. How many quotes should we be using in essays generally? How about for text response?
4. How many words should text response essays be?
5. How do you improve your essay writing skills?

Thanks ;D
great advice by everyone :) thought it may be helpful to get another perspective
1. Know your text (obviously) but also know the details by taking notes and looking through any other resources. From here you can create "quote" banks + evidence that you can use in your essays. Practicing writing essays would definitely by the most important in order to get better but make sure you have someone to give you feedback after each one and try to ammend those mistakes in the next essay.

2. Use the quotes - you will generally memorise them without too much effort like this. Writing your first few essays open book is alright.

3. Maybe around 4-5 big quotes but you can sprinkle the paragraph with other 1-2 word quotes.

4. 1000+ obviously quality > quantity. However say two essays are the same quality but one is 1500 words but the other is 800 the 1500 will get the higher score, but maybe by only 0.5-1 marks/10

5. Practice -> get feedback --> implement feedback
Title: Re: VCE English Question Thread
Post by: snowisawesome on March 05, 2018, 10:16:12 pm
Thanks sine and clarke54321 ;D
Just wondering, for a book like Medea since the entire book is a play, can we use any word from the book as use it as a quote?
Title: Re: VCE English Question Thread
Post by: Sine on March 05, 2018, 10:21:02 pm
Thanks sine and clarke54321 ;D
Just wondering, for a book like Medea since the entire book is a play, can we use any word from the book as use it as a quote?
Yup you definitely can. Just make sure it fits in your essay and not forced.
Title: Re: VCE English Question Thread
Post by: snowisawesome on March 05, 2018, 10:28:52 pm
Yup you definitely can. Just make sure it fits in your essay and not forced.
What do you mean by "not forced"?
Title: Re: VCE English Question Thread
Post by: Sine on March 05, 2018, 10:30:43 pm
What do you mean by "not forced"?
As in you don't want to be chucking in  "quotes" which aren't relevant just for the sake of increasing your quote count or using that particular quote.
Title: Re: VCE English Question Thread
Post by: snowisawesome on March 05, 2018, 10:33:55 pm
As in you don't want to be chucking in  "quotes" which aren't relevant just for the sake of increasing your quote count or using that particular quote.
So can quotes be as little as 2 words long?
Title: Re: VCE English Question Thread
Post by: MissSmiley on March 06, 2018, 06:07:05 pm
So can quotes be as little as 2 words long?
Definitely! :)
Title: Re: VCE English Question Thread
Post by: sdfg on March 06, 2018, 06:30:07 pm
So can quotes be as little as 2 words long?

As MissSmiley said, definitely, but don't use this as an excuse to memorise 2 words of a quote and not the whole thing. Only use 2 word quotes if they provide substantial material for analysis.
Title: Re: VCE English Question Thread
Post by: Yertle the Turtle on March 06, 2018, 07:39:42 pm
So can quotes be as little as 2 words long?
As little as one word, just remember to make sure that they are appropriate and sufficiently embedded in the essay, in other words they are part of the sentence.
Title: Re: VCE English Question Thread
Post by: Sine on March 06, 2018, 07:42:15 pm
So can quotes be as little as 2 words long?
as everyone else has already said- yes definitely you can

but the most improtant thing is knowing when and how to use certain quotes in your essay
Title: Re: VCE English Question Thread
Post by: emmawatsonlover990 on March 08, 2018, 09:04:10 pm
Does anyone have any ideas on what to write in a paragraph about the juxtaposing ideas of the theme of life and death in 'Year of Wonders'? 

;) ;) ;) Thank you in advance
Title: Re: VCE English Question Thread
Post by: MissSmiley on March 10, 2018, 02:48:16 pm
Hi everyone! :)

Could I just ask what you need to talk about in a conclusion to make it really strong and memorable in your text response?
Yes it's supposed to be really short and you have to signpost your points really succinctly, but other than that, does you last line need to be about how the text values, ideas, themes, etc apply to us as 21st century people or does the last line have to be specific to the text just analysing it holistically and relating it to the essay prompt?

Would love to hear how people approach their conclusion and how they end their piece! :)

Thank you so much!!
Title: Re: VCE English Question Thread
Post by: sdfg on March 10, 2018, 03:22:13 pm
Hi everyone! :)

Could I just ask what you need to talk about in a conclusion to make it really strong and memorable in your text response?
Yes it's supposed to be really short and you have to signpost your points really succinctly, but other than that, does you last line need to be about how the text values, ideas, themes, etc apply to us as 21st century people or does the last line have to be specific to the text just analysing it holistically and relating it to the essay prompt?

Would love to hear how people approach their conclusion and how they end their piece! :)

Thank you so much!!

I try to think of conclusions as a 'mic-drop' moment, where after for the most part of my essay talking about the specifics, I go BOOM this is the big picture bitches, then proceed to drop the mic and walk off the stage. Don't know how helpful of a tip this is but that's what I always thought of when writing conclusions and it worked well for me!!

Oh, and don't talk about how a text is applicable to the 21st century in the conclusions unless you've already discussed it in your body. A conclusion's function is exactly as its name suggests - to conclude, not to introduce new ideas.
Title: Re: VCE English Question Thread
Post by: Sine on March 10, 2018, 03:29:17 pm
Hi everyone! :)

Could I just ask what you need to talk about in a conclusion to make it really strong and memorable in your text response?
Yes it's supposed to be really short and you have to signpost your points really succinctly, but other than that, does you last line need to be about how the text values, ideas, themes, etc apply to us as 21st century people or does the last line have to be specific to the text just analysing it holistically and relating it to the essay prompt?

Would love to hear how people approach their conclusion and how they end their piece! :)

Thank you so much!!
other than a structural component of the essay it really won't alter your mark too much. The assesor usually knows what mark well before the conclusion. I think a good conclusion would as you say summarise all the important points along with NOT including any other new ideas. You could allude to some historical context of the text too.
Title: Re: VCE English Question Thread
Post by: Guideme on March 12, 2018, 10:06:06 am
So i am writing a short story in relation to the film "Rear Window" about Miss Lonelyhearts's feelings and actions when she gets sexually assaulted, who do you think the target audience would be?
Title: Re: VCE English Question Thread
Post by: Sine on March 12, 2018, 02:09:37 pm
So i am writing a short story in relation to the film "Rear Window" about Miss Lonelyhearts's feelings and actions when she gets sexually assaulted, who do you think the target audience would be?

I haven't studied the text so just going off what you wrote. I'm thinking anyone you want to educate on what those feelings are like and her mindset on why those actions were chosen and taken. Or if the actions were impulsive why were they so? Could also be for an audience who may have had similar experiences in order to make sure they are not isolated feel alone.
Title: Re: VCE English Question Thread
Post by: snowisawesome on March 12, 2018, 02:20:02 pm
"In the play medea, the crucial conflict is between reason and passion". Discuss

For the prompt above, would I lose marks in an essay if i had a few quotes that weren't directly about reason and passion?
Title: Re: VCE English Question Thread
Post by: Poet on March 12, 2018, 02:25:34 pm
"In the play medea, the crucial conflict is between reason and passion". Discuss

For the prompt above, would I lose marks in an essay if i had a few quotes that weren't directly about reason and passion?

If you use quotes in an essay, they don't have to be related. You just have to find a way to tie them in and make them somewhat relevant to whatever you're writing on, such as describing a character (Medea's passion is evident in her [insert quote here] and erratic conduct), etc.
Just make sure you're not just plonking quotes in that you think you can get away with. Quality over quantity, always. :)
Title: Re: VCE English Question Thread
Post by: clarke54321 on March 12, 2018, 02:26:22 pm
"In the play medea, the crucial conflict is between reason and passion". Discuss

For the prompt above, would I lose marks in an essay if i had a few quotes that weren't directly about reason and passion?

No, this wouldn’t be the case. You may have other quotes (not related to passion or reason) that work to corroborate the points that do in fact hold some relevance.
Title: Re: VCE English Question Thread
Post by: TheLlama on March 12, 2018, 05:29:04 pm
For the prompt above, would I lose marks in an essay if i had a few quotes that weren't directly about reason and passion?

Remember that you don't really 'lose' marks - assessors are looking to justify giving you a certain score, and they want to find reasons to give you marks. Put a little differently, your reader will be asking why you've selected a particular quote (or quotes) and what it adds to your reading of the text. If anything, you're going to need to use quotes that go beyond the whole question of reason/passion in order to get into the nuances of the text and, in turn, show that you've formed a slightly more complex reading.

There's also more important thing - notice how this topic talks about the "crucial conflict"? That implies that there's more to this question than just reason vs passion. So quite the contrary: a strong response would ideally go a little beyond that limited focus!
Title: Re: VCE English Question Thread
Post by: pha0015 on March 14, 2018, 10:11:25 pm
should you use quotes in a creative essay?
Title: Re: VCE English Question Thread
Post by: Yertle the Turtle on March 14, 2018, 10:19:28 pm
should you use quotes in a creative essay?
Yes. In a creative piece you should always use either footprints of the text (embedding themes, concepts, ideas directly from the text, and hinting at the text, but not directly quoting), or quotes. If it is a creative essay, then use quotes, if other types of creative, probably just use the footprints.
Title: Re: VCE English Question Thread
Post by: lilyrosee on March 14, 2018, 11:06:04 pm
should you use quotes in a creative essay?

I did not use direct quotes from the text in my creative piece as the task is to use your own language/writing in a creative response to the text, not to use the author's own words. However, in your explanation you can quote from the text, perhaps discussing how an idea for your creative piece stemmed from a quote in the text etc.

I hope this helps
Title: Re: VCE English Question Thread
Post by: TheLlama on March 15, 2018, 09:39:38 am
should you use quotes in a creative essay?

Generally, I'd suggest steering clear of quoting from the text in a creative piece.

Remember that this task is largely about showing you understand the ideas in the text as well as presenting your abilities as a creative writer. So while you might be borrowing and adapting ideas, imagery, metaphors and literary devices used by the author/director/playwright you're emulating, a cut-and-paste approach usually doesn't work well. Pieces that tend to quote the book, for example, just sound like you're copying, rather than adapting.

When you justify your choices, you can explain the link between the two (for example, "In choosing to employ images associated with... I was heavily influenced by the author's use of..." and make reference to what they do).

There is, however, a nice way to use a quote: as an opening line of your piece. If you have something that encapsulates what you want to say, you can have it as a single line at the top of the page, quoted. You might notice that sometimes books/chapters of books do this!
Title: Re: VCE English Question Thread
Post by: Poet on March 15, 2018, 05:54:55 pm
Can I use quotes from famous philosophers from Greek society in order to drive my point in my Medea essays? if so how can I effectively incorporate these quotes?

I agree with sdfg. Don't feel the need to use a bunch of quotes from outside the text, because the thing assessors are looking for is actually your knowledge and understanding of what you're meant to be studying - not your skill in remembering your year 9 history lessons! Keep it on topic, and try to "drive your point(s)" home with quotes from Medea. It's a lot better to fill your brain up with relevant quotes from the text than irrelevant quotes from ancient philosophers! :)
Title: Re: VCE English Question Thread
Post by: clarke54321 on March 15, 2018, 08:14:29 pm
Can I use quotes from famous philosophers from Greek society in order to drive my point in my Medea essays? if so how can I effectively incorporate these quotes?

I'd strongly suggest that you don't include external evidence, regardless of its seeming relevance. Note that the following is a key aspect of VCAA's section A criteria, use of textual evidence to support the interpretation. VCAA is primarily interested in the interpretations that you can build from the world of the play. That is, which views and values are confined to its own context? The context that Euripides established?
Title: Re: VCE English Question Thread
Post by: MissSmiley on March 16, 2018, 08:42:54 pm
Hi everyone!

Could I just ask if this interpretation of the quote "a rock or wave or sea would pay more attention to a counsel of friends than she does" (From the prologue of Medea) is right?

Shows that Medea does not position herself with nature already --> foreshadows her plan to kill her children, that is a defiance of natural order

Or this unclear?

Is there a better interpretation of this quote?

Thank you so much!! :)
Title: Re: VCE English Question Thread
Post by: Arithmetic on March 17, 2018, 01:55:13 pm
Hey guys,

We have an english sac coming up on Monday, it is a comaprative language analysis. We were given the articles yesterday and one of the articles is written by an author called Emily Ritchie. But the author does not write from her own perspective, but quotes and talks about another lady called Germaine Greer. I'm confused as to who's point of view I should discuss in the essay. The other article is written by the editor of the publisher.

Thanks y'all,
Title: Re: VCE English Question Thread
Post by: Poet on March 17, 2018, 01:57:35 pm
Hi everyone!

Could I just ask if this interpretation of the quote "a rock or wave or sea would pay more attention to a counsel of friends than she does" (From the prologue of Medea) is right?

Shows that Medea does not position herself with nature already --> foreshadows her plan to kill her children, that is a defiance of natural order

Or this unclear?

Is there a better interpretation of this quote?

Thank you so much!! :)

Hello MissSmiley!

I saw this quote as a lot more opaque, and I didn't seem to find any deep symbolism in it. I saw this quote as merely meaning that Medea took no heed of her friend's ministrations or words of comfort - she was distraught, and did not want any comfort. She didn't listen to anyone who told her anything that did not align with her bitter thoughts.

Your interpretation, in saying that Medea did not "position herself with nature already", is basically implying that Medea herself consciously knows that she is going to do something that the other women see as unnatural - the murder of your own children. I like the thought of this act as being a defiance of the natural order, that this might be the first indication of Medea's unnatural behaviour, but I don't feel like this quote foreshadows any plan on Medea's behalf to kill her children.

That's just some of my thoughts. I hope you find this meaningful. :)
Title: Re: VCE English Question Thread
Post by: MissSmiley on March 17, 2018, 02:27:15 pm
Hello MissSmiley!

I saw this quote as a lot more opaque, and I didn't seem to find any deep symbolism in it. I saw this quote as merely meaning that Medea took no heed of her friend's ministrations or words of comfort - she was distraught, and did not want any comfort. She didn't listen to anyone who told her anything that did not align with her bitter thoughts.

Your interpretation, in saying that Medea did not "position herself with nature already", is basically implying that Medea herself consciously knows that she is going to do something that the other women see as unnatural - the murder of your own children. I like the thought of this act as being a defiance of the natural order, that this might be the first indication of Medea's unnatural behaviour, but I don't feel like this quote foreshadows any plan on Medea's behalf to kill her children.

That's just some of my thoughts. I hope you find this meaningful. :)
Great!
Thank you so much, secretly_a_poet!
Of course it's meaningful!! :)
So I'm just overthinking it probably!

Thanks a lot! :)
Title: Re: VCE English Question Thread
Post by: MissSmiley on March 18, 2018, 07:06:17 pm
Hi guys!

I'd love to hear a response as soon as possible, because I have my Medea SAC tomorrow!

Do you think there could be a question based on literary techniques (symbolism, metaphors, etc) used in the play?
Obviously more to it with including character and everything, but could there be one like this?
VCAA hasn't given one of this type since so long, that's why I put it out there  ;)

Thanks guys!

Would love to hear your thoughts about this! :)
Title: Re: VCE English Question Thread
Post by: clarke54321 on March 18, 2018, 07:20:03 pm
Hi guys!

I'd love to hear a response as soon as possible, because I have my Medea SAC tomorrow!

Do you think there could be a question based on literary techniques (symbolism, metaphors, etc) used in the play?
Obviously more to it with including character and everything, but could there be one like this?
VCAA hasn't given one of this type since so long, that's why I put it out there  ;)

Thanks guys!

Would love to hear your thoughts about this! :)

Hi MissSmiley  :D

I highly doubt that you'd be given a prompt based around a literary technique alone. The closest Medea prompt I've seen, in relation to symbolism and metaphors, concerns the role of the children in the play. Also, I have come across one pertaining to the role of the Chorus.

But try not to stress about this too much. It's an unlikely possibility. Just to be sure, however, have a few literary techniques at the back of your mind, which you can easily link to key views and values if necessary.

All the very best with the SAC!
Title: Re: VCE English Question Thread
Post by: MissSmiley on March 18, 2018, 07:47:30 pm
Hi MissSmiley  :D

I highly doubt that you'd be given a prompt based around a literary technique alone. The closest Medea prompt I've seen, in relation to symbolism and metaphors, concerns the role of the children in the play. Also, I have come across one pertaining to the role of the Chorus.

But try not to stress about this too much. It's an unlikely possibility. Just to be sure, however, have a few literary techniques at the back of your mind, which you can easily link to key views and values if necessary.

All the very best with the SAC!
Great! Thanks a lot, Clarke!!  :D
Also, could I just ask, if the topic focuses on one character, for e.g "Jason's capacity for evil stems from his weaknesses than from strength." Do you agree?,
Can you divert the discussion of this topic to include other characters (such as the other men) as well? as long you discuss Jason throughout your piece?
Would you signpost that in the intro?
Because obviously you want to include other characters and Medea as well, otherwise it would be pretty boring right? just talking about Jason?

It's easier when the topic's focused on Medea and any other ideas about the play, but made me scratch my head about this one!  :)

Thanks a lot!!  :D

Title: Re: VCE English Question Thread
Post by: clarke54321 on March 18, 2018, 07:54:28 pm
Also, could I just ask, if the topic focuses on one character, for e.g "Jason's capacity for evil stems from his weaknesses than from strength." Do you agree?,
Can you divert the discussion of this topic to include other characters (such as the other men) as well? as long you discuss Jason throughout your piece?

Absolutely. In fact, you'd be penalised if you didn't. However, you must ensure that you are making coherent links back to Jason's character. Ideally, you'd be using these other male characters (and female characters, don't forget them) as a means of illuminating different aspects of Jason's character. Again, it's unlikely that a prompt would be this narrow in scope.

Would you signpost that in the intro?

This would depend on your particular arguments. But for logicality purposes, yes. You don't want examiners to get an off impression too early.
Title: Re: VCE English Question Thread
Post by: jsgraha on March 24, 2018, 04:32:26 pm
Hi guys, I will have my text response SAC on Monday and it is stressing me out. I gave few essays to different people and they gave me different feedback. One said my introductory has so many sentences and wants me to use simple words and the other even wants me to put more sentences (and super vocabulary) to show that I understand greatly the topic. could you share your thought on how to tackle this.

thanks

JS
Title: Re: VCE English Question Thread
Post by: MissSmiley on March 24, 2018, 05:22:59 pm
Hi guys, I will have my text response SAC on Monday and it is stressing me out. I gave few essays to different people and they gave me different feedback. One said my introductory has so many sentences and wants me to use simple words and the other even wants me to put more sentences (and super vocabulary) to show that I understand greatly the topic. could you share your thought on how to tackle this.

thanks

JS
Hi jsgraha!
Firstly, don't stress out so much! It always happens with English that you stress out so much before the SAC (and I mean soooo much!!) but during the actual SAC and even coming out of it, you're like what? Why did I even stress out so much?!! You are so prepared and you unnecessarily stress out, so I'm sure you'll be fine! Because it's great to see you're writing practice essays. Just with what you said,

1. You should definitely keep your introduction short! It should only be 4-5 sentences long. 1 1/2 sentences maybe to provide context but at the same time to relate it to the prompt, the next line your one line contention and then straight to brief signposting and done!

2. Considering the SAC's on Monday, just pick a handful of good or 'advanced vocab' that you could use to describe characters, their actions or just any words to do with the world of the text. Only a handful though!! Or if you're feeling confident that you've memorised quotes and are ready to write any paragraphs, then consider memorising some sentences from all the essays that you've written and the sentences or ideas your teacher liked. But don't worry at all if you keep your language simple and don't use complicated vocab. Assessors (and probably teachers) don't have time to get a dictionary beside them and search up every word that they don't know! :) So aim for clarity and throughout the year, just read widely and the complexity will automatically come! :) So having a handful of good words (consider those that would suit any theme or question you get) is enough for Monday. :)

All the very best!! I'm sure you'll get what I mean when you come out of the SAC and go wow! I didn't need to stress so much! Because they have to put in there at least 1 topic that'll grab your attention to most than the other topic you get :)
Title: Re: VCE English Question Thread
Post by: jsgraha on March 24, 2018, 09:24:46 pm
Thank you MissSmiley - my teacher did not give any valuable feedback, just number and after writing several essays I am still stuck at 8. Thank you for your advice anyway

Cheers
JS
Title: Re: VCE English Question Thread
Post by: Guideme on March 25, 2018, 10:55:47 pm
Hi everyone,

So in class we are studying a text 'Extinction' and my teacher has been pointing out parts of the play that include some irony, however i am not so sure how it will be useful or how to use it in our text response essays. i would like maybe a sampple sentence involving any sought of irony or any tips on when to embed them into our essays.

Thank you
Title: Re: VCE English Question Thread
Post by: MissSmiley on March 26, 2018, 03:45:46 pm
Hi everyone,

So in class we are studying a text 'Extinction' and my teacher has been pointing out parts of the play that include some irony, however i am not so sure how it will be useful or how to use it in our text response essays. i would like maybe a sampple sentence involving any sought of irony or any tips on when to embed them into our essays.

Thank you
Irony's a great way to tell your teacher and the assessor that you understand the linguistics side of your text. So yes it's the ideas, themes, etc, but every linguistic feature/ figurative language has a purpose! Often the purpose is to connect it to these central ideas.

You can use some sentence starters:
1. The use of irony in...
2. (the character's name)'s ironical declaration that...(quote from text) exemplifies...
3. The author's use of irony serves the purpose of...

For example, (this is from Medea by Euripides, but):
The tutor's ironical belief that "[Medea's] little ones are under no threat" highlights his innocence in not being able to recognise Medea's savagery in her plots to commit filicide and kill her own children.
or something like that. (Also, it highlights Medea's ability to cover her villanous character and show that's she's not actually a criminal. In this way, she fools other people around her, hence the irony when they are fooled.)

I know this isn't from Extinction, but hopefully you get the idea how to put irony in a sentence.

And yes! Use it! It's another way to stand out if you weave it in seamlessly and if it relates to the essay prompt :)

Good luck!


Title: Re: VCE English Question Thread
Post by: clarke54321 on March 26, 2018, 04:32:35 pm
Hi everyone,

So in class we are studying a text 'Extinction' and my teacher has been pointing out parts of the play that include some irony, however i am not so sure how it will be useful or how to use it in our text response essays. i would like maybe a sampple sentence involving any sought of irony or any tips on when to embed them into our essays.

Thank you

On top of MissSmiley's excellent advice, I'd encourage you to think of irony as an illuminating tool. I also have a 'Medea' snippet, which might illustrate the point more clearly.

.......Quite literally, Jason seeks to mute and thereby subjugate the self-autonomy of all women, who are palpably defined and restrained by their biological physicality. Hence, Jason’s socially ironic assessment that Medea has ‘known justice’ in Hellas, impels audiences to question the truth of this assertion.......

Within the confines of 'Medea,' Jason is clearly a misogynist. And so the fact that he tells Medea that she has known justice, which defies all of his comments/actions, is absurd and more to the point, ironic. This irony thus assists the playwright in foregrounding Jason's immoral character.

All the best  :)
Title: Re: VCE English Question Thread
Post by: pha0015 on March 27, 2018, 07:27:24 pm
What's the difference between language analysis and analysing arguments? I get what it means as words, but how does it holistically affect how we write our language analysis... ahem... argument analysis? Is it still mostly similar to language analysis but with a focus on how language aids in conveying individual arguments?
Title: Re: VCE English Question Thread
Post by: MissSmiley on March 27, 2018, 07:38:14 pm
What's the difference between language analysis and analysing arguments? I get what it means as words, but how does it holistically affect how we write our language analysis... ahem... argument analysis? Is it still mostly similar to language analysis but with a focus on how language aids in conveying individual arguments?
Exactly the way you've interpreted it!! :)
Yeah, like you said, more focus on arguments rather than just a shopping list of linguistic techniques used in the texts. I think until last year people used to fall under the trap of basing their whole paragraph on language devices (you know like similes, personification, etc) and then talk about what argument or value they serve.
But now the wording's changed, it's gonna force us to analyse arguments and the progression of the author's ideas/ views and then use linguistic devices as a support only. - to back up the argument and contextualise the evidence according to the argument.
So this means forcing us to make each topic sentence of every para ideas based :)
Which is excellent!! :) It allows for more critical thinking :)
Title: Re: VCE English Question Thread
Post by: emmawatsonlover990 on March 28, 2018, 09:30:38 am
Suggestion for current issues in the Australian media? (From September 1 2017 until present)
Title: Re: VCE English Question Thread
Post by: MissSmiley on April 01, 2018, 09:00:58 pm
Hi guys!

Just wanted to ask what is the best way to structure your statement of intention for your Creative SAC?
Do you have to write about the author's choice of certain features that they used in the original text or is the statement of intention all based on what techniques you used?
So I've read some sentences like "Similar to how the author conveyed (something) through symbolism, I have used a symbol..."
This does acknowledge the author's techniques, but only briefly. So is that all you need to do to make sure you're linking it back to the original text as well in your SOI ?
Because it wouldn't be advisable to dedicate one whole para of your SOI to analyse the original author's devices, etc right? So should we limit this?

Thank you! :)
Title: Re: VCE English Question Thread
Post by: clarke54321 on April 01, 2018, 09:39:53 pm
Hi guys!

Just wanted to ask what is the best way to structure your statement of intention for your Creative SAC?
Do you have to write about the author's choice of certain features that they used in the original text or is the statement of intention all based on what techniques you used?
So I've read some sentences like "Similar to how the author conveyed (something) through symbolism, I have used a symbol..."
This does acknowledge the author's techniques, but only briefly. So is that all you need to do to make sure you're linking it back to the original text as well in your SOI ?
Because it wouldn't be advisable to dedicate one whole para of your SOI to analyse the original author's devices, etc right? So should we limit this?

Thank you! :)


Hello!

Like sdfg has already outlined, the creative response is highly individual to each school. In turn, I'd strongly recommend that you closely follow the preferences of your teacher.

As concerns the crux of your questions, you should absolutely be making reference to the original author. However, this should not merely be a name drop. That is, if you are using a sentence to the effect of, In line with author X, I too have made use of ......, and fail to elaborate on why you have mimicked this particular style, you may lose some easy marks. Always consider, what effect does this technique achieve? How does this technique allow me to better engage with the concerns/world of the text? Just note that you should always be using tentative language here.

Check with your teacher, but in terms of structuring the author's devices, I would avoid keeping it to the one paragraph. While this isn't the worst thing, it certainly seems more natural if you can stagger these across paragraphs. What I felt was more effective was dedicating each paragraph to a 'big idea' of my piece, and then analysing the effect of my techniques (and by extension, the author's) in relation to that main idea.

Hopefully this helps you in some way  :D
Title: Re: VCE English Question Thread
Post by: MissSmiley on April 02, 2018, 12:02:29 am
Hello!

Like sdfg has already outlined, the creative response is highly individual to each school. In turn, I'd strongly recommend that you closely follow the preferences of your teacher.

As concerns the crux of your questions, you should absolutely be making reference to the original author. However, this should not merely be a name drop. That is, if you are using a sentence to the effect of, In line with author X, I too have made use of ......, and fail to elaborate on why you have mimicked this particular style, you may lose some easy marks. Always consider, what effect does this technique achieve? How does this technique allow me to better engage with the concerns/world of the text? Just note that you should always be using tentative language here.

Check with your teacher, but in terms of structuring the author's devices, I would avoid keeping it to the one paragraph. While this isn't the worst thing, it certainly seems more natural if you can stagger these across paragraphs. What I felt was more effective was dedicating each paragraph to a 'big idea' of my piece, and then analysing the effect of my techniques (and by extension, the author's) in relation to that main idea.

Hopefully this helps you in some way  :D

To be honest, your teacher is the only person who can answer this question for you. Every school runs the creative SAC differently so any answer here can steer you in the completely wrong way.

But from my perspective, no, the point of the statement of intention is to explain your intention, not somebody else's, which you in the first place can never know and explain with absolute certainty.

Thanks a lot clarke54321 and sdfg for giving me your thoughts!

Yes I agree, sdfg, it does vary from school to school so I'll follow the structure that our school will give us.
And you're 100% right Clarke!  :D I really like the 'big ideas' approach for every para and then supporting it with examples! Yes just like a text response, it'll be better to include the original author's values as we go rather than a separate para!

Thanks so much guys! :)
Title: Re: VCE English Question Thread
Post by: MissSmiley on April 04, 2018, 10:42:03 pm
Hi guys,
Could I just ask how long your statement of intention needs to be?
If you had to write your creative piece and SOI in 65-70 mins, then taking this into consideration, should your creative piece be around double the length or more of your SOI?
Obviously more marks are awarded for the creative piece, so it's wise to write more for this, but what's the proportion?

Thank you! :)
Title: VCE English Question Thread
Post by: Lear on April 04, 2018, 10:49:56 pm
Mine was roughly 600 words  (creative was 1600) however we had 100 minutes to write so maybe not a fair example
Title: Re: VCE English Question Thread
Post by: Sine on April 04, 2018, 10:52:11 pm
Hi guys,
Could I just ask how long your statement of intention needs to be?
If you had to write your creative piece and SOI in 65-70 mins, then taking this into consideration, should your creative piece be around double the length or more of your SOI?
Obviously more marks are awarded for the creative piece, so it's wise to write more for this, but what's the proportion?

Thank you! :)

depends on what your teacher wants so it's best to ask them
Lear has gone 600 but my teacher's were perfectly fine with anything  around 200-300
Title: Re: VCE English Question Thread
Post by: Guideme on April 06, 2018, 06:57:09 pm
In class my teacher has identified all of the foreshadowing and ironies but i dont know how to incorporate them in my essay. I feel like i if i incorporate them in my essay it will go off topic .
So can anyone provide me a sample sentence with irony and perhaps any tips to incorporate irony and foreshadowing in my analysis to prevent myself from going off-topic

Thank you in advance ! :)
Title: Re: VCE English Question Thread
Post by: clarke54321 on April 06, 2018, 07:15:48 pm
In class my teacher has identified all of the foreshadowing and ironies but i dont know how to incorporate them in my essay. I feel like i if i incorporate them in my essay it will go off topic .
So can anyone provide me a sample sentence with irony and perhaps any tips to incorporate irony and foreshadowing in my analysis to prevent myself from going off-topic

Thank you in advance ! :)

Hello!

This question came up recently in the forums, but I can't seem to find it. Nevertheless, here is an example of irony used in the context of Medea:

Quite literally, Jason seeks to mute and thereby subjugate the self-autonomy of all women, who are palpably defined and restrained by their biological physicality. Hence, Jason’s socially ironic assessment that Medea has ‘known justice’ in Hellas, impels audiences to challenge the truth of this assertion.


As you can see from above, a reference to irony is used for the purpose of illuminating something. In this case, we are trying to foreground the extent of Jason's insincerity. He claims that Medea has known justice, yet negates this in nearly all of his other actions. This is by no means detracting from my argument, but adding to it (here it concerns the double standards existent in Athenian society).

Please feel free to clarify if you are still doubtful  :)
Title: Re: VCE English Question Thread
Post by: Guideme on April 21, 2018, 07:50:28 pm
Hi,
When i write essays, my teacher always gives me feedback "try to be more concise and not to be too wordy", but i am not sure how to become more concise in my writing and reduce the wordiness. So can anyone give me any advice.

Thank You in advance
Title: Re: VCE English Question Thread
Post by: Sine on April 21, 2018, 07:57:27 pm
Hi,
When i write essays, my teacher always gives me feedback "try to be more concise and not to be too wordy", but i am not sure how to become more concise in my writing and reduce the wordiness. So can anyone give me any advice.

Thank You in advance
basically removing anything that is not 100% necessary (or efficient) for the purpose of the essay. Try to read over your past essays and look for any areas where you may have repeated something, used lengthy sentences to explain something you could've said in fewer words, any fluff etc
Title: Re: VCE English Question Thread
Post by: sophiayim on May 01, 2018, 07:27:29 pm
Hi there, i was just curious as to how you would compare two articles and one visual in a block method for a comparative language analysis? Thank you
Title: Re: VCE English Question Thread
Post by: clarke54321 on May 01, 2018, 07:53:38 pm
Hi there, i was just curious as to how you would compare two articles and one visual in a block method for a comparative language analysis? Thank you

Hello,

If you have articles of equivalent size, you may choose to set out your essay in the following way:

BP1- Article 1
BP2- Article 1
BP3- Article 2 (+linking topic sentence)
BP4- Article 2 (+linking topic sentence)

If one article is greater than the other, it could follow this pattern:

BP 1- Article 1
BP 2- Article 1
BP 3- Article 1
BP 4- Article 2 (+linking topic sentence)

In terms of the visual, if it is complimenting one article in particular, then link it to a relevant body paragraph (one relating to that specific article). However, if the visual is on its own (like in the 2016 VCAA paper), then you could link it to an article which shares a similar contention. For the latter option it is ultimately writer's choice.

All the best :)
Title: Re: VCE English Question Thread
Post by: PhoenixxFire on May 02, 2018, 09:19:14 am
How specific does our audience need to be for argument analysis? Like is it okay to say 'people interested in X issue' or is that too generic. I never quite know how to say who the audience is. Sometimes I can identify the audience if it's obvious like footy fans or parents but if its just like an article on the environment in a widely read newspaper then can I just say that the audience is people who care about the environment or is that too broad?
Title: Re: VCE English Question Thread
Post by: sdfg on May 02, 2018, 10:20:25 am
How specific does our audience need to be for argument analysis? Like is it okay to say 'people interested in X issue' or is that too generic. I never quite know how to say who the audience is. Sometimes I can identify the audience if it's obvious like footy fans or parents but if its just like an article on the environment in a widely read newspaper then can I just say that the audience is people who care about the environment or is that too broad?

Nope, people interested in X issue is too general. Rarely ever do authors target an audience that broad. What is their stance, for or against? Identifying the author's intent should also help. Also, do remember that there could be more than one 'target audience', and if the author's well known, that could be used to gauge what their target audience is.

Title: Re: VCE English Question Thread
Post by: clarke54321 on May 02, 2018, 10:21:51 am
How specific does our audience need to be for argument analysis? Like is it okay to say 'people interested in X issue' or is that too generic. I never quite know how to say who the audience is. Sometimes I can identify the audience if it's obvious like footy fans or parents but if its just like an article on the environment in a widely read newspaper then can I just say that the audience is people who care about the environment or is that too broad?

Reference to audience doesn't need to be overly specific. It can most definitely be, 'people who care about the environment.' Just ensure that any kind of speculation made regarding audience is marked with tentative language. That is, always try and insert the word, likely, before assumptions. We cannot know for sure exactly who will be reading article X. Obviously as you progress through your analysis, you can continue to draw implications between the language and the audience used.

Hopefully this helps :)
Title: Re: VCE English Question Thread
Post by: pha0015 on May 04, 2018, 05:44:25 pm
For language analysis, my teacher told us to focus our topic sentences on the intended effect on the audience, rather than an argument, as it'd help centre our piece towards the effect on the audience rather than recounting the article. Would you recommend using this formula, even though I don't think I have much of a problem with retelling the article? Would high scoring students use this formula?
Title: Re: VCE English Question Thread
Post by: Lear on May 04, 2018, 06:04:37 pm

For language analysis, my teacher told us to focus our topic sentences on the intended effect on the audience, rather than an argument, as it'd help centre our piece towards the effect on the audience rather than recounting the article. Would you recommend using this formula, even though I don't think I have much of a problem with retelling the article? Would high scoring students use this formula?

Our teachers told us that it is better to begin paragraphs with identifying arguments as it shows an understanding of the arguments presented by the author. The change in study design has shifted the focus from just language to arguments. I think it might be a good idea to start with something like ‘in order to engender a sense of.... the author posits that...’ doing both.


This only my understanding and might not be accurate.
Title: Re: VCE English Question Thread
Post by: pha0015 on May 04, 2018, 06:20:21 pm
Our teachers told us that it is better to begin paragraphs with identifying arguments as it shows an understanding of the arguments presented by the author. The change in study design has shifted the focus from just language to arguments. I think it might be a good idea to start with something like ‘in order to engender a sense of.... the author posits that...’ doing both.


This only my understanding and might not be accurate.

Yes, I usually do something along the lines of what you said, just not as sophisticated. Thanks
Title: Re: VCE English Question Thread
Post by: clarke54321 on May 04, 2018, 07:12:22 pm
For language analysis, my teacher told us to focus our topic sentences on the intended effect on the audience, rather than an argument, as it'd help centre our piece towards the effect on the audience rather than recounting the article. Would you recommend using this formula, even though I don't think I have much of a problem with retelling the article? Would high scoring students use this formula?

As Lear suggested, it is much more intuitive to frame your paragraph around a central argument. While the main purpose of argument analysis is to analyse the intended effect on the audience, students must first be aware of the argument (and language choices used to articulate this argument) that seeks to kindle a reaction in audiences. A nice method to follow is this:

Topic Sentence- What is the main argument? (You may decide to couple this with an accompanying technique, which bolsters the argument)
Next Sentence- Evidence (how this language is working to achieve an intended audience effect)
Sentences to follow- Repeat
Linking Sentence- What is the relevance of this argument in relation to the contention?

So ultimately, the audience effect is your 'homing point.' Try and always link your analysis to this, but frame your paragraph around the development of the argument itself.


Title: Re: VCE English Question Thread
Post by: Lear on May 06, 2018, 05:48:44 pm
Not sure if the answers to my questions are dependent on text. But i’m studying Medea.

When writing a text response what audience are we to consider? Modern or Classical era?

Also, what is the structure of paragraphs? I’ve always thought that we create 3 ‘arguments’ to answer the question. However,  apparently there is freedom to slowly build up to a final, singular argument through a string of paragraphs that don’t necessarily strictly answer the essay question as long these ultimately together present a thorough and reasoned response to it.
Title: Re: VCE English Question Thread
Post by: Poet on May 06, 2018, 05:59:50 pm
Not sure if the answers to my questions are dependent on text. But i’m studying Medea.

When writing a text response what audience are we to consider? Modern or Classical era?

Also, what is the structure of paragraphs? I’ve always thought that we create 3 ‘arguments’ to answer the question. However,  apparently there is freedom to slowly build up to a final, singular argument through a string of paragraphs that don’t necessarily strictly answer the essay question as long these ultimately together present a thorough and reasoned response to it.

Hey! Me too. You enjoying Medea so far? :)

I think we're meant to mainly take into account the audience the play was originally meant for - a society who would not have sympathized with Medea's character.

And typically, the paragraph structure would be your solid 3 argument basis. It's a bit of a risk to not answer the question until your conclusion - if you don't finish your essay or somehow mess it up, you would get a worse mark than the typical 3-argument response. But in that, you should be integrating each argument so that they can flow into one another and you can include small parts in your main arguments that could come together as a final piece of evidence in your conclusion. :)
Title: Re: VCE English Question Thread
Post by: fruitbowl34 on May 08, 2018, 06:47:08 pm
Hi! I was just wondering if anyone knows what average SAC scores and exam score I need to get a study score of 30, thank you!
Title: Re: VCE English Question Thread
Post by: sdfg on May 08, 2018, 08:12:56 pm
Hi! I was just wondering if anyone knows what average SAC scores and exam score I need to get a study score of 30, thank you!

SAC averages don't matter much in the grand scheme of things - your exam performance, cohort strength, ranking are what really determine your SS in the end. Generally speaking though, a C+ average in SACs and the exam should easily get you a 30, if not higher.
Title: Re: VCE English Question Thread
Post by: MissSmiley on May 08, 2018, 08:28:53 pm
SAC averages don't matter much in the grand scheme of things - your exam performance, cohort strength, ranking are what really determine your SS in the end. Generally speaking though, a C+ average in SACs and the exam should easily get you a 30, if not higher.
Hey!
Do you know where to find the graded distribution?
I'm really stressing out over English, I just want to make sure if I'm on the right track.
What SAC scores do you need to get to get like 40-43?

Everyone says you need to get an A+ average for everything (unit 3, 4 and exam) and I agree with this so I'm trying hard to make sure I fulfil this, but I really don't get how VCAA considers 'A+' to be like something 84% and above?
Please please correct me if I'm right!!
But isn't A+ 90% and above? That's the case for every subject at my school.
If I go by my school's grading system, then my friend who graduated last year was really unhappy with her scores, (she averaged 75-80 and maybe only 1 85%) but still she pulled a 42 ! :)
Obviously she must have done really really well in exams, (probs somewhere in the 80-10 range for every essay) - correct me if I'm wrong please! This is just me imagining her exam scores, I didn't ask her)
So how do we interpret this?
In this case is VCAA's grading system the answer? just because if considers 85% to be an A+ ?

Also, taking into consideration that our teachers mark harshly, what SAC scores would I need to get a 42 or 43?

I know I really shouldn't be falling into all of this, but you know I just can't control my mind and my hands typing all this!!  ;D

Would love to read a reply / any other experiences that people had.
Thanks guys :)
Title: Re: VCE English Question Thread
Post by: Poet on May 08, 2018, 08:40:32 pm
Hey!
Do you know where to find the graded distribution?
I'm really stressing out over English, I just want to make sure if I'm on the right track.
What SAC scores do you need to get to get like 40-43?

Everyone says you need to get an A+ average for everything (unit 3, 4 and exam) and I agree with this so I'm trying hard to make sure I fulfil this, but I really don't get how VCAA considers 'A+' to be like something 84% and above?
Please please correct me if I'm right!!
But isn't A+ 90% and above? That's the case for every subject at my school.
If I go by my school's grading system, then my friend who graduated last year was really unhappy with her scores, (she averaged 75-80 and maybe only 1 85%) but still she pulled a 42 ! :)
Obviously she must have done really really well in exams, (probs somewhere in the 80-10 range for every essay) - correct me if I'm wrong please! This is just me imagining her exam scores, I didn't ask her)
So how do we interpret this?
In this case is VCAA's grading system the answer? just because if considers 85% to be an A+ ?

Also, taking into consideration that our teachers mark harshly, what SAC scores would I need to get a 42 or 43?

I know I really shouldn't be falling into all of this, but you know I just can't control my mind and my hands typing all this!!  ;D

Would love to read a reply / any other experiences that people had.
Thanks guys :)

Hey MissSmiley,
When talking about getting a study score in English above 40, we're definitely talking about being in the higher end of things (in the top 9%) of the state, so the best idea is just to do the best you can. Keep aiming for 100%. But don't worry to much about score numbers such as 84% or 90% - the grade doesn't really matter to VCAA - only your ranking in comparison to your cohort and the rest of the state.
You can keep looking at this stuff on VCAA's website - their FAQ is really helpful for this sort of stuff. :)
Title: Re: VCE English Question Thread
Post by: MissSmiley on May 08, 2018, 08:47:46 pm
Hey MissSmiley,
When talking about getting a study score in English above 40, we're definitely talking about being in the higher end of things (in the top 9%) of the state, so the best idea is just to do the best you can. Keep aiming for 100%. But don't worry to much about score numbers such as 84% or 90% - the grade doesn't really matter to VCAA - only your ranking in comparison to your cohort and the rest of the state.
You can keep looking at this stuff on VCAA's website - their FAQ is really helpful for this sort of stuff. :)
Yup! Definitely the correct way is to aim for very high SAC marks!
Thanks poet :)
Title: Re: VCE English Question Thread
Post by: pha0015 on May 11, 2018, 05:53:56 pm
This is potentially a stupid question, but for the book Medea and Other Plays, is the text response only on Medea or is it inclusive of the other plays?
Title: Re: VCE English Question Thread
Post by: PhoenixxFire on May 11, 2018, 05:54:57 pm
Only Medea
Title: Re: VCE English Question Thread
Post by: Maya24 on May 12, 2018, 03:22:26 pm
What sac scores would you need to get a 30-35 study score?
Title: Re: VCE English Question Thread
Post by: Sine on May 12, 2018, 03:29:58 pm
What sac scores would you need to get a 30-35 study score?
No one can give actual sac scores since it would depend on the strength of your cohort/difficulty of sacs but aim for anywhere between the 50th to 75th percentile of an average cohort. If you have a strong cohort you could afford to be ranked a little lower. Along with a similar exam score.
Title: Re: VCE English Question Thread
Post by: Maya24 on May 12, 2018, 03:36:59 pm
No one can give actual sac scores since it would depend on the strength of your cohort/difficulty of sacs but aim for anywhere between the 50th to 75th percentile of an average cohort. If you have a strong cohort you could afford to be ranked a little lower. Along with a similar exam score.

Thank you
Title: Re: VCE English Question Thread
Post by: I\'m Not A Robot on May 14, 2018, 07:53:16 pm
Hey everyone, i was wondering if somebody could please take a look at this article we got given to annotate for our practice SAC. I'm struggling to find the actual contention of the author along with their arguments. It all seems to be sort of jumbled up. (btw- this is for a language analysis/comparison). Thanks heaps in advance. xD

For 60 years, cruelty to animals on live export ships has been kept secret.

There’s no ‘nice’ way to send animals halfway around the world to be killed for meat.

A live sheep isn’t ‘cargo’. He breathes. He thinks. He can suffer.  This simple fact may be lost on wealthy live export companies. But it wasn’t lost on one concerned crew member who was horrified by the suffering he witnessed on live export voyages...

Over 5 routine shipments carrying sheep from Australia to the Middle East, he carried out his duties. But also, he filmed — risking everything to show the world a glimpse of what has been hidden and ‘accepted’ by this industry for decades.

On these enormous ships, sheep are packed so tightly that many can’t lie down to rest. Nor can they all access food and water. Overcrowded pens make identifying sick and injured animals near impossible.  None of this is legal. Add to this extreme heat, exhaustion, rough seas, poor ventilation and millions of litres of untreated waste, and a typical live export voyage quickly becomes the perfect storm for death and suffering.

Over the years, more than 3 million Australian animals have died, often horrifically, on these death ships.

But evidence now shows that  it is the ‘survivors’ who suffer worst of all. They face the same gruelling nightmare, only their suffering never ends.

The evidence also shows that the Australian Government cannot legally continue to grant export licenses under these conditions.

This footage exposes an industry acting unlawfully. It reveals what millions of gentle animals have endured for decades, in secret.

And with it — a chance to end the cruelty.

More than 100,000 litres of urine and faeces accumulates on a typical live export ship every day sheep are on board. The ship won’t be ‘washed out’ until after they’ve disembarked.

As the northern summer kicks in, on-board conditions can turn catastrophic.

When temperatures soar — and predictably they do —  weeks of untreated waste buildup ‘melts’ into a thick, deadly soup. Any animal needing to lie down to rest risks being buried in excrement.

Corrosive ammonia chokes the air and burns the eyes and throats of those on board. Distressed animals rapidly overheat. Their hearts race as they gasp for oxygen. Trapped in what is essentially a ‘giant oven’, extremely heat stressed animals collapse before  literally being cooked alive. Other heat stressed sheep may die slowly over the following days. Those who survive a ‘heat event’ will continue to suffer in this putrid bog, now littered with the decaying bodies of their dead companions.

Imagine being born on a live export ship. The noisy vibrations the only sound you’ll ever hear. The inescapable stench of faeces the only air you’ll ever breathe…

These babies aren’t supposed to be born at all. Live export companies are legally required to ensure all ewes are pregnancy tested by ultrasound and certified not pregnant. And yet, heavily pregnant ewes are being shipped for slaughter like every other animal. Only — they’ll endure the trauma of live export while going into labour and giving birth.

Very few lambs born on a live export ship will ever see the outside world. Many will be  trampled, lose their mothers, or be killed by distressed crew members who are routinely ordered to slit their throats.

All this suffering is in aide of one thing: making wealthy live export companies even wealthier. And as it turns out, ‘business as usual’ on one of these ships to the Middle East isn’t just cruel, in many ways it’s illegal.
Title: Re: VCE English Question Thread
Post by: sdfg on May 14, 2018, 08:45:58 pm
Hints in blue. :)

Hey everyone, i was wondering if somebody could please take a look at this article we got given to annotate for our practice SAC. I'm struggling to find the actual contention of the author along with their arguments. It all seems to be sort of jumbled up. (btw- this is for a language analysis/comparison). Thanks heaps in advance. xD

For 60 years, cruelty to animals on live export ships has been kept secret.

There’s no ‘nice’ way to send animals halfway around the world to be killed for meat.

A live sheep isn’t ‘cargo’. He breathes. He thinks. He can suffer.  This simple fact may be lost on wealthy live export companies. But it wasn’t lost on one concerned crew member who was horrified by the suffering he witnessed on live export voyages...

Over 5 routine shipments carrying sheep from Australia to the Middle East, he carried out his duties. But also, he filmed — risking everything to show the world a glimpse of what has been hidden and ‘accepted’ by this industry for decades.

On these enormous ships, sheep are packed so tightly that many can’t lie down to rest. Nor can they all access food and water. Overcrowded pens make identifying sick and injured animals near impossible.  None of this is legal. Add to this extreme heat, exhaustion, rough seas, poor ventilation and millions of litres of untreated waste, and a typical live export voyage quickly becomes the perfect storm for death and suffering.

Over the years, more than 3 million Australian animals have died, often horrifically, on these death ships.

But evidence now shows that  it is the ‘survivors’ who suffer worst of all. They face the same gruelling nightmare, only their suffering never ends.

The evidence also shows that the Australian Government cannot legally continue to grant export licenses under these conditions.

This footage exposes an industry acting unlawfully. It reveals what millions of gentle animals have endured for decades, in secret.

And with it — a chance to end the cruelty.

More than 100,000 litres of urine and faeces accumulates on a typical live export ship every day sheep are on board. The ship won’t be ‘washed out’ until after they’ve disembarked.

As the northern summer kicks in, on-board conditions can turn catastrophic.

When temperatures soar — and predictably they do —  weeks of untreated waste buildup ‘melts’ into a thick, deadly soup. Any animal needing to lie down to rest risks being buried in excrement.

Corrosive ammonia chokes the air and burns the eyes and throats of those on board. Distressed animals rapidly overheat. Their hearts race as they gasp for oxygen. Trapped in what is essentially a ‘giant oven’, extremely heat stressed animals collapse before  literally being cooked alive. Other heat stressed sheep may die slowly over the following days. Those who survive a ‘heat event’ will continue to suffer in this putrid bog, now littered with the decaying bodies of their dead companions.

Imagine being born on a live export ship. The noisy vibrations the only sound you’ll ever hear. The inescapable stench of faeces the only air you’ll ever breathe…

These babies aren’t supposed to be born at all. Live export companies are legally required to ensure all ewes are pregnancy tested by ultrasound and certified not pregnant. And yet, heavily pregnant ewes are being shipped for slaughter like every other animal. Only — they’ll endure the trauma of live export while going into labour and giving birth.

Very few lambs born on a live export ship will ever see the outside world. Many will be  trampled, lose their mothers, or be killed by distressed crew members who are routinely ordered to slit their throats.

All this suffering is in aide of one thing: making wealthy live export companies even wealthier. And as it turns out, ‘business as usual’ on one of these ships to the Middle East isn’t just cruel, in many ways it’s illegal.
Title: Re: VCE English Question Thread
Post by: I\'m Not A Robot on May 14, 2018, 10:13:40 pm
Thanks sdfg :)

Also... would you say that the contention in this article is:
"support live exports", "Australia is the only country that actively works in overseas markets to help improve animal welfare conditions" or "If Australia was  to stop exporting livestock, global animal welfare standards will unquestionably decline".
...or something else.

i still can't pinpoint the contention of some of these articles

Support Live Exports
Australia's livestock export industry is committed to working with the Federal Minister for Agriculture, the Australian Government and our trading partners to deliver world class animal welfare outcomes for Australian livestock in all export markets.

Australia is the only country, of the more than 100 countries across the world that export livestock, that actively works in overseas markets to help improve animal welfare conditions. If Australia was to stop exporting livestock, global animal welfare standards will unquestionably decline.

Facts about livestock exports
Fact: If Australia was to stop exporting livestock, animal welfare conditions in overseas countries would decline.
Australia is the only country in the world that actively works in overseas markets to improve animal welfare conditions. We are also the only country in the world that attempts to regulate livestock exports, all the way from Australian farms to feedlots and abattoirs overseas. Our producer levy funds millions of dollars worth of training, education, research and development to improve animal welfare conditions during voyages and in overseas markets. If we were to stop exporting livestock, animal welfare conditions overseas would decline.

Fact: Boxed and chilled meat exports cannot replace livestock exports.
Many countries prefer live animals over boxed or chilled meat. This is often due to the challenge of transporting, storing and refrigerating meat, or due to cultural, religious and financial reasons. If Australia was to stop exporting live animals, the gap would not be filled by Australian chilled or boxed meat, but rather from live animals from other countries.

In 2007, for instance, Australia could not meet the Middle East’s demand for live animals, so animals were imported from Sudan, Somalia and Iran – countries that do not share Australia’s commitment to animal welfare, and critically, may also pose animal disease risks. Boxed and chilled meat exports cannot replace livestock exports.

Fact: The livestock export industry supports thousands of Australian jobs & is helping to provide protein to some of the world’s poorest people.
The Australian livestock export industry provides 13,000 jobs, including 11,000 in rural areas, to Australian workers and in some parts of Australia, is the entire backbone of the community and economy. The supply of Australian livestock also ensures hundreds of thousands of households across Asia and the Middle East have access to essential and affordable protein.

These countries do not have the resources or geography to efficiently produce livestock to feed their people, and Australia is able to meet this demand through livestock exports. The Federal Government has just released its Asian Century White Paper, which shows Australia has an important role to play in providing food to Asia’s growing population – and livestock exports can be part of the food solution.
Title: Re: VCE English Question Thread
Post by: literally lauren on May 15, 2018, 05:10:30 pm
i still can't pinpoint the contention of some of these articles
Here's an easy formula for finding the contention of any article. I'll use this one as an example.

How to find contentions - a 5 step process
1. What is the issue?
Even if you're not entirely sure what the author is arguing for/against, you'll be able to tell what they're talking about.
For example, the article above is obviously about the issue of live animal exports. HOWEVER, we don't want to oversimplify the issue, because that will make the process of finding the contention much harder. Instead, we need to be as specific as possible about what the issue is.
I like to do this by starting the phrase with "whether..." For example: whether Australia should stop live animal exports.
That gives us a nice, specific issue to take into step 2.

2. Does the author think the issue is GOOD or BAD?
Again, even if you can't come up with a really impressive statement about the contention straight away, you'll know whether the author is broadly in favour or against the core issue.
In this case, the author is against the idea of stopping live animal exports, thus he views the issue as a BAD thing.

3. In what way is the issue good/bad according to the author?
This is where we need to get more descriptive with our vocabulary (because if you literally wrote 'the author thinks the issue is a bad thing' for your SAC/exam, they would tear you and your essay to pieces).
There are SO MANY different ways something can be 'good' or 'bad', and as always, we're trying to be as specific as possible. If nothing comes to mind right away, consulting a thesaurus can be useful here:
(https://i.imgur.com/f9WxE9R.png)(https://i.imgur.com/UQLzrh3.png)
In this case, the author seems to be suggesting that the issue (i.e. Aus. stopping live animal exports) would be foolish, counterintuitive, and harmful.

4. Why is this good/bad, according to the author?
Now we need to start delving into the reasons for the author's stance. In this case, he is suggesting that stopping the exports is bad/counterintuitive because it would harm Australia, the overseas market, and the welfare standards of the livestock. In simpler terms: it is bad because it is disadvantageous for everyone involved. This leads us to the final step...

5. What does the author want to happen? What is their 'best case scenario' outcome?
Here, we need to think about the author's goal - what would be a better proposal or idea? Sometimes this is explicitly told to us (e.g. at the end of the article, he writes "livestock exports can be part of the food solution") but other times it will be more implicit (e.g. this article suggests that stopping live exports would be a bad idea, and that therefore live exports should continue).

To turn this into a sentence you can use in your essay, use the following formula:
The author contends that (1.) is (3.) because (4.), and so (5.)

For example:
The author contends that Australia ceasing its live animal exports is a counterintuitive idea because doing so would be harmful for Australia, other nations, and the welfare of livestock, and so Australia should instead support the export industry so that it can be part of the "food solution."

This gives you a nice, sophisticated summary of the contention, and that formula should work for virtually any persuasive piece. Remember to vary the wording as needed, and if you get really confident with this, you don't have to stick to the formula at all. But these elements are more or less what the assessors will be looking for, so if you can give that to them in a clear and concise sentence like the one above, they will be in a great mood reading the rest of your essay! :)

Also, one more quick thing:
... would you say that the contention in this article is:
"support live exports", "Australia is the only country that actively works in overseas markets to help improve animal welfare conditions" or "If Australia was  to stop exporting livestock, global animal welfare standards will unquestionably decline".
...or something else.
When identifying the author's contention, avoid using quotes!!! Some teachers are okay with this (and I've used a small one in the sample sentence above ("food solution")) but in general, you will be at a disadvantage in the exam if you are looking for phrases in the article that summarise the contention for you!

Instead, try and rely on your own ability to paraphrase, or use a formula like the one I've outlined here. It might be tougher at first, but it will make it much easier for you if you get some tricky material. ;)
Title: Re: VCE English Question Thread
Post by: vceme on May 15, 2018, 10:15:39 pm
Hey everyone!
Could anyone who is doing The White Tiger or is just an overall champ help me with unpacking this prompt! I'm currently trying to do some practice sacs:

“Balram is portrayed as a flawed hero in The White Tiger, but a hero nonetheless”

-Yes, Balram is a hero for breaking out of the 'darkness' but he is ultimately a sociopath whose flawed actions are justified by readers as a victim of India's corruption/inequality etc. therefore readers classify him as a hero.

I'm not too sure if i've gone the right path in challenging the prompt. Any help would be appreciated! Also wrote a prac introduction for this:

Aravind Adiga, author of ‘The White Tiger’ critiques India’s prospering globalisation and democracy for enshrouding a socio-economic system that is plagued by a culture of servitude and rampant political and economic corruption. The protagonist, Balram Halwai narrates his bloodstained struggle of emancipating himself from utter poverty and destitution to becoming a successful entrepreneur. Succeeding in his desire for freedom out of the darkness, Balram is characterised as a hero despite his morally dubious acts. However, Balram is ultimately a sociopath whose flawed acts are justified by readers as a victim of India’s systematic inequality.

Title: Re: VCE English Question Thread
Post by: MissSmiley on May 20, 2018, 09:21:25 pm
Hi everyone!

I was just wondering for Argument Analysis, do you always need to write about say a pre-buttal in your first para?
For example, in last year's exam, Walker was saying things like "oh no!" I hear you say and "she has got another idea" etc --> this would be the pre-buttal

In class we discussed how the first para (also when she says how her school's truly organic and basically she's proud) would be the first argument. It's likely to make readers take her side.
My teacher told us to dedicate a whole para (it can be short, but still recognised as an argument) on the pre-buttal.

Is this ok though? I've never considered analysing things like this, because I always get that feeling of 'stagnancy.' It's like fine we get it, she's trying to get readers on her side. this sounds too generalised, doesn't it?

I wrote a practice piece and then my teacher told me to analyse the first para in the newsletter message as my first argument.
I only mentioned the tone in that para, just to start my piece off, but I didn't dedicate a whole para to this.
But still managed to find 3 arguments from last year's exam (from Walker's writing)

So, is it a good idea to dedicate the first para to the pre-buttal or not?

Thanks so much! :)
Title: Re: VCE English Question Thread
Post by: sdfg on May 20, 2018, 09:41:52 pm
Hi everyone!

I was just wondering for Argument Analysis, do you always need to write about say a pre-buttal in your first para?
For example, in last year's exam, Walker was saying things like "oh no!" I hear you say and "she has got another idea" etc --> this would be the pre-buttal

In class we discussed how the first para (also when she says how her school's truly organic and basically she's proud) would be the first argument. It's likely to make readers take her side.
My teacher told us to dedicate a whole para (it can be short, but still recognised as an argument) on the pre-buttal.

Is this ok though? I've never considered analysing things like this, because I always get that feeling of 'stagnancy.' It's like fine we get it, she's trying to get readers on her side. this sounds too generalised, doesn't it?

I wrote a practice piece and then my teacher told me to analyse the first para in the newsletter message as my first argument.
I only mentioned the tone in that para, just to start my piece off, but I didn't dedicate a whole para to this.
But still managed to find 3 arguments from last year's exam (from Walker's writing)

So, is it a good idea to dedicate the first para to the pre-buttal or not?

Thanks so much! :)

For your SAC, I'll say do it because your teacher seems pretty persistent about it and they're the marking your work in the end, but for the exam, I'll say it depends and you'll have to be selective about whether or not to include it. Consider its significance to the author's argument (e.g are they a polarising public figure or from a demographic that usually isn't taken seriously? What's the relevance of the statement made to the broader issue?) and make a judgement from there.







Title: Re: VCE English Question Thread
Post by: BNard on May 22, 2018, 12:11:50 pm
Hey everyone!
Could anyone who is doing The White Tiger or is just an overall champ help me with unpacking this prompt! I'm currently trying to do some practice sacs:

“Balram is portrayed as a flawed hero in The White Tiger, but a hero nonetheless”

-Yes, Balram is a hero for breaking out of the 'darkness' but he is ultimately a sociopath whose flawed actions are justified by readers as a victim of India's corruption/inequality etc. therefore readers classify him as a hero.
This is slightly confused – you appear to be both challenging (‘ultimately a sociopath’) and not challenging (‘readers classify him as a hero’) the prompt, which is a tricky path to walk.
Quote
I'm not too sure if i've gone the right path in challenging the prompt. Any help would be appreciated! Also wrote a prac introduction for this:

Aravind Adiga, author of ‘The White Tiger’ critiques India’s prospering globalisation and democracy for enshrouding a socio-economic system that is plagued by a culture of servitude and rampant political and economic corruption.
I would aim for a less convoluted introductory sentence. Don’t be afraid to have a less specific sentence first, and then expand on it in the rest of your intro. To make it easy, you can have a format ready to go that allows you to substitute the topic. 
E.g. Aravind Adiga’s The White Tiger explores ________, exposing ___________.
OR Aravind Adiga’s social critique, The White Tiger, explores ____
They’re not the most revolutionary sentence starters, but they don’t have to be! It’s better to be clear and brief than confuse the examiner with overly complicated language.
Quote
The protagonist, Balram Halwai narrates his bloodstained struggle of emancipating himself from utter poverty and destitution to becoming a successful entrepreneur.
I would rephrase:
‘In narrating his struggle to escape the entrenched slavery that represses India’s subaltern, flawed protagonist Balram Halwai describes his rise to the world of ‘entrepreneurs’ as a story of inspiration and heroism.’
Quote
Succeeding in his desire for freedom out of the darkness, Balram is characterised as a hero despite his morally dubious acts. However, Balram is ultimately a sociopath whose flawed acts are justified by readers as a victim of India’s systematic inequality.

Once again, this contention is a little bit murky. Are you trying to say that readers can’t see that Balram is really a sociopath, because that becomes confusing (as you are also a reader). Also, remember that it’s a good idea to at least briefly (and not too blatantly) outline the three arguments that you will use to support your contention in subsequent paragraphs.
I would choose to argue that Balram’s narration fails to disguise his fundamental moral corruption, therefore rendering him an ‘antihero’, neither hero or villain. You could support this with discussion about how the picture of hopelessness he created in describing his upbringing aims to convince readers that he is a victim of a society in which animalistic corruption rules. You could then discuss how it is his own narration and choice to present himself as a victim that exposes him as an unforgiveable ‘murderer’. You could then talk about how despite his efforts to shape his story into that of a triumphant escape from a life of oppression, the only escape that Balram has truly made is that from any moral compass that he once adhered to.

Overall - there is some good vocab in here, it just needs to be rejigged a bit to improve clarity.  This topic is an absolute classic TWT prompt - I'd suggest that you really try to craft a great essay on it because there's a good chance you'll be able to recycle some of the material if you write on this text in the exam :)
Title: Re: VCE English Question Thread
Post by: vceme on May 24, 2018, 03:38:22 pm
This is slightly confused – you appear to be both challenging (‘ultimately a sociopath’) and not challenging (‘readers classify him as a hero’) the prompt, which is a tricky path to walk.I would aim for a less convoluted introductory sentence. Don’t be afraid to have a less specific sentence first, and then expand on it in the rest of your intro. To make it easy, you can have a format ready to go that allows you to substitute the topic. 
E.g. Aravind Adiga’s The White Tiger explores ________, exposing ___________.
OR Aravind Adiga’s social critique, The White Tiger, explores ____
They’re not the most revolutionary sentence starters, but they don’t have to be! It’s better to be clear and brief than confuse the examiner with overly complicated language.I would rephrase:
‘In narrating his struggle to escape the entrenched slavery that represses India’s subaltern, flawed protagonist Balram Halwai describes his rise to the world of ‘entrepreneurs’ as a story of inspiration and heroism.’
Once again, this contention is a little bit murky. Are you trying to say that readers can’t see that Balram is really a sociopath, because that becomes confusing (as you are also a reader). Also, remember that it’s a good idea to at least briefly (and not too blatantly) outline the three arguments that you will use to support your contention in subsequent paragraphs.
I would choose to argue that Balram’s narration fails to disguise his fundamental moral corruption, therefore rendering him an ‘antihero’, neither hero or villain. You could support this with discussion about how the picture of hopelessness he created in describing his upbringing aims to convince readers that he is a victim of a society in which animalistic corruption rules. You could then discuss how it is his own narration and choice to present himself as a victim that exposes him as an unforgiveable ‘murderer’. You could then talk about how despite his efforts to shape his story into that of a triumphant escape from a life of oppression, the only escape that Balram has truly made is that from any moral compass that he once adhered to.

Overall - there is some good vocab in here, it just needs to be rejigged a bit to improve clarity.  This topic is an absolute classic TWT prompt - I'd suggest that you really try to craft a great essay on it because there's a good chance you'll be able to recycle some of the material if you write on this text in the exam :)

Hey!
Thank you so much for taking the time to dissect my introduction and contention. I do admit that my contention was confusing and needed more improvement for clarity. I have fortunately revised my contention so its less wishy-washy :P  My teacher also told me that short sentences are great to enforce my point! so I definitely will take that feedback on. I am currently working on this essay with my revised contention and I will consider your perspective on this prompt. Thank you so much!!!!  :)
Title: Re: VCE English Question Thread
Post by: DBA-144 on May 24, 2018, 09:54:49 pm
 

  This topic is an absolute classic TWT prompt - I'd suggest that you really try to craft a great essay on it because there's a good chance you'll be able to recycle some of the material if you write on this text in the exam :)


What do you mean by TWT prompt? Please clarify further, this seems pretty interesting.
Title: Re: VCE English Question Thread
Post by: vceme on May 24, 2018, 10:07:50 pm


What do you mean by TWT prompt? Please clarify further, this seems pretty interesting.

TWT prompt stands for 'The White Tiger' prompt being. The intro I wrote was for the novel 'The White Tiger'.
Title: Re: VCE English Question Thread
Post by: vceme on May 24, 2018, 10:10:20 pm
Hi everyone!
My teacher says I need to put more textual evidence despite my attempts at embedding quotes into my paragraphs. I've seen past high-scoring essays done for our SACs that explain an important passage but how do I avoid the whole re-telling thing! Also, in a TR how much of author's intention statements should I put in, or rather how do I avoid my TR essay from sounding like a language analysis on a novel. Thank you!!!
Title: Re: VCE English Question Thread
Post by: DBA-144 on May 25, 2018, 04:31:17 pm
TWT prompt stands for 'The White Tiger' prompt being. The intro I wrote was for the novel 'The White Tiger'.


oops.  :-[ Sorry. Silly me.  :P

What are the different types of prompts? I know that there is structural, thematic, character? What other types are there?

Further, how should we deal with prompts which include quotes directly from the book? Do we include them in our introduction and deal with the implications/repercussions of the quotation in our work? Or, do we include the quote's context in the intro and then base our body paragraphs on what this means in the text as a whole?

TIA.
Title: Re: VCE English Question Thread
Post by: DipsetForever on May 27, 2018, 05:04:15 pm
Language Analysis Question: What does it mean to have "perceptive" understanding of arguments/views expresssed? and how would you demonstrate it without writing your own thoughts and opinions?
Title: Re: VCE English Question Thread
Post by: MissSmiley on May 27, 2018, 05:13:44 pm
Language Analysis Question: What does it mean to have "perceptive" understanding of arguments/views expresssed? and how would you demonstrate it without writing your own thoughts and opinions?
It just means trying to get into the readers' shoes and feeling what they would feel. Not you as a neutral reader, but the stakeholders in the issue.
(e.g. government ministers, animal rights activists, etc)
How they would feel + act as a result of reading the arguments that the author has introduced.
Analysing language perceptively also means trying to figure out vocab choices and other fine details that the writer uses to argue their point.
Thanks :)
Title: Re: VCE English Question Thread
Post by: vceme on May 28, 2018, 07:04:44 am


oops.  :-[ Sorry. Silly me.  :P

What are the different types of prompts? I know that there is structural, thematic, character? What other types are there?

Further, how should we deal with prompts which include quotes directly from the book? Do we include them in our introduction and deal with the implications/repercussions of the quotation in our work? Or, do we include the quote's context in the intro and then base our body paragraphs on what this means in the text as a whole?

TIA.

I think you covered all the types of prompts, but anyone correct me if I'm wrong!
My teacher says that you should mention the quote anywhere in your body paragraphs. You should include where the quote has come from (at least who said it) as this demonstrates textual knowledge. You should probably talk about the implications of this quote for a higher scoring essay  ;D (Again, still in year 12 so I might be wrong  :) )
Title: Re: VCE English Question Thread
Post by: DipsetForever on May 29, 2018, 10:45:45 pm
My english teacher hands the class the text we must analyse a week before our SAC... is this bad as we don’t have the same conditions when it comes to the exam?
Title: Re: VCE English Question Thread
Post by: S200 on May 29, 2018, 10:51:11 pm
My english teacher hands the class the text we must analyse a week before our SAC... is this bad as we don’t have the same conditions when it comes to the exam?
Is this Language Analysis?

If so, well we have the same.

As long as your teacher makes you aware of good exam practice it should be Ok.

Try doing some timed responses as practice by yourself...
Title: VCE English Question Thread
Post by: Lear on May 30, 2018, 09:27:19 pm
EDIT: Realised VATE Medea documentation is copy right material. Please ignore.
Title: Re: VCE English Question Thread
Post by: Umattty on June 02, 2018, 07:05:17 pm
Hi Everyone,  ;D

This is my first post, so hopefully this is the correct place to post my English Question.

For Argument Analysis, am i allowed to analyse the placement of the main contention and the placement of argument?

For instance, if i was to analyse an article that included the main contention at the end of the article can i say that it shows that the writer has considered both sides of the issues before coming to a conclusion, eliciting a reasoned tone. Thus, inclining readers to support his argument as it is based on a thoughtful and balanced approah.

Hopefully this makes sense!

Also, Thanks in advance  :) ;)
Title: Re: VCE English Question Thread
Post by: S200 on June 02, 2018, 07:26:22 pm
For Argument Analysis, am i allowed to analyse the placement of the main contention and the placement of argument?
For instance, if i was to analyse an article that included the main contention at the end of the article can i say that it shows that the writer has considered both sides of the issues before coming to a conclusion, eliciting a reasoned tone. Thus, inclining readers to support his argument as it is based on a thoughtful and balanced approach

Yes!
This is actually an entirely recommended technique, as it exemplifies your understanding of the piece and it's arguments.
Title: Re: VCE English Question Thread
Post by: clarke54321 on June 02, 2018, 07:29:57 pm
Hi Everyone,  ;D

This is my first post, so hopefully this is the correct place to post my English Question.

For Argument Analysis, am i allowed to analyse the placement of the main contention and the placement of argument?

For instance, if i was to analyse an article that included the main contention at the end of the article can i say that it shows that the writer has considered both sides of the issues before coming to a conclusion, eliciting a reasoned tone. Thus, inclining readers to support his argument as it is based on a thoughtful and balanced approah.

Hopefully this makes sense!

Also, Thanks in advance  :) ;)

Hello and welcome to AN  :)

This is most definitely the right place to be posting English questions. Thank-you for considering this.

Absolutely. If a contention comes through at the end of an article, there is most likely to be a good reason for this. And like you've mentioned, a writer may be using their article to lead readers through alternative perspectives and rely on logic to prove their stance as the ultimate one. It's the same way that you could analyse a contention that comes through in the title. These pieces are often more exigent in nature and are seeking to express caution early on. As concerns argument, these are often chronological and should form the basis of your body paragraphs as such.

Hopefully this helps.
Title: Re: VCE English Question Thread
Post by: smamsmo22 on June 03, 2018, 12:16:34 pm
Hi, just a couple of language analysis-related questions;

1. Is the analysis on the exam usually comparative/involving texts from 2 different authors?? I know that was the case last year and I was wondering if that's the norm now, I was always under the impression it was a single text by one author, but clearly it isn't always. If anyone can clarify that would be great !!
2. Is there a preference between setting up your analysis as chronologically (i.e.; going through the text from start to end and breaking up paragraphs where thought appropriate) or organising it into main author arguments (i.e. finding 2/3/4 main recurring points used by the author to argue their contention and analysing the language surrounding these specific ideas for each paragraph, regardless of where they appear chronologically in the text)? I have had different teachers teach me different things, and I've become comfortable with the chronological style but it seems the latter is more popular with teachers at my school at least?? Does it add to the sophistication and coherence of your piece?? (I know ultimately its my piece and my decision but if I can maximise the quality of my piece I'm willing to rethink my structure !!

Thanks
Title: Re: VCE English Question Thread
Post by: S200 on June 03, 2018, 10:20:52 pm
-snip-
As always, English being subjective, this isn't necessarily correct. But this is what my teacher told me (paraphrased)...

1.) No-one really knows, except VCAA. But it's a great idea to practice both types (single piece and multi). My teachers thought? - most likely another 2 piece, that is, an Article, with an image as part of it, and a comment (possible letter to editor) in reply. Key advice was that images are always included for a reason.

2.) Best analysis structure that we were taught depended on the article/s being analysed.
Single is pretty obvious.
Double article of equal length? The recommendation was to pick similar arguments in both texts (when possible). You can then analyse the differences in persuasive techniques to the best advantage.
Have two texts but it's the article/comment scenario? Similar to the double, but you focus analysis on the article, drawing in the comment where you can to compare and contrast. Remember! Chronology is not imperative, and analysing the article as a whole is a much more preferred technique compared to just combing through and spouting out language devices when you hit them...


I hope this helps, but I would be remiss to leave without mentioning these articles/posts... :D

EDIT - Pertinent quote regarding #2.
Quote from: Aidan in his Video
If you’re having trouble organising this logically, some students find it easier to think of it chronologically. But just remember: this is a slightly easier approach, and something that a lot of students do. So it might be useful to structure it in a slightly more complicated way, to really wow the examiners.
Title: Re: VCE English Question Thread
Post by: smamsmo22 on June 04, 2018, 04:41:42 pm
As always, English being subjective, this isn't necessarily correct. But this is what my teacher told me (paraphrased)...

1.) No-one really knows, except VCAA. But it's a great idea to practice both types (single piece and multi). My teachers thought? - most likely another 2 piece, that is, an Article, with an image as part of it, and a comment (possible letter to editor) in reply. Key advice was that images are always included for a reason.

2.) Best analysis structure that we were taught depended on the article/s being analysed.
Single is pretty obvious.
Double article of equal length? The recommendation was to pick similar arguments in both texts (when possible). You can then analyse the differences in persuasive techniques to the best advantage.
Have two texts but it's the article/comment scenario? Similar to the double, but you focus analysis on the article, drawing in the comment where you can to compare and contrast. Remember! Chronology is not imperative, and analysing the article as a whole is a much more preferred technique compared to just combing through and spouting out language devices when you hit them...


I hope this helps, but I would be remiss to leave without mentioning these articles/posts... :D

EDIT - Pertinent quote regarding #2.

Thanks so much for this!!!
Again regarding #2, are you recommending analysing both articles at once? i.e.; mentioning both articles in the introduction then having paragraphs organised by common ideas during which you're constantly referencing (+ comparing) both articles? Or analyse one, then analyse the other while making comparisons? Thanks again and sorry if this question is unclear!
Title: Re: VCE English Question Thread
Post by: S200 on June 04, 2018, 04:45:32 pm
Thanks so much for this!!!
Again regarding #2, are you recommending analysing both articles at once? i.e.; mentioning both articles in the introduction then having paragraphs organised by common ideas during which you're constantly referencing (+ comparing) both articles? Or analyse one, then analyse the other while making comparisons? Thanks again and sorry if this question is unclear!
Yes. Common ideas, and analysing how each author presents them differently (techniques).
Title: Re: VCE English Question Thread
Post by: MissSmiley on June 04, 2018, 07:45:26 pm
Hi all,

I'm feeling really stressed about my Argument Analysis SAC, and especially when I handed a practice piece and my teacher said I need to work on 'argument development / construction'...
How can I develop a 'comprehensive and complex understanding of the arguments presented in the text and how they've been constructed to position audiences' ?

It's really annoying, because I'm analysing three arguments in the main piece, then always comparing and contrasting, and she's happy with my analysing of techniques,
but what else does she want?

Could someone please please help?!

I'm really very scared and would love to hear some ideas on how I can write 'comprehensively'

Thanks so much guys!!
Title: Re: VCE English Question Thread
Post by: clarke54321 on June 04, 2018, 08:14:56 pm
Hi all,

I'm feeling really stressed about my Argument Analysis SAC, and especially when I handed a practice piece and my teacher said I need to work on 'argument development / construction'...
How can I develop a 'comprehensive and complex understanding of the arguments presented in the text and how they've been constructed to position audiences' ?

It's really annoying, because I'm analysing three arguments in the main piece, then always comparing and contrasting, and she's happy with my analysing of techniques,
but what else does she want?

Could someone please please help?!

I'm really very scared and would love to hear some ideas on how I can write 'comprehensively'

Thanks so much guys!!


The best way to demonstrate evidence of 'argument development,' is through clever use of linking language. Take this paragraph as an example:

Singh, however, scornfully undermines Brown’s idealistic future with the M-I, by casting the card as a deceptive invention. In employing a seemingly jocular pun in her headline, ‘M-I card- or YOUR Card,’ Singh seeks to quickly engage reader attention and then unveil a more serious concern. Indeed, the capitalised and thus protruding word ‘YOUR,’ endeavours to engender a sense of exigency in readers, who are compelled to question just how personal and secure the card is. This alarm created by Singh thereby diverges from the tranquil and assured manner of Brown, who maintains the card will bring ‘peace of mind for everyone.’ Given this palpable contrast, readers are thus inclined to receive Brown’s carefree future with an inkling of scepticism. Such an inclination is bolstered by Singh, who constructs a mock job interview to allude to the card’s discriminatory nature. With the employer bluntly declaring that the employee ‘don’t call [them]’ after analysing the M-I, Singh indicates that the private information held on the cards may have the potential to jeopardise future job prospects for Australians. In turn, Singh attempts to inspire acute levels of doubt in readers, who are positioned to perceive the card as an inequitable and invasive tool. Hence, far from the ‘improved’ future postulated by Brown, Singh establishes the M-I as a prospect that will thwart the opportunities of some.

Here, the red text seeks to demonstrate the way the writer has gone about developing their argument. Each point of evidence is linked together in some way. This makes it easier for analysis to come together in a cohesive fashion and clarify just how the author is conveying their main point. Hopefully this helps :)
Title: Re: VCE English Question Thread
Post by: MissSmiley on June 04, 2018, 08:37:55 pm
The best way to demonstrate evidence of 'argument development,' is through clever use of linking language. Take this paragraph as an example:

Singh, however, scornfully undermines Brown’s idealistic future with the M-I, by casting the card as a deceptive invention. In employing a seemingly jocular pun in her headline, ‘M-I card- or YOUR Card,’ Singh seeks to quickly engage reader attention and then unveil a more serious concern. Indeed, the capitalised and thus protruding word ‘YOUR,’ endeavours to engender a sense of exigency in readers, who are compelled to question just how personal and secure the card is. This alarm created by Singh thereby diverges from the tranquil and assured manner of Brown, who maintains the card will bring ‘peace of mind for everyone.’ Given this palpable contrast, readers are thus inclined to receive Brown’s carefree future with an inkling of scepticism. Such an inclination is bolstered by Singh, who constructs a mock job interview to allude to the card’s discriminatory nature. With the employer bluntly declaring that the employee ‘don’t call [them]’ after analysing the M-I, Singh indicates that the private information held on the cards may have the potential to jeopardise future job prospects for Australians. In turn, Singh attempts to inspire acute levels of doubt in readers, who are positioned to perceive the card as an inequitable and invasive tool. Hence, far from the ‘improved’ future postulated by Brown, Singh establishes the M-I as a prospect that will thwart the opportunities of some.

Here, the red text seeks to demonstrate the way the writer has gone about developing their argument. Each point of evidence is linked together in some way. This makes it easier for analysis to come together in a cohesive fashion and clarify just how the author is conveying their main point. Hopefully this helps :)
Thanks heaps, Clarke!!
This is such a well-written para!! --> You're an absolute GENIUS !!  :D
This does help, because now I can see the difference between effect of reader and the writer's own choices about why they've positioned certain things to be the way they are
I'll keep practising :)
Thanks so much!
Title: Re: VCE English Question Thread
Post by: imanerd on June 13, 2018, 07:13:52 pm
Hello Everyone  :),
i have a few questions regarding the structure of a argumentative analysis essay
 1. My teacher tells us to not write our contention in our introduction because apparently it will be too 'long'. Furthermore, she said it is good way to start our body paragraph with a contention. Tbh, i find the structure to be very arkward. So, in the exam should i write the contention in the introduction or the body? Or does it not matter?

2. Another thing my teacher told me, is that if we intend to analyse the articles title in the body paragraph, we are not allowed to state the title in the introduction because apparently our essay will sound repetitive. Again, for the exam should i include the title in the introduction if i analyse it in my body paragraphs?

3. One last question, i have been trying to understand the difference between invoke and evoke. So i would appreciate if someone is willing to explain the differences between them.

Thanks!
Title: Re: VCE English Question Thread
Post by: imanerd on June 13, 2018, 09:13:31 pm
1. To preface, contention = the main argument of the article/material, and whatever points the author use to support the contention are called the supporting points/arguments/sub-points/whatever else you want to call it. You should definitely state the contention in the introduction as that's where you'll frame the direction of your essay, and the point of your essay is to analyse how the author furthers their contetion. What you wouldn't do is outline the entire line of argument, from start to finish with all the details; the articles you'll be given are going to be too long for this, and even if you manage to squeeze it all in in a succinct a manner, you'll sacrifice too much detail that there'll be no point in doing it in the first place. Depending on how you structure your essay, the supporting points should be the topic sentences of your body paragraph (and not the contention). Are you sure that's what your teacher told you and you're not misunderstanding them, because honestly it's a bit odd for your teacher to tell you to do the exact opposite of what the common consensus is to do.

2. Always state the title in the introduction. The only time where you wouldn't restate the title again is when you're analysing it at the start at the first body paragraph (where in that case it'll be a bit repetitive if you do).

3. For invoke, think invoking the devil or "I invoke thee" or something like that. Evoke, on the hand, is bringing out something that's already there (e.g your feelings - you have them somewhere deep inside you and provided the correct stimulus, they come out to the front). For argument analysis, you'll probably only ever use evoke.

Hope that helps!

Thank you for answering my questions. 
i am sure my teacher told me not to state the contention in the introduction because she gave us list of what we can and cant write in the introduction and contention was in it. i wont include it in for the SAC, but i will definitely include it in the exam. i guess it sucks to go to a disadvantaged school where only one student gets 40+ for English each year lol

 Furthermore, i am still quite sceptical about the difference of invoke and evoke, even though my teacher explained it in class lol. From reading your answer, does invoke mean bring up something that is tangible ? Whereas, evoke means bringing up something that cannot be touched?
Title: Re: VCE English Question Thread
Post by: DipsetForever on June 19, 2018, 09:36:58 pm
Langauge Analysis - Help with reader/audience effect

Tips on writing a more concise (and less generic?) reader response.


Title: Re: VCE English Question Thread
Post by: lifeisgood on June 20, 2018, 09:52:22 am
im doing an oral presentation for English next term. i just don't know what to do though i was either thinking of doing it on homelessness or refugees on manus island.
would this be a could contention:
1. homelessness is a growing problem in Australia and more should be done to tackle this issue.
yeh im not sure about that one. its a very good topic but i don't know what argue or what to include in the arguments.

does anyone have any suggestion?
Title: Re: VCE English Question Thread
Post by: MAGGOT on June 20, 2018, 10:31:21 am
im doing an oral presentation for English next term. i just don't know what to do though i was either thinking of doing it on homelessness or refugees on manus island.
would this be a could contention:
1. homelessness is a growing problem in Australia and more should be done to tackle this issue.
yeh im not sure about that one. its a very good topic but i don't know what argue or what to include in the arguments.

does anyone have any suggestion?

First off, jot down your ideas associated to homelessness in Australia. Research is a person's best friend. Possible ideas you can talk about is that:
-the rate of the Australian population is increasing drastically and there isn't sufficient land
-a lot of people get kicked out due to being different and that their friends or families don't accept them
-the homeless have no where to go it's a lose lose situation for them
-have more job opportunities
-a lot homelessness is due to domestic violence, alcohol, gambling and lack of finances and maybe these areas need more advertisement on top of what the Australian government already has 
I can't think anymore on top of my head :(
Title: Re: VCE English Question Thread
Post by: lifeisgood on June 20, 2018, 09:33:54 pm
Those are some pretty good ideas. I've done some  research. And I was thinking about arguing that homelessness is complex and cannot be simply solved by funding more public housing. Would you that be good.
Soz
Title: Re: VCE English Question Thread
Post by: OZLexico on June 20, 2018, 09:53:28 pm
I also like that statement as a frame/starting point for examining that issue - have you thought of looking at/researching the solutions that are used in other countries (ie similar economies like Canada? Denmark?). If you've got time you might like to check out a book called "White Man's Burden". Its about foreign aid but has some alarming analysis about why throwing money at things doesn't work.  I think the author's name is Easterley, not sure of the spelling(?)
Title: Re: VCE English Question Thread
Post by: fruitbowl34 on June 22, 2018, 08:19:45 pm
Hi!! I'm having some trouble coming up with a contention for my oral presentation. I know my main argument is that reality tv shows can severely psychologically harm it's participants, but I can't seem to come up with a good contention. Any ideas would be greatly appreciated!
Title: Re: VCE English Question Thread
Post by: lifeisgood on June 24, 2018, 09:14:44 am
Hi!! I'm having some trouble coming up with a contention for my oral presentation. I know my main argument is that reality tv shows can severely psychologically harm it's participants, but I can't seem to come up with a good contention. Any ideas would be greatly appreciated!
Hi You could try either:
1.reality tv has a negative implication on society
2. Reality has harmful effect on teens
3. Reality t.v. is really just a distortion of our society/skews the truth
4. reality tv does more harm than good.
5.reality shows don't provide anything valuable to our society.
6.theres nothing new just more and more stupidity.is entirely based on swearing, crying and arguments, and often violence, drunkenness and sex.(I think that sums it up)
7.Reality shows are getting worse
8.Tv producers are just getting lazy.
9.All reality shows are basically the same.
The last few I just got lazy😂😂.but it's so true. Anyway hoped that helped.
Title: Re: VCE English Question Thread
Post by: smamsmo22 on June 28, 2018, 05:19:09 pm
Hi, I had a few questions concerning the oral presentation.

Is there a rule/general consensus regarding taking on a persona to make your speech? I thought taking on a different identity and setting to do your oral was perfectly acceptable and potentially beneficial, however a teacher was advising students against it. Is there any known preference of VCAA examiners?? Or at least, is there anything that suggests that they don't want you to take on a persona?

Also, is there a preferred way to incorporate your rebuttal? We've been given a general structure of intro, 2 argument paragraphs, rebuttal paragraph, 3rd argument paragraph, conclusion, but I've also been advised by teachers that you can incorporate your rebuttal into one of your argument paragraphs? Would this be ok or is it better to just go with the separated structure.
(I know that these are somewhat subjective questions but if anyone has been strongly advised against/for some of these techniques or knows if VCAA takes any particular stance it would be really helpful !!)

Thanks
Title: Re: VCE English Question Thread
Post by: vceme on June 29, 2018, 03:10:19 pm
Hi, I had a few questions concerning the oral presentation.

Is there a rule/general consensus regarding taking on a persona to make your speech? I thought taking on a different identity and setting to do your oral was perfectly acceptable and potentially beneficial, however a teacher was advising students against it. Is there any known preference of VCAA examiners?? Or at least, is there anything that suggests that they don't want you to take on a persona?

Also, is there a preferred way to incorporate your rebuttal? We've been given a general structure of intro, 2 argument paragraphs, rebuttal paragraph, 3rd argument paragraph, conclusion, but I've also been advised by teachers that you can incorporate your rebuttal into one of your argument paragraphs? Would this be ok or is it better to just go with the separated structure.
(I know that these are somewhat subjective questions but if anyone has been strongly advised against/for some of these techniques or knows if VCAA takes any particular stance it would be really helpful !!)

Thanks

Hey,
I think your teacher advises against it because its more harder to do. The oral presentation is only done as a SAC and not on the exam so you should be doing in preference of your teachers not VCAA examiners. So I would recommend to follow your teachers advice and such.
My oral structure was

Hook + Intro
P1: Argument 1
P2: Argument 2
P3: rebuttal
Conclusion
Title: Re: VCE English Question Thread
Post by: smamsmo22 on June 29, 2018, 04:52:30 pm
Hey,
I think your teacher advises against it because its more harder to do. The oral presentation is only done as a SAC and not on the exam so you should be doing in preference of your teachers not VCAA examiners. So I would recommend to follow your teachers advice and such.
My oral structure was

Hook + Intro
P1: Argument 1
P2: Argument 2
P3: rebuttal
Conclusion

Thanks; the reason I was asking about VCAA was because we have external assessors conducting our orals. (: But thank you anyway !!
Title: Re: VCE English Question Thread
Post by: Google123 on June 30, 2018, 06:00:28 pm
Hi,
Could someone be able to check my oral presentation topic idea?
I'm doing it on crime prevention and I'm opposing the building of more prisons, but instead proposing that there needs to be reforms to the criminal justice system, to prevent crime.
My arguments were that
1. Paying for prisons and etc costs a lot for taxpayers
2. Building prisons in rural areas limits communication between prisoners and their families (this is not good for rehabilitation)
3. My solutions (like alternative punishment options)

Is my second argument ok?
Would really appreciate if someone could check this.
Thank you!
Title: Re: VCE English Question Thread
Post by: Google123 on June 30, 2018, 06:39:16 pm
The second argument seems really good- It shows that you have clearly thought about all the possible implications of your topic which is what teachers usually look for in my experience. You may also like to argue that since a prison environment does not promote rehabilitation, there are more repeat offenders, which impacts the audience directly as these people are in the public and may cause problems. This like your taxpayer's argument provides a direct link to the audience and how it would affect them. Also, have some arguments for why your solution is superior to the current system (which I am sure you already do). Hope that is clear :)

Good Luck!
Thank you very much for your reply!
Yes I'll have the arguments that you've suggested as well.
Thank you! :)
Title: Re: VCE English Question Thread
Post by: moq418 on July 01, 2018, 06:29:24 pm
I have a essay question relating to crucible:
“The workings of power at the core of these texts are at the expense of individuals”
Discuss this statement and how it is represented in your prescribed text and the two stimulus text provided.

I have started the essay up to my introduction but im really confused and dont get the stimulus texts given can you please help me understand what the two video (stimulus text is talking about relating to people, politics and power and there motivations.
text 1 the informant https://www.youtube.com/watch?v=-qYnOHU4_Fg
text 2 future fund : https://www.youtube.com/watch?v=Ud8ZisH2IYI&ab_channel=whisper0555

i dont understand i have watch the videos many times but i dont know how they relate to the crucible when sharing representation of people, politics, power and there  motivations. Also is there a smaple essay similar to this question i can look to get a idea thanks. please help me !
Thank you so much!!
Title: Re: VCE English Question Thread
Post by: vceme on July 01, 2018, 08:26:57 pm
Hi,
Could someone be able to check my oral presentation topic idea?
I'm doing it on crime prevention and I'm opposing the building of more prisons, but instead proposing that there needs to be reforms to the criminal justice system, to prevent crime.
My arguments were that
1. Paying for prisons and etc costs a lot for taxpayers
2. Building prisons in rural areas limits communication between prisoners and their families (this is not good for rehabilitation)
3. My solutions (like alternative punishment options)

Is my second argument ok?
Would really appreciate if someone could check this.
Thank you!

Yep, find statistics on prisons  built in rural areas.
Use heavy, emotive language! and link back to your topic
Make sure you have a rebuttal which your first one is kinda and use persuasive language.
Good luck!!!
Title: Re: VCE English Question Thread
Post by: Google123 on July 05, 2018, 04:58:44 pm
Hi guys,
I just wanted to know if it's ok not to have a powerpoint presentation when you're giving your oral?
I'm giving mine to Cabinet members of the Government and so I haven't prepared a powerpoint.

Do you get more marks if you have one?
But it probably won't suit my setting, will it?
Thank you!
Title: Re: VCE English Question Thread
Post by: Sine on July 05, 2018, 05:03:22 pm
Hi guys,
I just wanted to know if it's ok not to have a powerpoint presentation when you're giving your oral?
I'm giving mine to Cabinet members of the Government and so I haven't prepared a powerpoint.

Do you get more marks if you have one?
But it probably won't suit my setting, will it?
Thank you!

depends on the school and the criteria that your teacher is assessing you on. You could just ask them if you would lose marks for it.

If you don't have a power point I would try to incorporate at least 1 sort of creative aspect in your speech.
Title: Re: VCE English Question Thread
Post by: OZLexico on July 14, 2018, 01:44:07 pm
Hi moq418, I hope these comments aren't too late to be helpful.. I've looked at your two videos that relate to "The Crucible".  In the first one (from "Billions") the main character is clearly putting on an act to frighten his employees.  His anger is shown in his loud, aggressive and commanding manner where he picks on an individual/makes an example of them to test their truth-telling. The two brief shots of the surveillance officer indicates there's another layer to this.  The boss has to look as if he's doing something to uncover 'the informant.' This open plan workplace also suggests a sort of egalitarian environment where there's mutual trust. This has clearly been betrayed by an unknown 'informant'.  The boss' strategy is to intimidate and frighten the staff into losing trust in each other as well as making them hope they wont be the next victim of the boss' anger.  How does this apply to "The Crucible"? You could look at the structure of authority and how it maintains respect and obedience from others (through fear and intimidation, making an example of individuals, like in "Billions" and/or through the appearance of doing something?).  The example from "The Hollow Men" suggests that decision making is subject to the interpretation (or spin) that officials are able to use - from manipulating money to using patriotic images in advertisements and the choice of paper stock used in a "Report." This ability to spin a proposal masks the true relationship between a governing body and the people. The scope of the Endowment Fund reflects the expected responsibilities of government but its dressed up as "a new initiative". In the course of this, the self-interest of the bureaucrats is also exposed ("why can't we have a very fast train?"). In relation to "The Crucible", think again about the structures of authority where leaders can be swayed by self-interested individuals and well motivated ones are forced to adapt to the judgement of their superiors.  This then has the effect of protecting the status quo.
Title: Re: VCE English Question Thread
Post by: pha0015 on July 17, 2018, 09:36:42 am
is it ok to have only one overarching argument for my oral presentation and then rebut the opposition's arguments? i.e. against abortion - fetus is human being therefore murder... then why normal cases as well as hard cases aren't exempt.

Do I also need to talk about the context of my topic? i.e. lack of unanimity of abortion laws in Australia?
Title: Re: VCE English Question Thread
Post by: a212345 on July 17, 2018, 10:19:31 am
Hi !
Just asking for some feedback on my Oral Presentation topic.
Its on the recent ban on selling Puppies and Kittens in Pet Stores in Victoria, due to Puppy farming and the ever growing amount of 
animals in pounds etc.

I just need some help with a over all contention, like what should I title my oral presentation?
" Should pet stores have banned the sale of Puppies and Kittens", " Adopt don't shop" etc. I don't know something like that
(Im all for banning the sale of Puppies and Kittens in pet stores and illegal unregistered breading and am against puppy farming as it is so cruel)

Also can i just talk about how I agree with the ban or do I have to look at both sides?
Any help would be greatly appreciated !

Thankyou
 
Title: Re: VCE English Question Thread
Post by: a212345 on July 17, 2018, 07:08:00 pm
It depends on what you are arguing. Are you arguing that selling pets in shops should be banned nationally or that everyone should adopt? You can incorporate both into your speech but it is a good idea to have one solid contention to work with.  You do have to look at both sides but you still argue for what your contention is and then rebut an opposing argument.

For example:
Contention: Pet shops should be banned from selling animals.
Introduction
Arguments: Cruelty issues, promotes puppy farming ect..
Rebuttal: You could incorporate the "adopt don't shop" notion here as a solution to an opposing argument that without pet stores people can't choose what pet they want and may then go down the illegal unregistered route, by explaining how adoption centers have a wide range of animals that need to be looked after.
Conclusion

Hope this clears a few things up for you. The key is to work out your contention then use all your other ideas to support that contention.

Thankyou so much for your help !!
 
Title: Re: VCE English Question Thread
Post by: OZLexico on July 19, 2018, 02:40:59 pm
Hi pha0015, I agree with Lilac - you need more than one argument and you definitely should mention the context of the issue.  As Lilac says, you need it in your introduction but you also need it in your Statement of Intention. You possibly need to do a little more research and look for current media discussion of the issue, see whether there are any recent conferences on women's health or any recent parliamentary focus on abortion. 
Title: Re: VCE English Question Thread
Post by: ddin on July 22, 2018, 10:13:21 pm
Hi. I don't know if this is the right place to ask but I have a question about predictions for this years exams:
Since this is just the second year with the comparisons of two texts section, do people think that the prompts will be quite simple like last years? Or are they likely to be significantly more difficult?
Thanks.
Title: Re: VCE English Question Thread
Post by: S200 on July 22, 2018, 10:18:27 pm
Well we had a TSSM presentation at our school a couple of weeks ago and they didn't say anything about increased difficulty or anything...

Obviously they aren't VCAA, but they are kinda authorities on the VCE scene...
Title: Re: VCE English Question Thread
Post by: Umattty on July 29, 2018, 06:48:19 pm
Does anyone have any tips on oral presentations?
i am an introvert who hates speaking in front of an audience :(!
i know this may sound stupid, but would it be easier for me to make eye contact if i didnt wear glasses, so that everyone looks blurry to me. haha  ;D ;D
Title: Re: VCE English Question Thread
Post by: PhoenixxFire on July 29, 2018, 07:16:39 pm
Does anyone have any tips on oral presentations?
i am an introvert who hates speaking in front of an audience :(!
i know this may sound stupid, but would it be easier for me to make eye contact if i didnt wear glasses, so that everyone looks blurry to me. haha  ;D ;D
I didn’t make eye contact with anyone during my oral speech. It’s only the teachers who are marking you and they can’t tell anyway - so long as you look at peoples faces you don’t need to actually make eye contact.

If you think it’ll be easier if you don’t wear glasses then go for it, make sure you practice your speech without them on though and make sure you can read your cue cards without them and still see well enough to look at people. Preferably do it with them on though, if you can.

General tips:
Practice your speech, not just saying it but practice looking up and around as if you’re looking at people, making hand gestures where appropriate, changing the volume/tone of your voice etc. The more you practice it, the less likely you are to screw up in the moment.
Title: Re: VCE English Question Thread
Post by: smamsmo22 on August 02, 2018, 10:07:49 pm
Hi, I feel like I'm asking more for consolation than advice, but nonetheless;
How have past year 12s found their ranking from SACs vs their study score (slash post exam rank I guess) compared/ how much opportunity is there for improvement? After being a top ranked English student for most of my schooling, I've ended up this year with a .. mediocre.. teacher (not blaming them completely, but I think it's made it difficult) and quite a few marks that I'm not happy with at all. Not bad marks, and I'd assume my rank isn't terrible, but just nowhere near what I thought I was capable of at the start of the year. Anyone got any advice/ happy stories about turning their English ranking around for the exam and getting a good study score?? I've tried to put as much effort as possible into English this year but I'm feeling quite discouraged atm and I really want to make the most out of the exam.
Thanks!
Title: Re: VCE English Question Thread
Post by: pha0015 on August 08, 2018, 02:34:34 pm
Has anyone watched the movie Into the Wild?

There's often text on the screen and I'm not sure if there's a literary term for it or not.

Also, what are some key cinematography techniques in the film and what do they represent? I often have a hard time recognising camera shots, angles even though they seem to be the most obvious ones. I know nothing about mise en scene except for when McCandless is dying. Then there's the music and background noise which I should be ok with. Even when I recognise techniques, I can't quite explain what they're supposed to convey and had to leech off the google for ideas i.e. aerial shot of natural landscape in which McCandless is but a speck illustrates how big of an adversary mother nature is (if I'm trying hard) or a close up shot of someone's face to emphasise emotion (this isn't enough, is it?).

On a similar note, how would one analyse the structure of Medea, since it's a play? I'm referring to things like pacing, positioning of characters, staging (deux ex machina)...

Thanks in advance
Title: Re: VCE English Question Thread
Post by: Maya24 on August 24, 2018, 06:42:50 pm
Are you allowed to refer to the background knowledge of a book when writing a text response? For example using the story about the beginning of the war in Ransom.
Title: Re: VCE English Question Thread
Post by: Poet on August 24, 2018, 09:40:54 pm
Are you allowed to refer to the background knowledge of a book when writing a text response? For example using the story about the beginning of the war in Ransom.
of course, you just have to ensure that it is relevant - don’t just spout the info because you know it - as well as being succinct. Don’t tell the history, because your arguments are meant to be based on the actual text, but you can use outside information as a stepping stone. :)
Title: Re: VCE English Question Thread
Post by: S200 on August 25, 2018, 12:49:26 am
Are you allowed to refer to the background knowledge of a book when writing a text response? For example using the story about the beginning of the war in Ransom.
Yeah, sure!
Giving historical context to the text is actually encouraged!
For instance, in my intro for the Ransom/Invictus matchup I am saying about how Malouf explores the themes central to his source, The Iliad (i.e Achilles wrath and honour). Although you technically shouldn't reference characters within the introduction, my teacher commended me on it because it is furthering the understanding of the historical context of the text... ;)
Title: Re: VCE English Question Thread
Post by: Google123 on August 26, 2018, 12:25:29 pm
Hi all,
I'm sorry if this has already been asked, but how should you structure your Argument Analysis essay with comparing a lot?
I've been writing essays and my teacher tells me I've got too much comparison which isn't really necessary for Section C.

How much should you compare within paragraphs? Could someone please suggest a good structure?

Thank you! :)
Title: Re: VCE English Question Thread
Post by: sdfg on August 26, 2018, 05:03:45 pm
Hi all,
I'm sorry if this has already been asked, but how should you structure your Argument Analysis essay with comparing a lot?
I've been writing essays and my teacher tells me I've got too much comparison which isn't really necessary for Section C.

How much should you compare within paragraphs? Could someone please suggest a good structure?

Thank you! :)

Comparing is only required for the argument analysis SAC; for the exam, you can choose whether or not to compare but it's generally advised to analyse the material independently (i.e. no comparison) because it's just easier that way. Personally, only time when I would compare for the exam would be if there's a really significant or interesting point of comparison.

If you want a comparative structure, though, here's what I was taught:     
1) Introduction: provide background, and introduce details (title, author, publication and pub. date) and contentions of all material.
2) For however many paragraphs necessary (rule of thumb: let the spread of your analysis reflect the spread of material), analyse article 1. This usually should be the longest article but if they're all the same length, make a judgement to which one should logically go first. No reference to any other article at this point. 
3) Analyse article 2 and intersperse your analysis with comparisons to article 1. Again, analyse for how many paragraphs necessary and do remember the time constraints.
4) Analyse article 3, with comparisons to 1 and 2.
5) Repeat 4) with any other material, if there's any and as above - analyse it if you were going to do it independently but then make comparisons to the other material you have talked about inbetween.
5) If time provides, a conclusion that ties everything back together. The statement you're making here should be about the differences/similarities between how the different authors argue their argument.
Title: Re: VCE English Question Thread
Post by: I\'m Not A Robot on August 31, 2018, 08:39:26 pm
Hey, so i've got my last SAC for English in about a weeks time (comparative between The Longest Memory and Black Diggers). My teachers tell me that it's much easier to do the quote comparison, rather than the actual prompt/statement given on the day. Only problem is, i've never really even done a comparison of quotes. SO i wanted to do some practice ones before i got started and my teacher gave me a list of practice 'prompts' to try.
One of which was:

"You knew your place old man. I liked you for that."- Sanders JNR (TLM)
And
"Thank god for the uniform and the chance to serve."- Archie (BD)

Could someone please help me create some points for a quote comparison like this. I'm not sure what exactly i'm meant to talk about.
I was thinking of using these very simple points for each paragraph in my essay:
- The slaves and the Indigenous seem to be blindly loyal to those that oppress them in their respective societies.
- Both authors question the value of loyalty in their texts as it generally does not always make the characters better off (still treated unequally, despite sacrifices).
- There are certain characters within both texts that treat the slaves and the Indigenous better than other characters.

These are probably pretty weak points, so i would really appreciate it if some stronger points were suggested as well.

Thanks heaps in advance :)
Title: Re: VCE English Question Thread
Post by: Lear on September 18, 2018, 07:07:40 pm
Hi,

I just wanted to get some insight from you all on how you believe/your teacher believes paragraphs should be done in text response and comparative essays.
For example one is where you do paragraphs based on ‘ideas’ which support the prompt while another is where you have a logical flow making one big argument answering the prompt.

Any insights are helpful.
Title: Re: VCE English Question Thread
Post by: vceme on September 18, 2018, 07:12:09 pm
Hi,

I just wanted to get some insight from you all on how you believe/your teacher believes paragraphs should be done in text response and comparative essays.
For example one is where you do paragraphs based on ‘ideas’ which support the prompt while another is where you have a logical flow making one big argument answering the prompt.

Any insights are helpful.

I haven't really heard of your examples to be honest, besides the 'integrated, 'block' approach stuff like that but here's my two cents. I think I would personally do my paragraphs based on my 'ideas' rather than having one big argument. Having different Ideas I guess makes your ideas seem more complex, and that you have a thorough and brilliant understanding of the text. Again, just a year 12 student tryna do their best so lol  ;D
Title: Re: VCE English Question Thread
Post by: sdfg on September 18, 2018, 07:38:53 pm
Hi,

I just wanted to get some insight from you all on how you believe/your teacher believes paragraphs should be done in text response and comparative essays.
For example one is where you do paragraphs based on ‘ideas’ which support the prompt while another is where you have a logical flow making one big argument answering the prompt.

Any insights are helpful.

Imo, it should be a combination of both of the approaches that you've mentioned as that way you're consistently responding to the prompt and arguing your interpretation. Have one big point that addresses the prompt and then smaller supporting points that support that big point. And if you do it correctly, you'll have in essence of one big argument answering the prompt (because all you're doing with the supporting points is arguing your big point) and paragraphs that are although on different ideas, will link together and respond to the prompt since all of them support that same main, big point.

Dunno if that made sense.  :-\
Title: Re: VCE English Question Thread
Post by: Lear on September 18, 2018, 07:46:13 pm
Imo, it should be a combination of both of the approaches that you've mentioned as that way you're consistently responding to the prompt and arguing your interpretation. Have one big point that addresses the prompt and then smaller supporting points that support that big point. And if you do it correctly, you'll have in essence of one big argument answering the prompt (because all you're doing with the supporting points is arguing your big point) and paragraphs that are although on different ideas, will link together and respond to the prompt because all of them support that same main, big point.

Dunno if that made sense.  :-\

That indeed makes a lot of sense. Clarke's essay on her Tips post exemplifies exactly what you've said and I think this is the best approach. Unfortunately it has been drilled into students in my schools to strictly base paragraphs on 'ideas' such as 'Euripides criticises excess love'. This basically creates a disjointed essay with 3 different 'arguments' for the prompt rather than a combined answer.
Title: Re: VCE English Question Thread
Post by: sdfg on September 18, 2018, 08:01:41 pm
That indeed makes a lot of sense. Clarke's essay on her Tips post exemplifies exactly what you've said and I think this is the best approach. Unfortunately it has been drilled into students in my schools to strictly base paragraphs on 'ideas' such as 'Euripides criticises excess love'. This basically creates a disjointed essay with 3 different 'arguments' for the prompt rather than a combined answer.

You still can base you body paragraphs on ideas (and ideally should) and have them all link together to form one cohesive essay. From your example, I think you're coming into trouble here because the ideas for your topic sentences are too broad. 'Euripides criticises excess love' is indeed a valid argument but it's Euripides' overarching argument and you should instead be basing your body paragraphs on the finer details.
Title: Re: VCE English Question Thread
Post by: Lear on September 18, 2018, 08:16:50 pm
Also, what's an ideal number of words for each essay during the exam?
I was thinking ~900
Title: Re: VCE English Question Thread
Post by: sdfg on September 18, 2018, 08:21:26 pm
Also, what's an ideal number of words for each essay during the exam?
I was thinking ~900

Obviously quality > quantity above all else, but ideally upwards of 1000 if you want to secure your chances at those 8 - 10 (i.e. 40+ SS). 900 would be fine if you're succinct and able to demonstrate detailed understanding of your texts/the analysis material in those 900 words, but 1000+ just to be on the safe side.
Title: Re: VCE English Question Thread
Post by: Google123 on September 18, 2018, 08:47:21 pm
Hi,

I just wanted to get some insight from you all on how you believe/your teacher believes paragraphs should be done in text response and comparative essays.
For example one is where you do paragraphs based on ‘ideas’ which support the prompt while another is where you have a logical flow making one big argument answering the prompt.

Any insights are helpful.
Hi Lear,
This is a really unique question and so could you help me out a little bit please?
But could you give an example of the second point of your question: "where you have a logical flow making one big argument answering the prompt."
An example will help clarify things so much for dummies like me!!  ;D
So for e.g last year's prompt: Is Medea an innocent victim?

Could you just tell me how you'd answer that with one big argument?
How would this be different to the 'ideas' para?

I'm sorry for asking too many questions, but I really need help!!!  :-\
Title: Re: VCE English Question Thread
Post by: DBA-144 on September 18, 2018, 09:44:35 pm
Obviously quality > quantity above all else, but ideally 1000+ if you want to secure your chances at those 8 - 10 (i.e. 40+ SS). 900 would be fine if you're succinct and able to demonstrate detailed understanding of your texts/the analysis material in those 900 words, but 1000+ just to be on the safe side.


Hey everyone,
tbh, 1000 words is probably too much to be writing for each  essay, given that you have 3 hours to write all of this. I'd suggest thay you try to keep it between 900-1000. This way, you should be able to write both a quantity that is competitive enough for a high study score, and that you don't veer too far away from the prompt. Remember that the prompt is your main priority. However, each to their own; some people can write really fast and really well in a limited amount of time. Just remember to keep it legible  :P

Re: Google123's question
The approach of 'one big argument' is not exactly encouraged, from my experience, seeing as it may fail to properly adress the key nuances the prompt, or the text, has. However, one should seek to form an overall contention, and then support their contention throughout the essay.

This is also why the separate argument structure is not the best; it struggles to make connections between the various aspects of the prompt.

Therefore, it looks like the better approach is the one Lear and sdfg are suggesting- to support an overarching contention with supporting sub arguments.

I would like to emphasise that this is my understanding only, and that it may be incorrect. Please feel free to correct me!
Title: Re: VCE English Question Thread
Post by: Sine on September 19, 2018, 12:23:20 pm
Also, what's an ideal number of words for each essay during the exam?
I was thinking ~900
no real ideal number imo but anywhere between 900+ is fine. Some people go beyond this I know it's nearly impossible but some have gone to around 1500 words words in an hour but that can happen when someone already has memorised certain paragraphs/essays.

A 1200 word essay of X quality will generally score better than a 900 word essay of X (same) quality.
Title: Re: VCE English Question Thread
Post by: smamsmo22 on September 24, 2018, 02:11:07 pm
Hi,
I was just confirming that for the end of year exam, we aren't expected to compare separate texts in the language analysis section? And incorporating comparisons or choosing not to will not have any impact on marks?

Thanks
Title: Re: VCE English Question Thread
Post by: clarke54321 on September 24, 2018, 03:40:47 pm
Hi,
I was just confirming that for the end of year exam, we aren't expected to compare separate texts in the language analysis section? And incorporating comparisons or choosing not to will not have any impact on marks?

Thanks

I can confirm that comparison is not required in the argument analysis. There is no explicit reference to this skill on VCAA’s marking scheme for section C. This isn’t to say, however, that you can’t use comparative ‘language’ in your introduction or body paragraphs. There might be times where it feels natural to bring the two authors/speakers together. It’s just important to know that this is by no means a necessity :)
Title: Re: VCE English Question Thread
Post by: smamsmo22 on September 30, 2018, 10:52:58 am
Hi, another question about language analysis.
I often find myself trying to analyse too many parts of a text, leading to my essay becoming somewhat convoluted and overall of a worse quality as I end up having to rush to finish. Does anyone have any tips on selecting parts to analyse (i.e., what to look for/prioritise in terms of particular techniques to enhance sophistication perhaps) and also any guidelines on (roughly) how many points/quotes/techniques should you be analysing per paragraph? I know some people prefer to write a large number of shorter paragraphs but I tend to write around 3 of a reasonable length, if anyone knows a good number of quotes/techniques to be implementing per paragraph just as a guide, that would be really helpful. I'm struggling with maintaining clarity in my pieces.
Thanks!
Title: Re: VCE English Question Thread
Post by: Seno72 on October 04, 2018, 07:12:20 am
Hi, another question about language analysis.
I often find myself trying to analyse too many parts of a text, leading to my essay becoming somewhat convoluted and overall of a worse quality as I end up having to rush to finish. Does anyone have any tips on selecting parts to analyse (i.e., what to look for/prioritise in terms of particular techniques to enhance sophistication perhaps) and also any guidelines on (roughly) how many points/quotes/techniques should you be analysing per paragraph? I know some people prefer to write a large number of shorter paragraphs but I tend to write around 3 of a reasonable length, if anyone knows a good number of quotes/techniques to be implementing per paragraph just as a guide, that would be really helpful. I'm struggling with maintaining clarity in my pieces.
Thanks!

Someone please answer this as well. Also considering that last year, the Lang Analysis exam only consisted of a large article, a small comment and a cartoon and we may get something similar this year, how are we meant to actually do the integrated method for 3-4 body paragraphs when you have hardly anything to actually analyse about the comment and the article? How our school just forced us to do integrated for lang analysis without telling us how to use in situations like this.

Cheers!
Title: Re: VCE English Question Thread
Post by: sophomania on October 04, 2018, 08:33:43 am
Hi, another question about language analysis.
I often find myself trying to analyse too many parts of a text, leading to my essay becoming somewhat convoluted and overall of a worse quality as I end up having to rush to finish. Does anyone have any tips on selecting parts to analyse (i.e., what to look for/prioritise in terms of particular techniques to enhance sophistication perhaps) and also any guidelines on (roughly) how many points/quotes/techniques should you be analysing per paragraph? I know some people prefer to write a large number of shorter paragraphs but I tend to write around 3 of a reasonable length, if anyone knows a good number of quotes/techniques to be implementing per paragraph just as a guide, that would be really helpful. I'm struggling with maintaining clarity in my pieces.
Thanks!
Someone please answer this as well. Also considering that last year, the Lang Analysis exam only consisted of a large article, a small comment and a cartoon and we may get something similar this year, how are we meant to actually do the integrated method for 3-4 body paragraphs when you have hardly anything to actually analyse about the comment and the article? How our school just forced us to do integrated for lang analysis without telling us how to use in situations like this.

Cheers!

Hello! It seems like some people are struggling with  argument analysis essays. So I'm just going to say what worked for me - it might not work for everyone and some people may disagree but you could give it a shot.

In terms of sounding too convoluted, it would help to have a clear structure in mind. In my opinion, it would be best to do structure your paragraphs by argument, as this is the easiest way to show how argument is being developed. Once you have identified the argument in the article, and you want to go analyse it for techniques and stuff, it would be best to select main techniques that actually contribute to the argument. These are usually techniques that are focused on bigger idea stuff, and are not so focused on language. So for example, which would be better to analyse - that rhetorical question or that appeal to a sense of community? Probably the appeal right? Also note that techniques such as appeals are usually supported by a range of other smaller techniques (which are more focused on language) - for example an appeal to a sense of community is supported by inclusive language.
Sorry if this doesn't make sense, but here's an example of generally how I would structure my paragraph:
- Topic sentence: Argument
- I would recommend analyse about two main techniques that strengthen the argument (e.g appeal to fear)
- further analyse these main techniques through explanation and also show how this main idea (i.e appeal to fear) is sustained through smaller supporting techniques (e.g statistics and negative connotations)

In terms of doing an integrated approach, I would group the texts by argument. I'm not too sure how to explain it so I'll give an example:
Say Text A has three main arguments: the health benefits, economic benefits, social benefits
Text B is a comment that talks about the economic disadvantages
Text C is a satirical cartoon about the social benefits
In this case, I would structure my essay like this:
Paragraph 1: Text A - talking about health benefits
Paragraph 2: Text A and Text B - economic
Paragraph 3: Text A and Text C - social

Hope this helps!
Title: Re: VCE English Question Thread
Post by: Seno72 on October 05, 2018, 11:06:46 am
Hello! It seems like some people are struggling with  argument analysis essays. So I'm just going to say what worked for me - it might not work for everyone and some people may disagree but you could give it a shot.

In terms of sounding too convoluted, it would help to have a clear structure in mind. In my opinion, it would be best to do structure your paragraphs by argument, as this is the easiest way to show how argument is being developed. Once you have identified the argument in the article, and you want to go analyse it for techniques and stuff, it would be best to select main techniques that actually contribute to the argument. These are usually techniques that are focused on bigger idea stuff, and are not so focused on language. So for example, which would be better to analyse - that rhetorical question or that appeal to a sense of community? Probably the appeal right? Also note that techniques such as appeals are usually supported by a range of other smaller techniques (which are more focused on language) - for example an appeal to a sense of community is supported by inclusive language.
Sorry if this doesn't make sense, but here's an example of generally how I would structure my paragraph:
- Topic sentence: Argument
- I would recommend analyse about two main techniques that strengthen the argument (e.g appeal to fear)
- further analyse these main techniques through explanation and also show how this main idea (i.e appeal to fear) is sustained through smaller supporting techniques (e.g statistics and negative connotations)

In terms of doing an integrated approach, I would group the texts by argument. I'm not too sure how to explain it so I'll give an example:
Say Text A has three main arguments: the health benefits, economic benefits, social benefits
Text B is a comment that talks about the economic disadvantages
Text C is a satirical cartoon about the social benefits
In this case, I would structure my essay like this:
Paragraph 1: Text A - talking about health benefits
Paragraph 2: Text A and Text B - economic
Paragraph 3: Text A and Text C - social

Hope this helps!


Thanks! I know everyone has their own approach, but when you have a cartoon, a large article and a small comment, which one would you reccomend? I would do block but that would mean doing paragraphs in under 70 mins, which is crazy (thats what my tutor reccomends). What my school reccomends is us do integrated method without telling us what to have as body paragraphs and they expect us to write 4 body paragraphs. Crazy.
Title: Re: VCE English Question Thread
Post by: userrrname on October 13, 2018, 01:22:30 pm
Hey... i just have a question. Since we're allowed to take our own dictionary into the exam, I was wondering if I'm allowed to fold the flaps of a few pages to help me remember where words are or some words. Would this be violating anything? Would it be considered cheating?
Title: Re: VCE English Question Thread
Post by: sophomania on October 13, 2018, 09:30:27 pm
Hey... i just have a question. Since we're allowed to take our own dictionary into the exam, I was wondering if I'm allowed to fold the flaps of a few pages to help me remember where words are or some words. Would this be violating anything? Would it be considered cheating?

No I wouldn't do that! Since you're not allowed to mark your dictionary, it might be considered cheating :/
Title: Re: VCE English Question Thread
Post by: dsabeta on October 13, 2018, 10:41:50 pm
Not sure if anyone would be able to help me out on this, but I'm a bit concerned about the translation of my text for the exam.
My school is doing Medea, and on the booklist (very stupidly) prescribed the wrong translation (Vellacott) for students to buy - I, however, had purchased the book myself from a bookshop, which is the Davie translation. Once school had started, my teacher realised that my version was actually the correct version that was specifically prescribed on the VCAA text list.
This initially wasn't seen as a very big deal, until we had an experienced external assessor come in to give us feedback on a trial exam she had marked for us. My friend (the only other person who has the correct translation) asked about this, and the assessor said that the markers typically go off the main version and tend to stick to that one. She also mentioned that the chances of all assessors having knowledge of the other translation is highly unlikely.
I've been working off of the Vellacott translation for my SACs and so forth, but as the exam approaches I'm unsure whether to base my quotes off of my copy (the correct translation) or not? I'm a bit worried that the examiners will be unaware of the quotes from the other translation, and will think that I'm pulling random quotes from thin air.. is this crazy to think?
I know this might seem like a silly question which has a self-explanatory answer, but any advice would be really helpful rn :(
Title: Re: VCE English Question Thread
Post by: PhoenixxFire on October 13, 2018, 11:31:23 pm
I’m unsure of how strict VCAA is on exact quotes when there are different translations (might be someone else on here who knows though) but definitely try and use quotes from the VCAA approved translation from now on.

Really sucks that your school messed that up!
Title: Re: VCE English Question Thread
Post by: sdfg on October 13, 2018, 11:52:27 pm
Not sure if anyone would be able to help me out on this, but I'm a bit concerned about the translation of my text for the exam.
My school is doing Medea, and on the booklist (very stupidly) prescribed the wrong translation (Vellacott) for students to buy - I, however, had purchased the book myself from a bookshop, which is the Davie translation. Once school had started, my teacher realised that my version was actually the correct version that was specifically prescribed on the VCAA text list.
This initially wasn't seen as a very big deal, until we had an experienced external assessor come in to give us feedback on a trial exam she had marked for us. My friend (the only other person who has the correct translation) asked about this, and the assessor said that the markers typically go off the main version and tend to stick to that one. She also mentioned that the chances of all assessors having knowledge of the other translation is highly unlikely.
I've been working off of the Vellacott translation for my SACs and so forth, but as the exam approaches I'm unsure whether to base my quotes off of my copy (the correct translation) or not? I'm a bit worried that the examiners will be unaware of the quotes from the other translation, and will think that I'm pulling random quotes from thin air.. is this crazy to think?
I know this might seem like a silly question which has a self-explanatory answer, but any advice would be really helpful rn :(

Either translation is fine. Vellacott was the original translation and they changed it to Davies afterwards, but some schools weren't happy because they've been teaching Vellacott all along so VCAA now accepts both translations. This is the reason why you won't find VCAA prompts for Medea which includes a quote from the play - cause it would unfairly disadvantage students who didn't study the translation the quote is from - unlike prompts for other texts which might.

So yeah, don't worry - you're fine. :)

Edit: From VCAA.
Title: Re: VCE English Question Thread
Post by: smamsmo22 on October 24, 2018, 08:30:14 am
Hi, just a question/advice request about language analysis.

I often receive feedback that I don't include enough analysis of 'specific language' and its (usually emotional) impact on the reader I was just trying to gain a better understanding of this/ how to fix this. Does this mean referring more to the impact of specific words and devices? Sorry this is so unclear but if anyone can direct me to tips/info about being more specific with language and literary devices that would be helpful I think  :-\
Title: VCE English Question Thread
Post by: Lear on October 24, 2018, 08:34:36 am
https://atarnotes.com/forum/index.php?topic=164759


This helped me immensely and I think it is exactly what you are looking for!
Title: Re: VCE English Question Thread
Post by: smamsmo22 on October 24, 2018, 08:56:07 am
https://atarnotes.com/forum/index.php?topic=164759


This helped me immensely and I think it is exactly what you are looking for!

Thank you! I've read this before but I'll definitely read again; it's definitely helpful (:

This probably sounds contradictory to my first but my analyses often lack discussion of the more 'traditional' literary devices; mine are usually centred around language and phrases. I know listing literary devices isn't the point of a language analysis but I feel like i should do it more. Does anyone have any tips about being specific with the impact of literary devices on the readers? Like.. I can identify rhetorical questions etc, but I feel that I sound too vague when describing their impact (i.e. heightens the concern of the reader by asking them to acknowledge their own habits..) idk. Hope that makes some sort of sense!!
Title: Re: VCE English Question Thread
Post by: lst1103 on October 24, 2018, 12:26:22 pm
Does anyone have any strategies to recognise arguments within articles for Section C? I struggle with this for some reason and it takes me far too long to detect them.
Title: Re: VCE English Question Thread
Post by: Lear on October 24, 2018, 12:47:47 pm
What helps me is reading it like a normal person, not a year 12 frantically trying to think about what their paragraphs should be based upon. I identify what this person is contending (what should happen, what you should believe, what you should do etc) and then think why?
For example, say the piece was on the need for more space exploration.
This person thinks that we should pursue space exploration because....... A1
Another reason is that ..... A2
Perhaps the biggest reason is that .... A3
Title: Re: VCE English Question Thread
Post by: Deliaaa454 on October 26, 2018, 07:31:30 pm
Hi, can anyone give me a rough estimate of what I need to score on the final exam for a 35+ study score?
I'm currently ranked just above average in my cohort (ranked about ~150 in the state) and has a B average for unit 3 and B+ for unit 4.
Thanks in advance.
Title: Re: VCE English Question Thread
Post by: marauder52 on October 29, 2018, 04:43:04 pm
How does one deal with the uncertainty of what prompts will be given on the exam?
I would say I am a decent english student but this is starting to get to me. I feel as though no matter how many quotes I memorise the prompts on the exam could absolutely screw me over. Especially for a play like Medea. There's just so much to know. How can one ensure all bases are covered?
Title: Re: VCE English Question Thread
Post by: Fergus6748 on October 29, 2018, 04:50:14 pm
How does one deal with the uncertainty of what prompts will be given on the exam?
I would say I am a decent english student but this is starting to get to me. I feel as though no matter how many quotes I memorise the prompts on the exam could absolutely screw me over. Especially for a play like Medea. There's just so much to know. How can one ensure all bases are covered?
Heya, I dont do VCE, but I can give it a crack. The best thing to do would be to go over you text to find the 3-4 strongest themes within it. Then sift througha bunch of past questions and write essay plans using the concepts. Since they are just concepts you should be able to adapt them to the question. Hope this helps!
Title: Re: VCE English Question Thread
Post by: ailenewu on October 29, 2018, 04:51:31 pm
a 'key player' = something the author positions in a certain way to strengthen their contention
then, discuss the various language features that support that sub-argument.

So for the sample piece on the facebook page, I had:
   - Paragraph 1: the importance of biodiversity and why it is vulnerable and in need of preservation (+ 1st visual, since it deals with this same sentiment)
   - Paragraph 2: the dangers of inaction and the damaging repercussions this can have
   - Paragraph 3: the need for honest reappraisal of the situation and for a genuine commitment to tangible goals (+ 2nd visual, since it can be linked to this idea too)

https://www.nytimes.com/2018/02/04/opinion/metoo-law-legal-system.html
http://theconversation.com/beyond-metoo-we-need-bystander-action-to-prevent-sexual-violence-91741

i just read some posts on using 'key players' as a way to structure a language analysis response for the english exam (section c) - ahhhh why did i not see this earlier?! i just wanted to do a little practice to make sure i have the whole concept down pact before wed. the articles above are the ones we had to do for a sac in school, and i've tried to identify the key players and how i would structure the arguments below:

1. survivors of sexual harassment
2. men in general (??)
3. women in general (??)
or i would group number 2 and 3 and make it the public of Australia/USA in general, but my teacher has told me to never mention 'the general public' as it is too vague...
3. the government bodies that can change laws

would these be the main players you would focus on? i'd love to know how you would approach these articles!
Title: Re: VCE English Question Thread
Post by: clarke54321 on October 29, 2018, 08:06:13 pm
How does one deal with the uncertainty of what prompts will be given on the exam?
I would say I am a decent english student but this is starting to get to me. I feel as though no matter how many quotes I memorise the prompts on the exam could absolutely screw me over. Especially for a play like Medea. There's just so much to know. How can one ensure all bases are covered?

The uncertainty you are experiencing is a very natural feeling! I felt exactly like this at this time last year.

Given that you feel quite confident in your writing abilities, and understanding of quotes, I'd encourage you to systematically work your way through a list of Medea prompts . That is, try and pair up prompts that could effectively pen the same essay. After a while, you will realise that you have very distinctive categories. Even where you have outliers, think hard to determine how you can manipulate the prompt to make it work for one of your categories. This does not mean that you aren't answering the question. It just means you are defining the scope of the question, so that you are able to write an essay that suits one of your main interpretations. This is how I used my time leading up to the exam. Above all, it gives you the security that no matter what VCAA throws at you, you'll be able to respond well.

All the very best for the exam :)
Title: Re: VCE English Question Thread
Post by: Maya24 on October 30, 2018, 01:56:10 pm
What do you guys recommend for last-minute English prep?
Title: Re: VCE English Question Thread
Post by: S200 on October 30, 2018, 02:05:43 pm
Grab a bunch of prompts and tear-em apart...
Title: Re: VCE English Question Thread
Post by: Poet on October 30, 2018, 02:17:56 pm
What do you guys recommend for last-minute English prep?
Hey! Have you skimmed through this "10 things to do before your English exam" thread by Lauren? If not, I think it's worth a peek. And if so, read it through again. S200's right - plan through prompts - but make sure you don't burn yourself out. Rest your mind a little and make sure everything's ticked off your list. :)
Title: Re: VCE English Question Thread
Post by: Maya24 on October 30, 2018, 02:29:38 pm
If there is more than one article in the language analysis, would we need to compare and contrast?
Title: Re: VCE English Question Thread
Post by: derpson on October 30, 2018, 02:45:05 pm
If there is more than one article in the language analysis, would we need to compare and contrast?

Nope.
Title: Re: VCE English Question Thread
Post by: clarke54321 on October 30, 2018, 03:40:13 pm
If there is more than one article in the language analysis, would we need to compare and contrast?


There is no requirement to do this under the VCAA criteria. However, you may find it helpful to incorporate comparative language in the introduction or relevant topic sentences for the purposes of coherency.
Title: Re: VCE English Question Thread
Post by: Lear on October 30, 2018, 04:29:27 pm
Sort of a technical question but are we allowed to change the tense/form of phrases using square brackets.
Such as this quote here 'I know that an impoverished friend is shunned'
Could we change this to '[he] knows that an impoverished friend is shunned in order to have it better fit a sentence as 'Through Jason Euripides illustrates the prevailing view of exiles in society as '[he] knows that an impoverished friend is shunned.

Or would it be preferable to mould the sentence around the quote?


EDIT: -___- the BBcode thinks I want to strikethrough when I use 's' between square brackets.


Title: Re: VCE English Question Thread
Post by: Poet on October 30, 2018, 04:33:03 pm
Sort of a technical question but are we allowed to change the tense/form of phrases using square brackets.
Such as this quote here 'I know that an impoverished friend is shunned'
Could we change this to '[he] know that an impoverished friend is shunned' in order to have it better fit a sentence as 'Through Jason Euripides illustrates the prevailing view of exiles in society as '[he] know that an impoverished friend is shunned.'
Yep! Just as long as you clearly indicate what you've changed. :)
Title: Re: VCE English Question Thread
Post by: Maya24 on October 30, 2018, 04:50:25 pm
For language analysis,  In the conclusion is it recommended to analyses how the writer ends their article or should we just summarise everything?
Thanks
Title: Re: VCE English Question Thread
Post by: clarke54321 on October 30, 2018, 04:52:45 pm
For language analysis,  In the conclusion is it recommended to analyses how the writer ends their article or should we just summarise everything?
Thanks

Some people use the final analysis to end their piece (and therefore include this as part of their conclusion), however it is not necessary. You can just summarise contrasting tones/contentions, etc. It's essentially writer's choice :)
Title: Re: VCE English Question Thread
Post by: anne13007 on October 30, 2018, 06:33:05 pm
When referring to the title of texts in Section A, B and C is it recommended to underline or quote the title? For example, Rear Window or 'Rear Window'.
Title: Re: VCE English Question Thread
Post by: clarke54321 on October 30, 2018, 06:36:46 pm
When referring to the title of texts in Section A, B and C is it recommended to underline or quote the title? For example, Rear Window or 'Rear Window'.

Yes it is recommended to do so! It just helps the examiner easily determine when you are referring to the text as a whole.
Title: Re: VCE English Question Thread
Post by: Lear on October 30, 2018, 07:42:54 pm
Can any of our forum legends predict tomorrow’s Medea topic with the knowledge of the past years?

My bet is on Moderation or Love!
Title: Re: VCE English Question Thread
Post by: clarke54321 on October 30, 2018, 08:02:56 pm
Can any of our forum legends predict tomorrow’s Medea topic with the knowledge of the past years?

My bet is on Moderation or Love!

Love is a good bet! They've never explicitly targeted this topic, so I think it's a likely possibility. I also wouldn't be surprised if the theme of pride came up, and how this relates to notions of revenge. Maybe ideas pertaining to trust will also appear? It's very difficult to say. But I think it's always comforting to know that whatever the prompt, you can always map your way back to the two core ideas of justice/revenge and emotion.
Title: Re: VCE English Question Thread
Post by: Maya24 on October 30, 2018, 08:05:41 pm
Can any of our forum legends predict tomorrow’s Medea topic with the knowledge of the past years?

My bet is on Moderation or Love!
My bet is on feminity and the roles of womens
Title: Re: VCE English Question Thread
Post by: Lear on October 30, 2018, 08:12:08 pm

My bet is on feminity and the roles of womens

Gender roles is also something that hasn’t come up recently!
Title: Re: VCE English Question Thread
Post by: clarke54321 on October 30, 2018, 08:17:35 pm
Gender roles is also something that hasn’t come up recently!

I'd say that last year's prompt sought to get at the oppression of women in a patriarchal society. My understanding is that VCAA are reluctant to explicitly target notions of gender, given that they are conscious of the fact that 'Medea' is only a proto-feminist text.
Title: Re: VCE English Question Thread
Post by: emilywrench on October 31, 2018, 03:58:53 pm
Hello!

I forgot to shade what text I was writing about in Section B of the exam (I shaded which prompt I was answering, just not that I was writing about Ransom/Invictus). Is this bad? Will they still mark it?
Title: Re: VCE English Question Thread
Post by: S200 on October 31, 2018, 04:00:08 pm
They should be able to figure it out pretty easily.
Not all texts have Mandela and Priam as characters :)

And yes, they will still mark it...
Title: Re: VCE English Question Thread
Post by: jb45 on November 01, 2018, 10:40:16 am
They should be able to figure it out pretty easily.
Not all texts have Mandela and Priam as characters :)

And yes, they will still mark it...
what if you ticked the wrong prompt?
Title: Re: VCE English Question Thread
Post by: clarke54321 on November 01, 2018, 10:42:44 am
what if you ticked the wrong prompt?

This also isn't too big of a worry. From the language/content of your piece, the examiners will be able to gauge which prompt you chose :)
Title: Re: VCE English Question Thread
Post by: Lenny-123 on November 01, 2018, 02:49:31 pm
I'd say that last year's prompt sought to get at the oppression of women in a patriarchal society. My understanding is that VCAA are reluctant to explicitly target notions of gender, given that they are conscious of the fact that 'Medea' is only a proto-feminist text.

Bombshells and The Penelopiad (Margaret Atwood) both v feminist
Title: Re: VCE English Question Thread
Post by: jacquieg on December 24, 2018, 10:37:54 am
Can someone please upload a PDF copy of 'The Golden Age' by Joan London for me.... i'll love you forever! I haven't got it yet and i'm about to go on a trip so i'd like to read ahead rather than rush through it on the last week of hols.
Title: Re: VCE English Question Thread
Post by: PhoenixxFire on December 24, 2018, 10:53:50 am
Can someone please upload a PDF copy of 'The Golden Age' by Joan London for me.... i'll love you forever! I haven't got it yet and i'm about to go on a trip so i'd like to read ahead rather than rush through it on the last week of hols.
That book is copyrighted so it can't be distributed on AN. You can buy a pdf version online on websites such as booktopia.
Title: Re: VCE English Question Thread
Post by: TheIllusion on January 17, 2019, 11:10:06 am
Hello!
For unit 3/4 English this year, I am required to read Island, The Golden Age, Ransom and to watch Invictus. For Island, our teacher strongly recommends taking notes for characters under the headings of fathers, mothers, sons, siblings, grandparents, married couples, as well as tradition, work, education, and environment. I have already read the texts, but am slightly unsure about the best way to take to take the notes.
Thanks.
Title: Re: VCE English Question Thread
Post by: clarke54321 on January 18, 2019, 10:01:22 am
Hello!
For unit 3/4 English this year, I am required to read Island, The Golden Age, Ransom and to watch Invictus. For Island, our teacher strongly recommends taking notes for characters under the headings of fathers, mothers, sons, siblings, grandparents, married couples, as well as tradition, work, education, and environment. I have already read the texts, but am slightly unsure about the best way to take to take the notes.
Thanks.

Hello,

Although I haven't read 'Island,' I'd suggest that you make notes on each of these headings chapter by chapter. If you try and grapple with the book as a whole, it may seem like an overwhelming task to localise the significant events, which encompass mothers and fathers, for example. Also, by making notes according to chapter, you will come to appreciate the subtle developments of character/theme. This is a really key skill in English.

All the best :)
Title: Re: VCE English Question Thread
Post by: jnylle on January 29, 2019, 07:25:31 pm
I have to write a statement of intention for a creative piece and was wondering if anyone has any great tips for writing an SOI or using the FLAP+C! (thank you in advanced!)  ;D
Title: Re: VCE English Question Thread
Post by: JR_StudyEd on February 04, 2019, 07:54:59 pm
For those who have had access to and used Edrolo for English in the past, was it helpful to you?
Title: Re: VCE English Question Thread
Post by: Sine on February 04, 2019, 08:59:06 pm
For those who have had access to and used Edrolo for English in the past, was it helpful to you?
not really tbh, I found that the best way to improve at the start would be to write essays and get the corrected so you can learn in which places you can improve. You could also just do essay plans which is also quite helpful as you can get through heaps of prompts without having to take the time to finish it.
Title: Re: VCE English Question Thread
Post by: clarke54321 on February 07, 2019, 10:11:47 am
For those who have had access to and used Edrolo for English in the past, was it helpful to you?

In my experience, Edrolo was a bit of a hit and miss. For example, I had an excellent teacher speak about the intricacies of Medea, and how context informed the plot and character intentions of Euripides. This specialised knowledge was something I couldn't really find online or in the classroom. And so, in this sense, it was definitely a valuable resource. However, when it came to the sample essays, the site became essentially redundant. The essays that they classified as 'high scoring' would probably earn you a 6-7 on a VCAA exam. Therefore, as Sine has mentioned, if you are looking for accurate feedback on writing, it is best to work closely with your teacher or a tutor.
Title: Re: VCE English Question Thread
Post by: Just another student on February 07, 2019, 06:35:37 pm
1) I was wondering if anyone has any resources for a text my school is studying called 'The Women of Troy.' The only resources I have are two prompts which my teacher gave us, does anyone have any other prompts? Or any other resources such as themes/quotes/exemplar essays.

2) This may sound stupid but I need some clarification on what your main idea is in each BP? Is it something the author is trying to convey?
Title: Re: VCE English Question Thread
Post by: You dont know me on February 12, 2019, 12:14:54 pm
Does anybody have detailed notes in relation to the book "The Golden Age"?

Would just like to read over them to see if I've missed anything important in the book.
Title: Re: VCE English Question Thread
Post by: lst1103 on February 14, 2019, 09:12:54 am
For the creative SAC, were people able to choose the direction of their story (alternate ending, journal entry, etc.) or was it set by the teacher?
Title: Re: VCE English Question Thread
Post by: Monkeymafia on February 15, 2019, 10:58:33 am
Can anyone let me know if the following analysis would be correct/if I could expand on it?

NASA's technology investments continue to make a different in the world around us.

In his description of the space technology corporation "NASA" being beneficial to the "world around us," the author attempts to inflate concerned citizens sense of apprehension for the chaos that may result if the government did not continue to invest in aerospace technology. Thereby, governmental departments may be more inclined to invest in NASA to reduce the concern amongst citizens or face scrutiny from the general public.
Title: Re: VCE English Question Thread
Post by: alanihale on February 26, 2019, 06:44:28 pm
How do you expand on or flesh-out ideas in a text response??
Title: Re: VCE English Question Thread
Post by: InnocentKid on February 27, 2019, 04:51:26 pm
so I got a analytical essay comin up  :-[ and I am legit no joke shit at English....

I need your help.  the text is "Night" by Ellie Wiesel
my teacher says my writing is pretty good and stuff but I actually feel like she's to afraid to tell me the truth ....idk. :'(
if u could give a brief read and tell me on ways to improve that would be amazing
thnx in advance   ;D

TOPIC: “In ‘Night’ Wiesel uses imagery to portray the horrors of his experiences. Discuss”



In Elie Wiesel’s memoir, ‘Night’, literary devices, such as imagery, are manipulated to discuss traumatic experiences faced throughout the time of the text. Figurative language is often used to explore the themes of night and darkness and can foreshadow key events throughout the text, while imagery and symbol around death, trauma, pain and suffering are also used to show the full depth of experience.

Within the text  Wiesel uses a variety of literary devices to depict how Night and Darkness affected him at the beginning of his journey. Wiesel brings up the theme of night  a number of times throughout the text to foreshadow demoralising events that were to come “ It was close to midnight”  Wiesel uses this concept of night  to communicate seemlislingy  endless darkeness  that was to come when his entire Jewish community would be transported of to Auschwitz. Wiesel  also portrays night as an enemy and a threat  during  the course of the text…he presents himself in a manner in which he “fears” night and as it  brings along darkness with it “The night seemed darker outside the gates of Auschwitz” the way in which he uses this sentence shows the significant impact “night” and “darkness” has had on him, the evil, pain and suffering that follows after. Ellie seems to be so afraid of Night and Darkness as he sees the product of the damage wherever he looks in the camp. Ellie using detailed imagery and metaphors, describes his fellow Jewish peers as already half dead “the shdows around me roused themselves as if from a deep sleep and left silently in every direction” this sentence has a significant impact on the audience showing them the quality of life Jews were forced to burden throughout the war and how they dealt with this. Wiesel uses his knowledge of writing to  manipulate literary devices such as imagery and metaphors to show how Night and Darkness took a horrendous  toll on him and his Jewish companions.


Wiesel also uses string figurative language and symbolism to explore the key concept of death in his memoir. Wiesel specifically focuses on “Death” as it was a fundamental aspect of his story, and affected nearly every Jew during the time of war. He illustrates this importance by using imagery and symbolism to truly give the reader an insight on the pain and suffering he experiences firsthand  his “ we were withered tress in the heart of the dessert”, Wiesels maudlin tone illustrates his thought on the matter, the result of the war and what they had become, the way they had developed and progressed as time passed.  Furthermore the way in which he synthesises his symbolism and imagery link perfectly to the event that were currently happening, this is exemplified when selection took place and certain individuals were selected, Wiesel describes them as “ wallowing in their despair” ,  “ trying to cling to hope” and struggling between “life and death”, as they were to be of no more in the near future, Wiesels intent  of this sentence was  for the reader to experience what  he felt at that time, empathy and sorrow for the “lost souls”.  Wiesel appropriately defines death in his memoir and the battles he and so many more faced against it, he tries to communicate to the reader his understanding of what he experienced in the most elaborate and meek format so that he may extinguish his agony.



An important factor in which everyone at the concentration camps experienced was the trauma that everyone grieved  during that time . Wiesel portrays trauma through linguistic methods in which he clearly reveals the horrors he experienced. At the beginning of the text Wiesels jumps straight to the inhuman acts he witnessed upon arriving in Auschwitz, “I pinched myself: Was I still alive? Was I awake? How was it possible that men, women, and children were being burned and that the world kept silent? No. All this would not be real. A nightmare perhaps … Soon I would wake up with a start, my heart pounding, and find that I was back in the room of my childhood”, the far – fetched  tone in which Wiesel expresses himself makes it distinctly obvious that he could not grasp of the acts being committed “how could this happen in 21st century quote” The disbelief Ellie shows suggests that  the Jews and Ellie  could never suspect that their “friends of yesterday” could allow something inhumane as this to happen. This would have deeply traumatized many, an act of betrayal upon them for simply being “different” both physically and culturally. Throughout the text traumatic events consistently took place, the fact of being kept in a concentration camp, enduring labour would have been traumatic enough for many, however certain events disturbed Ellie more than others. Wiesel emphasises this by adding in more depth to these parts than he would to others, for example Wiesel dedicates a whole page to “Juliek” the boy who played the violin ( Beethoven ) , Wiesel at the time was “crushed under the weight of other bodies” and “biting his was to search for air” and was distressed and close to mere death, a traumatic experience however “Juliek” who was also in the same position  releaves Wiesel when he plays the violin for the last time  expressing his thoughts and giving a deep and contextual meaning “his soul had become his bow”, this was both physical and emotionally traumatising for Wiesel especially when he wakes to find Juliek deceased the following morning. Other traumatising events include the moment in time when Wisels own father was beaten to death in front of him and he coudnt do anything to save him “ he called out to me and I had nmot answered” this would possibly be one of the momest stressful and traumatising events that took place in the text, it would have left Ellie with permanent psychological scars for life as he deeply cared for his father “ I remained more than an hour leaning over him, looking at him, etching his bloody, broken face”  Unfortunately for many Jews this was a common traumatic  hurdle many had  to fight when losing their loved ones. Wiesel does an excellent job at providing the audience with the traumatic experiences he had to endure.

In Night, produced by Ellie Wiesel. Literary devices are harnessed to create a deep and powerul meaning  text which accurately depicts the traumatic and horrific experiences Wiesels faced at his time in Concentration Camps during WW2. Wiesel uses his knowledge of literature to explore themes like night and darkness in which he gives detailed evidence of how it forshadows certain events to come, he also explores death and trauma with imagery and figurative language to give the reader a vivid image of the “hellish” experiences Jews took.
Title: Re: VCE English Question Thread
Post by: TTanner01 on March 05, 2019, 08:52:43 pm
Anyone know how the introduction of an essay is structured?

I usually do Broad statement of what the book is about --> Contention --> Body paragraphs

However, my teacher said that I shouldn't even have body paragraphs mentioned in the introduction, which is confusing because it was always emphasised that I should when I was in year 7 all the way to year 11.

Any help?
Cheers!
Title: Re: VCE English Question Thread
Post by: SmartWorker on March 11, 2019, 12:48:47 pm
Hi,

Does anyone have notes in regard to 'Night' by Elie Wieel
Title: Re: VCE English Question Thread
Post by: gdog123 on March 15, 2019, 04:51:44 pm
Hey there,

I am doing Behind the Beautiful Forevers Creative Essay soon. I'm really struggling to get an A+ does anyone have any ones they have did, so I can use it for ideas please?
Title: Re: VCE English Question Thread
Post by: Sohan on March 16, 2019, 09:01:54 pm
Hello guys, i am doing my creative piece on the "The Return" from Island by Alistair Macleod. So i am writing about Angus explaining his mother why he had to send the drunkard brother out. can anyone give me ideas about what to write and can also anyone check my written explanation.
Thanks
Title: Re: VCE English Question Thread
Post by: OZLexico on March 17, 2019, 11:02:52 am
Hi Sohan, I think you have a good idea to use Angus' conversation with his mother about his rejection of the drunkard brother.  However, I think this would work better as an extension of the existing scene on p 87 instead of making it a kind of add on epilogue at the end of the story.  So Angus' wife's return to the sitting room would be delayed while Angus and his mother discuss what has happened in Montreal that has caused Angus to send his brother back to the family home in Cape Breton.  To plan this out, you should look again at what you know of the drunkard brother (his working in the mine along with his father and four other brothers, his desperate attempts to recover the body of Andrew from the mine collapse).  (Not sure if you want to draw on details of a mine accident in a different story - "The Vastness of the Dark" see p 51-54) You can also draw in other aspects of the characters' lives - the seven generations in Cape Breton, the masculine companionship of the mining, and the communal bathing at the end of the work day. The mother also seems opposed to Angus' very different life in Montreal, as if this displays disloyalty to the family heritage.  Her views appear (to her) to be vindicated by the suicide of her other well educated son - the doctor, Alex (the 10 year old narrator Alex is probably named in his memory).  Then, it's up to you to work out your part of the story - is the brother's alcoholism a direct result of Andrew's death in the mine collapse? Why might this be? Is he feeling some guilt or responsibility for the brother's death? Why would leaving Cape Breton for Montreal be a good idea for him and what kind of support might Angus have tried to provide?  Once in Montreal, what went wrong?  What happened that made Angus think his alcoholic brother would be better back at home with their parents? What has been the role of Angus' wife in this?
As for your Written Explanation, I think you should get some of your own creative piece done first, then modify what you've written in your Explanation, to ensure that the two pieces are consistent.  I think your plan is to enhance the reader's understanding of relationships in Angus' family, the bond to the environment of Cape Breton and the damage to that bond when family members leave ("we just can't live in a clan system any more").  Give some thought to the aspects of MacLeod's writing style and his use of naturalistic informal conversation, supported with brief vivid description of the environment (the white sea birds and the black scars the mine makes on the green hillsides). Keep in mind the ambiguous title of the story too - whose "return" does it refer to?       
Title: Re: VCE English Question Thread
Post by: Sohan on March 17, 2019, 04:29:00 pm
Hi Sohan, I think you have a good idea to use Angus' conversation with his mother about his rejection of the drunkard brother.  However, I think this would work better as an extension of the existing scene on p 87 instead of making it a kind of add on epilogue at the end of the story.  So Angus' wife's return to the sitting room would be delayed while Angus and his mother discuss what has happened in Montreal that has caused Angus to send his brother back to the family home in Cape Breton.  To plan this out, you should look again at what you know of the drunkard brother (his working in the mine along with his father and four other brothers, his desperate attempts to recover the body of Andrew from the mine collapse).  (Not sure if you want to draw on details of a mine accident in a different story - "The Vastness of the Dark" see p 51-54) You can also draw in other aspects of the characters' lives - the seven generations in Cape Breton, the masculine companionship of the mining, and the communal bathing at the end of the work day. The mother also seems opposed to Angus' very different life in Montreal, as if this displays disloyalty to the family heritage.  Her views appear (to her) to be vindicated by the suicide of her other well educated son - the doctor, Alex (the 10 year old narrator Alex is probably named in his memory).  Then, it's up to you to work out your part of the story - is the brother's alcoholism a direct result of Andrew's death in the mine collapse? Why might this be? Is he feeling some guilt or responsibility for the brother's death? Why would leaving Cape Breton for Montreal be a good idea for him and what kind of support might Angus have tried to provide?  Once in Montreal, what went wrong?  What happened that made Angus think his alcoholic brother would be better back at home with their parents? What has been the role of Angus' wife in this?
As for your Written Explanation, I think you should get some of your own creative piece done first, then modify what you've written in your Explanation, to ensure that the two pieces are consistent.  I think your plan is to enhance the reader's understanding of relationships in Angus' family, the bond to the environment of Cape Breton and the damage to that bond when family members leave ("we just can't live in a clan system any more").  Give some thought to the aspects of MacLeod's writing style and his use of naturalistic informal conversation, supported with brief vivid description of the environment (the white sea birds and the black scars the mine makes on the green hillsides). Keep in mind the ambiguous title of the story too - whose "return" does it refer to?       

Thanks a lot OZLexcio really appreciated.
Title: Re: VCE English Question Thread
Post by: hayden2101 on March 30, 2019, 04:57:12 pm
How do i write an introduction on The Golden Age for this topic????

Topic: At The Royal Perth Hospital and The Golden Age, Frank struggles with more than just polio. Discuss
Title: Re: VCE English Question Thread
Post by: f0od on April 02, 2019, 05:19:25 pm
I recently got my first SAC (creative) back and I got a B. I was wondering if there is still a chance for me to get a 40+ ss in english ://
Title: Re: VCE English Question Thread
Post by: Sine on April 02, 2019, 09:21:35 pm
I recently got my first SAC (creative) back and I got a B. I was wondering if there is still a chance for me to get a 40+ ss in english ://
Your sac rank is what is important not your actual sac grade. Also there should be more sacs this semester and for the rest of the year to improve your ranking.
Title: Re: VCE English Question Thread
Post by: EllingtonFeint on April 09, 2019, 12:50:35 pm
Hey,
I'm a little stuck on some arguments for this essay I'm writing for an analytical on Euripides' Women of Troy.
The prompt is...
“Euripides continues the Greek literary tradition of portraying Helen of Troy as the root of all evil.” To what extent do you agree?
So, some ideas for paragraphs that I have are...
-It's never actually been a Greek literary tradition to portray Helen as the root of all evil (But how could I back this up? I would have to refer to external sources and that's not good, is it..,?)
-Euripides does allow Helen to be perceived as evil, however, she is not the ROOT of ALL evil.
-However, Euripides also portrays Helen as a powerless victim of the events of Troy so she can't be totally evil.
-Perhaps Euripides doesn't have intentionally portray her as evil or as a victim. Perhaps he simply portrays her as human.

Please tell me what you think of my arguments. I'm still uncertain as to how to write that first argument... :/
Looking forward to any tips or feedback :)
Title: Re: VCE English Question Thread
Post by: SmartWorker on April 14, 2019, 04:25:51 pm
Hi everyone,

So I got back my SAC on Night and I received a 8/10. My teacher suggested areas I had too improve on was written expression (my sentences were too long and needed to proofread especially around punctuation). I was wondering how I can improve this?

Thanks.
Title: Re: VCE English Question Thread
Post by: Peas on April 15, 2019, 12:09:38 pm
How do you create a quote bank? I have many quotes for the novel I am studying but I am struggling to organise them. Should I order them under different themes, ideas, characters, etc.? I also find that many quotes do not fall into one specific category but rather overlap with each other so what would you do?
Title: Re: VCE English Question Thread
Post by: jemima.allpress on April 15, 2019, 05:22:01 pm
Anyone have any tips on how to improve vocabulary?

At the moment, my marks are around 7-8/10, but my teacher says with more sophisticated and improved vocab, my essays will go beyond that.
Title: Re: VCE English Question Thread
Post by: Stephanie Joyce on April 15, 2019, 06:38:07 pm
How do you create a quote bank? I have many quotes for the novel I am studying but I am struggling to organise them. Should I order them under different themes, ideas, characters, etc.? I also find that many quotes do not fall into one specific category but rather overlap with each other so what would you do?


Hey,

I find that organising quotes into quote banks is sometimes difficult but there are two ways you can approach this in terms of the overlap. The first is to organise a couple of banks that deal with Themes and another that deals with Characters and then have quotes go wherever is relevant. Maybe that means that a quote is under more than one heading, but that might take a bit more work and could be quite time-consuming. Bearing this in mind, consider having a bank in the form of a table with three columns; the first should have the quote, the second some basic analysis so you understand how you might talk about the quote in an essay and the third with ideas about what the quote is relevant to (eg, dot point the themes/characters this would apply to). Then whenever you are looking for a quote for a certain character/idea/theme you can Ctrl+F (or Command+F for a Mac) the word you are looking for and easily find all the entries that work for that theme.

Hope that helps,
Steph
Title: Re: VCE English Question Thread
Post by: Stephanie Joyce on April 16, 2019, 05:18:37 pm
Anyone have any tips on how to improve vocabulary?

At the moment, my marks are around 7-8/10, but my teacher says with more sophisticated and improved vocab, my essays will go beyond that.

Hey,

The best way to improve vocab is to read, read, READ. Whenever you see a word you don't know, search it up and then try to use it in conversation to get familiar with it. Otherwise, a more direct method might be to try and create a word bank for words that you use a lot in your essays and then actively try to add new synonyms into your work for more diversity of language.

Hope this helps!
Title: Re: VCE English Question Thread
Post by: jemima.allpress on April 17, 2019, 09:37:18 am
Hey,

The best way to improve vocab is to read, read, READ. Whenever you see a word you don't know, search it up and then try to use it in conversation to get familiar with it. Otherwise, a more direct method might be to try and create a word bank for words that you use a lot in your essays and then actively try to add new synonyms into your work for more diversity of language.

Hope this helps!

Thanks heaps Stephanie Joyce. That's very helpful :D
I like the idea of creating a word bank with words that I always use in my essays because I always feel as though I am repeating myself.  ::)
Title: Re: VCE English Question Thread
Post by: nxthxnsxx on April 22, 2019, 11:29:49 am
Hello there!

I'm currently compiling a word bank for my essays (meaning that I'm grouping my new vocabulary according to the three main types of essays to be tested in the final exams). I was wondering if anyone had any vocabulary resources for text analysis and comparative essays. I know that people say that it's a matter of personal opinion in creating these word banks and to do further reading to actively consolidate the vocabulary, but I've done so already, but I feel as if my text response and comparative word lists were lacking... Any help would be appreciated! Thanks!!
Title: Re: VCE English Question Thread
Post by: gary123 on April 23, 2019, 08:01:21 pm
Hello, text response question.
My teacher REALLY emphasises on incorporating analysis of a text in relation to the world of the author - meaning not just their implicit values, but also how their life and experience during their time influenced their writing. E.g for In Cold Blood, the author focuses on appealing to readers compassion for a killer(based on true events) who has suffered a chaotic upbringing and violent childhood(family divorce, parental abuse, abandonment), with this culminating into the cold blooded actions he does one day. In this text, the author himself also has experienced a chaotic upbringing and violent childhood and what my teacher says is that I should analyse with reference the to the fact the author has experienced the same thing as the character. However I don't see the relevance of bringing the events of the authors personal life into my essays and am hoping for some clarification on this because I know to analyse authorial intent and values but not their personal life??? My friend's tutor said you shouldn't analyse in reference to the authors life. Thanks!
Title: Re: VCE English Question Thread
Post by: OZLexico on April 25, 2019, 09:45:34 am
Hi gary123, I sort of agree with your friend's tutor to keep the author's life experiences out of your essay writing, however, I think there is a place for the social (and maybe personal) context of the book.  As you are writing your essays under time pressures, there's only limited attention you can give this aspect of your response to a text essay topic. Capote goes to a lot of effort to represent the normal-ness of the murder victims and this is in strong contrast to his own disfunctional childhood and forms a social commentary on the admirable conventions of an average family when there are clearly plenty of examples of family disfunction out there.  Despite the stability and normal-ness of the victims, this fails to protect them from violence. It's interesting too, that Capote's focus on the killers is fairly single minded until later in the text when you see them sharing the prison with several other cold blooded killers horribly similar to them.
Title: Re: VCE English Question Thread
Post by: lach_chau04 on April 25, 2019, 07:41:59 pm
Guys helpppppppppp.

I want to prepare early for Year 12 VCE English and I'm in Year 10. What do I do?

Also, are there any documents out there that suggest good words to use for Year 12 English?

pm me pls - any advice would be helpful :)
Title: Re: VCE English Question Thread
Post by: kepsinlove on May 03, 2019, 11:53:42 am
Hi! We're studying Persepolis at my school and I was wondering how to approach "How" questions - like "How does Satrapi expose the dangers of power in her graphic novel". What sort of paragraph ideas and topic sentences are they looking for? Do they all need to be construction based, like "Through depicting everyday Iranians without distinguishable features, Satrapi suggests that they are equally maltreated under the regime"
OR can I have normal topic sentences like "Satrapi suggests that it is those lower in the social heirarchy that are the most vulnerable to misuse of power"

Please help!
Title: Re: VCE English Question Thread
Post by: undefined on May 04, 2019, 11:28:40 pm
Hey. For discuss questions, I know you're not meant to completely agree nor disagree with the topic, but does that mean you always have to discuss why it is true/untrue to an extent? Or can you just explore the prompt itself and indirectly agree with it?
For example, in EvangelionZeta's A Farewell To Arms text response,
Quote
A Farewell to Arms is concerned with Frederic Henry's growing understanding of both love and war.  Discuss.

Within its sweeping war-drama narrative, Ernest Hemingway’s A Farewell to Arms centralises the figure of Frederic Henry, whose growing understanding of both love and the conflicts surrounding him serves as the novel’s main thematic focus.
they do not talk about whether the prompt is true or false to an extent, but rather explore the prompt itself (and sort of indirectly agree with it).

Also for most of the sample essay examples in the english resources thread, they do not mention metalanguage or the author's intent which I've been told is important. Is it just because they're like 10 years old or is it not required to incorporate those kinds of things in text responses?
Title: Re: VCE English Question Thread
Post by: alanihale on May 05, 2019, 12:08:18 pm
Can anybody help me understand what on earth the contention of this cartoon is?? I'm not sure where it sits and what it's trying to do/show?
https://www.inkcinct.com.au/web-pages/cartoons/2019/2019-159P--Paleo-restaurant-vegan-protest,-cave-man---AUSTRALIA-8th-April.png
(it's in relation to the vegan protests in melb - the other two articles are pro-veganism but only one of them is critical of their methods)
Thanks!
Title: Re: VCE English Question Thread
Post by: NomotivationF on May 05, 2019, 12:20:27 pm
Hey. For discuss questions, I know you're not meant to completely agree nor disagree with the topic, but does that mean you always have to discuss why it is true/untrue to an extent? Or can you just explore the prompt itself and indirectly agree with it?
For example, in EvangelionZeta's A Farewell To Arms text response, they do not talk about whether the prompt is true or false to an extent, but rather explore the prompt itself (and sort of indirectly agree with it).

Also for most of the sample essay examples in the english resources thread, they do not mention metalanguage or the author's intent which I've been told is important. Is it just because they're like 10 years old or is it not required to incorporate those kinds of things in text responses?

My English teacher has told me that when a discuss question is asked, most of the time they want you to fully agree with the statement. However, if the statement given is outrageously wrong, you can almost create your own contention that disagrees with the statement provided and go on to write an essay fully disagreeing with the topic
Title: Re: VCE English Question Thread
Post by: NomotivationF on May 05, 2019, 12:25:23 pm
Can anybody help me understand what on earth the contention of this cartoon is?? I'm not sure where it sits and what it's trying to do/show?
https://www.inkcinct.com.au/web-pages/cartoons/2019/2019-159P--Paleo-restaurant-vegan-protest,-cave-man---AUSTRALIA-8th-April.png
(it's in relation to the vegan protests in melb - the other two articles are pro-veganism but only one of them is critical of their methods)
Thanks!

I’m assuming this is for language analysis. To me it seems like this article is attacking both sides by saying people that are anti-veganism are living in the past (hence the caveman). But you could also argue it to be attack vegan protesters and the way they go about protesting inside peoples restaunts or something like that. It would be easier to analyse this with the context of the article and where it sits within the article.
Title: Re: VCE English Question Thread
Post by: JR_StudyEd on May 08, 2019, 02:48:52 pm
Which of the three examinable areas of study (comparative, text response/analytical and argument analysis) is the most difficult piece to write? Which one requires the most work? (Obviously I know there won't be an objective answer to this given the fundamental subjectivity of English itself)
Title: Re: VCE English Question Thread
Post by: ErnieTheBirdi on May 11, 2019, 08:09:53 pm
Hey, I was wondering if anyone could help me with how to write and possibly ACE a language analysis? I would like to know, in what ways I could improve on language analysis and how to write it well enough to get high marks. Also, I have an English tutor that tells me to write a language analysis one way and my teacher at school tells me to do it differently, what should I do? Thankyou
Title: Re: VCE English Question Thread
Post by: OZLexico on May 13, 2019, 11:07:49 am
This seems really tough that you seem to be getting conflicting advice.  I think you should have another  look at your previous language analysis writing and your teacher's assessment comments.  What do they say is the weakness in your writing?  This will give your tutor something to focus on too.  You can always go back to your teacher and have a session with them so they can give you more advice about possible improvements.   Otherwise I'd suggest lots of practice analysis pieces - there are lots to choose from here on the Language Analysis Club bit, or you can look for your own examples (short ones like letters to the editor can be found on the sites of online newspapers).     
Title: Re: VCE English Question Thread
Post by: callum123 on May 14, 2019, 07:18:11 pm
Hey, I was wondering if anyone could help me with how to write and possibly ACE a language analysis? I would like to know, in what ways I could improve on language analysis and how to write it well enough to get high marks. Also, I have an English tutor that tells me to write a language analysis one way and my teacher at school tells me to do it differently, what should I do? Thankyou

My main advice for Language Analysis is to follow the WHAT-HOW-WHY process of analysis!
What is the author doing (identify the persuasive technique, quote her)
How does this position the audience to feel?
Why does the author want the audience to feel this way (what part of the contention is being pushed?

Another top tip is to be SPECIFIC with your discussion! Pick a quote and discuss what THAT quote is doing- not just all rhetorical questions in general!
Good luck!
Title: Re: VCE English Question Thread
Post by: gary123 on June 12, 2019, 11:26:22 pm
hi i just want to get some sort of explicit confirmation as to whether what my teacher expects in an la essay is true
basically if you are two or more texts, you have to focus on comparing the texts ie comparative analysis so he completely opposes blocking and wants integration structure. He used the 2016 vcaa report where there was an example of students offering 'astute insights' through comparison. Even though he references this he ignores the fact that there is no 'explicit expectation' to do this.
The criteria for our sac emphasised on how we compared the two texts given and I used block so I'm definitely not getting the mark i want.
the way he insists on his view makes me think i've been approaching la wrong but i doubt it since the task is to analyse langauge not compare.
Could I get some clarification on this for the end of year exam please? Is it okay to use blocking since its the most convenient for two or more articles?
Thanks
Title: Re: VCE English Question Thread
Post by: Matthew_Whelan on June 13, 2019, 12:24:18 am
hi i just want to get some sort of explicit confirmation as to whether what my teacher expects in an la essay is true
basically if you are two or more texts, you have to focus on comparing the texts ie comparative analysis so he completely opposes blocking and wants integration structure. He used the 2016 vcaa report where there was an example of students offering 'astute insights' through comparison. Even though he references this he ignores the fact that there is no 'explicit expectation' to do this.
The criteria for our sac emphasised on how we compared the two texts given and I used block so I'm definitely not getting the mark i want.
the way he insists on his view makes me think i've been approaching la wrong but i doubt it since the task is to analyse langauge not compare.
Could I get some clarification on this for the end of year exam please? Is it okay to use blocking since its the most convenient for two or more articles?
Thanks

This is just my opinion of course, but i think the integrated style works more effectively when there are two or more things to discuss. The opportunity for comparison is looking at how the authors have similar/different contentions, or the way they attempt to persuade, even compare their different audiences.
By comparing these features you show a better understanding of how they are both attempting to persuade. I think an integrated approach allows for side by side contrast which helps with this. A block approach can be effective but is less sophisticated (usually) than the integrated.

However, usually it's just one 'main' article and one or more smaller segments (comment or cartoon, ect) which means you mostly focus on one piece anyway. Points of comparison would be when you analyse a sub topic of the issue at hand that is addressed by both pieces.
Personally I break my paragraphs down by points of discussion (sub-arguments/topics) then analyse the langauge used to convey those points by the authors, and the subsequent effect.
Overall, to appease your teacher I'd stick with what he wants, but on the exam I'd expect them to be less fussed.
Title: Re: VCE English Question Thread
Post by: spursy on June 27, 2019, 05:33:56 pm
can someone explain whether this is for or against pill testing
Title: Re: VCE English Question Thread
Post by: AHomicidalDuck on June 28, 2019, 09:01:24 pm
I was wondering if I passed all my outcomes for Unit 1 English but fail the exam, am I at risk of failing the unit?
Title: Re: VCE English Question Thread
Post by: Calebark on June 28, 2019, 09:17:23 pm
can someone explain whether this is for or against pill testing

When examining a comic like this, try to imagine yourself as one of the characters. In this case, the (presumed) parents. What are you meant to feel looking at your child's corpse? Well, you're meant to feel despondent -- you're sad. Obviously, you don't want to be sad, so try and think of a way, relating to the context of the comic, that would've prevented you feeling this way. For this image, it'd having your child live. If they knew the pill was dangerous beforehand, perhaps they wouldn't have taken it, and they'd still be alive.

The comic is for pill testing, and is saying that without pill testing, the only way to know is by taking it, which could be fatal,

I was wondering if I passed all my outcomes for Unit 1 English but fail the exam, am I at risk of failing the unit?

I believe that as Unit 1 and 2 are purely internal (that is, not set by VCAA), then it's at your school's discretion. The way to go would be to an English teacher how it works at your school :)
Title: Re: VCE English Question Thread
Post by: milanander on June 29, 2019, 08:26:19 pm
Yesterday I got told that I had failed english this semester - as in, legitimately failed, so I'm getting an 'N' for unit 1. I averaged around 30% in my outcomes, and while I haven't gotten the results for our midyear exams back I won't be surprised if I failed that as well.

I'm really stressing out because I'm honestly doing anything I can think of but I've not seen any improvement at all. I've been doing wider reading and getting extra resources and reading over essays from way more talented students, even getting a tutor, but my score just isn't going up. I honestly feel like I might fail VCE, considering you must pass both 1/2 and 3/4 english, and it's not looking good for me atm.

Would really love some advice on how to effective study for this subject and see improvement. Also, how fked would I be if I also failed unit 2? Would I be not allowed to continue up to year 12?

any advice is appreciated!!
Title: Re: VCE English Question Thread
Post by: Matthew_Whelan on June 29, 2019, 09:44:03 pm
Yesterday I got told that I had failed english this semester - as in, legitimately failed, so I'm getting an 'N' for unit 1. I averaged around 30% in my outcomes, and while I haven't gotten the results for our midyear exams back I won't be surprised if I failed that as well.

I'm really stressing out because I'm honestly doing anything I can think of but I've not seen any improvement at all. I've been doing wider reading and getting extra resources and reading over essays from way more talented students, even getting a tutor, but my score just isn't going up. I honestly feel like I might fail VCE, considering you must pass both 1/2 and 3/4 english, and it's not looking good for me atm.

Would really love some advice on how to effective study for this subject and see improvement. Also, how fked would I be if I also failed unit 2? Would I be not allowed to continue up to year 12?

any advice is appreciated!!
Hey,
From what I've heard you can repeat the SAC until you attain a pass mark (but your original is the one that's weighed). I can't say for others but I find breaking down prompts, maybe with fellow classmates or your teacher will help as it will show you what you need to derive from it. I also find practising essays invaluable as it teaches you what you do know and don't; and it helps improve your expression and fluency. Another thing is to know your texts, so read them thoroughly and underline quotes or whatever is needed for you to get a good understanding of it. I suggest for Unit 3/4 to do this pre-emptively during the Summer holidays if you can.
I think if you are failing then speak to your teacher and do whatever you can to get a passing mark, I'm pretty sure you'll be fine.
Title: Re: VCE English Question Thread
Post by: Just another student on August 17, 2019, 12:50:49 pm
Hi there,

So for unit 3 I got 80/100 marks
and for my oral presentation I only got 26/40 :((((
I feel so discouraged now.

If I manage to get 56+/60 for my comparative essay and smash the exam, is there still hope for a ~45 study score?

Also how roughly how many practice essays did past high scorers do in prep for exam?

Thanks
Title: Re: VCE English Question Thread
Post by: Sine on August 17, 2019, 12:54:06 pm
Hi there,

So for unit 3 I got 80/100 marks
and for my oral presentation I only got 26/40 :((((
I feel so discouraged now.

If I manage to get 56+/60 for my comparative essay and smash the exam, is there still hope for a ~45 study score?

Also how roughly how many practice essays did past high scorers do in prep for exam?

Thanks
Depends on your sac ranking - which is more important than your actual scores so no one can really say whether you can get 45+.

As for how many prac exams - it will really vary between everyone - it's just important that you are getting better with each essay so that means getting someone to look at your essays and tell you what areas you can improve and also looking at really good sample essays.
Title: Re: VCE English Question Thread
Post by: w0lfqu33n89 on August 20, 2019, 11:41:37 am
hi all! in need some help planning my essay for Twelve Angry Men.
My topic sentence is "how does twelve angry men suggest that prejudice and self interest are the key enemies of justice."

I am in the middle of planning my essay. And I have chosen these as  my topic sentences:
- prejudice
- self - interest
- Juror 8.

Much help is appreciated even if its just a suggestion.

God bless you people xxxx
Title: Re: VCE English Question Thread
Post by: cherryblossoms on September 01, 2019, 09:07:01 pm
For text responses, which type of prompts is 'better' to write about, character-based prompts or thematic prompts? I know there isn't necessarily a 'better' one, but I can't choose between two prompts for a comparative essay.
Title: Re: VCE English Question Thread
Post by: Matthew_Whelan on September 01, 2019, 09:32:27 pm
For text responses, which type of prompts is 'better' to write about, character-based prompts or thematic prompts? I know there isn't necessarily a 'better' one, but I can't choose between two prompts for a comparative essay.

It mostly depends which theme/characters you have studied more throroughly, eg. if you have written a tonne of "leadership themed" essays then you are more likely to write an in-depth essay on leadership than if you did a theme you haven't looked at much.
I'd suggest to be prepared for whatever gets thrown at you but go with whichever you feel more confident with. Hope that helps.  ;)
Title: Re: VCE English Question Thread
Post by: han0002 on September 02, 2019, 12:01:48 pm
help with this essay question pls Comparative, 'Tracks' and 'Charlie's Country'

What, if anything, do Robyn and Charlie learn from their solo experiences?
Title: Re: VCE English Question Thread
Post by: JR_StudyEd on September 11, 2019, 08:34:10 pm
What is meant by ‘structural features’ of a text?
Title: Re: VCE English Question Thread
Post by: han0002 on September 12, 2019, 03:56:34 pm
What do 'Tracks' and 'Charlie's Country' indicate about the importance of having control over one's life? pls help
Title: Re: VCE English Question Thread
Post by: caffinatedloz on September 12, 2019, 07:16:46 pm
What do 'Tracks' and 'Charlie's Country' indicate about the importance of having control over one's life? pls help

Hey han0002,
I've seen you post this same question a few times around the forum. Do you want to try explaining what you already know and think, and others may be then able to jump in to let you know if you're on the right track and give you some extra insight?
Title: Re: VCE English Question Thread
Post by: james88 on September 18, 2019, 10:44:15 pm
URGENT HELP PLS

I need help for aN AA piece.

What would be the effect if having a rebuttal at the beginning of a piece?

Also why would an article explore/argue for both sides of the argument?

It would be great if someone could PM me back please as I really need help.

Much appreciated.
Title: Re: VCE English Question Thread
Post by: Matthew_Whelan on September 18, 2019, 11:11:50 pm
URGENT HELP PLS

I need help for aN AA piece.

What would be the effect if having a rebuttal at the beginning of a piece?

Also why would an article explore/argue for both sides of the argument?

It would be great if someone could PM me back please as I really need help.

Much appreciated.

Hi there,
Having a rebuttal is usually to render the opposing argument as erroneous or to highlight a flaw in it, while simultaneously reinforcing your own contention.

A few reasons a person might argue both sides is that they are discussing the issue and are not vehemently opposed to one side or another, maintaining a neutral stance.
 Otherwise it can convey a reasonable perception of the issue to the reader as the writer is willing to consider both sides of the argument, it also conveys a greater understanding of the issue at hand.
Title: Re: VCE English Question Thread
Post by: james88 on September 18, 2019, 11:14:00 pm
Thank you for that. Would you mind looking at the article for me and helping me out? Also what would be the tone of a neutral stance?

Much appreciated.
Title: Re: VCE English Question Thread
Post by: AnonymooseUser on September 23, 2019, 04:16:59 pm
Hello all,

Not sure if it belongs here, but I just have two questions
- How many quotes per Section A paragraph is generally considered a good amount?
- Is it bad to have an average essay length in the 750-850 word range?

Thanks
Title: Re: VCE English Question Thread
Post by: Loren_T on September 23, 2019, 04:58:22 pm
Hi
How many pages should I aim to do for the comparative essay in the exam- like 3 pages in 30 mins or something like that?
I can write pretty fast but should I try to take my time a bit to get my structure and wording right?
Title: Re: VCE English Question Thread
Post by: ErnieTheBirdi on September 23, 2019, 09:44:41 pm
Hey everyone on the ATAR Notes Forum!
I was just wondering does anyone have any tips for doing an English Oral? Although I am only in Year 10 this year I would like to really 'nail' my oral skills.
Also when doing an introduction should I mention my arguments? What would be the best way/structure to delivering my oral?
Thanks, everyone!
Title: Re: VCE English Question Thread
Post by: eliser24 on September 25, 2019, 12:03:59 pm
When it comes to memorising quotes for the exam, I was think of memorising 20 shortish quotes for each novel/play, but what sort of quotes should I memorise, should I memorise by theme, by character, or any other way? thank you :)
Title: Re: VCE English Question Thread
Post by: hello876 on September 26, 2019, 12:49:15 am
Hi, I am struggling with how to approach 'how' text response prompts. For example, "How does Hitchcock create an atmosphere of suspicion in Rear Window?" How differently do I approach this to that of a discuss question?
Title: Re: VCE English Question Thread
Post by: SmartWorker on September 27, 2019, 05:58:32 pm
Hi, I am struggling with how to approach 'how' text response prompts. For example, "How does Hitchcock create an atmosphere of suspicion in Rear Window?" How differently do I approach this to that of a discuss question?

Not an expert, but I believe it is referring to the ways in which Hitchcock is able to create this atmosphere.

So maybe look into:
- Characterisation
- Setting
- Language (figurative)

 :) :)
Title: Re: VCE English Question Thread
Post by: SmartWorker on September 28, 2019, 08:59:16 pm
Hi there,

Any suggestions on how I can improve my grammar and punctuation + sentence flow and expression. Please link me to some guides or textbooks that may help with this. I am open to all methods and willing to try them.

THANK YOU IN ADVANCE.  :) :) :) :) :)
Title: Re: VCE English Question Thread
Post by: Poet on October 01, 2019, 07:48:01 pm
Hi there,

Any suggestions on how I can improve my grammar and punctuation + sentence flow and expression. Please link me to some guides or textbooks that may help with this. I am open to all methods and willing to try them.

THANK YOU IN ADVANCE.  :) :) :) :) :)
Hi there,
As with everything, grammar and punctuation come with practice. (So do sentence flow and expression, but let's focus on the first two for now). For me personally, I found reading as a past-time a useful way of building up an almost subconscious record of the do's and don't's. Whether it be fiction or a science textbook, just reading a little bit every day will help you improve your overall writings. As well as this, try to use correct punctuation and grammar in everything. If you don't already, try texting and emailing full words and sentences instead of abbreviating. Make it a habit to write everything in the most succinct and precise language possible.

Sentence flow and expression can be helped with the same method, but that's a fairly passive form of practice. In regards to essays and more structured writings, make sure that you are planning out the main ideas you have in mind beforehand, using topic sentences and doing your best to be as concise as possible. Also, this will sound strange, but have you ever tried watching a movie or video and writing down what the characters are saying? Sentence flow and expression can most easily be identified in natural, fluent speech - why not transcribe it and see what you come up with?

You can find plenty of links from universities and schools with advice on how to improve these things, but Khan Academy is a go-to for exercises and quizzes on more advanced concepts. In particular, they have a lesson plan on grammar.

Hope this helps! :))
Title: Re: VCE English Question Thread
Post by: EmadMo on October 03, 2019, 08:34:18 pm
For Ransom and Invictus, I’m writing an essay on the importance of humility and was wondering if I could do a body Paragraph about how both Ransom and Invictus explore the dangers or detrimental impacts one faces when they lack humility. I was thinking about discussing Achilles and the National Sports Council but I’m not sure how I could link them to my point. Could someone help?
Title: Re: VCE English Question Thread
Post by: maryamalmufti on October 05, 2019, 04:06:42 am
you could take about the "self- consuming rage" Achilles went through after what he did to hector as a detrimental impact. Not sure about the sports council though.
Title: Re: VCE English Question Thread
Post by: SmartWorker on October 05, 2019, 06:45:37 pm
Hi there,
As with everything, grammar and punctuation come with practice. (So do sentence flow and expression, but let's focus on the first two for now). For me personally, I found reading as a past-time a useful way of building up an almost subconscious record of the do's and don't's. Whether it be fiction or a science textbook, just reading a little bit every day will help you improve your overall writings. As well as this, try to use correct punctuation and grammar in everything. If you don't already, try texting and emailing full words and sentences instead of abbreviating. Make it a habit to write everything in the most succinct and precise language possible.

Sentence flow and expression can be helped with the same method, but that's a fairly passive form of practice. In regards to essays and more structured writings, make sure that you are planning out the main ideas you have in mind beforehand, using topic sentences and doing your best to be as concise as possible. Also, this will sound strange, but have you ever tried watching a movie or video and writing down what the characters are saying? Sentence flow and expression can most easily be identified in natural, fluent speech - why not transcribe it and see what you come up with?

You can find plenty of links from universities and schools with advice on how to improve these things, but Khan Academy is a go-to for exercises and quizzes on more advanced concepts. In particular, they have a lesson plan on grammar.

Hope this helps! :))

Thank you so much. I will definitely try Khan Academy.
Title: Re: VCE English Question Thread
Post by: Otter on October 07, 2019, 10:01:20 am
Would a block paragraph structure in the comparative section negatively impact the number of marks your essay receives? Is it better to weave between the two texts?
Title: Re: VCE English Question Thread
Post by: Just another student on October 11, 2019, 07:46:32 pm
Hi AN
Can someone provide a a rough estimate of what I must get on the exam /60 for a 41-43 study score if I have sac scores of 80/100 and 75/100 (U3 & 4 respectively).
Title: Re: VCE English Question Thread
Post by: alanihale on October 13, 2019, 09:00:13 pm
Does anyone have any idea in regards to what a middle, an upper and a high range script would be equal to out of 10?? (From VCAA exam reports)
Thanks,
Title: Re: VCE English Question Thread
Post by: yourfriendlyneighbourhoodghost on October 15, 2019, 08:25:19 am
Hi, I am struggling with how to approach 'how' text response prompts. For example, "How does Hitchcock create an atmosphere of suspicion in Rear Window?" How differently do I approach this to that of a discuss question?

If I were were to answer a prompt like that, I would be looking at the portrayals of suspicion, the structural elements that create suspicion, the dialogue and symbols. I hope that helps (:
Title: Re: VCE English Question Thread
Post by: yourfriendlyneighbourhoodghost on October 15, 2019, 08:32:57 am
Would a block paragraph structure in the comparative section negatively impact the number of marks your essay receives? Is it better to weave between the two texts?

Block is good for consolidating your understanding of the two texts before you undergo your comparison paragraph. I think this is a good method when praticising, but for me, more comparison would be permitted if the integrated approach was used, nevertheless, the block approach is better than nothing so definitely use that if you feel most comfortable with it.
Title: Re: VCE English Question Thread
Post by: angrybiscuit on October 15, 2019, 11:52:07 am
Would a block paragraph structure in the comparative section negatively impact the number of marks your essay receives? Is it better to weave between the two texts?

It is absolutely important that you mention both texts in each of your paragraphs (at least this is what teachers have said.. not sure about vcaa markers). Also always have a 50:50 ratio when talking about texts in the paragraphs. It's okay to sometimes have a 20:80 ratio but you need to analyse both texts to a similar depth and degree. You wouldn't want an essay that largely focuses on one text only!
Title: Re: VCE English Question Thread
Post by: Just another student on October 15, 2019, 05:51:19 pm
In general, can the analyzing argument essay be shorter than the other 2 essays? Around 700 words? As I have heard that analyzing fewer techniques with greater explanation and effect is better then a long, less analyzed essay.   
Title: Re: VCE English Question Thread
Post by: randomnobody69420 on October 22, 2019, 01:13:09 pm
How many comparisons per paragraph is ideal for comparative? Right now I usually put in one similarity and 2 differences, sometimes a comparison of author values at the end. Is 3-4 too little?
Title: Re: VCE English Question Thread
Post by: Matthew_Whelan on October 22, 2019, 01:29:58 pm
One insightful comparison is worth more than 10 meaningless comparisons, however, there should be a few at least.
Title: Re: VCE English Question Thread
Post by: TheIllusion on October 24, 2019, 12:18:28 pm
Hey all!!,
Currently doing some English prep.
I was told by one of my English teachers that I would not need to write an conclusion for any of the sections of the English exam.
What do you guys think?
Title: Re: VCE English Question Thread
Post by: yourfriendlyneighbourhoodghost on October 24, 2019, 03:20:36 pm
Hey all!!,
Currently doing some English prep.
I was told by one of my English teachers that I would not need to write an conclusion for any of the sections of the English exam.
What do you guys think?

Hi, my teacher and many of the other English teachers at the school, said you must have a conclusion in order to satisfy the structure of the essay which gives your marks. (:
Title: Re: VCE English Question Thread
Post by: TheIllusion on October 25, 2019, 10:29:48 am
Hey yourfriendlyneighbourhoodghost,
Thanks for your reply. Helps a lot.
Title: Re: VCE English Question Thread
Post by: Sai Ramishetty on October 26, 2019, 10:51:12 am
Hi, my teacher and many of the other English teachers at the school, said you must have a conclusion in order to satisfy the structure of the essay which gives your marks. (:
I think this is not really applicable to Section C because the contentions are not ours. So I don’t think we need a conclusion. Note: Please correct me if I am wrong!
Title: Re: VCE English Question Thread
Post by: darkz on October 26, 2019, 11:18:40 am
I think this is not really applicable to Section C because the contentions are not ours. So I don’t think we need a conclusion. Note: Please correct me if I am wrong!

While it isn't necessary, it does make the essay look more complete - just one or two sentences summing it up - so won't take more than 1 minute either way.
Title: Re: VCE English Question Thread
Post by: RB04 on October 27, 2019, 02:23:47 pm
I am struggling with the theme of the way history repeats itself in Nine Days. The only examples I can think of is Connie and Charlotte's pregnancies before marriage and maybe the loss that Kip and Alec experience and what it teaches them. There is also the two pairs of twins, but I don't know what to say about them to make it a full body paragraph. If anyone has knowledge on Nine Days by Toni Jordan, I would really appreciate any help
Title: Re: VCE English Question Thread
Post by: NavyBlues9 on October 27, 2019, 08:02:43 pm
Hi, when I'm for example I'm writing Brooks's novel. Is it okay to have the 's or should it just be s' ?? Thanks!
Title: Re: VCE English Question Thread
Post by: Matthew_Whelan on October 27, 2019, 09:34:42 pm
I am struggling with the theme of the way history repeats itself in Nine Days. The only examples I can think of is Connie and Charlotte's pregnancies before marriage and maybe the loss that Kip and Alec experience and what it teaches them. There is also the two pairs of twins, but I don't know what to say about them to make it a full body paragraph. If anyone has knowledge on Nine Days by Toni Jordan, I would really appreciate any help

I think Nine Days doesn’t repeat history, but rather contrasts between the different time periods to highlight what has/hasn’t changed over time. Connie and Charlotte are parallels by their circumstances (pregnancy and single, although different reasons for both), yet Charlotte was able to choose whether she wanted to keep the child or not, whereas Connie was pressured into an abortion by Jean.

Kip and Alec similarly contrast between the generations. Kip as a child sees very little prospects for the future following Toms death as a result of him leaving school, being poor and the war drawing closer, yet creates a fulfilling life despite the odds. Alec on the other hand sees very little limitations to his future.

However, some things remain unchanged by time, such as human nature. Although societal views had changed, the conflict and love in people’s lives is consistent over time.
Title: Re: VCE English Question Thread
Post by: RB04 on October 28, 2019, 12:19:08 pm
I think Nine Days doesn’t repeat history, but rather contrasts between the different time periods to highlight what has/hasn’t changed over time. Connie and Charlotte are parallels by their circumstances (pregnancy and single, although different reasons for both), yet Charlotte was able to choose whether she wanted to keep the child or not, whereas Connie was pressured into an abortion by Jean.

Kip and Alec similarly contrast between the generations. Kip as a child sees very little prospects for the future following Toms death as a result of him leaving school, being poor and the war drawing closer, yet creates a fulfilling life despite the odds. Alec on the other hand sees very little limitations to his future.

However, some things remain unchanged by time, such as human nature. Although societal views had changed, the conflict and love in people’s lives is consistent over time.

This was very helpful. My teacher used the phrase 'history repeating itself' to describe it. But this makes a lot more sense. Thank you!
Title: Re: VCE English Question Thread
Post by: NavyBlues9 on October 28, 2019, 08:11:47 pm
Hi, when I'm for example I'm writing Brooks's novel. Is it okay to have the 's or should it just be s' ?? Thanks!
Can anyone help me out with this?
Title: Re: VCE English Question Thread
Post by: Sai Ramishetty on October 28, 2019, 09:19:57 pm
Can anyone help me out with this?
Brooks’
Title: Re: VCE English Question Thread
Post by: Coloratura on October 30, 2019, 02:46:25 pm
If you forgot to include a visual analysis in Section C of the English exam, how much will this impact your score?
Title: Re: VCE English Question Thread
Post by: jeremychoo on October 31, 2019, 06:42:28 pm
If you forgot to include a visual analysis in Section C of the English exam, how much will this impact your score?

It's not ideal but also really depends on the rest of your essay, so it's hard to say. I honestly wouldn't be worrying and stressing about what might have happened in the English exam and focusing on the upcoming exams that you can actually do something about.
Title: Re: VCE English Question Thread
Post by: ArtyDreams on November 09, 2019, 05:36:59 pm
HI! Just a question out of curiosity - (Year 11 english currently)

How important is it to have framework in our introduction for the Text Reponses or Comparatives? (i.e. mentioning what we are going to talk about in each of our body paragraphs?) My teachers have always told me to do it, but I feel like it just takes up so much time and makes my introductions look really cloggy. I'm scared to not do it as I dont want to lose marks for something as simple as that, but whats the deal in year 12?

Would appreciate any answers :)
Thanks!!!
Title: Re: VCE English Question Thread
Post by: Matthew_Whelan on November 09, 2019, 06:03:27 pm
HI! Just a question out of curiosity - (Year 11 english currently)

How important is it to have framework in our introduction for the Text Reponses or Comparatives? (i.e. mentioning what we are going to talk about in each of our body paragraphs?) My teachers have always told me to do it, but I feel like it just takes up so much time and makes my introductions look really cloggy. I'm scared to not do it as I dont want to lose marks for something as simple as that, but whats the deal in year 12?

Would appreciate any answers :)
Thanks!!!


Not sure if this was effective but I’d introduce the text(s) with a bit of background info, express my contention regarding the prompt (in a way that answers the prompt) and give 2-3 sentences as a general indication of what I’ll be talking about (sub arguments). Usually a succinct intro is effective but you want it to give your assessor a clear indication of how you’ll be addressing the prompt and what your view is.

With comparative I always have a sentence with a “both texts...” to highlight the similarity aspect and a “whereas ...” sentence to show understanding of differences.
Title: Re: VCE English Question Thread
Post by: ArtyDreams on November 09, 2019, 09:08:14 pm
Thank you!

Also one more thing, in a text reponse, how do you approach a question that ends with 'to what extent do you agree?'
Whats the best way to choose body paragraphs for this?

I understand that in a 'discuss' question youd talk for and against the topic, what different is this ^ ?
Title: Re: VCE English Question Thread
Post by: Matthew_Whelan on November 09, 2019, 11:19:33 pm
Thank you!

Also one more thing, in a text reponse, how do you approach a question that ends with 'to what extent do you agree?'
Whats the best way to choose body paragraphs for this?

I understand that in a 'discuss' question youd talk for and against the topic, what different is this ^ ?

To what extent do you agree is different as it’s not simply discussing the topic but having to argue something. Eg. Love is more important to [character] than greed, to what extend do you agree? You take a stance which either refutes or supports the assertion in the prompt. It also shows more complexity of thinking to argue both sides but there will usually be one side that is more obvious so don’t sit on the fence.

Discuss gives more freedom in terms of you just analysing that theme/character/idea in the text. But you might disagree with the prompt which you can discuss (with evidence). I do mine by first paragraph supporting my contention, and the second or third paragraph offering a different insight to the prompt. They’re pretty similar but “what extent do you agree?” stipulates an agreement or disagreement.

So for example in Nine Days if the prompt asks about family’s importance to characters you could talk about characters that prioritise their family like Kip, then also discuss those that don’t prioritise family, and if this is a result of another motive such as self interest or love. And the implications what they prioritise reveals the values of the characters and even the author (characters sometimes reflect the views/values of the author). You don’t have to completely agree or disagree with VCAA’s assertion and as long as you back your argument up with textual evidence (quotes/scenes) then you’re fine.

(I think this was a bit repetitive I think I answered you’re question  :) )
Title: Re: VCE English Question Thread
Post by: ArtyDreams on November 10, 2019, 03:40:25 pm
To what extent do you agree is different as it’s not simply discussing the topic but having to argue something. Eg. Love is more important to [character] than greed, to what extend do you agree? You take a stance which either refutes or supports the assertion in the prompt. It also shows more complexity of thinking to argue both sides but there will usually be one side that is more obvious so don’t sit on the fence.

Discuss gives more freedom in terms of you just analysing that theme/character/idea in the text. But you might disagree with the prompt which you can discuss (with evidence). I do mine by first paragraph supporting my contention, and the second or third paragraph offering a different insight to the prompt. They’re pretty similar but “what extent do you agree?” stipulates an agreement or disagreement.

So for example in Nine Days if the prompt asks about family’s importance to characters you could talk about characters that prioritise their family like Kip, then also discuss those that don’t prioritise family, and if this is a result of another motive such as self interest or love. And the implications what they prioritise reveals the values of the characters and even the author (characters sometimes reflect the views/values of the author). You don’t have to completely agree or disagree with VCAA’s assertion and as long as you back your argument up with textual evidence (quotes/scenes) then you’re fine.

(I think this was a bit repetitive I think I answered you’re question  :) )

thank you so much for your help. I think I get the general idea of it now!!
Title: Re: VCE English Question Thread
Post by: natalie03 on January 12, 2020, 07:27:43 pm
Hi,
Has anyone studied the graphic novel Maus? I would love some help finding some notes to get a better understanding of the book
Thank youuu xoxo
Title: Re: VCE English Question Thread
Post by: SmartWorker on January 17, 2020, 07:18:12 pm
Hi,
Has anyone studied the graphic novel Maus? I would love some help finding some notes to get a better understanding of the book
Thank youuu xoxo

Search up Sparknotes or Shoomp
Title: Re: VCE English Question Thread
Post by: Mudasser.abb on February 10, 2020, 06:07:19 pm
How do you get better at creative writing and any tips for studying English?
Title: Re: VCE English Question Thread
Post by: Sine on February 10, 2020, 10:26:53 pm
How do you get better at creative writing and any tips for studying English?
I didn't study too much creative writing in VCE so can't help too much with that.

As for overall English study, I believe practice essays are one of your most important techniques. So before sacs, I would be writing out a bunch of essays and getting them marked and receiving feedback from teacher's. Then use that feedback for your next essay. To improve vocab I would after writing essays see if I was using a word too much and then look for alternatives for that particular word - also if there was any area I found a bit clunky in expression I would try to rewrite more eloquently.

As for actually wirting your essays I think having a plan before you start is so important - even if you spend 5 minutes on this if you have this done you will have a direction for your essay. If you have written many essays before once you have the plan the essay basically writes itself as you will be reusing ideas from older essays you have written.

Some people will say to memorise essays - I didn't do this although I kind of unconsciously memorised paragraphs or parts of paragraphs which I used many times. So the memory would just come by me using a certain idea many times. However, students definitely do well with memorising essays - personally I don't recommend it though.
Title: Re: VCE English Question Thread
Post by: lr2020 on February 18, 2020, 05:35:32 pm
Hello,

I'm not sure if this is the right place to ask, but we're doing a creative piece on Peter Skrzynecki's Old/New World collection. Our prompt says to choose one of his poems and rewrite it as a narrative (story).

1. I'm not sure where to draw the line with this in terms of how much we can add to it (isn't that kinda the point of a creative?) and when it is too much of our own work to be considered 'rewriting' the poem.

2. Also, is rewriting the poem (still as a narrative) from another character's point of view incorrect in terms of responding to the prompt?

I'd really appreciate some feedback on understanding this prompt asap!
Thanks! :D
Title: Re: VCE English Question Thread
Post by: PranatiP on March 26, 2020, 09:42:59 pm
Hey guys!
I am looking for an English tutor who is really experienced that can help me achieve a really high score, but affordable (preferably less than $60 per hour).
I am studying the movie 'Rear Window' and the texts 'The 7 stages of grieving' and 'longest memory'.
If your teacher, or a VCAA examinor is tutoring within the price range, please let me know!!

I really need help with writing my essays, as it lacks sophistication, making it hard for me to achieve a 40+ study score in English!
Thanks!!!
Title: Re: VCE English Question Thread
Post by: essynedj on April 09, 2020, 05:52:11 pm
hi! would it be better to split the visual analysis of one visual in two paragraphs or just all of the visual analysis into one paragraph? thankyou! :)
Title: Re: VCE English Question Thread
Post by: natalie03 on April 17, 2020, 12:03:10 pm
For English, I have the choice of doing either a monologue or narrative towards a selection of poems from love dreaming. I was thinking of doing a monologue but I have no idea how to start it. if someone could give me a guide I would love that, thank you!!
Title: Re: VCE English Question Thread
Post by: whys on April 26, 2020, 02:48:21 pm
When we are saying that an author uses formal language, how do we use evidence for that? I usually get stuck on trying to quote formal language, because the entirety of the article is formal, so I don't know where exactly in the article I should quote from that displays overtly formal language.

Also, I was wondering how to incorporate tonal analysis in AA. In how much detail do we need to analyse the tone, and how do we do this?
Title: Re: VCE English Question Thread
Post by: Bri MT on April 26, 2020, 03:03:03 pm
When we are saying that an author uses formal language, how do we use evidence for that? I usually get stuck on trying to quote formal language, because the entirety of the article is formal, so I don't know where exactly in the article I should quote from that displays overtly formal language.

Also, I was wondering how to incorporate tonal analysis in AA. In how much detail do we need to analyse the tone, and how do we do this?

Formal vs informal langauge is a big thing in eng lang so you might find it useful to look at some eng lang notes for features of formal vs informal. Off the top of my head here the main things to look out for:
- word choice (standard English words (e.g. dog not doggo) , archaic language, etc.)
- sentence structuring (more complex and/or complicated sentences, adhering to grammar conventions etc.)

If you have difficult finding those resources lmk and I'm happy to give you more detailed advice

hi! would it be better to split the visual analysis of one visual in two paragraphs or just all of the visual analysis into one paragraph? thankyou! :)

This is a hard question to answer on its own as doing a split/integrated approach depends on how you are structuring the rest of your piece and how you want to conduct your analysis. If you're looking for a generic rule I imagine that 1 paragraph would be more common than two but I'm not super familiar with VCE English assessment so hopefully someone else can chip in here for you :)

Hey guys!
I am looking for an English tutor who is really experienced that can help me achieve a really high score, but affordable (preferably less than $60 per hour).
I am studying the movie 'Rear Window' and the texts 'The 7 stages of grieving' and 'longest memory'.
If your teacher, or a VCAA examinor is tutoring within the price range, please let me know!!

I really need help with writing my essays, as it lacks sophistication, making it hard for me to achieve a 40+ study score in English!
Thanks!!!


I recommend you look in the tutoring section of the forums and post there instead of in the English section to help you find someone



I'm not English-focused generally but I hope this helps you all out :)
Title: Re: VCE English Question Thread
Post by: SmartWorker on April 26, 2020, 04:49:42 pm
Formal vs informal langauge is a big thing in eng lang so you might find it useful to look at some eng lang notes for features of formal vs informal. Off the top of my head here the main things to look out for:
- word choice (standard English words (e.g. dog not doggo) , archaic language, etc.)
- sentence structuring (more complex and/or complicated sentences, adhering to grammar conventions etc.)

If you have difficult finding those resources lmk and I'm happy to give you more detailed advice

Hey Bri MT,

I find I have the same problem, can you please elaborate what you mean?

Thank you
Title: Re: VCE English Question Thread
Post by: whys on April 26, 2020, 05:09:40 pm
Hey Bri MT,

I find I have the same problem, can you please elaborate what you mean?

Thank you
I'm not Bri MT, but a little discussion with my eng lang friends gave me this information:
- the use of honorifics is formal
- agentless/agented passive
For example: I ate the chocolates.
Agented passive: The chocolates were eaten by me.
Agentless passive: The chocolates were eaten.
This makes it more objective (the sentence structure is object, verb then subject/no subject instead of the subject first)
This is probably something you don't need to know for mainstream, but it's good to know :)

Other examples of formality (from uncle Google):
- third person
- lack of slang/colloquial language
- no contractions (i.e. do not)
- avoids abbreviating words/phrases
- avoids emotive punctuation

Btw still confused with the detail we need when analysing tone - if anyone has any info feel free to share :P
Title: Re: VCE English Question Thread
Post by: Bri MT on April 27, 2020, 12:12:36 pm
Hey Bri MT,

I find I have the same problem, can you please elaborate what you mean?

Thank you

Whys has some great things to look out for in their post so definitely recommend you read that. Some of what I'm going to go into here is too detailed for you to talk about well in VCE English but may help to inform your ability to detect and construct formality.

Let's take for example these:

1. "The green sea turtle chelonia mydas is a national icon and efforts to preserve this species must be undertaken by parliament. "

2. "Govt must SAVE the green sea turtle NOW! It's a national icon."

1 is more formal than 2 and there are a few features we could pull apart to look at that.

- 1 uses compound complex sentence structure whereas 2 uses simple sentences.
- 2 uses abbreviations ("Govt")
- 2 breaks standard English conventions on capitalisation
- In highly formal writing "!" won't show up much, if at all
- Very formal language tends to be less overtly emotive
- 1 uses more formal language
- As whys mentioned, passive sentences are more formal "Green sea turtle must be saved by parliament" is passive, "Parliament must save the green sea turtle" is active. My physics teacher used to say that to check if something is agentless passive, see if you can add "by zombies" onto the end. For example, "the chocolates were eaten by zombies" works as does "Green sea turtles must be saved by zombies" so that tells us that "the chocolates were eaten" and "green sea turtles must be saved" are both agentless passive.


Note: It's important to recognise that informal to formal is a spectrum with a bunch of elements feeding into it, so it's important to consider a range of elements when deciding how formal something is. Don't for example, just rule it out because it has a feature that's more informal than formal o vice versa - consider it holistically.

My advice would be to go for the most stand-out features first. For example, if I was reading "The purchaser must henceforth remove any materials comprising, derived from, or relating to the purchase and cannot reinstall any component thereof subject to the agreement (XI(iv))" I wouldn't use that "cannot" was written rather than "can not" as evidence of formality and would use stronger evidence instead.

Re: Tone thought I'd add in this piece here
Formal language might be used to add to a sense of authority or credibility, informal language might be used to create a sense of friendliness or humour. There's also many more ways formal and informal language can be used so don't think you're restricted to what I've mentioned here. Imo it's more important to "feel" to the tone and understand how that's constructed, why etc. rather than focusing on specific tone lists. That being said, tone lists can be a good resource if you struggle to intuitively feel tone or to find words that specifically and accurately describe it.


Again, I'm not overly familiar with VCE English assessment so take what I say with a grain of salt and pick out the bits useful to you :)
Title: Re: VCE English Question Thread
Post by: J_Rho on April 27, 2020, 01:10:41 pm
My past English teacher told me I should be writing 3 essays a fortnight for the rest of the year (1Text response, 1 Lang Alanysis and 1 Comparative), but my class won't cover comparative until roughly mid/end of next term. Can I work ahead and start writing comparative essays? How would you guys recommend developing ideas outside of class because obviously a lot of idea and theme discussion is done in class time
Title: Re: VCE English Question Thread
Post by: rani_b on April 30, 2020, 10:40:06 pm
Btw still confused with the detail we need when analysing tone - if anyone has any info feel free to share :P

An argument analysis piece without analysing tone may be good. An argument analysis piece that describes the tone is better. An argument analysis that picks up on shifts in tone and why these tones are being used at certain points is excellent.

I would always recommend slotting in a simple tone descriptor in the introduction:
An example from my analysis on the 2012 VCAA article: In a speech to teachers, librarians and senior students, Mrs Elliot enthusiastically extols the benefits of e-books...Elliot’s sincere tone coupled with her personal experience as a retired teacher-librarian encourages her audience to...

Or: "In response, Dr Peter Laikis’ letter to the editor (titled “Off the planet”) scathingly repudiates Yergon’s advocacy, labelling it as a misguided endeavour and a waste of resources."

This is an easy way to identify tone and tick that box.

The next step is to identify a shift in tone. Writers often do this to signal a change in argument.

Here's an example (in response to the 2018 VCAA article). My first paragraph opened with "Jenkins opens his review by lauding the strong community atmosphere in Benmore..."

My second paragraph opened with "Jenkins’ tone shifts from admiring to caustic as he goes on to censure Calmer Coffee..." In this case, his whole tone had changed to rather critical when it had previously been warm, and this matches his argument about an apparently terrible coffee shop.

Finally, don't forget to discuss the impact of this tonal change.

Example (in response to 2010 VCAA article): "But this tone modulates from formal to disapproving in the series of rhetorical questions that follow - "quote...." Here, Lee elicits guilt in audience members, a feeling strengthened by..."
Title: Re: VCE English Question Thread
Post by: whys on May 01, 2020, 07:47:11 am
snip

Wow, thank you so much!!!!! :D
Title: Re: VCE English Question Thread
Post by: RescueToasts on May 12, 2020, 10:28:52 am
I'm stuck on my English remote essay, the topic sentence is
“He was waiting for the rage to fill him that would be equal at last to the outrage he was committing” Malouf warns against seeking revenge to overcome grief. Discuss.

Quite stuck on the first body paragraph, anything you guys have to say??
Title: Re: VCE English Question Thread
Post by: s110820 on May 12, 2020, 10:42:16 am
Hi RescueToasts,

I can definitely help you if you would like! Can you let me know what your text and introduction are, please?

Kind regards,

Darcy Dillon.
Title: Re: VCE English Question Thread
Post by: sarah15 on May 12, 2020, 06:29:06 pm
Hello! Any tips on how to show rather than tell for a creative response?
Title: Re: VCE English Question Thread
Post by: J_Rho on May 12, 2020, 06:46:17 pm
Hello! Any tips on how to show rather than tell for a creative response?

Hi Sarah!
One thing I was told was don't tell the audience your character is excited/angry/sad, describe the behaviours and/or thoughts of an excited/angry/sad person. Same goes for things like describing the setting, don't say the grass was green and it was a nice summer day, explain the grass and the day through the senses. Don't just state what you want the audience to know, describe it with imagery and symbolism! Here are some good (and more specific example than the ones i just gave)

Lisa's Study Guides: How to achieve an A+ in creative
Example 1
Tell: Katie was very happy.
Show: Katie’s face lifted. Little wrinkles appeared around her bright eyes, her dimples made an appearance that dug into her cheeks as a big grin emerged to show her perfect teeth.

Example 2
Tell: She felt horrible for the weeping children.
Show: Guilt throbbed inside her as she stared at the weeping children. Her heart pounded against her chest, her hands trembling beside her still body, her brain screaming at her to do something.

Example 3
Tell: I was scared.
Show: I hear my breathing; heavy, and rapid. I shut my eyes tightly. I can feel goosebumps running up my arms and down my back.

To test whether or not you are ‘telling’ instead of ‘showing’, think about whether or not your sentence leaves room for questions. In Example 1, ‘Katie was very happy’ would leave the reader thinking – what thought or action showed that she was happy? Whereas ‘show’ demonstrated that she was happy without directly stating it.

The key is to go into the finer details!
Title: Re: VCE English Question Thread
Post by: s110820 on May 18, 2020, 10:11:53 am
Hi :)

I was just wondering if anyone has any advice about how I can memorise two sets of Shakespeare quotes (Hamlet and King Lear) for my analytical essay external exam? If you could help, it would be extremely appreciated. Yes, I'm a QCE student but I tried to ask on the QCE English/Literature forums to no avail :(

I'm also just kinda worried that I may "mix up" the quotations as the two texts that I am studying for English and Literature are extremely similar in the themes and main ideas. Do you guys have any advice of how I can avoid this?

Thank you so much and kind regards,

Darcy Dillon.
Title: Re: VCE English Question Thread
Post by: wsdm on June 17, 2020, 08:41:54 pm
How would you signpost topic sentences in a text response essay without going off-topic or without sounding mechanical (doesn't flow well)?
Title: Re: VCE English Question Thread
Post by: Coolgalbornin03Lo on July 02, 2020, 09:27:31 am
I was looking in the essay marking thread to practise my skills by mentally “marking” some other essays and checking against other AN users feedback to that person.

What exactly is close analysis? Is it just in depth analysing?
Title: Re: VCE English Question Thread
Post by: jborn007 on July 02, 2020, 10:52:26 am
How would you signpost topic sentences in a text response essay without going off-topic or without sounding mechanical (doesn't flow well)?
tbh signposting would be useful for argument analysis and it's not necessary for text response.
Title: Re: VCE English Question Thread
Post by: jborn007 on July 02, 2020, 10:59:10 am
I was looking in the essay marking thread to practise my skills by mentally “marking” some other essays and checking against other AN users feedback to that person.

What exactly is close analysis? Is it just in depth analysing?
fyi close analysis is not the same as argument analysis. I believe to the best of my knowledge that close analysis is one of the two essay students write in the VCAA exam for LITERATURE, not English, and it's primary goal is to elaborate on the writer's language and it creates meaning. If you're originally referring to argument analysis, it's primary goal is to analyse the various techniques the writer intended to deploy and the intended response the writer seeks to evoke from a specifically targeted audience. It's one of the three essays students write in the VCAA exam for ENGLISH
Title: Re: VCE English Question Thread
Post by: Coolgalbornin03Lo on July 02, 2020, 02:29:52 pm
fyi close analysis is not the same as argument analysis. I believe to the best of my knowledge that close analysis is one of the two essay students write in the VCAA exam for LITERATURE, not English, and it's primary goal is to elaborate on the writer's language and it creates meaning. If you're originally referring to argument analysis, it's primary goal is to analyse the various techniques the writer intended to deploy and the intended response the writer seeks to evoke from a specifically targeted audience. It's one of the three essays students write in the VCAA exam for ENGLISH

Sorry I don’t think I phrased it right. I meant people that were marking the language analysis said this is great close analysis and what VCAA markers look for. I’m just wondering what IS close analysis, if you were to implement it in a language analysis. I know what LA is I’m a year 12, that came off a little rude.....
Title: Re: VCE English Question Thread
Post by: 1729 on July 02, 2020, 11:08:06 pm
How would you signpost topic sentences in a text response essay without going off-topic or without sounding mechanical (doesn't flow well)?
I honestly only have a vague idea of what a signpost is so I'm sorry if this is wrong. If the paragraph is in the middle you can kind of reference an idea from the previous paragraph and link it to your next paragraph so you can start talking about your next point. I'd start your signpost with connecting words (eg. furthermore, moreover, similar). Usually though, my topic sentence kind of is my signpost due to the word limit. In that case you might want to review the last sentence of the previous paragraph and tweak it in a way so that the first sentence of your next paragraph flows better. In my opinion, using similar or the same words from your thesis helps reinforce the transitions as they're ultimately arguing the same major point. Just make sure you don't introduce it like "the next point is" or "the purpose is" because those aren't subtle enough and may sound clunky. I also found This. If you scroll down a bit it talks about signposts and how to use them with topic sentences.

Title: Re: VCE English Question Thread
Post by: jkfleur on July 08, 2020, 09:01:17 am
hi, is anyone studying wordsworth poetry for section A of the exam this year?
other than the ten or so poems my school has prescribed for study, is it worth it to be familiar with all the poems in the collection (selected by Seamus Heaney)? many of the poems are a few pages long so I'm finding it daunting to make myself look at them..

Title: Re: VCE English Question Thread
Post by: jborn007 on July 08, 2020, 07:50:40 pm
hi, is anyone studying wordsworth poetry for section A of the exam this year?
other than the ten or so poems my school has prescribed for study, is it worth it to be familiar with all the poems in the collection (selected by Seamus Heaney)? many of the poems are a few pages long so I'm finding it daunting to make myself look at them..


nah, quality > quantity
Title: Re: VCE English Question Thread
Post by: uwuuu on July 09, 2020, 11:52:08 am
Hi I was wondering if regards to the end of year english exam, are we expected to write 4 body paragraphs for both the comparative and analytical? Will we lose marks if we write 3 lengthy body paragraphs instead?
Title: Re: VCE English Question Thread
Post by: darkz on July 09, 2020, 01:21:36 pm
Hi I was wondering if regards to the end of year english exam, are we expected to write 4 body paragraphs for both the comparative and analytical? Will we lose marks if we write 3 lengthy body paragraphs instead?

For text response/ comparative, the normal length is three paragraphs. In terms of language analysis, structure depends on how you determine your arguments. e.g. you could have seven small paragraphs or just three chunky paragraphs - structure is up to you.
Title: Re: VCE English Question Thread
Post by: Sine on July 09, 2020, 01:34:15 pm
Hi I was wondering if regards to the end of year english exam, are we expected to write 4 body paragraphs for both the comparative and analytical? Will we lose marks if we write 3 lengthy body paragraphs instead?
Nah 3-4 is fine and perfectly normal.  Although it isn't marked directly I think you start to fall into a trap if you go to 2 paragraphs since assessors will say your analysis is too narrow. Going with too many paragraphs will give the impression that your analysis is only superficial.
Title: Re: VCE English Question Thread
Post by: Coolgalbornin03Lo on July 09, 2020, 07:09:16 pm
How to overcome a bias teacher who hates you? She’s constantly marking me down and there’s nothing I can do about it. She doesn’t help me if I ask but she’ll help other people and hand back their work even people who hand it in late! I get my work back less than 12 hours before the SAC and it’s so vague. Should I get a tutor for exams? I can’t dob in on this teacher because I need to get good unit 4 marks and making her hate me more would just screw it up. I was aiming for above 40 study score but it’s not looking possible if my teacher refuses to cooperate.
Title: Re: VCE English Question Thread
Post by: The Cat In The Hat on July 10, 2020, 11:08:39 am
How to overcome a bias teacher who hates you? She’s constantly marking me down and there’s nothing I can do about it. She doesn’t help me if I ask but she’ll help other people and hand back their work even people who hand it in late! I get my work back less than 12 hours before the SAC and it’s so vague. Should I get a tutor for exams? I can’t dob in on this teacher because I need to get good unit 4 marks and making her hate me more would just screw it up. I was aiming for above 40 study score but it’s not looking possible if my teacher refuses to cooperate.
My easy answer is if she helps other people, get them to hand in your work and get feedback, if it's not for an assessment. She should be fair.
I think you should talk to another teacher or something - this is definitely not fair in any way, shape or form (assuming you're reporting it how it is, of course :) no offense)
If the teacher isn't fair someone needs to know about it. You could try talking to the teacher too, maybe - perhaps it's just misunderstanding or something? Say what your goal is and try to be friendly and non-confrontational.
Spoiler
I don't know what to do really... never been in situation myself... though I had a teacher I didn't like and the feeling was thoroughly mutual but still marked fair (though there was bias in class as to who she'd help etc.) And then I didn't do very well because I went, oh well, I hate this subj. now coz teacher and useless work to do; didn't turn out well.
Title: Re: VCE English Question Thread
Post by: chantal.duffy on July 13, 2020, 06:16:34 pm
Hello! Hope everyone is well. I'm from Qld and this is our first year of ATAR so we have zero past papers to refer to. The book I'm studying for English is The Poisonwood Bible by Barbara Kingsolver and my teacher has told us that HSC & VCE students study it. I've tried really hard by combing back through lots of practice papers to find some questions to help my revision and I've had no luck in finding any reference to the book. Could anyone possibly direct me to where some past papers are on the Poisonwood Bible for VCE? It's quite possible that the syllabus has been changed or something, or potentially I misheard my teacher when she said VCE. It could also be in another one of the kinds of English subjects, so if you know which one, please let me know.
Any help would be much, much appreciated! Thank you :)
Title: Re: VCE English Question Thread
Post by: The Cat In The Hat on July 13, 2020, 06:36:47 pm
I'm no authority, but I don't remember seeing that name on the list when I looked through it for English to see what our teacher was picking from. I don't remember ever seeing that name before and I probably would remember it, but again, I don't know because I looked before the start of this year.
Having looked at the book list I cannot see it on there - for VCE English, mind. I don't know about others.
You can check the list of all the books for other subjects - I found it by searching 'vce english texts'. It was the first return.
Hope that helps!
Title: Re: VCE English Question Thread
Post by: chantal.duffy on July 13, 2020, 07:53:58 pm
I'm no authority, but I don't remember seeing that name on the list when I looked through it for English to see what our teacher was picking from. I don't remember ever seeing that name before and I probably would remember it, but again, I don't know because I looked before the start of this year.
Having looked at the book list I cannot see it on there - for VCE English, mind. I don't know about others.
You can check the list of all the books for other subjects - I found it by searching 'vce english texts'. It was the first return.
Hope that helps!

OK, Thanks for letting me know! Really appreciate it. Best of luck with everything :)
Title: Re: VCE English Question Thread
Post by: vchs on July 20, 2020, 04:11:51 pm
Does anyone have the PDF of The Dressmaker and LitCharts PDF of the Dressmaker?
Title: Re: VCE English Question Thread
Post by: ArtyDreams on July 20, 2020, 09:29:49 pm
Just a quick question: do we need to analyse every single quote we use in a text response essay? Or are we allowed to add quotes to 'fill' sentences (if that makes sense, I'll leave an example down below.)

Is just that my teachers always tell me that I need to use more quotes, yet I don't think its possible for me to analyse every quote in detail?

Using it as a filler: Hecuba remains optimistic that Astyanax will grow up up to be the saviour of Troy, thus instilling hope in herself and Andromache
OR Astyanax had to ‘give up [his] life,’ due to his ‘fathers courage,’ to prevent revenges occurring in the future.

Analysing it: Andromache represents the Athenian vision of a ‘perfect wife,’ who never ‘wanted to leave [Hector’s] home.’ By referring to their shared house as Hector’s, it is evident that men withhold all tangible properties, and women have no value for themselves.

Thank you!
Title: Re: VCE English Question Thread
Post by: Owlbird83 on July 20, 2020, 10:00:45 pm
Just a quick question: do we need to analyse every single quote we use in a text response essay? Or are we allowed to add quotes to 'fill' sentences (if that makes sense, I'll leave an example down below.)

Is just that my teachers always tell me that I need to use more quotes, yet I don't think its possible for me to analyse every quote in detail?

Using it as a filler: Hecuba remains optimistic that Astyanax will grow up up to be the saviour of Troy, thus instilling hope in herself and Andromache
OR Astyanax had to ‘give up [his] life,’ due to his ‘fathers courage,’ to prevent revenges occurring in the future.

Analysing it: Andromache represents the Athenian vision of a ‘perfect wife,’ who never ‘wanted to leave [Hector’s] home.’ By referring to their shared house as Hector’s, it is evident that men withhold all tangible properties, and women have no value for themselves.

Thank you!

I'm pretty sure it's fine to use the drop in quotes without analysing them, I did it often. As long as you have enough quotes that are being analysed it's okay.
Title: Re: VCE English Question Thread
Post by: The Cat In The Hat on July 21, 2020, 02:47:04 pm
I'm pretty sure it's fine to use the drop in quotes without analysing them, I did it often. As long as you have enough quotes that are being analysed it's okay.
Also. I also. But I don't get very good marks, so I don't know.
Title: Re: VCE English Question Thread
Post by: Coolgalbornin03Lo on July 29, 2020, 06:54:31 pm
How do you use nominalisation and incorporate it into text response essays? Is it also necessary for comparitive?

Also in text response what is authorial intent? Is it like how the audience intends to position (although never use this word) the audience in language analysis?
Title: Re: VCE English Question Thread
Post by: whys on July 29, 2020, 07:14:59 pm
How do you use nominalisation and incorporate it into text response essays? Is it also necessary for comparitive?

Also in text response what is authorial intent? Is it like how the audience intends to position (although never use this word) the audience in language analysis?
1. It's not exactly necessary, but it's good to use if you find yourself lapsing into summative-style writing. You basically turn the verb into a noun. Example:
Atwood portrays Penelope as a principally flawed character who is a product of the patriarchy around her.
Atwood's portrayal of Penelope as a principally flawed character who is a product of the patriarchy around her suggests...
It basically forces you to analyse.

2. Authorial intent is the views and values the writer is trying to communicate through certain things. It's basically their overall, broader comment on life. Drawing upon my previous example, Atwood's portrayal of Penelope as "" suggests that women often fall victim to societal pressures, forcing them to take extreme measures to ensure their survival in a misogynistic world radically opposed to their existence. Authorial intent is the broader statement they are making through certain characters and themes, in essence.

Hope this helped!
(I used The Penelopiad to draw from, as that's one of the texts I'm studying. The same stuff applies to all texts though. :P)
Title: Re: VCE English Question Thread
Post by: Coolgalbornin03Lo on August 12, 2020, 10:43:37 pm
I actually can’t write a text response on a film (rear window) and with exams fast approaching idk what to do. Should I try thematic paragraphs? Like instead of writing to a prompt write big paragraphs of each them with lots of quotes and analyse. I just don’t understand authorial intent or anything 😞 I’m writing an essay online for my teacher Rn but it’s just so bad I can attach it if you’d like.

Spoiler
Prompt: the appearance between appearance and reality is central to the film

The views Hitchcock encourages the audience to form are based on the character's appearance. The male gaze through which Rear Window is delivered facilitates this as Jeff's interpretation of characters becomes the audiences.  Lisa introduces herself while turning on lights as “Lisa.Carol.Fremont”. The dull lighting coupled with dialogue suggests that Lisa is portraying the best version of herself by illuminating herself, welcoming any onlookers. Hitchcock is highlighting the idea that females at the time were seen as objects but wanted to take control of exactly how they were seen. Hitchcock continued to lead the audience to make conclusions based on what they saw through the characterisation of Miss Torso and Miss “Lonelyhearts”. Many of the “bathing beauties” which Jefferies saw through his window were given qualities solely based on what Jefferies saw of them. Miss Lonelyhearts' introduction was accompanied by sombre music and Stella wondering if “anyone in the neighbourhood who'd” look “in her direction”. This accompanying non diegetic sound encouraged the audience to view her loneliness as misfortune. Hitchcock draws light on the fact that in a post cold war society it was seen as “abnormal” not to have company. Hitchcock has Jefferies apartment looking down on Miss Lonelyhearts signifying that Jefferies was above Miss Lonelyhearts and dissimilar therefore dissimilar to her. By leading the audience to take what is presented to them at face value Hitchcock intends for the audience to see the stark contrast with reality.

[/spoiler= first paragraph ]
Title: Re: VCE English Question Thread
Post by: The Cat In The Hat on August 17, 2020, 10:16:33 pm
Hi, In English we are currently studying the play Medea and I was wondering if anyone had any useful resources that I could use for an essay that I need to write. In particular around the topic of Jason's betrayal and how this contributes towards the tragedy of Medea? Any help would be greatly appreciated.
Hi,
Have you looked at the Free Notes section of AN? You could search it up with Medea as the keyword - in fact, having done such a search, without specifying anything else either, I come up with five results. They may help, they may not - I'm not doing Medea so I haven't looked at them.
Title: Re: VCE English Question Thread
Post by: amanaazim on September 06, 2020, 01:07:28 pm
what is the difference between presenting argument analysis essay and a language analysis argument. how do they differ in terms of structure.
Title: Re: VCE English Question Thread
Post by: ArtyDreams on September 06, 2020, 11:01:31 pm
Hi AN! Does anyone have any tips on choosing body paragraph ideas for comparative prompts (or text response). I feel like I always break up my paragraphs in strange ways, that end up being too complicated and messy.

Also maybe like a formula to decide body paragraphs? (As an maths/science person I really struggle with the freedom of deciding body paragraphs.)

For example, I chose these body paragraphs for this prompt: 'How does Ransom and The Queen show that leaders must change with changing times?' These were my paragraphs, but I always feel that these don't have enough depth.
1. New and contrasting relationships were formed in both texts, allowing leaders and outsiders to grow from each other, eventually causing changes in the leader’s personal beliefs.
2. The ability of a leader to step outside traditional roles in order to face adversity drives the plotline for both narratives, as changing times call for new ways to act.
3. Furthermore, as plotlines of both works drive forward, it becomes evident that the ability of leaders to change themselves leaves a positive impact on others

Am I being too plot based with my paragraph division? How can I improve this?

Thank you so much  :D Would really appreciate any help.
Title: Re: VCE English Question Thread
Post by: blueycan on September 07, 2020, 12:23:47 am
//

hey artydreams! so when it comes to my essay writing, i tend to list off ideas on how i can best support my contention in a way takes more than surface-level plot info into consideration. in the examples i've provided below, where the prompt discusses power, structures, oppression, marginalisation, i took those key ideas and pulled out examples from the text i could work with (prejudice, patriarchy, conforming)
when working with topic sentences i find that overarching themes and ideas are easier to discuss than plot-based or character-based topic sentences, as i am able to draw from many areas of the text that don't necessarily need to be in chronological order. i feel like very specific topic sentences often leave no wiggle room for broader analysis or interpretation  (in my pg my topic sentence revolves around prejudice > i've explored the contrasting explorations of prejudice in both texts, one text exploring racial prejudice and the other exploring religious prejudice)

as for a methodical approach to topic sentences, finding those key words in your contention and linking them to key ideas explored within the texts can help you with forming the arguments that are deeper than surface level/plot points

tldr; i'd focus on theme and idea based topic sentences over plot or character based, as it allows you to explore a range of evidence and not necessarily in a way that retells the plot, and gives you more wiggle room to broaden your ideas instead of being restricted to one idea

my example ;

Prompt: Power in society is reinforced through structures designed to oppress marginalised groups. How is this explored in both texts?
Contention: Both ‘To Kill a Mockingbird’ (TKAM) and ‘The Crucible’ are representations of how societies oppress marginalised groups to maintain the places of those in power.

1. The contrasting explorations of prejudice within both texts reveals how biased thought affects one’s beliefs and actions.
2. The patriarchal social environments evident in both ‘To Kill a Mockingbird’ and ‘The Crucible’ depict how societies value men in powerful positions instead of women.
3. Both authors’ exploration of the consequences people suffer for not conforming represent the broader power the majority have in influencing the actions of others.

i'm not sure if this will apply to you really bc i'm in year 11 butttt hopefully this made some kind of sense and helped you in SOME way honestly not sure lmao but wish you the best (:
Title: Re: VCE English Question Thread
Post by: Coolgalbornin03Lo on September 07, 2020, 12:52:33 pm
Hi AN! Does anyone have any tips on choosing body paragraph ideas for comparative prompts (or text response). I feel like I always break up my paragraphs in strange ways, that end up being too complicated and messy.

Also maybe like a formula to decide body paragraphs? (As an maths/science person I really struggle with the freedom of deciding body paragraphs.)

For example, I chose these body paragraphs for this prompt: 'How does Ransom and The Queen show that leaders must change with changing times?' These were my paragraphs, but I always feel that these don't have enough depth.
1. New and contrasting relationships were formed in both texts, allowing leaders and outsiders to grow from each other, eventually causing changes in the leader’s personal beliefs.
2. The ability of a leader to step outside traditional roles in order to face adversity drives the plotline for both narratives, as changing times call for new ways to act.
3. Furthermore, as plotlines of both works drive forward, it becomes evident that the ability of leaders to change themselves leaves a positive impact on others

Am I being too plot based with my paragraph division? How can I improve this?

Thank you so much  :D Would really appreciate any help.

I did ransom and invictus last year year 11. For arguments I usually think of ideas in both texts which are similar and then make them a general thing.

1. In both Ransom and Invictus leaders made familial sacrifice in order to lead their country in the right direction.

I know this isn’t relevant to the queen btw!!! But Changing times and familial sacrifice- they must sacrifice their family in order to properly fufill this role of leaders

2. As the story progresses both Mandela and Priam must compromise their personal beliefs in order to......

In ransom (don’t quote me this was a year ago). Prism had to change his ideals to fit with the Trojans or something? He had to alter what him and his family firmly believed to be correct in order to continue.

Haha please take this advice with a grain of salt I’m a year 12. But I think of arguments as points underneath the contention- stuff which backs it up.

In maths Methods terms I guess the arguments would be a response to a show that question? Prove that does Ransom and The Queen show that leaders must change with changing times?'
Title: Re: VCE English Question Thread
Post by: ArtyDreams on September 07, 2020, 09:42:04 pm
Thank you both so much! This really helps  :D
Title: Re: VCE English Question Thread
Post by: TheEagle on September 08, 2020, 01:01:48 am
Now that there's only ~60 days until the English exam, I am starting to worry. Does anyone have any tips to prepare for the whole exam though I still have a comparative sac to do in a few weeks time. How do I be all round prepared for all 3 sections? I am really worried
Title: Re: VCE English Question Thread
Post by: blueycan on September 08, 2020, 06:08:48 pm
//

hi! it might be worth posting a new thread in the VCE English and EAL board with your questions. more people will be able to see your questions and respond, and that way, others can view the thread in the future and be able to learn from the advice as well.
i found this AN article that may help you as well
best of luck !! (:
Title: Re: VCE English Question Thread
Post by: MEH0010 on September 09, 2020, 05:22:19 pm
Hi.
I've just got a quick question.
For example, with this prompt: Compare how Tracks and Charlie's Country expose the limitations of isolation and the lack of human contact. Can i mostly disagree with the prompt and talk two paragraphs on the advantages and one paragraph on the limitations.
Thank you  :)
Title: Re: VCE English Question Thread
Post by: The Cat In The Hat on September 09, 2020, 05:30:29 pm
I don't think so? Maybe it could work if you split it 50-50, but I don't think doing it the way you're saying really answers the question. But you should get some more knowledgeable advice, I don't know.
Title: Re: VCE English Question Thread
Post by: Jm179 on September 09, 2020, 09:35:07 pm
Hi,
I am trying to write a text response essay about Medea(The Play) on the prompt 'Jason as the real Criminal' I was hoping someone could shed some light and give me some ideas that I could write for my body paragraphs.

Any help would greatly be appreciated!?
Title: Re: VCE English Question Thread
Post by: uwuuu on September 12, 2020, 07:16:03 pm
Does anyone have any exempliar essays of The Dressmaker/The Crucible? It would be very helpful to see a high level essay I can look up work towards :)
Title: Re: VCE English Question Thread
Post by: vchs on September 13, 2020, 12:50:35 pm
Does anyone have quotes (themed-sorted if possible) for The Crucible and The Dressmaker?
Title: Re: VCE English Question Thread
Post by: Coolgalbornin03Lo on September 13, 2020, 05:15:11 pm
In comparative are we comparing concepts or authorial intent? I’m confused?

What separates a good from a great comparative in terms of analysis not language used?
Title: Re: VCE English Question Thread
Post by: Coolgalbornin03Lo on September 14, 2020, 03:27:03 pm
For a comparative when relating the argument back to contention at end of the paragraph what exactly do you say in the sentence?
Title: Re: VCE English Question Thread
Post by: whys on September 14, 2020, 03:38:20 pm
For a comparative when relating the argument back to contention at end of the paragraph what exactly do you say in the sentence?
Your concluding statement should be a discovery. What have you learnt from what you've discussed in that paragraph? Don't make it a simple repetition of the topic sentence - the assessors already know that from the first sentence of your paragraph so repeating your topic sentence with slightly different wording won't help advance your essay. Doing that can make your writing and essay feel quite repetitive (I'm not saying you're doing this, I'm just saying to avoid this!) and circular, which is what you DON'T want. What you DO want is to conclude what the overall conclusion is from your paragraph. The best way to do this is to link it back to authorial intent - what are the authors trying to say about what you've talked about in your paragraph? This will make the assessor feel like they're going through a well-structured essay that is able to make relevant and apt links at the end of each paragraph that talks about what you've found after your discussion. For example, a paragraph could be talking about how characters take control of their own narratives and what this power entails. Your last sentence could say something like 'Both writers use storytelling as a means to liberate their characters from their one-dimensional portrays in history.' See how it's not a repetition of the topic sentence? Rather, it talks about the power of taking control of the narrative and what this means for the characters. Alternatively, you can make a sweeping views and values statement, which is a bit more authorial agenda-y.

Hope this helped!

EDIT: the example I provided was random and generic, obviously in an actual essay you might want to make it more specific to the texts/characters/themes that you've specifically discussed!
Title: Re: VCE English Question Thread
Post by: Coolgalbornin03Lo on September 14, 2020, 06:15:54 pm

The best way to do this is to link it back to authorial intent - what are the authors trying to say about what you've talked about in your paragraph? This will make the assessor feel like they're going through a well-structured essay that is able to make relevant and apt links at the end of each paragraph that talks about what you've found after your discussion.


What if I’ve already been doing throughout the essay? Is that bad structure?
Title: Re: VCE English Question Thread
Post by: Annon4589 on September 21, 2020, 03:56:51 pm
Hey guys,

Since VCAA has stipulated that for section C we do not need to write comparatively, if multiple texts are given, does that mean we can focus on each text individually and still receive a 10, even though this approach may lack sophistication when compared to other students who compare effectively?

Much appreciated
Title: Re: VCE English Question Thread
Post by: The Cat In The Hat on September 21, 2020, 05:24:18 pm
Hey guys,

Since VCAA has stipulated that for section C we do not need to write comparatively, if multiple texts are given, does that mean we can focus on each text individually and still receive a 10, even though this approach may lack sophistication when compared to other students who compare effectively?

Much appreciated
I believe so, but my personal opinion is that it might be harder to get that 10 if we don't compare. However, I'm not sure; don't take my word for it :)
Title: Re: VCE English Question Thread
Post by: rukayabal on September 26, 2020, 10:11:06 am
Hi all,
If I choose a quote prompt, and I forget to integrate one of the quotes in the essay, would I lose marks? I mean like a great amount of marks?
Title: Re: VCE English Question Thread
Post by: The Cat In The Hat on September 26, 2020, 03:08:58 pm
Hi all,
If I choose a quote prompt, and I forget to integrate one of the quotes in the essay, would I lose marks? I mean like a great amount of marks?
To add to that question- are we meant to integrate quotes from such a prompt into the essay? In my trial exam I didn't...
Title: Re: VCE English Question Thread
Post by: whys on September 26, 2020, 04:17:22 pm
Hi all,
If I choose a quote prompt, and I forget to integrate one of the quotes in the essay, would I lose marks? I mean like a great amount of marks?
Not sure if you'd lose a 'great amount' of marks, but I'm certain you'd lose at least 1-2. Not integrating the quote could show the assessor that you don't fully understand the extent of the prompt (even if it was just a simple mistake of forgetting!), as the quotes often encompass some sort of answer to the prompt.

To add to that question- are we meant to integrate quotes from such a prompt into the essay? In my trial exam I didn't...
Yes, that is the purpose of including quotes as part of a prompt. It's okay, your trial exam is only for learning purposes so best that you forgot it now rather than in the real exam!
Title: Re: VCE English Question Thread
Post by: Coolgalbornin03Lo on September 26, 2020, 05:02:12 pm
How many trial exams is everyone’s school holding for English? We only have one and it’s the first week back. What’s scary is our schools SACs have never been one hour in length so what’s gonna happen to the kids who haven’t written since June? I think it’s a little mean.....not to mention teachers aren’t taking work this holidays!!!!

How has anyone here overcome teachers not reading work but still wanting to prepare for the exam?

They said we can prepare for exam next term but we go back to school 4 weeks before the exam and I have other  subjects. I know I’m being unreasonable but I think it would be nice for those who would like feedback now to be able to access a teacher :(
Title: Re: VCE English Question Thread
Post by: The Cat In The Hat on September 26, 2020, 06:20:21 pm
How many trial exams is everyone’s school holding for English? We only have one and it’s the first week back. What’s scary is our schools SACs have never been one hour in length so what’s gonna happen to the kids who haven’t written since June? I think it’s a little mean.....not to mention teachers aren’t taking work this holidays!!!!

How has anyone here overcome teachers not reading work but still wanting to prepare for the exam?

They said we can prepare for exam next term but we go back to school 4 weeks before the exam and I have other  subjects. I know I’m being unreasonable but I think it would be nice for those who would like feedback now to be able to access a teacher :(
My school's only holding one; I'm also doing ones of my own with my study group. You could try putting your work up on the work submissions and marking board here on AN? People would probably look at it, even though it's not the ideal situation, obviously.

Personally, I overcome it by simply writing stuff that just isn't marked. Not as good but still...
Title: Re: VCE English Question Thread
Post by: Chocolatemilkshake on September 26, 2020, 06:25:31 pm
How has anyone here overcome teachers not reading work but still wanting to prepare for the exam?
They said we can prepare for exam next term but we go back to school 4 weeks before the exam and I have other  subjects. I know I’m being unreasonable but I think it would be nice for those who would like feedback now to be able to access a teacher :(
Hi Elle just wanted to say I understand how you feel my teacher is not AT ALL prompt with returning essay feedback  :'(. I'd encourage you to reach out to previous teachers you've had in english (if you've had a good relationship with them) if that's allowed at your school. You can also exchange work with friends (I have an english "buddy" I have worked with this year and we exchange essays all the time which is beneficial for both of us and a fantastic way to get new ideas and perspectives on the texts. I'd encourage you to find someone at a similar level, it's amazing how helpful this is). You could also reach out to any past students (if you know any) who are good at english and are happy to mark some work. And there are ANers who are always happy to help too.

Overall, you can definitely still do it! Just try to keep up the motivation and maybe ease into study (making mind maps, light rereading/memorising quotes, writing plans, etc) before you go into writing full essays under strict time, hopefully this will increase your motivation and confidence as the exam nears (and be less daunting). And yes you have other subjects but english is a primary four so it's important that you prioritise it equally (I know how hard this is as a student who would pick 5 science exams over an english essay any day of the week aha).

How many trial exams is everyone’s school holding for English? We only have one and it’s the first week back. What’s scary is our schools SACs have never been one hour in length so what’s gonna happen to the kids who haven’t written since June? I think it’s a little mean.....not to mention teachers aren’t taking work this holidays!!!!
I have one trial exam but I've already done it (we did it at home). Also don't be too worried about not having written a SAC under full time, as you practice and do your trial exam this won't disadvantage you. As long as you continue to work on timing over the next month and a half!
Good luck you've got this!!  8)
Title: Re: VCE English Question Thread
Post by: a weaponized ikea chair on September 29, 2020, 08:20:43 am
What are VCAA rules on using archaic words? I could not really find anything on it.

I sometimes use words such as therein, whereunto, forbye.
Title: Re: VCE English Question Thread
Post by: The Cat In The Hat on September 29, 2020, 09:00:42 am
What are VCAA rules on using archaic words? I could not really find anything on it.

I sometimes use words such as therein, whereunto, forbye.
Dunno, but I use 'em too - I never heard a rule against it, if used correctly. ???
Title: Re: VCE English Question Thread
Post by: angrybiscuit on September 29, 2020, 10:41:48 am
What are VCAA rules on using quotes with vulgar words? In a context where removing it is difficult? Should I avoid completely or use square brackets to alter it?
Title: Re: VCE English Question Thread
Post by: The Cat In The Hat on September 29, 2020, 11:04:38 am
What are VCAA rules on using quotes with vulgar words? In a context where removing it is difficult? Should I avoid completely or use square brackets to alter it?
It possibly depends on what the words are and what context they are in, but I'd advise the use of square brackets just in case.
Title: Re: VCE English Question Thread
Post by: sarah15 on October 04, 2020, 08:34:21 am
hi everyone! there is just over a month till the exam eek!!!
i have spent the holidays getting familiar with all my texts as my teacher advised but i am stressed about the limited time left to write practice essays. can any of you advise how many essays to do a week to get as much practice in as possible w/out getting burn out?? thanks!
Title: Re: VCE English Question Thread
Post by: The Cat In The Hat on October 04, 2020, 08:53:23 am
hi everyone! there is just over a month till the exam eek!!!
i have spent the holidays getting familiar with all my texts as my teacher advised but i am stressed about the limited time left to write practice essays. can any of you advise how many essays to do a week to get as much practice in as possible w/out getting burn out?? thanks!
It probably depends on the person. Perhaps you should try just doing paragraphs etc. as well as full essays, just to mix it up a bit. Also do full exams. At least, I found that a full exam, doing the last essay in it, was quite difficult. It probably also depends on your workload with other subjects. Personally, I'm going to aim to do 1-2 full exams a week (once school finishes for me), and very little else. But that doesn't work for others necessarily.
Title: Re: VCE English Question Thread
Post by: tiredandstressed on October 04, 2020, 12:01:34 pm
hi everyone! there is just over a month till the exam eek!!!
i have spent the holidays getting familiar with all my texts as my teacher advised but i am stressed about the limited time left to write practice essays. can any of you advise how many essays to do a week to get as much practice in as possible w/out getting burn out?? thanks!
Essay plans > Essays
The reason is
1. You will burn yourself out
2. Essay plans are easier to memorise compares to essays
(however, you should write full essays for language analysis)
Try to tackle as many prompts that you can, and try to have an overall general essay plan for each theme
E.g. For Medea (revenge, social hierarchy, gender roles, power, emotions v logic etc.)
This will be more effective than just writing essay
More importantly, try to write timed essays (but you don't need to take out three hour, just try to do 1 essay 1 hour blocks)
Title: Re: VCE English Question Thread
Post by: Geoo on October 04, 2020, 06:38:24 pm
Really dumb question, but I tend to handwrite my Z's in cursive form, which is more like a 3 that I connect with other letters to form words. Do many teachers recognise the cursive z instead of the printed form?
For some reason i'm just a bit nervous to write how I normally do since I rarely see others using the cursive Z instead of the regular text z.

Should I change to this "z", since it's more standard and potentially easier to read?
Title: Re: VCE English Question Thread
Post by: Coolgalbornin03Lo on October 04, 2020, 06:53:39 pm
Hi guys. I was wondering how many pieces of evidence necessary for a comparative body paragraph? My teacher said one from each text and one of them should be a literary device. Is this enough? It’s difficult to fit more as the paragraphs are huge! I wrote my first one today by hand and to time in 70 minutes- I know eons but it was a struggle.
Title: Re: VCE English Question Thread
Post by: tiredandstressed on October 05, 2020, 01:02:55 am
Hi guys. I was wondering how many pieces of evidence necessary for a comparative body paragraph? My teacher said one from each text and one of them should be a literary device. Is this enough? It’s difficult to fit more as the paragraphs are huge! I wrote my first one today by hand and to time in 70 minutes- I know eons but it was a struggle.
I would aim for at least 2-3 quotes per text, or less quotes for a more sophisticated literary deceive.
Examiners don't count your quotes, quality > quantity. 
As long as you show a high level of understanding of the text will insightful explanations your paragraphs will score highly.
I will provide you with some paragraphs that got me a <8/10 in practice essays (marked by my teacher, who was an exam marker)

Spoiler
Analyse how memory is depicted as simultaneously inescapable and unbearable in The Longest Memory and Black Diggers.
However, although memory can be seen as inescapable and unbearable, Wright demonstrates how characters can gain permanently from the pain of memory whereas D’Aguiar suggests this can only be temporary. Wright explores how mutual grief can make the memory of “the worst thing on earth” bearable he demonstrates through the Bloke with a Glass of Wine that “in those hideous moments in hell on Earth… you realise I belong” here Wright reveals that through mateship on the battlefield the Indigenous Australians were accepted, and thus by grieving together with the white Australians they could bear the pain of the war. Wright intends on opening Act Two with the Bloke with a Glass of Wine to illustrate that in the “living hell” of war the next generation of black diggers were no longer  “woebegone [failures]” but rather could share pride with the White Australians “for being an Aussie”, resultantly revealing that trauma “brought [them] together”. Conversely, D’Aguiar demonstrates that through love Chapel could only momentarily endure his experiences of oppression. D’Aguiar highlights Chapel and Lydia’s relationship allowed them to be happy evident when Lydia reveals “that his love for me is such that no one… can stop him doing what is necessary for us to be together” demonstrating Chapel was able to bear the pain whilst being a slave with no ideal future. Despite their passion, D’Aguiar depicts the couple as “two star-crossed lovers” foreshadowing their doom and eventual separation. Here, D’Aguiar reveals although Chapel feels “joy; not fear” he is denied a future, and his relationship with Lydia, his safe haven, was only temporary. Consequently, D’Aguiar illustrates that, unlike The Bloke with a Glass of Wine, Chapel was unable to sustain a future, robbed of his life as “the whip ate into him” his death a result of prejudice, his inability to combat his enslavement. Therefore, although both texts differ in their portrayal of traumatic experiences and their memories, they are united in their assertion that those experiences and memories are fundamental components of our existence. 

Spoiler
Compare the impact of one group of people assuming mastery over another in both text.
However, both texts reveal the importance of an equitable society, Wright demonstrates the impact of equality being long-lasting, whereas D’Aguiar demonstrates the near impossibly of reaching equality. Wright explores how mutual grief can make the memory of “the worst thing on earth” enable for equality to exist he demonstrates through the Bloke with a Glass of Wine that “in those hideous moments in hell on Earth… you realise I belong” here Wright reveals that through mateship on the battlefield the Indigenous Australians were accepted, and thus the possibility for equality and the abandonment of master groups. Additionally, through the implementation of the Correspondence he articulates his resentment of the hypocritical treatment towards the Indigenous Australians which he condemns as a “gross injustice” his use of letters intends to highlight to the audience the existence of the letters suggests the need and potential of the Indigenous Australians having “equal rights with the white community”. Accordingly, Wright establishes the urgency to abolish the concept of master groups and the commencement of an equitable society. Conversely, D’Aguiar demonstrates that fighting for change although necessary, is often difficult to achieve. Comparable to Wright’s letters of the Correspondence, D’Aguiar also employs structural use of The Virginian, that demonstrate near impossibility of reaching equality, as symbolised by the non-chronological order of the editorials. The editor promotes the unequal standards of slaver and slave where he suggests “They are, quite literally, not like us” and that ultimately there will be “no end to slavery”. Here, D’Aguiar exposes the unjust attitudes the white American possess, but also reveals that accomplishment of equality is merely a dream. The editor challenges Lydia’s notion “slavery is dying as an institution” by a blunt comment “She exhibits a love for blacks that clouds her ability to reason” despite the prejudiced claim, in the end Lydia is unable to free Chapel from slavery, consequently D’Aguiar exposes that although the disbandment of master groups is necessary for the improvement of society, the power imbalances remain heavily entrenched. Thus, although both texts differ in their portrayal of societies reaching equality, they are united in their assertion that for humanity to develop to its full potential, equality of all individuals must exist and be protected
From memory thats how I wrote in the exam and I got an A+ so hope that helps!
Title: Re: VCE English Question Thread
Post by: The Cat In The Hat on October 05, 2020, 09:37:41 am
What's the difference - and how can I fix it - between an 8/10 and a 9-10/10 for a comparative? Or a text response, for that matter. Or an AA. What strategies should I use to pick up my marks from all around 8/10 (well, AA 7/10) to 9/10?
Title: Re: VCE English Question Thread
Post by: tiredandstressed on October 05, 2020, 01:09:53 pm
What's the difference - and how can I fix it - between an 8/10 and a 9-10/10 for a comparative? Or a text response, for that matter. Or an AA. What strategies should I use to pick up my marks from all around 8/10 (well, AA 7/10) to 9/10?
This is a very interesting question, but also a challenging one.
You probably have realised your English teacher won't tell you what a 10/10 response is like, because that is difficult to explain. In contrast, to maths and science where its very easy to reward marks, in English it's different. I encourage to read the rubric for each essay here: https://www.vcaa.vic.edu.au/Documents/exams/english/english-crit-descriptors-w.pdf
To score a 9-10 you must: Text response
Section A: Text response
Demonstrates a close and perceptive reading of the text, exploring complexities of its concepts and construction
Demonstrates an understanding of the implications of the topic, using an appropriate strategy for dealing with it, and exploring its complexity from the basis of the text
Develops a cogent, controlled and well-substantiated discussion using precise and expressive language
Section B: Comparative text response
Demonstrates through comparison an insightful grasp of the ideas and issues, and conceptual complexity, of the texts Achieves an assured, cohesively structured piece of writing that presents a sophisticated comparison that explores the implications of the topic
Makes fluent and effective use of language
Section C: Language analysis
Shows a perceptive understanding of the arguments presented and the points of view expressed
Demonstrate sophisticated insight into the ways in which written and visual language is used to
persuade
Uses language fluently and effectively
Title: Re: VCE English Question Thread
Post by: rozmaaate on October 05, 2020, 04:40:23 pm
Hey guys not sure if this questions belongs on this forum but could someone please explain why the correct answer for the MC in last years GAT is “c”?

Thanks in advance
Title: Re: VCE English Question Thread
Post by: a weaponized ikea chair on October 05, 2020, 09:30:26 pm
Hey guys not sure if this questions belongs on this forum but could someone please explain why the correct answer for the MC in last years GAT is “c”?

Thanks in advance

***Not in year 12****

a Doesn't make sense because they are performing the action.
b Doesn't make sense because the target cannot be a dog.
d Doesn't make sense at all.

The only relevant answer is c, since the phrase concerns them most.

This could be completely wrong, I am in year 10.
Title: Re: VCE English Question Thread
Post by: Annon4589 on October 07, 2020, 05:35:11 pm
Hi guys,

By any chance has anyone done or has got the 2020 Insight or Boobook  English Exam?
Title: Re: VCE English Question Thread
Post by: The Cat In The Hat on October 10, 2020, 10:17:35 am
Hi guys,

By any chance has anyone done or has got the 2020 Insight or Boobook  English Exam?
I have done the Boobook one.

I have a question- In doing an AA, if the author's name is really long (the example I'm doing now is 'Kennedy-Khan') do we have to use the whole surname or just part of it? especially if it's hyphenated.
Title: Re: VCE English Question Thread
Post by: darkz on October 10, 2020, 11:11:11 am
I have done the Boobook one.

I have a question- In doing an AA, if the author's name is really long (the example I'm doing now is 'Kennedy-Khan') do we have to use the whole surname or just part of it? especially if it's hyphenated.

Well, you should write out the entire name - but don't stress too much, since I accidentally used the first name for one of the authors throughout my AA in the exam and still did really well hehe.
Title: Re: VCE English Question Thread
Post by: Coolgalbornin03Lo on October 10, 2020, 08:49:17 pm
What’s the perfect amount of evidence and quotes in Each essay.

In comparative I do one Technique per text cause the paragraphs are huge but if cause a few more “quotes” sprinkled in.

Text response I usually do 1-2 pieces of analyses evidence 1 a quote and one a cinematic technique with little “quotes” littered throughout.

Language analysis for the visual I find at least 3 visual techniques and in the paragraphs 2-3 analysed  things + quotes which are explained.
Title: Re: VCE English Question Thread
Post by: tiredandstressed on October 11, 2020, 12:28:15 am
What’s the perfect amount of evidence and quotes in Each essay.

In comparative I do one Technique per text cause the paragraphs are huge but if cause a few more “quotes” sprinkled in.

Text response I usually do 1-2 pieces of analyses evidence 1 a quote and one a cinematic technique with little “quotes” littered throughout.

Language analysis for the visual I find at least 3 visual techniques and in the paragraphs 2-3 analysed  things + quotes which are explained.
You asked this question before in the forum, you just re-worded it.
What do you mean by "technique" (i.e. literary devices). And do you mean 1 technique per paragraph or for the whole essay
I would encourage a mix of literary devices, construction elements and quotes.
You should have a minimum of one quote per text per paragraph but a discussion of literary devices is great as well.
My tip is three pieces of evidence per text for a paragraph, so that's roughly six pieces of evidence per paragraph.
You are right the paragraphs do get huge- so it is important to be concise and choose the best & appropriate evidence.
1 literary device in a paragraph is not sufficient, the inclusion of quotes is required always.
That's what I did and I did decent in the final exam :)
Title: Re: VCE English Question Thread
Post by: ZACCO003 on October 12, 2020, 09:12:30 pm
Hello,

I'm confused as to this topic and how to plan it to write an essay, it is on the lieutenant written by Kate Grenville, the topic is “By the time the snag had unraveled the whole thing, it was too late to mend”. The Lieutenant is a story of regret and remorse. HELP please it makes now sense to me.

Thanks in advance,

Title: Re: VCE English Question Thread
Post by: The Cat In The Hat on October 15, 2020, 09:13:28 pm
My teacher's said I'm not writing long enough essays (to get those 19-20/20s) in exam conditions; my average is right to the end of three pages (per essay), medium sized to maybe a bit small handwriting, with some crossing out (maybe more than average deletion...). How do I increase the length without just getting excessively verbose? Also said that while the depth of ideas is good, I need to increase the sophistication and command of language etc.... how do I do all this?
Thanks! :)
Also if anyone has any tips for effective exam revision when aiming to get 19/20 for each essay, I'd like to hear them. :)
Title: Re: VCE English Question Thread
Post by: Coolgalbornin03Lo on October 17, 2020, 08:51:06 pm
Do you guys have every single essay you write get marked?

I was thinking about doing a practise English exam this Friday and then another one next Monday (3 days later) but I doubt my teacher will be able to mark them in that time.
Title: Re: VCE English Question Thread
Post by: GodNifty on October 17, 2020, 09:59:37 pm
Do you guys have every single essay you write get marked?

I was thinking about doing a practise English exam this Friday and then another one next Monday (3 days later) but I doubt my teacher will be able to mark them in that time.
Assuming you go to a relatively mid to large school, I would advise you to send your practise English exam to all the year 12 English teachers. If you don't have a lot of year 12 teachers, then I guess your best bet is to send your year7-year11 English teachers with your analysing argument essay.
Title: Re: VCE English Question Thread
Post by: Coolgalbornin03Lo on October 19, 2020, 10:14:59 pm
Having a lot of trouble finding examples in my second comparative text for these two really hard sub themes (education).

How do I go about this?
Title: Re: VCE English Question Thread
Post by: whys on October 19, 2020, 10:20:58 pm
Having a lot of trouble finding examples in my second comparative text for these two really hard sub themes (education).

How do I go about this?
I think the best way is to reread your texts. You don't need to read all of them from start to end, just key scenes that showcase those themes or major aspects of those themes. Or, if you've made notes throughout the year, look back on them to find those quotes. Some themes can be quite obscure and there may only be a few quotes, or the theme may be explored in a subtle fashion, so sometimes it's not as easy as flicking through the text to find them. I would ask a study group, or you can approach your teacher and ask for help too. Try googling some critical essays on your text and find ones that relate to your themes - this can kickstart your analysis and help you find your footing again. Remember that themes from essay prompts will always be linked to BOTH texts in a significant manner, so there should be no shortage of examples and interpretations of these examples to develop a well-thought-out response.
Title: Re: VCE English Question Thread
Post by: ArtyDreams on October 20, 2020, 01:36:51 pm
- Do examiners give half marks for exams? i.e. 8.5s?

- Are we able to have ‘discussions’ within body paragraphs? For example, rather than having 2 paragraphs agree with the prompt and one paragraph disagree, can one body paragraph itself have for and against?
Something like this: While this suggests the women's lives were miserable, it also shows the progressing of societal views, and that the world has become a more accepting place for them.
Title: Re: VCE English Question Thread
Post by: The Cat In The Hat on October 20, 2020, 01:48:21 pm
- Do examiners give half marks for exams? i.e. 8.5s?

- Are we able to have ‘discussions’ within body paragraphs? For example, rather than having 2 paragraphs agree with the prompt and one paragraph disagree, can one body paragraph itself have for and against?
Something like this: While this suggests the women's lives were miserable, it also shows the progressing of societal views, and that the world has become a more accepting place for them.
Fairly sure no half marks.

I don't know if we're really able to have discussions; it probably depends on how it's handled/what the topic is etc. It could provide a more nuanced view, especially in a comparative, but I think it would need careful treatment. (Although note, I'm just a year 12 as well :)))
Title: Re: VCE English Question Thread
Post by: hel256 on October 20, 2020, 03:12:20 pm
- Do examiners give half marks for exams? i.e. 8.5s?

- Are we able to have ‘discussions’ within body paragraphs? For example, rather than having 2 paragraphs agree with the prompt and one paragraph disagree, can one body paragraph itself have for and against?
Something like this: While this suggests the women's lives were miserable, it also shows the progressing of societal views, and that the world has become a more accepting place for them.

I've done things similar in my own essays and haven't suffered for it.
Remember that there is no 'set' structure for how to write an essay; the essential components are ideas, the efficacy of which they are conveyed depends on your expression/fluency. This main idea for a paragraph demonstrates a nuanced understanding of your text, which is what the examiners are looking for, so its perfectly fine to do if you believe it to be the best way in which to present your argument.

(I'm also in Year 12, but have had this message of flexible structure hammered into me by my English teacher, who is a VCAA examiner)
Title: Re: VCE English Question Thread
Post by: a weaponized ikea chair on October 24, 2020, 10:30:28 am
How many body paragraphs are we expected to have for an essay? Three or four?
Title: Re: VCE English Question Thread
Post by: ArtyDreams on October 24, 2020, 10:46:33 am
How many body paragraphs are we expected to have for an essay? Three or four?

I don't think theres an expectation to have a certain number of body paragraphs - I think it depends on the type of prompt you get - and how best to organise your ideas. Quality is definetely more important than quantity, if you can have 3 well written, anaytical paragraphs, it is plenty - and better than having four paragraphs that only scratch the surface of analysis.
Sometimes 3 is sufficient, but if you tend to have more ideas 4 might be helpful too. I personally stick with 3 due to time constraints, but if I feel like the essay is better divided up into 4 body paragraphs I do that too.

Hope this helps!
Title: Re: VCE English Question Thread
Post by: Coolgalbornin03Lo on October 29, 2020, 03:15:11 pm
Does anyone have experience with the company boo book? My school hired them to company exams and results were dodgy. I’m wondering if it’s because my cohort was so large or....?
Title: Re: VCE English Question Thread
Post by: The Cat In The Hat on October 30, 2020, 10:39:44 am
Does anyone have experience with the company boo book? My school hired them to company exams and results were dodgy. I’m wondering if it’s because my cohort was so large or....?
Our cohort was kinda small (19 kids) and we'd been having them all year, so perhaps it's the difference? We got a page of feedback each (for the three sections thus a paragraph or so).
Title: Re: VCE English Question Thread
Post by: whys on October 30, 2020, 08:34:44 pm
Hey everyone,
Would we be marked down for long essays? I know this is a bit of a weird question but I've found that all the essays I write in exam conditions end up being around 1350 words... I really don't want to lose marks for the length of my essays but I have trouble reducing the amount of what I'm writing. I don't have any issues with rambling/going off-topic, etc, I just write a lot. From the feedback I've gotten my essays are around 8-10/10 but I don't know if VCAA assessors would mark me down for the length of my essays.

Thank you!
Title: Re: VCE English Question Thread
Post by: The Cat In The Hat on October 30, 2020, 08:49:26 pm
Hey everyone,
Would we be marked down for long essays? I know this is a bit of a weird question but I've found that all the essays I write in exam conditions end up being around 1350 words... I really don't want to lose marks for the length of my essays but I have trouble reducing the amount of what I'm writing. I don't have any issues with rambling/going off-topic, etc, I just write a lot. From the feedback I've gotten my essays are around 8-10/10 but I don't know if VCAA assessors would mark me down for the length of my essays.

Thank you!
The only advice I can give you is that my teacher told me that to get the really top marks in the exam (ie going from 18/20 to 19 or 20/20) I need to lengthen my essays, which are currently around three full pages (handwritten) with reasonably small handwriting (although not tiny). So no, I don't think you'll get marked down.
Title: Re: VCE English Question Thread
Post by: tiredandstressed on October 30, 2020, 09:39:46 pm
Hey everyone,
Would we be marked down for long essays? I know this is a bit of a weird question but I've found that all the essays I write in exam conditions end up being around 1350 words... I really don't want to lose marks for the length of my essays but I have trouble reducing the amount of what I'm writing. I don't have any issues with rambling/going off-topic, etc, I just write a lot. From the feedback I've gotten my essays are around 8-10/10 but I don't know if VCAA assessors would mark me down for the length of my essays.
Thank you!
Unless you are either repeating yourself or using long quotes I don't this is an issue
I only could write around ~900 words in an hour, so it's great you can get to 1350 as long as you're not going over time then you should be ok/
However, 1350 is quite a bit over 1000 (the typical maximum) how long are your intros and conclusions (they really shouldn't be more than 4 sentences) but considering you're getting positive feedback I wouldn't stress.
Edit your own essays and see if you find anything that could be removed or written more concisely!
Title: Re: VCE English Question Thread
Post by: Coolgalbornin03Lo on October 30, 2020, 09:46:54 pm
Unless you are either repeating yourself or using long quotes I don't this is an issue
I only could write around ~900 words in an hour, so it's great you can get to 1350 as long as you're not going over time then you should be ok/
However, 1350 is quite a bit over 1000 (the typical maximum) how long are your intros and conclusions (they really shouldn't be more than 4 sentences) but considering you're getting positive feedback I wouldn't stress.
Edit your own essays and see if you find anything that could be removed or written more concisely!

My introductions are long but my teacher has never been opposed to them. It’s the length of the one which I posted and you commented on one time.
Title: Re: VCE English Question Thread
Post by: Sine on October 30, 2020, 09:47:36 pm
Hey everyone,
Would we be marked down for long essays? I know this is a bit of a weird question but I've found that all the essays I write in exam conditions end up being around 1350 words... I really don't want to lose marks for the length of my essays but I have trouble reducing the amount of what I'm writing. I don't have any issues with rambling/going off-topic, etc, I just write a lot. From the feedback I've gotten my essays are around 8-10/10 but I don't know if VCAA assessors would mark me down for the length of my essays.

Thank you!
The actual word count wouldn't be the reasons you lose marks. If you are just writing fluff, going on tangents and overall just not able to deliver your point concisely then you fall into problems. If the 1300+ words are of high quality you should be fine.

Title: Re: VCE English Question Thread
Post by: whys on October 30, 2020, 09:58:59 pm
Thank you everyone for your input! My intros are about ~150 words for TR and Comparative, and a bit shorter (100-ish) for AA. Conclusions usually 3-4 sentences. The bulk of my essay is the body paragraphs. I'll definitely try work on being more concise and work on elaborating on more specific examples and restricting myself. I feel like if I spent more time planning rather than writing I could have a shorter, higher quality essay rather than one that's longer, since I barely plan and sort of 'write with the flow' if that makes sense.

Thanks again!
Title: Re: VCE English Question Thread
Post by: Coolgalbornin03Lo on October 31, 2020, 08:22:41 pm
Is an introduction which is one page for L.A too much? My writing is quite big and this is in on the VCAA paper. I went to a lecture and a VCAA assessor said paragraphs shouldn’t be a page long.
Title: Re: VCE English Question Thread
Post by: Coolgalbornin03Lo on October 31, 2020, 09:02:46 pm
Is an introduction which is one page for L.A too much? My writing is quite big and this is in on the VCAA paper. I went to a lecture and a VCAA assessor said paragraphs shouldn’t be a page long.

Sorry to double post but in text response what are “implications”. I’m watching a VATE revision lecture and it states to get an 8/10 you must consider the “implications” of the topics.
Title: Re: VCE English Question Thread
Post by: tiredandstressed on October 31, 2020, 09:20:23 pm
Is an introduction which is one page for L.A too much? My writing is quite big and this is in on the VCAA paper. I went to a lecture and a VCAA assessor said paragraphs shouldn’t be a page long.
My introductions were only like 3-5 sentences max, how long does it take you to write
If you can write the intro within 5 minutes it shouldn't be an issue, it not really an issue if your intro is too long, but more so how much time does it take.
Title: Re: VCE English Question Thread
Post by: Snow Leopard on October 31, 2020, 10:38:38 pm
Hey everyone,
Would we be marked down for long essays? I know this is a bit of a weird question but I've found that all the essays I write in exam conditions end up being around 1350 words... I really don't want to lose marks for the length of my essays but I have trouble reducing the amount of what I'm writing. I don't have any issues with rambling/going off-topic, etc, I just write a lot. From the feedback I've gotten my essays are around 8-10/10 but I don't know if VCAA assessors would mark me down for the length of my essays.

Thank you!
Hey whys,
my English teacher (who has marked VCE Eng exams before) says that the examiners don't care if you go over the word limit as long as what you're writing is quality.
Title: Re: VCE English Question Thread
Post by: Coolgalbornin03Lo on November 01, 2020, 08:58:55 am
For section C must you always say what that technique makes the audience feel and do or sometimes just feel?
Title: Re: VCE English Question Thread
Post by: tiredandstressed on November 01, 2020, 12:19:06 pm
For section C must you always say what that technique makes the audience feel and do or sometimes just feel?
From my experience, stronger essays does both how it makes the audience feel & do (putting it in simple terms)
For instance "the author's appeal to pathos aims to manipulate the audience to believe that if actions are not done to address climate change, their children's future is at jeopardy. Specifically, her emotional plea encourages the audience to support her pursuit to unite and continue to protest against the government until the mining industry is reformed"
You don't necessarily have to do it for every technique, just make sure the "do" is specific to the technique, the link from feel -> do should be logical and when an examiner reads your essay they should see the flow and links of logic to go from the technique -> feelings -> audience's actions
Title: Re: VCE English Question Thread
Post by: Coolgalbornin03Lo on November 01, 2020, 11:25:57 pm
Thanks tired and stressed!

Hey everyone sorry for more questions but:

How do you write with enough time to proof read? How much time is required for adequate proof reading? I’m worried my errors could be what brings me down from an 8 to a 7
Title: Re: VCE English Question Thread
Post by: The Cat In The Hat on November 02, 2020, 12:49:42 pm
Thanks tired and stressed!

Hey everyone sorry for more questions but:

How do you write with enough time to proof read? How much time is required for adequate proof reading? I’m worried my errors could be what brings me down from an 8 to a 7
The only thing I can think of is to somehow write faster... I don't know though. I don't proofread my essays except inasmuch as is done while writing them - should I somehow write faster and take time at the end to proofread? I mean sometimes I skim but apart from that nothing. Would that boost my grade, though? (Sorry to add onto your question Coolgal.)
Title: Re: VCE English Question Thread
Post by: Sine on November 02, 2020, 02:21:36 pm
Thanks tired and stressed!

Hey everyone sorry for more questions but:

How do you write with enough time to proof read? How much time is required for adequate proof reading? I’m worried my errors could be what brings me down from an 8 to a 7
I don't think you need a leave a heap of time for proofreading. I think ~5 minutes for each essay is probably more than enough.

For English, your proofreading is going to be likely just fixing some spelling errors, making it a bit more clear, fixing expression, fix grammar, fix duplicated/missed words etc. You can't exactly change the whole point of your essay in the last few minutes.
Title: Re: VCE English Question Thread
Post by: Coolgalbornin03Lo on November 02, 2020, 02:56:12 pm
I don't think you need a leave a heap of time for proofreading. I think ~5 minutes for each essay is probably more than enough.

For English, your proofreading is going to be likely just fixing some spelling errors, making it a bit more clear, fixing expression, fix grammar, fix duplicated/missed words etc. You can't exactly change the whole point of your essay in the last few minutes.

Oh okay. I’ve just been told that these small errors can add up taking you from high marks to lower ones because of the “messiness” or whatever- thank you for cleaning that up!!!!
Title: Re: VCE English Question Thread
Post by: Coolgalbornin03Lo on November 03, 2020, 10:23:24 am
How do you answer “to what extent” or “how does author xyz” prompts?

For the to what extent should you go agree, agree, agree but here’s another possibility erring on the othersides POV?
Title: Re: VCE English Question Thread
Post by: The Cat In The Hat on November 03, 2020, 10:56:36 am
If there's a 'to what extent' I generally answer it same as I do a Revs 'to what extent' question - start out with a nice statement like 'To a large extent...' and so on.
I would go either agree, agree, disagree or agree, disagree, disagree (so for instance starting 'To some extent, Austen uses hyperbole in drawing her characters in order to comment on the flawed perception of marriage in her community; however, her liberal use of satire and...' [probably a really bad sentence, as I'm not writing off an actual prompt but it serves my purpose]), generally not using 'To some extent' but sometimes using it if it works for my purpose. But starting out with the 'To __ extent' is apparently an instant tick from the examiner (at least in Revs) as you've started out by answering the question.
As to the 'how does author' prompts? I have no idea. I've never answered any... should I've??
Title: Re: VCE English Question Thread
Post by: tiredandstressed on November 03, 2020, 11:02:42 am
How do you answer “to what extent” or “how does author xyz” prompts?

For the to what extent should you go agree, agree, agree but here’s another possibility erring on the othersides POV?
To what extent questions are my fave tbh
Pretty much decide do you want to 'mainly agree' or 'mainly disagree' the prompt
The structure I usually do is
BP1: Yes
BP2: No
BP3: But- challenge the prompt, look at it at a different unique angle, provide complexity in your discussion
However, I used this structure since year 11 and always got positive feedback- but you have to be careful not to contradict yourself; that was a skill I worked on for two years, probz not feasible in a week.
So my advice is to come to a contention (mostly agree or mostly disagree) and make sure you discuss both sides of the prompt, stronger essays will go beyond and provide a new original angle, a unique perspective that flips the prompt.
As for how does the author...
These were my least favourite as it was hard to develop an opinion (for the most part you can't disagree)
- Discuss the effect it has on the audience
- How does the author -> link back to the author's intentions
- Discuss unique outlooks the prompt suggests
Title: Re: VCE English Question Thread
Post by: Coolgalbornin03Lo on November 03, 2020, 11:08:33 am
Thanks!!!!!!!

My other questions was is writing full practise exams necessary? I’ve done many 1 hour essays and a few 2 in 2 hour essays but I’ve only done one FULL practise.
Title: Re: VCE English Question Thread
Post by: tiredandstressed on November 03, 2020, 11:16:55 am
Thanks!!!!!!!

My other questions was is writing full practise exams necessary? I’ve done many 1 hour essays and a few 2 in 2 hour essays but I’ve only done one FULL practise.
I only did like two three-hour practice exams (coz quite frankly who has the time for that) with only one week till the exam, there probably isn't really a benefit to do a three-hour practise exam, if you plan to do one I would recommend to sit it at the same time as the exam 9:00 am-12:45 pm but by no later than Friday.
At this point your revision should be dedicated to be preparing for as many prompts in Section A and B and completing practice timed Section C's.
So I think just doing the one hour or two-hour sessions should be fine, you don't want to burn yourself out, in the end of the day you have other exams to study for!
All the best,
T&S
Title: Re: VCE English Question Thread
Post by: Geoo on November 05, 2020, 10:13:18 pm
I plan on doing one more LA essay, should I do the 2019 section c, or pick one from the older study designs?

Are those before 2017 still relevant for section C?
Title: Re: VCE English Question Thread
Post by: Coolgalbornin03Lo on November 05, 2020, 11:36:53 pm
I plan on doing one more LA essay, should I do the 2019 section c, or pick one from the older study designs?

Are those before 2017 still relevant for section C?

The ones before 2017 (like 14,15,16) are really good and I enjoyed them! 2016 is a personal favourite because of the visual!!!! I’d go for that one if I were you but it’s your choice whether you do 2019 or an earlier one :)

If you don’t do 2019 you can just do a plan then look through the examiner report :) good luck!!!

Also I think what’s different is in past study designs it was just language analysis (so the examiner report may reflect that) whilst in the current design it’s language and argument analysis so maybe the texts won’t have this but I never found this problem!!! I wrote on these two months ago but my teacher told me this last week :)
Title: Re: VCE English Question Thread
Post by: Coolgalbornin03Lo on November 06, 2020, 06:43:44 pm
I forgot to ask my teacher this and I’ll never see her again :'(

What does “implications” mean on 8+/10 on text response? I know I can’t change my writing this close but I was wondering is that view and values and authorial intent I’ve been incorporating or.......?
Title: Re: VCE English Question Thread
Post by: Coolgalbornin03Lo on November 09, 2020, 02:11:25 pm
Do “how this is shown” prompts always mean how ideas are conveyed through techniques or can I say how this is shown through concepts?
Title: Re: VCE English Question Thread
Post by: tiredandstressed on November 09, 2020, 02:50:11 pm
Do “how this is shown” prompts always mean how ideas are conveyed through techniques or can I say how this is shown through concepts?
I reckon both- the prompt does not restrict you to just discuss techniques, you can analyse how big pictures ideas are depicted to "how this is shown" prompts
Title: Re: VCE English Question Thread
Post by: Coolgalbornin03Lo on November 09, 2020, 06:03:31 pm
I reckon both- the prompt does not restrict you to just discuss techniques, you can analyse how big pictures ideas are depicted to "how this is shown" prompts

Thank you so much!!! This is what I’ve been doing but when doing some last minute planning I was like o- “maybe I’ve been a fool....why hasn’t my teachers said anything”
Title: Re: VCE English Question Thread
Post by: a weaponized ikea chair on November 09, 2020, 07:00:20 pm
Anyone got any synonyms for idea/notion? I can only use 'idea' and 'notion' so much. Like for instance:

Author Jane Doe conveys the idea/notion that...

Thanks
Title: Re: VCE English Question Thread
Post by: The Cat In The Hat on November 09, 2020, 08:02:37 pm
Anyone got any synonyms for idea/notion? I can only use 'idea' and 'notion' so much. Like for instance:

Author Jane Doe conveys the idea/notion that...

Thanks
concept
belief
conviction
perception
opinion

Those are only what I can think of off the top of my head. Hope it helps!
Title: Re: VCE English Question Thread
Post by: MoonChild1234 on November 09, 2020, 08:03:48 pm
Anyone got any synonyms for idea/notion? I can only use 'idea' and 'notion' so much. Like for instance:

Author Jane Doe conveys the idea/notion that...

Thanks

not really perfect fits but:
-ideology
- viewpoint
-stance
Title: Re: VCE English Question Thread
Post by: FortniteGode876 on November 11, 2020, 02:11:47 am
Hello!

I read this from the 2019 examiner's report and was wondering if anyone knows what happens to those who base their essay off another high scoring one from a text guide like atarnotes or tsfx. Will they get a zero?
What if your essay is very similar to the guide (not the same as it is a different prompt but is 80% similar)

Title: Re: VCE English Question Thread
Post by: Coolgalbornin03Lo on November 11, 2020, 07:36:22 am
Hello!

I read this from the 2019 examiner's report and was wondering if anyone knows what happens to those who base their essay off another high scoring one from a text guide like atarnotes or tsfx. Will they get a zero?
What if your essay is very similar to the guide (not the same as it is a different prompt but is 80% similar)

This scenario sounds like plagiarism, and although no ones ever said that’s bad for English people on AN have alluded to the fact if you memorise the work must be your own. If the assessor was the author of that study guide they may know but otherwise please just relax it’s now over and we are almost free :)
Title: Re: VCE English Question Thread
Post by: FortniteGode876 on November 11, 2020, 09:17:52 am
This scenario sounds like plagiarism, and although no ones ever said that’s bad for English people on AN have alluded to the fact if you memorise the work must be your own. If the assessor was the author of that study guide they may know but otherwise please just relax it’s now over and we are almost free :)


The author was my tutor who is also a friend. He told me that a commercial tutoring company asked for his resources and he gave them his essays and they included it on their study guide. I did alter it tho obviously since it was a different prompt
Title: Re: VCE English Question Thread
Post by: a weaponized ikea chair on December 23, 2020, 12:07:14 pm
Does a simile have to have like or as? I looked it up but couldn't find a clear definition.

I got into a debate with a friend over whether "he seemed to move like a tiger" was a simile. I argued it was a simile since it directly compared two things, but my friend says it wasn't.

Any insight into this?
Title: Re: VCE English Question Thread
Post by: Jinju-san on December 25, 2020, 02:04:48 pm
Hey guys!

This might be a weird question..
But does anyone who studied ‘The Lieutenant’ (Kate Grenville) this year have any useful notes or analysis they could provide as a helpful resource?
Thanks!
Title: Re: VCE English Question Thread
Post by: zoharreznik on January 05, 2021, 06:57:27 pm
Dear all upcoming Year 12 students,
If you’re worried about English, don’t be. It can’t be worse than being ranked 2-3 in a cohort of 100 students and then bringing them all down by getting an A on the exam 🥴
Title: Re: VCE English Question Thread
Post by: The Cat In The Hat on January 06, 2021, 07:32:56 am
Dear all upcoming Year 12 students,
If you’re worried about English, don’t be. It can’t be worse than being ranked 2-3 in a cohort of 100 students and then bringing them all down by getting an A on the exam 🥴
You weren't the only one... our cohort was smaller, but I was ranked first.... I suspect I pulled down a good many of my classmates. :-\
~
Dear all upcoming Year 12 students,
If you hate English, work at it. Truly. Halfway through the year - despite absolutely loving one of my texts, Pride and Prejudice (absolutely recommend), I was still adamantly refusing to do essays or really any work for English. Then I began to try and do the weekly essays the teacher set.
And I found that actually, I enjoyed writing essays, seeing the small connections everyone else seemed to have missed. Polishing my prose until it was sharp and focused. Writing an essay without redundancy and in a set time.
English was very disliked. But turning around and enjoying it halfway through the year, liking to write, to improve, completely changed everything.
I suppose I could go back and look at how not changing anything would've changed my study score and ATAR. At a rough estimate, I would've got possibly 35 - rather than 40 - as a study score, dropping my aggregate five points.
My ATAR would have gone from 85.00 to 82.15. And if - as mine is - your ATAR is on the dicey edge of might get in, might not, it's absolutely worth that extra time and effort.
And in the process, you might even learn to enjoy it.
Title: Re: VCE English Question Thread
Post by: kayzee on February 18, 2021, 07:06:02 am
Hello  :)

I'm a year 12 English Student and we have a SAC coming up...I'm new to ATAR Notes and not sure if I'm using it quite right but woud love some help...
Please see my topic on Cognitive Dissonance and The Lieutenant (Kate Grenville). I really need some help on my essay start if anyone has time to check it out. Thanks!
Title: Re: VCE English Question Thread
Post by: thenuttyprofessor on March 07, 2021, 07:07:54 pm
https://atarnotes.com/forum/index.php?topic=194214.msg1190068;topicseen#msg1190068

Can someone please chuck us some feedback on these? Would be a great help!
Title: Re: VCE English Question Thread
Post by: ErnieTheBirdi on April 16, 2021, 09:50:08 pm
Rear Window Argues that it is more important to be right than to be ethical. Do you agree?

Contention: Rear Window argues that it is more imperative to be unerring than to be righteous, however it must be acknowledged that whilst the film primarily suggests this, the ethics that are infused here must be considered.

Intro:


(Yes)Bp1:  In Hitchcock’s rear window it appears that the notion of ‘being correct’ is quite significant. Although not directly demonstrated in the film, this is shown through Jeff’s reluctance to take no for an answer, especially when it comes to the mystery disappearance of Mrs. Anna Thorwald.

(Yes)Bp2: The idea of “being right” throughout the movie is demonstrated to have multiple meanings, whether that is about one’s morals or being accurate about an issue. Hitchcock demonstrates this to his audience through many instances through Jeff and Lisa’s determination and journey in solving the case.


(No)Bp3: Despite the fact the film does primarily demonstrate to its audience that to be right is more important, ethics still plays a critical role throughout the film, especially in Jeff’s life.

Someone pls help, do these arguments work and flow ??????? Its been so long
Title: Re: VCE English Question Thread
Post by: biology1234 on May 30, 2021, 05:43:26 pm
Arguments on why political correctness unites people.
Title: Re: VCE English Question Thread
Post by: lm21074 on May 30, 2021, 07:04:06 pm
Arguments on why political correctness unites people.
See my response in the EngLang thread. Other people may respond too.

Just a heads up - posting the same query in multiple threads will not get you an answer quicker. Please refrain from doing so. Thanks :)
Title: Re: VCE English Question Thread
Post by: S_L1003 on September 11, 2021, 12:50:26 pm
Hi! I have finished all my English sacs and now I'm just studying for the exam - I have been writing timed essays, sometimes one other times two, for LA, TR and argument analysis, in all my spares and on weekends. I probably average about one a day. I can't really tell if I have improved at all, and I feel for that amount of effort I should be at a level higher than I am. I am doing ransom + the queen and all the light this year.

I know I have to work to get better, and so I am trying, but I'd really like to get 40+ for eng this year - could I have help on the best ways to study?
Title: Re: VCE English Question Thread
Post by: Billuminati on September 11, 2021, 12:58:58 pm
Hi! I have finished all my English sacs and now I'm just studying for the exam - I have been writing timed essays, sometimes one other times two, for LA, TR and argument analysis, in all my spares and on weekends. I probably average about one a day. I can't really tell if I have improved at all, and I feel for that amount of effort I should be at a level higher than I am. I am doing ransom + the queen and all the light this year.

I know I have to work to get better, and so I am trying, but I'd really like to get 40+ for eng this year - could I have help on the best ways to study?

Please see the attached screenshot for my advice to some redditors asking for VCE English exam prep tips. I’m aware I did different texts to you, but I think it may still be beneficial.

https://drive.google.com/file/d/1pT42Uw0_efS8KcxuasVWUH6rTQwzax41/view?usp=drivesdk
Title: Re: VCE English Question Thread
Post by: S_L1003 on September 11, 2021, 04:34:27 pm
Please see the attached screenshot for my advice to some redditors asking for VCE English exam prep tips. I’m aware I did different texts to you, but I think it may still be beneficial.

https://drive.google.com/file/d/1pT42Uw0_efS8KcxuasVWUH6rTQwzax41/view?usp=drivesdk

Thanks so much - this was really helpful!
Title: Re: VCE English Question Thread
Post by: Jinju-san on September 14, 2021, 07:26:10 pm
Hey guys!
Hope exam prep is going well for everyone!
I just remembered recently how one of my English teachers in year 8 advised me to start writing 4-5 body paragraphs in each essay, because she said that doing so sets you up well for VCE English.
Is it recommended that people write more than the usual 3 body paragraphs?
None of my other English teachers have advised me to do this, and I definitely can’t see myself doing this on an exam, but I’m just curious to know whether it is recommended by other VCE teachers and students or not.

Many thanks!
Title: Re: VCE English Question Thread
Post by: tiredandstressed on September 14, 2021, 10:05:53 pm
Hey guys!
Hope exam prep is going well for everyone!
I just remembered recently how one of my English teachers in year 8 advised me to start writing 4-5 body paragraphs in each essay, because she said that doing so sets you up well for VCE English.
Is it recommended that people write more than the usual 3 body paragraphs?
None of my other English teachers have advised me to do this, and I definitely can’t see myself doing this on an exam, but I’m just curious to know whether it is recommended by other VCE teachers and students or not.

Many thanks!
Nowhere is it said by VCCA that four or more paragraphs will be advantageous to the student, so no I don't think it would beneficial.
For some essays, maybe a 4 paragraph structure might be a better approach (however is not always necessary) I have seen some 9/10 -10/10 essays by VCCA that only used three-body paragraphs so stick with the structure you are used to.
Students who use four paragraphs have properly taken the time to learn and adapt the essay style to do, so this is something you cannot do out of the blue.
Hope this helps
-T&S
Title: Re: VCE English Question Thread
Post by: biology1234 on September 16, 2021, 08:57:09 pm
hi, what type of connected speech is the word binged?
Would it be Elision?
Title: Re: VCE English Question Thread
Post by: biology1234 on September 16, 2021, 09:07:52 pm
also what would fav be from favourite
Title: Re: VCE English Question Thread
Post by: sgrace on October 23, 2021, 05:28:55 pm
Does anyone have any tips for memorising quotes? I think I've left it a bit late but any advice helps :)
Title: Re: VCE English Question Thread
Post by: Slime_Bob on April 08, 2022, 06:28:49 pm
Hi there, I'm a complete newbie to ATAR notes, so forgive me if I make any formatting errors or have posted in the wrong thread.

I'm currently a yr 11 student doing 1/2 English, and I'm not doing bad at it, but not exactly doing... great. I want to get better and my plan is to write more essays. The online problem is I can't find many essay prompts about the book we're currently focusing on ('Ransom' by David Malouf) so if anyone knows any places to find writing prompts on that topic I would be super happy about that.

Also, once I do finish an essay, is the best way to get it checked is to talk to a teacher or self-check it via a rubric, or...?
Title: Re: VCE English Question Thread
Post by: emilydd123 on May 14, 2022, 02:44:41 pm
I'm really really stressed.
Prior to year 12 I'd never ever dropped below an A in English and now suddenly on my first two SACs I've gotten B's. I've been communicating with my teacher, getting feedback and getting her to mark my work. She says it's really good and only gives me minor things to work on and now I can't get passed a B and I'm distraught. I know my hopes for a 40 are well and truly gone which was my goal for the year :( I just don't know what to do my parents are going to be mad this has happened again.